You are on page 1of 1001

2017-18 100 &

op kers
Class 12 T
By E ran culty
-JE Fa r
IIT enior emie .
S fP r es
o titut
Ins

PHYSICS
FOR JEE MAIN & ADVANCED
SECOND
EDITION

Exhaustive Theory
(Now Revised)

Formula Sheet
9000+ Problems
based on latest JEE pattern

2500 + 1000 (New) Problems


of previous 35 years of
AIEEE (JEE Main) and IIT-JEE (JEE Adv)

5000+Illustrations and Solved Examples


Detailed Solutions
of all problems available

Topic Covered Plancess Concepts


Tips & Tricks, Facts, Notes, Misconceptions,
Geometrical Optics Key Take Aways, Problem Solving Tactics

PlancEssential
Questions recommended for revision
16. GEOMETRICAL
OPTICS

1. INTRODUCTION
Light is a form of radiant energy; that is, energy is emitted by the excited atoms or molecules that can cause the
sensation of vision by a normal human eye.
The branch of physics that deals with the phenomena of light is called optics. There are two branches of optics:
(a) ray optics and (b) wave optics.

SOME DEFINITIONS
(a) Ray: The ‘path’ along which the light travels is called a ray. The rays are represented by straight lines with
arrows directed towards the direction of travel of light.
(b) Beam: A bundle of rays is called a beam. A beam is parallel when its rays are parallel, it is divergent when its
rays spread out from a point, and it is convergent when its rays meet at a point.

Object and image


If the rays from a point on an object actually diverge from it and fall on the mirror, then the object is the real object
of the mirror. If the rays incident on the mirror does not start from a point but appear to converge at a point, then
that point is the virtual object of the mirror.

O O O O
Real Virtual object
Real object Virtual object object

O O
Real Virtual
object object

Figure 16.1
1 6 . 2 | Geometrical Optics

If the rays converge at a point after an interaction with a surface, then a real image will be formed, and if the rays
diverge after an interaction with a surface, a virtual image will be formed.

O I I O

(a) (b)

Figure 16.2

Real object, virtual object, real image, virtual image: In Fig. 16.2 (a), the object is real, while the image is virtual.
In Fig. 16.2 (b), the object is virtual, while the image is real.

2. REFLECTION OF LIGHT

Definition
When the light falling on a surface turns back into the same medium, it means it is reflected. The angle made by
the incident ray with the normal to the reflecting or refracting surface is called the angle of incidence, and the angle
made by the reflected or refracted ray with normal is called the angle of reflection or refraction.

2.1 Laws of Reflection


(a) When the incident ray, the reflected ray and the normal to the reflecting
surface at the point of incidence lie in the same plane, it is called the ir ir
plane of incidence.  

(b) The angle of incidence is equal to the angle of reflection ∠i =∠r .


Figure 16.3
2.2 Deviation of Ray
The deviation is defined as the angle between the directions of the incident ray and the reflected ray (or the
emergent ray). It is generally denoted by δ .

 (angle of deviation)
N 180°
B
A
i i

O 90° i (angle of
A’ incidence)

Figure 16.4

Here, ∠A'OB = δ = ∠AOA'− ∠AOB = 1800 − 2i


or =δ 1800 − 2i
P hysi cs | 16.3

PLANCESS CONCEPTS

The above two laws of reflection can be applied to the reflecting surfaces that are not even horizontal.
The following Fig. 16.5 illustrates this point.

r
i
i
r r
i

Figure 16.5

Vaibhav Gupta (JEE 2009 AIR 54)

3. REFLECTION FROM A PLANE SURFACE (OR PLANE MIRROR)


Almost everybody is familiar with the image formed by a plane mirror. If the object is real, the image formed by a
plane mirror is virtual, erect, of same size of the original object and in the same distance from the mirror.

B B’

O I
A A’

Figure 16.6

If an object is placed in front of a mirror as shown in Fig. 16.6, we get its image in the
mirror due to the reflection of light.
(a) The distance between the object and the mirror = the distance between the
image and the mirror.
O I
(b) The line joining the object point with its image is normal to the reflecting
surface.
(c) The image is laterally inverted (left–right inversion). Figure 16.7

(d) The size of the image is same as that of the object.


(e) For a real object, the image is virtual, and for a virtual object, the image is real.
(f) For a fixed incident light ray, if the mirror is rotated by an angle θ , the reflected ray turns through an angle
2 θ . If plane mirror is rotated through about an axis perpendicular to plane of mirror, then the reflected ray
image spot does not rotate.
1 6 . 4 | Geometrical Optics

Y
N N’ R’
R i-2
I
i 
I

i 
i i
X


Figure 16.8

(g) The minimum size of a plane mirror required to see the full-size image of a person by himself is half the size
of that person.

x
F
B
x
C
(x+y)

D G
y
E
Man
Figure 16.9

(h) A plane mirror behaves like a window to the virtual world.

B
Real Space Vitual Space

Figure 16.10

PLANCESS CONCEPTS

To find the location of the image of an object from an inclined plane mirror, you have to see the
perpendicular distance of that object from the mirror.
I

M O I

O
Correct Wrong
OM=Ml OM=Ml
Figure 16.11

Vaibhav Krishnan (JEE 2009 AIR 22)


P hysi cs | 16.5

Illustration 1: A point source of light S, placed at a distance L in front of the center


of a mirror of width d, hangs vertically on the wall. Assume that a man walks in
S
front of the mirror along a line parallel to the mirror at a distance 2L from it as d
shown in the Fig. 16.12. The greatest distance over which he can see the image of
the light source in the mirror is L

(A) d/2 (B) d (C) 2d (D) 3d (JEE MAIN) 2L

Sol: As the man is walking parallel to the mirror, the image of the point object S Figure 16.12
thus formed will also move relative to the man. We construct the ray diagram to
G
obtain the position of the image from the man.
C
The ray diagram is shown in the Fig. 16.13.
A H
d D
HI
= AB
= d; DS
= CD
=
2 S
E
d I
since AH =
2AD ; ∴ GH =
2CD =
2 =d B
2
F
Similarly IJ = d; ∴ GJ = GH + HI + IJ; =d + d + d = 3d J
Figure 16.13

Illustration 2: Two plane mirrors M1 and M2 are inclined at an angle θ as shown M₂ 2


in the Fig. 16.14. A ray of light 1, which is parallel to M1, strikes M2, and after two
reflections, ray 2 becomes parallel to M2. Find the angle θ .
 (JEE MAIN)

Sol: The angle of reflection is equal to angle of incidence about the normal. If ray 
M₁
makes angle α with the normal then the angle made with the surface is θ =90-α.
Completing the ray diagram for multiple reflection we get the angle θ. Figure 16.14

Different angles are as shown in the following Fig. 16.15. In triangle ABC, 
B 
θ 180°
θ + θ +=  =90°-


θ= 60°   
A
C
Figure 16.15

Illustration 3: A ray of light is travelling at an angle of 20° above the horizontal


plane. At what angle with the horizontal plane must a plane mirror be placed in its
path so that it becomes vertically upward after the reflection? (JEE MAIN) 20° 20°
Figure 16.16
Sol: The angle of incidence and reflection are similar with respect to the normal. To

make the ray reflect vertically upwards, we need to incline the mirror at an angle
 90 − i 
θ =  where I is the angle of incidence.
 2 
90 − 20
⇒ θ= = 35°
2

3.1 Velocity of Image Formed by a Plane Mirror


X OM → x coordinate of the object relative to the mirror.
1 6 . 6 | Geometrical Optics

X IM → x coordinate of the image relative to the mirror.


dx IM dxOM
Differentiating, = − ⇒ v IM =
−VOM
dt dt (XOMO) (,0) (XIMO)
⇒ Velocity of the image relative to the mirror
= - velocity of the object relative to the mirror.
Figure 16.17

Illustration 4: Find the velocity of the image when the object and mirror both are moving toward each other with
the velocities 2 m/s and 3 m/s, respectively. (JEE MAIN)

Sol: As both the object and mirrors are moving towards each other with a constant speed. The velocity of object
with respect to the mirror and velocity of image with respect to the mirror are equal in magnitude but opposite in
direction.
Here, v OM = − v IM

− ( v I − vM )
v O − vM =
⇒ ( +2m / s ) − ( −3m / s ) =− v I + ( −3 )
⇒ vI =
−8m / s

3.2 Image Formation by Multiple Reflection


Case I: When the mirrors are parallel to each other Figure 16.18 shows an image formed by an object placed at
a distance y, from M1 and at a distance x from M2. The number of images formed by the parallel plane mirrors is
infinite.
M₁ M₂
I₁’ I I₂

y y x

(2x+y) (2x+y)
Figure 16.18

Case II: When the mirrors are inclined at an angle θ. 


(a) All the images formed by the two mirrors lie on a circle with center C (an
intersection point of the two mirrors). Here, if the angle between these two 2- 
mirrors is θ , then an image will be formed on a circle at an angle 2π − θ . If
the angle θ is less, the number of images formed is high.
Figure 16.19
(b) If n is the number of image, then
If (i) 3600 / θ is even 

(ii) 3600 / θ is odd and object is kept symmetrically 
 then
= n (360 / θ) − 1
0

 3600 
and n =   for all other conditions.
 θ 
 
P hysi cs | 16.7

PLANCESS CONCEPTS

The number of images formed by two mutually perpendicular (θ=90o) M1

mirrors is three. All these three images will lie on a circle with center at
C, the point of intersection of mirrors M1 and M2, and whose radius is I1 a O
equal to the distance between C and object O. b

In fact, whatever be the angle, all the images lie on a circle. C M2

I3 I2
=90
o

Figure 16.20
Vijay Senapathi (JEE 2011 AIR 71)

4. SPHERICAL MIRRORS
A spherical mirror is a smooth reflecting surface that forms a part of a spherical surface. If reflection takes place
from the inner reflecting surface, then the mirror is called a concave mirror. If the reflection takes place from the
outer surface, it is called a convex mirror. The reflection of a light from a concave and a convex mirror is shown in
the Fig. 16.21.

i Light
r
C F C
F P
P

M’ M’
Concave Mirror Convex Mirror

Figure 16.21

4.1 Parameters Associated with the Spherical Mirror


A pole (P) or vertex is the geometrical center of a reflecting surface. Center of curvature C is the center of the
sphere, of which the mirror is a part. Radius of curvature R is equal to the distance between P and C of the mirror
and is the radius of the sphere, of which the mirror is a part. The principal axis is the line CP that passes through P
and C. If a ray of light is emitted from an object at infinity so that the beam of light is parallel to the principal axis,
an image is formed at principal focus F after reflection. Focal length, ƒ , is the distance between P and F along the
principal axis. When a beam of light is incident parallel to the principal axis, the reflected rays converge on F in a
concave mirror and diverge from F in a convex mirror after reflection. Aperture of a spherical mirror is the effective
diameter MM’ of the light-reflecting area in the mirror. When the aperture of a mirror is small, the focal length is
equal to half the radius of curvature.

4.2 Sign Convention


The following sign conventions based on coordinate geometry are used:
1 6 . 8 | Geometrical Optics

(a) The rays of light travel from the left to the right direction.
+y
(b) All the distances measured from the pole and in the direction of light 
toward the right of the pole are positive. The distances measured in the Light
opposite direction of light toward the left of the pole are negative.
C F P
(c) The transverse distances, above the principal axis, are positive and the +x
principal axis are negative.
Negative Positive
(d) If condition (1) in sign conventions is followed, this sign convention
follows the right-hand Cartesian coordinate system. -y

4.3 Rules for Image Formation Figure 16.22

The following rules are used for locating the image of an object by considering the reflection of three types of rays
based on laws of reflection:

(a) A ray incident parallel to the principal axis will pass through the principal focus after reflection in the case of
a concave mirror and will appear to originate from the focus in the case of a convex mirror.

(b) A ray that passes through the principal focus of a concave mirror, or that passes toward the principal focus of
a convex mirror, travels parallel to the principal axis after reflection.

(c) A ray that passes through the center of curvature of a concave mirror or toward the center of curvature of a
convex mirror is reflected from the mirror along the same path.

4.4 Image Formation by a Concave Mirror (for Real Object)

S. No. Position of Object Diagram Position of Image Nature of Image

F At the principal
focus F or in the Real, inverted and
1 Infinity focal plane diminished
Image problem
F

Figure 16.23

Between infinity C Real, inverted and


2 Between F and C
and C F diminished

Figure 16.24
P hysi cs | 16.9

S. No. Position of Object Diagram Position of Image Nature of Image

C F Real, inverted and


3 C C of same size as the
object.

Figure 16.25

Between C and
infinity Real, inverted and
4 Between F and C C F
magnified

Figure 16.26

At F or in the focal Real, inverted and


5 At infinity
plane highly magnified
F

Figure 16.27

Virtual, erect and


6 Between F and P F P Behind the mirror
magnified.

Figure 16.28

4.5 Image Formation by a Convex Mirror


An image is formed between the pole and the focus for all the 
positions of the real object except when the objects are at infinity in Light
which case the image is formed at F in the focal plane. The image
formed is virtual, erect and diminished. The ray diagram for the
formation of image I of object O after reflection from a convex mirror O P I F C
is shown in the Fig. 16.29. u v

Figure 16.29
PLANCESS CONCEPTS

Image formed by a convex mirror is always virtual, erect and diminished; no matter where the object is
placed (except for virtual objects).
Anurag Saraf (JEE 2011 AIR 226)
1 6 . 1 0 | Geometrical Optics

4.6 Mirror Formula


If an object is placed at a distance u from the pole of a mirror, its image is formed at a distance v from the pole, and
1 1 1
its focal length f is given by + = where f = R/2 (only for paraxial rays).
u v ƒ
According to the sign conventions, f and R are negative for a concave mirror and are positive for a convex mirror.
The power of a mirror, P, measured in units of dioptres is given by

1 100
P=
− Where ƒ is in metres =
− Where ƒ is in centimetres
ƒ ƒ

Illustration 5: A convex mirror has a radius of curvature of 20 cm. Find the position of the image of an object
placed at a distance of 12 cm from the mirror. (JEE MAIN)
1 1 2
Sol: The position of the image is found using formula + =. 
u v R
According to sign convention, if the object is placed to the left side of

the pole, object distance is considered to be negative.


The situation is shown in the Fig. 16.30. Here, u = −12 cm and O I C
R = +20 cm.

1 1 2 1 2 1 2 1 11
We have, + = or = − = − =
u v R v R u 20 cm −12cm 60 cm
Figure 16.30
60
⇒ v = cm .
11
The positive sign of v shows that the image is formed on the right side of the mirror and is a virtual image.

4.7 Magnification by Mirror


If an object of linear size O is placed vertically on the axis of a concave or convex mirror at a distance u from the
pole and its image of size I is formed at a distance v from the pole, then the lateral or transverse magnification, m,
I v
is given by m= = − 
0 u
A negative magnification indicates that the image is inverted with respect to the object, whereas a positive
magnification implies that the image is erect with respect to the object.

Illustration 6: A concave and a convex mirror of focal lengths 10 cm 


and 15 cm, respectively, are placed at a distance of 70 cm from each B
other. An object AB of height 2 cm is placed at a distance of 30 cm from 20cm
the concave mirror. First, a ray is incident on a concave mirror and then
on the convex mirror. Find the size, position and nature of the image.
30cm
 (JEE MAIN)
70cm
1 1 1
Sol: The position of the image is found using formula + =.
u v f Figure 16.31
The image formed by concave mirror is the object for the convex mirror.
For a concave mirror, u=
−30cm, f =
−10cm
P hysi cs | 16.11

1 1 1 1 1 −1 B
Using + = ⇒ − = ; ⇒ v=
−15cm
v u f v 30 10 A’
A'B' − v −15 A
Now, = = ; ⇒ A'B' =
−1cm
AB u −30 B’
An image formed by the first reflection will be real, inverted and diminished.
Figure 16.32
For a convex mirror,
1 1 1
u' = +15cm Using
−55cm, f = + =
v ' u' f '
55cm
1 1 1 A’ A’’
⇒ − = ⇒ v' =165 / 14cm
v ' 55 15
 165  B’ B’’
 
A"B" v' 14   3 
Now, =− = − ⇒ A"B" =   ( −1 ) =−0.2cm Figure 16.33
A'B' u' ( )
−55  14 
The final image will be virtual and diminished.

E
Illustration 7: An object ABED is placed in front of a concave B
mirror beyond the center of curvature C as shown in the Fig.
16.34. State the shape of the image.  (JEE ADVANCE) A D C F
P

Sol: As the object is placed beyond center of the curvature,


the image thus formed will lie between center of curvature
and pole. The position of the image is found using formula Figure 16.34
1 1 1
+ =
u v f
The object is placed beyond C. Hence, the image will be
real and it will lie between C and F. Furthermore, u, v and D’ A’
P
f all are negative; hence, the mirror formula will become C F
B’
1 1 1 1 1 1 u− ƒ ƒ E’
− − =− or =− = or v=
v u ƒ v ƒ u uƒ ƒ
1− .
u
Now uAB > uED ; ∴ v AB < vED VAB
 −v  VED
and mAB < mED  as m =  .
 u  Figure 16.35
Therefore, the shape of the image will be as shown in the
Fig. 16.39. Also note that

v AB < uAB and vED < uED , So, mAB < 1 and mED < 1 .

4.8 Relation between Object and Image Velocity


1 1 1 .
Differentiating equation + =
v u ƒ
1dv1du 1 1
⇒ − − = 0 ⇒ − V − V= 0 VIM ⇒ velocity of image w.r.t. mirror
2 IM OM
v dt u2dt
2
v u2
v2
⇒ VIM = − VOM VOM ⇒ velocity of object w.r.t. mirror
u2
⇒ VIM = −m2 VOM
1 6 . 1 2 | Geometrical Optics

Illustration 8: A mirror with a radius of curvature of 20 cm and an object that is placed at a distance 15 cm from
the mirror both are moving with the velocities 1 m/s and 10 m/s as shown in the Fig. 16.36. Find the velocity of the
image. (JEE MAIN)
Sol: As the object and the mirror are moving away from each other with different speed, the magnification of the
v2
image will also change. The velocity of image will be Vim = − Vom .
u2 -1
1 1 2 1ms
The position of the image is found using formula + = .
u v R 10ms
-1

1 1 2 1 1 1
Using + = ⇒ − =− ⇒ v=−30cm
v u R v 15 10 o
15ms
2 2
v v
Now, using Vim =
− Vom ⇒ ( Vi − Vm ) =
− ( V0 − Vm )
u2 u 2
Figure 16.36

( −30 )  −10 − −1 
2

⇒ V
=i − (1 ) 2 
( ) ( ) 45 cm / s .
⇒ Vi =
( −15)
So the image will move with the velocity of 45 cm/s.

Illustration 9: A gun of mass m1 fires a bullet of mass m2 with a horizontal speed v0. The gun is fitted with a concave
mirror of focal length f facing toward a receding bullet. Find the speed of separations of the bullet and the image
just after the gun was fired. (JEE ADVANCED)
Sol: The bullet when leave the gun it moves in direction opposite the motion of gun. As there are no external
forces acting on the bullet, then the momentum of the system can be conserved. The velocity of image will be
v2
Vim = − Vom .
u2
Let v1 be the speed of the gun (or the mirror) just after the firing of bullet. From conservation of linear momentum,
m 2 v0 = m 1 v1 ⇒ v = m2 v 0  … (i)
1
m1
du
m1
Now, is the rate at which the distance between mirror and bullet is increasing
dt
= v +v 
1 0 … (ii) V0 V1

dv  v 2  du m2
We know that ∴ =   .
dt  u2  dt
v2 2 Figure 16.37
Here, = m = 1 (as at the time of firing bullet is at pole).
u2
dv du
∴ = v1 + v 0 .
= … (iii)
dt dt
dv
Here, is the rate at which the distance between the image (of bullet) and the mirror is increasing. Hence, if v 2
dt
is the absolute velocity of image (toward right), then
dv
v 2 − v1 = = v1 + v 0 ⇒ v 2 2v1 + v 0 
= … (iv)
dt
Therefore, the speed of separation of the bullet and the image will be
vr = v 2 + v 0 = 2v1 + v 0 + v 0
v r 2 ( v1 + v 0 ) .
=
 m 
Substituting the value of v1 from Eq. (i), we have v r 2  1 + 2  v 0 .
=
 m1 
P hysi cs | 16.13

5. REFRACTION OF LIGHT AND LAWS OF REFRACTION


(a) The deviation or bending of light rays from their original path while travelling from one medium to another
is called refraction.
(b) If the refracted ray bends away from the normal, then the second medium is said to be RARER as compared
to the first medium, and the speed increases.

N N
A A

i Rarer i Denser
B B
S S’ S r S’
r Denser C
Rarer
N’ C N’

Figure 16.38

If the refracted ray bends toward the normal, then the second medium is said to be DENSER compared to the first,
and the speed decreases.
Deviation due to refraction δ = i−r .

PLANCESS CONCEPTS

In general, light will travel in straight lines and the deviation occurs only when there is a change of
medium (or refractive index (RI)).
B Rajiv Reddy (JEE 2012 AIR 11)

5.1 Laws of Refraction


The incident ray (AB), the normal (NN’) to the refracting surface (SS’) at the point of incidence (B), and the refracted
ray (BC) all lie in the same plane called the plane of incidence or the plane of refraction.

5.2 Snell’s Law


For any two given media and for light of given wavelength, i n1,v1111

r n2,v2222
sin i sin i 1 µ v λ
= cons tant ; = µ2 = 2 = 1 = 1 .
sin r sin r µ1 v 2 λ2
Figure 16.39
1
µ2 = RI of the second medium with respect to the first medium;

µ1 = RI of the first medium with respect to air or absolute

RI = c v1 ;

µ2 = RI of the second medium with respect to air or absolute RI = c v 2 ;

v1 , v 2 are the speeds of light in the first and the second medium, respectively;

λ1 , λ2 are the wavelengths of light in the first and the second medium, respectively;
c=the speed of light in air (or vacuum) = 3x108 m/s.
1 6 . 1 4 | Geometrical Optics

Note:
(i) The higher the value of RI, the denser (optically) the medium is.
(ii) The frequency of light does not change during refraction.
(iii) The refractive index of the medium relative to air = µr εr .

5.3 Refraction through a Transparent Sheet

Let the ray is incident at face AB. 1 i1


x
Apply the Snell’s law at faces AB and CD. A B
µ2 sini2 
µ1 sini1 = … (i)
µ3 sini3 
µ2 sini2 = … (ii) i2
2
(The angle of incidence for face CD is i2 )
From Eqs (i) & (ii), µ1 sini1 =
µ3 sini3
µ1
or sini3 = sini1  … (iii) C D
µ3 3 i3

Figure 16.40

The incident ray and emerging ray are parallel. It shows that the deviation of ray is not affected by the refractive
t sin ( i1 − i2 )
index of the sheet; it depends by µ1 and µ3 . µ2 only causes lateral displacement which is given by x = .
cosi2

At a glance:

i1 i1
1 B Rarer 1 B Denser
2 Denser 2 Rarer
i2

i1>i2 i1>i2
V2<V1 V2<V1
2<1 2<1
2<1 2<1

Figure 16.41
P hysi cs | 16.15

PLANCESS CONCEPTS

In general, the speed of light in any medium is less than that in vacuum. The refractive index µ of a
medium is defined as,
Speed of light in vaccum c
µ = .
Speed of light in medium v

Anand K (JEE 2011 AIR 47)

Illustration 10: A light beam passes from medium 1 to medium 2. Show that the emerging beam is parallel to the
incident beam. (JEE MAIN)
Sol: When a ray of light enters from one medium to other medium of different refractive index, then according
µ sini
to Snell’s law we get 1 = where i is the angle of incidence and r is the angle of refraction. The first refracted
µ2 sinr
ray is incident on other surface. So the angle o incidence on the second surface is equal to the angle of refraction
from first surface.
Applying the Snell’s law at A and B, µ1 sini1 =
µ2 sini2

µ1 sini2 i3
Or = … (i)
µ2 sini1 i2
i2
Similarly, µ2 sini2 =
µ1 sini3
i1
µ1 sini2
∴ = … (ii)
µ2 sini3
1 2 1
From Eqs (i) and (ii) i3 = i1 ; i.e. the emergent ray is parallel to the incident ray.
Figure 16.42

Illustration 11: The refractive index of glass with respect to water is (9/8) and the refractive index of glass with
respect to air is (3/2). Find the refractive index of water with respect to air. (JEE MAIN)
µ2
Sol: The refractive index of medium one with respect to medium two is given by µ 2 = .
1 µ1

To find the refractive index of water with respect to air, we need to obtain the ratio between w µg and aµg

Given=
w µg and aµg 3 / 2 .
9/8=

As a µg × gµ w × w µa =1

1 a µg
∴ =aµ w =aµg ×g µ w =
w µa w µg
3/2 4
∴ a µw
= =
9/8 3
1 6 . 1 6 | Geometrical Optics

Illustration 12: (i) Find the speed of light of wavelength l = 780 nm (in air) in a medium of refractive index µ = 1.55.
(ii) What is the wavelength of this light in the given medium? (JEE MAIN)
c
Sol: In a medium of refractive index µ the velocity of wave is given by v = and the wavelength of wave is given
µ
λair
by λmedium =
µ

c 3.0 x108
(i) v= = = 1.94 x108 m / s
µ 1.55

λ 780
(ii) λmedium = air = =503nm .
µ 1.55

5.4 Image Due to Refraction at a Plane Surface


Consider the situation given in the Fig. 16.43. A point object O is placed in a medium of refractive index µ1 . An
another medium of refractive index µ2 has its boundary at PA. Consider two rays OP and OA originating from
O. Let OP fall perpendicularly on PA and OA fall on PA at a small angle i with the normal. OP enters the second
medium without deviating, and OA enters by making an angle r with the normal. When produced backward, these
rays meet at I that is the virtual image of O. If i and r are small,
PA
sin
=i tan
=i ;
PO
I
PA
and sin
= r tan
= r .
PI
O O r
µ2 sini
Thus, =
µ1 sinr i i
I
i i
 PA   PI  PI
= = .  . ... (i) P r
A 1 P 1
 PO   PA  PO A
2 2
Suppose medium 2 is air and an r
observer looks at the image from this r
medium. The real depth of the object
inside medium 1 is PO, whereas the
depth as it appears to the observer is (a) (b)
PI. Writing µ2 =1 and µ1 =µ , Eq. (i)
gives, Figure 16.43

1 apparent depth real depth


= or, µ= .
µ real depth apparent depth

The image shifts closer to the observer’s eye by an amount

 PO − PI   PI   1
= PO − PI = 
OI  PO
= 1 −  PO or, ∆t =  1 −  t, where t is the thickness of the medium over
 PO   PO   µ 

the object, and ∆t is the apparent shift in its position toward the observer. Note that ∆t is positive in Fig16.43 (a)
and negative in Fig. 16.43 (b).
P hysi cs | 16.17

Different Scenarios at a Glance:

O
D x
C x
Air i
Air
Medium A B
i r A Medium
I
r

O
(a) ( b)

Figure 16.44

Illustration 13: A printed page is kept pressed by a glass cube ( µ =1.5 ) of edge 6.0 cm. By what amount will the
printed letters appear to be shifted when viewed from the top? (JEE MAIN)

Sol: As the glass of thickness t = 6 cm is kept on the page, the image of letter appears to be shifted closer to eye
 1
by ∆t =  1 −  t
 µ 
The thickness of the cube = t = 6.0 cm. The shift in its position of the printed letters is
 1  1 
∆t =  1 −  × t =  1 −  x6.0 cm = 2.0 cm .
 µ   1.5 

5.5 Shift Due to a Glass Slab (Double Refraction from Plane Surfaces)
(a) Normal Shift:

Refer Fig 16.45: An object is placed at O. Plane surface CD forms  C E


its image (virtual) at I1. This image acts as an object for EF which initially
forms the image (virtual) at I. Distance OI is called the normal shift, and
its value is N
M
 1  µ −1
OI
= 1 −  t ⇒  t
 µ   µ  I₁ O I A B

Proof: Let OA = x

AI1 = µx (Refraction from CD) 


BI1 =µx + t D F
BI1 t t
BI= = x+ (Refraction from EF) (a)
µ µ Figure 16.45
 t  1
∴ OI = (AB + OA) − BI =(t + x) −  x +  = 1 −  t .
 µ  µ
1 6 . 1 8 | Geometrical Optics

(b) Lateral Shift: We have already discussed that ray MA is parallel


to ray BN, but the emergent ray is displaced laterally by a  N
distance d, which depends on µ , t and i, and its value is given
by the relation
i
  r B
cos i
d t 1 −
=  sin i . i-r
 2 2  A r
 µ − sin i  i
C

d
Proof: D
AC t
AB
= =
cos r cos r
( as=
AC t ) .
M

t
Now, =d AB sin ( i=
− r) sin i cos r − cos i sin r 
cos r  t
Figure 16.46
or =d t sin i − cos i tan r  ... (i)

sin i sin i
Further µ
= = or sin r
sir r µ
sini
∴ tanr =
µ − sin2 i
2

 
cos i
d t 1 −
Substituting in Eq. (i), we get=  sin i .
 µ 2
− sin2 
i
 

6cm
Illustration 14: A point object O is placed in front of a concave
mirror of focal length 10 cm. A glass slab with a refractive
index of µ =3 / 2 and thickness of 6 cm is inserted between an
object and a mirror. Find the position of the final image when
the distance x shown in the Fig. 16.47 is: (a) 5cm (b) 20 cm
 (JEE MAIN)

Sol: As the glass slab of thickness t = 10 cm is kept in front


of the mirror, the image of object from the slab appears to be
X
 1
shifted closer to mirror by ∆t =  1 −  t . The position of the 32 cm
 µ
1 1 1 Figure 16.47
image is given by + = where u is the distance of object,
v u ƒ
f is the focal length of the mirror and v is the distance of the
image from mirror.
 1  2
As we know that the normal shift produced by a glass slab is ∆x =  1 −  t =  1 −  (6) = 2cm ;
 µ  3
i.e. for the mirror, the object is placed at a distance (32 − ∆x) =30cm from it. Now apply the mirror formula,
1 1 1 1 1 1
+ = ⇒ − =⇒ v = −15cm .
v u ƒ v 30 10

(i) When x = 5cm: The light falls on the slab on its return path as shown in the Fig. 16.48, but the slab will again
shift it by a distance of ∆x =2cm . Hence, the final real image is formed at a distance of (15+2) =17 cm from the
mirror.
P hysi cs | 16.19

15 cm

I
x
x

15 cm
Figure 16.48

(ii) When x = 20 cm: The final image is at a distance of 17 cm from the mirror in this case also, but it is virtual.

6. CRITICAL ANGLE AND TOTAL INTERNAL REFLECTION


When a ray of light passes from an optically denser medium (a medium with larger µ ) to an optically rarer medium
sin i µ
(a medium with smaller µ ), we have = 2 < 1 .
sin r µ1
When we gradually increase the value of i, the corresponding r value also increases, and at a certain point, r
becomes 90°. Let the angle of incidence for this case be θc called the critical angle of the given pair of media. If i
is increased further, there is no r that can satisfy the Snell’s law. Then, all the light waves are reflected back into the
first medium. This is called total internal reflection (TIR). Generally, the critical angle of a medium is quoted for light
travelling from the denser to the rarer medium. In this case, µ2 = 1.
sin θc 1
When substituting µ1 =µ , then the Snell’s law becomes 0
=
sin90 µ
or sin θc= (1 / µ ) or =θc sin−1 (1 / µ ) .

Illustration 16: A point source of light is placed at the bottom of a tank filled with water up to the level of 80 cm.
Find the area of the surface of water through which light from the source emerges out. Assume that the refractive
index is equal to 1.33. (JEE ADVANCED)

Sol: When light travels from a denser medium (water) to the rarer medium (air) for angle of incidence i greater than
1
critical angle, than by Snell’s law we get sin C = .
µ
Q P R
Let the light emerges out of a circular area of radius r as shown in the figure. 90
o

1
Step 1. Using sin C = , We get
n
1
sin
= C= 0.7513
1.33
1
=
∴ C sin–1−(0.7519)
∴ C = sin = 48.750
(0.7519)= 48.75°

Step 2. From ∆POR,


PR r O (Source of light)
tan=
C =
OP 80
0 Figure 16.49
∴ =r 80tanC
∴ r= 80 tanC
= 80 tan(48.75=
= 80tan(48.75°) )80 ×80
= x1.14
1.14 = 91.2cm.
= 91.2cm

3.14 x ( 91.2 ) =
2
πr 2 =
Step 3. ∴ Area through which light emerges out = 26116.76 cm2 =2.6m2 .
1 6 . 2 0 | Geometrical Optics

Illustration 17: The critical angle for water is 48.2°. Find its refractive index. (JEE MAIN)
1
Sol: By Snell’s law we get µ = .


sin C Denser
1 1 i> C O
=µ = = 1.34 . Rarer
sin θc sin48.2°
Day-to-day life: Due to heating of the earth, the refractive
index of air near the surface of the earth is lesser than
that above the earth. Light waves from a distant object
reach the surface of earth at an angle of i > θc , so that the
TIR will take place, and the image of an object creates an Mirage
illusion of water near the object. Figure 16.50

7. REFRACTION AT THE SPHERICAL SURFACES

Refraction at a single spherical surface


If the boundary between two transparent media is curved either as a convex or as a concave spherical surface, an
object O in a medium of refractive index µ1 forms an image I in a medium of refractive index µ2 as shown in the
Fig. 16.51. When the ray from O is incident at an angle i on the medium of refractive index µ1 , the refracted ray
forms an image I after refraction in the medium of refractive index µ2 at angle r according to the Snell’s law.
µ2 sin r .
µ1 sini =

N
r
2
i 
1 1
  
O P C I I C P

u  V
u

Figure 16.51

µ2 µ1µ2 − µ1
The relation between u=OP and v=IP is given by − = .
v u R

PLANCESS CONCEPTS

The relation is valid for a single spherical surface or plane refracting surfaces, and the sign convention
for the spherical mirrors and spherical refracting surfaces are the same.
µ2 µ1µ2 − µ1
The refraction formula − = can also be applied to plane refraction surfaces with R = ∞.
v u R
dactual
Let us derive dapp = using this.
µ
µ2 µ1µ2 − µ1
Applying − = with proper sign and values, we get
v u R
P hysi cs | 16.21

PLANCESS CONCEPTS

1 µ 1−µ d
− = = 0 or v =

v −d ∞ µ R= 2
+ve
d 1
i.e. the image of object O is formed at a distance on the
same side. µ
d

dactual
or dapp = .
µ
O
Figure 16.52

Anurag Saraf (JEE 2011 AIR 226)

Illustration 18: A sunshine recorder globe of 30 cm diameter is made of glass of refractive index n = 1.5. A ray of
light enters the globe parallel to the axis. Find the position from the center of the sphere where the ray crosses the
principle axis. (JEE ADVANCED)

Sol: As the light enters from air to the globe of refractive index n than for the refraction at the surface we have
µ1 µ2µ2 − µ1
relation + = .
u v R
First refraction (from the rarer to the denser medium): Here, u = −∞ , n2 = 1.5, R = +15cm .
n2 n1n2 − n1
Using the relation, + =
v u R 
n2  n2 − n1  n1
i.e. =   +
v  R  u P₁ C P₂ I’ I₁
R
1.5 1.5 − 1 1 1 u’
⇒ = + = ;
v 15 ( −∞ ) 30 V
⇒ v = 45cm . Figure 16.53

Second refraction (from the denser to the rarer medium):


Here, R=-15cm, u’= (45-30) =15cm.
n1 n2 n1 − n2
Using the relation, − =
v' u' R
1 1.5 1 − 1.5 1 30
i.e. − = = ⇒ v'
= = 7.5cm .
v' 5 −15 30 7

∴ Distance at which the image is formed from the center of the globe is (15+7.5) =22.5 cm.

Illustration 19: Locate the image of a point object O in the situation shown in the figure. Point C denotes the
center of curvature of the separating surface. (JEE ADVANCED)
Sol: According to the sign convention as the object is placed left to the pole, the distance of it is considered to be
µ2 µ1µ2 − µ1
negative. Using the formula − = we get the distance of image from the centre of curvature.
v u R
1 6 . 2 2 | Geometrical Optics

Here, u=-15cm, R=30cm, µ1 =1 and µ2 =1.5 We have


 = 1.0  = 1.5
µ2 µ1
µ2 − µ1
− =
v u R C
O
1.5 1.0 1.5 − 1
⇒ − = 15 cm 30 cm
v −15cm 30cm
1.5 0.5 1
⇒ = − ; ⇒ v=-30cm
v 30 cm 15 cm
The image is formed 30 cm left to the spherical surface and is Figure 16.54
virtual.

Illustration 20: A glass sphere of radius R=10 cm is kept


inside water. A point object O is placed at 20 cm from A as
shown in the figure. Find the position and nature of the image C
when seen from other side of the sphere and also draw the A B
O
ray diagram. Assume µg=3/2 and µw= 4/3. 10 cm
 (JEE ADVANCED)
20 cm
Sol: As the light passes through medium of different
Figure 16.55
refractive indices at each refraction the position of image
µ2 µ1µ2 − µ1
of point object is found by − = .
v u R
A ray of light from object O gets refracted twice. The
P M
direction of this light ray is from the left to right. Hence, the
2 1 2
distances measured in this direction are positive. 1
A B
µ2 µ1
µ2 − µ1
When applying − = twice with proper signs, O
v u R

we have 3 / 2 − 4 / 3 =
3/2−4 /3
AI1 −20 10
+ve
or AI1 = −30cm .

Now, the first image I1 acts an object for the second surface, Figure 16.56
where
BI1 =
u= −50cm .
−(30 + 20) =
4 /3 3/2 4 /3−3/2 .
∴ − = N
BI2 −50 −10 M
P
−100 cm, i.e. the final image I2 is virtual
∴ BI2 =
and is formed at a distance of 100 cm (toward
left) from B. The ray diagram is shown in the I2 I1 O A C B
figure.
20 cm
The following points should be taken into
30 cm
account while drawing the ray diagram.
100 cm
(i) At P, the ray travels from a rarer to a denser
medium. Hence, it will bend toward normal PC. Figure 16.57
At M, it travels from a denser to a rarer medium
and hence moves away from normal MC.
(ii) PM ray when extended backward meets at I1
, and MN ray when extended meets at I2 .
P hysi cs | 16.23

7.1 Lateral Magnification


The lateral magnification may be obtained with the help of the 1 2
adjacent Fig. 16.58, where two rays from the tip of an object
of height h0 meet at the corresponding point on an image of ho
1 C I
height h1. One ray passes through the center of curvature of O M 1 hI
the spherical surface so that its direction is unchanged. The
path of the second ray is obtained from the Snell’s law. With
the paraxial approximation,
u v
h0 hi
sin θ1 ≈ and sin θ2 ≈ .
u v Figure 16.58
Combining these equations with the Snell’s law, then,
h  h  h  µ  v 
µ1  0  =
µ2  i  or i =  1    .
u v h0  µ2   u 
hi
The lateral magnification m is the ratio of the image height to the object height or .
h0
 µ1   v  hi  µ1   v 
We, therefore, obtain     i.e. =   
 µ2   u  ho  µ2   u 

PLANCESS CONCEPTS

Here, v is +ve, u is −ve, hi is −ve, h0 is +ve (the distances measured above the axis are positive). So, if we
µ v
put these sign conventions, in Eq. (vi), we obtain the same result m = 1 .
µ2 u
Chinmay S Purandare (JEE 2012 AIR 698)

Illustration 21: Find the size of the image formed in the


situation shown in the figure.  (JEE MAIN) =1  = 1.33
Sol: Here as the object is placed left to the pole beyond 1.0 cm C
center of the curvature, then according to sign convention
O
the distance of object is considered to be negative. 20 cm
µ2
µ2 − µ1 µ1
The distance of image is found by − =
v u R 40 cm
Here, u=−40cm, R=−20cm, µ1 = 1, µ2 = 1.33.
Figure 16.59
µ2 µ1
µ2 − µ1
We have − =
v u R
1.33 1 1.33 − 1 1.33 1 0.33
⇒ − = ⇒ =
− −
v −40cm −20cm v 40cm 20cm

⇒ v = −32cm.

h2 µ1 v h2 −32cm
The magnification is =
m = or = + 6.0 cm. .
⇒ h2 =
h1 µ2u 1.0cm 1.33x( −40cm)
The image is erect.
1 6 . 2 4 | Geometrical Optics

7.2 Refraction through Lenses


A lens is made of a transparent material with two refracting surfaces such that at least one of these is a curved one.
The convex lens is thicker in the middle, and the concave lens is thinner in the middle.
The plano-convex and plano-concave lenses have one plane surface, and the other surfaces are convex and
concave, respectively. Different types of typical lenses are shown in the Fig. 16.60.

Convex Concave Plano Plano Convexo


Convex Concave Concave

Figure 16.60

7.2.1 Parameters Associated with Lens


The center of curvature C is the center of the sphere, of which the curved surface of the lens is a part. The radius of
curvature of either surface of the lens is the radius of the sphere, of which the curved surface is a part.


C1 O
F
C2 O
C1 C2

Figure 16.61

The optical center, O, of a lens is a point through which the ray does not get deviated. The principal axis is a line
passing through the center (s) of curvature and the optical center. If the object is at infinity, the image is formed at
the principal focus and vice versa. The focal length is the distance between the optical center of a lens and a point
on which a parallel beam of light converges or appears to converge. The aperture of a lens is the effective diameter
of its light-transmitting area. The intensity of the image is directly proportional to the square of the aperture.

7.2.2 Rules of Image Formation


(a) A ray that passes through the optical center does not get deviated through the lens.
(b) A ray incident parallel to the principal axis after refraction through the lens either passes through the focus or
appears to pass through the focus after extrapolation.
(c) A ray that passes through the focus or directed toward it becomes parallel to the principal axis after refraction.
The rays of light from a point of object intersect or appear to intersect after refraction through the lens and
form an image. If the rays actually intersect, then the image is real, and if the rays appear to intersect, then
the image is virtual.
P hysi cs | 16.25

7.3 Image Formation by Convex Lens


S. No Position of object Diagram Position of Nature of image
image

Real, inverted and


1 Infinity F Principal focus diminished

Figure 16.62

F Real, inverted and


Between 2F and ∞
2 2F Between F and 2F diminished
2F F

Figure 16.63

Real, inverted and


F
of same size as
3 2F 2F 2F 2F
2F F the object

Figure 16.64

F Real, inverted and


4 Between F and 2F 2F F 2F Beyond 2F magnified

Figure 16.65

Real, inverted and


5 F F Infinity highly magnified

Figure 16.66

Virtual, erect and


Between F and Same side as the magnified
6 F F
optical center optical center

Figure 16.67

A lens has two foci, which is not a case in a mirror.


1 6 . 2 6 | Geometrical Optics

First focus (F1): If an object (real in case of a convex, virtual for concave) is placed at the first focus (F1), the image
of this object is formed at infinity, or we can say through F1 it becomes parallel to the principal axis after refraction
from the lens. The distance from the first focal length is ƒ1 .

Incident light

P P F1
F1

+ve
First focus

Second focus
P F2 F2 P

Figure 16.68

Second focus or principal focus ( F2 ): A narrow beam of light that travels parallel to the principal axis either
converge (in case of a convex lens) or diverge (in case of a concave lens) at a refraction (r) from the lens. This point
F2 is called the second or principal focus. If the rays converge, the lens is a converging lens, and if the rays diverge,
then the lens is a diverging lens. It can be seen from the Fig. 16.68 that ƒ1 is negative for a convex lens and positive
for a concave lens. But ƒ 2 is positive for a convex lens and negative for a concave lens.

ƒ1 =ƒ 2 if the media on the two sides of a thin lens have the same refractive index.

We mainly concern with the second focus ƒ 2 . Thus, wherever we write the focal length ƒ, it means the second or
principal focal length. Therefore, ƒ = ƒ 2 and, hence, ƒ is positive for a convex lens and negative for a concave lens.

7.4 Image Formation by a Concave Lens


An image I formed by a concave lens of a real object O
occurred beyond F and the optical center is virtual, erect and
diminished. The image is formed at F if the object is at infinity. O F I

7.5 Lens Formulae Figure 16.69

If an object is in a medium of refractive index µ1 at a distance u from the optical center of a lens having radii of
curvature R1 and R2 and of refractive index µ2 , its image is formed at a distance v from the optical center, then

1 1  µ2  1 1 
− =  − 1 
 −  .
v u  µ1  R1 R 2 

1 1 1  µ2  1 1 
If is the focal length of the lens, then = − =  − 1 
 −  .
ƒ v u  µ1  R1 R 2 
P hysi cs | 16.27

•• If µ is the refractive index of the material the lens is made of with respect to the surrounding medium air,
1 1 1  1 1 
then = − = ( µ − 1 )  −  .
ƒ v u  R1 R 2 
•• If I and O are the lateral or transverse size of the image and object, respectively, the magnification m is given
I v
by, m = = .
O u
1 100
•• The power P of the lens is given by=
P = dioptre.
ƒ(m) ƒ(cm)

PLANCESS CONCEPTS

Suppose m is positive, it implies Incident light


R1>0
v and u are of same sign, i.e. the
object and its image are on the R2 R2>0
same side (left side), which implies C2 C1
that the image of a real object is
virtual. Thus, m=+2; it implies R1
that the image is virtual, erect and
magnified to two times the actual
(a)
size, and v = 2 u . Similarly,
Incident light
1 R1>0
m = − implies that the image is
2 R1 R2>0
real, inverted and diminished, and C2 C1
1
v = u.
2 R2
For a converging lens, R1 is

positive and R 2 is negative. (b)


 1 1 
Therefore,  −  is positive, Figure 16.70
 R1 R 2 
and if the lens is placed in air,

the value of ( µ − 1 ) is also positive. Hence, the focal length ƒ of


a converging lens is positive. For a diverging lens, however, R1 is
negative and R 2 is positive at the focal length ƒ .

 R
The focal length of a mirror  ƒM =  depends only on the radius of
 2
curvature R, while that of a lens depends on µ1 , µ2 ,R1 and R 2 . Thus,
when a lens and a mirror both are immersed in a liquid, the focal
length lens changes, whereas that of the mirror remains unchanged.
Suppose µ2 < µ1 , i.e. the refractive index of the medium (in which
the lens is placed) is more than the refractive index of the material Figure 16.71
 µ2 
the lens is made of, then  − 1  becomes negative, i.e. the lens’
 µ1 
behavior changes. A converging lens behaves as a diverging lens and vice versa. An air bubble in water

seems to be a convex lens, but behaves as a concave (diverging) lens.


Yashwanth Sandupatla (JEE 2012 AIR 821)
1 6 . 2 8 | Geometrical Optics

Illustration 22: The focal length of a convex lens in air is 10 cm. Find its focal length in water. Assume
=µg 3 / 2 and
= µ w 4 / 3  (JEE MAIN)

 1 1 
Sol: The focal length of lens is given by
1 1 1
= − =
f v u
( 1µ
2
)
− 1  −  .
 R1 R 2 
1  

ƒ air
(
= µg − 1 )  R1 − R1   ... (i)
 1 2 

1  µg  1 1 
and = − 1   −  ... (ii)
ƒ water  µw   R1 R 2 
 

Dividing Eq. (i) by Eq. (ii), we get


ƒ water
=
(µ g −1 )
ƒ air (µ g / µw − 1 )
Substituting the values, we get ƒ water =
(3 / 2 − 1) ƒ ; 4 air =4x10 =40cm .

3/2  air
 − 1
4/3 

O
Illustration 23: An image I of point object O is formed by a lens whose
optical axis is AB as shown in the Fig. 16.72.
(a) State whether it is a convex or a concave lens? A B
(b) Draw a ray diagram to locate the lens and its focus. i
Figure 16.72
 (JEE ADVANCED)

Sol: For the convex lens, the image of the object is real, formed on the opposite site and is always inverted while
for concave lens the image of an object is always virtual, erect and on the same side of object.
(a) (i) A concave lens always forms an erect image. The given
image I is on the other side of the optical axis. Hence, the lens O M
is convex. F
A
(ii) Join O with I. Line OI cuts the optical axis AB at pole P of the
B
P
lens. The dotted line shows the position of the lens.
I
Then, draw a line parallel to AB from point O. It cuts the dotted line
at M and join M with I. Line MI cuts the optical axis at focus (F) of
Figure 16.73
the lens.

Illustration 24: Find the distance of an object from a convex lens if image is magnified two times the actual size.
The focal length of the lens is 10 cm. (JEE MAIN)
1 1 1
Sol: For the convex lens, the position of the object from the lens is given by − =.
v u ƒ
A convex lens forms both type of images, i.e. real and virtual. Since the type of the image is not mentioned here,
we have to consider both the cases.

When the image is real: In this case, v is positive and u is negative with v = 2 u .

Thus, if u=−x then v=2x and ƒ =10 cm.

1 1 1
Substituting in − =,
v u ƒ
P hysi cs | 16.29

1 1 1 3 1
We get + = or =
2x x 10 2x 10
∴ x=15cm
x =15 cm; it implies that the object lies between F and 2F.
When the image is virtual: In this case, v and u both are negative. So let, u = − y then v = −2y and ƒ = 10cm .
1 1 1 1 1 1
Substituting in − = We get, + =
v u ƒ −2y y 10
1 1
or = ; ∴ y=5cm.
2y 10
y=5 cm; it implies that the object lies between F and P.

7.6 Displacement Method to Determine the Focal Length of a Convex Lens


If the distance d between an object and a screen is
Screen
greater than four times the focal length of a convex
lens, then there are two possible positions of the lens
between the object and the screen at which a sharp
image of the object is formed on the screen. This
method is called displacement method and is used in Object
laboratory to determine the focal length of a convex
lens.
To prove this, let us take an object that is placed at a u d-u
distance u from a convex lens of a focal length f. The d
distance between the image and the lens v = (d−u).
Figure 16.74
1 1 1
From the lens formula, − = .
v u ƒ

1 1 1
We have − = ⇒ u2 − du + df =
0
d − u −u ƒ

d ± d ( d − 4f )
∴ u=
2
Now, there are following possibilities:
(i) If d<4 ƒ , then u is imaginary.
d
(ii) If d=4 ƒ , then u= = 2ƒ . Hence, there is only one possible position and the minimum distance between an
2
object and its real image in case of a convex lens is 4 f .
d + d ( d − 4ƒ ) d − d ( d − 4ƒ )
(iii) If d > 4ƒ , there are two possible positions of lens at distances and , for which
an real image is formed on the screen. 2 2

(iv) If I1 is the image length in the first position of the object and I2 is the image length in the second position, then

the object length O is given by O = I1 I2 .


Proof:
d + d ( d − 4ƒ ) d − d ( d − 4ƒ )
u1 = ∴ v1 =d − u1 =
2 2
1 6 . 3 0 | Geometrical Optics

d − d ( d − 4ƒ ) d + d ( d − 4ƒ )
u2 = ∴ v1 =d − u2 =
2 2

I1 I2 v1 v2
Now m=
1m2 =x x
O O u1 u2

I1 I2
Substituting the values, we=
get 1=or O I1 I2
O2

7.7 Lenses in Contact


If two thin lenses of focal lengths ƒ1 and ƒ 2 are placed in contact, the equivalent focal length F of this combination
1
is given by = 1 1 .
+
F ƒ1 ƒ 2
If one surface of the lens is coated with silver, the effective focal length F of the combination is given by
1 2 1 , where
= + ƒ l is the focal length of the lens, and ƒm is the focal length of the silvered surface.
F ƒl ƒm

When two lenses of focal lengths ƒ1 and ƒ 2 are kept distance d apart from each other, the focal length of this

combination is given by 1 = 1 + 1 − d
F ƒl ƒ2 ƒl ƒ 2
1 1 1 d/µ
If there is a medium of refractive index µ between the lenses, the equivalent focal length F is = + − .
F ƒl ƒ 2 ƒl ƒ 2
1 n 1
Note: When more than two lenses in contact, the equivalent focal length is given by the formula, = ∑
F i=1 ƒ i
Here, ƒ1, ƒ 2 , … should be substituted with their respective signs.

Important Observation: To find the position of an image when one R₁ R₂


face of a lens is coated with silver.
The given system finally behaves as mirror, whose focal length is given
by O I
1 2
1 / v +1 /u =
1 / f.
+ve
1 2 ( µ2 / µ1 ) 2 ( µ2 / µ1 − 1 ) .
= − u
f R2 R1
Figure 16.75

PLANCESS CONCEPTS

A lens made of three different materials has three focal lengths. Thus, for a given object, there are three
images.
1
R R f 2
  or 2f 2f
3
f

Figure 16.76
P hysi cs | 16.31

PLANCESS CONCEPTS

Memory zone

f f f f

In contact In contact
(A) (B)

Figure 16.77

f
(A) The resultant focal length in this case is .
2
(B) The resultant focal length in this case is ∞. This is because the optical axes of both parts have been
inverted.
Anurag Saraf (JEE 2011 AIR 226)

Illustration 25: A double concave lens made up of glass of refractive index 1.6 has radii of curvature of 40 cm and
60 cm. Calculate its focal length in air. (JEE MAIN)

 1 1 
of glass with respect to air.
1
Sol: For double concave lens, the focal length is given by =
ƒ
( α
)
µg − 1  −  where aµg = n is refractive index
 R1 R 2 

11  11 11 
Using the relation (n
=
=(n−−1)
1) −− .HereR
.HereR11 =
−−40cm,
= 40cm,RR22 =
++60cm
= 60cmand 1.6 .
and xx =
1.6
=
ƒƒ RR11 RR22 

1  1 1   60 + 40  1
We get (1.6 1) 
=− − =− 0.6  =−
ƒ  −40 60   60x40  40

i.e. ƒ = −40 cm .

2
Illustration 26: A biconvex lens has a focal length times the radius of curvature of either surface. Calculate
refractive index of lens material. 3 (JEE MAIN)

1  1 1 
Sol: The biconvex or double convex lens, we can find the refractive index of material using (n − 1) 
= −  .
ƒ  R1 R 2 
2
Here,=
ƒ R= R1 R = and R 2 R .
3
1  1 1  3  1 1  2(n − 1)
Using =(n − 1)  −  , we get =(n − 1)  +  =
ƒ R
 1 R 2  2R R R  R

3 3 7
⇒ (n − 1 ) = 4
or n = + 1 = = 1.75 .
4 4

Illustration 27: A glass convex lens has a power of 10.0 D. When this lens is fully immersed in a liquid, it acts a
concave lens of focal length 50 cm. Calculate the refractive index of the liquid (Assume aµg =1.5 ).
 (JEE ADVANCED)
1 6 . 3 2 | Geometrical Optics

 1 1 
Sol: Power of convex lens is given by P =
100
f
1
where =
ƒ
( a
)
µg − 1  −  . When lens is immersed inside the
 R1 R 2 
1  µg  1
a
1 
liquid, it now behaves as concave lens thus the focal length of the lens is given as = a − 1   − 
f  µ   R1 R 2 
 l 
100 100
P
= f
∴= = 10 cm .
ƒ(in cm) 10
   1 1 
Now
1
ƒ
= ( µ − 1)  R1 − R1  ;
a
g
1 2
1
10
= 0.5  − 
 R1 R 2 

 1 1  1
∴  −  =  ... (i)
 R1 R 2  5
When fully immersed in a liquid, ƒ = −50 cm

1  µg  1 1   µg  1
a a
∴ −=  − 1   − =  − 1 x
50  a µ   R1 R 2   a µ  5
 l   l 
a a
µg 1 µg 1 9 a 10 a 10
∴ − 1 =− ; or =− + 1 = or µl = × µg = ×1.5 =1.67
a
µl 10 a
µl 10 10 9 9

Illustration 28: A thin plano-convex lens of focal length ƒ is split into two halves. One
of the halves is shifted along the optical axis as shown in the Fig. 16.78. The separation
between the object and image planes is 1.8 m. The magnification of the image formed
by one of the half lens is 2. Find the focal length of the lens and the distance between
the two halves. Draw the ray diagram of image formation.  (JEE ADVANCED)
Sol: When the lens is cut in two halves along principle axis, the focal length of both the
halves is equal as the original len.
1.8 m
For both the halves, the position of the object and the image is same. There is difference
Figure 16.78
only in magnification. Magnification for one of the halves is given as 2 (>1). This is the

first one, because v > u . So, for the first half,

=v / u 2=
or v 2u

Let u =
− x, then v=
+2x

and u + v =
1.8m .
d
=i.e. 3x 1.8m
= or x 0.6m
B1
Hence, u=−0.6m and v= +1.2m f=0.4m
1 f=0.4m
1 1 1 1 1 1 B2
Using = − = − = ƒ =0.4m A (A1,A2)
ƒ v u 1.2 −0.6 0.4
1 1 1 2
For the second half,
= −
ƒ 1.2 − d −(0.6 + d) B
1 1 1
or = + .
0.4 1.2 − d (0.6 + d)
0.6 m 0.6 m 0.6 m
By solving this, we get d = 0.6m Figure 16.79
P hysi cs | 16.33

v 0.6 1
Magnification for the second half will be m2 = = = − ,
u −(1.2) 2
v 1.2
and for the first half is m1 = = = −2
u −(0.6)

The ray diagram is shown in the Fig. 16.79.

Illustration 29: A converging lens of focal length 5.0 cm is placed in contact with a diverging lens of focal length
10.0 cm. Find the combined focal length of the system. (JEE MAIN)
1
Sol: The focal length of the combination of lenses is given by = 1 1
+
F ƒ1 ƒ 2
Here, ƒ1 =+5.0cm and ƒ 2 =−10.0cm

Therefore, the combined focal length F is given by


1 1 1 1 1 1
=+ = − =
+ ; ∴ F=
+10.0 cm
F ƒ1 ƒ 2 5.0 10.0 10.0

i.e. this combination behaves as a converging lens of focal length 10.0 cm.

7.8 Power of an Optical Instrument


By the optical power of an instrument (whether it is a lens, mirror or a refractive surface), we measure the ability of
the instrument to deviate the path of rays passing through it. If the instrument converges the rays parallel to the
principal axis, its power is positive, and if it diverges the rays, it is a negative power.

f1 f2
Figure 16.80

The shorter the focal length of a lens (or a mirror), the more it converges or diverges light. As shown in the Fig.
16.80, ƒ1 < ƒ 2 , and hence, the power P1 > P2, because bending of the light ray in case 1 is more than that in case
2. For a lens,
1 −1
P ( in dioptre) = , and for a mirror, P ( in dioptre) =
ƒ (metre) ƒ (metre)

At a glance:

Nature of lens/ Focal length Power Converging/ Ray diagram


mirror 1 1 diverging
(ƒ) PL = , PM = −
f f

Convex lens +ve +ve Converging

Figure 16.81
1 6 . 3 4 | Geometrical Optics

Nature of lens/ Focal length Power Converging/ Ray diagram


mirror 1 1 diverging
(ƒ) PL = , PM = −
f f

Concave mirror −ve +ve Converging

Figure 16.82

Diverging
Concave lens −ve −ve

Figure 16.83

Convex mirror +ve −ve diverging

Figure 16.84

PLANCESS CONCEPTS

Both convex lens and concave mirror have positive power and are converging in nature, whereas both
concave lens and convex mirror have negative power and are diverging in nature.
Vaibhav Krishnan (JEE 2009 AIR 22)

4
Illustration 30: A spherical convex surface separates an object and an image space of refractive index 1.0 and .
3
If the radius of curvature of the surface is 10 cm, find its power.  (JEE ADVANCED)
1
Sol: For the convex lens power of magnification is given by P = 40 cm
f
1  µ2  1 1 
where = − 1 
 − 
f  µ1  R1 R 2 
We have to find where do the parallel rays converge (or diverge) on
the principal axis and call it the focus and the corresponding length F
is the focal length.
1 2
µ2 µ1
µ2 − µ1
Using − = .
v u R Figure 16.85
P hysi cs | 16.35

4 / 3 1.0 4 / 3 − 1.0
With proper values and signs, we have −= ; or
= ƒ 40cm
= 0.4m
ƒ ∞ +10
Since, the rays are converging, its power should be positive. Hence,
+1 1
P ( in=
dioptre) = ; or P 2.5 dioptre
=
ƒ (metre) 0.4

Illustration 31: Two lenses of focal length 20 cm and −25cm are placed in contact. Find the total power of this
combination. (JEE MAIN)

Sol: The power of combination of lenses is given by P = P1 + P2 = 1 + 1 .


f1 f2

100 100
ƒ=
1 20cm, ƒ=
2 −25cm ; ∴ P=
1 = = 5D
ƒ1 20
100
P2 = =−4D ; ∴ P =P1 + P2 =5 − 4 =1.0 D
−25

8. PRISM
The figure shows the cross section of a prism. AB and AC represent the refracting surfaces. The angle BAC is
the angle of the prism. Assume that the prism is placed in air. A ray PQ, incident on a refracting surface AB,
gets refracted along QR. The angle of incidence and A
the angle of refraction are i and r, respectively. The ray
QR is incident on the surface AC. Here, the light travels U
from an optically denser medium to an optically rarer T

medium. If the angle of incidence r’ is not greater than
the critical angle, then the ray is refracted in air along I r r’ I’
RS. The angle of refraction is i’. The angle i’ is also called Q R
the angle of emergence. If the prism were not present, V
the incident ray would have passed un-deviated along P S
PQTU. Due to the presence of the prism, the final ray B C
travels along RS. The angle UTS= δ is called angle of Figure 16.86
deviation. From triangle TQR,

∠ UTS = ∠TQR + ∠ TRQ

or δ = ( ∠TQV − ∠RQV ) + ( ∠TRV − ∠QRV )

=( i − r ) + ( i'− r' ) =( i + i' ) − (r + r' )  .… (i)


Now, the sum of four angles of the quadrangle AQVR is 360°. The angles AQV and ARV both are 90°. Thus,
= 180° .
A + ∠ QVR
Also, from the triangle QRV, r + r '+ ∠QVR
= 180°
Hence r + r ' =A
Substituting in Eq. (i) δ = i + i'− A
1 6 . 3 6 | Geometrical Optics

8.1 Angle of Minimum Deviation


The angle i’ is determined by the angle of incidence i. Thus, the angle of deviation δ is also determined by i. For a
particular value of i, δ is minimum. In this case, the ray passes symmetrically through the prism, so that i=i’.

 
I I’ m
Q R

S
i₁=i₂ i1
R₁=R₂

Figure 16.87

8.2 Relation between the Refractive index and the Angle of Minimum Deviation
Let the angle of minimum deviation be δm . For a minimum deviation, i=i’ and r=r’.

We have δm = i + i'− A = 2i − A

A + δm
or i =  ... (i)
2
Also r + r' =A; or r = A/2 ... (ii)
sin i
The refractive index is µ= .
sin r

A + δm
sin
µ= 2
Using (i) and (ii) .
A
sin
2
If the angle of prism A is small, δm is also small. Then, the equation becomes
A + δm

µ =
2
A
⇒ δm = ( µ − 1) A .
2

Illustration 32: The angle of minimum deviation from a prism is 37°. If the angle of prism is 53°, find the refractive
index of the material of the prism. (JEE MAIN)
A + δm
sin
Sol: For prism, the refractive angle is given by µ = 2
A
sin
2
A + δm 530 + 370
sin sin
2 2 sin 450

= µ = = = 1.58 .
A 53 0
sin 26.50
sin sin
2 2
P hysi cs | 16.37

Illustration 33: A ray of light passes through a glass prism such that the angle of incidence is equal to the angle
of emergence. If the angle of emergence is ¾ times the angle of the prism, then calculate the angle of deviation
when the angle of prism is 30°.  (JEE MAIN)
Sol: For prism, we have relation i + e= A + δ, where i and e are angle of incidence and emergence respectively. A
is angle of prism and δ is angle of minimum deviation
3
Given that i=e, e = 30° .
=   A and A
4
Using the relation i + e= A + δ, we get δ =i + e − A =e + e − A =2e − A
3
= 2 x  A − A
= 0.5A 0
= 0.5 X 30= 150 .
4

8.3 Condition of No Emergence


In this section, we want to find the condition such that a ray of light entering the face AB does not come out of the
face AC for any angle iI , i.e. TIR takes place on AC
A ; ∴
r1 + r2 = r2= A − r1

or (r2 )min= A − (r1 )


max
.  ... (x)

Now, r1 will be maximum when iI is maximum, and the maximum value of iI is 90°

sin ( iI ) sin900
Hence,
= µ = max
sin (rI ) sin (r1 )
max max

max
1
sir (rI ) = = sin θc ; ∴
µ
(rI )max = θc .

From Eq. (x) (r2 )min= A − θc . … (xi)

Now, if the minimum value of r2 is greater than θC, then obviously all the values of r2 will be greater than θC and TIR
will take place under all the conditions. Thus, the condition of no emergence is,
(r2 )min > θc or A − θc > θc  …. (xii)
or A > 2θc .
A


K
M i2
i1 r1 r2 N

B C

Figure 16.88
1 6 . 3 8 | Geometrical Optics

PLANCESS CONCEPTS

Equation r1 + r2 =
A can be applied at any of the three vertices. For example in the Fig. 16.89, r1 + r2 =
B.

r1
r2

B C

Figure 16.89
Anand K (JEE 2011 AIR 47)

9. DISPERSION OF LIGHT
When a beam of white light that consists of spectrum of various wavelengths  D
ranging from long wavelengths in red color to short wavelengths in violet color
passes through a prism, it is split into its constituent colors. This phenomenon
is called dispersion. The dispersion of light takes place because the refractive
R
O
Y
G
index µ of a medium depends upon the wavelength λ of light according to the B C B
I
V
B
Cauchy’s relation µ  A + 2 , where µ is maximum for violet and minimum for Figure 16.90
λ
red color.

9.1 Angle of Dispersion


It is defined as an angle between emerging violet and red color 
( ( ) )
rays, i.e. angle of dispersion is givenαby= δαv =− δvR −; δR ; = A µ
= VA−µµVR −.µR

Red

Voilet
9.2 Dispersive Powers Figure 16.91

The ratio of the angle of dispersion to the angle of deviation of the mean yellow color, δ, of the ray produced by
any prism is called the dispersive power ω of the material of the prism
δ − δR
Dispersive power = ω =   v .
δ
µ − µR
Also, = ω =   v , where µ v , µR and µ are the refractive indices of violet, red and mean yellow colors respectively.
µ −1

9.3 Deviation without Dispersion


If two suitable prisms of small angle made up of two different transparent materials are combined and placed
opposite to each other, then the net dispersion is equal to zero, and these prisms are called an achromatic pair of
prisms that produce a deviation of ray of light, but without any dispersion.

For the two prisms of angle A and A’ and refractive indices for violet and red colors µv , µR and µv ' µR ', respectively,
( ) (
the net dispersion = µ v − µR A + µ v '− µR ' A' = 0 )
P hysi cs | 16.39

A' = −
( µ v − µR ) A
( µ 'v − µ 'R )
If ω and ω’ are dispersive powers of these prisms, and δ and δ ' are their mean deviations, then

( µ v − µR ) µ − 1 A + µ 'v − µ 'R µ '− 1 A' =0


( ) ( )
(µ − 1) µ '− 1

ωδ+ω’δ’=0.

9.4 Dispersion without Deviation


If the deviation produced by the first prism for the mean ray is equal and opposite to that produced by the second
prism, the combination of such two prisms produces dispersion without any deviation. For zone net deviation,
δ+δ’=0.
( µ − 1 ) A , (− sign denotes that the second prism is inverted),
( µ − 1 ) A + ( µ '− 1 ) A' = 0 ; A' = −
( µ '− 1 )
where μ and μ’ are refractive indices for the mean color for prisms of angles A and A’, respectively.
Note: Most of the problems of prisms can be easily solved by drawing proper ray diagram and then applying laws
of geometry with the basic knowledge of prism formulae.

Illustration 34: The angle of a prism is A, and its one surface is coated with silver.
A light ray falling at an angle of incidence 2A on the first surface returns back A
A
through the same path after suffering reflection at the second silvered surface. 90-
2A
Find the refractive index of material. (JEE MAIN) A
sin i
Sol: According to Snell’s law we have µ = . And the angle of refraction r
=90-i sin r
Figure 16.92

Given i = 2A. From the figure, it can be obtained that r = A.


sin i sin 2A 2sinA cos A

= µ = = = 2cos A .
sin r sin A sinA

Illustration 35: A crown glass prism of angle 5° is to be combined with a glass prism in such a way that the mean
ray passes undeviated. Find (i) the angle of the flint glass prism needed and (ii) the angular dispersion produced
by the combination when white light passes through it. Refractive indices for red, yellow and violet color light are
1.514, 1.523, respectively, for crown glass and 1.613, 1.620 and 1.632, respectively, for flint glass.
 (JEE ADVANCED)
( )
Sol: For the angle of minimum deviation, we have the relation δ = µ − 1 A . As prism of two different materials are
δ'
joined together, the ratio gives the angle of minimum deviation for flint glass is obtained.
δ
The deviation produced by the crown prism is δ = ( µ − 1) A , and by the flint prism is δ ' = ( µ '− 1) A' .
The prisms are placed at their inverted angles with respect to each other. The deviations are also in opposite
directions. Thus, the net deviation is
D = δ − δ ' = ( µ − 1 ) A − ( µ '− 1 ) A'  … (i)
(
(i) If the net deviation for the mean ray is zero, µ − 1 A = ) (µ '− 1) A'
or A'
=
(µ −= 1)
A
1.517 − 1 0
= x5 4.20 .
( µ '− 1 ) 1.620 − 1
1 6 . 4 0 | Geometrical Optics

(ii) The angular dispersion produced by the crown prism is δ v − δr = (µv − µr ) A , and that by the flint prism is
δ 'v − δ 'r = ( µ 'v − µ 'r ) A' .

The net angular dispersion is


δ = ( µ v − µr ) A − ( µ 'v − µ 'r ) A' = (1.523 − 1.514 ) x50 − (1.632 − 1.613) x 4.20 = −0.03480 .
The angular dispersion is 0.03480 .

Illustration 36: An isosceles glass prism has one of its faces coated with silver. A
A ray of light is incident normally on the other face (which has an equal size to
the silvered face). The ray of light is reflected twice on the same-sized faces and 180-20
then emerges through the base of the prism perpendicularly. Find the angles
of prism.  D
 (JEE ADVANCED) r₂ E
Sol: The angles of prism add up to 180o. r₂

r1 = 0 ∴ r2 = A = 1800 − 2θ  ... (i)


F r₃
∠DFE = 1800 − 900 − 2r2 = 180 − 90 − 360 + 4θ = 4θ − 270 
0 0 0 0
... (ii)
r₃
∴ r3 900 − ∠DFE
= = 3600 − 4θ  ... (iii)  G 
B C
= 900 −=
∠BFG θ 900 − r3 ; or =
r3 θ  … (iv)
From Eqs (iii) and (iv), Figure 16.93

=5θ 3600 ;=∴ θ 720 and 1800 − 2θ =360 .


∴ Angles of prism are 72o, 72o and 36o.

10. DEFECTS OF IMAGES


The relations developed regarding the formation of images by lenses and mirrors are approximate, which do not
produce focused, perfect image of objects. The imperfections or defects in such images are called aberrations.
If the defect in image is due to such approximations, these can produce spherical aberration apart from coma,
distortion, astigmatism and so on. If the defect in image is due to the dispersion of white light into constituent
colors similar to that in prism, it is called chromatic aberration producing blurred colored image of the object. The
spherical and chromatic aberrations are briefly described in the following subsections:

10.1 Spherical Aberration


This aberration is produced due to the spherical nature of a lens or
a mirror. The rays of light from point object when incident near the
principal axis called paraxial rays are focused at a larger distance at
point IM, and the rays incident near the periphery or margin of the lens
are focused near the lens at IP as shown in the Fig. 16.94. It gives a rise O
IP
IM
to a defocused long image of a point object. The spherical aberration
can be reduced by any one of the following methods:
(a) Using two thin convex lenses of focal lengths ƒ1 and ƒ 2 separated Figure 16.94
at a distance d = ƒ1 − ƒ 2 .
(b) Using a plano-convex lens with its convex surface facing the incident or emergent light, whichever is more
parallel.
P hysi cs | 16.41

(c) Either using a lens of large focal length or using a specially designed aplanatic lens, crossed lens or parabolic
reflectors.

10.2 Chromatic Aberration


As the refractive index for different wavelengths or colors comprising white or any composite light is different,
the image of an object illuminated by white light is colored due to the chromatic aberration. Such a defect can
by removed by using an achromatic doublet comprising a convex and a concave lens of focal lengths ƒ1 and ƒ 2
and dispersive powers ω1 and ω2 such that the ratio of their focal lengths is equal to the ratio of their dispersive
powers.

ω1 ω2 ω1 ƒ
+ 0
= or − 1 ,
=
ƒ1 ƒ2 ω2 ƒ2
µ v − µr
where dispersive=
power ω and µ v , µr and µ are respective indices for wavelengths of violet, red and
µ −1
mean yellow colors of white light.

11. OPTICAL INSTRUMENTS


Optical instruments are used to assist the eye in viewing an object. Let us first discuss about the human eye and
the mechanism through which we see.

11.1 The Eye


The eye has a nearly spherical shape of diameter 1 inch each. Following are some of the terms related to the eye.
(a) Cornea – The front portion of the eye is more sharply Suspensory ligament
curved and is covered by a transparent protective
Vitreous humor
membrane called the cornea. Iris

(b) Aqueous humor – Behind the cornea, there is a space Retina


filled with a liquid called aqueous humor.
Aqueous
humour
(c) Crystalline lens – The part just behind aqueous humor Pupil
is called crystalline lens.
Cornea
(d) Iris – It is the muscular diaphragm between the
aqueous humor and lens and is the colored part that Ciliary muscle
we see in the eye.
Figure 16.95
(e) Pupil – The small hole in the iris is called the pupil.
Varying aperture of the pupil controls the amount of
light entering into the eye with the help of iris.
(f) Retina – This is a screen-like structure on which the eye forms an image. The retina contains rods and cones
that receive light signal.
(g) Accommodation – When the eye is focused on a distant object, the ciliary muscles are relaxed so that
the focal length of the eye lens has its maximum value that is equal to its distance from the retina. The
parallel rays that enter into the eyes are focused on the retina, and we see the object clearly. When the
eye is focused on a closer object, the ciliary muscles are strained and the focal length of the eye lens
decreases. The ciliary muscles adjust the focal length in such a way that the image is again formed on
the retina and we see the object clearly. This process of adjusting focal length is called accommodation.
However, the muscles cannot be strained beyond a limit, and hence, if the object is brought too
close to the eye, the focal length cannot be adjusted to form the image on the retina. Thus, there is a
minimum distance for the clear vision of an object.
1 6 . 4 2 | Geometrical Optics

The nearest point at which the image can be formed on the retina is called the near point of the eye. The distance
of the near point from the eye is called the least distance for clear vision. This varies from person to person and
with age. At a young age (say below 10 years), the muscles are strong and flexible and can bear more strain. The
near point may be as close as 7−8 cm at this age. In old age, the muscles cannot bear more strain and the near
point shifts to large values, say 1 to 2 m or even more. We shall discuss about these defects of vision and use
of glasses in a later section. The average value of the least distance for clear vision for a normal eye is generally
25 cm.

11.2 Apparent Size


The size of an object is related to the
size of the image formed on the retina.
A larger image on the retina activates h
larger number of rods and cones 
attached to it, and the object looks  Image
larger. As it is clear from the Fig. 16.96,
if an object is taken away from the eye,
the size of the image on the retina
decreases, and hence, the same object
looks smaller. Furthermore the size
of the image on the retina is roughly 
proportional to the angle subtended 
Image
by the object on the eye. This angle
is called the visual angle, and optical
instruments are used to increase this Figure 16.96
artificially in order to improve the
clarity.

Illustration 37: Two boys, the one is 52 inches tall and the other 55 inches tall, are standing at distances 4.0 m and
5.0 m, respectively, from an human eye. Which boy will appear taller? (JEE MAIN)

height of object
Sol: The angle subtended by any object is given by α = .
Dis tance of object from observer
The boy which subtends the larger angle will appear taller.

52inch
The angle subtended by the first boy on the eye is=
α1 = 13inch / m .
4.0m
52inch
And the angle subtended by the second boy is =
α2 = 11inch / m .
5.0m
As α1 > α2 , the first boy will look taller when seen through the eye.

12. SIMPLE MICROSCOPE


When we view an object with naked eyes, the object must be placed somewhere between infinity and the near
point. The angle subtended on the eye is maximum when the object is placed at the near point. This angle is
h
θ0 = ,  … (i)
D
where h is the size of the object, and D is the least distance for clear vision.
P hysi cs | 16.43

This angle can further be increased if a converging lens of short focal length that is called a simple microscope or
a magnifier is placed just in front of the eye.

h o h 
F F uo
D
f D
(a) (b) (c)
Figure 16.97

Suppose, the lens has a focal length ƒ that is lesser than D and let us move the object to the first focal point F. The
eye receives the rays that come from infinity. The actual size of the image is infinite, but the angle subtended on
the lens (and hence on the eye) is
h
θ =  … (ii)
ƒ
As ƒ < D, Eqs (i) and (ii) show that θ > θ0 . Hence, the eye perceives a larger image than it could have had without
the microscope. Because the image is occurred at infinity, the ciliary muscles are least strained to focus the final
image on the retina. This is called normal adjustment. The magnifying power of a microscope is θ / θ0 , where θ
is the angle subtended by the image on the eye when the microscope is used, and θ0 is the angle subtended on
the naked eye when the object is placed at the near point. This is also known as the angular magnification. Thus,
the magnifying power is a factor by which the image on the retina can be enlarged by using the microscope.
In the normal adjustment, the magnifying power of a simple microscope is by Eqs. (i) and (ii),
θ h/ƒ D
m
= = or m
=
θ0 h / D ƒ

If ƒ < D , the magnifying power is greater than 1.


The magnifying power can further be increased by moving the object more closer to the lens. Suppose we move
the object to a distance u0 from the lens such that the virtual, erect image is formed at the near point, although the
eye is strained, it can still see the image clearly. The distance u0 is calculated using the lens formula,
1 1 1
= − .
u v ƒ
Here, v=−D and u=−u0, so that
1 1 1 D D
=
− − ; or, 1+
=  … (iii)
−u0 D ƒ u0 ƒ

The angle subtended by the image on the lens (and hence on the eye) is θ ' =h
u0
In this case, the angular magnification or magnifying power is
θ ' h / u0 D D
m= = = = 1+
θ0 h / D u0 ƒ

The above equations show that the magnification is high when the focal length ƒ is small. However, due to
several other aberrations, the image becomes too defective at a large magnification with simple microscope.
Approximately, a magnification up to 4 is trouble-free.
The magnifying power is measured in a unit X; therefore, if a magnifier produces an angular magnification of 10,
it is called as 10 X magnifier.
1 6 . 4 4 | Geometrical Optics

13. COMPOUND MICROSCOPE


The Fig. 16.98 shows a simplified version of a compound microscope and the ray diagram of image formation. It
consists of two converging lenses set coaxially. The one that faces the object is called the objective, and the one
that is close to the eye is called the eyepiece or ocular. The objective has a smaller aperture and smaller focal length
than the eyepiece. The distance between the objective and the eyepiece can be varied by appropriate screws fixed
on the panel of the microscope.

Eyepiece
Vo Uo

uo fo

h F
’
h’
Objective

Image

Figure 16.98

The object is placed at a distance u0 from the objective which is slightly greater than its focal length ƒ 0 . A real
image and an inverted image are formed at a distance v0 on the other side of the objective. This image becomes
the object for the eyepiece. For normal adjustment, the position of the eyepiece is adjusted such that the image
formed by the objective falls in the focal plane of the eyepiece. Then, the final image is formed at infinity. It is
erect with respect to the first image and, hence, inverted with respect to the object. The eye is least strained in this
adjustment as it has to focus the parallel rays coming toward it. The position of the eyepiece can also be adjusted
in such a way that the final virtual image is formed at the near point. The angular magnification is increased in this
case. The ray diagram in the Fig. 16.98 refers to this case.
The eyepiece acts as a simple microscope effectively used to view the first image. Thus, the magnification by a
compound microscope is a two-step process. In the first step, the objective produces a magnified image of the
given object. In the second step, the eyepiece produces an angular magnification. The overall angular magnification
is the product of the two.

Magnifying power

Refer to the figure, if an object of height h is seen by the naked eye and placed at the near point, the largest image
h
is formed on the retina. The angle formed by the object on the eye in this situation is θ0 = .  … (i)
D
When a compound microscope is used, the final image subtends an angle θ’ on the eyepiece (and hence on the
h'
eye) given by θ ' =  (ii)
ue '
Where h’ is the height of the first image, and ue is the distance between the first image and the eyepiece.
The magnifying power of the compound microscope is, therefore,

θ ' h' D  h'   D 


=
m = x=      … (iii)
θ0 ue h  h   ue 
P hysi cs | 16.45

h' v v
Also from the figure − 0 = 
= … (iv)
h u0 u

Now, D/ue is the magnifying power of the eyepiece that acts as a simple microscope. Using the equations given
above, in normal adjustment, this value becomes D / ƒ e when the image is formed at infinity and 1 + D / ƒ e
when the image is formed at the least distance for clear vision, i.e. at D. Thus, for the normal adjustment, the
v D 
magnifying power of the compound microscope is, by Eq. (iii), m =   when the image is formed at infinity
u  ƒ e 
v D 
and is=
m  1 +  when the final image is formed at the least distance for clear vision.
u ƒ e 

Using lens equation for the objective,


1 1 1 v v v v
− = ⇒ 1− = ⇒ = 1−
v u ƒ u ƒ0 u ƒ0
v
In general, the focal length of the objective is very small, so that >> 1. Furthermore, the first image is close to
ƒ0
the eyepiece, so that v ≈ l , where l is the tube length (distance between the objective and the eyepiece). Thus,
v v v l
= 1− ≈− ≈− .
u ƒ0 ƒ0 ƒ0
If these conditions are satisfied, for the normal adjustment, the magnifying power of the compound microscope
l D l  D 
is m = − when the image is formed at infinity and is m = − 1 +
  when the final image is formed at
ƒ0 ƒ0 ƒ0  ƒ 0 
the least distance for clear vision.
In an actual compound microscope, all the objectives and the eyepieces consist of a combination of several lenses
instead of a single lens assumed in the simplified version.

Illustration 38: A compound microscope has an objective of focal length 1 cm and an eyepiece of focal length 2.5
cm. An object has to be placed at a distance of 1.2 cm away from the objective for normal adjustment.
(i) Find the angular magnification.
(ii) Find the length of the microscope tube. (JEE ADVANCED)
1 1 1
Sol: As the objective lens of microscope is convex lens, the focal length is obtained as = − and magnification
f v u
v
is m = . The length of microscopic tube is given as L = v + ƒ e where v is the distance of image formed by objective.
u
(i) If the first image is formed at a distance v from the objective, we get
1 1 1
− = or,
= v 6 cm.
v ( −1.2cm) 1cm
v D 6cm 25cm
The angular magnification in normal adjustment is m =
=− . −50 .
=
u ƒe 1.2cm 2.5cm
(ii) For normal adjustment, the first image must be in the focal plane of the eyepiece. The length of the tube is,
therefore, L = v + ƒ e =6 cm + 2.5 cm = 8.5 cm.

14. TELESCOPES
A microscope is used to view the object placed close to it, i.e. within few centimeters. To look at the distant objects
such as stars, planets and a distant tree, we use telescope. There are three types of telescopes that are used.
1 6 . 4 6 | Geometrical Optics

(A) Astronomical Telescope


The Fig. 16.99 shows the construction and working principle of a simplified version of an astronomical telescope.

’
P’ E
P P’’ O ’ 
Q’

Figure 16.99

The telescope consists of two converging lenses placed coaxially. The one that faces the distant object is called the
objective, and it has larger aperture and focal length. The other is called the eyepiece, as it is placed closer to the
eye and has smaller aperture and focal length. The lenses are fixed in tubes. The eyepiece tube can slide within the
objective tube so that the distance between the objective and the eyepiece can be changed.
When the telescope is directed toward a distant object PQ, the objective forms a real image of that object in its
focal plane. If the point P is on the principal axis, the image point P’ is at the second focus of the objective. The
rays from Q are focused at Q’. The eyepiece forms a magnified virtual image P”Q” of P’Q’. This image is finally seen
by the eye. In normal adjustment, the position is adjusted such that the final image is formed at infinity. In such a
case, the first image P’Q’ is formed in the first focal plane of the eyepiece. The eye is least strained to focus this final
image. The image can be brought closer by pushing the eyepiece closer to the first image. A maximum angular
magnification is produced when the final image is formed at the near point.

Magnifying Power

Let the objective and the eyepiece have focal lengths ƒ 0 and ƒ e , respectively, and the object is placed at a large
distance u0 from the objective. The object PQ in the Fig. 16.99 subtends an angle α on the objective. Since the
object is at infinity, the angle it would subtend on the eye, if there were no telescope, is α’.
As u0 is very large, the first image P’Q’ is formed in the focal plane of the objective.

P'Q' P'Q'
From the figure α = α ' ≈ tan α ' = =  ... (i)
OP' ƒ0

The final image P”Q” subtends an angle β on the eyepiece (and hence on the eye). From the triangle P’Q’E,,

P'Q' β ƒ0
β ≈ tan β= ⇒ =  ... (ii)
EP' α EP'

If the telescope is adjusted for normal adjustment so that the final image is formed at infinity, the first image P’Q’
must be in the focal plane of the eyepiece.
Then ΕΡ ' = ƒ e .

β ƒ
Thus, Eq. (ii) becomes = 0 .  … (iii)
α ƒe

The angular magnification or the magnifying power of the telescope is


P hysi cs | 16.47

Angle subtended by the final image on the eye


m= .
Angle subtended by the object on the unaided eye

The angles β and α are formed on the opposite sides of the axis. Hence, the signs of these angles are opposite,
β β
and β / α is negative. Hence, m = = − .
α α
ƒ
Using Eq. (iii), m = − 0 .
ƒe
If the telescope is adjusted so that the final image is formed at the near point of the eye, the angular magnification
is further increased. Let us apply the lens equation to the eyepiece in this case.
Here, u = −EP’ and v =−EP” = −D.
1 1 1
The lens equation is − =
v u ƒ
1 1 1 1 1 1 ƒe + D
⇒ − = ⇒ = + = . ... (iv)
−D −EP' ƒ e −EP' ƒ e D ƒeD

β ƒ0 ( ƒe + D )
By Eq. (ii), =
α ƒeD

β β ƒ0 ( ƒe + D ) ƒ  ƒ 
The magnification is m = =
− =
− − 0  1 + e 
=
α α ƒeD ƒe  D

Length of the Telescope


From the Fig. 16.106, we see that the length of the telescope is L =OP'+ P'E' =ƒ 0 + P'E.

For normal adjustment, Ρ ' Ε = ƒ e so that L = ƒ 0 + ƒ e . For adjustment for the near-point vision, we get, by Eq. (iv),
ƒe ƒeD
Ρ'Ε = , so that the length is L =ƒ 0 + .
ƒe + D ƒe + D

14.1 Resolving Power of a Telescope


The resolving power of a microscope is defined as the reciprocal of the distance between two objects, which can
be resolved when seen through the microscope. It depends on the wavelength λ of the light, the refractive index
µ of the medium between the object and the objective of the microscope and the angle θ subtended by a radius
of the objective on one of the object. It is given by
1 2µ sin θ
R
= =
∆d λ
To increase the resolving power, the objective and the object are kept immersed in oil. It increases µ and hence R.
The resolving power of a telescope is defined as the reciprocal of the angular separation between two distant
objects which are just resolved when viewed through a telescope. It is given by
1 a
R
= = ,
∆θ 1.22 λ
where a is the diameter of the objective of the telescope. The telescopes with larger objective aperture (1 m or
more) are used in astronomical studies.

15. DEFECTS OF VISION


As described earlier, the ciliary muscles control the curvature of the lens in the eye and hence can change the
effective focal length of the system. When the muscles are fully relaxed, the focal length is maximum. When the
1 6 . 4 8 | Geometrical Optics

muscles are strained, the curvature of the lens increases and the focal length decreases. For clear vision, the image
must be formed on the retina. The image distance is, therefore, fixed for clear vision, and it equals the distance of
the retina from the eye lens. It is about 2.5 cm for a grown-up person. If we apply the lens formula to the eye, the
magnitudes of the object distance, the image distance and the effective focal length satisfy

1 1 1 1 1 1
+ = or = −  ... (i)
v 0 u0 ƒ u0 ƒ v 0

Here, v0 is fixed, and hence by changing ƒ , the eye is focused on the objects placed at different values of u0. We
see from Eq. (i) that when ƒ increases, u0 increases, and when ƒ decreases, u0 decreases. The maximum distance
one can see is

1 1 1
= − ,  ... (ii)
umax ƒmax v 0

where ƒmax is the maximum focal length possible for the eye lens.
The focal length is maximum when the ciliary muscles are fully relaxed. In a normal eye, this focal length equals the
distance v0 from the lens to the retina. Thus,
v0 =
ƒmax by (ii) , umax =
∞.
Theoretically, a person can have clear vision of the objects placed at any large distance from the eye. For the closer
objects, u is smaller, and hence, f should be smaller. The smallest distance at which a person can have a clear vision
is related to the minimum possible focal length ƒ . The ciliary muscles are most strained in this position. By Eq. (ii),
the closest distance for clear vision is given by
1 1 1
= − . ... (iii)
umin ƒmin v 0
For an average grown-up person, umin should be around 25 cm or
less. This is a convenient distance at which one can hold an object in
his/her hand and can see. Thus, a normal eye can clearly see objects
placed in the range from about 25 cm from the eye to a large distance
of the order of several kilometers. The nearest point and the farthest
point up to which an eye can clearly see are called the near point and
the far point. For a normal eye, the distance of the near point should
be around 25 cm or less, and the far point should be at infinity. We (A) Defective-eye
now describe some common defects of vision.
Figure 16.100
By the eye lens, real, inverted and diminished image is formed
at retina.
The common defects of vision are as follows:
(a) Myopia or short sightedness: The distant objects are not
clearly visible in this defect. The image of a distant object
is formed before the retina.
The defect can be remedied by using a concave lens. (B) Defective-eye

Figure 16.101

Illustration 39: A nearsighted man can clearly see the objects up to a distance of 1.5 m. Calculate the power of the
lens of the spectacles necessary for the remedy of this defect.  (JEE MAIN)
Sol: As the man has near sighted vision, he need to wear concave lens which can form virtual and erect images.
1
The power of magnification of lens is P = .
ƒ
P hysi cs | 16.49

The lens should form a virtual image of a distant object at 1.5 m from the lens. Thus, it should be a divergent

lens, and its focal length should be −1.5 m. Hence,


1 1 −1
ƒ = − 1.5m ⇒ P= = − m = −0.67D.
ƒ 1.5

(b) Hypermetropia or far sightedness: The near objects are not clearly visible in this defect. The image of a near
object is formed behind the retina.

I
O

(A) Deflected-eye (B) Corrected-eye

Figure 16.102

This defect is remedied by using a convex lens.

(c) Presbyopia: In this defect, both near and far objects are not clearly visible. This is remedied either by using
two separate lenses or by using a single spectacle having bifocal lenses.

(d) Astigmatism: In this defect, the eye cannot see objects in two orthogonal (perpendicular) directions clearly
simultaneously. This defect is remedied by using a cylindrical lens.

PLANCESS CONCEPTS

While testing your eye by reading a chart, if doctor finds it to 6/12, it implies that you can read a letter
from 6 m which the normal eye can read from 12 m. Thus, 6/6 is the normal eye sight.
1
Worth Knowing: The persistence of vision is s, i.e. if the time interval between two consecutive light
10
rays is less than 0.1 s, the eye cannot distinguish them separately. Hence, the fps (frames per second) of
a video should be more than 10.
Anurag Saraf (JEE 2011 AIR 226)
1 6 . 5 0 | Geometrical Optics

PROBLEM-SOLVING TACTICS

 1. Of u, v and f, any two values will be known to us and we will be asked to find the third. In such type of
problems, two cases are possible.
Case 1: When signs of all the three will be known to us from the given information, substitute all the three with the
known sign; then, we can get only the numerical value of the unknown (i.e. the third quantity) without sign.
Case 2: When the sign of the third unknown quantity is not known to us, substitute only the known quantities with
sign. Then, the numerical value of the unknown with its respective sign can be obtained.
 2. The experiments show that if the boundaries of the media are parallel, the emergent ray CD, although
laterally displaced, is parallel to the incident ray AB if µ1 =µ5 . We can also directly apply the Snell’s law
( µ sini =
cons tant ) in media 1 and 5, i.e. µ1 sini1 =
µ5 sini5 µ1 sini1 =
µ2 sini2 =
µ3 sini3 =
.... =
µi sinii

i5
C

B
i1

A
1 2 3 4 5

Figure 16.103

Notice that an apparent depth is multiple of either µ or 1/ µ . It can be find out by knowing whether the medium
through which light is entering is a denser or rarer medium.

 3. Sometimes only a part of a prism will be given. To solve such problems, first complete the prism and then
solve the problems.

o
50


o o o o
60 70 60 70

Figure 16.104
P hysi cs | 16.51

FORMULAE SHEET

S. No KEY CONCEPTS DESCRIPTIONS


1 Law of (i) The incident ray (AB), the reflected ray (BC) and normal (NN’) to the surface
reflection (SS’) of reflection at the point of incidence (B) lie in the same plane. This plane is
called the plane of incidence (also the plane of reflection).
(ii) The angle of incidence (the angle between the normal and the incident ray)
and reflection angle (the angle between the reflected ray and the normal) are
equal.
∠ i =∠r . A C
N

i r
S N’ S’
B

Figure 16.105

2 Object (a) Real: Point from which rays actually diverge.


(b) Virtual: Point toward which rays appear to converge.

3 Image The image is decided by the reflected or refracted rays only. The point image for
a mirror is that point
(i) Toward which the rays reflected from the mirror actually converge (real image),
OR
(ii) From which the reflected rays appear to diverge (virtual image).

4 Characteristics (a) The size of the image is the same as that of the object.
of reflection by
(b) For a real object, the image is virtual, and for a virtual object, the image is real.
a plane mirror
(c) For a fixed incident light ray, if the mirror is rotated through an angle θ , the
reflected ray turns through an angle of 2 θ .
5 Spherical
mirrors A
A

C F P
C F P

B B
Concave Convex

Figure 16.106

6 Paraxial rays Rays that form very small angle with principal axis are called paraxial rays.
1 6 . 5 2 | Geometrical Optics

S. No KEY CONCEPTS DESCRIPTIONS


7 Sign convention We follow the Cartesian coordinate system convention according to which
(a) The pole of the mirror is the origin.
(b) The direction of the incident rays is a positive x-axis.
(c) Vertically up is positive y-axis.
Note: According to this, a convention radius of curvature and focus of concave
mirror are negative and of convex mirror are positive.
8 Mirror formula f=x-coordinate of focus;

1 1 1
= + , where v=x-coordinate of the image;
f v u

u=x-coordinate of the object.


Note: Valid only for paraxial rays.
9 Transverse h2 = y -coordinate of the image
magnification
h2 v
m= = − .
h1 u

h1 = y -coordinate of the object.


(both are perpendicular to the principle axis of the mirror)
10 Optical power 1
Optical power of a mirror (in dioptres) = − , where f is the focal length (in m) with
a respective sign. f

REFRACTION – PLANE SURFACE


1 Laws of (i) The incident ray (AB), the normal (NN’) to the refracting surface (II’) at the point
refraction (at of incidence (B) and the refracted ray (BC) all lie in the same plane called the plane
any refracting of incidence or the plane of refraction.
surface)
sin i
(ii) = Cons tant : for any two given media and light of a given wavelength.
sin r

sin i n2 v1 λ1
This is the Snell’s law. = 1 n=
2 = =
sin r n1 v 2 λ2

Note: The frequency of light does not change during refraction.


2 Deviation of angle of
a ray due to deviation
refraction
i -i-r


r

Figure 16.107
P hysi cs | 16.53

S. No KEY CONCEPTS DESCRIPTIONS


3 Refraction (i) Emerged ray is parallel to the incident ray, if medium is same on both sides.
through a
parallel slab t sin(i − r)
(ii) Lateral shift x = ,
cos r

where t= thickness of the slab.


A
N
i
B Air
Glass(M)
r
t N’

o
90
C X
i
D
Figure 16.108

Note: Emerged ray is not parallel to the incident ray if the media on both the sides
are different.
4 Apparent depth At near normal incidence,
of a submerged
object µ2
h' = h
µ1 II
I

h’
h
x

O
Figure 16.109

Note: h and h’ are always measured from the surface.


5 Critical angle (i) Ray travels from a denser to a rarer medium.
& total internal
(ii) The angle of incidence should be greater than the critical angle (i>c).
reflection (TIR.)
N N N
nr
Critical angle C = sin−1
ni r
Rarer
I I’
c i>C
i
Denser
O
N’ N’ N’
                       Figure 16.110
1 6 . 5 4 | Geometrical Optics

S. No KEY CONCEPTS DESCRIPTIONS


6 Refraction
1. δ = (i + i') − (r + r ') .
through prism P
2. r + r ' =.
A
A
3. Variation in δ versus (shown in diagram). i

i’
r1 r2
4. There is one and only one angle of incidence,
for which the angle of deviation is minimum.

Q R
When δ = δm then i = i' & r = r ' , Figure 16.111
the ray passes symmetrically through the prism,
and then (where n=absolute RI of glass),
 A + δm  
sin 
2  .
n 
A

min
sin  
2

min
o
min i=i’ 90
Figure 16.112

Note: When the prism is dipped in a medium, then (where n=RI of glass w.r.t.
medium).

( )
5. For a thin prism, A < 100 ;  δ= (n − 1)A .
6. Dispersion of light: The angular splitting of a ray of white light into a
number of components when it is refracted in a medium other than air is called
dispersion of light.
7. Angle of dispersion: An angle between the rays of the extreme colors in the
refracted (dispersed) light is called angle of dispersion. θ = δ v − δr .
8. Dispersive power ( ω) of the medium of the material of prism.
Angular dispersion
( ω) =Derivation of mean ray ( yellow )
 v
i n r
For a small-angled prism, (A<10°) 
Mean ray
δ v − δR nv − nR nv + nR v
=ω = = ;n ,
δy n−1 2

where nv , nR and n are RI of the material Figure 16.113


for violet, red and yellow colors, respectively.
P hysi cs | 16.55

S. No KEY CONCEPTS DESCRIPTIONS


9. Combination of two prisms:
(i) Achromatic combination: It is used for deviation without dispersion. Condition
for this is (nv − n) A = (n'v − n'r ) A'.

 n + nR   n' + n'R 
Net mean Deviation=  v − 1 A −  v − 1 A'.
 2   2 
Or ωδ= + ω ' δ ' 0 where ω, ω ' are dispersive powers for the two prisms and δ, δ '
are the mean deviations.
(ii) Direct vision combination: It is used to produce dispersion without deviation;
 n + nR   n'v + n'R 
condition for this is  v − 1=
A  − 1 A'.
 2   2 
Net angle of dispersion (nv − n) A − (n'v − n'r ) A'.

REFRACTION AT A SPERICAL SURFACE


1
µ2 µ1µ2 − µ1
(a) − = ; v, u a R are kept with sign. As v = PI
v u R

u = −PO
R = PC 1 2
(Note radius is with sign).
+ve x
O P C I
µ1 v
(b) m =
µ2u
Figure 16.114

2 Lens formula
1 1 1
(a) − =
v u f
1 1
1  
( µ − 1)  R1 − R1 

(b) =
f  1 2 
+ve

v
(c) m =
u
Figure 16.115
1 6 . 5 6 | Geometrical Optics

Solved Examples

JEE Main/Boards µ in the direction of one of the diameters. At what


distance from the center of the sphere, will the rays be
Example 1: A person sees the point A on the rim at the focused? Assume that µ < 2.
bottom of a cylindrical vessel when the vessel is empty
Sol: For refraction of light ray from surface of sphere,
through the telescope T. When the vessel is completely
filled with a liquid of refractive index 1.5, he observes µ µ µ1 − µ2
the distance of image is obtained by 1 − 2 = .
a mark at the center B, of the bottom, without moving v r R
the telescope or the vessel. What is the height of the The ray refracted at one surface becomes object for the
vessel if the diameter of its cross section is 10 cm? opposite surface.
For the first refraction surface:
Sol: The vessel is filled with a liquid of refractive index
µ 1 = 1, µ 2 = µ ,u = −∞ , v = ?, r = + R
µ when the ray from B reaches the telescope.
µ 1 µ −1 µR
∴ µ sin i = 1.0 x sin r , ... (i) ∴ − = v
⇒=
v −∞ R µ −1
where ∠r =∠ADC, ∠i =∠BDC
For refraction at the second surface:
R 2R
=sin i = , sin r I’ serves as an object for the second surface. Now,
R 2 + h2 h2 + (2R)2 µ1 =µ (since light travels from sphere to air) and
µ2 = 1,
where h is the height of the liquid, and R is the radius
of the vessel.  µR  R (2 − µ )
u ==
BI' +  − 2R  =
T  µ −1  µ −1
E
(It is positive because light travels from the left to the
right, and the distance is also in the same direction.)
r v = ?, r = − R .

1 µ (µ − 1) 1 − µ R (2 − µ )
 = 1.5 D
i ∴ − = or v
=
h v R (2 − µ) −R 2 (µ − 1)

This distance is positive and is referred from the second


surface.
A B C
∴ Distance of the focal point from the center
5 cm 5 cm
R (2 − µ ) µR
= = +R .
Substituting these values in Eq. (i), 2 (µ − 1) 2 (µ − 1)

R 2R 2 R 2 + h2
µ×
= = ⇒ µ . Q
R 2 + h2 h2 + ( 2R ) h2 + ( 2R )
2 2 P
R

 µ2 − 1 
( ) ( )
A C B I I’
4R 2 µ2 −=
1 h2 4 − µ2 ⇒=h 2R  
2
 4 − µ 

(1.5)2 − 1 5
=h 10 = 10 = 8.45cm . Example 3: A rectangular glass block of thickness
4 − (1.5)2 7
10 cm and refractive index 1.5 is placed over a small
coin. A beaker is filled with water of refractive index 4/3
Example 2: A parallel beam of light rays is incident on to a height of 10 cm and is placed over the block.
a transparent sphere of radius R and a refractive index
P hysi cs | 16.57

(i) Find the apparent position of the object when it is 1 1 −1 3


viewed from normal incidence. = sin−= sin
= 480 .
a
µw 4
(ii) Draw a neat ray diagram.
Obviously, therefore, TIR takes place earlier at the
(iii) If the eye is slowly moved away from the normal, at water–air interface.
a certain position, the object is found to disappear due
to the TIR. At which surface, does this happen and why?
Example 4: How long will the light take in travelling a
distance of 500 m in water? Given that µ for water is 4/3
Sol: The image of coin formed at upper surface of block,
and the velocity of light in vacuum is 3x1010 cm/s. Also
becomes object for beaker containing water. The image
calculate the equivalent path.
µ1 µ2 µ1 − µ2
thus formed at distance v is given by − = . c
v r R Sol: The velocity of light in a medium is given as v = .
4 µ
For the first surface: µ= , µ= 1.5,
2
3 1
The optical path travelled by light is  = µ × d where d
u= −10 cm, R = ∞, v = ? is the distance travelled in water.
1
4 4 . We know that
− 1.5
3 − 1.5 3 80
= ⇒ v1 =
− cm Velocity of light in vaccum
v1 −10 ∞ 9 µ= .
Velocity of light in water
I’ serves as an object for the second surface. For the
second surface: 4 3x1010
= .
Note: This is an alternative to the apparent depth 3 Velocity of light in water
relation. Velocity of light in water = 2.25 x 1010cm/s.
 80  170
u= −BI' = − (BA + AI') =−  10 + =− cm 500x100
 9  9 Time
= taken = 2.22x10−6 sec.
2.25x1010
4
µ1 = , µ2 =1, R =∞ , v 2 =?
3 Equivalent optical path
4 4
1− = µ x dis tance travelled in water
1
∴ − 3= 3 ⇒ v= 14.2cm
2
v2 170 ∞ 4
− = = x500 666.64m.
9 3
θc: critical angle for glass–water interface
Example 5: In the figure shown, for an angle of
1 1 incidence i at the top surface, what is the minimum
= sin−1 ⇒ θc =sin−1 refractive index needed for TIR at the vertical face?
w w
µg µa × aµg
i
Air
B
r


c
c

1 −1 8
= sin−1 .= sin
= 62.70 Sol: Total internal reflection of light occurs inside body
3 3 9
x when the angle of incidence is greater than critical
4 2
1
The critical angle for water–air interface angle θC and according to Snell’s law θC = sin−1  
µ
Applying the Snell’s law at the top surface,
1 6 . 5 8 | Geometrical Optics

sini .
µ sin r = ... (i)
For TIR, the vertical face A

µ sin θc =
1 i
Using geometry, θc= 90° − r i

µ sin(90 − r)= 1 r 1 
⇒ µ cos = ... (ii)

On squaring and adding Eqs (i) and (ii), we get

∴ µ2 sin2 r + µ2 cos2 r = 1 + sin2 i

2
⇒ µ= 1 + sin i. 

Example 6: A point source of light is placed at the


bottom of a tank containing a liquid (refractive index =
Sol: The angles of prism add up to 180o.
µ) up to a depth h. A bright circular spot is seen on the
surface of the liquid. Find the radius of this bright spot. α 900 − A
From the figure, =
=i 900 − α ; =i 900 −=
α A … (i)
Sol: The light waves incident on the surface of the water
0
at an incident angle greater than critical angle will get Also, =β 90 −=2i 900 − 2A
reflected in side water. This light waves forms cone in
γ 900 −=
and = β 2A
the volume of the tank. Thus we get relation of critical
R Thus, γ = r = 2A
angle as tan θC =
h From geometry,
180
R A+γ+
= γ 1800. or =
A = 360 .
5

c c
Example 8: A lens has a power of +5 dioptre in air.
h
What will be its power if completely immersed in water?

3 4
Given µ=
g ; µ=
w
.
2 3
S
Sol: According to the lens maker’s formula the focal
Rays coming out of the source and incident at an angle
greater than θc will be reflected back into the liquid; 1  µ1  1 1 
length 1/f is = − 1   − 
therefore, the corresponding region on the surface will f  µ2   R1 R 2 
appear dark. As it is obvious from the figure, the radius
of the bright spot is given by Let fa and fw be the focal lengths of the lens in air and
water, respectively, then,
h sin θc hsin θ 1 µw
R h tan =
= θc R
⇒= =Pa = and Pw .
cos θc 1 − sin θc2
fa fw
1 h = = 20cm .
fa 0.2m
Since
= sin θc ; =∴R .
µ 2
µ −1 Using the lens maker’s formula,
Example 7: The cross section of the glass prism has the  
1
form of an isosceles triangle. One of the equal faces is Pa =
fa
(
= µg − 1 ) R1 − R1   ... (i)
coated with silver. A ray of light incident normally on  1 2 
the other equal face and after getting reflected twice
1  µg  1 1 
emerges through the base of prism along the normal. = − 1  − 
Find the angle of the prism. fw  µ w   R1 R 2 
 
P hysi cs | 16.59

Example 10: Two equiconvex lenses of focal lengths 30


1  µg − µ w  1 1  cm and 70 cm, made of material of refractive index =
P=
w =    −  …(ii)
fw  µ w   R1 R 2  1.5, are held in contact coaxially by a rubber band round
 
their edges. A liquid of refractive index 1.3 is introduced
Dividing Eq. (ii) by Eq. (i), we get,
in the space between the lenses filling it completely.

=
Pw
=
(
µg − µ w 1 1
.
) Find the position of the image of a luminous point
object placed on the axis of the combination lens at a
Pa µ µ − 1
w g (
3 µw ) distance of 90 cm from it.
 1   3  +5
⇒ Pw Pa   =
=  D Sol: This system is combination of three lenses. Two
 3  4  4 lenses of glass one lens of liquid. Add the powers to
get total power.
Example 9: The distance between two point sources
of light is 24 cm. Find out where you would place a
R1 = R 2 = f1= 30cm (As µ= 1.5) .
converging lens of focal length 9 cm, so that the images
of both the sources are formed at the same point. Similarly, R 3= R 4= f2= 70cm .
Sol: For lens the distance of the image formed from The focal length of the liquid lens (in air),
1 1 1
the lens is given by − = where u, v and f are
1  1 
v u f
distance of object, distance of image and focal length f3
= ( µ − 1)  R1 − 
R 3 
 2

respectively.

(1.3 − 1)  −130 − 70
1 
1 1 1  1
For S1 : − = =  = −
ν1 −x 9   70 1 2 3 4

1 1 1 ν
∴ = −  … (i) (b) m =
ν1 9 x u
1 1 1
For S2 : − =
ν2 − ( 24 − x ) 9
 … (ii)
∴m1 =
(5.0 − 4.0 ) =
−0.25 ,
1 1 1
( −4.0 )
∴ = − .
ν2 9 24 − x
and m2 =
(5.0 − 1.0 ) = −4.00 .
f=9 cm ( −1.0 )
Hence, both the images are real and inverted, the first
S1 S2 is magnification −0.25, and the second is −4.00.

JEE Advanced/Boards
x 24-x
Since, the sign convention for S1 and S2 is just opposite. Example 1: A 4-cm-thick layer of water covers a 6-cm-
Hence, thick glass slab. A coin is placed at the bottom of the
slab and is being observed from the air side along the
ν1 = −ν2 . normal to the surface. Find the apparent position of the
1 1 coin from the surface.
⇒ =

ν1 ν2 Air
1 1 1 1 Water
∴ −= − 4cm h1
9 x 24 − x 9
Solving this equation, we get x = 6 cm. Therefore, the Glass
lens should be kept at a distance of 6 cm from either 6cm h2
of the object. Coin

Sol: As the thick layer of water is placed over the


glass slab. The coin placed beneath the glass slab will
1 6 . 6 0 | Geometrical Optics

appear to shift upwards due to both glass and water by focal length for crown glass f ω
distance s. This apparent shift is thus given by = = −
focal length for flint glass f '
ω'
 1   1  9 325 5 1 5
s = s1 + s2 =  1 −  × h1 +  1 − ×h =
− × − ;
= = −  … (i)
µ2  2 520 9 8 8f
'
 µ1   f

The total apparent shift is As the focal length of the combination is 100 cm,

 1   1  1 1 1 1 5 3 1
s = h1  1 −  + h2  1 − = + = − = = .
 F f f ' f 8f 8f 100
 µ1   µ2 
3
37.5cm .
f = × 100 =
 1   1  8
s = 4 1 −  + 6 1 −  = 3 cm.
 4 / 3   3 /2 −8 −8 75
f' = × f= × = −60 cm .
5 5 2
Air The achromatic doublet requires a convex lens of focal
length 37.5 cm made of crown glass and a concave lens
4cm h1 Water of focal length 60 cm made of flint glass.
h

Glass Example 3: A 20-cm-thick slab of glass of refractive


6cm h2 S index 1.5 is placed in front of a plane mirror and a pin
is placed in front of it in the air at a distance of 40 cm
Coin from the mirror. Find the position of the image.
Thus, h = h1 + h2 − s = 4 + 6 − 3 .
Sol: The slab of thickness t forms the image of the
= 7.0 cm. object O at the point O’.
The slab shifts the image of object by ( t − t µ ) . O’
Example 2: An achromatic lens of focal length 100 cm serves as an object for the plane mirror.
is made up of crown and flint glass lenses. Find the
Object distance for the mirror is MO.
focal length of each lens. Given that for crown glass
µν =1.5245, µr =1.5155 and for flint glass µ'ν =1.659
and µ'r =1.641.

Sol: The focal length of the combination of lens is given


1 1 1 O O’ M I” I’
by = + .
F f1 f2
If µ and µ’ are mean refractive indices for crown and
flint glasses, respectively, then,
µν + µr 1.5245 + 1.5155 20 100
=µ = = 1.52 . = 40 − = cm .
2 2 3 3
µ'ν + µr' 1.659 + 1.641 Since in a plane mirror, object distance = image
=µ' = = 1.65 .
2 2 distance.
The dispersive powers ω and ω ' for crown and flint 100
glass, respectively, are MI' MO
= = '
cm .
3
µν − µr 1.5245 − 1.5155 0.009 9 Now, I' serves as an object for the slab again.
=ω = = =
µ −1 1.52 − 1 0.52 520 20 20
I' is shifted to I" by 20 − =cm .
1.5 3
µ'ν − µ'r 1.659 − 1.641 0.018 9
=ω' =
'
µ −1
= =
1.65 − 1 0.65 325 ()
∴ Distance of the final image I" from the mirror
100 20 80
To have an achromatic combination, = − = cm .
3 3 3
P hysi cs | 16.61

Example 4: The convex surface of a thin concavo– For the image to be informed at the same point as the
convex lens of glass of refractive index 1.5 has a radius object
of curvature 20 cm. The concave surface has a radius
u=2F =2 × 7.5 =15cm.
of curvature of 60 cm. The convex side is coated with
silver and placed at a horizontal surface as shown in The object should be placed at a distance of 15 cm
the figure. from the lens on the optical axis.
(b) If fw is the focal length of lens in water, the focal
length F’ of this combination is given by

1 1 1 1 1 1
= + + + +
F' fw fg fm fg fw

1 2 2 1
(a) Where a pin should be placed on the optical axis = + +  ... (ii)
F ' fw fg fm
such that its image is formed at the same place?
=
(b) If the concave part is filled with water of refractive fg 60 = cm, fm 10 cm .
index 4/3, find the distance through which the pin
The value of fw is calculated by using the relation,
should be moved, so that the image of the pin again
coincides with pin.  1 1 
1
fw
= ( a
)
µ w − 1  − 
 R1 R 2 
Sol: When the convex side of the concavo–convex lens
is coated with silver, the combination becomes a mirror.
This combination consist two lenses and one mirror 1 4  1  1
⇒ = − 1  =
placed close to each other. The powers of all the three fw  3 60
  180
will be added. When the water is filled on concave side,
fw = 180 cm.
we get plano-convex water lens whose focal length
is found by lens makers formula. This combination Substituting these values in Eq. (ii), we get
consists of four lenses (two lenses of glass and two
1 2 2 1 90
lenses of water) and one mirror. = + + ; F =
F' 180 60 10 13
(a) The refraction takes place from the first surface, 2 × 90 180
reflection from the lower surface and finally refraction
'
u= 2F
= = cm.
13 13
from the first surface of focal lengths fg , fm and fg ,
respectively. The combined focal length F is given by Displacement of the pin

1 1 1 1 2 1 180 15
= + + = +  … (i) =u − u' =15 − = =1.14 cm.
F fg fm fg fg fm 13 13

=fm R=
2 /2 20
= / 2 10 cm. . Example 5: The radius of curvature of the convex face
of plano-convex lens is 12 cm, and its µ =1.5.
The value of fg can be obtained by using the formula
(a) Find the focal length of the lens.
 1 1 
1
fg
= ( a
)
µg − 1  − 
 R1 R 2 
The plane surface of the lens is coated with silver.
(b) At what distance from the lens, will the parallel rays
incident on the convex surface converge?
 1 1 
=(1.5 − 1)  20 − 
60  (c) Sketch the ray diagram to locate the image, when a

point object is placed on the axis at a distance of 20 cm
fg = 60 cm. from the lens.

Substituting these values in Eq. (i), (d) Calculate the image distance when the object is
placed as in (c).
1 2 1 2
= + =
F 60 10 15 Sol: Use the lens maker’s formula to find the focal
15 length of the plane-convex lens. When the plane side of
F
= = 7.5cm. the lens is coated with silver, the combination becomes
2
1 6 . 6 2 | Geometrical Optics

a mirror. This combination consist two lenses and one 1 1 1


+ =.
mirror placed close to each other. The powers of all the ν u F
three will be added.
For F = 12 cm, u = +20cm and ν =?
(a) The focal length f of lens of refractive index µ is
given by the formula. 1 1 1 1
= − =
ν 12 20 30
1  1 
f
= ( µ − 1)  R1 − .
R 2 
ν =30 cm.
 1
A real image is formed.
In case of plano-convex lens,
R1 = ∞ , R 2 = −12cm, µ =1.5 Example 6: An object O is placed at a distance of 15
cm from a convex lens A of focal length 10 cm and its
1 1  image I1 is formed on a screen on the other side of the
∴ =
f
(1.5 − 1)  12 
lens. A concave lens B is now placed midway between A
 
and I1 , then the screen is moved back 10 cm to receive
f = 24 cm. a clear image I2.
(b) The focal length, fm, of plane-silvered surface is Find (a) The focal length of the concave lens and
infinity. The focal length F of the plano-convex lens,
when the plane surface is coated with silver is given by (b) Linear magnification of the final image.

1 1 1 1 2 1 Sol: The convex lens forms the image of the object at


= + + = + point I1. This image acts as the object for concave lens
F f fm f f ∞
and the final image is formed at I1’.
f 24
∴ F= = = 12cm. For refraction through convex lens,
2 2
u1 = OP = − 15cm .
Such a lens behaves like a concave mirror of focal
length 12 cm. The parallel rays converge at a distance f =+10 cm, ν1 =PI1 .
of 12 cm from the silvered surface. 1 1 1
Using − =
ν u f

A B

P Q I1 I2

12 cm
15 cm 15 cm 15 cm 10 cm
(c) The figure shows the ray diagram of the image
formed by this lens when the object is placed at a 1 1 1 1  1  3−2 1
= + = + = =
distance of 20 cm from the lens. The light is incident ν1 f u 10  −15  30 30
from the right to the left.
⇒ ν1 = 30 cm .
I1 serves as a virtual object for lens B. For refraction
through lens B,

I
u2 =
QI1 =
PI1 − 15 =
30 − 15 =
+15cm,
O
ν2 =15 + 10 =+25cm .
1 1 1 1 1 3−5 2
=− = − = =−
f ν u 25 15 75 75
(d) For a lens, 75
f=
− −37.5cm,
=
2
P hysi cs | 16.63

The negative sign implies it is a concave lens.


and µ2 =− ( 0.6 + 0.4 ) =−1.0 cm
ν1 ν2
Linear magnification =
u1
×
u2 ( 4 / 3) − 1 ( 4 / 3) − 1
=
30 5 10
ν2 ( −1) −0.2
= × = − =−3.33
15 3 3 ν2 =−0.5 cm
The negative sign shows that the final image is The final image I2 is formed at a distance of 0.5 cm to
inverted. the left of the second surface P2 . The final image is at
a distance of 0.5 − 0.4 = 0.1cm to the left of the first
Example 7: A parallel beam of light travelling in water surface as shown in figure.
( µ =4 / 3) is refracted by a spherical air bubble of (b) The ray diagram is already shown in the figure drawn.
radius 2 mm placed in water. Assume that the light rays
to be paraxial.
Example 8: An object is placed at a distance of 12 cm
(a) Find the position of the image due to refraction at to the left of a diverging lens of focal length −6.0 cm. A
the first surface and the position of the final image. converging lens with a focal length of 12.0 cm is placed
(b) Draw a ray diagram showing the positions of both at a distance d to the right of the diverging lens. Find the
images. distance d that corresponds to a final image at infinity.

f = 6.0 cm f = 12 cm

=

I2 P1
P2

 = (4/3)

0.6 cm 4 cm 12 cm D
5 cm
Sol: The concave lens forms the image of the object at
Sol: The image of the object formed by the first point say I1. This image acts as the object for convex
refraction by the water-glass surface acts as the object lens and the final image is formed at say I1’.
for the second refraction at glass-water surface. 1 1 1
Applying lens formula − = twice, we have
(a) For the refraction from a single spherical surface, ν u f
we have 1 1 1
− = ... (i)
µ2

µ1 ( µ2 − µ1 ) .
=
ν1 −12 −6

ν u R 1 1 1
− = ... (ii)
Let P1 be pole at the first surface and P2 to be pole at ∞ ν1 − d 12

the second surface. At P, Solving Eqs (i) and (ii), we have
µ1 =( 4 / 3) ; µ2 =1; R = +0.2cm, u1 = ∞ . ν1 =− 4cm .
1 ( 4 / 3) 1 − ( 4 / 3) And d = 8cm .
So − = .
ν1 ∞ +0.2
Example 9: A solid glass sphere with a radius R and
∴ ν1 =−0.6 cm .
a refractive index of 1.5 is coated with silver over a
The first surface will form a virtual image I1 at a distance hemisphere. A small object is located on the axis of the
0.6 cm to the left of P1 as shown in the figure. sphere at a distance 2R to the left of the vertex of the
un-silvered hemisphere. Find the position of the final
This image acts as an object for the second surface. So image after all refractions and reflections have taken
for the second surface at P2 , place.
µ1 =1, µ2 =( 4 / 3 ) , R =−0.2cm
1 6 . 6 4 | Geometrical Optics

Sol: The image formed by first refraction at air-glass 1 1 1 2


Second reflection: Using + = = with proper
spherical surface acts as the object for the concave ν u f R
mirror. The image formed by the mirror acts as the sign conventions, we have
object for spherical glass-air spherical surface.
1 1 2 R
The ray of light first gets refracted, then reflected and + =− ∴ ν2 = −
then again refracted. For the first refraction and then ν2 ∞ R 2
reflection, the ray of light travels from the left to the
µ2 µ1 µ2 − µ1
right, while for the last refraction it travels from the Third refraction: Again using − =
right to the left. Hence, the sign convention will change ν u R
accordingly. with reversed sign convention, we have

I2
I3
O O
R R

2R
1.5 R R/2
µ2 µ1 µ2 − µ1
First refraction: Using − = with proper 1.0 1.5 1.0 − 1.5
ν u R − =
ν3 −1.5R −R
sign conventions, we have
⇒ ν3 =−2R ;
1.5 1.0 1.5 − 1.0 i.e. final image is formed on the vertex of the silvered
− = ∴ ν1 =∞ .
ν1 −2R +R face.

JEE Main/Boards

Exercise 1 Q.4 State the conditions for TIR of light to take place.
Calculate the speed of light in a medium, whose critical
angle is 45o.
Q.1 A ray of light incident on an equilateral glass prism
shows a minimum deviation of 30°. Calculate the speed
Q.5 An object is placed at the focus of concave lens.
of light through the glass prism.
Where will its image be formed?

Q.2 Where an object should be placed from a


Q.6 Draw a graph to show the variation of the angle of
converging lens of focal length 20 cm so as to obtain a
deviation ‘D’ with that of the angle of incidence ‘I’ for
real image of magnification 2?
a monochromatic ray of light passing through a glass
prism refracting angle ‘A’. Hence, deduce the relation.
Q.3 What changes in the focal length of (i) a concave
mirror and (ii) a convex lens occur, when the incident
Q.7 Draw a ray diagram of an astronomical telescope
violet light on them is replaced with red light?
in the normal adjustment position. Write down the
expression for its magnifying power.
P hysi cs | 16.65

Q.8 A spherical surface of radius of curvature R separates 1 1 1


Q.16 Derive the lens formula, = − for a concave
a rarer and a denser medium. Complete the path of the f v u
incident ray of light, showing the formation of the real lens, using the necessary ray diagram.
image. Hence, derive the relation connecting an object
Two lenses of power 10 D and −5 D are placed in
distance ‘u’, image distance ‘v’, radius of curvature R
contact.
and the refractive indices n1 and n2 of the two media.
Briefly explain how the focal length of a convex lens (i) Calculate the power of the new lens.
changes, with an increase in wavelength of incident
(ii) Where should an object be held from the lens so as
light.
to obtain a virtual image of magnification 2?

Q.9 (a) Draw a labelled ray diagram to show the


Q.17 Two thin lenses of power +6 D and −2 D are in
formation of the image by a compound microscope.
contact. What is the focal length of the combination?
Write the expression for its magnifying power.
(b) How does the resolving power of a compound Q.18 Define the refractive index of a transparent
microscope change and when? medium. A ray of light passes through a triangular
(i) Refractive index of the medium between the object prism. Plot a graph showing the variation of the angle
and the objective lens increases and of deviation with an angle of incidence.
(ii) Wavelength of the radiation used is increased.
Q.19 (a) (i) Draw a labeled ray diagram to show the
formation of image in an astronomical telescope for a
Q.10 Draw a labeled ray diagram to show the image
distant object.
formation I of a refractive-type astronomical telescope.
Why should the diameter of the objective of a telescope (ii) Write three distinct advantages of a reflecting-type
be large? telescope over a refracting-type telescope.
(b) A convex lens of focal length 10 cm is placed
Q.11 A beam of light converges to a point P. A lens coaxially 5 cm away from a concave lens of focal length
is placed in the path of the convergent beam at a 10 cm. If an object is placed 30 cm in front of the convex
distance of 12 cm from P. At what point, does the beam lens, find the position of the final image formed by the
converge if the lens is combined system.
(i) A convex lens of focal length 20 cm.
Q.20 A converging lens is kept coaxially in contact with
(ii) A concave lens of focal length 16cm?
a diverging lens – both the lenses being of equal focal
Do the required calculations. lengths. What is the focal length of the combination?

Q.12 A double convex lens of glass of refractive index Q.21 (a) (i) Draw a neat labeled ray diagram of an
1.6 has its both surfaces of equal radii of curvature of astronomical telescope in the normal adjustment.
30 cm each. An object of height 5 cm is placed at a Explain briefly its working.
distance of 12.5 cm from the lens. Calculate the size of
(ii) An astronomical telescope uses two lenses of power
the image found.
10 D and 1 D. What is its magnifying power in the
normal adjustment?
Q.13 Why does the bluish color predominate in a clear
(b) (i) Draw a neat labeled ray diagram of a compound
sky?
microscope. Explain briefly its working.

Q.14 How does the angle of minimum deviation of (ii) Why must both the objective and the eyepiece of a
a glass prism of a refractive index 1.5 change, if it is compound microscope have short focal lengths?
immersed in a liquid of refractive index 1.3?
Q.22 Draw a labeled ray diagram of a reflecting
Q.15 Draw a labeled ray diagram, showing the image telescope. Mention its two advantages over the
formation of an astronomical telescope in the normal refracting telescope.
adjustment position. Write the expression for its
magnifying power.
1 6 . 6 6 | Geometrical Optics

Q.23 (i) An object is placed between two plane mirrors (C) The boy cannot see his feet.
inclined at 60o to each other. How many images do
(D) The boy cannot see neither his hair nor his feet.
you expect to see?
(ii) An object is placed between two plane parallel Q.5 A point source of light S is placed in front of
mirrors. Why do the distant images get fainter and two large mirrors as shown. Which of the following
fainter? observers will see only one image of S?

Q.24 The magnifying power of a compound microscope S


C
B
is 20. The focal length of its eye piece is 3 cm. Calculate
the magnification produced by the objective lens.
30o

Q.25 An astronomical telescope having a magnifying A


power of 8 consists of two thin lenses 45 cm apart. Find
the focal length of the lenses.
(A) Only A (B) Only C
(C) Both A and C (B) Both B and C
Exercise 2
Single Correct Choice Type Q.6 In the figure shown if the object ‘O’ moves toward
the plane mirror, then the image I (which is formed after
successive reflections from M1 & M2 , respectively) will
Q.1 Two plane mirrors are inclined at 70o . A ray incident move:
on one mirror at an angle of θ after reflection falls on
the second mirror and is reflected from there parallel to (A) Toward right
the first mirror, θ is: (B) Toward left
o o
(A) 50 (B) 45 (C) With zero velocity O

(C) 30o (D) 55o (D) Cannot be determined M2 M1

Q.2 There are two plane mirror with reflecting surfaces Q.7 A point source of light is 60 cm away from a screen
facing each other. Both the mirrors are moving with and is kept at the focus of a concave mirror that reflects
the speed of ν away from each other. A point object is light on the screen. The focal length of the mirror is 20
placed between the mirrors. The velocity of the image cm. The ratio of average intensities of the illumination
formed due to the nth reflection will be on the screen when the mirror is present and when the
(A) nv (B) 2nv mirror is removed is:

(C) 3nv (D) 4nv (A) 36:1 (B) 37:1


(C) 49:1 (D) 10:1
Q.3 A man of height ‘h’ is walking away from a street
lamp with a constant speed ‘ ν ’. The height of the street Q.8 A concave mirror is placed on a horizontal table,
lamp is 3h. The rate at which the length of the man’s with its axis directed vertically upward. Let O be the
shadow is increasing when he is at a distance of 10h pole of the mirror and C its center of curvature. A point
from the base of the street lamp is: object is placed at C. It has a real image, also located at
(A) ν / 2 (B) ν / 3 C (a condition called auto-collimation). If the mirror is
now filled with water, the image will be:
(C) 2ν (D) ν / 6
(A) Real and will remain at C
Q.4 A boy of height 1.5 m with his eye level at 1.4 m (B) Real and located at a point between C and ∞
stands before a plane mirror of length 0.75 m fixed on (C) Virtual and located at a point between C and O
the wall. The height of the lower edge of the mirror
above the floor is 0.8 m. Then, (D) Real and located at a point between C and O

(A) The boy will see his full image.


Q.9 In the diagram shown below, a point source O is
(B) The boy cannot see his hair. placed vertically below the center of a circular plane
P hysi cs | 16.67

mirror. The light rays starting from the source are Q.13 When the object is at distances u1 and u2 , the
reflected from the mirror such that a circular area A on images formed by the same lens are real and virtual,
the ground receives light. Now, a glass slab is placed respectively, and of same size. Then, the focal length of
between the mirror and the source O. What will be the the lens is:
magnitude of the new area on the ground receiving
1 1
light? (A) uu
2 1 2
(B) (u + u2 )
2 1
Circular plane mirror Circular plane mirror
(C) u1u2 (D) 2 (u1 + u2 )

Q.14 Parallel beam of light is incident on a system of


two convex lenses of focal lengths f1 = 20 cm and f2 =
O O
10 cm. What should be the distance between the two
Circular plane mirror lenses so that rays after refraction from both lenses get
un-deviated?
(A) A
(B) Greater than A
(C) Less than A
(D) Cannot say, as the information given is insufficient

Q.10 In the figure ABC is the cross section of a right-


f1
angled prism, and BCDE is the cross section of a glass
slab. The value of θ , so that light incident normally
(A) 60 cm (B) 30 cm (C) 90 cm (D) 40 cm
on the face AB does not cross the face BC, is (given
sin−1 ( 3 / 5 ) = 37o )
Q.15 An object is placed at a distance of 15 cm from a
convex lens of a focal length 10 cm. On the other side
B E of the lens, a convex mirror is placed at its focus such
that the image formed by the combination coincides
with the object itself. The focal length of the convex
3/2
6/2

mirror is
l


n
n

A C D

(A) θ ≤ 37o (B) θ > 37o


(C) θ ≤ 53o (D) θ < 53o O

Q.11 A small source of light is 4 m below the surface


of a liquid of refractive index 5/3. In order to cut off
all the light coming out of liquid surface, the minimum (A) 20 cm (B) 10 cm (C) 15 cm (D) 30 cm
diameter of the disc placed on the surface of liquid is:
(A) 3m (B) 4m (C) 6m (D) ∞ Q.16 Look at the ray diagram shown, what will be the
focal length of the 1st and the 2nd lens, if the incident
light ray is parallel to emergent ray.
Q.12 A point source of light is placed at a distance h
below the surface of a large deep lake. What is the st
1
nd
2
percentage of light energy that escapes directly from Incident
the water surface? µ of the water=4/3? (Neglect partial
Emergent
reflection)
(A) 50% (B) 25% (C) 20% (D) 17% 5 cm 5 cm
1 6 . 6 8 | Geometrical Optics

(A) −5cm and −10cm (B) +5cm and +10cm π π


(A) cm (B) cm
(C) −5cm and +5cm (D) +5cm and +5cm 8 12
5π π
(C) cm (D) cm
Q.17 A point object is kept at the first focus of a convex 36 7
lens. If the lens starts moving toward right with a
constant velocity, the image will Q.22 Light ray is incident on a prism of angle A= 60°
and refractive index µ = 2 . The angle of incidence at
(A) Always move toward right
which the emergent ray grazes the surface is given by
(B) Always move toward left  3 −1 1 − 3 
(C) First move toward right and then toward left. (A) sin−1   (B) sin−1  
 2   2 
   
(D) First move toward left and then toward right.    2 
3
(C) sin−1   (D) sin−1  
 2   3
 
V
Object
Previous Years’ Questions
F

Q.1 A student measures the focal length of convex lens


by putting an object pin at a distance ‘u’ from the lens
Q.18 A ray incident at an angle 53° on a prism emerges and measuring the distance ‘v’ of the image pin. The
at an angle of 37° as shown. If the angle of incidence graph between ‘u’ and ‘v’ plotted by the student should
is 50° , which of the following is a possible value of the look like (2002)
angle of emergence
v (cm) v (cm)

o
37 (A) (B)
o
53

O u (cm) O u (cm)

v (cm) v (cm)

(A) 35° (B) 42° (C) 40° (D) 38° (C) (D)

3
Q.19 A prism has a refractive index and a refracting O u (cm) O u (cm)
2
angle 90°. Find the minimum deviation produced by Q.2 An experiment is performed to find the refractive
the prism. index of glass using a travelling microscope. In this
experiment, distances are measured by  (2003)
(A) 40° (B) 45° (C) 30° (D) 49°
(A) A vernier scale provided on the microscope
Q.20 A certain prism is found to produce a minimum (B) A standard laboratory scale
deviation of 38° . It produces a deviation of 44° when
(C) A meter scale provided on the microscope
the angle of incidence is either 42° or 62° . What is the
angle of incidence when it is undergoing a minimum (D) A screw gauge provided on the microscope
deviation?
(A) 45° (B) 49° (C) 40° (D) 55° Q.3 Two transparent media of refractive indices
µ1 and µ3 have a solid lens shaped transparent
material of refractive index µ2 between them as shown
Q.21 A thin prism of angle 5° is placed at a distance
in the figures in Column II. A ray traversing these media
of 10 cm from the object. What is the distance of the
is also shown in the figures. In Column I, the different
image from the object? (Given µ of prism = 1.5)
P hysi cs | 16.69

relationships between µ1 , µ2and µ3 are given. Match (A) The ray gets totally internally reflected at face CD.
them to the ray diagram shown in Column II. (2007) (B) The ray comes out through face AD.
(C) The angle between the incident ray and the
Column I Column II emergent ray is 90o.
(D) The angle between the incident ray and the
(A) µ1 < µ2 (p) emergent ray is 120o.

2 1
3 Q.5 The focal length of a thin biconvex lens is 20cm.
When an object is moved from a distance of 25 cm
in front of it to 50 cm, the magnification of its image
(B) µ1 > µ2 m
(q) changes from m25 to m50 . The ratio 25 is (2005)
m50

Q.6 An object at distance 2.4 m in front of a lens forms


2 1
3 a sharp image on a film at distance 12 cm behind the
lens. A 1-cm-thick glass plate of refractive index 1.50 is
interposed between lens and film with its plane faces
(r) parallel to film. At what distance (from lens), should the
(C) µ2 =µ3 object be shifted to be in sharp focus on film? (2009)
(A) 7.2 m (B) 2.4 m (C) 3.2 m (D) 5.6m
3 1
2
Q.7 A spectrometer gives the following reading when
used to measure the angle of prism.
(D) µ (s)
Main scale reading: 58.5°
Vernier scale reading: 09 divisions
1
3 2 Given that 1 division on the main scale corresponds
to 0.5°. Total divisions on the vernier scale are 30 and
(t) match with 29 divisions of the main scale. The angle of
the prism from the above data (2011)
(A) 58.59° (B) 58.77°
3
1 (C) 58.65° (D) 59°
2

Q.8 An initially parallel cylindrical beam travels in a


medium of refractive index µ ( I ) = µ0 + µ2 I , where µ0
Q.4 A ray OP of monochromatic light is incident on and µ2 are positive constants, and I is the intensity of
the face AB of prism ABCD near vertex B at an incident the light beam. The intensity of the beam is decreasing
angle of 60o (see figure) If the refractive index of the with an increasing radius.
material of the prism is 3 , which of the following is As the beam enters the medium, it will (2013)
(are) correct? (2009)
(A) Diverge
B
(B) Converge
0 o (C) Diverge near the axis and converge near the
60
periphery
P
C (D) Travel as a cylindrical beam 4
o
135

Q.9 The initial shape of the wave in front of the beam is


90
o
o  (2000)
75
A (A) Convex
D
1 6 . 7 0 | Geometrical Optics

(B) Concave (A) The spectrum of visible light whose frequency is


more than that of green light will come out to the air
(C) Convex near the axis and concave near the periphery
medium.
(D) Planar
(B) The entire spectrum of visible light will come out of
the water at various angles to the normal.
Q.10 An object 2.4 m in front of a lens forms a sharp
image on a film 12 cm behind the lens. A glass plate (C) The entire spectrum of visible light will come out of
1cm thick, of refractive index 1.50 is interposed between the water at an angle of 90° to the normal.
lens and film with its plane faces parallel to film. At (D) The spectrum of visible light whose frequency is
what distance (from lens) should object be shifted to less than that of green light will come out to the air
be in sharp focus on film?  (2012) medium.
(A) 7.2 m (B) 2.4 m (C) 3.2 m (D) 5.6 m
Q.15 Monochromatic light is incident on a glass prism
Q.11 The graph between angle of deviation ( δ ) of angle A. If the refractive index of the material of the
and angle of incidence (i) for a triangular prism is prism is µ , a ray, incident at an angle θ , on the face AB
represented by:  (2013) would get transmitted through the face AC of the prism
provided:  (2015)

(A)  
(B)
 A

O i O i B C

     1  
(C) (D) (A) θ − sin−1 µ sin  A − sin−1    
   µ   
O O 
i i   1  
(B) θ > cos−1 µ sin  A + sin−1    
   µ   
Q.12 Diameter of plano-convex lens is 6 cm and
thickness at the centre is 3 mm. If speed of light in    1  
material of lens is 2 × 108 m / s , the focal length of the (C) θ < cos−1 µ sin  A + sin−1    
   µ   
lens is:  (2013)
(A) 20 cm (B) 30 cm (C) 10 cm (D) 15 cm    1  
(D) θ > sin−1 µ sin  A − sin−1    
   µ   
 3
Q.13 A thin convex lens made from crown glass  µ = 
 2
has focal length f. When it is measured in two different Q.16 Assuming human pupil to have a radius of 0.25
cm and a comfortable viewing distance of 25 cm, the
4 5
liquids having refractive indices and , it has the minimum separation between two objects that human
3 3 eye can resolve at 500 nm wavelength is:  (2015)
focal lengths f1 and f2 respectively. The correct relation
between the focal lengths is:  (2014) (A) 30 µm (B) 100 µm

(A) f2 > f and f1 becomes negative (C) 300 µm (D) 1 µm

(B) f1 and f2 both become negative


Q.17 In an experiment for determination of refractive
(C) f1= f2 < f index of glass of a prism by i − δ , plot, it was found that
(D) f1 > f and f2 becomes negative a ray incident at angle 35° , suffers a deviation of 40°
and that it emerges at angle 79° . In that case which of
the following is closest to the maximum possible value
Q.14 A green light is incident from the water to the
of the refractive index ?  (2016)
air – water interface at the critical angle ( θ ) . Select the
correct statement (2014) (A) 1.6 (B) 1.7 (C) 1.8 (D) 1.5
P hysi cs | 16.71

JEE Advanced/Boards

Exercise 1 Q.5 A balloon is rising up along the axis of a concave


mirror of radius of curvature 20 m. A ball is dropped
from the balloon at a height 15 m from the mirror when
Q.1 Two flat mirrors have their reflecting surfaces the balloon has velocity 20 m/s. Find the speed of the
facing each other, with an edge of one mirror in contact image of the ball formed by a concave mirror after 4 s?
with an edge of the other so that the angle between [Take: g = 10 m / s2 ]
the mirrors is 60° . Find all the angular positions of
the image with respect to x-axis. Take the case when
Q.6 An observer whose least distance of distinct vision
a point object is between the mirrors at (1, 1). Point of
is ‘d’ views his own face in a convex mirror of radius
intersections is (0, 0) and 1st mirror is along the x-axis.
or curvature ‘r’. Prove that the magnification produced
r
Q.2 In the figure shown, AB is a plane mirror of length cannot exceed .
40 cm placed at a height 40 cm from ground. There d + d2 + r 2
is a light source S at a point on the ground. Find the
minimum and maximum height of a man (eye height) Q.7 A surveyor on one bank of canal observes the
required to see the image of the source if he is standing images of the 4-inch mark and 17-ft mark on a vertical
at a point P on the ground as shown in the Fig. staff, which is partially immersed in the water and held
A against the bank directly opposite to him. He see that
40 cm

the reflected and refracted rays come from the same


point which is the center of the canal. If the 17-ft mark
B and the surveyor’s eye are both 6 ft above the water
level, estimate the width of the canal, assuming that the
40 cm

S P refractive index of the water is 4/3. Zero mark is at the


bottom of the canal.
20 cm 40 cm

Q.8 A ray of light travelling in air is incident at a grazing


Q.3 A plane mirror of circular shape with radius r=20 angle (incident angle = 90° ) on a long rectangular slab
cm is fixed to the ceiling. A bulb is placed on the axis of of a transparent medium of thickness t−1.0 (see figure).
the mirror. A circular area of radius R=1 on the floor is The point of incidence is the origin A (0, 0). The medium
illuminated after the reflection of light from the mirror. has a variable index of refraction n(y) given by
The height of the room is 3 m. What is the maximum
1/2
distance from the center of the mirror and the bulb so n( y ) =
ky 3/2 + 1 1.0 m−3/2 ,
, where k =
that the required area is illuminated?  
the refractive index of air is 1.0
Q.4 A concave mirror of focal length 20 cm is cut into y
two parts from the middle, and the two parts are moved Air P(x₁, y₁)
perpendicularly by a distance 1 cm from the previous
principal axis AB. Find the distance between the images
t. 1m

formed by the two parts?


B(x, y)
M1 Medium x
10 cm (0, 0) Air

A
1 cm
B
(i) Obtain a relation between the slope (dy/dx) of the
O 1 cm trajectory of the ray at a point B (x, y) in the medium
and the incident angle at that point.
(ii) Find the value of n sin i.
M2
(iii) Obtain an equation for the trajectory y(x) of the ray
in the medium.
1 6 . 7 2 | Geometrical Optics

(iv) Determine the coordinate ( x1 y1 ) of point P, where


the ray intersects the upper surface of the slab–air I
boundary. o
37
(v) Indicate the path of the ray subsequently.
M
Q.9 A uniform, horizontal beam of light is incident O 10 m
upon a quarter cylinder of radius R = 5 cm and has W
a refractive index 2 / 3 . A patch on the table at a Q.14 The diagram shows five isosceles right-angled
distance ‘x’ from the cylinder is unilluminated. Find the prisms. A light ray incident at 90° at the first face
value of ‘x’? emerges at the same angle with the normal from
the last face. Find the relation between the refractive
indices?

Q.15 Two rays travelling parallel to the principal axis

2 4
R X 1 3 5

Q.10 An opaque cylindrical tank with an open top has


strike a large plano-convex lens of a refractive index
diameter of 3.00 m and is fully filled with water. When
of 1.60. If the convex face is spherical, a ray near
the sunlight reaches at an angle of 37° above the
the edge does not pass through the focal point
horizon, sunlight ceases to illuminate any part of the
(spherical aberration occurs). If this face has a radius
bottom of the tank. How deep the tank is?
of curvature of 20.0 cm and the two rays are at
= h1 0.50
= cm and h2 12.0 cm from the principal axis,
Q.11 A beam of parallel rays of width b propagates in then find the difference in the positions where they
a glass at an angle θ to its plane face. The beam width cross the principal axis.
after it enters into air through this face is _______ if the
refractive index of glass is µ .

C x
b Glass R

Air
Q.16 A room contains air in which the speed of sound
Q.12 A parallel beam of light falls normally on the first is 340m/s. The walls of the room are made of concrete,
face of a prism of a small angle. At the second face, it is in which the speed of sound is 1700m/s. (a) Find the
partly transmitted and partly reflected, and the reflected critical angle for the TIR of sound at the concrete–air
beam strikes at the first face again and emerges from boundary. (b) In which medium must the sound be
it in a direction by making an angle 6°30' with the undergone the TIR?
reversed direction of the incident beam. The refracted
beam has undergone a deviation of 1°15' from the
Q.17 A rod made of glass ( µ =1.5 ) and of square cross
original direction. Find the refractive index of the glass
section is bent as shown in the figure. A parallel beam
and the angle of the prism.
of light falls perpendicularly on the plane flat surface A.
Referring to the diagram, d is the width of a side, and
Q.13 A light ray I is incident on a plane mirror M. The R is the radius of inner semicircle. Find the maximum
mirror is rotated in the direction as shown in the figure d
by an arrow at a frequency 9 / π rev / sec . The light value of ratio so that all the light rays entering the
R
reflected by the mirror is received on the wall W at a glass through surface A emerge from the glass through
distance 10 m from the axis of rotation. When the angle surface B.
of incidence becomes 37o, find the speed of the spot (a
point) on the wall?
P hysi cs | 16.73

of the bulb. (c) At this 7.50-m distance, a convex lens is


set up with its axis pointing toward the bulb. The lens
has a circular face with a diameter of 15.0 cm and a
focal length of 30.0 cm. Find the diameter of the image
d of the bulb formed on a screen kept at the location of
the image. (d) Find the light intensity at the image.

A 3
Q.23 A thin plano-convex lens fits exactly into a plano-
concave lens with their plane surface parallel to each
other as shown in the figure. The radius of curvature
Q.18 A prism of refractive index 2 has a refracting of the curved surface R = 30 cm. The lenses are
angle of 30° . One of the refracting surfaces of the made of different material having a refractive index
prism is polished. For the beam of monochromatic light =µ1 3 / 2and
= µ2 5 / 4 as shown in the Fig.
to retrace its path, find the angle of incidence on the
refracting surface.

1=3/2 1=5/4
Q.19 An equilateral prism deviates a ray by 23° for
two angles of incidence differing by 23° . Find µ of the
prism?

Q.20 A ray is incident on a glass sphere as shown


in the figure The opposite surface of the sphere is
partially coated with silver. If the net deviation of the
ray transmitted at the partially silvered surface is 1/3rd
(i) If plane surface of the plano-convex lens is coated
of the net deviation suffered by the ray reflected at the
with silver, then calculate the equivalent focal length
partially silvered surface (after emerging out of the
of this system and also the nature of this equivalent
sphere), find the refractive index of the sphere.
mirror.

Partially (ii) An object having a transverse length of 5 cm in


60
o
silvered placed on the axis of equivalent mirror (in par 1) at
a distance 15 cm from the equivalent mirror along
the principal axis. Find the transverse magnification
produced by the equivalent mirror.

Q.24 Two identical convex lenses L1 and L2 are placed


at a distance of 20 cm from each other on the common
principal axis. The focal length of each lens is 15 cm,
and lens L2 is placed right to lens L1. A point object
Q.21 Two thin similar watch glass pieces are joined
is placed at a distance of 20 cm left to lens L1 on
together front to front, with rear convex portion is
the common principal axis of two lenses. Find where
coated with silver, and the combination of glass pieces
a convex mirror of radius of curvature 5 cm should
is placed at a distance α =60 cm from a screen. A
be placed to the right of L2 so that the final image
small object is placed normal to the optical axis of the
coincides with the object?
combination such that its two times magnified image
is formed on the screen. If air between the glass pieces
is replaced by water ( µ =4 / 3) , calculate the distance Q.25 A thin equiconvex lens of glass of a refractive
through which the object must be displaced so that a index µ =3 / 2 and a focal length 0.3 m in air is sealed
sharp image is again formed on the screen. into an opening at one end of a tank filled with water
( µ =4 / 3) . On the opposite side of the lens, a mirror
is placed inside the tank on the tank wall perpendicular
Q.22 A spherical light bulb with a diameter of 3.0 cm
to the lens axis, as shown in the figure. The distance
radiates light equally in all directions, with a power of
4.5π W. (a) Find the light intensity at the surface of the between the lens and the mirror is 0.8 m. A small object
is placed outside the tank in front of the lens at a
bulb. (b) Find the light intensity 7.50 m from the center
distance of 0.9 m from the lens along its axis. Find the
1 6 . 7 4 | Geometrical Optics

position (relative to the lens) of the image of the object (A) f/2, I/2 (B) f, I/4
formed by the system.
(C) 3f/4, I/2 (D) f, 3I/4
0.9 m 0.8 m
Q.5 An object is placed in front of a thin convex lens
of focal length 30 cm, and a plane mirror is placed 15
cm behind the lens. If the final image of the object

Mirror
coincides with the object, the distance of the object
from the lens is
(A) 60 cm (B) 30 cm
(C) 15 cm (D) 25 cm

Q.6 A converging lens of a focal length 20 cm and


Exercise 2 diameter 5 cm is cut along line AB. The part of the lens
shaded in the diagram is used to form an image of a
Single Correct Choice Type point P placed 30 cm away from it on line XY, which is
perpendicular to the plane of the lens. The image of P
Q.1 An object is placed in front of a convex mirror will be formed.
at a distance of 50 cm. A plane mirror is introduced
covering the lower half of the convex mirror. If the
distance between the object and the plane mirror is 30
2 cm 2 cm
cm, there is no gap between the images formed by the B
two mirrors. The radius of the convex mirror is: 5 cm A
X P Y
(A) 12.5 cm (B) 25 cm
(C) 50 cm (D) 100 cm 30 cm

Q.2 An infinitely long rod lies along the axis of a (A) 0.5 cm above XY (B) 1 cm below XY
concave mirror of a focal length f. the near end of the
rod is at a distance u>f from the mirror. Its image will (C) On XY (D) 1.5 cm below XY
have a length.
Q.7 A screen is placed 90 cm away from an object. The
(A) f 2 / (u − f ) (B) uf / (u − f) image of the object on the screen is formed by a convex
lens at two different locations separated by 20 cm. The
(C) f 2 / (u + f ) (D) uf / (u + f ) focal length of the lens is
(A) 18 cm (B) 21.4 cm
Q.3 A luminous point object is moving along the
principal axis of a concave mirror of a focal length 12 (C) 60 cm (D) 85.6 cm
cm toward it. When its distance from mirror is 20 cm,
its velocity is 4 cm/s. The velocity of the image in cm/s Q.8 In the above problem, if the sizes of the images
at that instant is: formed on the screen are 6 cm and 3 cm, then the
(A) 6 toward the mirror height of the object is nearly:

(B) 6 away from the mirror (A) 4.2 cm (B) 4.5 cm

(C) 9 away from the mirror (C) 5 cm (D) 9 cm

(D) 9 toward the mirror


Q.9 A concave mirror cannot form

Q.4 A thin lens has a focal length f, and its aperture (A) A virtual image of a virtual object
has a diameter d. It forms an image of intensity I. Now (B) A virtual image of a real object
the central part of the aperture up to diameter (d/2)
is blocked by an opaque paper. The focal length and (C) A real image of a real object
image intensity would change to (D) A real image of a virtual object
P hysi cs | 16.75

Multiple Correct Choice Type

Q.10 A reflecting surface is represented by the equation 1 2


2L  πx 
= y sin   ,0 ≤ × ≤ L . A ray travelling horizontally
π  L 
becomes vertical after reflection. The coordinates of
3
the point(s) where this ray is incident is

(A) µ1 < µ2 < µ3 (B) µ12 − µ22 > µ32


y
(C) µ12 − µ32 > µ22 (D) µ12 + µ22 > µ32

Q.13 For refraction through a small angle prism, the


angle of deviation:
(A) Increases with an increase in RI of prism.
(B) Will decrease with an increase in RI of prism.
 L 2L   L 3L  (C) Is directly proportional to the angle of prism.
(A)  ,  (B)  , 
4 π  3 π  (D) Will be 2 D for a ray of RI = 2.4 if it is d for a ray of
   
RI = 1.2.
 3L 2L   2L 3L 
(C)  ,  (D)  , 
 4 π   3 π 
    Q.14 For the refraction of light through a prism
(A) For every angle of deviation, there are two angles
Q.11 In the figure shown, consider the first reflection at of incidence.
the plane mirror and second at the convex mirror. AB
(B) The light travelling inside an equilateral prism is
is object.
necessarily parallel to the base when prism is set for a
minimum deviation.
V
(C) There are two angles of incidence for a maximum
deviation.
A B C
(D) The angle of minimum deviation will increase if
refractive index of prism is increased keeping the
10cm 10cm 120cm outside medium unchanged if µp > µs .
50cm
Q.15 A man of height 170 cm wants to see his complete
image in a plane mirror (while standing). His eyes are at
(A) The second image is real, inverted by 1/5th a height of 160 cm from the ground.
magnification w.r.t. AB.
(A) Minimum length of the mirror=80 cm.
(B) The second image is virtual and erect by 1/5th
(B) Minimum length of the mirror=85 cm.
magnification w.r.t. AB.
(C) Bottom of the mirror should be at a height 80 cm
(C) The second image moves toward the convex mirror.
or less.
(D) The second image moves away from the convex
(D) Bottom of the mirror should be at a height 85 cm.
mirror.

Q.16 A flat mirror M is arranged parallel to a wall W


Q.12 In the diagram shown, a ray of light is incident
at a distance l from it. The light produced by a point
on the interface between 1 and 2 at an angle slightly
sources S kept on the wall is reflected by the mirror and
greater than the critical angle. The light undergoes
produces a light spot on the wall. The mirror moves
TIR at this interface. After that, the light ray falls at
with a velocity v toward the wall.
interfaces 1 and 3, and again it undergoes TIR. Which
of the following relations should hold true?
1 6 . 7 6 | Geometrical Optics

Wall O
W
S
B
l A
V

M I

(A) The spot of light will move with the speed v on the Q.19 A lens is placed in the XYZ coordinate system such
wall. that its optical center is the origin and the principal axis
is along the x-axis. The focal length of the lens is 20 cm.
(B) The spot of light will not move on the wall. A point object has been placed at the point (-40 cm, +1
(C) As the mirror comes closer, the spot of light will cm, −1 cm). Which of the following are correct about
become larger and shift away from the wall with a coordinates of the image?
speed larger than v. (A) x = 40 cm (B) y =+1 cm
(D) The size of the light spot on the wall remains the (C) z = +1 cm (D) z=−1 cm
same.
Q.20 Which of the following can form a diminished,
Q.17 Two reflecting media are separated by a spherical virtual and erect image of your face.
interface as shown in the figure. PP’ is the principal axis;
µ1 and µ2 are the medium of refraction, respectively, (A) Converging mirror (B) Diverging mirror
then, (C) Converging lens (D) Diverging lens

2 1 Assertion Reasoning Type

P P
Q.21 Statement-I: If a source of light is placed in front
of rough wall, its image is not seen.
Statement-II: The wall does not reflect light.
(A) If µ2 > µ1 , then there cannot be a real image of a
real project. (A) Statement-I is true, and statement-II is true;
statement-II is correct explanation for statement-I
(B) If µ2 > µ1 , then there cannot be a real image of a
virtual object. (B) Statement-I is true, and statement-II is true; statement
-II is NOT the correct explanation for statement-I.
(C) If µ1 > µ2 , then there cannot be a virtual image of a
virtual object. (C) Statement-I is true, and statement-II is false.
(D) If µ1 > µ2 , then there cannot be a real image of a (D) Statement-I is false, and statement-II is true.
real object.
Q.22 Statement-I: As the distance x of a parallel ray
Q.18 A luminous point object is placed at O. whose from axis increases, the focal length decreases
image is formed at I as shown in the figure. AB is the
optical axis. Which of the following statements are
correct? X
(A) If a lens is used to obtain an image, the lens must
be converging. P A C Axis

(B) If a mirror is used to obtain an image, the mirror


must be a convex mirror having a pole at the point of
intersection of lines OI and AB. Statement-II: As x increases, the distance from the
pole to the point of intersection of a reflected ray with
(C) Position of the principal focus of mirror cannot be
the principal axis decreases.
found.
(A) Statement-I is true, and statement-II is true;
(D) I is a real image.
statement-II is correct explanation for statement-I.
P hysi cs | 16.77

(B) Statement-I is true, and statement-II is true; (A) Statement-I is true, statement-II is true, and
statement-II is NOT the correct explanation for statement-II is the correct explanation for statement-I.
statement-I.
(B) Statement-I is true, statement-II is true, and
(C) Statement-I is true, and statement-II is false. statement-II is NOT the correct explanation for
statement-I.
(D) Statement-I is false, and statement-II is true.
(C) Statement-I is true, and statement-II is false.
Q.23 Statement-I: When an object dipped in a liquid is (D) Statement-I is false, and statement-II is true.
viewed normally, the distance between the image and
the object is independent of the height of the liquid
Comprehension Type
above the object.
Statement-II: The normal shift is independent of Paragraph 1: Spherical aberration in spherical mirrors
the location of the slab between the object and the is a defect that is due to the dependence of focal length
observer. ‘f’ on the angle of incidence ' θ ' as shown in the figure
is given by
(A) Statement-I is true, statement-II is true, and
statement-II is the correct explanation for statement-I. R
f=
R− sec θ ,
2
(B) Statement-I is true, statement-II is true, and
statement-II is NOT the correct explanation for where R is radius of curvature of mirror and q is the angle
statement-I. of incidence. The rays that are close to the principal axis
are called marginal rays. As a result, different rays focus
(C) Statement-I is true, and statement-II is false. at different points and the image of a point object is
(D) Statement-I is false, and statement-II is true. not a point.

Q.24 Statement-I: When two plane mirrors are kept Q.26 If fp and fm represent the focal length of paraxial
perpendicular to each other as shown in the figure (O is and marginal rays, respectively, then the correct
the point object), three images will be formed. relationship is:
(A) fp = fm (B) fp > fm
(C) fp < fm (D) none

Q.27 If the angle of incidence is 60o, then the focal


O length of this ray is:
(A) R (B) R/2
(C) 2R (D) 0
Statement-II: In case of a multiple reflection, the image
of one surface can act as an object for the next surface. Paragraph 2: A student is performing Young’s double
(A) Statement-I is true, statement-II is true, and slit experiment. There are two slits S1 and S2. The
statement-II is the correct explanation for statement-I. distance between them is d. There is a large screen
at a distance D (D >>d) from the slits. The setup is
(B) Statement-I is true, statement-II is true, and shown in the following figure. A parallel beam of light
statement-II is NOT the correct explanation for is incident on it. A monochromatic light of wavelength
statement-I. λ is used. The initial phase difference between the two
(C) Statement-I is true, and statement-II is false. slits behave as two coherent sources of light is zero.
The intensities of light waves on the screen coming
(D) Statement-I is false, and statement-II is true. out of S1 and S2 are same, i.e. I0 . In this situation, the
principal maximum is formed at point P. At the point
Q.25 Statement-I: Keeping a point object fixed, if a on screen where the principal maximum is formed, the
plane mirror is moved, the image will definitely move. phase difference between two interfering waves is zero.
Statement-II: In case of a plane mirror, the distance
between a point object and its image from a given
point on mirror is equal.
1 6 . 7 8 | Geometrical Optics

point P (see figure). The value of α is


t (µ − 1) t (µ − 1)
Screen
S (A) sin−1 (B) cos−1
d d
O d P
S t (µ − 1)D tD
(C) sin−1 (D) sin−1
d d
D

Q.28 The total deviation suffered by the ray falling on Q.34 Match the Column
the mirror at an angle of incidence 60o is
Column I Column II
(A) 180o (B) 90o
(A) Conversing system (p) Convex lens
(C) Cannot be determined (D) None
(B) Concave lens (q) Concave lens
(C) A virtual image is formed by (r) Concave mirror
Q.29 For paraxial rays, focal length approximately is
(D) Magnification < 1 is possible (s) Convex mirror
(A) R (B) R/2
with
(C) 2R (D) none

Q.30 Which of the following statements are correct Previous Years’ Questions
regarding spherical aberration:
(A) It can be completely eliminated. Q.1 A student performed the experiment of
determination of the focal length of a concave mirror
(B) It cannot be completely eliminated, but it can be by u-v method using an optical bench of length 1.5
minimized by allowing either paraxial or marginal rays m. The focal length of the mirror used is 24 cm. The
to hit the mirror. maximum error in the location of the image can be 0.2
(C) It is reduced by taking mirrors with large aperture. cm. The five sets of (u, v) values recorded by the student
(in cm) are: (42, 56), (48, 48), (60, 40), (66, 33) and (78,
(D) None of these. 39). The data set (s) that cannot come from experiment
and is (are) incorrectly recorded, is (are) (1999)
Q.31 Initially, the distance of third minima from principal
(A) (42, 56) (B) (48, 48)
maxima will be
(C) (66, 33) (D) (78, 39)
3λD 3λD
(A) (B)
2d d
Q.2 A light beam travels from Region I to Region IV
5λD 5λD (See figure). The refractive index in Regions I, II, III and
(C) (D) n n
4d 2d IV are n0 , 0 and 0 , respectively. The angle of incidence
6 8
Q.32 A glass slab of thickness t and refractive index θ for which the beam misses entering Region IV (as in
µ is introduced before S2 . Now, P does not remain the figure): (2004)
the point of principal maximum. Suppose the principal
Region I Region II Region III Region IV
maximum forms at a point P’ on screen, then PP’ is
equal to n0 n0 n0
n0 
tD ( µ − 1 ) tD ( µ − 1 ) 2 6 8
(A) (B)
d 2d
0 0.2 m 0.6 m
D (µ − 1) D (µ − 1)
(C) (D) 3 1
t d (A) sin−1   (B) sin−1  
4 8
Q.33 Use the statement given in previous question. 1 1
Now, a parallel beam is incident at an angle α w.r.t. line (C) sin−1   (D) sin−1  
4
  3
OP, such that the principal maximum again comes at
P hysi cs | 16.79

Q.3 An optical component and an object s placed (C) The intensity of the characteristics X-rays depends
along its optical axis are given in column I. The distance on the electrical power given to the X-ray tube.
between the object and the component can be varied.
(D) Cutoff wavelength of the continuous X-rays depends
The properties of images are given in column II. Match
on the energy of the electrons in the X-ray tube.
all the properties of images from column II with the
appropriate components given in column I. Indicate
your answer by darkening the appropriate bubbles of
1 1
the 4 x 4 matrix given in the ORS. (2006) Air Air
(A) (B)
Meta-material Meta-material 2
Column I Column II 2

(A)

S (p) Real image

1 1
(B)
Air Air
S (q) Virtual image (C) (D)
Meta-material Meta-material
2 2

(C)

(r) Magnified image Paragraph: Most materials have a refractive index n >1.
S
Therefore, when a light ray from air enters a naturally
sin θ1 n2
occurring material, then by the Snell’s law, =
sin θ2 n2
(D) , it is understood that the refracted ray bends toward
S
(s) Image at infinity
the normal, but it never emerges on the same side
of the normal as the incident ray. According to the
electromagnetism, the refractive index of the medium is
c
given by the relation, n =   = ± ε1µ1 , where c is the
Q.4 Two beams of red and violet colors are pass v
separately through a prism (angle of the prism is 60o). speed of electromagnetic waves in vacuum, v is its speed
In the position of minimum deviation, the angle of in the medium, ε1 and µ1 are the relative permittivity
refraction will be (2007) and permeability of the medium, respectively. In a
normal material, both ε1 and µ1 are positive, implying
(A) 30o for both the colors positive n for the medium. When both ε1 and µ1 are
(B) Greater for the violet color negative, one must choose the negative root of n. Such
materials with negative refractive indices can now be
(C) Greater for the red color artificially prepared and are called meta-materials.
(D) Equal but not 30o for both the colors They exhibit a significantly different optical behavior,
without violating any physical laws. Since n is negative,
it results in a change in the direction of propagation
Q.5 Which one of the following statements is WRONG
of the refracted light. However, similar to the normal
in the context of X-rays generated from an X-ray tube?
materials, the frequency of light remains unchanged
 (2001)
upon refraction even in meta-materials. (2012)
(A) The wavelength of the characteristics X-rays
decreases when the atomic number of the target
increases.
(B) The cutoff wavelength of the continuous X-rays
depends on the atomic number of the target.
1 6 . 8 0 | Geometrical Optics

Q.6 For light incident from air on a meta-material, the (B) Real and at a distance of 16 cm from the mirror
appropriate ray diagram is  (C) Virtual and at a distance of 20 cm from the mirror
(D) Real and at a distance of 20 cm from the mirror

1 1 Q.10 The image of an object approaching a convex


mirror of a radius of curvature 20 m along its optical
(A) (B)
25 50
Air Air

Meta-material Meta-material axis moves from m to m in 30 s. What is the


2 3 7
2 speed of the object in km/h? (2011)

Q.11 A bi-convex lens is formed with two thin plano-


convex lenses as shown in the figure. Refractive index n of
1 1 the first lens is 1.5 and that of the second lens is 1.2. Both
(C)
Air
(D)
Air the curved surfaces are of the same radius of curvature R
Meta-material Meta-material
= 14 cm. For this bi-convex lens, for an object distance of
2 2 40 cm, the image distance will be –  (2012)

n = 1.5 n = 1.2
Q.7 Choose the correct statement.

(A) The speed of light in the meta-material is v= c n .

c
(B) The speed of light in the meta-material is v= .
n
(C) The speed of light in the meta-materials is v = c. R = 14 cm
(D) The wavelength of the light in the meta-material (A) – 280.0 cm (B) 40.0 cm (C) 21.5 cm (D) 13.3 cm
( λm ) is given by λm =λair n , where λair is the
wavelength of the light in air. Q.12 A transparent slab of thickness d has a refractive
index n (z) that increases with z. Here z is the vertical
distance inside the slab, measured from the top. The slab
Q.8 A biconvex lens is formed with two thin plano- is placed between two media with uniform refractive
convex lenses as shown in the figure. Refractive index n indices n1 and n2 ( > n1 ) , as shown in the figure. A ray
of the first lens is 1.5 and that of the second lens is 1.2. of light is incident with angle θ1 from medium 1 and
Both curved surfaces are of same radius of curvature emerges in medium 2 with refraction angle θf with a
R = 14 cm. For this biconvex lens, for an object distance lateral displacement l. (2016)
of 40 cm, the image distance will be (2009)
Which of the following statement (s) is (are) true?
n = 1.5 n = 1.2
1
n1 = constant 1

n(z)

z
d
R = 14 cm

Q.9 A biconvex lens of focal length 15 cm is in front of


n2 = constant l 2
a plane mirror. The distance between the lens and the
mirror is 10 cm. A small object is kept at a distance of

30 cm from the lens. The final image is  (2011)
(A) Virtual and at a distance of 16 cm from the mirror
P hysi cs | 16.81

(A) n1 sin
= θi n2 sin θf Q.16 A right angled prism of refractive index µ1, is
placed in a rectangular block of refractive index µ2,
(B) n1 sin θ=
i (n2 − n1 ) sin θf which is surrounded by a medium of refractive index
(C) l is independent of n2 µ3 , as shown in the figure. A ray of light ‘e’ enters the
(D) l is dependent on n (z) rectangular block at normal incidence. Depending
upon the relationships between µ1 , µ2 and µ3 , it takes
one of the four possible paths ‘ef’, ‘eg’, ‘eh’, or ‘ei’.
Q.13 A small object is placed 50 cm to the left of a thin
convex lens of focal length 30 cm. A convex spherical f
mirror of radius of curvature 100 cm is placed to the
right of the lens at a distance of 50 cm. The mirror is 45
o

tilted such that the axis of the mirror is at an angle e g


θ= 30° to the axis of the lens, as shown in the figure. 1
 (2015) i h
f = 30 cm
2 3


x
(-50, 0) (0, 0) Match the paths in list I with conditions of refractive
R = 100 cm indices in list II and select the correct answer using the
codes given below the lists: (2013)
50 cm

(50+503, -50) List I List II


P. 1. µ1 > 2 µ2
If the origin of the coordinate system is taken to be at e→ f
the centre of the lens, the coordinates (in cm) of the
point (x, y) at which the image is formed are (2016) Q. 2. µ2 > µ1 and µ2 > µ3
e→ g

(
(A) 25, 25 3 ) (
(B) 125 / 3, 25 / 3 ) R.
e→ h
3. µ1 = µ2

(C) (50 − 25 3 , 25 ) (D) (0, 0)


S. 4. µ2 < µ1 < 2 µ2 and
e→ i
Q.14 A ray of light travelling in the direction
1 ˆ
2
i + 3 ˆj( ) µ 2 > µ3

is incident on a plane mirror. After reflection, it travels

along the direction


2
(
1 ˆ
)
i + 3 ˆj . The angle of incidence Codes:

is (2013) P Q R S
A 2 3 1 4
(A) 30° (B) 45° (C) 60° (D) 75°
B 1 2 4 3
Q.15 The image of an object, formed by a plano-convex C 4 1 2 3
lens at a distance of 8 m behind the lens, is real and is
D 2 3 4 1
one-third the size of the object. The wavelength of light
2
inside the lens is times the wavelength in free space.
3
The radius of the curved surface of the lens is  (2013) Q.17 A transparent thin film of uniform thickness
and refractive index n1 = 1.4 is coated on the convex
(A) 1 m (B) 2 m (C) 3 m (D) 4 m
spherical surface of radius R at one end of a long solid
glass cylinder of refractive index n2 = 1.5 , as shown
in the figure. Rays of light parallel to the axis of the
cylinder traversing through the film from air to glass
1 6 . 8 2 | Geometrical Optics

get focused at distance f1 from the film, while rays of


List I List II
light traversing from glass to air get focused at distance
f2 from the film. Then (2014) R -r
n1

n2
Air
S r

(A) f1 = 3R (B) f1 = 2.8R

(C) f2 = 2R (D) f2 = 1.4R

Code:
Q.18 A point source S is placed at the bottom of a (A) P-1, Q-2, R-3, S-4
transparent block of height 10 mm and refractive index
2.72. It is immersed in a lower refractive index liquid as (B) P-2, Q-4, R-3, S-1
shown in the figure. It is found that the light emerging (C) P-4, Q-1,R-2, S-3
from the block to the liquid forms a circular bright spot
(D) P-2, Q-1, R-3, S-4
of diameter 11.54 mm on the top of the block. The
refractive index of the liquid is  (2014)
Q.20 Consider a concave mirror and a convex lens
Liquid (refractive index = 1.5) of focal length 10 cm each,
separated by a distance of 50 cm in air (refractive
Block index = 1) as shown in the figure. An object is placed
S at a distance of 15 cm from the mirror. Its erect image
(A) 1.21 (B) 1.30 (C) 1.36 (D) 1.42 formed by this combination has magnification M1 .
When the set-up is kept in a medium of refractive index
Q.19 Four combinations of two thin lenses are given in 7/6, the magnification becomes M2 . The magnitude
list I. The radius of curvature of all curved surfaces is r M2
is (2015)
and the refractive index of all the lenses is 1.5. Match M1
lens combinations in List I with their focal length in
list II and select the correct answer using the code given
below the lists.  (2014) Q.21 Two identical glass rods S1 and S2 (refractive
index = 1.5) have one convex end of radius of curvature
10 cm. They are placed with the curved surfaces at a
List I List II
distance d as shown in the figure, with their axes (shown
P 2r by the dashed line) aligned. When a point source of
light P is placed inside rod S1 on its axis at a distance of
50 cm from the curved face, the light rays emanating
from it are found to be parallel to the axis inside S2 .
The distance d is  (2015)

Q r/2
S1 P S2

50 cm d

(A) 60 cm (B) 70 cm
(C) 80 cm (D) 90 cm
P hysi cs | 16.83

Q.22 A monochromatic beam of light is incident at (C) NA of S1 placed in air is the same as that of S2
60° on one face of an equilateral prism of refractive 4
index n and emerges from the opposite face making an immersed in liquid of refractive index .
15
angle θ (n) with the normal (see the figure). For n = 3
dθ (D) NA of S1 placed in air is the same as that of S2 placed
the value of θ is 60° and = m. The value of m is in water.
dn
 (2015)
Q.24 A parallel beam of light is incident from air at an
angle α on the side PQ of a right angled triangular
prism of refractive index n = 2 . Light undergoes total

internal reflection in the prism at the face PR when α
o
60
has a minimum value of 45° . The angle θ of the prism
is  (2016)
P



Paragraph 1: Light guidance in an optical fiber can
be understood by considering a structure comprising
of thin solid glass cylinder of refractive index n1
n =2
surrounded by a medium of lower refractive index n2.
R
The light guidance in the structure takes place due to Q
successive total internal reflections at the interface of
the media n1 and n2 as shown in the figure. All rays (A) 15° (B) 22.5° (C) 30° (D) 45°
with the angle of incidence i less than a particular value
im are confined in the medium of refractive index n1. Q.25 A plano-convex lens is made of a material of
The numerical aperture (NA) of the structure is defined refractive index n. When a small object is placed 30
as sin im cm away in front of the curved surface of the lens, an
image of double the size of the object is produced.
n1 > n2 Due to reflection from the convex surface of the lens,
another faint image is observed at a distance of 10 cm
Air
away from the lens. Which of the following statement(s)
Cladding n2


is(are) true?  (2016)
Core
i n1 (A) The refractive index of the lens is 2.5
(B) The radius of curvature of the convex surface is
45 cm
(C) The faint image is erect and real
(D) The focal length of the lens is 20 cm
Q.23 For two structures namely S1 with n1 = 45 / 4
and n2 = 3 / 2 and S2 with n1 = 8 / 5 and n2 = 7 / 5
and taking the refractive index of water to be 4/3 and
that of air to be 1, the correct option(s) is(are)  (2015)

(A) NA of S1 immersed in water is the same as that of


16
S2 immersed in a liquid of refractive index
3 15
6
(B) NA of S1 immersed in liquid of refractive index
is the same as that of S2 immersed in water 15
1 6 . 8 4 | Geometrical Optics

PlancEssential Questions
JEE Main/Boards JEE Advanced/Boards
Exercise 1 Exercise 1
Q.11 Q.12 Q.24 Q.3 Q.4 Q.8
Q.13 Q.22 Q.23

Exercise 2 Exercise 2
Q. 3 Q.5 Q.7 Q.6 Q.9 Q.11
Q.8 Q.10 Q.18 Q.24 Q.25 Q.30
Q.23 Q.27 Q.37 Q.31 Q.33

Answer Key

JEE Main/ Boards


Exercise 1
Q.1 2.12 × 108 ms−1 Q.2 u=−30 cm
8
3 × 10
Q.4 m / s Q.5 The image is formed on the same side of object.
2
Q.9 (b) (i) When the refractive index of the medium increases, the resolving power increases.
(ii) When the wavelength of the radiation increases, the resolving power decreases.
Q.11 For convex lens +7.5 cm, for concave lens +48 cm.
Q.12 A virtual image of height 10 cm is formed at a distance of 25 cm from the lens on the same side of the object.
Q.14 The angle of deviation is decreased.
Q.16 (i) +5 D; (ii) 10 cm Q.17 25 cm Q.20 feq = ∞
Q.21 (ii) -10 Q.23 (a) 5 Q.24 2.14
Q.25
= f0 40cm;
= fe 5cm

Exercise 2

Single Correct Choice Type


Q.1 A Q.2 B Q.3 A Q.4 C Q.5 B Q.6 A
Q.7 D Q.8 D Q.9 A Q.10 A Q.11 C Q.12 D
P hysi cs | 16.85

Q.13 B Q.14 B Q.15 B Q.16 C Q.17 D Q.18 D


Q.19 C Q.20 B Q.21 C Q.22 A

Previous Years’ Questions


Q.1 D Q.2 A Q.3 A → p, r; B → q, s, t; C → p, r, t; D → q, s
Q.4 A,B,C Q.5 6 Q.6 D Q.7 C Q.8 B Q.9 D
Q.10 D Q.11 B Q.12 B Q.13 D Q.14 D Q.15 D
Q.16 A Q.17 D

JEE Advanced/Boards
Exercise 1
Q.1 (i) 75o (ii) 165° (iii) 195° (iv) 285° (v) 315°
Q.2 160 cm; 320 cm Q.3 75 cm Q.4 2 cm Q.5 80 m/s Q.7 16 ft
dy
(iii) y = k 2 ( x / 4 ) (iv) 4.0,1;
4
Q.8 (i) tan=
θ = cot i (ii)1
dx
(v) It will become parallels to x-axis Q.9 5 cm Q.10 h=5.95 m Q.11 Same Q.12 1.625
Q.13 1000 m/s Q.14 µ12 + µ32 + µ52 = 2 + µ22 + µ24 Q.15 9 m

1 d 1 43
Q.16 (a) sin−1   (b) air Q.17   = Q.18 45° Q.19
5  R max 2 5
Q.20 3 Q.21 15 cm toward the combination
2 2
Q.22 (a) 5000 W / m (b) 0.02 W / m (c) 0.214 cm (d) 24.56 W / m2
Q.23 +60, +4/5 Q.24 5.9 cm, 10.9 cm
Q.25 90 cm from the lens toward right

Exercise 2

Single Correct Choice Type

Q.1 B Q.2 A Q.3 C Q.4 D Q.5 B Q.6 D


Q.7 B Q.8 A Q.9 A

Multiple Correct Choice Type


Q.10 B, D Q.11 B, C Q.12 B, C Q.13 A, C Q.14 A, B, D Q.15 B, C
Q.16 B, D Q.17 A, C Q.18 A, D Q.19 A, C Q.20 B, D

Assertion Reasoning Type


Q.21 C Q.22 A Q.23 D Q.24 D Q.25 D

Comprehension Type
Q.26 B Q.27 D Q.28 D Q.29 B Q.30 B Q.31 D
Q.32 A Q.33 A
1 6 . 8 6 | Geometrical Optics

Match the Column


Q.34 A → p, r; B → q, s; C → p, q, r, s; D → p, q, r, s

Previous Years’ Questions


Q.1 C, D Q.2 B Q.3 A → p, q, r, s; B → q; C → p, q, r, s; D → p, q, r, s
Q.4 A Q.5 B Q.6 C Q.7 B Q.8 B Q.9 B
Q.10 3 Q.11 B Q.12 A, C, D Q.13 A Q.14 A Q.15 C
Q.16 D Q.17 A, C Q.18 C Q.19 B Q.20 7 Q.21 B
Q.22 2 Q.23 A, C Q.24 A Q.25 A, D

Solutions

JEE Main/Boards (ii) Focal length of a concave lens depends on the


refractive index µ of the medium which in-turn
depends upon the wavelength of light. µ decreases with
Exercise 1 increasing wavelength. So for red light µ will decrease.
1  1 1 
 A + δm   90  1 As per lens maker’s formula  = (µ − 1)  −   as
sin   sin  
2 2  f  R1 R 2  
Sol 1:=
µ  =   =  = 2 2
A  60  1 / 2 µ decrease, f increase.
sin sin  
2  2 
Here A = 60 , δm = 30 .
o o Sol 4: Total internal reflection (TIR) takes place when
light travels from denser medium towards rarer medium
c c 3 × 108 and at the interface the angle of incidence exceeds θc,
Now μ = ⇒v= ⇒v= = 2.12×108 ms-1
v µ 2 the critical angle, and the incident beam is completely
reflected at the boundary (interface). Critical angle
1 1 1  µ 
Sol 2: Lens Formula: − =. Let object is θc =sin−1  Rarer 
v u f µ
 Denser 
placed at distance x from lens and image is found
1 1 1 c
at distance y from lens. For real image v is positive 45= sin−1   ⇒ = ⇒ µ= = 2
µ
  µ 2 v
v +y
v= + y, u = − x, m = == −2 (For real image m
u −x c 3 × 108
1 1 1 1 1 ⇒ v= = = 2.12 × 108 ms−1
is negative) ⇒ y = 2x …..(i) − = − = (f is 2 2
v u y −x 20
positive for convex lens)
Sol 5: Lens formula.
1 1 1 3 1 1 1 1
⇒ + = (using (1)) ⇒ = ⇒ x= = 30 cm − (for concave lens f is ( − ) ve)
2x x 20 2x 20 v u f
1 1 1
u=-30 cm Now for u = − f, − =
v −f −f
1 1 1 2 f
Sol 3: (i) Focal length of a concave mirror is independent =− − =− ⇒ v =−
v f f f 2
of the medium and wavelength of light. So there will
not be any change. Image is virtual, diminished and on the same side as
object.
P hysi cs | 16.87

Sol 6:  Putting these expressions with proper signs, in eqn. A,


n1 n2n2 − n1
we get + =
m −u v R1

1  1 1 
Lens Maker’s Formula: (n− 1) 
= − 
f  R1 R 2 
L1
L1=L2 As wavelength of light increases, the refractive index n
r1=r2 decreases and from the lens maker formula we see that,
as n decreases, f increases.
The variation of angle of deviation δ with the angle
of incidence i of the ray incident on the first refracting
surface of the prism is shown in figure. For one angle of Sol 9: (i) Magnifying power:
incidence it has a minimum value δmin . At this value the
ray passes symmetrically through the prism. vD 
m=   for normal adjustment
u  fe 
Sol 7: For relaxed eye, intermediate image should lie at v D
first focus of eye piece or ue = fe
m
=  1 +  for final image at D, least distance for
u fe 
B f0 fe clear vision.
1 2µ sin θ
(ii) Resolving Power: =
R =
∆d λ
Fo Fe
A O A’ F’e E
µ → Refractive index of the medium between the
B’ object and the objective.
λ → Wavelength of light.

f0 Eye piece
Magnification M∞ = Vo Uo
fe
uo fo
Sol 8: Reflection from a spherical surface: Here n1 < n2 . h F
Ray leaves point O and focuses at point I. Snell’s Law at ’
point P n1 sin= θ1 n2 sin θ2 or n1 θ1 = n2 θ2 (For small
h’

angles)
Objective

P
Q₁
 B Q₂ 
O M C I Image
n₁ n₂
Here we see that
u R
(i) as µ increases, R increases.
V

From geometry of figure


Q

θ1 = α + β, β = θ2 + γ
 P’ E
n1 P P’’ O ’ 
Eliminating θ1and θ2 we get =
β ( α + β) + γ
n2 O’

Or n1 α + n2=
γ (n2 − n1 ) β ……….(A)
S
Now angle at C is=
β =(S arc(PM))
R
S S Q’’
Also in paraxial approximation
= α and
= γ
u v
(ii) as λ increases, R decreases.
1 6 . 8 8 | Geometrical Optics

Sol 10: Refracting astronomical telescope: It consists of Sol 14: Angle of minimum deviation δm and angle of
an objective lens of a large focal length (fo) and large
aperture, also an eye lens of small aperture and focal  A + δm 
sin  
length.  2 
prism A are related as, µ =
(i) Magnification when final image is formed at D, A
sin  
fo  fe  2
⇒m=−  1 +  and length of telescope, Glass prism of refractive index 1.5 is immersed in a
fe  D
liquid of refractive index 1.3 so the relative refractive
fe D 1.5
L = |fo| + index of the prism decreases. µ=' = 1.15
fe + D 1.3
So as per above equation as A is constant for a prism,
Sol 11: (i) Lens formula as µ decreases, δm also decreases.

1 1 1
− = Sol 15: [Refer question 7 solution]
v u f
u=+12 cm, f =
+20 cm
Sol 16: Consider an object O placed at a distance u
1 1 1 1 1 1 1 1 8
⇒ = + = + =  + = from a convex lens as shown in figure. Let its image I
v u f 12 20 4  3 5  4 × 15 after two refractions from spherical surfaces of radii R1
⇒v=+7.5 cm (positive) and R2 (negative) be formed at a distance v
from the lens. Let v1 be the distance of image formed
1 1 1 4 −3 1
(ii) f = −16 cm ⇒ = − = = by refraction from the refracting surface of radius R1.
v 12 16 48 48 This image acts as an object for the second surface.
⇒v=+48 cm Using,

µ2 µ1 µ2 − µ1
1  1 1  − =
Sol 12: = (µ − 1)  −  v u R
f  R1 R 2  Incident light

Here R1 = +30 cm ;R 2 = −30 cm; µ = 1.6

1  1  2 1 R₂ R₁
⇒ = (0.6 )   × 2= = ⇒ f + 25 cm
f 30
  50 25
O C₂ 1 1 C₁ I
1 1 1 1 1 1 2
u=12.5 cm, = + = − = −
v f u 25 12.5 25
h2 v −25
⇒v= −25 cm , m = = = = 2 +ve
h1 u −12.5
u v
⇒ h2 =2h1 =2 × 5 cm =10 cm
µ2 µ1µ2 − µ1
Image is virtual and erect, on the same side as the twice, we have − = 
....(i) ... (i)
v1 u R
object.
µ1 µ2 µ1 − µ2
and − = ....(ii) ... (ii)
v v1 −R 2
Sol 13: Predominance of bluish colour in a clear sky
is due to the phenomena of scattering of light in the Adding Eqs. (i) and (ii) and then simplifying, we get
atmosphere around earth. If size of the air particles
are smaller than the wavelength, the scattering is 1 1  µ2  1 1 
− =  − 1  −  .....(iii) ... (iii)
proportional to 1 / λ 4 . This is the Rayleigh’s law of v u 
scattering. The light of short wavelengths are strongly  µ1  R1 R 2 
scattered by the air molecules and reach the observer. This expression relates the image distance v of the
image formed by a thin lens to the object distance u
Among the shorter wavelengths, the colour blue is
and to the thin lens properties (index of refraction and
present in large proportion in sunlight.
radii of curvature). It is valid only for paraxial rays and
P hysi cs | 16.89

only when the lens thickness is much less then R1 and Sol 19: (a) (i)
R2. The focal length f of a thin lens is the image distance Q
that corresponds to an object at infinity. So, putting
u= ∞ and v = f in the above equation, we have
 P’ E
P P’’ O ’ 
1  µ2  1 1 
= − 1 
 −  ....(iv) O’
f  µ1  R1 R 2 
If the refractive index of the material of the lens is µ
and it is placed in air, µ2 =µ and µ1 =1 so that Eq. (iv)
becomes
Q’’
1  1 1  (ii) Reflector telescope advantages:
= (µ − 1)  −  ....(v)
f  R1 R 2  1. Reflector telescopes do not suffer from chromatic
This is called the lens maker’s formula because it can aberration because all wavelengths will reflect off the
be used to determine the values of R1 and R2 that are mirror in the same way.
needed for a given refractive index and a desired focal 2. Support for the objective mirror is all along the back
length f. side so they can be made very BIG.
Combining Eqs. (iii) and (v), we get 3. Reflector telescopes are cheaper to make than
1 1 1 refractors of the same size.
− = ....(vi) which is known as the lens formula.
v u f 4. Because light is reflecting off the objective, rather
(i) P = P1 + P2 = 10D − 5D = 5D than passing through it, only one side of the reflector
telescope’s objective needs to be perfect.
1 1 v
(ii)f = = =0.2m =20 cm; m =+ 2= (b) f1 = +10 cm, f2 = -10 cm, u = -30 cm.
P 5D u
⇒v= 2u 1 1 1 1 1 2
= + = − = ⇒ v1 =15 cm
v1 f1 u1 10 30 30
For virtual image m is positive
So for the concave lens
1 1 1 1 1 1
= + ⇒ − =
v f u 2u u 20 u2 =
+(15 − 5) cm =
+10 cm
1 1
⇒− = ⇒ u = −10 cm B f0 fe
2u 20

Sol 17:
Fo Fe
1 1 O A’ F’e E
P = P1 + P2 = 6 − 2 = 4D ⇒ f = = = 0.25m A
P 4D B’
⇒f= 25cm

Sol 18: The speed of light in vacuum is a universal 


constant denoted by c. When a light wave travels in a 1 1 1 1 1
transparent material, the speed is decreased by a factor =+ = + 0 ⇒ v2 =
= ∞
v 2 f2 u2 −10 10
µ , called the refractive index of the material.
speed of lightin vacuum Final image will be formed at infinity.
µ=
speed of light in the material 1 1 1 1 1
Sol 20: = + = − =0 ⇒ F =∞
For graph refer figure of question 6. F f1 f2 f f

Sol 21: (i) Astronomical Telescope for normal


adjustment
It consists of two converging lenses placed coaxially.
1 6 . 9 0 | Geometrical Optics

The one facing the distant object is called the objective eyepiece. The final image is then formed at infinity.
and has a large aperture and a large focal length. The It is erect with respect to the first image and hence,
other is called the eyepiece, as the eye is placed close inverted with respect to the object. The eye is least
to it. It has a smaller aperture and a smaller focal length. strained in this adjustment as it has to focus the parallel
The lenses are fixed in tubes. The eyepiece tube can rays coming to it. The position of the eyepiece can also
slide within the objective tube so that the separation be adjusted in such a way that the final virtual image is
between the objective and the eyepiece may be formed at the near point. The angular magnification is
changed. increased in this case. The ray diagram in figure refers
to this case.
Eyeplece
vo ue
(ii) Magnifying power of a Compound microscope is
vo fo
v D 
− 0   → normal adjustment and
m=
h F u0  fe 
’
h’
v  D
Objective − 0  1 +  → final image at D.
m=
u0  fe 

Now for large magnification, m is to be large, so fe


should be small and u0 should be small. Now object
Image is placed at a distance u0 from the objective which is
slightly greater than its focal length fo. So for u0 to be
When the telescope is directed towards a distant object
small, fo should also be small.
PQ, the objective forms a real image of the object in its
focal plane. If the point P is on the principal axis, the
image point P’ is at the second focus of the objective. Sol 22: Refer question 19.(a).(ii)
The rays coming from Q are focused at Q’. The eyepiece
forms a magnified virtual image P”Q” of P’Q’. This Sol 23: When two plane mirrors are placed at an angle
image is finally seen by the eye. In normal adjustment, θ to each other, the object is kept between them, then
the position is so adjusted that the final image is 360ο
formed at infinity. In such a case, the first image P’Q’ is the numbers of images observed is n = . If n is
θ
formed in the first focal plane of the eyepiece. The eye
even then number of image is (n-1).
is least strained to focus this final image. The image
can be brought closer by pushing the eyepiece closer 360ο 360ο
So here = = 6⇒ n = 6 − 1 = 5
to the first image. Maximum angular magnification is θ 60
produced when the final image is formed at the near (b) At each reflection some of the light energy is lost
point. due to absorption at the mirror surface. So the intensity
f p 10D of the reflected ray goes on decreasing at multiple
(ii) m =− 0 =− e =− =−10 reflections due to parallel mirrors.
fe pο 1D
Here the objective has large focal length and smaller v D 
Sol 24: For compound microscope m = − ο  
Power. uο  fe 
(normal adjustment)
(b) (i) Figure shows a simplified version of a compound
microscope and the ray diagram for image formation.  v   25 cm  v ο −60 −12
−20 =  − ο    ⇒− = = = −2.4
It consists of two converging lenses arranged coaxially.  u
 ο   3cm  uο 25 5
The one facing the object is called the objective and
the one close to the eye is called the eyepiece or ocular. For final image at least distance:

The object is placed at a distance u0 from the objective v  D v  25 


− ο  1 +  ⇒ − 20 = − ο  1 +
m= 
which is slightly greater than its focal length f0 . A real 
uο  
fe  uο  3 
and inverted image is formed at a distance v 0 on the
other side of the objective. This image works as the vο −20 −5 × 3 v
object for the eyepiece. For normal adjustment, the ⇒− = = ⇒ − ο = −2.14
uο 28 / 3 7 uο
position of the eyepiece is so adjusted that the image
formed by the objective falls in the focal plane of the
P hysi cs | 16.91

−fο left with speed 3v. So its second image will more away
Sol 25: Astronomical telescope =
m → normal towards right with speed 3v w.r.t mirror 2. Hence speed
fe
adjustment w.r.t O will be 3v+v=4v
fo Hence for nth image v image = 2nv
⇒ −8 = − ⇒ fo = 8fe
fe

L = fο + fe = 45 cm ⇒ 8fe + fe = 45 cm h 3h
Sol 3: (A) tan θ= = lim
⇒ fe 5cm=
= and fe 40 cm y 10 h + y x →∞

⇒ 10h + y= 3y ⇒ y = 5h
Exercise 2 For general case
L₁
Single Correct Choice Type
M₂
3h
Sol 1: (A)
L₂ M₁ h v

δ1 180o − 2θ; =
= δ2 180o − 2=
θ ' 70o + 90o − θ ' S
10h = x y

3h h
o
70 tan θ = = ⇒ 3y = x + y ⇒ 2y = x
x +y y

dy dx dy 1
N’ ⇒ 2. = ⇒ = .v
90 -’ dt dt dt 2
o

’ 2
1
o
70
N
 ’ Sol 4: (C) A is head and E is feet of man. C is the eye.
The mirror can be placed anywhere between the centre

line BF (of AC) and DG (of CE) to get full image from

head to feet.
So here CE=1.4 m. So DE should be 0.7 m. But mirror is
⇒ θ ' =20 ο 0.8 m from ground so feet will not be visible. The upper
edge of mirror is at height (0.8+0.75) m equal to 1.55 m
α = 180o − 70o − (90o − θ ') = 180o − 70o − 70o which is more than BE.
α 40o ⇒=
= θ 90o −=
α 50o
A
x
Sol 2: (B) With respect to mirror1 the object is going B
F
array from mirror. So first image will also more away x
C
w.r.t mirror 1 with same speed v. So with respect to (x + y)
object O the image speed is y

D
1 2 G
y
E
O Man

AC 0.1
BE = BC + CE = + CE = + 1.4
2 2
= 0.05 + 1.4 = 1.45m

So head will be visible.


v image1 = v image1,mirror1 + v mirror 1,obj = v + v = 2v

Now this image becomes object for mirror 2. With


respect to mirror 2 the image is going away towards
1 6 . 9 2 | Geometrical Optics

Sol 5: (B) Rays from S going right from the normal will When the mirror is present
not reach the bottom horizontal mirror as they will hit
the inclined mirror and get deviated. So C will see only Pθ12 h P
Intensity = and tan θ = ⇒=
the image formed by inclined mirror. 4 πh1 2 20 4 π × (20)2

Intensity when mirror present


Sol 6: (A) Object O moves towards M1 so image 1 due Ratio =
Intensity when mirror object
to M1 will move towards left i.e. towards M2 .
For M2 we have formula for speed of image as P P
+2
 v 2  du 4 π × (20) 4 π × (60)2 10
dv = =
= −   . So negative sign means final image P 1
dt  u2  dt
  4 π × (20)2
2 due to M2 will move opposite to image 1 i.e. towards
right. Sol 8: (D) As a result of water the apparent height of
source will be beyond C, at C’. OC’ = Rμ. So its image I’
Sol 7: (D) Intensity incident will not be formed at C but it will be formed between
C and focus F of the mirror. But again in the return
Pθ2
= path of rays they will be again refracted at water to
4 × (area on which lightis incident)
air boundary and final image I will be further shifted
When the mirror is not present, light is reaching the downwards towards O.
screen up to height h. Maximum area on which light is
incident = πh2
C
Pθ2
Intensity = and
4 πh2 I’
h Ph 2 I R
P
tan θ = ⇒
= = F
60 2
4 πh × (60)2
4 π × (60)2


O

 Sol 9: (A) The slab will cause a lateral shift in the
incident rays as well as in the reflected rays from the
circular mirror MM’. Now the angle of emergence θ1
will be equal to the angle of incidence in case of a slab.

M M’
h
 C 1 1 B
s  60
h

D A

1
P Q O
h
 The rays reaching the edge M of the circular mirror
F after passing through the glass slab will be leaving the
20 source O at a greater angle (θ1 ) with the normal as
compared to the angle (θ) when there is no slab. But
due to symmetry of incident and reflected rays, the
reflected rays from the edge M, after passing through
the slab will reach the some point Q on the ground
where they would have reached when there was no
slab.
P hysi cs | 16.93

Here we have, OQ=QP, both without and with slab, r


between source O and mirror MM’.

h
Sol 10: (A) B E

c


90 -
o

O
Ω = 2π(1 − cos θc )
3 6
n= n=

2 5 If total intensity is I then, intensity per unit solid angle is
I
A C D .So intensity through the circular area is,
4 π.
6 /5
90 − θ ≥ θc = sin−1  ; I
3/2 I
= .2π (1 − cos θc )

4
90 − θ ≥ sin−1   ;
5 I' (1 − cos 48.59 ) I
ο ο =⇒ = ⇒ 16.9 %
90 − θ ≥ 53 ⇒ θ ≤ 37 I 2 I

1 1 1 1 1 u1 − f
Sol 13: (B) = + = − =
Sol 11: (C) v1 f ( − u1 ) f u1 f u1
1 f u1 f u2
=h 4m ; θc = sin−1   =v1 = and v 2
µ u1 − f u2 − f
 1  −1  3  v1
= sin−1=
  sin
=   37
ο v −f
Now m1= = =
5 / 3 5 u −u1 u1 − f
r 3h 3 × 4m −f
tan θc = ⇒ r = htan 37ο = = and m2
= = .also | m1 | | m2 |
h 4 4 u2 − f
r r
Now m1 is negative (real image) and m2 is positive
(virtual image). So we have,
h f −f
= ⇒ u2 − f =−u1 + f
c u1 − f u2 − f
c
u1 + u2
⇒ u1 + u2 = 2f ⇒ f =
2
O
⇒ diameter = 6m
2r = Sol 14: (B) The image formed by first lens will lie at its
second lens focus. This image will act as an object for
the second lens. For the rays to become parallel after
−1 1 3 passing through the second lens, the object for second
Sol 12: (D)
= θc sin= sin−1=
  48.59
ο
µ 4
  lens should lie on its first focus. Thus the distance
between the two lenses will be equal to sum of their
Solid angle subtended at source of light O by the
focal lengths.
circular area of radius r is
D = f1 + f2 = 20 cm + 10 cm = 30 cm
1 6 . 9 4 | Geometrical Optics

Sol 15: (B) Image formed by lens be at distance v1 from Speed of image with respect to object is
1 1 1 1 1 3−2 1 v I,O
= v I,L + vL,O
lens. = + = + = =
v1 f1 u1 10 −15 30 30
 dv 
=v I,O   + ( − v)
v1 = 30 cm from lens.  dt (towards left) (towards right)

Sol 18: (D) At first refracting surface we have,


sini1 = µ sin r1 . So as i1 decreases, r1 also decreases.
O
Now for prism r1 + r2 =A (constant). So as r1 decreases,
10 cm r2 increases. At the second refracting surface we have,
µ sin r2 =sin i2 . So as r2 increases, i2 also increases. So
out of all choices D is most appropriate as amount of
For convex lens, u2 = +(30 - 10) cm = +20 cm
increase in i2 should be less than amount of decrease
v2= +20 cm, because rays retrace their path after in i1.
reflection.
1 1 1 1 1 1 1 Sol 19: (C) For prism with refracting angle A, we have
+ =⇒ = + = ⇒ f2 =+ 10 cm
v 2 u2 f2 f2 20 20 10
 A + δm 
sin   o
= µ  2=  ⇒ 3 sin (90 + δm ) / 2
Sol 16: (C) For first lens, convergent ray becomes A 2 sin 45o
parallel to principal axis after refraction. sin
2
So f1 = +5 cm. 3  90o + δ  90o + δ
=
⇒ sin  m 
⇒= m
60 o
For second lens, ray parallel to principal axis becomes 2  2  2
 
convergent and parallel to incident ray.
⇒=δm 120o −= 90o 30o


h

h
 A + δm A + 38ο
Sol 20: (B) At min deviation
= im =
X 2 2

5 cm Now δ= (i1 + i2 ) − A; Here i1= 42o


, δ 44o
i2 62ο=
and=
So focal will length of second lens will be x as shown ⇒ 44 ο= (42ο + 62ο ) − A ⇒ A= 104 ο − 44 ο= 60o
in figure.
h h 60o + 38o
tan θ = = ⇒ x = 5 cm ⇒ f2 = x = 5 cm ⇒ im
= = 49ο
5 x 2

Sol 17: (D) Let as work in the frame of reference Sol 21: (C)

1 1 1 M
attached to the lens. Lens formula: − =
v u f
Differentiating w.r.t. time,
dv du
− v −2 − ( −u−2 ) =0 ; (f is constant) I
x O
dt dt P

dv  v 2  du δ = (µ − 1) A; µ = 1.5
⇒ =  
dt  u2  dt
o 2.5 5 π
δ 0.5 × 5=
= 2.5o ⇒ =
δ =
π =
π
Initially when u=f, v → ∞ so speed image is very large 180 360 72
and finally when u → ∞ , v → f and the speed of image
OP ≈ OM ≈ IM = 10 cm; ∆x = δ × (OM) = 10 δ cm
is very low (nearly zero). With respect to lens, as object
moves left, the image also moves left.
P hysi cs | 16.95

π 5π Sol 4: (A, B, C) Using Snell’s law


∆x= × 10 cm= cm
72 36
B
O o
60
Sol 22: (A) At second refracting surface o
A P 135
µ sin r2 =
1sin 90 C
o
1 1 60
o 45
⇒ sin r2 ==
µ 2
90o
45ο
⇒ r2 =  i r1 r2
o 75o
30
⇒ r1 = A − r2 = 2
A
ο ο ο
= 60 − 45 = 15

⇒ At first refracting surface , sin i1 =


µ sin r1
3 −1 3 −1
⇒ sin
= i1 15ο
2 .sin = 2. = 1 1
2 2 2 sin–1 < sin–1
3 2
 3 −1
sin−1 
⇒ i1 = . Net deviations is 90º
 2 
 

Sol 5:
Previous Years’ Questions
1 1 1
− =
v u f
1 1 1
Sol 1: – = = constant u u u u+ f 
v u f or − 1 = or = 
v f v  f 
Sol 2: (A) An experiment is performed to find the
v  f 
refractive index of glass using a travelling microscope. ∴ m ==  
In this experiment, distances are measured by a vernier u u+ f 
scale provided on the microscope.
 20 
m25  −25 + 20 
Sol 3: (A) → since µ1 < µ2 , the ray of light will bend = = 6
m30  20 
towards normal after first refraction.  
 −50 + 20 
(B) → µ1 > µ2 , the ray of light will bend away from the
normal after first refraction. ∴ Answer is 6.
(C) → µ2 =µ3 means in second refraction there will be
no change in the path of ray of light. Sol 6: (D) Case I: u = – 240 cm, v = 12, by lens formula

(D) → Since µ2 > µ3 , ray of light will bend away from the 1 7
=
normal after second refraction. f 80

Therefore the correct options are as under. 1 35


Case II: v = 12 – =
3 3
(A) → p, r
2 1
(normal shift = 1 – = )
(B) → q, s, t 3 3
(C) → p, r, t 7
f=
(D) → q, s 80

u = 5.6
1 6 . 9 6 | Geometrical Optics

1 1  1 
Sol 7: (C) L.C =
60 f
= ( µ − 1)  R1 − 
R 2 
 1
9
Total Reading = 585 + = 58.65
60 1 3  1 
=  − 1  
f 2   15 
Sol 8: (B) As intensity is maximum at axis.
f = 30 cm
\ µ will be maximum and speed will be minimum on
the axis of the beam.
Sol 13: (D)
fm
=
(µ − 1)
∴Beam will converge.
f  µ 
 − 1 
µ
 m 
Sol 9: (D) For a parallel cylindrical beam, wave front will
be planar. 3 
f1  − 1
2
 = 
=
⇒ 4
Sol 10: (D) Case I: u = -240 cm, v = 12, by Lens formula f 3/2 
 − 1
1 7 4/3 
=
f 80
⇒ f1 =
4f
1 35
Case II: v = 12 − =
3 3 3 
 − 1
f2 2
 2 1
 Normal shift = 1- =  =   = −5
 3 3 f 3/2 
 − 1
 5 / 3 
7
f=
80 ⇒ f2 < 0
u = 5.6

Sol 14: (D) As frequency of visible light increases


Sol 11: (B) Self-explanatory refractive index increases. With the increase of
refractive index critical angle decreases. So that light
Sol 12: (B) having frequency greater than green will get total
internal reflection and the light having frequency less
R 2 = d2 + (R − t )
2
than green will pass to air.

2
 t Sol 15: (D) At face AB,
R 2 − d2= R 2 1 − 
 R  sin θ = µ sin r 

At face ACr ' < θc A


R 
d r r’
1
A − r < sin−1 B C
t µ
R-t
1
∴ r > A − sin−1
µ
 1
∴ sin r > sin  A − sin−1 
d2 2t  µ
1− =1−
R 2 R
sin θ  1
> sin  A − sin−1 
(3) =
2
90 µ  µ 
R
= = 15 cm
2 × ( 0.3 ) 6
  1 
θ > sin−1 µ sin  A − sin−1  
  µ 
P hysi cs | 16.97

Sol 2:
Sol 16: (A) θ =1.22 λ
D

(
Minimum separation = 25 × 10−2 =
θ 30 µm )
Sol 17: (D) δ = i + e − A ⇒ A = 74°
120 cm

 A + δmin  A G
sin   D
 2  5 sin  37° + δmin 
=µ =  
A 3  2  40 cm 40 cm
sin  
2  F
B  C
5 5
µmax can be , so µ will be less than 40 cm
40 cm
3 3 
Since δmin will be less than 40° , so P
S
5 5
µ< sin 57° < sin 60° ⇒ µ < 1.446 20 cm 40 cm
3 3
By similar triangles
So the nearest possible value of µ should be 1.5
∆ BCD  ∆ BFE; so EF =
3 × CD
Because BF= 3 ×BC

= EF 120 cm ⇒
= EP 160 cm
JEE Advanced/Boards
∴ Minimum height of eye is 160 cm.
Exercise 1 And similarly maximum height will be
E'P = 80 + 3 × 80 = 320cm
 3 −1 3 −1
Sol
= 1: OB OP
= sin15 ο
2=
 
 2 2  2
  Sol 3: By property of similar triangles,
Number of image
M P
y 20 cm
B
 
O
(1, 1) x
120
o
x
3m
B S
o
60 A 20 20
P x
(0, 0) (1, 0) 3-x
c

(1, -1) D
I1
360 360
n
= − 1= − 1= 5
θ θ
x 300 cm
∆ MAB  ∆MCD; =
20 cm y

y=
+ 20 100 cm;= ⇒ y 80 cm
20 × 300 cm
⇒x
= = 75cm
80
1 6 . 9 8 | Geometrical Optics

Sol 4: Relation between speed of image and speed of object


M1 for lens is

axis of M₁ 10 cm  v 2  du  ( −30)2 
dv dv
I₂ =  2 . ⇒
=   . 20m s−1 (downward)
dt  u  dt dt  ( −15)2 
1cm O 1cm    
A dv
1cm 1cm (4) × 20 ms−1 =
= 80m s−1 (downward)
dt
I₁
axis of M₂
Sol 6: Refer theory
M2

1 1 1 Sol 7: Apply Snell’s law:


+ = ;f = −20 cm ,u =−10 cm
v u f
4
1 1 1 1 1 1 2 −1 1 1=
× sin θ sin(90 − ϕ)
− =− ⇒ = − = = ⇒v=20 3
v 10 20 v 10 20 20 20
d 4 d×3
v 20 1× = ×
M= =
− =2 2 3
u ( −10) 30 + d 1024 + d2

Image will be erect with respect to the axis of each


mirror. Distance between images is 2 cm. 1024 + d2= 4 36 + d2

Sol 5: OS=15 m; OC =20 m


2 2
6 +d
Height of ball after 4 s is, S
6 ft
 Vertical shaft

1 feet = 12 inch
C  1' = 12"
Speed s = 20m/s d d
S 90°-
2'
10
3
2
1024+d
9

1 1'
H = 15 + 20 × 4 − × 10 × 16 = 15 + 80 − 80 = 15m 4”
2 o
3

Speed of ball after 4 s is, sball = 20 − 10 × 4


On squaring both sides,
⇒ sball = −20 ms−1
20 − 40 =
1024+9d2=16(36+d2) ; 1024+9d2=576+16d2
So ball is moving downward with speed of 20 ms-1 and
is at height 15 m above the mirror. 448
7d2=448; d2= = 64
20 7
So u=-15 m, and f = m = −10 m
−2 d= 8 feet, width=2d=16 feet

1 1 1
Mirror formula + = gives Sol 8: At any point by Snell’s law
v u f
1sin 90 =n(y). sin (90o − θ);
1 1 1 1 1 2 − 3 −1
= − = − = = (kg3/2 + 1)1/2 cos θ
1=
v −10 ( −15) 15 −10 30 30
⇒v= −30 cm
P hysi cs | 16.99

Here angle of incidence at B(x, y) is i = 90° – θ dy


[From (1)
= tan θ = 1 ⇒ θ = 45ο [From (i) put
put yy =
= 1]
1]
dx
y
sin 45o 1
r (1 1)1/2 =2 ⇒
n =+ =
sin r 2
 ⇒ sin r = 1 ⇒ r = 90ο
i B(x, y)
⇒ Ray of right becomes parallel to x axis.
x
(0, 0) Sol 9: At first refracting surface the rays will pass un-
deviated. At the second refracting surface the rays are
refracting from denser to rarer medium and hence
suffer Total Internal Refraction if i > θc
1
cos θ =
3/2
(ky + 1)1/2
dy 1 ky 3/2 + 1 − 1
⇒ tan θ= = = = ky 3/2 B
dx 2 1
c
(i) Now as
dy
=i 90o − θ ⇒=
θ 90o − i So = tan=
θ cot i R h
dx
(ii) Initial angle at air is 90° and n=1. At point B(x, y) c 90-c
angle of incidence is i.
So we have by Snell’s law A D P
C
x
1.sin 90ο = n sin i
⇒ n sin i = 1  3
1
=θc sin−1  =⇒ θc sin−1=
  60ο
dy µ  2 
(iii) Now, = k y 3/ 4 [From (i)]
[From(1)]  
dx
y x = 90o − θc
From right ∆ BCD, ∠BCD
y dy
dy
x yy ++1/ 4
1/ 4
⇒ ∫∫ 3/ 4 ==
⇒ ∫∫ kk dx
dx ⇒ +1 / 4 ==
⇒ kk xx
0y
y 3/ 4 0 +1 / 4 o 3
0 0 From ∆ ABD,=
h R sin θ=
c R sin 60= R
kk xx x
4
4 2
⇒ +1/ 4
⇒ yy +1/=4 ⇒ yy kk 22  x  …(ii)
= .......(2)
= 4 ⇒ =  4  .......(2) 3 3R
4 4
  o o
=DC h cot (90= − θc ) h cot30
= R= . 3
2 2
(iv) For the point P, we have y=1.0 m k=1 R
AP
= R , AD= R cos= θc
4 4 2
2x x
⇒y= k   gives 1 =   ⇒ x = 4.0 m R 3R
4 4 AC =AD + DC = + =2R;
2 2
(v) At P, we have x = pc = AC − AP = 2R − R = R = 5cm
⇒x=
5m
y
r
P
(0, 1)
(2, 1)
45o

x
1 6 . 1 0 0 | Geometrical Optics

Sol 10: o
For reflection of reflected ray at first face of prism
37

A
r

h r i
r
=1 15'
o

(90-2r)

2r
3.0 m o
6 30'
ο
sin i sin 37 4 3 3/5
=µ⇒ = µ = ⇒ sin r = =
sin r sin r 3 h +9 2 4/3
1 3 400
⇒ = ⇒ 400 = 9 (h2 + 9) ⇒ h2 = −9 6.5
2
h +9 30 9 µ sin 2r =sin 6.5 = π
180 1
319

= h = 5.95 m
9 6.5π 6.5π
2rµ
⇒= rµ
⇒=  … (iii)
180 360

From (i), (ii) & (iii) we get


sin i
Sol 11: = µ All the ray in the bean are deviated
sin r 1.25 π
by same angle so width of beam will not change after 1.25 = (µ − 1) A = µ A − A .....(4) … (iv)
180π = (µ − 1) A = µ A − A .....(4)
it goes over to air. 1806.5 π
and 6.5 π = µ A.....(5)

and 360 = µ A.....(5) … (v)
360

i ii Subtract (iv) from (v) to get


L
  Glass (6.5 − 2.5)π π
= A⇒A = rad = 2 ο
Air 360 90
r r and put value of A in (v) to get

6.5 π 90
=µ × ⇒
= µ 1.625
360 π

Sol 13: For a given incident ray, if the mirror is rotated


ο ο
Sol 12: δ = (µ − 1) A = 1 15' = 1.25 = (µ − 1) A  … (i) through an angle θ , then the reflected ray turns
through an angle of 2 θ . So if angular speed of mirror
A. Here r1 = 0,
Now for prism r1 + r2 = is ω then the angular speed by which the reflected ray
r2 = r is rotated is 2 ω.
⇒ r =A  ... (ii) 9 r e v 18 rad
ωrefl = 2 × ω = 2 × = × 2π
π sec π sec
36 rad s−1
ωrefl =
µ12 + µ32 sin2 r5 = 1 + µ22 ....(C)
From (iv)
µ3 cos r5 =
µ 4 sin r7 ...(D) P hysi cs | 16.101

(C)and(D)gives
When
µ12 + µ32 = 1 + µ22 + µ24 sin2 r7 .......(E)
vref vB From (v) µ5 sinr9 =
µ 4 cos r7 ......(F)
N
B (E) and (F) gives
µ12 + µ32 + µ52 sin2 r9 = 1 + µ22 + µ24 .....(G)
i From (vi) µ5 cos r9 =
1 ....(H)
(G) and (H) gives
i i ⇒ µ12 + µ32 + µ52 = 2 + µ22 + µ24
90-i
M A Sol 15: Snell’s law at spherical surface for the first ray
O µ sin i1 = sin r1
10 m

i = 37ο , we have
OA
OB = vB
cos(90 − i) r1
OA 10 10 i2
= = = m vref h1
sin i sin 37 3 / 5  R
90-i i1
50 x1
⇒ OB =m C
3 i
⇒ speed vrefl = ωrefl × [OB]

50 i  h1  h1 R2
⇒ v refl =36 × m s−1 =600m s−1 ⇒ 1.6 = ⇒ h12=
+ x12
3 
R  h12 + x12 2.56
v ref v ref 600
vB
= = =
cos(90 − i) sin i 3 / 5 202
⇒ 0.52 + x22= ⇒ x2= 12.49 m
= 200 × 5= 1000 m / s. 2.56

Similarly for second ray


Sol 14: 1 sin90 = µ1 sin r1 ........(i)
µ1 sin r2 =
µ2 sin r3 ........(ii)  h2  h2 R2
⇒ 1.6 =  ⇒ h22=
+ x22
µ2 sin r4 =
µ3 sin r5 ........(iii) R  h22 + x22 2.56

µ3 sin r6 =
µ 4 sin r7 ........(iv) 202
⇒ x22= − 122 ⇒ x2= 3.5 m
µ 4 sin r8 =
µ5 sin r9 ........(v) 2.56
µ5 sin r10 =
1 sin 90........(v i) ⇒ ∆x2= x1 − x2= 9 m

Also as all prisms are isosceles right angled prisms, we Sol 16: (a) For total internal reflection at the concrete-
have air interface we have critical angle

r1 + r2 = r3 + r4 = r5 + r6 = r7 + r8 = r9 + r10 = 90o.....(viii) sin θc µ2 v1 340m / s


= = =
From equations (i), (ii) and (viii) we get, sin90 µ1 v 2 1700m / s

1 = µ1 sinr1 ; µ2 sin r3 =
µ1 cos r1 1 1
⇒ sin θc = ⇒ θc = sin−1  
⇒ µ12 = 1 + µ22 2
sin r3 .....(A) 5 5
From (iii) µ3 sin r5 = µ2 cos r3 ......(B)
(b) The concrete is a rarer medium for sound because
(A) and (B) gives the speed of sound is higher in concrete, while air will
µ12 + µ32 sin2 r5 = 1 + µ22 ....(C) be denser medium for sound as the speed of sound is
lower in air. So for TIR, sound must travel in air which is
From (iv)
denser medium in this case.
µ3 cos r5 =
µ 4 sin r7 ...(D)
(C)and(D)gives
µ12 + µ32 = 1 + µ22 + µ24 sin2 r7 .......(E)
From (v) µ5 sinr9 =
µ 4 cos r7 ......(F)
1 6 . 1 0 2 | Geometrical Optics

Sol 17: The outer most ray of the beam , ray 1, will Sol 19: Angle of deviation is gives as δ= (i1 + i2 ) − A
be tangential to the circular surface of rod at point P
Hence i2 – i1 = 23°, A =60°, δ =23°
and hence angle of incidence is 90°, hence greater than
critical angle, and hence will travel tangentially at all ⇒
⇒ 23
⇒ 23°°==
23°=
ii1 + i − 60
+ ii222 −
i11 + − 60
60
points of the circular portion from P to P’. ⇒ ii1 + i == 83 °  ...(1) … (i)

⇒ i11 + + ii222 83 °
= 83° ...(1)...(1)
⇒ iii22 −
⇒ − ii11 == 23
23°°°  ...(2)
...(2) … (ii)
i ⇒ 2 − i1 = 23 ...(2)
From (i) and (ii) we get i1 = 30°, i2=53°
R Snell’s law at first refracting surface.
P Q Q’ P’

o
60

d
i1 i2
A B r1 r2

ray 1 ray 2 1 2 o o
60 60
The inner most ray of the beam, ray 2, will be incident
on the inner circular surface at angle i. 1 1
sini1 =µ sinr1 ⇒ =µ sinr1 ⇒ sinr1 =
From the geometry of figure we see that, 2 2µ
CQ = R; CA= R +d Snell’s law at second refracting surface.
R=(R+d) cos (90 - i) ⇒ R = (R + d) sin i 1 4 4
µ sinr2 = sini2 ⇒ sinr2 = . =
Here angle i will be the least of all angles of incidence µ 5 5µ
of ray 2 during its path inside the critical rod. So, if i is Now r1+r2=60°
greater than critical angle then ray 2 will surfer TIR at all
point in circular rod.

1 R 1 1 1 d A
sini > ⇒ > ⇒ > ⇒1+ < µ
µ R+d µ d µ R
1+ i
R
R+d
d d
⇒ < (µ − 1) ⇒   = (µ − 1) = 1.5 − 1
R  R max
d 1
⇒  =
 R max 2 90-i
Q R C
Sol 18: In a prime r1+r2=A;
Here r =r, r =0o, A=30o ⇒ r =30o  …..(i)
1 2 3 3
⇒ sin(r1 + r2 ) = ⇒ sinr1 cosr2 + cosr1 sinr2 = ⇒ μ
At first refracting surface 2 2
1 16 1 4 3
sin i 1 1 ⇒ . 1− + 1− . =
= µ = 2 ⇒ sin i = (sin30o ) 2 = 2   = 2µ 25µ 2
4µ 5µ
2 2
sin r 2
  2

⇒ i =45
43
=
5
P hysi cs | 16.103

Sol 20: Deviation suffered by the transmitted ray is Sol 21: When air is filled between two similar glass
pieces.
δδ=
= 1 (60oo
(60 − ) + (60oo
− rr1 ) + (60 − − rr11 ))
1 1
120 − 2r1 ...........(1)
...........(1) Pe 2PL + PM
=
δδ11=
= 120 − 2r 1
… (i)
Deviation suffered by the reflected ray after emerging
1 1  µL − µm  1 1 
out of sphere. = PL = where   − 
fL fL  µm  R1 R 2 

o o o
δ= (60o − r1 ) + (180o − 2r1 ) + (60o − r1 )
2
δ=
2 (60 − r1 ) + (180 − 2r1 ) + (60 − r1 )
δ2 300oo − 4r1 ..........(2)
= 1  1 − 1  1 1  1
δ2 300 − 4r1 …(ii)
= ..........(2) ⇒   −  ⇒ = 0
fL  1  R −R  fL
1
Now δ1 = δ2 ⇒ 3δ1 =δ2 .......(3)
 …(iii) air air
3
From (i), (ii) and (iii) we get = air +

1 R
30
o Then PL=0 PM = − where fM = = −
fM 2

(since mirror is concave)

i r Given that screen is placed at a distance 60 cm from the


combination
v
m=− ⇒ Two times magnification means
u
3(120° − 2r=
1 ) 300° − 4r1 m=-2, then v=2u ⇒ 60=2u ⇒ v=30cm
⇒ 360° − 6r=
1 300° − 4r1
 1 1 R
⇒ +2r1 = +60° ⇒ r1 = 30o ......(4) …(iv) For equivalent combination Pe =− ⇒ fe =− = −
fe Pe 2
Snell’s law of point P
Apply mirror formula to the equivalent combination of
mirror.
o
60
0 u
P
r1 Q I
r1 i1=60o v=60 cm
R R Screen
r1
1 1 1
+ =
v u fe
R
1 1 1 1 1 2 R
r1 ⇒ + = ⇒− − = −  since fe = − 
−60 −30 fe 60 30 R 2

R=40cm
o
air air
60
= w +

sin60 3 /2
=µ⇒µ= ⇒µ= 3 Again if air between the glass pieces is replaced by
sinr1 1/2 water.

Pe 2PL + PM
=
1 6 . 1 0 4 | Geometrical Optics

1 1  µ1 − 1   1 1  (d) Light intensity at image is the intensity focused by


=PL = where    −  the lens.
fL fL  1  40 − 40 
1 1 0.02 × π (7.5)2 × 10−4 W
⇒ ( µ w =4 / 3) ⇒ PL = ⇒ PM =− Iimage =
2
s
60 fM  0.214  −4 2
4π   × 10 m
 2 
1 R
Where = − (for concave mirror) Iimage = 24.56 Wm−2
fM 2

1 R
= − (for concave mirror) Sol 23: Focal length of Plano - concave lens
fM 2
2 2 1 1 1
PM= = = (µ1 − 1)  − 
R 40 f1 ∞ R
1 3  1  1
⇒ Pe = 2PL + PM = − 1 −  = − ⇒ f1 =−60 cm
f1  2  30  60
1 2 5 1
2× + = = Focal length of plano-convex lens
60 40 60 12
1 1 1 
1 = (µ2 − 1)  − 
− ⇒ fe =−12 cm f2  ∞ −R 
fe
1 5  1  1
1 1 1 =  − 1  = ⇒ f2 = 120 cm
Again apply + = f1  4 30
  120
v u fe

1 1 1 (i) Plane surface of Plano-convex lens is silvered. So the


⇒ + = ⇒ u =−15 cm equivalent focal length of the system,
−60 u −12
1 1 1 1 1 1
When air is filled between the gap, object distance=30 Power = = + + + +
−F f1 f2 −fm f2 f1
when water is filled between the gas, object distance=15
cm, Then, object is displaced by 15 cm towards the fm = focal length of planemirror = ∞
combination. 1 2 2 2 2 −1
⇒ =+ = − + =
−F f1 f2 60 120 60
⇒ +F = +60 cm
Sol 22: (a) At surface light intensity
4.5 πW 1.125 × 10 4 4 −2 Focal length = 60 cm
I = 4.5 πW ⇒ I 1.125 × 10
Wm −2
= I 4 π.(1.5)2 4.5
×=10π−W
4 2
m ⇒ I 1.125
2.25 × 10 Wm−2
4
The equivalent system behaves as a convex mirror.
= I 4 π.(1.5)=2
× 10 −4
m2 ⇒I 2.25 Wm
= 5000 4 πWm −22 × 10 −4 m2
.(1.5) 2.25
= 5000 Wm−−22 1 1 1
(ii) Mirror formula + =. Hence u = -15 cm,
5000
= 4.5 πWWm v u f
= IP = 4.5 πW 0.02Wm−2 −2 f = +60 cm
=(b) IP4 π.(7.5)
= 4.52 m
πW2 0.02Wm−2
= IP 4= π.(7.5)22 m22 0.02Wm 1 1 1 1 4 1
u = 4 π.(7.5)
−7.5m = m
−750cm ⇒ = − = + =
u = −7.5m = −750cm v 60 −15 60 60 2
(c)
f =u += −7.5m =
30cm −750cm
f = +30cm ⇒v= +12cm
f = +30cm
v −12 4
Magnification, m =− = ⇒m=
1 1 1 1 1 1 1 1 14 7 u −15 5
− = ⇒ = + = − = =
v u f v f u 30 750 750 375
375 Sol 24: Image from L1:
⇒ v = cm,
7 L1 L2

v 375 / 7 1 I2
Magnification, m = = ⇒m= −
u −750 14
20 cm
P2 R
3.0 cm
M C I1
⇒ diameter of imageof bulb,d1 = = 0.214 cm
14 P1
20 cm 40 cm
X
P hysi cs | 16.105

1 1 1 1 1 4 −3 1 Add (iii) and (iv)


= + = − = =
v1 f u 15 20 60 60 3 4 3
− −1 +
⇒ v1 =
60 cm from L1 1 1 2 3 2
+ =
v 6 30 30
For lens L2: 1 2 1 −1
⇒ =− = ⇒ v6 =
−90 cm
1 1 1 8+3 v 6 90 30 90
u2 =
+40 cm, =+ =
v 2 15 40 120 from lens (right from lens)
120 So final image is 90 cm right of lens.
⇒ v2 =
+ cm from L2
11
This final image should lie at the centre of curvature of
convex mirror, so MC = R = 5 cm Exercise 2
120 120 − 55
So,P2M= P2C2 − MC= − 5= cm Single Correct Choice Type
11 11
65 Sol 1: (B) Distance of image due to plane mirror from

= x cm
= 5.91cm
11 object will be 60 cm. So OO’ =60 cm. So distance of
image from convex mirror PO’ =[OO’-OP]=10 cm
Sol 25: For equi-convex lens radius R= f= 30 cm
Refraction at surface I: Air to glass
3 M P
−1
3/2 1 2  ..............(i) ... (i) O O’
− =
v1 ( −90) +30 cm
30 cm 20 cm
Refraction at surface II: Glass to water M’

4 3
− 1 1 1
4/3 3/2 3 2  ..............(ii) ⇒Mirror formula + = will give focal length of
− = ... (ii) v u f
v2 v1 −30 cm
mirror by putting u=-50 cm, v=+10 cm,
Add (i) and (ii)
1 1 1 4 2 25
3 4 3 So, = − = = ⇒ f=
−1 − + f 10 50 50 25 2
4 1 2 3 2
+ =
3v 2 90 30 Radius of curvature R=2f =25 cm
4 2 1
⇒ =− ⇒ v2 =
+120 cm
3v 2 90 90
near end
For mirror, image of lens acts as object. F

For mirror u3=+ (120-80) = 40 cm (right from mirror)


So v3 = -40 cm (left from mirror)
Sol 2: (A) Mirror formula
Refraction from surface II after reflection from mirror.
1 1 1 1 1 1 u−f fu
u4=-40 cm (left from surface II) + = ⇒ = − = ⇒v= .
v u f v f u fu u−f
3 4
− At near end |u|> |f|. u and f both are negative .
3/2 4/3 2 3  ... (iii)
− = .........(iii)
v5 −40 +30 So v is negative. At far end we have u = −∞ . So mirror
Refraction at surface I: Glass to air 1 1 1 1
formula gives = − = ⇒ v ∞ =f .
3 v ∞ f −∞ f
1−
1 3/2 2 .........(iv) For near point |vnear| > |f|.
− =  ... (iv)
v6 v5 −30
1 6 . 1 0 6 | Geometrical Optics

Image length 1 1 1
Sol 6: (D) Lens formula − =; u = -30 cm,
v u f
fu fu − uf + f 2
| v near | − | v ∞=| f
−=
u−f u−f f = 20 cm
2
f
⇒ ∆I =
u−f
2 cm
Sol 3: (C) Velocity of image
P
 v  du dv
2  v  2 h1 0.5 cm
dv
−  ;
= =
−  .4 cm s−1 . axis
dt  u2  dt dt  20 2 
   
h2
From mirror formula
1 1 1 3−2 1
1 1 1 1 1 1 1 1 3 −5 1 = − = = ⇒ v = 60
=− ⇒ = − = − = =− v 20 30 60 60
v f u v −12 −20 20 12 60 30 v 60
Magnification m = ⇒ m = =−2 ⇒ h2 =−2h1
dv  900  −1
u −30
⇒v=−30 cm ⇒ =−  4 cm s = −9 cm s−1
dt  400  So h2 =−2 × 0.5cm =−1cm (below axis)
So Image of P is 1.5 cm below XY.
d2
Sol 4: (D) Area of mirror, A1 = π .
4
Sol 7: (B) Distance between object and screen is D,
Area left after putting opaque, displacement of lens is d, and so focal length of lens is
d2 d2 d2 D2 − d2 902 − 202
A2 = A1 − π d2 = π d2 − π d2 =f = = 21.4 cm
A2 = A1 − π 16 = π 4 − π 16 4D 4 × 90
16 4 16
d2  1  3 πd22 3
⇒ A2 = π d2  1 − 1  =
3 πd =3 A1
⇒ A2 = π 4  1 − 4  =4 4 = 4 A1 Sol 8: (A) Object size
4  4 4 4 4

3 O= I1 × I2 = 6 cm× 3 cm
Focal length will not change and intensity become I.
4
O = 4.24 cm

Sol 5: (B) Rays should fall normally on plane mirror.


This will happen if rays become parallel to principal axis 1 1 1
Sol 9: (A) + =
after passing through lens. So OL = f=30 cm. v u f
Let u = + x (virtual) and |f| = -f (concave mirror)
1 1 1 −x − | f | |f|x
⇒ =− − = ⇒v= → ( −)ve
v |f| x |f|x −x − | f |

L So v is always negative when u is positive (+x).

15 cm Multiple Correct Choice Type

Sol 10: (B, D) Slope of reflecting surface at the desired


point will be tan 45 = 1

dy  πx  πx π L
= 2cos   = 1 ⇒ = ⇒x=
dx  L  L 3 3

 L  2L π 3L
⇒ y=  sin  =
3 π 3 π
P hysi cs | 16.107

Sol 11: (B, C) Length of object is AB = (50-20) cm δ +A


=30 cm. After first reflection from plane mirror AB is sin  m 
δ µ  2 
inverted to B1A1 with distance from convex mirror as If m is minimum deviation, p =
shown in figure. µs A
sin  
2
Image of A in convex mirror
So as µp increases , δm also increases.
v1
60 cm
u1 Sol 15: (B, C) The minimum length of a plane mirror to
H
B1 A1 A’ B’ see one’s full height in it is , where H is the height of
2
u2 = 90cm v2 man. The mirror can be placed anywhere between the
centre line BF (of AC) and DG (of CE). Eye is at C.

1 1 1 1 1 1 1 A
=− ⇒ = − = ⇒ v1 =
+30cm x
v1 f u1 v1 60 −60 30 F
B
x
Image of B in convex mirror C
(x + y)
y
1 1 1 1 1 3+2 1
= − = − = =
v 2 f u2 60 −90 180 36 D
G
⇒ v2 =
+36 cm ⇒ A'B' =
(36 − 30)cm =
6 cm y
E
th
1 Man
Second image A’B’ is virtual and   of magnification
w. r. t. AB and erect. 5
Sol 16: (B, D) The distance PQ1 and PQ2 will not change
1
as the mirror MM’ moves with speed v perpendicular
2
i to its length.
P S P Q1 Wall Q2
i

i’ i’

Now formula for speed of image for convex mirror


V
dv v 2 du
is, = . As object moves towards mirror, the M’ M
dt u2 dt
image also moves towards the mirror.
Sol 17: (A, C) µ2 > µ1 Rays from real object will be
deviated away from radius of curvature and hence
Sol 12: (B, C) i + i’ =90o from figure will becomes more diverging. For virtual object the
At point P, µ1 sin i > µ2  … (i) deviated rays may converge on the principle axis.
µ1 > µ2 : For virtual object the deviated rays will
At Q, µ1 sini' > µ3 or µ1 cosi > µ3  … (ii)
converge on principle axis.
Squaring and adding (1) and (2) to get
For real object the refracted ray will deviate towards
µ12 > µ22 + µ32 ⇒ µ12 − µ22 > µ32 ⇒ µ12 − µ32 > µ22 radius of curvature and may coverage on principle
axis.
Sol 13: (A, C) Angle of deviation δ= ( µ -1)A.
Sol 18: (A, D) The image formed by a convex mirror is
always, virtual and erect. So convex mirror cannot form
Sol 14: (A, B, D) Angle of deviation δ= (i1 + i2 ) − A . i1
inverted image of OA. Option B and C are ruled out.
is angle of incidence and i2 is angle of emergence and
angle of incidence and emergence are interchangeable.
1 6 . 1 0 8 | Geometrical Optics

Sol 19: (A, C) u = -40 cm; f = +20 cm R


f =R− sec60 = R − R = 0
2
1 1 1 1 1 1 1 1 1
− = ⇒ = + = − =
v u f v f u 20 40 40
v 40 = 180o − 2 × 60=
Sol 28: (D) Deviation o
60o . Ans (D)
⇒ v =+40 cm, and magnification m = =
u −40
⇒ m =−1 ⇒ y and z coordinate’s values will change o
their sign but magnitude will remain the same. 60
o
60
⇒ y image =
−1 , zimage =
+1 R
C
Sol 20: (B, D) Convex mirror and concave lens for
diminished, virtual, correct image of object.

Assertion Reasoning Type


R R
R − sec0 =
Sol 29: (B) f =
Sol 21: (C) The wall does reflect light, but the reflection is 2 2
irregular. Reflected rays are deviated in different directions.
Sol 30: (B) Spherical aberration cannot be completely
eliminated, but it can be minimized by allowing either
Sol 22: (A) This phenomena is called spherical paraxial or marginal rays to hit the mirror.
aberration. The rays close to the principal axis are
focused at the geometrical focus F of the mirror as
given by mirror formula. The rays farthest from the Sol 31: (D)
principal axis are focused at a point somewhat closer (2n − 1)λD
to mirror. = ymin = (n 1,2,...........)
2d
dactual 5λD
Sol 23: (D) For object in liquid. dapparent = For n =
+3, ymin =
µ 2d
 1
For a slab: normal shift ∆x= t  1 −  where t is Sol 32: (A) Shift in the fringe due to the glass slab is
thickness of slab.  µ
D
∆y = (µ − 1)t where t is thickness of glass slab. Due to
d
Sol 24: (D) Two image will be formed one for each glass slab path of ray from S2 gets increased by (µ − 1)t .
mirror.

Sol 25: (D) If a plane mirror is moved such that its Sol 33: (A) Path of rays 1 is more than path of ray 2 by
perpendicular distance from the point object does not a distance dsin α . Draw perpendicular S2M from S2 to
change, then the image will not move. ray 1.

Comprehension Type S1

Sol 26: (B) Paraxial rays are focused at the geometrical 


focus F of the mirror. The marginal rays are focused at a O
P
point F’ somewhat closer to the mirror.
1
S2
Sol 27: (D) y
2

O I
P
X ∠MS2S1 =
α and MS1 =
d sin α
-40

This path difference is suffered before passing the slits


S1 & S2. After passing through the slits, path of ray from
P hysi cs | 16.109

S2 is increased by (µ − 1)t . For net path difference to be hc


zero at point P we have, Sol 5: (B) lcutoff = (independent of atomic number)
eV
S1
M Sol 6: (C) The refractive index n for meta-material is
negative.
 d
 sin θ1
1 Hence is negative.
sin θ2
S2
Thus if θ1 is negative, θ2 will be negative. So the current
2 choice is C.

d sin α = (µ − 1)t c c
Sol 7: (B) N= = v= which is choice B
 (µ − 1)t  v n
⇒ α =sin−1  
 d  v
Also frequency ν = since v remains unchanged
λ
Match the Column v air v m
=
λair λm
Sol 34: A → p, r; B → q, s; C → p, q, r, s; D → p, q, r, s
vm vm c
⇒ λm = λair × =λair × ×
v air c v air

Previous Years’ Questions =λair ×


nair  c
 v = 
nm  n

1 1 1 λair
Sol 1: (C, D)
f
=
v
+ (mirror formula)
u n
= (
 nm n=
and nair 1 )
F = – 24 cm
So choice D is wrong

Sol 2: (B) Critical angle from region III to region IV


 1   1 
n0 / 8 3 Sol 8: (B) PT = (1.5 – 1)  – 0  + (1.2 – 1) 0 – 
sin=
θc =  14   –14 
n0 / 6 4
0.5 0.2 1
= + =
Now applying Snell’s law in region I and region III 14 14 20

n0 f = + 20 cm
n0 sin
= θ sin θC
6
1 1 1
1 13 1 – =
or sin
= θ sin=
θC = v –40 20
6 64 8
1 1 1 1
1 = – =
∴ θ =sin−1   v 20 40 40
8
\ v = 40 cm
Sol 3: (A, C, D) In case of concave mirror or convex lens
image can be real, virtual, diminished, magnified or of
same size. Sol 9: (B) Object is placed at distance 2f from the lens.
So first image I1 will be formed at distance 2f on other
(B) In case of convex mirror image is always virtual (for
side. This image I1 will behave like a virtual object for
real object).
mirror. The second image I2 will be formed at distance
20 cm in front of the mirror, or at distance 10 cm to the
Sol 4: (A) At minimum deviation ( δ = δm ) : left hand side of the lens.
A 60o
r1= r2= = = 30o (For both colours)
2 2
1 6 . 1 1 0 | Geometrical Optics

Now applying lens formula Hence, l will depend on n (z) but not on n2.

10 cm

O
I3
I1 n1 = constant 1 1
I2
6 cm
n(z)
30 cm 10 cm 20 cm
z
d
1 1 1 d
− =
v u f
1 1 1
∴ − = n2 = constant l
v +10 +15 2

or v = 6 cm
Therefore, the final image is at distance 16 cm from the
mirror. But, this image will be real.
This is because ray of light is travelling from right to Sol 13: (A) First Image I1 from the lens will be formed
left. at 75 cm to the right of the lens.
Taking the mirror to be straight, the image I1 after
50
Sol 10: (3) For v1 = m, u1 = – 25 m reflection will be formed at 50 cm to the left of the
7 mirror.
25
v2 = m, u2 = – 50 m On rotation of mirror by 30° the final image is I3.
3
25 18 So x = 50 – 50 cos 60° =25 cm.
Speed of object = × = 3 km/h.
30 5 and y = 50 sin 60° =25 3 cm.

I3
Sol 11: (B)

For the combination


=
1 ( µ1 − 1) + ( µ2 − 1) 50 cm

feq R R 30
o

feq = 20 o
30
I2 (0, 0)
Here u = - 40, f = 20

v = 40 50 cm

Sol 14: (A) Let angle between the directions of incident


Sol 12: (A, C, D) From Snell’s Law ray and reflected ray be θ

θi n ( d) sin=
n1 sin
= θd n2 sin θf
The deviation of ray in the slab will depend on n (z)
cos=
θ (
1 ˆ
2
i + 3 ˆj )


cos=
θ
2 (
1 ˆ 1
) (
i + 3 ˆj . ˆi + 3 ˆj
2 )
P hysi cs | 16.111

1 ∴ f1 = 3R
cos θ = −
2 For glass to air.
θ 120°
= 1 1.4 − 1.5 1 − 1.4
= +
f2 −R −R
Sol 15: (C) ∴ f2 =
2R

r 5.77
Sol 18: (C) tan θc = = ≈ 3
h 10
Liquid r
24 m 8m
C C h
λa 3
=
µ =
λm 2 µ
⇒ sin θc = 
µb
1 µ −1 1
⇒= =
f R 2R 1
µ 2.72 ×= 1.36
⇒=
2
1 1 1
⇒ = −
f v u 1  1 
Sol 19: (B)
f
= ( µ − 1)  R1 − 
R 2 
1 1 1  1
⇒ − =
8 −24 2R

3+1 1
⇒ =
24 2R f=R f = 2R f = -2R

⇒R=
3m
1 1 1
Use = +
feq f1 f2
Sol 16: (D) P → (2); Q → (3); R → (4) ; S → (1)
1 1 1 2 R
P. µ2 > µ1 .... (towards normal) (P) = + = ;f =
feq R R R eq 2
µ2 > µ3 .... (away from normal)
Q. µ1 =µ2 .... (No change in path) 1 1 1 1
(Q) = + = ; feq = R
feq 2R 2R R
∠ i= 0 ⇒ ∠r = 0 on the block.
R. µ1 > µ2 .... (Away from the normal) 1 1 1 1
(R) =
− − − ; feq =
= −R
µ2 > µ3 .... (Away from the normal) feq 2R 2R R

1 µ1
µ1 × =µ2 sinr ⇒ sin r = . 1 1 1 1
(S) = − = ; f =2R
2 2 µ2 feq R 2R 2R eq
Since sin r < 1 ⇒ µ1 < 2 µ2

S. For TIR: Sol 20: Image by mirror is formed at 30 cm from


mirror at its right and finally by the combination it is
1 µ2
45° > C ⇒ sin 45° > sin C ⇒ > ⇒ µ1 > 2 µ2 formed at 20 cm on right of the lens. So in air medium,
2 µ1 magnification by lens is unity. In second medium,
7
Sol 17: (A, C) For air to glass µ = . focal length of the lens is given by,
6
1.5 1.4 − 1 1.5 − 1.4
= +
f1 R R
1 6 . 1 1 2 | Geometrical Optics

 1  Sol 23: (A, C) θ ≥ c


1 (1.5 − 1)  R1
 −
10 =  1 R 2  ⇒ f 35 cm ⇒ 90° − r ≥ c
1  1.5  1
− 1   −
1  2 ⇒ sin ( 90° − r ) ≥ c
f  

7/6   R1 R 2  ⇒ cos r ≥ sin c
So in second medium, final image is formed at 140 cm to sin i n1 n
Using = and sin c = 2
the right of the lens. Second medium does not change sinr nm n1
M Mm M n12 − n22
the magnification by mirror.=
So 2 = 2 2
7 We get, sin2 im =
M1 Mm M 2
nm
1 1

Putting values, we get, correct options as A & C


Sol 21: (B) For Ist refraction
n2
1 1.5 1 − 1.5
− = nm  n1
v −50 −10 r
i
⇒v=
50 cm

For IInd refraction

1.5 1 1.5 − 1 Sol 24: (A) i = β + θ


− =
∞ −x +10 For α= 45° ; by Snell’s law,
⇒x=
20 cm
P
⇒d=
70 cm

3 
Sol 22: Snell’s Law on 1st surface: = n sin r1 
2
i
3
sin r1 =  …(i) n =2
2n
Q R
4n2 − 3
3
⇒ cos r1 =1 − =
4n2 2n
1 × sin=
45° 2 sin β

r1 + r2 = 60°  …. (ii) ⇒ β= 30°

Snell’s Law on 2nd surface: For TIR on face PR,

n sin =
r2 sin θ  1 
β + θ = θc = sin−1   = 45°
Using equation (i) and (ii)  2

nsin ( 60° − r1=


) sin θ ⇒ θ= 45° − β= 15° .
 3 1 
n cos r1 − sin r1  =sin θ Sol 25: (A, D) For refraction through lens,
 2 2 
1 1 1 v
d  3  2  dθ − = and −2 =
  4n − 3 − 1  = cos θ v −30 f u
dn  4    dn
∴ v =−2u =60 cm
For θ= 60° and n = 3
∴f =+20 cm

⇒ 2
=
dn
P hysi cs | 16.113

For reflection
1 1 2
+ = ⇒ R = 30 cm
10 −30 R

(n − 1)  R1  =
1 1
 
=
  f 20
5
∴n =
2
The faint image is erect and virtual.
2017-18 100 &
op kers
Class 12 T
By E ran culty
-JE Fa r
IIT enior emie .
S fP r es
o titut
Ins

PHYSICS
FOR JEE MAIN & ADVANCED
SECOND
EDITION

Exhaustive Theory
(Now Revised)

Formula Sheet
9000+ Problems
based on latest JEE pattern

2500 + 1000 (New) Problems


of previous 35 years of
AIEEE (JEE Main) and IIT-JEE (JEE Adv)

5000+Illustrations and Solved Examples


Detailed Solutions
of all problems available

Topic Covered Plancess Concepts


Tips & Tricks, Facts, Notes, Misconceptions,
Wave Optics Key Take Aways, Problem Solving Tactics

PlancEssential
Questions recommended for revision
17. WAV E O P T I C S

1. INTRODUCTION
There have always been controversies over the nature of light. Some theories believe light to be a wave whereas
some believe it to be a particle. Newton, the greatest among the great, believed that light is a collection of particles.
He believed that these particles travel from a source of light in straight lines when it is not under the influence of
external forces. This was one of the strongest evidence of the particle nature of light.
A Dutch physicist named Huygens (1629 – 1695), suggested that light may have a wave nature. The apparent
rectilinear propagation of light explained by Newton may be just due to the fact that the wavelength of light may
be much smaller than the dimensions of these obstacles. This proposal remained a dump for almost a century.
Newton’s theory was then challenged by the Young’s double slit experiment in 1801. A series of experiments on
diffraction of light conducted by French scientist Fresnel were some of the activities that put an end to the particle
nature of light and established the wave nature of light.
The twist came around when the wave nature of light failed to explain the photoelectric effect in which light again
behaved as particles. This again brought up the question whether light had a wave or a particle nature and an
acceptance was eventually reached that light is of dual nature – particle and wave. In this material, we will focus on
the study of the wave nature of light.
Key point – Light waves need no material medium to travel. They can propagate in vacuum.

1.1 Nature of Light Waves


Light waves are transverse, i.e. disturbance of the medium is perpendicular to the direction of propagation of the
wave. Hence they can be polarized. If a plane light wave is travelling in the x-direction, the electric field may be
along the y or z direction or any other direction in y-z plane. The equation of such a monochromatic light wave can
E E0 sin ω ( t − x v )
be written as =
The speed of light is generally denoted by c. When light travels in a transparent material, the speed is decreased
by a factor µ, which is called as the refractive index of the material.
speed of light in vaccum
µ=
speed of light in the material

The frequency of visible light varies from about 3800 × 1011 Hz to about 7800 × 1011 Hz .

Colour Wave-length
Red 620-780 nm
Orange 590-620 nm
Yellow 570-590nm
1 7 . 2 | Wave Optics

Colour Wave-length
Green 500-570 nm
Blue 450-500 nm
Violet 380-450 nm

Light of a single wavelength is called monochromatic light.

Illustration 1: The refractive index of glass is 1.5. Find the speed of light in glass.  (JEE MAIN)
speed of light in vaccum
Sol: µ =
speed of light in the material

speed of light in vaccum 3.0 × 108 ms−1


Thus, speed of light in glass = = = 2.0 × 108 ms−1
µ 1.5

2. HUYGENS’S WAVE THEORY


It has following two basic postulates:
(a) Consider all the points on a primary wave-front to be the sources of light, which emit disturbance known as
secondary disturbance.
(b) Tangent envelop to all secondary wavelets gives the position of the new wave-front.
Huygens’ Principle may be stated in its most general form as follows:
Various points of an arbitrary surface, when reached by a wave front, become secondary sources of light emitting
secondary wavelets. The disturbance beyond the surface result from the superposition of these secondary wavelets.

Huygens’ Construction
Huygens, the Dutch physicist and astronomer of the seventeenth century, gave a beautiful F1 F2
geometrical description of wave propagation. We can guess that, he must have seen water
waves many times in the canals of his native place Holland. A stick placed in water, oscillated
up and down, becomes a source of waves. Since the surface of water is two dimensional, the
resulting wave fronts would be circles instead of spheres. At each point on such a circle, the
water level moves up and down. Huygens’ idea is that we can think of every such oscillating A1 A2
point on a wave front as a new source of waves. According to Huygens’ principle, what we
observe is the result of adding up the waves from all these different sources. These are called B1 B2
secondary waves or wavelets. Huygens’ Principle is illustrated in (Figure) as the simple case
of a plane wave.
C1 C2
(a) At time t=0, we have a wave front F1 ,F1 separates those parts of the medium that are
undisturbed from those where the wave has already reached.
D1 D2
(b) Each point on F1 acts like a new source and sends out a spherical wave. After a time ‘t’,
each of these will have radius vt. These spheres are the secondary wavelets.
(c) After a time t, the disturbance would now have reached all points within the region
covered by all these secondary waves. The boundary of this region is the new wavefront Figure 17.1
F2 . Notice that F2 is a surface tangent to all the spheres. It is called the forward envelop
of these secondary wavelets.
(d) The secondary wavelets from the point A1 on F1 touches F2 at A2 . According to Huygens, A1 A2 is a ray. It
is perpendicular to the wavefronts F1 and F2 and has length vt. This implies that rays are perpendicular to
wavefronts. Further, the time taken for light to travel between two wavefronts is the same along any ray. In our
P hysi cs | 17.3

examples, the speed ‘v’ of the wave has been taken to be the same at all points in the medium. In this case,
we can say that the distance between two wavefronts is the same measured along any ray.
(e) This geometrical construction can be repeated starting with F2 to get the next wavefront F3 a time t later, and
so on. This is known as Huygens’ construction.
Huygens’ construction can be understood physically for waves in a material medium, like the surface of water. Each
oscillating particle can set its neighbors into oscillation, and therefore acts as a secondary source. But what if there
is no medium, such as for light travelling in vacuum? The mathematical theory, which cannot be given here, shows
that the same geometrical construction work in this case as well.

3. INTERFERENCE
When two waves of the same frequency move along the same direction in a medium, they superimpose and give
rise to a phenomena called interference. Points of constructive interference have maximum intensity while points
of destructive interference have minimum intensity.

3.1 Coherent and Incoherent Sources


Two light sources of light waves are coherent if the initial phase difference between the waves emitted by the
sources remains constant with time. If it changes randomly with time, the sources are said to be incoherent. Two
waves produce an interference pattern only if they originate from coherent sources.

3.2 Intensity and Superposition of Waves


If two waves
= y1 A1 sin ( ωt ) &
= y 2 A2 sin ( ωt + θ ) are superimposed, resultant wave is given by
= y R sin ( ωt + θ )

Where R 2 = A12 + A22 + 2A1 A2 cos φ or I = I1 + I2 + 2 ( ) ( )


I1 I2 cos φ I ∝ A2 and tan θ =
A2 sin φ
A1 + A2 cos φ

For maxima, cos φ= 1, φ= 2nπ n= 0,1,2

For minima, cos φ = −1, φ = (2n − 1)π n = 1,2,...

( )
2
(=A1 + A2 )
2
φ Imax I1 + I2
If I=
1 I=
2 I0 ; I1 = 410 cos2 =;
2 Imin
( A1 − A2 ) ( )
2 2
I1 − I2

Note: Consider two coherent sources S1 and S2 . Suppose two waves emanating S1
from these two sources superimpose at point P. The phase difference between them
at P is φ (which is constant). If the amplitude due to two individual sources at P is A1 P
S2
and A2 , then resultant amplitude at P will be, A = A12 + A22 + 2A1 A2 cos φ
Figure 17.2

Similarly the resultant intensity at P is given by, I = I1 + I2 + 2 I1 I2 cos φ . Here, I1 and


I2 are the intensities due to independent sources. If the sources are incoherent then
resultant intensity at P is given by, =
I I1 + I2

Illustration 2: Light from two sources, each of same frequency and in same direction, but with intensity in the
ration 4:1 interfere. Find ratio of maximum to minimum intensity.  (JEE MAIN)

Sol: Interference, amplitudes added and subtracted not the intensity, =


A A1 + A2 .
1 7 . 4 | Wave Optics

2
 I1 
2  +1 
Imax  I1 + I2 
2
 I2  2+1
=    =  =  9:1
Imin  I − I   I1   2 −1
 1 2 
−1 
 I2
 

3.3 Conditions for Interference


(a) Sources should be coherent i.e. the phase difference between them should be constant. For this, frequency of
sources should be the same.
(b) The amplitudes of both the waves should be nearly equal so as to obtain bright and dark fingers of maximum
contrast.
(c) The two sources should be very close to each other.
(d) The two sources of slits should be very narrow otherwise a broad source will be equivalent to a number of
narrow sources emitting their own overlapping wavelets.
If the two sources are obtained from a single parent source by splitting the light into two narrow sources, they
form coherent sources which produce sustained interference pattern due to a constant phase difference between
the waves.

Illustration 3: In a Young’s experiment, the interference pattern is found to have an intensity ratio between the
bright and dark fringes as 9:1, find out:  (JEE MAIN)
(i) The ratio of intensities
(ii) Amplitude of two interfering waves.

Sol: In Interference, A2 ∝ I , =
A A1 ± A2 .

( )
2
Imax 9 I1 + I2 9 I1 + I2 3
= ; = ; = ; I1 + I2 = 3 I1 − 3 I2
Imin 1
( ) 1 1
2
I1 − I2 I1 − I2

I1 I1 a I1 a1
−2 I1 =
−4 I2 ; =
2
I2 1
; =
4
I2 1
(
; I ∝ a2 , 1 =
a2
)
I2
;
a2
4 2
==
1 1

Illustration 4: Two coherent monochromatic light beams of intensity I & 4I are superimposed. What is the max &
min possible intensities in the resulting wave?  (JEE MAIN)

Sol: In Interference, A2 ∝ I , =
A A1 ± A2 .

( ) =( I + 4I ) =(3 I )
2 2 2
Imax = I1 + I2 2 =9I

( ) =( I − 4I ) =I
2 2
Imin = I1 − I2

3.4 Young’s Double Slit Experiment


The experiment consists of a parallel beam of monochromatic light from slit S which is incident on two narrow
pinhole or slits S1 and S2 separated by a small distance d. The wavelets emitted from these sources superimpose
at the screen placed in front of these slits to produce an alternate dark and bright fringe pattern at points on the
P hysi cs | 17.5

screen depending upon whether these waves reach with a phase difference φ = (2n – 1)π producing destructive
interference or φ= 2nπ producing constructive interference respectively. If the screen is placed at a perpendicular
distance D from the middle point of the slits, the point O on the screen lies at the right bisector of S1 S2 and is
equidistant from S1 and S2. The intensity at O is maximum. Consider a point P located at a distance xn from O on
the screen as shown in the figure. The path difference of waves reaching at point P from S2 and S1 is given by Path
x d
difference = S2P − S1P = n
D
S1P = XP

S=
2P S2 X + XP

⇒ S2P − S1P = S2 X = dsin θ

⇒ path difference =
dsin θ
xn
As θ is small sin θ ≈=
tan θ {D >> d}
D
xn d
∴ S2P − S1P =
D
The point P will be bright or of a maximum intensity when the path difference is an integral multiple of wavelength
x d
λ or φ = 2nπ = nλ ; ∴ S2P − S1P =n = nλ
D
The bright fringes are thus observed at distance
P
λD 2λD 3λD nλD
=x1 = ,x = , x3 ,......,
= xn
d 2 d d d
S1 Xn
The distance between consecutive bright fringes,
 
nλD λD λD
=
β − (n − 1 ) = d
d d d X

The point P will be dark or of minimum intensity when the path S2


difference is an odd multiple of half wavelength or φ − ( 2n − 1 ) π ;
D
x d
∴ S2P − S1P =n = nλ Figure 17.3
D

The bright fringes are thus observed at distances

λD 2λD 3λD nλD


=x1 = , x2 = , x3 ,......,
= xn
d d d d
nλD λD λD
The distance between consecutive bright fringes,=
β − (n − 1 ) =
d d d
The point P will be dark or of minimum intensity when the path difference is an odd multiple of half wavelength or
xn d λ
=
φ ( 2n − 1) π ; ∴ S2P − S1P = = ( 2n − 1 )
D 2

Where n is an integer ∴xn = ( 2n − 1) λ2dD .

The dark fringes will be observed at distance


= x1′
λD
= , x2′
3λD
= , x3′
5λD
,.......,
= xn′
( 2n − 1) λD
2d 2d 2d 2d
1 7 . 6 | Wave Optics

Fringe width: The spacing between any two consecutive bright or two dark fringes is equal and is called the fringe
width.

The distance between two consecutive dark fringes = β =


( 2n − 1) λD − ( 2n − 3) λD = λD
2d 2d d
λD
∴Fringe width = β =
d
If a thin transparent plate of thickness t and refractive index µ is introduced in the path S1P of one of the interfering
waves, the entire fringes pattern is shifted through a constant distance. The path S1P in air is increased to an air
path equal to S1P + ( µ − 1 ) t
xn d
δ S2P − S1P + ( µ − 1 ) =
∴ The path difference = t  S2P − S1P − ( µ − 1=
)t − (µ − 1) t
D
xn .d D
For nth maxima at a distance x'n = − ( µ − 1 ) t = nλ ; xn = nλ + ( µ − 1 ) t 
D d 

Thus, when a thin transparent plate of thickness t and refractive index µ is introduced in one of the paths of the
xd
waves, the path difference changes by .
D
xd D
∴ = ( µ − 1 ) t; x = (µ − 1) t
D d
D (µ − 1) t
∴The central maxima shifts by a distance equal to .
d

PLANCESS CONCEPTS

In Young’s double slit experiment, it is important to note that energy is just redistributed over the surface
of screen. It is still conserved! More energy is taken by points near bright fringes whereas dark fringes
have almost no energy.
B Rajiv Reddy (JEE 2012, AIR 11)

Illustration 5: S1 and S2 are two coherent sources of frequently ‘f’ each. ( θ1 =θ2 =0° ) Vsound = 330m s . Find f
(i) So that there is constructive interference at ‘P’
(ii) So that there is destructive interference at ‘P’ (JEE MAIN)
λ
Sol: Path difference for constructive and destructive interference must be λ and respectively.
2
For constructive interference,
S1 3m P
2π 2 2 330
K∆x = 2nπ ; × 2 = 2nπ ; λ = ; V =λf ⇒ V = f=
; f × n For destructive interference,
λ n n 2
4m
2π 1 ( 2n + 1) V 330 × ( 2n + 1 )
K∆x= ( 2n + 1) π ; λ
.2 = ( 2n + 1) π ; λ
=
4
;=
f =
λ 4 S2
Figure 17.4

Illustration 6: In a Young’s double slit experiment, the separation between the slits is 0.10 mm, the wavelength of
light used is 600 nm and the interference pattern is observed on a screen 1.0 m away. Find the separation between
the successive bright fringes.  (JEE MAIN)
P hysi cs | 17.7

Sol: Path difference needs to be λ.


Dλ 1.0m × 600 × 10−9 m
The separation between the successive bright fringes is, ω
= = = 6.0 × 10−3 m
= 6.0mm
d 0.10 × 10 −3 m

Assumptions: 2. Since d<<D, we can assume that intensities at P due to independent sources S1 and S2 are almost
equal. or I=
1 I=
2 I0 (say)

Illustration 7: When a plastic thin film of refractive index 1.45 is placed in the path of one of the interfering waves,
then the central fringe is displaced through width of five fringes. Find the thickness of the film, if the wavelength of
light is 5890 A.  (JEE MAIN)

Sol: Path difference due to introducing of thin film.

β β ( 0.45 ) t 5 × 589010−10
X 0= ( µ − 1 ) t ⇒ 5p= ∴ t=
0.45
=6.544 × 10−4 cm
λ 5890 × 10−10

Illustration 8: Laser light of wavelength 630 nm, incident on a pair of slits produces an interference pattern in
which bright fringes are separated by 8.1 mm. A second light produces an interference pattern in which the bright
fringes are separated by 7.2 mm. Find the wavelength of the second light.  (JEE ADVANCED)

Sol: The separation between the successive bright fringes, β ∝ λ .

λ1= 630nm= 630 × 10−9 m; β1= 8.1mm= 8.1 × 10−3 m; β2= 7.2mm= 7.2 × 10−3 m

λ2 β2 β2 7.2 × 10−3 8
= ; λ2 = λ1 ; λ2 = × 630 × 10−9 =× 630 × 10−9 =
560 × 10−9 =
560nm
λ1 β1 β1 8.1 × 10 −3 9

Illustration 9: In a Young’s double slit experiment, the two slits are illuminated by light of wavelength 5890
Angstrom and the distance between the fringes obtained on the screen is 0.2° . If the whole apparatus is immersed
in water, the angular fringe width will be: (the refractive index of water is 4 3 ). (JEE ADVANCED)

Sol: Angular fringe width, ω ∝ λ .


( ωa )water λ water ( ωa )water λ
λ d;
ωa = ∴ ωa ∝ λ ⇒ = ⇒ = ⇒ ( ωa ) 0.15°
=
ωa λ ωa µ water λ water

Illustration 10: In a YDSE, D=1m, d=1mm and λ =1 2mm  (JEE ADVANCED)


(i) Find the distance between the first and the central maxima of the screen.
(ii) Find the no. of maxima and minima obtained on the screen.
dy
Sol: Here, sinθ <<1, not applicable. Hence =
∆P dtan
= θ is used.
D
d
= dsin θ ;
(i) D>>d Hence ∆P = 2,
λ
d
Clearly, n << = 2 is not possible for any value of n.
λ
dy
Hence ∆p = cannot be used.
D
λ 1
For 1st maxima, ∆p = dsin θ = λ ⇒ sin θ= = ; ⇒ θ= 30°
d 2
1 7 . 8 | Wave Optics

1
Hence,=y D tan
= θ meter
3
(ii) Maximum path difference S1 y

∆Pmax ==
d 1mm
d
D
S2
d
⇒ Highest order maxima, nmax =
 = 2
λ 
Figure 17.5

d 1
And highest order minima nmin =  +  = 2
λ 2

Total no. of maxima ∗


= 2nmax + 1= 5 ∗
( central maxima)
Total no. of minima=
= 2n=
min 4

Illustration 11: Monochromatic light of wavelength 5000 A° is used in a Y.D.S.E., with slit-width, d=1mm, distance
between screen and slits, D is 1m. If the intensities at the two slits are,
= I1 4I
= 0 ,I2 I0, find  (JEE ADVANCED)

(i) Fringe width β


(ii) Distance of 5th minima from the central maxima on the screen.
1
(iii) Intensity at y = mm
3
(iv) Distance of the 1000th maxima from the central maxima on the screen.
(v) Distance of the 5000th maxima from the central maxima on the screen.

Sol: Refer to the formulas:-

λD 5000 × 10−10 × 1
(i) =
β = = 0.5mm
d 1 × 10−3
λD
( 2n 1)
(ii) y =−
2d
,n =
5 ⇒y =2.25mm

1 d.y
(iii)
= At y mm, y << D Hence ∆p =
3 D

2π dy 4 π
∆φ= ∆p= 2π =
λ λD 3
Now resultant intensity

I I1 + I2 + 2 I1 I2 cos ∆φ ; 4I0 + I0 + 2 4I20 cos ∆φ
= = 5I0 + 4I0 cos = 3I0
3
d 10−3
(iv)
= = 2000
λ 0.5 × 10−6

n=1000 is not <<2000

Hence now ∆p =d sin θ must be used


λ 1
Hence, d sin θ = nλ = 1000λ ⇒=
sin θ 1000
= ⇒ θ= 30°
d 2
P hysi cs | 17.9

1 Maximum intensity
Y=D tan θ = meter
3 4l0
d
(v) Highest order maxima nmax
= =  2000
λ 
Hence, n=5000 is not possible
Intensity variation on screen: If I0 is the intensity
of light beam coming from each slit, the resultant O
intensity at a point where they have a phase Minimum intensity
difference of φ is Fringe width

φ 2π ( dsin θ ) Figure 17.6


I cos2 , where φ
4I0 =
2 λ

For interference in reflected rays


1 1 2
When µ2 > µ1 , µ3; condition for
 1
(i) Maxima: 2tµ2 cosr =  n −  λ , n = 1,2,....
 2 2
t
(ii) Minima: 2tµ2 cosr =λ
n , n =1,2,....

When µ2 is in between µ1 & µ2 , condition for 3


(i) Maxima: 2tµ2 cosr =λ
n , n =1,2,.... 3 4
Figure 17.7
 1
(ii) Minima: 2tµ2 cosr =  n −  λ , n = 1,2,....
 2
For interference in transmitted rays, 3 and 4 conditions are interchanged for maxima and minima in both cases
of µ2 .

Illustration 12: When a plastic thin film of refractive index 1.45 is placed in the path of one of the interfering waves
then the central fringe is displaced through width of five fringes. Find the thickness of the film, if the wavelength of
light is 5890 Å. (JEE MAIN)

Sol: Path difference due to introducing of thin film.

β β ( 0.45 ) t 5 × 5890 10−10


∴ X 0= ( µ − 1 ) t ⇒ 5p= ; ∴ t=
0.45
= 6.544 × 10−4 cm
λ 5890 × 10−10

Useful tips: If two slits have unequal sizes (they correspond to intensity). The intensity of the resultant is

( I ) + ( I ) + (2 )
2 2
I= 1 2 I1 I2 cos φ

( ) (
I= I1 + I2 + 2 I1 I2 cos φ= k S1 + S2 + 2 S1S2 cos φ )
Where S1 & S2 is the size of slits

λ2 λ2 φ
Coherence length, Icoh = ; Coherence radius ρcoh = β=
∆λ φ 2
1 7 . 1 0 | Wave Optics

3.5 Optical Path


Actual distance travelled by light in a medium is called geometrical path ( ∆x ) . Consider a light wave given by the
equation
= E E0 sin ( ωt − kx + φ )
ω
If the light travels by ∆x , its phase changes by k∆x =∆x , where ω , the frequency of light does not depend on the
v c
medium, but v, the speed of light depends on the medium as v = .
µ
w
Consequently, change in phase ∆φ= k ∆x=
c
( µ∆x )
It is clear that a wave travelling a distance ∆x in a medium of refractive index µ suffers the same phase change as
when it travels a distance µ∆x in vacuum i.e. a path length of ∆x in medium of refractive index µ is equivalent to a
path length of µ∆x in vacuum.
The quantity µ∆x is called the optical path length of light, ∆xopt . And in terms of optical path length, phase
ω 2π
difference would be given by. ∆φ= ∆x = ∆x
c opt λ0 opt
where λ0 =wavelength of light in vacuum.
However in terms of the geometrical path length ∆x ,
ω 2π
∆φ
=
c
( )
µ∆x=
λ
∆x

 λ 
Where λ = wavelength of light in the medium  λ = 0  .
 µ 

PLANCESS CONCEPTS

Optical path must always be linked to phase of wave, so that it’s more convincing and useful. Only
learning manually will make it confusing and annoying.
If a material of thickness t interrupts the path of light and distance measured from position of an end of
material, then the phase of wave which is found at a distance x(<t) through the material will be same to
phase at distance (mu), if there was no material.
Fringe width (w) is the distance between two successive maximas or minimas. It is given by,
λD
=w ; or w ∝ λ .Two conclusions can be drawn from this relation:-
d
(i) If a YDSE apparatus is immersed in a liquid of refractive index µ , then wavelength of light and hence,
fringe width decreases µ times.

(ii) If white light is used in place of a monochromatic light then coloured fringes are obtained on the
screen with red fringes of larger size than that of violet, because λred > λ violet .
Vaibhav Gupta (JEE 2009, AIR 54)

Illustration 13: The wavelength of light coming from a sodium source is 589 nm. What will be its wavelength in
water? Refractive index of water =1.33.  (JEE MAIN)

Sol: The wavelength in water is λ = λ0 µ , where λ0 is the wavelength in vacuum and µ is the refractive index of
589
water. Thu=
λ = 443nm
1.33
P hysi cs | 17.11

3.6 Interference from Thin Films


Interference effects are commonly observed in thin films, such as thin layers of oil o
180 phase
on water or the thin surface of a soap bubble. The various colors observed when change No phase
white light is incident on such films result from the interference of waves reflected change
from the two surfaces of the film. Consider a film of uniform thickness t and index 1
2
of refraction µ as shown in the figure. Let us assume that the rays travelling in air are Air air< film
nearly normal to the two surfaces of the film. To determine whether the reflected
A
rays interfere constructively or destructively, we first note the following facts. Film
λ r
(i) The wavelength of light in a medium whose refractive index is µ is, λµ =
Where λ is the wavelength of light in vacuum (on air) µ B
(ii) If a wave is reflected from a denser medium, it undergoes a phase change of Air
180° . Let us apply these rules to the film shown in figure. The path difference
between the two rays. 1 and 2 is 2t while the phase difference between them Figure 17.8
λµ
is 180° . Hence, condition of constructive interference will be, = 2t ( 2n − 1 )
 1 λ 2
or, 2µt=  n −  λ as λµ =
 2 µ
Similarly, condition of destructive interference will be 2µt = nλ ; n=0, 1, 2, …

Illustration 14: Find the minimum thickness of a film which will strongly reflect the light of wavelength 589 nm.
The refractive index of the material of the film is 1.25.  (JEE MAIN)

Sol: Path difference due to introducing of thin film.


For strong reflection, the least optical path difference introduced by the film should be λ 2 . The optical path
difference between the waves reflected from the two surface of the film is 2µd .
λ 589nm
Thus, for strong reflection, 2µd =λ 2 or,=
d = = 118nm.
4µ 4 × 1.25

3.7 YDSE with Glass Slab

Path difference produced by a slab


Consider two light rays 1 and 2 moving in air parallel to each other. If a slab
, t
of refractive index µ and thickness t is inserted between the path of one of
the rays then a path difference ∆x = ( µ − 1 ) t is produced among them. This


1
can be shown as under,
Speed of light in air = c x
c 2
Speed of light in medium = Time taken by ray 1 to cross the slab,
µ
t µt Figure 17.9
t1 =
= and time taken by ray 2 cross the same thickness t in air will
c/µ c
t
be, t2
= as t1 > t2
c
Difference in time ∆t = t1 − t2 = ( µ − 1) ct
Optical path length: Now we can show that a thickness t in a medium of refractive index µ is equivalent to a length
µt in vacuum (or air). This is called optical path length. Thus,
Optical path length= µt
1 7 . 1 2 | Wave Optics

Shifting of fringes: Suppose a glass slab of thickness t and refractive


index µ is inserted onto the path of the ray emanating from source S1 . P

Then, the whole fringe pattern shifts upwards by a distance


( µ − 1) tD y
d
S1
. This can be shown as under Geometric path difference between S2P and
yd
S1P is, ∆x1 = S2P − S1P = Path difference produced by the glass slab
D
S2
∆x2 = ( µ − 1 ) t

Note: Due to the glass slab, path of ray 1 gets increased by ∆x2 . Therefore,
Figure 17.10
net path difference between the two rays is,
yd
∆x = ∆x1 − ∆x2 or, x
∆= − (µ − 1) t
D
yd
For nth maxima on upper side, or, − ( µ − 1 ) t = nλ ;
D

nλD ( µ − 1 ) tD
∴ y= +
d d

Earlier, it was
nλD
; Shift =
( µ − 1) tD
d d

Following three points are important with regard to Eq. above


(a) Shift is independent of n, (the order of the fringe), i.e.,
Shift of zero order maximum = shift of 7th order maximum
Shift of 5th order maximum = shift of 9th order minimum and so on.
(b) Shift in independent of λ , i.e., if white light is used then,
Shift of red colour fringes = shift of violet colour fringes.

(c) Number of fringes shifted = =


shift (=
µ − 1 ) tD d ( µ − 1 ) t
fringes width λD d λ

These numbers are inversely proportional to λ . This is because the shift is the same for all colours but the fringe
width of the colour having smaller value of λ is small, so more number of fringes of this colour will shift.

Illustration 15: In a YDSE with d=1mm and D=1m, slabs of ( t = 1µm, µ = 3) and ( t = 0.5 µm, µ = 2 ) are introduced
in front of the upper slit and the lower slit respectively. Find the shift in the fringes pattern. (JEE MAIN)

Sol: Path difference due to introducing of thin film.

Optical path for light coming from upper slit S1 is S1P + 1µm ( 2 − 1=
) S2P + 0.5µm
Similarly optical path for light coming from S2 is S2P + 0.5µm ( 2 − 1 ) = S2P + 0.5µm
yd
Path difference ∆=
p (S2P + 0.5µm) − (S1P + 2µm) = (S2P − S1P ) − 1.5µm = D
− 1.5µm

1.5µm
For central bright ∆p =0 ⇒=
y × 1m
= 1.5mm.
1mm
The whole pattern is shifted by 1.5mm upwards.
P hysi cs | 17.13

Illustration 16: Bichromatic light is used in YDSE having wavelength 4 7


l1 = 400 mm and l2 = 700 mm. Find minimum order of l1 which overlaps with 12
l 2.  (JEE ADVANCED) 3 1 
Sol: Fringe width depends on wavelength. 4
2
n1 λ1D n2 λ2D
Let n1 bright find of λ1 overlaps with λ2 . Then, = Or, 2
d d 1
n1 λ2 700 7 n 7 1
= = = . The ratio 1 = implies that 7th bright fringes of λ1
n2 λ1 400 4 n2 4 y=0 y=0
2 1
will overlap with 4th bright fringes of λ2 . Similarly 14th of λ1 will overlap with
Figure 17.11
8 of λ2 and so on.
th

So the minimum order of λ1 which overlaps with λ2 is 7.

Illustration 17: In YDSE, find the thickness of a glass slab ( µ =1.5 ) which should be placed before the upper slit S1
so that the central maximum now lies at a point where 5th bright fringe was lying earlier (before inserting the slab).
Wavelength of light used is 5000 Å.  (JEE MAIN)

Sol: Path difference due to introducing of thin film.


According to the question, Shift = 5 (fringe width)


( µ − 1) tD =
5λD
y
∴=

=
25000
=50,000 Å
d d µ − 1 1.5 − 1

Tip: If the light reaching P is direct (not reflected) from two sources then P will be a bright fringe if the path
difference = nλ
λ
= ( 2n + 1 ) → because the
If the light reaching P after reflection forms a bright fringe (at P) then path difference
2
λ
reflection causes an additional path difference of (or phase difference = π rad.) If the interference occurs due to
2
reflected light, central fringe (or ring in Newton’s rings) will be dark. If the interference occurs due to transmitted
light, central fringe (or ring in Newton’s rings) will be bright.

3.8 YDSE with Oblique Incidence


In YDSE, ray is incident on the slit at an inclination of θ0 to the axis of symmetry of the experimental set-up for
points above the central point on the screen, (say for P1 )
p dsin θ0 + ( S2P1 − S1P1 ) 
∆= Screen

⇒∆
=p dsin θ0 + dsin θ1 (if d<<D)
P1
S1
and for points below O on the screen, (say for P2 )

∆p
= ( dsin θ0 + S2P2 ) − S1P2= dsin θ0 − ( S1P2 − S2P2 )
0
3
1
2 O
P2
dsin4
∆p dsin θ0 − dsin θ2 (if d<<D)
⇒= S2

O’
We obtain central maxima at a point where, ∆p =0.
( d sin θ0 − d sin θ2 ) =0 or, θ2 =θ0 .
Figure 17.12
1 7 . 1 4 | Wave Optics

This corresponds to the point O’ in the diagram. Hence, we finally have the path difference as
d ( sin θ0 + sin θ ) − for points above O

∆p d ( sin θ0 − sin θ ) − for points between O & O'
=

d ( sin θ − sin θο ) − for points below O'

Illustration 18: In a YDSE with D=1m, d=1mm, light of wavelength 500 nm is incident at an angle of 0.57° w.r.t.
the axis of symmetry of the experimental set up. If the centre of symmetry of screen is O as shown, find:
(i) The position of central maxima.
(ii) Intensity at point O in terms of intensity of central maxima I0 .
S1 P
(iii) Number of maxima lying between O and the central maxima.  (JEE MAIN) 0.57
o y

S2 O

Sol: Path difference at central maxima =0.


Figure 17.13
 0.57 
(i) θ = θ0 = 0.57°
⇒ y⇒ = −Dtanθ
D tan θ==‒Dθ
−Dθ==‒1 meter × 
−1meter rad  ⇒ y =−1cm
 57 
(ii) For point 0, θ =0

Hence, ∆=
p dsin θ0  dθ=
0 ( )= 20 × (500nm)
1mm × 10−2 rad = 10,000nm

Hence point O correspond to 20th maxima ⇒ intensity at O=I0

(iii) 19 maxima lie between central maxima and O, excluding maxima at O and central maxima.

4. DIFFRACTION
The phenomenon of bending of light around the corners of an obstacle or Light A’
an aperture into the region of the geometrical shadow of the obstacle is
called diffraction of light. The diffraction of light is more pronounced when
A
the dimension of the obstacle/aperture is comparable to the wavelength of

Screen
the wave. S Obstacle

4.1 Diffraction of Light Due to Single Slit B

Diverging light from monochromatic source S is made parallel after refraction


Light B’
through convex lens L1 . The refracted light from L1 is propagated in the form
of plane wave front WW’. The plane wave front WW’ is incident on the slit Figure 17.14
AB of width ‘d’. According to Huygens’ Principle, each point of slit AB acts as
a source of secondary disturbance of wavelets.
Path difference: To find the path difference between the secondary wavelets originating from corresponding
points A and B of the plane wave front, draw AN perpendicular on BB’. The path difference between these wavelets
originating from A and B is BN.
P hysi cs | 17.15

L2
L1
W A 
P
x
d  
S Q
C
N B

W’ B

DL Screen
2

Figure 17.15

BN
From ∆BAN,
AB
sin θ
= Or, =
BN AB sin θ ∴Path difference, BN=d sinθ  dθ ( θ is small)
(a) For Minima: If the path difference is equal to one wavelength i.e., BN= d sinθ = λ , position P will be of
minimum intensity. Hence, for first minima, d sinθ1 =λ
Or sinθ1 =λ d …(i)
Or θ1 =λ d ( θ1 is very small) …(ii)
Similarly, if BN= 2λ ,
Thus, for second minima, dsin θ2 = 2λ Or sin θ2 = 2λ d ;  sinθ2 =θ2 Or θ2 = 2λ d

In general, for minima, dsin θm = mλ Or sin θm = mλ d



Since θm is small, so sin θm = θm ∴ θm = (here θ we use is in radians)
d
Where, θm is the angle giving direction of the mth order minima and m=1,2,3,…. Is an integer.

Note: Condition for minima → Path difference between two waves should be mλ , where m is an integer.
(b) For secondary maxima: If path difference, BN= d sin θ is an odd multiple of λ 2 ,

 2m + 1   2m + 1  λ
i.e. dsin
= θm   λ or sin θm = 
 2   2 d

 2m + 1  λ
Since θm is small, so sinθm =θm ; ∴ θm =  …(iv)
 2 d
M=1 ,2, 3, …. Is an integer
Let f be the focal length of lens L2 and the distance of first minima -3 -2 - - 2 3
x
on either side of the central maxima be x. Then, tan θ = -3 
f -2 - 2 3
Since the lens L2 is very close to the slit, so f=D d d d d d d

x x
∴ tan θ = Since θ is very small, so tan θ ≈ sin θ ∴ sin θ =  …(i)
d d Figure 17.16
y
Also, for first minima, dsin θ = λ or sin θ =  …(ii)
d
1 7 . 1 6 | Wave Optics

x λ λD
From eqns. (i) and (ii), we have = or x =  …(iii)
D d d

This is the distance of first minima on either side from the centre of the central maximum. Width of central maximum
2λD
is given by: ∴ 2x =
d
Diffraction pattern due to a single slit consists of a central maximum flanked by alternate minima and secondary
maxima is shown in figure.
Note: That sin θ =0 corresponds to central maxima while sinθ = π , corresponds to first minima.
Diffraction grating: It consists of a large number of equally spaced parallel slits. If light is incident normally on a
transmission grating, the direction of principal maxima is given by dsin θ = nλ
Here d is the distance between two consecutive slits and is called the grating element.
N=1, 2, 3,…. is the order of principal maximas.
Resolving power of the diffraction grating: The diffraction grating is most useful for measuring wavelengths
accurately. Like the prism, the diffraction grating can be used to disperse a spectrum into its wavelength components.
Of the two devices, the grating is the more precise if one wants to distinguish two closely spaced wavelengths.
For two nearly equal wavelengths λ1 and λ2 , between which a diffraction grating can just barely distinguish, the
resolving power R of the grating is defined as
λ λ
=R =
λ2 − λ1 ∆λ
where, λ = ( λ1 + λ2 ) 2 and ∆λ = λ2 − λ1 .

Illustration 19: A parallel beam of monochromatic light of wavelength 450 nm passes through a long slit of width
0.2 mm. Find the angular divergence in which most of the light is diffracted.  (JEE MAIN)

Sol: Most of the light is diffracted between the two first order minima. These minima occur at angles given by
bsinθ = ±λ
450 × 10−9 m
Or, sin θ = ± λ b =
± ±2.25 × 10 −3 Or, θ = ±2.25 × 10−3 rad
=
0.2 × 10 −3 m

= 4.5 × 10−3 rad .


The angular divergence

4.2 Diffraction by a Circular Aperture


Mathematical analysis shows that the first dark ring is formed by the light diffracted from the hole at an angle θ
λ
with the axis, where sin θ =1.22 Here, λ is the wavelength of the light used and b is the diameter of the hole. If
b λD
the screen is at a distance D(D>>b) from the hole, the radius of the first dark ring is R = 1.22
b

(a)

(b)
b

D P hysi cs | 17.17

(a)

(b)

Figure 17.17

If the light transmitted by the hole is converged by a converging lens at the screen placed at the focal plane of this
λf
lens, the radius of the first dark ring is R = 1.22
b
As most of the light coming from the hole is concentrated within the first dark ring, this radius is also called the
radius of the diffraction disc.

Illustration 20: A beam of light of wavelength 590nm is focused by a converging lens of diameter 10.0 cm at a
distance of 20 cm from it. Find the diameter of the disc image formed.  (JEE MAIN)

Sol: The angular radius of the central bright disc in a diffraction pattern from circular aperture is given by
1.22λ 1.22 × 590 × 10−9 m
sin θ = = = 0.7 × 10 −5 rad.
b −2
10.0 × 10 m

The radius of the bright disc is 0.7 × 10−5 × 20cm =1.4 × 10−4 cm

= 2.8 × 10 −4 cm
The diameter of the disc image

4.3 Diffraction of X-Rays by Crystals


The arrangement of atoms in a crystal of NaCl is shown
in the above figure. Each unit cell is a cube of length
of edge a. If an incident x-ray beam makes an angle θ X-rays
with one of the planes, the beam can be reflected from
both the planes. However, the beam reflected from the
lower plane travels farther than the beam reflected
from the upper plane.
Crystal
The effective path difference is 2d sinθ . The two X-rays
beams reinforce each other (constructive interference) tube
when this path difference is equal to some integer Photographic
multiple of λ . The same is true for reflection from the Collimator
film
entire family of parallel planes. Hence, the condition
for constructive interference (maxima in the reflected Figure 17.18
beam) is 2dsin θ= mλ where, m=1, 2, 3,…is an integer. Incident
Reflected
This condition is known as Bragg’s Law, after W.L. Bragg beam
beam
(1890-1971), who first derived the relationship. If the
wavelength and diffraction angle are measured, the
above equation can be used to calculate the spacing Upper plane
 
between atomic planes. d
Lower plane
Note: Each fringe in Young’s Double Split Experiment
d sin 
has equal intensity while in diffraction, the intensity
falls as the fringe order increases. Figure 17.19
1 7 . 1 8 | Wave Optics

Important Points:
(a) Types of diffraction: The diffraction phenomenon is divided into two types viz. Fresnel diffraction and
Fraunhofer diffraction. In the first type, either the source or the screen or both are at a finite distance from
the diffracting device (obstacle of aperture). In the second type, both the source and screen are effectively at
an infinite distance from the diffracting device. Fraunhofer diffraction is a particular limiting case of Fresnel
diffraction.
(b) Difference between interference and diffraction: Both interference and diffraction are the results of
superposition of waves, so they are often present simultaneously, as in Young’s double slit experiment.
However interference is the result of superposition of waves from two different wave fronts while diffraction
results due to superposition of wavelets from different points of the same wave front.

PLANCESS CONCEPTS

X-ray diffraction is used in crystals to find inter-atomic distance owing to the fact that wavelengths of
x-rays are of order of inter-atomic distance, the required condition for diffraction.
Nivvedan (JEE 2009, AIR 113)

5. RESOLVING POWER OF OPTICAL INSTRUMENTS


When the two images cannot be distinguished, they are said to be un-resolved. If the images are well distinguished.
They are said to be well resolved. On the other hand, if the images are just distinguished, they are said to be just
resolved.

D
 

(a)
Figure 17.20

Rayleigh Criterion: According to Rayleigh, two objects or points are just resolved if the position of the central
maximum of the image of one object coincides with the first minimum of the image of the other object as shown
in figure (a).

5.1 Limit of Resolution


The minimum distance of separation between two points so that they can be seen as separate (or just resolved) by
the optical instrument is known as its limit of resolution. Diffraction of light limits the ability of optical instruments.
P hysi cs | 17.19

PLANCESS CONCEPTS

Resolving Power: The ability of an optical instrument to form distinctly separate images of the two
closely placed points or objects is called its resolving power. Resolving power is also defined as reciprocal
of the limit of resolution,
1
R.P. =
Limit of resolution
Smaller the limit of resolution of an optical instrument, larger is its resolving power and vice-versa.
Resolving Power of Eyes: Since eye lens is a converging lens, the limit of resolution of the human eye
1.22γ
is that of the objective lens of a telescope i.e. limit of resolution of the eye, α =
D
Where, D= diameter of the pupil of the eye.
1 D
Resolving power of the eye= =
α 1.22λ
Resolving power of an astronomical telescope: Resolving power of a telescope,
1 D
R.P. = or R.P. =
Limit of resolution 1.22λ

Chinmay S Purandare (JEE 2012, AIR 698)

Illustration 21: A star is seen through a telescope having objective lens diameter as 203.2cm. If the wavelength of
light coming from a star is 6600 Å, find (i) the limit of resolution of the telescope and (ii) the resolving power of the
telescope.  (JEE MAIN)
1 D
Sol: Resolving power of the eye= = 6600 × 10−8 cm =
Here, D=203.2cm and λ = 6600  Å = 6.6 × 10 −5 cm
α 1.22λ
1.22λ 1.22 × 6.6 × 10−5
(i) Limit of resolution of telescope, α = = = 3.96 × 10−7 rad
D 203.2

1 D 1
(ii) Resolving power of telescope,= = = 2.53 × 106
α 1.22λ 3.96 × 10−7
1
Resolving power of a microscope: Resolving power of a microscope= i.e.
dmin
2nsin β
R.P.microscope =
1.22λ

6. SCATTERING OF LIGHT
Scattering of light is a phenomenon in which a part of a parallel beam of light appears in directions other than the
incident radiation when passed through a gas.
Process: Absorption of light by gas molecules followed by its re-radiation in different directions.
The strength of scattering depends on the following:
(a) Loss of energy in the light beam as it passes through the gas
(b) Wavelength of light
(c) Size of the particles that cause scattering
1 7 . 2 0 | Wave Optics

Key point: If these particles are smaller than the wavelength, the scattering is proportional to 1 λ 4 . This is known
as Rayleigh’s law of scattering. Thus, red is scattered the least and violet is scattered the most. This is why, red
signals are used to indicate dangers. Such a signal transmit to large distance without an appreciable loss due to
scattering.
Practical example of scattering: The blue color of the sky is caused by the scattering of sunlight by the molecules
in the atmosphere. This scattering, called Rayleigh scattering, is more effective at short wavelengths (the blue
end of the visible spectrum). Therefore, the light scattered down to the earth at a large angle with respect to the
direction of the sun’s light, is predominantly in the blue end of the spectrum.

7. POLARIZATION OF LIGHT
The process of splitting of light into two directions is known as polarization.
y

Direction of
propagation of light

Figure 17.21

Phenomenon of polarization: The phenomenon of restricting the vibrations of a light vector of the electric field
vector in a particular direction in a plane perpendicular to the direction of propagation of light is called polarization
of light. Tourmaline crystal is used to polarize the light and hence it is called polarizer.
y
P
y

x
x
z
Source Unpolarised Polarised
light z light
Q

Pictorial representation of polarised light shown figure (A) and (B)

(Vibrations in the (Vibrations  to the


plane of paper) plane of paper)

(A) (B)
Figure 17.22
P hysi cs | 17.21

7.1 Unpolarized Light


(a) An ordinary beam of light consists of a large number of waves emitted by the atoms or molecules of the
light source. Each atom produces a wave with its own orientation of electric vector E. However, because all
directions are equally probable, the resulting electromagnetic wave is a superposition of waves produced by
individual atomic sources. This wave is called as an unpolarized light wave.
(b) All the vibrations of an unpolarized light at a given instant can be resolved into two mutually perpendicular
directions and hence an unpolarized light is equivalent to superposition of two mutually perpendicular
identical plane polarized lights.

7.2 Plane Polarized Light


(a) If somehow we confine the vibrations of electric field vector in one direction perpendicular to the direction
of wave motion of propagation of wave, the light is said to be plane polarized and the plane containing the
direction of vibration and wave motion is called the plane of polarization.
(b) If an unpolarised light is converted into plane polarized light, its intensity reduces to half.
(c) Polarization is a proof of the wave nature of light.

Partially polarized light: If in case of unpolarised light, electric field vector in some plane is either more or less
than in its perpendicular plane, the light is said to be partially polarized.

7.3 Polaroids
Polaroid
A polaroid is a device used to produce plane polarized light. The direction
perpendicular to the direction of the alignment of the molecules of the
Polaroid is known as pass-axis or the polarizing direction of the Polaroid.
Note: A Polaroid used to examine the polarized light is known as analyzer.
Plane polarised
light

7.4 Malus’ Law Figure 17.23

This law states that the intensity of the polarised light transmitted through Q
the analyser varies as the square of the cosine of the angle between the plane
of transmission of the analyser and the plane of the polariser. Resolve E into
E
two components: We know, intensity ∝ ( Amplitude )
2 P
V

∴ Intensity of the transmitted light through the analyser is given by E sin  E cos 
Iα (Ecos θ ) or=
2 2 2
I kE cos θ ; O
Figure 17.24
But kE2 = I0 (intensity of the incident polarised light)

=I I0 cos2 θ Or I ∝ cos2 θ which is Malus Law.

PLANCESS CONCEPTS

Plane polarized light is used for chemical purposes in measuring optical rotations of various chemical
compounds. It can also be used for stating the difference in enantiomeric compounds.
Nitin Chandrol (JEE 2012, AIR 134)
1 7 . 2 2 | Wave Optics

7.5 BREWSTER’S LAW


According to this law, the refractive index of the refractive medium (n) is numerically equal to the tangent of the
angle of polarization ( IB ) . i.e. n = tanIB

Illustration 22: What is the polarizing angle of a medium of refractive index 1.732? (JEE MAIN)

Sol: As per Brewster’s law, n = taniB or iB= tan−1 n


= tan−1 1.732
= 60°

PROBLEM-SOLVING TACTICS
 1. Most of the questions in JEE are related to Young’s Double slit experiment with minor variations. For any such
problem, drawing a rough figure and writing down the given parameters is a good idea before solving the
question.
 2. Wave optics has a lot of derivations. It is advisable to remember the end results for faster problem solving.
 3. Use the concept of optical path carefully and check for phase relations.
 4. Only direct formulae related questions are asked from sections of diffraction, polarizations and scattering so
these formulae must be learnt.

FORMULAE SHEET

S. No. Term Description


1 Wave front It is the locus of points in the medium which at any instant are vibrating in
the same phase.
2 Huygens’ Principle 1 Each point on the given primary wave front acts as a source of
secondary wavelets spreading out disturbance in all direction.
2 The tangential plane to these secondary wavelets constitutes the
new wave front.
3 Interference It is the phenomenon of non-uniform distribution of energy in the
medium due to superposition of two light waves.
4 Condition of maximum intensity φ= 2nπ or x = nλ , where n=0,1,2,3,….

6 Condition of minimum intensity


=
φ ( 2n + 1) π or=x ( 2n + 1) y 2 where n=0,1,2,3…

7 Ratio of maximum and minimum


( a1 + a2 )
2
intensity Imax
=
Imin
( a1 − a2 )
2

8 Distance of nth bright fringe from


nDλ
centre of the screen yn = ,
d
where d is the separation distance between two coherent sources of light,
D is the distance between screen and slit, λ is the wavelength used.
P hysi cs | 17.23

S. No. Term Description


9 Angular position of nth bright yn nλ
fringe θn= =
D d

10 Distance of nth dark fringes from


centre of the screen yn′ =
( 2n + 1)Dλ
2d

11 Angular position of nth dark fringe


θn′ =
yn′
=
( 2n + 1) λ
D 2d

12 Fringe width Dλ
β=
d

Diffraction and polarization of light

S. No. Term Description


1 Diffraction It is the phenomenon of bending of light waves round the sharp corners
and spreading into the regions of the geometrical shadow of the object.
2 Single slit diffraction nλ
Condition for dark fringes is sin θ = where n =±1, ±2, ±3, ±4....., a is
a

the width of the slit and q is the angle of diffraction. Condition for bright
( 2n + 1) λ
fringes is sin θ =
2a
3 Width of central maximum is 2λD
θ0 = , where D is the distance between the slit and the screen.
a

4 Diffraction grating The arrangement of large number of narrow rectangular slits of equal
width placed side by side parallel to each other. The condition for maxima
in the interference pattern at the angle θ is dsin θ = nλ where n=0, 1, 2,
3, 4…..
6 Resolving power of the grating For nearly two equal wavelengths λ1 and λ 2 between which a diffraction
λ λ
grating can just barely distinguish, resolving power
= is R =
λ1 − λ2 ∆λ
where λ = ( λ1 + λ2 ) 2

7 Diffraction of X-Rays by crystals The condition for constructive interference is 2dsin θ = nλ

8 Polarisation It is the phenomenon due to which vibrations of light are restricted in a


particular plane.
9 Brewster’s law µ =tan p where µ is refractive index of medium and p is the angle of
polarisation.

10 Law of Malus
= I I0 cos2 θ where I is the intensity of emergent light from analyser, I0 is
the intensity of incident plane polarised light and θ is the angle between
planes of transmission of the analyser and the polarizer.
1 7 . 2 4 | Wave Optics

Solved Examples

JEE Main/Boards β=
D
( )
λ=
1 × 5896 × 10−10
= 2.948 × 10−4 m
= 0.3mm
d 2 × 10 −3

Example 1: In a YDSE, if the source consists of two Now if the system is immersed in water
wavelengths λ1 =4000Å and λ2 =4000 Å, find the
distance from the centre where the fringes disappear, ( β )w λw v
= = as v =
fλ andf =
const. 
if d 1cm;D
= = 1m . β λ c
 c
Sol: The fringes disappear when the maxima of λ1 Or ( β ) = ( β µ ) as µ = 
w
 v 
p p 1
fall over the minima of λ2 . That is − =
λ1 λ2 2 ∴ ( β=
) ( 0.3) × (3 =
4) 0.225mm
w
where p is the optical path difference at the point or
λ1 λ2 Example 4: A beam of light consisting of two
p=
2 ( λ1 − λ2 ) wavelengths 6500 Å and 5200 Å is used to obtain
interference fringes in a Young’s double slit experiment.
Here,
= λ1 4000Å,
= λ2 4002Å
(a) Find the distance of the third bright fringe on the
∴p =0.04cm screen from central maxima for 6500 Å wavelength.
In YDSE (b) What is the least distance from the central maxima
where the bright fringes due to both the wavelengths
dy D (1000 )
p=
D
∴ y=
d
p=
10
( 0.4 ) = 40mm coincide? The distance between slits is 2 mm and
distance between slits and screen is 120 cm.

Example 2: Light from a source consists of two


Sol: Calculate the fringe width of both light individually.
wavelengths λ1 =6500 Å and λ2 = 5200 Å . If D = 2m
and d=6.5 mm, find the minimum value of y ( ≠ 0 ) d = 2 × 10−3 m,D = 120cm = 1.2m, λ1 = 6500Å
where the maxima of both the wavelengths coincides.
= 6.5 × 10−7 , λ2 = 5200 Å= 5.2 × 10−7 m
Sol: The point where maxima coincides, consider n1th n1 λ1D 3 × 6.5 × 10−7 × 1.2
maxima of λ1 coincide with n2th maxima of λ2 = x =
(a) n1 = 3, 1 d 2 × 10−3
Then yn1 = yn2 =1.17 × 10−3 m =1.17mm
n1 λ1D n2 λ2D n1 λ1 5200 4 (b) x = n1β1 = n2β2 where n1 and n2 are number of bright
Or = or = = =
d d n2 λ2 6500 5 fringes which coincide for respective wavelengths

Thus, fourth maxima of λ1 coincides with fifth maxima n1 λ1D n2 λ2D


λ1 and λ2 . = or n1 λ1 = n2 λ2 .
of λ2 The minima value of y ( ≠ 0 ) is d d
n1 λ 2 5200Å 4

given=
by y =
−6
(
4λ1D 4 0.65 × 10 ( 2 )
= 0.8mm
) or = = =
n2 λ1 6500Å 5
d 6.5 × 10 −3
min. value of n1 = 4 and n2 = 5
Example 3: In Young’s experiment for interference of Distance of a point at fringes coincide
light, two narrow slits 0.2 cm apart are illuminated by
yellow light ( λ =5896Å ) . What would be the fringe n1 λ1D 4 × 6500 × 10−10 × 1.2
= = 1.56mm
width on a screen placed 1m from the plane of slits. d 2 × 10−3
What will be the fringes width if the whole system is
immersed in water ( µ =4 3) ? Example 5: What is the effect on the interference fringes
in a YDSE due to each of the following operations.
Sol: Wavelength changes inside water, hence
(a) The screen is moved away from the plane of the slits
d = 2 × 10−3 m, λ = 5896 × 10−10 m andD = 1m ,
P hysi cs | 17.25

(b) The (monochromatic) source is replaced by another a


= 10−3 m
µm 1.33,d
=
(monochromatic) source of shorter wavelength
1.33 × 6300 × 10−10
(c) The separation between the two slits is increased
= β′ = 0.63mm
1.33 × 10−3
(d) The monochromatic source is replaced by source of
(b) If t is the required minimum thickness of sheet of
white light
refractive index m µg with respect to the medium, the
(e) The whole experimental setup is placed in a medium
of refractive index µ shift in path of the fringes =
D
d
= m µg − 1 t ( )
Sol: Recall the formulas for YDSE and analysis the The shift required to bring adjacent minima on the axis
changes, half width β ' Dλ
 λ = = =
(a) Angular separation  =  of the fringes remains 2 2 2a µ d
 d m

constant. But the linear separation or fringes width


increases in proportion to the distance (D) from the Equating, =
β' Dλ
2 2 µ d d
a
=
D m
µg − 1 t ( )
screen. m

(b) As λ decreases, fringe width ( ω ∝ λ ) decreases λ λ


= t =
 1
(c) As d increases, fringe width  ω ∝  decreases (m
2 µg − 1 µm a
) 2
 a
µ 
 g − 1 a µm
 d  a µm 
 
(d) If a white light is used in the double slit experiment, −10
the different colours will be split up on the viewing = λ 6300 × 10
t = = 1.575µm
screen according to their wavelengths. The violet
end of the spectrum (with the shortest wavelengths)
(a a
2 µg − µm 2 )
(1.53 − 1.33)
is closer to the central fringes, with the other colours
being further away in order. Example 7: A beam of light consisting of two
λ wavelengths 6500 Å and 5200 Å is used to obtain
(e) Since in a medium, the wavelength of light is λ ′ = , interference fringes in a Young’s double slit experiment.
µ
λ ′D λD (a) Find the distance of the third bright fringe on the
therefore the fringe width is given by= ω =
d µd screen from the central maxima for the wavelength
6500 Å. (b) What is the least distance from the central
Example 6: A double slit apparatus is immersed in a maxima where the bright fringes due to both the
liquid of refractive index 1.33. It has a slit separation of wavelength coincide? The distance between the slits is
1.0mm and the distance between the plane of slits and 2 mm and the distance between the plane of the slits
screen is 1.33m. The slits are illuminated by a parallel and screen is 120cm.
beam of light whose wavelength is 6300 Å. One of the
slits is covered by a thin glass sheet of refractive index Sol: According to the theory of interference, position y
1.53.
D
(a) Find the fringe width
of a point on the screen is given by= y
d
( ∆x ) and as
(b) Find the smallest thickness of the sheet required for 3rd maximum ∆x = 3λ
to bring the adjacent minima at the position of earlier
central maxima. y
=
D
d
( )
3λ=
120
0.2
(
3 × 6500 × 10−8 cm
= 0.117cm )
Sol: Change in the wavelength occur due to liquid,
also as β = ( Dλ d ) , y = 3β i.e.
= β ( y=
3) 0.039cm.
hence parameter depending on wavelength will change,
(b) If n is the least number of fringes of λ1 ( =6500Å ) ,
(a) If the double slit apparatus is immersed in a liquid
medium of refractive index a µm then fringe width which are coincident with (n + 1 ) of smaller wavelength
λD n + 1 β λ1
β′ =
a
µmd
λ2 (=5200 Å), y ′ = nβ = (n + 1) β′, i.e., n
= =
β′ λ 2

= = 10−3 m ; λ =6300Å,D = 1.33m


λ 1.33,d λ2 5200 5200
or n
= = = 4
λ1 − λ2 6500 − 5200 1300
1 7 . 2 6 | Wave Optics

So y ′ = 4β = 4 × 0.039 = 0.156cm. As the intensity is proportional to the square of the


( Amax ) (3A )
2 2
Imax
amplitude,= = = 9
Example 8: In a YDSE, λ =60nm ;= D 2m;= λ 6mm. Imin
( Amin )
2
A2
Find the position of a point lying between third maxima
and third minima where the intensity is three-fourth of
Example 10: In a single slit diffraction experiment first
the maximum intensity of the screen.
minima for λ1 – 660 nm coincides with first maxima for
Sol: Formula for intensity in terms of phase difference, wavelength λ2. Calculate λ2.

φ Sol: Position of minima on diffraction pattern is given


Using equation I = 4I0 cos2 by dsin θ − nλ
2

here I
=
3
= ( 4I0 ) 3I0
φ
∴ cos =
3 For first minima of λ1, we have dsin θ1 = (1) λ1
4 2 2
φ π π λ
Thus, = nπ ± or φ= 2nπ ± or sin θ1 = 1  …(i)
2 6 3 d
The first maxima approximately lies between first and
2π 2π  d.yn  second maxima. For wavelength λ2, its position will be,
Since=φ = p  
λ λ  D 
3 3λ
dsin θ2 = λ2 ∴ sin θ2 = 2  …(ii)
2π dyn π  1  λD 2 2d
∴ = 2nπ ± or y=
n n ± 
λ D 3  6 d
The two will coincide if θ1 =θ2 or sin θ1 = sin θ2
For the point lying between third minima and third λ1 3λ2 2 2
maxima, = or λ2 = λ1 = × 660nm =440nm
d 2d 3 3
 1  λD 17 λD
N=3 and y=
3 3 −  or y 3 =
 6  d 6 d

λ 0.6 × 10−6 m;D


Putting = = 2m;d
= 6mm
JEE Advanced/Boards

y3
(
= 5.67mm
−6
)
17 0.6 × 10 m ( 2 )
Example 1: Two monochromatic coherent sources of
wavelength 5000 Å are placed along the line normal to
6 6 × 10−3 the screen as shown in the figure.

Example 9: The width of one of the two slits in a P


Young’s double slit experiment is double of the other
slit. Assuming that the amplitude of the light coming
from a slit is proportional to the slit-width, find the O
ratio of the maximum to the minimum intensity in the S1 S2
interference pattern.
d D
Sol: Suppose the amplitude of the light wave coming
from the narrower slit is A and that coming from the (a) Determine the condition for maxima at point P.
wider slit is 2A. The maximum intensity occurs at a
(b) Find the order of the central bright fringe if
place where constructive amplitude is the sum of the
d=0.5.mm, D=1m.
individual amplitudes. Thus, Amax = 2A + A = 3A
The minimum intensity occurs at a place where Sol: Consider the given geometry and get the optical
destructive interference takes place. The resultant path difference at P is p = S1P − S2P = dcos θ
amplitude is then the difference of the individual
amplitudes. Thus, Amin = 2A − A = A
P hysi cs | 17.27

The integers between 5.71 and 10 are 6,7,8,9 and 10.


P
∴ The wavelength which forms maxima at P2 are
y λ1 =4000Å for n = 10

O λ 2 =4444 Å for n = 9
S1 S2 λ3 =5000Å for n = 8

d D λ4 =5714 Å for n = 7
λ5 = 6666Å for n = 6
θ2
Since cos θ = 1 − when θ is small, Therefore,
2 Example 3: A monochromatic beam of λ =5000Å is
 θ2   y2  incident on two narrow slits separated by a distance of
p = d  1 −  = d 1 −
  Where D + d ≈ D 7.5 × 10−4 m to produce interference pattern on a screen
 2   2D 
2
placed at a distance of 1.5 m from the slits in Young’s
 nλ  double slit experiment. A thin uniform glass plate of
For nth maxima, p = nλ =
∴ y D 2 1 −  µ =1.5 and 2.5 × 10−6 m thickness is placed normal to
 d 
the beam between one of the slits and the screen.
(b) At the central maxima, θ =0 (a) Find the internal shift of the central maxima.
d 0.5 (b) If the intensity at the central maxima is I0 , find the
∴ p = d = nλ or n= = = 1000
λ 0.5 × 10−3 intensity at this point after the glass plate is introduced.
(c) If the glass plate is removed and the slits are also
Example 2: In a YDSE conducted with white light (4000
illuminated by an additional light of 4000 Å , find the
Å-7000 Å), consider two points P1 and P2 on the screen
least distance from the central maxima wavelengths
at y1 =0.2mm and y 2 =1.6mm, respectively. Determine
will coincides.
the wavelength which forms maxima at these points.
Sol: Consider the theory of path difference on
Sol: Start from the formulas maxima where, the optical introducing the plate as,
path difference at P1 is
d 7.5 × 10−4 m, µ
(a) D=1.5m, = = 2.5 × 10−6 m
= 1.5,t
Lateral shift of central maxima
P2

D 1.5 × 0.5 × 2.5 × 10−6


( µ − 1 )=
S1 y2=1.5mm
t = 2.5 × 10−3 m
P1 d 7.5 × 10 −4
d=1cm y1=0.2mm
= 2.5mm

S2
(b) Path difference caused by sheet at

4m O= x= ( µ − 1) t = (1.5 − 1) × 2.5 × 10−6


= 1.25 × 10−6 m
dy1  10 
p1 = =  ( 0.2 ) =
5 × 10 −4 mm =
5000Å In the Corresponding phase difference
D  4000 
visible range 4000-7000 Å,
2πx 2π × 1.25 × 10−6
=φ= = = 5π
5000 5000 λ 5000 × 10−10
n1
= = 1.25 and = n2 = 0.714 The only
4000 7000
integer between 0.714 and 1.25 is 1. A2 = a12 + a22 + 2a1a2 cos φ,

∴ The wavelength which forms maxima at P is λ Let a=


1 a=
2 a
=5000Å
dy 2  10 
p2 = =   (1.6 ) =
4 × 10−3 mm =
40000Å
D  4000 
40000 40000
Here
= n1 = 10 and
= n2 = 5.71
4000 7000
1 7 . 2 8 | Wave Optics

P
S1

 
d O
O
S2

Path difference, ∆x , for a point P is given by,


∴ A2 = a2 + a2 + 2a2 cos (5π ) = 2a2 − 2a2 = 0
x dsin θ
∆= ( for small θ,tan θ=sinθ )
A2 = 0, there will be minima at O.
Phase difference
(c) Let n1th maxima of λ1 =5000 Å will coincide With n2th
2π 2π 2π
maxima of λ2 =4000 Å =φ = ( ∆x ) = ( dsin θ ) = × (150 sin θ ) =π sin θ
λ λ 300
n1 λ1D n2 λ2D Resultant intensity due to both slits
=
d d
=I =I1 + I2 + 2 I1 I2 cos φ
n1 λ 2 4000Å 4
= = =
n2 λ1 5000Å 5 As, I1 = I2

For least distance where bright fringes coincide,  π sin θ 


I = 2I1 1 + cos ( π sin θ )  = 4I1 × cos2  
n1 4,n
= = 2 5  2 

4 × 5000 × 10−10 × 1.5  π sin θ 


∴x = = 4 × 10 −3 m For I to be maximum, cos2   =1
7.5 × 10 −4  2 
 π sin θ 
Example 4: In an interference arrangement similar ∴ Imax = 4I1 = I0 or I = I0 cos2  
to Young’s double slit experiment, slits S1 and S2 are  2 
illuminated with coherent microwave sources each = I0 cos2 ( 0=
of frequency 1 MHz. The sources are synchronized to
(a) For =
θ 0,I ) I0
have zero phase difference. The slits are separated by
π I
a distance d=150m. The intensity I0 is measured as (b) For θ= 30°,I= I0 cos2  = 0
a function of θ with respect to the axial line passing 4 2
through the middle point of the slits. If I0 is maximum π
(c) For θ= 90°,I= I0 cos2  = 0.
intensity, calculate I ( θ ) for 2
(a) θ =0
Example 5: In a modified Young’s double slit experiment,
(b) θ= 30° and
a monochromatic uniform and parallel beam of light
(c) θ= 90° 10
of wavelength 6000 Å and intensity W m2 is
π
incident normally on two circular apertures A and B
Sol: Addition of Amplitudes of different wavelength.
of radii 0.001m and 0.002 m respectively. A perfectly
Wavelength of microwaves λ is given by transparent film of thickness 2000 Å and refractive
c 3 × 108 index 1.5 for wavelength 6000 Å is placed in front of the
λ= = = 300m . aperture A. Calculate the power in watts received at the
f 106
focal spot F of the lens. The lens is symmetrically placed
with respect to the aperture. Assume that 10% of the
power received by each aperture goes in the original
direction and is brought to the focal spot.
P hysi cs | 17.29

Sol: First get the intensities and then apply Power= of the power received by each aperture goes in the
IxArea original direction and is brought to the focal spot.
If E is the energy incident in time t for surface area a,
E Sol: As intensity and power are defined as
then intensity I =
at E E
I= , P= and P=IS
E St t
Power = or Power = Ia
t So power received at A and B is respectively,
10
× π × ( 0.001 )= 10−5 W
2
Power received at A
= P= 10 2
A
π PA= × π(0.001)= 10−5 W and
π
10
× π × ( 0.002 ) = 4 × 10−5 W
2
Power received at B= PB = 10
π PB = × π(0.002)2 = 4 × 10−5 W
As only 10% of the power goes in the original direction, π
and as only 10% of the incident power passes,
10
PA′ = × 10−5 =10−6 W 10
100 PA′ = × 10−5 =10−6 W and
10 100
PB′ = × 4 × 10−5 = 4 × 10−6 W
100 10
Pa′ = × 4 × 10−5 = 4 × 10−6 W
Path difference due to film of thickness t and refractive 100
index µ = ∆x = ( µ − 1 ) t
Now due to 10% of the introduction of film the path
2π difference produced
Phase difference produced = φ =
λ
(µ − 1) t
L

φ ) × 2000 × 10−10 3π
(1.5 − 1= A
6000 × 10 −10
F
If I1 is intensity of A and I2 is intensity of B, the
resultant intensity I due to interference is given by, B

I = I1 + I2 + 2 I1 I2 cos φ
∆x = ( µ − 1) t = (1.5 − 1) × 2000 = 1000Å

The resultant power, P, at area a of focal spot is given by, 2π 2π π


So, =
φ
λ
( ∆=
x)
6000
× 1000
=
3
P = Ia = I A a + IBb + 2 I A IB × a × cos φ
But as in interference,
P = PA′ + PB′ + 2 PA′ PB′ .cos φ
I = I1 + I2 + 2 I1 I2 cos φ
= (10 ) + ( 4 × 10 )
−6 −6
+ 2 × 1 × 4 × 10 − 12 × cos
π
3 and S is the area of focal sport,
 1 
( )

10−6 1 + 4  4 ×   =×
= 7 10−6 Watt. P =IS =I A S + IBS + 2S I A IB cos φ
  2 

i.e., P = PA′ + PB′ + 2 PA′ PB′ cos ( π 3)


Example 6: In a modified Young’s double slit experiment,
a monochromatic uniform and parallel beam of light
of wavelength 6000 Å and intensity (10/π)W/m2 is or, P = 10−6 1 + 4 + 2
 ( )
1 × 4 × (1 2 )  = 7 × 10−6 watt.

incident normally on two circular apertures A and B
of radii 0.001 m and 0.002 m respectively. A perfectly
Example 7: In an interference arrangement similar
transparent film of thickness 2000 Å and refractive
to Young’s double-slit experiment, slits S1 and S2 are
index 1.5 for wavelength 6000 Å is placed in front of
illuminated with coherent microwave sources each
aperture A. Calculate the power (in watts) received at
of frequency 1 MHz. The sources are synchronized to
the focal spot F of the lens. The lens is symmetrically
have zero phase difference. The slits are separated by
placed with respect to the aperture. Assume that 10%
distance d=150.0m. The intensity I( 0 ) is measured as a
1 7 . 3 0 | Wave Optics

function of θ, where θ is defined as shown in figure. If Sol: Path difference at O, S1O − S2O =2λ i.e., maximum
I0 is maximum intensity, calculate I( 0 ) for (a) θ = 0° (b) intensity is obtained at O. Next maxima will be obtained
θ= 30° and (c) θ= 90° at point P where,

Sol: For microwaves, as c = fλ , λ Or dcosθ = λ


S1P − S2P =

λ=
c 3 × 108
= = 300m
Or ( 2λ ) cos θ = λ
f 106 1
Or cos θ =
2
And as ∆=
x dsin θ
∴ θ= 60°
2π 2π
φ= (dsin θ) = (150 sin θ) =π sin θ
λ 300
P
So, I = I1 + I2 + 2 ( )
I1 I2 cos φ X
O
S1 S2
P
S1 d
y D
 
d PO
Now in ∆S1PO , = tan θ or
{

S1O
S2 x
x
D = tan60
= ° 3 ; ∴ x=
3D
d
With I1 = I2 and φ = π sin θ reduced to Note: At point O, the path difference is 2λ , i.e., we
obtain second order maxima. At point P, where path
IR= 2I1 1 + cos ( π sin θ )=
 4I1 cos2 ( π sin θ 2 )
 difference is λ (i.e., x = 3D ), we get first order maxima.

(
and as IR will be max. when cos2  π ( sin θ ) 2 = max = 1
 
The next, i.e., zero order maxima will be obtained where
path difference, i.e., dcos θ =0 or θ= 90° , x = ∞ . So,
So that, ( IR ) = 4I
= 1 I0 (given) our answer i.e., finite distance of x should be x = 3D ,
max
corresponding to first order maxima.
And hence,
= I I0 cos2 ( π sin θ ) 2  …(i)

0 cos ( 0 )
θ =0 I I=
So (a) If = 2
I0 Example 9: An interference is observed due to two
coherent sources S1 placed at origin and S2 placed
(b) If θ= 30°=I I0 cos2 ( π =
4) ( I0 2 ) at ( 0,3λ ,0 ) . Here λ is the wavelength of the sources.
(c) If θ= 90° s =I I0 cos2 ( π =
2) 0 A detector D is moved along the positive x-axis. Find
x-coordinates on the x-axis (excluding x=0 and x = ∞ )
where maximum intensity is observed.
Example 8: Two coherent narrow slits emitting lights
of wavelength λ in the same phase are placed parallel
Sol: At x = 0, path difference is 3λ . Hence, third order
to each other at a small separation of 2λ . The light is
maxima will be obtained. At x = ∞ , path difference is
collected on a screen S which is placed at a distance
zero. Hence, zero order maxima is obtained in between
(D>> λ ) from the slit S1 as shown in figure. Find the
first and second order maximas.
finite distance x such that the intensity at P is equal to
intensity at O. Y

P
X S2
S1 S2
O 3
2

D S1 X
P
S X
P hysi cs | 17.31

(b) Position of central maximum is shifted upwards by a


First order maxima: S2P − S1P =
λ Or x2 + 9λ2 − x =λ
distance
( µ2 − 1) tD
Or x 2 + 9λ 2 = x + λ d
Squaring both sides, we get  µ3 
 − 1  tD
2 2 2
x + 9λ = x + λ + 2xλ 2
(c)
( µ2 − 1) tD =  µ1 
Solving this, we get x= 4λ d d
µ3
Second order maxima: S2P − S1P =
2λ ∴ = µ2 or µ3 = µ1µ2
µ1
Or x2 + 9λ2 − x = 2λ φ
(d) I = Imax cos2  
2
Or x 2 + 9λ 2 = ( x + 2λ )
 2π  φ π φ
Squaring both sides, we get x2 + 9λ2 = x2 + 4λ2 + 4xλ Where
= φ   ∆x or=   ∆x I ∝ cos2  
λ
  2 λ
  2
5
Solving, we get x= λ= 1.25λ In the first and third case, ∆x =0 while in second case,
4
∆x = ( µ2 − 1 ) t . Therefore, the desired ratio is
Hence, the desired x coordinates are,
 π ( µ2 − 1 ) t 
x 1.25λ and x= 4λ
= I1 : I2 : I3 = I : cos2  :1
 λ 
Example 10: A young double slit apparatus is immersed
in a liquid of refractive index µ1 . The slit plane touches Example 11: In a Young’s experiment, the light source
the liquid surface. A parallel beam of monochromatic is at distance I=
1 20µm and I=2 40µm from the slits.
light of wavelength λ (in air) is incident normally on the A light of wavelength λ =500 nm is incident on slits
slits. separated at a distance 10µm . A screen is placed at a
distance D=2m away from the slits as shown in figure.
Find
(a) The values of θ relative to the central line where
S2 maxima appear on the screen?
d
O (b) How many maxima will appear on the screen?
S2
P
S l1
D S1
l2 
(a) Find the fringe width. d
C

(b) If one of the slits (say, S2 ) is covered by a transparent S2


slab of refractive index µ2 and thickness t as shown, find D
the new position of the central maxima.
(c) Now the other slit S1 is also covered by a slab of (c) What should be the minimum thickness of a slab of
same thickness and refractive index µ3 as shown in the refractive index 1.5 be placed on the path of one of the
figure due to which the central maxima recovers its rays so that minima occurs at C?
position. Find the value of µ3 .
Sol: For the maxima, total path difference is to be
(d) Find the ratio of intensities at O in the three consider here, as some path difference already exist in
conditions (a), (b) and (c). the two beams before coming to the slits.

Sol: wavelength changes inside the liquid, (a) The optical path difference between the beams

λ1D λD
arriving at P, ∆x = (l2 − l1 ) + dsin θ The condition for
(a) Fringe width=
w = maximum intensity is,
d µ1d
∆x = nλ n = 0 ± 1, ±2,...
1 7 . 3 2 | Wave Optics

1 1 Hence, number of maxima =60-20=40


Thus, sin θ =  ∆x − (l2 − l1 )  = nλ − (l2 − l1 ) 
d   d 
(c) At C, phase difference,
1 n × 500 × 10−9 − 20 × 10−6 
=
10 × 10 −6  
 2π 
 λ 

φ=   (l2 − l1 )= 

−9
 500 × 10 

( −6
)
 20 × 10 = 80π

n    n 
= 2 Hence, θ sin−1 2 
− 1  ;= − 1  Hence, maximum intensity will appear at C. For
 40    40  λ
minimum intensity at C, ( µ − 1 ) t =
2
(b) sin θ ≤ 1
λ 500 × 10−9
Or t
= = = 500mm
2 (µ − 1) 2 × 0.5
n 
∴ −1 ≤ 2  − 1  ≤ 1 Or −20 ≤ (n − 40 ) ≤ 20
 40 
Or 20 ≤ n ≤ 60

JEE Main/Boards

Exercise 1 Q.6 What is a wavefront? What is the geometrical


shape of a wave front of light emerging out of a convex
lens, when point source is placed at its focus? Using
Q.1 State the essential condition for diffraction of light Huygens’s principles show that, for a parallel beam
to occur. The light of wavelength 600mn is incident incident on a reflecting surface, the angle of reflection
normally on a slit of width 3mm. Calculate the linear is equal to the angle of incidence.
width of central maximum on a screen kept 3m away
from the slit.
Q.7 Two slits in Young’s double slit experiment are
illuminated by two different lamps emitting light of the
Q.2 (a) State the postulates of Huygens’s wave theory. same wavelength. Will you observe the interference
(b) Draw the type of wave front that correspond to a pattern? Justify your answer.
beam of light (i) coming from a very far-off source and
(ii) diverging from a point source. Find the ratio of intensities at two points on a screen in
Young’s double slit experiment, when waves from the
two slits have path difference of (i) 0 (ii) λ 4
Q.3 In a single slit diffraction pattern, how does the
angular width of central maximum change, when (i)
slit width is decreased (ii) distance between the slit Q.8 Two narrow slits are illumination by a single
and screen in increased and (iii) light of smaller visible monochromatic source. Name the pattern obtained on
wavelength is used? Justify your answer in each case. the screen. One of the slits is now completely covered.
What is the name of the pattern now obtained on the
screen? Draw intensity pattern obtained in the two
Q.4 Derive Snell’s law of refraction using Huygens’s cases. Also write two difference between the patterns
wave theory. obtained in the above two cases.

Q.5 Explain with reason, how the resolving power of a Q.9 Using Huygens’s Principle, draw a diagram to
compound microscope will change when (i) frequency show propagation of a wave-front originating from
of the incident light on the objective lens in increased, a monochromatic point source. Describe diffraction
(ii) focal length of the objective lens is increased, and of light due to a single slit. Explain formation of a
(iii) aperture of the objective lens is increased. pattern of fringes obtained on the screen and plot
showing variation of intensity with angle θ in single slit
diffraction.
P hysi cs | 17.33

Q.10 What are coherent sources of light? State two Q.17 When light travels from a rarer to a denser
conditions for two light sources to be coherent. Derive medium, the speed decreases. Does this decrease in
a mathematical expression for the width of interference speed imply a decrease in the energy carried by the
fringes obtained in Young’s double slit. light wave? Justify your answer.

Q.11 Define resolving power of a compound Q.18 In Young’s double slit experiment, the two slits
microscope. How does the resolving power of a 0.12 mm apart are illuminated by monochromatic light
compound microscope change when of wavelength 420 nm. The screen is 1.0 m away from
the slits.
(i) refractive index of the medium between the object
and objective lens increases? (a) Find the distance of the second (i) bright fringes, (ii)
dark fringes from the central maximum.
(ii) wavelength of the radiation used in increased?
(b) How will the fringes pattern change if the screen is
Q.12 State one feature by which the phenomenon moved away from the slits?
of interference can be distinguished from that of
diffraction. A parallel beam of light of wavelength Q.19 How does an unpolarised light get polarized
600nm is incident normally on a slit of width ‘a’. If the when passes through a polaroid?
distance between the slits and the screen is 0.8 m and
Two polaroids are set in crossed position. A third
the distance of 2nd order minimum from the centre of
Polaroid is placed between the two making an angle
the screen is 15 mm. Calculate the width of the slit.
θ with the pass axis of the first Polaroid. Write the
expression for the intensity of light transmitted from
Q.13 How would the angular separation of interference the second Polaroid. In what orientations will the
fringes in Young’s double slit experiment change when transmitted intensity be (i) minimum and (ii) maximum?
the distance between the slits and screen is doubled?
Q.20 How does the angular separation between fringes
Q.14 Define the term ‘linearly polarized light’. When in single-slit diffraction experiment change when the
does the intensity of transmitted light become distance of separation between the slit and screen is
maximum, when a polaroid sheet is rotated between doubled?
two crossed polaroids?
Q.21 For the same value of angle of incidence, the
Q.15 In Young’s double slit experiment, monochromatic angle of refraction in three media A, B and C are 15° ,
light of wave length 630nm illuminates the pair of slits 25° and 35° respectively. In which medium would the
and produce an interference pattern in which two velocity of light be minimum?
consecutive bright fringes are separated by 8.1mm.
Another source of monochromatic light produces the
Q.22 (a) In Young’s double slit experiment, derive
interference pattern in which the two consecutive bright
the condition for (i) constructive interference and (ii)
fringes are separated by 7.2 mm. Find the wavelength
destructive interference at a point in the screen.
of light from the second source. What is the effect on
the interference fringes if the monochromatic source is (b) A beam of light consisting of two wavelengths,
replaces by a source of white light? 800nm and 600nm is used to obtain in the interference
fringes in a Young’s double slit experiment on a screen
placed 1.4 m away. If the two slits are separated by
Q.16 (a) In a single slit diffraction experiment, a slit
0.28 mm, calculate the least distance from the central
of width ‘d’ is illuminated by red light of wavelength
bright maximum where the bright fringes of the two
650nm. For what value of ‘d’ will
wavelengths coincide.
(i) the first minimum fall at an angle of diffraction of
30° , and
Q.23 (a) How does an unpolarized light incident on
(ii) the first maximum fall at an angle of diffraction of light on polaroid get polarized?
30° ?
Describe briefly, with the help of a necessary diagram,
(b) Why does the intensity of the secondary maximum the polarization of light by reflecting from a transparent
becomes less as compared to the central maximum? medium.
1 7 . 3 4 | Wave Optics

(b) Two polaroids ‘A’ and ‘B’ are kept in crossed position. Q.4 In Young’s double slit experiment, the wavelength
How should a third polaroid ‘C’ be placed between them of red light is 7800Å and that of blue is 5200 Å. The
so that the intensity of polarized light transmitted by value of n for which nth bright band due to red light
polaroid B reduce to 1/8th of the intensity of unpolarized
coincides with (n + 1 ) bright band due to blue light is:
th
light incident on A?
(A) 1 (B) 2 (C) 3 (D) 4
Q.24 Two sources of intensity I1 and I2 undergo
interference in Young’s double slit experiment. Show Q.5 Two identical narrow slits S1 and S2 are illuminated
2 by light of wavelength λ from a point source P. If, as
Imax  a + a2 
that =  1  shown in the diagram above, the light is then allowed
Imin  a1 − a2  to fall on a screen, and if n is a positive integer, the
Where a1 and a2 are the amplitudes of disturbance for condition for destructive interference at Q is
two sources S1 and S2 .
l1 S1
Q.25 Two coherent waves of equal amplitude produce l3
P
interference pattern in Young’s double slit experiment.
l2
What is the ratio of intensity at a point where phase S2 l4 Q
different is π 2 to intensity at centre.
(A) (l1 − l2 ) = ( 2n + 1) λ 2
(B) (l3 − l4 ) = ( 2n + 1 ) λ 2
Exercise 2
(C) (l1 + l2 ) − (l3 + l4 ) =nλ
Single Correct Choice Type
(D) (l1 + l3 ) − (l2 + l4 ) = ( 2n + 1 ) λ 2

Q.1 Two coherent monochromatic light beams of


intensities I and 4I are superposed. The maximum and Q.6 In a young’s double slit experiment, a small detector
minimum possible intensities in the resulting beam are: measures an intensity of illumination of I units at the
centre of the fringe pattern. If one of the two (identical)
(A) 5I and I (B) 5I and 3I slits is now covered, the measured intensity will be
(C) 9I and I (D) 9I and 3I (A) 2I (B) I (C) I/4 (D) I/2

Q.2 When light is refracted into a denser medium, Q.7 In a Young’s double slit experiment D equals the
(A) Its wavelength and frequency both increase distance of screen and d is the separation between the
slit. The distance of the nearest point to the central
(B) Its wavelength increase but frequency remains maximum where the intensity is same as that due to a
unchanged single slit, is equal to
(C) Its wavelength decreases but frequency remain Dλ Dλ
unchanged (A) (B)
d 2d
(D) It wavelength and frequency both decrease. Dλ 2Dλ
(C) (D)
3d d
Q.3 In YDSE how many maxima can be obtained on
the screen if wavelength of light used is 200nm and Q.8 A plane wavefront AB is incident on a concave
d=700nm: mirror as shown.
(A) 12 (B) 7
A
(C) 18 (D) None of these

B
Then, the wavefront just after reflection is
P hysi cs | 17.35

   Q.12 Light of wavelength λ in air enters a medium of


(A) (B) (C) refractive index µ . Two points in this medium, lying
along the path of this light, are at a distance x apart.
(D) None of the above The phase difference between these points is:
2πµx 2πx
(A) (B)
Q.9 In a Young’s double slit experiment, first maxima λ µλ
is observed at a fixed point P on the screen. Now the 2π ( µ − 1 ) x 2πx
screen is continuously moved away from the plane of (C) (D)
slits. The ratio of intensity at point P to the intensity at
λ ( − 1) λ
µ
point O (center of the screen)
Q.13 In YDSE, the source placed symmetrically with
respect to the slit is now moved parallel to the plane of
the slits so that it is closer to the upper slit, as shown.
Then,
P
O S
S1

S2

(A) Remains constant (A) The fringe width will remain and fringe pattern will
shift down
(B) Keeps on decreasing
(B) The fringe width will remain same but fringe pattern
(C) First decrease and then increases will shift up
(D) First decreases and then becomes constant (C) The fringe width will decrease and fringe pattern will
shift down
Q.10 In the figure shown if a parallel beam of white light (D) The fringe width will remain same but fringe pattern
is incident on the plane of the slits then the distance of will shift down
the white spot on the screen from O is [Assumed d<<D
λ <<d]
Q.14 In a YDSE with two identical slits, when the upper
slit is covered with a thin, perfectly transparent sheet
d 2d/3 of mica, the intensity at the centre of screen recs to
O 75% of the initial value. Second minima is observed to
the above this point and third maxima below it. Which
of the following can not be a possible value of phase
D
difference caused by the mica sheet.
(A) 0 (B) d/2 π 13π
(A) (B)
(C) d/3 (D) d/6 3 3
17π 11π
(C) (D)
Q.11 In Young’s double slit arrangement, water is filled 3 3
in the space between screen and slits. Then:
(A) Fringe pattern shifts upwards but fringes width Q.15 Two monochromatic and coherent point sources
remain unchanged. of light are placed at a certain distance from each
other in the horizontal plane. The locus of all those
(B) Fringe width decreases and central bright fringe points in the horizontal plane which have constructive
shift upwards. interference will be:
(C) Fringe width increases and central bright fringe (A) A hyperbola
does not shift.
(B) Family of hyperbolas
(D) Fringe width decreases and central bright fringe
does not shift. (C) Family of straight lines
(D) Family of parabolas
1 7 . 3 6 | Wave Optics

Q.16 A circular planar wire loop is dipped in a soap Q.19 In a Young’s double slit experiment, green light
solution and after taking it out, held with its plane is incident on the two slits. The interference pattern is
vertical in air. Assuming thickness of film at the top observed on a screen. Which of the following changes
to be very small, as sunlight falls on the soap film, & would cause the observed fringes to be more closely
observer receive reflected light spaced?
(A) The top portion appears dark while the first colour
Fringes
to be observed as one moves down is red
(B) The top portion appears violet while the first colour
to be observed as one moves down is indigo
(C) The top portion appears dark while the first colour
to be observed as one moves down is violet
(D) The top portion appears dark while the first colour
to be observed as one moves down is depends on the Incoming
refractive index of the soap solution. light waves

Q.17 A thin film of thickness t and index of refraction (A) Reducing the separation between the slits
1.33 coats a glass with index of refraction 1.50. What is (B) Using blue light instead of green light
the least thickness t that will strongly reflect light with
(C) Used red light instead of green light
wavelength 600nm incident normally?
(D) Moving the light source further away from the slits.
(A) 225nm (B) 300nm
(C) 400nm (D) 450nm
Q.20 In the previous question, films of thickness t A
and tB and refractive indices µ A and µB , are placed in
Q.18 Spherical wave fronts shown in figure, strike a front of A and B respectively. If µ A t A =
µB tB , the central
plane mirror. Reflected wavefronts will be as shown in maximum will:
(A) Not shift
(B) Shift towards A
(C) Shift towards B
(D) Option (B), if tB > t A ; option (C) if tB < t A

Q.21 To make the central fringe at the centre O, a mica


sheet of refractive index 1.5 is introduced. Choose the
correct statement(s).

d S1
S
d O
(A) (B)
S2 D>>d

(A) The thickness of sheet is 2 ( )


2 − 1 d in front of S1 .

(B) The thickness of sheet is ( )


2 − 1 d in front of S2 .

(C)
(D) (C) The thickness of sheet is 2 2 in front of S1

( )
(D) The thickness of sheet is 2 2 − 1 d in front of S1 .
P hysi cs | 17.37

Previous Years’ Questions λ  λ 


(A) sin−1   (B) sin−1  
d
   2d 
Q.1 In Young’s double slit experiment, the separation
between the slits is halved and the distance between  λ   λ 
(C) sin−1   (D) sin−1  
the slits and the screen is doubled. The fringe width is  3d   4d 
 (1981)
Q.7 A narrow monochromatic beam of light intensity I
(A) Unchanged (B) Halved
is incident on a glass plate as shown in figure. Another
(C) Doubled (D) Quadrupled identical glass plate is kept close to the first one-and
parallel to it. Each glass plate reflects 25 per cent of
Q.2 Two coherent monochromatic light beams of the light incident on it transmits the remaining. Find
intensities I and 4I are superimposed. The maximum the ratio of the minimum and maximum intensities
and minimum possible intensities in the resulting beam in the interference pattern formed by the two beams
are (1988) obtained after one reflection at each plate.  (1990)

(A) 5I and I (B) 5I and 3I


(C) 9I and I (D) 9I and 3I

Q.3 In a double slit experiment instead of taking slits l


of equal widths, one slit is made twice as wide as the
other, then in the interference pattern. (2000) 1 2
(A) The intensities of both the maxima and the minima
increase Q.8 Angular width of central maximum in the
Fraunhofer diffraction pattern of a slit is measured. The
(B) The intensity of the maxima increases and the slit is illuminated by light of wavelength 6000 Å . When
minima has zero the slit is illuminated by light of another wavelength,
(C) The intensity of the maxima decreases and that the angular width decreases by 30%. Calculate the
minima increases. wavelength of this light. The same decrease in the
angular width of central maximum is obtained when
(D) The intensity of the maxima decreases and the
the original apparatus is immersed in a liquid. Find
minima has zero.
refractive index of the liquid.  (1996)

Q.4 Two beams of light having intensities I and 4I


Q.9 A double slit apparatus is immersed in a liquid of
interfere to produce a fringe pattern on a screen. The
refractive index 1.33. it has slit separation of 1mm and
phase difference between the beams is π 2 at point A
distance between the plane of slits and screen is 1.33m.
and π at point B. Then the difference between resultant
The slits are illuminated by a parallel beam of light
intensities at A and B is (2001)
whose wavelength in air is 6300 Å . (1996)
(A) 2I (B) 4I
(a) Calculate the fringes width.
(C) 5I (D) 7I
(b) One of the slits of the apparatus is covered by a thin
glass sheet of refractive index 1.53. Find the smallest
Q.5 In a YDSE bi-chromatic light of wavelengths 400nm thickness of the sheet to bring the adjacent minimum
and 560nm is used. The distance between the slits is as the axis.
0.1mm and the distance between the plane of the slits
and the screen is 1 m. The minimum distance between
Q.10 In a Young’s double slit experiment, two
two successive regions of complete darkness is (2004)
wavelengths of 500nm and 700nm were used. What
(A) 4mm (B) 5.6mm is the minimum distance from the central maximum
where their maximas coincide again? Take D d = 103 .
(C) 14mm (D) 28mm
Symbols have their usual meanings. (2004)

Q.6 In Young’s double slit experiment intensity at a point


is (1/4) of the maximum intensity. Angular position of
this point is (2005)
1 7 . 3 8 | Wave Optics

Q.11 A beam of unpolarised light of intensity I0 is a rotation of Polaroid through 30° makes the two
passed through a polaroid A and then through another beams appear equally bright. If the initial intensities of
polaroid B which is oriented so that its principal plane the two beams are I A and IB respectively, then I A / IB
makes an angle of 45° relative to that of A. The intensity equals:  (2014)
of the emergent light is: (2013)
(A) 1 (B) 1/3
(A) I0 / 2 (B) I0 / 4
(C) 3 (D) 3/2
(C) I0 / 8 (D) I0
Q.14 The box of pin hole camera, of length L, has a
Q.12 Two coherent point sources S1 and S2 are hole of radius a. It is assumed that when the hole is
separated by a small distance ‘d’ as shown. The fringes illuminated by a parallel beam of light of wavelength λ
obtained on the screen will be: (2013) the spread of the spot (obtained on the opposite wall
of the camera) is the sum of its geometrical spread and
d
the spread due to diffraction. The spot would then have
S1 S2 Screen its minimum size ( say bmin ) when:  (2016)

D  2 λ2 
(A) a= λ L and bmin =  
 L 
(A) Straight lines (B) Semi-circles  
(B) a= λ L and bmin
= 4 λL
(C) Concentric circles (D) Points
λ2
(C) a = and bmin
= 4 λL
Q.13 Two beams, A and B, of plane polarized light with L
mutually perpendicular planes of polarization are seen λ2  2 λ2 
through a polaroid. From the position when the beam A (D) a = and bmin =  
L  L 
has maximum intensity (and beam B has zero intensity),  

JEE Advanced/Boards

Exercise 1 Q.3 In young’s double slit experiment the slits are 0.5
mm apart and the interference is observed on a screen
at a distance of 100cm from the slit. It is found that
Q.1 Two coherent waves are described by the the 9th bright fringe is at a distance of 7.5mm from the
expressions. second dark fringe from the centre of the fringe pattern
 2πx1 π on same side. Find the wavelength of the light used.
=E1 E0 sin  − 2πft + 
 λ 6 
Q.4 Light of wavelength 520nm passing through a
 2πx2 π double slit, produce interference pattern of relative
=E1 E0 sin  − 2πft + 
 λ 8 intensity versus deflection angle θ as shown in the
figure. Find the separation d between the slits.
Determine the relationship between x1 and x2 that
produces constructive interference when the two waves
Relative intensity

are superposed.

Q.2 In a Young’s double slit experiment for interference


of light, the slits are 0.2 cm apart and are illuminated
by yellow light ( λ =600nm) . What would be the fringe
width on a screen placed 1m from the plane of slits if
O
the whole system is immersed in water of index 4/3? 0.75 1 2 3  (degree)
P hysi cs | 17.39

Q.5 In a YDSE apparatus, d=1mm, λ =600nm and Considering λ as 4mm, calculate the position of
D=1m. The slits individually produce same intensity maxima and draw shape of interference pattern. Take
on the screen. Find the minimum distance between initial phase difference between the two sources to be
two points on the screen having 75% intensity of the zero.
maximum intensity.
Q.11 Two radio antennas radiating wave in phase
Q.6 The distance between two slits is a YDSE apparatus are located at points A and B , 200m apart (Figure).
is 3mm. The distance of the screen from the slits is 1 The radio waves have a frequency of 5.80MHz. A
m. Microwaves of wavelength 1mm are incident on the radio receiver is moved out from point B along a line
plane of the slits normally. Find the distance of the first perpendicular to the line connecting A and B (line BC
maxima on the screen from the central maxima. Also shown in figure). At what distance from B will there be
find the total number of maxima on the screen. destructive interference?

A
Q.7 One slit of a double slit experiment is covered by
a thin glass plate of refractive index 1.4 and the other
by a thin glass plate of refractive index 1.7. The point 200m
on the screen, where central bright fringe was formed
before the introduction of the glass sheets, is now
occupied by the 5th bright fringe. Assuming that both B
C
the glass plates have same thickness and wavelength of
light used is 4800Å, find their thickness.
Q.12 A ray of light of intensity I is incident on a parallel
Q.8 A monochromatic light of λ =5000Å is incident glass-slab at a point A as shown in figure. It undergoes
on two slits separated by a distance of 5 × 10−4 m. partial reflection and refraction. At each reflection 20%
The interference pattern is seen on a screen placed of incident energy is reflected. The rays AB and A′ B′
at a distance of 1m from the slits. A thin glass plate undergo interference. Find the ratio Imax Imin .
of thickness 1.5 × 10−6 m & refractive index µ =1.5
is placed between one of the slits & the screen. Find B
B’
the intensity at the centre of the screen, if the intensity
there is I0 in the absence of the plate. Also find the A A’
internal shift of the central maximum.

Q.9 One radio transmitter A operating at 60.0 MHz is


10.0 m from another similar transmitter B that is 180°
out of the phase with transmitter. A. How far must an [Neglect the absorption of light]
observe move from transmitter A toward transmitter B
along the line connecting A and B to reach the nearest
point where the two beams are in phase? Q.13 If the slits of the double slit were moved
symmetrically apart with relative velocity v, calculate
the number of fringes passing per unit time at a
Q.10 Two microwaves coherent point sources emitting distance x from the centre of the fringes system formed
waves of wavelength λ are placed at 5λ distance apart. on a screen y distance away from the double slits if
The interference is being observed on a flat non- wavelength of light is λ . Assume y>>d & d>> λ .
reflecting surface along a line passing through one
source, in a direction perpendicular to the line joining
the two sources (refer figure). Q.14 A thin glass plate of thickness t and refractive
index µ is inserted between screen & one of the slits
S1 Observation in a Young’s experiment. If the intensity at the centre of
the screen is I, what was the intensity at the same point
surface
S prior to the introduction of the sheet?

S2 Q.15 In Young’s experiment, the source is red light


of wavelength 7 × 10−7 m. When a thin glass plate of
1 7 . 4 0 | Wave Optics

refractive index 1.5 at this wavelength is put in the path while that from L1 and L2 to O is 1.30m. The screen at
of one of the interfering beams, the central bright fringe O is normal SO.
shifts by 10−3 m to the position previously occupied by
the 5th bright fringe. Find the thickness of the plate. (i) If the third intensity maximum occurs at the point A
When the source is now changed to green light of on the screen, find the distance OA.
wavelength 5 × 10−7 m, the central fringe shift to a
position initially occupied by the 6th bright fringe due
to red light without the plate. Find the refractive index A
of glass for the green light. Also estimate the change in S
fringe width due to the change in wavelength. L1
S 0.5mm
Q.16 In a Young’s experiment, the upper slit is covered O
by a thin glass plate of refractive index 1.4 while the
lower slit is covered by another glass plate having the L2
Screen
same thickness as the first one but having refractive
index 1.7. Interference pattern is observed using light
of wavelength 5400 Å . It is found that the point P on
the screen where the central maximum (n=0) fell before 0.15m 1.30m
the glass plates were inserted now has ¾ the original (ii) If the gap between L1 & L2 is reduced from its
intensity. It is further observed that what used to be the original value of 0.5 mm, will the distance OA increase,
5th maximum earlier, lies below (Absorption of light by decrease or remain the same?
glass plate may be neglected).
Q.19 A coherent parallel beam of microwave of
Q.17 A screen is at a distance D=80cm from a wavelength λ =0.5 mm falls on a Young’s double slit
diaphragm having two narrow slits S1 is and S2 which apparatus. The separation between the slits is 1.0 mm.
are d=2 mm apart. Slit S1 is covered by a transparent The intensity of microwaves in measured on screen
sheet of thickness t1 =2.5 µm and S2 by another sheet placed parallel to the plane of the slits at a distance of
of thickness t2 =1.25 µm as shown in figure. Both 1.0m from it, as shown in the figure.
sheets are made of same material having refractive
index µ =1.40. Water is filled in space between (a) If the incident beam falls normally on the double slit
diaphragm and screen. A monochromatic light beam apparatus, find the y-coordinates of all the interference
of wavelength λ =5000 Å is incident normally on the minima on the screen.
diaphragm. Assuming intensity of beam to be uniform y
and slits of equal width, calculate ratio of intensity at C
to maximum intensity of interference pattern obtained
on the screen, where C is foot of perpendicular bisector
of S1 S2 . (Refractive index of water, µ w =4/3) 30
o
d-1.0m
x
t1
S1 D-1.0m

C
S2
(b) if the incident beam makes an angle of 30° with the
t2 x-axis (as in the dotted arrow shown in the figure), find
the y-coordinates of the first minima on either side of
Q.18 In the figure shown S is a monochromatic point the central maximum.
source emitting light of wavelength=500nm. A thin
lens of circular shape and focal length 0.10 m is cut into Q.20 In a YDSE with visible monochromatic light two
identical halves L1 and L2 by a plane passing through thin transparent sheets are used in front of the slits
a diameter. The two halves are placed symmetrically S1 and S2 with µ1 =1.6 and µ2 =1.4 respectively. If
about the central axis SO with a gap of 0.5 mm. The both sheets have thickness t, the central maximum is
distance along the axis from the S to L1 and L2 is 0.15m, observed at a distance of 5mm from centre O. Now
P hysi cs | 17.41

the sheets are replaced by two sheets of same material Q.3 In Young’s double slit experiment, the two slits act as
µ + µ2 coherent sources of equal amplitude A and wavelength
refractive index 1 but having thickness t1 & t2 λ . In another experiment with the same setup the two
2
t +t slits are sources of equal amplitude A and wavelength
such that t = 1 2 . Now central maximum is observed
2 λ but are incoherent. The ratio of the average intensity
at distance of 8mm from centre O on the same side as of light at the midpoint of the screen in the first case to
before. Find the thickness t1 (in µm ) [Given: d=1mm. that in the second case is
D=1m].
(A) 1:1 (B) 2:1
(1,t,t1)
P (C) 4:1 (D) None of these
S1

d Q.4 Two point monochromatic and coherent sources


O of light wavelength λ are placed on the dotted line in
S2 front of a large screen. The source emit waves in phase
(2,t,t2) with each other. The distance between S1 and S2 is ‘d’
D while their distance from the screen is much larger.
Then,
(1) If d= 7 λ 2 , O will be a minima

Exercise 2 (2) If d= 4.3 λ , there will be a total of 8 minima on y axis.


(3) If d= 7λ , O will be a maxima.
Single Correct Choice Type
y
Q.1 Figure shows plane waves refracted from air to
water using Huygens’s principle a, b, c, d, e are lengths
on the diagram. The refractive index of water w.r.t. air
is the ratio: S1 S2 O

(4) If d= λ , there will be only one maxima on the screen.


Which is the set of correct statement
(A) 1, 2, & 3 (B) 2, 3 & 4
(C) 1, 2, 3 & 4 (D) 1, 3 & 4
a b
Air C Q.5 Two slits are separated by 0.3 mm. A beam of 500nm
d c light strikes the slits producing an interference pattern.
Water The number of maxima observed in the angular range-
0° < θ < 30° .
(A) a/e (B) b/e (C) b/d (D) d/b

Q.2 In a YDSE, the central bright fringe can be identified:
(A) As it has greater intensity than the other bright
(A) 300 (B) 150 (C) 599 (D) 601
fringes.
(B) As it is wider than the other bright fringes.
Q.6 In the above question of the light incident is
(C) As it is narrower than the other bright fringes. monochromatic and point O is a maxima, then the
wavelength of the light incident cannot be
(D) By using white light instead of single wavelength
light. d2 d2 d2 d2
(A) (B) (C) (D)
3D 6D 12D 18D
1 7 . 4 2 | Wave Optics

Q.7 In the figure shown in YDSE, a parallel beam of


light is incident on the slit from a medium of refractive
index n1 . The wavelength of light in this medium is λ1 . t, 2
A transparent slab of thickness ‘t’ and refractive index y
d
n3 is put infront of one slit. The medium between
the screen and the plane of the slits is n2 . The phase , t2
difference between the light waves reaching point ‘O’
D
(symmetrical, relative to the slits) is:
tD
n1 (A) Zero (B)
n3 d
n2
tD
(C) − (D) None
O d

1 Multiple Correct Choice Type

2π Q.10 In a YDSE apparatus, if we use white light then:


(A) (n − n2 ) t (B) 2λπ (n3 − n2 ) t
n1 λ1 3 1 (A) The fringe next to the central will be red

2πn1  n3  2πn1 (B) The central fringe will be white


(C)  − 1  t (D)
n2 λ1  n2 λ1
(n3 − n1 ) t (C) The fringe next to the central will be violet

(D) There will not be a completely dark fringe.
Q.8 In A YDSE experiment if a slab whose refractive
index can be varied is placed in front of one of the slits Q.11 If one of the slits of a standard YDSE apparatus
then the variation of resultant intensity at mid-point is covered by a thin parallel sided glass slab so that it
of screen with ' µ ' will be best represented by ( µ ≥ 1 ) . transmit only one half of the light intensity of the other,
[Assumes slits of equal width and there is no absorption then:
by slab]
(A) The fringe pattern will get shifted towards the
covered slit

I0 I0 I0 (B) The fringe


I0 pattern will get shifted away from the
covered slit
(A) (B) (C) (C) The(D)
bright fringes will be less bright and the dark
ones will be more bright
  (D) The fringe width will
 remain unchanged.
=1 =1 =1 =1

Q.12 In an interference arrangement similar to Young’s


I0 I0 double-slit experiment, the slits S1 & S2 are illuminated
with coherent microwave sources, each of frequency
(C) (D) 106 Hz. The sources are synchronized to have zero
phase difference. The slits are separated by a distance
   d=150.0m. The intensity I( θ ) is measured as a function
=1 =1 of θ at a large distance from S1 & S2 , where θ is defined
as shown if I0 is the maximum intensity then
Q.9 In the YDSE shown the two slits are covered with
thin sheets having thickness t & 2t and refractive index
2 µ and µ . Find the position (y) of central maxima S1


d

S2
P hysi cs | 17.43

I( θ ) for 0 ≤ θ ≤ 90° is given by: (C) Statement-I is true, statement-II is false


I0 (D) Statement-I is false, statement-II is true
( θ)
(A) I=
2
for θ=30°

I0 Q.17 Statement-I: In YDSE, as shown in figure, central


( θ)
(B) I=
4
for θ=90°
bright fringe is formed at O. If a liquid is filled between
plane of slits and screen, the central bright fringe is
( θ ) I0 for θ=0°
(C) I=
shifted in upward direction.
(D) I ( θ ) is constant for all values of θ
Statement-II: If path difference at O increases
y-coordinate of central bright fringe will change.
Q.13 To observe a sustained interference pattern
formed by two light waves, it is not necessary that they S1
must have:
O
(A) The same frequency
(B) Same amplitude S2
(C) A constant phase difference
(D) The same intensity Q.18 Statement-I: In glass, red light travels faster than
blue light.
Q.14 If the source of light used in a Young’s Double Slit Statement-II: Red light has a wavelength longer than
Experiment is changed from red to blue, then blue.
(A) The fringes will become brighter
(B) Consecutive fringes will come closer Q.19 Statement-I: In standard YDSE set up with visible
light, the position on screen where phase difference is
(C) The number of maxima formed on the screen zero appears bright
increases
Statement-II: In YDSE set up magnitude of
(D) The central bright fringe will become a dark fringe. electromagnetic field at central bright fringe is not
varying with time.
Q.15 In a Young’s double-slit experiment, let A and B
be the two slits. A thin film of thickness t and refractive Comprehension Type
index µ is placed in front of A. Let β =fringe width. The
central maximum will shift: The figure shows a schematic diagram showing the
arrangement of Young’s Double Slit Experiment:
(A) towards A (B) towards B
β β
(C) by t ( µ − 1 ) (D) by µt S S1
λ λ a
d O
Q.16 In a standard YDSE apparatus a thin film S2 Screen
( µ 1.5,t − 2.1µm) is placed in front of upper slit. How
= 
far above or below the centre point of the screen are D
two nearest maxima located? Take D=1m, d=1 mm, Q.20 Choose the correct statement(s) related to the
λ =4500Å .(Symbols have usual meaning) wavelength of light used
(A) 1.5mm (B) 0.6mm (A) Larger the wavelength of light larger the fringe
(C) 0.15mm (D) 0.3mm width
(B) The position of central maxima depends on the
Assertion Reasoning Type wavelength of light used
(A) Statement-I is true, statement-II is true and (C) If white light is used in YDSE, then the violet forms
statement-II is correct explanation for statement-I its first maxima closest to the central maxima
(B) Statement-I is true, statement-II is true and (D) The central maxima of all the wavelength coincide
statement-II is NOT the correct explanation for statement-I
1 7 . 4 4 | Wave Optics

Q.21 If the distance D is varied, then choose the correct maximum occurred previously remain unchanged. The
statement(s) minimum thickness of the glass-plate is  (2002)
(A) The angular fringe width does not change 2λ
(A) 2λ (B)
(B) The fringe width change in direct proportion 3
λ
(C) The change in fringe width is same for all wavelengths (C) (D) λ
3
(D) The position of central maxima remains unhanged
Q.5 The phases of the light wave at c, d, e and f are
Q.22 If the distance d is varied, then identify the correct φc , φd , φe and φf respectively. It is given that φc ≠ φf 
statement  (2007)

(A) The angular width does not change (A) φc cannot be equal to φd
(B) The fringe width changes in inverse proportion (B) φd cannot be equal to φe
(C) The positions of all maxima change (C) ( φd − φf ) is equal to ( φc − φe )
(D) The positions of all minima change (D) ( φd − φc ) is not equal to ( φf − φe )

Q.6 Shows four situations of standard Young’s


Previous Years’ Questions doubles slit arrangement with the screen placed
away from the slits S1 and S2 . In each of these cases
Q.1 A narrow slit of width 1mm is illuminated by λ λ
monochromatic light of wavelength 600nm. The S1=P0 S2P0 ,S1P1 − S=
2P1 and S1P2 − S2P2 = , where
4 3
distance between the first minima on either side of a
λ is the wavelength of the light used. In the cases B,
screen at a distance of 2 m is.  (1994)
C and D, a transparent sheet of refractive index µ
(A) 1.2 cm (B) 1.2 mm and thickness t is pasted on slit S2 . The thickness of
the sheets are different in different cases. The phase
(C) 2.4 cm (D) 2.4 mm
difference between the light waves reaching a point P
on the screen from the two slits is donated by δ (P )
Q.2 A parallel monochromatic beam of light is incident and the intensity by I (P). Match each situation given in
normally on a narrow slit. A diffraction pattern is formed Column I with the statement(s) in Column II valid that
on a screen placed perpendicular to the direction of the situation. (2009)
incident beam. At the first minimum of the diffraction
pattern, the phase difference between the rays coming
from the two edges of the slit is  (1998) Column I Column II

(A) Zero (B) π 2 S2 P2


(A)
P1
P0 (p) δ(P0 ) =
0
(C) π (D) 2 π S1

Q.3 In a Young’s double slit experiment, 12 fringes are S2 P2


(q) δ(P1 ) =
0
λ
(B) ( µ − 1 ) t =
P1
observed to be formed in a certain segment of the P0
4 S1
screen when light of wavelength 600nm is used. If the
wavelength of light is changed to 400nm, number of
fringes observed in the same segment of the screen is S (r) I(P1 ) = 0
given by  (2001) λ 2 P2
(
(C) µ − 1 t = )
2 S1
P1
P0

(A) 12 (B) 18
(C) 24 (D) 30

S2 P2 (s) I(P0 ) > I(P1 )

Q.4 In the ideal double-slit experiment, when a glass-


(
(D) µ − 1 t = ) 4
P1
P0
S1
plate (refractive index 1.5) of thickness t is introduced
in the path of one of the interfering beams (wavelength
λ ), the intensity at the position where the central (t) I(P2 ) > I(P1 )
P hysi cs | 17.45

Q.7 In the Young’s double slit experiment, the in one plane. The radiant powers of sources A and B
interference pattern is found to have an intensity ratio are 90 W and 180 W respectively. The beam from C is
between the bright and dark fringes as 9. This implies of intensity 20 W m2 . Calculate intensity at P on the
that  (1982) screen.
C
(A) The intensities at the screen due to the two slits are
5 units and 4 units respectively.
o
60
(B) The intensities at the screen due to the two slits are A P
4 units and 1 unit respectively 60
o

(C) The amplitude ratio is 3


(D) The amplitude ratio is 2 S
B

Q.8 White light is used to illuminate the two slits in a


Young’s double slit experiment. The separation between Q.11 In a modified Young’s double slit experiment, a
the slits is b and the screen is at a distance d (>>b) from monochromatic uniform and parallel beam of light of
the slits. At a point on the screen directly in front of one wavelength 6000 Å and intensity (10 π ) Wm2 is incident
of the slits, certain wavelengths are missing. Some of normally on two apertures A and B of radii 0.001 m
these missing wavelengths are and 0.002 m respectively. A perfectly transparent film
of thickness 2000 Å and refractive index 1.5 for the
(A) λ =b2 d (B) λ =2b2 d wavelength of 6000 Å is placed in front of aperture A
(C) λ =b2 3d (D) λ =2b2 3d (see figure). Calculate the power (in W) received at the
focal spot F of the lens. The lens is symmetrically placed
with respect to the apertures. Assume that 10% of the
Q.9 In an interference arrangement similar to Young’s
power received by each aperture goes in the original
double-slit experiment, the slits S1 and S2 are
direction and is brought to the focal spot. (1989)
illuminated with coherent microwave sources, each
of frequency 106 Hz. The sources are synchronized to
have zero phase difference. The slits are separated by a
A
distance d=150.0m. The intensity I ( θ ) is measured as F
a function of θ where θ is defined as shown. If I0 is the
maximum intensity then I B

S1
d/2
Q.12 In Young’s experiment, the source is red light
 of wavelength 7 × 10−7 m. When a thin glass plate
of refractive index 1.5 at this wavelength is put in
d/2 the path of one of the interfering beams, the central
bright fringe shift by 10−3 m to the position previously
S2 occupied by the 5th bright fringe. Find the thickness of
the plate. When the sources is now changed to green
light of wavelength 5 × 10−7 m, the central fringe shifts
( θ ) for 0 ≤ θ ≤ 90° is given by (1995)
to a position initially occupied by the 6th bright fringe
due to red light. Find the refractive index of glass for
(A) I ( θ )= I0 2 for θ= 30°
green light. Also estimate the change in fringe width
(B) I ( θ )= I0 4 for θ= 90° due to change in wavelength.  (1997)
(C) I ( θ )= I0 for θ = 0°
Q.13 In a Young’s experiment, the upper slit is covered by
(D) I ( θ )= is constant for all values of θ
a thin glass plate of refractive index 1.4 while the lower
slit is covered by another glass plate, having the same
Q.10 Screen S is illuminated by two point sources A thickness as the first one but having refractive index 1.7.
and B. Another source C sends a parallel beam of light Interference pattern is observed using light of wavelength
towards point P on the screen (see figure). Line AP is 5400 Å . It is found that the point P on the screen, where
normal to the screen and the lines AP, BP and CP are
1 7 . 4 6 | Wave Optics

the central maximum (n=0) fall before the glass plates (C) From the central maximum, 3rd maximum of λ2
3 overlaps with 5th minimum of λ1
were inserted, now has the original intensity. It is
4 (D) The angular separation of fringes for λ1 is greater
further observed that what used to be the fifth maximum than λ2
earlier lies below the point P while the sixth minima lies
above P. Calculate the thickness of glass plate. (Absorption
of light by glass plate may be neglected). (1997) Q.16 A Young’s double slit interference arrangement
with slits S1 and S2 is immersed in water (refractive
index = 4/3) as shown in the figure. The positions
Q.14 In the Young’s double slit experiment using of maxima on the surface of water are given by
a monochromatic light of wavelength λ , the path 2
x= p2m2 λ2 − d2 , where λ is the wavelength of light in
difference (in terms of an integer n) corresponding to air (refractive index = 1), 2d is the separation between
any point having half the peak intensity is (2013)
the slits and m is an integer. The value of p is (2015)
λ λ
(A) ( 2n + 1 ) (B) ( 2n + 1 )
2 4
λ λ
(C) ( 2n + 1 ) (D) ( 2n + 1 )
8 16
S1
Q.15 A light source, which emits two wavelengths d
λ1 =400 mm and λ2 = 600 nm, is used in a Young’s x Air
double slit experiment. If recorded fringe widths for d
λ1 and λ2 are β1 and β2 and the number of fringes S2 Water
for them within a distance y on one side of the central
maximum are m1 and m2 , respectively, then  (2014)
(A) β2 > β1
(B) m1 > m2

PlancEssential Questions
JEE Main/Boards JEE Advanced/Boards
Exercise 1 Exercise 1
Q. 12 Q.15 Q.16 Q.2 Q.3 Q.8
Q.18 Q.22 Q.23 Q.12 Q.15

Exercise 2 Exercise 2
Q.4 Q.5 Q.8 Q.4 Q.7
Q.15 Q.16

Previous Years’ Questions Previous Years’ Questions


Q.5 Q.7 Q.8 Q.3 Q.9
Q.9
P hysi cs | 17.47

Answer Key

JEE Main /Boards Q.15 560nm, when the monochromatic source is


replaced by a source of white light; the fringe width
Exercise 1 would change.
Q.16 (a) (i) 1300nm; (ii) 1950nm
Q.1 1.2 × 10−3
(b) Intensity of secondary maximum is lesser as compared
Q.3 (i) Angular width increases (ii) no change (iii)
to central maxima
angular width increases
Q.17 No, Energy carried by a wave depends on the
Q.5 (i) Resolving power increases (ii) remains unchanged
amplitude of the wave, not on the speed of wave
(iii) resolving power increases
propagation.
Q.7 No, Ratio=2:1
Q.18 (a) (i) 0.007m, (ii) 0.00525m (b) If screen is moved
I0 away from the slits fringe pattern will shrink.
Q.9 Intensity becomes
4
1
−4
Q.12 6.4 × 10 mm Q.25
2
Q.13 Fringe width becomes twice

Exercise 2

Single Correct Question


Q.1 C Q.2 C Q.3 B Q.4 B Q.5 D Q.6 C
Q.7 C Q.8 C Q.9 C Q.10 D Q.11 D Q.12 A
Q.13 D Q.14 A Q.15 B Q.16 C Q.17 A Q.18 C
Q.19 B Q.20 D Q.21 A

Previous Years’ Questions


Q.1 D Q.2 C Q.3 A Q.4 B Q.5 D Q.6 C
Q.7 1 49 Q.8 (a) 4200 Angstrom, (b) 1.4 Q.9 (a) 0.63mm, (b) 1.579 µm Q.10 3.5mm
Q.11 B Q.12 C Q.13 B Q.14 B

JEE Advanced/Boards
Exercise 1
 1 
Q.1  n −  λ= x1 − x2 Q.2 0.225mm
 48 

Q.3 5000 Å Q.4 1.99 × 10−2 mm
Q.5 0.2 mm Q.6 35.35 cm app., 5
Q.7 8 µm Q.8 0, 1.5mm

32 9
Q.9 1.25m Q.10 48, 21, , ,0 m.m
3 2
Q.11 760m, 21.8m, 89.4m, 19.6m
1 7 . 4 8 | Wave Optics

x
Q.12 81:1 Q.13 v
λy

 π (µ − 1) t  400
Q.14 I0 = I sec2   Q.15 7 µm , 1.6, µm (decrease)
 λ  7

Q.16 9.3 µm Q.17 3/4
1 3
Q.18 (i) 1 mm (ii) increase Q.19 (a) ,; (b) No shift
15 4
Q.20 33

Exercise 2

Single Correct Choice Type

Q.1 C Q.2 D Q.3 B Q.4 C Q.5 C Q.6 A


Q.7 A Q.8 C Q.9 B

Multiple Correct Choice Type

Q.10 B, C, D Q.11 A, C, D Q.12 A, C Q.13 B, D Q.14 B, C Q.15 A, C


Q.16 C, D

Assertion Reasoning Type

Q.17 D Q.18 A Q.19 C

Comprehension Type

Q.20 A, C, D Q.21 A, B, D Q.22 B, D

Previous Years’ Questions


Q.1 D Q.2 D Q.3 B Q.4 A Q.5 C

Q.6 A → p, s; B → q; C → t; D → s Q.7 B, D Q.8 A, C Q.9 A, C

Q.10 13.97 W m2 Q.11 7 × 10−6 W Q.12 (a) 7 × 10−6 m ; (b) 1.6; (c) −5.71 × 10−5 m

Q.13 9.3 µm Q.14 B Q.15 A, B, C Q.16 3

P hysi cs | 17.49

Solutions

JEE Main/Boards (ii) Doesn’t depend on D


(iii) If λ decreases, θ decreases
Exercise 1
Sol 4: According to Huygens theory each point on the
Sol 1: Size of obstacle must be comparable to leading surface of a wave disturbance may be regarded
2λD as a secondary source of spherical wave, which
wavelength of light width = themselves progress with the speed of light in the
d
medium & whose envelope at later times constitutes
2 × 6 × 10−9 × 3 × 102 the new wave front.
= = 1.2 × 10–3 m.
3 × 10−3

Sol 2: (a) Wave front is the locus of all particles of


the medium which vibrate in same phase and where
disturbances reach at the same point of time.
Consider all the point on a primary wave front to be
sources of light, which emit disturbances known as
secondary disturbances.

Tangent envelope to all secondary wavelets gives the


position of new wave front.
(b)
(i)  DG 
DM = Vtt = Vt  
 Vi 

n 
DM =  i  DG
 nt 

ni sin θt
⇒ =
nt sin θi

D
Sol 5: R. P. =
1.22 λ
(ii) (i) If f increases, λ decreases
R. P. increases
(ii) R. P. doesn’t depend on f.
(iii) If D increases, R. P increases

Sol 6: Wave front is the locus of points having the same


phase (a line or a curve, etc)

λ
Sol 3: θ ∝
d
(i) If d decrease, θ increases
1 7 . 5 0 | Wave Optics

Sol 7: If the wavelength of both the sources is same, Sol 10: Two sources are said to be coherent if their
then interference may not be possible as even phase frequencies are equal and they have a constant phase
difference must be constant difference. Two independent sources of light cannot be
coherent
I = I1 + I2 + 2 I1 I2 cos θ

(i) θ = 0; I1 = 4I0;
I1
(ii) θ = 90° = 2 ; I2 = 2I0
I2

Sol 8: With 2 slits → interference pattern

xn d
S 2P – S 1 P =
D
 δ xn 
 = 
 d D
With 1 slit → diffraction pattern xn d
If = nλ
D
we will observe maximum intensity
xn d λ
If = (2n + 1)
λ 2
we will observe minimum intensity.

In first case, the maximum intensity is constant as we Sol 11: Resolving power of an instrument is its capacity
go from centre. to resolve 2 points which are close together
In second case, the intensities at maximum decrease as (i) It doesn’t depend on μ of the medium
we go from centre.
(ii) It’s inversely proportional to λ of light.
In first case, the fringe length is fixed. In second case,
the fringe angle is fixed.
Sol 12: Difference between interference and diffraction:
Interference is due to superposition of two distinct waves
Sol 9: coming from two coherent sources. Diffraction is produced
as a result of superposition of the secondary wavelets
coming from different parts of the same wavefront.
Numerical: Here, λ = 600nm = 600 × 10−19 = 6 × 10−7 m
D
= 0.8m, = = 15 × 10−3 m
x 15mm
n 2,a
= = ?
In diffraction pattern x
 a = nλ
D
nλD 2 × 6 ×10−7 × 0.8
a
= = = 6.4 ×10−5 m
x 15 × 10 −3

Sol 13: Angular separation of interference fringes in


YDSE depends only on λ, d but not on D.
P hysi cs | 17.51

Sol 14: Linearly polarised light is light in which all sin i


the electric field of all the photons are confined to 1 µ=
sin r
direction perpendicular to direction of wave.
C0 1
I = I0cos2q C= as C ∝
µ µ
I ∝ cos2q
⇒ C ∝ sin r.
θ = 180°
Minimum for r = 15°.
λD
Sol 15: β =
d Sol 22: (a) d sin θ = nλ (for constructive)
β λ
⇒ β α λ; 1 = 1  1
β2 λ2 d sin θ =  n +  λ (for destructive)
 2
If white light is used, there will no complete darkness
as all colours will not be out of phase at a single point
centre will be brightest as all colours will be in phase at
that point.


Sol 16: First maxima → θ =
2d
λ
First minima → θ =
d
λ1D
(b) b1 =
hc d
Sol 17: E = hν = . c value decrease & λ also decreases
λ
8 × 10−7 × 1.4
maintaining the frequency constant. So E is constant. = = 4 × 10–3 m = 4 mm
−4
2.8 × 10
λD
Sol 18: β = λ2D 6 × 10−7 × 1.4
d b2 = = = 3 mm.
d 2.8 × 10−4
(a) (i) 2nd bright : y = 2b
3b1 = 4b2
β
(ii) 1st dark : y = 3rd bright of 1st light = 4th bright of 2nd light
2

(b) If D increases Sol 23: (a) The transparent medium allows components
of E only in 1 direction & reflects all its perpendicular
β decreases, so fringe width increases.
components.
I0
Sol 19: The centre reflects the components perpendicular (b) As A & B are crossed, I0 = →
to the direction 2
I0 I0
I = I0 cos2q & cos2θ =
2 8

Sol 20: In single slit diffraction angular fringe width Sol 24: Imin = I1 + I2 + 2 I1 I2 .cos φ & I = a2
depends only of λ, d but not on D.
Imax = a12 + a22 + 2a1a2 (1) = (a1 + a2)2
Sol 21:
Imin = a12 + a22 + 2a1a2 ( −1) = (a1 − a2 )2

Sol 25: I0 = 4I1


I0
I'0 = I1 + I1 + 2I1cos90° = 2I1 =
2
1 7 . 5 2 | Wave Optics

Exercise 2 Phase difference =



3
Single Correct Choice Type λD
for 2π →
d
Sol 1: (C) I = I1 + I2 + 2 I1 I2 cosφ 2π λD
for →
Imax → cos φ = 1 3 3d

I = 9I
Sol 8: (C)
Imin → cos φ = –1
Imin = I

c
Sol 2: (C) γ =
λ
In denser medium, c decreases but frequency remains
the same. Sol 9: (C)
∴ λ also decreases

Sol 3: (B) Maximum path difference = 100 nm = 3.5 λ


So, we can get pd of –3λ, –2λ, …., 3λ.
i.e. 7 maxima.

nλ1D (n + 1) λ2D as D increases


Sol 4: (B) =
d d
I1 moves away from O.
n(2200) = (n + 1)(5200)
∴ I first decreases, then increases.
⇒ 3n = 2(n + 1)
⇒n=2 Sol 10: (D)

(2n + 1) λ
Sol 5: (D) (l1 + l3) – (l2 + l4) =
2
  ↓   ↓
path by path by
  S1  
S2

Sol 6: (C) Let intensity due to single slit by I1. By two


2d d d
slits we get I. OI = − =
3 2 6
⇒ I = I1+I2+2 I1 I2 cos(φ)
d nλD
and φ = 0º at centre. =
6 d
⇒ I = 4I1
d2
I ⇒λ=
⇒ I1 = 6nD
4
d2
So is not possible.
Sol 7: (C) Let I be intensity due to single slit. 3D
I=I+I+2 I.I cos φ
2λD
Sol 11: (D) β =
−I −1 d
⇒ cos φ = =
2I 2 as c decreases, λ also decreases
⇒ φ = 120º
so β decreases but there won’t be any shift.
P hysi cs | 17.53

Sol 12: (A) d = µx S1P – S2P = nλ


Family of hyperbolas with n as variable.

Sol 16: (C) First coloured to be received is violet. As


µx 2πµx frequency of violet is high, lred is high and
phase difference = =
λ λ
µred > µviolet

Sol 13: (D)

λ
Sol 17: (A) = 1.33 t
2
300
⇒t= = 225 nm
1.33

Fringe width will not change it depends only on λ, d, D. Sol 18: (C) Image will coincide with S but on opposite
side.
To get Pd = 0, S1O > S2O.
So O will be below O1 pattern will shift downwards.

Sol 14: (A) Imax = 4I0


75% of Imax = 3I0
Sol 19: (B) They will get closer, if we use light of lower λ.
3I0 = I0 + I0 + 2I0.cosφ
i.e. using blue light.
1
cos φ = If ‘d’ decreases, β increases
2
β doesn’t depend on distance between source and slits.
⇒ φ = 2nπ ± 60º

It’s between Sol 20: (D) P1 = µAtA + tB

3π 6π P2 = tA + µBtB
Pd = tA.tB
↓ ↓
If tA > tB → towards B
2nd minima 3rd maxima
[same as in previous question]
π
only is not possible in the options If tB < tA → towards A
3

Sol 21: (A) If we put mica sheet in front of S1.


( 2 – 1)d = (µ – 1)t

⇒ t = 2( 2 – 1)d

In front of S2

Sol 15: (B) ( 2 – 1)d + (µ – 1)t = nλ.


1 7 . 5 4 | Wave Optics

Previous Years’ Questions or


2n – 1
=
7
=
14
=…
2m – 1 5 10
λD i.e., 4 th minima of 400 nm coincides with 3rd minima
Sol 1: (D) ω =
d of 560 nm.
d is halved and D is doubled Location of this minima is,
∴Fringe width ω will become four times. (2 × 4 − 1)(1000)(400 × 10−6 )
Y1 = = 14 mm
2 × 0.1
Sol 2: (C) Imax = ( I1 + I2 )2 = ( 4I + I )2 = 9I Next 11th minima of 400 nm will coincide with 8th
minima of 560 nm.
Imin = ( I1 – I2 )2 = ( 4I – I )2 = I
Location of this minima is,
(2 × 11 − 1)(1000)(400 × 10−6 )
Sol 3: (A) In interference we know that Y1 = = 42 mm
2 × 0.1
Imax = ( I1 + I2 )2 and Imin =( I1 ~ I2 )2 ∴ Required distance = Y2 – Y1 =28 mm

Under normal conditions (when the widths of both the


slits are equal) φ
Sol 6: (C) I = Imax cos2  
2
I1 ≈ I2 = I (say)
Imax φ
∴Imax = 4I and Imin = 0 ∴ = Imax cos2
4 2
When the width of one of the slits is increased. Intensity φ 1
due to that slit would increase, while that of the other cos =
2 2
will remain same. So, let :
φ π
I1 = I and I2 = ηI (η > 1) or =
2 3
Then, Imax = I (1 + η )2 > 4I
2π  2π 
∴φ= =   Dx ... (i)
And Imin = I ( η – 1)2 > 0 3  λ 
∴ Intensity of both maxima and minima is increased. where Dx =d sin q
Substituting in Eq. (i), we get
Sol 4: (B) I(φ) = I1 + I2 + 2 I1 I2 cos φ …(i)
 λ
sin θ =
Here, I1 = I and I2 = 4I 3d
π  λ 
At point A, φ = ∴ IA = I + 4I = 5I or θ = sin–1  
2  3d 
At point B, φ = π ∴ IB = I + 4I – 4I = I
∴ IA – IB = 4I Sol 7: Each plate reflects 25% and transmits 75%.

Note: Equation (i) for resultant intensity can be applied Incident beam has an intensity I. This beam undergoes
only when the sources are coherent. In the question multiple reflections and refractions. The corresponding
it is given that the rays interfere. Interference takes intensity after each reflection and refraction
place only when the sources are coherent. That is why (transmission) are shown in figure.
we applied equation number (i). When the sources are
3
incoherent, the resultant intensity is given by I = I2 + I2
9 64 3
64 16
Sol 5: (D) Let nth minima of 400 nm coincides with mth
minima of 560 nm, then 9
 16
 400   560 
(2n – 1)   = (2m – 1)   4
3
 2   2  4
P hysi cs | 17.55

Interference pattern is to take place between rays 1 (4737 × 10 –10 m)(1.33m)


and 2. ω= = 6.3×10–4 m
(1 × 10 –3 m)
I1 = I/4 and I2 = 9I/64 ω = 0.63 mm
2
Imin  I – I  1 (b) Let t be the thickness of the glass slab
∴ =  1 2 
=
Imax  I + I  49 Path difference due to glass slab at centre O.
 1 2 

Sol 8: Given λ = 6000 Å


Let b be the width of slit and D the distance between
screen and slit.
First minima is obtained at b sin θ = l
or bθ = λ as sin θ = θ  µglass   1.53 
Dx =  – 1 t =  – 1 t
 µliquid   1.33 
 
or Dx = 0.15t
Now, for the intensity to be minimum at O, this path
λ'
difference should be equal to
2
λ'
λ ∴ Dx =
orθ = Angular width of first maxima = 2θ 2
b 4737
2λ or 0.15 t = Å
= ∝l 2
b
∴ t = 15790 Å
Angular width will decrease by 30% when λ is also
or t = 1.579 mm
decreased by 30%.
Therefore, new wavelength
Sol 10: Let n1 bright fringe corresponding to
  30   wavelength l1 = 500 nm coincides with n2 bright fringe
λ’ = (6000) –   6000  Å corresponding to wavelength l2 = 700 nm.
  100  
λ’ = 4200 Å λ1D λ2D
\ n1 = n2
(b) When the apparatus is immersed in a liquid of d d
refractive index µ, the wavelength is decreased µ times. n1 λ2 7
or = =
Therefore, n2 λ1 5
6000Å
4200 Å = This implies that 7th maxima of l1 coincides with 5th
µ maxima of l2. Similarly 14th maxima of l1 will coincide
6000 with 10th maxima of l2 and so an.
\µ = or µ = 1.429 ≈ 1.43
4200 n1 λ1D
∴Minimum distance = = 7×5×10–7×103
d
Sol 9: Given, µ = 1.33, d = 1 mm, D = 1.33 m,
= 3.5 × 10–3 m = 3.5 mm
λ = 6300 Å
I0
(a) Wavelength of light in the given liquid: Sol 11: (B) Intensity of light transmitted by A =
2
λ 6300
λ’ = = Å = 4737 Å = 4737 × 10–10 m According to Malus law, the intensity of light transmitted
µ 1.33
by B
λ 'D
∴ Fringe width, ω = I0 I0 I0
d =
2
cos
= 2
θ
2
( 45°)
cos2=
4
1 7 . 5 6 | Wave Optics

Sol 12: (C) Consider a point P on the screen. The λD λ


path difference between the waves from S1 and S2 Sol 2: β = ; λ = 0
d µ
on reaching P is ( S2 P − S1 P ) . This path difference is
6 × 10−7 1
constant for every point on a circle of radius x with ∴β= ×3× = 2.25 × 10–4 m.
P0 as the centre. Hence the figures will be concentric 4 2 × 10−3
circles with common centre at P0 .
9λ D
Sol 3: 9th Bright fringe =
d
1.5λD
P 2nd dark fringe =
d
λ ×1
x 7. 5 × 10–3 = (9 - 1. 5)
5 × 10−4

P0 ⇒ λ = 5 × 10–7 m = 5000 Å.
S1 S2
Sol 4: Pd = d sin q
Note that S1 and S2 are point sources and (not slit
sources as in Young’s experiment). d sin θ =
λ
2
π
Sol 13: (B) d × 0. 75 = 2. 6 × 10–7
180°
I A cos2 30 = IB cos2 60
⇒ d = 2 × 10–2 mm.
IA 1
=
IB 3 Sol 5: Imax = 4I0
75% of Imax = 3I0
Sol 14: (B) We know that
Geometrical spread = a 3I0 = I0 + I0 + 2 I0 I0 cos φ
λL
and diffraction spread = 1
a ⇒ cos φ =
2
λL
So spot size(b) = a + φ = 60° , – 60°, 120°
a
λD
λL 360 →
For minimum spot size a = ⇒ a = λL d
a
λD
120° → = 0.2 mm.
and bmin = λL + λL = 4λL 3d

Sol 6: Possible pd for maxima → –2l1…. . . 2l


i. e. 5
JEE Advanced/Boards
d sin θ = λ for 1st maxima 

Exercise 1 ⇒ 3 sin θ = 1
1
sin θ =
Sol 1: For constructing interference 3

φ = 2xp 1
⇒ tan θ =
8
2π(x1 − x2 ) π π
= − + 2nπ
λ 8 6 y = tan θ = 0. 353 m.
 1 
⇒ (x1 – x2) =  n − λ .
 48 
P hysi cs | 17.57

Sol 7: (m1 – m2) t = Pd = 5l x


Sol 13: n =
β
5 × (48 × 10−8 ) xd
t= = 8 µm n=
0.3 λD
Sol 8: I0 = 4I1 dn x d(d) xv
= =
Pd = (y – 1) t dt λD dt λy

= 0. 5 × 1. 5 × 10–6
Sol 14: Let intensity of individual slit be I1
= 7.5 × 10–7 m
I0 = 4I1
λ = 5 × 10–7 m
with glass plate
Pd = 1. 5 l
(µ − 1)t
∴ I0 = 0 φ = 2π ×
λ
λD I = 2I1 + 2I1 cos f
Shift = 1.5 = 1.5 mm
d
I0
I= (1 + cos φ)
c 3 × 108 2
Sol 9: =
λ= = 5m
ν 6 × 107 2I
⇒ I0 =
λ φ
Dp must be 2cos2
2 2
λ
10 – x – x = (2n + 1)
2 (µ − 1)t 5λrD
Sol 15: f1 = × 2π ; 10–3 = 5b1 =
10 – 2x = 2.5 or 7.5 λr d

to get minimum x and φ = 10π


10 – 2x = 7.5 (µ − 1)t
f2 = × 2π
10 − 7.5 λg
=x = 1.25m
2
Dyc = 6b1

Sol 10: 25λ2 + d2 − d = nλ


Sol 16: Pd = (m1 – m2) t
Possible values of
3
I1 = I
n = 5, 4, …. . 1 4 max
for each value of d, we will get a circle with S1 as center. ⇒ φ = 60° or – 60° ± 2 np

λ φ must lie between 10π, 11π.


Sol 11: | AB – BC | = (2n + 1)
2
AB = 200 m
c 3 × 108
λ= =
ν 5.8 × 106

4 4 16
Sol 12: I A 'B' = × I = I
5 5 25

16 16
1+ +2
Imax 25 25
= = 81 : 1
Imin 16 16
1+ −2
2 5 25
1 7 . 5 8 | Wave Optics

Sol 17: φ =
(µ2 − µ1 ) (t2 − t1 )
× 2π
Exercise 2
λ0
Single Correct Choice Type
Ic = 2I0 + 2I0 cos f
Imax = 4I0. b d
Sol 1: (C) =
Vair Vwater
Sol 18: S will have 2 images which will act as sources Vair µ
b
and is similar to YDSE ⇒ = = water
d Vwater µair
1 1 1
− =
v u f
Sol 2: (D) For monochromatic light, Imax and fringe
f = 10 cm width is constant.
u = – 15 cm so, we use white light to determine central maximum.
1 1 1 1
= − =
v 10 15 30 Sol 3: (B) Case-I → I1 = I0+I0+2 I0 I0 cos(0º) = 4I0
v
d = 0. 5 × = 1mm Case-II → I2 = I0+I0+2 I0 I0 cos(90º) = 2I0
u
I1
Sol 19: Minima possible when =2
I2
λ
(a) Pd = (2n + 1)
2
Sol 4: (C) At O, Pd = S1S2 = d
i. e. – 0.75, – 0.25, 0.25, 0.75
(2n + 1)λ
d sin θ = pd if d = → 0 → minima
2
y = D tan q d = nλ → 0 → maxima
0.25 0.75 1 3 if d = 4.3 λ,
sin θ = , ⇒ ,
1 1 4 4
Possible minima
1 3
⇒ tan θ = , → –3.5λ, –2.5λ, …. 3.5λ.
15 4
i.e. 8 points.
(b) We need to find the initial Pd
Sol 5: (C)

D/ 3

D/ 3
d
Pd = d sin 30° = = 0.5 mm = l
2
So, there will be no shift.

λD
Sol 20: Let β =
d
5 Pd
= Pd = (m1 – m2)t
β λ
8 (µ − 1) (t2 − t1 )
=
β λ
P hysi cs | 17.59

Pd = d sinθ Sol 11: (A, C, D) Imax = I1 + kI1 + 2 k I1cosφ; (cosf→ 1)


d 3
= = × 10–4 m = I1(1 + k + 2 k ) and k < 1
2 2
3 Imin = I1(1 + k – 2 k )
= × 5 × 10–7 × 103 m = 300 λ
10 so Imax < I0 and Imin > 0
So, possible maxima
β doesn’t change.
–299λ, –298λ, ……. 299λ
Fringes will shift towards covered slit.
i.e. 599 maxima
yd
c 3 × 108
Sol 12: (A, C) λ = = = 300 m
Sol 6: (A) In the YDSE experiment, ∆x = , ν 106
D
yd λ
d=
for the maxima, ∆x = n λ ⇒ = nλ 2
D
n λD d
⇒ y = . In the question, y = .
d 6
2
d n λD d
Then,= ⇒
= λ where, n = 1, 2 , 3 , 4….
6 d 6nD
I1 = I1 + I2 + 2 I1 I2 cos(0º)
Sol 7: (A) Pd = n3t – n2t
I0 = 4I1
(n3 t − n2 t) 2π λ
φ= × 2n = (n − n2 )t If θ = 90º ; φ = ;I=0
λ0 n1 λ1 3 2
d λ
If θ = 30º; DPd = = ; φ = 90º
Sol 8: (C) Pd = (µ – 1) t 2 4
(µ − 1)t I0
φ= × 2π I = 2I1 =
λ 2
I = I0 + I0 + 2I0cosφ
Sol 13: (B, D) They must have same frequency and
constant φ.
They need not have same A1I.

Sol 9: (B) Pd = 2µt + t – µ(2t)


Pd = t Sol 14: (B, C) lred > lblue
y D Dt ∴ bred > bblue
= ⇒y=
Pd d d Fringe length decreases.
So, no. of maxima increases.

Multiple Correct Choice Type Sol 15: (A, C) Central maxima will shift towards A as
(µ – 1)t is added before A.
Sol 10: (B, C, D) Central fringe will white as phase Dx = (µ – 1)t
difference = 0 for all colours.
We can’t get completely dark fringe as all colours will (µ − 1)t
Dy = × β.
not have phase difference = 0 at a single point. λ
1 7 . 6 0 | Wave Optics

lviolet < lred, lviolet is minimum in visible region.


So, violet maxima is closest.

2λD 2 × 4.5 × 10−7


Sol 16: (C, D) β = = = 9 × 10–4 m
d 10−3
λD
= 4.5 × 10–4 m.
d
1
Pd = (µ – 1)t = 0.5 × 2.1 × 10–6 Sol 21: (A, B, D) β ∝
D
Pd Angular fringe width doesn’t depend on D.
φ=
λ Central fringe doesn’t change from O.
−7
10.5 × 10 1
= =2+
4.5 × 10 −7 3 Sol 22: (B, D) bx angular fringe width depends on ’d’

λd 2λd Position of central maxima doesn’t change.


So ,
3D 3D Rest all maxima, minima positions change.

Assertion Reasoning Type


Previous Years’ Questions
Sol 17: (D) Statement-I is false as path difference will
be zero.
Sol 1: (D) For first dark fring on either side d sin θ = λ or
PS = µS1O dy yD
1
=λ∴y=
PS = µS2O D d
2

Pd = 0 Therefore distance between two dark fringes on either


2yD
side = 2y =
c d
Sol 18: (A) ν = ; l > lblue
λ red
And µred > µblue
So, light speed of red > light speed of blue.

Sol 19: (C) Electromagnetic field at a point depends


also on time.
It’s magnitude depends with time
So statement-II is false. Substituting that values, we have

2(600 × 10 –6 mm)(2 × 103 mm)


Comprehension Type Distance = = 2.4 mm
(1.0mm)
2λd
Sol 20: (A, C, D) β =
D
So β ∝ λ
Central maxima is always at O in this case.
P hysi cs | 17.61

Sol 2: (D) At First minima, b sin θ = l For minimum value of t, n = 1


λ λ
\t = = = 2l
µ –1 1.5 – 1

Sol 5: (C) All points on a wavefront are at the same


phase.
\ fd = fc and ff = fe
\ fd – ff = fc – fe

y Sol 6: (A) → (p, s) → Intensity at P0 is maximum. It will


or bθ = λ or b   = l continuously decrease from P0 towards P2.
D 
λD λb (B) → (q) → Path difference due to slap will be
or y = or =λ …(i) compensated by geometrical path difference. Hence
b D
δ(P1) = 0
Now, at P (First minima) path difference between the λ λ λ λ
(C) → (t) → δ(P1) = , δ(P1) = – = and δ(P2)
rays reaching from two edges (A and B) will be 2 2 4 4
λ λ λ
= – = . When path difference increases from
λb λb 2 3 6
Dx = (Compare with Dx = in YDSE)
D D λ
0 to , intensity will decrease from maximum to zero.
or Dx = l[From eq. (i)] 2
Hence in this case, I(P2) > I(P1) > I(P0)
Corresponding phase difference (φ) will be
(D) → (r) → Intensity is zero at P1
 2π  2π
φ =   .Dx, φ = .λ = 2p
( )
2
 λ  λ I1 + I2  I / I +1
2
Imax
Sol 7: (B, D) = =  1 2  =
Imin
( )
2  I / I –1
λD I1 – I2  1 2 
Sol 3: (B) Fringe width, ω = ∝l
d
9 (Given)
When the wavelength is decreased from 600 nm to 400
I1
4 Solving this, we have =4
nm, fringe width will also decrease by a factor of or I2
6
2 A1
or the number of fringes in the same segment will But I ∝ A2∴ =2
3 A2
increase by a factor of 3/2.
b
Therefore, number of fringes observed in the same Sol 8: (A, C) At P (directly infrom of S1) y =
2
3
segment = 12 × = 18 \Path difference,
2
Note: since ω ∝ λ, if YDSE apparatus is immersed in a y.(b)
DX = S2P – S1P =
liquid of refractive index µ, the wavelength λ, and thus d
the fringe width will decrease µ times.
b
  (b)
2 b2
Sol 4: (A) Path dirrerence due to slab should be =   =
d 2d
integeral multiple of λ or ∆ x = nλ
or (µ – 1)t = nλ n = 1, 2, 3

or t =
µ –1
1 7 . 6 2 | Wave Optics

90 180
= 2
+ cos60º + 20 cos 60º
4 π(3) 4 π(1.5)2

= 0.79 + 3.18 + 10
b
= 13.97 W/m2
2
Sol 11: Power received by aperture A,
10
PA = I( πrA2 ) = (π) (0.001)2 = 10–5 W
π
Those wavelengths will be missing for which Power received by aperture B,
λ1 3λ2 5λ3 10
DX = , , … PB = I( πrB2 ) = (π) (0.002)2 = 4×10–5 W
2 2 2 π
Only 10% of PA and PB goes to the original direction
b2 2∆x b2
\l1 = 2Dx = l2 = =
d 3 3d Hence, 10% of PA = 10–6 = P1 (say)

2∆x b2 and 10% of PB = 4 × 10–6 = P2 (say)


l3 = =
5 5d Path difference created by slab
δ Dx = (µ – 1)t = (1.5 – 1) (2000) = 1000Å
Sol 9: (A, C) The intensity of light is I(θ) = I0 cos2  
2
Corresponding phase difference,
2π  2π 
where δ = (Dπ) =   (d sin θ)
λ  λ  2π 2π π
φ= . Dx = × 1000 =
(a) for θ = 30º λ 6000 3
Now, resultant power at the focal point
c 3 × 108
λ= = =300 m and d = 150 m
v 106 P = P1 + P2 + 2 P1P2 cos φ
 2π  1 π
δ=   (150)   = π
 300  2 2 = 10–6+4×10–6+ 2 (10 –6 )(4 × 10 –6 ) cos
3
δ π
∴ = = 7 × 10–6 W
2 4
π I
∴I(θ) = I0 cos2  4  = 0 [option (a)] Sol 12: (a) Path difference due to the glass slab,
  2
Dx = (µ – 1)t = (1.5 – 1)t = 0.5t
(b) For θ = 90º
Due to this slab, 5 red fringes have been shifted
 2π  upwards.
δ=   (150) (1) = p
 300  Therefore, Dx = 5lred or 0.5t = (5) (7 × 10–7m)
δ π ∴t = thickness of glass slab =7×10–6m
or = and I(θ) = 0
2 2
(b) Let µ’ be the refractive index for green light then
δ
(c) For θ = 0º, δ = 0 or =0 Dx’ = (µ’ – 1)
2
∴I(θ) = I0 [option (c)] Now the shifting is of 6 fringes of red light. Therefore,
Dx’ = 6lred
Sol 10: Resultant intensity at P
∴(µ’ – 1)t = 6lred
I p = IA + IB + IC (6)(7 × 10 –7 )
∴(µ’ – 1) = = 0.6
PA PB 7 × 10 –6
= + cos 60º + IC cos 60º ∴µ’ = 1.6
2
4 π(PA) 4 π(PB)2
P hysi cs | 17.63

(c) In part (a), shifting of 5 bright fringes was equal to 3


Intensity at O is given I and since
10–3 m. Which implies that 4 max
5wred = 10–3 m φ
I(φ) = Imax cos2  
(Here ω = Fringe width) 2
10 –3 3 φ
∴wred = m = 0.2 × 10–3 m ∴ I = Imax cos2  
5 4 max 2
λD
Now since ω = or ω ∝ l 3 φ
d or = cos2    …(iv)
4 2
ωgreen λgreen
∴ =
ωred λred From Equation (iii) and (iv), we find that
λ
λgreen  5 × 10 –7  ∆=
∴ wgreen = wred = (0.2×10–3)   6
 7 × 10 –7 
λred   λ 31
i.e., Dx = 5λ + + = λ = 0.3t
6 6
wgreen = 0.143 × 10–3 m
31λ (31)(5400 × 10 –10 )
∴Dω = wgreen – wred = (0.143 – 0.2) × 10 m –3 ∴t = =
6(0.3) 1.8

Dω = – 5.71 × 10–5 m or t = 9.3 × 10–6 m = 9.3 mm

Sol 13: m1 = 1.4 and m2 = 1.7 and let t be the thickness Imax φ
of each glass plates. Sol 14: (B) = Im cos2  
2 2
Path difference at O, due to insertion of glass plates φ 1
will be ⇒ cos   =
2
  2
φ π
⇒ =
2 4
π

= φ
2
( 2n + 1)
λ λ π λ
x
⇒ ∆= =
φ × ( 2n + 1=
) ( 2n + 1 )
2π 2π 2 4


Dx = (m2 – m1)t = (1.7 – 1.4)t = 0.3t  …(i) Sol 15: (A, B, C) β =
d
Now, since 5th maxima (earlier) lies below O and 6th
minima lies above O.
 λ2 > λ1 ⇒ β2 > β1

λ Also m1 β1= m2 β2 ⇒ m1 > m2


This path difference should lie between 5λ and 5λ +
2
D   D 
So, let Dx = 5λ + ∆  …(ii) Also 3   ( 600 nm) = ( 2 × 5 − 1)  2d  400 nm
d  
λ λ
Where ∆ < Angular width θ =
2 d
Due to the path difference Dx, the phase difference at
O will Sol 16: For maxima,
2π 2π
φ= Dx = (5λ + ∆) 4
λ λ d2 + x2 − d2 + x2 =m λ , m is an integer
3
2π 2
= (10π + .∆)  …(iii) So, x= 9m2 λ2 − d2
λ
∴p =3
2017-18 100 &
op kers
Class 12 T
By E ran culty
-JE Fa r
IIT enior emie .
S fP r es
o titut
Ins

PHYSICS
FOR JEE MAIN & ADVANCED
SECOND
EDITION

Exhaustive Theory
(Now Revised)

Formula Sheet
9000+ Problems
based on latest JEE pattern

2500 + 1000 (New) Problems


of previous 35 years of
AIEEE (JEE Main) and IIT-JEE (JEE Adv)

5000+Illustrations and Solved Examples


Detailed Solutions
of all problems available

Topic Covered Plancess Concepts


Tips & Tricks, Facts, Notes, Misconceptions,
Electric Charges, Key Take Aways, Problem Solving Tactics
Forces and Fields
PlancEssential
Questions recommended for revision
18. ELECTRIC CHARGES,
FORCES AND FIELDS

1. INTRODUCTION
You must have felt the attraction of hair of your hand when you bring it near to your Television screen. Did you ever
think of cause behind it? These all are the electric charges and their properties. Now we will extend our concept to
electric charges and their effects.

1.1 Nature of Electricity


The atomic structure shows that matter is electrical in nature i.e. matter contains particles of electricity viz. protons
and electrons. Whether a given body shows electricity (i.e. charge) or not depends upon the relative number of
these particles in the body.
(a) If the number of protons is equal to the number of electrons in a body, the resultant charge is zero and the
body will be electrically neutral. For example, the paper of this book is electrically neutral (i.e. exhibits no
charge) because it has the same number of protons and electrons.
(b) If from a neutral body, some *electrons are removed, the protons outnumber the electrons. Consequently, the body
attains a positive charge. Hence, a positively charged body has deficit of electrons from the normal due share.

2. TYPES OF CHARGES
Depending upon whether electrons are removed or added to a body, there are two types of charges viz
(i) Positive charge (ii) Negative charge
If a glass rod is rubbed with silk, some electrons pass from glass rod to silk. As a result,
the glass rod becomes positively charged and silk attains an equal negative charge as
+
- - +
shown in Fig. 18.1. It is because silk gains as many electrons as lost by the glass rod. It - - +
can be shown experimentally that like charges repel each other while unlike charges
+
attract each other. In other words, if the two charges are of the same nature (i.e., both Silk Glass rod
positive or both negative), the force between them is of repulsion. On the other hand, if
one charge is positive and the other is negative, the force between them is of attraction.
Figure 18.1
The following points may be noted:
(a) The charges are not created by the rubbing action. There is merely transfer of electrons from one body to
another.
(b) Electrons are transferred from glass rod to silk due to rubbing because we have done external work. Thus law
of conservation of energy holds.
(c) The mass of negatively charged silk will increase and that of glass rod will decrease. It is because silk has
gained electrons while glass rod has lost electrons.
1 8 . 2 | Electric Charges, Forces and Fields

3. PROPERTIES OF CHARGE
(a) Charge is a scalar quantity
(b) Charge is transferable
(c) Charge is conserved
(d) Charge is quantized
(e) Like point charges repel each other while unlike point charges attract each other.
(f) A charged body may attract a neutral body or an oppositely charged body but it always repels a similarly
charged body
(g) Note: Repulsion is a sure test of electrification whereas attraction may not be.
(h) Charge is always associated with mass, i.e. charges cannot exist without mass though mass can exist without
charge.
(i) Charge is relatively invariant: This means that charge is independent of frame of reference, i.e, charge on a
body does not change whatever be its speed. This property is worth mentioning as in contrast to charge, the
mass of a body depends on its speed and increases with increase in speed.
( j) A charge at rest produces only electric field around itself; a charge having uniform motion produces electric
as well as magnetic field around itself while a charge having acceleration emits electromagnetic radiation also
in addition to producing electric and magnetic fields.

4. ELECTROSTATICS
The branch of physics which deals with charges at rest is called electrostatics. When a glass rod is rubbed with
silk and then separated, the former becomes positively charged and the latter attains equal negative charge. It is
because during rubbing, some electrons are transferred from glass to silk. Since glass rod and silk are separated by
an insulating medium (i.e. air), they retain the charges. In other words, the charges on them are static or stationary.
Note that the word ‘electrostatic’ means charges at rest.

5. CONDUCTORS AND INSULATORS


In general, the substances are divided into the following two classes on the basis of their ability to conduct electric
charges:
(a) Conductors: Those substances through which electric charges can flow easily are called conductors e.g., silver,
copper, aluminum, mercury, etc. In a metallic conductor, there are a large number of free electrons which act as
charge carries. However, in a liquid conductor, both positive and negative ions are the charge carries. When a
positively charged body is brought close to or touches a neutral conductor (metallic), the free electrons (charge
carriers) in the conductor move quickly towards this positive charge. On the other hand, if a negatively charged
body is brought close to or touches a neutral conductor, the free electrons in the conductor move away from the
negative charge that is brought close.
(b) Insulators: Those substances through which electric charges cannot flow are called insulators e.g., glass, rubber,
mica etc. When such materials are charged by rubbing, only the area that is rubbed becomes charged and there is
no tendency of the charge to move into other regions of the substance. It is because there are practically no free
electrons in an insulator.

6. CHARGING OF A BODY
A body can be charged by means of (a) friction, (b) conduction, (c) induction, (d) thermionic ionization, (e)
photoelectric effect and (f) field emission.
P hysi cs | 18.3

(a) Charging by Friction: When a neutral body is After rubbing


+++++++ -
-----
-
rubbed with another neutral body (at least one of
+ + -
A B + A + -
- B -
-
-
them should be insulator) then some electrons are
+ +
+ +
+++++++
-
-----
-
transferred from one body to another. The body which
gains electrons becomes negatively charged and the Neutral
Figure 18.2
other becomes positively charged.
(b) Conduction (flow): There are two types of materials in nature.
(i) Conductor: Materials which have large number of +++ +++
free electrons.
++ ++
++ ++
(ii) Insulator or Dielectric or Nonconductors: +++
++ A B
Materials which do not have free electrons. ++
+++ +++
When a charged conductor is connected with a A ++
++
++
++
neutral conductor, then charge flows from one body
to another body. In case of two charged conductors, A B
charge flows from higher potential to lower potential. Figure 18.3
The charge stops flowing when the potential of the two
bodies become same.
Note: If two identical shaped conductors kept at large distance are connected to each other, then they will have
equal charges finally.
(c) Induction: When a charged particle is taken near to a neutral Induced charge
object, then the electrons move to one side and there is excess of
electrons on that side making it negatively charged and deficiency on -- +++
-- +
the other side making that side positively charged. Hence charges
+ + + -
+ + + -q - +
- +q
appear on two sides of the body (although total charge of the body + + + -
- ++
-
is still zero). This phenomenon is called induction and the charge
produced by it is called induced charge. Figure 18.4

A body can be charged by induction in following two ways.


++
-------
+ --
Method-I: The potential of conductor A becomes zero after earthing. -
- + -
- -
-
+ + + --- +
--- -
-
To make potential zero some electrons flow from the Earth to the + + + -
- + -
- -
-
+ + + --- +
-
-
-- --
conductor A and now connection is removed making it negatively -- ----
A B
charged.
Figure 18.5

Method-II: The conductor which has included charge on it, is


++ ++
+ +
connected to a neutral conductor which makes the flow of charge
-
-
--- + +
+ + + + +
such that their potentials become equal and now they are
+ + + -
- + +
+ + + --- + +
disconnected making the neutral conductor charged. A B
Figure 18.6

(d) Thermo-ionic emission: When the metal is heated at a high temperature then some electrons of metals are
ejected and the metal gets ionized. It becomes positively charged.

e- e- e-

e- + + + + + + e-
+ + + + + +
+ + + + + +

Heat
Figure 18.7
1 8 . 4 | Electric Charges, Forces and Fields

(e) Photoelectric effect: When light of sufficiently high frequency is incident on Light
metal surface then some electrons come out and metal gets ionized.
e- e- e-

+ + + + e-
+ + + +
+ + + +

Figure 18.8
(f) Field emission: When electric field of large magnitude is applied near the
metal surface then some electrons come out from the metal surface and hence e-
the metal gets positively charged. E
e-

e-

Figure 18.9
7. UNIT OF ELECTRIC CHARGE
We know that a positively charged body has deficit of electrons and a negatively charged body has excess of
electrons from normal due share. Since the charge on an electron is very small, it is not convenient to select it as the
unit of charge. In practice, coulomb is used as the unit of charge, i.e., SI unit of charge is coulomb abbreviated as C.
The charge on one electron in coulomb is =-1.6×10-19C
Note that charge on an electron has been found experimentally.

8. QUANTIZATION OF ELECTRIC CHARGE


The charge on an electron (-e=1.6×10-19C) or on a proton (+e=+1.6×10-19C) is minimum. We know that charge on a
body is due to loss or gain of electrons by the body. Since a body cannot lose or gain a fraction of an electron, the
charge on a body must be an integral multiple of electronic charge ±e . In other words, charge on a body can only
be q = ±ne where n=1, 2, 3, 4, and e=1.6×10-19C. This is called quantization of charge.

The fact that all free charges are integral multiple of electronic charge (e=1.6×10-19C) is known as quantization of
electric charge.
∴ Charge on a body, q = ±ne

Where n=1, 2, 3…….. and e=1.6×10-19C


Suppose you measure the charge on a tiny body as +4.5×10-19C. This measurement is not correct because measured
value is not an integral multiple of minimum charge (i.e., 1.6×10-19C).
Note: (i) The quantization of charge shows that charge is discrete in nature and not of continuous nature.

(ii) Since the charge on an electron is so small (e=1.6×10-19C), we normally do not notice its discreteness in
macroscopic charge (1µC charge requires about 1013 electrons) which thus seems continuous.

9. CONSERVATION OF ELECTRIC CHARGE


Just as total linear momentum of an isolated system always remains constant, similarly, the total electric charge
of an isolated system always remains constant. This is called law of conservation of charge and may be stated as
under: The total electric charge of an isolated system always remains constant.
In any physical process, the charges may get transferred from one part of the system to the other but total or net
charge remains the same. In other words, charges can neither be created nor destroyed. No violation of this law
has ever been found and it is as firmly established as the laws of conservation of linear momentum and energy.
P hysi cs | 18.5

Electrostatic Force-Coulomb’s Law


F = Electrostatic force
q1q2
q = Electric charge Fs =k
r2
r= Distance between charge centers
k= Coulomb constant 9.0×109N.m2/C2

r
F₂₁ F₁₂

Unlike charges attract.


+ -
q₁ q₂

+ F₂₁ F₁₂
+
q₁ q₂
Like charges repel.

+ F₁₂
-
F₂₁
q₁ q₂

Figure 18.10
   
F21 is the force on charge 1 due to 2 and r12 =
r1 − r 2 =−r 21

PLANCESS CONCEPTS

In few problems of electrostatics, Lami’s theorem is very useful.  F2
According to this theorem, “if three concurrent forces  
   F1
F1 ,F2 and F3 as shown in Fig. 18.11 are in equilibrium or if 
  
F1 + F2 + F3 = 0 , then 

F1 F2 F3 
F3
= =
sin α sin β sin γ
Nivvedan (JEE 2009 AIR 113) Figure 18.11

10. RELATIVE PERMITTIVITY OR DIELECTRIC CONSTANT


Permittivity is the property of a medium and affects the magnitude of force between two point charges. Air or vacuum
has a minimum value of permittivity. The absolute (or actual) permittivity of air or vacuum is 8.854 × 10−12 C2N−1m−2
. The absolute permittivity ε of all other insulating materials is greater than ε0 . The ratio ε / ε0 is called relative
permittivity of the material and is denoted by K or ( εr ).

ε Absolute permittivity of medium


K
= = It may be noted that the relative permittivity is also called
ε0 Absolute permittivity of air ( or vacuum)
dielectric constant.
1 q1q2
Another Definition. Force between two charges in air (or vacuum) is Fair = [See Fig. 18.12]
4 πε0 r 2

Force between the same two charges held same distance apart in a medium of absolute permittivity ε is
1 8 . 6 | Electric Charges, Forces and Fields

1 q1q2 1 q1q2
=Fm = [see Fig. 18.12]
4 πε r 2 4 πε0K r 2

q1 0 q2 q1 K q2

r r

(i) (ii)
Figure 18.12
Fair ε
∴ = = K =Relative permittivity of the medium
Fm ε0

Hence, relative permittivity (or dielectric constant) of a medium may be defined as the ratio of force between two
charges separated by a certain distance in air (or vacuum) to the force between the same charges separated by the
same distance in the medium.

Discussion. The following points may be noted:


(a) For air or vacuum, K =ε0 / ε0 =1. For all other insulating materials, the value of K is more than 1.

Fm = Fair / K . This implies that force between two charges is decreased when air is replaced by other insulating
(b)
medium. For example, K for water is 80. It means that for the same charges ( q1 ,q2 ) and same distance (r), the
force between two charges in water is 1/80th of that in air.
Fair ε
(c) K is number; being the ratio of two absolute permitivities. K = K=
Fmed ε0

Comparison of Electrical Force with the Gravitational Force.


(a) Both electrical and gravitational forces follow the inverse square law.
(b) Both can act in vacuum also.
(c) Electrical forces may be attractive or repulsive but gravitational force is always attractive.
(d) Electrical forces are much stronger than gravitational forces.
(e) Both are central as well as conservative forces.
(f) Both the forces obey Newton’s third law.

11. SUPERPOSITION OF ELECTROSTATIC FORCE


If in a region, more than 2 charges are present, then the net force acting on a particular charge will be the vector
sum of the individual contribution of all other charges present in region, presence of any other charge in space
cannot affect the force applied by a particular charge.
     
F1.net = F12 + F13 + F14 + F15 + ......... + F1n ,

Illustration 1: Two identical balls each having a density ρ are suspended from a common point by two insulating
strings of equal length. Both the balls have equal mass and charge. In equilibrium, each string makes an angle θ
with vertical. Now, both the balls are immersed in a liquid. As a result, the angle θ does not change. The density of
the liquid is σ . Find the dielectric constant of the liquid. (JEE ADVANCED)

Sol: Inside the liquid, up thrust would act but simultaneously, electric force would also weaken due to dielectric of
the liquid.
P hysi cs | 18.7

In vacuum each ball is in equilibrium under the


following three forces:
 
(i) Tension, (ii) Electric force and (iii) Weight.
 T  T’
So, Lami’s theorem can be applied.
Fe Fe‘
Fe
In the liquid, Fe' = Where, K=dielectric constant
K
W W’
of liquid and W’=W-up thrust
In vacuum In liquid
Applying Lami’s theorem in vacuum
Figure 18.13

W Fe W F
= or = e  … (i)
( ο
sin 90 + θ ) (
sin 180 − θ ο
) cos θ sin θ

W' F'
Similarly in liquid = e  ... (ii)
cos θ sin θ
W Fe W  F 
Dividing Eq.(i) by Eq.(ii), we get = or K =  as e = k 
W' F' W − upthrust  Fe' 
e  
Vρg ρ
(V=volume of ball) Or K =
Vρg − Vσg ρ−σ

Note: In the liquid Fe and W have been changed. Therefore, T will also change.

Illustration 2: A non-conducting rod of length L with a uniform positive charge density λ and a total charge Q is
lying along the x-axis, as illustrated in Fig. 18.14.  (JEE ADVANCED)
Calculate the force at a point P located along the axis of the rod and a distant x0 from one end of the rod.

Sol: Consider rod as large number of small charges and apply y


principle of superposition of forces. dq=dx’
dx’
The linear charge density is uniform and is given by λ =Q / L .
x’
The amount of charge contained in a small segment of length dF
x
dx' is dq = λdx' .
P L
Since the source carries a positive charge Q, the force at P points Q X0
in the negative x direction, and the unit vector that points from
the source to P is r̂ = ˆi . The contribution to the electric field due Figure 18.14
to dq is
 Q dq Q λdx' ˆ 1 Q2dx' ˆ
dF =
4 πεο r 2
rˆ =
4 πεο x' 2
− i =− ( )
4 πεο Lx' 2
i

Integrating over the entire length leads to

  1 Q2 xο + L dx' 1 Q2  1 1 ˆ 1 Q2
ˆi = ˆi
F=∫ dF =

4 πεο L ∫xο x' 2
−  − 
4 πεο L  x ο x ο + L 
i=

4 πεο x ο (L + x ο )

 1 Q2 ˆ
Notice that when P is very far away from the rod, xo >> L and the above expression becomes F ≈ − i
4 πεο x2
ο

The result is to be expected since at sufficiently far distance away, the distinction between a continuous charge
distribution and a point charge diminishes.
1 8 . 8 | Electric Charges, Forces and Fields

12. ELECTRIC FIELD


A charged particle cannot directly interact with another particle kept at a distance. A charge produces something
called an electric field in the space around it and this electric field exerts a force on any other charge (except the
source charge itself) placed in it.
Thus, the region surrounding a charge or distribution of charge in which its electrical effects can be observed is
called the electric field of
 the charge or distribution of charge. Electric field at a point can be defined in terms of
either a vector function E called ‘electric field strength’ or a scalar function V called ‘electric potential’. The electric
field can also be visualized graphically in terms of ‘lines of force’. The field propagates through space with the
speed of light, c. Thus, if a charge is suddenly moved, the force it exerts on another charge a distance  r away does
not change until a time r/c later. In our forgoing discussion we will see that electric field strength E and electric
potential V are interrelated. It is similar to a case where the acceleration, velocity and displacement of a particle are
related to each other.


12.1 Electric Field Strength (E)
Like its gravitational counterpart, the
 electric field strength (often called electric field) at a point in an electric field is
defined as the electrostatic force Fe per unit positive charge. Thus, if the electrostatic
 force experienced by a small
  Fe
test charge q0 is Fe , then field strength at that point is defined as, E = lim
q0 →0 q
0
( q0 → 0 so that it doesn’t interfere with the electrical field)

The electric field is a vector quantity and its direction is the same as the direction of the force Fe on a positive test
charge. The SI unit of electric
 field is N/C. Here it should be noted that the test charge q0 does not disturb other
charges which produces E . With the concept of electric field, our description of electric interactions has two parts.
First, a given charge distribution acts as a source of electric field. Second, the electric field exerts a force on any
charge that is present in this field.

An electric field leads to a force



Suppose there is an electric field strength
 E at some point in an electric field, then the electrostatic force acting on
a charge +q is qE in the direction of E , while on the charge –q it is qE in the opposite direction of E .
 
The electric field at a point is a vector quantity. Suppose E1 is the field at a point due to a charge q1 and E2
is the
 field
 at the same point due to a charge q2 . The resultant field when both the charges are present is
=E E1 + E2
If the given charges distribution is continuous, we can use the technique of integration to find the resultant electric
field at a point.

Illustration 3: A uniform electric field E is created between two parallel charged plates as shown in Fig. 18.15. An
electron enters the field symmetrically between the plates with a speed v ο . The length of each plate is l. Find the
angle of deviation of the path of the electron as it comes out of the field.  (JEE MAIN)

Sol: Electron gains velocity in the vertical direction due to field between the plates.
eE
The acceleration of the electron is a = in the upward direction. The horizontal velocity remains v ο as there is
m
l
no acceleration in this direction. Thus, the time taken in crossing the field is t = . … (i)
vο
The upward component of the velocity of the electron as it emerges
from the field region is
+ + + + + + 
eEl
v=
y at
=
mv ο V₀ E
- - - - - -
The horizontal component of the velocity remains v x = v ο .
Figure 18.15
P hysi cs | 18.9

The angle θ made by the resultant velocity with the original direction is given by
vy eEl eEl
tan=
θ = . Thus, the electron deviates by an angle θ =tan−1 .
v x mv 2 mv ο2
ο

PLANCESS CONCEPTS

Charge Densities
It is of three types:
(i) Linear charge density: It is defined as charge per unit length, i.e.
q
λ = its S.I. unit is coulomb/ metre and dimensional formula is  ATL−1 
l  
(ii) Surface charge density: It is defined as charge per unit area, i.e.
q
σ= its S.I. unit is coulomb / metre2 and dimensional formula is  ATL−2 
A  
(iii) Volume charge density: It is defined as charge per unit volume i.e.
q
ρ= its S.I. unit is coulomb / metre3 and dimensional formula is  ATL−3 
V  
Nitin Chandrol (JEE 2012 AIR 134)

12.2 Electric Fields Due to Continuous Charge Distributions


 1 dq
The electric field at a point P due to each charge element dq is given by Coulomb’s law: dE = rˆ
4 πεο r 2

Where r is the distance from dq to P and r̂ is the corresponding unit vector. Using the superposition principle, the
  1 dq
total electric field E is the vector sum (integral) of all these infinitesimal contributions: E =
4 πεο ∫v r 2

This is an example of a vector integral which consists of three separate integrations, one for each component of
the electric field.

12.3 Electric Field Due to a Point Charge


q₀
r
The electric field produced by a point charge q can be obtained in general

q Fe
terms from Coulomb’s law. First note that the magnitude of the force exerted
by the charge q on a test charge q0 is, 
1 qq0 q E
Fe = .
4 πεο r 2 
1 q E
Then divide this value by q0 to obtain the magnitude of the field: E = . q
4 πεο r 2
 Figure 18.16
If q is positive, E is directed away from q. On the other hand, if q is negative,
then E is directed towards q.

12.4 Electric Field Due to a Ring of Charge


A conducting ring of radius R has a total charge q uniformly distributed over its circumference. We are interested
in finding the electric field at point P that lies on the axis of the ring at a distance x from its center. We divide the
ring into infinitesimal segments of length dl. Each segment has a charge dq and acts as a point charge source of
electric field.
1 8 . 1 0 | Electric Charges, Forces and Fields


Let dE be the electric field from one such segment; the net
y
 dl,dq
electric field at p is then the sum of all contributions dE
from all the segments that make up the ring. If we r
consider two ring segments at top and  bottom of the R
r = X² +R²
ring, we see that the contributions dE to the field at P
from these segments have the same x-component but 90⁰
opposite y-components. Hence, the total y-component  dE x
of field due to this pair of segments is zero. When we add O x P 
up the contributions
 from all such pairs of segments, the 
dE
total field E will have only a component along the ring’s q dEy
symmetry axis (the x-axis) with no component
perpendicular to that axis (i.e. no y or z component). So Figure 18.17
the field at P is described completely by its x component
Ex .

 q  1 dq
Calculation of Ex dq =   .dl ; dE = .
 2πR  4 πεο r 2

dEx dEcos
 1   dq   x 

 1  ( dq) x
∴= = θ 
 4 πε   2 
2 
=
 4 πε  .

( )
32
 ο   x + R   x2 + R 2   ο  x2 + R 2

x  1  qx
∫ dE=x dq ; or Ex = 
32 ∫
∴ E= 
( )
x
( ) 4 πε 32
4 πεο x2 + R 2  ο  x2 + R 2

From the above expression, we can see that


(a) Ex = 0 at x=0, i.e., field is zero at the center of the ring. We Ex
should expect this, charges on opposite sides of the ring would
push in opposite directions on a test charge at the center, and
the forces would add to zero.
Emax
1 q
(b) Ex = . for x >> R i.e., when the point P is much farther
4 πεο x2
from the ring, its field is the same as that of a point charge.

To an observer far from the ring, the ring would appear like a R X
2
point, and the electric field reflects this.
Figure 18.18
dEx
(c) Ex will be maximum where = 0 . Differentiating Ex w.r.t. x
dx
R 2  1 q 
and putting it equal to zero we get x = and Emax comes out to be,  . .
2 3
3  4 πεο R 2 

12.5 Electric Field Due to a Line Charge


Positive charge q is distributed uniformly along a line with length 2a, lying along the y-axis between y=-a and
y=+a. We are here interested in finding the electric field at point P on x-axis.

q q 1 dq q dy
λ =charge per unit length= λdy = dy ; dE =
dq = . =
2a 2a 4 πεο r 2 4 πεο 2a x2 + y 2 ( )
q xdy
dEx dEcos
= = θ .
4 πεο
( )
3/2
2a x2 + y 2
P hysi cs | 18.11

q ydy
dEy = −dEsin θ = .
4 πεο
( )
3/2 r = X² +R²
2a x2 + y 2

dy
1 qx a dy q 1
∴ Ex =. ∫ .
4 πεο 2a
( )
4 πεο x x2 + a2
− a 3/2 r
x2 + y 2 y

P dE x
1 q a ydy 
and Ey = . ∫ 0 O 
x
4 πεο 2a −a 2
( )
3/2 x
2
x +y
dEy
Thus, electric field is along x-axis only and which has a
magnitude,

q
Ex =  … (i) Figure 18.19
4 πεο x x2 + a2

From the above expression, we can see that:

1 q
(a) If x >> a, Ex = . , i.e., if point P is very far from the line charge, the field at P is the same as that of a
4 πεο x2
point charge.
(b) Now assume that, we make the line of charge longer and longer, adding charge in proportion to the total
length so that λ , the charge per unit length remains constant. In this case Eq(i) can be written as,
1  q 1 λ
Ex = . . =
2πεο  2a  x x2 / a2 + 1 2πεο x x2 / a2 + 1

λ
= x2 / a2 → 0 asaa
Now, >>x,x, Ex =
2πεο x

13. ELECTRIC FIELD LINES


An electric line of force is an imaginary smooth curve in an electric field along which a free, isolated unit positive
charge moves.

Properties
(a) Electric lines of force start at a positive and terminate at a negative charge.
(b) A tangent to a line of force at any point gives the direction of the force on positive charge and hence direction
of electric field at that point.
(c) No two lines of force can intersect one another.
(d) The lines of force are crowded in the region of larger intensity and further apart in the region of weak field.
(e) Lines of force leave the surface of a conductor normally.
(f) Electric lines of force do not pass through a closed conductor.
1 8 . 1 2 | Electric Charges, Forces and Fields

Field of some special classes


We here highlight the following charge distributions.
(a) Single positive or negative charge (Fig. 18.20
(a) and (b))- The field lines of a single positive
charge are radially outward while those of a
single negative charge are radially inward. q>0 q<0
(b) Two equal positive charges (Fig18.20 (c))- the (a) (b)
field lines around a system of two positive
charges (q, q) give a vivid pictorial description
of their mutual repulsion.
(c) Two equal and opposite charges (Fig18.20
(d))-The field around the configuration of two +q +q +q -q
equal and opposite charges (q,-q), a dipole,
show clearly the mutual attraction between (c)
the charges.
Figure 18.20
Properties:
(a) Line of force originates out from a positive charge and terminates on a negative
charge. If there is only one positive charge then lines start from positive +B
charge and terminate at ∞ . If there is only one negative charge then lines
start from ∞ and terminate at negative charge.
(b) The electric intensity at a point is the number of lines of force streaming +A
through per unit area normal to the direction of the intensity at that point. EA > EB
The intensity will be more where the density of lines is more.
Figure 18.21
(c) Number of lines originating (terminating) from (on) is directly proportional
to the magnitude of the charge.
Note: A charge particle need not follow an Electric field lines.
(a) Electric field lines of resultant electric field can never intersect with each other.
(b) Electric field lines produced by static charges do not form close loop.
(c) Electric field lines end or start perpendicularly on the surface of a conductor.
(d) Electric field lines never enter in to conductors.

Illustration 4: Consider the situation shown in Fig. 18.22. What are the signs of
q1 and q2 ? If the lines are drawn in proportion to the charge, what is the ratio
q1 / q2 ?  (JEE MAIN)

Sol: Use properties of field lines.  q₂


The basic concept of this question is that number density is directly proportional
to electric field. If we take the entire area of the sphere around the charge, then
area will be the same. Now, we just have to count the number of lines originating
from the two charges.
q₁
In case of point charges, E ∝ q
Thus, E1=
/ E2 q1=
/ q2 n1=
/ n2 6=
/ 18 1 / 3
Figure 18.22
However, this problem can also be seen by flux. Why don’t you try it as an
exercise?Plus, q1 has to be negative, while q2 would be positive.
P hysi cs | 18.13

14. ELECTRIC FLUX


The strength of an electric field is proportional to the number of field lines per unit area.
The number of electric field lines that penetrates a given surface is called an “electric
flux,” which we denote as ΦE . The electric field can therefore be thought of as the
number of lines per unit area.

E
In Fig. 18.23 shows Electric field lines passing through a surface of area A. 
 A
Consider the surface shown in Fig. 18.24. Let A = Anˆ be defined as the area vector
having a magnitude of the area of the surface, A, and pointing  in the normal
direction, n̂ . If the surface is placed in a uniform electric field E that points in the
same direction as n̂ , i.e., perpendicular to the surface A, the flux through the surface is Figure 18.23
  
ΦE= E.A= E.nA ˆ = EA

On the other hand,  if the electric field E makes an angle θ with n̂ , the electric flux
becomes Φ= E E.A
= E. Acos= θ En A
 
Where En = E.nˆ is the component of E perpendicular to the surface. 
A
Note that with the definition for the normal vector n̂ , the electric flux ΦE is positive if 
the electric field lines are leaving the surface, and negative if entering the surface.

E

In general, a surface S can be curved and the electric field E may vary over the surface.
We shall be interested in the case where the surface is closed. A closed surface is a
surface which completely encloses a volume. In order to compute the electric  flux, we
divide the surface into a large number of infinitesimal area elements ∆ Ai = ∆Ainˆ i , as Figure 18.24
shown in Fig. 18.25. Note that for a closed surface, the unit vector n̂i is chosen to point
in the outward normal direction.

Electric field is passing through an area element ∆ Ai , making an angle θ with the

Al 
El
normal of the surface.
   
The electric flux through ∆ Ai is ∆ΦE= Ei .∆ Ai= Ei ∆Ai cos θ
The total flux through the entire surface
 can be obtained by summing over all the
area elements. Taking the limit ∆ Ai → 0 and the number of elements to infinity, we
   
lim ∑ Ei .dAi ∫ E.dA
S
have ∆ΦE =
=
∆Ai →0
In order to evaluate the above
  integral, we must first specify the surface and then Figure 18.25
sum over the dot product E.dA. .

Let ∆ A1 = ∆A1rˆ be
An area element on the surface of a sphere S1 of radius r1 , as shown in Fig. 18.26.
The area element ∆A subtends a solid angle ∆Ω. The solid angle ∆Ω 
 A₂
subtended by ∆ A1 =
∆A1rˆ at the center of the sphere is defined as 
∆A1  E₂
∆Ω ≡ r₂
r12 
r₁ EL S2
Solid angles are dimensionless quantities measured in steradians (sr). Since An
Q
the surface area of the sphere S1 is 4 πr12 , the total solid angle subtended by
the sphere is

4 πr12
Ω= = 4π Figure 18.26
r12

In Fig. 18.26, the area element ∆ A2 makes an angle θ with the radial unit vector r̂ , then the solid angle subtended
by ∆A2 is
1 8 . 1 4 | Electric Charges, Forces and Fields


∆ A2 .rˆ ∆A2 cos θ ∆A2n
=
∆Ω = =
r22 r22 r22


Illustration 5: A non - uniform electric field given by
= E 3.0xiˆ + 4.0ˆj pierces the Gaussian cube shown in Fig. 18.28
(E is in newton per coulomb and x is in meters.) What is the electric flux through the right face, the left face, and
the top face?  (JEE ADVANCED)
 
Sol: We can find the flux through the surface by integrating the scalar product E.dA over each face.

Right face: An area vector A is always  perpendicular to its surface and always points away from the interior of a
Gaussian surface. Thus, the vector dA for the right face of the cube must point in the positive direction of the x
axis. In unit-vector notation,

dA = dAi. ˆ The flux Φ , through the right face is then
 
(
= ∫ 3.0xiˆ + =
Φ ∫ E.dA
= )( )
4.0ˆj . dA ˆi ∫ (3.0x )( dA ) ˆi ⋅ ˆi + ( 4.0 )( dA )=  ∫ ( 3.0xdA=
ˆj ⋅ ˆi  + 0 ) 3.0 ∫ xdA.

We are about to integrate over the right face, but we note that x has the same value everywhere on that face-
namely, x=3.0m. This means we can substitute that constant value for x. Then
= ∫ (3.0 ) dA 9.0∫ dA.
Φr 3.0=

The integral ∫ dA merely gives us the area A=4.0 m2 of the right face; so
=Φr ( 9.0N
= / C ) ( 4.0m2 ) 36N.m2 / C . 
y
Left face: The procedure for finding the flux through the left face
is the same as that for the rightface. However, two factors change. Gaussian
(i) The differential areavector dA points in the negative direction of surface

the x axis, and thus dA = −dAi. ˆ (ii) The term x again appears in our
integration, and it is again constant over the face being considered. X
However, on the left face, x=1.0m. With these two changes, we find
that the flux Φl through the left face is
z x=1.0m x=3.0m
Φ1 =−12N ⋅ m2 / C.
Figure 18.27

Top face: The differential area vector
 dA points in the positive
ˆ The flux Φ through the
direction of the y axis, and thus dA = +dAj. l
top face is then

=
Φ1 ∫ (3.0xiˆ + =
4.0ˆj ) ⋅ ( dAjˆ ) ∫ ( 3.0x )( dA ) ˆi ⋅ ˆj + ( 4.0 )( dA ) ˆj.ˆj =
  ∫ ( 0 + 4.0dA ) =
4.0 ∫=
dA 16 N ⋅ m2 / C .

15. GAUSS’ LAW 


dA E
Consider a positive point charge Q located at the center of a sphere of radius r, as 

shown in Fig. 18.28. The electric field due to the charge Q= is E Q / 4 πε0r 2 rˆ , ( ) r
which points in the radial direction. We enclose the charge by an imaginary sphere
of radius r called the “Gaussian surface”.
A spherical Gaussian surface enclosing a charge Q. Gaussian
surface

Figure 18.28
P hysi cs | 18.15

Inspherical coordinates, a small surface area element on the sphere is given by


z
= r 2 sin θdθdφrˆ 
dA

A small area element on the surface of a sphere of radius


  r.  1 Q 2
dΦE = E ⋅dA = EdA = 
Thus the net electric flux through the area element is  4 πε r 2 
 ο 
(
r sin θdθdφ ) V²sindd

   1 Q 2 Q rd
dΦE = E ⋅dA = EdA = 
 4 πε r 2 
 ο 
(
r sin θd=
θdφ
4 πεο) sin θdθdφ a2 + b2


Q y
=The totalsinflux dφ a2 +the
θdθthrough b2 entire surface is
4 πεο

  Q z 2π Q
Φ=
E ∫∫ ⋅dA
 E =
4 πεο ∫ο sin θdθ∫ο d=
φ
εο

s x
Figure 18.29

The same result can also be obtained by noting that a sphere of radius r has
a surface area A= 4 πr 2 , and since the magnitude of the electric field at any 
E
point on the spherical surface =is E Q / 4 πεοr 2 , the electric flux through the
surface is
   1 Q Q
2
ΦE = ∫∫ E ⋅ dA= E
∫∫ dA= EA=  4πε r2 4πr = ε S₃ S₂ S₁
s s  ο  ο + Q

In the above, we have chosen a sphere to be the Gaussian surface. However,


it turns out that the shape of the closed surface can be arbitrarily chosen. For
the surfaces shown in Fig. 18.30, the same result ( ΦE= Q / εο ) is obtained.
Whether the choice is S1 ,S2 or S3 .
The statement that the net flux through any closed surface is proportional Figure 18.30
to the net charge enclosed is known as Gauss’s law. Mathematically, Gauss’s
law is expressed as
  q
∫∫ dA ε (Gauss’s law)
ene
=ΦE  E
=
s ο
Where qene is the net charge inside the surface. One way to explain why Gauss’s law holds is that the number
of field lines that leave the charge is independent of the shape of the imaginary Gaussian surface we choose to
enclose the charge.

Illustration 6: Fig. 18.31 shows five charged lumps of plastic and an electrically
neutral coin. The cross section of a Gaussian surface S is indicated. What is the s
net electric flux through the surface if q1 = q4 = +3.1nC, q2 = q5 = −5.9nC, +
and q3 = −3.1nC? Five plastic objects, each with an electric charge, and a coin, q¹
which has no net charge. A Gaussian surface, shown in cross section, encloses -
three of the charged objects and the coin.  (JEE MAIN) q³
+ q²
- -
Sol: In Gauss’s law, only enclosed charges used to calculate the flux. q⁴
q⁵
The net flux Φ through the surface depends on the net charge qenc enclosed
by surface S. Figure 18.31

The coin does not contribute to Φ because it is neutral and thus contains equal amounts of positive and negative
charge. Charges q4 and q5 do not contribute because they are outside surface S. Thus, qenc is q1 + q2 + q3 and
gives us
qenc q1 + q2 + q3 +3.1 × 10−9 C − 5.9 × 10−9 C − 3.1 × 10−9 C
=
Φ = = = −670N.m2 / C.
εο εο 8.85 × 10−12 C2 / N ⋅ m2
1 8 . 1 6 | Electric Charges, Forces and Fields

Conclude: The minus sign shows that the net flux through the surface is inward and thus that the net charge within
the surface is negative.

Illustration 7: Find the flux through the disk shown in Fig. 18.32. The line joining the charge to the center of the
disk is perpendicular to the disk. (JEE MAIN)

Sol: The electric flux through the disk cannot be found by the equation
 
φ = E ⋅ A = EA cos θ If we wish to use the basic formula, we can divide the x

disk into small rings as shown in Fig. 18.33 and find the electric field due q 4
to charge at all the rings: R
3

φ = ∫ E.ds. Here we divide the entire disk into thin ring and find the flux
due to the charge through the thin ring. Figure 18.32

the electric field due to the point charge at the location of the ring shown is given by
kq
E=
(16 / 9 ) R2 + x2 .
E
As we discussed before, the area of the ring is 2πxdx . But dx 
the electric field is not normal to the ring. The angle can be
found as shown:
x
4R / 3   
cos θ = , φ= ∫ E ⋅ds ,
( )
16R 2 / 9 + x2
q 4
3
R

0.75R kq × ( 4R / 3) × 2πxdx q
φ ∫0= .

 2
(
2


)2 2
x + 16R / 9  × x + 16R / 9 ( ) 10εο Figure 18.33

Illustration 8: An infinitely long rod of negligible radius has a uniform 


E
charge density λ . Calculate the electric field at a distance r from the wire.
 (JEE MAIN)

Sol: We shall solve the problem by following the steps outlined above. 
E=O



(a) An infinitely long rod possesses cylindrical symmetry. a
(b) The charge density is uniformly distributed throughout the length,

and the electric field E must point radially away from the symmetry
axis of the rod (Fig. 18.34). The magnitude of the electric field is Figure 18.34
constant on cylindrical surface of radius r. Therefore, we choose a
coaxial cylinder as our Gaussian surface. 
E3
(c) Field lines for an infinite uniformly charged rod (the symmetry axis  
dA3 E2
of the rod and the Gaussian cylinder are perpendicular to plane of
the page.)
s3 
(d) The amount of charge enclosed by the Gaussian surface, a cylinder r dA2
of radius r and length  (Fig. 18.35), is qenc = λ . +++ +++
(e) As indicated in Fig. 18.36, the Gaussian surface consists of three

dA1 s1 s2
parts: a two ends S1 and S2 plus the curved side wall S3. The flux
through the Gaussian surface is 
Gaussian
        
E1 ⋅ dA1 + E2 ⋅ dA2 + E3 ⋅ dA3 = 0 + 0 + E3 A3 = E ( 2π r  ) E1 surface
Φ=E ∫∫

E.dA
= ∫∫ ∫∫ ∫∫
S S1 S2 S3 Figure 18.35
P hysi cs | 18.17

Where we have set E3 = E . As can be seen from the  Fig. 18.35,


 no flux
passes through the ends since the area vectors dA1 . and dA2 are perpendicular
to the electric field which points in the radial direction.  E
λ
(f) Applying Gauss’s Law gives E ( 2πr ) = λ / εο , or E = r
r
2πεοr
The result is in complete agreement with that obtained in equation
Figure 18.36
using Coulomb’s law. Notice that the result is independent of the
length  of the cylinder, and only depends on the inverse of the
distance r from the symmetry axis. The qualitative behavior of E as a
function of r is plotted in Fig. 18.36.

Illustration 9: Consider an infinitely large non-conduction plane in the xy-plane with uniform surface charge
density σ . Determine the electric field everywhere in space.  (JEE MAIN) 
z
E y

Sol: (i) An infinitely large plane possesses a planar symmetry.


(ii) Since
 the charge is uniformly distributed on the surface, the electric
field E must point perpendicularly away from the plane, E = Ekˆ . The
magnitude of the electric field is constant on planes parallel to the non-
conducting plane. x
We choose our Gaussian surface to be a cylinder, which is often referred 
E
to as a “pillbox”
The pillbox also consists of three parts: two end-caps S1 and S2, and a
Figure 18.37
curved side S3.
(ii) Since the surface charge distribution is uniform, the charge enclosed 
E1
by the Gaussian “pillbox” is qenc = σA , where A=A1=A2 is the area of the 
dA1 
end-caps. E3

(iv) The total flux through the Gaussian pillbox flux is Gaussian pillbox
s1

        s3 dA3
Φ=E ∫∫ E ⋅ dA= 
 ∫∫ E1 ⋅ dA1 + 
∫∫ E2 ⋅ dA2 + 
∫∫ E3 ⋅ dA3 + + + + + + + +
S S1 S2 S3 + + + + + + + + + +
= E1 A1 + E2 A2 + =
0 (E1 + E2 ) A + + + + + + + + + + +

Since the two ends are at the same distance from the plane, by symmetry,
s2

the magnitude of the electric field must be the same: E=


1 E=2 E . Hence, 
the total flux can be rewritten as dA2

E2
ΦE =2EA
Figure 18.38
qenc σA
(v) By applying Gauss’s law, we obtain 2EA = = Which gives
σ εο εο
E=
2εο
 σ
k̂, z > 0
  2ε Ez
In unit-vector notation, we have E =  ο

− σ k̂, z < 0

2o
 2εο

Thus, we see that the electric field due to an infinite large non-conducting
z

plane is uniform in space. The result, plotted in Fig. 18.39, is the same
as that obtained using Coulomb’s law. Note again the discontinuity in - 
2o
electric field as we cross the plane:
Figure 18.39
σ  σ  σ
∆Ez= Ez + − Ez −= − − =
2εο  2εο  εο
1 8 . 1 8 | Electric Charges, Forces and Fields

Illustration 10: A thin spherical shell of radius a has a charge +Q evenly


distributed over its surface. Find the electric field both inside and outside
the shell.  (JEE MAIN)

E

Sol: Apply Gauss’s law, as the charge distribution is symmetric.


The charge distribution is spherically symmetric, with a surface charge 
E=O


density
= σ Q /= As Q / 4 πa2 , where As = 4 πa2 is the surface area of
the sphere. The electric field E must be radially symmetric and directed
a
outward (Fig. 18.40). We treat the regions r ≤ a and r ≥ a separately.
Electric field for uniform spherical shell of charge

Case 1: r ≤ a We choose our Gaussian surface to be a sphere of radius


Figure 18.40
r ≤ a , as shown in Fig. 18.41 (a).
The charge enclosed by the Gaussian surface is qenc = 0 since all the
charge is located on the surface of the shell. Thus, from Gauss’s law, a r
ΦE qenc / εο , we conclude E=0, r<a
=

Case 2: r ≥ a In this case, the Gaussian surface is a sphere of radius r ≥ a Gaussian


, as shown in Fig. 18.42 (b). Since the radius of the “Gaussian sphere” is sufface
greater than the radius of the spherical shell, all the charge is enclosed:
(a)
qenc = Q

 
Since the flux through the Gaussian surface is ΦE = ∫∫ E ⋅ dA=

S
(
EA= E 4 πr 2 )
Q Q
By applying Gauss’s law, we obtain
= E = ke , r ≥ a a
2
4 πεοr r2 Gaussian

Note that the field outside the sphere is the same as if all the charges r sufface

were concentrated at the center of the sphere. The qualitative behavior


of E as a function of r is plotted in Fig. 18.42 showing electric field as a (b)
function of r due to a uniformly charged spherical shell. Figure 18.41
As in the case of a non-conducting charged plane, we again see a
discontinuity in E as we cross the boundary at r=a. The change, from
E
outer to the inner surface, is given by
ke Q
E= 2
Q σ r
∆E = E+ − E− = −0=
2 εο
4 πε a
ο

Illustration 11: Non-Conducting Solid Sphere Figure 18.42

An electric charge +Q is uniformly distributed throughout a non-conducting solid sphere of radius a. Determine
the electric field everywhere inside and outside the sphere.  (JEE MAIN)

Sol: For non-conducting object. Charge distributed thoughout the mass.


The charge distribution is spherically symmetric with the charge density given by
Q Q
ρ= =
V ( 4 / 3) πa3

Where V is the volume of the sphere. In this case, the electric field E is radially
symmetric and directed outward. The magnitude of the electric field is constant on
spherical surfaces of radius r. The regions r ≤ aandr ≥ a shall be studied separately.

Case 1: r ≤ a
P hysi cs | 18.19

We choose our Gaussian surface to be a sphere of radius r ≤ a , as shown in


Fig. 18.41 (a).
Fig. 18.41 (b) shows Gaussian surface for uniformly charged solid sphere, for (a)
r
r ≤ a , and (b) r > a .
 
( )
∫∫ E ⋅dA= EA= E 4πr With uniform
The flux through the Gaussian surface is ΦE = 
S
2 a

4   r3 
∫ dV =
charge distribution, the charge enclosed is qenc =ρ ρ  πr3  =
ρV = Q 
2   a3  Gaussian suface
V  
Figure 18.43
Which is proportional to the volume enclosed by the Gaussian surface. Applying
Gauss’s law

= (
ΦE qenc / εο , we obtain E 4 πr 2= ) ρ 4 3
 πr  or=
εο  3 
E =
ρr Qr
3εο 4 πε a3
r≤a
ο 
Case 2: r ≥ a E

In this case, our Gaussian surface is a sphere of radius r ≥ a , as shown in Fig. 18.44
. Since the radius of the Gaussian surface is greater than the radius of the sphere

all the charge is enclosed in our Gaussian surface: qenc = Q . With the electric flux Gaussian surface
through the Gaussian surface given by ΦE= E 4 πr ( 2
) , upon applying Gauss’s law, Figure 18.44

we obtain
E
( )
E 4 πr 2 = Q / εο=
, or E
Q
= ke ,
4 πεοr 2
Q
r2
r>a
E=
keQ
2
r
The field outside the sphere is the same as if all the charges were concentrated
at the center of the sphere. The qualitative behavior of E as a function of r is
r
plotted in Fig. 18.45. a

Figure 18.45

PROBLEM-SOLVING TACTICS
The following steps may be useful when applying Gauss’s law:
(a) Identify the symmetry associated with the charge distribution.
(b) Determine the direction of the electric field, and a “Gaussian surface” on which the magnitude of the electric
field is constant over portions of the surface.
(c) Divide the space into different regions associated with the charge distribution. For each region, calculate qenc
, the charge enclosed by the Gaussian surface.
(d) Calculate the electric flux ΦE through the Gaussian surface for each region.
(e) Equate ΦE with qenc / εο , and deduce the magnitude of the electric field.
In this chapter, we have discussed how electric field can be calculated for both the discrete and continuous charge
 1 qi
distributions. For the former, we apply the superposition principle: E = ∑ rˆ
4 πεο i r 2 i
i
 1 dq
For the latter, we must evaluate the vector integral E = ∫ rˆ
4 πεο r 2
1 8 . 2 0 | Electric Charges, Forces and Fields

Where r is the distance from dq to the field point P and r̂ is the corresponding unit vector. To complete the
integration, we shall follow the procedure outlined below:
 1 dq
(a) Start with dE = rˆ
4 πεο r 2
 λd (length)

(b) Rewrite the charge element dq as dq= σdA ( area)

 ρdV ( volume )

Depending on whether the charge is distributed over a length, an area, or a volume.



(c) Substituting dq into the expression for dE .
(d) Specify an appropriate coordinate system (Cartesian, cylindrical or spherical) and express the differential
element ( d ,dA or dV ) and r in terms of the coordinates (see table below for summary.)

Cartesian (x,y,z) (
Cylindrical ρ, φ, z ) (
Spherical r, θ, φ )
Dl dx, dy, dz
dρ, ρdφ, dz dr,rdθ,r sin θdφ
dA dxdy, dydz, dzdx
dρ dz, ρdφdz, ρdφdρ rdrdθ,r sin θdrdφ,r 2 sin θdθdφ
dV dxdydz
ρd ρdφdz r 2 sin θ dr d θ d φ

Differential elements of length, area and volume in different coordinates



(a) Rewrite dE in terms of the integration variable(s), and apply symmetry argument to identify non-vanishing
component(s) of the electric field.

(b) Complete the integration to obtain E .
In the Table below we illustrate how the above methodologies can be utilized to compute the electric field for an
infinite line charge, a ring of charge and a uniformly charged disk.

Line charge Ring of charge Uniformly charged disk

y  z 
dE dE

P dE P
P
r dp
dq
z r z
y
(1) Figure r’ y
R y
‘ R r’
x’ dr’
x dq
x
o dx’
L
Figure 18.46 Figure 18.47 x
Figure 18.48

(2) Express dq in
terms of charge dq = λdx' dq = λd dq = σdA
density
(3) write down dE λdx' λdl σdA
dE = k e dE = k e dE = k e
'2 2
r r r2
P hysi cs | 18.21

Line charge Ring of charge Uniformly charged disk


(4) Rewrite r and
dx' d=
 Rdφ ' dA = 2πr 'dr '
the differential
element in terms y z z
cos θ = cos θ = cos θ =
of the appropriate r' r r
coordinates
r'
= x'2 + y 2 r r'2 + z 2
=r R 2 + z2 =
(5) Apply symmetry
argument dEy dEcos θ
= dEy dEcos θ
= dEy dEcos θ
=
to identify
non-vanishing λydx' λRzdφ ' 2πσzr 'dr '
= ke = ke = ke
(x )
3/2
component(s) of dE
(r ' + z )
3/2
(R )
'2 2 3/2
+y 2
+ z2
2 2

(6) Integrate to
get E
+  /2 dx Rλz R r 'dr '
Ey= k e λy ∫ = Ez k e ∫ dφ ' Ez = 2πσk e z ∫
( ) ( ) (r )
−  /2 3/2 3/2 0 3/2
x2 + y 2 R 2 + z2 '2
+ z2

2k e λ /2 == k e
( 2πRλ ) z z
= 2πσk e  −
z 

= z 
(R )
3/2
y 2
+ z2  z + R2
2

(  / 2)
2
+ y2
Qz
ke
(R )
3/2
2
+ z2

Uniformly Charged solid


System Infinite line of charge Infinite plane of charge
sphere

+ + + + + + +
+ + + + + + +
+++++++++++++ + + + + + + +
+ + + + + + + a
Figure Figure 18.49 + + + + + + +
+ + + + + + +
+ + + + + + +

Figure 18.50
Figure 18.51

Identify the symmetry Cylindrical Planar Spherical


Determine the z

direction of E E 
E y 
E

+++++++++++++

x

E

Figure 18.52 Figure 18.54


Figure 18.53

Divide the space into r>0 Z >0 and z<0


different regions r ≤ a and r ≥ a
1 8 . 2 2 | Electric Charges, Forces and Fields

Choose Gaussian  
E1
surface E3
 

dA1 
dA3 E2 E3


s1 r
s3 Gaussian pillbox
r dA2 
s3 dA3 Gaussian
+++ +++ + + + + + + + + shere
 + + + + + + + + + + a
dA1 s1 s2 + + + + + + + + + + +

Gaussian s2 Concentric shpere

E1 surface

dA2 Figure 18.57

Figure 18.55 E2

Figure 18.56

Calculate electric flux


ΦE = E(2πrl) ΦE = EA + EA= 2EA ΦE = E(4 πr 2 )

Calculate enclosed  (r /a)3


charge qin qenc = λl qenc = σA qenc = Q r≤a
Q r≥a

Apply Gauss’s law


 Qr
Φ=E qin / ε0 to find  , r≤a
λ σ 3
E E= E=  4 πε0 a
2πε0r 2ε0 E=
 Q r≥a
 4 πε r 2
 0

FORMULAE SHEET

Electric Charges, Forces and Fields

S. No Term Description

Charge Charges are of two types


1 (a) Positive charge (b) Negative charge
Like charges repel each other and unlike charges attract each other.
2 Properties of charge 1. Quantization:-q=ne where n=0, 1, 2…… and e is charge of an electron.

∑q
2. Additive: −qnet =
3. Conservation: - total charge of an isolated system is constant
3 Coulomb’s law
The mutual electrostatic force between the charges q1 and q2 separated by a
distance r is given by Force on the charge q1 F = Kq q r / r 2
1 1 2 12

Where r12 is the unit vector in the direction from q2 and q1 .
For more than two charges in the system, the force acting on any charge is
vector sum of the coulomb force from each of the other charges. This is called
principle of superposition for q1 , q2 , q3 ….. qn Charges are present in the system.
P hysi cs | 18.23

S. No Term Description

4 Electric Field -The region around a particular charge in which its electrical effects can be
observed is called the electric field of the charge
-Electric field has its own existence and is present even if there is no charge to
experience the electric force.
5 Electric field
Intensity E=F/ q0 Where F is the electric force experienced by the test charge q0 at this
point. It is a vector quantity.
Some points to note on this
1. Electric field lines extend away from the positive charge and towards the
negative charge.
2. Electric field produces the force so if a charge q is placed in the electric field
E, the force experienced by the charge is F=qE
3. Principle of superposition also applies to electric field so
E = E1 + E2 + E3 + E4 + .....
 KQ r
Electric field intensity due to point charge E =
r2
Where r is the distance from the point charge and r is the unit vector along the
direction from source to point.

Electric Field Intensities due to various Charge Distributions

Name/Type Formula Note Graph

Point Charge •• q is source charge



•• r is vector drawn from source
Kq  Kq 
r= r charge to the test point. E
2 r3
r
r

Infinitely long line λ is linear charge density


charge ••
λ  2Kλ  (assumed uniform) E
r= r
2πε0r r •• r is perpendicular distance of
point from line charge r

•• r is radial unit vector drawn from
the charge to test point

Uniformly Charged Ring •• Q is total charge of the ring E

•• x=distance of point on the axis Emax


KQx from centre of the ring.
E=
(R 2 + X 2 )3/2 R r
•• Electric field is always along the 2
Ecentre = 0
axis.
1 8 . 2 4 | Electric Charges, Forces and Fields

Infinitely large non- σ is surface charge density


•• E
conducting thin sheet (assumed uniform)
σ
n̂  /20
2ε0 •• n is unit normal vector
•• Electric field intensity is r
independent of distance

Infinitely large charged σ is surface charge density


•• E
conducting sheet (assumed uniform)
σ
n̂  /0
ε0 •• n is unit normal vector
•• Electric field intensity is r
independent of distance

Uniformly charged (i) for r ≥ R •• R is radius of the sphere


hollow conducting/non  E
 KQ •• r is vector drawn from centre of
conducting sphere or 2
E=
2
rˆ the sphere to the test point. KQ/R
solid conducting sphere r
•• Sphere acts like a point charge
placed at the centre for point R r
(ii) for r<R
 outside the sphere.
E=0 
•• E is always along radial direction.
•• Q is total charge ( =σ 4 πR 2 ).
( σ = Surface charge density)

Uniformly charged solid 


(i) for r ≥ R •• r is vector drawn from centre of
non conducting sphere the sphere to the test point.
E
(insulating material)  KQ 2
E= rˆ •• Sphere acts like a point charge
KQ/R
2
r
placed at the centre for points
outside the sphere. R r
(ii) for r ≤ R 
 KQ  •• E is always along radial direction
ρ 
=E = r r 4
R 3 3ε0 •• Q is total charge ( =ρ πR 3 ).
3
( ρ =volume charge density)
•• Inside the sphere E ∝ r
•• Outside the sphere E ∝ 1 / r 2

Note: (i) Net charge on a conductor remains only on the outer surface of a conductor.
(ii) On the surface of spherical conductors charge is uniformly distributed.
P hysi cs | 18.25

Solved Examples

JEE Main/Boards Sol: Formula based.


On giving electrons, body acquires-ve charge and to
Example 1: A block having mass m and charge –q is acquire a net charge of 4pc
resting on a frictionless plane at a distance L from fixed
large non-conducting infinite sheet of uniform charge q = 4 x10−12 C
density s as shown in figure. Discuss the motion of the e = 1.6 x10−19 C
block assuming that collision of the block with the sheet
q 4x10−12
is perfectly elastic. Is it SHM? ⇒n= = = 2.5 x107
e 1.6x10−19
2.5 x107 electrons will have to be given.
R
Sheet
Example 3: What is the value of charge on a body if it
m qE has an excess of 1.5 x 107 electrons?

Sol: Electrons are negatively charged


L
mg n = 1.5 x107 and the body has excess of electrons
⇒ it is- vely charged and charge on it is q=ne
Sol: Electric force produced by sheet will accelerate the 1.5 x107 x1.6 x10−19 C
⇒q=
block towards the sheet producing an acceleration.
⇒q=
2.4pC
Acceleration will be uniform because electric field E due
to the sheet is uniform
Example 4: When 1022 electrons are removed from a
F qE neutral metal sphere, what is the charge on the sphere?
a = = , where E =
σ / 2ε0
m m
Sol: Loss of electrons make a body positively charged.
As initially the block is at rest and acceleration is
constant, from second equation of motion, time taken On removing electrons, body acquires +ve charge and
by the block to reach the wall its value is
q=ne = 1022 x1.6 x10−19 =1600 coulomb.
1 2 2L 2mL 4mLε0
=L at i.e.,
= t = =
2 a aE aσ
Example 5: Calculate the coulomb force between two
As collision with the wall is perfectly elastic, the block α -particles separated by a distance of 3.2 x10−15 m.
will rebound with same speed and as now its motion
is opposite to the acceleration, it will come to rest after Sol: Charge on α -particle
travelling same distance L in same time t. After stopping
We have qα = 3.2 x10−19 C
+ 2e =
it will be again accelerated towards the wall and so the
block will execute oscillatory motion with ‘span’ L and  
time period. A B
3.2 x 10 ¹⁵m
2mL 4mLε0
T 2t
= = 2 = 2
aE aσ
However, as the restoring force F=qE is constant and
1 qα qα 3.2 x10−19 x 3.2 x10−19
not proportional to displacement x, the motion is not F= = 9 x109 x
4 πε0 r 2 3.2 x 3.2 x10−30
simple harmonic.
=90 N (repulsive)
Example 2: How many electrons must be given to a
neutral body so that it could acquire a charge of 4.0 pC? Example 6: Consider two identical spheres P and Q with
charge q on each. A third sphere R of the same size but
1 8 . 2 6 | Electric Charges, Forces and Fields

uncharged is successively brought in contact with the Let the charges divided on the two objects be q and Q-q
two spheres. What is the new force of repulsion between q(Q − q)
so that the force between them is f = K
P and Q? r2
dF
For maximum force, =0
Sol: Charge on two spheres will be equally divided on dq
two sphere each times on touching. d  q(Q − q) 
K =0
When R is kept in contact with R, charge q is equally dq  r2 
distributed between P and R.
q K d
Charge on P = ⇒ [q(Q − q)] =
0
2 r 2 dq
q
Charge on R = d
2 ⇒ [qQ − q2 ] =
0
dq
When R is kept in contact with Q, total charge will again
Q
be equally distributed. ⇒ Q − 2q = 0 ⇒ q =
2
q + (q / 2) 3q
Charge
= on Q =
2 4 i.e, the charge must be equally divided.
3q
Charge on R =
4 Example 9: Two identical point charges of magnitude
Initial force of repulsion between P and Q Q are kept at a distance r from each other. A third
point charge q is placed on the line joining the above
1  qxq  two charges, such that all the three charges are in
F(say)  
4 πε0  r 2  equilibrium. What is the sign, magnitude and position
of the third charge?
Final force of repulsion between P and Q
Sol: For equilibrium, net F on each charge = 0
 q 3q 
x Let identical charges Q be placed at A and B and
1 1 2 4  3
=F =  2  F another charge q is at a distance x from A so that it is in
4 πε0  r  8



 equilibrium.

Example 7: Compare the electrostatic force and +Q q +Q


gravitational force taking two protons. A B
r
x
Sol: Simply apply the formula for Gravitational and
Electrostatic force
2
1 e2 mp
Fe = ; Fg = G ; Mass of proton = 1.67×10-27kg ∴ Force on q due to charge at A in the + X direction
4 πε0 r 2
r2
1 Qq
e e = and force on a due to charge at B in the-X
4 πε0 x2
A B 1 Qq
r direction =
4 πε0 (r − x)2
mp mp
r
A B For equilibrium, these two forces must be equal i.e.,
Fe 1 1 r
⇒ 1.24 x1036
= = = or x If q was a negative charge, the
Fg x 2
(r − x)2 2
direction of force due to q at B would be in-X and at A

Example 8: A charge Q is to be divided on two objects. in +X direction.


What should be the value of the charges on the two
objects, so that the force between them can be But, if all the three charges are of same nature, there
maximum? would be repulsion between charges at A and B also.
Hence to have equilibrium among three charges, Q must
Sol: If a + b constant, than a x b is maximum when a = b.
be opposite of q so that force of attraction between Q
P hysi cs | 18.27

and q=force of repulsion between Q and q. JEE Advanced/Boards


Q2 Oq Qq
= =
i.e.. 4 πε r 2 4 πε r 2 r
2 Example 1: An arc of radius r subtends an angle θ at
the center with x-axis in a Cartesian coordinate system.
0 0
4 πε0  
2 A charge is distributed over the arc such that the linear
Q charge density is λ . Calculate the electric field at the
∴q =
4 origin.

Y
Example 10: A charge Q is uniformly distributed on
the circumference of a circular ring of radius a. Find the rd
intensity of electric field at a point at a distance x from
the center on the axis of ring. d

X
Sol: Consider a small part of the ring. All points on the 

r
dE
ring are symmetrical to any point on the axis of the ring.
Given situation is depicted in the figure. Consider an Sol: Consider small element on the arc as point charge
infinitesimal element at point A on the circumference of and then proceed by integrating for all such points.
the ring. Let charge on this element be dq. The The electric charge distributed on the portion of the

magnitude of the intensity of electric field dE at a point arc making an angle d φ is dQ= λrdφ . The electric field
P situated at a distance x from the center on its axis is,
produced due to this portion at the origin will be,
1 dq dq kλr.dφ 
= dE = k . dE = The electric field vector dE of this portion
4 πε0 AP2 (a + x2 )
2
r2
of the arc is indicated in the diagram.

A
dE has two components

kλrdφ  kλr.dφ 
a
dE cos  dEx = − cos φ i and dEy =− cos φ j
2 2
 p r r
X   kλ 0  kλ 
0
dE ∴ Ex = ∫ cos φdφ i =− [sin φ]0 iθ
dE sin  r 0 r
 kλ 
Its direction is from A to P. Now consider two ∴ Ex = − sin φ i (θ not φ)
r

components of dE (i) dE sin θ , parallel to the axis of  kλ  kλ 
Now, Ey= ∫ sin φdφ j= [cos φ]00 jθ
the ring and (ii) dE cos θ , parallel to the axis. r r
Here it is clear that in the vector sum of intensities due  kλ 
∴ Ey = [ (cos θ − 1) j ]
to all such elements taken all over the circumference, r
the dE sin θ components of the diametrically opposite  kλ  
elements will cancel each other as they are mutually ∴
= Ey [(1 − sin θ) i + (cos θ − 1) j]
r
opposite. Hence only dEcos θ components should be
considered for integration. ( î component is just - sin θ)
∴ The total intensity of electric field at point P, Example 2: Two small spheres each having mass m kg
and charge q coulomb are suspended from a point by
dq
OP dq x insulating threads each 1 metre long but of negligible
= ∫ dEcos θ = ∫ k E= k ∫
(a + x ) AP
2 2
(a + x2 )
2 1
mass. If θ is the angle each string makes with the
(a2 + x2 ) 2
x dxQ vertical when equilibrium has been attained, show that
=∴E k = ∫ dq
3 surface 3 =q2 (4mgl2 sin2 θ tan θ) 4 πε0.
(a2 + x2 )2 (a2 + x2 )2
Sol: Gravitational as well as electrostatic force act on
1 xQ each sphere.
=
4 πε0 3
(a2 + x2 ) 2 Consider two small spheres A and B each of mass
1 8 . 2 8 | Electric Charges, Forces and Fields

m kg and charge q coulomb. When the two spheres


are suspended from point S by two threads each of
length I, they repel each other and when equilibrium is
attained, each string makes an angle θ with the vertical q + = +10C
[See figure.]. O
S 5 cm
D C
10 cm
A B

Sol: Charge is symmetric to all faces of the cube, hence
by symmetry each face would have equal flux passing
l l though it.
Here, q =+10µC =10−5 C

x
T T O
qA qB
F A B F
0.16m
q O q
Consider that the charge q is at a distance of 5cm
mg mg from the square ABCD of each side 10 cm [figure]. The
square ABCD can be considered as one of the six faces
Each of the two spheres is acted upon by the following
of a cube of each side 10cm. Then, according to Gauss’s
three forces:
theorem, total electric flux through all the six faces of
q
(i) The electrostatic force of repulsion f directed away the cube, φ =
ε0
from each other.
(ii) The weight mg of the sphere acting vertically
downwards.
(iii) The tension T in the string directed towards point S. 
I x III - II
Since the two spheres are in equilibrium, the three
forces acting on a sphere can be represented by the
three sides of the ∆ AOS taken in order. For sphere A,
we have at equilibrium by Lami’s theorem
Obviously, the flux through the square ABCD will be
F mg T
= =  … (i) 1 1 q
OA SO AS =
φ x=
φ x
6 6 ε0
Here, OA =lsin θ ; SO =lcos θ And AB =2AO =2lsin θ
1 10−5
= = x 1.88 x105 N m2 C−1
1 qxq 1 q2 6 8.854x10−12
and F = ⋅ = ⋅
4 πε0 AB2 4 πε0 4l2 sin2 θ
Example 4: Two large thin metal plates are parallel and
OA
From equation (i), we have F = mgx close to each other as shown in the figure. On their
SO inner faces, the plates have surface charge densities
1 q2
lsin θ of opposite signs and of magnitude 17.0 × 10–22 Cm–2.
or ⋅ =mgx What is E (i) to the left of the plates, (ii) to the right of
4 πε0 4l2 sin2 θ lcos θ
the plates and (iii) between the plates?
or q2 (4mgl2 sin2 θ tan θ)4 πε0
=
Sol: Apply formula for Electric field intensity due to
Example 3: A point charge + 10 µ C is at a distance 5 charged plate.
cm directly above the center of a square of side 10 cm Here σ =17.0 x10−22 Cm−2
as shown in Fig. What is the magnitude of the electric
flux through the square? (i) To the left of plates: The region I is to the left of the
plates. Therefore, the electric field to the left of plates
is zero.
P hysi cs | 18.29

(ii) To the right of plates: The region II is to the right Example 7: Electric field in the above figure is directed
of the plates. Again, the electric field in the region II is along + x direction and given by =Ex 5Ax + 2B , where E
zero. is in NC−1 and x is in meter. A and B are constants with
dimensions.
(iii) Between the two plates, the electric field given by
Y
σ 17.0x10−22
E =
= = 1.92x10−10 NC−1
ε0 8.854x10−12
M N
Example 5: A parallel plate capacitor is to be designed
X

with a voltage rating 1 kV, using a material of dielectric -10 cm


constant 3 and dielectric strength about 107 Vm–1.
Z

(Dielectric strength is the maximum electric field a Taking A= 10NC−1 m−1 and B = 5 NC−1 , Calculate
material can tolerate without breakdown, i.e., without
(i) The electric flux through the cube.
starting to conduct electricity through partial ionization.)
For safety, we should like the field never to exceed, say (ii) Net charge enclosed within the cube.
10% of the dielectric strength. What minimum area of
the plates is required to have a capacitance of 50 pF? Sol: Vector rotation of area and Gauss’s Law for net
enclosed charged is applied.
Sol: Maximum field strength should be 10% of the
Ex 5Ax + 2B . The electric field at face
(i) Given =
dielectric strength of the material,.
M where x=0 is E1 = 2B The electric field at face N where
10% of the given field i.e. 107 Cm−1 x = 10cm=0.010m is E2 =5A x 0.10+ 2B= 0.5A+2B
Given E = 0.1 x107 Cm−1 The electric flux through face M is

dV V φ1 = E1 .S1 = E1S1 cos π = −E1S1
Using E = − i.e. E = , we get
dr r = −2Bxl2 where 1=10 cm= 0.01 m
V 1000
r
= = = 10−3 m The electric flux through face N
E 0.1 x107 
φ2 = E2 ⋅ S2 = E2S1 cos 0 = (0.5 A + 2B)l2
∈0 ∈r A
Using C = , we get Net electric flux φ = φ1 + φ2
d
−12 −3
Cd Cr (450 x10 )(10 )
=
=A = = 19 cm2 . = −2Bl2 + (0.5A + 2B)l2 = 0.5Al2
−12
∈0 ∈r ∈0 ∈r 8.854 x10 x 3
= 0.5
= x10 x (0.10)2 5 x10−2 Vm

Example 6: The electrostatic force on a small sphere (ii) If θ is net charge enclosed within the cube, then by
of charge 0.4 µ C due to another small sphere of 1
Gauss’s theorem φ = q
charge-0.8 µ C in air is 0.2 N. ε0
(i) What is the distance between the two spheres? φ = ε0 φ = 8.85 x10−12 x5 x10−2 C = 4.425 x10−13 C
(ii) What is the force on the second sphere due to the
first? Example 8: Four electric charges, +q,+q, -q and –q are
respectively placed on the vertices A, B, C and D of
Sol: Consider each sphere as a point change and apply square. The length of the square is a. Calculate the
Coulomb’s law. intensity of the resultant electric field at the center.
(i) Force on charge 1 due to charge 2 is given by the +q +q
relation
A B

qq O
F12 = 9 x109 1 2 EB+ED EA+EC
r2
F 0.2

E
⇒ r 2 = 12 = D C
(9 x10 )q1q2 (9 x10 )(0.8 x10−6 )(0.4 x10−6 )
9 9
-q -q
Sol: Apply Superposition of electrostatic forces.
i.e. r=0.12m
All the electric charges are equidistant from the center
(ii) F12 = 0.2 N, Attractive F21=F12.
= F12
1 8 . 3 0 | Electric Charges, Forces and Fields

O. If r is the distance of vertices from the center, we E is the resultant of E’ and E’’. It is evident from the
kq geometry of the figure that,
have, E=A E=
B E=C E=
D
r2 8k 2q2
The directions of these electric fields are as shown in E2 =E'2 + E''2 = Using
figure. r4
a q
If E’ is the resultant field of EB and ED r= ,E = 4 2k
2 a2
kq
E' = EB + ED = 2
r2

JEE Main/Boards

Exercise 1 which to which?)


(ii) Is there a transfer of mass from wool to polythene?
Q.1 Electrostatic force between two charges is called
central force. Why?
Q.11 Give two properties of electric lines of force.
Sketch them for an isolated positive charge.
Q.2 In Coulomb’s law, on what factors the value of
electrostatic force constant k depends?
Q.12 An infinite line charge produces a field of 9 x 10 4
N/C at a distance of 2 cm. Calculate the linear charge
Q.3 Define dielectric constant of a medium. density.

Q.4 Dielectric constant of water is 80. What is its Q.13 Calculate the Coulomb’s force between a proton
permittivity? and electron separated by 0.8 x 10−15 m.

Q.5 State the principle of superposition of forces in Q.14 If the distance between two equal point charges is
electrostatics. doubled and their individual charges are also doubled,
what would happen to the force between them?
Q.6 How many electrons must be removed from a
conductor, so that it acquires a charge of 3.5nC? Q.15 Which is bigger, a coulomb or charge on an
electron? How many electronic charge form one
Q.7 A point charge of 10−7 coulomb is situated at the coulomb of charge?
center of a cube of 1 m side. Calculate the electric flux
through its surface. Q.16 What is the amount of charge possessed by 1kg
of electrons? Given that mass of an electron is 9.1 x
Q.8 Find the electric flux through each face of a hollow 10−31 kg.
cube of side 10 cm, if a charge of 8.854 µ C is placed at
its center. Q.17 Four charges +q, ..+q,-q,-q are placed respectively
at the four corners of a square of side a. Find the
Q.9 What is the force between two small charged magnitude and direction of the electric field at the
spheres having charges of 2 x 10−7 C and 3 x 10−7 C center of the square.
placed 30 cm apart in air?
Q.18 Four point charges qA = 2 µC, qB
Q.10 A polythene piece rubbed with wool is found to =−5µC, qC = 2µC and qD =5µC are located at corners
have a negative charge of 3 x 10−7 C. of a square ABCD of side 10 cm. What is the force on a
charge of 1 µ C placed at the center of the square?
(i) Estimate the number of electrons transferred (from
P hysi cs | 18.31

Q.19 Two point charges qA = 3µC are


3 µC and qB = Q.27 Define electric flux, Write its S.I. unit, A charge q is
located 20 cm apart in vacuum. enclosed by a spherical surface of radius R. If the radius
is reduced to half, how would the electric flux through
(i) What is the electric field at the midpoint O of the line
the surface charge?
AB joining the two charges?
(ii) If a negative test charge of magnitude 1.5 x 10−9 C Q.28 A positive point charge (+q) is kept in the vicinity
is placed at this point, what is the force experienced by of an uncharged conducting plate. Sketch electric field
the test charge? lines originating from the point on to the surface of the
 plate.
Q.20 Consider a uniform electric field E=3 x 103 i N/C.
Derive the expression of the electric field at the surface
(i) What is the flux of this field through a square of 10
of a charged conductor.
cm on a side whose plane is parallel to the yz plane?
(ii) What is the flux through the same square if the
normal to its plane makes a 60o angle with the x-axis? Exercise 2
Single Correct Question
Q.21 A point charge +10 µ C is at 5 cm
a distance of 5 cm directly above
Q.1 A point charge 50µC is located in the XY plane
the center of a square of side 10  
at the point of position vector r= 2 i + 3 j what is the
m
cm, as shown in figure What is the 0  
10 c

magnitude of the electric flux electric field at the point of position vector= r 8i +5 j .
through the square? (A) 1200V/m (B) 0.04V/m
10 cm
(C) 900V/m (D) 4500V/m
Q.22 Show that the electric field at the surface of a
σ  Q.2 A point charge q is placed at origin. Let EA, EB and EC
charged conductor is given by E = n , where σ is the
 ε 0 be the electric field at three points A (1, 2, 3), B (1, 1,-1)
surface charge density and n is a unit vector normal to and C (2, 2, 2) due to charge q Then
the surface in the outward direction. [i] EA ⊥ EB [ii] EB |= 4 | EC | Select the correct alternative
(A) Only [i] is correct
Q.23 A copper atom consists of copper nucleus
surrounded by 29 electrons. The atomic weight of (B) (B) only [ii] is correct
copper is 63.5 g Let us now take two pieces of copper (C) Both [i] and [ii] are correct
weighing 10 g. Let us transfer one electron from one
piece. What will be the Coulomb force between the two (D) (D) both [i] and [ii] are wrong
pieces after the transfer of electrons, if they are 1 cm
apart? Avogadro number = 6 × 1023 C mol–1, charge on Q.3 Two identical point charges are placed at a
an electron = 1.6 × 10–19 C. separation of l. P is a point on the line joining the
charges, at a distance x from any one charge, The field
Q.24 Two fixed point charged 4Q and 2Q are separated at P is E. E is plotted against x for values of x from close
by a distance x. Where a third point charge q should be to zero to slightly less than l. Which of the following
placed for it to be in equilibrium? best represents the resulting curve?
E E
Q.25 It is required to hold four equal point charges +q (A) (B)
in equilibrium at the corners of a square. Find the point
charge that will do this, if placed at the center of the O L
x
O L
x
square.
E E
Q.26 Four point charges, each having a charge q are
(C)
placed on the four corners A, B, C and D of a regular L (D) L
pentagon ABCEDE. The distance of each corner from O
x
O
x
the center is a. Find the electric field at the center of
the pentagon.
1 8 . 3 2 | Electric Charges, Forces and Fields

Q.4 Four charges are arranged at the corners of a equilibrium of the particle will be stable
square ABCD, as shown. The force on a +ve charge kept R
at the center of the square is  (A) for all values of H (B) only if H>
A B 2
(A) Zero
+q -q
R R
(C) only if H < (D) only if H=
(B) Along diagonal AC 2 2
(C) Along diagonal BD
Q.9 Point P lies on the axis of a dipole. If the dipole is
-2q +2q
(D) Perpendicular to the side AB
rotated by 90o anti-clock wise, the electric field vector
C D

E at P will rotate by
(A) 90o Clock wise (B) 180o
Q.5 Two free positive charges 4q and q are a distance l
apart. What charge Q is needed to achieve equilibrium (C) 90o Anti clock wise (D) no ne
for the entire system and where should it be placed
from charge q ? Q.10 The Fig. shows
4 l the electric field lines in
(A) Q =
9
q(negative)at
3 the vicinity of two point q₁
charges. Which one of q₂
4 l l
(B) Q = q(positive)at (C) Q = q(positive)at the following statements
9 3 3 concerning this situation
l is true?
(D) Q = q(negative)at
3 (A) q1 is negative and q2 is positive

Q.6 A small particle of mass m and charge –q is placed (B) The magnitude of the ratio ( q2 / q1 ) is less than one
at point P on the axis of uniformly charged ring and (C) Both q1 and q2 have the same sign of charge
released. If R >> x, the particle will undergo oscillation
(D) The electric field is strongest midway between the
along the axis of symmetry with an angular frequency
charges.
that is equal to
Q
Q.11 Electric flux through a surface of area 100 m2
R
P lying in the xy plane is (in V-m) if E =
ˆi + 2ˆj + 3kˆ
X (A) 100 (B) 141.4
(C) 173.2 (D) 200
qQ qQx
(B)

(A)
4 πε0mR 3 4 πε0mR 4 Q.12 An infinite, uniformly
charged sheet with surface
qQ qQx charge density σ cuts through
(C) (D)
a spherical Gaussian surface of
3
R
4 πε0mR 4 πε0mR 4
radius R at a distance x from its X
center, as shown in the Fig.
Q.7 Which of the following is a volt: 18.80. The electric flux Φ
(A) Erg per cm through the Gaussian surface is

(B) Joule per coulomb πR 2 σ 2 2


(A) (B) 2π(R − x )
(C) Erg per ampere ε0 σ ε0

(D) Newton/(Coulomb x m2 ) (C)


π(R − x)2
(D)
π(R 2 − x2 )2 σ
σ ε0 ε0
Q.8 A charged particle having some mass is resting
in equilibrium at a height H above the center of a Q.13 Two identical small conducting spheres, having
uniformly charged non-conducting horizontal ring charges of opposite sign, attract each other with a
of radius R. The force of gravity acts downwards. The force of 0.108 N when separated by 0.5 m. The spheres
P hysi cs | 18.33

are connected by a conducting wire, which is then electric field. The lines of force follow the path(s) shown
removed, and thereafter, they repel each other with a in figure as (1996)
force of 0.036 N. The initial charges on the spheres are 1 1
(A) ± 5 x10 −6
C and  15 x10 −6
C 2 2

(B) ± 1.0 x10−6 C and  3.0 x10−6 C 3 3

(C) ± 2.0 x10−6 C and  6.0 x10−6 C


4 4

(A) 1 (B) 2 (C)3 (D) 4


(D) ± 0.5 x10−6 C and  1.5 x10−6 C

Q.6 An electron of mass me , initially at rest, moves


Previous Years’ Questions through a certain distance in a uniform electric field
in time t1 . A proton of mass mp , also initially at rest,
Q.1 An alpha particle of energy 5 MeV is scattered takes time t2 to move through an equal distance in this
through 180o by a fixed uranium nucleus. The distance uniform electric field. Neglecting the effect of gravity,
of closest approach is of the order of  (1981) the ratio t2 / t1 is nearly equal to.  (1997)
(A) l  (B) 10−10 cm (A) 1 (B) (mp / me )1/2

(C) 10−12 cm (D) 10−15 cm (C) (me / mp )1/2 (D) 1836

Q.2 Two equal negative charges – q are fixed at points Q.7 A non-conducting ring of radius 0.5 m carries a total
(0,-a) and (0,a) on y-axis. A positive charge Q is released charge of 1.11×10-10 C distributed non-uniformly on its
from rest at the point (2a, 0) on the x-axis. The charge Q
circumference producing an electric field E everywhere
will  (1984)  
in space. The value of the integral ∫ tt ==∞
0
−E ⋅ dl (l=0 being
(A) Execute simple harmonic motion about the origin
center of the ring) in volt is  (1997)
(B) Move to the origin and remain at rest
(A) +2 (B) -1 (C) -2 (D) zero
(C) Move to infinity
(D) Execute oscillatory but not simple harmonic motion Q.8 Three charges Q, +q and +q are placed at the
vertices of a right angle triangle (isosceles triangle) as
Q.3 A charge q is placed at the centre of the line joining shown. The net electrostatic energy of the configuration
two equal charges Q. The system of the three charges is zero, if Q is equal to  (2000)
will be in equilibrium if q is equal to  (1987) −q −2q
(A) (B) (C)-2q (D) +q
Q Q Q Q 1+ 2 2+ 2
(A) − (B) − (C) + (D) +
2 4 4 2
Q.9 Three positive charges of equal value q are placed
Q.4 The at the vertices of an equilateral triangle. The resulting
 magnitude of electric
Q

field E in the annular region of a lines of force should be sketched as in (2001)
charged cylindrical capacitor 
 (1996)
(B)
(A) Is same throughout (A)

(B) Is higher near the outer


cylinder than near the inner
-q +q
a
cylinder
(C) Varies as 1/r where r is the distance from the axis
(D) Varies as 1/ r 2 where r is the distance from the axis
(C)
(D)
Q.5 A metallic solid sphere is placed in a uniform
1 8 . 3 4 | Electric Charges, Forces and Fields

Q.10 A metallic shell has a point charge q kept inside its q2


cavity. Which one of the following diagrams correctly +q1
represent the electric lines of force? (2003)

-q1
(A) (B)
(A) Negative and distributed uniformly over the surface
of the sphere
(B) Negative and appears only at the point on the
sphere closest to the point charge

(C) (D) (C) Negative and distributed non-uniformly over the


entire surface of the sphere
(D) zero

Q.11 Six charges, three positive and three negative Q.15 A spherical portion has been
of equal magnitude are to be placed at the vertices removed from a solid sphere having a
of a regular hexagon such that the electric field at charge distributed uniformly in its
O is double the electric field when only one positive volume as shown in the figure. The
charge of same magnitude is placed at R. Which of the electric field inside the emptied space
following arrangements of charge is possible for, P, Q, is  (2007)
R, S, T and U respectively?  (2004)
(A) Zero everywhere (B) Non-zero and uniform
(A) +, -, +, -, -,+ (B) +, -, +, -, +, -
(C) Non-uniform (D) Zero only at its center
(C) +, +, -, +, -, - (D) -, +, +, -, +, -

a
Q.12 Consider the charge configur- P Q Q.16 A disk of radius having a uniformly distributed
4
ation and a spherical Gaussian charge 6C and 6C is placed in the x-y plane with its
surface as shown in the figure. U O R
When calculating the flux of the  −a 
center at  ,0,0  . A rod of length a carrying a
electric field over the spherical  2 
surface, the electric field will be due
S
uniformly distributed charge 8C is placed on the x-axis
T
to  (2004)
a 5a
(A) q2 (B) Only the positive charges from x = to x = . Two point charges -7C and 3C
4 4
(C) All the charges (D) + q1 and − q1  a −a   −3a 3a 
are placed at  , ,0  and  , ,0  . Respectively.
4 4   4 4 
Q.13 Three infinitely long charge sheets are placed as
Consider a cubical surface formed by six surfaces
shown in figure. The electric field at point P is (2005)
a a a
z x=
± ,y= ± . The electric flux through this
± ,z=
2 2 2
cubical surface is
 z =3a
P (2009)
-2 z =0
x y
- z =-a

2σ  2σ  4σ  4σ 
(A) k (B) − k (C) k (D) − k x
ε0 ε0 ε0 ε0

Q.14 Consider a neutral conducting sphere. A positive


point charge is placed outside the sphere. The net −2C 2C 10C 12C
(A) (B) (C) (D)
charge on the sphere is then (2007) ε0 ε0 ε0 ε0
P hysi cs | 18.35

Q.17 Three concentric metallic spherical shells of Q.21 In a uniformly charged sphere of total charge Q
radii R, 2R and 3R are given charges Q1 , Q2 and Q3 and radius R, the electric field E is plotted as a function
respectively. It is found that the surface charge densities of distance from the centre. The graph which would
on the outer surfaces of the shells are equal. Then the correspond to the above will be (2012)
ratio of the charges given to the shells, Q1 :Q2 : Q3 is E E
 (2009)
(A) (B)
(A) 1:2:3 (B) 1:3:5 (C) 1:4:9 (D) 1:8:18

Q.18 A tiny spherical oil drop carrying a net charge q is


R r R r

balanced in still air with a vertical uniform electric field


E E
81π
of strength x105 Vm−1 , When the field is switched
7
off, the drop is observed to fall with terminal velocity
(C) (D)

2 x10−3 ms−1 . Given g= 9.8ms−2 , viscosity of the air


=1.8 x 10−5 Ns m−2 and the density of oil = 900 kg m-3,
R r R r

the magnitude of q is (2010)


Q.22 Two charges, each equal to q, are kept at x = −a
(A) 1.6 x10-19C (B) 3.2 x10-19C and x = a on the x-axis. A particle of mass m and charge
q
q0 = is placed at the origin. If charge q0 is given a
(C) 4.8 x10-19C (D) 8.0 x10-19C 2
small displacement (y << a) along the y-axis, the net
force acting on the particle is proportional to:  (2013)
Q.19 A uniformly charged thin spherical shell of radius 1
R carries uniform surface charge density of σ per (A) – y (B)
y
unit area. It is made of two hemispherical shells, held
together by pressing them with force F (see figure). F is (C) −
1
(D) y
proportional to (2010) y

Q.23 A long cylindrical shell carries positive surface


F F charge σ in the upper half and negative surface charge
- σ in the lower half. The electric field lines around
the cylinder will look like figure given in: (figures are
schematic and not drawn to scale)  (2015)
1 2 2 1 2 1 σ2 1 σ2
(A) σR (B) σ R (C) (D)
ε0 ε0 ε0 R ε0 R 2
+
++ ++ +
++ ++

(A) (B)
+ +
 + +
Q.20 Consider an electric field E = E0 x , where E0 is a
+ + + +
- -
- - - -
-- --
- -

constant. The flux through the shaded area (as shown


-
--
-
--

in the figure) due to this field is  (2011)


z
+
(a,0,a) (a,a,a) ++ ++ +
++ ++
(C) (D)
+ + + +
+ + + +
-
- -
-
- -
--
-
--
-
-
--
-
--

y
(0,0,0) (0,a,0)

x
E0 a2
(A) 2E0 a2 (B) 2E0 a2 (C) E0 a2 (D)
2
1 8 . 3 6 | Electric Charges, Forces and Fields

JEE Advanced/Boards

Exercise 1 Q.7 A charge +Q is uniformly distributed over a thin


ring with radius R. A negative point charge –Q and
Q.1 A negative point charge 2q and a positive charge q mass m starts from rest at a point far away from the
are fixed at a distance l apart. Where should a positive center of the ring and moves towards the center. Find
test charge Q be placed on the line connecting the the velocity of this particle at the moment it passes
charge for it to be in equilibrium? What is the nature through the center of the ring.
of the equilibrium with respect to longitudinal motion?
Q.8 A point charge +q & mass 100 gm experiences a
Q.2 Draw E-r graph for 0<r<b, force of 100 N at a point at a distance 20 cm from a
if two point charges a & b are long infinite uniformly charged wire. If it is released find
located r distance apart, when its speed when it is at a distance 40 cm from wire

Q.9 consider the configuration of a system of four


0

charges each of value +q. Find the work done by


a b
(i) Both are + ve r
(ii) Bothe are – ve external agent in changing the configuration of the
system from figure (i) and figure (ii).
(iii) a is + ve and b is – ve +q
+q +q
(iv) a is – ve and b is + ve
a

Q.3 A clock face has negative charges –q, -2q, -3q, +q +q


…….., , -12q fixed at the position of the corresponding
+q +q
numerals on the dial. The clock hands do not disturb +q
the net field due to point charges. At what time does a
(I) (II)
the hour hand point in the same direction as electric
field at the center of the dial. Q.10 Two identical particles of mass m carry charge
Q each. Initially one is at rest on a smooth horizontal
Q.4 A charge + 10−9 C is located at the origin in free plane and the other is projected along the plane
space & another charge Q at (2, 0, 0). If the X-component directly towards the first from a large distance with an
of the electric field at (3, 1, 1) is zero, calculate the value initial speed V. find the closest distance of approach.
of Q. Is the Y-component zero at (3, 1, 1)? R

Q.5 A simple pendulum of length l and  a


bob mass m is hanging in front of a +
+
large non - conducting sheet having +
surface charge density σ . If suddenly a + Q
+
charge +q is given to the bob & it is + l Q.11 A particle of mass m and negative charge q is
released from the position shown in +
thrown in a gravity free space with speed u from the
figure. Find the maximum angle
+
+ point A on the large non conducting charged sheet
through which the string is deflected with surface charge density σ , as shown in figure. Find
from vertical. the maximum distance from A on sheet where the
particle can strike.
Q.6 A particle of mass m and charge –q moves along +
+
a diameter of a uniformly charged sphere of radius R +
and carrying a total charge + Q. Find the frequency of +
S.H.M. of the particle if the amplitude does not exceed + u
+ 
R. +
+
+ A

P hysi cs | 18.37

Q.12 The length of each side of a cubical closed surface Find the smallest value of the speed v such that the
is l. If charge q is situated on one of the vertices of the particle does not return to P.
cube, then find the flux passing through shaded face of
the cube. Q.18 2 small balls having the same mass & charge &
located on the same vertical at heights h1 & h2 are
thrown in the same direction along the horizontal at
the same velocity v. The 1st ball touches the ground
at a distance l from the initial vertical. At what height
q will the 2nd ball be at this instant? The air drag & the
charges induced should be neglected.
Q.13 A point charge Q is located on the axis of a disc
of radius R at a distance a from the plane of the disc. Q.19 Two identical balls of charges q1 & q2 initially have
If one fourth (1/4th) of the flux from the charge passes equal velocity of the same magnitude and direction.
through the disc, then find the relation between a & R. After a uniform electric field is applied for some time,
the direction of the velocity of the first ball changes
by 60o and the magnitude is reduced by half. The
Q.14 Two thin conducting shells of radii R and 3R are
direction of the velocity of the second ball changes by
shown in figure. The outer shell carries a charge +Q
90o . In what proportion will the velocity of the second
and the inner shell is neutral. The inner shell is earthed
ball changes?
with the help of switch S. find the charge attained by
the inner shell.
+Q Q.20 Small identical balls with equal charges are fixed
3R
at vertices of regular 2008- g on with side a. At a certain
R S instant, one of the balls is released & a sufficiently long
time interval later, the ball adjacent to the first released
ball is freed. The kinetic energies of the released balls
are found to differ by K at a sufficiently long distance
from the polygon. Determine the charge q of each part.
Q.15 Consider three identical metal spheres A, B and
C. spheres A carries charge + 6q and sphere B carries
Q.21 A non - conducting ring of mass m and radius R
charge -3q. Sphere C carries no charge. Spheres A and
is charged as shown. The charged density i.e. charge
B are touched together and then separated. Sphere C is
per unit length is λ . It is then placed on a rough non
then touched to sphere A and separated from it. Finally
- conducting horizontal surface plane. At time t=0, a
the sphere C is touched to sphere B and separated from
uniform electric field E = E0i is switched on and the ring
it. Find the final charge on the sphere C.
start rolling without sliding. Determine the frictional
force (magnitude and direction) acting on the ring,
Q.16 Six charges are placed at the vertices of a regular when it starts moving.
hexagon as shown in the figure. Find the electric field y
on the line passing through O and perpendicular to
the plane of the figure as a function of distance x from +
+
point O. +
+Q -Q + x
-
-
-
-
-Q +Q
O

+Q -Q Q.22 Find the electric field at the center of semicircular


ring shown in figure.
a

Q.17 A circular ring of radius R with uniform positive charge


Y
density λ per unit length is fixed in the Y-Z plane with its
center at the origin O. A particle of mass m and positive -q - - - + + q
-- +
charge q is projected from the point P ( 3 R,0,0) on the - +
- +
positive X-axis directly towards O, with initial velocity v. -
X
R
1 8 . 3 8 | Electric Charges, Forces and Fields

Q.23 Two concentric rings, one of radius ‘a’ and the Q.2 Select the correct statement: (Only force on a
other of radius ‘b’ have the charges +q and −(2 / 5)−3/2 particle is due to electric field)
q respectively as shown in the figure. Find the ratio
(A) A charged particle always moves along the electric
b/a if a charge particle placed on the axis at z=a is in
lines of force.
equilibrium.
qB=-(2/5)
-3/2
q (B) A charged particle may move along the line of force.
b
(C) A charged particle never moves along the line of
force.
a z=a
(D) A charged particle moves along the line of force
qA=+q
only if released from rest.

Q.24 A positive charge Q is uniformly distributed Q.3 A conducting sphere of radius r has a charge. Then
throughout the volume of a non - conducting sphere
of radius R. A point mass having charge +q and mass (A) The charge is uniformly distributed over its surface,
m is fired towards the center of the sphere with velocity if there is an external electric field.
v from a point at distance r(r > R) from the center of (B) Distribution of charge over its surface will be non-
the sphere. Find the minimum velocity v so that it can uniform if no external electric field exists in space.
penetrate R/2 distance of the sphere. Neglect any
resistance other than electric interaction. Charge on the (C) Electric field strength inside the sphere will be equal
small mass remain constant throughout the motion. to zero only when no external electric field exists.
(D) Potential at every point of the sphere must be same.
Q.25 A cavity of radius r is present inside a solid
dielectric sphere of radius R, having a volume charge Multiple Correct Choice Type
density of ρ . The distance between the centers of the
sphere and the cavity is a. An electron e is kept inside Q.4 Two fixed charges 4Q (positive) and Q (negative)
the cavity at an angle θ = 45o as shown. How long will are located at A and B, the distance AB being 3 m.
it take to touch the sphere again?
+ 4Q -Q

A 3m B

(A) The point P where the resultant field due to both is


e
r 
a zero is on AB outside AB.
(B) The point P where the resultant field due to both is
zero is on AB inside AB.
(C) If a positive charge is placed at P and displaced
slightly along AB it will execute oscillation
Exercise 2 (D) If a negative charge is placed at P and displaced
slightly along AB it will execute oscillations.
Single Correct Choice Type
Q.5 Three point charges Q, 4Q and 16Q are placed on
Q.1 Mid way between the two equal and similar charges, a straight line 9 cm long. Charges are placed in such
we placed the third equal and similar charge. Which of a way that the system has minimum potential energy.
the following statements is correct, concerning to the Then
equilibrium along the line joining the charges
(A) 4Q and 16Q must be at the ends and Q at a distance
(A) The third charge experienced a net force inclined to of 3 cm from the 16Q.
the line joining the charges.
(B) 4Q and 16Q must be at the ends and Q at a distance
(B) The third charge is in stable equilibrium. of 6 cm from the 16Q.
(C) The third charge is in unstable equilibrium. (C) Electric field at the position of Q is zero.
(D) The third charge experiences a net force Q
(D) Electric field at the position of Q is .
perpendicular to the line joining the charges. 4πε0
P hysi cs | 18.39

Q.6 Two infinite sheets of uniform charge density Qnet


+σ and − σ are parallel to each other as shown in the
enclosed
=
φE ∫ E.dA
=
ε0
Fig. 18.103, Electric field at the
Qnet enclosed
= 0 implies
= φE 0
+ -
+ -
+ + - -
+ - Q.11: Statement-I: In a given situation of arrangement
+ -
+ -
+ - of charges, an extra charge is placed outside the
+ - Q
Gaussian surface. In the Gauss Theorem ∫ E.ds = in Qin
(A) Points to the left or to the right of the sheets is zero. ε0
remains unchanged whereas electric field E at the site
(B) Midpoint between the sheets is zero.
of the element is changed.
(C) Midpoint of the sheets is σ / ε0 and is directed
towards right. Statement-II: Electric field E at any point on the
(D) Midpoint of the sheet is 2 σ / ε0 and is directed Gaussian surface is due to inside charge only.
towards right.
Q.12: Statement-I: The flux crossing through a closed
Q.7 A particle of mass m and charge q is thrown in surface is independent of the location of enclosed
a region where uniform gravitational field and electric charge.
field are present. The path of particle Statement-II: Upon the displacement of charges within
(A) May be a straight line (B) May be a circle a closed surface, the E at any point on surface does not
charge.
(C) May be a parabola (D) May be a hyperbola

Assertion Reasoning Type Previous Years’ Questions


(A) Statement-I is true, statement-II is true and
statement-II is correct explanation for statement-I Paragraph: (Q.1-Q.4) The nuclear charge (Ze) is non-
uniformly distributed within a nucleus of radius R.
(B) Statement-I is true, statement-II is true and The charge density p(r) (change per unit volume) is
statement-II is NOT the correct explanation for statement-I dependent only on the radial distance r from the center
(C) Statement-I is true, statement-II is false. of the nucleus as shown in figure, the electric field is
only along the radial direction.
(D) Statement-I is false, statement-II is true.
(r)
Q.8: Statement-I: A positive point charge initially
d
at rest in a uniform electric field starts moving along
electric lines of forces. (Neglect all other forces except
electric forces)
Statement-II: Electric lines of force represents path of r
charged particle which is released from rest in it. a R

Q.1 The electric field r=R is  (2008)


Q.9: Statement-I: For a non-uniformly charged thin
(A) Independent of a
circular ring with net charge zero, the electric potential
at each point on axis of the ring is zero. (B) Directly proportional to a
Statement-II: For a non-uniformly charged thin circular (C) Directly proportional to a2
ring with net charges zero, the electric field at any point (D) Inversely proportional to a
on axis of the ring is zero.

Q.2 For a=0, the value of d (maximum value of p as


Q.10: Statement-I: If a concentric spherical Gaussian
shown in the figure) is (2008)
surface is drawn inside this spherical shell of charge,
electric field (E) at each point of surface must be zero. 3Ze 3Ze 4Ze Ze
(A) (B) (C) (D)
3 3 3
tatement-II: In accordance with Gauss’s law 4 πR πR 3πR 3πR 3
1 8 . 4 0 | Electric Charges, Forces and Fields

Q.3 The electric field within the nucleus is generally Q.8 A spherical metal shell A of radius R A and a solid
observed to be linearly dependent on r. This implies metal sphere B of radius RB ( < R A ) are kept for apart
 (2008) and each is given charge +Q. Now they are connected
R 2R by a thin mental wire. Then (2011)
(A) a=0 (B) a = (C) a = R (D) a =
2 3
(A) Einside
A = 0 (B) Q A > QB
Q.4 Under the influence of the coulomb field of charge σA RB
(C) = (D) Eonsurface < EBonsurface
+Q, a charge –q is moving around it in an elliptical σB RA A

orbit. Find out the correct statement(s).  (2008)


(A) The angular momentum of the charge –q is constant Q.9 A cubical region of side a has its centre at the origin.
It encloses three fixed point charges , -q at (0, -a/4, 0), +
(B) The linear momentum of the charge –q is constant 3q at (0, 0, 0) and -q at (0, +a/4, 0). Choose the correct
(C) The angular velocity of the charge –q is constant option(s)  (2012)
(D) The linear speed of the charge –q is constant. (A) The net electric flux crossing the plane x = +a/2 is
equal to the net electric flux crossing the plane x = −a/2
Q.5 A positively charged thin metal ring of radius R is (B) The net electric flux crossing the plane y = +a/2 is
fixed in the x-y plane with its centre at the origin O. A more than the net electric flux crossing the plane y =
negatively charged particle P is released from rest at −a/2.
the point (0, 0, z 0 ) where z 0 >0. Then the motion of P is q
(C) The net electric flux crossing the entire region is
 (1998) ε0
(D) The net electric flux crossing the plane
(A) Periodic for all values of z 0 satisfying 0< z 0 < ∞ .
z = +a/2 is equal to the net electric flux crossing the
(B) Simple harmonic for all values of z 0 satisfying 0<
plane x = +a/2.
z 0 ≤ R.
(C) Approximately simple harmonic provided z 0 <<R. Q.10 An infinitely long solid cylinder of radius R has
(D) Such that P crosses O and continues to move along a uniform volume charge density ρ . It has a spherical
the negative z-axis towards z=- ∞ . cavity of radius R/2 with its centre on the axis of the
cylinder, as shown in the figure. The magnitude of the
electric field at the point P, which is at a distance 2R
Q.6 A non-conducting solid sphere of radius R is
from the axis of the cylinder, is given by the expression
uniformly charged. The magnitude of the electric field
due to the sphere at a distance r from its center.(1998) 23 ρ R
. The value of k is (2012)
16k ε0
(A) Increases as r increases for r<R z

(B) Decreases as r increases for 0 <r< ∞


(C) Decreases as r increases for R<r< ∞
(D) Is discontinuous at r=R R
R/2
P
Q.7 A few electric field lines for a y

system of two charges Q1 and 2R

Q2 fixed at two different points Q1 Q2


x
on the x-axis are shown in the
figure. These lines suggest that 
 (2010)
Q.11 Two non-conducting solid spheres of radii R and
(A) Q1 > Q2 2R, having uniform volume charge densities ρ1 and ρ2
respectively, touch each other. The net electric field at
(B) Q1 < Q2 a distance 2R from the centre of the smaller sphere,
(C) At a finite distances to the left Q1 the electric field along the line joining the centre of the spheres is zero.
is zero. The ratio ρ1 / ρ2 can be  (2013)
(D) At a finite distance to the right of Q2 the electric 32 32
(A) – 4 (B) − (C) (D) 4
field is zero. 25 25
P hysi cs | 18.41

Q. 12 Two non-conducting spheres of radii R1 and R2 Q.15 Four charges Q1, Q2, Q3 and Q4 of same magnitude
and carrying uniform volume charge densities + ρ and are fixed along the x axis at x = -2a, - a, +a and +2a,
− ρ , respectively, are placed such that they partially respectively. A positive charge q is placed on the
overlap, as shown in the figure. At all points in the positive y axis at a distance b > 0. Four options of the
overlapping region,  (2013) signs of these charges are given in List I. The direction
of the forces on the charge q is given in List II. Match
List I with List II and select the correct answer using the
 - code given below the lists. (2014)
R2
R1
List I List II
P. Q1, Q2, Q3, Q4 all positive 1. +x
(A) The electrostatic field is zero
Q. Q1, Q2 positive , Q3, Q4 negative 2. -x
(B) The electrostatic potential is constant
R. Q1, Q4 positive , Q2, Q3 negative 3. +y
(C) The electrostatic field is constant in magnitude
S. Q1, Q3 positive , Q2, Q4 negative 4. -y
(D) The electrostatic field has same direction
Codes:
Q.13 Let E1 (r ) , E2 (r ) and E3 (r ) be the respective (A) P-3, Q-1, R-4, S-2 (B) P-4, Q-2, R-3, S-1
electric fields at a distance r from a point charge Q, an
(C) P-3, Q-1, R-2, S-4 (D) P-4, Q-2, R-1, S-3
infinitely long wire with constant linear charge density
λ , and an infinite plane with uniform surface charge
1 ( r0 ) E=
2 ( r0 ) E3 ( r0 ) at a given distance
(+0, b)
density σ . If E=
q
r0 , then  (2014)
λ
(A) Q= 4 σ πr02 (B) r0 =
2πσ
(C) E1 (r0 / 2 ) = 2E2 (r0 / 2 ) (D) E2 (r0 / 2 ) = 4E3 (r0 / 2 ) Q1 Q2 Q3 Q4
(-2a, 0) (-a, 0) (+a, 0) (-2a, 0)

Q.14 Charges Q, 2Q and 4Q are uniformly distributed Q.16 The figures below depict two situations in which
in three dielectric solid spheres 1, 2 and 3 of radii R/2, two infinitely long static line charges of constant
R and 2R respectively, as shown in figure. If magnitudes positive line charge density λ are kept parallel to each
of the electric fields at point P at a distance R from other. In their resulting electric field, point charges
the centre of spheres 1, 2 and 3 are E1 ,E2 and E3 q and -q are kept in equilibrium between them. The
respectively, then (2014) point charges are confined to move in the x direction
(A) E1 > E2 > E3 (B) E3 > E1 > E2 only. If they are given a small displacement about their
equilibrium positions, then the correct statement(s) is
(C) E2 > E1 > E3 (D) E3 > E2 > E1 (are)  (2015)
P
P (A) Both charges execute simple harmonic motion.
R R
(B) Both charges will continue moving in the direction
2Q
Q
of their displacement.
R/2
(C) Charge +q executes simple harmonic motion while
charge - q continues moving in the direction of its
Sphere 1 Sphere 2
displacement.
(D) Charge -q executes simple harmonic motion while
charge +q continues moving in the direction of its
P
R
displacement.
4Q
Q.17 Consider a uniform spherical charge distribution
of radius R1 centred at the origin O. In this distribution,
2R
a spherical cavity of radius R2, centred at P with distance
OP= a= R1 − R 2 (see figure) is made. If the electric field
Sphere 3
1 8 . 4 2 | Electric Charges, Forces and Fields

   
inside the cavity at position r is E ( r ) , then the correct (C) E is uniform, its magnitude is independent of a but

statement(s) is(are)  (2015) its direction depends on a

(D) E is uniform and both its magnitude and direction

    depend on a

x x
+q -q
R1
O

(A) E is uniform, its magnitude is independent of R 2
 
but its direction depends on r (B) E is uniform, its
magnitude depends on R 2 and its direction depends

on r

PlancEssential Questions
JEE Main/Boards JEE Advanced/Boards
Exercise 1 Exercise 1
Q. 17 Q.18 Q.19 Q.4 Q.20 Q.23
Q.23 Q.24 Q.25

Exercise 2 Exercise 2
Q. 1 Q.3 Q.6

Previous Years’ Questions Previous Years’ Questions


Q.7 Q.8 Q.11 Q.1 Q.2 Q.3
Q.4 Q.5 Q.8

Answer Key

JEE Main/Boards 5 2 −1
Q.8 1.67 ×10 Nm C
Q.9 6 ×10−3 N(repulsive)
Exercise 1
Q.10 (i) 2 × 1012, from wool to polythene,
Q.2 System of Units and nature of medium
(ii) Yes, but of a negligible amount (=2 ×1018 kg in the
10
Q.6 2.1875 ×10 example).
Q.7 1.13 ×10 4 Nm2C−1 Q.12 0.1 µC/m
P hysi cs | 18.43

Q.13 -360 N
Q.21 22 × 105 Nm2 / C
Q.14 No change
Q.23 2.06 × 1018 N (attractive)
18
Q.15 One coulomb, 6.25 ×10
Q.24 At a distance 2a/3 from the charge +4 q; Q=4q/9
Q.16 1.76 ×1011 C (negative)
Q.17 4 2kq / a2 1+2 2
Q.25 q (negative)
Q.18 Zero N 4
Q.26 kq / a2 along OE
Q.19 (i) 5.4 × 106 Nm−1 alongOB
Q.27 No change
(ii) 8.1 × 10−3 N alongOA
Q.28 (i) dV=4E, (ii) Vc > VA
Q.20 (i) 30Nm2 / C , (ii) 15Nm2 / C

Exercise 2
Q.1 D Q.2 C Q.3 D Q.4 D Q.5 A Q.6 A
Q.7 B Q.8 B Q.9 A Q.10 B Q.11 C Q.12 D
Q.13 B

Previous Years’ Questions


Q.1 C Q.2 D Q.3 B Q.4 C Q.5 B Q.6 B
Q.7 A Q.8 B Q.9 C Q.10 C Q.11 D Q.12 C
Q.13 B Q.14 D Q.15 B Q.16 A Q.17 B Q.18 D
Q.19 A Q.20 C Q.21 C Q.22 D Q.23 D

JEE Advanced/Boards
Exercise 1
Q.1 =
a (1 + 2) , the equilibrium will be stable
E E E E

Q.2 (i) a (ii) a (iii) a (iv) a




br br br br

3
 3 
Q.3 9:30 Q.4 −3   × 10−9 C , No field along y-axis
 11 
 

 σq0  1 qQ
Q.5 2 tan−1  Q.6
 2ε mg  2π 4 πε mR3
 0  0
2kQ2
Q.7 Q.8 20 ln2
mR

kq2 4KQ2
Q.9 −
a
(
3− 2 )

Q.10 r =
mV 2
1 8 . 4 4 | Electric Charges, Forces and Fields

2ε0u2m q
Q.11 Q.12
qσ 24 ∈0

R
Q.13 a = Q.14 -Q/3
3

Q.15 1.125 q Q.16 0

2
λq 
Q.17 Q.18 H2 = h1 + h2 − g  
2ε0m V

v
Q.19 Q.20 4 πε0Ka
3

4kq
Q.21 λ RE0 ˆi Q.22 − î
πR 2

2kQq  −1 11 
Q.23 2 Q.24  + 
m  r 8R 

6 2mr ε0
Q.25
epa

Exercise 2

Single Correct Choice Type

Q.1 B Q.2 B Q.3 D

Multiple Correct Choice Type

Q.4 A, D Q.5 B, C Q.6 A, C Q.7 A, C

Assertion Reasoning Type

Q.8 C Q.9 C Q.10 D Q.11 C Q.12 C

Previous Years’ Questions


Q.1 A Q.2 B Q.3 C Q.4 A, C Q.5 A, C Q.6 A

Q.7 A, D Q.8 A, B, C, D Q.9 A, C, D Q.10 6 Q.11 B, D Q.12 C, D

Q.13 C Q.14 C Q.15 A Q.16 C


P hysi cs | 18.45

Solutions

JEE Main/Boards 10−7


= = 1.13 × 104 Nm2 C-1
8.854 × 10−12
Exercise 1
Sol 8:
Sol 1: Because, the forces act towards or away from
centre of the charge

Electric flux through all surfaces


+ - −6
Sol 2: The value of a quantity depends on the units it’s charge enclosed 8.854 × 10
cube = = =106 Nm2 C-1
1 εo 8.854 × 10 −12
been given. Electrostatic force constant k = where
4πε
ε = permittivity of medium flux through one surface
∴ k is dependent on nature of medium 1
= (106) Nm2 C-1 = 1.67 × 105 Nm2 C-1
6
Sol 3: Dielectric constant of a medium is the ratio of (By symmetry)
permittivity of medium to permittivity of vacuum,
ε Sol 9:
30cm
k=
ε0 -7
2 x 10 C
-7
3 x 10 C

Sol 4: Given, dielectric constant = 80 k.q1q2 9 × 109 × 6 × 10−14


F= = = 6 × 10-3N (repulsive)
⇒ ε = 80 × ε0 = 80 × 8.854 × 10-12 r2 9 × 10−2
= 0.708 × 10-9 C2/N-m2
Sol 10: (i) q = ne
Sol 5: If a system contains many number of particles q 3 × 10−7
⇒n= = ≅ 2 × 1012
then the force on the system is the sum of forces on e 1.6 × 10−19
the particles.
     electrons should be present in polythene.
F = F1 + F2 + F3 + ..... Fn .
∴ Direction of flow of electrons is from wool to
polythene.
Sol 6: q = ne
(ii) Yes, since electrons have mass, there is mass transfer.
q 3.5 × 10−9
⇒n= = = 2.1875 × 1010 electrons
e 1.6 × 10−19
Sol 11: Properties:
Sol 7: → Electric lines of forces start at a positive charge and
terminate at a negative charge.
→ No two lines of forces can intersect one another.

Electric flux through the surfaces of cube


charge enclosed +
=
∈o
1 8 . 4 6 | Electric Charges, Forces and Fields

Sol 12: mass of electrons = 9.1 × 10-31 kg


1
No. of electron =
9.1 × 10−31

Charge of 1 kg of electrons = n.e
1
= × 1.6 × 10-19 C= 1.76 × 1011 C
−31
9.1 × 10
r
Electric flux through the imaginary cylinder Sol 17:

= charge enclosed
+q +q
0 a 0
εo

⇒ E(2prl) = λ a
εo
0 0
1 λ
⇒E= .
2π ε o r -q -q

λ = 2 π εo E r Electric field due +q at center


⇒ λ=2p × (8.854 × 10-12) × 9 × 104 × 2 × 10-2 kq kq 2kq
= = =
⇒ λ = 10-7 C/m r2  a 
2
a2
⇒ λ = 0.1 µC/m  
 2
Addition of the four vectors gives field 2 2 (Eq)
proton electron downward
Sol 13: + –
-15
0.8×10 m kq
∴ Electric field = 4 2 .
k.q1q2 a2
F=
r2
Sol 18: 10 cm
9 × 109 × (1.6 × 10−19 ) × ( −1.6 × 10−19 ) A B
= 5c
(0.8 × 10−15 )2 2c
C 10 cm
= -360 N (attractive)
2c
kq1q2 5c
Sol 14: F1 = D C
r2
k(2q1 )(2q2 ) kq1q2 We can see that the forces acting on 1µC are pairs of
F2 = = = F1 forces with equal magnitude and opposite direction
(2r)2 r2 
∴ Net force = 0
∴ No change is observed. 20 cm
A B
3c o -3c
Sol 15: Electron charge 1.6 × 10-19 C << 1 C
10 cm
∴ Coulomb is bigger
1 Sol 19: (i) Electric field at O
q
q = ne⇒ n = =
e 1.6 × 10−19 k(3mC)  k( −3ì C)  2k(3ì C)
= î +  ( −ˆi)  = î
2 2
= 6.25 × 1018 electrons are required (10cm)  (10cm)  10−2
= 5.4 × 106 Nm-1 along OB
Sol 16: Given,
(ii) If a charge of -1.5 × 10-9 C is placed at O, then the
1 kg of electrons force it experiences=E×q= –8.1 × 10-3 N along OA
P hysi cs | 18.47

Sol 20: z
electric field as in direction of n̂

>
3
E=3x10 iN/c
 σ
10 cm ⇒E= ⋅ nˆ
ε0
x
cm
Sol 23: No. of copper molecules
10
109
= × 6.023 × 1023 = 0.95 × 1023 atoms
y 63.59

(i) Flux= E.A =3×103 î × (10-2) î =30 Nm2/C No. of electrons transferred

  ˆi 3 ˆj  0.95 × 1023
= = 0.95 × 1021 electrons
(ii) Flux= E.A = 3×103 î (10-2)  +  100
2 2 

Charge of the pieces = n.e.
= 15 Nm2/C
= 1.52 × 102 = 152 C

Sol 21: Force between the two pieces

kq1q2 9 × 109 × (152) × ( −152)


= =
r2 10−4
5cm

10 Sol 24:
5cm
10
10 n
10 9Q Q
10
region (1) region (2) region (3)
Construct a Gaussian surface as shown
The electric flux through the surfaces of cube = Charge q should be negative to achieve equilibrium
charge enclosed Also if charge is placed in region (1) or (3) the charge
εo will attract the charge in the middle while the other
positive charge pushes the middle charge towards q.
= 10µC = 4π × 9 × 109 × 10 × 10-6 = 36π × 104 Nm2 C-1 So only region (2) is appropriate
εo
Let distance between 4Q and q be ‘d’ then for
Flux through one plate (bottom plate) equilibrium
1 ∴
= × (total/flux) ( Symmetry) k(4Q)(Q) (k)(4Q)(q)
6 =
2
1 x d2
= × 10µC = 6π × 104 Nm2 C-1 = 2 × 105 Nm2 C–1
6 εo q d2
=
Q x2
Sol 22:
k(4Q)(Q) (k)(Q)(q)
Also =
2
x (x − d)2
q 4(x − d)2
>

n
dA ⇒ =
Q x2
d
⇒ d2 = 4(x – d)2 ⇒ x = d ±
2
qenclosed 2x q 4 4Q
⇒d= or d = 2x and = ⇒q=
flux = E. πr 2 = (gauss law) 3 Q 9 9
∈0
−4Q
∴ q ∴q= (negative charge)
⇒E= σ ( σ= ) 9
ε0 πr 2
1 8 . 4 8 | Electric Charges, Forces and Fields

Sol 25: kq
2 2 Sol 27: Electric flux is the rate of flow of the electric
kq
2 2 field through a given area
2a a 
q
φ = E.A
2
kq q
2
a SI units of flux is Volt-meter
Electric flux is independent of the radius of spherical
qenclosed
surface since flux = (Gauss law)
q q εo
∴ No change will be observed.
The force on one charge due to others is

2 2 Sol 28: - -
 kq2   kq2  kq2 1  kq2
=  2  + 2  + =  2+ 
 a   a  2a2  2  q2 - -
   
- -
The charge to be placed at the center should be
O - -
negative and let value be Q
tq - -
2
kQ(q) kq  1 - -
2
=  2+ 
 a 
2
a  2 - -
 
 2 For derivation of the expression, please refer the theory.
1 + 2 2 
⇒Q=   q
 4 
 
(1 + 2 2) Exercise 2
∴Q=– q
4
Sol 1: (D)
Sol 26: B C
Q Q 3 (2,3)
(50

Eq
c
)

A Ea
Q a 0
2
Ea Ea QO
E (8,-5)
kq
E=
a2 (
Direction of field = 6iˆ − 8ˆj m )
The system will be stable if a force Eq is placed at O distance = 10 m

along EO ( symmetric and equal forces are acting)
kq 9 × 109 × (90µc)
Magnitude of field = = = 4500 V/m
∴ By adding a force Eq along EO and OE we get r2 102
Eq
Sol 2: (C)
Eq=Ea
Eq C
Eq
B
A
Eq O
Eq
Eq

∴ final electric field is E along OE


 
kq EA is parallel to OA and similarly are others.
E=
a2  
OA . OB = 1 + 2 – 3 = 0
P hysi cs | 18.49

    ∴ˆ
⇒ OA ⊥ OB ⇒ EA ⊥ EB ( j components get cancelled while integration)
 
OB  = 3 OC  = 2 3 kQx
    ⇒E=
(R + x2 )3/2
2

1 1 E 1
⇒ EB ∝  , EC ∝  ⇒ C = kQx
OB
2
OC
2 EB 4 ⇒E= if R ≫> x
R3
⇒ F = moa = - Eq
P kQq
Sol 3: (D) ⇒a=− .x
q x q m0R 3
kQq
at x=0 w2 =
E=- m0R 3
Qq
E ⇒ω=
4 π ε0m0R 3
x /2 at x = 1
at x = 1/2
E =- Sol 7: (B) Volt = joule/coulomb
E=0
(Since volt is S.I. unit of electric potential =W/q)
kq kq
E= − kQx( +q)
p
x 2
( − x)2 Sol 8: (B) F = -mg
(R 2 + x2 )3/2

Sol 4: (D) dF
A D if < 0 then the particle is in stable equilibrium
+q -q dx
3
2E (due to =2q) (R 2 + x2 )3/2 − (R 2 + x2 )1/2 (2x2 )
E (due to -q) ⇒ 2 <0
(R + x2 )3
2

E (due to +q) ⇒ R2 + x2 – 3x2 < 0


2E R R
⇒x> or x < −
The vector sum gives field in direction perpendicular 2 2
to AB
R
∴ Only if x > , the equilibrium will be stable.
2
Sol 5: (A) Refer question 24 of exercise I JEE Main

Sol 9: (A) Initial Final


Sol 6: (A)
2 2
+

R +X
R - +  P
 P
 
x P E - 
E
The field vector is rotated by 90° clockwise

Electric field due to ring at x is Sol 10: (B) q1 is positive, (emission of field lines), q2 is
negative, (termination of field lines).
=E= ∫ dE = ∫ dEcos θ ˆi + ∫ dEsin θ ˆj q2
= =
7 number of linesabsorbed
<1
Q Q q1 10 number of linesemitted
K cos θ dq ˆ K sin θ dq ˆ
⇒E= ∫ 2 2
i+ ∫
2 2
j Electric field is strongest at some point closer to q2.
0 (x + R ) 0 (x + R )

Q
x
⇒E= ∫ (x2 + R2 )3/2 dq + 0
0
1 8 . 5 0 | Electric Charges, Forces and Fields

Sol 11: (C) z q1 kq1q2


substituting q2 = in = 0.108N gives
3 r2

>

>

>
E = i+ 2j + 3k
y ⇒ q1 = ±3 × 10-6 C and q2 = ±1 × 10-6 C

x
A = 100 m
2
Previous Years’ Questions
Only z-component of field is responsible for flux Sol 1: (C) From conservation of mechanical energy
through plate
 Decrease in kinetic energy = increase in potential
⇒ Flux= E.A = (iˆ + 2ˆj + 3k)
ˆ . (100 k̂ ) = 173.2 V-m energy
1 (Ze)(2e)
or =5MeV=5×1.6×10–13 J
Sol 12: (D) 4πε0 rmin

2Ze2
R
∴ rmin = 1
x 4πε0 5 × 1.6 × 10 –13

(9 × 109 )(2)(92)(1.6 × 10 –19 )2


= ( Z = 92)
5 × 1.6 × 10 –13

qenclosed = σ. Aenclosed
+Ze +2e
Aenclosed = pr2 = π(R2 – x2)
qenclosed π(R 2 − x2 ) σ rmin
∴ Flux through sphere = =
∈o ∈0
= 5.3 × 10–14 m
Sol 13: (B) q1 q2 = 5.3 × 10–12 cm

0.5m i.e., rmin is of the order of 10–12 cm

Sol 2: (D) Motion is simple harmonic only if Q is


kq1q2
= 0.108 N released from a point not very far from the origin on
(0.5)2 x-axis. Otherwise motion is periodic but not simple
When connected with a wire, the charges on them will harmonic.
be distributed equally giving
−q1 + q2 Sol 3: (B) Since, q is at the centre of two charges Q and
q= on each sphere Q, net force on it is zero, whatever the magnitude and sign
2
of charge on it.
(Since one of them is negative)
kq2 Q q Q
= 0.036 N
(0.5)2 For the equilibrium of Q, q should be negative because
q2 0.036 1 other charge Q will repel it, so q should attract it.
⇒ = = Simultaneously these attractions and repulsions should
q1q2 0.108 3
be equal.
4q1q2
⇒ (q2 – q1)2 = 1 QQ 1 Qq
3 4πε0 r 2 4πε0 (r / 2)2
10q1q2
⇒ q22 + q12 – =0 Q
3 or q =
q1 q2 4
⇒ q2 = or q1 = Q
3 3 or with sign q = -
4
P hysi cs | 18.51

Sol 4: (C) The magnitude of electric field at a distance Sol 11: (D) According to option (d) the electric field
r from the axis is given as: due to P and S and due to Q and T add to zero. While
λ
due to U and R will be added up.
E=
2πε0r
1 Sol. 12: (C) At any point over the spherical Gaussian
i.e., E ∝ surface, net electric field is the vector sum of electric
r
fields due to +q1, –q1 and q2. Don’t confuse with the
Here, λ is the charge per unit length of the capacitor. electric flux which is zero (net) passing over the Gaussian
surface as the net charge enclosing the surface is zero.
Sol 5: (B) Electric Field lines never enter a metallic
conductor (E = 0, inside a conductor) and they fall Sol 13: (B) All the three plates will produce electric field
normally on the surface of a metallic conductor at P along negative z-axis, Hence,
(because whole surface is at same potential and lines
are perpendicular to equipotential surface) →  σ 2σ σ  2σ
Ep =  + +  (– k̂ )= – k̂

 0 2ε 0 2ε 
0 ε0
Sol 6: (B) Electrostatic force, Fe = eE (for both the
∴ Correct answer is (b)
particles)
But acceleration of electron, ae = Fe/me and acceleration
Sol 14: (D) Charge will be induced in the conducting
of proton, ap = Fe/mp
sphere, but net charge on it will be zero.
1 1
S= ae t2 = ap t22 ∴ Option (d) is correct.
2 1 2
t2 ae mp Sol 15: (B) Inside the cavity, field at any point is uniform
∴ = =
t1 ap me and non-zero.

 =0 → →  =0
Therefore, correct option is (b).
Sol 7: (A) − ∫ E .d = ∫ dV = V (centre) – V (infinity)
 =∞  =∞ Sol 16: (A) Total enclosed charge as already shown is
But V(infinity) = 0 6C 8C
 =0 → →
qnet = + – 7C = – 2C
2 4
∴ − ∫ E .d corresponds to potential at centre of ring. qnet –2C
 =∞
From Gauss theorem, net flux, φnet = =
ε0 ε0
1 q (9 × 109 )(1.11 × 10 –10 )
And V(centre) = . = =2 volt
4πε0 R 0.5 Sol 17: (B) (36R )
2

2
-16R 2
Sol 8: (B) Net electrostatic energy of the configuration S=(16R )
will be -Q1

 q.q Q.q Q.q  Q1


1
U=K  + +  Here, K=
 a 2a a  4πε0

–2q
Putting U = 0 we get, Q =
2+ 2

Sol 9: (C) Electric lines of force never form closed loops. Q1 = σ(4pR2) = 4psR2
Q2 = 16psR2 – Q1 = 12psR2
Sol 10: (C) Electric field is zero everywhere inside a
∴ Q3 = 36psR2 – 16psR2 = 20psR2
metal (conductor) i.e., field lines do not enter a metal.
Simultaneously these are perpendicular to a metal Q1 : Q2 : Q3 = 1 : 3 : 5
surface (equipotential surface).
1 8 . 5 2 | Electric Charges, Forces and Fields

Sol 18: (D) qE = mg  … (i) → →


Here, θ is the angle between E and S
6phrv = mg →
In this question θ = 45°, because S is perpendicular
4 3
pr rg = mg  … (ii) to surface.
3
1/3 E = E0
 3mg 
∴r=   … (iii)
 4 πρg  S = ( 2 a)(a) = 2 a2

Substituting the value of r in Eq. (i) we get, ∴ φ = (E0)( 2 a2) cos 45° = E0a2
1/3
 3mg   1
6phv   = mg Q
Sol 21: (C) Einside =  r
 4 πρg   4 π ε R 3 
 0 
 3mg 
or (6phv)3  4 πρg  =(mg)3  1 Q
Eoutside =  r
   4 πε R 3 
 0 
Again substituting mg = qE we get. ∴ E
 3 
(qE)2 =   (6phv)
3

 4 πρg 
r
1/2 R
 3 
or qE =   (6phv)3/2 Sol 22: (D)
 4 πρg 
k.q.q / 2 y kq2 y
1  3 
1/2 Fnet 2 Fcos θ = 2
= . = ( y << a)
∴q= (6phv)3/2  a2 + y 2 
  a2 + y 2 a3
 
E  4 πρg   
F   F
Substituting the values we get
q/2
7 3
q= × 216π3 y
81π × 105 4 π × 900 × 9.8
q a a q
× (1.8 × 10 –5 × 2 × 10 –3 )3

= 8.0 × 10–19 C Sol 23: (D) It originates from +Ve charge and terminates
at - Ve charge. It can not form close loop.
1 2
Sol 19: (A) Electrical force per unit area = eE
2 0
1  σ 2 σ2 JEE Advanced/Boards
= e0   =
2 ε  2ε0
 0
Exercise 1
Projected area = pR2
x
 σ2  Sol 1:
∴ Net electrical force =   (pR2)
 2ε  -2q Q q
 0 

In equilibrium, this force should be equal to the applied
force. For equilibrium x >  and Q should be positive
balancing force equations,
πσ2R 2 σ2R 2
∴F= or F ∝ k(2q)(Q) kq(Q)
2ε0 ε0 =
2
(x) (x −  )2
→ →
Sol 20: (C) Electric flux, φ = E . S 2
  1  1
⇒ 1 −  = ⇒1- =±
or φ = ES cos q  x  2 x 2
P hysi cs | 18.53

 2 ±1 ⇒ θ = 15°
⇒ = ⇒x=2± 2 
x 2 The hour hand should be midway of between 9 and 10
∴ Time =
9 : 30
x >  ⇒ x = (2 + 2) 

(1 + 2 )  from q Sol 4: )
1,1
(3,
It is in stable equilibrium w.r.t. longitudinal motion q Q
(0,0,0) (2,0,0)
Sol 2:

 kQ  3iˆ + ˆj + kˆ 
Eq =  
(32 + 12 + 12 )  11 
E
 kQ  ˆi − ˆj − kˆ 
r EQ =  
 12 + 12 + 12   3 
 
 
(i) (ii) At P x-Component of field is zero

  3kq −QK
⇒ (EP + EQ )x = 0 ⇒ =
( 11 ) ( 3)
3 3

3
 3 
⇒ Q = -3   × 10-9 C
 11 
 
y-component has zero field.
(iii) (iv)
σ
Sol 3: -12q Sol 5: Electric field due to plate =
2∈0
-q
(Non-conducting plate)
-2q
The force that is being applied on bob = Eq

Q T
Direction of electric field at center is
Eq
Eq
mg
12E 6E mg
6E
9E 3E

6E Change in gravitational potential = mg  (1 – cosθ)
6E
Change in electrical potential = Eq  sin q
6E 6E
6E
mg  (2 sin2 θ /2) = Eq  (2 sin θ /2 cos θ/2)
 Eq
ER ⇒ tan θ/2 =
mg

ER  σq 
6E 6 ⇒ θ = 2 tan-1  
≡ ≡ 0.1  2 ε0mg 
6+6+6 3 6(2+ 3 )

1
tan θ =
2+ 3
tan θ = 2 - 3
1 8 . 5 4 | Electric Charges, Forces and Fields

Sol 6: +++ Sol 8:


+ +

+
+ + -q

+
+ + ++ +

++
+ ++

+ + + +
Q
+ ++ + + +

+++
.Q
+ + + +++ +
+ ++ +++
++ + 

+
+ +
++ E=
+ + 2r

At any point x from center, the acceleration of the


r2
charge is  −λ 
V = - ∫ Edr =  nr 
Eq  2π  r1
a=-
m λ r
DV = ln 1
But, electric field at the point is 2π r2
qencloses
E.(4pr2) = (Gauss’s law) By energy conservation
ε0
1 λ r 
Q 4 3 mv2 = ln  2 
⇒ E.4pr2 = 1 . . πr 2
ε0 4 3 3
2π  r1 
πR
3
Qr λ  r2  2Er
v= ln   ⇒ v = .ln2
⇒E= πm  r1  m
4 π ε0R3

−Qq 2 × 100 × 0.2


v= .ln2
⇒ a= .x 0.1
4x ε0R 2
⇒ v = 20 ln2
Qq
⇒∴ω= a
4 π ε0R3 Sol 9:
+q +q
ω 1 Qq
⇒f= =
2π 2π 4 π ε0R3
Q

Sol 7: +Q +q
+q

-Q Initial configuration
-Q kq2 kq2
t>t Initial potential energy = ×4+ ×2
t>0 a 2a
=(4 + 2 ) kq2 /a
kQ
Electric potential at center of ring = +q
R
By energy conservation, 2a

1 kQ(Q) +q +q
mv2 =
2 R 2a

+q
2kQ2
⇒v= Final configuration
mR
Final potential energy
kq2 kq2
= ×4+ × 2 = (2 2 + 1) kq2 /a
2a 2a

work done = Uf − Ui
P hysi cs | 18.55

Construct Gaussian surface as below


= (( ) (
2 2 +1 − 4 + 2 ))
kq2
a
(
=− 3− 2 ) kq2
a Construct gaussian surface

Sol 10: Q Q
← V Initial
Q Q
← ←
V′ V′
1
r such that the original is th
of it
8
At closest distance of approach qenclosed
Flux =
By momentum conservation V ′ = V/2 ε0
flux through one forth of one surface
By energy conservation,
1 1 q
1 1  V′  kQ2
2 = . = q (By symmetry)
mV2 = m   ×2+ 4 6 ε0 24 ε0
2 2 2 r

1 1 Sol 13: R
⇒ kQ2 = mV2
r 4
4KQ2 a
⇒r=
2
mV
Q
Sol 11: +
+ Take an elemental part with thickness dr as below
+ E cos
+ a E
+ r dx
+ 
4
a
Eq 
a=
m
Q
u2 sin2θ
⇒ Maximum horizontal distance = The electric field at the elemental part is
a
u 2 kQ
⇒ Hmax = E=
a a + r2
2

flux through the element dφ = E. dA cos θ


mu2 2 ε0mu2
⇒ Hmax = = R Ea ( 2πr )
Eq σq ⇒ φ =∫ dr
(a )
1/2
∴ σ 0 2
+r 2
( E= for non-conducting plate)
2 ε0 R
r
⇒ φ = 2pkQa ∫ (a2 + r2 )3/2 dr
Sol 12: 0
R
 
 (a2 + r 2 )−1/2 
⇒ φ = apkQ  
 1 
 − 
 2 0
q
 1 1
⇒ φ = apkQ [-2]  2 − 
2 1/2
 (a + R ) a
1 8 . 5 6 | Electric Charges, Forces and Fields

 a  Sol 16: y
⇒ φ = 2pkQ 1 − 2 2 1/2 
 (a + R )  +Q
Given -Q x
a
-Q
φ= Q x
4 ε0 +Q
+Q a
 a 
⇒ Q  1 − 2  =
2 1/2 
Q
z
Q/2 -Q
2 ε0  (a + R )  4 ε0
Consider electric field due to +Q charges,
a 1 R
⇒ = ⇒ 3a2 = R2 ⇒ a = We will get,
(a2 + R 2 )1/2 2 3
 kQ2 x
E+ Q = ( +ˆj)
2 2
a +x a + x2
2
Sol 14:
+ 3R + + while due to negative charges,
E+q EQ-q
+
 kQ2 x
R q E−Q = ( −ˆj)
2 2
q a +x 2
a +x 2

∴ Electric field at point on the y-axis


 
= E+ Q + E−Q = 0
The potential at r = R is zero
k(Q − q) kq kq Sol 17: x
⇒ + - =0
3R 3R R
V( 3R, 0) P y
Q
⇒q= (negative)
3
z
R
Sol 15:

C
+6q –3q Initial electric potential energy = q.VP
A B
kQ kQq kλ (2πR)q
= q. = = = λq
R 2 + ( 3R)2 2R 2R 4 ε0

kQ
q = λq
+ 1.5q + 1.5q O
Final potential energy =
I. A B C R 2 ε0
for minimum velocity, final kinetic energy = 0
By conservation of Energy,
K.E.i + P.Ei = K.E.f + P.E.f
+0.75q +1.5q +0.75q
1
A +B C ⇒ mv2 + λq = 0 + λq
II. 2 4 ε0 2 ε0

λq
⇒v=
2 ε 0m

+0.75q
 1.5 + 0.75
2 
q

1.125q Sol 18: Consider the two balls of system, the only
external force is gravitational force. Initial position of
III. A +B C
h1 + h2
COM is at . The vertical distance moved by
2
∴ Charge on C = 1.125 q
P hysi cs | 18.57

 1 
2 Sol 20: +q
+q
COM during time t =   is h = g  
v 2 v
+q
+q
Final height of COM is
2

h1 + h2 − g  
H= V
2 2008-Gm

If the COM is at H, and one particle is on ground then the The potential energy of the system be U = U12 + U13 +
2(HCOM ) − H1 …… U20062007
height of the other will be at height H2 =
2 −1 The K.E. of the first ball after being released for a long

2
time
⇒ H2 = h1 + h2 – g  
v By energy conservation

∴ Height at which the body is located is K.Ef + P.Ef = K.Ei + P.Ei


2 ⇒ K1 = 0 + (P.Ei - P.Ef )

H = h1 + h2 – g   ⇒ K1 = U12 + U13 + U14 + …… U2007
v
The K.E. of second ball after being released for a long
time
Sol 19: I. t = 0 → ΙΙ. t = t
By energy conservation
v
q1 q1
K2 = (P.E.’ i – P.E’f )
o
60

V/2 = U23 + U24 + U25 + ….. + U2 2007


= U12 + U13 + U14 + …. + U1 2006
v
q2 q2 ∴
( Un n + 1 = Un-1 n by symmetry)
kq2 ∴
⇒ K1 – K2 = U1 2007 = = K( K1 – K2 = K Given)
1
V 2
Acceleration of first ball in x-direction due to a
⇒q= 4 π ε0 aK
3V ˆ
field = (a1 )x = (i)
4t

3V Sol 21:
Similarly (a1 )y = ( −ˆj) ++
4t
>

E0 i +
T
−V ˆ +
(a2)x = (i)
t
−V' ˆ f
(a2)y = ( j)
t Friction acts in forward direction decreases angular
 acceleration and increasing linear acceleration
 E.q1 3v 3v ˆ
a1 = =- î - j
m 4t 4t ⇒ f = ma

 −3mv 3mv ˆ and Te – f. R = Iα = mR2α


⇒E = î - j
4q1 t 4q1 t also a = Rα

 Eq2 −3vq2ˆi 3vq2 ˆ −v v1 ˆ Te
a2 = = - j = î - j ⇒f=
m 4q1 t 4q1 t t t 2R
Q
q2 4 3V  4  v but Te = ∫ dTe =
⇒ = and V1 =   = ∫ E0dq.R cos θ
q1 3 4 3 3 0
1 8 . 5 8 | Electric Charges, Forces and Fields

π /2
2
Sol 23: -3/2q
⇒ Te = 2 × ∫ E0 λ.R
0
cos θ dθ
 
-
2
5
(2 is multiplied considering –ve changes also) a
π /2
= 2 × E0 λ R2 sin θ  b O z=a
0

⇒ Te = 2E0 lR2
+q
⇒ f = E0 lR î
 kQ(a)
dq
EA = ˆ
(i)
E0dq (a + a2 )3/2
2

 kQ'(a)
EB = ( −ˆi)
(b + a2 )3/2
2
Rcos
d
   
E = EA + EB = 0 (given)
Q −Q'
⇒ =
Sol 22: 2 3/2
(2a ) (b + a2 )3/2
2

- ++ +
-q
---- ++ q 5
- - ++ ⇒ b2 + a2 = 2a2  
-
---- + 2
---
E +
---
E + b
+ ⇒ b = 2a ⇒ =2
a
ER
E+1
E+ Sol 24: ++
++++
+
++ R ++
dw +++
+ + ++ + + vq
+++ +++ +
+ ++
 k r
d
+ + dq
dEtsin + + +
 ++ +++ +
+ +
r
+
dEt + +++ +R/2 + dr
dEtcos ++ ++
R
The x-component of field = ∫ dE sin θ
+
∴ potential at R/2 is
π /2
k.dq k Q1
= ∫ sinθ = λ ∫ sin θ dθ k.dq
Q
kdq
R 2 R 0 V= ∫ dV = ∫ R 
+ ∫ r
0 Q'
 
kλ π /2 kλ kq 2kq 2
=  − cos θ  = (1) = =
R  0 R  πR  πR 2 (element part is a hollow sphere of rad radius r)
R 
 2 
R /2 R
The y-component of positive charges’ field cancels the k.ρ.4 πr 2dr kρ4 π r 2dr  Q 
y-component field of negative charges’ field.
= ⇒V ∫ R 
+ ∫
r
=  ρ 
3
(4 / 3)πR 
0 R /2 
 
∴ The total electric field will be 2
R
   4kQ R
Etotal = E+ + E− = ( −ˆi) 2kρ  r3  2  r2 
πR 2 ⇒V= × 4π   + 4pρk  
R  3   2  R
0
2

2k Q 9 kQ 11 kQ
⇒V= + =
R 8 8 R 8 R
P hysi cs | 18.59

By energy conservation we get, (Included charge)


K.E.i + P.E.i = K.E.f + P.E.f  ρa 
⇒ Erequired = along OO'
1 kQq 3ε0
11kQq
⇒ mv2 + =0+
2 r 8R It is true for any point inside cavity.

2kQq  −1 11 
⇒V=  +  Exercise 2
m  r 8R 
Single Correct Choice Type
Sol 25:
The electric field inside the cavity will be Sol 1: (B)
kQ
E= along OO’ (proof next page)
a2
+q +q +q

e
r
---  The charge in the middle experiences force along the
line.
1
Oa 0E

The equilibrium is stable along the line connecting


charges while
The distance the electron has to travel is The equilibrium is unstable along the line perpendicular
2a cos θ = 2r to the line of charges
s = 1/2 at 2 ∴ Only option B is correct (given consider equilibrium
only along line joining charges)

Sol 2: (B) It is not necessary for particle to move along


f i
lines of force. Lines of forces only denote the direction
of force that exists on particle.
  ∴ Option B is correct (It may move in a uniform electric
field)
2s 2 2r ∴ Eq To contradict option D, take negative charge.
⇒t= = [ a= ]
a Eq m
m
Sol 3: (D) Charge won’t be uniformly distributed if there
6 2mr ε0 is an external field, even in an external electric field, the
⇒t= 2m 2r = field strength inside sphere is zero (by Gauss law)
1 ρ × (4 / 3) πa3 eρa
e. Potential must be same at every point of sphere
4 π ε0 a2
  
Erequired = Ewhole - Ecut
Multiple Correct Choice Type

P Sol 4: (A, D)
= 1
0
P
+4Q -Q
O
1
0 A B

   The resultant electric field will be zero at point closer to
= OP - O'P = OO' B and outside AB (by analysing directions of field and
kq 4 magnitudes)
Field at center of cavity = where q = ρpa3
2
a 3
1 8 . 6 0 | Electric Charges, Forces and Fields

If a positive charge is placed at P and distributed, the qenclosed σπr 2


positive charge either goes towards, -Q or moves away ⇒ flux = E.(pr2) = =
ε0 ε0
∴ d2 vP
from –Q but won’t oscillate ( > 0) (unstable ⇒ E = σ = towards right
dx2 ε0
∴ d2Un
equilibrium) while negative charge oscillate (
d2 x Sol 7: (A, C)
d2UP
= − < 0) (Stable equilibrium) E
d2 x v
Sol 5: (B, C) For the system to be at minimum potential m
energy, the higher charged particles should be far apart.
Eq ∴
a= + g ( F = ma = Eq + mg)
x m
Eq
if + g = 0 ⇒ (linear motion)
4Q Q 16Q m
9 cm Eq
if + g = k (constant) ⇒ parabolic motion
m
and now potential energy
(k)(64Q2 ) k(16Q2 ) k4Q2 Assertion Reasoning Type
U= + +
d2 d−x x
dU +16 4 Sol 8: (C) Electric lines of force represent the force
=0⇒ =
dx (d − x)2
x2 acting on particle at that point
⇒ ±2x = d – x
⇒ x ± 2x = d Sol 9: (C) Refer to question 30 of exercise - III

⇒ x = d/3 or x = -d
Sol 10: (D) Drawing Gaussian surface won’t change
9 electric field.
⇒ x= = 3cm.
3
Field at Q is Sol 11: (C) Statement-I is true, since Qenclosed is same but
k(4Q) k(16Q) E at that site changes depending on external charge.
= - =0 But Gauss law is still valid since the flux by the external
2
(3cm) (6cm)2
change is zero.

Sol 6: (A, C) Q+ -
+ - Sol 12: (C) Statement-I is true by Gauss law.
t -
+
- Statement-II is false since distance between the point
+
+ - charge and the site decreases which changes electric
+ - field.
+ -
+ -
Previous Years’ Questions
qenclosed
flux = E (2πr 2 ) = = 0 =0
ε0 ε0 Sol 1: (A) At r = R. From Gauss’s law
∴E=0 qnet Ze
E (4pR2) = =
Also for any point between plates ε0 ε0
+ -
1 Ze
+ - or E =
+ - 4πε0 R 2
+ -
O
+ E

- E is independent of a.
+ -
+ -
+ -
P hysi cs | 18.61

Sol 2: (B) For a = 0 1 Qq


Fe = qE= . .z  ….(i)
4πε0 (R 2 + Z 2 )3/2 0
0
 d 
ρ(r) =  – .r + d 
 R  Similarly, when it crosses the origin, the force is again
towards centre O.
R
 d  Thus, the motion of the particle is periodic for all values
Now ∫ (4 πr 2 )  d – r  dr = net charge = Ze.
a  R  of z0 lying between 0 and ∞.
 Secondly, if z0 <<R, (R2 + z 20 )3/2 = R3
d
1 Qq
Fe = . .z [From Eq. (i)]
4πε0 R 3 0
i.e., the restoring force Fe ∝ – z0. Hence, the motion of
r the particle will be simple harmonic. (Here negative
R
3Ze sign implies that the force is towards its mean position.)
Solving this equation, we get d =
πR3 1 Q
Sol 6: (A) Inside the sphere E = r
Sol 3: (C) In case of solid sphere of charge of uniform 4πε0 R 3
volume density
( r )
⇒ E ∝ r for r ≤ R
1 q
E= . .r i.e., E at centre = 0 as r = 0
4πε0 R 3
1 Q
or E ∝ r and E at surface = . as r = R
4πε0 R 2
Thus, for E to be linearly Outside the sphere
t
dependent on r, volume charge R
1 Q 1
density should be constant. E= . for r ≥ R or E ∝
4πε0 r 2 r2
or a = R
Thus, variation of electric field (E) with distance (r) from
the centre will be as shown
Sol 4: (A, C) Net torque on (–q) about a point (say P) E
lying over +Q is zero. Therefore, angular momentum of
(–q) about point P should remain constant. 1 Q
E=
40 R2
Sol 5: (A, C) Let Q be the charge on the ring, the Er
E
1
2
negative charge –q is released from point P (0, 0, z0). r
The electric field at P due to the charged ring will be t
O
along positive z-axis and its magnitude will be r=R

y Sol 7: (A, D) From the behaviour of electric lines, we


E can say that Q1 is positive and Q2 is negative. Further,
Q |Q1| > |Q2|
-q
O z At some finite distance to the right of Q2, electric field
P(0,0z0) will be zero. Because electric field due to Q1 is towards
R
right (away from Q1) and due to Q2 is towards left
F0 (towards Q2). But since magnitude of Q1 is more, the
two fields may cancel each other because distance of
1 Qz 0 that point form Q1 will also be more
E=
4πε0 (R + z 2 )3/2
2
0
Sol 8: (A, B, C, D) Inside a conducting shell electric field
E = 0 at centre of the ring because z0 = 0 is always zero. Therefore, option (a) is correct. When the
two are connected, their potentials become the same.
Force on charge at P will be towards centre as shown,
and its magnitude is ∴ VA = VB
1 8 . 6 2 | Electric Charges, Forces and Fields

QA QB  1 Q
or =  V= 
RA RB  4 πε0 R 

Since, RA > RB ∴ QA > QB P2 P1
1
σR
Potential is also equal to, V = , VA = VB 2R 2
ε0
σA RA
∴ σARA = σBRB or = or σA < σB
σB RB
Sol 12: (C, D) In triangle PC1 C2
σ
Electric field on surface, E = or E ∝ s   
ε0 r2= d + r1
Since σA < σB ∴ EA < EB
The electrostatic field at point P is
qin q
Sol 9: (A, C, D) φout = =  4   4  
ε0 ε0 K  ρ π R13  r2 K  ρ π R32  ( − r1 )
By symmetry
=E  3  +  3 
3 3
R1 R2
Sol 10: (1) + (2) = Complete cylinder 4  
E=K ρ π ( r2 − r1 )
E1 + E2 =
E 3
 ρ 
ρ × πR 2 ρR E= d
=E = 3 ε0
2πε0 (2R) 4ε0
 
3  r2 r1 -
4π R  1 ρR P
E2 = ρ ×   × = C2
( )
C1
3 2
  4 π ε0 4R 2 24 × 4 ε0 R1 R2

ρR  1  ρ R 23 23 ρ R
E1 =E − E2 ⇒ 1 − = = d
4 ε0  24  4 ε0 4 × 6 16 ε0 × 6

Q λ σ
1 Sol 13: (C) = =
2
4 π ε0r0 2 π ε0 r0 2 ε0

2 r  Q r  λ r  σ
E1  0 
= = , E2  0  = ,E3  0 
 2  π ε0r0  2  π ε0r0  2  2 ε0
2

r  r 
∴ E1  0  =
2E2  0 
2 2
Sol 11: (B, D)

1 ρ1 ( 4 / 3) π R
3
ρ R Sol 14: (C)
At point P1 , = 2
4 π ε0 4R 2 3 ε0 Q
For point outside dielectric sphere E =
4 π ε0 r 2
ρ1 R ρ2 R r
= For point inside dielectric sphere E = Es
12 3 R
ρ1 Exact Ratio E1 : E2 : E3 = 2 : 4 : 1
=4
ρ2
ρ1 ( 4 / 3) πR3 ρ2 ( 4 / 3) π 8R3
At point P2 , + 0
= Sol 15: (A)
( 2R ) (5R )
2 2
P: By Q1 and Q4, Q3 and Q2 F is in +y
ρ1 32
∴ =

ρ2 25 Q: By Q1 and Q4, Q2 and Q3 F is in + ve x.
R: By Q1 and Q4, F is in + ve y
P hysi cs | 18.63

By Q2 and Q3, F is in –ve y


But later has more magnitude, since its closer to (0, b).
Therefore net force is in –y
S: By Q1 and Q4, F is in +ve x and by Q2 and Q3, F is in –x,
but later is more in magnitude, since its closer to (0, b).
Therefore net force is in –ve x.
(+0, b)

Q1 Q2 Q3 Q4
(-2a, 0) (-a, 0) (+a, 0) (-2a, 0)

Sol 16: (C) In Case I:

λq λq
F=
2 π ε0 ( r + x )
ˆi + = ( )
2 π ε0 ( r − x )
−ˆi λq
( )
x −ˆi
π ε0 r 2

Hence +q, charge will performs SHM with time period

π r 2 ε0 m
T= 2 π
λq

In case II: Resultant force will act along the direction of


displacement.

 ρ
Sol 17: (D) E = CC
3 ε0 1 2
C1 ⇒ Centre of sphere and C2 ⇒ centre of cavity.
2017-18 100 &
op kers
Class 12 T
By E ran culty
-JE Fa r
IIT enior emie .
S fP r es
o titut
Ins

PHYSICS
FOR JEE MAIN & ADVANCED
SECOND
EDITION

Exhaustive Theory
(Now Revised)

Formula Sheet
9000+ Problems
based on latest JEE pattern

2500 + 1000 (New) Problems


of previous 35 years of
AIEEE (JEE Main) and IIT-JEE (JEE Adv)

5000+Illustrations and Solved Examples


Detailed Solutions
of all problems available

Topic Covered Plancess Concepts


Tips & Tricks, Facts, Notes, Misconceptions,
Electric Potential Key Take Aways, Problem Solving Tactics
and Capacitance
PlancEssential
Questions recommended for revision
19. ELECTRIC
POTENTIAL AND
C A PA C I TA N C E
1. INTRODUCTION
Associating a potential energy with a force allows us to apply the principle of the conservation of mechanical
energy to closed systems involving the force. The principle allows us to calculate the results of experiments for
which force calculations alone would be very difficult. In this chapter, we first define this type of potential energy
and then put it to use. We will also discuss the behavior of conductors which can store the charges in it, in the
presence of the electric field.

2. ELECTRIC POTENTIAL
An electric field at any point can be defined in two different ways:

(i) by the field strength E , and
(ii) by the electric potential V at the point under consideration.

Both E and V are functions of position and there is a fixed relationship between these two. Of these the field
strength E is a vector quantity while the electric potential V is a scalar quantity. The electric potential at any point
in an electric field is defined as the potential energy per unit charge, and similarly, the field strength is defined as
the force per unit charge. Thus,
U
V= or U = q0 V
q0
The SI unit of potential is volt (V) which is equal to joule per coulomb. So,
1V 1volt
= = 1J=/ C 1 joule / coulomb
According to the definition of potential energy, the work done by the electrostatic force in displacing a test charge
q0 from point a to point b in an electric field is defined as the negative of change in potential energy between them,
or ∆U =−Wa−b ∴ Ub − Ua = −Wa−b
Dividing this equation by qο ,

Ub Ua W W U
− − a−b or Va − Vb =a−b since V =
=
qο qο qο qο qο

Thus, the work done per unit charge by the electric force when a charge body moves from point a to point b is equal
to the potential at point a minus the potential at point b. We sometimes abbreviate this difference as Vab
= Va − Vb .
1 9 . 2 | Electric Potential and Capacitance

Another way to interpret the potential difference Vab is that it is , equal to the work that must be done by an external
force to move a unit positive charge from point b to point a against the electric force. Thus,
( Wb−a )externalforce
Va − Vb =
qο

PLANCESS CONCEPTS

1 1 q dq
In the equation V =
4 πεο
∑ ri
4 πε ∫ r
or V =
, if the whole charge is at equal distance ro from the
i i ο
1 qnet
point where V is to be evaluated, then we can write, V = . , where qnet is the algebraic sum of
4 πεο rο
all the charges of which the system is made.
Vaibhav Gupta (JEE 2009 AIR 54)

Illustration 1: Consider a non-conducting rod of length  having a uniform  y


charge density λ . Find the electric potential at P at a perpendicular distance
P
y above the midpoint of the rod  (JEE MAIN)

Sol: The potential due to the small length element dx of the rod at a point p ’

dq 1
at distance r apart is given by=
dV × where dq is the charge on dx. y
4 πεo r r’
∂V
The electric field due to the rod is given by E = − .
∂r x’
x
Consider a differential element of length dx’ which carries a charge dq = λdx' O dx’
. As shown in Fig.19.1, the source element is located at (x’, 0), while the field L
point P is located on the y-axis at (0,y).
Figure 19.1

(x )
1/2
'2 2
The distance from dx’ to P =
is r +y .
1 dq 1 λdx'
Its contribution to the potential is given=
by dV =
4 πεο r 4 πεο
(x )
1/2
'2
+ y2

Taking V to be zero at infinity, the total potential due to the entire rod is
 /2 V(y)/VO
λ  /2 dx' λ
=V ∫− /2 = ln  x'+ x' 2 + y 2  
4 πεο '2
x +y 2 4 πε ο   −  /2 4

3
 
 (  / 2) + (  / 2) + y
2 2
λ 
= ln   2
4 πεο 
 (
−  / 2) + (  / 2) + y2
2

 1

Where we have used the integration formula -4 -2 2 4


y/t

dx'
∫ = In  x' + x' 2 + y 2  Figure 19.2
'2
x +y 2  

A plot of V ( y ) / Vο , where Vο = λ / 4 πεο , as a function of y /  is shown in Fig. 19.2. (Electric potential along the axis
that passes through the midpoint of a non-conducting rod.)
P hysi cs | 19.3

In the limit   y , the potential becomes


 2   
 (  / 2 ) +  / 2 1 + ( 2y /  )   1 + 1 + ( 2y /  )
2
λ λ 
V = ln  4 πε ln 
4 πεο  
 (
−  / 2 ) +  / 2 1 + ( 2y /  )   −1 + 1 + ( 2y /  )
2 2
ο 
  

λ  2  λ  2  λ 
≈ ln   = ln   = ln  
4 πεο  2y 2 / 2  4 πεο  y 2  2πεο  y 

∂V λ /2
The corresponding electric field can be obtained as Ey =
− =
∂y 2πεο y
(  / 2)
2
+ y2

Illustration 2: Consider a uniformly charged ring of radius R and charge 


density λ . What is the electric potential at a distance z from the central Z
axis?  (JEE MAIN)
r
The Fig. 19.3 shows a non-conducting ring of radius R with uniform
charge density λ Z
y

Sol: The point lies along the axis of the ring hence, the potential at the R
dq dp
point due to the charge on the ring is given by= ∫=
dV ∫ where
V x
4 πε r o Figure 19.3
r is the distance of point from ring. As the point lies on the z axis, the
∂V
field due to ring is given by EZ = − .
∂z
 Rdφ ' on the ring. The element carries a charge dq =
Consider a small differential element d= λ d =λRdφ ' , and its
1 dq 1 λRdφ '
contribution to the electric potential at =
P is dV =
4 πεο r 4 πεο R 2 + z2
1 λR 1 2πλR 1 Q
The electric potential at P due to the entire ring is =
V ∫ dV
= ∫ d=
φ' =
4 πεο 2
R +Z 2 4 πεο 2
R +z 2 4 πε ο R + z2
2

Where we have substituted Q = 2πRλ for the total charge on the ring. In the limit z  R,
1 Q
The potential approaches its “point-charge” limit: V≈
4 πεο z
∂V ∂  1 Q  1 Qz
The z-component of the electric field may be obtained as E2 =
− − 
= =
∂z  4 πεο 2 
∂z 4
( )
2 πε 3/2
R +z  ο R 2 + z2

Illustration 3: Consider a uniformly charged disk of radius R and charge density σ z

lying in the xy-plane. What is the electric potential at a distance z from the P
central axis?  (JEE ADVANCED)
dq 
r z
Sol: The disc can be assumed to be composed of many co-centric rings. Thus
dq
the potential due to small ring is dV = where r is the distance from the R
r’
y
4 πεor dr’
surface of ring. As the point lie on the z axis the field due to ring is given by
∂V
EZ = − . x
∂z Figure 19.4
1 9 . 4 | Electric Potential and Capacitance

Consider a small differential circular ring element of radius r’ and width dr’. The charge on the ring is
dq' =σdA' =σ ( 2πr 'dr ' ) . The field at point P located along the z-axis a distance z from the plane of the disk is to
(r )
1/2
'2
be calculated. From the Fig. 19.4, we also see that the distance from a point on the ring to P is=r + z2 .

1 dq 1 σ ( 2πr 'dr ' )


Therefore, the contribution to the electric potential at =
P is dV =
4 πεο r 4 πεο r' 2 + z 2

By summing over all the rings that make up the disk, we have
R
σ R 2πr 'dr ' σ  '2 2 σ  2
=V ∫ο = r + z= R + z2 − z 
4 πεο '2 2 2ε 
  ο 2ε 
 
r +z ο ο

1/2
2 2
 R2   R2 
In the limit z  R , R +Z = z 1 +  = z 1 + + ....  ,
 z 2   2z 2 
  
And the potential simplifies to the point-charge limit:

V≈
σ R2
. =
1 σ πR
=
2
( )
1 Q
2εο 2 z 4 πεο z 4 πεο z

As expected, at large distance, the potential due to a non-conducting charged disk is the same as that of a point
charge Q. A comparison of the electric potentials of the disk and a point charge is shown in Fig. 19.5.

V/VO
6
5
4
3 Point charge
2 Disk
1
Z/R’
-3 -2.5 -2 -1.5 -1 -0.5 0.5 1 1.5 2 2.5 3

Figure 19.5

The electric potential is measured in terms of=


Vο Q / 4 πεοR .
Note that the electric potential at the center of the disk (z=0) is finite, and its value is
σR Q R 1 2Q
Vc
= = .= = 2Vο
2εο πR 2εο 4 πεο R
2

This is the amount of work that needs to be done to bring a unit charge from infinity and place it at the center of
the disk.
The corresponding electric field at P can be obtained as:

∂V σ z z 
Ez =
− =  − 
∂Z 2εο  z R 2 + z2 

In the limit R  z, the above equation becomes E2 =
σ / 2εο , which is the electric field for an infinitely large non-
conducting sheet.

Illustration 4: Consider a metallic spherical shell of radius ‘a’ and charge Q, as shown in Fig. 19.6.
(a) Find the electric potential everywhere.
(b) Calculate the potential energy of the system. (JEE MAIN)
P hysi cs | 19.5

Sol: The sphere is symmetrical body. Thus the potential at the surface is constant while  ++++++
the potential changes outside of the sphere at a distance a < r < ∞ . ++ +
+  +
 Q + E=0 +
  r̂, r > a + +
E =  4 πεοr 2 + +
 + +
 0, r<a + +
+ +
B 
++ +
The electric potential may be calculated by V − V =− ∫ E.ds ++ ++
B A A ++++
r Q 1 Q Q Figure 19.6
For r>a, we have V (r ) − V ( ∞ ) =− ∫ dr ' = =k e

4 πεοr '2 4 πεο r r

We have chosen V ( ∞ ) =0 as our reference point. On the other hand, for r<a,  V
the potential becomes KeQ
KeQ r
a r
V (r ) − V ( ∞ ) =− ∫ drE (r > a) − ∫ drE(r < a)
a
∞ a

a Q 1 Q Q r
− ∫ dr
= = ke
= a

4 πεοr 2 4 πεο a a

A plot of the electric potential is shown in Fig. 19.7. Note that potential V is Figure 19.7
constant inside a conductor.

Illustration 5: An insulated solid sphere of radius a has a uniform charge density ρ . Compute the electric potential
everywhere.  (JEE MAIN)
Sol:
 Q
r̂, r > a a
  4 πε r 2
E= ο
 P2
 Qr r̂, r < a
 4 πε r 3 r
 ο
P1
For r > a,
Figure 19.8
r Q 1 Q Q
V1 (r ) − V ( ∞ ) =− ∫ dr ' = =k e

4 πεοr '2 4 πεο r r

On the other hand, the electric potential at P2 inside the sphere is given by V
a r a Q r Qr 3keQ
V2 (r ) − V ( ∞ ) =− ∫ drE (r > a) − ∫ E (r < a) =− ∫ dr − ∫ dr ' r' 2a
∞ a ∞ 2 a 3
4 πεοr 4 πεο a
 ke Q
1 Q r  2 a
=
1 Q

1 Q1 2
4 πεο a 4 πεο a 2
3
2
r − a= ( 
8πεο a 
)
3 − 2 
a 
a r
Q r2  Figure 19.9
= ke  3 − 
2a  a2 
A plot of electric potential as a function of r is given in Fig. 19.9.
1 9 . 6 | Electric Potential and Capacitance

2.1 Deriving Electric Field from the Electric Potential



In previous equations,
 we established the relation between E and V. If we consider two points which are separated
by a small distance ds , the following differential form is obtained:
 
dV = −E.ds
 
In Cartesian coordinates, E = Ex ˆi + Ey ˆj + E2kˆ and ds = dxiˆ + dyjˆ + dzkˆ , we have
dV = (E ˆi + E ˆj + E kˆ ) . (dxiˆ + dyjˆ + dzkˆ ) =
x y 2 Ex dx + Ey dy + Ez dz
Which implies

∂V ∂V ∂V
Ex =
− ,E =
− ,E =

∂x y ∂y z ∂z

By introducing a differential quantity called the “del (gradient) operator”


∂ ˆ ∂ ˆ ∂ ˆ
=
∇ i+ j+ k
∂x ∂y ∂z
The electric field can be written as
  ∂V ˆ ∂V ˆ ∂V ˆ   ∂ ∂ ∂ 
E = Ex ˆi + Ey ˆj + E2kˆ = −  i+ j+ k  = −  ˆi + ˆj+ kˆ  V = −∇V
 ∂x ∂y ∂z   ∂x ∂y ∂z 

E = −∇V

3. EQUIPOTENTIAL SURFACES
The equipotential surfaces in an electric field have the same basic idea as topographic maps used by civil engineers
or mountain climbers. On a topographic map, contour lines are drawn passing through the points having the
same elevation. The potential energy of a mass m does not change along a contour line as the elevation is same
everywhere.
By analogy to contour lines on a topographic map, an equipotential surface is a three dimensional surface on which
the electric potential V is the same at every point on it. An equipotential surface has the following characteristics.
(a) Potential difference between any two points in an equipotential surface is zero.
(b) If a test charge q0 is moved from one point to the other on such a surface, the electric potential energy q0V
remains constant.
(c) No work is done by the electric force when the test charge is moved along this surface.
(d) Two equipotential surfaces can never intersect each other because otherwise the point of intersection will
have two potentials which is of course not acceptable.
(e) As the work done by electric force is zero when a test charge is moved along the equipotential surface, it

follows that E must be perpendicular to the surface at every point so that the electric force qο E will always
be perpendicular to the displacement of a charge moving on the surface, causing the work done to be 0.
Thus, field lines and equipotential surfaces are always mutually perpendicular. Some equipotential surfaces
are shown in Fig. 19.10.
The equipotential surfaces are a family of concentric spheres for a point charge or a sphere of charge and are a
family of concentric cylinders for a line of charge or cylinder of charge. For a special case of a uniform field, where
the field lines are straight, parallel and equally spaced the equipotential are parallel planes perpendicular to the
field lines.
P hysi cs | 19.7

10V 10V
20V 20V
30V 30V
40V 40V 
E
+ -

40V 30V 20V

Figure 19.10

PLANCESS CONCEPTS

While drawing the equipotential surfaces we should keep in mind the two main points.
• These are perpendicular to field lines at all places.
• Field lines always flow from higher potential to lower potential.
Anurag Saraf (JEE 2011 AIR 226)

Illustration 6: Suppose the electric potential due to a certain charge distribution can be written in Cartesian
Coordinates as V ( x,=
y, z ) Ax2 y 2 + Bxyz where A,B and C are constants. What is the associated electric field?
 (JEE MAIN)
∂V
Sol: The field is given by Er = − where r is the respective Cartesian co-ordinate.
∂r
∂V ∂V ∂V
Ex = − = −2Axy 2 − Byz ; Ey = − =−2Ax2 y − Bxz ; Ez = − = −Bxy
∂x ∂y ∂z

Therefore, the electric field is E = ( ) ( )
−2Axy 2 − Byz ˆi − 2Ax2 y + Bxz ˆj − Bxykˆ

4. ELECTRIC POTENTIAL ENERGY


The electric force between two charges is directed along the line of the charges and is proportional to the inverse
square of their separation, the same as the gravitational force between two masses. Like the gravitational force, the
electric force is conservative, so there is a potential energy function U associated with it.
When a charged particle moves in an electric field, the field exerts a force that can do work on the particle. This
work can always be expressed in terms of electric potential energy. Just as gravitational potential energy depends
on the height of a mass above the earth’s surface,
 electric potential energy depends on the position of the charged
particle in the electric field. When a force F acts on a particle that moves from point a to point b, the work Wa→b
b  b
done by the force is given by, = Wa→b ∫= F.ds ∫ F cos θ ds
a a

Here Wa→b is the work done in displacing the particle from a to b by the conservative force (here electrostatic)
not by us. Moreover we can see from Eq. (i) that if Wa→b is positive, the change in potential energy ∆U is negative
and the potential energy decreases. So, whenever the work done by a conservative force is positive, the potential
energy of the system decreases and vice-versa. That’s what happens when a particle is thrown upwards, the work
done by gravity is negative, and the potential energy increases.
1 9 . 8 | Electric Potential and Capacitance

4.1 Potential Energy in a System of Charges


If a system of charges is assembled by an external agent, then ∆U =−W =+ Wext .

P
That is, the change in potential energy of the system is the work that must be put
in by an external agent to assemble the configuration. A simple example is lifting a
mass m through a height h. The work done by an external agent you, is +mgh (The q2
gravitational field does work -mgh). The charges are brought in from infinity without r12
acceleration i.e. they are at rest at the end of the process. Let’s start with just two
charges q1 and q2. Let the potential due to q1 at a point P be V1 (See Fig. 19.11).

The work W2 done by an agent in bringing the second charge q2 from infinity to P q1
is then W2 = q2 V1 . (No work is required to set up the first charge and W1 = 0). Since
V1 q1 / 4 πε0r12 , where q1 and r12 is the distance measured from q1 to P, we have
= Figure 19.11

1 q1q2
U=
12 W
=2
4 πε0 r12
If q1 and q2 have the same sign, positive work must be done to overcome P
the electrostatic repulsion and the potential energy of the system is
positive, U12 > 0. On the other hand, if the signs are opposite, then r23
U12 < 0 due to the attractive force between the charges. To add a q2
r12
third charge q 3 to the system, the work required is
q3  q1 q2 
W3= q3 ( V1 + V2 )=
q2
 + 
4 πεο  r13 r23 
r13
The potential energy of this configuration is then q1

1  q1q2 q1q3 q2q3  Figure 19.12


U = W2 + W3 =  + +  = U12 + U13 + U23
4 πεο  r12 r13 r23 

The equation shows that the total potential energy is simply the sum of the contributions from distinct pairs.
1 N N qiq j
Generalizing to a system of N charges, we have U = ∑∑
4 πεο =i 1=j 1 rij
j >i
Where the constraint j>i is placed to avoid double counting each pair. Alternatively, one may count each pair twice
and divide the result by 2. This leads to
 
1 N N qiq j 1 N  1 N q j  1 N
=U = ∑∑
8πεο =i 1=j 1 rij
∑ q=  ∑  ∑ q V (r )i
2=i 1 i  4 πεο =j 1 rij  2=i 1 i
 
j≠i  j≠i 

Where V (ri ) , the quantity in the parenthesis is the potential at ri (location of qi) due to all the other charges.

4.2 Continuous Charge Distribution



q
If the charge distribution is continuous, the potential at a point P can be found by summing 
>

r
over the contributions from individual differential elements of charge dq.

Consider the charge distribution shown in Fig. 19.13. Taking infinity as our reference point with
1 dq r
zero potential, the electric potential at P due to dq is dV =
4 πεο r
P
1 dq 
Summing over contributions from all differential elements, we have V = ∫ E
4 πεο r
Figure 19.13
P hysi cs | 19.9

Illustration 7: Find the potential due to a uniformly charged sphere of radius R and charge 
per unit volume ρ at different points in space.  (JEE MAIN) 

R
q ρR3
Sol: The field due to uniformly charged solid sphere
= is E = . The potential
4 πεor 2 3εor 2
inside the sphere is always constant while outside sphere at a distance R < r < ∞ is given
r r Figure 19.14
by  V1  =
R ∫R Edl
For a point outside the sphere, we can integrate the electric field outside to obtain an expression for the electric
field outside.
r  
3
r ρR ρR 3
∫ V= − ∫ E.dl =

−∫

3εο x2
dx =
3εοr

The dots represent a spherically symmetric distribution of charge of radius R, whose 


volume charge density ρ is a constant.
For finding the potential at an inside point, we integrate as before. Keep in mind
that the lower limit in this integration cannot be infinity, as infinity does not lie v3kQ
inside the sphere. But lower limit can be assumed to be R where potential is not 2R
zero but a known value.

Vin r   r ρx (
ρ R2 − r2 ) ;V = ρR 2 (
ρ 3R 2 − r 2)
∫VR
dV − ∫ E.dl =
=
R
−∫
R 3ε
dx =
6ε ο R
3εο
⇒ Vr =
6ε ο
.
r
ο -R R
This potential is plotted in Fig. 19.15. Figure 19.15

Illustration 8: A non-conducting disc of radius a and uniform positive surface charge density σ is placed on the
ground with its axis vertical. A particle of mass m and positive charge q is dropped, along the axis of the disc from
a height H with zero initial velocity. The particle has q / m =ε
4 οg / σ
(a) Find the value of H if the particle just reaches the disc.
(b) Sketch the potential energy of the particle as a function of its height and find its equilibrium position. 
 (JEE ADVANCED)

Sol: It is known that, for non conducting disc the potential at a point situated at height H above  P qm
σ  2
the disc is given by=
Vp a + H2 − H where a is the radius of the disc. When the charge of
2εο   H
the particle of mass m falls along the axis of the disc, the change in the gravitational portential
energy is equal to gain in its electric potential energy. As we required minimum H, the kinetic a O
energy, will be zero.
σ  2 Figure 19.16
As we have derived in the theory,=
Vp a + H2 − H
2εο  
σa
Potential at centre, (O) will be Vο = (H=0)
2εο
(a) Particle is released from P and it just reaches point O. Therefore, from observation of mechanical energy
0 because K=
Decrease in gravitational potential energy = increase in electrostatic potential energy ( ∆KE = i K=
f 0)

 q  σ  2 2 
or gH   
∴ mgH = q  Vο − Vρ  =  a − a + H + H  … (i)

 m   2εο   
1 9 . 1 0 | Electric Potential and Capacitance

q 4εο g qσ
= ∴ 2g
=
m σ 2εοm
Substituting in Eq. (i), we get

H H
= 2g a + H − a2 + H2  or = ( a + H) − a2 + H2 or
gH a2 + H2 =a +
  2 2
H2 3 4
or a2 + H2 = a2 +
4
+ aH or H2 = aH or H = a and H = 0 ∴ H =
4 3
( 4 / 3) a
(b) Potential energy of the particle at height H = Electrostatic potential energy + gravitational potential energy
U qV + mgH
=
Here V=potential at height H
σq  2
=U a + H2 − H + mgH  … (ii)
2εο  

−dU
At equilibrium position,
= F = 0
dH
Differentiating E.q. (ii) w.r.t.H

σq  1  1   σq 
or mg +   ( 2H) − 1 =0  = 2mg
2εο  2  a2 + H2   2εο 

 H  2H
∴ mg + 2mg  − 1 =0 or 1 + −2 =0
 a2 + H2  a2 + H2

2H H2 1 a
= 1 or = or 3H2 = a2 or H =
2
a +H 2 2
a +H 2 4 3
U
From Eq. (ii), we can see that,
a
U = 2mga at H = 0 and
= U U=
min 3mga at H = 2mga
3
Therefore, U-H graph will be as shown. 3mga
a
Note that at H = , U is minimum.
3 O aA 3 H
a
Therefore, H = is stable equilibrium position. Figure 19.17
3

5. ELECTRIC DIPOLE
An electric dipole is a system of equal and opposite charges separated
 by a fixed distance. Every electric dipole is
characterized by its electric dipole moment which is a vector P directed from the negative to the positive charge.
The magnitude of dipole moment is, P=(2a)q; Here, 2a is the distance between the two charges.

5.1 Electric Potential and Field Due to an Electric Dipole



Consider an electric dipole lying along positive y-direction with its center at origin. -q P
+q

P = 2aqjˆ 2a
Figure 19.18
P hysi cs | 19.11

The electric potential due to this dipole at point A(x, y, z) as shown y


is simply the sum of the potentials due to the two charges. Thus, 2 2
x + z + (y-a)
2 A (x, y, z)
 
1  q q 
V  − 
+q
4 πεο  2 +
x + ( y − a) + z2 x2 + ( y + a) + z2
2 2 2 2 2
 x + z + (y+a)
  a

By differentiating this function, we obtain the electric field of the a


dipole. -q -
  z
 
∂V q  x x  Figure 19.19
Ex
= =  −
∂x 4 πεο   2 3/2 3/2 
2  x2 + y + a 2 + z2  
  x + ( y − a) + z  ( )
2
  

 
 
∂V q  y − a y + a 
E= =  −
y
∂y 4 πεο   2 3/2 3/2 
2  x2 + y + a + z2  
  x + ( y − a) + z  ( )
2 2
  

 
 
∂V q  z x 
Ez
= =  − 
∂z 4 πεο   2 2
3/2
 2
3/2

  x + ( y − a) + z  x + ( y + a) + z
2 2 2
  
Special Cases
(i) On the axis of the dipole (say, along y-axis) X=0, z=0

q  1 1  2aq
∴V
=  =
− 
4 πεο  y − a y + a  4 πεο y 2 − a2 ( )
p
or V= (as 2aq = p) i.e., at a distance r from the Centre of the dipole(y=r)
4 πεο y 2 − a2( )
p p
V≈ or Vaxis = (for r  a )
( 2
4 πεο r − a 2
) 4 πεοr 2

V is positive when the point under consideration is towards positive charge and negative if it is towards negative charge.
Moreover the components of electric field are as under,

Ex 0,E
= = z 0 (as x = 0, z = 0)

 
q  1 1  4ayq 1 2py
and Ey
= − = or Ey =
4 πεο  y − a
 ( ) ( y + a)  4πεο y 2 − a2 ( ) 4 πεο
(y )
2 2 2 2
2
− a2

Note that Ey is along positive y-direction or parallel to P . Further, at a distance r from the centre of the dipole (y=r).

1 2pr 1 2p
Ey = or Eaxis ≈ . for r  a
4 πεο
(r ) 4 πεο r3
2
2
− a2

(ii) On the perpendicular bisector of dipole


Say along x-axis (it may be along z-axis also). y = 0, z = 0
1 9 . 1 2 | Electric Potential and Capacitance

1  q q 
=∴V  =
−  0 or V⊥ bi sector = 0
4 πεο  x2 + a2 x2 + a2 

Moreover the components of electric field are as under,=


Ex 0=
Ez 0
 
q  −a a  −2aq 1 P
and
= Ey =  − = ; Ey = .
3/2 
4 πεο  2
( ) ( ) ( ) 4 πεο
(x )
3/2 3/2 3/2
x + a2 x2 + a2  4 πεο x2 + a2 2
+ a2
 

Here, negative sign implies that the electric field is along negative y-direction or antiparallel to P . Further, at a
distance r from the Centre of dipole (x = r), the magnitude of electric field is,

1 P 1 P
E= or E⊥ bisector ≈ . (for r  a )
4 πεο
(r ) 4 πεο r3
3/2
2
+ a2

5.2 Force on Dipole



Suppose an electric dipole of dipole moment P = 2aq is  
 E
placed in a uniform electric field E at an angle θ . Here, θ +q  
    F1 P
is the angle between P and E . A force F1 = qE will act on a
A
positive charge and
    O
F2 = −qE on negative charge. Since, F1 andF2 are equal in a

   -q

magnitude but opposite in direction. Hence, F1 + F2 = 0 or F2
E

Fnet = 0 B

Thus, net force on a dipole in uniform electric field is zero. Figure 19.20
While in non-uniform electric field it may or may not be zero.

5.3 Torque on Dipole


     
The torque of F1 about O, τ1= OA × F1= q OA × E ( )
       
And torque of F2 about O is, τ 2 =OB × F 2 =−q OB = ×E q BO × E ( ) ( )
The net torque acting on the dipole is,
              
( ) (
τ = τ1 + τ2 = q OA × E + q BO × E = q OA × BO = ) (
× E q BA × E ) ( ) or τ =P xE

Thus, the magnitude of torque= is τ PEsin θ . The direction of torque is perpendicular to the plane of
 paper inwards.
Further this torque is zero at θ =0 or θ =180 , i.e., when the dipole is parallel or antiparallel to E and maximum
ο ο

at θ =90ο .

5.4 Potential Energy of Dipole


   
When an electric dipole is placed in an electric field E , a torque τ = P × E acts on it.  +q
If we rotate the dipole through a small angle dθ , the work done by the torque is, o
90
dW =τdθ ; dW =−PEsin θdθ
The work is negative as the rotation dθ is opposite to the torque. The change in
electric potential energy of the dipole is therefore. -q
dU =−dW =PEsin θdθ Figure 19.21
P hysi cs | 19.13

Now, at angle θ = 90°, the electric potential energy of the dipole may be assumed to be zero as net work done by
the electric forces in bringing the dipole from infinity to this position will be zero.
Integrating, =
dU PEsin θdθ

From 900 to θ , we have =


θ
∫ ο dU
90
θ
∫90ο PEsin θdθ ( )
or U ( θ ) − U 90ο = PE  − cos θ  ο
θ
90
 
∴ U ( θ ) = −PEcos θ = −P ⋅ E
If the dipole is rotated from an angle θ1 to θ2 then

Work done by external forces = U ( θ2 ) − U ( θ1 ) or Wext.forces =


−PEcos θ2 − ( −PEcos θ1 )

= PE ( cos θ1 − cos θ2 ) and work done by electric forces, Welectric force =− Wext.force =PE ( cos θ2 − cos θ1 )
Or Wext.forces

Illustration 9: Figure 19.22 is a graph of Ex, the x component of the electric field,  EX(V/m)
versus position along the x axis. Find and graph V(x). Assume V = 0; V at x = 0m.  2000
(JEE MAIN)
1000 V=-Area
Sol: The potential V(x) is given by V=E•dx=Area of the shaded region of the graph.
The potential difference is the negative of the area under the curve. 0 x(m)
 0 1 x
Ex is positive throughout this region of space, meaning that E points in the positive
x direction. Figure 19.22

If we integrate from x-0, then Vi = V(x − 0) − 0 . The potential for x>0 is the negative of the triangular area under the
Ex curve. We can see that Ex = 1000xV / m , where x is in meters(m). Thus,
Vt= V ( x )= 0 − (Area under the Ex curve) v(v)
x
1 1
− ( x )(1000x ) =
1 2
× base × height = −500x2 V.
2 2
Figure 19.23 shows that the electric potential in this region of space is parabolic, 1000
decreasing from 0 V at x=0 m to -2000V at x=2m. v(x) = -500x V
2

2000
The electric field points in the direction in which V is decreasing. We’ll soon see that
as this is a general rule.
Figure 19.23

Illustration 10: The electric potential at any point on the central axis of a uniformly charged disk is given by
σ  2 2 
= V  z + R − z .
2εο  
Starting with this expression, derive an expression for the electric field at any point on the axis of the disk. 
 (JEE MAIN)
∂V
Sol: As the point lie on the z axis the field due to ring is given by EZ = − .
∂z

Conceptualize/Classify: We want  the electric field E as a function of distance z along the axis of the disk. For
any value of z, the direction of E must be along that axis because the disk has circular symmetry about that axis.

Thus, we want the component Ez of E in the direction of z. This component is the negative of the rate at which the
electric potential changes with distance z.
Compute: Thus, from the previous equations, we can write

∂V σ d 2 σ  z 
Ez =
− =
− 2
 z +R =− z  ; 1 − .
∂z 2εο dz   
2εο  
z + R2
2

1 9 . 1 4 | Electric Potential and Capacitance

6. CAPACITOR
A capacitor is a combination of two conductors placed close to each other. It is used to store energy electrostatically
in an electric field. The ‘non-conducting’ dielectric acts to increase the capacitor’s charge capacity. Capacitors are
widely used as parts of electrical circuits in many common electrical devices. Unlike a resistor, a capacitor does not
dissipate energy. Instead, a capacitor stores energy in the form of an electrostatic field between its plates.
The physics of capacitors can be utilized to any scenario involving electric fields. For example, Earth’s atmospheric
electric field is analyzed by meterologists as being produced by a huge spherical capacitor that partially discharges
via lightning. The charge that is collected as they slide along snow can be modeled as being stored in a capacitor
that frequently discharges as sparks.

7. CAPACITANCE
There are 2 conductors in a capacitor. One conductor has positive charge (positive plate)
and the other has an equal and opposite negative charge (negative plate). The charge
on the positive plate is called the charge on the capacitor and the potential difference
between the plates is called the potential of the capacitor. For a given capacitor, the
charge Q on the capacitor is proportional to the potential difference V between the Figure 19.24
plates Thus, Q∝V or, Q=CV.
The proportionality constant C is called the capacitance of the capacitor. It depends on the shape, size and
geometrical placing of the conductors and the medium between them.
The SI unit of capacitance is coulomb per volt which is written as Farad. The symbol F is used for it.

Illustration 12: A Capacitor gets a charge of 60µC when it is connected to a battery of emf 12V. Calculate the
capacitance of the capacitor. (JEE MAIN)
Q
Sol: The capacitance is given by C =
V
The potential difference between the plates is the same as the emf of the battery which is 12V.
Q 60µC
Thus, the capacitance is C = = = 5µF.
V 12V

8. CALCULATING CAPACITANCE
To calcutate the capacitance of a capacitor once we know its geometry:-
(a) Assume a charges q and –q on the plates ;
(b) Calculate the electric field E between the plates in terms of this charge, using Gauss’ law;
(c) Knowing E , calculate the potential difference V between the plates.
(d) Calculate C.

9. TYPES OF CAPACITORS

9.1 Parallel Plate Capacitor


d
A parallel-plate capacitor contains two large plane plates parallel to each other  +Q
with a small separation between them. Suppose, the area of each of the surfaces -Q
is A and the separation between the two plates is d. Also, assume that vacuum
fills the space between the plates. Figure 19.25
P hysi cs | 19.15

Q
The magnitude of charge density on each of these surfaces is given by σ =
A
B
Let us draw a small area A parallel to the plates and in between them,  V+ A’ +Q
draw a cylinder with A as a cross-section and terminate it by another + + + + + + + +
symmetrically situated area A’ inside the positive plate. The flux through d A
A’ and through the curved part inside the plate is zero as the electric E
- - - - - - - -
field is zero inside a conductor. The flux through the curved part outside V- A -Q
the plates is also zero as the direction of the field E is parallel to this
surface. Figure 19.26

  Q
The flux through A is φ = E. ∆ A = E∆A . The only charge inside the Gaussian surface is ∆Q = σ∆A = ∆A.
A
  Q Q
From Gauss’s law, ∫ E.dS
= Qin / ε0 or, E∆A= ∆A ; or, E = .
ε0 A ε0 A
B 
The potential difference between the plates is V = V+ − V− =− ∫ E.dr.
A
  B
Qd
E.dr = ∫ E dr =
−Edr and V = Ed =
ε0 A
A
Q Qε0 A ε0 A
The capacitance of the parallel-plate capacitor is =
C = = .
V Qd d

Illustration 13: Calculate the capacitance of a parallel-plate capacitor having 20cm x 20cm square plates separated
by a distance of 1.0mm.  (JEE MAIN)
ε0 A
Sol: The capacitance is given by C = .
d
ε A 8.85 × 10−12 Fm−1 × 400 × 10−4 m2
The capacitance is C =0 = 3.54 × 10−10 F =
= 350pF.
d 1 × 10−3 m

9.2 Cylindrical Capacitor V-


A cylindrical capacitor consists of a solid or a hollow cylindrical conductor  V+
surrounded by another coaxial hollow cylindrical conductor. Let l be the length of
the cylinders and R1 and R2 be the radii of the inner and outer cylinders respectively.
If the cylinders are long as compared to the separation between them, the electric
field at a point between the cylinders will be radial and its magnitude will depend x
P
L
R1
only on the distance of the point from the axis. E r R2

To calculate the electric field at the point P, at a distance r, draw a coaxial cylinder
Q
of length x through the point. A Gaussian surface is made by the cylinder and its
two cross sections. The flux through the cross sections is zero as the electric field
is radial wherever it exists and hence is parallel to the cross sections. -Q
 
The flux through the curved part is φ= E∫ dS= E2πrx = ⇒ φ ∫= E .dS ∫ EdS . Figure 19.27

Q Q  Q
The charge enclosed by the Gaussian surface is =
Qin x. , E2=
πrx  x  / ε0 or,
= E
l  l  2 πε 0rl

B  R1
Q Q R
The potential difference between the cylinders is V = − ∫ E.dr =
V+ − V− = − ∫ dr = ln 2 .
A R2
2πε0rl 2πε0l R1
Q 2πε0l
The capacitance is =
C =
V ln R / R
2 ( 1
)
1 9 . 1 6 | Electric Potential and Capacitance

9.3 Spherical Capacitor


A Spherical Capacitor consists of two concentric spherical shells, of radii a and b. As a Gaussian surface we draw a

sphere of radius r concentric with the two shells q = ε0EA = ( )


ε0E 4 πr 2 , in which the area of the spherical Gaussian
1
surface is given by 4 πr 2 . We solve equation for E, obtaining E = , which we recognize as the expression for
4 πε0r 2
the electric field due to a uniform spherical charge distribution
+
q a dr q 1 1  q b −a
∫ Eds =
V= − ∫b r2 =  − =   , Substituted – dr for ds

4 πε0 4 πε0  a b  4 πε0  ab 

ab
C= 4 πε0 (Spherical capacitor).
b−a

Isolated Sphere
We can assign a capacitance to a single isolated spherical conductor of radius R by assuming that the “missing
a
plate” is a conducting sphere of infinite radius. C= 4 πε0 .
1−a/b
If we then let b ∞ and substitute R for a, we find C= 4 πε0R (isolated sphere)

PLANCESS CONCEPTS

• Earth can be considered as a capacitor with infinite capacitance.


• When a conductor is grounded and charge flows into the earth, the ground is still at potential zero.
GV Abhinav (JEE 2012 AIR 329)

Illustration 14: Three identical metallic plates are kept parallel to one another at a separation of a and b. The outer
plates are connected by a thin conducting wire and a charge Q is placed on the central plate. Find final charges on
all the six surfaces. (JEE ADVANCED)
Sol: The charge on any plate is computed using q = CV where C is the capacitance  1 2 3 4 5 6

of the plate and V is the potential difference applied on it. The chrge induced in
plates A and C are such that total q(A) + q(C) =0. Plates A and C are at the same
potential.
Let the charge distribution in all the six faces be as shown in Fig. 19.28. While a b
distributing the charge on different faces, we have used the fact that opposite faces
have equal and opposite charges on them.
Figure 19.28
Net charge on plates A and C is zero.
0 Or q2 + q3 =
Hence, q2 − q1 + q3 + q1 − Q = Q ….. (i) 
(Q-q1)
q2 -q2 -q1 q3
Further A anc C at same potentials. Hence, VB − VA = VB − VC or E1a = E2b
(q-Q1)
q1 Q − q1 Qb
.a = .b (A= Area of plates); q1a = ( Q − q1 ) b ∴ q1 = ….. (ii)
Aε0 Aε0 a+b
E1
Electric field inside any conducting plate (say inside C) is zero. Therefore, E2
q2 q1 q1 Q − q1 q1 − Q q3 a b
− + + + − 0
=
2Aε0 2Aε0 2Aε0 2Aε0 2Aε0 2Aε0
Figure 19.29
P hysi cs | 19.17

Qb Q
Solving these three equations, we get q=
1 , q=
2 q=
3
a+b 2

10. COMBINATION OF CAPACITORS


+C- +C- +C- +C-n
The desired value of capacitance can be obtained by combining more than one  Q Q Q Q
capacitor. We shall discuss two ways in which more then one capacitor can be V1 V2 V3 Vn
connected.

10.1 Series Combination of capacitors:


+ -
There is equal amount of charge Q deposited on each capacitor; but the potential V
difference between their plates is different (if the capacitance of each of the Figure 19.30
capacitors is different)
Q Q Q Q Q
V = V1 + V2 + V3 + .......... + Vn = + + + ..... + (from C = ) ;
C1 C2 C3 Cn V
V 1 1 1 1
∴ = + + + ..... +
Q C1 C2 C3 Cn

If C is the resultant capacitance of C1, C2……, Cn which are connected

V 1 1 1 1 1 1
In series then = ; ∴ = + + + ..... +
Q C C C1 C2 C3 Cn

PLANCESS CONCEPTS

The value of C is smaller than the smallest of the values of the capacitors which are connected in series.
B Rajiv Reddy (JEE 2012 AIR 111)

10.2 Parallel Combination of Capacitors


In this arrangement of the capacitors the charge accumulated on each of the capacitors is different while the
potential difference between them is the same and is equal to potential difference between the common points A
and B of such a connection.
Now,
= Q1 C=
1 V,Q 2 C=
2 V,Q3 C3 V
The total electric charge, Q = Q1 + Q2 + Q3 = ( C1 + C2 + C3 ) V
Q
In general, C = = C1 + C2 + C3 + .....,
V
Hence, by connecting capacitors in a parallel combination, a resultant capacitance can be obtained, whose value is
equal to the sum of all capacitances connected in parallel.
•• When a potential difference V is applied across several capacitors connected in parallel, it is applied across each
capacitor. The total charge q stored on the capacitors is the sum of the charges stored on all the capacitors.
•• Capacitors connected in parallel can be replaced with an equivalent capacitor that has the same total charge q
and the same potential difference V as the actual capacitors.
•• When a potential difference V is applied across serveral capacitors connected in series, the capacitors have
identical charge q. The sum of the potential differences across all the capacitors is equal to the applied potential
difference V.
1 9 . 1 8 | Electric Potential and Capacitance

Illustration 15: Three capacitors each of capacitance 9pF are connected in series.
(a) What is the total capacitance of the combination?
(b) What is the potential difference across each capacitor if the combination is connected to a 120V supply?
 (JEE MAIN)
−1
 1 1 1 
Sol: The equivalent capacitance in series combination is given by Ceq = 
C
+ +  .
 1 C2 C3 
q
The potential difference applied across each capacitor is given by V = where C is the capacitance of the capacitor.
C
1 1 1 1 1 1 1 1
(a) Here, = + + = + + = 3x
Ceq C1 C2 C3 C C C C
1 1 1 −12
i.e. = 3x = =
or Ceq 3x10
= F 3pF.
Ceq 9x10 −12 3x10 −12

q q q  q q
(b) Here V1 + V2 + V3 =
120 i.e. + += 120 ∴C
= or
= V 
C1 C2 C3  V C
3q 3xq
i.e. = 120 i.e. = 120 or q = 360x10−12 F
C 9x10 −12

q 360x10−12
∴ P.D. across a capacitor= = = 40V
C 9x10−12
Short cut
120
Since three equal capacitors are across 120V, so p.d. across each capacitor
= = 40V
3

10 F 20 F
Illustration 16: Calculate the charge on each capacitor shown in Fig. 19.31. (JEE MAIN)
C1C2
Sol: The equivalent capccitance is given by C =
C1 + C2
The two capacitors are joined in series. Their equivalent capacitance

1
Is given by =
1
+
1
or, =
C
C1C2
=
(10µF )( 20µF=) 20
µF.
30 V
Figure 19.31
C C1 C2 C1 + C2 30µF 3

The charge supplied by the battery is Q = CV


In series combination, each capacitor has equal charge and this charge equals the charge supplied by the battery.
 20 
 µF  ( 30V ) =µ
Thus, each capacitor has a charge of 200 µC . = 200 F
 3 

Illustration 17: Find the equivalent capacitance of the combination  P


shown in Fig. 19.32 between the points P and N. (JEE MAIN)
10 F 20 F
Sol: The circuit is the combination of series and parallel connection of
capacitors. The equivalent capacitance of series combination is given Q 30 F
CeqC2 N
by C = whereas the equivalent capacitance of the parallel
Ceq + C2 Figure 19.32

connection is given by Ceq


= C1 + C3 .
P hysi cs | 19.19

The 10µF and 20µF capacitors are connected in parallel. Their equivalent capacitance is 10µF +20µF = 30µF.
We can replace the 10µF and the 20µF capacitors by a single capacitor of capacitance 30µF between P and Q.

This is connected in series with the given 30µF capacitor. The equivalent capacitance C of this combination is given
1 1 1
by = + or,C = 15µF .
C 30µF 30µF

11. ENERGY STORED IN AN ELECTRIC FIELD OF A CAPACITOR


The work that needs to be done by an external force to charge a capacitor is visualized as electric potential energy
U stored in the electric field between the plates. This energy can be recovered by discharging the capacitor in a
circuit.
Suppose that, at a given instant, a charge q’ has been transferred from one plate of a capacitor to the other. The
potential difference V’ between the plates at that instant is q’/C. The work required to increase the charge by dq’ i
q
dW V'dq'
= = dq'
C
1 q q2
The work required to bring the total capacitor charge up to a final value q is
= w ∫= dW ∫ q'dq'
= .
C 0 2C

q2
This work is stored as potential energy U in the capacitor, so that we can also write this as U = (Potential energy).
2C
•• The potential energy of a charged capacitor may be viewed as being stored in the electrc field between its
plates.

Illustration 18: A parallel plate air capacitor is made using two plates 
+ - + -
0.2 m square, spaced 1 cm apart. It is connected to a 50V battery.
 (JEE ADVANCED)
+ - + -
(a) What is the capacitance? E E
(b) What is the charge on each plate? + - + -
d1 d2
(c) What is the energy stored in the capacitor?
(d) What is the electric field between the plates? Figure 19.33

(e) If the battery is disconnected and then the plates are pulled apart to a separation of 2cm, what are the
answers to the above parts?

Sol: The capacitance, charge on capacitor, energy stored in capacitor, the electric field between plates of capacitor,
εo A 1 2 V
are given=by C = ; Q CV;
= U CV and = E respectively.
d 2 d
ε0 A 8.85X10−12 X0.2X0.2
(a) =
C0 = = ; C0 3.54X10−5 µF
d0 0.01

(b) Q=
0 C0 V
=0 (3.54X10 −5 X50)µ=
C 1.77X10 −3 µC

1
(c) U0
= = C V 2 1 / 2(3.54X10−11 )(50)2 ; U0 = 4.42X10−8 J.
2 0 0
V0 50
(d) E=
0 = = 5000V / m.
d0 0.01
1 9 . 2 0 | Electric Potential and Capacitance

(e) If the battery is disconnected, the charge on the capacitor plates remain constant while the potential difference
between plates can change.
Aε0 Q Q0
C
= = : 1.77X10−5 µF ; =
Q Q= 1.77X10 −3 µC ; V
= = = 2V= 100Volts. ;
2d0 0
C C0 / 2 0

2
1 Q2 1 Q0 2V0
U
= = = 2U = 8.84X10−8 J , =
E = E= 5000V / m.
2 C 2 ( C0 / 2 ) 0
2d0 0

Work has to be against the attraction of plates when they are separated. This gets stored in the energy of the
capacitor.

Illustration 19: Find the energy stored in a capacitor of capacitance 100 µF when it is charged to a potential
difference of 20V.  (JEE MAIN)
1 2
Sol: The energy stored in the capacitor is given by U = CV .
2
1 1
The energy stored in the capacitor is U =CV 2 =(100µF )( 20V ) =
2
0.02J
2 2

C C
Illustration 20: Prove that in charging a capacitor half of the  + -
S S
energy supplied by the battery is dissipated in the form of q

heat.  (JEE ADVANCED)
V
Sol: When the swich S is closed, q=CV charge is stored in the
V +
capacitor. Charge transferred from the battery is also q. Hence, q
Figure 19.34
energy supplied by the battery = qV = (CV)(V) = CV 2 Half, of
1 1
its Energy, i.e., CV 2 is stored in the capacitor and the remaining 50% or CV 2 is dissipated as heat.
2 2

11.1 Energy Density


Neglecting fringing, the electric field in a parallel-plate capacitor has same value at all points between the plates.
The energy density u, which is the potential energy per unit volume between the plates should also be uniform. We
U CV 2
can find u by dividing the total potential energy by the volume Ad of the spacebetween
betweenthe
thePlat
plates
es =
u = ,.
2
Ad 2Ad
1 V
C = ε0A/d, this result becomes u= ε  
2 0d
1
V/d equals the electric field magnitude E; so u= ε E2 (Energy density )
2 0
Although we derived this result for the special case of a parallel-plate capacitor, it holds generally, whatever may
be the source of the electric field.
P hysi cs | 19.21

12. FORCE BETWEEN THE PLATES OF A CAPACITOR


q -q
Consider a parallel plate capacitor with plate area A. Suppose a positive charge q is given to  + -
one plate and a negative charge – q to the other plate. The electric field on the negative plate + -
due to positive charge is + -
σ q q
+ -
=E =  σ= + -
2ε0 2Aε0 A
+ -
q2 -
The magnitude of force on the charge in negative plate is =
F qE
= +
2Aε0 Figure 19.35
q2
This is the force with both the plates attract each other. Thus, F =
2Aε0

13. DIELECTRICS
If the medium between the plates of a capacitor is filled with an insulating substance (dielectric), the electric field
due to the charges plates induces a net dipole moment in 
the dielectric. This effect called polarization, gives rise to + -
+ - +- + -
a field in the opposite direction. Due to this, the potential +
+
-
-
+ -
+- + -
difference between the plates is reduced. Consequently, the +
+
-
-
+ -
+- + -
capacitance C increases from its value C0 when there is no + - +- + -
+ - + -
medium (vacuum). The dielectric constant K is defined by + - +- + -
+ - + -
Cdielectric + - +- + -
C + -
the relation
= K = + -
+ -
+- + -
C vacuum C0 + -
(a) (b)
Where C vacuum = capacity of a capacitor when there is Electric field lines with The induced charges on
vacuum between the plates. Thus, the dielectric constant of vacuum between the the faces of the dielectric
a substance is the factor by which the capacitance increases plates reduce the electric field

from its vacuum value, when dielectric is inserted fully Figure 19.36
between the plates.

13.1 Dielectrics at an Atomic View


What happens, in atomic and molecular terms, when we put a dielectric in an electric field? There are two possibilities,
depending on the type of molecule:-
(a) Polar Dielectrics: The molecules of some dielectrics, like water, have permanent electric dipole moments.
In such cases, the electric dipoles tend to line up with an external electric field because the molecules are
continuously jostling each other as a result of their random thermal motion. This alignment becomes more
complete as the magnitude of the applied field is increased (or as the temperature, and thus the jostling, are
decreased). The alignment of the electric dipoles produces an electric field that is directed opposite to the
applied field and is smaller in magnitude.

(b) Nonpolar moments: Even nonpolar molecules acquire dipole moments by induction when placed in an
external electric field. This occurs because the external field tends to “stretch” the molecules, slightly separating
the centres of negative and positive charge.
1 9 . 2 2 | Electric Potential and Capacitance

+
+

+
+
+

+
+ +

+
+

+
+

+
+

+
+

+
+

+
+

(a) (b)
Figure 19.37

Molecules with a permanent electric dipole An electric field is applied producing partial
moment, showing their random orientation in alignment of the dipoles. Thermal agitation
the absence of an external electric field. prevents complete alignment

+ - + -
+ -+ -+ -+ -+ - + + -
+ - + -
+ - +   -
-+ -+ -+ -+ E0 E +
 + - + -
E0=0 +  - + -
E0 
+ -+ -+ -+ -+ - + E0 + -
+ - + -
(a) (b) (c)

Figure 19.38

(i) A nonpolar dielectric slab. The circles represent the electrically neutral atoms within the slab.
(ii) An electric field is applied via charged capacitor plates; the field slightly stretches the atoms, separating the
centres of positive and negative charge.

(iii) The separation produces surface charges on the slab faces. These charges set up a field E ’, which opposes
   
the applied field E0 . The resultant field E inside the dielectric (the vector sum of E0 and E ’) has the same

direction as E0 but a smaller magnitude.

PLANCESS CONCEPTS

Real Capacitors
• Real capacitors used in circuits are parallel plate capacitors with a dielectric material inserted in
between the plates and rolled into a cylinder.
• This model reduces the size to get maximum capacitance.
Nitin Chandrol (JEE 2012 AIR 134)
P hysi cs | 19.23

Illustration 21: A parallel plate capacitor is to be designed with a voltage rating 1kV, using a material of dielectric
constant 3 and dielectric strength about 107 Vm-1. (Dielectric strength is the maximum electric field a material
can tolerate without breakdown, i.e. without starting to conduct electricity through partial ionization.) For safety,
we should like the field never to exceed, say 10% of the dielectric strength. What minimum area of the plates is
required to have a capacitance of 50pF?  (JEE ADVANCED)

C ⋅D
Sol: The area of the capacitor is given by A = where ε = ε0 ⋅ εr lim is the permittivity of the medium.
ε x →∞

10% of the given field i.e. 107 Vm-1. Given E=0.1x107 Cm-1.

−dV V V 1000
Using E = ; i.e. E = , weget r =
we get; = = 10−3 m
dr r E 0.1x107

Using C =
ε0 εr A
, we get;
= A =
Cd Cr
=
( 450x10 )
−12
(10 =)
−13

19cm2
d ε0 εr ε0 εr 8.854x10 −3
×3

Illustration 22: A parallel plate capacitor with air between the plates has a capacitance of 8pF (1pF = 10−12 F.) What
will be the capacitance if the distance between the plates is reduced by half, and the space between them is filled
with a substance of dielectric constant 6? (JEE MAIN)
kεA
Sol: The capacitance is C = where k is the dielectric constant. If d reduces to ½ and k changes to 6 then new
d
capacitance is 12 times the original capacitance.
ε0 εr A
Using C’ = C = , C' ∈r C = 12(8 × 10‒12) = 96 × 10‒12F = 96pF
d

Illustration 23: Two parallel-plate capacitors, each of capacitance 40 µF , are connected in series. The space
between the plates of one capacitor is filled with a dielectric material of dielectric constant K=4. Find the equivalent
capacitance of the system. (JEE ADVANCED)
C1C2
Sol: The equivalent capacitance of series connection is Ceq = .
C1 + C2
The capacitance of the capacitor with the dielectric is C1 = KC0 = 4 × 40 µF = 160 µF.
The other capacitor has capacitance C2 = 40 µF.

As they are connected in series, the equivalent capacitance is C=


C1C2
=
(160µF )( 40µF=) 32µF
C1 + C2 200µF

13.4 Dielectrics and Gauss law


 
From parallel plate capacitor with a dielectric equation we can write Gauss’ law in the form ε0 ∫ K E.dA = q (Gauss’
law with dielectric)
This equation, although derived for a parallel-plate capacitor, is true generally and is the most general form in
which Gauss’ law can be written.
Some more information about capacitors
(a) If charge is held constant, i.e. battery disconnected and dielectric is inserted between plates.

(i) Charge remains unchanged, i.e., q = q0, as in an isolated system, charge is conserved.
(ii)
Capacity increases, i.e. C = KC0, as by presence of a dielectric capacity becomes K times.
q q0
(iii)
P.D. between the plate decreases, i.e., V=(V0/ K) V= = as q c KC0  as,
= q0 and =
C KC
1 9 . 2 4 | Electric Potential and Capacitance

q q0
V= = as q
= q0 and =
c KC0 
C KC

V V E  V V0 
E between the plates decreases, i.e., E = (E0 / K ) , as, =
(iv) E = 0= 0 = 0 and E=
as V 0 
d Kd K  K d
Energy stored in the capacitor decreases, i.e. U = (U0 / K ) .
(v)
2
q2 q 0 U0
=U = = as=
q q0 and =
C KC0 
2C Kd K

q0 q
+++++ +++++ +++++ +++++

C0 , V0 , E0 , U0 C, V, E, U K

- - - - - - - - - - - - - - - - - - - -

(a) (b)
Figure 19.39

(b) If potential is held constant, i.e., battery remains attached and dielectric in inserted between plates.

(i) PD remains constant, i.e. V = V0, as battery is a source of constant potential difference.

(ii)
Capacity increases, i.e., C = KC0, as by presence of a dielectric capacity becomes K times.

(iii)
Charge on capacitor increases, i.e., q = Kq0 as=
q CV
= (KC0 )=
V Kq0 as q
=0 C0 V 

 V  V0  V0 
(iv)
Electric field remains unchanged, i.e., E = E0,=
E =  = E0 as =
V V0 and = E0 
d d  d 
(v)
Energy stored in the capacitor increases, i.e., U = KU0

1 2 1
((KC0 )( V0 ) ; 1=
2
As,
= U = CV  1 
2 2 = KU as C KC
= 0 and U0 C V 2
2 0 2 0 0 

Illustration 24: A parallel-plate capacitor has plate area A and  b


plate separation d. The space between the plates is filled up to
a thickness x(<d) with a dielectric of dielectric constant K. d c
Calculate the capacitance of the system.  (JEE ADVANCED) x

a
Figure 19.40

Sol: As the distance between the plates is filled with dielectric material upto a distance x<d, this system represent
CC
two capacitors connected in series. Thus equivalent capacitance is Ceq = 1 2 where each capacitance depends
C1 + C2
on the dielectric constant K and distance from the plate of capacitor.
The situation is shown in Fig. 19.40. The given system is equivalent to the series combination of two capacitors, one
between a and c and the other between c and b.
Here c represents the upper surface of the dielectric. This is because the potential at the upper surface of the
dielectric is constant and we can imagine a thin metal plate being placed there.
P hysi cs | 19.25

Kε0 A ε0 A
The capacitance of the capacitor between a and c is C1 = and that between c and b is C2 = .
x d−x
C1C2 Kε0 A
The equivalent capacitance
= is C =
C1 + C2 Kd − x (K − 1 )

Illustration 25: In the situation shown in Fig. 19.41 the area of the 
plates is A. The dielectric slab is released from rest. Prove that the
slab will execute periodic motion and find its time period. Mass of
the slab is m.  (JEE ADVANCED) V K d

Sol: As the slab is moving in constant electric field applied x


between the plates of capacitors, the force acting on it is constant.
F L
The acceleration is obtained by a = .When the slab moves
m Figure 19.41
completly out side the plates the electric force pulls the slab slides
2s
inside the plates of capacitor. The time period of oscillation is given by t = where s is the distance travelled by
a
dU ε0bV 2 (K − 1 )
the slab. After using, F = − , where U→ Potential
Constantenergy.
force,Constant force, F =
dx 2d
A
Here, b width
= = of plate
L
F ε 0 ( A / l ) V 2 (K − 1 )
∴ Acceleration of slab a = or a=
m 2md
The equilibrium position of the slab is, at the instant when the slab is fully inside the plates. So, the slab will execute
oscillations in the phases as shown in Fig. 19.42.
x x
(a) (b) (c)

x x
(a) (b) (c)

x
(d) (e)
Figure 19.42
T
= time taken to reach from position (a) to position (b) = t (say) ; using Sx = 1 at2
using
4 (d) 2(e)
1
using S = at2 2s
2
2 (l − x )
we ha ve t =
4 (l − x ) mdl a
; (S = l − x )
2s
Wehahave
we ve t = ; = (S = l − x ) = 2 (l − x ) 4 (l − x ) mdl
a
ε (A/l)V2 (K − 1)
ε0 AV 2 (K − 1 )
= =
2 (l − x ) 4 (l − x ) mdl ε0 AV 2 (K − 1 )
0
ε
2md 0( A/l) V2 (K −1 )
= =
ε
0( A/l) V2 (K −1 )
ε0 AV (K − 1 )
2
2md

TheThe desired
desired time
time
2md
period
period is, is, =
T 4t
= 8
(L − x ) mdl
ε0 AV 2 (K − 1 )
1 9 . 2 6 | Electric Potential and Capacitance

Illustration 26: Three concentric conducting shells A, B and C of radii a, b and c are  C
B
a shown in Fig. 19.43. A dielectric of dielectric constant K is filled between A and B,
find the capacitance between A and C. (JEE ADVANCED) A

Sol: The potential difference in the region dr between the shells is given by a
∆V = ∫ E ⋅ dr where E is the electric field. Capacitance is the ratio of charge and
b
potential difference. Let the sphere A have a charge q. When the dielectric is filled
between A and B, the electric field will change in this region. Therefore the potential c
difference and hence the capacitance of the system will change. So, first find the
electric field E(r) in the region a ≤ r ≤ c . Then find the potential difference (V) Figure 19.43
q q
between A and C and finally the capacitance of the system is C == , E(r) for a≤r ≤b
  V 4 πε r 2
q q 0
) forfor =
a≤r ≤b for b ≤ r ≤ c . Using, dv = − ∫ E.dr
2
4 πε0r 4 πε0r 2
b q b q
Here potential difference between A and C is, V = VA − VC = ∫ .dr − ∫ dr
2
a
4 πε0Kr a
4 πε0r 2
q  1  1 1   1 1  q  (b − a ) ( c − b )  q
=   − + = −   =
+  c (b − a) + Ka ( c − b ) 
4 πε0  K  a b   b c   4 πε0  Kab bc  4 πε0kabc  

q 4 πε0Kabc
The desired capacitance is C
= =
V Ka ( c − b ) + c (b − a)

14. R-C CIRCUITS


To understand the charging of a capacitor in C-R circuit, let us first consider the charging of a capacitor without
resistance. q0=CV
C
+ -

S V V

Figure 19.44

Consider a capacitor connected to a battery of emf V through a switch S. When we close the  q
switch, the capacitor gets charged immediately. Charging takes no time. A charge q0 = CV
q0
comes in the capacitor as soon as switch is closed and the q-t graph in this case is a straight
line parallel to t-axis as shown.

Figure 19.45

But if there is some resistance in the circuit charging takes some time. Because resistance  C
opposes the charging (or current flow in the circuit).
Final charge (called steady state charge) is still q0 but it is acquired after a long period of
time.

( )
S V
− t/ τc
The q-t equation in this case is,=
q q0 l − e
Figure 19.46
P hysi cs | 19.27

Here, q0 = CV and τc = CR = time constant. q


q0
q-t graph is an exponentially increasing graph. The charge q increases 
0.632 q0
exponentially from 0 to q0.
From the graph and equation we see that at t = 0, q = 0 and at t = ∞, q = q0
t=C t
Figure 19.47

14.1 Charging
When a capacitor C is connected to a battery through a resistance R, the plates of a capacitor will acquire equal and
opposite charge and the potential difference across it becomes equal to the emf of the battery. The process (called
charging) takes sometime and during this time there is an electric current through the resistance. If at any time t,
I is the current through the resistance R and q is the charge on capacitor C, the equation of emf for the circuit will
be VC= + VR E, i.e., V=+ IR E

But I = (dq/ dt) and q = CV

dq q q dq t dt
so, R + =E or ∫0 ( CE − q) = − ∫0 CR
dt C

q q0 l − e− t/CR
Which on solving for q gives= ( ) with q 0 = CE ( for t = ∞ ) …..(i)

This is the required result and from this it is clear that:


(a) During charging, charge on the capacitor increases from 0 to q0 (=CE) non-linearly.
(b) The density CR is called capacitive time constant τ of the circuit [as it has dimensions of time] and physically
represents the time in which charge on the capacitor reaches 0.632 times of its maximum value during
charging.
(c) During charging current at any time t in the circuit will be


dq d 
I == 
dt dt 
(
q0 l − e− t/CR  =
 ) E
I0 e− t/CR with I0 =
R
( at t =
0)

i.e., initially it acts as short circuit or as a simple conducting wire. If t → ∞ , I → 0 , i.e., it acts as open circuit or
as a broken wire.

14.2 Discharging
If a charged capacitor C having charge q0 is discharged through a resistance R, then at any time t,
V = IR ( −dq / dt ) and q =
but as I = CV

dq q q dq t dt
R + =
dt C
0 i,e., ∫q0 dt = −∫
0 CR
or q = q0 e− t/CR

This is the required result and from this it is clear that


(a) During discharging, the charge on capacitor decreases exponentially from q0 to 0
(b) The capacitive time constant τ =CR is the time in which charge becomes (l/e), i.e., 0.368 times of its initial
value (q0)
(c) During discharging current at any time t in the circuit:


I I=
dq
dq
−− =
=
dt
dt
=
dd
dt
dt
((
−− qq00ee−−t/CR
t/CR
=
= ))
I0I0ee−−t/CR
t/CR
with
EE
= and its direction is opposite to that of charging.
withI0I0 =
RR
1 9 . 2 8 | Electric Potential and Capacitance

(d) As in discharging of a capacitor through a resistance q = q0 e− t/CR i.e., R = t


Cloge (q0 / q)
So resistance R can be determined from the value of t and ( q / q0 ) , i.e. ( V0 / V ) . Using this concept in
laboratory we determine the value of high resistances (  MΩ ) by the so called ‘Leakage method’.

Definition of τc At t = τc , q = q0 (1-e-1) = 0.632 q0


Hence, τc can be defined as the time in which 63.2% charging is over in a C-R circuit. Note that τc is the time.
time  or CR  = M0L0 T  .
Hence,  τc  =
 

15. VAN DE GRAAFF GENERATOR


+ + + ++ ++
Van de Graaff generator is a device that can generate a potential  Pulley

+
Metal brush

++

++
difference of a few million volts. The highly intense electric field

++ +++

+ ++++
produced in this matching is used to accelerate charged particles, Insulating belt
which can then be used to study the composition of matter at the
+
+ to carry and
+
microscopic level. + +

+
+ deliver charge

+
+
+ Insulating
Principle: let us presume (See Fig. 19.48) that charge Q is +
+ supporting
+
residing on an isolated conducting shell, having radius R. + column
+
+ Motor driven
+
There is another conducting sphere having charge q and radius Metal brush
+++ pulley
equal to r at the centre of the above mentioned spherical shell (r<R)
delivering Grounded
charge from metal base
The electric potential on the spherical shell of radius R is equal to source Principle of construction of
Van de Graaff generation
kQ kq
V
=R +
R R Figure 19.48
kQ kq
The electric potential on the surface of the sphere of radius r is, =
Vr +
R r
kQ kq kQ kq 1 1 
Therefore, the potential difference between the surfaces of the two sphere is, Vr − VR =+ − − = kq  − 
R r R R r R
The above equation shows that the smaller sphere is at a higher electric potential in comparison to the larger
spherical shell. If the smaller sphere is brought in contact with the larger spherical shell then electric charge will
flow from the smaller sphere towards the larger spherical shell. Note if charge can be continuously transferred to
the smaller sphere in some way, it will keep getting accumulated on the larger shell, thereby increasing its electric
potential to a very high value.
Construction: S is a large spherical conducting shell with a radius of a few meters. This is erected to suitable
height over the insulating pillars, C1 and C2. A long narrow belt of insulating material moves continuously between
2 pulleys P1 and P2 as shown in the Fig. 19.48. B1 and B2 are two sharply pointed brushes fixed near pulleys, P1 and
P2 respectively, such that B2 touches the belt near the pulley P2. B1 is called the spray brush and B2 is called the
collector brush.
Working: The spray brush is given a positive potential w.r.t. the earth by high tension source known as E.H.T.
around the sharp points of B1, the high potential causes the air to ionize and this sprays positive charges on the
belt. As the belt moves, and reaches the sphere S, the collecting brush B2, which touches the belt near pulley P2,
collects the positive charge, which spreads on the outer surface of S. the uncharged belt returns down and again
collects the positive charge from B1. As the belt moves continuously between P1 and P2 the positive charge starts
accumulating on sphere S and the charge due to ionization is minimized by enclosing the metallic shell in an earth
connected steel tank filled with air at high pressure. The charge particles may accelerate in this large potential to
high kinetic energies of the order of more than 2 MeV.
P hysi cs | 19.29

PROBLEM-SOLVING TACTICS

Below we illustrate how the above methodologies can be employed to compute the electric potential for a line of
charge, a ring of charge and a uniformly charged disk.

Charged Rod Charged Ring Charged disk


Figure y z z
P P P
r dq 
’ z
Z r
y y
r’  R y
R r’
dr’
x’ x dp
x
O dx’
L x

Figure 19.49 Figure 19.50 Figure 19.51

(2) Express dq in dq = λdl dq = σdA


dq = λdx'
terms of charge
density
(3) Substitute dq into
λdx' dV = k e
λdl
dV = k e
σdA
expression for dV dV = k e
r r r

(4) Rewrite r and dl


= Rdφ
dx' dA = 2πr'dr'
the differential
element in terms =r x' 2 + y 2 =r R 2 + z2 =r r' 2 + z 2
of the appropriate
coordinates
(5) Rewrite dV
λdx' λRdφ' 2πσr'dr'
dV = k e dV = k e dV = k e
(x' 2 + y 2 )1/2 (R 2 + z 2 )1/2 (r' 2 + z 2 )1/2

(6) Integrate to get V


λ dx1
 /2 Rλ ' R r'dr'
V=
4 πε0 ∫− /2= V ke
(R + z 2 )1/2
2 ∫ dφ V k e 2πσ ∫
=
'2
x +y 2
(r' 2 + z 2 )1/2 0

(2πRλ )
=
λ
ln = ke = 2k e πσ  z 2 + R 2 − z 
4 πε0 R 2 + z2  
 ( / 2) + ( / 2)2 + y 2  Q 2k eQ  2 2 
= ke =  z +R − z 
  R2  
 −( / 2) + ( / 2)2 + y 2  R + z2
2
 

Derive E from V ∂V k Q ∂V
∂V
Ey = − Ez =
− = e Z Ez = −
∂y ∂z
∂z
( )
3/2
R + z2
2
λ /2 2k eQ  z z 
= =  − 
2πε0 y R 2  z 
(  / 2)
2
+ y2 z + R2
2

Point- charge limit k eQ k eQ k eQ


for E Ey ≈ y >>  Ez ≈ z >> R Ez ≈ z >> R
2
y z 2
z2
1 9 . 3 0 | Electric Potential and Capacitance

For any given combination, one may proceed as follows:


Step 1: Identify the two points between which the equivalent capacitance is to be calculated. Call any one of them
as P and the other as N.
Step 2: Connect (mentally) a battery between P and N with the positive terminal connected to P and the negative
terminal to N. Send a charge +Q from the positive terminal of the battery.
Step 3: Write the charges appearing on each of the plates of the capacitors. The charge conservation principle
may be used. The facing surfaces of a capacitor will always have equal and opposite charges. Assume variables
Q1, Q2 …, etc., for charges wherever needed.
Step 4: Take the potential of the negative terminal N to be zero and that of the positive terminal P to be V. Write
the potential of each of the plates. If necessary, assume variables V1, V2….
Step 5: Write the capacitor equation Q = CV for each capacitor. Eliminate Q1, Q2 …and V1, V2…., etc., to obtain the
equivalent capacitance C=Q/V.

FORMULAE SHEET

S. No FORMULA

1. q = CV
 
2. ε0 ∫ E.dA =
q.

f 
3. ∫ E.dS.
V1 − V2 =
i

+ d
4. =V ∫=
− ∫0 ds
Eds E= Ed

5. q = ε0EA.

ε0 A
6. C= (parallel-plate capacitor)
d

L
7. C= 2πε0 (cylindrical capacitor)
ln (b / a)

8.
ab
C= 4 πε0 (spherical capacitor)
b−a

9. C= 4 πε0R (isolated sphere)

n
10. Ceq = ∑ C J (n capacitors in parallel)
j=1

n
1 1
11. =∑ (n capacitors in series)
Ceq j=1 C J
P hysi cs | 19.31

S. No FORMULA

12. 1 2 (potential energy)


U= CV
2

13. q2 (potential energy)


U=
2C

1
14. u= ε E2 (energy density)
2 0

q
15. E=
4 πKε0r 2

 
16. ε0 ∫ KE.dA =
q (Gauss’ law with dielectric).

ε0bV 2 (K − 1 )
17. Force on a Dielectric Slab inside a Capacitor F =
2d

Electric Potential Formulae

S. No Term Description
1 Electric Potential ∆U = -W Where ∆U = Change in potential energy and W= Work done by the electric lines of
energy force.

For a system of two particles U(r) = q1q2 / 4πεr

Where r is the separation between the charges.

We assume U to be zero at infinity.

Similarly for a system of n charges

U = Sum of potential energy of all the distinct pairs in the system

For example for three charges

U = (1 / 4 πε )(q1q2 / r12 + q2q3 / r23 + q1q3 / r13 )

2 Electric PE of a = qV where V is the potential.


charge
3 Electric Potential Like Electric field intensity is used to define the electric field; we can also use Electric Potential
to define the field. Potential at any point P is equal to the work done per unit test charge by
the external agent in moving the test charge from the reference point (without Change in KE)

Vp = Wext / q. So for a point charge V0 = Q / 4πεr

Where r is the distance of the point from charge.


1 9 . 3 2 | Electric Potential and Capacitance

S. No Term Description
4. Some points 1. It is scalar quantity
about Electric
2. Potential at a point due to system of charges will be obtained by the summation of potential
potential
of each charge at that point

V = V 1 + V2 + V3 + V4

3. Electric forces are conservative forcew so work done by the electric force between two points
is independent of the path taken

4. V2 − V1 =
− E.dr ∫
5. In Cartesian coordinates system

dV =
−E.dr; dV =
−(Ex dx + Ey dy + Ez dz)
So Ex =
∂V / ∂x, Ey =
∂V / ∂y and Ez =
∂V / ∂z,

( ) ( ) (
Also E = ∂V / ∂x i + ∂V / ∂y i + ∂V / ∂z k  ) 
6. Surface where electric potential is same everywhere is called equipotential surface.

Electric field components parallel to equipotential surface are always zero.


5 Electric dipole A combination of two charges +q and –q separated by a distance d has a dipole moment
p = qd, where d is the vector joining negative to positive charge.
6 Electric potential
=V (1 / 4 πε) × (pcos θ / r 2 )
due to dipole
Where r is the distance from the center and θ is angle made by the line from the axis of dipole.
7 Electric field due
Eθ (1 / 4 πε )(psin θ / r3 );=
= Er (1 / 4 πε ) × (2pcos θ / r3 )
to dipole
Total
= E Eθ2 +=
Er 2 (p/ 4 πε r3 )( (3cos2 θ + 1)
Torque on dipole = p × E

Potential Energy U = ‒p.E


8 Few more points
1. ∫ E.dl over closed path is zero

2. Electric potential in the spherical charge conductor is Q / 4πεR where R is the radius of the
shell and the potential is same everywhere in the conductor.

3. Conductor surface is a equipotential surface

Electric potential due to various charge distributions

Name/Type Formula Note Graph


Point Charge •• q is source charge
Kq
r •• r is the distance of the point from the V
point charge.
r

Ring (uniform/non •• Q is source charge V


KQ
uniform charge At centre,
R •• x is distance of the point from centre.
distribution)
KQ
At axis,
R 2 + x2
P hysi cs | 19.33

Name/Type Formula Note Graph


Uniformly charged •• R is radius of sphere V
KQ
hollow conducting/ For r ≥ R, V =
R •• r is distance of the point from centre
non - conducting KQ/R
KQ of the sphere
sphere or solid For r ≤ R, V = 2
conducting sphere R •• Q is total charge ( ( =σ 4 πR )
r
R

Uniformly charged KQ •• R is radius of sphere V


solid non - conducting For r ≥ R, V =
r •• r is distance of point from centre of 3KQ/2R
sphere (insulating
For r ≤ R, the sphere. KQ/R
material)
4 3
•• Q is total charge ( =ρ πR )
3 3
KQ(3R 2 − r 2 ) ••   Vcentre = V
r
V= R
2R3 2 surface
ρ •• Inside sphere potential varies
= (3R 2 − r 2 ) parabolically.
6ε 0
•• Outside potential varies
hyperbolically.

Line charge Not defined •• Absolute potential is not defined

•• Potential difference between


two points is given by formula
VB − VA =−2Kλ ln(rB / rA ) , where
lambda is the charge per unit length

Infinite nonconducting Not defined •• Absolute potential Is not defined


thin sheet
•• Potential difference between
two points is given by formula
σ
VB − VA =
− (r − r ) , where
2ε0 B A
sigma is the charge density

Infinite charged Not defined •• Absolute potential is not defined


conducting thin sheet
•• Potential difference between
two point is given by formula
σ
VB − VA =
− (rB − rA ) , where
ε0
sigma is the charge density

 d
Electric dipole moment: p = qdzˆ , where two charges of charge ±q are placed along the z axis at z = ±
2
 1 p
Electric dipole field: Along the z axis (z>>d): E = zˆ , in the +z direction.
2πε0 z 3

 1 p
Along the x axis(x>>d): E = xˆ in the +x direction.
2πε0 x 3

 1 p
Along the x axis(y>>d): E = yˆ in the +y direction.
2πε0 y 3
1 9 . 3 4 | Electric Potential and Capacitance

Solved Examples

JEE Main/Boards Potential difference across C4 and 200 PF is in the ratio


2:1 i.e. 200 V across C4
Example 1: Obtain the equivalent capacitance of the ∴ Charge on C4 = C4 V4; =100 × 200 × 1012 = 2 × 10‒8 C
network in figure. For a 300V supply, determine the
Potential difference across C1=100V
charge and voltage across each capacitor.
100pF Charge on C1= C1 × V1 = 100 × 100 × 10‒12 =1 × 10‒8 C
Potential difference across C2 and C3 is 50V each
C1 ∴ Charge on C2 or C3= C2V2 = 200 × 50 × 1012 = 10‒8 C.

200pF 200pF
Example 2: The connection shown in figure are
+ established with the switch S open. How much charge
C2 100pF C3 300V will flow through the switch if it is closed?
-

C4 1 2

Sol: The circuit is made up of series and parallel + 24V 1F S 2F
combinations of the capacitors. The charge on the -
capacitor is given by q = CV. 2F 1F

100pF
4 3

100pF C1 (a)

C4 C2 C3
-Q1 Q1 Q2 -Q2
200pF 200pF 1 2
1F 2F
+ 24V S
300 V
- 0
2F V0 1F
4 3


Q2 Q2
The equivalent circuit is as shown below: (b)
100pF
Sol: Find the initial charge on capacitors. After the
100pF switch is closed, 1 and 2 become parallel and 3 and 4
become parallel.
C4
When the switch is open, capacitors (2) and (3) are in
100pF 2
µF. The charge appering on
series. Their equivalent capacitance is
3
300 V The charge appering on each of these capacitors is,
2


therefore, 24V × µF = 16µC


3
The equivalent capacitance of (1) and (4), which are
100pF 200pF 2
also connected in series, is also µF and the charge
3
300 V on each on each of these capacitors is also 16 µC . The
total charge on the two plates of (1) and (4) connected
to the switch is, therefore, zero.
P hysi cs | 19.35

The situation when the switch S is closed is shown in The equivalent capacitance between A being equivalent
figure. Let the charges be distributed as shown in the to the original ladder, the equivalent capacitance is
figure. Q1 and Q2 are arbitrarily chosen for the positive also C
plate of (1) and (2). CC1
Thus, C1= C + or C1C + C12 = C2 + 2CC1
Take the potential at the negative terminal to the zero C + C1
and at the switch to be V0
Or C12 − CC1 − C2 =
0,
Writing equations for the capacitors (i), (ii), (iii) and (iv).
Q1 = (24V ‒ V0) × 1µF … (i) C + C2 + 4C2 1 + 5
Giving C1
= = C
Q2 = (24V ‒ V0) × 2µF  … (ii) 2 2
Q1 = V0 × 1µF  ... (iii) Negative value of C1 is rejected.

Q2 = V0 × 2µF  … (iv)
Example 4: A parallel-plate capacitor has plates of area
From (i) and (iii), V0 =12V, Thus, from (iii) and (iv), 200 cm2 and separation between the plates 1.00 mm.
What potential difference will be developed if a charge
Q1= 12µC and Q= 24µC .
2 of 1.00 nC (i.e., 1.00 × 10‒9C) is given to the capacitor? If
The charge on the two plates of (1) and (4) which are the plate separation is now increased to 2.00 mm, what
connected to the switch is, therefore Q2 − Q1 =12µC . will be the new potential difference?
When the switch was open, this charge was zero. Thus, ε A
12µC of charge has passed through the switch after it Sol: Capacitance is given by C = 0 and the potential
d
was closed. Q
difference is given by V =
C
Example 3: Find the capacitance of the infinite ladder The capacitance of the capacitor is
shown in figure.
ε0 A 200x10−4 m2
C P C C C= = 8.85x10−12 Fm−1 x
A d 1x10−3 m
−9
= 0.177x10
= F 0.177nF.
C C C C The potential difference between the plates is
B
Q Q 1nC
V
= = = 5.65 Volts.
C 0.177nF
Sol: Capacitance between points P and Q is same as If the separation is increased from 1.00 mm to 2.00 mm,
that between A and B. The equivalent capacitance of the capacitance is decreased by a factor of 2. Thus, the
the circuit is deduced by reducing it to simple network. new potential difference will be 5.65 volts × 2 = 11.3 volts.
The simplified circuit is a parallel combination of the
capacitors whose capacitance is given by
Example 5: An isolated sphere has a capacitance of 50pF.
CC1
Ceq= C + . (a) Calculate its radius.
C + C1
(b) How much charge should be placed on it to raise its
As the ladder is infinitely long, the capacitance of the potential to 104 V?
ladder to the right of the points P, Q is the same as
that of the ladder to the right of the points A, B. If the Sol: For sphere, radius R is given by C= 4 πε0R . To
equivalent capacitance of the ladder is C, the given raise the potential to 104 V, the charge to be placed on
ladder may be replaced by the connections shown in sphere is given by Q = CV.
figure.
C P (a) The capacitance of an isolated sphere is
A
R
C= 4 πε0R. Thus, 50x10−12 F =
9x109 mF−1
−12
C C1 =or R 50x10
= x9x109 m 45cm.
B
Q (b) Q = 50X10−12 FX10 4 =
= CV V 0.5µC
1 9 . 3 6 | Electric Potential and Capacitance

Example 6: A parallel-plate capacitor of capacitance of charge induced in other plates. The capacitance is
100 µF if connected to a power supply of 200V. A Aε
dielectric slab of dielectric constant 5 is now inserted between any two surfaces is given by C = 0 .
d
into the gap between the plates. Suppose the negative terminal of the battery gives a
(a) Find the extra charge flown through the power charge – Q to the plate B. As the situation is symmetric
supply and the work done by the supply. on the two sides of B, the two faces of the plate B
will share equal charge – Q/2 each. From Gauss’s law,
(b) Find the charge in the electrostatic energy of the
the facing surfaces will have charge Q/2 each. As the
electric field in the capacitor.
positive terminal of the battery has supplied just this
much charge (+Q) to A and C, the outer surfaces of A
Sol: After inserting the dielectric the capacitance is
and C will have no charge. The distribution will be as
changed, thus the charge stored in the capacitor is
shown in figure. The capacitance between the plates A
given by Q = CV and work done by supply is CV2 = qV.
and B is
1 2
The energy stored in the capacitor is given by U = CV Aε0 200 × 10−4 m−2
2 C ==× 8.85 10−12 F / m×
d 2 × 10−4 m
(a) The original capacitance was 100µF. The charge on
= 8.85 = X10−10 F 0.885nF
the capacitor before the insertion of the dielectric was,
therefore, Q1 = 1000F × 200V = 20mC Thus, Q 8.85nF
= = x 20V 17.7nC
After the dielectric slab is introduced, the capacitance is The distribution of charge on various surfaces may be
increased to 500µF. The new charge on the capacitor is, written from figure.
100mC .
Therefore, 500µ F x 200v = Q
The equivalent capacitance is = 1.77nF
The charge flown through the power supply is, 20V
80 mC .
therefore, 100 mC − 20 mC =
Example 8: The emf of the cell in the circuit is 12 volts and
The work done by the power supply is 200V × 80mC = 16J. the capacitors are: C1 = 1µF, C2 = 3µF, C3 = 2µF, C4 = 4µF
(b) the electrostatic field energy of the capacitor without Calculate the charge on each capacitor and the total
the dielectric slab is charge drawn from the cell when
1 2 1
U1 =
2
CV =
2
(100µF ) X ( 200V ) =
2J (a) The switch s is closed
(b) The switch s is open.
And that after the slab is inserted is
1 C1 C2
(500µF ) X ( 200V ) =
2
U2 = 10J thus, the energy is
2
increased by 8J.

C3 C4
Example 7: Each of the three plates shown in figure has
an area of 200cm2 on one side and the gap between
the adjacent plates is 0.2mm. The emf of the battery is Cell
+ -
20V. Find the distribution of charge on various surfaces
of the plates. What is the equivalent capacitance of the 12 V
system between the terminal points?
Sol: When the switch is closed the equivalent
+
-
+
- 20V
capacitance is given by C=
( C1 + C3 )( C2 + C4 )
( C1 + C3 ) + ( C2 + C4 )
and the charge stored in the circuit is given by Q=CV.
Q/2

Q/2
-Q/2

-Q/2

When the switch is open the capacitance is given


A B C A B C
C1C2 C3C 4
by C
= + . The charge stored in the
(a) (b) C1 + C2 C3 + C 4
Sol: As the charge distribute symmetrically around capacitor is given by
the central plate. Due to this there is equal amount
(a) Switch S is closed:
P hysi cs | 19.37

C
=
( C1 + C3 )( C2 + C4 ) ; C
=
3×7
= 2.1µF ;
C1= charge on C2

( C1 + C3 ) + ( C2 + C4 ) 3+7  CC
= 3 4
 8
C +C  V = × 12 =16µC
 3  6
Total charge drawn from the cell is: 4

Q = CV = 2.1µF × 12 volts = 25.2µC C3 & C4 are in series and the potential difference across
combination is 12 volts charge on
C1 , C3 are in parallel and C2, C4 are in parallel.
C3= charge on C4
C1 C2
Q  CC  8
= 3 4  V = × 12 =16µC
C +C 6
 3 4 
C3 C4
Example 9: Two capacitors A and B with capacities 3 µF
and 2 µF are changed to a potential difference of 100V
+ -12V and 180V respectively. The plates of the capacitors are
connected as shown in the figure with one wire of each
Charge on C1 capacitor free. The upper plate of A is positive and that
of B is negative. An uncharged 2 µF capacitor C with
C1 1
Q1 = Q= × 25.2µC = 8.4µC lead wires falls on the free ends to complete the circuit.
C1 + C3 1+2 Calculate:
Charge on C3 q2
B + - C
C3 2 3 4
Q3= Q= × 25.2µC= 16.8µC
C1 + C3 1+2
+ 2 5 -
3F q1 2F
Charge on C2 -
1 6 q +
3

C2 3
Q2= Q= × 25.2µC= 10.8µC
C2 + C 4 3+ 4 A D

Charge on C4 (i) The final charge on the three capacitors, and

C4 4 (ii) The amount of electrostatic energy stored in the


Q 4= Q= × 25.2µC= 14.4µC system before and after the completion of the circuit.
C2 + C 4 3+ 4
(c) switch S is open : Sol: The charge stored in each capacitor when they
C1C2 C3C 4 1X3 2X4 25 are not connected to each other is given by q = CV.
=C + C= + = µF When the capacitors are connected to each other then
C1 + C2 C3 + C 4 1 + 3 2 + 4 12
the charge stored in each the capacitor is obtained by
Total charge drawn from battery is: applying Kirchoff’s 2nd law to the circuit. The energy
25 1
Q =CV = × 12 =25µC stored in the capacitor is given by U = CV 2
12 2

C1 C2 (i) charge on capacitor A, before joining with an


uncharged capacitor,
qA= CV= (100) + 3µc= 300µc
C3 C4 Similarly charge on capacitor B,
qB= 180 × 2µc= 360µc
12V
+ - Let q1, q2 and q3 be the charges on the three capacitors
after joining them as shown.
C1 and C2 are in series and the potential difference From conservation of charge,
across combination is 12 volts charge on
1 9 . 3 8 | Electric Potential and Capacitance

Net charge on plates 2 and 3 before joining (c) The total electrical energy stored after joining
= Net charge after joining 200V
+ -
∴300 =q1 + q2  …… (i)
6F
Similarly, net charge on plates 4 and 5 before
S
Joining = Net charge after joining
100V
−360 =−q2 − q3 ; 360
= q2 + q3  ...…(ii) + -

Applying Kirchoff’s 2nd law in loop ABCDA, 4F


q1 q2 q3 How do you account for the difference in energies in
− + 0
= (b) and (c) ?
3 2 2
0
2q1 − 3q2 + 3q3 = ..… (iii) Sol: When the capacitors are connected to each other
From equations (i), (ii) and (iii), then the total charge stored in the circuit is given by
q1 =90µC,q2 =90µC and q3 =150µC Q = Q1 + Q2 = C1 V1 + C2 V2 . The energy stored in each
1 2
(iii) (a) Electrostatic energy stored before completing capacitor is given by U = CV .
2
the circuit,
V1 = 100V,C1 = 4µF , Q1 = 4 × 10 −6 × 100
C1 V1 =
 1 2
 U = CV  = 4.74x10 J = 47.4mJ.
-2
2 = 4 × 10−4 C ; V2 = 200V,C2 = 6µF
 
2F Q2 =C2 V2 =6X10 −6 × 200 =12 × 10 −4 C
C When the plates with like charges are connected
together, both capacitors have the same potential after
+ - redistribution of charge.
3F 2F
A (a) Total charge = =
Q Q1 + Q2
B
=( 4 + 12 ) × 10−41.6 × 10−3 C
Capacitance of the combination in parallel
(b) Electrostatic energy stored after completing the = C = C1 + C2 = ( 4 + 6 ) µF = 10µF
circuit,
Potential across each capacitor after joining
1
( ) 1
2
Uf = 90X10−6 q 1.6 × 10−3
2 3x10−6 = V= = = 160V
C 10 × 10−6
1
( ) 1
2
+ 90x10−6 (b) Electrical energies U1 and U2 before joining are
2 2x10−6
given as :
1
( ) 1  1 q
2
150x10−6
+
2 2x10 −6
;  U =


2 c2 
1
U1 =
2
1
C1 V12 =
2
(
× 4X10 −6 × (100 ) =
2
)
2X10−2 J

= 90x10−4 J = 90mJ 1
U2 =
2
1
C2 V22 =
2
(
× 6X10−6 × ( 200 ) =
2
)
12X10−2 J

Example 10: A capacitor of capacitance 4 µ F is Energy before joining


charged to a potential difference of 100V and another = U=
i U1 + U2 ; = (12 + 2) × 10−2 = 0.14J
of capacitance 6 µ F is charged to a potential of 200V.
These capacitances are now joined with plates of like (c) Electrical energy after joining of capacitors
charges connected together. Calculate:
1
=Uf = × 10 × 0 −6 × (160)2 =0.128J
(a) The total potential across each after joining 2
(b) The total electrical energy stored before joining The stored electrical energy after joining is less by
(0.14-0.128) J i.e. 0.012J. the energy is dissipated as heat
energy through the connecting wires as current flows.
P hysi cs | 19.39

Example 11: What charges will flow thorugh A, B and = q1 (C)(P.D)


= (2)(30)µC
C in the directions shown in Fig. 19.69 when switch S is
Similarly, P.D. across 3 µF capacitor is same as that
closed?
between 60V battery. Hence, q =2 = 180µC. Now
(3)(60)
A let qA charge goes to the upper plate of 2 µF capacitor.
30V 2F Initially it had a charge +q and finally charge on it is
B + q1 . Hence, q1= q + qA or qA = q1 − q = 60 − 180
-S
=−48µC
3F
60V 48F
C -
48C
Sol: The charges stored in capacitor is given by Q = CV. - 30V
- 2F
CC
The equivalent capacitance is Ceq = 1 2 . To find the 48C +
C1 + C2
charges flowing in each capacitor we use the Kirchoff’s 72C
- 60V
2nd law. - 3F
Let us draw two figures as shown in figure and find the 72C C
charge on both the capacitors before closing the switch
and after closing the switch. Refer figure (a) when
Similary, charge qB goes to the upper plate of 3 µF
switch is open: both capacitors are in series. Hence,
capacitor and lower plate of 2 µF capacitor. Initially
their equivalent capacitance is,
both the plates had a charge +q, -q or zero. And finally
C1C2 (2)(3) 6 they have a charge (q2 − q1 ) .
Ceq= = = µF
C1 + C2 2+3 5
Hence, q2 − q1 = qB + 0
Therefore, charge on both capacitors will be same.
∴qB = q2 − q1
Hence, using q = CV, we get
= 180 − 60

30V
+ A = 120µC Initially it had a charge-q and finally −q2 .
q 2F 30V 2F
- Hence, qc =q − q2 =108 − 180 =−72µC
B
+ B
q 3F 3F Example 12: If 100 volts of potential difference is
60V - 60V
C applied between a and b in the circuit of figure, find the
potential difference between c & d.

(30 + 60 )  56  µC=
 
q= 108µC Sol: The charge stored in each capacitor is given by
  Q = CV. To find the potential difference at point’s c and
Refer figure (b), when switch is closed: let q1 and q2 be d we apply Kirchoff’s law to the circuit.
the charges (in µC ) on two capacitors. Then applying The charge distribution on different plates is shown in
Kirchoff’s 2nd law in upper and lower loop, we have figure. Suppose charge Q1 + Q2 is given by the positive
q1 terminal of the battery, out of which Q1 resides on the
30 − 0 Or q1 = 60µC;
=
2 positive plate of capacitor (1) and Q2 on that of (2). The
q2 remaining plates will have charges as shown in the
60 − 0 Or q2 = 180µC
= figure. Take the potential at the point b to be zero. The
3
potential at a will be 100V. Let the potentials at points
Charges q1 and q2 can be calculated alternatively by c and d be Vc and Vd respectively. Writing the equation
seeing that upper plate of 2 µF capacitor is connected Q=CV for the four capacitors, we get,
with positive terminal of 30V battery. Therefore, they
are at the same potential. Similarly, the lower plate of Q1 = 6µF × 100V = 600µC  …..(i)
this capacitor is at the same potential as that of the Q2 = 6µF × (100V − Vc )  …..(ii)
negative terminal of 30V battery. So, we can say that
Q2 = 6µF × (Vc − Vd )  …..(iii)
P.D. across 2 µF capacitor is also 30V.
Q2 = 6µF × Vd−  …..(iv)
1 9 . 4 0 | Electric Potential and Capacitance

From( ii(ii)
From ) andand((iii)
iii) 100V − Vc = Vc − Vd JEE Advanced/Boards
or 2Vc − Vd =
100V  (v)
…..…..(v)
Example 1: Two capacitors C1 = 1 µF and C2 = 4µF
And from ( iii ) and ( iv ) Vc − Vd =
Vd
are charged to a potential difference of 100 volts and
or Vc = 2Vd−  .. ( vi )
……..(vi) 200 volts respectively. The charged capacitors are now
connected to each other with terminals of opposite
From (v) and (vi)
sign connected together. What is the
100 200
 Fro m ( v(v)
From )= and ((vi)
and vi) Vd = V and Vc V V
3 3 V1
100
So that
So that Vc − Vd = V
3
+ + -- -- --
C1 C2 C1 C2
Example.13 A regular hexagon of side 10 cm has + + + + + + + +
a charge 5 µ C at each of its vertices. Calculate the
potential at the center of the hexagon.
q q V1

(a) Final charge on each capacitor in steady state?


q q (b) Decrease in the energy of the system?
O
Sol: The capacitors are connected parallel to each other.
q 10 cm q Thus the equivalent capacitance is given by C = C1+C2.
The charges stored in capacitor is given by Q = CV. The
Sol: The potential at the center of the hexagon is
1
due to 6 charges placed at its vertices. It is given by energy stored in each capacitor is given by U = CV 2 .
2
 1 q initial charge on C
= C V
= 100 µ F ;
V= 6 ×  1 1 1
 4 πε r 
 0  Initial charge on C
= 2 V2 800µF ; C1 V1 < C2 V2
C2 =
6F
When the terminals of opposite polarity are connected
a c
together, the magnitude of net charge finally is equal
6F 6F to the difference of magnitude of charges before
connection.
b d (Charge on C2 )i - (charge on C1 ) i
6F
(a) = (charge on C2 )f - (charge on C1 ) f
Let V be the final common potential difference across
Q2 -Q2 each.
a c
The charges will be redistributed and the system attains
Q 2 Q2 a steady state when potential difference across each
100V 1 3
-Q1 -Q2 capacitor becomes same.
b d
-Q2 Q2 C2 V2 − C1 V1 =C2 V + C1 V
(b) C1 V2 − C1 V2 800 − 100
=V = = 140 volts
Total potential at O is given by, C2 + C1 5

 1 q 5 × 10−6 Note that because C1 V1 < C2 V2 , the final charge


V =6×  = 6 × (9 × 109 ) × = 2.7 × 106 volt.
 4 πε r  0.1
 0  polarities are same as that of C2 before connection.
Final charge on C
=1 C1=
V 140µC
Final charge on C
= 2 C2=
V 560µC
Loss of energy= Ui − Uf
P hysi cs | 19.41

Loss of energy
Example 3: A 10 µF condenser C is charged through
1 1 1 1 resistance R of 0.1 MΩ from a battery of 1.5V. Find the
= C1 V12 + C2 V22 − C1 V 2 − C2 V 2
2 2 2 2 time required for the capacitor to get charged upto
0.75V for the circuits shown below.
1 1 1
1 (100 ) + 4 ( 200 ) − (1 + 4 )(140 )
2 2 2
=
2 2 2 R S
= 36000
=µJ 0.036J
E C C R
Note: the energy is lost as heat in the connected wires S
due to the temporary currents that flow while the
charge is being redistributed.
Sol: For RC circuit, the charge stored in the capacitor is
Example 2: Four identical metal plates are located in
given by
= q q0 1 − e− t/RC  . Taking log on both sides
the air at equal separation d as shown. The area of each  
plate is a. Calculate the effective capacitance of the we can get the value of time t.
arrangement across A and B.
(a) In the case of charging of a capacitor C through the
A
resistance R,
q
=q q0 1 − e− t/RC  ; e− t/RC= 1 −
  q0
B
q V 0.75 1
For a capacitor, q = CV = = =
q0 V0 1.5 2
Sol: The plates are connected in parallel combination
with each other. The equivalent capacitance is given =
by t RClog
= e2 (0.1 x106 )x(10x10−6 )loge 2
C = C1 +C2 +C3. The capacitance between A and B= is t RClog
= e 2 0.693 second.
2C
given by CAB = (b) In the case of the capacitor C being connected
3
directly to the battery initially, it acts like short circuit.
A
The capacitor will get charged instantaneously at t = 0
1 secs.
2
P
3 Hence, time cannot be calculated as per the requirement
4 of the question.
5 B
6 Example 4: Twelve capacitors, each having a capacitance
C, are connected to form a cube. Find the equivalent
capacitance between the diagonally opposite corners
such as A and B.
2 1 P 5 6 f
B B
A B
e
d
3 4 h Q/3
c b
F

Let us call the isolated plate as P.A capacitor is formed


Q/6
A g
A Q/3 E
by a pair of parallel plates facing each other. Hence we a

have three capacitor formed by the pairs (1,2), (3,4) and Sol: Applying the Kirchoff’s second law across diagonal
(5,6). The surface 2 and 3 are at same potential as that points A and B to find the charge distribution across
of A. The arrangement can be redrawn as a network of the branch of the circuit, and find the equivalent
three capacitors. capacitance.
2C 2ε A0 Suppose the points A and B are connected to a battey.
C=
AB =
3 3 d The charges appearing on some of the capacitors are
1 9 . 4 2 | Electric Potential and Capacitance

shown in figure suppose the positive terminal of the Let length and breadth of the capacitor be l and b
battery supplies a charge +Q through the point A. respectively and d be the distance between the plates
This charge is divided on the three plates connected as shown in figure then consider a strip at a distance x
to A. Looking from A, the three sides of the cube of width dx.
have identical properties and hence, the charge will
Now QR=x tan θ
be equally distributed on the three plates. Each of the
capacitors a, b and c will receive a charge Q/3. The And PQ = d-x tan θ ; where
negative terminal of the battey supplies a charge –Q tan θ =d/l,Capacitance of PQ
through the point B. This is again divided equally on
L
the three plates connected to B. Each of the capacitors
d, e and f gets equal charge Q/3.
A b
Now consider the capacitors g and h. As the three
plates connected to the point E form an isolated
system, their total charge must be zero. The negative
plate of the capacitor has a charge -Q/3. The two plates k1 ε0 (b dx) k1 ε0 (b dx)
= dC1 =
of g and h connected to E should have a total charge d − x tan θ xd
d−
Q/3. By symmetry, these two plates should have equal l
charges and hence each of these has a charge Q/6. The k1 ε0b / dx k1 ε0 A
capacitors a, g and d have charges Q/3, Q/6 and Q/3 = dC1 =
d (l − x ) d (l − x )
respectively.
k 2 ε0b ( dx )
And dC2 = capacitance of QR dC2 = ;
We have, VA − VB = ( VA − VE ) + ( VE − VF ) + ( VF − VB ) d tan θ
k 2 ε0 A ( dx )  d
Q / 3 Q / 6 Q / 3 5Q dC2 = … ∴tan θ = 
= + + = Xd  l
C C C 6C
Q 6 Now dC1 and dC2 are in series. Therefore, their
Ceq =
= C.
VA − VB 5 resulatant capacity dC will be given by

1 1 1 1 d (l − x ) X.d
Example 5: The Capacitance of a parallel plate = + , =then +
1 0 ( ) 1 0 A ( dx )
capacitor with plate area A and separation d is C. The dc dC 1 dC 2 dC K ε A dx K ε
space between the plates is filled with two wedges of
dielectric constants K1 and K 2 respectively (figure).=
Find 1 d  l − x X  d K 2 (l − x ) + K1 X 
 +  =
the capacitance of the resulting capacitor. dc ε0 A ( dx )  K1 K 2  ε0 AK1K 2 ( dx )

A
ε0 AK1K 2 ε0 AK1K 2
dC = dx , dC = dx
d K 2 (l − x ) + K1 X  d K 2l + (K1 − K1 ) X 
K2

d All such elemental capacitor representing DC are


connected in parallel.
K1

Now the capacitance of the given parallel plate capacitor


is obtained by adding such infinitesimal capacitors
Sol: As represented in the figure the dielectric material parallel from
are connected in series with each other across diagonal. x =L
X = 0 to X = L. i.e. C = ∫ dC ;
Consider one thin strip of the capacitor. The equivalent x =0
1 1 1 P
capacitance is given by = + . dc1
dC dC1 dC2
K1
These strips are connected to each other parallely such Q
that the equivalent capacitance of the capacitor is given dc2
d
by K2
x =L
C= ∫ dC
R

x =0 dx x
L
P hysi cs | 19.43

L
ε0 AK1K 2 KK11KK22εε00AA KK connected to a battery which supplies a positive charge
= ∫ d K l + (K dx ; CC == In 22
In
ln of magnitude Q to the upper plate. Find the rise in the
0  2 1 − K 2 ) X  (KK11 −−KK22) dd KK11 level of the liquid in the space between the plates.

Example 6: Figure (a) shows a parallel-plate capacitor Sol: The dielectric liquid experiences a electric force due
having square plates of edge a and plate-separtion d. to which it rises to the height h. This force is balanced
The gap between the plates is filled with a dielectric by weight of liquid.
medium of dielectric constant K which varies parallel to
an edge as =
K K 0 + ax  1
The situation is shown in figure. A charge −Q  1 −  is
 K
a a
 1
induced on the upper surface of the liquid and Q  1 − 
 K 
d K
at the surface in contact with the lower plate. The net
 1 Q
(x)
dx charge on the lower plate is −Q + Q  1 −  =− .
(a)
 K K
Consider the equilibrium of the liquid in the volume
(b)

Where K and α are constants and x is the distance from


the left end. Calculate the capacitance. ABCD. The forces on this liquid are
(a) The force due to the electric field at CD,
Sol: As the medium varies, consider a small strip of
the dielectric medium such that its capacitance is (b) The weight of the liquid,

dC =
(K0 + αx ) ε0adx . These strips are connected
(c) The force due to atmospheric pressure and
d (d) The force due to the pressure of the liquid below AB.
in parallel so the equivalent capacitance is given by As AB is in the same horizontal level as the outside
a surface, the pressure here is the same as the atmospheric
C = ∫ dC where a is the length of plates. pressure. The forces in (c) and (d), therefore, balance
0
each other. Hence, for equilibrium, the forces in (a) and
Consider a small strip of width dx at a separation x from
(b) should balance each other.
the left end. This strip forms a small capacitor of plate
Q
area adx. Its capacitance is dC =
(K0 + αx ) ε0adx The electric field at CD due to the charge Q is E1 =
2Aε0
d
The given capacitor may be divided into such strips with In the downward direction.
x varying from 0 to A. All these strips are connected in Q
parallel. The capacitance of the given capacitor is, The field at CD due to the charge –Q/K is E2 =
2Aε0K
Q Also in the downward direction.
D+ - + + + + + + +
- - -Q(1-1/K) (K + 1 ) Q .
The net field at CD is E1 + E2 =
h
A B 2Aε0K

 1
The force on the charge −Q  1 −  at CD is
-Q/K
 K

=
 1  (K + 1 ) Q
F Q 1 −  ; =
(K 2
)
− 1 Q2
a
(K0 + αx ) ε0adx ; 2
ε0 a  aα   K  2Aε0K 2Aε0K 2
C= ∫ d
= K +
d  0 2 

0 In the upward direction. The weight of the liquid
considered is hAρg.
Example 7: A parallel-plate capacitor is placed in such
a way that its plates are horizontal and the lower plate
thus, hAρg =
(K 2
)
− 1 Q2
; or h=
(K 2
)
− 1 Q2
is dipped into a liquid of dielectric constant K and 2 2 2
2Aε0K 2A K ε0 ρg
density ρ . Each plate has an area A. the plates are now
1 9 . 4 4 | Electric Potential and Capacitance

JEE Main/Boards

Exercise 1 Q.7 What capacitance is required to store an energy of


100kW h at a potential difference of 104 V?
Q.1 Calculate the equivalent capacitance in the
following circuit Q.8 A capacitor is filled with two dielectrics of the
same dimensions but of dielectric constants 2 and 3
4.5F respectively. Find the ratio of capacitances in the two
3F 3F
arrangements shown in figure A and B.

4.5F A B A B

Q.2 Find the potential of a sphere having charge of + - + - + -


5 µC and capacitance of 1nF.

d
Q. 3 Find the equivalent capacitance between A and B
in the given figure. Take C2= 10µF and B in the given (A) d/4 d/2 d/4
(B)
figure. Take C2= 10µF and C1 ,C3 ,C 4 ,C5 each equal to
4µF
C4 Q.9 Two metal plates separated by a distance d
constitute a parallel plate capacitor. A metal slab of
thickness (d/2) and same area as the plate is inserted
A B
between the plates. What is the ratio of the capacitances
C1 C2 C3 in the two cases?

C5
K1 C1 d/2
d
K2 C2 d/2
Q.4 Find the value of C if the equivalent capacitance
between the points A and B in the given figure is 1µF
(A)
1F
A C
A/2 A/2
4F
6F d K1 K2
8F
C3 C4
12F

2F (B)
2F Q.10 Keeping the voltage of the charging source
B
constant, what would be the percentage change in
the energy stored in a parallel plate capacitor if the
separation between its plates were to be decreased by
Q.5 A 4x10-6F capacitor is charged by a 200V supply. It 10%?
is then disconnected from the supply and is connected
across another uncharged 2x10-6F capacitor. How much
Q.11 A parallel plate air capacitor with its plates spaced
energy of the first capacitor is lost?
2cm apart is charged to a potential of 300 volts. What
will be the electric field intensity inside the capacitor,
Q.6 The plates of a parallel plate capacitor of area if the plates are moved apart to a distance of 5cm
100cm2 each and are separated by 2.0mm. The capacitor without disconnecting the power source? Calculate the
is charged by a 100V supply. Find energy store by the change in energy of the capacitor. Area of the plates is
capacitor. equal to A = 100cm2. Also solve the problem assuming
P hysi cs | 19.45

the entire operation was done after disconnecting the Q.16 Two charge particulars one is electron and other is
power source. Account for the change in energy in both negatively charged ion have a velocity directed parallel

(
the cases. ∈0 =9x10−12 SI units ) to the plates. They are sent separately into the field.
Both the electron and the ion have reveived their initial
kinetic energy by passing the same potential difference.
Q.12 Find the charge on the capacitor C in the following Which of the two particles will travel a greater distance
circuit in steady state. (parallel to plates shown by ‘I’) before hitting the
12V 2 positively charged plate, if both fly into the capacitor
at a point that is exactly in the middle of the distance
4
between the plates?
2C
6
L
+
V
Q.13 Three uncharged capacitors of capacitance C1,C2
and C3 are connected as shown in figure to one another _
and to points A, B and D potentials (φA), (φB) and (φD).
Determine the potential (φO) at point O. Q.17 A 3 µF capacitor is charged to a potential of 300
volt and 2 µF is charged to 200 volt. The capacitor
A are connected so that the plates of same polarity
C1 are connected together. What is the final potential
O difference between the plates of the capacitor after they
C2 C3 are connected? If instead of this the plates of opposite
B D polarity were joined together, what amount of charge
will flow and from which capacitor does it come?

Q.14 Determine the current through the battery in the


Q.18 In the following circuit, internal resistance of the
circuit shown.
battery r = 1Ω .
E K

C1 C1 R=2 C3
R1 C2 C2 C4
A B
R2 R=3
R3 E
R=1
(i) Immediately after the key K is closed and
(ii) In a long time interval, assuming that the parameters E = 4V, C1 = 8µF , C2 = 2µF , C3 = 6µF , C 4 = 4µF .
of the circuit are known.
Find the charge on plate of each capacitor.

Q.15 The lower plate of a parallel plate capacitor lies


Q.19 In the network given in the figure a potential
on an insulating plane. The upper plate is suspended
difference of 10 volts is applied across the two points
from one end of a balance. The two plates are joined
A and B find;
together by a thin wire and subsequently disconnected.
The balance is then counterpoised. A voltage V=5000 5F 4F
volt is applied between the plates. What additional A
8F 2F
mass should be placed to maintain balance? The 3F
distance between the plates is d=5mm and the area of 8F
6F 6F
each plate is A=100cm2? [all the elements other than B
plates are massless and nonconducting] (i) The potential difference across the capacitor of 2 µF .
(ii) The charge in both of 6 µF capacitors.

Rigid road
1 9 . 4 6 | Electric Potential and Capacitance

Q.20 In the figure shown here, each capacitance C1 in Q.26 A capacitor of capacitance C1=1.0µF withstands
the network is 3 µF and each capacitance C2 is 2µF the maximum voltage V1=6.0 kV while a capacitor of
capacitance C2 = 2.0µF withstands maximum voltage
(i) Compute the equivalent capacitance of the network
V2 = 4.0 kV. What voltage will the system of these two
between the points a and b.
capacitors withstand, if they are connected in series?
(ii) Calculate charge on each capacitors nearest to a and
b when Vab =900volt.
Q.27 Two identical parallel plate air capacitors are
(iii) With 900 volt across a and b, compute Vcd connected in one case in parallel and in the other in
series. In each case the plates of one capacitor are
C1 C1 C C1
a brought closer together by a distance a and the plates
of the other are moved apart by the same distance a.
C2 C2 C1
How will the total capacitance of each system change
b as result of such manipulation ?
C1 C1 D C1

Q.28 A 3 mega ohm resistor and 1 µF capacitor are


Q.21 If you have serval 2.0 µF capacitors, each capable connected in a single loop circuit with a source of
of withstantding 200 volts without breakdown, constant 4 volt. At one second after the connection is
how would you assemble a combination having an made what are the rates at which;
equivalent capacitance of; (i) The charge on the capacitor is increasing
(a) 0.40 µF or of (ii) Energy is being stored in the capacitor
(b) 1.2 µF , each capable of withstanding 100 volts. (iii) Joule heat is appearing in the resistor
(iv) Energy is being delived by the source
Q.22 Six 1 µF capacitors are so arranged that their
equivalent capacitance is 0.70 µF . If a potential
Q.29 A capacitor of capacity 1 µF is connected in closed
difference of 600 volt is applied to the combination,
series circuit with a resistance of 107 ohms, an open key
what charge will appear on each capacitor?
and a cell of 2V with negligible internal resistance:

Q.23 Two condensers are in parallel and the energy (i) When the key is switched on at time t=0, find;
of the combination is 103 J, when the difference of (a) The time constant for the circuit.
potential between their terminals is 2 volts. With the
(b) The charge on the capacitor at steady state.
same two condensers in series, the energy is 1.6x102 J
for the same differences of potential across the series (c) Time taken to deposit charge equaling half that at
combination. What are their capacities? steady state.
(ii) If after fully charging the capacitor, the cell is shorted
Q.24 A capacitor of capacitance C1=1.0 µF charged by zero resistance at time t=0, find the charge on the
upto a voltage V = 110V is connected in a parallel to capacitor at t = 50s.
the terminals of a circuit consisting of two uncharged
capacitor connected in series and possessing the
Q.30 An electric dipole, when held at 30o with respect
capacitance C2 = 2.0 µF and C3 = 3.0 µF . What charge
to a uniform electric field of 30 × 10 4 NC−1 experiences
will flow through the connecting wires?
a torque of 27 × 1026 N m. Calculate the dipole moment
of the dipole.
Q.25 Two capacitors A and B are connected in series
across a 100V supply and it is observed that the potential
Q.31 A regular hexagon of side 10 cm has a charge 5 µ
difference across them are 60V and 40V. A capacitor
C at each of its vertices. Calculate the potential at the
of 2 µF capacitance is now connected in parallel with
center of the hexagon.
A and the potential difference across with A and the
potential difference acorss B rise to 90V. Determine the
capacitance of A and B. Q.32 What is the work done in moving a 2 µ C point
charge from corner A to B of a square ABCD, when a 10
µ C charge exists at the center of the square?
P hysi cs | 19.47

Q.33 The potential at a point 0.1 m from an isolated Q.2 In the circuit shown, a potential difference of 60V
point charge is +100 volt. Find the nature and is applied across AB. The potential difference between
magnitude of the point charge. the point the point M and N is

A 2C
Q.34 Two charges equal to +20 µ C and -10 µ C are M
placed at points 6 cm apart. Find the value of the
60V
potential at a point distant 4 cm on the right bisector C C
of the line joining the two charges.
B N
2C
Q.35 A system has two charges qA = 2.5 × 10‒9 C and qB
= ‒2.5 × 10‒7C located at points A: (0, 0,-15) cm and B: (A) 10V (B) 15V (C) 20V (D) 30V
(0, 0, +15) cm, respectively. What are the total charge
and electric dipole moment of the system? Q.3 In the circuit shown in figure, the ratio of charges
on 5 µF and 4 µF capacitor:
Q.36 A test charge ‘q’ is moved without acceleration
2F
from A to C along the path from A to B and then from 3F
B to C in electric field E as shown in the figure. (i) 5F
Calculate the potential difference between A and C. (ii)
At which point (of the two) is the electric potential more
and why? 4F
(2, 3)
B E 6V
(A) 4/5 (B) 3/5 (C) 3/8 (D) ½

(2, 0) (6, 0) Q.4 From a supply of identical capacitors rated 8 µF ,


C A 250V the minimum number of capacitros required to
form a composite 16 µF , 1000V is:

Q.37 An electric dipole is held in a uniform electric field. (A) 2 (B) 4 (C) 16 (D) 32
(i) Show that the net force acting on it is zero.
Q.5 In the circuit shown, the energy stored in 1 µF
(ii) The dipole is aligned parallel to the field. capacitor is
3F 5F
Find the work done in rotating it through an angle of
180°.
1F

4F
Exercise 2
24V

Single Correct Choice Type


(A) 40 µJ (B) 64 µJ (C) 32 µJ (D) None

Q.1 A capacitor of capacitance C is charged to a potential


Q.6 If charge on left plane of the 5 µF capacitor in
difference V from a cell and then disconnected from
the circuit segment shown in the figure is -20 µC , the
it. A charge +Q is now given to its positive plate. The
charge on the right plate of 3 µF capacitor is
potential difference across the capacitor is now
3F
Q 2F
(A) V (B) V+ 5F
C

Q Q
(C) V+ (D) V- , if V< CV 4F
2C C
(A) +8.57 µC (B) -8.57 µC
(C) +11.42 µC (D) -11.42 µC
1 9 . 4 8 | Electric Potential and Capacitance

Q.7 What is equivalent capacitance of the system of Previous Years’ Questions


capacitors between A & B.
A Q.1 Seven capacitors each capacitance 2 µF are
connected in a configuration to obtain an effective
10
C C C capacitance µF . Which of the following combinations
11
B will achieve the desired result?  (1990)
(A) 7.6C (B) 1.6C (C) C (D) none

Q.8 Three capacitors 2 µF , 3 µF and 5 µF can withstand


(A) (B)
voltages to 3V, 2V and IV respectively. Their series
combination can withstand a maximum voltage equal
to
(A) 5Volts (B) (31/6) Volts
(C) (26/5) Volts (D) None (A) (B)
(A) (B)
(C) (D)
Q.9 A parallel plate capacitor has an electric field
of 105V/m between the plates. If the charge on the
capacitor plate is 1 µC , then the force on each capacitor
plate is
(A) 0.1 N (B)
(A)
0.05 N (C) 0.02 N (D) 0.01 N (B)
(C) (D)
(C) (D)

Q.10 A capacitor is connected to a battery. The force


of attraction between the plates when the separation
between them is halved.
(A) Remains the same   (B) Becomes eight times
(C) (D)
(C) Becomes four times   (D) Becomes two times

d
Q.11 A parallel plate capacitor is
make as shown in figure. Find Q.2 A parallel combination of 0.1MΩ resistor and
capacirance and equivalent dielectric k1 k2
a 10 µF capacitor is connected across a 1.5V source
constant. of negligible resistance. The time required for the
capacitor to get charged upto 0.75V is approximately
d/2 d/2
(in second)  (1997)
2 ε0 A(k1k 2 ) 4 ε0 A(k1k 2 )
(A) (B) (A) Infinite (B) loge2 (C) log102 (D) Zero
k1 + k 2 k1 + k 2
Q.3 For the circuit shown, which of the following
5 ε0 A(k1k 2 ) 3 ε0 A(k1k 2 )
(C) (D) statement is true? (1999)
k1k 2 k1 + k 2
V1 = 30 V V2 = 20 V
+ - + -

Q.12 A capacitor stores 60 µC charge when connected S1


C1= 2p F
S3
C2= 2p F
S2
across a battery. When the gap between the plates is
filled with a dielectric, a charge of 120 µC flows through
the battery. The dielectric constant of the material (A) With S1 closed, V1=15V, V2=20V
inserted is :
(B) With S3 closed,V1= V2 = 25V
(A) 1 (B) 2 (C) 3 (D) None
(C) With S1 and S2 closed, V1 = V2 = 0
(D) With S3 closed, V1 = 30 V, V2 = 20V
P hysi cs | 19.49

Q.4 Consider the situation shown in the figure. The Q.8 4 charges are placed each at distance ‘a’ from
capacitor A has a charge q on it whereas B is uncharged. origin. The dipole moment of configuration is (1983)
The charge appearing on the capacitor B after a long y
time after the switch is closed is (2001)
3q
q
+ -
+ - x
+ - -2q -2q
+ - S
+ - q
+ -
A B
(A) Zero (B) q/2 (C) q (D) 2q (A) 2 q a ˆj (B) 3 q a ˆj
(C) 2 a q [iˆ + ˆj] (D) None
Q.5 A parallel plate capacitor C with plates of unit are
and separation d is filled with a liquid of dielectric Q.9 n small drops of same size are charged to V volts
d each. If they coalesce to form a single large drop, then
constant K=2. The level of liquid is initially. Suppose
3 its potential will be (1985)
the liquid level decreases at a constant speed V, the (A) V/n (B) Vn (C) Vn1/3 (D) Vn2/3
time constant as a function of time t is (2008)
Q.10 A hollow metal sphere of radius 5 cm is charged
such that the potential on its surface is 10 V. The
potential at the center of the sphere is (1998)
C
(A) 0 V
d R
d (B) 10 V
3
(C) Same as at point 5 cm away from the surface outside
sphere.
(D) Same as a point 25 cm away from the surface.

(A)
6ε0R
(B)
(15d + 9vt ) ε0R Q.11 If the electric potential of the inner metal sphere
5d + 3vt 2d2 − 3dvt − 9v 2 t2 is 10 volt & that of the outer shell is 5 volt, then the
potential at the center will be: (1999)

(C)
6ε0R
(D)
(15d − 9vt ) ε0R (A) 10 volt (B) 5 volt (C) 15 volt (D) 0
5d − 3vt 2
2d + 3dvt − 9v t 2 2

Q.12 Three concentric metallic spherical shell A, B


Q.6 When a negative charge is released and moves in and C or radii a, b and c (a<b<c) have surface charge
electric field, it moves towards a position of  (1982) densities - σ , + σ , and - σ respectively. The potential
of shell A is : (1986)
(A) Lower electric potential and lower potential energy
(A) ( σ ε0 ) [a+b-c] (B) ( σ ε0 ) [a-b+c]
(B) Lower electric potential and higher potential energy
(C) ( σ ε0 ) [b-a-c] (D) None
(C) Higher electric potential and lower potential energy
(D) Higher electric potential and higher potential energy Q.13 A hollow metal sphere of radius 5 cm is charged
such that the potential on its surface is 10 V. The
Q.7 An infinite non - conducting sheet of charge has potential at the centre of the sphere is (1983)
a surface charge density of 10–7 C/m2. The separation (A) Zero
between two equipotential surfaces near the sheet
(B) 10 V
whose potential differ by 5V is (1983)
(C) Same as at a point 5 cm away from the surface
(A) 0.88 cm (B) 0.88 mm (C) 0.88 m (D) 5 × 10‒7 m
(D) Same as at a point 25 cm away from the surface
1 9 . 5 0 | Electric Potential and Capacitance

Q.14 A solid conducting sphere having a charge Q is Then the potential at the origin due to the above
surrounded by an uncharged concentric conducting system of charges is  (1998)
hollow spherical shell. Let the potential difference q
between the surface of the solid sphere and that of the (A) Zero (B) 8πε x ln2
0 0
outer surface of the hollow shell be V. If the shell is now
given a change of -3Q, the new potential difference q ln(2)
between the same two surfaces is  (1989) (C) Infinite (D)
4 πε0 x0
(A) V (B) 2 V (C) 4 V (D) -2 V
Q.19 Two identical metal plates are given positive
Q.15 Two identical thin rings, each of radius R, are charges Q1 and Q2 (< Q1 ) respectively. If they are
coaxially placed a distance R apart. If Q1 and Q2 are now brought close together to form a parallel plate
respectively the charges uniformly spread on the two capacitor with capacitance C, the potential difference
rings, the work done in moving a charge q from the between them is  (1999)
center of one ring to that of the other is (1992)
(A) (Q1 + Q2 ) / 2C (B) (Q1 + Q2 ) / C
(A) Zero
(C) (Q1 − Q2 ) / C (D) (Q1 − Q2 ) / 2C
(B) q(Q1 − Q2 )( 2 − 1) / 2(4 πε0R)

(C) q 2(Q1 + Q2 ) / (4 πε0R) Q.20 A parallel plate capacitor of area A, plate


separation d and capacitance C is filled with three
(D) q(Q1 / Q2 )( 2 + 1) 2(4 πε0R)
different dielectric materials having dielectric constants
K1 ,K 2 and K 3 as shown, If a single dielectric material
Q.16 Two point charges +q and –q are held fixed at (-d, is to be used to have the same capacitance C in this
0) and (d, 0) respectively of a x-y co-ordinate system. capacitor then its dielectric constant K is given by
Then  (1995)  (2000)
A/2 A/2
(A) The electric field E at all point on the x-axis has the
d
same direction. K1 K2 2
d
(B) Work has to be done in bringing a test charge from K3
∞ to the origin
(C) Electric field at all point on y-axis is along x-axis
(D) The dipole moment is 2qd along the x-axis 1 1 1 1 1 1 1
(A) = + + (B)
= +
K K1 K 2 2K 3 K K1 + K 2 2K 3
Q.17 A parallel plate capacitor of capacitance C is 1 K1K 2 K1K 3 K K
connected to a battery and is charged to a potential (C)
= + 2K 3 =(D) K + 2 3
K K1 + K 2 K1 + K 3 K 2 + K 3
difference V. Another capacitor of capacitance 2C is
similarly charged to a potential difference 2V. The
charging battery is now disconnected and the capacitors Q.21 A uniform electric field pointing in positive
are connected in parallel to each other in such a way x-direction exists in a region. Let A be origin, B be he
that the positive terminal of one is connected to the point on the x-axis at x=+1 cm and C be the point on
negative terminal of the other. The final energy of the y-axis at y=+1 cm. Then the potentials at the points
configuration is  (1995) A, B and C satisfy  (2001)
(A) VA < VB (B) VA > VB
3 25 2 9
(A) Zero (B) CV 2 (C) CV (D) CV 2 (C) VA < VC (D) VA > VC
2 6 2

Q.18 A charge +q is fixed at each of the points Q.22 Two equal point charges are fixed at x=-a and
=x x= 0, x 3x=
0 , x 5x 0 .....∞ on the x-axis x=+a on the x-axis. Another point charge Q is placed at
and a charge –q is fixed at each of the points the origin. The change in the electrical potential energy
=x 2x= 0, x 4x=0, x 6x0 .....∞ . Here, x0 is a positive of Q, when it is displaced by a small distance x along
constant. Take the electric potential at a point due to a the x-axis, is approximately proportional to (2002)
charge Q at a distance r from it to be Q/ 4 πε0r .
(A) x (B) x2 (C) x3 (D) 1/x
P hysi cs | 19.51

Q.23 Two identical capacitors, have the same y


capacitance C. One of them is charged to potential
V1 and the other to V2 . Likely charged plates are B
then connected. Then, the decrease in energy of the
combined system is (2002) C
x
60o O
1 1
(A) C(V12 − V22 ) (B) C(V12 + V22 )
4 4
A
1 1
(C) C(V1 − V2 )2 (D) C(V1 + V2 )2
4 4
q
(A) The electric field at point O is directed
2
Q.24 A long, hollow conducting cylinder is kept coaxially along the negative x-axis 8 πε 0R
inside another long, hollow conducting cylinder of
larger radius. Both the cylinders are initially electrically (B) The potential energy of the system is zero
neutral.  (2007)
(C) The magnitude of the force between the charges at
(A) A potential difference appears between the two
q2
cylinders when a charge density is given to the inner C and B is
cylinder 54 πε0R 2
q
(B) A potential difference appears between the two (D) The potential at point O is
cylinders when a charge density is given to the outer
12πε0R
cylinder
Q.27 A parallel plate capacitor C with plates of unit
(C) No potential difference appears between the two area and separation d is filled with a liquid of dielectric
cylinders when a uniform line charge is kept along the
d
axis of the cylinders constant K = 2. The level of liquid is initially. Suppose
3
(D) No potential difference appears between the two the liquid level decreases at a constant speed v, the
cylinders when same charge density is given to both time constant as a function of time t is  (2008)
the cylinders

Q.25 Positive and negative point charges of equal


 a  −a  C
magnitude are kept at 0,0,  and  0,0,  ,
 2  2  R
d d
respectively. The work done by the electric field when
3
another positive point charge in moved from (-a, 0, 0)
to (0, a, 0) is  (2007)
(A) Positive
(B) Negative
(C) Zero 6ε0R (15d + 9vt)ε0R
(A) (B)
(D) Depends on the path connecting the initial and final 5d + 3vt 2d2 − 3dvt − 9v 2 t2
position
6ε0R (15d − 9vt)ε0R
(C) (D)
q q 2q 5d − 3vt 2d2 + 3dvt − 9v 2 t2
Q.26 Consider a system of three charges , and −
3 3 3
placed at points A, B and C respectively, as shown in the
figure. Take O to the center of the circle of radius R and
angle CAB= 60o  (2008)
1 9 . 5 2 | Electric Potential and Capacitance

Q.28 A 2µF capacitor is charged as shown in the figure. Q.33 A uniformly charged solid sphere of radius R
The percentage of its stored energy dissipated after the has potential V0 (measured with respect to ∞ ) on
switch S is turned to potion 2 is (2011)
its surface. For this sphere the equipotential surfaces
1 2 3V0 5 V0 3 V0 V
with potentials , , and 0 have radius
2 4 4 4
R1 ,R 2 ,R3 and R 4 respectively. Then  (2015)

2F 8F (A) R1 ≠ 0 and (R 2 − R1 ) > (R 4 − R 3 )


(B) R1 = 0 and R 2 < (R 4 − R 3 )
(C) 2R < R 4

(A) 0% (B) 20% (C) 75% (D) 80% (D) R1 = 0 and R 2 > (R 4 − R 3 )

Q.29 Two capacitors C1 and C2 are charged to 120 V Q.34 In the given circuit, charge Q2 on the 2 µ F
and 200 V respectively. It isfound that by connecting capacitor changes as C is varied from 1 µ F to 3 µ F . Q2
them together the potential on each one can be as a function of ‘C’ is given properly by : (figures are
madezero. Then:  (2013) drawn schematically and are not to scale) (2015)

(A) 3C1 = 5C2 (B) 3C1 + 5C2 =


0
(C) 9C1 = 4 C2 (D) 5C1 = 3C2 1F
C
Q.30 A charge Q is uniformly distributed over a long 2F
rod AB oflength L as shown in the figure. The electric
potential at the point O lying at a distance L from the
end A is:  (2013) E

Charge Charge

O A B
(A) Q2 (B) Q2
L L
C C
1F 3F 1F 3F
3Q Q
(A) (B)
4 π ε0 L 4 π ε0 L ln 2 Charge Charge
Q ln 2 Q
(C) (D) (C) Q2 (D) Q2
4 π ε0 L 8 π ε0 L
C C
1F 3F 1F 3F
Q.31 A parallel plate capacitor is made of two circular
plates separated by a distance of 5 mm and with a
dielectric of dielectric constant 2.2 between them. When
the electric field in the dielectric is 3 × 104 V/m, the charge Q.35 A combination of capacitors is set up as shown in
density of the positive plate will be close to:  (2014) the figure. The magnitude of the electric field, due to
a point charge Q (having a charge equal to the sum of
(A) 3 × 10 4 C / m2 (B) 6 × 10 4 C / m2 the charges on the 4 µF and 9 µF capacitors), at a point
(C) 6 × 10−7 C / m2 (D) 3 × 10−7 C / m2 distant 30 m from it, would equal:  (2016)

 (A) 360 N/C (B) 420 N/C


Q.32 Assume that an electric field E = 30 x2 ˆi exists in
(C) 480 N/C (D) 240 N/C
space. Then the potential difference VA − V0 , where V0
is the potential at the origin and VA the potential at
x = 2 m is: (2014)
(A) – 80 V (B) 80 V
(C) 120 V (D) None of these
P hysi cs | 19.53

JEE Advanced/Boards

Exercise 1 Q.5 Find the charge on the capacitor C=1 µF in the


circuit shown in the figure.
Q.1 A parallel plate capacitor has plate area 100 cm2 and
place separation 1 cm. A glass plate (k = 6) of thickness C 1F
6 mm and an ebonite plate (k = 4) are inserted to fill the
gap between the plates. Find new capacitance. 6F 4F
8F

Q.2 In the given network (See figure) if Potential 2F 2F 12F
difference between p and q is 2V and C2=3C1. Then find
difference between a & b.
Q.6 The figure shows a circuit consisting of four capacitors.
+ -
Find the effective capacitance between X and Y

b 1F 1F
a C2 1F
C1 X Y
p 2F
q
C1 C2
Q.7 Five identical capacitor plates, each of area A are
Q.3 Find field produced in the circuit shown in figure on
arranged such that adjacent plates are at a distance‘d’
closing the switch S.
apart, the plates are connected to a source of emf V as
2F
shown in figure. The charge on plate 1 is ------ and that
+20C -20C on plate 4 is ----
S
+50C -50C
1 2 3 4 5 -
5F V
+

Q.4 In the following circuit (See figure), the resultant


capacitance between A and B is 1 µF . Find equivalent
capacitance of the circuit. Q.8 Find the capacitance of the system shown in figure.
Plate area = A
1F 1F 10V
d k=1 k=2
1F 1F
1F k=3 k=4
1F
1F d

1F 1F 1F


1F 1F C=1F 1F
Q.9 Figure shows three concentric conducting spherical
shells with inner and outer shells earthed and the
1F 1F 1F 1F 1F
middle shell is given a charge q. find the electrostatic
energy of the system stored in the region I and II.

3.5r

q
2.5r I
II
III
1 9 . 5 4 | Electric Potential and Capacitance

Q.10 Find the ratio between the erergy stored in 5µF (a) Charge on capacitor as a function of time.
capacitor to the 4 µF capacitor in the given circuit in
(b) Energy of the capacitor at t=20s.
stedy state.
5 5F Q.15 In the circuit shown in figure the capacitance of
each capacitor is equal to C and resistance R. One of
the capacitor was charged to a voltage V and then at
the moment t=0 was shorted by means of the switch
4F 4
S. find: R

2V
C C
Q.11 A solid conducting sphere of radius 10cm is S
enclosed by a thin metallic shell of radius 20cm. a W
charge q=20 µC is given to the inner sphere. Find
the heat generated in the process, the inner sphere is (a) The current in the circuit as a function of time t.
connected to the shell by a conducting wire. (b) The amount of generated heat.

Q.12 In the circuit shown here, At the steady state, the Q.16 The two identical parallel plates are given charges
charge on the capacitor is ------------. as shown in figure. If the plate area of either face of each
R1 plate is A and separation between plates is d, then find
the amount of heat liberated after closing the switch.
C
E R3 +3q +q
R2

Q.13 For the arrangement shown in the figure, the key


is closed at t=0. C2 is initially uncharged while C1 has a
charge of 2 µC S

30 C2

2F Q.17 Five identical conducting plates 1, 2, 3, 4 & 5 are


30 60 fixed parallel to and equidistant from each other (see
1F C1 figure). Plates 2 & 5 are connected by a conductor while
1 & 3 are joined by another conductor. The junction of
1 & 3 and the plate 4 are connected to a source of
K 9V
constant e.m.f. V0 find:
(a) Find the current coming out of the battery just after 5
switch is closed. 4 -
3
(b) Find the charge on the capacitors in the steady state
condition. 2
1 +
Q.14 In the circuit shown in figure R1=R2=6R3= 300M Ω , (i) The effective capacity of the system between the
C=0.01 µF and E=10V. The switch is closed at t=0, find terminals of the source.
S
(ii) The charges on plates 3 & 5. Given
R1 R3 D = Distance between any 2 successive plate &
R2 A = Area of either face of each plate.

E C
P hysi cs | 19.55

Q.18 Find the charge flown through the switch from A dielectric of dielectric  S
to B when it is closed. constant (or relative
permittivity) 3. Find
V A C B C
3F 3F 10V 6F the ratio of the total
5V 3F electrostatic enegry
6F A B stored in both capacitors
10V 5V 6F before and after the introduction of the dielectric.

Q.23 A parallel plate capacitor has plates with area A &


Q.19 Three capacitors of 2 µF , 3 µF and 5 µF are separation d. A battey charges the plates to a potential
independently charged with batteries of emf’s 5V, difference of V0. The battery is then disconnected
20V and 10V respectively. After disconnecting from & a di-electric slab of constant K & thickness d is
the voltage source. These capacitors are connected introduced. Calculate the positive work done by the
as shown in figure with their positive polarity plates system (capacitor+slab) on the man who introduces
connected to A and negative polarity is earthed. Now the slab.
a battery of 20V and an uncharged capacitor of 4 µF
capacitance are connected to the junction A as shown Q.24 A parallel plate capacitor is filled by a di-electric
with a switch S. when switch is closed, find: whose relative permittivity varies with the applied
voltage according to the law= α V, where α =1 per
3F volt. The same (but containing no di-electric) capacitor
2F
charged to a voltage V=156 volt is connected in parallel
A 20 V 4F to the first “non-linear” uncharged capacitor. Determine
5F the final voltage Vf across the capacitors.

(a) The potential of the junction A. Q.25 Two parallel plate capacitors A & B have the same
separation d=8.85+104m between the plates. The plate
(b) Final charge on all four capacitros. areas of A & B are 0.04 m2 & 0.02m2 respectively. A
slab of di-electric constant (relative permittivity) K=9
Q.20 In the circuit shown in figure, find the amount of has dimensions such that it can exactly fill the space
heat generated when switch s is closed. between the plates of capacitor B.
2F S
A B A B

5V 10V 3F

10V
4F
(i) The di-electric slab is placed inside A and A is then
Q.21 the connection shown in figure are established charged to a potential difference of 110volt. Calculate
with the switch S open. How much charge will flow the capacitance of A and the energy stored in it.
through the switch if it is closed? (ii) The battery is disconnectd & then the di-electric slab
is removed from A. find the work done by the external
1F 2F agency in removing the slab from A.

24V
S (iii) The same di-electric slab is now placed inside B,
filling it completely. The two capacitors A & B are then
2F 1F
connected. Calculate the energy stored in the system.

Q.26 Two square metallic plates of 1m side are kept 0.01m


Q.22 The figure shows two identical parallel plate apart, like a parallel plate capacitor, in air in such a way that
capacitors connected to a battery with the switch S one of their edges is perpendicular, to an oil surface in a
closed. The switch is now opened and the free space tank filled with an insulating oil. The plates are connected
between the plates of the capacitors is filled with a
1 9 . 5 6 | Electric Potential and Capacitance

to a battery of e.m.f. 500 volt. The plates are then lowered Q.32 A dipole is placed at origin of coordinate system
vertically into the oil at a speed of 0.001 m/s. calculate the as shown in figure, find the electric field at point P (0, y).
current drawn from the battery during the process. y
[Di-electric constant of oil= ε0 =8.85x10 C / N m ]
12 2 2 2
P (0,y)

Q.27 A 10 µF and 20 µF capacitor are connected to a 10V


x
cell in parallel for some time after which the capacitors P 45
o

are disconnected from the cell and reconnected at


t=0 with each other, in series, through wires of finite
resistance. The +ve plate of the first capacitor. Draw the Q.33 Two concentric rings of radii r and 2r are placed
graph which best describes the charge on the +ve plate with center at origin. Two charges +q each are fixed at
of the 20 µF capacitor with increasing time. the diametrically opposite points of the rings as shown
in figure. Smaller ring is now rotated by an angle 90c
Q.28 A capacitor of capacitance C0 is charged to a about Z-axis then it is again rotated by 90o about
potential V0 and then isolated. A small capacitor C is Y-axis. Find the work done by electrostatic force in each
then charged from C0, discharged and charged again, step. If finally larger ring is rotated by 90o about X-axis,
the process being repeated in times. The potential of the find the total work required to perform all three steps.
large capacitor has now fallen to V. Find the capacitance y
of the small capacitor. If V0=100volts, V=35volts, find +q
the value of n for C0=0.2 µF and C=0.01075 µF . Is it
possible to remove charge on C0 this way? +q

Q.29 In the figure shown initially switch is open for a x


long time. Now the switch is closed at t=0. Find the
charge on the rightmost capacitor as a function of time +q
given that it was initially unchanged.
z +q

R S
V R C C Q.34 A cone made of insulating material has a total
charge Q spread uniformly over its sloping surface.
Calculate the energy required to take a test charge q
from infinity to apex A of cone.
Q.30 Two capacitors A and B with capacitor are connected
as shown in figure with one wire from each capacitor
free. The upper plate of A is positive and that of B is A
negative. An uncharged 2 µF capacitor C with lead wires
falls on the free ends to complete the circuit. Calculate: AB=L
B
C 2F
The slant length is L.
+ -
3F 2F
A 100V B 180V Q.35 A non-conducting disc of radius a and uniform
positive surface charge density σ is placed on the
(i) The final charges on the three capacitors (ii) The ground, with its axis vertical. A particle of mass m &
amount of electrostatic energy stored in the system positive charge q is dropped, along the axis of the disc,
before and after the completion of the circuit. from a height H with zero initial velocity. The particle
q 4 ε0 g
has = .
m σ
Q.31 Three charges 0.1 coulomb each are placed on
the comers of an equilateral triangle of side 1 m. If the (a) Find the value of H if the particle just reaches the
energy is supplied to this system at the rate of 1 kW, disc.
how much time would be required to move one of the (b) Sketch the potential energy of the particle as a
charges onto the midpoint of the line joining the other function of its height and find its equilibrium position.
two?
P hysi cs | 19.57

Exercise 2 Q.6 Four metallic plates are arranged as shown in the


figure. If the distance between each plate is d, The
Single Correct Choice Type capacitance of the given system between points A and
B is (Given d<<A)

Q.1 Two capacitor having capacitance 8 µF and 16 d


µF have breaking voltage 20V and 80V. They are d
combined in series. The maximum charge they can A
d
B
store individually in the combination is:
(A) 160 µC (B) 200 µC
ε0 A 2ε0 A 3ε0 A 4 ε0 A
(C) 1280 µC (D) None of these (A)   (B)   (C)   (D)
d d d d

Q.2 Three plate A, B and C each of area Q.7 Find the equivalent capacitance across A and B
0.1m2 are separated by 0.885mm from A B
each other as shown in the figure. A C 10V
d
10V battery is used to charge the d
A B
system. The energy stored in the system is d
(A) 1µ J (B) 10-1µ J (C) 10-2µ J (D) 10-3µ J
ε0 A 2ε0 A 3ε0 A 4 ε0 A
(A)   (B)   (C)   (D)
d d d d
Q.3 A capacitor of capacitance C is initially charge to a
potential difference of V. Now it is connected to battery
Q.8 The diagram shows four capacitors with
of 2V with opposite polarity. The ratio of heat generated
capacitances and break down voltage as mentioned.
to the final erergy stored in the capacitor will be
What should be the maximum value of the external emf
(A) 1.75 (B) 2.25 (C) 2.5 (D) ½ source such that no capacitor breaks down? [Hint: First
of all find out the break down voltage of each branch.
Q.4 Five conducting parallel plates having E After that compare them.]
area A and separation between them A B
123 45 3C; 1kV 2C; 2kV
being d, are placed as shown in the figure.
Plate number 2 and 4 are connected with 1 2
a wire and between point A and B, a cell
of emf E is connected. The charge flown 7C; 1kV 3C; 2kV
E
3 ε0 AE 2 ε0 AE 4ε0 AE ε0 AE
(A) (B) (C) (D)
4 d 3 d d 2d (A) 2.5kV    (B) 10/3kV   (C) 3kV   (D) 1kV

Q.5 The plates S and T of an uncharged parallel plate Q.9 A conducting body 1 has some initial charge Q, and
capacitor are connected acorss a battey. The battery is its capacitance is C. There are two other conducting
then disconnected in a system as shown in the figure. bodies, 2 and 3, having capacitances: C2=2C and C3→ ∞.
The system shown is in equilibrium. All the strings are Bodies 2 and 3 are initially uncharged. “Body 2 is touched
insulating and massless. The magnitude of charge on with body 1 and touched with body 3, and the removed.”
one of the capacitor plates is: [Area of plates = A] This process is repeated N times. Then, the charge on
k S T body 1 at the end must be
KCE
(A) Q/3n (B) Q/3n-1
K +1
m (C) Q/2n (D) None

4mgAε0
(A) 2mgAε0 (B) Q.10 In the adjoining figure, capacitor (1) and (2) have
k a capacitance ‘C’ each. When the dielectric of dielectric
2mgAε0 constant K is inserted between the plates of one of the
(C) mgAε0 (D) capacitor, the total charge flowing through battery is
k
1 9 . 5 8 | Electric Potential and Capacitance

Conductor Dielectric Q.13 Condenser A has a capacity of 15µF when it is


filled with a medium of dielectric constant 15. Another
condenser B has a capcity 1 µF with air between the
plates. Both are charged separately by a battery of
100V. After charging, both are connected in parallel
without the battery and the dielectric material being
removed. The common potential now is
x-0 x-0 x-2d x-3d x-4d x-5d C

(A) From B to C; n
n
KCE
(B) From C to B
K +1 n
S

(K − 1)CE 9V
(C) From B to C;
2(K + 1) (A) 400V (B) 800V

(K − 1)CE (C) 1200V (D) 1600V


(D) From C to B
2(K + 1)
Q.14 Two identical capacitors 1 and 2 are connected
in series to a battery as shown in figure. Capacitor
Q.11 The distance between plates of a parallel plate 2 contains a dielectric slab of dielectric constant
capacitor is 5d. Let the positively charged plate is at k as shown. Q1 and Q2 are the charges stored in the
X = 0 and negatively charged plate is at X = 5d. Two capacitors. Now the dielectric slab is removed and the
slabs one of conductor and other of a dielectric of corresponding charges are Q’1 and Q’2. Then
equal thickness d are inserted between the plates as
E C
shown in figure. B

Potential versus distance graph will look like: C C C


A
D
v v 1 2

Q1′ k +1 Q′ k + 1
(A) (B) (A) = (B) 2 =
Q1 k Q2 2

x x Q2′ k +1 Q′ k
(C) = (D) 1 =
v v Q2 2k Q1 2

Q.15 Four identical plates 1, 2, 3 and 4 are placed


(C) (D) parallel to each other at equal distance as shown in the
figure. Plates 1 and 4 are joined together and the space
x x between 2 and 3 is filled with a dielectric of dielectric
constant k=2. The capacitance of the system between 1
and 3 & 2 and 4 are C1 and C2 respectively.
Q.12 The distance between the plates of a charged
parallel plate capacitor is 5cm and electric field inside C1
The ratio is:
the plates is 200V cm-1. An uncharged metal bar of width C2
2cm is fully immersed into the capacitor. The length of 1
the metal bar is same as that of plate of capacitor. The
voltage across capacitor after the immersion of the bar 2
is 3
(A) Zero (B) 400V 4
(C) 600V (D) 100V 5 3 5
(A) (B) 1 (C) (D)
3 5 7
P hysi cs | 19.59

Q.16 A charged capacitor is allowed to discharge Q.20 A charged capacitor is allowed to discharge
through a resistance 2Ω by closing the switch S at through a resistor by closing the key at the instant t=0.
the instant t = 0. At time t = ln2 µs , the reading of the At the instant t=(ln 4) µS , the reading of the ammeter
ammeter falls half of its initial value. The resistance of falls half the initial value. The resistance of the ammeter
the ammeter is equal to is equal to
0.5F (A) 1 MΩ    (B) 1 Ω    (C) 2 Ω   (D) 2 MΩ
+ -
S
Multiple Correct Choice Type
A
2 Q.21 Two capacitors of 2 µF and 3 µF are charged to 150
(A) 0  (B) 2Ω   (C) 2KΩ   (D) 2MΩ volt and 120 volt respectively. The plates of capacitor
are connected as shown in the figure. A discharged
capacitor of capacity 1.5 µF falls to the free ends of the
Q.17 A capacitor C = 100 µF is connected to three wire. Then
resistor each of resistance 1k Ω and a battery of emf 1.5F
9V. The switch S has been closed for long time so as
to charge the capacitor. When switch S is opened, the
capacitor discharges with time constant +
2F
-
3F
150V 120V
C-0.5F - +
A
+ -
(A) Charge on the 1.5 µF capacitors is 180 µC

A (B) Charge on the 2 µ F capacitor is 120 µ C


2
(C) Positive charge flows through A from right to left.
(A) 33ms    (B) 5ms   (C) 3.3ms    (D) 50ms
(D) Positive charge flows thorugh A from left to right.

Q.18 In the circuit shown, when the key k is pressed


Q.22 In the circuit shown, each capacitor has a
at time t=0, which of the following statements about
capacitance C. The emf of the cell is E. If the switch S is
current I in the resistor AB is true.
closed C
K B
C C
2V 1000
1000 + -
E

(A) Positive charge will flow out of the positive terminal


(A) I = 2mA at all t of the cell.
(B) I oscillates between 1mA and 2mA (B) Positive charge will enter the positive terminal of
(C) I = 1mA at all t the cell
(D) At t = 0, I = 2mA and with time it goes to 1mA (C) The amount of charge flowing through the cell will
be CE.
Q.19 In the R-C circuit shown in the figure the total (D) The amount of charge flowing through the cell will
energy of 3.6x10-3 J is dissipated in the 10 Ω resistor be 4/3 CE.
when the switch S is closed. The initial charge on the
capacitor is
Q.23 A circuit shown in the figure consists of a battery of
S emf 10V and two capacitance C1 and C2 of capacitances
2F 10 1.0 µF and 2.0 µF respectively. Which of the options are
correct?
A B
C1 e C2
60
(A) 60 µC (B) 120 µC (C) 60 2µC (D) µC
2
1 9 . 6 0 | Electric Potential and Capacitance

(A) Charge on capacitor C1 is equal to charge on Q.27 A parallel plate air-core capacitor is connected
capacitor C2 across a source of constant potential difference. When a
dielectric plate is introduced between the two plates then:
(B) Voltage across capacitor C1 is 5V
(A) Some charge from the capacitor will flow back into
(C) Voltage across capacitor C2 is 10V
the source.
(D) Energy stored in capacitor C1 is two times the energy
(B) Some extra charge from the sourch will flow back
stored in capacitor C2.
into the capacitor.

Q.24 If Q is the charge on the plates of a capacitor of (C) The electric field intensity between the two plates
capacitance C, V the potential difference between the does not charge.
plates, A the area of each plate and d the distance (D) The electric field intensity between the two plates
between the plates, The force of attraction between the will decrease.
plates is

1  Q2  1  CV 2  Q.28 A parallel plate capacitor of plate area A and plate


(A)   (B)   separation d is charged to potential difference V and
2  ε0 A  2  d 
then the battery is disconnected. A slab of dielectric
constant K is then inserted between the plates of the
1  CV 2  1  Q2  capacitor so as to fill the space between the plates. If Q,
(C)   (D)  
2  Aε0  4  πε d2  E and W denote respectively, the magnitude of charge
 0 
on each plate, the electric field between the plates (after
the slab is inserted) and the work done on the system,
Q.25 Four capacitors and a battery are connected as
in question, in the process of inserting the slab, then
shown. The potential drop across the 7 µF capacitor is
6V. Then the; ε0 AV ε0KAV
(A) Q = (B) Q =
12F d d

V ε0 AV 2  1
E (C) E = (D)
= W 1 − 
7F Kd 2d  K
3.9F
3F Q.29 A parallel plate capacitor has a parallel slab
of copper inserted between and parallel to the two
(A) Potential difference across the 3 µF capacitor is 10V plates, without touching the plates. The capacity of the
capacitor after the introduction of the copper sheet is:
(B) Charge on the 3 µF capacitor is 42 µC
(A) minimum when the copper slab touches one of the
(C) Emf on the battery is 30V plates.
(D) Potential difference across the 12 µF capacitor is 10V (B) Maximum when the copper slab touches one of the
plates.
Q.26 The capacitance of a parallel plate capacitor is C
(C) Invariant for all positions of the slab between the
when the region between the plates has air. This region
plates.
is now filled with a dielectric slab of dielectric constant
k. the capacitor is connected to a cell of emf E, and the (D) Greater than that before introducing the slab.
slab is taken out
(A) Charge CE (k-1) flows through the cell Q.30 The plates of a parallel plate capacitor with no
dielectric are connected to a voltage source. Now a
(B) Energy E2C (k-1) is absorbed by the cell. dielectric of dielectric constant K is inserted to fill the
(C) The energy stored in the capacitor is reduced by whole space between the plates with voltage source
E2C (k-1) remaining connected to the capacitor.
1 (A) The energy stored in the capacitor will become
(D) The external agent has to do E2C(k − 1) amount of
work to take the slab out. 2 K-times
(B) The electric field inside the capacitor will decrease
to K-times
P hysi cs | 19.61

(C) The force of attraction between the plates will Assertion Reasoning Type
increase to K2 times.
(A) Statement-I is true, statement-II is true and
(D) The charge on the capacitor will increase to K-times statement-2 is correct explanation for statement-I.
(B) Statement-I is true, statement-II is true and
Q.31 A parallel-plate capacitor is connected to a cell. Its statement-II is not the correct explatnation for
positive plate A and its negative palte B have charges +Q statement-I.
and –Q respectively. A third plate C, identical to A and
(C) Statement-I is true, statement-II is false.
B, with charge+Q, is now introduced midway between
A and B, parallel to them, which of the following are (D) Statement-I is false, statement-II is true.
correct?
3Q Q.34 Statement-I: The electrostatic force between the
(A) The charge on the inner face of B is now −
2 plates of a charged isolated capacitor decreases when
dielectric fills whole space between plates.
(B) There is no change in the potential difference
between A and B. Statement-II: The electric field between the plates of a
(C) The potential difference between A and C is one- charged isolated capacitance decreases when dielectric
third of the potential difference between B and C. fills whole space between plates.

(D) The charge on the inner face of A is now Q/2. y y

(A) (B)
Q.32 The circuit shown in the figure consists of a battery
of emf ε =10V; a capacitor of capacitance C=1.0 µF and (d,0) (3d,0)
x x
(d,0) (3d,0)
three resistor of values R1=2 Ω , R2=2 Ω and R3=1 Ω .
Initially the capacitor is completely uncharged and the y
y
switch S is open. The switch S is closed at t=0.
R1 (C)
(D)
S x
(d,0) (3d,0) x
(d, 0) (3d,0)
 R2 R3

Q.35 Statement-I: If temperature is increased, the


(A) The current through resistor R3 at the moment the dielectric constant of a polar dielectrc decreases
switch closed is zero. whereas that of a non-polar dielectric does not change
significantly.
(B) The current through resistor R3 a long time after
the switch closed is 5A Statement-II: The magnitude of dipole moment of
(C) The ratio of current through R1 and R2 is always individual polar molecule decreases significantly with
constant. increase in temperature.

(D) The maximum charge on the capacitor during the


operation is 5 µC Q.36 Statement-I: The heat produced by a resistor in
any time t during the charging of a capacitor in a series
circuit is half the energy stored in the capacitor by that
Q.33 In the circuit shown in figure C1=C2=2 µF Then time.
charge stored in
Statement-II: Current in the circuit is equal to the rate
1 2 3
of increase in charge on the capacitor.
C1 C2

2 1 3

120V

(A) Capacitor C1 is zero (B) Capacitor C2 is zero


(C) Both capacitor is zero (D) Capacitor C1 is 40 µC
1 9 . 6 2 | Electric Potential and Capacitance

Comprehension Type Q.40 A long time after closing the switch


(A) Voltage drop acorss the capacitor is E
Paragraph 1: Two capacitors each having area A and
plate separation‘d’ are connected as shown in the E
(B) Current through the battery is
circuit. Each capacitor carries charge Q0/2. R1 + R 2
Q0 2
1  R 2E 
2 (C) Energy stores in the capacitor is C  
2  R1 + R 2 

(D) Current through the capacitor becomes zero.


Q0
2 Paragraph 3: The charge across the capacitor in two
different RC circuit 1 and 2 are plotted as shown in
The plates of one capacitor are slowly pulled apart by figure.
an external agent till the separation between them q
becomes 2d. The other capacitor is not disturbed. qmax 1
2
Q.37 The force applied by the external agent when the
separation between the plates is ‘X’ is given by O t

Q 02 d Q 02 d Q20 Q 02 d
(A) (B) (C) (D) Q.41 Choose the correct statement (S) related to the
3 Aε0 12 Aε0 2 Aε0 6 Aε0
two circuits.
(A) Both the capacitors are charged to the same charge.
Q.38 At this position the potential difference across the
capacitors is given by (B) The emf’s of cells in both the circuit are equal.

Q0 xd Q0 xd (C) The emf’s of the cell may be different.


(A) (B)
2Aε0 (x + d) Aε0 (x + d) (D) The emf’s E1 is more than E2

Q0 xd Q0 d
(C) (D) Q.42 Identify the correct statement (s) related to the R1,
4Aε0 (x + d) 2Aε0 x
R2, C1 and C2 of the two RC circuits.

(A) R1 > R2 if E1 =E2 (B) C1< C2 If E1=E2


Paragraph 2: In the circuit as shown in figure the switch
is closed at t = 0. R1 C2
(C) R1C1> R2C2 (D) <
R1 R2 C1

E C R2 Previous Years’ Questions


S
Q.1 A parallel plate capacitor of capacitance C is
Q.39 At the instant of closing the switch connected to a battery and is charged to a potential
(A) The battery delivers maximum current difference V. Another capacitor of capacitance 2C is
similarly charged to a potential difference 2V. The
(B) No current flows through C charging battery is now disconnected and the capacitors
(C) Voltage drop across R2 is zero are connected in parallel to each other in such a way
that the positive terminal of one is connected to the
(D) The current through the battery decreases with time
negative terminal of the other. The final energy of the
and finally becomes zero.
configuration is  (1995)

3 2 25 2 9 2
(A) Zero (B) CV (C) CV (D) CV
2 6 2
P hysi cs | 19.63

Q.2 Two identical metal plates are given positive charges Q.6 A parallel plate air capacitor is connected to a
Q1 and Q2(<Q1) respectively. If they are now brought battery. The quantities charge, voltage, electric field
close together to form a parallel plate capacitor with and energy associated with this capacitor are given by
capacitance C, the potential difference between them Q0, V0, E0 and U0 respectively. A dielectric slab is now
is  (1999) introduced to fill the space between the plates with the
battery still in connection. The corresponding quantities
(A) (Q1+Q2)/2C (B) (Q1+Q2)/C
now given by Q, V, E and U are related to the previous
(C) (Q1-Q2)/C (D) (Q1-Q2)/2C one as  (1985)
(A) Q>Q0 (B) V>V0
Q.3 A parallel plate capacitor of area A, plate separation
d and capacitance C is filled with three different (C) E>E0 (D) U> U0
dielectric materials having dielectric constants K1, K2
and K3 as shown. If a single dielectric material is to be Q.7 A parallel plate capacitor is charged and the
used to have the same capacitance C in this capacitor charging battery is then disconnected. If the plates
then its dielectric constant K is given by  (2000) of the capacitor are moved farther apart by means of
insulating handles  (1987)
A/2 A/2
(A) The charge on the capacitor increases
d
K1 K2 2 (B) The voltage across the plates increases
d (C) The capacitance increases
K3
(D) The electrostatic energy stored in the capacitor
increases.

A = Area of plates Q.8 A parallel plate capacitor of plate area A and plate
separation d is charged to potential difference V and
1 1 1 1 1 1 1 then the battery is disconnected. A slab of dielectric
(A) = + + (B)
= + constant K is then inserted between the plates of the
K K1 K 2 2K 3 K K1 + K 2 2K 3
capacitor so as to fill the space between the plates. If Q,
E and W denote respectively, the magnitude of charge
1 K1K 2 K1K 3 K K on each plate, the electric field between the plates (after
(C)
= + 2K 3 (D)
= K + 2 3
K K1 + K 2 K1 + K 3 K 2 + K 3 the slab in inserted), and work done on the system, in
question, in the process of inserting the slab, then
Q.4 Two identical capacitors, have the same capacitance  (1991)
ε0 AV ε0KAV
C. One to them is charged to potential V1 and the other (A) Q = (B) Q =
to V2. Likely charged plates are then connected. Then, d d
the decrease in energy of the combined system is
V ε0 AV 2  1
 (2002) (c) E = (D)
= W 1 − 
Kd 2d  K 
1
( )
(A) C V1 − V2 (B) C V1 + V2
4
2 2 1
4
2
( 2
)
Q.9 A dielectric slab of thickness d is inserted in a
1 1
C ( V1 − V2 ) (D) C ( V1 + V2 )
2 2
(C) parallel plate capacitor whose negative plate is at X=0
4 4 and positive plate is at X=3d. the slab is equidistant
from the plates. The capacitor is given some charge. As
1 2 X goes from 0 to 3d  (1998)
Q.5 A 2 µF capacitor is charged as 
shown in the figure. The percentage
(A) The magnitude of the electric field remains the same
of its stored energy dissipated after
the switch S is turned to position 2 is 2F 8F (B) The direction of the electric field remains the same
 (2011) (C) The electric potential increases continuously
(A) 0% (B) 20%  (D) The electric potential increases at first, then
(C) 75% (D) 80% decreases and again increases
1 9 . 6 4 | Electric Potential and Capacitance

Q.10 Two point charges Q and –Q/4 are separated by a (C) The direction of electric field at all points on the axis
distance x. Then (2001) will be along the axis.
Q x -Q/4 (D) If the ring is placed inside a uniform external electric
field then net torque and force acting on the ring would
(A) Potential is zero at a point on the axis which is x/3 be zero.
on the right side of the charge –Q/4.
Q.14 An electric dipole is placed at the center of a
(B) Potential is zero at a point on the axis which is x/5
sphere. Mark the correct answer (2003)
on the left side of the charge –Q/4.
(A) The flux of the electric field through the sphere is zero
(C) Electric field is zero at a point on the axis which is at
a distance x on the right side of the charge –Q/4. (B) The electric field is zero at every point of the sphere.
(D) There exists two points on the axis where electric (C) The electric potential is zero everywhere on the
field is zero. sphere.
(D) The electric potential is zero on a circle on the surface.
Q.11 An electric charge 10−8 C is placed at the point
(4m, 7m, 2m). At the point (1m, 3m, 2m), the electric
Q.15 For the situation shown in the figure below
 (2000)
(assume r>> length of dipole) mark out the correct
(A) Potential will be 18 V statement (s) (2002)
(B) Field has no Y-component p
Q (small dipole)
(C) Field will be along Z-axis r
(D) Potential will be 1.8 V (A) Force acting on the dipole is zero
pQ
Q.12 Potential at a point A is 3 volt and at a point B is (B) Force acting on the dipole is approximately
7 volt, an electron is moving towards A from B. (2004) and is acting upward 4 πε0r 3

(A) It must have some K.E. at B to reach A pQ


(C) Torque acting on the dipole is in clockwise
(B) It need not have any K.E at B to reach A direction 4 πε0r 2
pQ
(C) to reach A it must have more than or equal to 4 eV (D) Torque acting on the dipole is in anti-
K. E at B. clockwise direction. 4 πε0r 2

(D) When it will reach A, it will have K.E. more than or at


least equal to 4eV if it was released from rest at B. Q.16 For the situation shown in the figure below, mark
out the correct statement (s) (2006)
Q.13 The figure shows a non - conducting ring which
has positive and negative charge non uniformly B
distributed on it such that the total charge is zero. q
Which of the following statements is true? (2007) d R

+ - Hollow neutral conductor


+ -
+ - q
+ - (A) Potential of the conductor is .
4 πε0 (d + R)
+
+ - -
+ O q
(B) Potential of the conductor is .
-
+ - axis 4 πε0 d
+ -
+
(C) Potential of the conductor can’t be determined as
-
nature of distribution of induced charges is not known
(A) The potential at all the points on the axis will be zero.
(D) Potential at point B due to induced charges is
(B) The electric field at all the points on the axis will be −qR
zero. 4 πε0 (d + R)d
P hysi cs | 19.65

Paragraph for Question No. 17 to 19 Q.18 The charge appearing on right side of plate 4 is
 (2009)
Four metallic plates are placed as shown in the figure.
Plate 2 is given a charge Q whereas all other plates are (A) Zero (B) Q/4 (C) -3Q/4 (D)+Q/2
uncharged. Plates 1 and 4 are jointed together. The
area of each plate is same. Q.19 The potential difference between plates 1 and 2 is
1 2 3 4  (2009)
Q
3 Qd Qd 3 Qd 3Qd
(A) (B) (C) (D)
2 ε0 A ε0 A 4 ε0 A ε0 A

(A) If statement-I is true, statement-II is true; statement


II is the correct explanation for statement-I.
d 2d d (B) If statement-I is true, statement-II is true;
statement- II is not a correct explanation for statement-I.
(C) If statement-I is true; statement-II is false.
Q.17 The charge appearing on the right on the right
side of plate 3 is (2009) (D) If statement-I is false; statement-II is true.
(A) Zero (B)+Q/4 (C) -3Q/4 (D)Q/2
Q.20 Statement-I: For practical purposes, the earth is
used as a reference at zero potential in electrical circuits.
Statement-II: The electrical potential of a sphere of
radius R with charge Q uniformly distributed on the
Q
surface is given by  (2008)
4 π ∈0 R

Q.21 Six point charges, each of the same magnitude q, are arranged in different manners as shown in column
II. In each case, a point M and line PQ passing through M (potential at infinity is zero) due to the given charge
distribution when it is at rest. Now, the whole system is set into rotation with a constant angular velocity about the
line PQ. Let B be the magnetic field at M and µ be the magnetic moment of the system in this condition. Assume
each rotating charge to be equivalent to a steady current.  (2009)

Column I Column II
(A) E = 0 (p) + - Q Charges are at the comers of a regular hexagon. M is at the centre
of the hexagon. PQ is perpendicular to the hexagon.
- +
M
P+ -
(B) V ≠ 0 (q) P Charges are on a line perpendicular to PQ at equal intervals. M is
the mid-point between the two innermost charges.
- + - + - +
M

(C) B = 0 (r) Q Charges are placed on two coplanar insulation rings at equal
+ - + intervals. M is the common centre of the rings. PQ is perpendicular
to the plane of the rings.
- M -

P +
1 9 . 6 6 | Electric Potential and Capacitance

Column I Column II
(D) µ ≠ 0 P Charges are placed at the corners of a rectangle of the sides a and
(s) + - 2a and at the mid points of the longer sides. M is at the centre of
the rectangle. PQ is parallel to the longer sides.
M
+ + - -
Q

(t) Charges are placed on two coplanar, identical insulating rings at


- + -
equal intervals. M is the mid points between the centres of the
P Q rings. PQ is perpendicular to the line joining the centres and
M coplanar to the rings.
- + -

Q.22 A parallel plate air capacitor is connected to a Q.25 A dielectric slab of thickness d is inserted in a
battery. The quantities charge, voltage, electric field parallel plate capacitor whose negative plate is at x=0
and energy associated with this capacitor are given and positive plate is at x=3d. The slab is equidistant
by Q0 , V0 ,E0 and U0 respectively. A dielectric slab is from the plates. The capacitor is given some charge. As
now introduced to fill the space between the plates x goes from 0 to 3d (1998)
with the battery still in connection. The corresponding
(A) The magnitude of the electric field remains the same
quantities now given by Q, V,Eand U are related to the
previous one as (1985) (B) The direction of the electric field remains the same

(A) Q > Q0 (B) V > V0 (C) E > E0 (D) U > U0 (C) The electric potential increases continuously
(D) The electric potential increases at first, then
Q.23 A parallel plate capacitor is charged and the decreases and again increases
charging battery is then disconnected. If the plates
of the capacitor are moved farther apart by means of Q.26 An elliptical cavity is carved within a perfect
insulating handles  (1987) conductor. A positive charge q is placed at the center of
(A) The charge on the capacitor increases. the cavity. The points A and B are on the cavity surface
as shown in the figure. Then  (1999)
(B) The voltage across the plates increases.
(C) The capacitance increases.
A
(D) The electrostatic energy stored in the capacitor
increases. q B

Q.24 A parallel plate capacitor of plate area A and plate


separation d is charged to potential difference V and
then the battery is disconnected. A slab of dielecric (A) Electric field near A in the cavity = electric field near
constant K is then inserted between the plates of the B in the cavity.
capacitor so as to fill the space between the plates
(B) Charge density at A= potential at B.
If Q, E and W denote respectively, the magnitude of
charge on each plate, the electric field between the (C) Potential at A=potential at B.
plates (after the slab is inserted), and work done on the (D) Total electric field flux through the surface of the
system, in question, in the process of inserting the slab, cavity is q/ ε0 .
then (1991)
ε0 AV ε0KAV
(A) Q = (B) Q =
d d

V ε0 AV 2  1
(C) E = (D)
= W 1 − 
Kd 2d  K 
P hysi cs | 19.67

Q.27 For spherical symmetrical charge distribution, Q.30 Six point charges are kept at the vertices of a
variation of electric potential with distance from center regular hexagon of side L and centre O, as shown in the
is given in diagram. Given that:  (2006) 1 q
figure. Given that K = , which of the following
q q 4 π ε0 L2
V= for r ≤ R 0 and V = for r ≥ R 0
4 πε0R 0 4 πε0r statement(s) is (are) correct ? (2012)

v F
F E
+q q
d
P
E3
r
r = R0
A S T D
O -2q
Then which option(s) are correct: +2q
E2
(A) Total charge within 2 R 0 is q
R
(B) Total electrostatic energy for r ≤ R 0 is zero
C
(C) At r = R 0 electric field is discontinuous. B
+q -q
(D) There will be no charge anywhere expect at r= R 0 .
(A) The electric field at O is 6K along OD.
(B) The potential at O is zero
Q.28 Which of the following statement(s) is/are correct?
 (2011) (C) The potential at all points on the line PR is same
(A) If the electric field due to a point charge varies as (D) The potential at all points on the line ST is same
−2.5 −2
r instead of r , then the Gauss’s law will still be valid
Q.31 In the circuit shown in the figure, there are two
(B) The Gauss’s law can be used to calculate the field
parallel plate capacitors each of capacitance C. The
distribution around an electric dipole
switch S1 is pressed first to fully charge the capacitor
(C) If the electric field between two point charges is C1 and then released. The switch S2 is then pressed to
zero somewhere, then the sign of the two charges is charge the capacitor C2 . After some time, S2 is released
the same and then S3 is pressed. After some time, (2013)
(D) The work done by the external force in moving a S1 S2 S3
unit positive charge from point A at potential VA to
point B at potential VB is (VB − VA )
C1 C2
2V0 V0
Q.29 In the given circuit, a charge of +80 µC is given
to the upper plate of the µF capacitor. Then in the
steady state, the charge on the upper plate of the 3µF
capacitor is (2012) (A) The charge on the upper plate of C1 is 2 C V0
(B) The charge on the upper plate of C1 is C V0
+80C
4F (C) The charge on the upper plate C1 is 0
(D) The charge on the upper plate of C1 is - C V0

2F 3F Q.32 A parallel plate capacitor has a dielectric slab of


dielectric constant K between its plates that covers
1/3 of the area of its plates, as shown in the figure.
The total capacitance of the capacitor is C while that
of the portion with dielectric in between is C1 . When
(A) +32 µC (B) +40 µC
the capacitor is charged, the plate area covered by
(C) +48 µC (D) +80 µC the dielectric gets charge Q1 and the rest of the area
gets charge Q2 . The electric field in the dielectric is E1
1 9 . 6 8 | Electric Potential and Capacitance

and that in the other portion is E2. Choose the correct d/2
option/options, ignoring edge effects  (2014)

E1 E1 1
(A) = 1 (B) = Q1 E1
E2 E2 K S/2

E2 2
Q1 3 C 2+K Q2
(C) = (D) =
Q2 K C1 K
+ S/2 -

Q.33 A parallel plate capacitor having plates of area 1


S and plate separation d, has capacitance C1 in air.
When two dielectrics of different relative permittivities d
( ε1 2 and=
= ε2 4 ) are introduced between the two
plates as shown in the figure, the capacitance becomes (A) 6/5   (B) 5/3   (C) 7/5    (D) 7/3
C
C2 . The ratio 2 is  (2015)
C1

PlancEssential Questions
JEE Main/Boards JEE Advanced/Boards

Exercise 1 Exercise 1
Q.1 Q.3 Q.4 Q.3 Q.5 Q.6

Q.11 Q.13 Q.14 Q.7 Q.14 Q.15

Q.18 Q.20 Q.29 Q.16 Q.19 Q.20

Exercise 2 Exercise 2
Q. 2 Q.3 Q.4 Q.2 Q.6 Q.12

Q.5 Q.6 Q.11 Q.14 Q.15 Q.16


Q.17 Q.18 Q.22
Q.33 Q.34
Previous Years’ Questions
Q.5 Q.23 Q.30
Previous Years’ Questions
Q.31 Q.32 Q.21 Q.27
P hysi cs | 19.69

Answer Key

(b) 3 parallel rows; each consisting of five;


JEE Main/Boards
2.0 µF capacitors in series
Exercise 1
Q.22 420 µC one one, 180 µC on two,
Q.1 1.29 µF
60 µC on remaining 3 capacitors
Q.2 5x103 V
Q.23 4x102 F,102 F
Q.3 4 µF
Q.24 60 µC
Q.4 1.39 µF
Q.25 0.16µF,0.24µF
Q.5 2.67 x10−2 J
Q.26 Vmax < 9 kV
−7
Q.6 2.21x10 J
Q.27 (i) In parallel → new capacitance greater than the
Q.7 7.2F initial one (ii) In series → unchanged
C 24
Q.8 = Q.28 (i) 0.9 µC / s ; (ii) 1.09 × 10−6 J / s ;
C' 25
Q.9 C = 2C0 (iii) 2.73x10−6 J / s ; (iv) 3.82 × 10−6 J / s

Q.10 11.1% energy is increased Q.29 (i) (a) 10s ; (b) 2 µC ; (c) 6.94s

Q.11 6x103 V/ m, ∆ E =−12.15 x10−8 J (ii)


= q 1.348 × 10−8 C
∆E =30.4X10−8 J Q.30 1.8 ×1024 Cm
Q.12 18 µC
Q.31 2.7 × 106 V
φA C1 + φB C2 + φD C3
Q.13 φ0 = Q.32 0
C1 + C2 + C3
Q.33 1.11 × 10−9 C positive
E E
Q.14 (i) (ii)
R1 R1 + R 3 Q.34 1.8 × 106 V
Q.15 4.5g Q.35 Total charge is zero. Dipole moment =7.5 × 10‒8m
along z-axis.
Q.16 Both will travel the same distance
Q.36 (i) dV=4E, (ii) Vc > VA
Q.17 260volts, 6x10-4C charge flows from 3 µF to 2 µF
Q.37 (i) 0
Q.18 12 µC , 3 µC , 9 µC , 6 µC

Q.19 (i) 2.04V; (ii) 6.12 µC

Q.21 (a) Five 2 µC capacitors in series;

Exercise 2
Q.1 C Q.2 D Q.3 C Q.4 D Q.5 C Q.6 A

Q.7 B Q.8 B Q.9 B Q.10 C Q.11 A Q.12 C


1 9 . 7 0 | Electric Potential and Capacitance

Previous Years’ Questions


Q.1 A Q.2 D Q.3 D Q.4 A Q.5 A Q.6 C
Q.7 B Q.8 A Q.9 D Q.10 B Q.11 A Q.12 C
Q.13 B Q.14 A Q.15 B Q.16 C Q.17 B Q.18 D
Q.19 D Q.20 D Q.21 B Q.22 B Q.23 C Q.24 A
Q.25 C Q.26 C Q.27 A Q.28 D Q.29 A Q.30 C
Q.31 C Q.32 D Q.33 B C Q.34 A Q.35 B

JEE Advanced/Boards
V0 1
Q.15 (a) I = e−2t/Rc ; (b) CV 2
Exercise 1 R 4 0

Q.1 44.25pF 1 q2d


Q.16
Q.2 30V 2 ε0 A
Q.3 0
5  ε0 A 
32 Q.17 (i)  ;
Q.4 µF 3  d 
23

Q.5 10 µC 4  ε0 AV  2  ε0 AV 
(ii) Q3 =   ; Q5 =  

3 d   3  d 
8
Q.6 µF Q.18 69 mC
3
Aε0 V 100
Q.7 Q.19 (a) Volts
d 7

25ε0 A (b) 28.56mC, 42.84mC,


Q.8
24 d 71.4mC, 22.88mC

Q.20 150 µJ
2
Q.9 2k(q − x) Q.21 16 µC
35r
Q.10 0.8
3
Q.22
Q.11 9J 5
1  1
 E  Q.23 W =Aε0 V02  1 −  '
Q.12 C  R 2  K
 R + R  3
 1 3 
Q.24 12Volt
7
Q.13 (a) A Q.25 (i) 0.2 × 10−8 F,1.2 × 10−5 J;
50
(ii) 4.84 × 10−5 J
(b) Q1 = 0
9µC, Q2 =
(iii) 1.1 × 10−5 J

Q.14 (a) 0.05 1 − e− t/2  µC ;


 

(b) 0.125 µJ
P hysi cs | 19.71

Q.26 4.425 × 10‒5 Amp Q.31 1.8 × 105 sec



Q.27 12V 2 kP  
Q.32 ( − i − 2 j)
q(C) 2y 3
200 2C 4
 8 4  Kq2
Q.33 Wfirst step =
 −  , Wsecondstep =
0, Wtotal =
0
6 3 5 r
O t
Qq
C Q.34
2π ∈0 L
 V 
1/n  4a
C C0  0
Q.28=  1  0.01078µ=
−= F,n 20,No Q.35 (a) H = ,
 V  3
   a
(b)U=mg 2 h2 + a2 − h equilibrium at h=
  3
Q.29 =q
CV
2
(
1 − e− t/RC )
Q.30 Q A =
90µC,QB =
150µC,QC =
210µC,Ui
= 47.4mJ,U
= f 18mJ

Exercise 2
Single Correct Choice Type

Q.1 A Q.2 B Q.3 B Q.4 B Q.5 A Q.6 B


Q.7 B Q.8 A Q.9 A Q.10 D Q.11 B Q.12 C
Q.13 B Q.14 C Q.15 B Q.16 A Q.17 D Q.18 D
Q.19 B Q.20 C

Multiple Correct Choice Type

Q.21 A, B, C Q.22 A, D Q.23 A, D Q.24 A, B Q.25 B, C, D Q.26 A, B, D


Q.27 B, C Q.28 A, C, D Q.29 C, D Q.30 A, C, D Q.31 A, B, C, D Q.32 A, B, C, D
Q.33 B,D

Assertion Reasoning Type

Q.34 D Q.35 C Q.36 D

Comprehension Type

Paragraph 1: Q.37 B Q.38 B Paragraph 2: Q.39 A, C Q.40 A, B, C


Paragraph 3: Q.41 A, C Q.42 D
1 9 . 7 2 | Electric Potential and Capacitance

Previous Years’ Questions


Q.3 D Q.4 C Q.5 D Q.6 A, D Q.7 B, D Q.8 A, C, D
Q.9 B, C Q.10 A, B, C Q.11 A Q.12 A, C Q.13 A Q.14 A, D
Q.15 B, C Q.16 A, D Q.17 B Q.18 B Q.19 C Q.20 B
Q.21 A → p, r, s; B → r, s; C → p, q, t; D → r, s Q.22 A, D Q.23 B, D Q.24 A, C, D
Q.25 B, C Q.26 C, D Q.27 A, B, C, D Q.28 C, D Q.29 C Q.30 A, B, C
Q.31 B, D Q.32 A, D Q.33 D

Solutions

JEE Main/Boards Sol 4:


4F
11F
Exercise 1 C D
CF
A G 6F 12F B
Sol 1: Parallel ⇒ (4.5 + 4.5) µ F = 9 µ F = C'2 E F

1 1 1 1 8F
Total ⇒ = + + 2F
Ctotal 3 3 9
Capacitance b/w C and D
3F C2 ' 3F −1
1 1 
= 4µ F +  +  µ F
1 7  6 12 
= ⇒ Ctotal = 9 / 7 µ F = 1.29 µF
Ctotal 9
= 4µ F + 3µ F
= 7µ F
Sol 2: We have
Let capacitance b/w E and F
Q =CV ⇒ 5 × 10−6 =C × V = 2 + 2 = 4µ F
−6
5 × 10 Net capa. b/w G and B
⇒ V= = 5000 V
−9
10 −1 −1
1 1  1
D =  +  µF + 1 +  µ F
C4 C3 8 4  7

Sol 3: A C2 B 8 7 64 + 21 85
=  +  µF = µF= µF
C1C5 3 8 24 24
E
Now from symmetry, we can argue that the charge at 1 24
Now 1= +
C2 must be zero and hence potential diff. across D and E. C 85

1 1 1 1 1 85
So (capacitance across ADB)−1 = + = + = ⇒ Cc= µ F= 1.39 µ F
C 4 C3 4 4 2 61

⇒ Capacitance across ADB = 2µ F


Similarly, Capacitance across AEB = 2µ F
So total capacitance = 2 + 2 = 4µ F
P hysi cs | 19.73

Sol 5: Now Q = 4 × 10−6 × 200 = 800HQ 1 1


Sol 8: (A) (Capacitance total ) = +
C1 C2
4F d d
-Q Q ⇒ +
400V 2 × k1ε0 A 2 × k 2 ε0 A

d 1 1


Capacitance −1 = . + 
-(Q-q)
2F
Q-q 2Q/3 2ε0 A  k1 k 2 
−1

 ⇒ CA
=
2ε0 A  1 1 
. + 
d  k1 k 2 
-q q
Q/3 (B) Capacitance Total = C1 + C2
2F

Now, an applying Kirchhoff’s law, k1ε0 (A / 2) k 2 .ε0 .(A / 2)


= +
Q −q q d d
− 0
=
4 µF 2 µF ε0 A
⇒ C=
B (k1 + k 2 ).
Q = 3q ⇒ q = Q / 3 2d
−1
1 1 1 1
Einitial = × C × V 2 = × 4 ×10−6 × (200)2 = 8 × 10−2 2ε0 A / d.  + 
2 2
So,= CA / CB =
 k1 k 2 
4Q 2 Q2 (k1 + k 2 ). ε0 A / 2d
Q 21 / 2C1 + Q 22 / 2C2 =
Efinal = +
9 × 2C1 2C2 × 9
CA 4 ⋅ k1k 2 4 × 2 × 3 24
 2
2 1  = = = =
= Q  +  CB (k + k ) 2
(5)2 25
 9C1 18C2  1 2

 2 1  64 × 10−2 ε0 A 1 1
(8 × 10−4 )2 
= +  = Sol 9: CA = 1
and CB −= +
 9 × 4 × 10−6 18 × 2 × 10−6  2×6 d C1 C2

= 5.33 × 10−2 J d d d 2ε A
= + = ⇒ CB = 0
4ε0 A 4ε0 A 2ε0 A d
⇒ Energy lost = 8 − 5.33 = 2.67 ×10−2 J

So CA / CB =1/ 2

ε0 A 8.854 × 10 −12 × 100 × 10−4


Sol 6:=
C = Sol 10: We have Q = CV
d 2 ×10−3
1
Now energy = × CV 2
8.854 × 10−11 2
= = 4.427 ×10−11 F
2 ε A ε A
V = Same, but C1 = 0 and C2 = 0
1 d 0.9d
So, E = × C × V2
2 So energy change
1 4.427
= × 4.427 × 10−11 × (100)2 = ×10−7 = 0.221µ J
2 2 1 ε A 1 ε A 1 ε A  1 
= − × 0 × V2 + × 0 × V2 = × 0 × V2 .  − 1
2 d 2 0.9d 2 d  0.9 
1 2
Sol 7: = CV = 100 × 103 × 3600 = Einitial [0.111]
2

200 × 103 × 3600 ⇒ 11.1 % energy is increased.


⇒ C= = 7.2 F
(104)2
ε0 A 9 × 10−12 × 100 × 10−4
Sol 11:=C =
d 2 × 10 −2
1 9 . 7 4 | Electric Potential and Capacitance

= 4.5 × 10‒12 F Sol 13: A


1 1
Change in energy = × C1 × V 2 − × C2 × V 2 q1 C1
2 2
1 2 O
= Einitial − Efinal = × V . C1 − C2  C2 q1 v q3C
2 0 3

B D
1 1 1
= × (300)2 . ε0 A  − 
2  d1 d2  We have q1 + q2 + q3 =
0

9 × 10 4 × 9 × 10−12 × 100 × 10−4  1 1 ⇒ C1 .(φ0 − φA ) + C2 .(φ0 − φB ) + C3 .(φ0 − φD )


= × − 
2 × 10−2 2 5
⇒ (C1 + C2 + C3 )φ0 = C1 φA + C2 φB + C3 φD
= 40.5 ×10−8 3 / 10 
C φ + C2 φB + C3 φD
−7 ⇒ φ0 =1 A
= 1.215 ×10 J C1 + C2 + C3
Q CV ε0 A 300
E
= = = ⋅V = = 60 × 102
ε0 A ε0 A d2 .ε0 A 5 × 10 −2 Sol 14: (i) Now, at just t = 0, charge at C1 = 0 and charge
at C2 = 0 so = V Q= / C 0 so potential diff. between
= 6 × 103 V / m AB = 0
Now in 2nd case , Q = cons tant i = E / R1 + R AB = E / R1
We have Q = CV (ii) After long time , no current through both capacitors
−12 −10
= 4.5 × 10 × 300 = 13.5 × 10 ⋅C E
E iR1 + iR3 ⇒ i =
⇒=
= Q = 1.35 × 10−9 ⋅ C R1 + R3

Now change in energy: Einitial − Efinal Q


Sol 15: E due to one plate =
2ε0 A
 Q 2 Q 2  Q 2  d1 d 
⇒  − =  . − 2 Q Q2
 2C 2C  2  ε A ε A  So force = Q. × =
 1 1  0 0  2ε0 A 2ε0 A
Q2
Q2 Q 2 .(d1 − d2 ) So=
T mg
=
= [d1 − d2 ] = 2ε0 A
2ε0 A 2ε0 A
C2 V 2
mg =
(1.35 × 10−9 )2 .(2 − 5) × 10−2 (1.35 × 10−9 )2 × ( −3) 2ε0 A
= =
2 × 9 × 10−12 × 10−2 2 × 9 × 10−12
C2 V 2 ε A.V 2 9 × 10−12 × 10−12 × (5 × 108 )2
m= = 0 =
−(1.35 × 10−9 )2 2ε0 A.g 2gd2 2 × 10 × (5 × 10−3 )2
= = −0.3 × 10−6 = −3 × 10−7 Joules
2 × 3 × 10−12
= 4.5 × 10−3 kg = 4.5 gm
12V 2
Sol 16: Let there, masses be me and mi
Sol 12: 1 1
4 Then Vd = me v e2 = × Mi × Vi2  …. (i)
2 2
6 Ee Ee
Now acceleratio of particles ⇒ = ae and = = ai
me Mi
Now no current will pass through the capacitor.
So, i = 12 / 8 = 3 / 2 A 1
So time to hit the plate ⇒ =
S ut + at2
2
Thus, potential diff. across capacitor
2d 2d.me
9V ⇒ Q = CV = 18µC.
= 6×3 / 2= ⇒ = t=
ae e Ee
P hysi cs | 19.75

2d 2d.Mi q1 q2 3
Similarly,=
ti = .. ⇒ = ⇒ q1 = q
ai Ee 3 2 2 2

, q1 780 µ C
q2 520 µ C =
=
Now distance = Ve × te
520 µ C
2d.me Potential difference = = 260 Volts
2Vd 2 µF
= ×
me Ee Thus the charge will flow from each positive terminal to
the negative terminal.
V
= 2d. (independent of mass)
Ee
Sol 18: No current will pass through, the upper portion,
Same distance can be found for the ion and hence the and voltage difference between the capacitors would
time is same. be same as ε .
−1 −1
 1 1   1 1
Sol 17: Q =
3 × 300 =
900 µ C. C total 
=
C +C
+  =  +  = 5µ F
 1 2 C3 + C 4   10 10 
And Q 2 =
2 × 200 =400 µ C.
(4)
Actual emf = ε − × 1 = ε − 1 = 3V
Q1 Q1-q (3 + 1)
+ -
So Q = CV = 5 µ F × 9 = 15µ C
q1 q3
Q2 Q2+q

Q1 − q (Q 2 + q)
So using Kirchhoff’s law, ⇒ − 0
=
3 2
q2 q1
2Q − 3Q 2
⇒ 2Q1 − 2q = 3Q 2 + 3q ⇒ q = 1
5

q1 Now, q1 + q2 = 15µ C  …(ii)


C1 q1 q2
and − 0
=
C1 C2
C1 q1 q2
⇒ = ⇒ q1 = 4q2  …. (i)
8 2

q1 + q2 = 900 − 400 = 500 µ C ⇒ q1 =


12 µ C, q2 =
3µ C

(from charge conservation) Also (q3 + q4 ) =


15
q1 q2 q1 3 q3 q4
Also − 0 ⇒ 2q1 = 3q2 ⇒
= = − 0
=
3 2 q2 2 c3 c4
⇒ q1 =
300µ C and q2 =
200 µ C
q3 q4 q3 3
= ⇒ =
q1 + q2 = 1300 µ C 6 4 q4 2

q1 ⇒ q3= 9µ C ⇒ q4= 6µ C

q1

q2
1 9 . 7 6 | Electric Potential and Capacitance

Sol 19: Diagram-1 Sol 20: (i)


2F C1 C1
4F q4 -q4 e c
q2 q2 a
5F q5 -q5
D
q1 -q1 C2 C2 C1
8F 8F
A G B
6F b
q6 -q6 C1 f C1 d
q3 -q3


3F q66F -q6

Diagram-2 C1
e q’’ c
a


2F q’’
q4 C2 CT ’
4F
2F
q4
5F 4F q5 b
-q’’C q’’ d
f
A 4F B
1
5F q5
4F


A B
3F 12F
e
a


3F 12F
Diagram-3 q’’
2.4F C2 CT’’

A 2.4F B b
f
5F


A 2.4F B
5F
−1
2.4CF =  1 + 1 
total  
 5 4.8  a
−1 −1
1 1  1 5  120
Ctotal
=  +  = 2.45µ F
 5 4.8   +  = = CT’’’
 5 24  49
−1
1 5  120 b
=
So,
 =Q+ CV
= = =
2.45 × 102.45µF µ C
= 24.5
 5 24  49
Now from diagram (3), capacitance
We have
= same ⇒ charge is equally divided. −1 −1
 1 1 1 2 1
24.5 CT =
 + +  = (6 / 7)µ C
 +  =
⇒ q=
2 q=
3 = 12.25 µ C 3 3 2 3 2
2
Now from diagram (2) 6 20
Now CT ' = + 2= µC
q4 + q5 =
q2 7 7
q4 q5  7 2
−1
 61   60 
−1
And − 0 ⇒ 2q4 − q5
= Now CT " = +  =  =  µC
2 4
 20 3   60   61 
12.25 × 2
So q4 = 12.25/3µC = 4.08µC and=
q5 = 8.166µ C
3 60 182
⇒ CT '"= + 2= µC
4.08µ C 61 61
(i) So voltage diff. =
= q / c = 2.04 V
2µ F
−1 −1
 61 1 1   61 2 
12.25µ C 6.12µC CTotal = + +  = 1 C
+  =µ
(ii) Charge = =  182 3 3   182 3 
2
[q3 =
12.25µ Cand2q6 =
q3 ]
P hysi cs | 19.77

(ii) Q = CV = 1× 900
= 900µC 7 7
VAC= × VCB ⇒ VAC ⇒ .VCB
3 3
(iii)
600 3
600V ⇒ VAC= 7 × = 420 V and V=
CB 600 × = 180 V
C 500V 10 10
900
1 C × 420 = 420 µ C
⇒ Q AC =CV =µ
600 C 2 300 C
And QBC= 180 × 1= 180 µ C
0 d 400V
300V And in each 3 , q will be equally distributed
q= 420 − 2 × 180 ( on each)
Q 900µC
=
1 1
900 Sol 23: We have × C1 × (2)2 + × C2 .(2)2 = 10 × 102
v
⇒ ∆= = 300V 2 2
3
300 ⇒ C1 + C2 =
500
Now between e and c = = 100V
3
Q2 Q2 1 1
So, between c and=
d 500 − 400V = 100V And 160
= + 320 Q 2  +
⇒= 
2C1 2C2 C
 1 C 2

Sol 21: (a) a b 1 1 V2
= V 2 × Ceq2  + =
All in series  C1 C2   1 1 
 + 
(b) 2  C1 C2 
V ⋅ C1 ⋅ C2
320 =
C1 + C2
a b C1 ⋅ C2
⇒ 80
= ⇒ C1 ⋅ C
= 2 40000  …(i)
500
Cnet =1.2F ⇒ On solving, 400 , C2 = 100
C1 =

Sol 24:
q2 -q2
q1 -q1

Cnet
= 1.2 µ F 2F 3F

Sol 22: By trial and error q3 -q3


110C -110C

1F

We have q3 + q1 = 110µC
A 1 C B
and q2 − q1 =
0
q3 q1 q2 q1 q2
 q1 and
⇒ q2 = − − 0 ⇒ q3 =
= +
A 1 C Ctot B 1 2 3 2 3
5 ⋅ q1 6 × 110
⇒ 110 − q1= ⇒ q1= = 60 ⇒ q1 = 60µC
A 1 C Ctot B 6 11

VAC + VCB =
600 ….. (i) So, q=
2 60µC and q5= 50µC

So, 110 − 50= 60µC charge passes through wire.
And 1× VAC = Ctot × VCB
1 9 . 7 8 | Electric Potential and Capacitance

40V (unchanged)
Sol 25: 100V 0
CA CB Ceq (parallel)= C1 + C2

100V CA 90 0  1 1  ε0 A.(2d)
= ε0 A.  + = 2
CB  d + a d − a  (d − a2 )

2µC  1 1  2ε A
Without any a difference , Ceq =
ε0 A ×  +  = 0
We have CA × 60 = CB × 40 ( q charge is same) d d d

Now 1 − a2 / d2 < 1 so the capacitance increases in case


CA 2 of parallel.
⇒ =
CB 3

Similarly Sol 28:


1F
CA + 2 9×3 25
⇒ = 9 ⇒ CA + 2= × CA = 2 = C q -q
CB 2 2 A
6
i 3 x 10 
4
= CA = CA 0.16µ F
⇒ =
25

CA 2
Similarly , = CB 0.24 µ F
⇒= 4V
CB 3
(i) We have , iR + q / c =
4
Vmax V 0
Sol 26: dq q 4
⇒ iCR + q =4C ⇒ + =
C1 C2 dt RC R
For Vmax .(inseries) , Q = same for both the capacitance (in q,t t
4 t/RC
∫ d q.e
t/RC 
R ∫0
series ) ⇒ = e .dt

0,0
So , C1 (Vmax − V)= C2 (V − 0)
4
q.e
= t/RC
=.(RC) et/RC − 1
R  
⇒ 1× (Vmax − V) = 2.(V − 0)
Vmax
⇒ Vmax − V =
2V ⇒ V = =
⇒ q 4C 1 − e− t/RC 
3  
Now V < 4kV dq  1  − t/RC
Now = 4C.   .e
Vmax dt  RC 
⇒ < 4 ⇒ Vmax < 12 kV
3
4
= × e−1/3 = 0.96 µ C / s
And Vmax − V < 6kV 3 × 10 6

2Vmax Q2
⇒ < 6kV ⇒ Vmax < 9 kV (ii) E= x ×
3 2c
Lower limit is 9 kV dE 1 dq q dq
⇒ = × 2 × q. = .
C1.C2 dt 2C dt C dt
Sol 27: Ceq (Series) =
(C1 + C2 ) 4C 1 − e−1/3 
=   . 4 × e−1/3 = 1.09 × 10−6 J / s
 1
−1
d+a d−a
−1
 2d 
−1
ε0 A C 3 × 106
1 
=  +  =

+  =   =
 C1 C2   ε0 A ε0 A   ε0 A  2d
 dq 
2
2
(iii) P = i R =   .R
Which is same as without any difference a.  dt 
= (0.96 µ C)2 .3 × 106
P hysi cs | 19.79

= (0.96)2 × 3 µ J = 2.73 µ J / s Sol 31: 5C

(iv) Energy = Energy in Resistance + Energy in capacitance 5C 5C


−6
= (2.73 + 1.09) × 10 J/s
= 3.82 × 10−6 J / s 10cm
cm
10 5C
Sol 29: 5C

5C
kq
Potential at center due to one charge =
r
9 × 109 × 5 × 10−6
2V = = 45 × 104 V
−1
10
(i)
Potential at center due to all the charges
(a) t = RC = 107 × 10–6 = 10 sec
(b) Q = CV = 2 × 1 µC = 2µC = 6 × Vq = 2.7 × 106 V

(c) Now , as in Ques. 29 above, q= 2 ×µ C [1 − e− t/10 ]


1 Sol 32:
⇒ = 1 − e− t/10
2 D C

⇒ et/10 = 2
10C
⇒ t = 10ln2 = 6.94 sec

(ii) A B
2C

We have q = q0 e− t/RC (k)(10µC) 2k


VA = = (10 µC)
a a
Now q= 2 × e−50/10 = 2.e−5 = 0.0135 µ C 2

(k)(10µC) 2k
Sol 30: VB = = (10 µC)
 a  a
 
-1

 2
NC

Work done is moving charge of 2µC = q(VB - VA) = 2mC


4
10

o
30
(0) = 0
x
30

+ 2
E=

Sol 33:
T = (Eq) (2  ) sin 30° q
T = E sin 30°. (q.2l) r = 0.1m
⇒ Dipole moment (D) v = 100 volts
26
T 27 × 10 kq 100 × 0.1
= q.2 l = = ⇒ = 100 ⇒ q =
Esin30° 30 × 10 4 × 1/ 2 r k
= 1.8 × 1024 cm = 1.1 × 10-9 C (positive)
1 9 . 8 0 | Electric Potential and Capacitance

Sol 34: ∴ Net force = Eq – Eq = 0 (Assuming length of dipole


is small)
4cm 
(ii) Ui = - P.E = -PE
P 
20c
6cm -10c Uf = P.E = PE
3cm 1cm Uf – Ui = 2PE = work done is rotating dipole

7cm
Exercise 2
q1 q2 q q 
Vp = K +k = k  1 + 2  Single Correct Choice Type
r1 r2  r1 r2 
q1 = −10µc , r1 = 1 cm Sol 1: (C)
q2 = 20µc , r2 = 7 cm CV + Q
-CV
Put the values in the equation for Vp, d
Vp = – 1.8 × 106 V

Sol 35:
-7 C
0
x1
5)

.5
,1

-2  σ1 CV   CV + Q CV  Q CV
,0

 +  = +  = +
(0

 02ε 2Aε0   2ε 0 A 2Aε0  2Aε 0 Aε0


-7 C
0 Q d d
x1
5)

.5 work done = E × d = + CV ×
,1

-2 (per unit charge) 2 Aε0 Aε0


,0
(0

Q 1 Q
Total charge of system = + C × V'× = V +
2C C 2C
= -2.5 × 10-7 C + 2.5 × 10-7 C = 0
−1
Dipole moment of system 1 1 1 
Sol 2: (D) Ceq =C  + +  =C + C / 2 =3C / 2
= -2.5 × 10 × (-15 – 15) × 10
-7 -2  C 2C 2C 
Q =×
60 3C / 2 =
90C
=-7.5 × 10-8 Cm (along –ve z-axis)
q1 q2
Now = = ; q1 2q2 and =
q1 + q2 90C
Sol 36: The motion perpendicular to electric field won’t C C/2
account in work
⇒ q2 =
30C. So potential difference
∴ Work done in moving charge from A to B is = Q=
/ C 30V
E(6 – 2) = 4E
∴ Potential difference between A and B = 4E also Sol 3: (C)
q3
VB = VC (since no parallel movement along E)
q2
⇒ VC – VA = 4E and VC > VA
q4
Sol 37: (i) q1
+ Eq
4F
6 Q
Eq -
P hysi cs | 19.81

1 1
−1
 10 
−1 q1 q2
Ceq =4 +  +  =4 +   =6.1µC. = ⇒ q1 = 2q2
4 2
3 7  21 
144µF
⇒ q2 = = 48µC
So qtot = q1 + q2 = (6 / 6.1)µC 3

Out of this again, this is divided in 5µF and 1µV


3F 1
-q
1
q q1 q2
= ⇒ q1 = 5q2
2F 5 1
5F 48
⇒ q6 = =µ8 C
-q q 6
Q2 8 × 8
so energy= = 32µJ
2 2
-q q =
4F 2C 2 × 1
q1 q2 q1 4
= ⇒ =
4 2.1 q2 2.1 Sol 6: (A) Charge on right should be positive and
q1 q2 q1 3
4 6 2.1 6 q1 + q2 = 20µC and = ⇒ =
q1 = × µC and q2 = × µC 3 4 q2 4
6.1 6.1 6.1 6.1
3 60
and also q3 + q4 =
q2 ⇒ q1 = × 20 = =8.57µC
7 7
q3 q4
and =
2 5 Sol 7: (B)
5 5 2.1 6
⇒ q4 = × q2 = × × µC
7 7 6.1 6.1
5 2.1 6
q4 7 × 6.1 × 6.1 3 A  A B

= =
q1 4×6 8 B C


6.1× 6.1 C

3C/2
Sol 4: (D) 4 times in series. Let each be x, then A B

x
= 16µF ⇒ x = 4µF
4 C
−1
Which is possible when 8 are connected in parallel  20 1  3C 8C
So eq ⇒  +  +C= +C=
⇒ 8 × 4 = 32  3C C  5 5

Sol 5: (C) −1
 1 1 1  5 1
Sol 8: (B) Ceq =  + +  − 1=  + 
6F  2 5 3  6 5
3F 6F −1
 31  30 30
 =  = = µF q C=
eq .Vmax .V ;
31 max
4F  30  31

24V
q 30
−1 V=
1 = .V < 3V ⇒ Vmax < 6.2 V
1 1 C 31× 2 max
 +  =2 and 2 + 4 =6µF
3 6
q 30 1
V2 == × .V < 2V ⇒ Vmax < 6.2 V
So charge = Q = 6µF × 24 = 144µC C 31 3 max

That much charge is divided in 4µF and 2µF (above eq. 30 1


V2 =× .Vmax < 1 Vmax < 31/ 6
capacitance) 31 3
1 9 . 8 2 | Electric Potential and Capacitance

Sol 9: (B)
σ
=105 V / m ⇒
σ
=0.5 × 105 V / m Previous Years’ Questions
ε0 2ε0

σ C1C2
Now, force= × Q= 0.5 × 105 × 1µC= 0.05 N Sol 1: (A) In series, C =
2ε0 C1 + C2

σ Q2
Sol 10: (C) Force = × Q=
2ε0 2Aε0

so Force ∝ Q 2
1F
Vmax 3V 2V 1V
O
2 3 5 10F
(1) (2) (3) (10)(1) 10
Cnet = = µF
Now, when d is halved ⇒ C is doubled; Q=CV ⇒ Q is 10 + 1 11
doubled.
So Force ∝ Q 2 ⇒ F becomes four times. Sol 2: (D) Since, the capacitor plates are directly
connected to the battery, it will take no time in charging

d/2 d/2  k + k2  C
Sol 11: (A) deff = + = d  1 
k1 k2  2k1k 2 

d 2k1k 2 R
∴ k eff = =
deff k1 + k 2
V
A 2 ∈0 A(k1k 2 )
Also, C =
∈0 =
deff k1 + k 2
Sol 3: (D) When S3 is closed, due to attraction with
opposite charge, no flow of charge takes place through
Sol 12: (C)
S3. Therefore, potential difference across capacitor
plates remains unchanged or V1 = 30V and V2 = 20V

Alternate solution
Charges on the capacitors are
q1 = (30)(2) = 60pC and
Capacitance increases = kC0
q2 = (20)(3) = 60pC or q1 = q2 = q(say)
Now,
The situation is similar as the two capacitors in series
qi =Ci V =60µC and qf =Cf V =180µC(60 + 120) are first charged with a battery of emf 50 V and then
⇒ Cf = kCi ⇒ k = Ci . V = 180µ C ⇒ k = 3 disconnected.

q q
+ - + -
2pF 2pF q=60pC q=60pC
+ - + -
V1=30V V1=20V

50V
∴ When S3 is closed, V1 = 30V and V2 = 20 V.
P hysi cs | 19.83

Sol 4: (A) Due to attraction with positive charge, the Dipole moment
negative charge on capacitor A will not flow through
the switch S. 4q
a/2
Sol 5: (A) After time t, thickness of liquid will
-4q
d 
remain  – vt 
3  a
P = 4q   ˆj = 2qa ˆj
Now, time constant as function of time: 2

ε0 (1)R kq VR
tc = CR = Sol 9: (D) Potential of each drop = V = ⇒q=
 d  d / 3 – vt R k
 d – – vt  + VR
 3  2 If n drops coalse, Q = nq = n
k

  Radius R’ = n1/3 R
 ε0 A 
 Applying C =  nkVR
 t  The potential of new drop = = Vn2/3
 d–t+  kn1/3R
 k 
6ε R Sol 10: (B) The potential at center is same as that of
=0
5d + 3Vt the surface.
∴ V = 10 Volts
Sol 6: (C) Option C is correct because electron gets
attracted towards positive charge (moves against Sol 11: (A)
electric field) larger the potential, lower the potential
energy for electron.

U = V(-q) b
10V

5V
a
Sol 7: (B) σ = 10-7 C/m2
Given, V = 5 volts

σ At center the potential will be same as the potential of


but V = .r the sphere enclosing it first
ε0
If it is any other potential, there will exist a electric field
ε0 (V) 8.854 × 10 −12
× 10 inside sphere contradicting gauss law.
⇒r= =
−7
σ 10
= 8.8 × 10-4 m = 0.88 mm Sol 12: (C)

Sol 8: (A) -
3q b
+
a
-2q -2q c -

(Center of charge) of negative charges is origin Potential at a due to charge of a = - (σ / ∈0 ) (a)

(center of charge) of positive charges is Potential at b due to charge of b = (σ / ∈0 ) (b)

3q(a) − q(a) q Potential at C due to charge of c = −(σ / ∈0 ) (c)


= σ
4q 2 ∴ Potential at a = (b – a – c)
∈0
1 9 . 8 4 | Electric Potential and Capacitance

Sol 13: (B) Electric potential at any point inside a hollow →


The electrical field E at all points on the x-axis will not
metallic sphere is constant. Therefore, if potential at
have the same direction.
surface is 10 V, potential at centre will also be 10 V.
For – d ≤ x ≤ d, electric field is along positive x-axis
while for all other points it is along negative x-axis. The
Sol 14: (A) In such situation potential difference →
depends only on the charge on inner sphere. Since, electric field E at all points on the y-axis will be parallel
charge on inner sphere is unchanged. Therefore, to the x-axis (i.e., î )
potential difference V will remain unchanged. The electrical potential at the origin due to both the
charges is zero, hence no work is done in bringing a
test charge from infinity of the origin.
1 Q1 1 Q2
Sol 15: (B) VC = VQ = VQ = +
1 1 2 4πε0 R 4πε0 R 2 Dipole moment is directed from the –q charge to the
+q charge (i.e.,– î direction)
1  Q2 
=  Q1 + 
4 πε0R  2 Sol 17: (B) The diagrammatic representation of given
problem is shown in figure.
1  Q1 
Similarly VC =  Q 2 +  The net charge shared between the two capacitors is
2 4 πε0R  2
Q’ = Q2 – Q1 = 4CV – CV = 3CV
Q1 Q2
+ - Q1=C1V1=CV
R 2 + -
R + -
+ -
+ -
C1 R C1

+ -
+ -
+ -
∴ DV = VC – VC + - Q2=C2V2=(2C)(2V)=4CV
1 2 + -
1  1 
= (Q1 – Q 2 ) – (Q1 – Q 2 ) The two capacitors will have the same potential, say V’
4 πε0R  2 
The net capacitance of the parallel combination of the
Q1 – Q 2 two capacitors will be
= ( 2 –1)
2(4 πε0R) C’ = C1 + C2 = C + 2C = 3C
The potential difference across the capacitors will be
W = qDV = q(Q1 – Q2)( 2 – 1)/ 2 (4pe0R)
Q' 3CV
V’ = = =V
C' 3C
Sol 16: (C) The diagrammatic representation of the
given question is shown in figure. The electrostatic energy of the capacitors will be
 1 1 3
Eq U’ = C’V’2 = (3C)V2 = CV2
2 2 2

E
 Sol 18: (D) Potential at origin will be given by
y
E-q
q 1 1 1 1 
V=  – + – + ....... 
4πε0  x0 2x0 3x0 4x0 
x
q 1  1 1 1  q
q -q = . 1– + – +.......  = ln(2)
4πε0 x0  2 3 4  4 πε 0 x0
(-d, 0) (d, 0)
P hysi cs | 19.85

→ → → Sol 21: (B) Potential decreases in the direction of


Sol 19: (D) Electric field within the plates=
E E Q1 + E Q2
electric field. Dotted lines are equipotential lines.
Q1 Q2 Q1 – Q 2
E = E1 – E2 = – ,E= y
2Aε0 2Aε0 2Aε0

∴ Potential difference between the plates



E C

x
A B
+Q1 +Q2
E2 E1 E

∴ VA = VC and VA > VB
1 2
 Q – Q2  Q1 – Q 2 Q1 – Q 2 q Q
VA – VB = Ed =  1 Sol 22: (B) q
 2Aε  d =  Aε  = 2C
 0 
2  0  x=–a x=0 x=+a
 d 
Initial Position

ε0 A
Sol 20: (D) Applying C= , we have q Q q
t t
d – t1 – t2 + 1 + 2 x=–a
K1 K 2 x=x x=a
ε0 (A / 2) Final position
d/2 d/2
d–d/2–d/2+ + 2KQq K.q.q
K1 K3 Ui = +
q 2a
ε0 (A / 2) Kε0 A
+ =  1 1  K.q.q
d/2 d/2 d and Uf = KQq 
d–d/2–d/2+ + +  +
K2 K3  a + x a – x  2a

Solving this equation, we get 1


Here, K =
4πε0
2KQqx2
A/2 A/2 A DU = Uf – Ut or |DU| =
d/2 d/2 a3
K1 K2
+  K d for x << a ∴ DU ∝ x2
K3 d/2 K3 d/2
Sol 23: (C) DU = decrease in potential energy
= Ui – Uf
In parallel 2
1 1  V + V2 
= C( V12 + V22 ) – (2C)  1 
K1K 3 K 2K 3 2 2  2 
K= +
K1 + K 3 K 2 + K3 1
= C(V1 – V2)2
4

Sol 24: (A) There will be an electric field between two


cylinders (using Gauss theorem). This electric field will
produce a potential difference,
1 9 . 8 6 | Electric Potential and Capacitance

Sol 25: (C) A = (–a, 0, 0), B = (0, a, 0) Sol 29: (A) 120C1 = 200C2
y
6C1 = 10C2
3C1 = 5C2
B -Q

Sol 30: (C)


x
A O
Q
z
x dx
Point charges is moved from A to B
VA = VB = 0 ∴ W = 0 x =2L k Q Q ln 2
=V ∫x=
=L x  L 
 
dx
4 π ε0 L
3
Sol 26: (C) Distance BC= AB sin 60°=(2R) = 3R
2
Sol 31: (C) By formula of electric field between the
1 (q / 3)(2q / 3) q 2 σ
∴ |FeBC|= = plates of a capacitor E =
4πε0 ( 3R) 2
54 πε0R 2 K ε0

⇒ σ = E K ε0 = 3 × 10 4 × 2.2 × 8.85 × 10 −12


Sol 27: (A) After time t, thickeness of liquid will =6.6 × 8.85 ×10−8
d  = 5.841× 10−7
remain  – vt 
 3 
≅ 6 × 10−7 C / m2
Now, time constant as function of time:

Sol 32: (A) E = 30 x2 ˆi
ε0 (1)R
tc = CR = dV = − ∫ E.dx
 d  d / 3 – vt
 d – – vt  +
 3  2 VA 2

∫ dV = − ∫ 30 x2 dx
  V0 0
 ε0 A  6ε0R
 ApplyingC =  = VA − V0 =
−80 volt
 t  5d + 3Vt
 d–t+ 
 k 
Sol 33: (B, C) The potential at the centre
Sol 28: (D) q1 = C1V = 2V = q 2
R 4 π r dr
Q 3 kQ 3 1
4 3 ∫0
= k = = =V ;k
This charge will remain constant after switch is shifted r 2 R 2 0 4 π ε0
πR
from position 1 to position 2. 3
1 q2 q2 q2
Ui = = = ∴ R1 = 0
2 Ci 2×2 4
kQ
1 q2 q2 q2 Potential at surface, V0 =
Uf = = = R
2 Cf 2 × 10 20
5V R
q2 Potential at R 2 = 0
⇒ R2 =
∴ Energy dissipated = Ui – Uf = 4 2
5
k Q 3 kQ 4R
 q2  Potential at R3 , = ⇒ R=
3
This energy dissipated  =  is 80% of the initial R3 4 R 3
 5 
 q 2  
stored energy  =  k Q kQ
 4  Similarly at R 4 , = ⇒ R 4 = 4R
  R4 4R
P hysi cs | 19.87

Sol 34: (A) Let the potential at P be V, Now q on=


C2 8C
= 2 24 C2
Then, C (E – V) = 1× V + 2 × V (we take C in µ F ) So total charge = (24 + 6)C1 = 30C1

V Same charge would be there at capacitor b/w a and b


1F 30 C1
So , ∆=
V = 30V
C P C1
2F
(50µ C )
2
(20 µ C)2
Sol 3: Einitial = +
2 × 2µ F 2 × 5µ F
E
CE 2500
Or, V = = 100 µJ + µJ = 350 µJ
3+C 10
2CE q1 q2 2
∴ Q2 = Now = ⇒ q1 = × 70 µ C = 20 µ C
3+C 2 5 7
q1
Sol 35: (B) Charge on 9 µF capacitor = 18 µC
Charge on 4 µF capacitor = 24 µC 2F
∴Q = 24 + 18 = 42 µ C

KQ 9 × 109 × 42 × 10−6 5F


=
∴ = 420 N / C q2
r2 (30 )
2

And thus , q=
2 50 µ C

Einitial = same (thus there would be no heat produced)


JEE Advanced/Boards
Sol 4:
Exercise 1 1F
C
A
6 mm 4 mm 8F
Sol 1: deff = + = 2 mm 4F 4F
6 4 4F
A
∴C =∈0 1F
deff

8.85 × 10−12 × 100 × 10−4


 8/3F 8F
= 44.25 pF
2 × 10−3
C
A
Sol 2: 8/3F 8/9F
+ - B
C
b A
a C2
C1 32/9F
q B
C1 C2
1 9
Now qpq =×
2 C2 =
6C1 Now, 1
= +
C 32
So voltage difference across b and p
32 − 9 1 32
= 6C1 / C1 = 6V ⇒ = C
⇒= µF
32 C 23
1 9 . 8 8 | Electric Potential and Capacitance

Sol 5: ∆ V =
10 V  4ε A  ε0 AV
4Q = (4C)V =  0  V ⇒ Q =
So, Q = CV = 10 × 1 µ C  d  d
(The ends of C = 1µ F are connected to the terminals of ε0 AV
the battery) So, the charge on plate 1 is and that on plate 4
d
2ε AV
is − 0
d
Sol 6: Suppose a charge q is given to the system.
q1 -q1 q4 -q4 C1.C2
Sol 8: Capacitance (left side):
C1 + C2
-q2 1
x 1 1 2 y 1× (A / 2).ε0 3 × (A / 2).ε0
q2 . 3Aε0 3Aε0
2 = d d = =
(A / 2).ε0 8d 8d
q3 -q3 4.
d
Then, q1 + q2 + q3 =q …… (i)
 Capacitance: (right side)
And q4 + q3 = q  ….. (ii)
2 × (A / 2).ε0 4 × (A / 2).ε0
Also , applying kirchoffs law in loop (i) .
d d 4 Aε0 2Aε0
−q1 q2 = = =
+ 0 ⇒ q1 = q2 …(iii)
= (A / 2).ε0 6 d 3d
6.
1 1 d
q3 q2 q4
And − − 0
= 3Aε0 2Aε0
2 2 1 So C=
total +
8d 3d
q3 2(q2 + q4 )
⇒=
 9 + 16  Aε0 25 Aε0
Now, 2q1 + q3 =
q (form (i)) =Ctotal  = 
 24  d 24 d
(q − q3 )
⇒ q=
2 q=
1
2
Sol 9:
 (q − q3 ) 
⇒ q3 2. 
= + q − q3 
 2  II
⇒ 4q3 =q − q3 + 2q − 2q3
I q2
⇒ q3 = 3q / 4 -x
p
1 3q
Now, ∆=
V 3q / 4 × = V
2 8 x
8 q-x
So Capacitance = q / ∆ V = µF -q+x
3

Sol 7: The Circuit can be converted to all branches have Now V at outer must = 0
the same capacitance and are connected in parallel.
k.( − x) kq k[( −q + x) + q2 ]
1 2 ⇒ + + 0
=
Q Q 3.5r 3.5r 3.5r

⇒ kq2 = 0 ⇒ q2 = 0
3 2
(A,1,3,5) (2,4,B) And now potential at inner cell = 0
Q Q
k.( −q + x) k(q − x) k.x kx
3 4 + + − 0
=
3.5r 2.5r 2.5r r
Q Q
5 4 k( −q + x) kq kx
⇒ + − 0
=
Q Q 3.5r 2.5r r
P hysi cs | 19.89

(x − q) q 20
1
+ −x =0 So Energy =∫ ε0 .E2 .(4 πr 2 ).dr
3.5 2.5 2
10

2(x − q) 2q
+ − x=0 ε0 dr
7 5 = . 4 π.k 2q2 20
10 ∫ r2
2
⇒ 10(x − q) + 14 q − 35 x =
0
⇒ 4q = 25x kq2  1 1  5kq2
= . −  =
2  0.1 0.2  2
⇒ x = 4q / 25
kx 5 × 9 × 109 × (20)2 × 10−12
So in region (I), E = =
r2 2
2.5r =9J
1
Energy = ∫ ε .E2 .(4 πr 2 ).dr
2 0 E
r Sol 12: i =
R1 + R3
2.5
1 2 1 2
= 2 ×ε0 × 4 π.kx ∫ r2 r .dr
So, ∆ v= R 3 .i=
R 3E
r
R1 + R 3
2
kx  2  3k 2 CR 3E
= 1 −  = 10r .x where x = 4q/25 Now, Q = C × ( ∆ V) =
2r  5  R1 + R 3
Similarly in region II
−kx k(q) k( −x + q) Sol 13: (A)
E
= + =
2 2
r r r2
2F
3.5r
1
⇒ Energy = ∫ ε0E2 (4 πr 2 )dr 30
2.5r
2 9V 2V
30 60 +
k(q − x)2  2 2  2k(q − x)2 1F
=  −  = -
2r 5 7  35r
O
-2V
Sol 10:=i 2 / 4 + =
5 2/9 -
or
2 10 +
So ∆ V for 4µ F =iR = 5 × =V
9 9
2 8 2µC
And ∆ V for 5 µ F = iR = 4 × = v (a) ∆V across 1µ=
F = 2V
9 9 1µ F
1 / 2 × 5 × (8 / 9)2 9−2
So Energy ratio = Now so current =
1 / 2 × 4 × (10 / 9)2 −1
 1 1 
30 +  + 
5 64 4  60 30 
= × = =0.8
4 100 5 7 7
= = = A 0.14 A
30 + 20 50
Sol 11: Now after connection all the 
charge will transfer to be outer shall. 30C (b) Current = 0; So, qC = 9 × 1 = 9µ C
1
Now, the heat generated would be And qC =( ∆V) × C2 =0 × C 2 =0
same as then energy of the region 100m 2

between the sphere and the shell 20m


(Electric field elsewhere is same)

kq
So, E =
r2
1 9 . 9 0 | Electric Potential and Capacitance

Sol 14: Q 2 (0.05)2 .[1 − e− t/2 ]2 × (µ c)2


(b) Energy = =
R1 i i1 2C 2 × 0.01(µ F)

5 × 0.05  2
= 1 − e−10 
R2 R2  2  
E + qC  0.125 µJ
-
q
⇒ E − iR1 − i1 R 3 − =0  ….. (i) Sol 15:
C
0 
And E − iR1 − i2R 2 = ….. (ii) q1
i= i1 + i2 …… (iii) (a)
 q2
q
So i2R 2 − i1R 3 − =0
C
q dq1 dq2
(i − i1 ) R 2 − i1R3 − 0
= =
=i =
C dt dt
q
iR 2 − i1 (R 2 + R3 ) − 0
= q1 q2
c And − iRC =
C C
R  q
⇒ iR1.  2 = i1 (R 2 + R 3 ) + and q1 + q2 =
CV
 R1  C

 q R q ⇒ q1 − iRC = CV − q1
⇒  E − i1R3 −  . 2 = i1(R 2 + R3 ) +
 C  R1 C dq1
⇒ 2q1 + .(RC) =
CV
dt
R  R .R  q  R 
⇒ E. 2= i1  R 2 + R3 + 3 2  + 1 +  2q1 dq1
R1  R1  C  R1  ⇒ + V /R
=
C dt
q
⇒ E.R 2= i1(R1.R 2 + R1.R 3 + R 3 .R 2 ) + .[R .R ] q,t t
C 1 2 ⇒ ∫
q0,0
(
d q1.e2t/RC = ) V
∫ R .e
0
2t/RC
.dt
2 dq q(R1 + R 2 )
⇒ = +
∑ R1.R 2 dt c.( ∑ R1.R 2 ) 2t/RC V (RC)  2t/RC 
⇒ q1.e= − q0 . . e −1
R 2  
t.(R1+R2 )  t.(R1+R2 ) 
 E.R 2  t c( ∑ R .R ) dt q,t  c.( ∑ R +R )  VC 
 ∑ R .R  ∫ e = ∫ d q.e
⇒  1 2 1 2
  ⇒ q=
1 q0 .e
−2t/RC
+ . 1 − e−2t/RC 
 1 2  0 0,0   2  

dq1  −2  −2/RC VC  2  −2t/RC
 t(R1+R2 )  t(R1+R2 ) = So, q0.  .e +   .e
E.R 2 C( ∑ R1.R 2 )  C( ∑ R1.R2 )  C( ∑ R1.R2 ) dt  RC  2  RC 
⇒ . e −1 = q.e
∑(R1.R 2 ) (R1 + R 2 )  
  −V V
= × 2.e−2t/RC + .e−2t/RC
R R
ECR 2  −t(R1 + R 2 )  dq
⇒ 1 − e =q V
(R1 + R 2 )  C( ∑ R1.R 2 )  = i=
− 1=+ .−2t/RC
dt R
(b) Heat generated = Einitial − Efinal
 − t(300 +300)×2 
 10 × 0.01× 300   0.01×106 [300×300 +300×50 + 300×50×186 ] 
q   . 1 − e   1  CV 2 1 
 300 + 300   1
  = × CV0 2 −  ×  × × 2 
2 2  2  C 
 
0.1 
= . 1 − e− t/2  = 0.05 1 − e− t/2  µC CV02 CV02 CV02
2    
= − =
2 4 4
P hysi cs | 19.91

3q q q So capacitance = Q / ∆V
Sol 16: E = − =
2Aε0 2Aε0 Aε0 = q / 3q / 5c
qd q 5C 5Aε0
⇒ Voltage = E × d= = = =
Aε0 C 3 3d
1
So energy initial = × C × V2 (ii) Charge on plate 3 ⇒ q4
2
2
q (from 2 and 3)
2q4 + q2 =
1 q q2 q2d
= × C ×  = = ⇒ q4 = − q / 10
2 C 2C 2Aε0
Now, we have ∆ v =
3q / 5C , So v 0 = 3q / 5C
and Efinal = 0 (both have q = 2q)
5V0 C
So Einitial = Efinal + heat ⇒ =q
3
q2d 5V0 Aε0
⇒ = Heat ⇒ .
=q
2Aε0 3d
Aε0 .V0
Sol 17: So, q4
= ⇒ on plate 3
6d
1
And charge on plate 5 = q1
2
-
3 Now, q1 + q2 =
q
4
5
⇒ q1 = 2q / 5
6
+
7 2 5V0 .Aε0 2 V0 Aε0
8 = × =
5 3d 3 d

Sol 18:

1 2 3 4 5 6 7
Qo Q2 Qo-Q2
3F
3F 10V 6F
5V
Q1-Qo 3F
-q1 7 q
2 8
Q1 A B
q1 1 -q3
q3 6F 5V 6F


q4 5 Qo
3 -q4 6 Qo 10V
A B
q2 4
-q2 Set up potential drop equations to get the values of Q0,
Q1, and Q2
Let q charge be given to the system. Now, close the switch ‘AB’ and draw a similar charge
distribution diagram.
q 
q1 + q2 = …. (i)
This will give you the new charges across all the plates.
q 
q3 + q4 + q2 = ….. (ii)
q3 q4 By using appropriate arithmetic, you can easily get to
− =0 ⇒ q3 =q4 …. (iii) the charge flown across the switch AB.
1 1 
q1 − q2 + q4 =
0
q2 
q1 + q4 = ….. (iv)
So on solving,
⇒ q2 = 3q / 5
So ∆ V =
3q / 5C (C = capacitance of each capacitors)
1 9 . 9 2 | Electric Potential and Capacitance

Sol 19: Only change would be there In q3

60C q3 = CV = 3µ F × 10 = 30µ F
-10C 10C So energy or heat generated = work done by battery -
energy stored in capacitor
A 20 V 4F
1
50C = 10 × 30 − × 3 × (10)2 = 300 − 150 = 150 µ J
2
−1 −1
 1 1 
Sol 21: Ceq = 1 +  +  + 1
3F   2 2 
V 2 V-20 2 2 4
1 = + = µF
20 V 4F 3 3 3
2F 3
4
5F So , Q =Ceq .V = × 24 =32 µ C
3
This will be divided equally, as the capacitor are
Charge is conserved
equal ⇒ Q = 16 µ C
(V − 0) × 3 + (V − 0) × 2 + (V − 0) × 5 + (V − 20) × 4 = 10 + 60 + 50
= 10 + 60 + 50 1
1 2
2
⇒ 14V − 80 =
120 24V 1
1 FF S 2
2 FF
+
+ 24V S
⇒ VA = 200 / 14 = 100 / 7 --
2'
2' FF 1
1 FF
300
(b) q1= 2 × (100 / 7) , q2 =3 × V = µC 4
4 3
3
7
5 × 100 300 (a)
(a)
And=
q3 = µC
7 7
-Q
-Q11 Q
Q11 Q
Q22 -Q
-Q22
 100 
= q4 = (VA − 20) × 4 µ F =  − 20  × 4µ F 1 2
 7  1
1FF 2FF
2
24V 1 S 2
+ 24V
+ S
−160 -- 0
= µ F = 22.88
7
0 2
2 FF V
V00 1
1FF
4
4 3
3
Q22 Q Q22
Sol 20: Q -Q
-Q11 Q
Q11
5
(b)
(b)

2F
10V 3F When the switch S is closed as shown in the figure
(b), then the charges stored across positive plates
5V

q2 capacitors 1 and be Q1 and Q2. The same charges will


0 - + (V+10) be distributed across negative plates of capacitor 3 and
4F 4. Let the potential at the negative terminal is zero, then
Now, q1 + q2 =
0 the charges across capacirots 1, 2, 3 and 4 will be

0 (Cons. of charge)
⇒ (V − 5)2 + (V + 10)4 = Q=
1 ( 24 − Vo ) × 1µF …(i)
⇒ 6V + 30 =
0 Q 2= ( 24 − Vo ) × 2µF  …(ii)

⇒ V = −5 Q 3= V0 × 1µF …(iii)

So q1 =−
( 5 − 5)2 =− 20 µ C Q 4 = V0 × 2µF …(iv)

And q2 = ( −5 + 10) × 4 = 20 µ C Thus from (i) and (iii) V0 = 12V and thus the charges be
n
After the switch is closed, the above eq s would s + Q1 =
Q3 =
12µF and Q 2 =
Q4 =
24µF
111 be valid , and hence q1 and q2 would be same.
P hysi cs | 19.93

Thus when switch is zero, 12µF charge is passed through Sol 24:
the switch. q1 C

Sol 22: Before opening the switch potential difference VF 0


across both the capacitors is V, as they are in parallel. q2
Hence, energy stored in them is,
VC
1 2
U= U= CV
A B
2 So, =
Q 156 × C
Thus, q1 + q2 =
156C
∴ UTotal =CV 2 =Ui
…(i)
⇒ CV + αCV 2 =156C
After opening the switch, potential difference across it
⇒ V 2 + V − 156 =0
is V and its capacity is 3C
⇒ (V − 12)( V + 13) =
0
1 2 3 2
∴ U
=A (3C)V
= CV ⇒ V = 12 Volts
2 2

In case of capacitor B, charge stored in it is q = CV and ε0 (A/ 2) K ε0 (A/ 2)


Sol 25:
= (i) C +
its capacity is also 3C. d d

8.85×10-12
q2 CV 2 = [0.02+ 0.18]
Therefore,
= UB = 8.85×10-4
2(3C) 6
= 0.2 ×10-8 F
2 2
3CV CV 1 2
∴ UTotal = + Energy = CV= 1.2 × 10−5 J
2 6 2
(ii) Q = CV
10 2 5CV 2
= = CV Uf  …(ii) ε0 A
6 3 Find the new capacitance, C' =
Ui 3 d
From eqs. (i) and (ii) U = 5 Q 2
Q 2
f Work done = – = 4.84 × 10−5 J
2C' 2C
Aε0 Aε0 AV0
Sol 23: C = , So Q = CV = × V0 = .ε 0
d d d
kc
AV0 ε0 Q2 Q2 (iii) C’ 2
So, Q = , Einitial = =
d 2C0 2C0
1 1 Aε
= × C0 V02 = . 0 × V02 Find the new charge distribution, and proceed as (i) & (ii)
2 2 d
Energy of system = 1.1×10-5 J
ε0 AV02
Einitial =
2d
2 Sol 26:
 Aε0  2
  V0
Q2 = d  V02 Aε0
E=
final = .
2Cf  Aε0  2k d (1-x)
2.k.  
 d 
x
Efinal + work done = Einitial

Aε0 V02 1 Aε V 2 Aε0 V02 Now area of air = 1× (1 − x)m2 = (1 − x)m2


 1
⇒ . + W = 0 0= ⇒ W  1− 
2d k 2d 2d  k = area of air = xm2
1 9 . 9 4 | Electric Potential and Capacitance

So both are in parallel connection. q1 q2


⇒ + − iR =
0
(1 − x).ε0 11× x.ε0 C 2C
Capacitance = +
d d
q1  q + 100  qd1
ε0 10x ε0 ε0 ⇒ +  1 + .R =
0
= + = 1 + 10x  C  2C  dt
d d d 
Now, Q = CV 3q1 50 dq1
⇒ ++ + .R = 0
dC dC 2CR RC dt
⇒ = i= .(V)
dt dt
ε d dq1 3
= V. 0 [1 + 10x] ⇒ + .q =
− 50 / RC
d dt dt 2CR 1
V.ε0 dx 500 × 10 × 0.001 q1t d
= × 10 × = −50
∫ d(q1.et/20R /3 ) = . et/2RC/3 .dt
RC ∫q
d dt 0.01× 8.85 × 10−12 ⇒
100,0
−10 −5
= 5 × 8.85 × 10 = 4.425 × 10 Amp.
−50 2CR  t/2CR /3−1 
⇒ × . e
Sol 27: We have Ceq= 30 µ F RC 3  

−100  (t/2RC/3) 
10F ⇒ q1et/2CR
= /3
− 100 e −1
3  

100 
q1 100.e− t/RC/3 −
⇒= . 1 − e− t/2RC/3 
3  

200 400 − t/2RC/3


⇒ q2 = q 1 + 100 µ C = + .e
3 3
20F

+ - 200
Charge
10V (C) 3

So, q = 30 × 10 = 300 µC
t
Thus, q on 20 µ=
F 200 µC
Sol 28: Q = C0V0
And q on 20 µ=
F 200 µC
Now,
Now,
q1 -q1 q1 q2

C0 C

-q2 q2
q1 q2 q1 q q C
So, − =0 ⇒ =2 ⇒ 1= 0
C0 C C0 C q2 C
Now, q2 − q1 = 200 − 100 = 100 µCNow,
C0
−dq1 dq2 So, q1
= (q1 + q2 ) (After first)
i= = − (C0 + C)
dt dt
q1 q2 C02 V0
And, + − iR =
0 = (After First)
10 20 (C0 + C)
P hysi cs | 19.95

After 2nd V 2dq1 2q1


⇒= + (from i and ii)
C0 C0 2R dt C
q1 = . .(C V )
(C0 + C) (C0 + C) 0 0
V t t/RC q1,t t/RC
 C0  2 ⇒
2R 0 ∫e = 0,0 ∫ d.(q1.e )
s= .C0 V0
 C + C 
 0  VC  t/RC 
⇒ . e. −1 =q1.et/RC
 C0  2  
Now after n ⇒  .C V = q1
 C + C  0 0
 0  VC 

= q1 1 − e− t/RC 
n 2  
q  C0 
voltage 1 V=
So= 0
 C + C 
35
C0  0  Sol 30:
n
 C0 
⇒   = 0.35
 C0 + C 
300C -360C
n 300C -360C
 0.2 
=   = 0.35 -300C 360C
 0.2 + 0.01075  -300C 360C

⇒ (0.094899)n = 0.35
qq1 1 -q-q
1
1
ln(0.35)
⇒ n= 
22 FF
ln(0.94899) -q33
-q -q-q
2 2
⇒ n = 20 33FF 22FF
q33 q2q2

Sol 29:
S −q3 q1 q2
i R
i1 (i) Now Kirchhoff’s law : − + 0
=
3 2 2
i3 q2 q1 = 3q
=2 3q1 + 2q3 …. (i)
V R 
C
i2 C and charge conservation
⇒ q1 − q3 = 300 µ C
i = i1 + i2 + i3 q2 − q3 = 60 µ C
dq1 dq
= i1= , 2 i2 = 3 60 − q3 =
 3 300 + q3  + 2q3
dt dt
q1 q2 = 180 − 3q3 = 900 + 3q 3 + 2q3 = −720 =
8q3
And − = 0 ⇒ q1 + q2  …. (i)
C C = q3 =
− 90 µ C
dq1 dq2
⇒ = = i= i And thus, =
q2 150 µ C
dt dt 1 2
And =
q1 210 µ C
d2
⇒ = i3R ⇒ q2 = i3RC 2
C 1 Q2 1 Q2
(ii) Einitial = × + ×
q2 2 C1 2 C2
⇒ i3 =  …. (ii)
RC 1 (300)2 1 (360)2
q = × + ×
V − iR − 1 = 0 2 3µ F 2 2µ F
C
= (300 × 50 + 360 × 90) µ J = 0.0474 J
(Kirchoff’s Law in bigger loop)

V q q (210)2 (90)2 (150)2


⇒ =i + 1 =i1 + i2 + i3 + 1 Efinal = + +
R RC RC 2×2 2×3 2×2
= (5625 + 1350 + 11025) µ J =0.018 J
1 9 . 9 6 | Electric Potential and Capacitance

Sol 31: 
  P   1  ˆ
E2 + 2k    3  ( − j)
+q  2 y 

e    kP
E = E1 + E2 = ( −ˆi − 2ˆj)
2y 3
a a
Sol 33:

+q +q I. +q
a
r
Initial +q
2
kq
Ui = ×3 r
a
a/2 a/2 Or
eq eq eq +q
r
Final
kq2 +q
kq2
Uf = + ×2
a a II. +q
 
2 r
2v
Work done = Uf – Ui

5kq2 3kq2 2kq2 r


= - =
a a a +q O r +q
Work done = Power × Time

2kq2 2r
⇒t=
ap +q
9 −2
2 × 9 × 10 × 10
= = 18×104 sec
1× 103
kq2 kq2 kq2 kq2
UI = ×2+ + ×2+
r 2r 3r 4r
Sol 32:
kq2 kq2 kq2
y UII = + + ×4
4r 2r 5r
P (0,y)
kq2  8 4 
Wfirst step = UII – UI =  − 
r 3 5
o x
45
 III.
P

E1 o,y (0,y) 


E2 2v

P r
 +
2 
P r
2

  P  1 2r
E1 = k   3 ( −ˆi)
 2  y 0
P hysi cs | 19.97

IV. Sol 35:


Eq
r q
mg
2r 2r
r H

r
UIII = UII ∴ wsecondstep = 0
dr
⇒ wthird = UIV – UIII – UII – UI - Wfirst step

∴ kldone
∴ Total work total =w first + wsec ond + w third =0 Initial electric potential = V
Q
kdq
Sol 34: Q = ∫ dv = ∫ (H2 + r2 )1/2
0

a
kσ2πr,dr  q 
⇒V= ∫ (H2 + r2 )1/2  where σ = 2 
A 0  πa 
L a
 (H2 + r 2 )1/2 
B ⇒ V = πσk  
Q  1/ 2  0
σ = surface charge density = take an elemental
πRL
part
dx
(
⇒ V = 2psk (H2 + a2 )1/2 − H = 2 )
Final electric potential Vf = 2psk (a)
 [substitute H = 0]
r
 By equation conservation,
x A mg H + 2psk ((H2 + a2)1/2 – H) q = (2pska)q

 q   q 
⇒ gH + σ   ((H2 + a2)1/2 –H) = (σ . a)  
dp 2 ε
 0  m  m2 ε0 

The potential due to this part at A is ⇒ H + 2 (H2 + a2)1/2 – H) = 2s


kdq kdq 2
dV = =  H H2
2 2  ⇒  a +  = H2 +a2 ⇒ 3 = aH
R +x  2 4
k
⇒ dV = . σ . 2pr d  4a
 ⇒H=
3
⇒ dV = kσ . 2psin θ d 
U = mg  2 H2 + a2 − H 
V
 
⇒V= L dU
∫ dV = kσ 2π sinθ ∫ d
=0⇒
2H
-1=0
0 dH H2 + a2
1 Q R a
= 2π L L ⇒H= (equilibrium)
4π ε0 πRL 3

Q
⇒V= U
2n ε0L

Energy required to bring the charge from infinity to


open = Uf – Ui a
H=
qQ 3
= 2π ε L
0
1 9 . 9 8 | Electric Potential and Capacitance

Exercise 2
⇒ 4.5CV 2 =
Heat generated
2
Single Correct Choice Type =ratio 4.5CV
= / 2CV 2 2.25

Sol 1: (A) Vmax 16/3 Sol 4: (B)


O oA
−1 d
1 1  16
Ceq =
 +  =µC then let vmax = the max (2, 4)
 8 16  3  oA
d (5)
voltage across the capacitors oA
d
16 (A, 1) oA (3, B)
So charge =Q= Cv max
= × v max
3 d
Now v =16 × v × 1  Q  
1 max
3 16  C  
E
v
= max < 80 ⇒ v max < 240V  2ε0 A   ε0 A 
3   
16 1  d   d 
and v 2 = × v max × Net capacitance =
3 8 3ε0 A
2 d
= v < 20 ⇒ v max < 30V
3 max 2 ε0 A
=
so vmax = 30V . 3 d
2 ε A 2 ε0 A
16 × 30 Charge flown =  0  E = E
Thus =
Q = 160µC 3 d  3 d
3  

Sol 5: (A) Force between plates = mg


Sol 2: (B) Ceq = 2C where C= capacitance of each plate.
σ σ2 A Q2
1 And A
× σ= = mg ⇒ = mg
So energy = × Ceq × V 2 2ε0 2ε0 2Aε0
2
(0.1)(8.85 × 10−12 ) ⇒ 2mgAε0 =Q
= CV 2 = × 102 = 10−1 µJ
−3
0.885 × 10
Sol 6: (B) Now across point c and d ∆V = 0
Sol 3: (B)

CV -CV CV 2CV q1 C


2V q2
Initial Final 2C

⇒ The capacitance is 0 for that


Now, initial energy + work done by battery
⇒ Ceq = C1 + C2 = 2Aε0 / d
= Final energy + heat generated

1
∴ × C × V 2 + 2V.(3CV)
2
1
= × C × (2V)2 + Heat generated
2
1 2
⇒ CV + 6CV 2 = 2CV 2 + Heat generated
2
P hysi cs | 19.99

Sol 7: (B) 2.1 Vmax .C 10


V=
2 = 0.3Vmax < 1 KV ⇒ Vmax < KV
3C 2C 7C 3

q1 -q1 q1 -q1 2.1Vmax .C 20


V'=
2 = 0.7Vmax < 2 KV ⇒ Vmax < KV
v2 v2 3C 7
= 2.5 KV

2
q2 -q2 q2 -q2 Sol 9: (A) q 1F
v2 v2

oA oA
d d

A B 1
q 1F
Let there be a charge Q on body 1.
oA
q1 q2 q1 1
d So = ⇒ =
2ε0 A 1 2 q2 2
Net Capacitance =
d and q1 + q2 = Q ⇒ q1 = Q / 3 where q = initial charge
−1 −1
 1 1   1 1  Q/ 3n
⇒q=
Sol 8: (A) Ceq =  +  + + 
 3C 2C   7C 3C 
EC
6C 21C 33C Sol 10: (D) Total charge now= E.C / 2 =
= + = =3.3C 2
5 10 10
Now after dielectric is inserted,
Q max = Vmax × Ceq = Vmax (3.3C)
kC.C kC kCE
Ceff
= = so
= Q net
q1 q2 q 4 (kC+ C) (k + 1) (k + 1)
Now
= = ; 1
6 / 5C 21/ 10C q2 7 KCE CE (K − 1)CE
so Q flowed = − = from B to A
4 k +1 2 2(k + 1)
Now q1 = × Vmax × (3.3C) =1.2 Vmax C
11
as capacitance increases.
7
q2 = × Vmax × (3.3C) =2.1Vmax C
11 Sol 11: (B) Voltage would be highest at x = 0. And
E = constant ⇒ V ∝ x so it would decrease linearly,
from x = 0 to x-d, will remain constant from x = d to
x = 2d.
c B

A
d
Sol 12: (C) 1000V 0V

1.2 Vmax .C ∆V = E × ∆d = 200 × 5 = 1000V.


V
=1 = 0.4Vmax < 1 KV ⇒ Vmax < 2.5 KV
3C
Now the electric field would be same so
1.2 Vmax .C 10
V'=
1 = 0.6Vmax < 2 KV ⇒ Vmax < KV ∆V = E × ∆d = 200 × 3 = 600V.
2C 3
1 9 . 1 0 0 | Electric Potential and Capacitance

Sol 13: (B) Q1 =


1500µC and Q 2 =
100µC q02 RC
⇒ ×R × × 1= 3.6 × 10−3 × 2
q1 q2 (RC) 2 2
q1 + q2 = 1600µC and = ⇒ q1 = q2 = 800µC
1 1 ⇒ q02 = 7.2 × 10 −3 × 2 × 10 −6
∆V= q / c= 800µC / 1µC= 800V
=14.4 × 10−9 =144 × 10−10
E.kC ⇒ q0 = 12 × 10 −5
Sol 14: (C) Q
= 1 Q=
2 Ceq=
.E
(k + 1)
dq  1
EC Q 2 ' k + 1 Sol 20: (C) q = q0 .e− t/ τ ⇒ = q0 .  −  .e− t/ τ ;
and Q ; dt  τ
= 1' Q=
2' Ceq
= .E =
2 Q2 2k
q
− 0 .e − t / τ
i=
τ
Sol 15: (B) From the symmetry, we can say that the
capacitance would be same. τ = RC = (r + 2)0.5 × 10−6 sec

1
and thus ×i i|t ln 4µs.
=
1.5F 2 =|t 0 =

300 C -360 C
1
-300 C +360 C τ = 2µs = (r + 2) × × 10−6 ⇒ r = 2Ω
2
1  q0   q0  − t/ τ
⇒ ×  −  =  −  .e = et/ τ = 2
2  τ   τ 
1.5F ⇒ ln 2 ⇒ ln 4 / τ
Sol 16: (A) q= q0 .e
q3− t/ τ -qwhere
3 τ (2 + r)0.5µ F
=
dq q0 − t/ τ 1.5F
⇒ =.e Multiple Correct Choice Type
dt z
300 -q -q2 -360 C

C1
-300 qC 2F ln2 × µs 3F +360 C Sol 21: (A, B, C) Now from charge conservation
e− t/ =
τ
1/ 2 1⇒ ln2
= t/τ⇒ = qln2
2
z q1 + q2 = 60µC ... (i); q3 − q1 = 300µC ... (ii)
r == 00)
⇒ τ = 1µs = (2 + r) × 1/ 2 ⇒ (r
⇒ q2 + q3 = 360µC
1.5F q1 q3 q2
q3 -q3 + − = 0 ; 3q1 + 4q
=3 2q2 ... (iii)
2 1.5 3

3(q3 − 300) + 4 q3 = 2(360 − q3 )


-q1 -q2
 q1
2F
q2
3F 7q3 − 900 = 720 − 2q3 ⇒ 9q3 = 1620
1620
⇒ q3 = = 180µC
9
and q1 =
−120 µC and q2 =
180µC
1
Sol 17: (D) R e q × C = × 100µF × 103 −1
2  1 1  2C 2CE
Sol 22: (A, D) Ceff =
 +  so charge =
= 0.5 × 105 × 10−6 = 0.05sec  C 2C  3 3

Final Ceff =
2C ⇒ charge =
2CE
Sol 18: (D) As at t = 0 the capacitor is assumed as a wire.
2CE − 2CE / 3 =
4CE / 3

dq q
Sol 19: (B)
= q q0 .e− t/RC ⇒= 0 .re−= t/RC
i Sol 23: (A, D)
dt RC
q0 1F v+10 2F
⇒=i .e− t/RC Now ∫ i2R.dt= 3.6 × 10−3 5 v O
−6
2 × 10 × 10
C1 C2
P hysi cs | 19.101

From charge conservation Q C = Q C 30


1 2 ∆V across 12 µF = 4 × = 10V
12
(5 − V) × C1 = (v + 10) × C2 ; 5 − V = (V + 10)2
CE/3 CE/3
−15 =
3V ⇒ V =−5V 2CE/3 + -

1 1
EC = × 1× (10)2 = 50µ J ; EC = × 2 × (5)2 = 25µ J  + - o
1 2 2 2 CE/3 CE/3

⇒ 2EC =
EC + - + -
1 1
E E
σ
Sol 24: (A, B) We have E = (by one plate)
2ε0
Sol 26: (A, B, D) Charge = KCV – CV=(K-1)CV
σ Q2
So force = . Q= Energy absorbed= (K − 1)CV 2
2ε0 2Aε0
1 (K − 1)
C2 V 2 d CV 2 Energy
energy = × K × C × V2 = .CV 2
Q = CV ⇒ F= × = 2 2
2Aε0 d 2d
1 1
Now × K × C × V 2 + work = × C × V 2 = (K − 1)CV 2
2 2
Sol 25: (B, C, D)
1
Ceq ⇒ Work
work =(K − 1)CV 2
12F 2

q1 Sol 27: (B, C) V = E d , so E=same.


7F same same
q2
3.9F
q1 Aε0 Aε0 V
3F Sol 28: (A, C, D) C = and Q = CV =
d d

Q CV V
Now Vnet = = = = E×d ⇒ E =V / Kd
KCV KCV C KC K
+ - + -
And energy initial + work done=energy final

 Q2 Q2
+ work done =
2C 2KC

V V Q2 1  CV 2  1 
Work
work =  − 1 =− 1 − 
2C K  2  K
−1
1 1
Ceq ' = 3.9 +  +  = 3.9 + 2.1 = 6µF Putting the value of "C" from the first line, we get
3 7
−1 ε0 AV 2  1 
1 1  =W 1 − 
Ceq =3.9 +  +  =4µF so q
= 4EµC. 2d  K 
 6 12 
−1
q1 q2 2.1  1 1 
q1 + q2 = 4E and = ⇒ q1 = × 4E Sol 29: (C, D) C=  + 
2.1 3.9 6 eq
 C1 C2 
= 0.7 × 2E = 1.4E −1
 x d−x−t
0.7 × 2EµC =  + 
so ∆V across 7 µF ⇒ =
q1/ 7 µF = 6  Aε Aε0 
7µF × 10  0
−1
⇒E=30V d−t  Aε0 
= =
⇒ q1 = 0.7 × 2 × 30 = 42µC.  Aε   
 0  d−t
q
∆V across 3 µ=
F = 14V ;
3 µF
1 9 . 1 0 2 | Electric Potential and Capacitance

Sol 30: (A, C, D) ∆V= iR= 5A × 1Ω= 5V.


CV KCV
So Q = CV = 1µF × 5 = 5µF

Sol 33: (B, D)


120 120
=i = = 40A
R e q  1 1 −1
 + 
2 2
V V
Now this I will be divided equally,=
so i1 20A,i
= 2 20A
Q=C’V=KCV
Now ( ∆ V2 ) =
0.... (B)
1 2 1 Q C = 20 × 2µC = 40µC.....(D)
Einitial = CV Efinal = × K × CV 2 = K × Einitial 1
2 2
As V =
E × d ,so
soEE=
=same
same Assertion Reasoning Type
same same

Q2 C2 V 2 Sol 34: (D) Statement-I is false electrostatic force


=F = K2
2Aε0 2AV0 remains the same.

Sol 31: (A, B, C, D) Sol 35: (C) Statement-II is false.


Q -Q q Q-q -2Q+q
2Q-q
Sol 36: (D) Q = (ε − iR)C ⇒ i = (ε − Q / C).1/ R
A B  -(Q-q)
Work done by battery = ε.Q = ε(ε − iR)
Q-q
Q 2 (ε − iR)2 C
And energy in capacitor = =
2C 2
d d/2 d/2
(ε − iR)2 C
= + iR.(ε − iR).C
2
i1 100 60

⇒ Statement-I is false.
120 C1 C2 V

Comprehension Type
i2 80 60

Sol 37, 38: (B, B)


Now the charge on outer surfaces should be same C1
q = Q −q ⇒ q = Q /2
So (A) =
−2Q + Q / 2 =
−3Q / 2
Charge on A=Q/2 and charge on B = 3Q/2 C2
So as E ∝ Q and V ∝ E ⇒ V ∝ Q and hence C is
correct.
C1.d q1 q2 Q 0 − q2 x
C2 = =⇒ =
Sol 32: (A, B, C, D) (A) at t=0 consider it as wire. x C1 C2 q2 d

10 10 10  
(B) = i= = = 5A Q0  Q 
Re q −1 2 x
 1 1 ⇒ 1+
=  0 
⇒ q2 =
1+  +  q2 d  1+ x 
2 2  
 d
(C) From kirchoff 's law
i1 R2
i1R1 = i2R 2 ⇒ = = cons tant
i2 R1
P hysi cs | 19.103

Now force Previous Years’ Questions


Sol 1: The diagrammatic representation of given
problem is shown in figure.

+ - Q1=C1V1=CV
+ -
+ -
+ -
+ -
t
d
+ -
x d-(x-t) + -
+ -
Q2 q22 Q 02 + - Q2=C2V2=(2C)(2V)=4CV
= = = + -
2Aε0 2Aε0 (1 + x/ d)2 2Aε
0
The net charge shared between the two capacitors is
Q 02 2d
1 Q’ = Q2 – Q1 = 4CV – CV = 3CV
And work done =
2Aε0 ∫ (1 + x/ d)2
d The two capacitors will have the same potential, say V’
2 2
Q0 d Q0 d The net capacitance of the parallel combination of the
= ×=
2Aε0 6 12Aε0 two capacitors will be
C’ = C1 + C2 = C + 2C = 3C
And potential difference = q1/C
The potential difference across the capacitors will be
Q0 Q 0 (x/ d) Q 0 x
q1 =
Q0 − = = Q' 3CV
1+ x / d 1+ x / d x + d V’ = = =V
C' 3C
Q 0 xd The electrostatic energy of the capacitors will be
potential q1 / C1 =
(x + d) A ε0 1 1 3
U’ = C’V’2 = (3C)V2 = CV2
2 2 2
Sol 39: (A, C) i=E/R1 → → →
Sol 2: Electric field within the plates=
E E Q1 + E Q2
E
Sol 40: (A, B, (B)
C) Current = 
R1 + R 2 E
E × R2
(A) Voltage
voltage = ;
(R1 + R 2 )
2
1 1  ER 2  +Q1 +Q2
(C) × CV 2 = × C × 
2 2

 (R1 + R 2 )   E2 E1

Sol 41: (A, C) (A) qmax = same.


1 2
(B) is wrong option as q = CV Now, C can be different
Q1 Q2 Q1 – Q 2
E = E1 – E2 = – ,E=
(D) Not necessarily, depends on R1 and R 2 also. 2Aε0 2Aε0 2Aε0

∴ Potential difference between the plates


Sol.42: (D) τ1 < τ2 ; R1C1 < R 2C2 ;
Now E1C1 = E2C2 (charge same)  Q – Q2  Q1 – Q 2 Q1 – Q 2
VA – VB = Ed =  1
 2Aε  d =  Aε  =
2C
R1 C2  0 
2  0 
⇒ C1 = C2 ⇒ R1 < R 2 ; <
R2 C1  d 
1 9 . 1 0 4 | Electric Potential and Capacitance

ε0 A ∴ V = V0, C > C0
Sol 3: (D) Applying C = ,
t1 t2 Q = CV∴ Q > Q0
d – t1 – t2 + +
K1 K 2
1 2
U= CV ∴ U > U0
we have 2
V
E= but V and d both are unchanged
A/2 A/2 A d
K1 d/2 K2 d/2 Therefore, E = E0
+  K d
K3 d/2 K3 d/2
Sol 7: (B, D) Charging battery is removed. Therefore,
q = constant Distance between the plates is increased.
Therefore, C decreases.
In parallel
q
ε0 (A / 2) Now, V = , q is constant and C is decreasing
C
d/2 d/2 Therefore, V should increase.
d–d/2–d/2+ +
K1 K3
1 q2
U= again q is constant and c is decreasing
ε0 (A / 2) Kε0 A 2 C
+ = Therefore U should increase.
d/2 d/2 d
d–d/2–d/2+ +
K2 K3
Sol 8: (A, C, D) Battery is revoved. Therefore, charge
K1K 3 K 2K 3
Solving this equation, we get K = + stored in the plates will remain constant
K1 + K 3 K 2 + K3
ε0 A
Q =CV = V
Sol 4: (C) DU = decrease in potential energy = Ui – Uf d
2 Q = constant.
1 1  V + V2 
= C( V12 + V22 ) – (2C)  1  Now, dielectric slab is inserted. Therefore, C will
2 2  2  increase. New capacity will be
1 ε0KA
= C(V1 – V2)2 C’ = KC =
4 d
Q V
Sol 5: (D) q1 = C1V = 2V = q V’ = =
C' K
This charge will remain constant after switch is shifted V' V
from position 1 to position 2. And new electric field E = =
d K.d
1 q2 q2 q2 Potential energy stored in the capacitor,
Ui = = =
2 Ci 2×2 4 1 ε AV 2
1 q2 q2 q2 Initially, Ui = 2 CV2 = 0
Uf = = = 2d
2 Cf 2 × 10 20 2
1 1  Kε A   V  ε0 AV 2
q2 Finally, Uf = C’V’2 =  0    =
∴ Energy dissipated = Ui – Uf = 2 2  d   K  2Kd
5
 q2  Work done on the system will be
This energy dissipated  =  is 80% of the initial
 5 
 q2   
ε0 AV 2  1 
stored energy  =  |DU| = 1– 
 4  2d  K 
 

Sol 6: (A, D) When dielectric slab is introduced capacity Sol 9: (B, C) The magnitude and direction of electric field
gets increased while potential difference remains at different points are shown in figure. The direction of
unchanged. the electric field remains the same. Hence, option (b) is
correct. Similarly, electric lines always flow from higher
P hysi cs | 19.105

to lower potential, therefore, electric potential increase if (d < n)


continuously as we move from x = 0 to x = 3d. kQ kQ 4x
⇒ - =0 ⇒ =d
q1 q1 d 4(x − d) 5

- + - + Electric field at P is zero


- +
- + d
- +
- + - +
+Q -Q/4
f
- +
- + E-Q/4 EQ
x
- +
- + - + kQ kQ
- E0/K + ⇒ =
2
E0 E0 d 4(d − x)2
x=0 x=e x=2e x=3d
±d
⇒d–x=
Therefore, option (c) is also correct. The variation of 2
electric field (E) and potential (V) with x will be as 3d d
follows ⇒x= or x =
2 2
E 2x
⇒d= or d = 2x
3
(since d > x)
d 2d 3d
x 
Sol 11: (A) r = (iˆ + 3ˆj + 2k)
ˆ - (4iˆ + 7ˆj + 2k)
ˆ

= −3iˆ − 4ˆj + 0kˆ



V |r | =5
k×q 9 × 109 × 10−8
B V= = = 18 v
A r 5

Field is parallel to r and thus has no 3-component

V0
x Sol 12: (A, C) It must have any K.E. at B to reach A
O d 2d 3d
Since
OA||BC and (Slope)OA > (Slope)AB UA – UB = -e (VA – VB) = +4eV
Because E0–d = E2d–3d (K.E.)f – (k.E.)i = UB – UA = -4eV
And E0–d > Ed–2d
Sol 13: (A)
Sol 10: (A, B, C) + -
+ -
+ -
PQ n P -Q/4 + -
+
+ - -
+ -
d + -
Potential at P is zero +
+ -
-

If (d>x)
By symmetry potential due to negative part =(potential
kQ kQ due to particle part). (Also every small charge is
⇒ - = 0 ⇒ d = 4d – 4x
d 4(d − x) equidistant from axis)

3d 4x ∴ Potential at all potential axis is zero


⇒x= =d=
4 3
1 9 . 1 0 6 | Electric Potential and Capacitance

Direction of field is perpendicular to axis and towards (It is independent of presence of conductor because
negative side induced charges provides zero potential at center)
There will be a torque when placed in uniform field Potential B = Potential at C
(Since electric field inside conduction is zero)
Sol 14: (A, D) Electric field due to dipole exists inside
⇒ V = constant
sphere only a circle on the sphere has zero potential
which is equidistant from poles of dipole ∴ Potential at B due induced charges
= Potential at B – potential at B due to q
kq kq −kqR
- = - =
+ qenclosed =0 4 π ∈0 (d + R) 4 π ∈0 d 4 π ∈0 (R + d)d
 Electric flux = 0
Sol 17: (B) Let the distribution of charges be
Zero potential
Sol 15: (B, C) (1) (2) (3) (4)
+q y x Q-x -Q+x y
r+ F

r -x -Q+x Q-x
Q 2

r+ d 2d d
-q
2F. 2
FR = (= 2F sin θ) =
2
a + 2
r + 2
2 Since the potential difference between plates (1) and
(4) is zero.
 kQ  kQ(P)
 2 .q  = − x.d Q − x(2d) (Q − x)d
r +  2
 (r + 2 )3/2
2 ⇒ + + =0
A ∈0 A ∈0 A ∈0
1 pQ 3Q
⇒ FR = . 3 (r >>  ) ⇒x= ∴ Option B
4π ε0 r 4
F.r ⇒ Charge on right side of plate 3 = Q – x = Q/4
Torque on dipole = (2  )
(r + 2 )1/2
2

= (=2 F cos q  ) Sol 18: (B) Also Charge is conserved on plate 1 and 4

r.(2 )  l Q  ⇒ y + (-x) + Q – x + y = 0 + 0
⇒T=  . .q 

 4 π ε0 r + 
2 2 1/2 2 2 Q
(r +  )  ⇒y= ∴ Option B
4
1 pQ Q
⇒T= (r >>  ) ∴ Charge on right side of plate 4 =
4π ε0 r 2 2

Sol 19: (C) Potential difference between (i) and (ii)


Sol 16: (A, D)
x ( d) 3Qd
= =
A ∈0 4A ∈0

q R B R C
Sol 20: (B) Obviously, both statements are correct. But,
Statement-II is not a correct explanation of statement-I.
kq
Potential at center of sphee =
4 π ∈0 (d + R) Sol 21: Refer theory on Superposition of electric field &
Ampere’s Loop Law.
P hysi cs | 19.107

Sol 22: (A, D) Q = KQ0 V → Constant Sol 30: (A, B, C)


+q -q
1 1 
=U =
2
(KC ) V2 =2
CV 2  k K U0
 
E3
E = E0 / k
+2q -2q
E1
O
1
Sol 23: (B, D) C ∝ E2
d

+q -q
Sol 24: (A, C, D) Q – remains constant.
2kq
∈0 A k ∈0 A E=
2 E=
3
C changes from to L2
d d 4kq
Hence, other variable also change. E1 =
L2
  
Sol 25: (B, C) Eall = E1 + E2 + E3

-Q 6kq 1 q
+Q = 6×
= 6K (Along OD)
=
L2 4 π ε0 L2

Vat O = 0

Vat line PR is zero because this line is equatorial axis for


three dipole.

x=0 d x=3d Sol 31: (B, D) After switch S1 is closed, C1 is charged


by 2CV0 when switch S2 is closed, C1 and C2 both
have upper plate charge CV0 .
When S3 is closed, the upper plate of C2 becomes
Q2  charged by −CV0 and lower plate by +CV0 .
Eoutside = 
A ∈0 
 → So, ( Q ) is wrong
Q2  Sol 32: (A, D) As E = V/d E1 / E2 = 1 (both parts have
Einside =
K A ∈0  common potential difference)

(B) is correct as the direction of field remain the same. Assume C0 be the capacitance without dielectric for
whole capacitor.
(C) As we are going in the opposite direction of electric
C0 2C0
field, potential would rise. k + C
=
3 3
C 2+k
Sol 26: (C, D) Using Gauss’s Law =
C1 k
  qin
Net =Flux ∫= E.ds
∈0 Q1 k
= .
It electric field is same at all the points of a surface, it is Q2 2
knows as equipotential surface.

Sol 27: (A, B, C, D) Refer Theory.

Sol 28: (C, D) Refer Theory.

Sol 29: (C) 2 µ F & 3 µ F are in parallel combination.


1 9 . 1 0 8 | Electric Potential and Capacitance

Sol 33: (D)

+80
-80
80
q2 =
2
x 80 = 32 q1 = 3 x 80 = 48
5 5
+32 +48
-32 -48

S
4 ε0
C10
= = 2 4 ε0 S
d/2 d

2 ε0 S ε0 S
=C20 = , C30
d d

1 1 1 d  1
= + = 1 + 
C'10 C10 C10 2 ε0S  2

4 ε0 S
⇒ C'10 =
3d

7 ε0 S
C2 =C30 + C'10 =
3d

C2 7
=
C1 3

C10 C20

C30
2017-18 100 &
op kers
Class 12 T
By E ran culty
-JE Fa r
IIT enior emie .
S fP r es
o titut
Ins

PHYSICS
FOR JEE MAIN & ADVANCED
SECOND
EDITION

Exhaustive Theory
(Now Revised)

Formula Sheet
9000+ Problems
based on latest JEE pattern

2500 + 1000 (New) Problems


of previous 35 years of
AIEEE (JEE Main) and IIT-JEE (JEE Adv)

5000+Illustrations and Solved Examples


Detailed Solutions
of all problems available

Topic Covered Plancess Concepts


Tips & Tricks, Facts, Notes, Misconceptions,
Current Electricity Key Take Aways, Problem Solving Tactics

PlancEssential
Questions recommended for revision
20. CURRENT ELECTRICITY

1. INTRODUCTION
Transfer of charge across a cross-section of a conducting medium constitutes an electric current. Any conductor in
general offers some resistance to the flow of electric current through it. This means an electric current cannot flow
continuously all by itself in a conductor. An electric source is needed to continuously drive electric current through
a conductor. Some work is done or energy is supplied by the source to drive the current in a conducting medium.
In this chapter we will study the laws and phenomena that govern the flow of electric current in conductors. We
will discuss the factors that affect the electrical properties of conductors, what constitutes an electric circuit and
the laws of division of current in various branches of a complicated network. Electricity has indeed transformed
our lives beyond imagination. Electric energy is used everywhere, right from the lights of our homes, our electronic
appliances, computers, automobiles, heavy machines used in our industries, hospitals, aircrafts etc. We will mainly
focus on direct current circuits and sources in this chapter. The techniques of circuit analysis developed in this
chapter form the backbone of electrical and electronics science and engineering.

2. ELECTRIC CURRENT
In this chapter we will be mainly dealing with current in a conducting medium. Electric current is defined as the
rate of flow of electric charge through a certain cross-section of a conductor. If there is to be an electric current
through a given surface, there must be a net flow of charge through the surface. The free electrons (conduction
electrons)in an isolated conductor are in random chaotic motion in all directions and on an average same number
of electrons passes through each side of any imaginary surface. Thus, the net charge passing through any surface
in any time interval is zero, and thus the current through the conductor is zero. However, if we connect the ends
of the conductor to a battery, an electric field is applied inside the conductor from positive terminal to negative
terminal, and the motion of the electrons is biased opposite to the electric field, with the result that an ordered

motion with a certain average velocity u opposite to the direction of electric field is superimposed on the chaotic
motion of the electrons. Thus there is a net flow of negative charge opposite to the electric field, or equivalently
flow of net positive charge in the direction of electric field. Thus an electric current flows through the conductor in
the direction of electric field.
If charge dq passes through an imaginary surface in time dt, then the current I through that surface is defined
dq
as I = (definition of current). The direction of the current is the direction of flow of positive charge carriers, or
dt
opposite to the flow of electrons.
Also, we can write dq = i dt. The charge that passes though the surface in a time interval extending from 0 to t is
t
given as:=q ∫=
dq ∫ idt
0
(the current i in general varies with time).
The SI unit for current is coulomb per second or the ampere (A), which is an SI base unit:
1 ampere = 1 A = 1 coulomb per second = 1 Cs-1.
2 0 . 2 | Current Electricity

3. CURRENT DENSITY
In general the electric current is distributed non-uniformly over the surface through which it passes. So to analyse
the current through an elementary surface of infinitesimal area at any point inside the conducting medium, we

introduce a current density vector j .The magnitude of current density vector at any point P, is equal to the ratio
of current dI through an elementary surface perpendicular to the direction of current at P to the area dS ⊥ of this

elementary surface. The direction of j is the same as the notion of dI at that point, or the direction of velocity

vector u of the ordered motion of positive charge carriers.
∆I
If ∆I be the current through the area ∆S ⊥ ,the magnitude of average current density is j = .
∆S ⊥
dI
The magnitude of current density at the point P is j = .
dS ⊥

S cos

Q 
P


Q S n
=i S
t
Figure 20.1: Current and current density

If the area dS is not perpendicular to the current dI through it, i.e. the normal to the area makes some angle θ with
the notion of the current, then the current density is given as,
dI
j= or,
= dI jdS cos θ
dS cos θ
  
If dS be the area vector corresponding to the area dS, we have dI = j.dS
 
For a finite area, I = ∫ j.dS

An electric current is not a vector quantity. It does not follow the laws of vector addition. The current density is a
vector quantity.

PLANCESS CONCEPTS

Direction of Current
•• Direction of drift of electrons is in the opposite direction of electric field in conducting wires.
•• It is not always along the length of the wire (direction of cross section). We take the component of the
velocity along the wire.
Yashwanth Sandupatla (JEE 2012, AIR 821)

4. DRIFT SPEED
A conductor contains a large number of loosely bound electrons called free electrons or conduction electrons.
These electrons move randomly in all directions within the entire volume of the conductor, (see Fig. 20.2) and in
this process keep on colliding with the atoms/molecules/ions of the conductor, changing their direction of motion
at each collision.
P hysi cs | 20.3

x y x y

(a) (b)

Figure 20.2: Random motion of electron inside conductor

When an electric field is applied inside the conductor, each electron experiences a force in the direction opposite
to the field. The chaotic motion of electrons gets biased in favour of this force. At each collision with a molecule,
the electron changes its direction of motion and moves with a random velocity but gains an additional velocity
eE
ve = τô in the direction opposite to the electric field till the next collision happens and the direction of its motion
m
again changes abruptly. As the average time τ between successive collisions is small, the electrons slowly and
steadily drift opposite to the applied field (see Fig. 20.3) with an average drift speed vd.
The distance drifted during successive collisions can be written as

= 1 a( τ)2 = 1  eE  ( τ)2
 
2 2 m 
 1  eE 
The drift speed will be given by the relation: vd= =  τ
τ 2 m 

E
A vd

Figure 20.3: Calculating drift speed

Current density can be expressed in terms of the drift speed. Consider a cylindrical conductor of cross-sectional
area A in which electric field E exists. Consider a length L= v d ∆t of the conductor. The volume of this portion is
Av d ∆t . If there are n free electrons per unit volume of the conductor, the number of free electrons in this portion
are nA v d Δt. All these electrons cross the area A in time Δt. Thus, the charge crossing this cross-section in time Δt is
∆Q
Q neAv d ∆t or current through the conductor is,=
∆= I = neAv d
∆t
I
Therefore current density is: =j = nev d
A

Illustration 1: If n =8.5 × 1028 m–3,how long does an electron take to drift from one end of 3 m long wire to its other
end? The area of cross section of the wire is 2.0 × 10-6 m2and it is carrying a current of 3.0 Ampere. (JEE MAIN)

Sol: For constant p.d. across conductor the electrons drifts with constant drift velocity across it. If we find the drift
velocity, the time to drift across wire of constant length is easily found out
Given that: (i) Number density n = 8.5 × 1028 m–3 (ii) Cross-sectional area A = 2.0 × 10-6m2
(iii) Current I = 3 A (iv) Charge on electron e = 1.6 × 10–19 C
Current in terms of drift speed is expressed as I = neAvd
I
⇒ vd = .
neA
Now time taken to cross the length  of the wire is:
2 0 . 4 | Current Electricity

 neA 8.5 × 1028 × 1.6 × 10−19 × 2.0 × 10−6 × 3


=
t = =
vd I 3

t = 2.7 × 104 s.

5. RESISTANCE AND RESISTIVITY


When same potential difference is applied across the ends of two conductors of same shape and size but made of
different materials, different currents flow through them. The property of the conductor that makes the difference
here is its electrical resistance. The resistance between any two points of a conductor is the ratio of applied potential
difference V across those points to the current I that flows through the conductor. Thus resistance R is given as:
V
R =
I
The SI unit for resistance is the volt per ampere. This unit occurs so often that we give it a special name, the ohm
(symbol Ω); that is,
1 ohm = 1 Ω = 1 volt per amp = 1 V/A.
E
The resistivity ρ of the material is defined as: ρñ=
=
j
V /m V  
Unit of resistivity
= = m = Ω .m. In vector form we can write: E = ρ J .
A /m2
A
Another property of material is conductivity σ . It is the reciprocal its resistivity, so
1
σ=
ρ
The SI unit conductivity is the reciprocal of Ω -m or mho-m-1 is sometimes used. (mho is ohm backwards).

5.1 Calculating Resistance from Resistivity


Resistance is a property of a conductor. Resistivity is the property of a material. If we know the resistivity of a
material such as copper, we can calculate the resistance of a conductor made of that material. Let A be the cross-
L
sectional area of a wire and L be its length, then resistance is R = ρ
A

Illustration 2: If a wire is stretched to ‘n’ times its original length how does the R change? On stretching, volume
remains constant. (JEE MAIN)

Sol: For any deformation in wire the resistance varies inversely w.r.t the area of cross-section of wire, but directly
w.r.t. the length of the conductor at the given instant of time.
As the volume remains constant, AL = A’L’ ⇒ AL = A’nL (Given L’ = nL)
A' 1 L'
⇒ =and = n
A n L
L R' L' A
Now R ∝ ⇒ = × = n × n = n2 ⇒R’ = n2 R
A R A' L

Illustration 3: Resistance of a hollow cylinder  (JEE ADVANCED)


Consider a hollow cylinder of length L and inner radius a and outer radius b, as shown in Fig. 20.4. The material
has resistivity ρ.
P hysi cs | 20.5

Figure 20.4

(a) Suppose a potential difference is applied between the ends of the cylinder that produces a current flowing
parallel to the axis. What is the resistance measured?
(b) If instead the potential difference is applied between the inner and outer surfaces so that current flows radially
outward, what is the resistance measured?

ρL
Sol: The resistance of the conductor is calculated as R = where A is the area of conductor perpendicular to the
A
direction of current. The larger the area, the lower the resistance offered to charge drifting across conductor.
(a) When a potential difference is applied between the ends of the cylinder, current flows parallel to the axis. In
this case, the cross-sectional area perpendicular to the current is A = π (b2 – a2), and the resistance is given by
ρL ρL
R= =
A π(b − a2 )
2

(b) When the current flows radially outwards, the area of cross-section perpendicular to the current will be the
area of cylindrical surface coaxial with the cylinder and its value will increase as the radius of the cylindrical
surface increases. Consider an elementary coaxial cylindrical shell of infinitesimal thickness having inner radius
ρdr
r and outer radius r + dr and length L. Its contribution to the resistance of the system is given by dR =
2πrL
where A = 2prL is the area normal to the direction of current flow. The total resistance of the system becomes
b
ρdr ρ b
=R ∫=
2πrL 2πL
log  
a
a

5.2 Colour Code for Carbon Resistors


The resistance value and percentage accuracy for carbon resistors are indicated by a colour code.
The resistor has a set of coloured bands on it. Each colour corresponds to a single digit number.
The first two bands indicate the first two significant digits of the resistance in ohms. The third band indicates the
decimal multiplier (10n) and the last band gives the tolerance or possible variation in the value of the resistance. If
the fourth band is absent, it implies that the tolerance is ± 20%.

Table 20.1: Resistor colour codes

Colour Number Multiplier Tolerance (%)


Black 0 1
Brown 1 101
Red 2 102
Orange 3 103
Yellow 4 104
Green 5 105
2 0 . 6 | Current Electricity

Blue 6 106
Violet 7 107
Grey 8 108
White 9 109
Gold 10–1 ± 5%
Silver 10–2 ± 20%
No colour ± 20%

Example

Blue Red Green Gold

62  105  5%

Figure 20.5: Calculating resistance by colour codes

Hint: An easy way to remember the sequence of colours is to memorize the following phrase: “B B ROY goes
Bombay via Gate Way”, the letters in capital (B, B, R, O, Y, G, B, V, G, W) are the first letters of the colours of the
table for Resistor colour code.
In the example shown in the Fig. 20.5, first colour is Blue and stands for numeric value 6, Red corresponds to 2,
Green corresponds to 105, Gold is ± 5%, so the value of resistance is (62 × 105 Ω) ± 5%

6. TEMPERATURE DEPENDENCE
Resistivity of a material changes with temperature. If α is the temperature coefficient of resistivity, then change in
resistivity with change in temperature is given as ρ – ρ0 =ρ0 α(T – T0 ) .
Here T0 is the initial temperature and r0 is the resistivity at that temperature. Usually T0 = 293 K (room temperature).
The sizes of degrees on Celsius or Kelvin scales are identical. We can use any of these scales in this equation as only
the temperature difference between two states is used here.

6.1 Thermistor
A thermistor is a type of resistor whose temperature coefficient of resistivity is quite large as compared to standard
resistor. A small change in temperature produces a large change in resistivity. In Negative Temperature Coefficient
(NTC) thermistors, resistance decreases with increase in temperature. In Positive Temperature Coefficient (PTC)
thermistors, resistance increases with increase in temperature. The thermistor is dipped in the bath whose
temperature is to be measured. The current is supplied by a battery. If the temperature increases, the current
changes because of the change in resistivity. Thus, by noting the change in the current, one can find the change in
temperature. A typical thermistor can easily measure a change in temperature of the order of 10–3 ºC.

Illustration 4: The resistance of a Platinum wire at ice point (0oC) =5 Ω and steam point (100oC) =5.23 Ω. When the
thermometer is inserted in a hot bath the resistance is 5.795 Ω. Find the temperature of hot bath?  (JEE MAIN)
P hysi cs | 20.7

Sol: The resistance of the wire varies linearly with change in temperature as RT =R0(1+∝ΔT).
Resistance of wire over temperature difference ∆T= 100oC is R100 = R0[1 + α(100-0)] ⇒ 5.23 = 5(1 + 100α)

0.23
⇒ α= For resistance of wire as 5.795 Ω the temperature of wire is found as
500

RT = 5[1 + α (T - 0)] = 5.795

 5.795  500
=
⇒t  − 1
 5  0.23
⇒ t = 345.65oC

6.2 Superconductor
Superconductor is a material which offers zero resistance to flow of electric current through it. Superconductivity is
a phenomena exhibited by certain materials wherein there resistivity drops abruptly to zero when they are cooled
below a certain temperature. This temperature is called the critical temperature for that material. Above the critical
temperature, the resistivity of the material has a non-zero value and increases with increase in temperature.
Resistivity

Resistivity

normal metal Super conductor

Temperature Tc Temperature

Figure 20.6: Variation of resistance with respect to temperature

7. OHM’S LAW
German scientist George Simon Ohm stated the following law known as Ohm’s law:
The current (I) flowing through a conductor is directly proportional to the potential difference (V) across its ends
provided the physical conditions (temperature, strain, etc.) do not change, i.e.
I ∝ V
V
or
= constant
= R
I
or V=IR
R is called the resistance of the given conductor. The quantity 1/R is called conductance.
Now for a wire of length L and area of cross-section A we have
L
V = I ρ
A
I V V
or = j= = σ
A Lρ L

or j = σE
2 0 . 8 | Current Electricity

Limitation of OHM’S Law


A material in which Ohm’s law is not valid is called non-ohmic material. The non-validity can be of following types:
(a) Voltage (V) is not proportional to current (I)
(b) The relation between ‘V’ and ‘I’ is not unique which means there is more than one value of V for the same
current E.g. Ga, As
(c) The relation between ‘V’ and ‘I’ depends on the sign of V. This mean, if ‘I’ is the current in a conducting
medium for a certain V, then on reversing the direction of V keeping its magnitude fixed, the current of the
same magnitude is not produced in the opposite direction. E.g. In a diode.

i
(mA)
i
-v v

v i
(A)
Figure 20.7: Variation of current with respect to voltage in non-ohmic devices

Illustration 5: Calculate the resistance of an aluminium wire of length 50cm and cross sectional area 2.0 mm2. The
ρ 2.6 × 10 −8 Ωm 
resistivity of aluminium is = (JEE MAIN)
ρL
Sol: The variation of resistance with length and area is shown as R =
A
L (2.6 × 10−8 Ωm) × (0.50m)
The resistance of the wire is R =ρ = =0.0065 Ω
A 2 × 10−6 m2

8. COMBINATION OF RESISTANCE

8.1 Series Combination


(a) Same current flows through each resistance (See Fig. 20.8).
R1 R2 R3

V1 V2 V3
V

Figure 20.8: Series combination of three resistances

(b) Voltage across each resistance is directly proportional to the value of resistance. V1=IR1, V2 =IR2
(c) Sum of the voltages across individual resistances is equal to the total voltage applied across the combination
i.e.
V = V1 + V2 + V3 + …………………..
V = IR1 + IR2 + IR3 + …………………
V
= R1 + R2 + R3 + …………… = R
I
where, R = equivalent resistance.
Note: If n resistances (each equal to R) are connected in series there resultant will be nR (See Fig. 20.9).
P hysi cs | 20.9

R1 R2

Figure 20.9: Series combination of two resistances

(d) For a series combination of two resistance


(i) Equivalent resistance R = R1 + R2
(ii) I = V / (R1 + R2)
R1 V
(iii) V1 (voltage across R1) = IR1 =
R1 + R 2
R2 V
(iv) V2 (voltage across R2) = IR2 =
R1 + R 2

8.2 Parallel Combination


(a) Same potential difference is applied across each resistance (See Fig. 20.10).
i1 R1

i2 R2
A B
i3

R3

V
Figure 20.10: Parallel combination of resistances

(b) Current in each resistance is inversely proportional to the value of resistance i.e.
V V V
=I1 = , I2 = , I3 etc.
R1 R2 R3

(c) Current flowing in the combination is the sum of the currents in individual resistances i.e. I=I1+I2+I3
V V V I 1 1 1 1
I= + + + ⇒ = = + + ……………..
R1 R 2 R 3 V R R1 R 2 R 3

where R = equivalent resistance.

Note:

(a) You are asked to find R and not in the question, so be careful.

(b) The equivalent resistance of parallel combination is less than the value of the lowest individual resistance in
the combination.
(c) For a parallel combination of two resistances

V(R1 + R 2 )
I = I 1 + I2 =
R1R 2
2 0 . 1 0 | Current Electricity

Note:
(a) If n resistance (each equal to R) are connected in parallel, their resultant will be R/n(See Fig. 20.11).

R1

i1
i2

R2

Figure 20.11: Parallel combination of two resistances

(b) If n resistance are connected in series and parallel respectively the ratio of their resultant resistances will be
(nR):(R/n) = n2.

8.3 Division of Current for Parallel Resistors


Consider the circuit shown in Fig. 20.12. Using ohm’s law on resistors R1 and R2,

i1 R1

a b
i2 R2

Figure 20.12: Current division

Va – Vb= I1R1 and Va – Vb = I2R2.

Thus, I1R1 = I2R2 or, I1 R1  …(i)


=
I2 R2
We see that the current is divided in resistors, connected in parallel, in inverse ratio of the resistances.
Also, we have I = I1 + I2  … (ii)
R2 R1
Solving (i) and (ii) we get I1 and I2 in terms of I: I1 = I and I2 = I
R1 +R 2 R1 +R 2

Illustration 6: (a) Three resistors 1 Ω , 2 Ω and 3 Ω are combined in series. What is the total resistance of the
combination?  (JEE MAIN)
(b) If the combination is connected to a battery of e.m.f. 12V and negligible internal resistance, obtain the potential
drop across each resistor.

Sol: (a) When resistances in any circuit when connected in series, the net resistance of the circuit increases, and
there is a constant potential drop across each resistance depending on amount of current passing through it.
(a) For series combination Req = R1 + R2 + R3, so we get Req = 1 + 2 + 3 = 6 Ω

V 12
(b) By Ohm’s Law current drawn from the battery,=I = = 2A
R eq 6

Again, by Ohm’s Law voltage drop across R1 is, V1 = IR1 = 2 × 1 = 2V


P hysi cs | 20.11

Voltage drop across R2 is, V2 = IR2 = 2 × 2 = 4V


Voltage drop across R3 is,V3 = IR3 = 2 × 3 = 6V.

Illustration 7: Given ‘n’ resistors each of resistance ‘R’. How will you combine them to get maximum and minimum
effective resistance? What is the ratio of maximum to minimum resistance? (JEE ADVANCED)

Sol: When resistances in any circuit when connected in series, the net resistance of the circuit increases while in
parallel connection the net resistance of the circuit decreases.
Equivalent resistance of n resistors each of resistance R,
(a) Connected in series is:
Rseries = Rmax = nR
1 1 n R
Connected in parallel is: = = ⇒ Rmin =
Rparallel Rmin R n

Rmax n
= nR × = n2
Rmin R

9. POWER
Consider a simple circuit wherein a resistance R is connected across the terminals of a battery. The potential
difference across the battery is V and a steady current I is drawn from it. The amount of charge dq that moves
between the terminals in time interval dt is equal to I dt. This charge dq moves through a decrease in potential of
magnitude V across the resistance, and thus its electric potential energy is decreased by an amount,
dU = dq V = dt V
By law of conservation of energy, this energy must be converted to some another form. In a resistance this “another
form” is nothing but heat energy. The temperature of the resistance rises as a result of heat generated in it when a
current is passed through it.
After crossing the resistance this charge again enters the battery at its negative terminal and emerges at the
positive terminal. In doing so, it moves through a rise in potential of magnitude V inside the battery, thus its electric
potential energy increases by the same amount dU = V I dt.
The battery thus performs work on the charge to increase its electric potential energy and this energy is dissipated
in the resistance in the form of heat.
The power P associated with this energy transfer is the rate of work done by the battery or the rate of production
of electrical energy, equal to the rate at which heat is dissipated:
dU
=
P = V I (Rate of electrical energy transfer).
dt

The unit of power that follows is the volt-ampere (V.A).

 J  C  J
We can write 1 V ⋅ A =  1  1  = 1 = 1 W .
 C  s  s

Also we can write voltage drop across resistance as V = I R

The rate of electrical energy dissipation in a resistance comes out to be


V2
P = I2 R or P = (resistive dissipation)
R
2 0 . 1 2 | Current Electricity

Caution: P = VI applies to electrical energy transfers of all kinds; P = I2R and P=V2/R apply only to the transfer of
electric potential energy to thermal energy in a resistance.

9.1 Joule’s Laws of Heating


When electric current flows through a resistor, the temperature of the resistor increases. If the potential difference
across the resistor is V and current I flows through it, the work done by the electric source in time t is
W = (potential difference) × (charge) = V (It) = (IR)(It) = I2Rt
This work is converted into thermal energy of the resistor or heat is produced in the resistor denoted by H. (actually,
this energy is not heat as it does not flow due to any temperature difference)

Illustration 8: Consider the following circuit, find power dissipated in the resistor R.
V2
Sol: Power dissipated in any electrical resistance is = = I2R
P IV = . Power supplied by the source is P = E I
R
R

i = 2A i

E = 12V.r=2
Figure 20.13

Total electrical power produced by the battery is


Pin = E I = 12 × 2 = 24W
The power dissipated in the internal resistance of battery is
I2r = 22 × 2 = 8W
The net power output from the battery Pout = 24 – 8 = 16W
The power dissipated in the resistor = 16 W

PLANCESS CONCEPTS

Power Method
•• Total power(power dissipated plus power generated) in a circuit is zero. This comes from principle of
conservation of energy.
•• In a circuit generally, battery supplies energy, and resistors always dissipate. But sometime battery
might also dissipate(in multi-source circuit)
•• Power is dissipated if current and voltage drop are in same direction of a given device and vice-versa.
If they are in opposite direction, power is supplied.
•• For a collection of devices, net power dissipated is product of voltage drop and current across the
terminals.
Vaibhav Gupta (JEE 2009, AIR 54)
P hysi cs | 20.13

9.2 Cell
A cell is an electric source, which converts chemical energy into electrical energy. A chemical solution called
electrolyte is filled inside the cell. Two metal rods called electrodes are immersed inside the electrolyte (see
Fig. 20.15). These electrodes exchange charges with the electrolyte. The electrode at the higher potential is called
positive electrode, and that at the lower potential is called negative electrode. When no current is drawn from the
cell, the potential difference between the positive and negative electrodes is called the electromotive force, emf (E).


Symbol of emf

Figure 20.14: Representation of cell

R -
i
+ -

Figure 20.15: Electrolytic cell

When an external resistance (R) is connected across the cell, a current I flows from positive to negative electrode
(terminal) through the external resistance. The same current flows though the cell from negative to positive terminal.
The resistance offered by the cell when the current flows through it is called the internal resistance (r) of the cell.
The internal resistance depends on the nature of the electrolyte, distance between the electrodes and the area of
the electrodes inside the electrolyte.
If current I flows through the cell, the potential drop across internal resistance will be I.r. Thus the potential difference
across the cell reduces by amount I.r from its value in open circuit, i.e. the emf E. Hence, the potential difference
across the cell when current I is drawn from it is
V = E – Ir
V is called the terminal potential difference.
E
The potential difference across external resistance R, is V = IR, thus, E -Ir = IR, or I =
R +r
(a) When current is drawn from a cell it is called the discharging of a cell. During discharging,
V = E – I r; V < E
(b) During charging of the cell, current enters the cell from positive terminal and emerges from negative terminal.
Energy is transferred to the cell by a DC source (see Fig. 20.16).

+ -
DC Charges
i
-

Figure 20.16: Charging of electrolytic cell

During charging V= E + I r; V > E


Note: When charging or discharging information is not given, take discharging action.
2 0 . 1 4 | Current Electricity

PLANCESS CONCEPTS

Battery with resistance


•• When current is drawn from a battery, the voltage across it is less than rated voltage.
•• We need to subtract the voltage drop due to internal resistance of the battery.
Nitin Chandrol (JEE 2012, AIR 1340)

Illustration 9: A storage battery of e.m.f.8.0V and internal resistance 0.5 Ω is being charged by a120V dc supply
using a resistor of 15.5 Ω. What is the terminal voltage of the battery during charging? What is the purpose of
having a series resistor in the charging circuit? (JEE ADVANCED)

Sol: The total potential difference in the circuit is the algebraic sum of voltage drop across the components in the
circuit. Using ohm’s law we get the potential difference across resistance.
The circuit is as shown in Fig. 20.17. During charging the total potential drop across the battery (including its
internal resistance) and the resistance connected in series will be equal to the voltage of the source.
V=E+Ir+IR

V −E 120 − 8 112
=I = = = 7 A Terminal voltage = 8 + 7 × 0.5 = 11.5 V
r + R 0.5 + 15.5 16
120V

1.5.5

8V 0.5

Figure 20.17

9.2.1 Combination of Cell in Series and Parallel

In series combination of cells (see Fig. 20.18) we have

....n cells
E1 r1 E2 r2 E3 r3 E4 r4

Figure 20.18: Series combination of cells

Enet = ∑ Ei and rnet = ∑ ri For parallel combination of two cells (see Fig. 20.19) we have

E2 r2
i2

i
i1
E1 r1

Figure 20.19: Parallel combination of cells


P hysi cs | 20.15

E1 − V E2 − V
VA − VB = V = E1 − i1r1 ⇒ i1 = Again, VA − VB = V = E2 − i2r2 ⇒ i2 =
r1 r2
E1 – V E 2 – V
i= i1 + i 2 ⇒ i= +
r1 r2
E E  1 1 
⇒ i =  1 + 2  −V + 
 r1 r2   r1 r2 

 1 1  E E 
⇒ V +  =  1 + 2  −i
 r1 r2   r1 r2 

 E1 E2 
 + 
 r1 r2  − i
⇒ V
=
1 1  1 1 
 +   + 
 r1 r2   r1 r2 
 Ei 
∑ 
ri  1 1
⇒ =
V Eeq − ireq where Eeq = and =∑
 1 req ri
∑ 
 ri 

Eeq is the equivalent emf and req is the equivalent internal resistance of the cell combination.
r
If all the cells are identical then for parallel combination, Net Emf = E; Net internal resistance is: req = for ’n’ cell
in parallel. n

Illustration 10: Two cells in series have emf 1.5V each and internal resistance 0.5 Wand 0.25 Ω respectively. They
are connected to external resistance R that is 2.25 Ω. Find the current in the circuit and potential difference across
the resultant cell. (JEE MAIN)

Sol: The total potential difference in the circuit is the algebraic sum of voltage drop across the components in the
circuit. Using ohm’s law we get the potential difference across resistance.

1.5V 1.5V

0.25 0.25

0.25

Figure 20.20

E = 1.5V, R = 2.25 W
r = 0.5 Ω, r’= 0.25 W
Total emf =1.5 + 1.5 = 3V ;
Total emf 3
Current I
= = = A 1 A ; V1 = E1 – Ir1 = 1.5 – 1 × 0.5 = 1 V
Total resis tance 0.5 + 0.25 + 2.25

V2 = E2– Ir2 = 1.5 – 1 × 0.25 = 1.25 V


Total potential difference across resultant cell is V =V1+V2 = 2.25 V
2 0 . 1 6 | Current Electricity

9.3 Maximum Power Transfer Theorem


For a cell of given emf and internal resistance, there is a certain value of external resistance R which when connected
across the cell will draw maximum power from it.
E
The current in the resistor R is I =
R +r
E2R
The power consumed in the resistance R is= 2
P I=R
(R + r )
2

dP dP E2 (R + r)2 − 2E2R(R + r)
For P to be maximum,
= = 0 or = 0
dR dR
( )
4
R+ r

E2 R E2r E2
Which gives R = r and=
Pmax = or=
Pmax
4r
(R + r ) (r + r )
2 2

Kilowatt-hour
It is commercial unit of electrical energy. It is known as 1 unit.
Thus, 1 KW – h = 1000 × 3600 J = 3.6 × 106 J

Total Power(W) x time(h) Total Power(W) x time(s)


Number of unit consumed = =
1000 3.6 × 106

10. KIRCHOFF’S LAWS

10.1 Kirchhoff’s First Law or Rule (The Junction Law or Kirchhoff’s Current Law)
This law is based on law of conservation of charge.
It states that the sum of all current smeeting at any point ( junction) must be equal to the sum of all current leaving
that point ( junction).
or
The algebraic sum of all the current meeting at a point ( junction) in a closed electrical circuit (for example see

Fig. 20.21) is zero. i.e. Ik = 0 … (i)

R2
R3
i3 i2
O
R4 i4 i5 i1 R1

R5

Figure 20.21

Here we need to choose the sign of each current Ik. Each term Ik is an algebraic quantity. The currents leaving a
junction and currents meeting at a junction are supposed to have opposite signs. For example if we assume former
to be positive then the latter has to be assumed negative and vice-versa(which one to be decided positive or
negative is immaterial). The sign convention once decided for a particular junction has to be followed for all the
currents at that junction.
P hysi cs | 20.17

Illustration 11: Consider a point or junction O in an electrical circuit. Let I1, I3 be the currents entering the point O
and I2, I4, I5 be the current leaving point O.

Sol: According to KCL, the algebraic sum of the current entering and leaving the junction is zero. So apply KCL to
the junction O.
According to Kirchhoff’s first law, I1 + I3 = I2 + I4 + I5 … (i)

Equation (i) can also be written as I1 + I3 + (–I2) + (–I4) + (–I5) = 0; I1 + I3 – I2 – I4 – I5 = 0 ( ∑ Ik = 0 )

Note: Kirchhoff’s First law is based on the law of conservation of charge i.e. on the fact that charge does not remain
accumulated at a junction or a point of a circuit.

10.2 Kirchhoff’s Second Law (The Loop Law or Kirchhoff’s Voltage Law)
This law is based on the law of conservation of energy.
The algebraic sum of all the potential differences ΔVk, along a closed loop in a circuit is zero.


Closed loop
∆Vk =
0

The rules for determining the sign of ΔV across a resistor and a battery are shown below:
Travel direction Travel direction
higher V I Lower V
I
Lower V higher V
a V=Vb-Va=-IR b a V=Vb-Va=+IR b

Travel direction Travel direction

Lower V  higher V higher V  Lower V


- + + -
a b a V=Vb-Va=- b
V=Vb-Va=+
Figure 20.22: Convention for determining ∆V

Note: The choice of direction of loop traversal is arbitrary. The same equation is obtained whether the closed loop
is traversed clockwise or anti-clockwise.

Illustration 12: A battery of 10V and negligible internal resistance is connected across the diagonally opposite
corners of a cubical network consisting of 12 resistors each of resistance 1 Ω . Find the equivalent resistance and
the current along each edge of the cube. (JEE MAIN)

Sol: Using KCL, the current distribution across each branch of the cube is as shown in Fig. 20.23. potential difference
across each branch is given by Ohm’s law. P.D. across diagonally opposite points equals equivalent resistance
multiplied by input current.

2i 6i

2i
i 2i

2i i i
1 2
2i
i
2i 3
6i

Figure 20.23
2 0 . 1 8 | Current Electricity

Current enters one end and leaves at diagonally opposite end.


Each branch has equal resistance, so by symmetry same current will flow through the three branches 1, 2 and 3.
If say 6i current enters the cubical network at one end and leaves the diagonally opposite end then the current
distribution in all the branches will be as shown in the adjacent Fig. 20.23.
Potential difference (P.D.) across the diagonally opposite ends taking any shortest path (any shortest path will have
three branches having currents 2i, i, and 2i)
2i × r +i × r+2ir=6iReq
5 5
5ir=6iReq; R e q= =
r Ω
6 6
Again, 6iReq = 10 V ⇒ i = 2 A

PLANCESS CONCEPTS

It is highly advised to solve numerous problems on Kirchhoff’s laws and ohm’s law to get hold over the
concepts and criticalities.
Chinmay S Purandare (JEE 2012, AIR 698)

11. WHEATSTONE BRIDGE


B i1-ig
Wheatstone bridge is an arrangement of four resistors in the form of a loop. The value
of one of the four resistors is unknown and is calculated in terms of the other three P Q
known resistors. ig
i1
Construction: It consists of four branches P, Q, R and S arranged to form a loop ABCD i2
(see Fig. 20.24). A source of e.m.f. E is connected across points A and C. A galvanometer G
is connected across points B and D. Unknown resistor is put in branch S. Out of the i
remaining three branches, one branch contains a variable resistance and the other two R S
K i1
branches have fixed resistors.
i i

Principle: When the battery is connected, the galvanometer shows the presence of
current Ig through it. The value of variable resistance R is adjusted in such a way that
the galvanometer shows no deflection. At this stage, the points B and D are at same Figure 20.24
potential and hence no current flows through the galvanometer. The Wheatstone
bridge at this stage is said to be balanced and ratio of P and Q is equal to ratio of R and S.

P R Q
i.e. = or S =   R Knowing P, Q and R, the value of S can be calculated.
Q S P

Illustration 13: Find the value of R in circuit (see Fig. 20.25) so that there is no current in the 50 Ω resistor.
 (JEE MAIN)
20
10
50

R 40

Figure 20.25
P hysi cs | 20.19

Sol: For Wheatstone network, when the bridge is in balanced condition the, ratio of resistance in the opposite
branches is equal.
Using Wheatstone bridge principle there will be no current in the 50 Ω resistor if the bridge is balanced.
10 Ω 20 Ω
= or R = 20 Ω
R 40 Ω

12. METRE BRIDGE OR SLIDE WIRE BRIDGE


P R
Metre Bridgeworks on the principle of Wheatstone bridge. i.e. =
Q S
Construction: Consider a circuit shown in Fig. 20.26. It consists of magnenin or + -
constantan wire AB of length 1 m (hence the name meter bridge). The wire is of a
uniform area of cross-section and is fixed on wooden board parallel to a metre  100-

scale. Let the resistance of the wire between A and J be P and that between J and A
J
B be Q. The resistance R between A and C is known and the resistance X between G
C and B is unknown. A galvanometer is connected between C and J. The jockey J
is slid on the wire AB till we get null point (zero reading of the galvanometer). If
length AJ is  then we have, R C X
P 
= Figure 20.26
Q 100 − 

By Wheatstone-bridge principle we get

P  R

= =
Q 100 −  X

 100 −  
or X= R
  
Thus, by knowing R and  we get the value of X.

13. POTENTIOMETER
A potentiometer is a device used to measure the potential difference across points in a circuit. It acts as an ideal
voltmeter, because it does not draw any current from the circuit element across which it is connected.
Construction: It consists of a long uniform mangen in or constantan wire AB fixed on a wooden board. The
schematic diagram of a potentiometer is shown in Fig. 20.27.

K
R B
1m

+ CS
100
A 1m
-

CS
1m

+ -
V
Figure 20.27: Potentiometer circuit
2 0 . 2 0 | Current Electricity

Actually, this long wire AB consist of four equal parts each 1 m long. These four parts of the wire are connected to
one another by thick copper strips (CS).
Principle: It works on the principle that potential difference across any part of a uniform wire is directly proportional
to the length of that part, when a constant current flows through the wire. According to Ohm’s law V = IR, but R ∝
 for uniform cross-section.
∴ V ∝ , provided I is constant.
We measure the length of the wire which has same potential difference as that of the circuit element. This length
is obtained with the help of a jockey connected to a galvanometer by ensuring zero deflection.

13.1 Comparison of E.M.F.S of Two Cells Using Potentiometer


(a) Comparison of EMF of Two Primary Cells. In the circuit shown, the cell of emf E1 is balanced for a length l1 on
the potentiometer wire. The cell of emf E2 is balanced for a length l2 on the potentiometer wire. Then we can
E 
write E1 ∝ 1 and E2 ∝  2 ∴ 1 = 1
. The emf of a cell can be determined by this method only when emf of
E2  2
another cell is known to us.

K Rh
+ -
()

P Q
E1
+ - A
C G
+ - B
E2

Figure 20.28: Deduction of cell e.m.f. using potentiometer circuit

Illustration 14: In a potentiometer, a cell of e.m.f 1.5V gives a balance point at 30cm length of the wire. Another
battery gives new balance point at 60 cm. Find e.m.f. of the battery.  (JEE MAIN)
Sol: For potentiometer circuit, E α . Thus we take the ratio of l1 to l2 to find E2.
E1 l1 l 60
From the formula = we get E2 = E1 × 2 = 1.5 × =3V
E2 l2 l1 30

13.2 Determination of Internal Resistance of a Cell Using Potentiometer


The arrangement for measuring the internal resistance r of a cell using potentiometer.
Procedure A
E + -
K
R₁

A B
J’ J

G
R
K₂
Figure 20.29: Determination of internal resistance of cell
P hysi cs | 20.21

Step 1: Key K1 is closed and key K2 is kept open. E.m.f. (E) of the cell is balanced for a length 1 on the potentiometer
wire. Therefore
E ∝ 1  .…(i)
Step 2: Both keys K1 and K2are closed. A known resistance (R) is connected across the cell. Potential difference
across the cell is balanced for a length  2 of the potentiometer wire. Therefore
V ∝  2  .…(ii)
Dividing (i) by (ii) we get,
E 1
=  ….(iii)
V 2

We know, internal resistance of a cell is given by,

E 
r =  – 1  R  …. (iv)
V 
Using (iii) in (iv), we get,

 
r =  1 – 1  R  …. (v)
 2 

Note: A potentiometer can be used to compare/determine unknown resistances and to calibrate ammeters and
voltmeter.

Illustration 15: A potentiometer with 1.5V cell is used for finding the internal resistance of a 1.2V cell. The balance
point corners at 65cm.If a resistor of 10 Ω is used as shown in the Fig. 20.30, the balance point changes to 55cm.
Calculate the internal resistance of the cell. (JEE MAIN)
1.5V

P Q

1.2V G
G

10
Figure 20.30

Sol: For potentiometer circuit, E ∝ 


or R ∝  Thus we take the ratio of resistance
Internal resistance of the cell can be calculated by the formula,

  – 2  65 – 55
=r  1 = R Ω 1.82 Ω .
× 10=
 2  55
2 0 . 2 2 | Current Electricity

13.3 Measurement of Unknown Small Resistance Using Potentiometer


The arrangement for measuring the unknown small resistance is shown in Fig. 20.31.

A
E - +
A

l₁ l₂
0 J 100
G
Rmk Rk
unknown known
resistance resistance

Figure 20.31: Potentiometer circuit to determine unknown resistance

Procedure: Close key K1. Find point Jon the wire AB of potentiometer such that, the galvanometer G gives no
l1 Ruk
deflection. Length AJ is l1 and length JB is l2. Using the principle of balanced Wheatstone bridge, we get, = ;
l2 Rk
l1
Ruk = Rk
l2

Precaution: Sliding of jockey should not be allowed. Itis not to be pressed roughly on the wire. It should be assured
that all connections are made with thick copper wires or strips. All connections should be tight. Key should be
opened after every reading to avoid unnecessary heating up of the wire.

Illustration 16: In a meter bridge the null point is found at 33.7 cm. If a resistance of 12 Ω is connected in parallel
with S, null point is 51.9 cm. Find R and S? (JEE MAIN)

Sol: For potentiometer circuit, E ∝ l or R ∝  . Thus we take the ratio of resistance


Case-I: Initial null point is  = 33.7cm
R  R 33.7 33.7
= = ; =
S 100 −  S 100 − 33.7 66.3

Case-II: New null point is  = 51.9 cm. Resistance S and 12 Ω are in parallel.

S × 12 R 51.9 R(S + 12) 51.9 33.7 (S + 12) 51.9


Seq = and = ;. = .;. =
S + 12 Seq 48.1 12S 48.1 66.3 12 48.1

0R RR 0 R0
S = 13.5 Ω and R = 6.86 Ω R1 + = +
2 2R + R 0 2

l
Illustration17: A resistance of R R = ρ   draws current form a potentiometer.The potentiometer has total
l A
resistance R0 R = ρ   . Drive an expression for the voltage across R when the sliding contact is in middle of the
A
potentiometer.  V (JEE MAIN)

R₀
A C
B
R

Figure 20.32
P hysi cs | 20.23

Sol: As the resistance R0 and R, are in parallel combination the equivalent resistance of circuit drops and using
ohm’s law we get the current through the resistance.
When the sliding contact is in the middle, only half of the resistance R0 is included between A and B. So, the total
RR 0
1 1 1 2 RR 0
resistance between A and B is R1, = + = ; R1 =
R1 R R 0 / 2 R 2R + R 0
R+ 0
2
0 R RR 0 R0
Total resistance (Req) between A and C, Req = R1 + = +
2 2R + R 0 2
V V 2V
Current drawn from supply is, i=
= =
Re q R 2R1 + R 0
R1 + 0
2
Potential drop across A-B will be

2V RR 0 2V RR 0 2VR
VR = iR1 = × = × =
2R1 + R 0 2R + R 0  RR 0  2R + R 0 4R + R 0
2   + R 0
 2R + R 0 

14. AMMETER AND VOLTMETER


14.1 Ammeter
Electric current in an electric circuit is measured by an Ammeter which is a modified form of a galvanometer. If a
galvanometer is directly used to measure the electric current flowing in the circuit, then two problems arise: (i) the
large resistance of the galvanometer will change the electric current in the circuit, and (ii) large amount of current
in the circuit may damage or burn the galvanometer. Thus, galvanometer cannot be directly used to measure the
current in the circuit. A device with practically zero or very low resistance is required for this purpose. Thus ammeter
should have low resistance.

Conversion of Galvanometer into Ammeter by using Shunt

RG
G
I A B
(I-I) r

Ammeter
Figure 20.33: Representation of Ammeter circuit

A galvanometer can be converted into an ammeter by connecting a low resistance called shunt, parallel to the
galvanometer.
Let RG and rs be the resistances of the galvanometer and the shunt respectively.
Let I be the maximum current to be measured by an ammeter in the circuit.
Let Ig be the maximum current that can flowt hrough the galvanometer corresponding to which galvanometer gives
the full scale deflection.
The resistance of the shunt must be such that the remaining current (I-Ig) should pass through the shunt.
Since RG and rs are in parallel, the potential difference across them is same.
 Ig 
i.e. IgRG = (I - Ig)rs or rs =  RG .
 I − Ig 
 
2 0 . 2 4 | Current Electricity

This is the required value of shunt resistance to convert a galvanometer into an ammeter of range 0 to I ampere.

Effective resistance of ammeter: The total effective resistance Reff of an ammeter is gives by

1 1 1 R +r R r
= + = G s or R eff = G s
R eff R G rs R Grs R G +rs

Since RG >> rs, so (RG+ rs) = RG

Hence R eff = rs
Thus, an ammeter is a low resistance device.

14.2 Voltmeter
Potential difference across a circuit element is measured by an instrument called voltmeter. The voltmeter is to be
connected in parallel to the circuit element across which potential difference is to be measured. The resistance of
the voltmeter has to be very large.

Conversion of galvanometer into voltmeter: A galvanometer is converted into a voltmeter by connecting a large
resistance in series with it.

Volmerter
R
A Rc B

Figure 20.34: Equivalent voltmeter circuit

Let RG and R be the resistances of the galvanometer and a resistor connected in series with it respectively. R is very
large as compared to RG.
Let V be the potential difference to be measured by the voltmeter.
Let Ig be the current for which the galvanometer gives the full scale deflection.
Potential difference across voltmeter is
IgR + IgRG= Ig(R + RG)
V=

V V
∴ R + R G = or R= − RG
Ig Ig
This is the value of resistance R which has to be connected in series to the galvanometer to convert it into a
voltmeter of range 0 to V volt.
Effective resistance of the voltmeter is given by Reff = (R + RG) which is very high.
Thus, voltmeter is a high resistance device.

Illustration 18: A galvanometer having a coil of resistance 12 Ω gives full scale deflection for a current of 4 mA.
How can it be converted into a voltmeter of range 0 to 24 V? (JEE MAIN)
Sol: When we connect a large resistance in series to the galvanometer, the current through it reduces. Thus such a
galvanometer can be connected across high potential difference.
P hysi cs | 20.25

Galvanometer is converted into voltmeter by connecting a large resistance say R in series to the coil. Value of R can
V 24
be calculated using the formula R = − RG = – 12 =5988 Ω
Ig 4 × 10 –3

15. POST OFFICE BOX

R1 B1 R2 C

1000 100 10 10 100 1000 B

R2 R1
ig

A G D
1 2 2 5 10 20 20 50 E
K1
X
5000 2000 2000 1000 500 200 200 100
D D

E K1
A’ K1 K2 B’

Figure 20.35: Post office box as equivalent Wheatstone bridge network

 R2 
Post office box is a form of Wheatstone bridge. The unknown resistance X is given as X = R  
 R1 
The arms having resistances R1 and R2 are called the ratio arms.
R2 1 1 1
The ratio can be adjusted to have values in multiples of 10, e.g. 1, 10, 100, 1000, , , and so on.
R1 10 100 1000
R2
The accuracy in the measured value of X depends on the selection of the ratio .
R1
R2
If is selected as 1, then the value of the unknown resistance is measured within accuracy of ± 1 Ω.
R1

R2 1
If is selected as , then the value of the unknown resistance is measured within accuracy of ± 0.1 Ω.
R1 10

R2 1
If is selected as , then the value of unknown resistance is measured within accuracy of ± 0.01 Ω and so on.
R1 100
2 0 . 2 6 | Current Electricity

Illustration 19: The value of an unknown resistance is obtained by using a post office box. Two consecutive
readings of R are observed as the galvanometer deflects in the opposite direction for three different value of R1.
These two values are recorded under the column I and II in the following observation table. (JEE MAIN)

S.No. R1 (Ω) R2 (Ω) R lies in between X=R(R2/ R1)

I Ω II Ω IΩ II Ω

1 10 10 16 17
2 100 10 163 164

3 1000 10 1638 1639

Determine the value of the unknown resistance.

Sol: The table listed above computes the value of unknown resistance for different values of ratio R2: R1. The
average of the final values will give the best estimate for the unknown resistance.
The observation table may be complete as follows:

S.No. R1 (Ω) R2 (Ω) R lies in between X=R(R2/ R1)

I Ω II Ω IΩ II Ω
1 10 10 16 17 16.0 17.0
2 100 10 163 164 16.3 16.4
3 1000 10 1638 1639 16.38 16.39

The value of the unknown resistance lies in-between 16.38 Ω and 16.39 Ω
16.38 + 16.39
The unknown value may be the average of the two=
i.e. X = 16.385 Ω .
2

PROBLEM-SOLVING TACTICS

(a) Never think of current in terms of electrons and get confused in direction or vector analysis. Instead, it would
be easier to think of current in terms of flow of positive charges because it is equivalent. However, my advice
is to remember the reality in questions which intentionally deals with this concept.
(b) While solving a problem always remember that it is the resistivity and conductivity that is same for materials
and not the resistance and conductance itself. This will help you avoid silly mistakes. Also note that Temperature
dependence formula is very analogous to length expansion formula.
(c) The problems related to devices are quite easy and always remember that they are always solved by ratios and
proportions. And one good thing is they do not require much of calculation too.
(d) Questions on combination of resistances can be solved more easily by identifying symmetry in breaking down
the problem to basic series and parallel circuit.Symmetry, in this context implies equal resistances or equal
distribution of current. This can help you in solving big complicated looking circuits.
(e) Questions related to power are easy and require use of law of conservation of energy.
P hysi cs | 20.27

(f) Applying Kirchhoff’s Rules: Kirchhoff’s rules can be used to analyse multi-loop circuit. The steps are
summarized below:
(i) Draw a circuit diagram, and label all the quantities, both known and unknown. The number of unknown
quantities must be equal to the number of linearly independent equations we obtain.
(ii) Assign a direction to the current in each branch of the circuit. (If the actual direction of current is opposite
to that assumed initially, the value of current obtained will be a negative number.)
(iii) Apply the junction rule to all but one of the junctions. (Applying the junction rule to the last junction will
not yield any independent equation.)
(iv) Apply the loop rule to the loops in the circuit.
Obtain as many independent equations using both the Kirchhoff’s Laws as there are number of unknowns.

FORMULAE SHEET

 1. Ohm’s Law V = IR


 
 2. Current flowing through cross-section area of conductor is I = J.dA ∫
q nALe
 3. The current is defined as =
I = = nAev d
t L / vd

E
 4. Current in terms of resistivity is defined as I = A
ρ
I
 5. Drift velocity for charge is v d =
ne A
neA
 6. Time taken by charge to drift across conductor is t =
I

 7. The current density of a conductor is J = (ne) v d

 8. Conductivity σ is related to current density as J = σ E


 

vd
 9. Mobility of charges ì =
E
E 1
 10. Resistivity of conductor is ρ= =
J σ
n
 11. For Series Network of resistance , net resistance R eq = ∑ Ri
i=1

 12. Voltage division in two resistances (see Fig. 20.36) I V1 V2

R1 R2
R1 R2
V = V1 +=
V2 where V1 = V and V2 V
R1 + R 2 R1 + R 2 
V
n 
1 1
For Parallel Network of resistance, net resistance is = ∑   Figure 20.36
R eq i=1  Ri 
2 0 . 2 8 | Current Electricity

 13. Current division in two resistances (see Fig. 20.37)

R2 R1

= I = I1 + I2, I = I and I2 I
1
R1 + R 2 R1 + R 2

I1 R1

I2 R2

Figure 20.37

 14. Kirchhoff’s Voltage Law: For closed loop ∑ V i= 0


 15. Kirchhoff’s Current Law: At junction ∑ I i= 0
∑Ei
 16. For circuit of series combination of cell, I = i where Ei and ri are values for ith cell.
∑ ri + R
i
E
∑ ri 1
 17. For circuit of parallel combination of cell, Eeq= i
and R eq= R +
1 1
∑r ∑r
i i

 18. Relation of resistance to the resistivity of conductor is R = ρ
A

 19. Temperature dependence of resistivity is expressed as ρ − ρ0 = ρ0 α (T − T0 )


V2
 20. Power consumed in any electrical device is P= IV and, resistive power dissipation=
P I2=
R
R
q
 21. For DC circuit of Resistance and Capacitance, (i) applied voltage E= + IR ,
C

(ii) Current in the circuit I = Ioe–t/τ where τC = ReqC is the time constant of the circuit

(iii) Charge stored by capacitor is q = Q (1 – e–t/t)

 22. For potentiometer E ∝  ⇒ R ∝ 


R1 R3
 23. For Wheatstone bridge network of resistance =
R2 R4
R 1 1
 24. For meter bridge, the ratio of resistance =
1
=
R2  2 100 − 1

E   
 25. Internal resistance of cell is given by r = − 1  R = 1 − 1  R
V   2 
P hysi cs | 20.29

Solved Examples

JEE Main/Boards Example 2: A potential difference of 200 volt is


maintained across a conductor of resistance 100ohm.
Example 1: 3 A current is flowing through two identical Calculate the number of electrons flowing through it in
conductors having diameter equal to 0.2 cm. These one second. Charge on electron, e = 1.6 × 10–19C
conductors are then split into three identical conductors
Sol: Number of electrons flowing per sec, through
each having 0.1cm diameter (see figure below) Calculate
conductor is the ratio of total charge flowing to the
the drift velocities in the thicker and the thinner
charge ofan electron.
conductors.
Here V = 200 volt; R = 100 ohm;

3A e = 1.6 × 10–19C
0.2 cm 0.1 cm V 200
3A Now =
I = = 2 ampere
R 100
The electron density = 7× 1028 m-3. All the conductors
are made of the same material. The electric charge on Charge flowing in t = 1 sec, q =I t = 2 × 1 = 2C
electron is equal to = 1.6 × 10–19C
Therefore, number of Ω electrons flowing through the
conductor flowing in 1 s,
Sol: At the junction the algebraic sum of currents is
zero. The drift velocity of charges is the ratio of current q 2
n= = = 1.25 × 1019
density to the charge density. e 1.6 × 10 −19

The current density in the thicker wire


I 3 3 Example 3: A negligibly small current is passed through
= = =
A πr 2
π(0.1 × 10−2 )2 a wire of length 15 m and uniform cross-section

Current density J = nevd


5 × 6 × 10−7
6.0 × 10 m2 ρ =
–7 = 2 × 107 Ωm
15
J L
∴ Vd = and its resistance is measured to be R = ρ .
ne A
3 What is the resistivity of the material at the temperature
=
π(0.1 × 10 ) × 7 × 1028 × 1.6 × 10−19
−2 2
of the experiment?

∴ V= 8.5 × 105 ms−1 Sol: The resistance of the conductor is calculated as


d
ρL
6A total current is flowing through the three identical R= .
A
conductors (As per Kirchhoff’s First Law).
L R×a
The current flowing through each of the wires =2A Using R = ρ , we get . ρ = .

a L
J' 5 × 6 × 10−7
∴ Vd ' = i.e. ρ = = 2 × 107 Ωm
ne 15
2
=
 0.1 
2
Example 4: Determine the current drawn from 12V
π × 10−2  × 7 × 1028 × 1.6 × 10−19 supply with internal resistance 0.5 Ω by the infinite
 2 
network shown in figure. Each resistance has 1 Ω
resistance.
= 2.3 × 10−4 ms−1
Sol: The equivalent resistance of the circuit is deduced
by reducing it to simple network. Using Ohm’s law we
2 0 . 3 0 | Current Electricity

get the current through network. Example 6: Calculate the current flowing through the
resistor R1 in the given circuit. R1 = 10Ω, R2 = 20Ω and
Let the total resistance of the circuit be Z and a set of
R3 = 30Ω The potentials of the points 1, 2 and 3 are
three resistor of
respectively, V1 = 10v. Calculate V2 = 6v and V3 = 5v.
R Z Calculate the potential at the junction.

Sol: Use the KVL and KCL at junction to get current


R
Z
 followed by application of Ohm’s law to get value of
R R
resistance.
R R O the junction point in the above circuit (see figure). The
potential at point 1, 2 and 3 is higher than the potential
value R each be connected to it as shown in the figure, existing at points 2 and 3. Hence, the direction of the
adding of these resistors will not change the value of Z current is from point 1to O, from O to 2 and from O
because the network is infinite then to point 3. The figure indicates the electric current and
their direction.
ZR ZR
Z =R+ + R = 2R + Z
Z +R Z +R Z= 2 +
l+ Z
Z 2
Or Z = 2 + i.e. Z 2 − 2Z − 2 = 0 V₂=20V
l+ Z
R₂=10
or Z =1 ± 3 or Z =1± 3 =2.73Ω 1
O
V₁=10V R₁=10
(value of Z cannot be negative). R₃=10
E 12 V₃=5V
Current drawn,
= I = = 3.71A 3
Z + r 2.73
Now for the 1O2 path, we have,
V1 - IR1 – l2R2 = V2
Example 5: A wire carries a current of 1.2 A, when a
potential difference of 1.8 V is applied across it. What ∴ 10 − 10I − 20I2 =
6
is its conductance? If the wire is of length 3 m and area ∴ 10I − 20I2 =
4 ….(i)
of cross-section 5.4 × 10–6 m2, calculate its conductivity. 
2
Sol: Resistivity is reciprocal of conductivity.
Here V = 1.8V1 = 1.2 A; R₂
l = 3 m and A = 5.4 × 10 m The resistance of wire.
–6 2
I
O
R₁
V 1.8 I-I₂
R= = = 1.5 Ω
1 1.2
Therefore conductance of wire. 3
1 1 For the 1O3 path, we have
G= = = 0.67Ω −1
R 1.5 10I + 30(I – I2) = 5

1 RA ∴ 40I − 30I2 = 5 ….(ii)


Now, R = ρ or ρ = 
A l Solving equation (i) and (ii), and we have
I I = 0.2 A
Also, conductivity of wire, σ =
ρ
Let V0 be the potential at point O, then
1 3
Or σ
= = = 3.7 × 105 Ω −1 m−1 10 – V0 = IR1

RA 1.5 × 5.4 × 10 6
∴ 10 – V0 = 2
∴ V0 = 8V
P hysi cs | 20.31

Example 7: Four identical cells, each of emf 2V, are The positive terminals of the cells E1 and E2 are connected
joined in parallel providing supply of current to external to the wire AE of resistance 10Ω and negative terminals
circuit consisting of two 15 Ω resistors joined in parallel. to the wire BD of resistance 4Ω. The resistance of 10Ω
The terminal voltage of the cells as read by an ideal is connected between the middle points F and C of the
voltmeter is 1.6 V. Calculate the internal resistance of wires AE and BD respectively. (See figure)
each cell.
I1 E1 = 2V r1 = 1
Sol: For parallel connection of n identical cells, the net A B
emf is equal to that of one cell and the net internal
resistance is the parallel combination of resistances of R 1 = 5
I1+I1 I1
r
all the cells req = . C1
n
Four cells are connected in parallel to the parallel R 1 = 5
combination of two 15 Ω resistors as shown in figure. E D
I2 r2 = 2
E r E2 = 1V

E r
r 10 4
∴ R1 =R 2 = =Ω5 ; R3 =R 4 = =Ω
2
I 2 2
15
The distribution of current in various branches is shown
in the figure.
15 In closed part ABCFA of the circuit
Let r be internal resistance of each cell and I be the
I1 × r1 + I1 × R 3 + (I1 × I2 )R + I1 × R 3 =
Ea
current I in the circuit. Since the cells are connected in
parallel. or I1 × 1 + I1 × 5 + (I1 × I2 ) × 10 + I1 × 2 =2
Total e.m.f in the circuit – e.m.f. one cell -2V
or 9I1 + 5I2 =
1 .…(i)

Further, total internal resistance of the cells is gives by
In closed part CDRFC of the circuit:
1 1 1 1 1 4
r= + + + = or r’ = r/4 I2 × r2 + I2 × R 2 + (I1 × I2 )R + I2 × R 4 =
E2
r' r r r r ' r4
or I2 × 2 + I2 × 5 + (I1 × I2 ) × 10 + I1 × 2 =1
Let R be resistance of the parallel combination of two
15 Ω resistors. Then, total external resistance,
10l1 + 9l2 = 1  ....(ii)
15 × 15
=R .75Ω E 
15 + 15 =
r'  − 1  R
Now, internal resistance of the parallel combination of V 
cells is given by Solving equation (i) and (ii) we have
E  r  2  14 1
r' =  − 1  R or =  − 1  × 7.5 or=
r 7.5 Ω I2 = A and I2 = A
V  4  1.6  121 121

Therefore, current thought resistance R,


Example 8: Two cell of e.m.f. 2V and 1 V and of internal 4  1  13
resistance 1Ω and 2Ω respectively have their positive I1 + I=
2
+ − =  A
121  121  121
terminals connected by a wire of 10Ω resistance and their
negative terminals by a wire of 4Ω resistance. Another coil Potential difference across the resistance R
of 10 Ω is connected between the middle points of these
wires. Find the potential difference across the 10Ω coil. 13
(I1 + I2 )R = × 10 = 1.0744 V
121
Sol: Use KVL and KCL and solve to get the current
through the resistances and total current through the
circuit. Use Ohm’s law to get the equivalent P.D. across R.
2 0 . 3 2 | Current Electricity

Example 9: (a) In a meter bridge (See figure), the 4 12


balance point is found to be at 39.5 cm from the end A,
B C D
when the resistor Y is of 12.5Ω. Determine the resistance A
of X Why are the connection between resistors in a
Wheatstone or Meter Bridge made of thick copper 4 6
strips? 16 V 1
X Y

B Sol: Compute Requ first than compute the current


through network. Then use Ohm’s law to get P.D. across
G
each arm.
(a) The network is a simple series and parallel
A C
D combination of resistors. First the two 4Ω resistors in
 
parallel are equivalent to a resistor =  4 × 4  Ω = 2Ω
4 + 4
In the way, the 12Ω and 6Ω resistor in parallel are
(b) Determine the balance point of the bridge above if  12 × 6 
equivalent to a resistor of   Ω = 4Ω .
X and Y interchanged.  12 + 6 
(c) What happens if the galvanometer and cell are The equivalent resistance R of the network is obtained
interchanged at the balance point of the bridge? by combining these resistors (2Ω and 4Ω) with 1Ω in
series, that is, R = 2Ω + 4Ω + 1Ω = 7Ω.
Would the galvanometer show any current?
(b) The total current I in the circuit is
Sol: For potentiometer circuit, E ∝  ⇒ R ∝  . Thus we
E 16V
take the ratio of resistances. = I = = 2A
R + r (7 + 1)Ω
1
(a) (i) Using X − R , we get
100 − I Consider the resistors between A and B. If I1 is the
current in one of the 4Ω resistors and I2 the current in
12.5(39.5) 12.5(39.5) the other, I1 × 4 = I2 × 4
=X = = 8.2Ω
(100 − 39.5) 6 That is l = l , which is otherwise obvious form the
1 2
symmetry of two arms.
(ii) Thick copper strips are used to reduce their resistance
because this resistance is not But I1 + I2 = I = 2A. Thus I1 = I2 = 1A

Accounted for in the calculations. That is current in each 4Ω resistor 1 A. Current in 1Ω


resistor between Band C would be 2 A.
(b) Interchanging R and X we get
Now consider the resistance between C and D. If 13 is
1 the current in the 12Ω resistor, and l4 in the 6Ω resistor,
R X = i.e. 100R − RI X I
100 − I
I3 × 12 = I 4 × 6, i.e., I 4 = 2b
R100 12.5 × 100
i.e. I =
= i.e. I But, I3 + I 4 − I =2A
R+X 12.5 + 8.2
4
2  4 
(c) In this case also the galvanometer will not show Thus, I3 =
= A  , I 4  A  A]
current. 3   3 3
l
That is the current in the 12 R = X resistor is (2/3)
Example 10: A network of resistors is connected to a 100 − l
16 V battery with internal resistance of 1Ω, as shown in R100
figure. A, while the current in the 6 l = resistor (4/3)A,
R+X
(a) Compute the equivalent resistance of the network.
(c) The voltage drops across AB is
(b) Obtain the current each resistor.
VAB = I1 × 4= 1A × 4Ω= 4V
(c) Obtain the voltage drops VAB, VBC, VCD,
P hysi cs | 20.33

This can also be obtained by multiplying the total 10(I1 + Ig ) − 15Ig − 5(I1 − Ig ) =
0
current between A and B by equivalent resistance
between A and B that is, 10I1 + 30Ig 5I2 =5 ; 20I1 + 6Ig =I2 =0
VAB= 2A × 2Ω= 4V (b) Considering the mesh ADCEA,
The voltage drop across BC is.
60I2 + 5(I1 − Ig ) =
10
VBC= 12Ω× 1Ω= 2V
65I=
2 + 5Ig 10 ; 13I=
2 + Ig 2
Finally, the voltage drop across CD is
(c) Multiplying Eq. (b) by 10
2
VCD= 12Ω × I3= 12Ω ×   A= 8V 20I1 + 60 Ig – 10I2 = 0
3
(d) Form Eqs. (d) and (a) we have
This can alternatively be obtained by multiplying total
current between C and D by the equivalent resistance I2 = 31.5Ig
between C and D that is, VCD= 2A × 4Ω= 8V
Substituting the value of l2 into Eq. (c), we get
Note that the total voltage drop across AD is
4V + 2V + 8V =
14V 2 ; 410.5 +=
13(31.5Ig ) + Ig = Ig 2 ; =
Ig 4.87mA

Thus the terminal voltage of battery is 14V, white its


emf is 16V. The loss of the voltage (=2V) is accounted Example 12: In a meter bridge (See figure), the null
for the resistance 1 Ω of the battery [2 A× 1Ω =2 V] . point is found at distance of 33.7cm from A. If now a
resistance of 12 Ω connected in parallel with S, the null
point occurs at 51.9cm. Determine the values of Rand S.
Example 11: The four arms of Wheatstone bridge (See
figure) have the following resistance: Sol: for meter bridge E ∝  ⇒ R ∝ 
AB =10Ω, BC =10Ω, CD =5Ω and DA =Ω From the first balance point, we get
A galvanometer of 15 R = 2Ω + 4Ω + 1Ω = 7Ω resistance R 3
is connected across BD. Calculate the current through = ….(i)
S 66.3 
the galvanometer when a potential difference of 10 V is R S
maintained across AC.
B
Sol: Use KVL for current distribution across each
component as shown in circuit (see figure) and solve to
G
get current through galvanometer.


I1 100I1
Considering the mesh BADB, we have A
Metre scale
C

100I1 + 15I, −60 I2 =


0
or 20I1 + 3I2 – 12I2 =0 ( )
K1
(a) I1-I1
E

100 10 After S is connected in parallel with a resistance of


I2
I1 12 Ω , the resistance across the gap changes form S to
12S
Seq where Seq =
I1 S + 12
10 I and hence the new balance condition now gives
60 I
I2-I1 51.9 R(S + 12) 33.7
I = . …..(ii)
48.1 12S 66.3 

Substituting the value of R/S form Eq. (i), we get


10V
Considering the mesh BCDB. We have 51.9 R(S + 12) 33.7
= .
48.1 12S 66.3
2 0 . 3 4 | Current Electricity

Which gives S = 13.5Ω. Using the value of R/S above, open. When both the keys are closed the balanced
we get R = 6.86Ω point is obtained at only 5 cm. Calculate.
(a) The emf of E1
Example 13: Calculate the potential difference between
(b) The internal resistance of E2
the plates A and B of the capacitor in adjacent circuit.
The resistance of the potentiometer wire RAB = 10Ω.
30 A B K1
IV
R2
20 10
A B
4V r G
F
Sol: Use KVL for current distribution across each
component and compute the p.d. across plates A and B. H M
R2 K1
The distribution of the current shown in figure. Applying
Kirchhoff’s second Law to the closed loop abedea Sol: For potentiometer circuit,
E ∝ . Use KVL and solve for the current in the circuit.
g
30 h A B
i
Use Ohm’s law to get internal resistance of cell E2.
(a) Resistance R1is connected into the circuit when both
the keys are open while R2 gets
c
e Disconnected
20 I1-I1 10 I
 ε1ρ 
=ε1 4V =
∴ ε2   I1
a b  R1 + R AB 
4V I
10
We have, ρ= = 20Ωm−4
0.5
= 10I = 20(I − I1 ) + 4 = 0 ; ∴ 30I − 20I1 = 4 .  4 + 20  −2
R1= 15Ω ∴ ε2=   31.25 × 10
For the dhge loop  5 + 10 
20(I − I1 ) + I =30I1 =0 ; ∴ 20I =50I1 =−1
R AB= 10Ω ∴ ε2= 1V
Solving equation (i) and (ii), we have,
=I1 0.1A
= and I 0.2 A. = = 31.25 × 10−2 m
I2 31.25cm
The pd between the two plates of the capacitor is equal (b) Resistance R1 gets short circuited when both of
to the pd between c and h point. Let 100l1 + 15l, – 6012 = the keys are closed, in other words resistance R1 is not
0 be the potential at point c and let 20l1 + lg – 212 = 0 be connected in the circuit while R2 gets connected.
the potential at h. For the path odh, we have Let V12 be the pd between point A and jockey which is
equal to the p.d. between points F and G of the circuit.
∴ Vc = 10 × 0.2 + 1 = Vb
∴ Vc =
Va ==
2 1+1 ε1 × I2 × ρ 4 × 5 × 10−2 × 20
=V12 =
∴ The potential difference between the two capacitor = 1 V R AB 10

4 10−2 V
∴ V12 =×
Example 14: A potentiometer circuit is shown in the
figure, the length of the potentiometer wire is equal to
Now, the pd. between point F and G = E2 – Ir
50 cm E1 = 4V. The internal resistance of the battery can
ignored. The values of the resistance R1 and R2 are 15 Ω Where 1 = current flowing through the battery E2. It can
and 5 Ω . The balanced point is obtained at a distance be seen from the diagram that current does flow from
of 31.25 cm from the end A, where both the keys are the circuit FHMG’F, even if the galvanometer shown
P hysi cs | 20.35

zero deflection which means that current flow the E2 The path AA’, AD and AB are obviously symmetrically
battery which results in a pd between point ‘F’ and ‘G’. placed in the network. Thus, the current in each must
be same say, I, Further, at the corners A’, B and D, the
E2 = Ir
incoming current I must split equally into the two
The pd should be equal to V12 using equation in the outgoing branches. In this manner, the current in all the
above result, we have, 12 edges of the cube are easily written down in terms
40 × 10 −2 =ε1 =Ir of I, using Kirchhoff’s a first rule and the symmetry in
the problem. Next take a closed loop, say, ABCC’EA,
The current flowing in the circuit FHMG’F’ is equal to. and apply Kirchhoff’s second rule:
ε1 = IR = (1/2)IR - IR + ε = 0
I=
R2 + r where R is the resistance of each edge and ε the emf of
ε2r  r  5
∴ 40 × 10−2 =ε2 − =ε2 1 −  battery. Thus, ε = IR
R2 + r  R 2 + r  2
The equivalent R eq of the network is
ε2 + r − r  R   5 
=ε2 − ∴ 40 × 10−2 ε2  2  =
= 1  ε 5
R2 + r  R 2 + r  5 + r  =
R eq = R
3I 6
5
∴ 5 +=
r = 12.5Ω For R = 1 Ω ,Req=(5/6)Ω and for ε = 10V , the
40 × 10 −2
total current (=3I) in the network is 3I = 10V/(5/6)Ω
∴ r = 12.5 − 5 = 7.5 Ω = 12A, i.e., I = 4A
The current flowing in each edge can now be read off
JEE Advanced/Boards from the figure.

Example 1: A battery of 10 V and negligible internal Example 2: A resistance of RΩ draws current from
resistance is connected across the diagonally opposite a potentiometer. The potentiometer has a total
corners of a cubical network consisting of 12 resistors resistance R0Ω (see figure). A voltage V is supplied to
each of resistance 1Ω (see figure below). Determine the the potentiometer. Derive an expression for the voltage
equivalent resistance of the network and the current across R When the sliding contact is in the middle of
along each edge of the cube. the potentiometer.
V
Sol: Use KVL and for p.d. across each component and
solve for current though each component.
The network is not reducible to a simple series and
parallel combinations of resistors. There is however, R
clear symmetry in the problem which we can exploit to A C
obtain the equivalent resistance of the network. B
3I
R
D’ C’ 6i
I/2
I/2 Sol: Use formulae for series and parallel combination of
A’ resistances. P.D. across each branch is given by Ohm’s law.
I While the slide is in the middle of the potentiometer
I/2
I/2 only half of its resistance (R0/2) will be between the
I points A and B. Hence, The total resistance between A
D C and B, say R1 will be given by the following expression:
I I/2
I/2
1 1 1 R 0R
A
B = + ; R1 =
3I 3 R1 R (R 0 / 2) R 0 + 2R
3I
2 0 . 3 6 | Current Electricity

The total resistance between A and C will be sum of Thus, the main current form the battery is
resistance between A and B and B and C, i.e. R1 + R0/2
100V
∴ The current flowing through the potentiometer will=i = 0.25A
be 400Ω

V 2V The potential drop across the 200 Ω resistance is,


=I =
R1 + R 0 / 2 2R1 + R 0 therefore, 200Ω × 0.25 A = 50 V and that across 300is
also 50 V. This is also the potential drop across the
voltmeter and hence the reading of voltmeter is 50 V.
The voltage V1 taken form the potentiometer will be the
product of current I and resistance R1,
Example 4: A galvanometer has a coil of resistance
 2V  100 Ω showing a full-scale deflection at 50 µA . What
= =
V1 IR 1   × R1 resistance should be added to use it as (a) a voltmeter
 2R1 + R 0  of range 50 V (b) an ammeter of range 10mA?
Substituting for R1, we have a
Sol: To convert galvanometer to voltmeter, connect a
2V R ×R large resistance in series with it. To convert galvanometer
V1 × 0
 R ×R  R 0 + 2R to ammeter, connect a small resistance in parallel with
2  0  + R 0 it. The current through the galvanometer should not
 R 0 + 2R  exceed its full scale current.

V1 =
2VR
or V1
2VR (a) When a potential difference of 50 V is applied across
2R + R 0 + 2R R 0 + 4R the voltmeter, full-scale deflection should take place.
Thus, 50 µA should go through the coil. We
Example 3: (a) Find the potential drops across the
two resistors shown in figure (a). (b) A voltmeter of 100
R 50 A 100
resistance 600Ω is used to measure the potential drop G G
10 A
across the 300Ω resistor (see figure (b)). What will be
the measured potential drop? r
50 V
600 
V add a resistance R in series with the given coil to achieve
300  200 
this (see figure (a)).
50V
300  200  We have, 50 µA =
100 V 100Ω + R

or =
R 106 Ω − 100Ω ≈ 106 Ω
100 V
(b) When a current of 10mA is passed through the
Sol: Find the equivalent resistance in both the circuit ammeter, 50µA should go through the coil. We add a
and then find the p.d. using Ohm’s law. resistance r in parallel to the coil to achieve this (see
100V figure (b)).
(a) The current in the circuit is = 0.2A
300Ω + 200Ω The current through the coil is
The potential drop across the 300Ω resistor is r
50 µA = (10mA) or r = 0.5Ω
300Ω × 0.2A = 60V r + 100Ω

Similarly, the drop across the 200Ω resistor is 40V.


(b) The equivalent resistance, when the voltmeter is
connected across 300Ω, is (see figure (b))

600Ω × 300Ω
=
R 200Ω + = 400Ω
600Ω + 300Ω
P hysi cs | 20.37

JEE Main/Boards

Exercise 1 if the room temperature is 27.0 o C ? Temperature


coefficient of resistance of nichrome averaged over the
Q.1 The storage battery of a car has an emf of 12V. temperature range involved is 1.70 × 10‒4 °C‒1.
If the internal resistance of the battery is 0.4 Ω, what
is the maximum current that can be drawn from the Q.9 Determine the current in each branch of the
battery? network shown in figure:

B
Q.2 A battery of emf 10V and internal resistance 3Ω
is connected to a resistor. If the current in the circuit 10 5
is 0.5A, what is the resistance of the resistor? What is
the terminal voltage of the battery when the circuit is
closed? A 5 D

Q.3 (a) There resistor 1 Ω, 2 Ω, and 3Ω are combined in 5


series. What is the total resistance of the combination? 10
D
(b) If the combination is connected to a battery of
10V
emf 12 V and negligible internal resistance, obtain the
potential drop across each resistor. 10

Q.4 (a) Three resistors 2Ω, 4Ω and 5Ω are combined in


parallel. What is the total resistance of the combination? Q.10 A storage battery of emf 8.0FV and internal
resistance 0.5Ω is being changed bya 120V DC supply
(b) If the combination is connected to battery of emf using a series resistor by a 15.5Ω. What is the terminal
20V and negligible internal resistance, determine the voltage of the battery during charging? What is the
current through each resistor, and the total current purpose of having a series resistor in the charging
drawn from the battery circuit?

Q.5 At room temperature (27.0 o C) the resistance of a Q.11 In potentiometer arrangement, a cell of emf 1.25V
heating element is 100Ω. What is the temperature of gives a balance point at 35.0cm length of the wire. If the
the element if the resistance is found a be 117Ω, given cell is replaced by another cell and the balance point
that the temperature coefficient of the material of the shifts to 63.0cm, what is the emf the second cell?
resistor is 1.70 × 10‒6 °C‒1.
Q.12 The number density of free electrons in a copper
Q.6 A negligibly small current is passed through a wire conductor as estimated is 8.5 × 1028m‒3. How long does
of length 15m and uniform cross–section 6.0 × 10‒7 m2 , an electron take to drift form one end of a wire 3.0m
and its resistance is measured to be 5.0Ω. What is the long to its other end? The area of cross- section of the
resistivity of the material oat the temperature of the wire is 2.0 × 106 m2 and it is carrying a current of 3.0A.
experiment?
Q.13 The earth’s surface has a negative surface charge
Q.7 A silver wire has a resistance of 2.1Ω at 27.5 o C , density of 10-9cm-2. The potential difference of 400kV
and a resistance of 2.7Ω at 100°C. Determine the between the top of the atmosphere and the surface
temperature coefficient of resistivity of silver. results (due to the low conductivity of the lower
atmosphere) in a current of only 1800A over the
Q.8 A heating element using nichrome connected to entire globe. If there were no mechanism of sustaining
a 230V supply drawn an initial current of 3.2A which atmosphere electric field, how much time (roughly)
settles after a few seconds to a steady value of 2.8A. would be required to neutralise the earth’s surface?
What is the steady temperature of the heating element (This never happens in practices because there is a
2 0 . 3 8 | Current Electricity

mechanism to replenish electric charges, namely the with internal resistance 0.5Ω by the infinite network
continual thunderstorms and lighting in different parts shown in figure. Each resistor has 1Ω resistance.
of the globe). (Radius of earth = 6.37 × 106 m)
1 1 1 1 1
Q.14 (a) Six lead-acid type of secondary cells each of 1 1 1
12V
emf 2.0V and internal resistance 0.015Ω are joined in 1.5 
series to provide a supply to a resistance of 8.5Ω. What 1 1 1 1 1
are the current drawn from the supply and its terminal
voltage? Q.19 Figure shows a potentiometer with a cell of 2.0V
(b) A secondary cell after long use has an emf of 1.9V and internal resistance 0.40Ω maintaining a potential
and a large internal resistance of 380Ω. What maximum drop across the resistor wire AB. A Standard cell which
current can be drawn from the cell? Could the cell drive maintains a constant emf of 1.02V (for every moderate
the starting motor of a car? up to a few mA) Give a balance point at 63.7cm length
of the wire. To ensure very low currents drawn from the
standard cell, a very high resistance of 600kΩ is put
Q.15 Two wire of equal length, one aluminium and the in series with it, which is shorted close to the balance
other of copper have the same resistance. Which of the point. The standard cell is then replaced by a cell of
two wire is lighter? Hence explain why aluminium wires unknown emf ε and the balance point found similarly,
are preferred for overhead power cables. turns out to be at 82.3cm length of the wire.
(ρA1 = 2.63 × 10‒8 Ωm, ρA2 = 1.72 × 10‒8 Ωm,
2V 0.4
Relative density of Al = 2.7, of Cu = 8.9)

Q.16 Answer the following questions: A B


(a) A steady current flow in a metallic conductor of non-
uniform cross-section. Which of these quantities is
constant along the conductor: current, current density, G
electric field drift speed? 600 k
(b) Is ohm’s law universally applicable for all conducting
(a) What is the values ε?
elements? If not, given examples of elements which do
not obey ohm’s law. (b) What purpose does the high resistance of 600 kΩ
have?
(c) A low voltage supply from which one needs high
currents must have very low internal resistance. Why? (c) Is the balance point affected by this high resistance?
(d) A high tension (HT) supply, of say 6kV, must have a (d) Is the balance point affected by the internal
very large internal resistance. Why? resistance of the driver cell?
(e) Would the method work in the above situation if
Q.17 Choose the correct alternative:
the driver cell of the potentiometer had an emf 1.0V
(a) Alloys of metals usually have (greater/less) resistivity instead of 2.0V?
than that of their constituent metals.
(f) Would the circuit work well for determining an
(b) Alloys usually have much (lower/higher) temperature extremely small emf, say of the order of a few mV (such
coefficients of resistance than pure metals. as the typical emf of a thermo-couple)? If not, how will
(c) The resistance of the alloy manganin (is nearly you modify the circuit?
independent of / increases rapidly with increases of)
temperature. Q.20 Figure shows a 2.0 V potentiometer used for the
determining of internal resistance of a 1.5 V cell.The
(d) The resistivity of a typical insulator (e.g., amber) is
balance point of the cell in open circuit is 76.3cm. When
greater than that of a metal by a factor of the order of
a resistor or 9.5 W is used in the external circuit of the
(1022\103).
cell, the balance point shifts to 64.8 cm length of the
potentiometer wire. Determine the internal resistance
Q.18 Determine the current drawn from a 12V supply of the cell.
P hysi cs | 20.39

2.0V Q.26 With two resistance R1 and R2 (> R1) in the two
gaps of a meter bridge, the balance point was found
1
to be m from the zero end. When a 6Ω resistance
3
is connected in series with the smaller of the two
2
A B resistance, the point is shifted to m from the same
3
1.5V
end. Calculate R1 and R2.
G
G
Q.27 A set of 4cells, each of emf 2 V and internal
9.5 resistance 1.05Ω, are connected across an external load
of 10Ω. with 2 rows, 2cells in each branch. Calculate the
Q.21 A galvanometer coil has a resistance of 12Ω and current in each branch and potential difference across
the meter shows full scale deflection for a current of 10Ω.
3mA. How will you convert the bridge into a voltmeter
of range 0 to 18 V? Q.28 In the circuit shown in figure E,F,G and H are cells
of emf 2,1,3 and 1 V respectively. The resistances 2,1,3
and 1Ω are their respective internal resistance.Calculate
Q.22 A galvanometer coil has resistance of 15Ω and
(a) the potential difference between B and D and (b) the
the meter shows full scale deflection for a current of
potential difference across the terminals of each of the
4mA. How will you convert the meter into an ammeter
cells G and H.
of range 0to 6 A?
2V 2 2V 2
B B
Q.23 A 10 m long wire of uniform cross-section of A
E
A
E
20Ω resistance is fitted in a potentiometer. This wire is 1V
F H
1V 1V
F H
1V
connected in series with a battery of 5volt, along with
an external resistance of 480Ω. If an unknown emf E is 1
2 1
1
2 1
balanced at 6.0 m length of this wire, calculate (i) the i1 i2
G G
potential gradient of the potentiometer wire, (ii) the D
3V
C D
3V
C
i2
value of the unknown emf E. 3 3

Q.24 (a) Three cells of emf 2.0V, 1.8V, and 1.5V are
connected in series. Their internal resistances are 0.05Ω, Exercise 2
0.7Ω and 1Ω, respectively. If the battery is connected
to an external resistor of 4Ω via a very low resistance Q.1 A current I flow through a uniform wire of diameter
ammeter, what would be the reading in the ammeter? d when the mean electron drift velocity is V. The same
current will flow through a wire of diameter d/2made
(b) If the three cells above were joined in parallel, of the same material if the mean drift velocity of the
would they be characterized by a definite and internal electron is:
resistance (independent of external circuit)?If not, how
will you obtain currents in different branches of the (A) V/4 (B) V/2 (C) 2V (D) 4V
circuit?
Q.2 A wire has a non- uniform cross- section as shown
Q.25 A galvanometer with a coil of resistance 12.0Ω in figure. A steady current flows through it. The drift
shows a full scale deflection for a current of 2.5 mA. speed of electrons at point p and q is Vp and VQ.
How will you convert the galvanometer into (a) an
ammeter of range 0 to 7.5A, (b) a voltmeter of range
0 to 10.0V. Determine the net resistance of the meter
in each case. When an ammeter is put in a circuit, does P Q
it read (slightly) less or more than the actual current in
the original circuit? When a voltmeter is put a cross a
part of circuit, does it read (slightly) less or more than (A) VP = VQ (B) VP < VQ

the original voltage drop? Explain.
(C)VP > VQ (D) Data insufficient

2 0 . 4 0 | Current Electricity

Q.3 A uniform copper wire carriers a current i amperes Q.7 Two batteries one of the emf 3 V, internal resistance
and has p carriers per cubic meter. The length of the wire 1 ohm and the other of emf 15 V, internal resistance
is l meters and its cross-section area is s meter2. If the 2 ohm are connected in series with a resistance R as
charge on a carrier is q coulombs, the drift velocity in shown. If the potential difference between a and b is
ms-1 is given by zero the resistance of R in ohm is
(A) i/lsp (B) i/psq (C) psq/i (D) i/pslq a b 1.5V,2

3V,1
Q.4 The current in a metallic conductor is plotted
against voltage at two different temperatures T1 and T2.
Which is correct
(A) 5 (B) 7 (C) 3 (D) 1
T1
Current

T2 Q.8 In the diagram resistance between any two


junctions is R. Equivalent resistance across terminals A
and B is
Voltage

(A) T1 > T2 (B) T1 < T2


(C) T1 = T2 (D) None of these

A B
Q.5 A battery consists of a variable number n of
identical cells having internal resistance connected in
11R 18R 7R 11R
series. The terminal of the battery are short circuited (A) (B) (C) (D)
and the current I measured. Which one of the graph 7 11 11 18
below shows the relationship between I and n?
Q.9 When electric bulbs of same power, but different
marked voltage are connected in series across the
power line, their brightness will be:
(A) I/A (B) I/A
(A) Proportional to their marked voltage
O n O
n (B) Inversely proportional to their marked voltage
(C) Proportional to the square of their marked voltage
(C) I/A (D) I/A (D) Inversely proportional to the square of their marked
voltage
n O n
O
Q.10 Two bulbs rated (25W – 220 V) and (100W – 220V)
are connected in series to a 440 V line. Which one is
Q.6 A circuit is comprised of eight identical batteries
likely to fuse?
and a resistor R = 0.8Ω. Each battery has an emf 1.0V
and internal resistance of 0.2Ω. The voltage difference (A) 25 W bulb (B) 100 W bulb
across any of the battery is
(C) Both bulbs (D) None

R=0.8 Q.11 A cell of emf E has an internal resistance r & is


connected to rheostat. When resistance R of rheostat
is changed correct graph of potential across it is (see
figure)
(A) 0.5 V (B) 1.0 V (C) 0 V (D) 2 V
V V
(A) (B)

R R

V V
V V
(A) (B)
P hysi cs | 20.41
R R

Q.17 By error, a student place moving-coil voltmeter


V V V (nearly ideal) in series with the resistance in a circuit
in order to read the current, as shown (see figure). The
(C) (D)
voltmeter reading will be
R R

E=12V, r=2
Q.12 The battery in the diagram is to be charged by the 4
generator G. The generator has a terminal voltage of V
120 volts when the charging current is 10 amperes. The
battery has an emf of 100 volts and an internal resistance (A) 0 (B) 4 V (C) 6 V (D) 12 V
of 1 ohm. In order to charge the battery at 10 amperes
charging current, the resistance R should be set at Q.18 Statement-I: Conductivity of a metallic conductor
20 decreases with increases in temperature.
Statement-II: On increasing temperature the number
i/2 of free electrons in the metallic conductor decreases.

i
G (A) Statement-I is true, statement-II is true and
i/2 Rg statement-II is correct explanation for statement-I.

(A) 0.1 Ω (B) 0.5 Ω (C) 1.0 Ω (D) 5.0 Ω (B) Statement-I is true, statement-II is true and
statement-II is NOT the correct explanation for
statement-I
Q.13 In a galvanometer, the deflection becomes one
half when the galvanometer is shunted by a 20Ω (C) Statement-I is true, statement-II is false.
resistor. The galvanometer resistance is (D) Statement-I is false, statement-II is true.
(A) 5 Ω (B) 10 Ω (C) 40 Ω (D) 20 Ω

Q.14 A galvanometer has a resistance of 20Ω and reads Previous Years’ Questions
full-scale when 0.2 V is applied across it. To convert it into
a 10 A ammeter, the galvanometer coil should have a Q.1 In the circuit shown in figure the heat produced the
5Ω resistor due to the current flowing through it is 10
(A) 0.01Ω resistor connected across it
cal/s. (1981)
(B) 0.02Ω resistor connected across it
4 6
(C) 200Ω resistor connected in series with it
(D) 2000Ω resistor connected in series with it

Q.15 A galvanometer coil has a resistance 90Ω and full 5


scale deflection current 10 mA. A 910Ω resistance is The heat generated in the 4Ω resistor is
connected in series with the galvanometer to make a
voltmeter. If the least count of the voltmeter is 0.1V, the (A) 1 cal/s (B) 2 cal/s
number of divisions on its scale is (C) 3 cal/s (D) 4 cal/s
(A) 90 (B) 91 (C) 100 (D) None
Q.2 The current i in the circuit (see figure) is
Q.16 In the circuit shown the resistance of voltmeter
is 10,000 ohm and that of ammeter is 20 ohm. The i
ammeter reading is 0.10 Amp and voltmeter reading is 2V


30
12 volt. Then R is equal to
30

R
A 30

1 1 1 1
V
(A) A (B) A (C) A (D) A
45 15 10 5
(A) 122 Ω (B) 140 Ω (C) 116 Ω (D) 100 Ω
2 0 . 4 2 | Current Electricity

Q.3 A battery of internal resistance 4Ω is connected (A) R1R2R5 = R3R4R6


to the network of resistances as shown in figure. In
order that the maximum power can be delivered to the 1 1 1 1
(B) += +
network, the value of R in Ω should be. (1995) R5 R 6 R1 + R 2 R 3 + R 4

(C) R1R4 = R2R3


R R
E R 6R R
R 4R
4 (D) R1R3 = R2R4

4 8 Q.7 A wire of length L and 3 identical cells of negligible


(A) (B) 2 (C) (D) 18 internal resistance are connected in series. Due to the
9 3
current, the temperature of the wire is raised by ΔT in
a time t. A number N of similar cells is now connected
Q.4 In the circuit shown in the figure, the current in series with a wire of the same material and cross-
through  (1998) section but of length 2L. The temperature of the wire is
3 2 2 raised by the same amount ΔT in the time. The value of
N is  (2001)
9V
(A) 4 (B) 6 (C) 8 (D) 9
8 8 4

2 2 2
Q.8 In the given circuit, with steady current, the potential
difference across the capacitor must be 
(A) The 3Ω resistor is 0.50A  (2001)
V R
(B) The 3Ω resistor is 0.25A
V C
(C) The 4Ω resistor is 0.50A
2V
(D) The 4Ω resistor is 0.25A

Q.5 In the circuit shown P ≠ R, the reading of (A) V (B) V/2 (C) V/3 (D) 2V/3
galvanometer is same with switch S open or closed.
Then  (1999)
Q.9 The effective resistance between point P and Q of
the electrical circuit shown in the figure is (2002)
P Q
2R 2R

2R


R G r r
P Q

2R
2R 2R
V

(A) IR = IG (B) IP = IG (C) IQ = IG (D) IQ = IG

2Rr 8R(R + r)
Q.6 In the given circuit, it is observed that the current I (A) (B)
R +r 3R + r
is independent of the value of the resistance R6. Then,
the resistance values must satisfy  (2001) 5R
(C) 2r + 4R (D) + 2r
R5 2

I R1 R3 Q.10 In shown arrangement of the experiment of the


R6 meter bridge if AC corresponding to null deflection of
galvanometer is x, what would be its value if the radius
R2 R4 of the wire AB is doubled? (2003)
P hysi cs | 20.43

Potential difference
25

V En volts
R1 R2 20
15
G 10
A B 5
C
0 50 50 150 200 250 300

(A) x (B) x/4 (C) 4x (D) 2x


Time in seconds

Q.11 The three resistance of equal value are arranged Q.13 The electric bulbs marked 25 W – 220 V and 100
in the different combinations shown below. Arrange W – 220 V are connected in series to a 440 V supply.
them in increasing order of power dissipation  (2003) Which of the bulbs will fuse ? (2012)

i i (A) Both (B) 100 W


(C) 25 W (D) Neither
I
II
Q.14 The supply voltage to a room is 120 V. The
resistance of the lead wires is 6 Ω . A 60 W bulb is
already switched on. What is the decrease of voltage
i i across the bulb, when a 240 W heater is switched on in
parallel to the bulb ?  (2013)
III (A) 2.9 Volt
IV (B) 13.3 Volt
(C) 10.04 Volt (D) Zero volt

Heater

II 6 Bulb

120 V
i

II IV Q.15 This question has statement-I and statement-II. Of


the four choices given after the statements, choose the
(A) III<II<IV<I (B) II<III<IV<I one that best describes the two statement.
(C) I<IV<III<II (D) I<III<II<IV Statement-I: Higher the range, greater is the resistance
of ammeter.
Q.12 The figure shows an experimental plot for Statement-II: To increase the range of ammeter,
discharging of a capacitor in an R-C circuit. The time additional shunt needs to be used across it. (2013)
constant τ of this circuit lies between :  (2012)
(A) Statement-I is true, statement-II is true, statement-II
(A) 150 sec and 200 sec is not the correct explanation of statement-I
(B) 0 and 50 sec (B) Statement-I is true, statement-II is false
(C) 50 sec and 100 sec (C) Statement-I is false, statement-II is true
(D) 100 sec and 150 sec (D) Statement-I is true, statement-II is true, statement-II
is the correct explanation of statement-I
2 0 . 4 4 | Current Electricity

Q.16 In a large building, there are 15 bulbs of 40 W, 5 Q.18 When 5V potential difference is applied across a
bulbs of 100 W, 5 fans of 80 W and 1 heater of 1kW. wire of length 0.1 m, the drift speed of electrons is 2.5
The voltage of the electric mains is 220 V. The minimum × 10‒4 ms‒1. If the electron density in the wire is 8 × 1028
capacity of the main fuse of the building will be: (2014) ms‒3, the resistivity of the material is close to:  (2015)
(A) 12 A (B) 14 A (C) 8 A (D) 10 A (A) 1.6 × 10‒7 Ωm (B) 1.6 × 10‒6 Ωm
(C) 1.6 × 10‒5 Ωm (D) 1.6 × 10‒8 Ωm
Q.17 In the circuit shown, the current in the 1 Ω resistor
is:  (2015)
Q.19 A galvanometer having a coil resistance of
(A) 0 A 100 Ω gives a full scale deflection, when a current of
1 mA is passed through it. The value of the resistance,
(B) 0.13 A, from Q to P
V
which can convert this galvanometer into ammeter
6V
(C) 0.13 A, from P to Q giving a full scale deflection for a current of 10 A is:
P
(D) 1.3 A, from P to Q 9V  (2016)
1 (A) 2 Ω (B) 0.1 Ω (C) 3 Ω (D) 0.01 Ω
 V+6
V-9
3 Q
5(=2+3)

JEE Advanced/Boards

Exercise 1 Q.3 Find the equivalent resistance of the circuit between


points A and B shown in figure is: (each branch is of
Q.1 A network of nine conductors connects six point A, resistance = 1Ω)
B, C, D, E and F as shown in figure. The figure denotes
resistances in ohms. Find the equivalent resistance
between A and D.
A B
A

2
1 1
1 1
Q.4 Find the current through 25V cell &power supplied
1 by 20V cell in the figure.
2 2 10V 5V 20V
B C 30V
1 25V
10

11
5

Q.2 Find the current I & voltage V in the circuit shown.


5 2 Q.5 In a cell of constant E.M.F. produces the same amount
60V of the heat during the same time in two independent
4 2 resistor R1 and R2, when they are separately connected
across the terminals of the cell, one after the another,
1
V 0.4
find the internal resistance of the cell.

20 V
4 8

2 10
P hysi cs | 20.45

Q.6 In the circuit shown in figure, all wires have equal Q.12 If switches S1, S2 and S3 in the figure are arranged
resistance r. Find the equivalent resistance between A such that current through the battery isminimum, find
and B. C the voltage across points A and B.
E
6 3
B
D 6 A
F
S2
1
A S1 9 1 1
24V S3
Q.7 Find the resistor in which maximum heat will be
produced. 5 5 B
6
4 Q.13 A battery of emf e0 = 10V is connected across a
2 1 m long uniform wire having 10Ω\m. Two cells of emf
V 𐐩1= 2V and 𐐩2 = 4Vhaving internal resistances 1Ω and
5Ω respectively are connected as shown in the figure. If
Q.8 For what value of R in circuit, current through 4Ω a galvanometer shown no deflection at the point P, find
resistance is zero. the distance of point P from the point a.
R 4

4V 2 10
6V
10V 0=10V
P
A B
1
Q.9 In the circuit shown in figure, the reading of
1=2V G
ammeter is the same with both switches open as with 5
both closed. Then find the resistance R.(ammeter is
2=4V
ideal)
100
Q.14 Potentiometer wire AB is 100 cm long and has a
V

total resistance of 10 ohm. If the galvanometer shows


R
50
+ – zero deflection at the position C, then find the value of
300 1.5V unknown resistance R.

Q.10 An electrical circuit is shown in the figure.


Calculate the potential difference across the resistance 10V
of 400 ohm, as will be measured by the voltmeter V of
resistance 400 ohm, either by applying Kirchhoff’s rules 40cm C
or otherwise. A B
V r=1
400  G
5V
100 100 200
R
100 
Q.15 In the figure for which value of R1 and R2 the
balance point for Jockey is at 40 cm from A. When R2 is
shunted by a resistance of 10Ω, balance shifts to 50 cm.
10V

Find R1 and R2(AB=1 m):


Q.11 Find the current (in mA) in the wire between
points A and B.
1k A 2k
R1 R2
G

2k 1k
B
A B

30V
2 0 . 4 6 | Current Electricity

Q.16 A part of a circuit is shown in figure. Here reading of Q.21 A network of resistance is constructed with R1 and
ammeter is 5 ampere and voltmeter is 96 V and voltmeter R2 as shown in the figure. The potential at the point
resistance is 480 ohm. The find the resistance R 1,2,3,…, N are V1,V2,V3,..., Vn respectively each having a
R potential k time smaller than previous one. Find:
A
V V0 R1 V1 R1 V2 R1 V3 VN-1 R1 VN

Q.17 A accumulator of emf 2 Volt and negligible V0=kV


1
R2 R2 R2 R2 R2
internal resistance is connected across a uniform wire
of length 10 m and resistance 30Ω The appropriate
terminals of a cell of emf 1.5 Volt and internal resistance
1Ω is connected of one end of the wire, and other
terminal of the cell is connected through sensitive
galvanometer to a slider on the wire. What length of R1 R2
(i) and in terms of k
the wire will be required to produce zero deflection of R2 R3
the galvanometer?
(ii) Current that passes through the resistance R2 nearest
How will the balancing change (a) when a coil of to the V0 in terms V0, k & R3.
resistance 5Ω is placed series with the accumulator,
(b) the cell of 1.5 volt is shunted with 5Ω resistor?
Q.22 A person decides to use his bath tub water to
Q.18 (a) The current density across a cylindrical generate electric power to run a 40 watt bulb. The bath
conductor of radius R varies according to the tube is located at a height of 10 m from the ground & it
 r holds 200 liters of water. If we install a water driven wheel
equation,
= J J0  1 −  where r is the distance from generator on the ground, at what rate should the water
 R drain from the bath tube to light bulb? How long can
the axis. Thus the current density is a maximum we keep the bulb on, if the bath tub was full initially. The
J0 at the axis r = 0 and decreases linearly to zero at efficiency of generator is 90%. (g = 10 m/s‒2)
the surface r = R. Calculate the current in terms of
J0 and conductor’s cross sectional area is A = πR2. Q.23 In the circuit shown in figure, calculate the
(b) Suppose that instead the current density is a following:
maximum J0 at the surface and decreases linearly to
r 36V
zero at the axis so that J = J0 . Calculate the current.
R 6 3
S
Q.19 What will be the change in the resistance of a a b
circuit consisting of five identical conductors if two 3 6
similar conductors are added as shown by the dashed
line in figure.
(i) Potential difference between point a and b when
switch S is open
(ii) Current through S in the circuit when S is closed.

Q.24 A rob length L and cross-section area A lies


Q.20 The current I though a rod of a certain metallic oxide
along the x-axis between x = 0 and x = L. The material
is given by I=2V5/2, where V is the potential difference
obeys Ohm‘s law and its resistivity varies along the rod
across it. The rod is connected in series with a resistance
according to ρ(x) = ρ0ex/L. The end of rod at x = 0 is at a
to a 6V battery of negligible internal resistance. What
potential V0 and it is zero at x = L.
value should the series resistance have so that:
(a) Find the total resistance of the rod and the current
(i) The current in the current is 0.44
in the wire.
(ii) The power dissipated in the rod is twice that
(b) Find the electric potential in the rod as a function
dissipated in the resistance.
of x.
P hysi cs | 20.47

Q.25 An ideal cell having a steady emf of 2 volt is Q.4 Resistance R1and R2 each 60Ω are connected in
connected across the potentiometer wire of length series as shown in figure. The Potential difference
10 m. The potentiometer wire of magnesium and between A and B is kept 120 volt. Then what will be the
having resistance of 11.5Ω/m. An another cell gives a reading of voltmeter connected between the point C
null point at 6.9 m. If a resistance of 5Ω is put in series and D if resistance of voltmeter is 120Ω.
with potentiometer wire, find the new position of the
null point. A B

R1 C R2
Exercise 2 V D

Single Correct Choice Type (A) 48V (B) 24V

Q.1 An insulating pipe of cross-section area ‘A’ contains (C) 40V (D) None of these
an electrolyte which has two types of ions → their charge
begin-e and +2 e. A potential difference applied between Q.5 The resistance of all the wires between any two
the ends of the pipe result in the drifting of the two types adjacent dots is R. The equivalent resistance between A
of ions, having drift speed = v ( -v e ion) and v/4 (+v e ion). and B as shown in figure is:
Both ions have the same number per unit volume = n. The
current flowing through the pipe is A

(A) nev A/2 (B) nev A/4


(C) 5nev A/2 (D) 3nev A/2
B
Q.2 Under what condition current passing through the
resistance R can be increased by short circuiting the (A)7/3 R (B) 7/6 R
battery of emf E2. The internal resistance of the two (C) 14/8 R (D) None of these
batteries are r1 and r2 respectively.

E1 r1 E2 r2 Q.6 In the circuit shown, what is the potential difference


VPQ?
2
Q
4V 3
R 2V
P
1 2 1V
(A) E2r1 > E1(R + r2) (B) E1r2 > E2(R + r1)

(C) E2r2 > E1(R + r2) (D) E1r1 > E2(R + r1) (A) +3V (B) +2V
(C) -2V (D) None of these
Q.3 Awire of length L and 3 identical cells of negligible
internal resistance are connected in series. Due to the Q.7 One end of a Nichrome wire of length 2L and
current, the temperature of the wire is raised by ΔT in cross-sectional area A is attached to an end of another
time t. N number of similar cells is now connected in Nichrome wire of length L and cross- sectional area
series with a wire of the same material and cross section 2A. If the free end of the longer wire is at an electric
but of length 2L. The temperature of the wire is raised potential of 8.0 volt, and the free end of the shorter
by the same amount ΔT in the same time t. The value wire is at an electric potential of 1.0 volt, the potential
of N is: at the junction of the two wire is equal to
(A) 4 (B) 6 (C) 8 (D) 9 (A) 2.4 V (B) 3.2 V (C) 4.5 V (D) 5.6 V
2 0 . 4 8 | Current Electricity

Q.8 Rate of dissipation Joule’s heat in resistance per Q.12 Which of the following quantities do not change
unit volume (Symbols have usual meaning) when an ohmic resistor connected to a battery is heated
due to the current?
(A) σE (B) σJ (C) JE (D) None of these
(A) Drift speed (B) Resistivity
Q.9 A millimetre of range 10 mA and resistance 9Ω is (C) Resistance (D) Number of free electrons
joined in a circuit as shown (see figure). The meter gives
full-scale deflection for current I when A and B are used
Q.13 Consider the circuit shown in the figure.
as its terminals, i.e. current enters leaves at A and leaves
at B (C is left isolated). The value of I is 5 3 A 3

(A) 100 mA (B) 900 mA


10 10 4
(C) 1 A (D) 1.1 A 28V

9 10mA 4 2 3

(A) The current in the 5 Ω resistor is 2 A


(B) The current in the 5 Ω resistor 1 A
0.1 0.9
(C) The potential difference VA − VB is a 10V
A B C
(D) The potential difference VA − VB is 5V

Q.10 In a balance Wheatstone bridge, current in the Q.14 In the circuit shown the readings of ammeter and
galvanometer is zero. It remains when: voltmeter are 4A and 20V respectively. The meters are
(i) Battery emf is increased non-ideal, then R is:
R
(ii) All resistance are increased by 10 ohms A
(iii) All resistance are made five times V
(iv) The battery and the galvanometer are interchanged (A) 5 Ω
(A) Only (i) is correct (B) Less than 5 Ω
(B) (i), (ii) and (iii) are correct (C) Greater than 5 Ω
(C) (i), (iii) and (iv) are correct (D) Between 4 Ω and 5 Ω
(D) (i) and (iii) are correct
Multiple Correct Choice Type
Q.11 A Wheatstone’s bridge is balanced with a
resistance of 625Ω in the third arm, where P, Q and S Q.15 A metallic conductor of irregular cross-section is
are in the 1st, 2ndand 4th arm respectively. If P and Q are as shown in the figure. A constant potential difference
interchanged, the resistance in the third arm has to is applied across the ends (1) and (2). Then:
be increased by 51Ω to secure balance. The unknown
resistance in the fourth arm is
(1) P Q (2)

P Q
(A) The current at the cross-section P equals the current
at cross-section Q
(B) The electric field intensity at P is less than that at Q.
S 625 (C) The rate of heat generated per unit time at Q is
greater that at P
(D) The number of electrons crossing per unit area of
(A) 625Ω (B) 650Ω (C) 676Ω (D) 600Ω
cross-section at P is less than that at Q.
P hysi cs | 20.49

Q.16 In the circuit shown E, F, G and H are cells of Q.20 Mark out the correct options.
e.m.f 2V,1V, 3V and 1V respectively and their internal
(A) An ammeter should have small resistance
resistance are 2 Ω ,1 Ω ,3 Ω and 1 Ω respectively.
(B) An ammeter should have large resistance
A (C) A voltage should have small resistance
F E (D) A voltage should have large resistance
D B
2 Assertion Reasoning Type
G H (A) Statement-I is true, statement-II is true and
C statement-II is correct explanation for statement-I.
(A) VD-VB=-2/13V (B) Statement-I is true, statement-II is true and
statement-II is true and statement-II is NOT the correct
(B) VD-VB=2/13V
explanation for statement-I
(C) VG=-21/13V=potential difference across G.
(C) Statement-I is true, statement-II is false.
(D) VH=19/13V=potential difference across H.
(D) Statement-I is false, statement-II is true

Q.17 A micrometre has a resistance of 100 Ω and a full


Q.21 Statement-I: When two conduction wires of
scale range of 50µA. It can be used as a voltmeter or a
different resistivity having same cross section area are
higher range ammeter provided a resistance is added to
joined in series, the electric field in them would be
it. Pick the correct range and resistance combination (s)
equal when they carry current.
(A) 50V range with 10kΩ resistance in series.
Statement-II: When wires are in series they carry equal
(B) 10V range with 200kΩ resistance in series. current.
(C) 5mA range with 1Ω resistance in parallel
Q.22 Statement-I: potential difference across the
(D) 10mA range with 1kΩ resistance in parallel.
terminals of a battery is always less than its emf .

Q.18 In a potentiometer wire experiment the emf of Statement-II: A battery always has some internal
a battery in the primary circuit is 20V and its internal resistance.
resistance is 5Ω. There is resistance box in series with the
battery and the potentiometer wire, whose resistance Q.23 Statement-I: Knowing that rating is done at
can be varied from 120Ω to 170Ω. Resistance of the steady of the filament, an electric bulb connected to a
potentiometer wire is 75 Ω. The following potential source having rated voltage consumes more than rated
difference can be measured using this potentiometer. power just after it is switched on.
(A) 5 V (B) 6V (C) 7V (D) 8 V Statement-II: When filament is at room temperature
its resistance is less than its resistance when the bulb is
Q.19 A current passes through an ohmic conductor fully illuminated.
of non-uniform cross-section. Which of the following
quantities are independent of the cross- section? Comprehension Type
(A) The charge crossing in a given time interval.
Paragraph 1: Two persons are pulling a square of side
(B) Drift speed a along one of the diameters horizontally to make it
rhombus. Plane of rhombus is always vertical and
(C) Current density
uniform magnetic field B exist perpendicular to plane.
(D) Free-electron density They start pulling at t = 0 and with constant velocity v.

Q.24 The induced emf in the frame when angle at


corner being pulled is 60º
Bav Bav
(A) Bav (B) 2Bav (C) (D)
2 4
2 0 . 5 0 | Current Electricity

Q.25 If the resistance of the frame is R the current Match the Columns
induced is
Q.31 Electrons are emitted by a hot filament and are
Bav 2Bav Bav Bav
(A) (B) (C) (D) accelerated by an electric field as shown in figure. The
2R 2R 2 4R two stops at the left ensure that the electron beam has
a uniform cross-section. Match the entries of column-I
Q.26 Finally square frame reduces to straight wire. The with column-II as electron move from A to B:
total charge flown is
E
a2B a2B aB aB
(A) (B) (C) (D)
R 2R R R
A B

Paragraph 2: In case of analysis of circuits, containing


cells, resistance and inductances two things are very
important one is conservation of charge which leads Column-I Column-II
to the fact that at any junction of circuit incoming
current is equal to outgoing current. The other thing (A) Speed of an electron (p) Increase
is that sum of voltage drop in a closed loop is equal to (B) Number of free (q) Decrease
zero. Inductors have a unique property by which they
Per unit Volume electrons
oppose the change in magnetic flux linked to them.
(C) current density (r) Remains same
The voltage drop across resistor is VR = IR and across
(D) Electric potential (s) any of the above is
dI
inductor is L . In the steady state current through possible
dt
inductor becomes constant which leads to zero voltage
drop across inductor. i.e. it behaves like short circuit.
Refer to circuit (see figure) E = 10V, R1 = 2 Ω , R2 = 3 Ω , Previous Years’ Questions
R3 = 6 Ω and L = 5H

S R1 I1 I2 R2 Q.1 Six equal resistance are connected between


points P, Q and R as shown in the figure. Then, the net
I3
resistance will be maximum between (2004)
E R3 L
P

Q.27 The current I1 just after pressing the switch S is


10 10 10 10
(A) A (B) A (C) A (D) A
8 5 12 6
Q R
Q.28 The current I1 long after pressing the switch S is
10 10 10 10 (A) P and Q (B) Q and R
(A) A (B) A (C) A (D) A
4 5 12 6
(C) P and R (D) Any two points

Q.29 The current I2 long after pressing the switch S is


10 10 10 10
(A) A (B) A (C) A (D) A
4 5 12 6

Q.30 The current through R2 just after releasing the


switch S is
10 10 10 10
(A) A (B) A (C) A (D) A
4 5 6 6
P hysi cs | 20.51

Q.2 For the post office box arrangement to determine (C) Current density across AB is double that of across BC
the value of unknown resistance, the unknown
(D) Potential difference across AB is 4 times that of
resistance should be connected between  (2004)
across BC
B C D

Q.6 Find the time constant for the given RC circuits in


correct (in µs). (2006)
V V
R1 C2
C1 R1
R2 C1
A

C2 R2
B1 C1 V

R1 C1

(A) B and C (B) C and D


(C) A and D (D) B1 and C1 R2 C2

Q.3 A moving coil galvanometer of resistance 100 R1 =


1Ω, R 2 =
2Ω, C1 =µ
4 F, C2 =
2µF.
Ω is used as ammeter using a resistance 0.1 Ω . The
maximum deflection current in the galvanometer is (A) 18,4,8/9 (B) 18, 8/9, 4
100µA. Find the current in circuit, so that the ammeter (C) 4, 18,8/9 (D) 4, 8/9,18
shown maximum deflection  (2005)
(A) 100.1 mA (B) 1000.1 mA Q.7 A circuit is connected as shown in the figure with
the switch S open. When the switch is closed, the total
(C) 10.01 mA (D) 10.1 mA
amount of charge that flows from Y to X is  (2007)

Q.4 Find out the value of current through 2 Ω resistance 3F 6F
X
for the given circuit. (2005)
S
3 6
Y
10V 5 10 20V

9V
2
(A) Zero (B) 54 µC (C) 27 µC (D) 81 µC
(A) 5A (B) 2A (C) Zero (D) 4A
Q.8 Figure shows resistor circuit configuration R1 ,R 2
Q.5 Two bars of radius r and 2r are kept in contact as and R3 connected to 3V battery. If the power dissipated
shown figure. An electric current l is passed through the configuration R1R 2 and R3is P1 ,P2 and P3respectively,
bars. Which one of following is correct? (2006) then  (2008)

l/2
1 1 1
2r l/2 1
1 1 3V
r 3V
1
l 1 1 3V 1 1
C 1
1 1 1
A B
R1 R2 R3
(A) Heat produced in bar BC is 4 times the heat
produced in bar AB
(A) P1 > P2 > P3 (B) P1 > P3 > P2
(B) Electric field in both halves is equal
(C) P2 > P1 > P3 (D) P3 > P2 > P1
2 0 . 5 2 | Current Electricity

Q.9 Consider thin square sheet of side L and thickness Q.12 A micrometre has a resistance of 100Ω and full
t, made of a material of resistivity ρ. The resistance scale range of 50 µF . If can be used as a voltmeter or as
between two opposite faces, shown by the shaded area a higher range ammeter provided a resistance is added
in the figure is  (2010) to it. Pick the correct range and resistance combination
(S) (1991)
(A) 50V range with 10 kΩ resistance in series
(B) 10V range with200 kΩ resistance in series
(C) 5mA range with 1Ω resistance in parallel
t
L (D) 10mA range 1Ω resistance in parallel

(A) Directly proportional to L. Q.13 When a potential difference is applied across. The
current passing through (1999)
(B) Directly proportional to t
(A) An insulator at 0 K is zero
(C) Independent of L
(B) A semiconductor at 0 K is zero
(D) Independent of t
(C) A metal at 0 K finite
Assertion Reasoning Type (D) A p-n diode at 300 K is finite, if it is reverse biased
(A) If statement-I is true, statement-II is true: statement-II
is the correct explanation for statement-I Q.14 For the circuit shown in the figure. (2009)
(B) If statement-I is true, statement-II is true: statement-II
is the not a correct explanation for statement-I 2k R1
(C) If statement-I is true: statement-II is false l

(D) If statement-I is false: statement-II is true 24V


6k R2
RL
1.5k

Q.10 Statement-I: In a meter bridge experiment, null (A) The current I through the battery is 7.5 mA
point for an unknown resistance is measured. Now,
the unknown resistance is put inside an enclosure (B) The potential difference across RL is 18V
maintained at a higher temperature. The null point can (C) Ratio of powers dissipated in RL and R2 is 3
be obtained at the same point as before by decreasing
(D) If R1and R2 are interchanged, magnitude of the
the value of the standard resistance.
power dissipated in RL will decrease by a factor of 9
Statement-II: Resistance of a metal increase with
increase in temperature. (2008)
Q.15 For the resistance network shown in the figure,
choose the correct option(s). (2012)
Q.11 Capacitor C1 of capacitance 1 µF and capacitor C2
2
of capacitance 2 µF are separately charged fully by a P
i2
S
common battery. The two capacitors are then separately 2 2
allowed to discharge through equal resistor at time
t = 0 (1989) 1 1
(A) The current in each of the two discharging circuit is 4 4
zero at t = 0 Q 4 T
(B) The current in the two discharging at circuits at i2
t = 0 are equal but not zero 12 V
(C) The current in the two discharging circuit at t = 0
are unequal (A) The current through PQ is zero
(B) I1 = 3 A
(D) Capacitor C1, loses 50% of its initial charge sooner
than C2 loses 50% of its initial charge (C) The potential at S is less than that at Q
(D) I2 = 2A
P hysi cs | 20.53

Q.16 The ideal batteries of emf V1 and V2 and three j(t) j(t)
resistances R1, R2 and R3 are connected as shown in
the figure. The current in resistance R2 would be zero if
 (2014) (A) (B)
(A) V=1 V2 and R= 1 R=2 R3
t
(B) =
V1 V2 and =
R1 2R
= 2 R3 (0, 0) (0, 0)
(C) =
V1 2V2 and 2R
= 1 2R
= 2 R3
j(t) j(t)
(D) 2V=
1 2V2 and 2R=
1 Rj(t)
=2 R3 j(t)

(C) j(t) (D) j(t)


V1 (A) (B)
R1 t
(0, 0) (0, 0)
t (A) t (B)
R(0,
2 0) (0, 0)
t
j(t) j(t)(0, 0) (0, 0)
V2
j(t) j(t)
j(t)
R3 j(t)
(C) (D)

t (C) t (D)
(A) (0, 0) (0, 0)
(B)
Q.17 During an experiment with a metre bridge, the
t
galvanometer shows a null point when the jockey t is (0, 0) t (0, 0)
pressed at 40.0 cm using a standard
(0, 0) resistance of 90 Ω , (0, 0)
as shown in the figure. The least count of the scale used
in the metre bridge is 1 mm.
j(t) The unknown resistance is j(t)
 (2014)
(A) 60 ± 0.15 Ω (C) (B) 135 ± 0.56 Ω (D)
(C) 60 ± 0.25 Ω (D) 135 ± 0.23 Ω
t t
(0, 0) (0, 0)

R 90
Q.19 Consider the identical galvanometers and two
identical resistors with resistance R. If the internal
resistance of the galvanometers RC < R/2, which of
40.0cm the following statement(s) about any one of the
galvanometers is(are) true ?  (2016)

Q.18 An infinite line charge of uniform electric (A) The maximum voltage range is obtained when all
charge density λ lies along the axis of an electrically the components are connected in series
conducting infinite cylindrical shell of radius R. At (B) The maximum voltage range is obtained when the
time t = 0, the space inside the cylinder is filled with a two resistors and one galvanometer are connected in
material of permittivity ε and electrical conductivity σ . series, and the second galvanometer is connected in
The electrical conduction in the material follows Ohm’s parallel to the first galvanometer
law. Which one of the following graphs best describes
the subsequent variation of the magnitude of current (C) The maximum current range is obtained when all
density j(t) at any point in the material? (2016) the components are connected in parallel
(D) The maximum current range is obtained when the
two galvanometers are connected in series and the
combination is connected in parallel with both the
resistors
2 0 . 5 4 | Current Electricity

Q.20 In the circuit shown below, the key is pressed at (A) The voltmeter displays – 5 V as soon as the key is
time t = 0. Which of the following statement(s) is (are) pressed, and displays +5 V after a long time
true?  (2016)
(B) The voltmeter will display 0 V at time t = ln 2 seconds
40 F 25 k
(C) The current in the ammeter becomes 1/e of the
- initial value after 1 second
V
+ (D) The current in the ammeter becomes zero after a
20 F long time
A 50 k

+ -
Key 5V

PlancEssential Questions
JEE Main/Boards JEE Advanced/Boards
Exercise 1 Exercise 1
Q. 8 Q.9 Q.11 Q.1 Q.2 Q.13
Q.19 Q.20 Q.24 Q. 23 Q.24
Q.28
Exercise 2
Exercise 2 Q.5 Q.9 Q.14
Q. 2 Q.4 Q.8 Q.15 Q.17 Q.18
Q.14 Q.21
Comprehension – 1 (Q.24-26),
Comprehension – 2 (Q.27-30)

Answer Key

JEE Main/Boards Q.4 (a) (20/19) Ω ,(b) 10A, 5A, 4A; 19A

Exercise 1 Q.5 1027 0 C

Q.1 30 A Q.6 2.0 × 10-7 Ω m

Q.2 17 Ω , 8.5V Q.7 0.0039 o C−1

Q.3 (a) 6 Ω (b) 2 V,4V ,6V Q.8 867 0 C


P hysi cs | 20.55

Q.9 current in branch AB = (4/17)A, in BC = (6/17)A, Q.17 (a) greater, (b) lower, (c) nearly independent of,
in CD = (-4/17)A, in AD(6/17)A, in BD = (-2/17)A, total (d) 1022.
current (10/17)A.
Q.18 Hint: Let X be the equivalent resistance of the
Q.10 11.5V infinite network. Clearly, 2+X(X+1) =X which given

Q.11 2.25V X= (1 + 3) Ω ; therefore the current is 3.7 A

Q.12 2.7 × 104 s (7.5 h) Q.19 (a) ε = 1.24V, (b) To reduce current through the
galvanometer when the movable contact is far from the
Q.13 ≈ 238 s. balance point, (c) No. (d) N truncated. (e) No

Q.14 (a) 1.4A, 11.9V, (b) 0.005 A Q.20 1.7 Ω

Q.15 ≈ 22 Aluminium is lighter, it is preferred for long Q.21 Resistance in series =5988 Ω
suspensions of cables Q.22 Shunt resistance=10 m Ω

Q.16 (a) Only current (because it is given to be steady). Q.23 0.12 volt
The rest depends on the area of cross-section inversely. Q.24 (a) 0.9 A, (b)(b) Yes we can do it. It is called The
(b) No, examples of non-ohmic elements; vacuum venin theorem. It can be represented as equivalent
diode, semiconductor diode. resistance and equivalent voltage.
Q.25 (a) shunt resistance = 400 × 10 which is also
−3

(c) Because the maximum current drawn a source = ε/r.


nearly its net resistance, reads slightly less. (b) Series
(d) Because, if the circuit is shorted (accidentally), resistance = 3988 Ω ; net resistance = 4000 Ω ; read
the current drawn will exceed safety limits, if internal slightly less
resistance is not large Q.26 2 Ω ,4 Ω
Q.27 3.6 volt

Exercise 2
Single Correct Choice Type

Q.1 D Q.2 C Q.3 B Q.4 B Q.5 D Q.6 C


Q.7 C Q.8 D Q.9 C Q.10 A Q.11 D Q.12 C
Q.13 D Q.14 B Q.15 C Q.16 D Q.17 D Q.18 C

Previous Years’ Questions


Q.1 B Q.2 C Q.3 B Q.4 D Q.5 A Q.6 C
Q.7 B Q.8 C Q.9 A Q.10 A Q.11 A Q.12 D
Q.13 C Q.14 C Q.15 C Q.16 A Q.17 B Q.18 C
Q.19 D

JEE Advanced/Boards

Exercise 1
22
Q.1 1 Ω Q.2 I = 2.5A, V= 2.5 V Q.3 Ω
35
3r
Q.4 12A-20W Q.5 R1R 2 Q.6 Ω
5
2 0 . 5 6 | Current Electricity

Q.7 4 Ω Q.8 1 Ω Q.9 600 Ω


Q.10 20/3V Q.11 7.5 A Q.12 1V
10
Q.13 46.67 cm Q.14 4 ohm Q.15 Ω, 5Ω
3
Q.16 20 ohm Q.17 7.5 m,8.7 m, 6.125 m Q.18 (a) J0A\3; (b) 2J0A\3
R1 5
Q.19 = Q.20 (i) 10.52 Ω ; (ii) 0.3125 Ω
R2 3

Q.21 (i)
(k − 1)2
;
k
(ii)
( )
(k − 1) \ k 2 V0
Q.22 4/9kgs–1, 450 sec
k (k − 1) R3

Q.23 (i) Vab = –12 V (ii) 3 amp from b to a

ρ0L  1 V0 A  e  V0 (e− x \L − e−1 )


Q.24 R = 1 −  ; I =  ; V Q.25 7.2 m
A  e ρ0L  e − 1  1 − e−1

Exercise 2

Single Correct Choice Type

Q.1 D Q.2 B Q.3 B Q.4 A Q.5 B Q.6 B

Q.7 A Q.8 C Q.9 C Q.10 C Q.11 B Q.12 D

Q.13 A Q.14 C

Multiple Correct Choice Type

Q.15 A, B, C, D Q.16 A, D Q.17 B, C Q.18 A, B, C Q.19 A, D Q.20 A, D

Assertion Reasoning type


Q.21 D Q.22 D Q.23 A

Comprehension Type
Q.24 B Q.25 B Q.26 A Q.27 A Q.28 A Q.29 D
Q.30 C

Match the Columns


Q.31 A → p, B → q, C → r, D → p

Previous Years’ Questions


Q.1 A Q.2 C Q.3 A Q.4 C Q.5 A Q.6 B
Q.7 C Q.8 C Q.9 C Q.10 D Q.11 B, D Q.12 B, C
Q.13 A, B, D Q.14 A, D Q.15 A, B, C, D Q.16 A, B, D Q.17 C Q.18 C
Q.19 A, C Q.20 A, B, C, D
P hysi cs | 20.57

Solutions

JEE Main/Boards t = 1027ºC

Exercise 1 Sol 6: R =
ρL
A
V 12
Sol 1: i = = = 30 A RA 5 × 6 × 10−7
R 0.4 ρ= = = 2 × 10–7 Wm
L 15
V
Sol 2: i = (let R be resistor) ∆r 2.7 – 2.1
r +R Sol 7: α = = = 0.0039ºC–1
∆t 100 – 27.5
10
0.5 =
3+R V V

∆r i1 i2 V 1 1
R = 17 W Sol 8: α = = =  − 
∆t ∆t ∆t  i1 i2 
V0 = V – ir = 10 – 3 (0.5) = 8.5 V
\ Terminal voltage = 8.5 V V1 1  230  1 1 
t= t0 +  −  = 27 +  − 
α  i1 i2  1.7 × 10  2.8 3.2 
−4

Sol 3: (a) r = r1 + r2 + r3= 1 + 2 + 3 = 6 W = 867ºC


V 12
(B) i = = = 2A
r 6 Sol 9: By symmetry iAB = iDC
V = ir = i [1, 2, 3] = 2 [1, 2, 3] = [2, 4, 6] iBC = iAD
∴ Potential drops are 2V, 4V, 6V Let current in circuit be i
Let iAB = i1
1 1 1 1 1 1 1
Sol 4: = + + = + + (i –i1)5 = 10(i1) + 5(i1–(i–i1) )
R r1 r2 r3 2 4 5
(from ABD)
1 19
= Si – Si1 = 10i1 + 10i1 – 5i
R 20
⇒ 25i1 = 10i
20
⇒R= W 2
19 ⇒ i1 = i
5
1 1 1 1 
i= V   = 20   = [10 5 4] Now, 10 = 10i + 10i1 + 5(i – i1)
r  2 4 5
2
= 15i + 5i1= 15i + i (5)
\ Current is 10A, 5A, 4A 5 1
V 20 10 = 17i
Total current = = = 19 A
R 20 10
i= A
19 17
2 4
Sol 5: rt = r0 + α(t – t0) iAB = iOC = i= A
3 17
117 = 100 + 1.7 × 10–4Dt 6
iAD = iBC = i – i1 = A
17
⇒Dt = 103 ºC
−4
⇒ t0 = t0 + Dt = 27 + 103 iCD = 2i1 – i = A
17
2 0 . 5 8 | Current Electricity

V1 − V2 120 − 8 Sol 16: (a) Only current is constant. Rest depends on


Sol 10: i = = area of cross-section inversely.
R +r 15.5 + 0.5
i=7A (b) Ohm’s law is non-universal for example non-ohmic
conductors like semi-conductors.
DVbattery = V + ir = 8 + 7(0.5) = 11.5 V
E 1
(c) i = ⇒ i∝
r r
V1 1
Sol 11: = So low resistance is necessary.
V2 2
(d) Because the circuit is shorted accidentally, the
2 63 current drawn will exceed safely limits.
⇒ V2 = V1× = 1.25 × = 2.25 V
1 35

Sol 17: (a) Alloys have greater resistance than their


I I constitutional metals.
So 12: = neVd; Vd =
A Ane
(b) Alloys have lower temperature coefficient of
 Ane 2 × 10−6 × 8.5 × 1028 × 1.6 × 10−19 × 3 resistance than pure metals.
t= = =
Vd I 3 (c) It is nearly independent
= 27200 sec ρ r ρ
(d) r = ⇒ 1 = 1
A r2 ρ2
4 3
Sol 13: Q = σA= σ . πr
3 e r >>> r2⇒ ρ1>>>ρ2
3 ⇒ 1022 is the correct answer.
Q 4 πσre 4 π × 10−9 × (6.37 × 106 )3
t= = =
i 3 i 3 1800 Note: (a), (b), (c) are all theoretical. Perfect reasoning
≈ 238 s
will be learnt in engineering. You may refer to energy
bands to understand it.
nV 6×2
Sol 14: (a) i = = = 1.4 A
R + nr 8.5 + 6(0.015) Sol 18: Let the resistance of circuit be R.

Terminal voltage = iR= 1.4 (8.5)= 11.9 V 1

V 1.9
(b) imax = = = 0.005 A
r 380 R 1
It can’t start a car as its current is very less. 1

ρ ρ
Sol 15: R = ; R∝ 1 R
A A ⇒R= +2⇒R= +2
1 1 R +1
m = d.Al +
R 1
m 1 d
A∝ ; ∝ ⇒R= 3 + 1; (R >0⇒ R ≠ – 3 + 1)
d A m
ρd V 12
⇒R∝ R = constant i= = ≈ 3.7 A
m r +R 3 + 1 + 0.5
⇒ m ∝ ρd
Sol 19: (a) Let length of potentiometer be 
ma 2.63 × 10−8 × 2.7
= = 0.464 ⇒ Standard cell voltage
mcu 1.72 × 10−8 × 8.9
2
VS = E
⇒ mAl < mCu 
Aluminium is lighter Assume  = 100 cm
For long suspension cables, Al is used as it is lighter. For standard potentiometer
P hysi cs | 20.59

1.02 × 100 0.2


⇒ξ = = 1.24 V Potential gradient = = 2 × 10–2 V/m
82.3 10
(b) 600 kΩ is used to reduce the galvanometer current, E = Potential gradient × length
when the movable content is far from the balance point.
= 2 × 10–2 × 6= 0.12 V
(c) No, the balance point where voltage of standard
cell equals voltage difference or potentiometer. Hence
current is zero so high resistance doesn’t affect it. V1 + V2 + V3
Sol 24: (a) i =
r1 + r2 + r3 + R
(d) Not much if the driver cell has small resistance
compared to potentiometer. Else it affects 2 + 1.8 + 1.5
= = 0.9 A
(e) No, as emf of driver cell is less than standard cell, 0.05 + 0.71 + 1 + 4
current through galvanometer is in opposite direction.
(b) Yes we can do it. It is called theorem. It can be
(f) No, if doesn’t work well as the gradation on represented as equivalent resistance and equivalent
potentiometer is low. To measure it keep a high voltage.
resistance in series to battery, to reduce voltage across
potentiometer.
Sol 25: (a) The resistance to be added in parallel

Sol 20: For 1 = 76.3 cm, V1 = 1.5V r.i 12 × 2.5 × 10−3


rs = = = 4 × 10–3 Ω = 4 mW
iA 7.5
2 64.8
⇒ V2 = × V 1= × 1.5= 1.274 V
1 76.3 ∴ Shunt resistance is 4 mW
Let current in 9.5Ω be i (b) The resistance to be added in series

⇒ (9.5) (i) = 1.27 W ⇒ i = 0.134 A V 10


R= = = 4kW
i 2.5 × 10−3
⇒ 1.274 = 1.5 – i(r)
⇒ r = 1.7 W \rseries = r – r9= 4000 – 12= 3988 W
Ammeter reads slightly less as all of iA doesn’t pass
Sol 21: Voltage across galvanometer through shunt. In derivation we assume iA passes
through shunt only for full deflection. Hence it reads
(V) = ir= 3 × 10–3 × 12= 36 mV
slightly less. Voltmeter reads less as some current passes
To convert it into a voltmeter of maximum value V0, through it, which reduces current in the measuring
resistance R should be add in series. element, which leads to reduced reading.
V0 = 18V
1
\ V0>> V ⇒ R >> r R1 3 1
Sol 26: = = ⇒ R2 = 2R1
V0 1.8 R2 1 2
1−
⇒R= = ≈ 6 kW 3
i 3 × 10−3
2
For accurate result, use R1 + 6
= 3 =2
V0 = i(r + R) ⇒ R = 5988 W R2 2
1−
3
Sol 22: i1R1 = i2R2
R1 + 6
⇒ = 2 ⇒ R1 = 2Ω, R2 = 4Ω,
i1R1 –3
4 × 10 × 15 2R1
⇒ R2 = = = 10–2Ω = 10 mW
i2 6
Sol 27: Let current in each branch be i
∆VP
Sol 23: Potential gradient = Current in 10Ω is 2i by symmetry

⇒ 2 + 2 = 10(2i) + 2 × 1.05 (i)
(ΔVP = Potential a cross potentiometer)
4
rP × ε 20 × 5 i= = 0.181 A
ΔVP = = = 0.2 V 22.1
rP + R 20 × 480
2 0 . 6 0 | Current Electricity

V10Ω = 20i = 3.6 V Sol 3 : (B) i = neVA


i i
V= =
Sol 28: Let VD = 0 neA pqs
3V – 1V = i1(3 + 1) + 2I (for loop BDC)
1V – 2V = i2 (2 + 1) + 2I (for loop BDA) Sol 4 : (B) V = iR
I = i1 + i2 V
=R
i
⇒ 2V = i1(4) + 2(i1 + i2) i T1
⇒ 6i1 + 2i2 = 2  … (i) T2
⇒ –1V = 3i1 + 2 (i1 + i2)
V
⇒ 2i1 + 5i2 = –1  … (ii)
 i 1
−5 6 Here slope of the graph is   =  
⇒ i2 = A (from (i) and (ii)) i1 = V R 
13 13
VD – VB = –2(I) = –2(i1 + i2) We know that as temperature increases, Resistance of
the metal increases.
 6 5  −2
= –2  −  = V  1   1 
 13 13  13 ∴   >   [From graph]
 R1   R 2 
6 21
DVG = EG–i1 RG= 3V – (iii) = V ⇒ R1< R2∴ T1 < T2
13 13

16 19
DVH = EH + i1RH= 1 + (i) = V Sol 5 : (D)
13 13

Exercise 2
Single Correcr Choice Type
 E + E2 + .......En 
i = 1
Sol 1 : (D) i = neVA  r + r + r + .....R 
1 2 3 
i1 V1 A1
= nE
i2 V2 A2 i= (for no R)
nr
i 1 = i2 E
i=
⇒ V1A1 = V2A2 r

 2 
V1(π d2) = V2  πd  ; V2 = 4V Sol 6 : (C) On simplifying
 4 
 
i 0.8 
Sol 2 : (C) i = neVA
∴ For constant n and e; 0.8 
⇒ V1A1 = V2A2
VP AQ i 0.8 
= ;AQ> AP
VQ AP
VP Current flowing through R is zero.
∴ > 1;⇒ VP> VQ
VQ 8
‘i’ in the circuit is = 5 amp.
1.6
P hysi cs | 20.61

P1 25
Potential difference between each cell is E – ir Sol 10 : (A) (i) i1 = = ≈ 0.1136 A
V1 220
1 – 5(0.2) = 0V
P2 100
i2 = = ≈ 0.4545 A
 3 + 15  V2 220V
Sol 7 : (C) i =  
1 + 2 +R  V12 2002
r1 = = = 1936 W
 18  P1 25
i = 
3+R 
V22 2002
r2 = = = 484 W
3V, 1 15V, 2 P2 100
a b
V 440
i= = = 182Å
r1 + r2 1936 + 484

∴ 25 W bulb fuses.
R
(ii) Alternative solution
Now writing the potential drop ab;
1
r∝
 18  P
3 – (1)   =0
3+R  r1 : r2 = P2 : P1 = 700 : 25 = 4 : 1
18 V1 : V2 = r1 : r2 = 4 : 1
⇒3= ;R = 3 W
3+R
4V 4
V1 = = V = 352 V
Sol 8 : (D) 4 +1 5

V V
V2 = = = 88 V
C D 4 +1 5
V1>Vrated , V2<Vrated

Hence 25W bulb fuses.


A B
1 2R RV
RCD = = Sol 11: (D) V =
1 1 3 r +R
+
2R R

42 R Sol 12 : (C) Let resistance be r


Similarly RDB =
3 VG − V
1 4R i=
RCB= ;RCB = r +R
1 1 7
+ 120 − 100
R 2R 2R 10 =
+ r +1
3 3
1 11R ⇒ r = 1Ω
RAB= ; RAB =
1 1 18
+
R 4R Sol 13: (D) Let voltage across galvanometer be V,
+R resistance r
7
V
V2 i1 =
Sol 9 : (C) R = ⇒ R ∝ V2 r
P
Now after shunning
∵ Connected in series, they have same current i
V
Power consume (PC) = i2R i2 =
r + 20
PC∝ R ⇒ PC∝ V2
2 0 . 6 2 | Current Electricity

i1 \ 2)(4)(p i2R)
Now P4 (i=
=
given i2 =
2
p5 = (i)2 (5)
1 1 1
⇒   = p4 1
or = P=
P5 10
= = 2 cal / s
2 r  r + 20 4
p5 5 5 5
⇒ r = 20 W

Sol 14: (B) To convert galvanometer to ammeter, we Sol 2: (C) The simplified circuit is shown in the figure
need to connect resistance across it. Let it be r max
current through galvanometer 30 
V 0.2
2V
30  30 
I= = = 10–2 A 2V
R 20 30  30 

ir = I0R0 30  .
.
10(r) = 10–2 × 20
2V 2V
60  20 
⇒ r = 2 × 10–2 W
30  =
2V 2V
30  60  = 20 
\ 0.02Ω resistor is connected across it.
.
.
Sol 15: (C) Total voltage (V0) = i(r + R)
= 10(90 + 910)mV = 10 V 2 1
Therefore current I ==I = A
\ Number of divisions 20 10
V0 10
= = = 100 Sol 3: (B) The given circuit is a balanced Whetstone’s
least count 0.1
bridge
C
Sol 16: (D) V = i(r + R) R 2R
12 = 0.1(20 + r) ⇒ r = 100 W 6R
A B
R R
R 6R R
E 2R
Sol 17: (D) Resistance of galvanometer, 4
R 4R
4R D
Rg>> r, R
4
\Vg≈ 12 V E

∴ It reads 12 V Thus no current will flow across 6R of the side CD. The
given circuit will now be equivalent to

Sol 18 : (C) On increase in temperature, number of free R 2R


electrons increase. But also the collisions will increase.
Hence conductivity decreases. 2R 4R
4

Previous Years’ Questions


Sol 1: (B) Sing, resistance in upper branch of the circuit
is twice the resistance in low lower branch. Hence,
current there will be half. For maximum power net external resistance
4 6 = Total internal resistance
i/2
or 2R = 4 or R = 2 Ω
.
.
5
P hysi cs | 20.63

Sol 4: (D) Net resistance of the circuit is 9 W. Therefore, in the second case,
∴ Current drawn from the battery. (NE)2
= t (2m)s ∆T  ... (i)
9 2R
= = 1A = current though 3 Ω resistor Dividing Eq. (ii) by (i), we get
9
PR (IR2 )R1 (7.5)2 (2) 25 N2
1 1
= 2=or N2 36=
or N 6
= = = 18
PR (I 2 (1.5) 3
2 R )R 22

3 A 2 C 2 Sol 8 (C) In steady state condition, no current will flow


though the capacitor C, current in the outer circuit,
1A i2 i4
9V 2V − V V
8 8 4
=i =
i1 i3 2R + R 3R  V R
2 B 2 D 2
V A B
Potential difference between A and B .
Potential difference between A and B is i=0 .
2V
VA − VB = 9 − (3 + 2) = 4V − 8i1 VA − V + V + ir =VB 2R
V V
∴ i1 =
0.5 A ∴ VB − VA =iR =  R =
3r
  3
∴ i2 =1 − 11 =0.5 A
Note: In this problem charge stored in the capacitor
Similarly potential difference between C and D V
can also be asked. Which is equal to q = C with
3
VC − VD = (VA − VB ) − i2 (2 + 2)
positive change on B side and negative on A side
=4 − 4i2 =4 − 4(0.5) =2V =8i3 because VB > VA
i3 = 0.25 A

i4 = i2 − i3 = 0.5 − 0.25 Sol 9: (A) The circuit can be drawn as follows:


2R 2R 2R 2R
i4 = 0.25 A
2R
Sol 5: (A) As there is no change in the reading of r r r r
galvanometer with switch S open or closed. It implies Q P Q
2R
that bridge is balanced. Current though S is zero and 2R 2R 2R 2R

IR I=
= G , IP IQ
4R
2Rr
Sol 6: (C) The statement indicates a balanced 2r R+r
Wheatstone bridge formed by R3, R4, R2 and R1. P Q P Q

R3 R1 4R
⇒ =
R 4 R2
AC
i.e. R1R4 = R3R2 Sol 10: (A) The ratio will remain unchanged.
CB
(3E)2 Sol 11: (A) P = i2R
Sol 7: (B) In the first case t = ms ΔT  ... (i)
R
 Current is same, so P∝R.
V2  2
H = t
In the first case it is 3r, in second case it is r in third
 R  3
r 3r
When length of the wire is doubled, resistance and case it is and in fourth case the net resistance is .
3 2
mass both are doubled.
R III < R II < R IV < R I ∴ PIII < PII < PIV < PI
2 0 . 6 4 | Current Electricity

Sol 12: (D) For discharging of an RC circuit, Sol 16: (A)

V = V0 e− t/ τ Item No. Power

V0 40 W bulb 15 600 Watt


So, when V =
2 100 W bulb 5 500 Watt
V0
= V0 e− t/ τ 80 W fan 5 400 watt
2
1000 W heater 1 1000 Watt
1 t t
ln =− ⇒τ=
2 τ ln2 Total Wattage = 2500 Watt
V0 So current capacity
From graph when V = , t – 100 s
2
100 P 2500 125
∴=τ = 144.3 sec =i = = = 11.36 ≅ 12 Amp
ln2 V 220 11

Sol 17: (B) Taking the potential at Q to be 0 and at P to


Sol 13: (C) Resistances of both the bulbs are be V, we apply Kirchhoff’s current law at Q :
V 2 2202
R
= = V+6 V V−9
1
P1 25 + + 0
=
3 1 5
V 2 2202 3
R
= 2 = V=
− −0.13 volt
=
P2 100 23
Hence R1 > R 2 The current will flow from Q to P.
V
When connected in series, the voltages divide in them 6V
in the ratio of their resistances. The voltage of 440 V P
divides in such a way that voltage across 25 W bulb will 9V
1
be more than 220 V.
V+6
V-9
Q
120 × 120 3 5(=2+3)
Sol 14: (C) Resistance of bulb
= = 240 Ω
60
120 × 120 Sol 18: (C) J = ne v d
Resistance of Heater
= = 60 Ω
240
A∆V
Voltage across bulb before is switched on, = nev d
ρA
120 ∆v
V1
= × 240 ∴ ρ=
246 nev 4
5
Voltage across bulb after heater is switched on, = = 1.56 × 10−4
28 −19
0.1 × 8 × 10 × 1.6 × 10 × 2.5 × 10−4
120
V2
= × 48 ≈ 1.6 × 10−5 Ω m
54
Decrease in the voltage is V1 − V2 = 10.04 V (approximately).
Sol 19: (D) For full scale deflection
Note: Here supply voltage is taken as rated voltage.
100 × ig = (i − ig )S
IgG
Sol 15: (C) For ammeter, S = Where ‘S’ is the required resistance
I − Ig
So for I to increase, S should decrease, so additional S 100 × 1 × 10−3
S=
can be connected across it. (10 − 10−3 )
S ≈ 0.01 Ω
P hysi cs | 20.65

JEE Advanced/Boards Let effective resistance across A, B be RAB


1 1 1
⇒ = +
Exercise 1 R AB 5+2 34 + 7

⇒ RAB = 6 W
Sol 1: Let voltage at A be VA
Let current from A to B be iAB. Similarly define iAC by the 1 1 1
Similarly = +
symmetry of circuit, iAB = iAC R CD 4+8 2 + 10
VB = VA – iAB (RAB) ⇒ RCD = 6 W

= VA – iAB(1) Let current be I

= VA – iAB Writing RVL (Kirchhoff Voltage law) equation

Similarly VC = VA – iAC 60 = 20 + IRAB + IRCD + I(4)

∴ iAC = iAB 40 = I(6 + 6 + 4)

⇒ VB = VC ⇒ I = 2.5 A

⇒ iBC = 0 VBA = IRAD= 2.5 (6) = 15 V


7 × VBA
⇒ We can ignore RBC. Voltage across 7Ω =
7 + 34
Similarly we can ignore Ref. Resultant circuit.
15 × 7
= = 2.5 V
A 42

3 Sol 3:
1 1 C D
O
1
1
A B
E F E
B 2 2 C

1 1 1 1
⇒ = + + G
R eff 2 1+ 2+1 1+ 2+1
By symmetry, iCE = iED
1 ⇒ Point E can be detached to your CED branch.
=1
R eff
Let resistance across CD be RCD
⇒ Equivalent resistance between AD = 1W
1 1 1 1 1
⇒ = + + ⇒ RCD = W
R CD 2 2 1 2
Sol 2:
1 1 1
= +
60V R AC 1 2
A 5 2 B + –
2
RAC = W
3
I 34 7 4
2
V RDB = W
20V 3
⇒ RACDB = RAC + RCD + RDB
4 8
1 2 2
= + +
2 10 2 3 3

11
RACDB = W
6
2 0 . 6 6 | Current Electricity

11
By symmetry, RAGB = RACDB = W = R12R2 + 2R1R2R + R2R2
6
RAEB = 1 + 1 = 2 W ⇒ R2 (R1 –R2) = R1R2(R1 –R2)

Let effective resistance be Reff. ⇒R= R1R 2


1 1 1 1
= + +
R eff. R ACDB R AEB R AGB Sol 6: Path ADCB is similar to path AFEB
6 6 1 ⇒ iAD = iET
= + +
11 11 2
Where iAD is current trough RAD.
35 22 ⇒ VD = VF
= ⇒ Reff= W
22 35
⇒ VFD = D

Sol 4: Let current through 11 Ω be i1 ⇒ Resistor FD can be removed


25 + 30 Similar EC can be removed
i1 = =5A
11 RADCB = 3r
25 – 20
Similarly i2 = = 1A RDFEB = 3r
5
25 + 5 RAB = r
i3 = =3A
10 Let effective resistance be Reff.
25 − 10
i4 = =3A 1 1 1 1
5 ⇒ = + +
R eff. R ADCB R AB R AFEB
i = i1 + i2 + i3 + i4= 5 + 1 + 3 + 3= 12 A
1 1 1
Power supplied = V × i = + +
3r r 3r
= – 20 × i2= – 20 × 1= – 20 W 3r
⇒ Reff. = W
Note: here V is taken – 20V as it opposes the direction 5
of current.
Sol 7: 5 3
Sol 5: Let internal resistance of battery be R 6
C

Current flown when R1 is connected 4


A B
V 2
i1 =
R1 + R

Power consumed V
1 1 1 1
V 2R1 = + +
P1 = i12R1 = R AB 2 6 5
(R + R1 )2
15
RAB = W
V 2R 2 13
Similarly P2 =
(R + R 2 )2 20
RBC = W
Given P1 = P2 9
Let current trough circuit be i
V 2R1 V 2R 2
⇒ = 15i
(R1 + R)2 (R + R 2 )2 ⇒ VAB = iRAB =
13
20i
⇒ R1R22 + 2R1R2R + R1R2 VBC =
13
Power in 4Ω is more than 5Ω across it as
P hysi cs | 20.67
1
 V2 V2  1.5 − VR 1.5 −
4Ω< 5Ω  > if r1 < r2  = = 3 = 7 × 10−2 A
r r2  300 300 18
 1 
Similarly P2Ω is greater than 6Ω, 5Ω across it Current through R iR = i0 - i

( VAB )
2 2 7 1 1
 15i  1 = × 10−2 – × 10 −2 = × 10−2 A
P2Ω = =   . 18 3 18
2Ω  13  2
1
225 i2 VR
P2Ω = Ω R= = 3 = 600 W
338 iR 1
× 10−2
(VBC )2
2
 20i  1 100 i2 18
P4Ω = =   =
4Ω  9  4 81
Sol 10 : Since resistance of voltage is 400 Ω, resistance
100 225 viewed by voltmeter R0
>
81 338 1 1 1
= +
⇒ 4Ω produces maximum power. R0 R RV

RV = 400 Ω, R = 400 W
Sol 8: R A 4
C ⇒ R0 = 200 W
2 Now we see there is a wheat stone bridge formed.
4V 6V
V
10V 400 
B 100 100 200
i4Ω = 0
100 
Given, ⇒ VAB = 6V
But VAB = VR + 4V
10V
⇒ VR = 2V (VR = Voltage across R)
Hence the middle 100 Ω can be ignored
10V – 6V
iAB = = 2A 200 20
2 ⇒ V200Ω = × 10 = V
200 + 100 3
iAB = iR = 2A (∵ i4Ω = 0) 20
\ Voltmeter resonances V
VR 2V 3
⇒R= = = 1W
iR 2A
Sol 11: Resistance of circuit

Sol 9 : When both switches open,  


 1  4
1.5 R = 2  = W
i= A 1+1 3
300 + 100 + 50  
1 2
1 V 30 45
i= × 10–2 A Current i = = = A
3 R 4 2
When both switches closed, 3
V 30
Voltage across R, Voltage across 2Ω = = = 15 V
2 2
1
VR = 1000 × i = V V 15
3 i2Ω = = A
r 2
Current in 200 Ω (i0)
45 15
i1Ω= i – i2Ω = – = 15 A
2 2
2 0 . 6 8 | Current Electricity

Current through AB = i1R –i2R 7


⇒ VAP = V
15 3
= 15 – = 7.5 A
2 R
RAB = 10 ×1m = 10 W
m
Sol 12: Since current is minimum, resistance should be
Let AP = xm
maximum.
RAP = 10xΩ
⇒ All switches should be open to present short circuit.
10x 10x
1 1 VAP = × V= × 10
RAB = = W R AB + 10Ω 10 + 10
1+1 2
VAP = 5 x
Resistance across switch S2:
7
1 9 But VAP = V
R1 = = W 3
1 1 2
+ 7 7
9 6+3 ⇒5x = ⇒x= m = 46.67 cm
3 15
Resistance of circuit
Sol 14: Potential drop across the 40 cm wire
1 9
R = 1 + 6 + R1 + RAB= 7 + + = 12 W
2 2 40
= × 10 v = 4 v
100
V 24
Current in circuit i = = =2A
R 12 Now, 5 – i r = i r = 4
1 ⇒ 5 − i= i r = 4
VAB = i . RAB = 2   = 1V
2 ⇒ i= 1A & R= 4Ω

Sol 13: 10V 10 Sol 15: R1 R2

G
A B
2V A B
1
G 9V
9V
X Y
5 R1 40
= ⇒ 3R1 = 2R2 … (i)
iG = 0 R2 60
⇒ iAX = 0 R1
=1
Note: here we should see XY as a system since current 1
leaving is zero. 1 1
+
R 2 10
⇒ Current entering is zero
VAB = Vxy R 2 (10)
R1 =
10 + R 2
Current in XY system
R1R2 = 10(R2 – R1) … (ii)
4−2 1
ixy = = A
5+1 3 From (i) and (ii)

1 7 10
Vxy = 2 + 1   = V R1 = Ω; R2 = 5W
3 3 3
P hysi cs | 20.69

Sol 16: RA R (b) 2V


V – 0.75
A
5 amp
V A
P
B
RV = 480 x

I = 5 ampere, V = 96 V 1.5V 1
V
 480  V + 1.25 5
Voltage diff. at R= 5   R = 96
 R + 480 
x(3)i = 1.25
25R = R + 480
(10 – x) 3i = 0.75; x = 6.125m
R = 20 W

Sol 18: (a)


Sol 17: 2V dr
V – 0.5

r
10 m
x V R
V + 1.5
1 G
Consider a small circular strip of width dr.
1.5V V Area of strip dA = 2ρrdr
Hence xi = 1.5 Current through strip di = 5 .dA
(v – x)i = 0.5
= J0  1 − r  2πr.dr
X = 7.5 m  
 R
V – 0.5 R
V – 0.5+i(5)  r
i= ∫ di = ∫ J0  1 − R  2πrdr
0
10 m R
A R 2 2 3
B = 2πJ0 (r − r ).dr = 2πJ0 ( r − r )
(a) V + 1.5
x V ∫ R 2 3R
G 0 0
2
 2 3  πJ R
1.5V = 2πJ0  R − R  = 0
 2 3R  3
 
 x 
1.5 = i   (30)  … (i) AJ0
 10  i= (πR2 = A)
3
 10 − x  r
0.5 – i(5) = i   30 (b) Here J = J0
 10  R
R
5 – 50i = i(350 – 30x) R
r 2π J0 3 2
⇒i= ∫ di = ∫ J0 2πrdr= r = πJ R2
5 = i (350 – 30x)  … (ii) 0
R 3 R 0 3 0

From (i) and (ii) 2AJ0


i=
5 = 350 i – 15 3
2
i=
35 Sol 19: Initial resistance R1 = 5R
Putting in (i) Suppose two new similar resistance are added, we get
2 a wheat stone bridge
1.5 = (x) (3); x = 8.75 m
35
2 0 . 7 0 | Current Electricity

R R 7

R P1 = 0.2(4) 2
R R 7
P1
P1 = = 0.1(4) 2
1 2
Resistance of bridge Reff. = =R
1 1
+ Pr= I2r = (0.2)2 . (4)5 .r
2R 2R
\Total resistance R2 = 3R 7
⇒ 6.1 (4) 2 = (0.2)2 (4)5 .r
Change in resistance = 5R – 3R = 2R
3
R1 : R2 = 5 : 3 1 −
⇒r= . (4) 2
0.4
Sol 20: Let resistance be r r = 0.3125W
(i) ⇒ V = V0 – ir
VN
⇒ i = 0.2 ( V0 – ir)5/2 Sol 21: R3 =
VN − 1
Given i = 0.44
VN−1 − VN
⇒ 0.44 = (6 – 0.44r)5/2 × 0.2 R1 =
VN − 1
2
 0.44  5 R1 VN−1 − VN
  = 6 – 0.44r =
 0.2  R3 VN
2
 0.44  5 R1
6− 
 0.2  6 − 1.37 =k–1
r= = = 10.52 W R3
0.44 0.44
V0 − V1 (k − 1) − V1
5 Now R1 = =
(ii) I = 0.2 V2 i i
V1 i
2 ⇒ =
 I 5 R1 (k − 1)
⇒V=  
 0.2  V1 V1
R2 = ⇒ i1 =
Power dissipated in rod P1 = VI i1 R2
7
 k − 1
P1 =
I5  V
V1 − V2  k  1
2 R1 = =
(0.2)5 i − i1 i − i1
3
Total power dissipated P = P  k − 1
2 1  V
V  R  1
Power supplied by battery Pb = V0I i– =
R2 R1
\ Pb = P
V  k − 1 i
7 2 i– =  
3 I5 3 I5
R2  k  R − 11
⇒ VoI= . ⇒6= .
2 2 2 2
V  k − 1
(0.2)5 (0.2)5 = i 
R2  k 
5 VR
⇒I = 0.2 . (4) 2 =6.4 A ⇒ R2 =
i(k − 1)
7 R1 (k − 1)2
 5 5 ⇒ =
R2 k
 0.2(4) 2 
  R2 R2 R1 k
P1 =   = × =
2 R3 R1 R3 (k − 1)
(0.2)5
P hysi cs | 20.71

Sol 22: Power dissipated Pd = 40 watt −


x
x
ρ0 e L
dm (b) rx = ∫ dx
Power generate Pg = n. ×g×h A
dt 0

h = efficiency
(rx = resistance till a distance x)
dm dm
= 0.9 × × 10 × 10 = 90
ρ0L  
x
dt dt −
= 1 − e L 
But Pg = Pd A  
 
dm ΔV= rx. I
⇒ 40 = 90
dt
ρ0L   ρ A
x
dm 4 − 1
⇒ = kg/s = 1 − e L . 0
dt 9 A   Lρ0 (1 − e−1 )
 
Mass of water = V × d
 − 
x
= 200 × 1 (1 litre = 1 kg)= 200 kg V0  1 − e L 
 
200
=
4 =  
−1
T 9 (1 − e )
T = 450 s = 7.5 minute
DV = V0 – V(R) (V(x) = Potential at x)

3 ⇒ V(R) = V0 – ΔV
Sol 23: (i) Va = × 36 = 12 V
3+6
6  − 
x
Vb = × 36 = 24 V V0  1 − e L 
3+6  
= V0 –  
−1
V 36 1−e
(ii) Va = = = 18 V
2 2
 −x 
 e L − e−1 
18 V(R)= V0 
i6Ω = = 3A −1 
6  1−e 
 
18
i3Ω = = 6A
3 Sol 25 : Voltage at 6.9 m
iab = i3Ω – i6Ω =6 – 3 = 3A 6.9
V1 = × 2 = 1.38 V
x
10

ρd ρ e 2 But V1 is voltage of another cell
Sol 24 : (a) dr= ;dr = 0 .dx
A A ⇒ Vcell = 1.38 V


x
L
The null point is V = 1.38 V
x e L
− Resistance of wire r w = ρ × l = 11.5 × 10 = 115 W
L ρ0 e L dx ρ0
r = ∫r = ∫0 = 0
Resistance at a distance R,
A A 1

L rx = 11.5 x
ρ0L Lρ0  e − 1  Voltage at x
r= . (1 − e−1 ) =  
A A  e  rx 11.5 × x 11.5x
Vx = ×V = (ii) =
rw + 5 120 60
V V0 V0 Ae
I= = =
r Lρ0  e −1 Lρ0 (e − 1) But Vx = Vcell
 
A  e 
11.5x
⇒ 1.38 = ⇒ x = 7.2 m
60
2 0 . 7 2 | Current Electricity

Exercise 2 E1 E1 + E2
∴ >
r1 + R r1 + R + r2
Single Correct Choice Type
E1r1 + E1R + E1r2>E1r1+E1R +E2r1 +E2R
Sol 1: (D) We have two types of charges –e and +2e E1r2> E2(R + r1)

+e Vd V2 (3)2
Vd  –e Sol 3: (B) Power initially = =
R R

When a potential different is applied both the charges


drift in opposite directions.
L
+x
Now when length is doubled,
\i = neVdA
Pfinal = 2 Pinitial,
(Vd ) neVd A
i(+ve) = (n) (2e) A=
4 2  3L 
Pfinal = 2 Pinitial, R = 
i(–e) = n(–e) (–Vd) . A = neVdA  A

itotal = i+2e + i–e V12 (3)2


∴ =2.
Rf R
3
i= neVdA
2 ∵V= 4 × (3)2 = 2 × 3
Try to understand why they are getting summed up!
N=6
Sol 2: (B) Initially
Sol 4: (A) A B
Writing the KVL;

60 A i1 60
E1 E2
r1 r2 i
i2 120
V
R B
Reff = 60 Ω + (60 || 120)= 60 Ω + 40 W
–E1 + ir1 – E2 + ir2 + iR = 0 Reff = 100 W
 E + E2  120 Volt
⇒ i =  1  amp i= = 1.2 amp
 r1 + r2 + R  100Ω

Now when E2 is short circuited; 60 i1 = 120 i2

–E1 + i1r1 + i1R = 0 i1 = 2i2  …..(i)

 E  and also at junction A; i = i1 + i2


⇒ i1 =  
 r1 + R  i = 2i2 + i2
Now given that i1>i
i2 = i = 0.4 amp
3
∴ V across voltmeter = (0.4) (120) = 48 V
P hysi cs | 20.73

Sol 5: (B) Now we can remove resister OO’ because of VP – VQ = 5i2 – i1 –1= 5 – 2 – 1
symmetry property.
VPQ = 2V

A
Sol 7: (A)
L, 2A
R O O’ 2L, A
R R A O B

ρ.2L ρ.2L
B R1 = R2 =
A 2A

 ρL  1  ρL 
R1 = 2   ; R2 = R x ∴ R = 
R/2 A 2  A

R R1 = 2R
2R/3
VA – VB 8 −1 7  14R 
i= = = =  
R1 + R 2
2R +
R 5R  5R 
2R/3
2R/3 2 2
VA – iR1 = V0
2R/3
B  14 
8 –   (2R) = V0
 5R 
A 28
V0 = 8 – = 2.4 V
5

P EeVd
7R/3 7R/3 = (7R/6) Sol 8: (C) =
V V
P
J = neVd ⇒ = EJ
V
B

Sol 6: (B) In Mesh A, Applying KVL; Sol 9: (C) IR = ir

2 I = 10 mA
Q
R = 9Ω + 0.9 Ω = 9.9 W
4 3
(A) (B) r = 0.1 W
2 10 × 9.9
1 i2 1 ⇒i= = 990 mA
P i1 O i X 0.1
Total current = i + I = 990 + 10 = 1A
i1 – 4 + 2i1 – 2 = 0
Sol 10: (C) For wheatstone bridge,
3i1 = 6
R 1R 4 = R 2 R 3
⇒ i1 = 2 amp
Now same for mesh 2;
R1 R2
– 2i2 + 1 – 3i2 + 4 = 0
⇒ i2 = 1 amp
Now applying voltage drop along POXQ; R3 R4

VP + i1 (1) – 2i2 + 1 – 3i2 = VQ


2 0 . 7 4 | Current Electricity

V0 20
It is independent of emf Sol 14: (C) 1f V was ideal, R = = = 5W
i0 4
Let r = KR
\ V is not ideal, i< i0
⇒ r1r4 = K2(R1R4) = K2(R2R3) = r2r3
⇒ R > R0
So it is still balance
⇒ R > 5W
Even if battery and galvanometer are interchanged, still
it is balanced. Multiple Correct Choice Type

Sol 11: (B) 625(P) = QS Sol 15: (A, B, C, D) Current is constant across the
(625 + 51) = PS cross-section.

P Q P 676 1 1
⇒ 625 . = 676   ; ⇒ = E∝ ; r∝
A A
Q P Q 625
1
P 26 ⇒ i2r∝
= A
Q 25
\ Heat at Q > heat a P
P 26
S = 625 = 625 × = 650 Ω
Q 25
Sol 16: (A, D)

Sol 12: (D) Number of free electrons is constant for 1V 2V


ohmic resistor.
i2
II
Sol 13: (A) i
5 C 3 A 3 D B
I
i1
3V 1V
28V 10 10 4
4 4 4
B
Let VD = 0
D
1
RAD = = 5W 3V – 1 V = i1 (3 + 1) + 2I (for loop I)
1 1
+
10 3 + 4 + 3 1V – 2 V = i2 (2 + 1) + 2I (for loop II)
1 I = i1 + i2
1 1 ⇒ 2V = i1(4) + 2(i1 + i2)
RCB = + = 5W
10 3 + 5 + 2
⇒ 6i1+2i2= 2  … (i)
28 – 1V = 3i2 +2 (i1 + i2)
i= = 2A
5+5+ 4
⇒ 2i1+ 5i2 = – 1  … (ii)
Current through 5Ω is 2A −5
⇒ i2 = A (from i and ii)
RCADB = 3+5+2 = 10W 13
6
∵ RCADB = RCB i1 = A
13
i VB – VD = 2(I) = 2(i1 + i2)
⇒ iCA = iCB = = 1A
2
 6 5  2
VCA = 1 × 3 = 3V = 2 −  = V
 13 13  13
VCB = 1 × 10 = 10V
−2
VAB = VCB – VCA = 10 – 3 ⇒ VD – VB = V
13
VA – VB = 7V
P hysi cs | 20.75

6 21 Sol 22: (D) If a battery of higher emf is placed across


DVG = EG – i1BG= 3V – (3) V = V it, e.m.f. of given battery < potential difference across it
13 13
6 19
∆’VH = EH + i1RH= 1 + (1) = V Sol 23: (A) Statement-II explains statement-I
13 13

Comprehension type

Sol 17 : (B, C) Let range of voltage be V Paragraph 1


Let resistance added be R
dA
Sol 24: (B) E = B
V = i(R + r) dt
For R >> r,  60º
V = iR;i = 50µA = 50 × 10 A -6
A1 x
For 200 kΩ, V = 10V
For 10 kΩ, V = 0.5V
Let range of ammeter be I
Let resistance in series be R
Voltage V = 50 × 10-6 × 100= 5 mV A = 4A1
V 1
For i = (R << r) A= . x ( x tan θ)
R 2
For R = 1Ω, I = 5mA
x2  60º 
For R = 1 kΩ, R >> r, so cannot be ammeter = tanθ =
θ = 30º 
2  2 
dA1 dx
Sol 18 : (A, B, C) For R = 120 W = x tan θ – = xv tanq
dt dt
Potential across potentiometers
x = a cosq
75(20) 75(20)
DVP = = = 7.5 V dA1
75 + 5 + 120 200 ⇒ = av sinq
dt
For V <DVP
dA
V can be measured = 4av sinq
dt
dA
Sol 19 : (A, D) Current is constant, hence charge ⇒E=B = 4avBsinθ= 4avBsin30º= 2avB
dt
crossing cross section
J = nevd E 2BaV
Sol 25: (B) i = =
J is not constant ⇒ vd variable R R
dA
B
Sol 20: (A, D) Ammeter should have small resistance Sol 26: (A) i = dt
and voltmeters large R

dQ B dA
= .
Assertion Reasoning Type dt R dt

iR 2 B B
Sol 21: (D) JE = (V = volume) ⇒DQ = ∆A = (a2)
V R R
⇒ E ∝ R2
a2B
∴ Statement-I is false DQ =
B
2 0 . 7 6 | Current Electricity

Paragraph 2 Sol 4: (C) Current in the respective loop will remain


confined in the loop itself.
Sol 27: (A) Just after pressing switch, I2 = 0 as inductor Therefore, current through 2Ω resistance=0
doesn’t pass current through it initially.
V 10 10
∴I1 = I3 = = = A Sol 5: (A) Current flowing through the bars is equal.
R1 + R 2 2+6 8
Now, the heat produced is given by
Sol 28: (A) After long time, inductor acts as short H = I2Rt or H ∝ R
circuit.
HAB R AB (1 \ 2r)2 1 1
R 3 I1 or = = (as R∝ ∝ )
∴I2 = HBC RBC (1 \ r) A r2
R3 + R 2
1
E 10 = or HBC = 4HAB
I1 = = = 2.5 A 4
R 2 × R3 3× 6
R1 + 2+
R 2 + R3 3+6
Sol 6: (B) . τ =CR

6 × 2.5  R 3 I1  τ (C1 + C2 )(R1 + R 2=


= ) 18 µs
Sol 29 : (D) I2 =  I2 = 
9  R 2 + R 3 
 C C  R R  8 2 8
τ2 =  1 2   1 2  = × = µs
5
I2 = A  C1 + C2   R1 + R 2  6 3 9
3
 RR  2
τ3 = (C1 + C2 )  1 2  = (6)   = 4µs
Sol 30: (C) Even after releasing switch, inductor still
 R1 + R 2  3
tries to continue the same current
10 Sol 7: (C) From Y to X charge flows to plates a and b.
⇒ I2 = A
6
(qa + qb )i =0, (qa + qb )f =27µC

Match the Columns 3µF 6µF


a b
+ – + –
Sol 31 : A → p; B → q; C → r; D → p 18µC 18µC
3 6

Previous Years’ Questions


9V
Initial Figure
5 4 3
Sol 1: =
(A) RPQ r and RPR = r
(When switch was open)
= r, R QR
11 11 11 3µF
a X b
6µF
+ – + –
∴ RPQ is maximum. 9µC 36µC
3 1A 6
Y
Sol 2: (C) BC, CD and BA are known resistance.
The unknown resistance is connected between A and D. 9V
Final Figure
S (When switch is closed)
Sol 3: (A)
(i–ig )
∴ 27 µC charge flows from Y to X.
. a G
ig b 2
Sol 8: (C) Applying P = V , R1 =1 Ω , R2 = 0.5 Ω
Vab= ig .G= (i − ig ) R
 G and R3= 2 Ω
∴ i =+
1 i
 Sg V1 = V2 = V3 = 3V

Substituting the values, we get, i = 100.1 m A (3)2


∴ P1 = =9W
1
P hysi cs | 20.77

(3)2 (3)2 10
P2
= = 18 W and=
P3 = 4.5W For option (B)
= R − 100 ≈ 200 kΩ
0.5 2 50 × 10−6

∴P2> P1> P3
Sol 13: (A, B, D) At 0 K, a semiconductor becomes
ρ(L) ρL ρ a perfect insulator. Therefore, at 0 K, if some
Sol 9: (C) R = =
R = = potential difference is applied across an insulator or
A tL t
semiconductor, current is zero. But a conductor will
i.e. R is independent of L. become a super conductor at 0 K. Therefore, current
will be infinite. In reverse biasing at 300 K through a
Sol 10: (D) With increase in temperature, the value of p-n junction diode, a small finite current flows due to
unknown resistance will increases. minority charge carriers.
R l1
In balanced Wheat stone bridge condition, =
X l2 6 × 1.5
Sol 14: (A, D) Rtotal= 2 + 3.2 kΩ
=
Hence R = Value of standard resistance 6 + 1.5
X = value of unknown resistance 24 V
(a) I
= = 7.5mA
= IR
l1 3.2kΩ 1
To take null point at same point or to remain
l2  RL 
R IR = 
unchanged should also remain unchanged.  I
 RL + R 2 
2
X
Therefore, if X is increasing R, should also increase. 1.5
I= × 7.5 = 1.5 mA
7.5
+ –
Sol 11: (B, D) The discharging IR = 6mA
L
−τ /CR
current in the circuit is, i = i0 e
R (b) VR
= (I=
R )(RL ) 9V
L L
V
Hence, i0 = initial current =
R PR IR2 R1 (7.5)2 (2) 25
(c)= 1
= 1
=
Here, V is the potential with which capacitor was PR I 2
R (1.5)2
(6) 3
2 R 2
charged. 2

Since, V and R for both the capacitors are same, initial (d) When R1 and R2 are inter changed then
discharging current will be same, but non-zero.
R 2RL 2 × 1.5 6
= = kΩ
Further, τc = CR R 2 + RL 3.5 7
C1< C2 or τC < τC Now potential difference across RL will be
1 2

or C1 loses its 50% of initial charge sooner the C2.  6/7 


= VL 24 =   3V Earlier it was 9V
6 + 6 / 7 
Sol 12: (B, C) To increase the range of ammeter a
parallel resistance (called shunt) is required which is V2
Since,
= P or P ∝ V 2
given by R
In new situation potential difference has been decreased
 i 
S =  x G three times. Therefore, power dissipated will decrease
 i − ig  by a factor of 9.
 
For option (C)
Sol 15: (A, B, C, D) The circuit can be simplified as
 50 × 10 −6 
S=   (100) = 1Ω 2 P 2 S 2
 5 × 10−3 − 50 × 10−6 
 
1 1
To change it in voltmeter, a high resistance R is put in
V 4 Q 4 T 4
series where R is given by R = −G
ig
12V
2 0 . 7 8 | Current Electricity

Because of symmetry no current through 1W I λσ  λ0 σ  −( σ / ε )t


So,
=j = =   e
2πr 2πεr  2πεr 
2 P 2 S 2 P 6 S

dr
4 Q 4 T 4 12

I1 12V I1 12V
r
12 × 6
R e q= = 4Ω
12 + 6

12 2
I1 = = 3A ; I2 = × 3 = 2A
4 3

Here we have
Sol 19: (A, C) For maximum voltage range across a
VS − VQ =
−4 i.e., VS < VQ galvanometer, all the elements must be connected in
series.
For maximum current range through a galvanometer,
R1 (V1 + V2 ) all the elements should be connected in parallel.
Sol 16: (A, B, D)
= V1 ⇒V
=1R 3 V2R1
R1 + R 3
R3 (V1 + V2 ) Sol 20: (A, B, C, D) At t = 0 voltage across each
V2
= ⇒V
=2R1 V2R 3
R1 + R 3 capacitor is zero, so reading of voltmeter is -5 Volt.
At t = ∞ , capacitors are fully charged. So for ideal
x
Sol 17: (C) R = 90 voltmeter, reading is 5 Volt.
100 − x
At transient state.
∴ R =60 Ω
t t
dR 100 5 −τ 5 −τ
= dx I1= e mA, I2= e and =
I I1 + I2
R (x)(100 − x) 50 25

100
dR
∴ = 60 0.25 Ω
0.1 ×= I2 40 F I2 25 k
(40)(60)
-
Sol 18: (C) For infinite the, V
+
λ I1
E=
2πεr
I 50 k I1 20 F
−λ
⇒ dV = dr
2πεr
Current through an elemental shell ; 5V

λ Where τ = 1 sec
dr
| dV | λσ
=I = 2πε= r
So I becomes 1/e times of the initial current after 1 sec.
dR 1 dr ε
×
σ 2πr The reading of voltmeter at any instant
This current is radially outwards so ;  − 
t

t
∆V40µf − ∆V50k=
Ω 5  1 − e τ  − 5e τ

d −λσ dλ σ  
 
( λ ) = ⇒ =−   dt
dt ε λ ε So at t = log2 sec, reading of voltmeter is zero.
⇒ λ=λ
0e−( σ / ε )t
2017-18 100 &
op kers
Class 12 T
By E ran culty
-JE Fa r
IIT enior emie .
S fP r es
o titut
Ins

PHYSICS
FOR JEE MAIN & ADVANCED
SECOND
EDITION

Exhaustive Theory
(Now Revised)

Formula Sheet
9000+ Problems
based on latest JEE pattern

2500 + 1000 (New) Problems


of previous 35 years of
AIEEE (JEE Main) and IIT-JEE (JEE Adv)

5000+Illustrations and Solved Examples


Detailed Solutions
of all problems available

Topic Covered Plancess Concepts


Tips & Tricks, Facts, Notes, Misconceptions,
Moving Charges Key Take Aways, Problem Solving Tactics
and Magnetism
PlancEssential
Questions recommended for revision
21. MOVING CHARGES
AND MAGNETISM

1. INTRODUCTION
In the previous chapters on electrostatics and current electricity, we have studied about the electric force and electric
field. Another important property associated with moving charges is the magnetic force and the magnetic field.
The current flowing in a conductor produces a magnetic field and any charge moving in this field will experience a
magnetic force which will depend on the velocity (both magnitude and direction) as well as on some property of
the field. We will study the properties and laws governing the magnetic field and magnetic force in detail in this
chapter.
There are a wide variety of industrial and medical applications of magnetic fields and forces. Common example, is
the use of electromagnet to lift heavy pieces of metal. Magnets are used in CD and DVD players, computer hard
drives,loud speakers, headphones, TVs, and telephones. We are surrounded by magnets. Right from our doorbells
to cars to security alarm systems and in our hospitals, magnets are being used everywhere.

2. LORENTZ FORCE: DEFINITION OF MAGNETIC FIELD B


If electric field and magnetic field occur simultaneously in a region then the force acting on a point charge q in

the region will depend both on the position of the charge as well as on its velocity. The force F will have two
  
components, viz. the electric force Fe and magnetic force Fm . The force Fe does not depend on the motion of

the charge, but only on its position, while Fm depends both on charge’s velocity and position (see Fig. 21.1). The
 
magnitude of Fe is qE and direction is along E (q is positive).
 
To know the direction and magnitude of Fm we introduced a vector B called magnetic flux density ormagnetic
induction, which characterizes the magnetic field at a particular
 Y
point. Experiments show that the force Fm isproportional to the

magnitude of charge q, to the velocity v of the charge and the B

magnitude of density B , this force being always perpendicular
 
to vector v as well as vector B . Also, if the charge moves along
 +q
the direction of B at a point then the magnetic force on it is zero.
O X
We can summarize all these experimental results with the
following vector equation: F = q( x B)
   Z
Fm = q v xB

That is, the force Fm on the point charge is equal to the Figure 21.1: Magnetic Force on a Moving Charge
2 1 . 2 | Moving Charges and Magnetism

 
charge q times the cross product of its velocity v and the field B (all measured in the same reference frame). Using

formula for the magnitude of cross product, we can write the magnitude of = Fm as Fm q vB sin θ where θ is the
 
angle between the velocity v and magnetic field B .
If angle θ is 90o, then the above relation for magnetic force can be used to define the magnitude of magnetic flux
density B as,
Fm
B =
q v⊥

where v ⊥ is the velocity component perpendicular to vector B .
  
Thus, the total electromagnetic force acting on charge q is given as, F= Fe + Fm
   
or F = qE + q[v × B]

This is called Lorentz force.

The unit of B is Tesla abbreviated as T. If q=1 C, v=1 ms-1, sin θ=1 for θ = 90o, and Fm = 1 N,then B=1 T = 1
Weber-m-2. Thus 1 Tesla is defined as the unit of magnetic field strength in S.I units which when acting on 1 C of
charge moving with a velocity of 1 ms-1 at right angles to the magnetic field exerts a force of 1 N in a direction
perpendicular to that of field and velocity vectors. C.G.S. units of magnetic field strength or magnetic induction is
1 gauss or 1 oersted. 1 gauss = 1 oersted= 10-4T.

Illustration 1: A 2 MeV proton is moving perpendicular to uniform magnetic field of 2.5 T. What is the magnetic
force on the proton? (Mass of proton = 1.6 x 10-27 kg) (JEE MAIN)

mp v 2
Sol: Kinetic energy of proton is K.E. = . 1 MeV=1.6 x 10-13J.
2
1
K.E = 2 MeV = 2 x 1.6 x 10-13J or mv2= 3.2 x 10-13J
2

2x3.2x10−13 2x3.2x10−13
∴V= = = 2 x 107 m s-1
m 1.6x10−27
Now, magnetic force on proton, F= ev B = 1.6 x 10-19 x 2 x 107 x 2.5 = 8.0 x 10-12 N


Illustration 2: A charged particle is projected in a magnetic field = B (3i + 4j)x10−2 T

The acceleration of the particle is found to be,=a (xi + 2j) ms−2 Find the value of x.  (JEE MAIN)

Sol: Magnetic force on a moving charge is perpendicular to the magnetic field. Therefore the dot product of force
and magnetic field vector is zero.
     
As we have read Fm ⊥ B i.e., the acceleration a ⊥ B or a ⋅B =0
8
or (xi + 2 j) ⋅ (3i + 4 j) x10 −2 =
0 ; (3x+8)x102=0 ∴ x= − ms−2
3

3. RELATION BETWEEN ELECTRIC AND MAGNETIC FIELD


Suppose in a particular inertial reference frame K, the electric field is zero and the magnetic field has a non-zero

finite value. A point charge is moving with some velocity v in the frame K and thus experiences a magnetic force,
and its velocity changes. Now suppose we have a frame K’ translating with respect to frame K withconstantvelocity

v . In the frame K’,the point charge is initially at rest, and so the magnetic force on it will be zero. Butas its velocity
changes in the K frame, its velocity changes in the K’ frame as well, i.e. it experiences a force in K’ frame as well.
P hysi cs | 21.3


This initial force on it is the force Fe due to electric field in the K’ frame.Thus the magnetic field in K frame appears
as a combination of electric field and magnetic field in K’ frame. The electric and magnetic fields are thus inter-
dependent. We introduce a single physical entity called electromagnetic field. Whether the electromagnetic field
will appear as electric field or magnetic field depends on the frame of reference. If we confine to a particular
reference frame, we can treat electric fieldand magnetic field as separate entities. A field which is constant in one
reference frame in the general case is found to vary in another reference frame.

4. MAGNETIC FIELD LINES


Magnetic field lines are used to represent the magnetic field in a region. The rules to construct the magnetic field
lines are:-
(a) The
 direction of tangent to a magnetic field line at a point gives the direction of magnetic flux density vector
B at that point.

(b) The density of the magnetic field lines at a point isproportional to the magnitude of vector B at that point. At
points where the field lines are closer together, the magnetic field is stronger.

PLANCESS CONCEPTS

•• In case of a bar magnet, the density of magnetic field lines is high at points near the poles, and the
density at pointsnear the center of the magnet is low.
•• If we place a magnetic compass at any point in the earth’s magnetic field, it will align itself in the
direction of the magnetic field lines.
Vaibhav Krishnan (JEE 2009 AIR 22)

PLANCESS CONCEPTS

•• Common misconception about magnetic field lines is that it is the path followed by a magnetic
north pole in a magnetic field.
•• This is not correct. It is the instantaneous direction of the magnetic force acting on the magnetic
north pole in the magnetic field.
Vaibhav Gupta (JEE 2009 AIR 54)

5. EARTH’S MAGNETIC FIELD


Magnetic field is present everywhere near the earth’s surface. The line of earth’s magnetic field lies in a vertical plane
coinciding with the magnetic north-south direction at that place i.e. the plane passing through the geomagnetic
poles. This plane is called the Magnetic Meridian. This plane is slightly inclined to the plane passing through the
geographic poles called the geographic meridian.The angle between the magnetic meridian and the geographic
meridian at a point is called the declination at that point. The earth’s magnetic poles are opposite to the geographic
poles i.e. at earth’s North Pole, its magnetic south pole is situated and vice versa.
In the magnetic meridian plane, the magnetic field vector of the earth at any point, is generally inclined to the
horizontalat that pointby an angle called the magneticdip at that point. If magnetic field of the earth at that point
is B and the dip is θ,
2 1 . 4 | Moving Charges and Magnetism

Bv = the vertical component of B in the magnetic meridian plane = B sinθ


BH = the horizontal component of B in the magnetic meridian plane = B cosθ.
BV
= tan θ
BH

6. MOTION OF CHARGED PARTICLE IN ELECTRIC AND MAGNETIC FIELD


6.1 Trajectory of a Charged Particle Moving in Uniform Electric Field
Let a positively charged particle having charge +q and mass m
Charged
enter at origin O with velocity v along X-direction in the region Y P
Plate
where electric field Eis along the Y-direction (see Fig. 21.2).
E
Force acting on the charge +q due to electric field E is given by – – – –
  O´
F = qE y
  +q
 F  qE
Acceleration of the charged particle
= is a = or a ...(i) O v C X
m m 
 Charged
The charged particle will accelerate in the direction of E and get + + + + Plate
deflected from its straight line path. x Screen

During its motion in the region of electric field, along x-axis we


Figure 21.2: Charged particle moving
have ux=v and ax= 0 and x = vt
in electric field
x
or t= …(ii)
v
qE
Along y axis we have, uy = 0, ay = (∴ Initially the particle was moving along x-direction)
m
1 2
y = a t
2 y

1  qE  2
∴ y=  t
2 m 
2
1  qE   x  qEx2
Using Eq, (ii), we get y =     or =y = Kx2 …(iii)
2  m  v  2mv 2
qE
where K= is a constant.Thus the charged particle moves along a parabolic trajectory.
2mv 2

6.2 Trajectory of a Charged Particle Movingin Uniform Magnetic Field



(a) Magnetic force acting on a charged  particle moving with velocity v parallel
x
x
(θ=0) or antiparallel (θ=180o) to B , will be zero. Thus the trajectory of the x
particle is a straight line. x B
x x
  x
(b) If velocity v of the particle is perpendicular to B i.e. θ =90o, then magnetic x
force is F= qvB and the direction of this force is always perpendicular to v. The x x
charged particle moves in a circular trajectory (see Fig. 21.3). x x
  q x x x
(c) If velocity v of the charged particle makes an angle θ with B , the particle 
moves in a helical path. The component v sin θ which is perpendicular to B
drives the charged particle along a circular  path whereas the component v Figure 21.3: Charged particle
cos θ , which is parallel or antiparallel to B , remains unchanged as there is moving in uniform magnetic
no magnetic force along the direction of B . Thus the charged particle moves field in electric field
along a helical path (see Fig. 21.4).
P hysi cs | 21.5

(d) The magnetic force on the component of velocity perpendicular to the magnetic field provides the centripetal
force to the charged particle to follow a circular trajectory of radius r.

mv 2⊥
qqv
v ⊥⊥ B =
r
mv ⊥
or r =
qB v sin  v
v ⊥ qB
Angular velocity, =
ω =
r m

qB 
Frequency f = B
2πm
Figure 21.4: Charged particle moving in
2πm helical path in uniform magnetic field
Time period T=
qB

Time period T is independent of v.

7.DISCOVERY OF ELECTRON

The Fig. 21.5 shows the simplified version of Thomson’s’ experiment. An electric field E is established
 in the region
between the deflecting plates by connecting a battery across their terminals. The magnetic field B in the region
between the deflecting plates is directed into the plane of the figure.

+
E
B Spot of
Filament light
x x x x x
x x x x x
Screen C

– Screen S
Glass
envelope
v
To vacuum pump

Figure 21.5: Thomson’s experimental set up

Charged particles (electrons) are emitted by a hot filament at the rear of the evacuated cathoderay tube and are
accelerated by an applied potential difference V. After they pass through a slit in screen C, they form a narrow
beam. They then pass through the region between the deflecting plates, headed towards the center of fluorescent
 
screen S, where they produce a spot of light. The crossed-fields E and B in the region between the deflecting

plates can deflect them from the center of the screen. By controlling the magnitude and directions of the fields, E

and B the deflection of the charged particles can be controlled.
 
When both the fields E and B are turned-off the beam of charged particles reaches the screen un-deflected.

When field E is turned-on the beam of charged particles is deflected.
  
Keeping the field E unchanged, field B is also turned-on.The magnitude of B is adjusted such that the deflection
2 1 . 6 | Moving Charges and Magnetism

of the charged particles becomes zero. In this situation the electric force on the charged particles is balanced by
the magnetic force.
  
q E = −q v ×B
  
or E =− v × B
 
The ratio of magnitudes of E and B in this situation gives the speed of the charged particles.
E
v =
B

When only field E is turned-on, the displacementof the charged particlesin the y-direction, when they reach the
end of the plates, as derived in article 6.1 is

q EL2
y =
2m v 2

where v is the particle’s speed along x-direction, mits mass, qits charge, and L is the length of the plates. The
direction of deflection of charged particles show that the particles are negatively charged.
Substituting the value of v in terms of E and B we get,

q B2 L2
y =
2mE

m B2 L2
or =
q 2 yE

Thus in this way the mass to charge ratio of electrons was discovered.

PLANCESS CONCEPTS

Charged particle motion as a points on wheel


•• 1. Suppose electric and magnetic field are perpendicular to each other and a charged particle is
projected perpendicular to magnetic field, its motion can be assumed as that of the motion of a
particle on a wheel
•• 2. The point could be inside, on or outside the wheel depending on the problem
•• 3. Suppose in this field it is projected in any other way (expect along the magnetic field) its horizontal
motion is still like that of a point on a wheel, while vertical motion will be uniform velocity motion
•• 4. To such problem, just resolve the particle velocity in to along the magnetic field and perpendicular
to it
•• 5. If electric field is not perpendicular, resolve it also into along and perpendicular to magnetic field
and solve accordingly.
Nitin Chandrol (JEE 2012 AIR 134)

8. HALL EFFECT
The Hall Effect is the production of a voltage difference (the Hall voltage) across acurrent carrying conductor, lying
in a magnetic field perpendicular to the current. The hall voltage is produced in the direction transverse to the
electric current in the conductor. It was discovered by Edwin Hall in 1879.Hall Effect allows us to find out whether the
P hysi cs | 21.7

charge carries in a conductor are positively or negatively charged


i i
and the number of charge carries per unit volume of the conductor.
 d
External magnetic field B , points into the plane of a copper strip +

-
E
of width d, carrying a current I as shown in Fig. 21.6.The magnetic
  
force Fm will act on each drifting electron, towards the right edge B B + -
of the strip. As the electrons accumulate on the right edge, positive + -
  
charges are induced on the left edge and an electric field E is vd vd
+ -
 
produced within the strip, directed from left to right.This field FB

FC FB

exerts an electric force Fe on each electron, towards the left edge + -

of the strip.The hall potential difference V across the width of the



strip,due to the electric field E isV=Ed.
When the electric and magnetic forces balance each other, eE=evdB Figure 21.6: Hall Effect in conductor
or E = vd B
J I
Thedrift speed vd is given as v=
d =
ne neA

BI Cross − sec tion Area


So we obtain n = where  (= ) is the thickness of the strip.
Ve Width

Illustration 3:Copper has 8.0 x 1028 conduction electrons per metre3. A copper wire of length 1 m and cross-
sectional area 8.0 x 10-6 m2 carrying a current and lying at right angle to magnetic field of strength 5 x 10-3 T
experiences a force of 8.0 x 10-2N. Calculate the drift velocity of free electrons in the wire. (JEE ADVANCED)

Sol: If v is the drift speed of electrons then the magnetic force on the wire is
o
=F qvBsin
= θ qvBsin90
= qvB
where q is the total charge of electrons in the wire.
n=8.0 x 1028 m-3
l= 1 m; A=8.0 x 10-6m2
Charge on each electron, e=1.6 x 10-19 C
Number of electrons in the copper wire = n x volume of wire = n(A l)
Total charge in the wire, q=n((A l)e or q=8.0 x 1028 x 8.0 x 10-6 x 1 x 1.6 x 10-19=1.024 x 105C

F 8.0x10−2
Using F=qvB sin θ, we have,
= ν = = 1.563 x 10-4m s-1
qBsin θ 1.024x105 x5x10−3 x sin90o

9. MAGNETIC FORCE ON A CURRENT CARRYING WIRE


Suppose in a conductor number of free electrons per unit volume is n, then in an infinitesimal volume dV in the
conductor, the total charge of free electrons will be
dq = ne dV
 
If the magnetic field at the location of the elementary volume is B , and the drift velocity of free electrons is v d then
the magnetic force on the elementary volume will be
  
=dF ne[v d × B]dV
Now we know that the current density is given as
2 1 . 8 | Moving Charges and Magnetism

 
j = nev d
  
So dF= [ j × B]dV
   
Introducing the vector d  in the direction of current we can write, j dV =∆ j S d =I d  . Here ΔS is the area of
cross-section and d the length of the elementary volume dV.
  
So = dF I [d  × B]
  
The total magnetic force on the conductor= is F I ∫ [d  × B]

For a thin straight wire of length L, if the field B is constant throughout the length of the wire and perpendicular
to it, we can write
F = I L B
   
In vector form we can write, =
F I L ×B , where L is a length vector that has magnitude L and is directed along the
wire segment in the direction of the (conventional) current.
L’
Few important points regarding the force on current carrying conductor -
in magnetic field are given below: L

(a) In a uniform magnetic field the force, dF= IBd sin θ , does not I

depend on the position vector r of the current element.Thus
this force is non-central. (Acentral force is a function of position Figure 21.7: Current carrying conductor in
  
uniform magnetic fieldr
vector r , F = f(r ) )
 
(b) Theforce dF is always perpendicular to the plane containing B
  
and d  .Vectors B and d  may or may not be perpendicular to
each other. I
(c) As explained above, the total magnetic force on the conductor is
  
= F I ∫ [d  × B]
 I
For uniform magnetic field, B can be taken out from the integral. -
   Fnet = 0
= F I  ∫ d   × B
 Figure 21.8: Current carrying loop in
According to the law of vector addition ∫ d  is equal to the length
 uniform magnetic field
vector L from initial to final point of the conductor as shown in
Fig. 21.7. For a conductor of any arbitrary shape the magnitude

of vector L is different from the actual length L’ of the conductor.
  
∴ F = IL xB
(d) For a current carrying closed loop of any arbitrary shape placed 
in a uniform magnetic field (see Fig. 21.8), I -
B
  
= F I  �  ×B 0
∫ d =

Uniform
field

Here as we add all the elementary vectors d  around the closed -
loop, the vector sum is zero because the final point is same as Fnet = 0
the initial point. net = 0
-

 Figure 21.9: Area vector of closed loop


∴ �∫ d  = 0 is in direction of uniform magnetic field
Thus the net magnetic force on a current loop in a uniform
magnetic field is always zero.
However, different parts of the loop do experience different net forces, although the vector some of all these
P hysi cs | 21.9

forces comes out to be zero.


So the loop may experience some infinitesimal contraction or
expansion, thus may be under tension. I
Although the resultant of magnetic forces acting on the loop is
zero, the resultant torque due the magnetic forces may not be zero.
Thus the torque on a loop in a uniform magnetic field is not always
-
zero. Fnet = 0
(e) When a current carrying closed loop is placed in a non-uniform  0
-
net

magnetic field, in the general case it will experience non-zero net


force as well as net torque. Figure 21.10: Area vector of closed loop
is perpendicular to uniform magnetic field
Even a conductor of arbitrary shape not forming a loop, will
magnetic field
experience a torque in a non-uniform field.
If the conductor is free to move, it will execute combined
translational and rotational motion.
(f) When a current carrying conductor or closed loop translates or rotates in a magnetic field, the kinetic energy
gained by it is, not due to the work done by magnetic forcesbut, at the expense of the energy supplied by the
electric source which is maintaining current in the conductor/loop.

I1 I1
I2 I2
-
Fnet

- -
Fnet  0 Fnet  0
 =0
-
net  0
-
net

Figure 21.11: Closed loop in non-uniform magnetic field

The net work done by magnetic forces acting on a current carrying conductor is zero.
Though it may appear that,
    
W= ∫ F.dr =∫ [ I ∫ (d  × B)].dr =
∆K

but actually the kinetic energy is supplied by the electric source.

Illustration 4: A wire 12 cm long and carrying a current of 2 A is placed perpendicular to a uniform magnetic field.
If a force of 0.8 N acts on it, calculate the value of the magnetic induction. (JEE MAIN)

Sol: This problem can be solved using formula


= F BI sin θ for force on current carrying wire in uniform magnetic
field.
 = 12 cm = 12 x 10-2 m ; I = 2 A ; F = 0.8 N; θ =90o

F 0.8
Using, F= BIlsin θ , we=
get B = = 3.3 T
I  sin θ 2x12x10−12 x sin90o
2 1 . 1 0 | Moving Charges and Magnetism

9.1 Fleming’s Left Hand Rule


If the thumb and the first two fingers of the left hand are stretched mutually perpendicular to each other and if the
first finger points in the direction of the magnetic field and the second middle finger points in the direction of the
current in the conductor, then the direction of thumb gives the direction of force on the conductor.

Field

Current

Figure 21.12: Fleming’s Left hand Rule

10. TORQUE ON A CURRENT LOOP


Let us consider a square loop PQRS having side  and area A=l2 -
B
-
B
(See Figure). Let us introduce a unit vector n̂ normal to the plane I -
F1
of the loop whose direction is related to the direction of current P Q
in the loop by the right-hand
 screw rule. Area of the loop can be
2
written in vector form as A =  n .
ˆ
- -
If current I in the loop is anti-clockwise then the vector n̂ will be 
F2 F4
directed along the perpendicular to the plane of the paper towards M N
the reader as shown in the Fig. 21.13. Suppose the loop is placed

in a uniform magnetic field B directed along the perpendicular I
to the plane of the paper towards the reader, i.e. along the vector S -
F3
R
n̂ .In this situation, the magnitude of magnetic force on each of
    
the branches of the loop will be IB, i.e. F=1 F=
2 F=
3 F=
4 I B . -
B
-
B
The direction of force on each branch can be found by Fleming’s
    Figure 21.13: Zero torque on closed
left hand rule. We can easily see that F1 = − F3 and F1 and F3 have
    loop in uniform magnetic field
same line of action. Similarly F2 = − F4 and F2 and F4 have same
line of action.So, the net force as well as the net torque on the loop PQRS is zero.
-
F1
Now suppose the loop is rotated through an angle θ about the -
B
-
B
lineMN as shown in Fig. 21.14).So the anglebetween vector n̂ and P’ I Q’
 P
B will beθ. In this situation each of the sides Q’R’ and S’P’ makes an Q
  
angle 90o-θ with the magnetic field B so that F= F= I  Bcos θ  
2 4
   
and again we have F2 = − F4 and F2 and F4 have same line of
- -
F2 F4
action. The side PQ shifts to P’Q’ and RS shifts to R’S’ such that M N
 
PQ || P'Q' and RS || R'S' so that F= 1 F=
3 I  B and again we have
   
F1 = − F3 , but the lines of action of F1 and F3 are displaced from
S R
I
each other by a distance of lsinθ. This forms a force couple, and S’
the torque due to it will have magnitude -
B -
-
B
F3
τ = (I  B)  sin θ = I 2 Bsin θ = I A B sin θ
Figure 21.14: 14 Non-zero torque on closed
This torque is directed along the line MN.
loop in uniform magnetic field
P hysi cs | 21.11

  
In vector form we can write τ= I A ×B
    
Defining magnetic dipole moment of the loop as M = I= A I Anˆ , we can write torque as τ= M×B
 
If the number of turns in the loop is N then we have,
= M NI=A NI Anˆ
Note that although this formula has been derived for a square loop, it comes out to be true for any shape of the
loop.

Illustration 5: A vertical circular coil of radius 0.1 m has moment of inertia as 1 x 10-1kg m2.It is free to rotate along
y-axis coinciding with its diameter. Initially axis of the coil and direction of magnetic field of 1 T are along x-axis.
The coil takes a quarter rotation. Find  (JEE ADVANCED)
(i) Magnetic field strength at the center of the coil. Current of 3.19 A flows through this coil having 200 turns.
(ii) Magnetic moments of the coil.
(iii) Torque at the initial and final positions of the coil.
(iv) Angular speed at the final position.
    dω dω
Sol: The torque on coil is τ = − M × B where M the magnetic moment of coil is. As torque τ = I α = I = I ω,
integrating equation of torque we get the angular velocity. dt d θ

µ NI (4 πx10−7 )(200)(3.19)
B = 0 , we have B = 4x10−6 T
(i) Using =
2R 2x0.1

(ii) Magnetic moment, m = NIA=NI (πR2)= 200 x 3.19 x πx(0.1)2 = 20Am2

(iii) Torqueτ=N1ABsin θ =m sin θ ; initially θ =0 so sin θ =0 and τ=0

Finally, θ =90o so sin θ =sin90o=1 i.e., τ =mB; i.e.τ=20x4x10-6x1=8x10-5Nm

dω dω dω dω dθ dω
(iv)
= Γ I and
= Γ mBsin θ ; I mBsin θ, But
= =x =ω Then, Iωdω=(mB sin θ )d θ
dt dt dt dθ dt dθ

ω π /2
Iω2 90
Integrating, we get I ∫ ωdω = mB ∫ sin θdθ i.e, = −mBcos θ | = mB
0 0
2 0
1/2 1/2
 2mB   2x8x10−5 
i.e.
= ω =    =4 x 10-2 rad s-1
 I   0.1 

Note:
(a) Never use Fleming left-hand rule or right hand rule while solving questions. It becomes cumbersome to
remember them precisely. Instead always find the direction of force by identifying the directions of motion
and the field and then take the cross-product.
 
(b) Also, torque can be directly calculated by formula M × B , where M is the magnetic dipolemoment as discussed
below.

11. MAGNETIC DIPOLE MOMENT


S N
Every current carrying loop behave like a magnetic dipole. It has two poles, north
(N) and south (S) similar to a bar magnet. (see Fig. 21.15) Magnetic field lines are
closed pathsdirected from the North Pole to the South Pole in the region outside
the magnetic dipole and from South Pole to North Pole inside the magnetic dipole. Figure 21.15: North and South
Pole of current coil
2 1 . 1 2 | Moving Charges and Magnetism

 
Each loop has magnetic dipole moment defined as M = NI A , where N is the number of turns in the loop, I is the
current in the loop andA is the area of cross-section of the loop.

For the direction of M any one of following methods can be used:
 
(a) The direction of M is from South Pole to North Poles we traverse inside the magnetic M
dipole. For a current loopthe North and the South Pole can be identified by the sense
of current. The side fromwhere the current seems to flow clockwise is the South Pole i
and the opposite side from where it seems to flow anticlockwise is theNorth Pole.
 
(b) Vector M is along the normal to the plane of the loop. The direction of M is related
to the direction of current in the loop by the right hand screw rule. Curl the fingers of
the right hand around the perimeter of the loop in the direction of current as shown
in Fig.21.16. Then thumb extendedperpendicular to the plane of the loop, points in
 Figure 21.16: Right
the direction of M . hand screw rule
The potential energy U of a magnetic dipole placed in a uniform magnetic field is
U= − MBcos θ
  M
or U = − M.B
For a bar magnet we define the magnetic dipole moment as
 
M= m

Here m is the pole strength of the bar magnet and vector  is directed from South Pole
to North Pole.
The unit of magnetic dipole moment is A-m2.
Figure 21.17: Direction of
The magnetic field at a large
 distance x on the magnetic axis of a bar magnet having magnetic moment
magnetic dipole moment M is

 µ0  2M 
B=  
4 π  x3 
The magnetic
 field at a large distance x on the perpendicular bisector of a bar magnet having magnetic dipole
moment M is

 µ0  M 
B= −  
4 π  x3 

Illustration 6: A square loop OABCO of side  carries a current I. It is placed as shown in Fig. 21.18. Find magnetic
moment of the loop. (JEE MAIN) z
A
Sol: The magnetic moment of the loop is M = I A for single turn. The direction

of M is related to the direction of current in the loop by the right hand screw rule. B
As discussed earlier, magnetic moments of the loop can be written as, i
  
( )
O
M = I BCxCO o
y
60
    3 C
Here, BC =  k CO = − cos60o ˆi −  sin60o j =− i − j x
2 2
  Figure 21.18
  −  i −  3 j   or
  I2
∴ M = I ( − k)x = M (j − 3 i)
  2 2  2
  
P hysi cs | 21.13

Illustration 7:Find the magnitude of magnetic moment of the current carrying C


loop ABCDEFA. Each side of the loop is 10 cm long and current in the loop is
i=20 A. (JEE ADVANCED)
D
Sol: The magnetic moment of the loop is M = I A for single turn. If a loop is B
divided into different parts, the magnetic moment of entire loop is vector sum
of the magnetic moments of its individual parts. A
E
By assuming two equal and opposite currents in BE, two current carrying loops
(ABEFA and BCDEB) are formed. Their magnetic moments are equal in magnitude
but perpendicular to each other. Hence, F

Mnet = M2 + M2 = 2M Figure 21.19

Where M=iA-(2.0)(0.1)(0.1)=0.02 A-m2


= Mnet ( 2)(0.02) A − m2
A
=0.028 A-m2

C D

B E

A F

Figure 21.20

12. BIOT-SAVART LAW


Biot-Savart law is gives the strength of the magnetic field at any point due to a i

current element. If infinitesimalcurrent element of length d  carries a current I,

the magnetic field or magnetic induction dB at any point P is given by Biot-Savart  
dl P
law as O 
  r
  µ0  I d  × r
= dB   ⋅
 4π  r3
 
Here r is the position vector from the center of the element of length d  to

the point of observation P. The direction of d  is along the direction of current I Figure 21.21 : Magnetic field
  due to current element dℓ
through it. If θ is the angle which r makes with the length d  of the conductor,
the magnitude of magnetic induction is given by

 µ I d (r sin θ)
dB = o
4π r3
 µ I d (sin θ)
dB = 0
4π r2
µ0
Here m0 is the permeability of free space and = 10−7 Tesla-meter/ampere.

  
The direction of dB is perpendicular to the plane containing current element d  and radius vector r which joins

d  to P.
 
The total magnetic induction due to the conductor is given by, B = ∫ dB .
2 1 . 1 4 | Moving Charges and Magnetism

The magnetic intensity H at any point in the magnetic field is related to the magnetic induction
B
as H = or B = µH where µ is permeability of the medium. The unit of magnetic intensity H
µ
is A-m-1
Maxwell’s Cork Screw Rule: If a right handed cork screw is rotated so that its tip moves in the
direction of flow of current through the conductor, then the direction of rotation of the head of
the screw gives the direction of magnetic field lines around the conductor.
Right Hand Rule: If we hold the conductor in the right hand such that the thumb is stretched
Figure 21.22:
in the direction of current, the direction in which the fingers curl gives the direction on the
Right hand
magnetic field.
thumb rule

12.1 Application of Biot-Savart Law


Biot-Savart law is used to find the magnetic field due to current carrying conductors.

12.1.1 Magnetic Induction Due to Infinitely Long Straight Current Carrying Conductor
Suppose the current I flows through a long straightcurrent carrying conductor. We intend to find the magnetic field

at point P at perpendicular distance r from the conductor. As shown in Fig. 21.23. the magnitude of field dB at P
due toan infinitesimal element of length d, is given by Biot-Savart law as:
 µ I d sin(90 + α )
dB
= dB = 0
4 πx2

where x is the distance between the current element and point P. The field dB is directed into the plane of the
figure and perpendicular to it.


d xd

x
I
d


dB
r A

Figure 21.23: Magnetic field due to infinitely long straight wire


r
Now from Fig. 21.23. it is clear that, d cos α = x dα and x = , so we can
write, cos α
µ I cos α dα
dB = 0  ……(i)
4π r I
π π
The conductor is infinitely long,so as the angle α varies from − to , 1
2 2 P
r 2
the infinitesimal element covers the infinite length of the conductor, and for

allinfinitesimal elements making-up the conductor the field dB is directed into

the plane of the figure. Thus we can add the magnitudes of dB due to all the O
infinitesimal elements to get the magnitude of total field as, Figure 21.24: Magnetic field
due to finite straight wire
P hysi cs | 21.15

π /2
µo I µo I
= B
4 π r −π /2 ∫=
cos α d α
2π r

12.1.2 A Straight Conductor of Finite Length


If a conductor of finite length subtends an angle α1on one side and α2on the other side ofperpendicular from point
P as shown in Fig. 21.24 then we can write,
α1
µo I µo I α µo I
=B ∫ cos=
α dα α 1
sin= [sin α1 + sin α2 ]  ….(ii)
4 πr −α 4 πr −α2 4 πr
2

12.1.3 At the End of a Straight Conductor of Infinite Length


π
In this case, the angle α varies from 0 to , and we can write
2
µo I π /2 µo I
= B
4 πr 0∫ = cos α dα
4 πr
I

12.1.4 At The End of a Straight Conductor of Finite Length



In this case, (see Fig. 21.25) the angle α varies from 0 to α, and we can write r P

α
µ0 I µo I sin α Figure 21.25: Magnetic
4 πr ∫
= B cos
= α dα
4 πr field at end of straight
0
wire of finite length

12.1.5 At a Point Along the Length of the Straight Conductor Near Its End

π π
In this case (see Fig. 21.26) α1 = and α2 =− , and thus equation (ii)gives B=0. Actually
2 2
in this case the value of α does not vary at all i.e. it is constant (at all points of the wire we
π
have α = ), thus dα = 0 and thus equation (i) gives dB = 0. I
2

Illustration 8: Calculate the magnetic field at the center of a coil in the form of a square
of side 4 cm carrying a current of 5A. (JEE MAIN)
r
Sol: Square loop can be considered as four wires each of length . Magnetic field due to
µ0 1
any one wire, at a the center is calculated=
as B1 sin θ1 + sin θ2  P
4π x 
Figure 21.26: Magnetic
A square coil carrying current is equivalent to four conductors of finite length. field along length of
straight wire
Step 1
Magnetic field at O due to conductor BC is A B

µ0 1
=B1 sin θ1 + sin θ2 
4π x  O
45o
45 a
o

Here θ1 =θ2 =450 ; I=5A,x=2 cm=2x10-2m

10−7 x5  10−7 x5x 2 D C


∴ B1
= sin 45o + sin 45o  = = 3.54x10−5 T
−2   −2
2x10 2x10 Figure 21.27
2 1 . 1 6 | Moving Charges and Magnetism

By symmetry, magnetic field intensity at O due to each arm will be same.Moreover, the direction of magnetic field
at O due to each arm of the square is same

Step 2
∴ Net magnetic field at O due to current carrying square,
B=4B1 =4 x 3.54 x 10-5Tor B=1.42 x 10-4T

12.1.6 Magnetic Field on the Axis of a Current Carrying Circular Arc

If a current I is flowing in a circular arc of radius R lying in the y-z plane with center at origin O and subtending an

angle ϕ at O, then the magneticfield dB at a point Pon x-axiswith coordinates (x, 0, 0) due to a small elementary arc

of length d=  R dθ at a distance r from P is given by Biot-Savart Law as:

 µ0 I d  × r
dB =  … (i)
4π r3
 
where r is a vector from midpoint of d  to P.

As shown in Fig. 21.28 the coordinates of d  are (0, R cosθ, R sinθ), where θ is the angle between the radius of the

arc through d  and the y-axis.

Rd
(0, Rcos, Rsin) BY
I R r

 P BX
X
O
(x, 0, 0)
BZ

Figure 21.28: Magnetic field at a point on the axis of current carrying arc

So we can write r = x ˆi − R cos θ ˆj − R sin θ kˆ  … (ii)

Magnitude
= r x2 + R 2  … (iii)

Let us express d  in Cartesian coordinates system as shown in Fig. 21.29.

d  =− R sin θ dθ ˆj + R cos θ dθ kˆ  … (iv)
Put (ii), (iii) and (iv) in (i) to get

 µ I ( −R sin θ d θ ˆj + R cos θ d θ k) ˆ × (x ˆi − R cos θ ˆj − R sin θ k)


ˆ
dB = 0
4π ( x2 + R 2 )3
 µ0 I
⇒ dB
= (R 2 d θ ˆi + x R cos θ d θ ˆj + xR sin θ d θ k)
ˆ
4 π (x + R 2 )3/2
2
P hysi cs | 21.17

Resultant magnetic field at P is Y


φ φ φ
 µ0 I -
= B (R 2 ∫ d θ ˆi + xR ∫ cos θ d θ ˆj + xR ∫ sin θ d θ k)
ˆ d =Rd - 
2 2 3/2
4 π(x + R ) 0 0 0 d
 µ0 I
=⇒ B [R 2 φ ˆi + xR sin φ ˆj + xR(1 − cos φ)k]
ˆ I d
2 2 3/2
4 π(x + R ) R
 
Thus B can be resolved into components parallel to the x, y and the z axes.

2
µ0 I R φ O
Bx =
2 2 3/2
4 π(x + R ) Figure 21.29: Vector is in
the YZ plane
µ0 I R x sin φ
By =
4 π(x2 + R 2 )3/2

µ0 I R x(1 − cos φ)
Bz =
4 π(x2 + R 2 )3/2
The field at center of the arc: At center x = 0, so
µ0 I φ
Bx =
4πR

By = 0
Bz = 0
Thus at the center the field is normal to the plane of the arc.
µ0 I
For a semicircular loop, the angle subtended at the center is φ = π, so B =
4r

12.1.7 Magnetic Field on the Axis of a Current Carrying Circular Loop


 
The field B on the axis of a current carrying circular loop (see Fig. 21.30) can be obtained from the expression of B
for a current carrying circular arc derived in the previous article by substituting the value of angle ϕ subtended at
the center as 2π.

 µ0 I
=∴ B [R 2 (2π) ˆi + xR sin2π ˆj + xR(1 − cos2π)k]
ˆ
4 π(x + R 2 )3/2
2 Y

 µ0 IR 2 ˆi
P
∴ B = X
2 2 3/2 (x, 0, 0) BX
2(x + R )
 Z
Thus field B is directed along the axis of the circular loop.
Figure 21.30: Magnetic field at a point
µ0NIR 2 on the axis of circular loop
For a coil havingN circular turns, B=
2(R 2 + x2 )3/2
The field at center of the coil:

µ0 NIR 2
At center x = 0, so B0 =
2R3
µ0 NI
∴ B0 =
2R
2 1 . 1 8 | Moving Charges and Magnetism

The direction of B at the center of circular current carrying arc or closed circular loop can be found as follows:
If we curl the fingers of the right hand in the direction of the current in the arc/loop, then the stretched thumb
points in the direction of the field at the center.
µ NIR 2
If the point P is at a very large distance from the coil,then x2>>R2, B = 0
2x3
µ NI A
If A is area of one turn of the coil, A=πR2 B = 0
2πx3
Illustration 9: A straight wire carrying a current of 12 A is bent into a semi-circular are of radius 2.0 cm as shown in
Fig. 21.31.(i) What is the direction and magnitude of magnetic field (B) at the center of the arc? (JEE ADVANCED)
(ii) Would the answer change if wire is bent in the opposite way?

Sol: For given arrangement of wire, the magnetic field at the 2.0
center due to the straight sections will be zero. The magnetic cm
field at center will be due to the semicircular wire. Direction O
of field depends on direction of current and determined by
right hand thumb rule.
(i) The wire is divided into three sections: (a) the straight Figure 21.31
section to be left (b) the straight section to the right and
(c) circular arc.

µ0 Idl sin θ
Step 1. Magnetic field due to a current carrying element at a point is given by dB =
4π r2

In the given case, angle between dl and r for the straight section is 0oor π .So sin 0 =sin π =0
Hence magnetic field at the center (O) of the arc due to straight sections is ZERO

Step 2. Magnetic field at the center due to current carrying semi-circular section is

1 µ0 2πI µ0 πI 10−7 x3.142x12


=B =x = =1.89x 10-4 T
2 4π r 4π r 2x10−12
The magnetic field is directed into the plane of the paper.
(ii) Direction of the field will be opposite to the found out in (i).

Illustration 10: A current path shaped as shown in Fig. 21.32 produces a magnetic field at P, the center of the arc.
If the arc subtends an angle of 30o and the radius of the arc is 0.6m, what are the magnitude and direction of the
field produced at P if the current is 3.0 A  (JEE ADVANCED)
A
µ Iφ
Sol: Magnetic field at the center P of arc CD is B = 0 , and due to straight C
4 πR
wires AC and DE is zero.

The magnetic field at P due to the straight segment AC and DE is zero, 30


o

    P E
because d is parallel to r along these paths, this means that d x r =0. D

Each length element d  along path CD is at the same distance from P,
Figure 21.32

hence B at P is due to segment CD which is given by

µ0 I µ0 I π µ0 I
B
= = φ ×=
4 πr 4 πr 6 24r
P hysi cs | 21.19

13. FORCE BETWEEN PARALLEL CURRENTS


Consider two long wires kept parallel to each other such that the
separation d between them is quite small as compared to their Y
lengths. Suppose currents I1 and I2 flow through the wires in the I1 I2
same direction (see Fig. 21.33). Consider a small element d of -
the wire carrying current I2.The magnetic field at d due to the B -
d X
wire carrying current dF
 µ0 I1 Z
is B
I1 = ˆ 
( − k) ...(i)
2πd d

( B is normal to and directed into the plane of the figure)
 Figure 21.33: Force between parallel currents
 
The magnetic force on this element is dF= I2d  × B= I2d(ˆj) × B( − k)
ˆ

ˆi) µ0 I1 I2 d ( − ˆi) (directed towards the wire carrying current I )



or, =
dF I2d B( −
= 1
2πd
Thus the wire carrying current I2 is attracted towards the wire carrying current I1. By Newton’s third law the force
acting on wire carrying current I1 will also be attractive.Thus the two wires are attracted towards each other.
The force per unit length on each of the wires due to the other wire will be,
dF µ0 I1 I2
=
d 2πd
Parallel currents attract each other, and antiparallel currents repel each other.

Note: Memorizing various formula of magnetic field due to ring and wire carrying current would easily help in
calculating magnetic field due to complicated wire systems. Also, be careful about the direction of field in every
problem you solve.

Illustration 11: A current of 10A flows through each two parallel long wires. The wires are 5 cm apart. Calculate the
force acting per unit length of each wire. Use the standard values of constants required.  (JEE MAIN)
F µ0 2I1 I2
Sol: Field of one wire exerts force on other wire and the force per unit length of wire is
= .
 4π d
Force acting per unit length of long conductor due to another long conductor parallel to it and carrying same
current.

dF µ0 2I1 I2 µ dF 10−7 x2x10x10


= ;I1=I2=10A, r=5 cm =5x10-2m, 0 =10-7TmA-1; = =4 x 10-4 N m-1
d 4 π d 4π d 5x10−2

Illustration 12:The wires which connect the battery of an automobile to its starting motor carry a current of 30A
(for a short time).What is the force per unit length between the wires, if they are 70 cm long and 1.5 cm apart? Is
the force attractive or repulsive? (JEE ADVANCED)

Sol: Field of one wire exerts force on other wire and the force per unit length of M
F µ 2I I
wire is = 0 1 2 .
 4π d 70 cm
Force depends on direction of current. Parallel currents attract while anti-parallel F1 F2
currents repel. 1.5 cm

dF µ0  2I1 I2 
=   ; I =I =300A; r=1.5cm=1.5x 10-2m
d 4 π  d  1 2
Figure 21.34
2 1 . 2 0 | Moving Charges and Magnetism

dF 10−7 x 2 x 300 x 300


∴ = 1.2Nm−1
d 1.5x10−2
Since current in both the wires flows in opposite direction, so the force is repulsive.

14. AMPERE’S LAW


This law is also called the ‘Theorem on Circulation of Vector B’.

According to this law the line integral or circulation of magnetic field vector B around a closed path is equal to µ0
times the algebraic sum of the currents enclosed by the closed path.
- -
  B.d=Bdcos
�∫ B ⋅ d  =
µ0 Ienc

The closed path is also called Amperian loop. I3


-
I1  B
Ienc is the algebraic sum of all the currents passing through -
d
the area enclosed by the closed path. Current is assumed
positive if it is along the direction associated with the
direction of the circumvention of the closed path through  I2
Direction of
the right-hand screw rule.If we curl the fingers of the Amperian contour
right hand around the closed path, in the direction of Loop circumvention
circumvention, the stretched thumb gives the positive
direction of current. The current in the opposite direction
Figure 21.35: Current enclosed by amperian loop
is negative.

For example in the Fig. 21.35 shown, the current directed out of the plane of the figure is positive, so we have Ienc
 
= I1 - I2; � µ0 ( I1 − I2 )
∫ B ⋅ d =

14.1 Limitations of Ampere’s Circuital Law


Ampere’s law is an important tool in calculating the magnetic field due to a current distribution. However this
usefulness is limited to only a few cases where the magnetic field is having a symmetrical distribution in space. The
Amperian loop is chosen in such a way that the magnetic field has a constant value along the loopand is directed
tangentially at all points of the loop.If such a choice of a loop is not possible, then Ampere’s law cannot be used to
find out the magnetic field. For example this law can’t be used to find the magnetic field at the center of a current
carrying loop.

Note: Ampere’s circuital law holds good for a closed path of any size and shape around a current carrying conductor.

14.2 Applications of Ampere’s Law

14.2.1 Magnetic field due to current carrying circular wire of infinite length
Let R be the radius of the infinite circular wire carrying current I. The magnetic field lines are concentric circles with
their centers on the axis of the wire.

(a) Magnetic field intensity at a point outside the wire

We intend to find magnetic field at a distance r> R from the axis of the wire. We choose a circular path of radius r
and center at the axis of the wire as the Amperian loop. B will be constant and tangential at all points of this loop.
Using Ampere’s law,
P hysi cs | 21.21

  0
�∫ B.d  = µ0 I or �∫ Bd cos0 = µ0 I

or B �
∫ d =B(2π r) =µ0 I
µ0 I
∴ B=  …(i)
2πr
Thus, the magnetic field intensity at a point outside the wire varies inversely as the distance of the point from the
axis of the wire.

r r>R

I r R
 r<R

Amperian
Loop

Figure 21.36: Circular cross-section of infinitely long straight wire

1
That is, B ∝
r
µ0 I
At the surface of the wire, r = R, so B=  …(ii)
2πR

(b) Magnetic field intensity at a point inside the wire

We intend to find magnetic field at a distance r< R from the axis of the wire. We choose a circular path of radius r

and center at the axis of the wire as the Amperian loop. B will be constant and tangential at all points of this loop.
Using Ampere’s law,
  0
�∫ B.d  = µ0 Ienc or �∫ Bd cos0 = µ0 Ienc

or B ∫ d =B(2π r) =µ0 Ienc

If the current is uniformly distributed throughout the cross – section of the wire, then we have
I Ir 2
Ienc= ( π r 2 )= B
πR 2 R2
Ir 2 B 1
∴ B(2π r) =µ0 r
r
B

R2

µ0 Ir
∴ B= r=R
2π R 2
Figure 21.37: Variation of field
Thus, B∝r with radial distance r
The variation of B with distance r from the axis of the wire is shown in Fig.21.37.

Illustration 13: Figure 21.38 shows the cross section of a long conducting cylinder with inner radius a=2.0 cm and
outer radius b=4.0 cm.The cylinder carries a current out of the page, and the magnitude of the current density in

the cross section is given by j = cr2, with c=3.0 x 106 A/m4and r in meters. What is the magnetic field B at appoint
that is 3.0 cm from the central axis of the cylindrical? (JEE ADVANCED)
2 1 . 2 2 | Moving Charges and Magnetism

Sol: The magnetic field in this case is symmetric. The field lines are concentric
circles. We choose a circular amperian loop coaxial with the cylinder. First find
 
the current enclosed for region a<x<r where r = 3 cm. Then use � ∫ B ⋅ ds =
µ0ienc

to find B .
r 2
We write the integral as=
ienc ∫=
JdA ∫ cr (2πr dr) a
a b
r
r r4  πc(r 4 − a4 )
2πc r3 dr =
= ∫ 2πc   =
a 4   a 2

The direction of integration indicated in Fig. 21.38 is (arbitrarily) clockwise.


Applying the right-hand rule for Ampere’s law to that loop, we find that we Figure 21.38
should take ienc as negative because the current is directed out of the page but
our thumb is directed into the page.

We next evaluate the left side of Ampere’s law exactly as we did in figure.
 
Then Ampere’s law, �
∫ B ⋅ ds =
µ0ienc ,
µ0 πc
Gives us B(2πr) =− (r 4 − a4 )
2
µ πc
− 0 (r 4 − a4 )
Solving for B and substituting known data yield B =
4 πr

= −
( 4π x10 −7
)(
T ⋅ m / A 3.0 x106 A / m4 ) x (0.030 m) 4
– ( 0.020m)
4
= − 2.0 x 10 −5 T
.

4 (0.030m)π  

Thus, the magnetic field B at a point 3.0 cm from the central axis has magnitude B=2.0 x 10-5 T and forms magnetic
field lines that are directed opposite our direction of integration, hence counterclockwise in figure.

14.2.2 Magnetic Field Inside a Solenoid


A solenoid is an insulated wire wound closely into multiple turnsto form a helix. The length of the solenoid is
assumed to be much larger than its diameter. At points very close to a turn, the magnetic field lines are almost
concentric circles.The fields due to adjacent turns at points near the axis add-up while fields at points away from
the axis cancel each other. If the solenoid is very tightly wound and its length is quite large, then the field inside it
is uniform and parallel to its axis, while field outside it will be zero.

x x x x x

Figure 21.39: Magnetic field lines inside solenoid


P hysi cs | 21.23

We can apply Ampere’s law to find the magnetic field inside the solenoid. We choose a rectangular Amperian
loop abcd partly inside the solenoid and partly outside it as shown in Fig. 21.40, its length lbeing parallel to the
solenoid’s axis.

d c

Amperian
Loop a b

B
           

Figure 21.40: Rectangular amperian loop

 
There are four sides of the rectangle. We write �∫ B.d  as the sum of four integrals, one for each side:

  b  c  d  a 

∫ B.d  = ∫a B.d  + ∫b B.d  + ∫c B.d  + ∫d B.d 
The sides bc and da do not contribute to the line integral as the magnetic field is perpendicular to these sides at
points inside the solenoid and at points outside the solenoid the magnetic field is zero. The side cd is completely
outside the solenoid and hence the magnetic field is zero at all its points. So the only side that contributes to the
line integral is ab.
 
Thus, we get �∫ B.d  = B = µ0 n I

Here I is the current through each turn of the solenoid and n is the number of turns per unit length of the solenoid.
The net current enclosed by the rectangle is n  I.
∴ B = µ0 n I

PLANCESS CONCEPTS

(a) Magnetic field inside a solenoid and coil


(i) Magnetic field is considered uniform throughout the solenoid, while it is not true for coil
(ii) This is because, solenoid is long, while coil is thin.
(iii) Thus, magnetic field lines look very symmetric inside a solenoid, and of nearly equal length,
while in a coil, the path are very different, and by Ampere’s law, their magnitude is different

(b) Magnetic field on the axis at the end of a long solenoid

(i) Think of an infinite solenoid, if you could take the midpoint at the axis of this solenoid then
µ nI
the magnetic field strength at that point from each side would be B = 0 the situation you
2
µ0nI
describe is like taking half of this infinite solenoid (as L>>d) and so B =
2
( µ0 =permeability of free space, n= number of coils in the solenoid, l= current)
Anurag Saraf (JEE 2011 AIR 226)
2 1 . 2 4 | Moving Charges and Magnetism

Illustration 14:A closely wound solenoid 80 cm long has 5 layers of winding of 400 turns each. The diameter of the
solenoid is 1.8 cm. if the current carries is 8.0 A, find the magnitude of B inside the solenoid near its center.
 (JEE MAIN)
µ0NI
Sol: For solenoid of length  the field at a point inside it is B = where N is the number of turns in solenoid.

Magnetic field induction at a point inside the solenoid is

µ0NI 4 π10−7 x(400x5)x8


=B = = 8 π x10-3T ≈ 2.5 x10-2 T
−2
 (80x10 )

Illustration 15: A solenoid is 2 m long and 3 cm in diameter. Ithas 5 layers of winding of 1000 turns each and
carries a current of 5A. What is the magnetic field at its center? (JEE MAIN)
µ0NI
Sol: For solenoid of length  the field at a point inside it is B = where N is the number of turns in solenoid.

Magnetic field at the center of a solenoid is given by,

µ0NI  5x1000 
B= =(4 π x10-7)   x5= 1.57 x 10 T
-2
l  2 

14.2.3 Magnetic field Inside a Toroid


Toroid is a circular solenoid. An insulated conducting wire is tightly wound on a ring (or torus) made ofnon-
conducting material to form a toroid. The magnetic field inside a toroid can be obtained by using Ampere’s law.
We choose a circularAmperian loop of radius rinside the toroid concentric with it.
 
�∫ ⋅ d  =�∫ Bd =B �∫ d =B(2πr) =µ0 Ienc
B

  
B

 



 r 
  
B

Figure 21.41: Magnetic field inside Toroid

If each turn of the toroid carries current I and the total number of turns in the toroid is N, then current enclosed by
the Amperian loop is NI.
µ NI
So 2π r B = µ0 N I or, B = 0
2πr
P hysi cs | 21.25

Illustration 16: A toroid of 4000 turns has outer radius of 26 cm and inner radius of 25 cm. If the current in the wire
is 10A, calculate the magnetic field of the toroid also in the inner air space of the toroid. (JEE ADVANCED)

Sol: For toroid the field at a pointinside it at radial distance r from its
µ NI
center is B = 0 where N is the number of turns in toroid.
2πr
25 + 26
Radius of toroid r = = 25.5cm = 25.510−2 m

cm
2

26
Length of toroid l=2 π r=2 π x (25.5 x 10-2= 51 x 10-2 π m 25
cm
4000
∴ Number of turns /unit length, n=
51 x10−2 π
Field in a toroid is given by

 4000 
B= µ0nI = 4 πx10 −2   x10 ;= 3.14 x 102T
 51 x10−2 π 
 

Field in the air space bounded by the toroid is zero because the field exists inside
the envelope of the winding of the toroid. Figure 21.42

15. MOVING COIL GALVANOMETER


Moving Coil Galvanometer is a device used to detect/measure small electric current flowing in an electric circuit.

Principle: When a current carrying loop or coil is placed in the uniform magnetic field, it experiences a torque and
thus starts rotating.

Construction: A moving coil galvanometer is shown in Fig. 21.43. It consists of a coil made of insulated copper
wire wound on a soft-iron cylinder. The coil is suspended by a spiral spring between two cylindrical shaped poles
of a permanent magnet.
The spring exerts a very small restoring torque on the coil. Soft iron core
Theory
Let B = Magnetic field N S
I = Current flowing through the coil
 = Length of coil
b = Breadth of the coil Coil
(xb) = A= Area of the coil Figure 21.43: Moving coil
galvanometer
N = Number of turns in the coil
When current flows through the coil, it experiences a torque, which is given by
τ=NIAB sinθ
where, θ is the angle between the normal to the plane of the coil and the direction of the magnetic field.
Initially, θ = 90o, so τ=NIAB …(i)
This torque is called deflecting torque.As the coil gets deflected, the spring is twisted and a restoring torque is
developed in it which is proportional to the angle of deflection φ
τres=kφ  …(ii)
2 1 . 2 6 | Moving Charges and Magnetism

Here k is a constant for a particular spring.


For equilibrium of the coil,
Deflecting torque= Restoring torque
i.e. NIAB=kφ  …(iii)


or I=  …(iv)
NAB
or I = Gφ  …(v)
k
where G = is Galvanometer constant
NAB
∴ I ∝ φ  …(vi)
Thus, the current flowing through the coil is directly proportional to the deflection of the coil. Hence we can
determine the current in the coil by measuring its deflection.

Use of a radial magnetic field in the moving coil galvanometer


A radial magnetic field, produced by cylindrical poles of permanent magnet is always parallel to the plane of the
coil of the galvanometer. Thus the angle between the normal to the coil and the magnetic field is always 90o. Thus
torque on the coil is τ = NIAB = kφ or I ∝φ. Thus, when radial magnetic field is used, the current in the coil is always
proportional to the deflection. Hence, a linear scale can be used to determine the currentin the coil.

Use of Galvanometer
(a) It is used to detect electric current in a circuit e.g., Wheatstone Bridge.
(b) It is convertedinto an ammeter by putting a small resistance parallel toit.
(c) It is converted into a voltmeter by putting a high resistance in series with it.
(d) It is used as an ohmmeter.

Sensitivity of a Galvanometer
A galvanometer is said to be sensitive if a small current flowing through its coil produces a large deflection in it.

(a) Current Sensitivity


The current sensitivity of a galvanometer is the deflection produced in the galvanometer per unit current
flowing through it.
φ NAB
i.e. Current sensitivity = =
I k
Current sensitivity of galvanometer can be increased either by
(i) Increasing the magnetic field B by using a strong permanent horse-shoe shaped magnet.
(ii) Increasing the number of turns N.
(iii) Increasing the area of the coil A. (but this will make the galvanometer bulky and ultimately less sensitive)
(iv) Using a spring having small value of restoring torque constant k.

(b) Voltage Sensitivity


Voltagesensitivity is the deflection produced in the galvanometer per unit voltage applied to it.
φ φ NBA
Voltage sensitivity = = i.e., voltage sensitivity = (R= resistance of the coil)
V IR kR
P hysi cs | 21.27

Voltage sensitivity can be increased by


(i) Increasing N
(ii) Increasing B
(iii) Increasing A
(iv) Decreasing k and
(v) Decreasing R.

Advantage of a moving coil galvanometer


(a) A minutely small current in the electric circuit can be detected using an extremely sensitively galvanometer.
(b) A linear scale can be used to read the current, since deflection of the coil is directly proportional to the current.
(c) The external magnetic fields (e.g. horizontal component of earth’s magnetic field) cannot effect the deflection
of the coil of the galvanometer,because the magnetic field of the permanent magnet is very strong. Thus the
galvanometer can be placed in any location.
(d) A dead beat type galvanometer is used.(The coil of a dead beat type galvanometer comes to rest quickly after
deflecting to its equilibrium position, i.e it does not oscillate)

16. CYCLOTRON
Cyclotron is a device used to accelerate positively
charged particles (like protons,α particles, deuteron, S
ions etc.) to acquire enough energy to carry out 
B
nuclear disintegrations. Deflecting High
plate D1
Principle: It works on the following principle: A  frequency
positively charged particle is made to accelerate oscillator
W
through an electric field and using a strong magnetic
Source of
field it is circled back to the region of the electric
positively
field, to accelerate it again and again to acquire
Target charged
sufficiently large amount of energy. D2
particle
Construction and Working: It consists of two
hollow D-shaped metallic chambers D1 and D2 called
dees. These dees are separated by a small gap where N
a source of positively charged particles is placed.
Dees are connected to high frequency oscillator,
which provides high frequency electric field across Figure 21.44: Cyclotron
the gap of the dees which accelerates the particles.
The magnetic field inside the dees is perpendicular to the plane of motion of particles and drives theminto a
circular path. Suppose the particles start from rest and are accelerated towards chamber D2. After completing a
semicircle, when the particles reach the gap of the dees again, thereversal of the polarity of electric field ensures
that the particlesareagain accelerated towards the other chamber D1 by the electric field. Radius of the circular path
increases with increase in speed, thusthe particles follow a spiral path (see Fig. 21.44)

Theory: The magnetic force on the positively charged particle provides the centripetal force to move in a circle of
radius r.
mv 2 mv
∴ qvB
= = or r  … (i)
r qB
2 1 . 2 8 | Moving Charges and Magnetism

Time taken by the particle to complete the semi-circle inside the dee,
dis tance πr π mv πm
t= = or t= × or t =  … (ii)
speed v v qB qB
This shows that time taken by the positively charged particle to complete any semi-circle (irrespective of its radius)
is same

(a) Time Period: Let T be the period of the high frequency electric field, then the polarities of dees will change
T
after time .
2
T
The particle will be accelerated if time taken by it to describe the semi-circle is equal to .
2
T πm 2πm
i.e. = t= or T =  … (iii)
2 qB qB
1 qB
(b) Cyclotron frequency: fc= =  … (iv)
T 2πm
qB
∴ Cyclotron angular frequency ω = 2πfc =  … (v)
m
1
(c) Energy gained: Energy gained by the positively charged particle in the cyclotron is given by E = mv 2
2
2
qBr 1  qBr  q2B2r 2
From eqn.(i), we have v= , then E= m x   or E =  … (vi)
m 2  m  2m

Maximum energy gained by the positively charged particle will depend on the maximum value of radius of
its path, i.e the radius of the dees.
 q2B2  2
Emax =  r  … (vii)
 2m  max
 

(d) Limitations of Cyclotron: Cyclotron cannot accelerate uncharged particles like neutron.
(e) Cyclotron cannot accelerate electrons because they have very small mass. Electrons start moving at a very
high speed when they gain small energy in the cyclotron. The frequency of oscillating electric field required
to keep them in phase with the electric field is very high, which is not feasible.
(f) The positively charged particle having large mass (i.e. ions) cannotbe accelerated after a certain speed in the
cyclotron. When the speed of ion becomes comparable to the speed of light,the mass of ion increases as per
the relation
m0
m= ,
v2
1−
c2
where m= mass of ion at velocity v, m0=mass of ion at rest,cis speed of light (3 x 108 ms-1)
Time taken by the ion to describe semi-circular path increases as mass increases.So as the mass increases, the
ion does not reach the gap between the two dees exactly at the instant the polarity is reversed and,it is not
be accelerated further.

Uses of a Cyclotron
(a) It is used to produce radioactive material for medical purposes.
(b) It is used to synthesize fresh substances.
(c) It is used to improve the quality of solids by adding ions.
(d) It is used to bombard the atomic nuclei with highly accelerated particles to study the nuclear reactions.
P hysi cs | 21.29

Note: Sections after this are not in the syllabus of JEE ADVANCED but they are important for understanding the
concepts completely.

Illustration 17:A cyclotron’s oscillator frequency is 10 MHz. What should be the operating magnetic field for
accelerating protons? If the radius of its dees is 60 cm. What is the kinetic energy (in MeV) of the proton beam
produced by the acceleration?  (JEE MAIN)

(e 1.60 x 10−19 C,m0 1.67 x=


= 10−27 kg, 1MeV 1.6 x 10−13 J )
Bq
Sol: The frequency of cyclotron is f = where q is the charge and m is the mass of the charged particle to be
2πm  m v2 
accelerated inside the cyclotron. The kinetic energy of the particle is   in eV .
 2e 
 
Cyclotron’s oscillator frequency should be same as the proton’s revolution frequency (in circular path)
Bq
∴ ƒ= or
2πm

2πmf
B=
q
(2)(22 / 7)(1.67 x10−27 )(10 x106 )
Substituting the values in SI units, we have B = =0.67 T
1.6 x10 −19
The emerging beam of proton moves with the velocity

v = ω r = 2πƒ r = 2 × π × 107 × 0.60


= 3.77 × 107 ms−1

( )
2
−27 7
 m v 2  1.67 × 10 × 3.77 × 10
Thus the kinetic
= energy (in MeV) is   = eV 7.42 MeV
 2e  2 × 1.6 × 10 −19
 

17. MAGNETIC POLES AND BAR MAGNET


Two isolated charges of opposite signs are placed near each other, to form an electric dipole characterized by

an electric dipole moment p .On the other hand in magnetism an isolated ‘magnetic charge’ does  not exist. The
simplest magnetic structure is the magnetic dipole, characterized by a magnetic dipole moment M .A current loop,
a bar magnet and a solenoid of finite length are examples of magnetic dipoles.
   
When a magnetic dipole is placed in an external magnetic field B , a torque act on it, given by τ =MxB


The magnetic field B due to a magnetic dipole at a point along its magnetic axis at (large) distance r from its center,

 µ0 2M
is B =
4π r3

A bar magnet has two poles (North and South) separated by a small distance.
N S
However, we cannot separate these poles apart. If a magnet is broken, the
fragments prove to be dipoles and not isolated poles. If we break up a magnet N S N S N S
into the electrons and nuclei that make up its atom, it will be found that even
these elementary particle a re magnetic dipoles. Figure 21.45: Poles of bar magnet

The poles of the bar magnet are modeled as follows:


(a) There are two types of magnetic charges; positive magnetic charge or North Pole and negative magnetic
charge or South Pole. Every Pole has a strength m. The unit of Pole strength is A-m.
2 1 . 3 0 | Moving Charges and Magnetism

 
(b) A magnetic charge placed in a magnetic field experiences a force, F = mB . The force on positive magnetic
charge is along the field and force on a negative magnetic charge is opposite to the field.
(c) A magnetic dipole is formed when a negative magnetic charge –m and a positive magnetic charge +m are
placed at a small separation d. The magnetic dipole moment is, M=md. The direction of M is from –m to +m.

Geometrical Length and magnetic Length


In bar magnet, the poles are located at points which are slightly inside the two ends. The distance between the
locations of the poles is called the magnetic length of the magnet. The distance between the ends is called the
geometrical length of the magnet.

Magnetic length

N S

Geometric length

Figure 21.46: Geometric and Magnetic length of a bar magnet

o
Illustration18: Calculate the magnetic induction at a point 1 A away from a proton, measured along its axis of
spin. The magnetic moment of the proton is 1.4 x 10-26 A-m2.  (JEE MAIN)

µ0 2M
Sol: On the axis of a magnetic dipole, magnetic induction is given by. =
B ⋅
4π r3

Substituting the values, we get B =


(10 ) (2) (1.4x10 ) = 2.8 x 10 T= 2.8 mT
−7 −26
-3

(10 )
3
−10

18. MAGNETIC SUSCEPTIBILITY


For paramagnetic and diamagnetic materialsthe intensity of magnetization is directly proportional to the magnetic
field intensity.
 
I = χmH

The proportionality constant χm is called the magnetic susceptibility of the material. I and H have the dimensions
of A-m-1 and the susceptibility χm is a dimensionless constant. For vacuum χm =0. For paramagnetic materials
χm > 0, and for diamagnetic materials χm < 0 are diamagnetic.

19. CURIES’S LAW


When the temperature increase, due to thermal agitation the magnetization I decreases for a given magnetic
intensity H,which means χm decreases as T increases. According to Curie’s law, the susceptibility of a paramagnetic
c
substance is inversely proportional to the absolute temperature: χm = where c is a constant called the curie
constant. T
P hysi cs | 21.31

The magnetization of ferromagnetic material also decreases with increase in temperature, and on reaching a certain
temperature,the ferromagnetic properties of the material disappear. This temperature is called Curie point (Tc). At
temperatures above Tcferromagnetic turns into a paramagnetic and its susceptibility varies with temperature as,
C'
χm =
T − Tc
where C’ is a constant.

20. PROPERTIES OF PARA -, DIA- AND FERRO-MAGNETISM


(a) Paramagnetic Substances: Example of such substances are platinum, aluminium, chromium, manganese,
CuSO4 solution, etc. They have the following properties:
(i) The substances, when placed in magnetic field, acquire
a feeble magnetisation in the same sense as the applied
field. Thus, the magnetic inductance inside the substance is N S
slightly greater than outside to it.
(ii) In a uniform magnetic field, these substances rotate until
their longest axes are parallel to the field.
Figure 21.47: Paramagnetic material
(iii) These substances are attracted towards regions of stronger
in strong magnetic field
magnetic field when placed in a non-uniform magnetic field.
(iv) Figure 21.47 shows a strong electromagnet in which one of the pole pieces is sharply pointed, while the
other is flat. Magnetic field is much stronger near the pointed pole than near flat pole. If a small piece
of paramagnetic material is suspended in this region, a force can be observed in the direction of arrow.
(v) If a paramagnetic liquid is filled in a narrow U-tube and one limb is placed in between the pole pieces of
an electromagnet such that the level of the liquid is in line with the field, then the liquid will rise in the
limb as the field is switched on.
(vi) For paramagnetic substances, the relative permeability µr is slightly greater than one.
(vii) At a given temperature the magnetic susceptibility χm does not change with the magnetizing field.
However it varies inversely as the absolute temperature. As temperature increases χm decreases. At
some higher temperature χm becomes negative and the substance become diamagnetic.
(b) Diamagnetic Substances: Examples of such substances are bismuth, antimony, gold, quartz, water, alcohol,
etc. They have the following properties:
(i) These substances, when placed in a magnetic field, acquire feeble
magnetization in a direction opposite to that of the applied field.
Thus, the lines of induction inside the substance are smaller than
those outside to it. N S
(ii) In a uniform field, these substances rotate until their longest axes
are normal to the field.
(iii) In a non-uniform field, these substances move from stronger to
weaker parts of the field.
(iv) If a diamagnetic liquid is filled in a narrow U-tube, and one limb is
placed in between the pole of an electromagnet, the level depresses
when the field is switched on.
Figure 21.48: Liquid column of
(v) The relative permeability µr is slightly less than 1.
paramagnetic substance in strong
(vi) The susceptibility χm of such substances is always negative. It is magnetic field
constant and does not vary with field or the temperature.
2 1 . 3 2 | Moving Charges and Magnetism

(c) Ferromagnetic Substances: Examples of such substances are iron, nickel, steel, cobalt and their alloys. These
substances resemble to a higher degree the paramagnetic substances with regards to their behaviour. They
have the following additional properties:
(i) These substances are strongly magnetized by even a weak magnetic field.
(ii) The relative permeability is very large and is of the
order of hundreds and thousands.
(iii) The susceptibility is positive and very large.
(iv) Susceptibility remains constant for very  small
N S

values of H , increases for larger values of H and
then decreases for very large values of H .
(v) Susceptibility decreases steadily with the rise of the
temperature. Above a certain temperature, known Figure 21.49: Diamagnetic substance
as Curie temperature, the ferromagnetic substances in magnetic field
become paramagnetic. For iron, it is 1000oC, 770oC
for steel, 360oC for nickel, and 1150oC for cobalt.

21. HYSTERESIS
Hysteresis is the dependence of the magnetic flux density B in a
ferromagnetic material not only on its current magnetizing field H, A
but also on its history of magnetization or residual magnetization.
C

When a ferromagnetic material is magnetized in one direction, and


then the applied magnetizing field is removed, then its magnetization -H0 D H0
H
will not be reduced to zero. It must be driven back to zero by a field O G
in the opposite direction. If an alternating magnetic field intensity is
applied to the material, its magnetization will trace out a loop called
F
a hysteresis loop.
The phenomena in which magnetic flux density (B) lags behind the E
magnetizing field (H) in a ferromagnetic material during cycles of
magnetization is called as hysteresis. Figure 21.50: Hysteresis loop of I vs H

PROBLEM-SOLVING TACTICS

(a) General advice for this section involves learning of formulae and avoiding silly mistakes. Also it would be
better to go by the usual algorithm of noting down known and unknown quantities and linking them.
(b) Much of manipulation and mathematical complexity is involved here which can’t be avoided.
P hysi cs | 21.33

FORMULAE SHEET
    
(a) Magnetic Force on a charge moving with velocity v in magnetic field B is F=
m q v × B . Magnitude is
= Fm q vB sin θ .
(b) Charged particle moving in uniform magnetic field

qB
(i) Angular velocity ω = 2πf =
m
2πm
(ii) Time period T=
qB

mv m 2qV 1 2mV
(iii) Radius
= r = =
qB qB m B q

mv ⊥ 2πm
(c) Helical Paths: Radius r = Pitch:=
p v=
⊥T v⊥
qB |q|B
(d) The cyclotron q B= 2πmfosc

   
(e) Crossed Fields: Lorentz Force F= q(E + v × B)

q E x2
(f) Trajectory of a charged particle in electric field y =
2m v 2
  
(g) Magnetic force on current element dF
= I d × B
  
(h) Magnetic force on a conductor in uniform field F= IL × B


(i) Magnetic dipole moment of a current coil having N turns pm = NI Anˆ

  
( j) Torque on a current coil =
τ pm × B

 
(k) Potential energy of current coil U = − pm .B

 µ I d  × r µ I d sin θ R1
(l) Biot-Savart Law dB = 0 , dB = 0 i i
4π r3 4π r2
 µ  Iθ R2
(m) Magnetic field at center of an arc subtending angle θ, B =  0 
 4π  R
µ0 NIR 2
(n) Magnetic field at a point on the axis of a N turn coil B = Figure 21.51
2
(z )
32
2 2
+R
µ NI
(o) Magnetic field at center of N turn coil B = 0
2 R

(p) Concentric coils with equal turns

(i) Similar currents flowing in the same direction


µ0 NI µ0 NI µ0  1 1 
Net magnetic field, =B +
= NI  + 
2 R1 2 R 2 2  R1 R 2 
2 1 . 3 4 | Moving Charges and Magnetism

(ii) Similar currents flowing in the opposite direction

µ0 NI µ0 NI
Net magnetic field, =B − R1
2 R1 2 R 2 i i
µ0  1 1  R2
= NI  − 
2  R1 R 2 

 (q) Mutually perpendicular coils Figure 21.52


 µ  2πI
Net Magnetic field, B = 2  0 
 4π  R Y
(r) Dispatched coils
B2
µ0 IR 2
Net Magnetic Field, B= 2 2B1
2 (R 2 + x3 )3/2
R
µ0 IR 2
=
2(x2 + R 2 )3/2
X
µ0 I
(s) Infinite straight wire B = i B1
2πR
µ0 I
(t) Semi-infinite straight wire B =
4 πR

(u) Force per unit length between two parallel currents separated by
dF µ0 I1 I2 Figure 21.53
distance d, =
d 2πd
 
(v) Ampere’s law � ∫ B.d  = µ0 Ienc
µ0 I
(w) Field inside infinite straight wire of circular cross-section B = r
2πR 2
(x) Magnetic Field inside long solenoid having n turns per unit length B = µ0 nI

µ0 NI
(y) Magnetic Field inside toroid having N turns B =
2πr
µ0 2M
(z) Magnetic field due to bar magnet at end-on position B =
4 π d3 Y

(aa) Magnetic field due to bar magnet at broadside-on position


µ M
B= 0
4 π d3 i R
o
90
kφ y=x
(ab) Moving Coil Galvanometer I =
NAB
B
(ac) Magnetic field Intensity H, in vacuum is, H = i
µ0
B X
(ad) Magnetic field Intensity H, in a medium is, H = (0, 0) R
µr µ0
Figure 21.54
P hysi cs | 21.35

Solved Examples

JEE Main/Boards 10-9C and moving in the x-y plane. Find the velocity of
the particle.
Example 1: A uniform magnetic fields of 30 mT
Sol: The force on the particle in external magnetic
exists in the +X direction. A particle of charge +e and   
mass1.67 x 10-27 kg is projected into the field along the field is F = q(v xB) . Take vector product of velocity and
+Y direction with a speed of 4.8 x 106 m/s magnetic field vector.

( )
(i) Find the force on the charged particle in magnitude 
= Given, B = 4 x10−3 k T,q 10−9 C
and direction

(ii) Find the force if the particle were negatively charged. and Magnetic force = Fm (4.0i + 3.0 j)10−10 N

(iii) Describe the nature of path followed by the particle Let Velocity of the particle in x-y plane be, v = ν x i + ν y j
  
in both the cases. Then From the relation, Fm = q(v xB)
Sol: The force on the particle in external magnetic We have,

( )( )
  
field is F = q(v xB) . Take vector product of velocity and (4.0i + 3.0j) x10 10−9  ν x i + ν y j x 4x10−3 k 
−10
=
 
magnetic field vector, and solve for force.
y (
= 4ν X10−12 j − 4ν 10−12 j
y x )
Comparing the coefficients of i and j we have,

v

F 4 x 10-10= 4ν y x10−12
∴ =ν y 102=
m / s 100m / s

x and 3.0 x 10-10= 4ν y x10−12


 
B ∴ ν x =−75m / s ; ∴ V = −75i + 100j

z
Example 3: Figure shows current loop having two
(i) Force acting on a charge particle moving in the circular arcs joined by two radial lines. Find the magnetic
magnetic field field B at the center O.
   
F = q(v xB) Magnetic field B = 30(mT)j i

Velocity of the charge particle V = 4.8 x106 (m / s) j D C

( )( )( )

F = 1.6 x10−19  4.8 x106 j x 30 x10−3 i  A B
 

F =230.4 x 10-16( −k̂ )N.
O
(ii) If the particle were negatively charged, the magnitude
Sol: Find magnetic field at the center O of concentric
of the force will be the same but the direction will be
µ Iθ
along (+z) direction. arcs AB and CD by B = 0 where θ is the angle
4 πR
(iii) As v ⊥ B, the path describe is a circle subtended at the center.
mv
R
= = (1.67x10−27 ) ⋅ (4.8x106 ) / Magnetic field at point O, due to wires CB and AD
qB
(1.6x10−19 ) ⋅ (30x10−3 ) = 1.67 m. will be zero. Magnetic field due to wire BA will be,
 θ  µ i  
B1 =    0  Direction of field B1 is coming out of
Example 2: A magnetic field of (4.0 x 10-3 k ) T exerts a  2π   2a 
force (4.0 i +3.0 j ) x 10-10N on a particle having a charge the plane of the figure. Similarly, field at O due to arc
2 1 . 3 6 | Moving Charges and Magnetism

of uniform magnetic field only, the particle performs


 θ  µ i 
DC will be, B2 =    0  mv
 2π   2a  uniform circular motion of radius r = .
 qB
Direction of field B2 is going into the plane of the Let i , j and k be unit vector along the positive directions
figure. The resultant field at O is of x, y and z axes. Q=charge on the particle=1.6 x 10-19C,
v=velocity of the charged particle
µ0 iθ(b − a)
B = B1 − B2 = Coming out of theplane, z
4 π ab

Example 4: A current of 2.00 A exist in a square loop of


edge 10.0 cm. Find the magnetic field B at the center of
the square loop.

Sol: The center of the loop is equidistant from all x


the sides, and can be considered as a point on the
perpendicular bisector of one side.The field at the point
y
due to one side is
µo Ia =(1.28 x 106)ms-1
B= 
2πd a2 + 4 d2 E = electric field intensity;

=(-102.4 x 103Vm-1) k B =magnetic induction of the
The magnetic field at the center due to the four sides
magnetic field = (8 x 10-2 Wbm-2) j
will be equal in magnitude and direction. The field due 
to one side will be ∴ Fe =electric force on the charge

µ0ia =qE=1.6 x 10-19(-102.4x103)N k = 163.84 x 10-16N(- k )


B1 =
2πd a2 + 4 d2 Fm =magnetic force on the charge = qv x B
= [1.6x10-19(1.28x106)(8x10-2)N]( i x j )= (163.84x10-16N)
Here, a=10 cm and d=a/2=5 cm.
( k )
 
µ0 (2 A)  10 cm

The two forces Fe and Fm are along z-axis and equal,
Thus, B1 =  
2π(5cm)  (10 cm)2 + 4 (5cm)2  opposite and collinear. The net force on the charge is
 
zero and hence the particle does not get deflection and
1 continues to travel along x-axis. (a) At time t=5x10-6s
=2 x 10-7 T mA-1 x 2 A x = 5·66 x 10-6 T
5 2 cm
x=(5 x 10-6)(1.28 x 106)=6.4m∴ Coordinates of the
Hence, the net field at the center of the loop will be particle = (6.4 m,0,0)
4 x 5·66 x 10-6T=22·6x10-6 T. (b) When the electric field is switched off, the particle
is in the uniform magnetic field perpendicular to its
velocity only and has a uniform circular motion in the
Example 5: A particle of mass 1 x 10-26kg and charge
x-z plane (i.e. the plane of velocity and magnetic force),
1.6 x 10-19C travelling with a velocity 1.28 x 106 ms-1
anticlockwise as seen along+ y axis.
in the +x direction enters a region in which uniform
magnetic field of induction B are present such that mv 2
Now, =qvB where r is the radius of the circle.
Ex=Ey=0, Ez = -102.4 kVm-1 and Bx = Bz=0. By=8 x 10-2. r
The particle enters this region at the origin at time
∴r =
mv
=
( )(
1x10−26 1.28 x106 )=1
t = 0. Determine the location (x, y and z coordinates)
of the particle at t= 5 x 10-6s. If the electric field is
qB ( )(
1.6x10−19 8x10−2 )
switched off at this instant (with the magnetic field still
The length of the arc traced by the particle in [(7.5-5)
present), what will be the position of the particle at
x 10-6s]
t = 7.45 x 10-6 s?
1
= (v)(T)=(1.28 x 1060)(2.45 x 10-6)=3.136m= π m=
Sol: In presence of simultaneous electric and magnetic circumference 2
   
( )
field, the Lorentz force is F= q E + (v× B) . Under action ∴ The particle has the coordinates (6,4,0,2m) as (x,y,z).
P hysi cs | 21.37


Example 6: The region between x=0 and x=L is filled Example 7: A uniform, constantmagnetic field B is
directed at an angle of 45° to the x-axis in the xy-plane.
with uniform, steady magnetic field B0k. A particle of
PQRS is a rigid, square wire frame carrying a steady
mass m, positive charge q and velocity v0 î travels along current I0 , with its center at the origin. O. At time t=0,
x-axis and enters the region of magnetic field. Neglect the frame is at rest in the position (shown the Figure)
gravity throughout the question. with its sides parallel to the x and y axes. Each side of
(i) Find the value of L if it emerges from the region of the frame is of mass M and length L.
magnetic field with its final velocity at an angle 30o to y
the initial velocity. S R
(i) Find the final velocity of the particle and the time
spent by it in the magnetic field, if the field now extents
up to x=2. 1L. x
O
Sol: The particle under action of uniform magnetic field
performs uniform circular motion. The magnetic force P Q
acting on it provides the centripetal force. The radius of
mv
the circular orbit is r = .
qB (a) What is the torque τ about O acting on the frame
(i) As the initial velocity of the particle is perpendicular due to the magnetic field?
to the field the particle will move along the arc of a (b) Find the angle by which the frame rotates under the
circle as shown. action of this torque in a short interval of time ∆t , and
v0 i x=L the axis about which this rotation occurs. ( ∆t is so short
o
30 that any variation i n the torque during this interval may
be neglected). Given moment of any variation in the
torque during this interval may be neglected). Given
r B0k moment of inertia of the frame about an axis through
r its center perpendicular to its p late is (4/3) ML2 .
o
30
  
Sol: The torque acting on loop is τ = M × B .

If r is the radius of the circle, then =


α
dω τ
= and
dt I
mv 20
= qv 0B0 Also from geometry, L=r sin 30° y
r
S R
mv 0
⇒r =2L or L =
2qB0  o
45 x
2.1mv 0
(ii) In this =
case L > r Hence the particle will O
2qB0
I0
complete a semi-circular path and emerge from the P Q
field with velocity v 0 ˆi as shown. Time spent by the
πr πm θ = ∫ ωdt (a) As magnetic field B is in x-y plane and
particle in the magnetic field =T = subtends an angle of 45° with x-axis.
v 0 qB0
=Bx Bcos
= 45° B 2
L>r
and
= B y Bsin
= 45° B 2

So in vector from

( ) ( )

=B ˆi B 2 + j B 2
v0 i 
and=
M I= ˆ I L2kˆ
0 Sk 0
The speed of the particle does not change due to the
magnetic field.
2 1 . 3 8 | Moving Charges and Magnetism

   B ˆ B ˆ Consider the figure.


so, τˆ= M × B= I0L2kˆ ×  i+ j
y
 2 2 
I0L2B
i.e.,
= τˆ
2
(
× −ˆi + ˆj ) 2
i.e., torque has magnitude I0L2B and is directed along
line QS from Q to S. 4
1
(b) As by theorem of perpendicular axes, moment of
inertia of the frame about QS, 3 P
x
1 14 2 2 2 a/2
I=
QS = I  ML= ML 3a/2
2 z 2 3  3 5
And as τ = Iα , z
    
( ) + (B ) + (B ) + (B ) + (B )
2 
τ I L B × 3 3 I0B BP = B1 2 3 4 5
α= = 0 = P P P P P
1 2L2M 2 M
 µ0i
As here α is constant, equations of circular motion are
where =
B1 ( ) P  3a 
( −ˆj)
valid and hence from 4π  
 2
1  µ0 i 
θ = ω0 t + αt2 with ω0 =0 we have
2 (Semi-infinite wire) =B2 ( ) P  3a 
( +kˆ ) (B ) 3
P
=0 ;
4 
1 2 1  3 I0B  3 I0B 2  2
 ( ∆t ) =
2  µ0 i
θ=
2
αt = 
22 M  4 M
∆t
( )
B4
=
P a
( −kˆ )
4 
2
Example 8: In the figure shown the magnetic field at
the point P.  µ i   1 1 ˆ  1 
⇒ BP = 0  −  +  j −  1 −  kˆ 
y 2a   3π π   3 

 2µ0i  1 ˆ ˆ   µ0i
BP
⇒=  j − k ⇒ BP
= 1 + π2
3a  π  3πa

Example 9: What is the smallest value of B that can


i be set up at the equator to permit a portion of speed
P 107 m s to circulate around the earth?
x
(a, 0) (2a, 0) (3a, 0) R = 1.67 × 10−27 kg .
6.4 × 106 m,mp =
 
i
Sol: Particle under action of force in uniform magnetic
z
field, moves in circular orbit whose radius is given by
Sol: The conductor forms two concentric semicircles mv
r= . For charged particle orbiting near earth with
Bq
and two straight wires. Find magnetic field at the center high velocity, the magnetic field can be obtained
µ0 Iθ rearranging above formula.
P due to concentric arcs by formula B = , and fields
4 πR mv
From the relation r =
µ0 I Bq
due to straight wires by formula B = and then add mv
4 πd We have B =
the fields due to individual parts. qr
Substituting the values,we have
P hysi cs | 21.39

B
=
(1.67 × 10 )(10 )=
−27 7

1.6 × 108 T f = q  V × B 


f = q V × B (( ))
µ I
((
= q −V0 i ×  4R − i + j 

))
= q −V0 ˆˆi ×  µ00 I −ˆˆi + ˆˆj 
(1.6 × 10 )(6.4 × 10 )
−19 6

 qV µ I 
 4R

qV µ I


 qV µ I  ( ) (−ˆˆi × −ˆˆi= ))
+ ˆˆj  qV0 µ0 I −kˆˆ(( ))
 4R  ( ) (
0 0 0 0
= 0 0
=  4R   − i × − i= +j  4R −k
 4R
 

JEE Advanced/Boards Example 2: A long horizontal wire AB, which is free to


move in a vertical plane and carries a steady current of
20 A, is in equilibrium at a height of 0.01 m over another
parallel long wire CD which is fixed in a horizontal plane
Example 1: A circular loop of radius R is bent along a
and carries a steady current of 30 A, as shown in figure
diameter and given a shape as shown in figure. One of
Show that when AB is slightlydeed it executes simple
the semi-circle (KNM) lies in the x-z plane and the other
harmonic motion. Find the period of oscillation.
one (KLM) in the y-z plane with their centers at origin.
Current I is flowing through each of the semi-circles as A B
shown in Figure.
Sol: The current carrying wire AB, experiences force
L due to the magnetic field created by wire CD. Find the
equation of motion of wire AB. If the force acting on
wire AB is restoring in nature and directly proportional
M y

i to its displacement from the equilibrium position,


then we compare the equation of acceleration with
N
the standard differential equation of SHM. Then time
x
i
ω
period of oscillation is given by T = 2π
K z
Let m be the
g
A particle of charge q is released at the origin with a
mass per unit length of wire AB. At a height x about the
velocity V = −V0 ˆi . Find the instantaneous force F in the
particle. Assume that space is gravity free. wire AB will be given by
Fm
Sol: For wire bent as shown the magnetic field at the i1=20A
µ I( π) A B
center is calculated as B = 0 , where π is the angle
4 πR Fg
X=d=0.01
subtended by the wire at center. The Lorentz force
   C i2=30A D
acting on particle is =
F q(v× B)
µ0i1i2
Magnetic field at the center of a circular wire of radius Fm = (upwards)  … (i)
µ I 2πx
R carrying a current I is given by B = 0
2R Wt. per unit of wire AB is Fg =mg (downwards) At x=d,
wire in equilibrium
In this problem, current are flowing in two semi-circles,
KLM in the y-z plane and KNM in the x-z plane. The µ0 i1i2
i.e., Fm =
Fg ⇒ mg
=
centers of these semi-circles coincide with the origin of 2π d
the Cartesian system of axes. µ0i1i2
mg
  ⇒ =  …(ii)
1  µ0 I  ˆ 1  µ0 I  ˆ d
( ) ( )
2
∴ BKLM 2πd
=   − i ∴ BKNM
=   −j
2  2R  2  2R 
When AB is deed, x decreases therefore, Fm will increase,
µ0 I Fg remains the same. Let
The total magnetic field at the origin is B=
0
4R
( −ˆi + ˆj)
AB is displaced by dx downwards.
It is given that a particle of charge q is released at the Differentiating equation (i) w.r. t.x, we get
origin with a velocity V = −V0 ˆi . The instantaneous force µ0 i1i2
dFm = − dx  …(iii)
acting on this particle is given by 2π x 2
i.e., restoring force, F=d Fm ∝ −dx
2 1 . 4 0 | Moving Charges and Magnetism

Hence the motion of wire is simple harmonic. From


µ0 I2 L xdx µ0 I2 a2 + L2
equation (ii) and (iii), we can write
π ∫0 a2 + x2
= ⇒F = ln
2π a2
 mg 
dFm = −   .dx (x=d)
 d  If the current in B is reversed, the magnetic field due to
g the two wires would be only along
∴Acceleration of wire, a=-   .dx
d Y
Hence period of oscillations B
R
dx disp. a
T=
2π 2π
= P(x,0,0)
a acc. O x
X

0.01 
⇒ T =2π d g =2π ⇒ T =0.2s a  
9.8 R BB BA
Z A
Example 3: A straight segment OC (of length L meter)
of a circuit carrying a current 1 amp is placed along the
x-axis. Two infinitely long straight wire A and B, each x- direction and the force on the current along
extending z = −∞ to + ∞ are fixed at y=-a meter and
x- direction will be zero.
y=+a meter respectively, as shown in the figure. If the
wires A and B each carry a current 1 amp into the plane
of the paper, obtain the expression for the force acting Example 4: Two long wires a and b, carrying equal
on segment OC. What will be the force on OC if the currents of 10.0 A, are placed parallel to each other
current in the wire B is reversed? with a separation of 4.00 cm between them as shown in
figure. Find the magnetic field B at each of the points
Y P, Q and R.
(0,a,0)
B 2.00 cm 2.00 cm 2.00 cm 2.00 cm
i i C
X P a Q b R
O

A Sol: Net field at a point will be the vector sum of the


Z (0,-a,0) fields due to the two wires.
i
The magnetic field at P due to the wire a has magnitude
Sol: Find the net field due to wires A and B at any point
on the wire OC.Find the force due to this field on a small µ0i 4 π × 10−7 TmA −1 × 10A
current element of wire OC at that point. Then integrate B1
= = = 1.00 × 10−4 T.
2πd 2π × 2 × 10−2 m
this expression to find force on wire OC.
Its direction will be perpendicular to the line shown and
Magnetic field BA produced at P(x,0, 0) due to wire, will point downward in the figure. The field at this point
BA =µ0 I 2πR, BB =µ0 I 2πR . due to the other wire has magnitude
Components of BA and BB along x-axis cancel, while
µ0i 4 π × 10−7 TmA −1 × 10A
those along y-axis add up to give total field. B2
= = = 0.33 × 10−4 T.
2πd −2
2π × 6 × 10 m
µ I 2µ0 I x µ0 I x
=B 2  0 = cos θ =
 2πR  2πR R π a + x2
2
( ) Its direction will be the same as that of B1 . Thus, the
resultant field will be 1.33 × 10−4 T also along the same
(along – y direction) direction.

The force dF acting on the current element is Similarly, the resultant magnetic field at R will be
= 1.33 × 10−4 T along the direction pointing upward in
dF = I(d xB) the figure.
µ0 I2x dx The magnetic field at point Q due to the two wires will
=dF = ∴ sin90° 1
π a2 + x2  have equal magnitudes but opposite directions and
hence the resultant field will be zero.
P hysi cs | 21.41

Example 5: A coil of radius R carries current I. Another According to law of conservation of angular momentum,
concentric coil of radius (r<<R) carries current i. Planes I1 ω1 = I2 ω2
of two coils are mutually perpendicular and both the
and according to law of conservation of energy,
coils are free to rotate about a common diameter. Find
maximum kinetic energy of smaller coil when both 1 1
I ω2 + I ω2 =U
the coils are released, masses of coils are M and m 2 1 1 2 2 2
respectively. From above equations, maximum kinetic energy of
1 smaller coil,
Sol: For rotating coils, kinetic energy is I ω2 .
2 1 UI1 µ0 πliMRr 2
2
I=
ω =
Each coil is a magnetic dipole and has a potential
energy in magnetic field due to other coil. This potential
(
2 2 2 I1 + I2 2 MR 2 + mr 2 )
energy is converted into kinetic energy as the dipole
moment of the coil aligns itself with the magnetic field. Example 6: A wire loop carrying a current I is placed in
the x-y plane as shown in Figure.
(a) If a particle with charge q and mass m is placed at
the centerP and given a velocity v along NP find its
instantaneous acceleration.
y
M
If a magnetic dipole having moment M be rotated
through angle θ from equilibrium position in a uniform
magnetic field B, work done on it is W= MB(1 − cos θ) .
q x
This work is stored in the system in the form of energy. 120
o

i
When system is release, dipole starts to rotate to P
occupy equilibrium position and the energy converts
a
into kinetic energy and kinetic energy of the system is
maximum when stored energy is completely released.
N

Magnetic induction, at centers due to current in larger


µ i (b) If an external uniform magnetic induction B = Biˆ is
coil B = 0 Magnetic dipole moment of smaller applied, find the force and torque acting on the loop.
2R
coil is iπr 2 . Initially planes of two coils are mutually
Sol: Find the net magnetic field at the point P due to the
perpendicular, therefore θ is 90° or energy of the arc and the straight wire and find the magnetic force on
q by rules of vector cross product. The magnetic force
system is U = ( )
iπr 2 B (1 − cos90° ) on a current loop in uniform magnetic field is zero.
µ0 Iiπr 2 The toque will be non-zero depending on the angle
U= between field and the area vector of the loop.
2R
When coils are released, both the coils start to rotate (a) As in case of current-carrying straight conductor
about their common diameter and their kinetic energies and arc, the magnitude of B is given by
are maximum when they become coplanar.
y
Moment of inertia of larger coil about axis of rotation is
M
1 1
I1 = mR 2 and that of smaller coil is I2 = mr 2 .
2 2 o
v
90
Since, two coils rotate due to their mutual interaction
only, therefore, if one coil rotates clockwise then the F q x
other rotates anticlockwise. i o P
120
Let angular velocities of larger and smaller coils be a
numerically equal to ω1 and ω2 respectively when they
become coplanar, N
2 1 . 4 2 | Moving Charges and Magnetism

µ0i µ 0 Iφ Example 7: A disc of radius R rotates at an angular


=B1
4 πd
( sin α + sinβ ) and B2 =
4 πr velocity ω about the axis perpendicular to its surface
and passing through its center. If the disc has a uniform
So in accordance with right hand screw rule, charge density σ , find the magnetic induction on the
 axis of rotation at a
( )
B=W
µ0 1
4 π ( acos60 )
ˆ and due to are
× 2sin60( −k)
Sol: The disc can be thought as made-up of elementary
 rings. When disc rotates about axis passing through
(B ) =
MN
µ0 I  2  ˆ
( )
×  π  +k
4π a  3 
center and perpendicular to plane of disc, then each
elementary ring constitutes a current. The magnetic
 field along axis of rotation due to each elementary ring
and hence net B at P due to the givenloop
is to be considered.
  
B BW + BA
= At distance r from the center of disc consider a ring of
radius r and width dr.
 µ 2I  π ˆ
⇒B
= 0
 3 −  ( −k)  …(i) Charge on the ring, dq =( 2πr dr ) σ
4π a  3
dq ωdq
Now as force on charged particle in a magnetic fields Current due to ring is dI = = = σωr dr
T 2π
is given by
Magnetic field due to ring at point P on axis is
  
= (
F q v ×B ) dB =
µ0 dlr 2
or
( )
 32
So=
here, F qvBsin90° along PF 2 r 2 + x2
µ0 σω R r3dr
 µ 2qvI  π
=B ∫=
dB ∫  …(i)
2
(r )
32
=i.e. F 0
 3 −  along PF 0 2
+x 2
4π a  3

 F 2qvI  π Putting r 2 + x2 =
t2 and 2r dr=2t dt and integrating (i)
a = 10−7
and so =  3 −  along PF we get
m a  3
      µ0 σω  R 2 + 2x2 
(b) As dF = =B  − 2x  .
∫ IdL ×B
IdL × B, so F = 2  R 2 + x2 

As here I and B are constant
Example 8: In the figure a charged sphere of mass m
=F I  ∫ dL =
× B 0 as ∫=
dL 0 
and charge q starts sliding from rest on a vertical fixed
circular track of radius R from the position shown. There
Further as area of coil, exists a uniform and constant horizontal magnetic field
 1 1  of induction B. The maximum force exerted by the track
=  πa2 − .2a sin 60° × a cos 60° kˆ
S on the sphere.
3 2 
π 3ˆ m
= a2  − k q
 3 4 
  π 3ˆ
So M = Ia2  −
= IS k
 3 4  X

B
   π 3 ˆ ˆ
and hence τ= M × B= Ia2B  −  k×i
 3 4 
( )
 π 3ˆ
i.e. τ Ia2B  −
=
3 4
(
 jN − m as kˆ × ˆi =ˆj . )
 
Sol: As the sphere moves along the circular track the
vector sum of radial component of magnetic force, the
P hysi cs | 21.43

normal reaction and the radial component of weight of By symmetry, U2 = −U1


the sphere provide the necessary centripetal force.
⇒ −∆U = work done
Fm = qvB , and directed radially outward.
l2
mv 2 mv 2
∴N − mgsin θ + qvB = ⇒
= N + mgsin θ − qvB
R R

Hence at θ = π 2
r
2mgR l1
⇒ Nmax= + mg − qB 2gR
R

= 3mg − qB 2gR. µ IIl b


− (U2 − U1 ) =
= 2 0 1 2 ln .
2π a
Example 9: What is the work done in transferring the The work done in transferring the wire from
wire from position (1) to position (2)? µ0 I1 I2l b
Position 1 to 2 = ln
Sol: While transfering wire from position 1 to position π a
2 find the change in the potential energy of the loop in
the field of the wire. This chage in potential energy will Example 10: A long, straight wire carries a current i. A
be equal to the work done. particle having a positive charge q and mass m, kept at
a distance x0 from the wire is projected towards it with
The loop can be considered as the combination of the a speed v. Find the minimum separation between the
number of elementary loops. The net current in the wire and the particle.
dotted wires is 0 as current in the neighboring loops
flowing through the same wire opposite in direction. Y
Consider an elementary loop of width dr at a distance
r from the wire
i

N x0
ma
O
2 P
A 2 m/s
A
B X
mg
The 'dµ ' magnetic moment of the elemental loop Sol: At minimum separation the x-component of
velocity of the particle will be zero. Find the acceleration
= I2ldr of the particle due to the magnetic force and solve to
get the expression for velocity and displacement.
The B at that point due to straight wire =
µ0 I1 2πr .
Let the particle be initially at P. Take the wire as the y-axis
b and the foot of perpendicular from P to the wire as the
origin. Take the line OP as the x-axis. We have, OP= X0.
l2 The magnetic field B at any point to the right of the
l1 a l a wire is along the negative z-axis. The magnetic force
on the particle is, therefore, in the x-y plane. As there
is no initial velocity along the z-axis, the motion will be
in the x-y plane. Also, its speed remains unchanged. As
1 2 the magnetic field is not uniform, the particle does not
go along a circle.
µ0 I1       µ i 
dU = −B.dµ = − I ldr(cos π) The force at time t is =F qv ×= B q(iv x + jv y ) ×  − 0 k 
2πr 2
 2πx 
[As dµ is anti-parallel to B.]  µ i  µ i
= jqv x 0 − iqv y 0 .
µ0 I1 I2l b 1 µ0 I1 I2l  a  2πx 2πx
U1
= ∫=
du
2π a r ∫ =dr

ln  
b
2 1 . 4 4 | Moving Charges and Magnetism

Fx µ0 qi µ y µy is a constant. Find the magnetic field at a point P at a


Thus ax = = − = −λ  …(i) distance x from the metal sheet.
m 2πm x x
P
µ0 qi
Where λ = .
2πm x

dv x dv x dx v x dv x
Also,=
ax = = . …(ii)
dt dx dt dx
Sol: Field due to the sheet will be symmetric. Field lines
As v 2x + v 2y =
v2 , will be parallel to the sheet at points near it. Select a
rectangular amperian loop and use Ampere’s Law to
2v x dv x + 2v y dv y =
0 find the field.

giving v x dv x = − v y dv y .  …(iii) Consider two strips A and C of the sheet situated


symmetrically on the two sides of P.The magnetic field
From (i), (ii) and (iii), at P due to the strip A is Ba perpendicular to AP and
that due to the strip C is Bc perpendicular to CP. The
v y dv y λv y dx dv y resultant of these two is parallel to the width AC of the
= or = .
dx x x λ sheet. The field due to the whole sheet will also be in
this direction. Suppose this field has magnitude B.
Initially x = x0 and v y = 0 . At minimum separation
from the wire, v x = 0 so that v y = − v . The field on the opposite side of the sheet at the
same distance will also be B but in opposite direction.
x
dx
−v dv y x v Applying Ampere’s law to the rectangle shown in figure.
Thus ∫ = ∫ or, ln = −
x0
x 0
λ x0 λ
2 πmv Ba
− B P
−v λ µ0qi x
or, x x=
= 0e x0 e P

BC x
C x
B
Example 11: Figure shows a cross section of a large O dl
l
metal sheet carrying an electric current along its (a) (b)
surface. The current in a strip of width dl is Kdl where K
1
2Bl = µ0Kl or, B= µ K Note that it is independent of x.
2 0

JEE Main/Boards

Exercise 1 Q.4 A horizontal overhead power line carries a current


of 90 A in east west direction. What is the magnitude
Q.1 A circular coil of wire consisting of 100 turns, each and direction of the magnetic field due to the current
of radius 8.0 cm carries current of 0.40 A. What is the 1.5 m below the line?
magnitude of the magnetic field B at the center of the
coil? Q.5 What is the magnitude of a magnetic force per unit
length on a wire carrying a current of 8 A and making an
Q.2 A long straight wire carries a current of 35 A. What angle of 30° with the direction of a uniform magnetic
is the magnitude of the field B at a point 20 cm from field of 0.15 T?
the wire?
Q.6 In a chamber, a uniform magnetic field of 6.5
Q.3 A long straight wire in the horizontal plane carrier G(1G= 10−4 T) is maintained. An electron is shot into
of 50 A in north to south direction. Give the magnitude the field with a speed of 4.8 × 106 ms−1 normal to the
and direction of Bat a point 2.5 m east of the wire. field. Explain why the path of the electron is a circle.
Determine the radius of the circular orbit.
P hysi cs | 21.45

Q.12 A circular coil of N turns and radius R carries a


(e =
1.6 × 10 −19
9.1 × 10−31 kg
C, me = ) current I. It is unwound and rewound to make another
coil of radius R/2. Current I remaining the same.
Q.7 (i) A circular coil of 30 turns and radius 8.0 cm Calculate the ratio of the magnetic moments of the
carrying a current of 6.0 A is suspended vertically in a new coil and the original coil.
uniform horizontal magnetic field of magnitude 1.0 T.
The field lines make an angle of 60o with the normal of
Q.13 A circular coil of 20 turns and radius 10 cm is
the coil. Calculate the magnitude of the counter torque
placed in a uniform magnetic field of 0.10 T normal to
that must be applied to prevent the coil from turning.
the plane of the coil. If the current in the coil is 5.0 A,
(ii) Would your answer change, if the circular coil in (a) what is the
were replaced by a planner coil of some irregular shape
(a) Total torque on the coil,
that encloses the same area? (All other particulars are
also unaltered.) (b) Total force on the coil
(c) Average force on each electron is the coil due to the
Q.8 Two concentric circular coils X and Y radii 16 cm magnetic field?
and 10 cm, respectively, lie in the same vertical plane
(The coil is made of copper wire of cross-sectional area
containing the north to south direction. Coil X has 20
10−5 m2 , and the free electron density in copper is
turns and carries a current 16 A; coil Y has 25 turns and
given to be about 1029 m−3 .)
carries a current of 18 A. The sense of the current in
X is anticlockwise, and clockwise in Y, for an observer
looking at the coils facing west. Give the magnitude Q.14 State the Biot-Savart law for the magnetic field
and direction of the net magnetic field due to the coils due to a current-carrying element. Use this law to
at their center. obtain a formula for magnetic field at the center of a
circular loop of radius a carrying a, steady current I.
Q.9 A straight horizontal conducting rod of length 0.45
m and mass 60 g is suspended by two vertical wires at Q.15 Give the formula for the magnetic field produced
its ends. A current of 5.0 A is set up in the rod through by a straight infinitely long current-carrying wire.
the wires. Describe the lines of field B in this case.
(a) What magnetic field should be set up normal to the
conductor in order that the tension in the wire is zero? Q.16 How much is the density B at the center of a long
solenoid?
(b) What will be the total tension in the wires if the
direction of current is reversed keeping the magnetic
field same as before? Q.17 A proton shot at normal to magnetic field describe
a circular path of radius R. If a deuteron ( H ) is to
1
2

Q.10 The wires which connect the battery of an move on the same path, what should be the ratio of the
automobile to its starting motor carry a current of 300 velocity of proton and the velocity of deuteron?
A (for a short time). What is the force per unit length
between its wires if they are 70 cm long and 1.5 cm
Q.18 State the principle of cyclotron.
apart? Is the force attractive of repulsive?

Q.19 A charge q is moving in a region where both the


Q.11 A uniform magnetic field of 1.5 T exists in a
magnetic field B and electric field E are simultaneously
cylindrical region of radius10.0 cm, its direction parallel
present. What is the Lorentz force acting on the charge?
to the axis along east to west. A wire carrying current
of 7.0 A in the north to south direction passes through
this region. What is the magnitude and direction of the Q.20 A charged particle moving in a straight line enters
force on the wire if, a uniform magnetic field at an angle of 45° . What will
be its path?
(a) The wire intersects the axis,
(b) The wire is turned from N-S to northeast-northwest Q.21 A current of 1A is flowing in the sides of an
direction,
equilateral triangle of side 4.5 × 10−2 m. Find the
(c) The wire in the N-S direction is lowered from the axis magnetic field at the centroid of the triangle.
by a distance of 6.0 cm?
2 1 . 4 6 | Moving Charges and Magnetism

A 10
m from the wire B, is zero. Calculate
11
l (i) the magnitude and direction of current in B (ii) the
magnitude of magnetic field induction at S
O
l (ii) the force per unit length of the wire B.
o
60
o 60
r

Exercise 2
B D C

Q.22 The radius of the first electron orbit of a hydrogen


atom is 0.5 Å. The electron moves in this orbit with a
uniform speed of 2.2 × 106 ms−1 . What is the magnetic Q.1 A current 1 ampere is flowing through each of
field produced at the center of the nucleus due to the the bent wires as shown figure. The magnitude and
motion of this electron? direction of magnetic field at O is

Q.23 A solenoid is 2 m long and 3 cm in diameter. It R’


has 5 layers of windings of 1000 turns each and carries O
a current of 5 A. What is the magnetic field at itscenter?
R
Use the standard value of µ0 .
µ0i  1 2  µ0i  1 3 
(A)  +  (B)  + 
Q.24 A proton entersa magnetic field of flux density 2.5 4  R R′  4  R R′ 
T with a velocity of 1.5 × 107 ms−1 at an angle of 30° with µ0i  1 µ0i  1 3 
3 
the field. Find the force on the proton. (C)  +  (D)  + 
8  R 2R ′  8  R R′ 

Q.25Two parallel wires one meter apart carry currents of


Q.2 Net magnetic field at the center of the circle O
1A and 3 respectively in opposite directions. Calculate
due to a current carrying loop as shown in figure is
the force per unit length acting between these wires.
( θ < 180°)
Q.26 A solenoid of length 0.4, and having 400 turns
of wire carries a current of 3 A. A thin coil having 10 i i O
turns of wire and radius 0.01 m carries a current 0.4 A.
Calculate the torque required to hold the coil in the
middle of the solenoid with its axis perpendicularto the
axis of the solenoid. (A) Zero
(B) Perpendicular to paper inwards
Q.27 In a circuit shown in figure a voltmeter reads
(C) Perpendicular to paper outwards
30 V, when it is connected across 400ohm resistance.
Calculate what the same voltmeter will read when (D) Is perpendicular to paper inwards if θ ≤ 90° and
connected across the 300 Ω resistance? perpendicular to paper outwards if 90° ≤ θ < 180°
v
Q.3 A charge particle A of charge q=2C has velocity
v=100 m/s. When it passes through point A and
300 400 has velocity in the direction shown. The strength of
magnetic field at point B due to this moving charge is
(r=2 m). v B
o
60v 30
r
Q.28 Two long straight parallel wires are 2m apart,
perpendicular to the plane of the paper. The wire A A
carries a current of 9.6 ampere directed into the plane
(A) 2.5 µT (B) 5.0 µT
of the paper. The wire B carries a current such that the
magnetic field induction at the point P, at a distance of (C) 2.0 µT (D) None
P hysi cs | 21.47

Q.4 Three rings, each having equal radius R, are placed Q.8 A particle of charge q and mass m starts moving
mutually perpendicular to each other and each having from the origin under the action of an electric field
its center at the origin of co-ordinates system. If current ˆ

= E E= 0 i and B B0 ˆi with velocity v = v 0 ˆj . The speed of
is flowing through each ring then the magnitude of the
magnetic field at the common center is the particle will become 2 v 0 after a time
2mv 0 2Bq
y (A) t = (B) t =
qE mv 0

3Bq 3 mv 0
(C) t = (D) t =
mv 0 qE
x
Q.9 An electron is projected with velocity v 0 in a uniform
electric field E perpendicular to the field. Again it is
z projected with velocity v 0 perpendicular to a uniform
magnetic field B. If r1 is initial radius of curvature just
µ0 I
(A) 3 (B) Zero after entering in the electric field and r2 in initial radius
2R of curvature just after entering in magnetic field then
(C) ( 2 −1 ) µ2RI
0
(D) ( 3− 2 ) µ2RI
0 the ratio r1 r2 is equal to

Bv 2 0 B Ev 0 Bv 0
(A) (B) (C) (D)
Q.5 Two concentric coils X and Y of radii 16 cm and E E B E
10 cm lie in the same vertical plane containing N-S
direction. X has 20 turns and carries 16 A. Y has 25 turns Q.10 A uniform magnetic field B = B0 ˆj exists in a space.
& carries 18 A. X has current in anticlockwise direction A particle of mass m and charge q is projected towards
and Y has current in clockwise direction for an observer, negative x-axis with speed v from the point (d, 0, 0).
looking at the coils facing the west. The magnitude of The maximum value v for which the particle does not
net magnetic field at their common center is hit y-z plane is

(A) 5π × 10−4 T towards west


(B) 13π × 10−4 T towards east 2B0 q B0 q B0 q B0 qd
(A) (B) (C) (D)
dm m 2dm 2m
(C) 13π × 10−4 T towards west
(D) 5π × 10−4 T towards east Q.11 Two protons move parallel to each other, keeping
distance r between them, both moving with same
Q.6 Equal current i is flowing in three infinitely long velocity v. Then the ratio of the electric and magnetic
wires along positive x, y and z directions. The magnetic force of interaction between them is.
field at a point (0, 0, -a) would be: (A) c2/v2 (B) 2c2/v2
µ i µ i
2πa
( )
(A) 0 ˆj − ˆi
2πa
( )
(B) 0 ˆi + ˆj (C) c2/2v2 (D) None

µ0i ˆ ˆ µ0i ˆ ˆ ˆ Q.12 Three ions H+ ,He+ and O +2 having same kinetic
(C)
2πa
i−j ( ) (D)
2πa
(
i + j+k ) energy pass through a region in which there width is a
uniform magnetic field perpendicular to their velocity,
Q.7 An electron is moving along positive x-axis. A then:
uniform electric field exists towards negatively y-axis. (A) H+ will be least deflected.
What should be the direction of magnetic field of
(B) He+ and O +2 will be deflected equally.
suitable magnitude so that net force of electron is
zero. (C) O +2 will be deflected most.
(A) Positive z-axis (B) Negative z-axis (D) all will be deflected equally.
(C) Positive y-axis (D) Negative y-axis
Q.13 An electron having kinetic energy T is moving in
a circular orbit of radius R perpendicular to a uniform
2 1 . 4 8 | Moving Charges and Magnetism

magnetic induction B. If kinetic energy is doubled and Q.18 A block of mass m & charge q is released on a
magnetic induction tripled, the radius will become. long smooth inclined plane magnetic field B is constant,
3R 3 uniform, horizontal and parallel to surface as shown.
(A) (B) R Find the time from start when block loses contact with
2 2
the surface.
2 4 mcos θ mcosec θ
(C) R (D) R (A) (B)
9 3 qB qB
mcot θ
Q.14 A charged particle moves in magnetic field (C) (D) None
 qB
B = 10iˆ with initial velocity u= 5i + 4j .
The path of the particle will be. Q.19 A metal ring of radius r=0.5m with its plane normal
to a uniform magnetic field B of induction 0.2T carries
(A) Straight line (B) Circle a current I=100A. The tension in Newton developed in
(C) Helical (D) None the ring is:
q
m
(
Q.15 A electron experiences a force 4.0iˆ + 3.0ˆj x10−13 N ) B

in a uniform magnetic field when its velocity is


2.5k × 107 ms−1 .When the velocity is redirected and 
becomes ( )
1.5iˆ − 2.0ˆj x107 ms−1 , the magnetic force
(A) 100 (B) 50
of the electron is zero. The magnetic field vector B is :
(C) 25 (D) 10
(A) −0.075iˆ + 0.1ˆj (B) 0.1iˆ + 0.075ˆj
(C) 0.075iˆ + 0.1ˆj + kˆ (D) 0.075iˆ + 0.1ˆj Q.20 In the shown a coil of single turn is wound on a
sphere of radius R and mass m. The plane of the coil is
Q.16 An electron moving with a velocity V1 = 2iˆ m / s parallel to the plane and lies in the equatorial plane of
at a point in a magnetic field experiences a force the sphere. Current in the coil is i. The value of B if the
sphere is in equilibrium is
F1 = −2ˆj N . If the electron is moving with a velocity
V = 2ˆj m / s at the same point, it experiences a force
2 B
F = + 2iˆ N . The force the electron would experience if it
2

were moving with a velocity V3 = 2kˆ m / s at the same
point is
mgcos θ mg
(A) Zero (B) 2kˆ N (A) (B)
πiR πiR
(C) −2kˆ N (D) Information is insufficient
mg tan θ mgsin θ
(C) (D)
Q.17 The direction of magnetic force on the electron as πiR πiR
shown in the diagram is along
Q.21 The magnetic moment of a circular orbit of radius
y ‘r’ carrying a charge ‘q’ and rotating with velocity v is
given by
x qvr qvr
(A) (B)
i 2π 2

(C) qvπr (D) qvπr 2


e
(A) y-axis (B) –y-axis (C) z-axis (D) –z-axis
d d d P hysi
d cs | 21.49
(a) (b)

Previous Years’ Questions


d d
Q.1 Two very long straight parallel wires carry steady x x’ x x’
currents I and –I respectively. The distance between the
wires is d. At a certain instant of time, a point charge q is
at a point equidistant from the two wires  in the plane of d d
the wires. Its instantaneous velocity V is perpendicular
(a) (b)
to this plane. The magnitude of the force due to the
magnetic field acting on the charge at this instant is
 (1998) Q.4 A non-planar loop of conducting wire carrying a
current I is placed as shown in the figure. Each of the
µ0 Iqv µ0 Iqv straight section of the loop is of length 2a. The magnetic
(A) (B)
2πd πd field due tothis loop at the point P(a,0,a) points in the
2µ0 Iqv direction (2001)
(C) (D) Zero z
πd y

Q.2 An infinitely long conductor PQR is bent to form a


right angle as shown in Figure. A current I flows through
PQR. The magnetic field due to this current at the point
M is H1. Now, another infinitely long straight conductor x
QS is connected at Q, so that current is I/2 in QR as well
as in QS, the current in PQ remaining uncharged. The
magnetic field at M is now H2. The ratio H1/H2 is given 1 1
by  (2000)
(A)
2
( −ˆj + kˆ ) (B)
3
( −ˆj + kˆ + ˆi )
1 ˆ ˆ ˆ 1 ˆ ˆ
M (C)
3
(
i + j+k ) (D)
2
(
i +k )
Q.5 A coil having N turns is wound tightly in the form of
90
o a spiral with inner and outer radii a and b respectively.
When a current I passes through the coil, the magnetic
 
P l Q 90o S field at the center is  (2001)
µ0NI 2µ0NI
(A) (B)
R b a
µ0NI b µ0 IN b
(C) log (D) log  
 2(b− a) a 2(b− a) a
(A) 1/2 (B) 1
(C) 2/3 (D) 2 Q.6 Two particles A and B of masses mA and mB
respectively and having the same charge are moving in
a plane. A uniform magnetic field exists perpendicular
Q.3 Two long parallel wire are at a distance 2d apart.
to this plane. The speeds of the particles are VA and
They carry steady equal currents flowing out of the
VB respectively and the trajectories are as shown in the
plane of the paper as shown. The variation of the
figure. Then (2001)
magnetic field B along the line XX’ is given by (2000)
A
B

x x’ x x

d d (A) mA v A < mB vB (B) mA v A > mB vB


d d
(a) (b) (C) mA < mB and v A < =
vB (D) mA m
=B and v A vB

d
2 1 . 5 0 | Moving Charges and Magnetism

Q.7 A long straight wire along the z-axis carries a current (A) I > III > II > IV (B) I > II > III > IV
Iin the negative z-direction. The magnetic vector field
(C) I > IV > II > III (D) III > IV > I > II
B at a point having coordinate (x,y) on the z=0 plane is
 (2002)

(A)
(
ˆ ˆ
µ0 I yi − xj
(B)
) ˆ ˆ
µ0 I xi − yj ( ) Q.11 An electron moving with a speed u along the
position x-axis at y=0 enters a region of uniform
2π(x2 + y 2 ) 2π(x2 + y 2 ) 
magnetic field B = −B0kˆ which exists to the right of

(C)
(
µ0 I xjˆ − yiˆ ) (D)
(
µ0 I xiˆ − yjˆ ) y-axis. The electron exits from the region after sometime
2
2π(x + y ) 2
2π(x + y ) 2 2 with the speed v at coordinate y, then (2004)

y
Q.8 A particle of mass m and charge q moves with a
constant velocity v along the positive x-direction. It
enters a region containing a uniform field B directed e- u
along the negative z-direction, extending from x=a x
to x=b. the minimum value of v required so that the
particle can just enter the region x>b is  (2002)

q (b − a ) B qaB q (b + a ) B (A) v>u, y<0


(A) qbB (B) (C) (D) (B) v=u, y>0
m m m 2m
(C) v>u, y>0 (D) v=u, v<0
Q.9 For a positively charged particle moving in a x-y 
plane initially along x-axis, there is a sudden change Q.12 A magnetic field B = −B0 ˆj exists in the region

in its path due to presence of electric and/or magnetic a<x<2a and B = −B0 ˆj , in the region 2a<x<3a, where B0
fields beyond P. The curved path is shown in the x-y
plane and is found to be non-circular. is a positive constant. A positive point charge moving

Which one of the following combinations is possible? with a velocity v = − v 0 ˆi , where v0 is a positive constant,
 y (2003) enters the magnetic field at x=a.
B0

P x
O
0 x
a 2a 3a
   
(A) E= 0;B= bjˆ + ckˆ (B) = ˆ =
E ai; B ckˆ + aiˆ
    -B0
(C) E= 0;B= cjˆ + bkˆ (D) = ˆ =
E ai; B ckˆ + bjˆ
The trajectory of the charge in this region can be like
 (2007)
Q.10 A current carrying loop is placed in a uniform
magnetic field in four different orientations, I, II, III and z z
IV, arrange them in the decreasing order of potential
energy  (2003) a 2a 3a
(A) x (B) x
a 2a 3a


n B B
z z

n
(I) (II)
(C) (D) a 2a 3a
 x x
n a 2a 3a

B B

n
(III) (IV)
P hysi cs | 21.51

Q.13 Which of the field patterns given in the figure Q.17 The coercivity of a small magnet where the
is valid for electric field as well as for magnetic field? ferromagnet gets demagnetized is 3 × 103 Am−1 . The
 (2011) current required to be passed in a solenoid of length 10
cm and number of turns 100, so that the magnet gets
demagnetized when inside the solenoid, is: (2014)
(A) (B) (A) 3A (B) 6A (C) 30 mA (D) 60 mA

Q.18 A rectangular loop of sides 10 cm and 5 cm carrying


a current I of 12 A is placed in different orientations as
shown in the figures below:
z z
(C) (D) I
B B
(A) (C)
I I I
y y
I
Q.14 A long insulated copper wire is closely wound as a x x

spiral of N turns. The spiral has inner radius a and outer z


radius b. The spiral lies in the X-Y plane and a steady z
current I flows through the wire. The Z-component of B
I
the magnetic field at the center of the spiral is  (2011) (B)
B
I (D)
I y
I y I
x
µ0NI b µ0NI b +a I I
(A) ln   (B) ln   x I
2(b − a)  a  2(b − a)  b − a 
z z
µ0NI
b µ NI  b + a 
(C) ln   (D) 0 lnI 
2b  a  B
2b  b − a  B
(A) (C)
I I I
Q.15 Proton, Deuteron and alpha particle of the same y y
kinetic energy are moving in circular I trajectories in a
x
constant magnetic field. The
x
radii of proton, deuteron
and alpha particle are respectively rp, rd and ra. Which z
one of the following relations is correct?
z  (2012)
B
(A) rα= rp= rd (B) rαB= rp < rd I

(B) I (D)
I y
(C) rα > rd > rp (D) Irα= rd > rp y I
I
x I
x I
Q.16 Two short bar magnets of length 1 cm each have If there is a uniform magnetic field of 0.3 T in the positive
magnetic moments 1.20 Am2 and 1.00 Am2 respectively. z direction , in which orientations the loop would be
They are placed on a horizontal table parallel to each in (i) stable equilibrium and (ii) unstable equilibrium?
other with their N poles pointing towards the South.  (2015)
They have a common magnetic equator and are
separated by a distance of 20.0 cm. The value of the (A) (a) and (c), respectively
resultant horizontal magnetic induction at the mid (B) (b) and (d), respectively
- point O of the line joining their centres is close to
(Horizontal component of earth’s magnetic induction is (C) (b) and (c), respectively
3.6 × 10−5 Wb / m2 ) (2013) (D) (a) and (b), respectively
−4 2 −4 2
(A) 2.56 × 10 Wb / m (B) 3.50 × 10 Wb / m
(C) 5.80 × 10−4 Wb / m2 (D) 3.6 × 10−5 Wb / m2
2 1 . 5 2 | Moving Charges and Magnetism

JEE Advanced/Boards

Exercise 1 when a current of 1amp enters in the loop and taken


out of it by two long wires as shown in the figure.

Q.1 A system of long four parallel conductors whose


sections with the plane of the drawing lie at the vertices
of a square there flow four equal currents. The directions O
of these currents are as follows:  45
o

1amp 90
o
I1 I2
1amp


Q.5 Find the magnetic induction at the origin in the


figure shown.
y
Those marked ⊗ point away from the reader, while
those marked with a dot point towards the reader.
How is the vector of magnetic induction directed at the i A i
center of the square? z i
i
x
Q.2 A long straight wire carriers a current of 10A directed
along the negative y-axis as shown in figure. A uniform
Q.6 Find themagnetic induction at point O, if the current
magnetic field B0 of magnitude 10-6T is directed parallel
carrying wire is in the shape shown in the figure.
to the x-axis. What is the resultant magnetic field at the
following points?
r I
z O
y r

Q.7 Find the magnitude of the magnetic induction


B of a magnetic field generated by a system of thin
x conductors along which a current I is flowing at a point
A(O,R,O), that is the center of a circular conductor of
i radius R. The ring is in the yz plane.

Y
(a) x=0, z=2m ;
(b) x=2m, z=0 R

(c) x=0, z=-0.5m R RR


X
R
Q.3 Find the magnetic field at the center P of square of I
side a shown in figure. Z

Q.8 A cylindrical conductor of radius R carriers a current


along its length. The current density J, however, is not
P uniform over the cross section of the conductor but is
i i a function of the radius according to J=br, where b is a
constant. Find an expression for the magnetic field B.
Q.4 What is the magnitude of magnetic field at the
(a) at r1 < R (b) at distance r2 < R , measured from the
center ‘O’ of loop of radius 2 m made of uniform wire axis
P hysi cs | 21.53

i that is given by β = αy where α is constant. Find the


total magnetic force on the loop if it carries current i.
R
Y
a

Q.9 Electric charge q is uniformly distributed over a rod B


a
of length L. The rod is placed parallel to a long wire
carrying a current I. The separation between the rod X
and the wire is a. Find the force needed to move the rod
Q.16 A particle of charge +q and mass m moving
along its lengths with a uniform velocity V.
under the influence of a uniform electric field E î and a
magnetic field B k̂ enters in I quadrant of a coordinate
Q.10 An electron moving with a velocity 5x106 ms−1ˆj system at a point (0, a) with initial velocity v î and leaves
in the uniform electric field of 5x107 Vm−1ˆj . Find the the quadrant at a point (2a, 0) with velocity −2v ˆj . Find
magnitude and direction of a minimum uniform Magnitude of electric field
magnetic field in tesla that will cause the electron to (a) Rate of work done by the electric field at point
move undeviated along it original path.
(b) (0, a) Rate of work done by both the fields at.

Q.11 A charged particle (charge q, mass m) has velocity (c) (2a, 0).
V0 at origin in +x direction. In space there is a uniform
magnetic field B in –z direction. Find the y coordinate of Q.17 A square current carrying loop made of thin wire
the particle when it crosses y axis. and having a mass m=10g can rotate without friction
with respect to the vertical axis OOI , passing through
Q.12 A proton beam passes without deviation through the center of the loop at right angles to two opposite
a region of space where there are uniform transverse sides of the loop. The loop is placed in a uniform
mutually perpendicular electric and magnetic field with magnetic field with an induction B=10-1T directed at
E and B. Then the beam strikes a grounded target. Find right angles to the plane of the drawing. A current
the force imparted by a beam on the target if the beam I=2A is flowing in the loop. Find the period of small
current is equal to I. oscillations that the loop performs about its position of
stable equilibrium.

Q.13 A conducting circular loop of radius r carriers a O B


constant current i. It is placed in a uniform magnetic
field B0 such that B0 is perpendicular to the plane of the
i
loop. Find the magnetic force acting on the loop.

Q.14 An arc of a circular loop of radius R is kept in the O1


horizontal plane and a constant magnetic field B is
applied in the vertical direction as shown in the figure. Q.18 An infinitely long straight wire carries a
If the carries current I then find the force on the arc. conventional current I as shown in the figure. The
rectangular loop carries a conventional current I’ in
x x x x x
x the clockwise direction. Find the net force on the
Bx
x x x x x rectangular loop.
I
x x x x x x

x x x o
x x x
90
x x x x x x
i’ c
x x x x x x

a
Q.15 A rectangular loop of wire is oriented with the
left corner at the origin, one edge along X-axis and b
the other edge along. Y-axis as shown in the figure. A
magnetic field is into the page and has a magnitude
2 1 . 5 4 | Moving Charges and Magnetism

Q.19 3 Infinitely long thin wires each carrying current i (b) If an external uniformmagnetic induction field
in the same direction, are in the x-y plane of a gravity B = Biˆ is applied, find the torque acting on the loop
free space. The central wire is along the y-axis while due to the field.
the other two are along x = ±d. (i) Find the locus of the
points for which the magnetic field B is zero. Q.23 (a) A rigid circular loop of radius r & mass m lies in
(ii) If the central wire is displaced along the Z-direction the xy plane on a flat table and has a current I flowing
by a small amount & released, show that it will execute in it. At this particular place, the earth’s magnetic field
simple harmonic motion. If the linear density of the B Bx ˆi + B y ˆj . How large must I be before one edge
is =
wires is λ , find the frequency of oscillation. of the loop will lift from table?
B Bx ˆi + Bzk.
(b) Repeat if,= ˆ
Q.20 Q charge isuniformly distributed over the same
surface of a right circular cone of semi-vertical angle Q.24 A conductor carrying a current is placed parallel
θ and height h. The cone is uniformly rotated about a current per unit width j0 and width d, as shown in the
its axis at angular velocity ω . Calculated associated Figure.
magnetic dipole moment.
z


 j0 h
h d  y

Q.21 Four long wires each carrying current I as shown


Find the force per unit length on the conductor.
in the figure are placed at the point A, B, C and D. Find
the magnitude and direction of
Q.25 The figure shows a conductor of weight 1.0N and
Y length L= 0.5m placed on a rough inclined plane making
an angle 300 with the horizontal so that conductor is
D(-a,a) + A(a,a)
perpendicular to a uniform horizontal magnetic field
X of induction B=0.10 T. The coefficient of static friction
C(-a,a) + B(a,-a) between the conductor and the plane is 0.1. A current
of I=10A flows through the conductor inside the plane
of this paper as shown. What is the force that should be
(i) Magnetic field at the center of the square. applied parallel to the inclined plane for sustaining the
conductor at rest?
(ii) Force per metre acting on wire at point D.

Q.26 An electron gun G emits electron of energy 2kev


Q.22 A wire loop carrying current I is placed in the X-Y
traveling in the (+) ve x-direction. The electron are
plane as shown in the figure.
required to hit the spot S where GS=0.1m & line GS
M makes an angle of 600 with the x-axis, as shown in the
Y figure. A uniform magnetic field B parallel to GS exists
a
V in the region outside to the electron gun. Find the
minimum value of B needed to make the electron hit S.
120o

I I P
a S
X B
N B
o
60
(a) If a particle with charge +Q and mass m is placed at
Gun X
the center P and given a velocity along NP (see figure).
Find its instantaneous acceleration.
P hysi cs | 21.55

Q.27 Two coils each of 100 turns are held such that one Exercise 2
lies in the vertical plane with their centers coinciding.
The radius of the vertical coil is 20cm and that of the Single Correct Choice Type
horizontal coil is 30cm. How would you neutralize the
magnetic field of the earth at their common center?
What is the current to be passed through each coil? Q.1 Two very long straight parallel wires, parallel to -y
Horizontal component of earth’s magnetic induction = direction, respectively. The wire are passes through the
3.49 x 10-5T and angle of dip=300. x-axis at the point (d, 0, 0) and (-d, 0, 0)respectively. The
graph of magnetic field z-component as one moves
along the x-axis from x=-d to x=+d, is best given by
Q.28 An infinite wire, placed along z-axis, has current
i1 in positive z-direction. A conducting rod placed in o x
xy plane parallel to y-axis has current i2 in positive (A) (B)
y-direction. The ends of the rod subtend +300 and -600 o
at the origin with positive x-direction. The rod is at a
distance a from the origin. Find net force on the rod.
(C) x (D) o x
o
Q.29 A square loop of wire of edge a carries a current i.
(a) Show that B for a point on the axis of the loop and a Q.2 A long thin walled pipe of radius R carries a current
distance x from its center is given by, I along its length. The current density is uniform over
4µ0ia2 the circumference of the pipe. The magnetic field at the
B= center of the pipe due to quarter portion of the pipe
( )( 4x )

1/2
π 4x2 + a2 2
+ 2a2 shown, is

(b) Can the result of the above problem be reduced to


give field at x=0?

Q.30 A straight segment OC (of length L meter) of a µ0 I 2 µ0 I


circuit carrying a current I amp is placed along the (A) (B)
4π R 2
π2R
x-axis. Two infinitely line straight wires A and B, each
extending from z = −∞ to + ∞ , are fixed by y=−a 2µ0 I 2
(C) (D) None
meter and y=+a meter respectively, as shown in the π2R
Figure.

Q.3 An electron (mass=9.1 x 10-31; charge=-1.6 x 10-19C)


y
experiences no deflection if subjected to an electric field
B of 3.2 x 105 V/m and a magnetic field of 2.0 x 10-3 Wb/
m2. Both the fields are normal to the path of electron
and to each other. If the electric field is removed, then
O x the electron will revolve in an orbit of radius:
C
(A) 45m (B) 4.5m (C) 0.45m (D) 0.045m
z A
Q.4 A particle of specific charge (charge/mass) α starts
moving from the origin under the action of an electric
If the wires A and B each carry a current I amp into
plane of the paper. Obtain the expression for the force field E = E0 ˆi and magnetic field B = B0kˆ . Its velocity at
acting on the segment OC. What will be the force OC if
current in the wire B is reversed?
( x , y 0 ) is ( 4iˆ − 3ˆj) . The value of x
0 0, 0
is:

13 αE0 16αB0
(A) (B)
2 B0 E0

25 5α
(C) (D)
2αE0 2B0
2 1 . 5 6 | Moving Charges and Magnetism

Q.5 A particle of specific charge (q/m) is projected from Q.9 A conducting wire bent in the form of a parabola
the origin of coordinates with initial velocity [ui-vj]. y 2 = 2x carriers a current i=2A as shown in figure. This
Uniform electric magnetic field exist in the region along wire is placed in a uniform magnetic field B = −4kˆ Tesla.
the +y direction, of magnitude E and B. The particle will The magnetic force on the wire is (in newton).
definitely return to the origin once if y(m) A
(A)  vB / 2πE  is an integer

( )
1/2 2 x(m)
(B) u2 + v 2 B / πE  is an integer
B
(C)  vB / πE  in an integer
(A) −16iˆ (B) 32iˆ
(D) uB / πE  is an integer.
(C) −32iˆ (D) 16iˆ

Q.6 Two particles of charges +Q and –Q are projected Q.10 A semicircular current carrying wire having radius
from the same point with a velocity v in a region of R is placed in x-y plane with its center at origin ‘O’.
uniform magnetic field B such that the velocity vector There is non-uniform magnetic field
makes an angle θ with the magnetic field. Their masses  B x
are M and 2M, respectively. Then, they will meet again B = o kˆ (here Bo is +ve constant) is existing in the
for the first time at a point whose distance from the 2R
region. The magnetic force acting on semicircular wire
point of projection is
will be along
(A) 2πMv cos θ / QB (B) 8πMv cos θ / QB Y
(C) πMv cos θ QB (D) 4 πMv cos θ / QB
i
Q.7 A particle with charge +Q and mass m enters a
magnetic field of magnitude B, existing only to the (-R,0,0) (+R,0,0) X
right of the boundary YZ. The direction of the motion
of the particle is perpendicular to the direction of B. Let Z
(A) –x-axis (B) +y-axis
2πM
T= . The time spent by the particle in the field
QB (C) –y-axis (D) +x-axis
will be
Q.11 A square loop ABCD, carrying a current I, is placed
(A) T θ (B) 2T θ
near and coplanar with a long straight conductor XY
 π + 2θ   π − 2θ  carrying a current I, the net force on the loop will be
(C) T   (D) T  
 2π   2π 
B C
+Q
Y
m x B i L
x A D
 x L/2 L
x
2µ0 Ii µ0 Ii 2µ0 Iil µ0 Iil
x (A) (B) (C) (D)
Z 3π 2π 3π 2π
Q.8 In the previous question, if the particle has-Q
charge, the time spend by the particle in the field will Q.12 A conducting ring of mass 2kg and radius 0.5m is
be placed on a smooth horizontal plane. The ring carries
a current i=4A. A horizontal magnetic field B=10T is
(A) T θ (B) 2T θ
switched on at time t=0 as shown in figure. The initial
 π + 2θ   π − 2θ  angular acceleration of the ring will be
(C) T   (D) T  
 2π   2π 
P hysi cs | 21.57

(A) Its x-coordinate can never be positive


 (B) Its x- and z-coordinates cannot both be zero at the
B
same time.
(C) Its z-coordinate can never be negative.
(D) Its y-coordinate will be proportional to the square
(A) 40π rad / s2 (B) 20π rad / s2 of its time of flight.
(C) 5π rad / s2 (D) 15π rad / s2
Multiple Correct Choice Type
Q.13 In the following hexagons, made up of two
different material P and Q, current enters and leaves Q.16 Which of the following statements is correct:
from points X and Y respectively. In which case the (A) A charged particle enters a region of uniform
magnetic field at its center is not zero. magnetic field at an angle 850 to magnetic lines of
Q
Y
Q
Y
force. The path of the particle is a circle.
Q P
P P (B) An electron and proton are moving with the same
(A) (B) kinetic energy along the same direction. When they
P Q Q Q pass through uniform magnetic field perpendicular to
X
P
X
P their direction of motion, they describe circular path.
(C) There is no change in the energy of a charged
Q P particle moving in a magnetic field although magnetic
Y Y
P Q force acts on it.
Q Q
(C) (D) (D) Two electrons enter with the same speed but in
P Q P Q opposite direction in a uniform transverse magnetic
X
P
X
P field. Then the two describe circle of the same radius
and these move in the same direction.
Q.14 Current flows through uniform, square frames as
shown. Q.17 Consider the magnetic field produced by a finitely
long current carrying wire.
In which case is the magnetic field at the center of the
frame not zero? (A) The lines of field will be concentric circles with
centers on the wire.
(B) There can be two points in the same plane where
(A) (B) magnetic fields are same.
(C) There can be large number of points where the
magnetic field is same.

(C) (D) (D) The magnetic field at a point is inversely proportional


to the distance of the point from the wire.

Q.15 In a region of space, a uniform magnetic field Q.18 A long straight wire carriers a current along the
B exists in the y-direction. A proton is fired from the x-axis. Consider the points A(0,1,0), B(0,1,1), C(1,0,1)
origin, with initial velocity v making a small angle α and D(1,1,1). Which of the following pairs of points will
with the y-direction in the yz plane. In the subsequent have magnetic field of the same magnitude?
motion of the proton, z (A) A and B (B) A and C (C) B and C (D) B and D

B
Q.19 Consider three quantities x=E/B, = y 1 / µ0 ε0
1
v and z = . Here, l is the length of a wire, C is a
CR

y capacitance and R is a resistance. All other symbols
O B
have standard meanings.
x
2 1 . 5 8 | Moving Charges and Magnetism

(A) x,y have the same dimensions (B) While deflecting in magnetic field its energy
gradually increases.
(B) y, z have the same dimension
(C) Only the component of magnetic field perpendicular
(C) z, x have the same dimensions
to the direction of motion
(D) None of the three pairs have the same dimensions.
of the charged particle is effective in deflecting it.
(D) Direction of deflecting force on the moving charged
Q.20 Two long thin, parallel conductors carrying equal
particle is perpendicular to its velocity.
currents in the same direction are fixed parallel to the
x-axis, one passing through y=a and the other through
Assertion Reasoning Type
y=-a. The resultant magnetic field due to the two
conductors at any point is B. Which of the following (A) Statement-I is true, statement-II is true and
are correct? Statement-II is correct explanation for Statement-I.
z (B) Statement-I is true, statement-II is true and statement-
II is NOT the correct explanation for statement-I.
(C) Statement-I is true, statement-II is false.
-a (D) Statement-I is false, statement-II is true.
O a y
i
i Q.24 Statement-I: A charged particle can never move
x
along a magnetic field line in absence of any other
force.
(A) B=0 for all points on the x-axis Statement-II: Force due to magnetic field is given by
  
(B) At all points on the y-axis, excluding the origin, B ( )
F = q v xB .
has only a z-component.
(C) At all points on the z-axis, excluding the origin, B Q.25 Statement-I : It is not possible for a charged
has only an x-component. particle to move in a circular path around a long
straight uncharged conductor carrying current under
Q.21 An electron is moving along the positive X-axis. the influence of its magnetic field alone.
You want to apply a magnetic field for a short time so Statement-II: The magnetic force (if nonzero) on a
that the electron may reverse its direction and move moving charged particle is normal to its velocity.
parallel to the negative X-axis. This can be done by
applying the magnetic field along.
Q.26 Statement-I: For a charged particle to pass through
(A) Y-axis (B) Z-axis a uniform electro-magnetic field without change in
velocity, its velocity vector must be perpendicular to
(C) Y-axis only (D) Z-axis only
the magnetic field.

Q.22 Two identical charged particles enter a uniform Statement-II: Net Lorentz force on the particle is given

magnetic field with same speed but at angles 300 and by=F q E + v xB 
600 with field. Let a, b and c be the ratio of their time
periods, radii and pitches of the helical paths then Q.27 Statement-I: Two long parallel conductors
carrying current in the same direction experience a
(A) abc=1 (B) abc > 1
force of attraction.
(C) abc < 1 (D) a=bc
Statement-II: The magnetic fields produced in the
space between the conductors are in the same direction.
Q.23 Consider the following statements regarding
a charged particle in a magnetic field. Which of the
Q.28 Statement-I: Ampere law can be used to find
statement are true :
magnetic field due to finite length of a straight current
(A) Starting with zero velocity, it accelerates in a carrying wire.
direction perpendicular to the magnetic field.
P hysi cs | 21.59

Statement-II: The magnetic field due to finite length Q.30 The magnetic field at C due to curved part is
of a straight current carrying wire is symmetric about µ0 I
the wire. (A) , directed into the plane of the paper

µ0 I
Q.29 Statement-I: A pendulum made of a non- (B) , directed towards you

conducting rigid massless rod of length  is attached to
a small sphere of a mass m and charge q. The pendulum µ0 I
(C) , directed towards you
is undergoing oscillations of small amplitude having 3α
time period T. Now a uniform horizontal magnetic field µ0 I
(D) , directed up the plane of the paper
out of plane of page is switched on. As a result of this 3α
change, the time period of oscillations will change.

 Q.31 A wire loop carrying a current I is shown in figure.


B The magnetic field induction at C due to straight part

is

m,q
a

120o
C
Statement-II: In the situation of statement-I, after the I I
magnetic field is switched on the tension in string will a
change (except when the bob is at extreme position).

Comprehension Type 3µ0 I


(A) , directed up the plane of the paper
2πα
Paragraph 1: Magnetic field intensity (B) due to current
µ0 I
carrying conductor can be calculated by use of Biot- (B) , directed into the plane of the paper
Savart law. Which is 6α
µ0 Idlxr µ0 I
B= , (C) , directed towards you
4π r3 6α
µ0 I  3 1 
(D)  −  towards you
R 2α  π 3 
I 
I  O R 
Q.32 The net magnetic field at C due to the current
carrying loop is directed into the plane of the paper
µ0 I
(A) Zero (B)
α
where dB is magnetic field due current element Idl at
µ0 I µ I 3µ0 I
a position r from current element. For straight wire (C) (D) B =
− 0 + ,
carrying current magnetic field at a distance R from 9α 6a 2πa
wire is
µ0 I Paragraph 2: A current carrying coil behave like short
=B
4π R
( sin α + sinβ ) magnet whose magnetic dipole moment M=nIA. Where
direction of M is taking along the direction of magnetic
And magnetic field due to a circular arc at its center is fields on its axis and n is no of turns A is area of coil and
µ0 I I is current flowing through coil. When such a coil is put
=B .θ in magnetic field (B) magnetic torque ( τ ) acts on it as
4 πR
τ = −MxB and potential energy of the current loop in
where θ angle of circular arc at center, R is radius of
the magnetic field is u=-M.B.
circular arc.
2 1 . 6 0 | Moving Charges and Magnetism

Q.33 A current of 3A is flowing in a plane circular coil of Q.35 In above question, to hold the current-carrying
radius 1cm and having 20 turns. The coil is placed in a coil with the normal to its plane making an angle of 900
uniform magnetic field of 0.5 Wbm-2. Then, the dipole with the direction of magnetic induction, the necessary
moment of the coil is torque is
(A) 3000Am2 (B) 0.3Am2 (A) 1500 Nm (B) 9.4 x 10-3 Nm
(C) 75 Am2 (D) 1.88x10−2 Am2 (C) 15 Nm (D) 150 Nm

Q.34 A current of 3A is flowing in a plane circular coil


of radius 1cm and having 20 turns. The coil is placed
in a uniform magnetic field of 0.5 Wbm-2. Then, the
P.E. of the magnetic dipole in the position of stable
equilibrium is
(A) -1500 J (B) -9.4 mJ
(C) +0.15 J (D) +1500 J

Match the Column

Q.36 Two wires each carrying a steady current I are shown in four configuration in column I. Some of the resulting
effects are described in column II. Match the statement in column I with the statements in column II and indicate
your answer by darkening appropriate bubbles in the 4 x 4 matrix given in the ORS.

Column I Image Column II


(A) Point P is situated midway (p) The magnetic fields (B) at P due to
between the wires the currents in the wires are in the same
P
direction.

(B) Point P is situated at the mid- (q) The magnetic fields (B) at P due to
point of the line joining the centers the current in the wires are in opposite
of the circular wires, which have directions.
same radii. P

(C) Point P is situated at the (r) There is no magnetic field at P.


mid-point of the line joining the
P
centers of the circular wires, which
have same radii.
P hysi cs | 21.61

Q.37 Six point charges, each of the same magnitude q, are arranged in different manners as shown in column II. In
each case, a point M and a line PQ passing through M are shown. Let E be the electric field and V be the electric
potential at M (potential at infinity is zero) due to the given charge distribution when it is at rest. Now, the whole
system is set into rotation with a constant angular velocity about the line PQ. Let B be the magnetic field at M and
µ be the magnetic moment of the system in this condition. Assume each rotating charge to be equivalent to a
steady current.

Column-I Image Column-II


(A) E=0 Charges are at the corners of a regular
+ - Q hexagon. M is the center of the hexagon. PQ
is perpendicular to the plane of the hexagon.
- +
M

P + -

(B) V ≠ 0 Charges are on a line perpendicular to PQ at


P
equal intervals. M is the midpoint between
- + - + - + the two innermost charges.
M

(C) B=0 Charges are placed at the corners of a


rectangle of sides a and 2a and at the mid
+ - +
Q points of the longer sides. M is at the center
of the rectangle. PQ is parallel to the longer
- M - sides.

P
+

- - Charges are placed at the corners of a


(D) µ ≠ 0 + rectangle of sides a and 2a and at the mid
M points of the longer sides. M is at the center
P Q of the rectangle. PQ is parallel to the longer
sides.
- + -
2 1 . 6 2 | Moving Charges and Magnetism

Previous Years’ Questions Q.3 A superconductor has TC(0)=100K . When a


magnetic field of 7.5 Tesla is applied, its TC decreases to
Q.1 Statement I: The sensitivity of a moving coli 75K. For this material one can definitely say that when
galvanometer is increased by placing a suitable (Note: T=Tesla)  (1987)
magnetic material as a core inside the coil.  (2008) (A) B=5T, TC(B)=80K
Statement II: Soft iron has a high magnetic permeability (B) B=5T, 75K < TC(B) < 100K
and cannot be easily magnetized or demagnetized.
(C) B=10T, 75K < TC(B) < 100K
(A) Statement-I is true, statement-II is true and
(D) B=10T, TC(B)=70K
Statement-II is correct explanation for Statement-I.
(B) Statement-I is true, statement-II is true and statement-
Q.4 A proton moving with a constant velocity passes
II is NOT the correct explanation for statement-I.
through a region of space without any change in its
(C) Statement-I is true, statement-II is false. velocity. If E and B represent the electric and magnetic
fields respectively. Then, this region of space may have
(D) Statement-I is false, statement-II is true.
 (1985)
Passage: (Q.2-Q.3) (A) E=0, B=0 (B) E=0, B≠0
Electrical resistance of certain material, known as (C) E≠0, B=0 (D) E≠0, B≠0
superconductors, changes abruptly from a non-zero
value to zero as their temperature is lowered below
Q.5 A particle of charge
a critical temperature TC ( 0 ) . An interesting property y 
+q and mass m moving v E
of superconductors is that their critical temperature P
under the influence of a 
becomes smaller than TC ( 0 ) if they are placed in a
uniform electric field Eiˆ
B
magnetic field i.e., the critical temperature TC (B ) is a
and uniform magnetic a
function of the magnetic strength B. The dependence
field Bkˆ follows a
of TC (B ) on B is shown in the Figure. Q
trajectory from P to Q as 2a 2v
x
shown in Figure. The
TC(B)
velocities at P and Q are
viˆ and − 2ˆj . Which of the following statement (s) is/are
TC(0) correct? (1991)

3  mv 2 
(A) E =  
4  qa 
O B
(B) Rate of work done by the electric field at P is
Q.2 In the graphs below, the resistance R of a
superconductor is shown as a function of its temperature 3  mv 2 
T for two different magnetic fields B1 (solid line) and  
4  a 
B2 (dashed line). If B2 is larger than B1 , which of the
following graphs shows the correct variation of R with (C) Rate of work done by the electric field at P is zero
T in these fields? (2010)
(D) Rate of work done by both the fields at Q is zero
R R B2
B1 Q.6 H+ ,He+ and O2+ all having the same kinetic energy
(A) (B) pass through a region in which there is a uniform
magnetic field perpendicular to their velocity. The
T T
masses of H+ ,He+ and O2+ are 1 amu, 4 amu and 16
O B2 B1 O
amu respectively. Then (1994)

R R (A) H+ will, be deflected most


B1
B2 (B) O2+ will be deflected most
(C) (D)
B1 B2
(C) He+ and O2+ will be deflected equally
O T O T
(D) All will be deflected equally
P hysi cs | 21.63

Q.7 Which of the following statement is (are) correct in (A) They will never come out of the magnetic field
the given Figure? (2006) region

infinitely long wire kept perpendicular (B) They will come out travelling along parallel axis
C
to the paper carrying current inwards B (C) They will come out at the same time
l1
l2 (D) They will come out at different times.
O O’
Q.10 Consider the motion of a positive point charge in
a region where there are
 simultaneous  uniform electric
A and magnetic fields E = E0 ˆj and B = B0 ˆj . At time t =

D 0, this charge has velocity v in the x-y plane, making
(A) Net force on the loop is zero. an angle θ with the x-axis. Which of the following
option(s) is(are) correct for time t > 0 ? (2012)
(B) Net torque on the loop is zero.
(A) If θ =0o , the charge moves in a circular path in the
(C) Loop will rotate clockwise about axis OO' when x-z plane.
seen from O
(B) If θ =0o , the charge undergoes helical motion with
(D) Loop will rotate anticlockwise about OO’ when seen constant pitch along the y-axis.
from O
(C) If θ =10o , the charge undergoes helical motion
with its pitch increasing with time, along the y-axis
Q.8 A particle of mass m and charge q. moving with
velocity v enters Region II normal to the boundary as (D) θ =90o , the charge undergoes linear but accelerated
shown in the Figure. Region II has a uniform magnetic motion along the y-axis.
field B perpendicular to the plane of the paper. The
length of the Region II is l. Choose the correct choice Q.11 A cylindrical cavity of diameter a exists inside
(s).  (2008) a cylinder of diameter 2a as shown in the figure.
Both the cylinder and the cavity are infinitely long. A
Region I Region II Region III
uniform current density J flows along the length. If the
x x x x magnitude of the magnetic field at the point P is given
x x x x N
by µ aJ , then the value of N is  (2012)
v
x x x x 12 0
x x x x
x x x x

a
l P O
qlB
(A) The particle enters Region III only if its velocity> .
m
(B) The particle enters Region III only if its velocity 2a
qlB Q.12 A loop carrying current I lies in the x-y plane as
v< .
m shown in the figure. The unit vector k̂ is coming out of
(C)Path length of the particle in Region II is maximum the plane of the paper. The magnetic moment of the
qlB current loop is - (2012)
when velocity v= . y
m
(D) Time spent in Region II is same for any velocity v as
long as the particle returns to Region I.
I
Q.9 An electron and a proton are moving on straight a x
parallel paths with same velocity. They enter a semi-
infinite region of uniform magnetic field perpendicular
a
to the velocity. Which of the following statement(s) is/
are true? (2011)
2 1 . 6 4 | Moving Charges and Magnetism

π  50πM
(C) The magnitude of the magnetic field units.
(A) a2 I kˆ (B)  + 1  a2 I kˆ 3Q
2 
100πM
(D) The magnitude of the magnetic field is
π  units 3Q
(C) −  + 1  a2 I kˆ (D) (2π + 1)a2 I kˆ
2 
Q.15 Two bodies, each of mass M, are kept fixed with a
Q.13 An infinitely long hollow conducting cylinder with separation 2L. A particle of mass m is projected from the
inner radius R/2 and outer radius R carries a uniform midpoint of the line joining their centres, perpendicular
current density along
 its length. The magnitude of the to the line. The gravitational constant is G. The correct
magnetic field, | B | as a function of the radial distance statement(s) is (are) (2013)
r from the axis is best represented by  (2012)
(A) The minimum initial velocity of the mass m to escape
GM
the gravitational field of the two bodies is 4
  L
(A) B (B) B
(B) The minimum initial velocity of the mass m to escape
GM
R/2 R the gravitational
R/2 R field of the two bodies is 2
L

  (C) The minimum initial velocity of the mass m to escape


(B) BB
(A) (B) B

B
(D) 2GM
(C) B the gravitational field of the two bodies is
L
R/2
R/2 RR
r (D) TheR
R/2 Renergy of the mass m remains constant.
R/2 R
Q.16 Two parallel wires in the plane of the paper are
  distance X0 apart. A point charge is moving with speed
(D) B  (D) B
(C) B u between the wires in the same plane at a distance X1
(B) B from one of the wires. When the wires carry current of
magnitude I in the same direction, the radius of curvature
r r R
R/2 R
R of the path of the point charge is R1. In contrast, if the
R/2 R currents I in the two wires have directions opposite to
each other, the radius of curvature of the path is R 2 .
X0 R1
 If = 3 , the value of is (2014)
(D) B X1 R2

Q.17 When d ≈ a but wires are not touching the loop,


r it is found that the net magnetic field on the axis of the
R
loop is zero at a height h above the loop. In that case
 (2014)
Q.14 A particle of mass M and positive charge Q, moving
with a constant velocity u1 = 4imsˆ −1 , enters a region of (A) Current in wire 1 and wire 2 is the direction PQ and
uniform static magnetic field normal to the x-y plane. RS, respectively and h ≈ a
The region of the magnetic field extends from x = 0 (B) Current in wire 1 and wire 2 is the direction PQ and
to x = L for all values of y. After passing through this SR, respectively and h ≈ a
region, the particle emerges on the other side after 10
 (C) Current in wire 1 and wire 2 is the direction PQ and
milliseconds with a velocity= u2 2( 3iˆ + ˆj)m / s−1 . The
SR, respectively and h ≈ 1.2a
correct statement(s) is (are)  (2013)
(D) Current in wire 1 and wire 2 is the direction PQ and
(A) The direction of the magnetic field is -z direction.
RS, respectively and h ≈ 1.2a
(B) The direction of the magnetic field is +z direction.
P hysi cs | 21.65

Q.18 Consider d >> a, and the loop is rotated about its Q.20 Consider two different metallic strips (1 and 2) of
diameter parallel to the wires by 30° from the position the same material. Their lengths are the same, widths
shown in the figure. If the currents in the wires are in are w1 and w2 and thicknesses are d1 and d2, respectively.
the opposite directions, the torque on the loop at its Two points K and M are symmetrically located on the
new position will be (assume that the net field due to opposite faces parallel to the x-y plane (see figure). V1
the wires is constant over the loop) (2014) and V2 are the potential differences between K and M
in strips 1 and 2, respectively. Then, for a given current I
µ0 I2a2 µ0 I2a2 flowing through them in a given magnetic field strength
(A) (B)
d 2d B, the correct statement(s) is(are)  (2015)
(A) If w1 = w2 and d1 = 2d, then V2 = 2V1
3µ0 I2a2 3µ0 I2a2
(C) (D) (B) If w1 = w2 and d1 = 2d2, then V2 = V1
d 2d
(C) If w1 = 2w2 and d1 = d2, then V2 = 2V1
Q.19 A conductor (shown in the figure) carrying constant (D) If w1 = 2w2 and d1 = d2, then V2 = V1
current
 I is kept in the x-y plane in a uniform magnetic
field B . If F is the magnitude of the total magnetic force Q.21 Consider two different metallic strips (1 and 2)
acting on the conductor, then the correct statement(s) of same dimensions (lengths  , with w and thickness
is (are). (2015) d) with carrier densities n1 and n2, respectively. Strip
y 1 is placed in magnetic field B1 and strip 2 is placed
in magnetic field B2, both along positive y-directions.
R R Then V1 and V2 are the potential differences developed
I /6 /4
x between K and M in strips 1 and 2, respectively.
L R R L Assuming that the current I is the same for both the
 strips, the correct option(s) is(are) (2015)
(A) If B is along ẑ, F ∝ (L + R)
 (A) If B1 = B2 and n1 = 2n2, then V2 = 2V1
(B) If B is along x̂, F = 0
 (B) If B1 = B2 and n1 = 2n2, then V2 = V1
(C) If B is along ŷ, F ∝ (L + R)
 (C) If B1 = 2B2 and n1 = n2, then V2 = 0.5V1
(D) If B is along ẑ, F = 0
(D) If B1 = 2B2 and n1 = n2, then V2 = V1

PlancEssential Questions
JEE Main/Boards JEE Advanced/Boards
Exercise 1 Exercise 1
Q. 7 Q.8 Q.12 Q.4 Q.5 Q.16 Q.19
Q.20 Q.25 Q.22 Q.25 Q.30
Q.26 Q.27
Exercise 2
Exercise 2 Q.1 Q.3 Q.11 Q.13
Q.2 Q.5 Q.20 Q.15 Q.19 Q.20 Q.22
Q.40 Q.42 Q.44 Q.45
Q.48 Q.50
2 1 . 6 6 | Moving Charges and Magnetism

Answer Key

JEE Main/Boards
Exercise 1
Q.1 π × 10−4 T ≈ 3.1 × 10−4 T Q.2 3.5 × 10−5 T
Q.3 4 × 10−6 T, vertically up Q.4 1.2 × 10−5 T, towards south
Q.5 0.6N m−1 Q.6 4.2cm
Q.7 (i) 3.1 Nm, (ii) No Q.8 5π × 10−4=
T 1.6 × 10−3 T towards west
Q.9 (a) A horizontal magnetic field to magnitude 0.26T normal to the conductor in such a direction that Fleming’s
left-hand rule gives a magnetic force upward. (b) 1.176N
Q.10 1.22N m−1
Q.11 (a) 2.1 N vertically downwards (b) 2.1N vertically downwards (c) 1.68N vertically downwards
Q.12 2:1
Q.13 (a) Zero (b) Zero (c) Force on each electron in evB=IB(nA)= 5 × 10−25 N .
Note: Answer (c) Denotes only the magnetic force.
µ0 I µ0 I
Q.14 B = Q.15 B =
2R 2πR
Q.16 B = µ0 I N where N is the number of turns per unit length and I is the current flowing through the solenoid.
   
Q.17 2:1 Q.19 F =qE + q(v × B)
Q.20 Circle Q.21 4 × 10−5 T
Q.22 B=14.1 Wb Q.23 1.57 × 10−2 T
Q.24 3 × 10−12 Q.25 6 × 10−7 Nm−1
Q.26 5.9 × 10−6 N m Q.27 22.5V
Q.28 (i) 8A (ii) 3 × 10−7 T (iii) 7.68 × 10 –6 Nm−1

Exercise 2
Q.1 D Q.2 C Q.3 A Q.4 A Q.5 A Q.6 A
Q.7 B Q.8 D Q.9 D Q.10 B Q.11 A Q.12 B
Q.13 C Q.14 C Q.15 A Q.16 A Q.17 A Q.18 C
Q.19 D Q.20 B Q.21 B

Previous Years’ Questions


Q.1 D Q.2 C Q.3 B Q.4 D Q.5 C Q.6 B
Q.7 A Q.8 B Q.9 B Q.10 C Q.11 D Q.12 A
Q.13 C Q.14 A Q.15 B Q.16 A Q.17 A Q.18 B
P hysi cs | 21.67

JEE Advanced/Boards
Exercise 1
Q.1 In the plane of the drawing from right to left
Q.2 (a) 0 (b) 1.41 × 10−6 T, 45° in xz plane, (c) 5 × 10−6 T , +x-direction
(1 − 2 2)µ0 I
Q.3 k̂ Q.4 zero
πa
µ0 I  3 ˆ 1 ˆ  µ0i  3 
Q.5  k + j  Q.6  π + 1
4R  4 π  4 πR  2 

µ0i µ0br12 µ0br3


Q.7 B
=
4 πR
(
2 2π2 − 2π + 1 ) Q.8 B1
= =
3
, B2
3r2

µοiqv
Q.9 Q.10 10kˆ
2πa
2mv 0 mEI
Q.11 Q.12
qB Be

Q.13 Zero Q.14 2 IRB ˆj

3mv 2 3mv 3
Q.15 F = αa2ijˆ Q.16 (a) , (b) , (c) zero
4qa 4a

m µ0 I I'c  1 1 
Q.17 T0 =
2π 0.57s
= Q.18  −  to the left
6IB 2π  a b 

d I µ0 Qω
Q.19 (i) z=0, x = ± (ii) Q.20 h2 tan2 θ
3 2πd πλ 4

µ0  4I  µ0  I2  1
Q.21 (i)   along Y-axis,  (ii)   10 ,tan4   + π with positive axis
4π  a   
4 π  2a  3

Qvµ0 I  3 3   π 3  2ˆ
Q.22(a)  − 1    (b)
= τ BI  − a j
m 6a  π   
 3 4 
mg mg µ0iJ0  d
Q.23 (a) I = (b) I = Q.24 tan−1   −kˆ ( )
( ) πrBx π  2h 
1/2
πr B2x + B2y

Q.25 0.62N<F<0.88N = 4.7 × 103 T
Q.26 Bmin

µ0 I1 I2
Q.27 i 1 0.1110A,i
= = 2 0.096A Q.28 In (3) along –ve z direction

µ0 I2  a2 
Q.29 (b) Yes Q.30 F = In   , zero
2π  L2 + a2 

Exercise 2

Single Correct Choice Type


Q.1 C Q.2 A Q.3 C Q.4 C Q.5 C Q.6 D
2 1 . 6 8 | Moving Charges and Magnetism

Q.7 C Q.8 D Q.9 B Q.10 A Q.11 A Q.12 A


Q.13 A Q.14 C Q.15 C

Multiple Correct Choice Type


Q.16 B, C Q.17 A Q.18 B, D Q.19 A, B, C Q.20 A, B, C, D Q.21 A, B
Q.22 A, D Q.23 C, D

Assertion Reasoning Type


Q.24 D Q.25 B Q.26 D Q.27 C Q.28 D Q.29 D

Comprehension Type

Paragraph 1:
Q.30 A Q.31 A Q.32D
Paragraph 2:
Q.33D Q.34 B Q.35 B

Matric Match Type or Match the Column


Q.36 A → q, r; B → p; C → q, r; D → q, or; A → q, r; B → p; C → q, r; D → q, s
Q.37 A → p, r, s; B → r, s; C → p, q; D → r,s

Previous Years’ Questions


Q.1 C Q.2 C Q.3 B Q.4 A, B, D Q.5 A, B, D Q.6 A, C
Q.7 A, C Q.8 A, C, D Q.9 B, D Q.10 C, D Q.11 5 Q.12 B
Q.13 D Q.14 A, C Q.15 B Q.16 3 Q.17 C Q.18 B
Q.19 A, B, C Q.20 A, D Q.21 A, C

Solutions

µ0 × 90 µ0
JEE Main/Boards Sol 4: B =
3
=
π
× 30 = 1.2 × 10–5T,
2π ×
2
Exercise 1 towards south.

µ0NI µ0 × 100 × 0.4 1


Sol 1: B = = = 3.1 × 10–4T Sol 5: F =I  ×B = 8 × 1 × 0.15 × = 0.6 Nm–1
2R 2
2 × 8 × 10−2
µ0 I a
Sol 6: F = ·
2
µ0 I µ0 × 35 a a2
Sol 2: B = = = 3.5 × 10–5T 4 π x2 + x2 +
2πr 1 4 2
2π ×
5 Since force is always perpendicular to velocity so path
will be a circle
µ0 I µ0 × 50
Sol 3: B = k̂ = k̂ = 4 × 10–6T,
2πr 5 mv 9.1 × 10−31 × 4.8 × 106
2π × R= = = 4.2 cm
2 qB 1.6 × 10−19 × 6.5 × 10−4
vertically upward.
P hysi cs | 21.69

Sol 7: (i) τ = M × B; M = INA  20 


=iB×  
τ = 6 × 30 × π (0.08)2 ×1 × sin60º = 3.1 Nm  100 

(ii) No = 7 × 0.2 × 1.5= 2.1 N vertically downwards.

−µ0 × 20 × 16 µ0 × 25 × 18 (c)
Sol 8: B = +
16 10 10
2× 2× 6
100 100
= 5 π × 10–4T toward west 8 8

Sol 9: For tension to be zero


(a) F = I  ×B = mg Effective length of wire is 16 cmin the magnetic field so
2mKE 
= 5 ×0.45 × B = F = i  ×B
qB
0.6 = 7 × 0.16 ×1.5
B= = 0.26 T
5 × 0.45 = 1.68 N downwards

(b) By force equilibrium


Sol 12: Length of wire = N × 2 π R
2T = Mg + F N × 2πR
Final no. of turns = = 2N
= 0.06 × 9.8 + 0.06 × 9.8 = 1.176 N R

2
Sol 10: I = 300A; Force per unit length = F Magnetic moment µ = INA

µ0i1i2 µ0 × (300)2 µ1 N1 A1 N × πR 2 2
F= = = 1.2 Nm–1 = = =
2πd 3 µ2 N2 A2 2 1
2π × × 10−2 R 
2 2N × π  
Since the direction of the current is in opposite direction 2
in the wire, the force will be repulsive in nature. Sol 13: N = 20
r = 0.1 m
Sol 11: B = 1.5T; r = 0.1m B = 0.1 T
I=5A
B = 1.5 T 
(a) τ = M × B = MB sin0º = 0

7A (b) F = i  × B

F = Total force is zero as  is zero for a closed loop
 
 (c) Force on each electron = q v × B
(a) F = i  × B IB
= eVB = = 5 × 10–25 N
= 7 × 0.2 ×1.5= 2.1 N vertically downwards. nA

(b) Sol 14: Refer page 21.11 to 21.14

 Sol 15: Refer page 21.11 and 21.12

 Sol 16: B at the centre is B = µ0NI


F = i 1 × B
N - number of turns

= i 1Bsinθ I - current
2 1 . 7 0 | Moving Charges and Magnetism

mv Sol 25: Force per unit length


Sol 17: R =
qB
µ0i2
RqB F= = 6 × 10–7 N/m
v= 2πd
m
vp RqB md
= =2 Sol 26: Magnetic field inside the solenoid is
vd mp RqB
400
B = µ0NI = µ0 × ×3
0.4
Sol 18: Refer page 21.25 to 21.26 = 4 π ×10–7 ×3 ×1000 = 12 π × 10–4 T

 Torque on the coil is τ = M × B = MB
 
Sol 19: F = q E + q v × B
= 0.4 × 10 × π (0.01)2 × 12 π × 10–4 =5.9 ×10–6 Nm

Sol 20: Its path will be a circle.


Sol 27: Let the resistance of the voltmeter be R
Voltage across 300 Ω = 60 – 30 V= 30 V
Sol 21: Magnetic field due to side BC is BBC
30
µ I  3 I= = 0.1 A
3 3 3µ0 I 300
BBC = 0  +  ; BBC =

4 πR  2 
2  4 πR Let equivalent resistance of voltmeter and 400Ω be Req
Magnetic field due to all sides will be equal so IReq = 30V

3 3µ0 I 0.10 Req = 30 V


Bnet =
4 πR R × 400
Req = 300 Ω =
R + 400
Sol 22: Electron moving in a circle will act like a loop
carrying current I. 3R + 1200 = 4R
q qω qv Resistance of voltmeter = R = 1200 Ω
So,I = = =
t 2π 2πR When voltmeter is connected to 300 Ω
µ0 I µ0 qv
So magnetic field at centre = B = = 1200 × 300
2R 4 πR 2 Req = = 240 Ω
1500

9 × 109 × 1.6 × 10−19 × 2.2 × 106 60 6


Thus B = 14.1 Wb i= A= A
( ) 640 64
2
0.5 × 10−10
6
Voltage measured = ×240 = 22.5 V
64
Sol 23: B = µ0 ni
Sol 28: (i) i = 9.6A
5000
= 4 π × 10 ×–7
× 5 = 1.57 ×10–2T
2 2m
⊗ ⊙
Sol 24: B = 2.5 T 10
m
v = 1.5 × 10 m/s 7
11
  1
F = qv × B = q × 1.5 × 107× 2.5 × 10
2 B at m from wire B is
2.5 11
= 1.6 × 10–19 ×1.5 ×107 × × 3 × 10–12
2
µ0 × 9.6 µ0 I
B= − 0
=
 12  10
2π   2π ×
 11  11
P hysi cs | 21.71

9.6 × 10 µ0
I= = 8A = µ0 × 1250= × 5000π = 5π × 10−4 T
12 4π
µ0i1i2
(ii) Force per unit length F = Sol 6: (A)
2π × 2 y
µ0 × 9.6 × 8
= = 7.68 × 10–6 Nm-1
4π a

x
Exercise 2
z
Sol 1: (D) Total magnetic field at point O
µ0i ˆ µ0i ˆ
Magnetic field = B = – i+ j
µ0  3 µ0  1 2πa 2πa
=B +
2R ' 4 2R 4

µ   3 1 Sol 7: (B) E = −K1ˆj ;
= 0  + 
8 R ' R  K1 is some constant

V = K 2ˆi
Sol 2: (C)
  
F = q V × B + qE = 0
    
⇒ V × B = − E ⇒ B = −k̂
R

Sol 8: (D) Final velocity of the particle

2
µ0 I  θ  ˆ  qEt 
Magnetic field B =   ( −k) =v= v 20 +  = 2v0
2R  2π   m 

  2
 qEt 
 µ0 q V × r µ qV sin θ v 20 + 2
 = 4 v0
Sol 3: (A) B = = 0  m 
4π r3 4π r2
2
µ0 2 × 100 × sin30º  qEt  2 3 mv 0
= × = 10−7 × 25 =2.5mT   = 3v 0 ⇒ t =
4π 4  m  qE

Sol 4: (A) Magnetic field at the common centre is Sol 9: (D)

µ0 I ˆ µ0 I ˆ µ0 I ˆ
i+ j+ k •
2R 2R 2R qE
µ0 I
= 3
2R V0

When electric field is applied


Sol 5: (A) Magnitude of magnetic field at the centre
mv 20
= qE
µ0 × 20 × 16 µ0 × 25 × 18 R1
=+ −
2 × 16 × 10 −2 2 × 10 × 10−2 mv 20
R1 =
qE
3
= – µ0 × 10 + µ0 × 2250
2 1 . 7 2 | Moving Charges and Magnetism

When magnetic field is applied mv 2mKE


Sol 13: (C) R = =
mv 0 qB qB
R2 =
qB
2m(2KE) 2
R’ = =R
R1 mv 20 qB v0 B q(3B) 9
= =
R2 qEmv 0 E
Sol 14: (C)
Sol 10: (B) Vy = 4
B0

Vx = 5
(d, 0, 0) B =10
V
Y-component of velocity will make the particle to move
in circle whereas x-component of velocity will make
particle move along x-axis.
For the particle to not hit y-z plane radius of the particle So motion is helical.
should be less than equal to d

mv Sol 15: (A) Force on a particle moving in magnetic field


R= ≤d  
qB0 is qv × B .
 
qB0 d (4iˆ + 3ˆj) × 10−13 =1.6 × 10–19× 2.5 ×107 (K × B)
vmax =
m
Force will be zero if direction of magnetic field and
velocity is same.
kq1q2 kq2 
Sol 11: (A) Electric force Fe= = So B = (0.6 î × –0.8 ˆj ) B
r2 r2
µ qv  ⇒ (4 î + 3 ˆj ) = 1.6 × 25 ( k̂ × (0.6 î – 0.8 ˆj ) B
Magnetic force = qv  0 ×  
2 
 4π r  ⇒ B = –0.075 î + 0.1 ˆj
 µ0  1
Fm= q2v2  4 π  2  
  r Sol 16: (A) Force acting on particle = q. v × B

Fe k 1 c2 ⇒ q.2 î × B = –2 ˆj
= = = 
Fm µ  v 2 ε0 µ0 v2 ⇒ B is in +ve z direction ( k̂ )
v 2  0 
 4π  Electric force on the particle is zero.

mv 2mKE So when v3 =2 k̂ , force is zero.


Sol 12: (B) R = =
qB qB
Sol 17: (A) Magnetic field is in (– k̂ ) direction
2KE × 1 × mp So direction of force
R = (mp = mass of proton)   
H+ eB F = q v ×B

2KE × 4 × mp F̂ = –[– î ×(– k̂ )]= ˆj


R =
He+ eB
F
Sol 18: (C)
2KE × 16 × mp
R =
O+2 2eB

So, R =R mgcos 
He+ O +2
mgsin 
P hysi cs | 21.73

F = qVB 2π 2πr
t= =
Particle will leave the inclined plane when ω v

F = mgcos θ ⇒ qvB = mgcos θ q.πr 2 v qrv


=M =
2πr 2
mgcos θ
v=
qB

Time taken to reach v is t Previous Years’ Questions


v = gsin θ t
v mgcot θ mcot θ Sol 1: (D) Net magnetic field due to both the wires will
t= = =
gsinθ qgB qB be downward as shown in the figure.


Sol 19: (D) y v
⊗ z
y
×
I –I

x
x
T B
T d
→ →
Since, angle between v and B is 180°.
 
F = I ∫ d × B
Therefore, magnetic force
= I (2r î )× (–0.2 k̂ ) = 20 ˆj → → →
Fm = q ( v × B ) = 0
Magnetic force is in +ve y direction
So balancing force on semi-circular ring we get Sol 2: (C) H1 = Magnetic field at M Due to PQ +
magnetic field at M due to QR
2T = 20 ⇒ T = 10N
But magnetic field at M due to QR = 0
Sol 20: (B) ∴ Magnetic field at M due to PQ (or due to current I
 in PQ)= H1
 B
B Now H2 = Magnetic field at M due to PQ
(current I) + magnetic field at M due to QS (current I/2)
+ magnetic field at M due to QR
H1 3 H 2
B =H1 + +0= H1 ; 1 =
2 2 H2 3
 Note: Magnetic field at any point lying on the current
B carrying straight conductor is zero.
Torque due to magnetic field will be balanced by gravity.
B=0
mgsin θ R = I× π R × B sin θ
2

mg
B=
πiR
i
Sol 21: (B) Magnetic field = I × A

q.πr 2
M=
t
2 1 . 7 4 | Moving Charges and Magnetism

Sol 3: (B) If the current flows out of the paper, the Note: This is a common practice, when by assuming
magnetic field at points to the right of the wire will be equal currents in opposite directions in an imaginary
upwards and to the left will be downwards as shown in wire (here AB) loops are completed and solution
figure. becomes easy.


B Sol 5: (C) Consider an element of thickness dr at a
i distance r from the centre. The number of turns in this
  N 
B element,dN =   dr
 b – a
B Magnetic field due to this element at the centre of the
coil will be
Now, let us come to the problem.
µ0 (dN)I µ0 IN dr
Magnetic field at C = 0 dB = = .
2r 2 b–a r
Magnetic field in region BX’ will be upwards (+ve) r =b
because all points lying in this region are to the right of µ0NI b
both the wires.
∴B= ∫ dB = 2(b – a)
ln  
a
r =a

X X’
C B
A
b
Magnetic field in region AC will be upwards dr
(+ve),because points are closer to A, compared to B. r
Similarly magnetic field in region BC will be downwards
a
(–ve).
Graph (B) satisfies all these conditions. Therefore,
correct answer is (B).

Sol 4: (D) The magnetic field at P(a, 0, a) due to the Note: The idea of this question is taken from question
loop is equal to the vector sum of the magnetic fields number 3.245 of IE Irodov.
produced by loops ABCDA and AFEBA as shown in the
figure. mv
Sol 6: (B) Radius of the circle =
Bq
C
or radius ∝ mv if B and q are same.
P(a, 0, a)
(Radius)A> (Radius)B; ∴ mAvA> mBvB


D B î Sol 7: (A) Magnetic field at P is B , perpendicular to OP
E
in the direction shown in figure.
ˆj
y
P(x, y)
F r
A
 
Magnetic field due to loop ABCDA will be along î and i  B
× x
due to loop AFE BA, along k̂ . Magnitude of magnetic o
field due to both the loops will be equal. Therefore,
1
direction of resultant magnetic field at P will be
2
( î + k̂ ).
P hysi cs | 21.75

→ Sol 11: (D) Magnetic force does not change the speed
So, B = B sin θ î – B cos θ ˆj of charged particle. Hence, v = u. Further magnetic field
on the electron in the given condition is along negative
µ0 I
Here, B = y-axis in the starting. Or it describes a circular path in
2πr clockwise direction. Hence, when it exits from the field,
y x y < 0.
sin θ = and cos θ =
r r Therefore, the correct option is (D)
→ µ0 I 1 µ I(yiˆ – xj)
ˆ
∴B = . (y î – x ˆj ) = 0 → → →
2π r 2
2π(x + y 2 )
2
Sol 12: (A) F m = q ( v × B )
(as r2 = x2 + y2) ∴ Correct option is (A)

Sol 8: (B) If (b – a) ≥ r Sol 13: (C) Correct answer is (C), because induced
electric field lines (produced by change in magnetic
(r = radius of circular path of particle)
field) and magnetic field lines form closed loops.
The particle cannot enter the region x > b.
So, to enter in the region x > b Sol 14: (A) If we take a small strip of dr at distance r
from centre, then number of turns in this strip would
mv q(b – a)B
r > (b – a)or > (b – a)or v >  N 
Bq m be, dN =   dr
b – a
Sol 9: (B) Electric field can deviate the path of the Magnetic field due to this element at the centre of the
particle in the shown direction only when →it is along coil will be
negative y-direction. In the given options E is either
µ0 (dN)I µ0NI dr
zero or along x-direction. Hence, it is the magnetic field dB = =
which is really responsible for its curved path. Options 2r (b – a) r
(a) and (c) cannot be accepted as the path will be r =b
µ0NI b
circular in that case. Option (d) is wrong because in that ∴B= ∫ dB = ln
n  
case component of net force on the particle also comes r =a 2(b – a) a
in k̂ direction which is not acceptable as the particle is
moving in x-y plane. Only in option (b) the particle can
Sol 15: (B)
move in x-y plane.
→ → → → 2mK
In option (d) F net = q E + q ( v × B ) r= ⇒r∝ m
Bq q
Initial velocity is along x-direction. So, let rα= rp < rd

v =v î

F net = qa î + q [(v î ) × (c k̂ + b ˆj )]
Sol 16: (A) Bnet = BM + BM + BH
1 2
= qa î – qvc ˆj + qvb k̂

µ0M1 µ0M2
= + + BH
In option (b) F net = q (a î ) + q[(v î ) × 4 πx 3
4 πx3

(c k̂ + a î )]= qa î – qvc ˆj µ0
= (M1 + M2 ) + BH
4 πx3
→ →→
Sol 10: (C) U = – MB = – MB cos q 10−7
→ = × 2.2 + 3.6 × 10−5
Here, M = magnetic moment of the loop 10 −3
→ →
θ = angle between M and B
= 2.56 × 10−4 Wb / m2
U is maximum when θ = 180° and minimum when θ = 0°.
So, as θ decreases from 180° to 0° its PE also decrease.
2 1 . 7 6 | Moving Charges and Magnetism

Sol 17: (A) µ0H =µ0ni µ0 × 10


= 10–6 î +
1
3 100 2π ×
3 × 10= × i ⇒=
i 3A 2
0.1
= 10–6 î + 4 × 10–7 × 10 î

Sol 18: (B) Since B is uniform,
 only torque acts on a
current carrying loop. =

τ (IA) × B = 5 ×10–6 î T
   
A = Ak for (b) and A = −Ak for (d).
Sol 3: Magnetic field can be found as the super position

∴ τ =0 for both these cases. of both given below.
  
The energy of the loop in the B field is: U = −IA ⋅ B ,
which is minimum for (b).

I
JEE Advanced /Boards y 45º 45º
a/2
Exercise 1
x
Sol 1: I1 = I2 = I3 = I4
z
⇒ F1 = F2 = F3 = F4 = F a
⇒ 2F Magnetic field due to loop = Bl

 
2F  
µ0 I  1 1 
= – 
  a   2
+   × 4kˆ
45º 2 
2 F  4π   
2
45º    2 

2F I1 I2
Resultant force will be 2 2 F from right to left 1 ⊗ F2+F4
⊗2
45º
Sol 2: Let magnetic field due to wire be Bw Fres
45º
(a)x = 0, z = 2m; F1+F3
µ0 I ˆi ⊙ ⊙3
B = B 0 + Bw = – + 10–6 î 4
2π × 2
µ0 I
= –10–7 ×10 î + 10–6 î =– × 2 ×4 k̂
2πa
=0
2 2µ0 I
(b)x = 2m, z = 0 = − k̂
πa
µ0 I
B = B 0 + Bw = k̂ + 10–6 î µ0 Ikˆ
2π × 2 Magnetic field due to infinite length wire = Bw =
a
µ I 2π  
B = 10–6 k̂ + 10–6 î = 2 ×10–6 T = 0 k̂ 2
πa
(c)x=0, z=–0.5m (1 − 2 2)µ0 I
Net magnetic field = k̂
πa
B= B0 + Bw
P hysi cs | 21.77

Sol 4: Sol 5: Magnetic Induction

y  µ I 1 µ I1 µ I
B = 0   kˆ + 0   kˆ + 0 ˆj
2 2(2R)  4  2R  4  4 πR
45º 1
x µ0 I  3 ˆ 1 ˆ 
1amp 1 =  k + j
4R  4 π 
z
i2
Sol 6: Magnetic Induction
π  π
1  µ0 I  3 2  µ I
i1 = 2 × 1 = amp B=   kˆ + 0 kˆ
2π 4 2R  2π  4 πR
 
 
1 3 µ0 I 3 ˆ µ0 I ˆ µ I  3π 
i2 = 1 – = amp = × k+ k = 0  + 1 kˆ
4 4 2R 4 4 πR 4 πR  2 
1
µ0    3π 
4  
Magnetic field due to i1 = B1 =–    2  k̂ Sol 7: Magnetic Induction
2 2  2π 
 
µ0    µ I µ I µ I
3
= − × k̂ B = 0 ˆi – 0 ˆi – 0 kˆ
8 2 4 2R 4 πR 4 πR
3 π
µ0    
 4   2  k̂ µ0 I µ0 I ˆ
Magnetic field due to i2 = B2 = = [2π − 1] ˆi – k
2 2 2π 4 πR 4 πR

3µ0 1 µ0 I
= × k̂ =  4 π2 + 1 − 4 π + 1 
4 πR  
8 2 4

µ0 I
Magnetic field due to wire in x-direction = B3 =
4 πR
(
2 2π2 − 2π + 1 )
µ0 × 1
B3 = (sin(–45º ) + sin90º ) kˆ
4π × 1 Sol 8: We will find magnetic field B by ampere’s law.
 
µ0  1 ˆ �∫ B.dl = µ0 IIN
B3 = 1 − k
4π  2
r
1 1m
1 2
1 (a)For r1< R
1amp
Magnetic field due to wire in negative y-direction =By B×2 π r1 = µ0 ( ∫ JdA )
 µ ×1   r1 
By = –  0 (sin(–45º ) + sin90º )  kˆ =μ0  br2πr dr 
 4π × 1   ∫0 
 
µ0  1 ˆ 2πbr13
= − 1 − k B×2 π r1 = µ0
4π  2 3
Net magnetic field = B = B1 + B2 + B3 + B4= 0 µ0br12
B=
3
2 1 . 7 8 | Moving Charges and Magnetism

0 Sol 11:
y
(b)

By ampere’s law
 r0 
B×2 π r2 = µ0 ∫ (JdA) = µ0  ∫ br2πr dr  x
0  q, m V
 
r03
B×2 π r2 = µ0 2πb
3
y coordinate is equal to twice the radius of the circle
µ0br03
B= y = 2R
3r2
mV0 2mV0
R= ⇒y=
qB qB
Sol 9:

i Sol 12: We know that velocity of charged particle = v


E
a =
B
mv
Force = Change in momentum per sec=
t
e mE I
I= ⇒F=
µ I t B·e
Magnetic Force = qVB = qV  0 
 2πa 
Sol 13: Force acting on a wire carrying current
 
Sol 10: F = I∫ d  × B

y Since B is uniform so
E= 5× 10
7  
F = I ( ∫ d ) × B

For a loop ∫ d  = 0
x
So F = 0
6
V= 5× 10 m/s z
Sol 14:
Magnetic force = qVB
Electric force = qE


j

When both forces are equal in magnitude and opposite xB
in direction then net force on charged particle is zero.
qVB = qE


R R i
I
E 5 × 107
B= = = 10 T


k
V 5 × 106 


 = 2Ri
and direction is in positive k̂ direction
 
Force = I ∫ d  × B

Since B is constant so
 
F = I ( ∫ d ) × B
 
F = I ·  ×B
P hysi cs | 21.79

Consider a loop PQRS placed in uniform magnetic


F= I ( 2Riˆ × ( −Bk)
ˆ
) field B in such a way that the normal to coil subtends
an angle θ to the direction of B when a current I flows
= I 2RBjˆ = 2IRBjˆ
through the loop clockwise.
The sides PQ and RS are perpendicular to the field
Sol 15: F = F1 + F2 + F3 + F4 and equal and opposite forces of magnitude I and B
= i∫(d 1 ×B1)+ i∫ (d  2×B2) + i∫ (d  3×B3) + act upwards and downwards respectively. Equal and
opposite forces act on sides QR and PS towards right
i∫ (d  4×B4) and left of coil.
 a  a
The resultant force is zero but resultant torque is not
=  i ∫ dyjˆ × (αy)( −k)
ˆ  + i(aiˆ × αy( −k))
ˆ + i dyjˆ × (αy)kˆ + I × 0
∫ zero. The forces on sides PQ and RS produce a torque
 
 0  0
due to a single turn which is given by
a2 ˆ αa2 ˆ τ = I2B sin θ
F1 = −i α i + i αa2ˆj + i i = i αa2ˆj
2 2
for small θ , sin θ ≈ θ
τ = I2Bθ  ... (i)
Sol 16: τ = Iα
B
xxxx
 m 2 m 2 
xxxx =  ×2 + × 2α
E î  4 12 4 4 
xxxx  

 1 1 m2  4  m2
a = m2  +  α =  =  ... (ii)
 24 8  8 3 6
2a By (i) and (ii)
2v
m2
I 2 Bθ = α
(a) Work done by Electric Field = Change in Kinetic 6
Energy 6 IB
α = θ
1 1 m
∫F.dx = m(2v)2 − m v 2
2 2 6 IB
ω2 =
3 m
qE × 2a= mv 2
2 m 10−2
Time period = 2π = 2π
3mv 2 6 IB 6 × 2 × 10−1
E=
4qa 1
=
2π = 0.57 sec
3 120
(b) Rate of work done = F.v = qE.v= mv 3
4a
(c) Work done by magnetic field is always zero. Sol 18: Net force acting on the loop = F

Work done by electric field = F.vˆ = qE î . (–2v ˆj )= 0 µ0 I I'c µ0 I I'c µ0 I I'c  1 1 
F= − =  − 
2πa 2πb 2π  a b 
Sol 17:
O This loop will experience attractive forces.

P 

Q B
i

S
O1 R


2 1 . 8 0 | Moving Charges and Magnetism

Sol 19: (i) z


cos θ =
d + z2
2
y
µ0i2 
F=
2π d2 + z 2

Resultant force is downward


i
−2µ0i2  z
Fnet = –2 Fcos θ = ·
x 2π d2 + z 2 d2 + z 2
d
−µ0i2 z
d Fnet =
π (d2 + z 2 )

For small z
Net Force at some point x, y is

µ0 I µ0 I µ0 I −µ0i2 z
Fnet = + + =0 Fnet = = λa
2π(x + d) 2πx 2π(x − d) π d2

1 1 1 µ0i2 
⇒ + + =0 ω =
n+d x x−d λπd2
2x 1 µ0i2 µ0
+ =0 ω 1 i
2
x −d 2 x F= = =
2π 2π λπd 2 2πd λπ

2x2 + x2 − d2
= 0 ⇒ 3x2 = d2
x(x2 − d2 ) Sol 20: l cos θ = h

d

x= ±
3
Net force will be zero only in x–y plane
y
d θ
i.e. when z = 0 and x = ±
3  h
(ii) z

⊗ z
h tan 
⊗ ⊗ x Take a ring at distance y from the top point of the cone.
Let the middle wire is displaced by z distance in positive Magnetic moment M = IA
z-direction. dM=
Attractive force acting on wire is F  
 Q 2πydy tan θ  ω
 ·  · π (y tan θ)2
 π(htan θ) h cos θ  2π
F F  
 cos θ 
θ z h
Qω tan3 θ Qω tan2 θ h4
= ∫ · y 3dy = ·
0 h2 tan θ h2 4
d
P hysi cs | 21.81

1 Sol 22: (a)


= Q ω tan2 θ h2 v
4 R

Sol 21: (i)BC= BA = BB = BD = B 60º 60º

µ0 I × 2 R
B=
2π 2 a 2
y
BD + BB
1 3 ˆ
⊙ 45º ⊗ 
v = v  ˆi + j
BC + BA 2 2 
x 
  2π  
⊙ ⊗ µ I  µ I  3 3  
B =  0  3 − 0  +   k̂
 2R  2π  4 πR  2 2  
   
   2 
Net magnetic field is µ I 3µ0 I  µ I 1 3
µ I ×2 B=  0 −  k̂ = 0  −  k̂
Bnet = B 2 = 0 along y-axis  6R 2πR  2R  3 π 
2π a  

qv  ˆ µ I1 3  ˆ
(ii)
F2 Force = qv×B =
2 
( )
 i + 3 ˆj × 0  − k
2R  3 π  

 
D A
F1 Qv µ0 I  3 3 
=  − 1
F1 m 6a  π 
 
(b)Net Torque = M × B = I AB ˆj
C B
µ0 I2  π 3a2  ˆ
F1 = = I  a2 − B j
2π(2a) 3 4 

µ0 I2  π 3 2 ˆ
F2 = = B I − a j
3 4 
2π(2 2a)  

F2 µ0 I2 λ  1 3 µ0 I2 λ
F x = F1 + = 1 +  = 8πa Sol 23: (a) Net Torque on the loop is
2 4 πa  2
τ = – MBx ˆj + MBy î = Ιπr 2 B2x + B2y  ...(i)
2 2
F2 µ I λ 1 µ0 I λ
F y = F1 – = 0 1 −  = 8πa
2 4 πa  2 By Torque balance mgr = τ  ...(ii)
By (i) and (ii)
µ0 I2  2
Net force = 1+3 ;  =1 mg
8πa I=
πr B2x + B2y
 µ0   I2 
=
 4 π   2a  10 (b) Net Torque is τ = –MBx ˆj
  
Radius = R = a | τ | = Iπr 2Bx
By torque balance
mgr = t ⇒ mgr = Iπr2 Bx
mg
I=
πrBx
2 1 . 8 2 | Moving Charges and Magnetism

Sol 24: Magnetic field due to sheet of width d and Sol 26:
infinite length at a distance h is given by S
d  0.1m
µ j
B = 0 0 tan–1  2  ˆi
π  h 
  v
  
 = ˆj 2
 
F = i  ×B
G v
iµ 0 j 0  d ˆ
F= tan−1   (–k)
π  2n 
v 3

Sol 25: 2
R
Electron will move in helical path with pitch = 0.1 m. For
minimum value of B particle should reach at point S in
a single revolution.
I
2πm
Time period T =
qB
v
So 0.1 = T
2
 
Force= I  × B = 10 × 0.5 × 0.1 v ·2πm
0.1 =
1 2·qB
Force = N upward on inclined plane
2
20πmv
B’ =
2q

20π 2 × 9.1 × 10−31 × 2000 × 1.6 × 10−19


3 3 =
 2 × 1.6 × 10−19

 
2 2
1
mg 
2 2 × 9.1 × 10−31 × 2000
1 = 10π
2 mg  N 1.6 × 10−19
2
 mgcos  B = 10π 2.275 × 10−4

B = 4.7 × 10–3 T
F
Sol 27: To neutralize the magnetic field, current in
3 vertical ring should be such that the magnitude of
3 3 magnetic field is 3.49×10–5 T and current in horizontal
4
4 ring should be such that the magnitude of magnetic
3 3 3 3 1
µ .1 + F = ⇒F= – field is × 3.49 × 10−5
4 4 40 3

3  3 For vertical ring


Fmin = 1 −  = 0.62 N
4  10 
µ0NI µ0 × 100 × I
B= =
3 3 3 2r 2 × 0.2
F= +
4 40
3.49×10–5 = µ0 × 250 I
Fmax = 0.88 N
P hysi cs | 21.83

3.49 × 10−5 3.49 × 10−5 Sol 29: (a) |B1| = |B2| = |B3| = |B4|
I= = = 0.111 A
µ0 × 250 4 π × 10−7 × 250

For horizontal ring x


2
µ0NI 1 −5 µ0 × 100 I
B= ⇒ × 3.49 × 10 =
2r 3 2 × 0.3 i
1 3 a
⇒ I = 0.096 A
4
Sol 28:
2
i1 a
x 
2
y
4
z y R
dB
 B1  B3
dy B13
 P
30º R y
x
60º (a, 0 , 0) ⊙  x
dB (a, 0 , 0) 
Qa
2
Q
y
P 2
a
x 
2
dB 2
 a z
dy 2 y
2 a
x 
2
 4
R y R
x
0) ⊙  x a x
dB (a, 0 , 0)
2 2
a
x 
2
2
Q
Force on dy element in x direction is  
 
= ∫ i2dy B sinθ µ0 I a a
∫ dF B1 = 

+ 

a2  a2 a2 
4 π x2 + 2 x2 + 2 x2 +
Rdθ µ0i1 4  2 2 
F = ∫ i2 · sin θ
cos θ 2πR
µ0 I a
B1 = ·
30 2
µ0i1i2 a a2
= ∫ tan θ dθ 4 π x2 + x2 +
2π −60
4 2

µi1i2 Resultant of B1 and B3 is B13 = 2B1cos θ


F= [log cos]30
−60

a
µi1i2 µi1i2 2 × µ0 I a 2
= = log 3 log3 ·
2π 4π B13 =
a2 2 a2
2 a2
4π x + x + x2 +
4 2 4
2 1 . 8 4 | Moving Charges and Magnetism

µ0 Ia2 µ0 I2 α
µ0 I2 α
= = ∫ cot θ dθ = ln(sin θ)90
1 π π
π
 a2   a2  2 2
4 π  x2 +   x2 + 
 4   2 
 µ0 I2  
a
= ln  − 1
µ0 I a2 2π  L2 + a2 

B13 =
1
 a2  2 If direction of current in B is reversed then resultant
π(4x2 + a2 )  x2 + 
 2  magnetic field will become horizontal and so net force

will be zero.
µ0 Ia2
Similarly B24 =
1
 a2  2
π(4x + a )  x2 + 
2 2

 2  Exercise 2
Net resultant = B13 + B24 Single Correct Choice Type

2µ0 Ia2 Sol 1: (C) Magnetic field at some x is given by


=
1
 a2  2
π(4x2 + a2 )  x2 + 
 2 
 4I
(b) Yes

Sol 30: Bres = 2Bcos θ (–d, 0, 0) (d, 0, 0)



Fres = I ∫ d  × Bres
I

R dθ 2 µ 0 I a µ0 (4 I) ˆ µ0 I ˆ
= ∫ I × sin θ cos θ ; sin α = k+ k
2πR L2 + a2 2π(d − x) 2π(d + x)

µ0 I  4 1  µ0 I  5d + 3x 
=  +  =  
2π  d − x d + x  2π  d2 − x2 

It corresponds to graph (c)

x Sol 2: (A) Magnetic field at the centre due to Rdθ


component is
B
 
R dx B 
 By 
⊗  d
R B Bx Rd
L
  I 
µ0  ·Rdθ 
a
Bx = ∫ dBx = ∫ 
2πR  cos θ = µ0 I
2πR 4 π2R

 I 
µ0  ·Rdθ 
By = ∫ dB y = ∫ 
2πR  sin θ = µ0 I
2πR 4 π2R
P hysi cs | 21.85

µ0 I Distance from the point of projection = tv cosθ


B= 1+1
2
4π R 4 πM 4 πMv cos θ
= v cosθ =
QB QB
E
Sol 3: (C) V = for no deflection to occur
B
3.2 × 105 Sol 7: (C)
V= = 1.6 × 108 m/s
2 × 10 −3

mv 9.1 × 10−31 × 1.6 × 108


R= = = 0.45 m 
qB 1.6 × 10−19 × 2 × 10−3
V 
  2
q
Sol 4: (C) = α V 
m
1 
Work done by electric field = qE0x0= m(25–0)
2 R
25m 25
x0 = =
2qE0 2αE0  π + 2θ   π + 2θ 
Time taken =  =  T
 ω   2π 
Sol 5: (C) Particle is moving in helix along y-axis. So
the time taken by particle to reach in x-z plane should Sol 8: (D)
be integral multiple of time taken to complete one
revolution.
V
y



  2
x 


V
z  π − 2θ   π − 2θ 
Time taken =   =  T
Helical motion of the particle  ω   2π 
2mv  2πm 
⇒ =  n
qE  qB  Sol 9: (B)

Bv
n= (2, 2)
πE

 Bv 
So   should be an integer 
 πE  

Sol 6: (D) Both particles will move in helix. They will


meet for the first time when mass m will complete two
(2, –2)
revolutions and mass 2m will complete one revolution.
Time taken to complete one rotation. Magnetic force is given by
2 × 2πM 2π2M  
=t1 = ; t2 dFm = i ∫d  × B = i∫d (– ˆj ) × (–4 k̂ ) = 4i∫d  î
QB QB
2 1 . 8 6 | Moving Charges and Magnetism

since  and B are perpendicular so τ = MBsin θ

dfm = 8∫dl î = 8 × 4 î = 32 î τ = I × π R2 × B = I α

MR 2 α
Sol 10: (A) I π R2 × B =
2
y 2 × 4 × π × 10
α =
2
= 40 π rad/sec2
d
I
Sol 13: (A) Let us assume that resistance of p material

x is ρ and that of Q is q.
(–R,0,0) (R,0,0)
2ρ + q 2q + ρ
i1 = i , i2 = i
3(ρ + q) 3(ρ + q)

i1 2ρ + q
z =
i2 2q + ρ
B0
F = I∫Rdθ(sin θ î + cos θ ˆj ) × (–R cos θ ) k̂ I Q Y
2R
IB0R π Q
=
2 ∫ (sin θ ˆi + cos θ ˆj) × (− cos θ k)
ˆ dθ P
0
π i1
= ∫ (sin θ cos θ ˆj – cos2 θ ˆi) dθ P Q
0 i
X P i2 II
sin2θ ˆj (1 + cos2θ)iˆ 
π
= ∫ –  dθ
 2 2 We know that B ∝ i
0 
π π π SoB1 = magnetic field due to I part
 − cos2θ  ˆ  θ  ˆ  sin2θ  ˆ
=   j −   i −  i B2 = magnetic field due to II part
 4 0  2 0  4 0
For the magnetic field to be zero B1 = –B2 should hold.
π π
=0– î − 0 = – î Bi i1 2ρ + q
2 2 But ∝ = ≠ –1
Bi i2 2q + ρ

Sol 11: (A) Refer Q.18 Exercise-I JEE Advanced. So magnetic field will not be zero at centre.In (B), (C)
and (D) i1 = i2 so magnetic field is zero at centre.
Sol 12: (A) Torque on the ring due to magnetic field is
Sol 14: (C)
i I i
i1

B
i2
II

In (A)
3 i
i1 = i ; i2 =
4 4
P hysi cs | 21.87

−µ0i1 3µ0i2 Sol 15: (A) So its x coordinate cannot be positive.


B= (sin θ1 + sin θ2 ) + (sin θ1 + sin θ2 )
l 
4π 4π z B
2 2
v sin 
(B) V
i
i1 
y
v cos 
B
x
i2
i
z

By symmetry i1 = i2and magnetic field will be cancelled


out by both the parts.
(C) i1 y

45º 45º
x
L i2 Its x- and z- coordinate will be zero when particle will
complete one revolution.
y - Coordinate = vcos α t

3 i
i1 = i ; i2 = Multiple Correct Choice Type
4 4

Let magnetic field due to sides of square be Bs Sol 16: (B, C) (A)Motion is helical in nature
3 i
−µ0 i1   3µ0  (B) They will follow circular path with radius
Bs = 4 1 1 4 1 + 1 
 +  k̂ +   2mKE
L  2 2 L
4π 4π  2 2 R=
2 2 qB
Bs = 0 (C) Work done by magnetic force is always zero.

But magnetic field due to 2 infinitely long wires is not


(D)
zero so net magnetic field is zero.
i B
(D) x x x x x x x x x
i1 x x x x x x x x x
x x x x x x x x x
V
x x x x x x x x x
i
x x x x x x x x x
V
i i2 x x x x x x x x x
i x x x x x x x x x
By symmetry i1 = i2 =
2
So magnetic field due to four sides of square will cancel
out. Magnetic field due to two infinitely long wires will
also cancel out as they are equal in magnitude and
opposite in direction.
µ0 I
So net magnetic field is zero. Sol 17: (A, B, C) B = (sin θ1 + sin θ2 )
2πr
2 1 . 8 8 | Moving Charges and Magnetism

Sol 20: (A, B, C, D) On x-axis

i µ 0I µ 0I
(A) B = – =0
2πa 2πa

(B) On y-axis say at (y, 0, 0)

−µ 0 I µ 0I
B= kˆ + kˆ
2π(a + y) 2π(a − y)

So except at origin, B has only z-components

By ampere’s law magnetic field on a ring with centre as z


wire is same. (C) B1
1
B ∝
/ as θ1 and θ2 are also dependent on r. Bnet
r

B2
Sol 18: (B, D) Magnetic field 1
y y
O
2
A(0,1,0) x

(0,1,1)B D(1,1,1)
x (D) B cannot has x-component as B is perpendicular to
direction of I.
C(1,0,1)
z
Sol 21: (A, B) This can be done by applying magnetic
field in y-axis or z-axis.
µ0 I µ0 I
at A = BA = ;BB = y y
2π × 1 2π 2

µ0 I µ0 I
BC = ;BD =
2π × 1 2π × 2
x
x
1
Sol 19: (A, B, C) = c2
µ0 ε0 z z

Sodimension of y is m/s
Sol 22: (A, D)
V
E
v = when E and B are both perpendicular and
B
perpendicular to velocity 30º
B
So dimension of x m/s V
Dimension of RC = sec 60º

So Z = has dimension m/s
CR
2πm T1
So x, y, z have same dimensions. Time period T = ; a= =1
qB T2
P hysi cs | 21.89

mv sin θ Consider a point P in space between two wires at a


radiiR =
qB distance r from one wire. The magnetic force due to
wire 1 is in positive z-axis direction whereas due to wire
R1 sin(30º ) 1 2 is in negative z-axis direction.
= =
R2 sin(60º ) 3
Sol 28: (D) Statement 1 is false as Ampere’s circuital
pitch = v cos α t law holds good for a closed path of any size and shape
around a current carrying conductor only if the relation
P1 v cos(30º )
= = 3 is independent of distance.
P2 v cos(60º )

abc = 1; a = bc Sol 29: (D) Since angular acceleration of the mass will
not change so time period will also remain the same.

Sol 23: (C, D) If velocity is zero, then magnetic force is


zero.Energy cannot increase in magnetic field as work 
done by magnetic force is zero.
 
F = q v × B ;So force is perpendicular to its velocity. F =q

Assertion Reasoning Type



mgsin  mgcos 
Sol 24: (D) If initially velocity of charged particle is in mg
the direction of magnetic field then force acting on it
is zero and particle will continue to move in the same
direction. So statement 1 is false. Comprehension Type

Paragraph 1
Sol 25: (B) Magnetic field at any point is in tangential
direction. So it is not possible for a particle to move in Sol 30: (A) Magnetic field due to curved part is
tangential direction by the action of magnetic force.
µ 0 I  2π  µ 0 I
B=  =
4 πa  3  6a

B Sol 31: (A)

60º
Sol 26: (D) It’s velocity vector must be perpendicular to
both magnetic field and electric field.
60º

 
Sol 27: (C) F = I ∫ d  × B
So force acting is attractive

µ0 I 3µ0 I
B= (sin60º + sin60º ) =
1 2 a 2πa

i1 i2 2

d Sol 32: (D) Net magnetic field at C is


•P
r µ0 I 3µ0 I
B=– +
6a 2πa
2 1 . 9 0 | Moving Charges and Magnetism

Paragraph 1 3Kq 3Kq


Electric potential = − =0
r r
Sol 33: (D) I = 3A Magnetic field is zero as current due to rotating charge
r = 0.04m is zero.
+ –
N = 20
F
B = 0.5 T
r
Dipole moment M = INA= 3× 20 × π (0.01)2 –
F
+
M

= 1.88 × 10 Am
–2 2

1 F
Sol 34: (B) PE = –1.88 ×10–2 × + –
2
= –9.4mJ Magnetic moment= INA= 0 × NA = 0

Sol 35: (B) Torque, τ =in AB sin 90o E=



Kq

Kq
+
Kq
+
Kq

Kq

Kq
≠0
2 2 2 2 2 2
2  5a   5a   3a   3a  a a
 1             
= 3 × 20 × π ×   × 0.5 × 1 2 2  2  2 2 2
 100 
P
3 × 3.14 × 10−3 Nm
= = 9.4 × 10 −3 Nm

– + – + – +
Match the Columns
a M

Sol 36: A → q, r; B → p; C → q, r; D → q, or; A → q, r;


B → p; C → q, r; D → q, s Q
τ = MB sin 90º= 9.4 ×10–3 Nm Kq Kq Kq Kq Kq Kq
V= − + − + − =0
5a 5a 3a 3a a a
(A) Magnetic field is in opposite direction. Since current
2 2 2 2 2 2
is in same direction so they will attract each other.
Magnetic field is equal in magnitude at P so magnetic B = 0 as current due to rotating charge is zero.
field at P is zero.
(B) µ = 0 as current due to rotating charge is zero.

Q
B B +• •+


P
F a
b M
Magnetic field at P is in the same direction.
•–
–•
Wires will attract as the current is in the same direction.
(C) Magnetic field at P is in opposite direction due to

+
two wires and has same magnitude. So net magnetic P
field is zero at P. Wires will attract each other as current E=0
is in the same direction.
Electric field will cancel out due to symmetry
(D) Magnetic field will be in opposite direction and wires
Kq Kq
will repel each other as current is in opposite sense. V=– ×3+ ×3≠0
a b

Sol 37: A → p, r, s; B → r, s; C → p, q; D → r, s B is not zero as current due to rotating charge is non-


zero.
Electric field is zero at point M
P hysi cs | 21.91

µ = INA →
Sol 4: (A, B, D) If both E and B are zero, then Fe and
as I ≠ 0 ⇒ µ ≠ 0 →
Fm both are zero. Hence, velocity may remain constant.
– + – Therefore, option (a) is correct.
If E = 0, B ≠ 0 but velocity is parallel or antiparallel
→ →
to magnetic field, then also Fe and Fm both are zero.
P Q Hence, option (b) is also correct.
→ →
If E ≠ 0, B ≠ 0 but Fe + Fm = 0, then also velocity may
– –
+ remain constant or option (d) is also correct.
Electric field is zero.By symmetry electric field will
cancel out each other. Sol 5: (A, B, D) Magnetic force does not do work. From
−Kq Kq work-energy theorem:
×4+ ×2 ≠ 0
V =  5a a  1
  WFe = ∆KE or (qE)(2a) = m[4v2 – v2]
 2  2 2
 
3  mv 2 
Let I be the current due to moving charge or E =  
4  qa 
2xµ0 Ia2  
µ0 I ≠0
So B = – 3 At P, rate of work done by electric field
2a 2 ( )
2a2 2 → →

µ = INA = F e . v = (qE)(v) cos 0°


µ = 2× Ia2 – Ia2= Ia2  3 mv 2  3  mv 3 
= q  v=  
 4 qa  4  a 
 

Previous Years’ Questions Therefore, option (b) is also correct. Rate of work done
→ →
at Q: of electric field = F e . v = (qE)(2v)cos 90° = 0 and
of magnetic field is always zero. Therefore, option (d) is
Sol 1: (C) cφ = BINA
also correct.
 BNA  →
∴ φ=  I Note that F e = qE î
 c 

mv P 2km
Sol 2: (C) If B2> B1, critical temperature, (at which Sol 6: (A, C) r = = =
resistance of semiconductors abruptly becomes zero) Bq Bq Bq
in case 2 will be less than compared to case1. m
i.e., r ∝
Using iron core, value of magnetic field increases. So, q
deflection increases for same current. Hence, sensitivity
If K and B are same.
increases.
Soft iron can be easily magnetized or demagnetized. 1 4 16
i.e., r :r :r = : : =1:2:3
H+ He+ O2 + 1 1 2
Sol 3: (D) With increase in temperature, TC is decreasing. Therefore, He+ and O2+ will be deflected equally but H+
having the least radius will be deflected most.
TC(0) = 100 K
TC = 75 K at B = 7.5 T
Hence, at B = 5 T, TC should lie between 75 K and 100 K.
Hence, the correct option should be (b).
2 1 . 9 2 | Moving Charges and Magnetism


Sol 7: (A, C) F BA = 0, because magnetic lines are
X X X
parallel to this wire.
→ X X X
F CD = 0, because magnetic lines are antiparallel to this
wire. ×
→ → –
F CB is perpendicular to paper outwards and F AD is
perpendicular to paper inwards. These two forces
(although calculated by integration)cancel each other
but produce a torque which tend to rotate the loop in
clockwise direction about an axis OO’. Te Tp
∴ Te< Tp, te = and tp =
2 2
BqI
Sol 8: (A, C, D) v =
m or te< tp
→ →
v ⊥ B in region II. Therefore, path of particle is circle
in region II. Sol 10: (C, D) Y
E0, B0
v

X X X

X X
X X
If θ =0 or 10o
X X then particle moves in helical path with increasing pitch
along Y-axis.
If θ =90o then magnetic force on the particle is
Particle enters in region III if, radius of circular path, r >l zero and particle moves along Y-axis with constant
acceleration.
mv
or >l
Bq
Sol 11: (5)
BqI
or v>
m a
a/2
BqI mv P O
If v = ,r= = I, particle will turn back and path
m Bq
length will be maximum. If particle returns to region I,

time spent in region II will be:


I = J × πa2
T πm
t= = , which is independent of v. µ0 J × πa2 µ0 J × π a2
2 Bq B= − ×
2πa 3a 4
2π ×
mv 2
Sol 9: (B, D) r = or r ∝ m
Bq
1 1 
∴ re< rp as me< mp ⇒B=
µ0 Ja  − 
 2 12 

2πm 5
Further, T = or T ∝ m ⇒B=
µ0 Ja ×
Bq 12
P hysi cs | 21.93

Sol 12: (B) M = I × Area of loop k̂ Sol 16: (3) Case – I

 πa2  π  Case-I Case-II


=I × a2 × × 4  kˆ =I × a2  + 1 kˆ
 4×2  2 

Sol 13: (D) I I I I


X0 X0
R/2
R
r< ; B=
0
2 P P
X0/3 X0/3
R
B at r = 1  µ0   3I 
2 B1 =   
2  2π 
 x0 
µ0 JR µ0 JR
⇒B= − =0 mv
2×2 2×2 R1 =
qB1
R
B at r > Case - II
2
Case-I Case-II
µ JR µ J × π R 2
⇒B=0 − 0 ×
2 2πr 4

µ0L  R 2 
=B r −  I I I I
2  4r  X0 X0

R
If we put r = ,B=0
2 P P
X0/3 X0/3
∴ B is continuous at r = R/2
mv
R1 =
Sol 14: (A, C) So magnetic field is along –ve, z-direction. qB2
πM
Time taken in the magnetic field =10 × 10−1 = R1 B2 1/3
6QB ⇒ = = =3
R2 B1 1/9
πM 1000πM 50πM
=B = =
−3
60 × 10 Q 60Q 3Q Sol 17: (C) The net magnetic field at the given point
will be zero if.
y  


 2 ( 3i + j ( | Bwires |=| Bloop |


o
30
o 30
µ0 I a µ0 Ia2
⇒ 2 × =
2 2 3/2
2π a2 + h2 a2 + h2 2(a + h )

x ⇒ h ≈ 1.2a


4i

Sol 15: (B) The direction of magnetic field at the given point due
to the loop is normally out of the plane. Therefore, the
−2GMm 1 GM net magnetic field due the both wires should be into
+ mv 2 =
0⇒v=
2 the plane. For this current in wire I should be along PQ
L 2 L
and that in wire RS should be along SR.
Note: The energy of mass ‘m’ means its kinetic energy
(KE) only and not the potential energy of interaction Sol 18: (B)
between m and the two bodies (of mass M each) – µ0 I µ0 I2a2
τ = MB sin θ = Iπa2 × 2 × sin 30o =
which is the potential energy of the system. 2πd 2d
2 1 . 9 4 | Moving Charges and Magnetism


Sol 19: (A, B, C) F = 2I(L + R)[iˆ × B]
ˆ

.
2(L+R)

Sol 20: (A, D) I1 = I2

⇒ neA1 v1 =
neA2 v 2
⇒ d1 w1 v1 =
d2 w2 v 2

Now, potential difference developed across MK


V = Bvw
V1 v1 w1 d2
⇒ = =
V2 v 2 w2 d1

Sol 21: (A, C) As I1 = I2

n1 w1d1 v1 = n2 w2d2 v 2

V2 B2 v 2 w2  B2 w2  n1 w1d1  B2n1
Now
= =   =
 
V1 B2 v1 w1  B1 w1  n2 w2d2  B1n2
2017-18 100 &
op kers
Class 12 T
By E ran culty
-JE Fa r
IIT enior emie .
S fP r es
o titut
Ins

PHYSICS
FOR JEE MAIN & ADVANCED
SECOND
EDITION

Exhaustive Theory
(Now Revised)

Formula Sheet
9000+ Problems
based on latest JEE pattern

2500 + 1000 (New) Problems


of previous 35 years of
AIEEE (JEE Main) and IIT-JEE (JEE Adv)

5000+Illustrations and Solved Examples


Detailed Solutions
of all problems available

Topic Covered Plancess Concepts


Tips & Tricks, Facts, Notes, Misconceptions,
Electromagnetic Induction Key Take Aways, Problem Solving Tactics
and Waves
PlancEssential
Questions recommended for revision
22. ELECTROMAGNETIC
INDUCTION AND
ELECTROMAGNETIC
WAV E S

1. INTRODUCTION
The phenomenon of electromagnetic induction has acquired prime importance in today’s world in the field of
Electrical and Electronics Engineering. We have studied that a current produces a magnetic field. The phenomenon
of electromagnetic induction is thereverse effect wherein a magnetic field produces a current. Applications of this
phenomenon are found in modern electric power generation and transmission systems and various electronic
devices. This phenomenon enables us to convert the kinetic energy of a coil rotating and/or translating in a
magnetic field into electrical energy. So, by applying this phenomenon, energy stored in various forms like, nuclear,
thermal, wind etc. can be converted into electrical energy. The operating principle of electric motors, generators
and transformers is based on this phenomenon. Other applications include musical instruments, induction stove
used in our kitchen, and induction furnace used in foundries.

2. MAGNETIC FIELD LINES AND MAGNETIC FLUX


Let us first discuss the concept of magnetic field lines and magnetic flux. We can represent any magnetic field
by magnetic field lines. Unlike the electric lines of force it is wrong to call them magnetic lines of force, because
they do not point in the direction of the force on a charge. The force on a moving charged particle is always
perpendicular to the magnetic field (or magnetic field lines) at the particle’s position.
The idea of magnetic field lines is the same as it is for electric field lines. The magnetic field
at any point is tangential to the field line at that point. Where the field lines are close, the B
magnitude of field is large, where
 the field lines are far apart, the field magnitude is small.
Also, because the direction B at each point is unique, field lines never intersect. Unlike the

electric field lines, magnetic lines form closed loops. ds
A
SI unit of magnetic field is Tesla (T). 1 T=104 Gauss.
 
()
Magnetic flux φ through an area ds in magnetic field B is defined as φ = B ⋅ ds … (i)
Physically it represents total lines of induction passing through a given
Figure 22.1:
Area, equation (i) can be written as Magnetic flux through
elementary area ds
=φ Bds cos θ  … (ii)
Where θ is angle between B and area vector ds. (see Figure 22.1) According to equation
(ii) flux can change not only due to magnetic field and area but also due to orientation of area w.r.t. B.
2 2 . 2 | Electromagnetic Induction and Electromagnetic Waves

Dimensional formula of flux is ML2 T −2 A −1 


 
Note down the following points regarding the magnetic flux:
(a) Magnetic flux is a scalar quantity (dot product of two vector quantities is scalar quantity)
(b) The SI unit of magnetic flux is tesla-meter2 (1T-m2). This unit is called weber (1 Wb)
1Wb=1Tm2=Nm/A
Thus, unit of magnetic field is also weber/m2(1Wb/m2), or 1 T=1Wb/m2

(c) In the special case in which B is uniform over a plane surface with total area S, than=
φB BA cos θ (see Figure
22.2)

 
B B


S

R = BS cos  R = BS

Figure 22.2: Determination of flux for relative orientation of B and S



If B is perpendicular to the surface, then cos θ =1 and fB=BS

Illustration 1: At certain location in the northern hemisphere, the earth’s magnetic field has a magnitude of 42 µT
and points downward at 570 to vertical. The flux through a horizontal surface of area 2.5m2 will be (cos 570 =0.545)
 (JEE MAIN)
  
Sol: The magnetic flux through any surface is φφ==BB•.•AA
Using the formula of flux
= φ BA cos θ
we get the flux through the area as φ= BA cos 570= 42 × 10−6 × 2.5 × 0.545= 57 ×10−6 Wb .

3. ELECTROMAGNETIC INDUCTION
If a magnet is brought to a coil which is connected with a galvanometer, an electric
current is produced in the circuit (See Figure 22.3). The direction of the current so G
induced in the circuit, is reversed when the magnet recedes away from the coil.
The current so produced lasts long, as there is relative motion between the magnet
and the coil. S N

It is shown that whenever the magnetic flux linked with a closed circuit changes, Figure 22.3: Induced current in
an induced e.m.f. is produced in the circuit and lasts as long as the flux changes. coil due to relative movement
Such currents are produced due to induced electromotive force and the of magnet
phenomenon is called electromagnetic induction. The magnitude and direction
of induced electromagnetic force is given by the following Faraday’s and Lenz’s
laws respectively.

3.1 Faraday’s First Law


Whenever the magnetic flux linked with a closed circuit changes, an induced electromotive force is produced which
produces an induced current in the circuit which lasts as long as the change lasts.
P hysi cs | 22.3

3.2 Faraday’s Second Law


−dφ
The induced e.m.f. is equal to negative of rate of change of flux through the circuit. e =
dt
The negative sign shows that the induced e.m.f. opposes the changes in the magnetic flux.
Ndφ
If the coil has N number of turns, then e = − .
dt

4. LENZ’S LAW
The direction of induced electromotive force is such that it opposes the cause that produces the electromagnetic
induction.
N ( φ2 − φ1 )
If the magnetic flux changes from φ1 to φ2 in time t, the average induced e.m.f. is given by e ( avg) = −
t
When the magnetic flux φ through a closed circuit of known resistance R changes, the quantity of induced charge
q can be found as below:
 ∆φ  e N  ∆φ  N  ∆φ  N∆φ Total change of flux
As e =−N  , i = =   ; q = i∆t =   ∆t = =
 ∆t  R R  ∆t  R  ∆t  R Resis tance

Furthermore, the direction of induced e.m.f. is that of the induced current. Lenz’s law follows from the law of
conservation of energy.

4.1 Fleming’s Right Hand Rule


It states that if the thumb and the first two fingers of the right hand are
C F1 F
stretched mutually perpendicular to each other and if the forefinger gives
the direction of the magnetic field and the thumb gives the direction of
l
motion of the conductor, then the central finger gives the direction of the v
induced current. F3 D E
The current in the above mentioned loop is in anticlockwise direction.
F2
If the loop CDEF (See Figure 22.4) is moved towards right with velocity
v, the induced current I will be flowing in clock wise direction and this Figure 22.4: Loop moving in
current will produce forces F1 and F2 on arms CF and DE respectively, magnetic field
which being equal and opposite will cancel. Force F3 on arm CD= BI l
where CD=l
 Blv  B2l2 v

= F3 Bl  =  where R is the resistance of closed loop.
 R  R
B2l2 v 2
Power to pull the loop = F3 v =
R
B2l2 v 2
This work is completely converted to heat due to current flowing in the heat produced in the loop = I2R = .
R
Problem Solving Tactic
Never try to use Fleming right hand rule while actually solving a problem. Instead always try to imagine situation
and apply Lenz’s law, which is very fundamental and easy to understand.
Illustration 2: Space is divided by the line AD into two regions. Region I is field free and the region II has a uniform
magnetic field B directed into the plane of paper.
ACD is semicircular conducting loop of radius r with center at O, the plane of the loop being in the plane of the
paper. The loop is now made to rotate with a constant angular velocity w about an axis passing through O and
perpendicular to the plane of the paper.
2 2 . 4 | Electromagnetic Induction and Electromagnetic Waves

The effective resistance of the loop is R. Region I Region II


(a) Obtained an expression for the magnitude of the induced current in the loop.  A
(b) Show the direction of the current when the loop is entering into the region II. 

(c) Plot a graph between the induced e.m.f. and the time of rotation for two periods of C O B
rotation.  (JEE MAIN)

Sol: The current induced in the loop is such that it opposes the change in the magnetic D
flux linked with the loop.
(a) When the loop is in region I, the magnetic flux linked with the loop is zero. When
Figure 22.5
the loop enters in magnetic field in region II.
The magnetic flux linked with it, is given by φ = BA
dφ d (BA ) dA
∴ e.m.f. induced is E =
− =− = −B (Numerically)  Region I Region II
dt dt dt
Let dθ be the angle by which the loop is rotated in time dt, then from Figure 22.6 E rd
dA= Area of the triangle OEA=(1/2)r•r d θ 
A
1 r × r dθ 1 2 dθ 1 2 e.m.f 1 Br 2 ω
E B .
∴= = Br = Br ω Using Ohm’s law, induct current I= = C
O B
2 dt 2 dt 2 R 2 R
r
Note: dA can also be calculated in the following way; The area corresponding 2π
(angle) is πr 2 . D

∴ Area corresponding to unit angle dθ =


πr 2
2π Figure 22.6
πr 2 πr 2 1 2
Area corresponding to angle dθ = ×dθ ; ∴ dA= × d=
θ r dθ
2π 2π 2

(b) According to Lenz’s law, the direction of current induced is to 1 2


Br 
oppose the change in magnetic flux. So that magnetic field 2
induced must be upward. In this way, the direction of current e
must be anticlockwise. t
O
(c) The graph is shown in Figure 22.7. When the loop enters  2 3 4
the magnetic field, the magnetic flux linked increases and    
e.m.f. e=(1/2) Br 2 ω is produced in one direction. When the loop 1 2
- Br 
2
comes out of the field, the flux decreases and e.m.f. is induced
in the opposite sense.
Figure 22.7

Illustration 3: Figure 22.8 shows a conducting loop placed near a long, straight wire carrying a current i as shown.
If the current increases continuously, find the direction of the induced current in the loop. (JEE MAIN)

Sol: According to Lenz’s law, the direction of the induced current is such that it
opposes the cause.
Let us put an arrow on the loop as in the Figure 22.8. The right-hand thumb rule i
shows that the positive normal to the loop is going into the plane of the diagram.
Also, the same rule shows that the magnetic field at the site of the loop
 due to the
current is also going into the plane of the diagram. Thus, B and ds are along the
  Figure 22.8
same direction everywhere so that the flux Φ ∫
Φ = B.ds is positive. If i increases, the

magnitude of Φ increases. Since Φ is positive and its magnitude increases, is positive. Thus, E is negative and
dt
hence, the current is negative. The current is, therefore, induced in the direction opposite to the arrow.
P hysi cs | 22.5

5. THE ORIGIN OF INDUCED E.M.F.


E.M.F. is defined as the external mechanism by which work is done per unit charge to maintain the electric field in
the wire so as to establish electric current in a conducting wire.
 
The flux ∫ B.ds can be changed by
(a) Keeping the magnetic field constant as time passes and moving whole or part of the loop
(b) Keeping the loop at rest changing the magnetic field
(c) Combination of (a) and (b), that is, by moving the loop (partly or wholly) as well as by changing the field.
The mechanism by which e.m.f. is produced is different in the two basic processes (a) and (b). We now study them
under the headings motional e.m.f. and induced electric field.

5.1 Motional E.M.F.


 
The Figure 22.9 below shows a rod PQ of length l moving in a magnetic field B with a constant velocity V . The
length of the rod is perpendicular to the magnetic field and the velocity is perpendicular to both the magnetic field
and the rod.
The magnetic force due to the random velocity is zero on the average. Thus, the magnetic field exerts an average
  
force dF=
b qv × B on each free electron, where q=-16×10-19C is the charge on the electron. This force is towards
QP and hence the free electrons will move towards P. Negative charge is accumulated at P and positive  charge

appears at Q. An electrostatic field E is developed within the wire from Q to P. This field exerts a force dFe = qE on
  
each free electron. The charge keeps on accumulating until a situation comes when Fb=Feor, qv × B =qE or, νB =E
After this, there is no resultant force on the free electrons of the wire PQ. The potential difference between the ends
Q and P is V=El=vBl
Thus, it is the magnetic force on the moving free electrons that maintains the potential difference V=vBl and hence
produces an e.m.f. E=vBl
As this e.m.f. is produced due to the motion of a conductor, it is called
motional e.m.f. Q +
If the ends P and Q are connected by an external resistor Figure 22.10 (a),
I
an electric field is produced in this resistor due to the potential difference. A
current is established in the circuit. The electrons flow P to Q via the external P -
circuit and this tries to neutralize the charges accumulated at P and Q. The
magnetic force qvB on the free electrons in the wire QP, however, drives the
electrons back from Q to P to maintain the potential difference and hence Figure 22.9 Motional emf in a
the current. conducting rod

+ i
Q vBl
I
v R R
r
P - i
i
(b)
x
(a)

Figure 22.10: (a) Current due to motional emf (b) Equivalent circuit showing induced emf and current in the loop
2 2 . 6 | Electromagnetic Induction and Electromagnetic Waves

Thus, we can replace the moving rod QP by battery of e.m.f. ν Bl with the positive terminal at Q and the negative
terminal at P. The resistance r of the rod QP may be treated as the internal resistance of the battery. Figure 22.10
(b) shows the equivalent circuit.
ν Bl
The current is i = in the clockwise direction (induced current).
R +r
We can also find the induced e.m.f. and the induced current in the loop in Fig.22.10 (a) from Faraday’s law of
electromagnetic induction. If x be the length of the circuit in the magnetic field at time t, the magnetic flux through
the area bounded by the loop is Φ =Blx.
dΦ dx
The magnitude of the induced e.m.f. is
= E = Bl = νBl.
dt dt
ν Bl
The current is i = . The direction of the current can be worked out from Lenz’s law.
R +r

Illustration 4:Figure 22.11 (a) shows a rectangular loop MNOP being pulled out of a magnetic field with a uniform
velocityvby applying an external force F. The length MN is equal to l and the total resistance of the loop is R. Find (a)
the current in the loop, (b) the magnetic force on the loop, (c) the external force F needed to maintain the velocity,
 (JEE ADVANCED)

F2
N O N O

v v
I F I F
M P M F2 P

(a) (b)
Figure 22.11

Sol: Due to the motion of the loop inside the magnetic field, the motional e.m.f. is induced in the loop. And the
  
magnetic force acting on the loop is F= I  × B

(a) The e.m.f. induced in the loop is due to the motion of the wire MN. The e.m.f. is E=vBl with the positive end at
E vBl
N and the negative end at M. The current is =i =
R R
inclockwise direction (see Figure 22.11 b).
    vB2l2
(b) The magnetic force on the wire MN is F 1= i l × B . The magnitude is F=
1 ilB
= and is opposite to the velocity
R
on the parts of the wire NO and PM, lying in the field, cancel each other. The resultant magnetic force on the loop
B2l2 ν
is, therefore, F1 = opposite to the velocity.
R
(c) To move the loop at a constant velocity, the resultant force on it should be zero. Thus, one should pull the loop
ν B2l2
with a force F= F1 =
R

5.2 Induced Electric Field



Consider a conducting loop placed at rest in a magnetic field B . Suppose, the field is constant till t=0 and then
changes with time. An induced current starts in the loop at t=0.
The free electrons were at rest till t=0(we are not interested in the random motion of the electrons). The magnetic
field cannot exert force on electrons at rest. Thus, the magnetic force cannot start the induced current. The electron
may be forced to move only by an electric field and hence we conclude that an electric field appears at t=0. This
P hysi cs | 22.7

electric field is produced by the changing magnetic field and not by charged particles according to the Coulomb’s
law or the Gauss’s law. The electric field produced by the changing magnetic field is non-electrostatic and non-
conservative in nature. We cannot define a potential corresponding to this field. We call it induced electric field. The
lines of induced electric field are curves. There are no starting and terminating points of the lines.
 
If E be the induced electric field, the force on a charge q placed in the field is qE . The work done per unit charge
  
as the charge moves through dl is E ⋅ dl .
The E.M.F. developed in the loop is,
 
ε=� ∫ E.dl.
Using Faraday’s Law of Induction,

dΦ   dΦ
ε=
− ∫ E.dl =
or � −
dt dt 
The presence of a conducting loop is not necessary to have an induced electric field. As long as B keeps changing,
the induced electric field is present. If a loop is there, the free electrons start drifting and consequently an induced
current results.
Note: Induced electric field is not similar to electrostatic field. The biggest difference is that electrostatic field is
conservative while the other one is not.

5.3 Induction Due to Motion of a Straight Rod in the Magnetic Field


Consider a straight conducting rod CD moving velocity v towards F C C’
right along a U shaped conductor in a uniform magnetic field B
directed into the page. The motion of the conductor CD resulting in
l
changing the area from CDEF to C’D’EF. It result in a change of area
CDD’C’ in the magnetic flux producing an increase in the magnetic
flux dφ as dφ =B.A
E D D’

If l is the length of rod CD, which moves with velocity ν in time dt, Figure 22.12 Change of flux linkage due to
change in area perpendicular to the field=CDD’C=l vdt. motion of conductor

∴ dφ = Blνdt

e
The magnitude of induced e.m.f.= = Blv
dt
Blν
If R is the resistance of loop, the induced current is I =
R
The direction of the induced current is given by Fleming’s right hand rule.

Illustration 5: In the Figure 22.13,the arm PQ of the rectangular conductor is


K M
moved from x=0, outwards. The uniform magnetic field is perpendicular to
the plane and extend from x=0 to x=b and is zero for x>b. Only the arm PQ S
possesses substantial resistance r. Consider the situation when the arm PQ is P
pulled outwards from x=0 to x=2b, and is then moved back to x=0 with I v
constant speed v. Obtain expressions for the flux, the induced e.m.f., the force Q
necessary to pull the arm and the power dissipated as Joule heat. Sketch the R
variation of these quantities with distance. (JEE ADVANCED)
x=0 x=b x=2b
Sol: In external magnetic field, the magnetic force acting on movable part Figure 22.13
B2  2 v
of coil is F = and power dissipated in the circuit is given by P= I2r.
r
Let us first consider the forward motion from x=0 to x=2b
The flux ΦB linked with the circuit SPQR is=
ΦB Bx 0 ≤ x=
< b Bb 0 ≤ b < 2b
2 2 . 8 | Electromagnetic Induction and Electromagnetic Waves

The induced e.m.f. is,  Outward Intward


dΦB K L M L K
i E =− =−Bv 0 ≤ x < b =0 0 ≤ x < 2b
dt B

Flux
When the induced e.m.f. is non-zero, the current I
B v
is (in magnitude) I = +B
r

EMF
The force required to keep the arm PQ in constant motion is IB.
-B
Its direction is to the left. In magnitude 2 2
2 2
-B  
B  v r
=F 0≤x<b

Force
r
= 0 0 ≤ x < 2b 2 2
-B  
2
The Joule heating loss is PJ = I r r
2 2
-B  
B2  2 v 2 r

Power
= 0≤x<b
r
= 0 0 ≤ x < 2b x= 0 b 2b b 0
One obtains similar expressions for the inward motion from x=2b to x=0. Figure 22.14

5.4 E.M.F. Due to Rotation in Magnetic Field


(a) Rod rotating in a magnetic field: If a linearly conducting rod of length l
moves with a velocity ν perpendicular to a magnetic field B, the induced dx
e.m.f. =Eo=Blv. L
 A
If the rod of length l is rotating in a magnetic field with angular velocity ω,
velocity of different parts is different and increases moving from O → A. Velocity
of element at a distance x from O is ωx.
Induced e.m.f. across element of length dx.
  2 

dE =( dx ) ⋅ ( ωx ) ⋅ B =ωB ⋅ ( x dx⇒
dE )
=ω B ∫ xdx =
ω B  
2 Figure 22.15: Rod rotating in
0  
uniform magnetic field
  2  1
⇒ ∫ dE = ωB ∫ xdx = ωB   ⇒ E =ωB2
2 2
0  
1
⇒ E
Direction= ωB2 is given by right hand thumb rule.
of e.m.f.
2

Illustration 6:A wheel with 10 metallic spokes each 0.5 m long is rotated with a speed of 120 rev/min in a plane
normal to the horizontal component of earth’s magnetic field HE at a place. If HE= 0.4 G at the place, what is the
induced e.m.f. between the axle and the rim of the wheel? Note that 1G=10-4 T. (JEE MAIN)

Sol: E.m.f. induced in a rod of length R rotating about one end in magnetic field is
1
E= ωBR 2
2
120 × π
Frequency of revolution ω=120 rev/s = m / s= 2π m/ s
60
1 1
× 2π × 0.4 × 10−4 × ( 0.5 ) = 6.28 × 10−5 V
2
∴ Induced e.m.f.= ωBR 2=
2 2
The number of spokes is immaterial because thee.m.f.’sacross the spokes are in parallel.
P hysi cs | 22.9

(b) Coil rotating in a magnetic field: If a rectangular conducting coil of area A and N turns is rotated in a
uniform magnetic field B with angular velocity ω, as shown in the Figure 22.16. 
As the coil rotates, an induced e.m.f. E, is produced due to change of flux. At any
instant, area vector of coil makes an angle θ with magnetic field, flux linked with coil
is φ = NBA cos θ where θ = ωt ⇒ φ = NBA cos ωt

−BAN ω sin ωt
=
dt
Using Faraday’s Law B
e = BAN ω sin ωt or =e e0 sin ωt
The induced e.m.f. has a sinusoidal variation with time and has a maximum value of
e0 =NBA ω .
Figure 22.16: Coil rotating
Such a coil converts mechanical energy into electrical energy. It provides the basic in uniform magnetic field
principle onwhich an alternating current (A.C.) generator is based.
(c) Change of area inside magnetic field changes: Let a
x
rectangular coil of width Land length x be placed inside a B
magnetic field flux linked with coil, A
φ BA=
= (BL ⋅ x ) L v
dφ dx
= = (BLv )
BL
dt dt
D C
According to Faraday’s Law e=-LBv or |e|=LvB
The direction of induced e.m.f. is given by Lenz’sLaw.
Figure 22.17: Coil moving in magnetic field
(d) Flux linked with coil also changes when magnetic field
over coil change with time.

Illustration 7:Flux associated with coil of resistance 10 Ω and number of turns 1000 is 5.5 × 10−4. If the flux reduces
to 5.5 × 10−5 wb in 0.1 s. The electromotive force and the current induced in the coil will be respectively. (JEE MAIN)
dφ φ − φ1
is E N= N 2
Sol: The induced e.m.f. in coil = where N is the number of turns in the coil.
dt t2 − t1
−4
Initial magnetic flux φ1= 5.5 × 10 Wb.
Final magnetic flux φ2 = 5 × 10 −5 Wb.
∴ Change in flux ( ) ( )
∆φ = φ2 − φ1 = 5 × 10 −5 − 5.5 × 10 −4 = −50 × 10 −5 Wb
Time interval for this change, ∆t =0.1 sec.

∴ Induced e.m.f. in the coil E =


−N
∆Φ
−1000 ×
=
(
−50 × 10−5
5V
=
)
∆t 0.1
E 5V
Resistance of the coil, R=10 Ω. Hence induced current in the coil is =
i = = 0.5 A
R 10 Ω

6. A NEW LOOK OF ELECTRIC POTENTIAL


Electric potential has meaning only for electric fields that are produced by static charges; it has no meaning for
electric fields that are produced by induction.
You can understand this statement qualitatively by considering what happens to a charged particle that makes
a single journey around the circular path. It starts at a certain point and, on its return to that same point, has
 
�∫ = 0.
E.ds
2 2 . 1 0 | Electromagnetic Induction and Electromagnetic Waves

However, when a changing magnetic flux is present, this integral is not zero but is −dΦB/d.Thus, assigning electric
potential to an induced electric field leads us to a contradiction. We must conclude that electric potential has no
meaning for electric fields associated with induction.

7. EDDY CURRENT
Consider a solid plate of metal which enters a region having a magnetic field (See Figure 22.18a). Consider a loop
drawn on the plate, a part of which is in the field.

B B

(a) (b)
Figure 22.18: Generation of eddy current in conductor

As the plate moves, the magnetic flux through the area bounded by the loop changes and hence, a current is
induced. There may be a number of such loops on the plate and hence currents are induced on the surface along a
variety of paths. Such currents are called eddy currents. The basic idea is that we do not have a definite conducting
loop to guide the induced current. The system itself looks for the loops on the surface along which eddy currents
are induced. Because of the eddy currents in the metal plate, thermal energy is produced in it. This energy comes
at the cost of the kinetic energy of the plate and the plate slows down. This is known as electromagnetic damping.
To reduce electromagnetic damping, one can cut slots in the plate (See Figure 22.18 (b)). This reduces the possible
paths of the eddy current considerably.

8. INDUCTORS
An inductor (symbol ) can be used to produce a desired magnetic field. We shall consider a long solenoid
(more specifically, a short length near the middle of a long solenoid) as our basic type of inductor.
If we establish a current i in the windings (turns) of the solenoid we are taking as our inductor, the current produces
NΦB
a magnetic flux ΦB through the central region of the inductor. The inductance of the inductors is then L =
i
(inductance defined)
In which N is the number of turns.
NΦBis called the magnetic flux linkage.
The inductance L is thus a measure of the flux linkage produced by the inductor per unit of current.
The SI unit of magnetic flux is the tesla-square meter (Tm2), the SI unit of inductance is henry (H)
1 henry= 1H=1T.m2/A.
P hysi cs | 22.11

8.1 Potential Difference acrossan Inductor


We can find the direction of self-induced e.m.f. across an inductor from Lenz’s law.

i (constant) I (increasing) i (decreasing)


a b a b a b
+ - + -
e e
dl
=0 dl 0 dl 0
dt > >
e=0 dt dt
Vab =0 Vab =0 Vab <0
(a) (b) (c)
Figure 22.19: Variation of current in inductor coil

8.2 Self Induction


When the current is increased or reduced in the coil, it results in a change of magnetic flux due to which an e.m.f.
is induced in the coil, andthis is called self-induced e.m.f. due to the phenomenon of self-induction. If a current I is
flowing in a coil, a magnetic flux φ is linked to the coil which is directly proportional to the current ∴ φ ∝ I or φ =LI
Where L is a constant of proportionality and is called self-inductance of the coil or simply inductance of the coil.
dφ dI
∴ E.M.F. induced in the coil, E =
− −L
=
dt dt
The self-inductance of a coil is the e.m.f. induced in it when the rate change of current is unity. The unit of inductance
is Henry (H). One Henry is defined as the inductance of a coil in which an e.m.f. of 1 volt is produced, when the
current in the coil is changing at the rate of one ampere per second (A/s). If a solenoid has n number of turns
per meter and l is its length with total number of turns N=n  and area of cross section A, its inductance L is
µ0N2 A
µ0n2 A =
L=

The SI unit of self-inductance L is weber-1 or volt second ampere-1 (Vs/A). It is given the special name Henry and is
 

abbreviated as H. If we have a coil or a solenoid of N turns, the flux through each turn is B ⋅ ds . If this flux changes,

an e.m.f. is induced in each turn. The net e.m.f. induced between the ends of the coil is the sum of all these.Thus,
d  
E = −N ∫ B.ds
dt
One can compare this with the previous equation to get the inductance.

Illustration 8: The inductor shown in Figure 22.20 has inductance of 0.54 H and carries
i
di a b
a current in the direction shown that is decreasing at a uniform rate = − 0.03 A / s.
dt
(a) Find the self-induced e.m.f. L
Figure 22.20
(b) Which end of the inductor,a or b, is at a higher potential?  (JEE MAIN)

Sol: The e.m.f. induced in an inductordue to self-inductance opposesthe change in current in it. As the current
decreases, the induced e.m.f. tries to increase the current, thus a will be at higher potential.
dI
(a) Self-inducede.m.f. E = −L =
dt
( −0.54 )( −0.03) V =
1.62 × 10−2 V
dI
(b) Potential difference between two ends of inductor is Vba = L −1.62 × 10−2 V
=
dt
Since Vba (Vb-Va) is negative. It implies that Va>Vb or a is at higher potential.
2 2 . 1 2 | Electromagnetic Induction and Electromagnetic Waves

Illustration 9: Consider the circuit shown in the following Figure 22.21. The sliding contact is being pulled towards
the right so that the resistance in the circuit is increasing. Resistance at time instance is found to be12 Ω. Will the
current be more than 0.50 A or less than it at this instant? (JEE ADVANCED)

Sol: As resistance in the circuit changes, the current through the inductor also changes. 20 mH
Thus e.m.f. is induced in the inductor.
For change in resistance, there is equivalent change in the value of current. Then
r
dI 6V
inducede.m.f. in inductor E = −L
dt
dI
dI 6V − L Figure 22.21
The net e.m.f. in the circuit is 6V − L and hence current in circuit is I = dt
dt 12 Ω
 ... (i)
Due to continuous increase in resistance, the current in the circuit decreases.
Therefore, at given time instant t, the ratio dI/dt decreases, which makes numerator of eqn (i) higher than 6 and
hence, the current in the circuit is larger than 0.5 A

Illustration 10: An average e.m.f. of 0.20V appears in a coil when the current in it is changed from 5.0 A in one
direction to 5.0 A in the opposite direction in 0.20 s. Find the self-inductance of the coil.  (JEE MAIN)
dI
Sol: Using the formula E = −L , we can find inductance of coil.
dt

(i) The average change in current w.r.t. time t,


dI
di
=
( −5.0 A ) − (5.0 A ) = −50 A / s.
dt
dt 0.20s
dI 0.2
(ii) Using formula E = −L we get 0.2 V = 50 × L ⇒ L = = 4.0 mH
dt 50

8.2.1 Self-Inductance in a Long Solenoid


Consider a long solenoid of radius r having n turns per unit length. Suppose a current i is passed through the
solenoid. The magnetic field produced inside the solenoid is B = µ0ni . The flux through each turn of the solenoid
 
∫ B.ds. = ( µ0ni. ) πr
2
is Φ =
dΦ di
The e.m.f. induced in each turn is − = −µ0nπr 2
dt dt
(
As there are nlturnsin length l of the solenoid, the net e.m.f. across a length l is ε = − (nl) µ 0nπr
2
) dtdi
di
Comparing with ε = −L µ0n2 πr 2l.
, the self-inductance is L =
dt
We see that the self-inductance depends only on geometrical factors.
A coil or a solenoid made from thick wire has negligible resistance, but a considerable self-inductance. Such an
element is called an ideal inductor and is indicated by the symbol .
The self-inductance e.m.f. in a coil opposes the change in the current that has induced it. This is in accordance with
Lenz’s law. If the current is increasing, the induced current will be opposite to the original current. If the current is
decreasing, the induced current will be along the original current.

8.3. Inductance of a Solenoid


Let us find the inductance of a uniformly wound solenoid having N turns and length l. Assume that l is much longer
than the radius of the windings and that the core of the solenoid is air. We can assume that the interior magnetic
field due to a current i is uniform and is given by equation,
N N
B= µ0   i Where n=
µ0ni = is the number of turns per unit length.
l
  l
P hysi cs | 22.13

NS
The magnetic flux through each turn is, φB =BS =µ0 i . Here, S is the cross-sectional area of the solenoid.
l
NφB N  µ0NSi  µ0N2S µ0N2S
Now,
= L =  =  ∴ L=
i i l  l l

This result shows that L depends on dimensions (S,l) and is proportional to the square of the number of turns. L∝N2
Because N=nl, we can also express the result in the form,
(nl)
2

L=
µ 0 µ0n2Sl =
s= µ0n2 V µ0n2 V
or L =
l
Here, V=Sl is the volume of the solenoid.

Illustration 11: Two inductors L1 and L2 are placed sufficientlyapart. Find out equivalent inductance when they are
connected (a) in series (b) in parallel.  (JEE MAIN)
Sol: For inductors, when they are connected in series, the inductance of the combination should increase, while for
parallel connection, the inductance of combination should decrease
(i) In series the induced current i flows in the both the inductors and the total magnetic-flux linked with them will
be equal to the sum of the fluxes linked with them individually, that is, Φ=L1i+L2i
If the equivalent inductance be L. then Φ=Li ∴ Li=L1+L2 or L=L1+L2
(ii) In parallel, let the induced currents in the two coils be i1 and i2 Then the total induced current is I= I1 + I2
dI dI1 dI2
∴ = +
dt dt dt
dI dI
In parallel, the induced e.m.f. across each coil will be the same. Hence, E =
− L1 1 =
− L2 2
dt dt
di
If the equivalent inductance be L, then E = − L
dt
E dI  dI dI  E E 1 1 1 L1L 2
∴ = − = −  1 + 2  = + or = + or L =
L dt  dt dt L
 1 2 L L L1 L 2 L1 + L2

8.5 Energy Stored in an Inductor


The energy of a capacitor is stored as electric field between its x
plates. Similarly, an inductor has the capability of storing
energy in its magnetic field.  
F2 B
A changing current in an inductor causes an e.m.f. between I
its terminals
The work done per unit time is power. 
dW dI dW dU 
P = =−eI =LI from dW =−dU or = F1
dt dt dt dt L
dU dI
We have, −LI
=
= or dU =
LIDI
dt dt
The total energy U supplied while the current increases from

zero to a final value i is 

i F3
1 2 1 2
= ∫ IdI
U L=
2
Li=
; U
2
Li
0
b
This is the expression for the energy stored in the magnetic
field of an inductor when a current i flows through it. The Figure 22.22: Loop pulled out of magnetic field
source of this energy is the external source of e.m.f. that
supplies the current.
2 2 . 1 4 | Electromagnetic Induction and Electromagnetic Waves

Energy transfer
The rate at which you do work on the loop as you pull it from the magnetic field:
B2L2 ν2
P = Fν = (rate of doing work).
R
The rate at which thermal energy appears in the loop as you pull it along at constant speed. P = i2R.
2
 BLν  B2L2 ν2
Or, P =
=  R (thermal energy rate), which is exactly equal to the rate at which you are doing work on
 R  R
the loop.
Thus, the work that you do in pulling the loop through the magnetic field appears as thermal energy in the loop.

9. L-R CIRCUITS
Consider an inductor having inductance L and a resistor R are connected in series which is connected in series
to a battery of e.m.f. E in series through a two way key A,B,S as shown in the circuit diagram. When the switch S
is connected to A, the current in the circuit grows from zero value. When the current starts growing through the
inductance, a back e.m.f. is induced in the coil due to self-induction which opposes the rate of growth of current in
the circuit. Similarly, when the switch S is connected to B by disconnecting the battery, the current begins to fall.
The current, however, does not fall to zero instantaneously due to the e.m.f. induced in the coil due to self-induction
which opposes and reduces the rate of decay of current in the circuit.

9.1 Growth of Current


If S is connected to A during the growth of current, let I be instantaneous
current at any time in the circuit. A back e.m.f. equal
dI dI L B
to L will develop in the circuit so that effective e.m.f. in the circuit is E − L S
dt dt R L A
which is equal to potential drop of IR across resistor.
dI dI dt E
∴ E −L IR or
= =
dt E − RI L
Figure 22.23: Charging of LR
Integrating this equation between the limits when the current is zero at time circuit

t=0 to the instantaneous current I at time t,

E
I t −RT 
dI dt
∫ E −=
RI ∫L =, I 
R
1 − e L 

0 0 
E
If I0 is maximum current, so that I0 =
R 
E
 −Rt  = I0
I=I0 when exp =
 or t ∝
 0= R
 L 
Thus, current I approaches a value I0 asymptotically and grows 0.63I0
exponentially to a value equal to E/R. The curve for growth of the
current in L-R circuit is shown in the Figure. I

L
When t = ,
R  t
 −R L 
×  1 e − 1  2.718 − 1 
=I I0 1 − e L R  = I0 1 − = =
I0   I0=
  0.63 I0 Figure 22.24: Rise of current in LR circuit
   e  e   2.178 
P hysi cs | 22.15

L
The current reaches a value which is equal to 63% of the maximum value I0 after a time of τ = from the beginning.
R
L
∴ Time constant of the circuit = τ =
R
The time constant τ of a circuit is the time during which the current rises from zero to 63% of its maximum value.
 −t 
I I0 1 − e τ 
∴=
 
 

PLANCESS CONCEPTS

Inductor as stabilizer:
(a) From L-R circuits, we can see that for sudden changes in voltages, there is a smooth and continuous
10V
changes in current through inductor. Io == E / R = 0.10 A
100Ω
(b) Thus, inductor is used as a current stabilizer in circuits.
(c) Froma mathematical point of view, for any kind of voltage input (even discontinuous), current is
acontinuous function.
If voltage is continuous, then current is a smooth function.
Vaibhav Gupta (JEE 2009, AIR 54)

Illustration 12: An inductor (L=20 mH), a resistor (R=100 Ω) and a battery (E=10V) are connected in series. Find
(a) the time constant, (b) the maximum current and (c) the time elapsed before the current reaches 99% of the
maximum value. (JEE MAIN)
L
Sol: For LR circuit the current is= ( )
It Io 1 − e− t/ τ where τ = is the time constant of the circuit and maximum
R
E
current I0 =
R
L 20mH
(a) The time constant is. τ= = = 0.20ms
R 100Ω
(b) The maximum current is

(c) when It=0.99I0,then solving equation of current for time t we get

( ) ( )
It =Io 1 − e− t/ τ ⇒ 0.99 I0 =I0 1 − e− t/ τ  ⇒ e− t / τ =
0.01

⇒t=  0.2 × log (1 × 10 ) =


e
2
0.92 s

9.2 Decay of Current


If the S is connected to B, the battery is disconnected. The current does
I0
not fall instantaneously from I0 to zero but decays slowly due to the
current induced in the coil is in the direction opposite to that of the I
dI
falling current. The induced e.m.f. in the induced will be equal to −L 037I0
dt
corresponding to the instantaneous current I in resistor R at that time 
O t
dI
∴ −L =RI Figure 22.25: Decay of current in LR circuit
dt
Rate of decay of current = dI = −  R  I or dI = −  R  dt
   
dt L  I L  
2 2 . 1 6 | Electromagnetic Induction and Electromagnetic Waves

When t=0, the current, I0 is maximum and the current at time t is I.


I t RT −t
dI R −
∴ ∫ I
− ∫ dt=
=
L0
∴ I I=
0 e
L I0 e τ
I0

L
Where τ = is the time constant of the circuit.
R
−R L
L × I0 I0
When t = I I0 e
,= L R= = = 0.371 I0
R e 2.718

The time constant τ is defined as the time interval during which the current decays to 37%of the maximum current
during the decay. The rate of decay of the current shows an exponential decay behavior as shown in the Figure
22.27.
1 2
The energy stored in an inductor of inductance L, when the current I is passing through it, is equal to L I which
2
is in the magnetic form. Such LC circuit produces harmonic oscillation in an electrical circuit in which the energy
changes from the electrical to magnetic and vice versa. Such oscillations can be sustained in an electrical circuit and
can continue for a long time with the sane amplitude if there is negligible resistance in the circuit.

PLANCESS CONCEPTS

The formula for current in l-r circuit is very similar to that of charge in r-c circuit.
The basic similarities are its form i.e. exponential function.
Also, listed here are some basic points about capacitor, inductor and resistor.
(a) Resistor resists flow of charge.
(b) Capacitor resists change in the charge but can hold ideally any amount of charge.
(c) Inductors do not resist charge but resist change in current and ideally it can allow any amount of
current flow.
Nitin Chandrol (JEE 2012, AIR 134)

Illustration 13: A 50 mH inductor is in series with a 10Ω resistor and a battery with an e.m.f. of 25V. At t=0 the
switch is closed. Find: (a) the time constant of the circuit. (b) how long it takes the current to rise to 90% of its final
value;(c) the rate at which energy is stored in the inductor; (d) power dissipated in the resistor. (JEE ADVANCED)

( )
It Io 1 − et/ τ where τ =L / R is time constant,and the energy
Sol: For LR circuit, the current at any time instant is=
1 dUL V2
stored in the inductor is UL = L I2 andpower dissipated in the circuit is P= L & P=R I2=
R IV = (a) The
2 dt R
time constant is τ = L/R = 5×10−3s.
(b) We need to find the time taken for I to reach 90% of I0 i.e. 0.9I=0.9 E/R.
0.9I
= 0 (
I0 1 − e− t / τ )
From this we find that exp ( −t / τ ) =0.1 ⇒ ( −t / τ ) =In ( 0.1 ) . Thus, t = −τ In ( 0.1 ) = 11.5 × 10−3 s
(c) The rate at which energy is supplied to the inductor is

dUL dI dI dU −Rt/L
=+ LI ; =+E / Le−Rt/L ; There fore PPLL == LL =
=I × E × ee− t/ τ
dt dt dt dt
E2  −t / τ −2 t / τ 
We now substitute for I to obtain
= PL e −e 
R
P hysi cs | 22.17

 −2 t/ τ 
(d) The power dissipated in the resistor is PR = I2R =
I20R  1 − 2e− t/ τ + e 
 
E IP 1
From equation
= (iii), Es Is E=
P IP or s . In general, E∝
EP Is I

Illustration 14: (i) Calculate the inductance of an air core solenoid containing 300 turns if the length of the
solenoid is 25.0 cm and its cross-sectional area is 4.00 cm2.
(ii) Calculate the self-induced e.m.f. in the solenoid if the current through it is decreasing at the rate of 50.0 A/s.
 (JEE MAIN)
µ0N2S dI
Sol: For air core solenoid, inductance is calculated as L = and the e.m.f. induced in solenoid is E = −L
l dt
µ0N2S
(i) from the formula of inductance, L = we ..have,
l

=L
( 4π × 10 ) (300 ) ( 4.00 × 10 )=
−7 2

H
−4

1.81 × 10 −4 H
(25.0 × 10 ) −2

dI
(ii) Here ,
dt
= −50.0 A / s using formula of e.m.f. we get, E = (
− 1.81 × 10 −4 ) ( −50.0 ) 9.05 × 10−3 V =
= 9.05 mV

10. ENERGY STORED IN A MAGNETIC FIELD


To derive a quantitative expression for that stored energy, consider a source of e.m.f. connected to a resistor R and
an inductor L
di
If each side is multiplied by i, we obtain=
ξ L + iR,
dt
di 2
ξi Li
Which has the following physical interpretation in terms of work and energy:= + i R,
dt
(a) If a differential amount of charge, dq passes through the battery of e.m.f. in time dt. The battery works on
it in the amount dq. The rate at which the battery does work is (dq)/dt, or i. Thus, the left side of equation
represents the rate at which the e.m.f. device delivers energy to the rest of the circuit.
(b) The term on the extreme right in the equation represents the rate at which energy appears as thermal energy
in the resistor.
(c) Energy that is delivered to the circuit does not appear as thermal energy,but by the conservation-of-energy
hypothesis, isstored in the magnetic field of the inductor. Because the equation represents the principle of
conservation of energy for RL circuits, the middle term must represent the rate (dUB/dt) at which magnetic
potential energy UB is stored in the magnetic field.
dUB di
Thus = Li . We can write this as dUB = Li di.
dt dt
UB i
2 1
Integrating yields ∫ dUB = ∫ Li di or UB = Li (magnetic energy), which represents the total energy stored by
0 0
2
inductor L carrying a current i.

Illustration 15: Calculate the energy stored in an inductor of inductance 50 mH when a current of 2.0 A is passed
through it.  (JEE MAIN)
1
Sol: In LR circuit, magnetic energy is stored in inductor is U=
L L × I2
2
The energy stored is U =
1 2 1
2
Li =
2
(
50 × 10−3 H ( 2.0 A ) =
2
)
0.10 J.
2 2 . 1 8 | Electromagnetic Induction and Electromagnetic Waves

Illustration 16: What inductance would be needed to store 1.0 kWh of energy in a coil carrying a 200 A current?
(1kWh=3.6×10−6J)  (JEE MAIN)
1
Sol: In LR circuit, magnetic energy stored in inductor is U=
L L × I2
2
We have, i=200 A and U=1kWh= 3.6 × 10−6 J

∴ Using formula of energy we get


=
2U 2 3.6 × 10
L= = 180 H
( 6
)
( 200 )
2
i2

11. ENERGY DENSITY OF A MAGNETIC FIELD


Consider a length l near the middle of a long solenoid of cross-sectional area A carrying current i; the volume
associated with this length is Al. The energy UB stored by the length l of the solenoid must lie entirely within this
volume because the magnetic field outside such a solenoid is approximately zero. Moreover, the stored energy
must be uniformly distributed within the solenoid because magnetic field is (approximately) uniform everywhere
inside.
UB
Thus, the energy stored per unit volume of the field is uB =
Al
1 LI2 L I2
Or, since UB = LI2 , we have =
UB = .
2 2Al I 2A
Here L is the inductance of length l of the solenoid.
L 1
Substituting for = µ0n2 A , we get uB= µ n2i2 where n is the number of turns per unit length. We know that
l 2 0
B2
B = µ0in, we can write this energy density as uB = (magnetic energy density).
2µ0

This equation gives the density of stored energy at any point where the magnitude of the magnetic field is B. Even
though we derived it by considering the special case of a solenoid, this equation holds for all magnetic fields, no
1
matter how they are generated. This equation is comparable to uE= ε E2
2 0
Which gives the energy density (in a vacuum) at any point in an electric field. Note that both uB and uE are
proportional to the square of the appropriate field magnitude, B or E.

Problem solving tactic


To solve the problems, one would need to learn many of the above formulae. For this, l simply advise that one
should make analogy with electric field, capacitors, etc.
1 1
UB = LI2 looks similar to CV 2 . A similar one for energy density formula is also available, where the electric field can
2 2
be replaced with magnetic field, and absolute permittivity with absolute permeability’s inverse.

13. MUTUAL INDUCTANCE i

Suppose two closed circuits are placed close to each other and a
current i is passed in one. It produces a magnetic field and this field
has a flux Φ through the area bounded by the other circuit. As the
magnetic field at a point is proportional to the current producing it,
R
we can write Φ=MI where M is a constant depending on the
geometrical shapes of the two circuits and their placing. This
Figure 22.26: Mutual inductance of two coil
P hysi cs | 22.19

constant is called mutual inductance of the given pair of circuits. If the same current i is passed in the second circuit
and the flux is calculated through the area bounded by the circuit, the same proportionality constant M appears. If
there ismore than one turn in a circuit, one has to add the flux through each turn before applying the above
equation.
If the current i in one circuit changes with time, the flux through the area bounded by the second circuit also
changes. Thus, an e.m.f. is induced in the second circuit. This phenomenon is called mutual induction. From
dΦ dI
di
theabove equation, the induced e.m.f. is E = − = −M
dt dt
dt

Illustration 17: A solenoid S1 is placed inside another S2


solenoid S2 as shown in Figure 22.27. The radii of the S1
inner and the outer solenoid are r1 are r2 respectively
and the numbers of turns per unit length are n1 and
n2 respectively. Consider a length l of each solenoid.
Calculate the mutual inductance between them.
 (JEE ADVANCED) Figure 22.27

Sol: The flux linked with the secondary coil due to primary coil, is φ =MI .

Suppose a current i is passed through the inner solenoid S1. A magnetic field B = µ0n1i is produced inside S1 where
the field outside of it is zero. The flux through each turn of S2 is Bπr12 =µ0n1i πr12
The total flux through all the turns in a length l of S2 is
Φ= ( )
µ0n1 I πr12 n2l = (
µ0n1n2 πr12 l I) µ0n1n2 πr12 l.
Thus, M =

… (i)

14. OSCILLATING L-C CIRCUITS


If a charged capacitor C is short-circuited through an inductor L, the charge and current in the circuit start oscillating
simple harmonically. If the resistance of the circuit is zero, no energy is dissipated as heat. We also assume an
idealized situation in which energy is not radiated away from the circuit. With these idealizations – zero resistance
and no radiation – the oscillations in the circuit persist indefinitely and the energy is transferred from the capacitor’s
electric field to the inductor’s magnetic field and back. The total energy associated with the circuit is constant. This
analogous to the transfer of energy in an oscillating mechanical systemfrom potential energy to kinetic energy and
back, with constant total energy. Later we will see that this analogy goes much further.
Let us now derive an equation for the oscillations in an L-C circuit.
i

t=0 b t=t c

+ +
C q0 L C q L
- -
a d

S S
(a) (b)
Figure 22.28: LC circuit

Refer Figure 22.28 (a): A capacitor is charged to a P.D. V0 = q0 C


Here, qo is the maximum charge on the capacitor. At time t=0, it is connected to an inductor through a switch S. At
time t=0, switch S is closed.
Refer Figure 22.28 (b): When the switch is closed, the capacitor starts discharging. Let at time t charge on the
capacitor is q (<q0) and since, it is further decreasing there is a current i in the circuit in the direction shown in
2 2 . 2 0 | Electromagnetic Induction and Electromagnetic Waves

Fig.22.28 (b). Later we will see that, as the charge is oscillating there may be a situation when q will be increasing,
but in that case, direction of the current is also reversed and the equation remains unchanged.
The potential difference across capacitor=potential difference across inductor, or
q  di 
Vb − Va = Vc − Vd ∴ = L   … (i)
C  dt 
 −dq  di d2q
Now, as the charge is decreasing, ∴ i=
  or =−
 dt  dt dt2
q  d2q  d2q  1 
Substituting in Eq. (i), we get −L 
=  or − q 
= … (ii)
C  dt2  dt 2
 LC 
 
 d2 x 2

This is the standard equation of simple harmonic motion  2 = − ω x  .
 dt 
 
1
Here, ω =  ... (iii)
LC
(ii), is q q0 at cos ( ωt ± φ )
The general solution of Eq.=
For example in our case φ=0 as q=q0 at t=0.
Hence,
= q q0 cos ωt  … (iv)
Thus, we can say that charge in the circuit oscillates simple harmonically with angular frequency given by Eq. (iii).
1 ω 1 1
Thus, ω = , f= = and T = = 2π Lc
LC 2π 2π Lc f
The oscillations of the L-C circuit are electromagnetic analog to the mechanical oscillations of a block-spring
system.
q UC
q0 2
q max
t
2C
t

i UL
2
i0 Li max
2
t
T T 3T 2T T T 3T T
0 2 2 0 4 2 2

Figure 22.29: LC oscillations

Illustration 18: Two conducting loops of radii R and r are concentric and coplanar. Find the
mutual inductances of the system of the two loops. Take R>>>r.  (JEE ADVANCED)

Sol: For current I in the circuit, magnetic field is produced around and at the center of the R r
coil. The flux linked with the smallerloopis the product of the magnetic field at the center due
to the bigger loop and the area of the smaller loop.
Consider a current I passing through the large loop. The magnetic field at the center of this I
µ I Figure 22.30
loop due to this current is B = 0
2R
Now since r is very small in comparison to R, value of B can be considered uniform over πr2 area of the inner loop.
∴ The flux linked with the smaller loop is given by
µ0 I µ0 π Ir 2 Φ µ0 πr
2
=
Φ .π
=r2 ; ∴ M= =
2R 2R I 2R
P hysi cs | 22.21

15. ELECTROMAGNETIC WAVES


It is known that in certain situations light may be described as electromagnetic wave. The wave equation for light
propagating in x-direction in vacuum is written as follows
= B00 sin ω ( t − x / c ) 
BE=E … (i)
Here E is the sinusoidally varying electric field at the position x at time t. The constant c is the speed of light in
vacuum. The electric field E is in the Y-Z plane, i.e., perpendicular to the direction of propagation.
There is also a sinusoidally varying magnetic field associated with the electric field when light propagates. This
magnetic field is perpendicular to the direction of propagation as well as to the electric field E. It is given by
B B0 sin ω ( t − x / c ) 
= … (ii)
Such a combination of mutually perpendicular electric and magnetic field is referred to as an electromagnetic wave
in vacuum.

16. MAXWELL DISPLACEMENT CURRENT


 
Ampere’s law is stated as �∫ . dl = µ0 Ii
B

… (iii)
Here i is the electric current crossing a surface bounded by a closed curve and the line integral of B (circulation) is
calculated along that closed curve. When the electric current at the surface does not change, this equation is valid.
This law tell us that an electric current produces magnetic field and gives a method to calculate the field.
Ampere’s law in this from is not valid if the electric field at the surface varies
with time. As an example, consider a parallel-plate capacitor with circular plates, 
being charged by a battery (Figure 22.31). If we place a compass needle in the S
space between the plates, the needle, in general, deflects. This shows that there
is a magnetic field in this region. Figure 22.31 also shows a closed curve γ which i
lies completely in the region between the plates. The plane surface S bounded
by this curve is also parallel to the plates and lies completely inside the region
between the plates. Figure 22.31
During the charging process, there is an electric current through the connecting wires. Charge is accumulated on
the plates and the electric field at the points on the
 surface
 S changes. It is observed that there is a magnetic field
at the points on the curve γ and the circulation B . dl . This equation gives a nonzero value. As no charge crosses
�∫
the surface S, the current I through the surface is zero. Hence,
 
�∫ . dl ≠ µ0 Ii
B … (iv)
Now, Ampere’s law (i) can be deduced from Biot-Savart law. We can calculate the magnetic field due to each current
element from Biot-Savart law and then its circulation along the closed curve γ . The circulationof the magnetic field
due to these current elements must satisfy equation (i). If we denotes this magnetic field by B' , then
 
�∫ . dl = 0 
B'
 
… (v)
This shows that the actual magnetic field B is different from the field B' produced by the electric currents only. So,
there must be some other source of magnetic field. This other source is nothing but the changing electric field. As
the capacitor gets charged, the electric field between the plates changes and this changing electric field produces
magnetic field.
It is known that a changing magnetic field produces an electric field. The relation between the two is given by
Faraday’s law
  d ΦB
�∫ E . dl = − dt  … (vi)
 
Here, ΦB = ∫ B.dS is the flux of the magnetic field through the area bounded by the closed curve. Along this curve
the circulation of E is calculated. Now we find that a changing electric field produces a magnetic field. The relation
2 2 . 2 2 | Electromagnetic Induction and Electromagnetic Waves

between the changing electric field and the magnetic field resulting from it is given by
  d ΦE  … (vii)
�∫ . dl =µ0 ε0 dt
B

 ΦE is the flux of the electric field through the area bounded by the closed curve along which the circulation
Here,
of B is calculated. Equation (iii) gives the magnetic field resulting from an electric current due to flow of charges.
Equation (vii) gives the magnetic field due to the changing electric field. If there exists an electric current as well as
a changing electric field, the resultant magnetic field is given by
   d ΦE 
�∫ B.dl = µ0 i + µ0 ε0  dt 
 
 
Or, �∫ B.dl =µ0 (i + id )  … (viii)

d ΦE
In the above equation id = ε0 is the displacement current.
dt

Illustration 19: For a charging parallel plate capacitor, prove that the displacement current across an area in the
region between the plates and parallel to it is equal to the conduction current in the connecting wires.

Sol: For electric flux ΦE associated with the surface of one of the parallel plates, the displacement current in and
d ΦE
across the area of the parallel plate is id = ε0 .
dt
The electric field between the plates is E = Q
ε0 A

Where Q is the charge accumulated at the positive plate. The flux of this field through the given area is
Q Q
Φ=
E A
×=
ε0 A ε0
d ΦE d  Q  dQ
The displacement current is id =
ε0 =
ε0  =
dt dt  ε0  dt
dQ
But is the rate at which the charge is carried to the positive plate through the connecting wire. Thus, id=ic
dt

17 MAXWELL’S EQUATIONS AND PLANE ELECTROMAGNETIC WAVES.


We can summarize the concepts of electricity and magnetism mathematically with the help of four fundamental
equations:
  q
Gauss’s law for electricity ∫ E ⋅ dS =
�  … (ix)
ε 0
 
Gauss’s law for magnetism �∫ ⋅ dS =
B 0 … (x)

  dΦB
Faraday’s law �∫ E ⋅ dl =− dt
 … (xi)

  dΦE
Ampere’s law �∫ B ⋅ dl = µ0i + ε0µ0 dt
 … (xii)

These equations are collectively known as Maxwell’s equations.


In vacuum, there are no charges and hence no conduction currents. Faraday’s law and Ampere’s law take the form
  dΦB


� E ⋅ dl =−
dt
 ... (xiii)
P hysi cs | 22.23

  dΦE
and �∫ B ⋅ dl =µ0 ε0 − dt
 … (xiv)

Respectively.
Let us check if these equations are satisfied by a plane electromagnetic wave given by

E= Ey = E0 .sin ω(t − x/ c) 
 … (xv)
and B= Bz = B0 sin ω(t − x/ c)

The wave described above propagates along the positive x-direction, the electric field remains along the y-direction
and the magnetic field along the z-direction. The magnitudes of the fields oscillate between ± E0 and ± B0
respectively. It is a linearly polarized light, polarized along the y-axis.
From the theory of the waves, we can prove the relations between electric and magnetic field represented in
equation (xv) as

E0 = c B0 .  … (xvi)
1
B0 =µ0 ε0 c E0 ⇒ µ0 ε0 =  … (xvii)
c2
1
Or, c =  … (xviii)
µ0 ε0
2π ω E 1
The wave number k = and speed of light in vacuum is c = = fλ = o =
λ k Bo εo µo
In general the speed of electromagnetic waves in the medium of electric permittivity ε and magnetic permeability
1
µ is v =
µε

Illustration 20: The maximum electric field in a plane electromagnetic wave is 900 N C-1. The wave is going in the
x-direction and the electric field is in the y-direction. Find the maximum magnetic field in the wave and its direction.

Sol: The magnetic field is found using the relation E0 = c B0


E0 900 NC−1
We have B0 = = = 3 × 10 −6 T.
c 3 × 108 ms−1
 
As E,B and the direction of propagation are mutually perpendicular, B should be along the z-direction.

18 ENERGY DENSITY AND INTENSITY IN ELECTROMAGNETIC WAVE


The electric and magnetic field in a plane electromagnetic wave are given by

=E E0 sin ω(t − x/ c) and


= B B0 sin ω(t − x/ c).
1
In any small volume dV, the energy of the electric field is UE= ε E2dV  … (xix)
2 0
1 2
And the energy of the magnetic field is UB = B dV  … (xx)
2µ0
1
Thus, the total energy is U = 1 2
ε0 E2dV + B dV  … (xxi)
2 2µ0

1 1 2 1 1 2
The energy density is u = ε0E2 + B = ε E2 sin2 ω(t − x/ c) + B sin2 ω(t − x/ c)  … (xxii)
2 2µ0 2 0 0 2µ0 0
If we take the average over a long time, the sin2 terms have an average value of ½ Thus,
2 2 . 2 4 | Electromagnetic Induction and Electromagnetic Waves

1 1 2
uau = ε0 E20 + B  … (xxiii)
4 4µ 0 0
From equations (xvi) and (xx)
2 2
1 1 2 ε0 c  E0  1
=E0 cB0 and
= µ0 ε0 so that, B0=   = ε0 E02
c2 4µ 0 4  c  4

Thus, the electric energy density is equal to the magnetic energy density in average.
1 1 1
Or, uav = ε E2 + ε E2 = ε E2  ... (xxiv)
4 0 0 4 0 0 2 0 0

Also, u = 1 B2 + 1 B2 = 1 B2 . ... (xxv)


av
4µ0 0 4µ0 0 2µ0 0

given by E (50 NC−1 )sin ω(t − x/ c) . Find the energy


Illustration 21: The electric field in an electromagnetic wave is =
contained in a cylinder of cross-section 10 cm and length 50 cm along the x-axis.
2

1
Sol: The energy of electric field is given by U
=E V ε E2 where V is the volume of the cylinder
2 0
1 1
The energy density is uav = ε0 E20 = × (8.55 × 10−12 C2N−1m−2 ) × (50 NC−1 )2 = 1 ⋅ 1 × 10−8 Jm−3
2 2
The volume of the cylinder is V=10 cm2 x 50 cm=5 x 10-4 m3.
The energy contained in this volume is U = (1 ⋅ 1 × 10−8 Jm−3 ) × (5 × 10−4 m3 ) = 5 ⋅ 5 × 10−12 J

Intensity
The energy crossing per unit area per unit time perpendicular to direction of propagation is called the intensity of
a wave.
ct

A x

Figure 22.32

Consider a cylindrical volume with area of cross-section A and length c ∆t along the X-axis (See Figure 22.32).
The energy contained in this cylinder crosses the area A in time ∆t as the wave propagates at speed c. The energy
contained is
=U uav (c ∆ t) A .

U
The intensity is of the wave =
is I = uav c.
A∆t
1
In terms of maximum electric field, the intensity is written as I= ε E2 c  … (xxvi)
2 0 0

Illustration 22: Find the intensity of the wave discussed in Illustration 3


1
Sol. The intensity of the wave in terms of electric field is given by I= ε0 E20 c . The intensity is
1
2
I = ε0 E20 c = (1.1 × 10−8 Jm−2 ) × (3 × 108 ms−1 ) = 3.3 Wm−2 .
2
P hysi cs | 22.25

19. MOMENTUM
The propagating electromagnetic wave also carries linear momentum with it. The linear momentum carried by the
U
portion of wave having energy U is given by p =  … (xxvii)
c
Thus, if the wave incident on a material surface is completely absorbed, it delivers energy U and momentum
p=U/c to the surface. If the wave is totally reflected, the momentum delivered to the surface of the material is 2U/c
because the momentum of the wave changes from p to –p. It follows that electromagnetic waves incident on a
surface exert a force on the surface.

20. ELECTROMAGNETIC SPECRUM AND RADIATION IN ATMOSPHERE


Maxwell’s equations are applicable for electromagnetic waves of all wavelengths. Visible light has wavelengths
roughly in the range 380 nm to 780 nm. Today we are familiar with electromagnetic waves having wavelengths
as small as 30 fm (1 fm=10-15 m) to as large as 30 km. Figure 22.33 shows the electromagnetic spectrum we are
familiar with.
4 -4 -B -B
3 x 10 m 3m 3 x 10 m 3 x 10 m 3 x 10 m Wavelength (m)

Infrared Ultraviolet Gamma rays

Radio waves X-rays

Microwaves Visible light


(e.g. radar)
4 6 8 10 12 14 16 18 20
10 10 10 10 10 10 10 10 10 Frequency (Hz)
Figure 22.33

The accelerated charge is the basic source of electromagnetic wave. This produces changing electric field and
changing magnetic field which constitute the wave. Among the electromagnetic waves, visible light is most familiar
to us. This is emitted by atoms under suitable conditions. An atom contains electrons and the light emission is
related to the acceleration of an electron inside the atom. The mechanism of emission of ultraviolet radiation is
similar to that for visible light.

PROBLEM SOLVING TACTIC


You can remember a single point that when uniform field is into the paper and the rod is moving to the right, i.e.
moving out of magnetic field, then higher potential is at the upper end with a difference of Bvl. By remembering
this single point you can change it whenever required according to actual situation by just reversing the sign. (E.g.
if field is out of the paper and all other conditions are same, then multiply a negative sign.)
2 2 . 2 6 | Electromagnetic Induction and Electromagnetic Waves

FORMULAE SHEET

 
(a) Flux of magnetic field through a surface: ΦB = ∫ B ⋅ ds
(b) Faraday’s law of electromagnetic induction

dΦB N ⋅ d ΦB
(i) in coil of single loop Eξ = − (ii) in coil of N loops ξE = − where E is induced E.M.F.
dt dt
    
(c) Motional E.M.F. ξE = − ∫ E ⋅ d  = ∫ (v ×B) ⋅  = vB 
vB 
(d) The magnitude of induced current is I =
R
  dΦB
(e) Electric field induced due to changing magnetic field ∫ E ⋅ d  =

dt
(  vB )
2

(f) Power P = F × v =
R
NΦB
(g) Self-inductance of a coil is L =
I
µ0n2 πr 2
(h) For infinitely long solenoid, self-inductance per unit length Lunit length =

dI
(i) Self-Induced e.m.f. ξE = −L
dt
( j) Series Inductors: L=L1+ L2 + …..

1 1 1
(k) Parallel Inductors: = + + .....
L L1 L 2

(l) For LR circuit


dI E
(i) Sourcee.m.f. is=E L + IR =
(ii) Growth of current is I (1 − e−t / τ )
dt R
E −t / τ L
(iii) Decay of current is i = (e ) (iv) Time constant τ =
R R
1 2
(m) Energy stored in an Inductor is U = LI
2
U B2
(n) Energy density in magnetic field is u =
B =
V 2µo
(o) In LC circuit

(i) The p.d. across each component is V= q= L  di  q q0 cos ( ωt ± φ )


 
(ii) Charge in capacitor
=
C  dt 
(iii) Frequency of oscillation ω = 1
LC
di2 di1
(p) E.m.f. due to Mutual Induction E1 = − M E2 = − M
dt dt
ω E 1
(q) Speed of electromagnetic wave: c = = fλ = o =
k Bo εo µo
1 1 2
(r) Energy density in electromagnetic wave uav = ε0 E02 = B
2 2µ0 0
1
(s) Intensity of wave in terms of maximum electric field is I= ε E2 c
2 0 0
P hysi cs | 22.27

Solved Examples

JEE Main/Boards d ΦB E
Using formula E = , I= & dq = I × dt
dt R
Example1:A coil made up of inductance L=50 µH and
resistance r=0.2 Ω is connected to a battery of e.m.f.=5.0
∆φ 8 × 10−3
When ∆t=0.01 s − B =
E= 0.8 V
=
V.A resistance R=10 Ω is connected parallel to the coil. ∆t 0.01
Now at some instant the connection of the battery is
E 0.8
switched off. Find the amount of heat generated in the I
= = = 0.16 A
coil after switching off the battery. R 5
& ∆q = I × ∆t = 0.16 × 0.01 = 1.6 × 10−3 C
Sol: In LR circuit, the magnetic energy is stored in
When ∆t=0.01 s
1
inductor and is U=L L × I2
2 ∆φ 8.0 × 10−3
Given: (i) L= 50 µH , (ii) r=0.2 Ω, − B =
E = 0.4 V
=
∆t 0.02
(iii) R=10 Ω
E 0.4
We want to find the fraction of energy lost by the I
= = = 0.8 A
inductor in the form of heat. R 5
& ∆q = I × ∆t = ( 0.08 )( 0.02 ) = 1.6 × 10−3 C
Total energy stored in the inductor is
2
1 2 1 V Example 3: A coil of area 2 m2 is placed in magnetic
UL =
= Li L 
2 0 2 r fieldwhich varies as=
B ( )
2t2 + 2 T with area vector in
the direction of B. What is the magnitudeof E.M.F.at
∴ Fraction of energy lost across inductor as heat
t=2s?
r LV 2 50 × 10 −6 × 52
= UL •= = =3.1×10−4J
(R + r ) 2r (R + r ) 2 × 0.2 (10 + 0.2) Sol: The rate of change of magnetic flux linked with the
= 3.1 × 10 J −4 dφ
coil is equal to the induced e.m.f. in the coil E = −
Example 2: A square loop ACDE of area 20 cm2 and dt
resistance 5 Ω is rotated in a magnetic field B=2T
through 1800
Find the magnitude of E, i and ∆q after time
(a) 0.01s and (b) in 0.02s.

Sol: When the loop is rotated in external magnetic


field, the change in flux linked with the loop induces
e.m.f. in it.
 
Let S be the area

vector of loop. Before rotation
 S is
in direction to B . After

rotating loop by 180o S is in
opposite direction to B . We want to find E.M.F. through the coil when

Hence, flux through the loop before rotation is t=2 s. If we find the rate of change of flux, we have
E.M.F.
φi = BS cos00 = 2 × 20 × 10−4 =4.0×10−3 Wb  ... (i)
For θ =0= , φ BA =
cos θ BA cos 0
o
=
& flux × 10−3 Wb
4.0passing through the loop...(1)
when it is rotated by dφ dB
Differentiating the above equation, we get = .A
180 ,
0
dt dt
φf =BS cos1800 =− 1 × 2 × 20 × 10−4 =4.0×10−3 Wb  ... (i) dB  dΦB 
E A. = A ( 4t + 4 )
⇒=  |=
E| 
−3 dt  dt 
= −4.0 × 10
Therefore, changeWbin flux, ...(2)
for =
A 2; | E=| 8t + 8
∆φB = φf − φi = − 8.0 × 10−3 Wb
When t=2 s, |E|=16+8=24 V
2 2 . 2 8 | Electromagnetic Induction and Electromagnetic Waves

Example 4: A current i=(3+2t)×10-2 Aincreases at a Example 5: What inductance would be needed to store
steady rate in a long straight wire. A small circular loop 1.0kWh of energy in a coil carrying a 200Acurrent?
of radius 10-3 m has its plane parallel to the wire and
placed at a distance of 1m from the wire. The resistance
(1kWh= 3.6 × 106 J )
of the loop is 8 mΩ. Find the magnitude and the 2U
Sol: The inductance in the coil is L =
direction of the induced current in the loop. i2
Given: (i) energy stored in inductor UL=1 kWh=3.6 MJ,
Sol: As the circular loop is small, the magnetic field (ii) Current =200 A.
through it can be assumed to be uniform, having
We want to find inductance of coil.
magnitude equal to that of the field at the center of the
circular loop, and flux associated with loop is φ = Bπr 2 . 1
The energy stored in inductor is UL = Li2
dφ 2
The emf induced in loop is E = .
dt The inductance is
1m 2U 2 × 3.6 × 106
∴L= = = 180 H
i2 ( 200 )
2

Example 6: The two rails of a railway track insulated


from each other and the ground are connected to a
millivolt-meter. What is the reading of the voltmeter
Wire Loop when a train travels at a speed of 108 kmh-1 along the
track? Given the vertical component of earth’s magnetic
field = 2 × 10−4 T& separation between the rails= 1m.
The arrangement is shown in Figure. The field due to
straight wire at the center of loop is: Sol: Here the train can be considered to move
µ 2I 2I perpendicular to the earth’s magnetic field. Due to
B= 0 =10 −7 × = 2I × 10−7 T
4π d 1 motion of the train, motional e.m.f. is induced in the
& flux linked with the loop is dφ
axle of train, given by E =
− =Bv sin θ
dt
( )
2
φ = BA= B × πr 2 = 2I × 10−7 × π × 10−3 Wb The train moves in a direction perpendicular to the
component of the earth’s magnetic field. So the flux
(Area of coil is very small so B over it can be taken to associated with the axle of train changes such that the
be constant) induced E.M.F. in axle is given by
E.M.F. Einduced in the loop due to change of current is dφ
E= − = Bv sin θ  ...(1) ... (i)
dφ dI dt
e= = 2π × 10 −13   
dt dt As (v × B) is parallel to  , θ =0o

 I =( 3 + 2t ) × 10−2 ∴ E=− Blv  ...(2) ... (ii)


where l=1m, BV= 2 × 10 −4
dI
So, = 2 × 10−2 As−1
dt 180 × 1000
=&v = 50ms−1 ...(3) ... (iii)
And hence, e = 2π × 10 −13
× 2 × 10 −2
= 1.26 × 10 −14
V 60 × 60 

Induced current in the loop From (i),(ii) & (iii)

E 1.26 × 10−14 E = 2 × 10−4 × 1 × 50 = 10 × 10−3 mV


I =
= = 1.6 × 10−11 A
R 8 × 10−4 ∴ Milli-voltmeter will read 10 mV when the train passes
with a speed of 108 km/h.
Due to an increase in the current in the wire, the flux
linked with the loop will increase.So in accordance
with Lenz’s law, the direction of the current induced Example 7: A very small circular loop of area 5 cm2
in the loop will be opposite of that in the wire, i.e., &resistance 2 Ω, and negligible inductance is initially
anticlockwise. coplanar and concentric, with a much larger fixed
circular loop of radius 10cm. A constant current of 1
P hysi cs | 22.29

Ais passed in the bigger loop and the smaller loop is is observed that when the terminal velocity is attained,
rotated with angular velocity wrad/s about a diameter. the power dissipated in R1 and R2 are 0.76 W and 1.2 W
Calculate (a) the flux linked with the smaller loop (b) respectively. Find the terminal velocity of the bar and
induced e.m.f. and current in the smaller loop as a the values of R1 and R2.
function of time.
A R2 C

 FM

IA
a - I +
b
mg

B D
R1
Sol: Current in the larger loop produces magnetic field
at the center of the loop. Magnetic flux is linked with Sol: The motional e.m.f. induced in the bar is E=ℓBv. The
the smaller loop. When the smaller loop is rotated, flux direction of induced current in the bar is as shown in
linked with it changes, and thus e.m.f. is induced in it. Figure. By Fleming’s left hand rule the ampere force on
the bar will be vertically upwards.
(a) The Figure represents the arrangement of coils.
When current passes through the larger loop, the field The bar falling freely under action of gravity will acquire
at the center of larger loop is, terminal velocity only when its motion is opposed by
magnetic force FM=Bil,
µ0 I µ 0 2π × I 2π × 1 Wb
B1 = = = 10 −7 × = 2π × 10 −6 Such that Bil=mg
2R 4π R 0.1 m2
0.2 × 9.8 9.8
is normal to the area of smaller loop. i.e., I
= = A
0.6 × 1 3
The smaller loop is rotating at angular velocityω. The total power dissipated in the circuit if E is the E.M.F.
Therefore the angle of rotation is θ = ωt w.r . to B linked with the coil is
The flux linked with the smaller loop at time t,
E × I = P = P1 + P2
φ2= B1S2 cos θ= (2π × 10 )(5 × 10 ) cos(ωt)
−6 −4
⇒ E
=
( 0.76
=
+ 1.20 )
0.6 V
−9
i.e., φ2 = π × 10 cos(ωt) Wb ( 9.8 / 3)
E 0.6
The E.M.F. E=l•BvT ∴ vT = = =1 ms−1
(b) The induced e.m.f. in the smaller loop, Bl 0.6 × 1
dφ2
E2 = −
dt
d
= −
dt
(
π × 10−9 cos ωt ) Using the formula of =
power P
V2
=
R
i.e., R
V2
P
i.e., E2 = π × 10−9 ω sin ωt For constant potential drop V1 =V2 =E

( 0.6 )=
2
And induced current in the smaller loop, E2 9
R= = Ω &,
E2 1 1
P1 0.76 19
I2= = πω × 10 −9 sin ωt A.
R 2
( 0.6 )=
2
E2
R=
2 = 0.3 Ω
Example 8: Two parallel vertical metallic rails AB and P2 1.20
CD are separated by 1 m. They are connected at the ∴ The terminal velocity of the rod is 1m/s & R1=0.47 Ω
two ends by resistances R1 and R2 as shown in Figure & R2=0.3 Ω
22.40. A horizontal metallic bar of mass 0.2 kg slides
without friction, vertically down the rails under the
Example 9: A square metal wire loop of side 10 cm and
action of gravity. There is a uniform horizontal magnetic
resistance 1 Ω is moved with a constant velocity V0 in
field of 0.6 T perpendicular to the plane of the rails. It
a uniform magnetic field of induction B=2 Wbm-2. The
2 2 . 3 0 | Electromagnetic Induction and Electromagnetic Waves

magnetic field lines are perpendicular to the plane of NS ES


the loop directed into the paper. The loop is connected For transformer the =
NP EP
to a network of resistors, each of 3 Ω. The resistances of
lead wire OS and PQ are negligible. What should be the E  4.6 × 1000 × 1000
s
speed of the loop so as to have a steady current 1 mA = N s
= N = 20,000
 Ep  p 230
in the loop? Give the direction of current in the loop.  
If Ip is current in primary, than the power in primary


coil is
V0
S PP = IP × EP = 6.9 kW
O 3 3
B 6.9 × 103
∴ Ip
= = 30 A ;
A C 230
3
P 3 Is Np 1000 1
3 &= = =
Q Ip Ns 20000 20

1 30
 Is = ×I = = 1.5 A ;
20 P 20
Sol: The network of resistors is a balanced wheatstone ∴ Current rating of the secondary coil is 1.5
bridge. The induced e.m.f. in the loop is E=BlV, where l
is one side of square loop, moving with speed v in the
magnetic field. Example 11: An infinitesimally small bar magnet of
dipole moment M is pointing and moving with the
The network mesh ASCQ is a balanced Wheatstone. So speed v in the x-direction. A small closed circular
there is no current through branch AC. conducting loop of radius ‘a’ and of negligible self-
Let R be the effective resistance of mesh ASCQ inductance lies in the y-z plane with its center at x=0,
and its axis coinciding with the x-axis. Find the force
6×6 opposing the motion of the magnet, if the resistance of
∴R = = 3Ω
6+6 the loop is R. Assume that the distance x of the magnet
Resistance of loop OSCQP =3+1=4 Ω from the center of the loop is much greater than a.

Let speed of loop through the field be V0 Sol: The flux linked with loop due to magnetic field of
bar magnet will decrease as the bar moves away from
∴ The induced E.M.F. in the loop is E=BlV0
the loop. The current induced in the loop will oppose
E = 2×0.1×V0 = 0.2V0 its cause i.e. will create a magnetic field at the location
of bar magnet such that the bar magnet is attracted
& using Ohm’s law the current in the circuit is towards the loop, thus bar magnet is decelerated.
E BlV0 0.2V0 Field due to the bar magnet at distance x (near the
I
= = =
R R 4 µ0 2M
4 × 10−3 loop) B =
−3
 I =10 A ⇒ V0 = =2 × 10−2 ms−1 4 π x3
0.2
Flux linked with the loop:
According to Fleming’s right hand rule direction of µ0 2M
φ = BA = πa2 ×
induced current in the loop is in clockwise direction. 4 π x3
e.m.f. induced in the loop:
Example 10: A power transformer is used to step up an dφ µ0 6π × Ma2 dx µ0 6πMa2
alternating e.m.f. from 230 V to 4.6kV to transmit 6.9KW E=
− = = v
dt 4 π x4 dt 4 π x 4
of power. If primary coil has 1000 turns, find
∴ Induced current:
(a) no. of turns in the secondary
E µ0 3πMa2 3µ Ma2
(b) the current rating of the secondary coil. I= = × ⋅v = 0 ⋅v
R 2π Rx 4 2Rx 4
Sol: For coil of transformer E ∝ N where E is induced (B) Find the opposing force
E.M.F. and N is number of turns in the coil.
The induced current develops field around it. As coil is
P hysi cs | 22.31

moving in the external field it will be opposed by the Example 13: A light beam travelling in the x - direction
force which is equal to heat dissipated in the coil due is described by the electric field
= Ey 300 sin ω(t − x / v)
to resistive force. . An electron is constrained to move along the
y-direction with the speed of 2.0 × 107 m/s. Find the
Heat dissipated in coil= Resistive force acting on coil
maximum electric force and the maximum magnetic
while it is in motion.
force on the electron.
I2R ;
∴ Fv = (Dimension of power)
2 Sol: The maximum force exerted by the wave is
I2R  3µ0Ma 
2 2 2 4
R 9 µ0M a v F=FE+FB= qE + qvB.
⇒ F= =   × v2 × = .
v  2Rx 4  v 4 Rx8
  (i) Maximum electric field E0 = 300 V / m

∴ Maximum electric force FE=qE0


Example 12: In an L-C circuit L=3.3 H and C=840 pF.
At t=0 charge on the capacitor is 105mC and maximum. = (1.6 × 10−19 )(300) = 4.8 × 10−17 N
Compute the following quantities at t=2.0 ms:
E
(a) The energy stored in the capacitor. (ii) From the equation, c = 0
B0
(b) The energy stored in the inductor. E
Maximum magnetic field B0 = 0
(c) The total energy in the circuit. c
300
=Or B0 = 10 −6 T
1 2 3.0 × 108
Sol: In LC circuit, the energy stored in inductor is Li
2 2 ∴ Maximum magnetic force FB= B0qv sin 90o=B0qv
and energy stored in capacitor is q .
2C
Substituting the values we have,
Given, L=3.3 H, C=840 ×10 F and qmax=105×10−6C
−12

Maximum magnetic force = (10−6 )(1.6 × 10−19 )(2.0 × 107 )


The circuit when connected to AC supply, oscillated and
= 3.2 × 10−18 N
the angular frequency of oscillations of circuit which is,
Hence total force is F= (4.8 + 0.32) × 10−17 N
1 1
ω
= = = 1.9 × 10 4 rad / s = 5.12 × 10−17 N
LC 3.3 × 840 × 10 −12

Charge stored in the capacitor at any time instant t is


given by,
= q q0 cos ωt
JEE Advanced/Boards
(a) At t= 2 × 10−3 s ; charge in capacitor is Example 1: A wire frame of area 3.92×10−4m and
resistance 20Ω is suspended from a 0.392 m long
(  )
q =105 × 10−6 cos 1.9 × 10 4  2 × 10−3 
 
thread. There is a uniform magnetic field of 0.784 T
−6 and the plane of wire-frame is perpendicular to the
=100.3 × 10 C =100 µC magnetic field. The frame is made to oscillate under
∴ Energy stored in the capacitor is gravity by displacing it through 2×10−2m from its initial
position along the direction of magnetic field. The
( )
2
1 q2 100.3 × 10 −6 plane of the frame is always along the direction of
UC
= = = 5.99 J the thread and does not rotate about it. What is the
2 C 2 × 840 × 10 −12
induced e.m.f. in a wire-frame as a function of time?
(c) Total energy in the circuit Also find the maximum current in the frame.

( )
2
−6
1 q0 105 × 10
2
Sol: As the wire frame oscillates in the magnetic field,
=U = = 6.56 J
2 C 2 × 840 × 10 −12 the angle between the area vector and the magnetic
field continuously varies. Thus, the flux linked with
(b) Energy stored in inductor in the given time the frame changes and e.m.f.and current is induced
=total energy in circuit – energy stored in capacitor in the frame. As the magnetic field is uniform, the net
magnetic force on the frame will be zero.
=6.56-6=0.56 J
The instantaneous flux through the frame when it is
displaced through an angle θ is given by
= Φ BA cos θ
2 2 . 3 2 | Electromagnetic Induction and Electromagnetic Waves

Substituting the values, we get

( 0.784 ) × (3.92 × 10−4 ) × 5 × (5 × 10−2 )


2
 =E sin10 t
 = 4 × 10−6 sin10 t
B
4 × 10−6 V and
⇒ Emax =


sin
Emax 4 × 10−6
g
mg mg cos
m
Imax = = = 2 × 10−7 A
R 20
Instantaneous induced e.m.f. to the coil is
Example 2: A variable magnetic field creates a
dΦ dθ constant e.m.f. E in a conductor ABCDA. The resistance
E=
− BA sin θ
=
dt dt of the portions ABC, CDA and AMC areR1, R2and R3,
sinceθ is very small respectively. What current will be recorded by the meter
M? The magnetic field is concentrated near the axis of

E = BA θ
dt
( sin θ = θ )  … (i) the circular conductor.

(B) Find the equation of motion & its solution


M
The force acting on the coil when it is displaced by A
small angle θ B
2 2
dx d x
m − mgsin θ or
= −gsin θ
= C
2
dt dt2
From Figure 22.43 the displacement of the coil is
x D
θ = ⇒ x = θ

d2 x d2 θ gθ
 = − gθ ⇒ =−
dt 2
dt 2 l Sol: Due to variable magnetic field, e.m.f. and current
are induced in the coil ABCDA.
Putting ω = ( g / l) , we get Let E1 and E2 be the e.m.f.s developed in ABC and CDA,
respectively. Then E1 + E2 =E.
d2 θ
+ ω2 θ = 0  … (ii) There is no net e.m.f. in the loop AMCBA as it does not
dt2
enclose the magnetic field. If E3 is the e.m.f. in AMC
This is the equation of S.H.M. then E1– E3=0. The equivalent circuit and distribution of
current is shown in Figure.
(C) Solve equation (i) to get EmaxandImax
By the loop rule R1 (x-y) + R2 x= E1+E2 =E
Solution of equation (ii) is given by θ = θ0 sin ω t
Substituting the value of θ in equation (i), we get x + -
d
(
E = BA θ0 sin ωt
dt 0
θ sin ωt) ( ) R2 R1
R3

M
= BA θ0 sin ωt ωθ0 cos ωt + +
 E2 E1
E BA ωθ20 sin2ωt
= ...(iii) ... (iii) - x-y - y
x
g  9.8  −1
Here
= ω =  =  5 rads
l  0.392  And R3 y- R1(x-y) =E3-E1 =0
x0 2 × 10−2 ER1
And θ0 = = = 5 × 10−2 rad Solving for y, y= .
l 0.392 R1R 2 + R 2R3 + R3R1
P hysi cs | 22.33

Example 3: A square loop of side ‘a’ and a straight, Sol: If n2 is the number of turns in secondary and φ2 is
infinite conductor are placed in the same plane with the flux linked through one turn, then the flux linked
two sides of the square parallel to the conductor. through the secondary is n2φ2.
The inductance and resistance are equal to L and R
Magnetic field inside any point of solenoid B = µ0 n1 i1
respectively. The frame is turned through 1800 about
where n1 is no turns in primary and i1is current in
the axis OO’. Find the electric charge that flows in the
primary.
square loop.
Flux through secondary having turns n2 is

n2 (BA ) =
O
b a n2 φ2 = µ0 n1n2i1 A
n2 φ2
⇒ M= µ0 n1n2 S
=
i1
4 π × 10−7 × 50 × 200 × 4 × 10−4
= = 5 × 10−4 H.
−2
O’ 10

di Example 5:A rectangular conducting loop in the vertical


Sol: For LR circuit, the total E.M.F. is =
E iR + L . And x-z plane has length L, width W, mass M and resistance
dt
the charge in the coil is q = I dt . ∫ R. It is dropped lengthwise from rest. At t=0 the bottom
of the loop is at a height h above the horizontal x-axis.
There is a uniform magnetic field B perpendicular to
By circuit equation iR =  ε − L di  where
dt  the x-z plane, below the x-axis. The bottom and top

of the loop cross this axis at t=t1 and t2 respectively.
di Obtain the expression for the velocity of the loop for
ε=induced e.m.f. and L = self-induced e.m.f.
dt time t1 ≤ t ≤ t2 .
Integrating above equation w.r.t time we get
Sol: The motional e.m.f. induces in the loop as it moves
di in the magnetic field. The direction of induced current
∫ Ri dt =ε
∫ dt − ∫ L dtdt will be such that the ampere force on the width of the
dφ f loop will be vertically upwards.
⇒ Rq = ∫ − dt dt − L[i]i =
φi − φf
For time t1, the loop is freely falling under gravity, so
(= ifinal 0 )
i initial 0,= velocity attained by loop at t=t1
⇒ q = ( φi − φf ) / R ν1= gt1= 2gh
Consider a strip at a distance x in the initial position. During the time t1 ≤ t ≤ t2 , flux linked with the loop is
µ I changing, so induced e.m.f.
Then B= 0 along the inward normal to the plane.
2πx

E=
− −BvW
=
µ I µ0 Ia dx dt
∴ dφi = 0 a dx cos0 =
2π x 2π x BvW
and induced current I = − clockwise
a +b
µ0 Ia dx µ0 Ia a + b R
2π b∫ x
=
⇒ φi = ln
2π b B2 vW 2
Magnetic force F=WIB = −
R
µ0 Ia 2a + b µ Ia 2a + b
Similarly φf =− ln ∴ φi − φf = 0 ln dv B2 vW 2
2π a+b 2π b So, m = mg −
dt R
µ0 Ia 2a + b
∴ q = ln mdv
2πR b dt = Integrating,
 B2 W 2 v  
 mg − 
 R  
Example 4: A straight solenoid has 50 turns per cm 
in primary and 200 turns in the secondary. The area mR  B2 ν W 2 
t=
− loge mg − +A
of cross-section of the solenoid is 4 cm2. Calculate the B2 W 2  R 
mutual inductance.
2 2 . 3 4 | Electromagnetic Induction and Electromagnetic Waves

At=t t1 , =
v v= gt1 to ampere force should be balanced by the net torque
1
of the external agent which is maintaining constant
 angular velocity of the rod.
mR B2 v 1 W 2 
A=
t1 + loge mg − 
B2 W 2 R
 
y A
Substituting for A,
x

d
 B2 vW 2 

dr

Ro
B2 W2  mg − 
− ( t −t1 ) R
= loge  

r
e mR mg FM
 B v1 W 2
2  
 mg −  O
 R 
Gives the expression for velocity of the loop in the
interval t1 ≤ t ≤ t2 . (a) As the terminals of the switch S are connected
between the points O and C, so the e.m.f. across the
switch is same as across the ends of the rod. Now to
Example 6:A metal rod OA of mass m and length l
calculate the e.m.f. across the rod, consider an element
is kept rotating with a constant angular speedwin a
of the rod of length dr at a distance r from O, then
vertical plane about a horizontal axis at the end O. The
free end A is arranged to slide without friction along dE = Bνdr = Brωdr ( as ν = rω)
a fixed conducting circular ring in the same plane as l
1
that of rotation. A uniform and constant magnetic so E =∫ Bωr dr = Bωl2 ………( i )
2
induction B is applied perpendicular and into the plane 0
And in accordance with Fleming’s right hand rule the
of rotation as shown in Figure. An inductor L and an
direction of current in the will be from A to Oand so O
external resistance R are connected through a switch S
will be at a higher potential (as inside a source of e.m.f.
between the point O and a point C on the ring to from
current flows from lower to higher potential)
an electrical circuit. Neglect the resistance of the ring
the rod. Initially, the switch is open. (b)(i) Treating the ring and rod rotating in the field as a
source of e.m.f. E given by equation (i), the equivalent
(a) What is the induced e.m.f. across the terminals of
circuit (when the switch S is closed) is as shown in
the switch?
Figure.
(b) The switch S is closed at time t=0
A
y
A S
 E
S 
O
R
C L
L C

(i) Obtain an expression for the current as a function


of time Applying Kirchhoff’s loop rule to it, keeping in mind
that current in the circuit is increasing, we get
(ii) In the steady state, obtained the time dependence
of the torque required to maintain the constant angular dI dI 1
E −=
IR − L 0 ;or= dt
speed, given that the rod OA was along the positive dt (E − IR ) L
x-axis at t=0. which on integration with initial condition I=0 at t=0
yields
Sol: As the rod rotates in uniform magnetic field,
motional e.m.f. is induced in it. When the switch is = (
I I0 1 − e− t/ τ with=)
I0
E
R
and= τ
L
R
closed, induced current flows in the coil. The direction
of current will be such that the torque on the rod due So substituting the value of E from Eqn. (i) we have
to ampere force will oppose the motion of the rod. The Bωl2 1 − e−(R /L )t 
= I … (ii)
torque, due to weight of the rod, and the torque due 2R  

P hysi cs | 22.35

Sol: As the rod moves in the magnetic field, motional


As in steady state I is independent of time, i.e., e− t/ τ →
e.m.f. is induced in it. The current in the rod will be
0 ⇒ t → ∞ ,so
such that the ampere force on it will be opposite to the
Bωl2 direction of motion. As the rod moves the resistance
Isteady state =I max = … (iii)
2R  of path increases. So to maintain constant current the
Now as the rod is rotating in a vertical plane so for the motional e.m.f. should also increase. So in turn, the
situation shown in Figure 22.48 it will experience torque velocity of the rod should increase.
in clockwise sense due to its own weight and also due Let F be the instantaneous force acting on the rod
to the magnetic force on it. So the torque on element MN at any instant t when the rod is at a distance x.
dr,=dτ (mg) × r cos θ + FM × r The instantaneous flux φ is given by φ = B × d × x
M The instantaneously induced e.m.f. is given by
i.e. dτ
=
l
( dr ) g × r cos θ + BI dr dφ  dx 
E=
− −Bd 
= 
 M  dt  dt 
× r  as = m dr andF=
M BI dr 
 l  The instantaneous total resistance of the circuit=R+2 λ x
So total torque acting on the rod Current in the circuit is

dx i (R + 2λx )
l 2
M  Mgl l E Bd  dx 
=  ∫ rdr
τ  g cos θ + BI= cos θ + BI = i (R + 2λx)  ⇒
=i dx =
l 0 2 2 ⇒R = (R + 2λx )  dt  dt Bd
dt Bd i (R + 2λx )
But as rod is rotating at constant angular velocity ω, i (R + 2λx )i.e., velocity =
Bd
θ =ωt and from equation (iii) =
I (
Bωl2 / 2R ) i.e., velocity =
Bd
2 4
Mgl B ωl The instantaneous acceleration
So, τ
= cos ωt + … (iv)
2 4R 
d2 x 2iλ  dx 
And hence the rod will rotate at constant angular = a =  
velocity ω if a torque having magnitude equal to that dt2 Bd  dt 
given by equation is applied to it in anticlockwise
=
( 2
)
2iλ  i R + 2λx  2i λ R + 2λx
=  
( )
sense.
Bd  Bd 2 2
 Bd

M
∴ Instantaneous applied force
2i2 λ (R + 2λx )
F ma
= = ×m
B d B2d2
R
FB2d2
From this equation i2 =
x 2mλ (R + 2λx )

FB2d2
N Heat produced per second = i2 (R + 2λx ) =
2mλ
i (R + 2λx )
Example 7: Two long parallel horizontal rails at Power W= F. v= F ×
Bd
distance d apart and each having a resistance λ per
2 2
HeatR.product
unit length, are joined at one end by a resistance A H FBHeatd productBd H FB2d2 Bd
Therefore,
= = × = = ×
perfectly conducting rod MN of mass m is free to work
slidedone W 2mwork (
λ FidoneR + 2 λ x )W 2m λ Fi ( + 2λx )
R
on rails without friction. There is a uniform magnetic
B3d3 B3d3
field of induction B normal to the plane of the paper = =
2mλ (R + 2λx ) 2mλ (R + 2λx )
and direct into the paper. A variable force F is applied
to the rod MN such that, as the rod moves, a constant
current flows through R.
Example 8: A metal rod of mass m can rotate about
(a) Find the velocity of the rod and the applied force F a horizontal axis O, sliding along a circular conductor
as function of the distance x of the rod from R. of radius a. The arrangement is located in a horizontal
(b) What fraction of the work done per second by F is and uniform magnetic field of induction B directed
converted into heat? perpendicular to the ring plane. The axis and the ring
2 2 . 3 6 | Electromagnetic Induction and Electromagnetic Waves

are connected to an e.m.f. source to form a circuit of between the rails d is large compared with x, the distance
resistance R. Deduce the relation according to which between the wires. Both wires and rails are made of a
the source e.m.f. must vary to make the rod rotate material of resistivity ρ per unit length. A magnetic flux
with a constant angular velocityω. Neglect the friction, of density B applied perpendicular to the rectangle
circuit inductance and ring resistance. made by the wires and rails. One wire is moved along
the rails with a uniform speed v while the other is held
E
stationary. Determine how the force on the stationary
wire varies with x and show that it vanishes for a value
µ0 v
of x approximately equal to .
t 4 πρ

x Rail 1
O mg
x dx
B
I v
d
Sol: As current flows in the rod due to the source e.m.f., B
it experiences torque due to ampere forces and starts Rail 2
rotating. The torque due to weight of the rod should
balance the torque due to ampere force to maintain
Sol: Due to motional e.m.f. current will be induced in
constant angular velocity. As torque due to weight of
rectangular loop. The stationary wire will be attracted
the rod varies with angular position the torque due to
by the moving wire, as well as it will experience a force
ampere force should also vary. So in turn, the current
due to the uniform magnetic field.
and thus source e.m.f. should also vary.
Let at any instant t, during the motion of second wire,
Inductance e.m.f. across the ends of the rod
a the second wire is at a distance x. The area of the
1
E =∫ dE =∫ Bωx dx = B ωa2 rectangle between the two wires is xd. Rate of change
0
2 of magnetic flux through the rectangle
Force on the rod if a current I flow through it:
dφ d dx
F= IaB =
dt dt
(=
B x d) B=
d
dt
Bvd

If the angular velocity is constant so that torque about


∴ Induced e.m.f.
O must vanish. Hence

a 1 dφ
mg sin ωt = I a2 B e=
− − Bvd
=
2 2 dt
So, the current induced in the rectangle I is given by
∴ Current required through the rod
E Bvd
mgsin ωt I= = −
I= R 2 (d + x ) ρ
aB
The force between the two wires due to current flow
This must be equal be to the current due to total e.m.f.
in the circuit µ0i1i2 µ0 2I2d
F
= .d
= ×
1 2πx 4π x
E − Bωa2 2
2 mg sin ωt
=I = ; µ  2d   Bvd 
R aB = 0   
4π  x   2 ( d + x ) ρ 
 
=∴E
1
2Ba
(
2 mgR sin ωt + B2 ωa2 ) The force F’, due to magnetic field on the stationary
wires
Example 9: Two long wires are placed on a pair of  Bvd  B2d2 v
parallel rails perpendicular to the wires. The spacing F' B=
= I d Bd  = 
 2 ( d + x ) ρ  2 ( d + x ) ρ
P hysi cs | 22.37

The former force on stationary wire will be directed


dP Li2  1 2 
towards left hand side because opposite currents repel = 0 ⇒ 0  − e− t/ τ + e−2t/ τ  =
0
dt τ  τ τ 
each other while the force due to magnetic field will be
directed toward right hand according to Fleming’s left 1
⇒ − e− t/ τ =
hand rule. 2

∴ Fresultant =
F'− F Putting in (i)
2 Li20  1 1  LE2 E2
B2d2 v µ  2d   Bdv  Pmax
=  − =  =
= − 0    τ  2 4  4R (L / R ) 4R
2
2 ( d + x ) ρ 4 π  x   2 ( d + x ) ρ 

B2d2 v  µ0 dv 
Example 11:A parallel-plate capacitor having plate area
= 1 − 
2 ( d + x ) ρ  4 π ( d + x ) ρ  A and plate separation d is joined to a battery of emf
V and internal resistance 2R, at t=0. Consider a plane
The force will be zero, when surface of area A, parallel to the plates and situated
µ0 dv µ0 v symmetrically between them. Find the displacement
= 1 or x = current through this surface as a function of time. [The
4π ( d + x ) ρ 4 πρ
charge on the capacitor at time t is given by q=CV
(Neglecting x in comparison with d). (1 − e− t / τ ) .where τ =CR]
d ΦE
Sol: id = ε0 is the displacement current, ϕEis
Example 10: An inductance L and a resistance R are dt
connected in series with a battery of e.m.f. . Find the the flux of the electric field between the plates of the
maximum rate at which the energy is stored in the capacitor.
magnetic field.
Given, q=CV (1 − e− t / τ )
Sol: Substitute the expression for instantaneous current q CV
∴ Surface charge density σ= = (1 − e−t/ τ )
in the LR series circuit in the formula for energy stored A A
in the inductor. Electric field between the plates of capacitor,
I σ CV
E
= = (1 − e− t / τ )
ε0 ε0 A

R +O Electric flux from the given area,


-C E
CV
L φE= EA= (1 − e− t / τ )
ε0
dφE
Displacement current, id = ε0
dt
The energy in the magnetic field at time t is, d  CV  CV −t/ τ
ε0  (1 − e−t/ τ ) =
Or, id = e
1 2 1 2
( )
2
=U = Li Li 1 − e−t/ τ dt  ε0  τ
2 2 0
The rate at which the energy is stored is Substituting,
= τ CR
= ′ where R ′ 2R

V − t/2CR
dU
P= =
dt
Li20 1 − e− t/ τ( )( −e )  − 1τ 
− t/ τ We have, id =
2R
e

Li20 ε0 A
=
τ
(e − t/ τ
)
− e−2t/ τ ... ( i ) Again substituting, C =
d
td
This rate will be maximum when V 2ε0 AR
id = e
2R
2 2 . 3 8 | Electromagnetic Induction and Electromagnetic Waves

JEE Main/Boards

Exercise 1 Q.9 A square copper coil of each side 8 cm consists of


100 turns. The coil is initially in vertically plane, such
Q.1 Can a person sitting in a moving train measure the that the plane of coil is normal to the uniform magnetic
potential difference between the ends of the axle by a field of induction 0.4 weber m-2. The coil is turned
sensitive voltmeter? through 1800 about a horizontal axis in 0.2s. Find the
induced e.m.f.

Q.2 A coil of mean area 500 cm2 and having 1000 turns
is held perpendicular to a uniform field of 4 × 10−4 T. Q.10 A 5 H inductor carries a steady current of 2 A. How
can a 50 V self-induced e.m.f. be made to appear in the
1
The coil is turned through 1800 in s. Calculate the inductor?
average induced e.m.f.. 10

Q.11 A conducting wire of 100 turns is wound over 1


Q.3 The self-inductance of an inductance coil having cm near the center of a solenoid of 100 dm length and
100 turns is 20 mH. Calculate the magnetic flux through 2 cm radius having 1000 turns. Calculate coefficient of
the cross-section of the coil corresponding to a current mutual inductance of the two solenoids.
of 4 mA. Also find the total flux.
Q.12 If the self-inductance of an air core inductor
Q.4 A rectangular loop of wire is being withdrawn out increases from 0.01 mH to 10 mH on introducing an
of the magnetic field with velocity v. The magnetic field iron core to it, what is the relative permeability of the
is perpendicular to the plane of paper. What will be the core used?
direction of induced current, in the loop?
Q.13 State Lenz’s law. The P Q
Q.5 A solenoidal coil has 50 turns per centimeter along closed loop PQRS is
its length and cross sectional area of 4 × 10−4 m2. 200 moving into uniform
turns of another wire is wound round the first solenoid magnetic field acting at
coaxially. The two coils are electrically insulted from right angle to the plane of
each other. Calculate the mutual inductance between the paper as shown in the
the two coils. Figure. State the direction
S R
in which the induced
Q.6 Calculate the mutual inductance between two coils, current flows in the loop.
when a current of 4.0 A changes to 8.0 A in 0.5 second
and induces an e.m.f. of 50 m V in the secondary coil. Q.14 A solenoid with an iron core and a bulb are
connected to a D.C. source. How does the brightness of
Q.7 In a car spark coil, an e.m.f. of 40,000 V is induced the bulb change, when the iron core is removed from
in the secondary coil when the primary coil current the solenoid?
changes from 4 A to 0 A in 10 µ s. Calculate the mutual
inductance between the primary secondary windings Q.15 What is induced e.m.f.? Write faraday’s law of
of this spark coil. electromagnetic induction. Express it mathematically.
A conducting rod of length ‘l’, with one end pivoted,
Q.8 A current of 10 A is flowing in a long straight wire is rotated with a uniform angular speed ' ω' in a vertical
situated near a rectangular coil. The two sides, of the plane, normal to a uniform magnetic field ‘B’. Deduce
coil, of length 0.2 m are parallel to the wire. One of an expression for the e.m.f. induced in this rod.
them is at a distance of 0.05m and the other is at a
distance of 0.10 m from the wire. The wire is in the plane
Q.16 A circular coil of radius 8 cm and 20 turns rotates
of the coil. Calculate the magnetic flux through the
about its vertical diameter with an angular speed of 50 s-1
rectangular coil. If the current decays uniformly to zero
in a uniform horizontal magnetic field of magnitude
in 0.02s, find the e.m.f. induced in the coil and indicate
3 × 10 −2 T. Find the maximum and average value of the
the direction in which the induced current flows.
e.m.f. induced in the coil.
P hysi cs | 22.39

Q.17 Define self-inductance and give its S.I. unit. Derive P R


an expression for self-inductance of a long, air-cored
solenoid of length l, radius r, and having N number of
 v
turns.

S
Q.18 A 0.5 m long metal rod PQ completes the
circuit as shown in the Figure. The area of the circuit Deduce an expression for
is perpendicular to the magnitude field of flux density
0.15 T. If the resistance of the total circuit is 3 Ω, calculate (i) The e.m.f. induced across the arm ‘RS’,
the force needed to move the rode in the direction as (ii) The external force required to move the arm, and
indicated with a constant speed of 2 ms-1.
(iii) The power dissipated as heat.
Q
Q.23 Define self-inductance of a coil. Write its S.I. units.

P Q 24 The identical loops, one of copper and the other


of aluminum, are rotated with the same angular speed
Q.19 What are eddy currents? How are these produced? in the same magnetic field. Compare (i) the induced
In what sense are eddy currents considered undesirable e.m.f. and (ii) the current produced in the two coils.
in a transformer and how are these reduced in such a Justify your answer.
device?
Q.25 Two bar magnets are quickly moved towards a
Q.20 The circuit arrangement given below shows that metallic loop connected across a capacitor ‘C’ as shown
when an a.c. passes through the coil A, the current in the Figure. Predict the polarity of the capacitor.
starts flowing in the coil B.
C

S N S N

Q.26 A bar magnetic is moved in the direction indicated


by the arrow between two coils PQ and CD. Predict the
directions of induced current in each coil.
Coil A Coil B
P Q C D
(i) State the underlying principle involved. N S
 
(ii) Mention two factors on which the current produced A A
in the coil B depends.

Q21 (i) State faraday’s law of electromagnetic induction.


Exercise 2
(ii) A jet plane is travelling towards west at a speed of Single Correct Choice Type
1800 km/h. what is the voltage difference developed
between the ends of the wing having a span of 25m, Q.1. A conducting loop of radius R is present in a
if the earth’s magnetic field at the location has a uniform magnetic field B perpendicular to the plane of
magnitude of 5 × 10 −4 T and the dip angle is 300? the ring. If radius R varies as a function of time‘t’, as
R=R0+t. The e.m.f. induced in the loop is
Q.22 (a) Write the two applications of eddy currents. (b)
Figure 22.57 shows a rectangular conducting lop PQSR
in which arm RS of length '  ' is movable. The loop is
kept in a uniform magnetic field ‘B’ directed downward R
B
perpendicular to the plane of the loop. The arm RS is
moved with a uniform speed ‘v’.
2 2 . 4 0 | Electromagnetic Induction and Electromagnetic Waves

(A) 2π(R0+t)B clockwise (B) π(R0+t)B clockwise (C) The current in each will remain the same
(C) 2π(R0+t)B anticlockwise (D) zero (D) The current in one will increase and in other will
decrease
Q.2 A square wire loop of 10.0 cm side lies at right
angle to a uniform magnetic field of 20T. A 10V light Q.6 The Figure shows an
bulb is in a series with the loop as shown in the Figure. isosceles triangle wire frame with B
The magnetic field is decreasing steadily to zero over apex angle equal to π / 2 . The o
frame starts entering into the 90
a time interval ∆t . The bulb will shine full brightness if v
∆t is equal to uniform magnetic field B with
Constant velocity v at t=0. The
t=0
longest side of the frame is
perpendicular to the direction of velocity. If i is the
instantaneous current through the frame then choose
the alternative showing the correct variation of i with
time.

i i
(A) (B)
(A) 20 ms (B) 0.02 ms t t
(C) 2 ms (D) 0.2 ms
i i
(C) (D)
Q.3 The dimensions of permeability of free space can
t t
be given by

(A) MLT −2 A −2  (B) MLA −2  Q.7 A thin wire of length 2 m is perpendicular to the xy
    
( )
plane. It is moved with velocity v = 2iˆ + 3ˆj + kˆ m / s
(C) ML−3 T2 A2 
 
(D) MLA −1 
  through a region of magnetic induction B= (ˆi + 2ˆj)
Wb/ m2. Then potential difference induced between the
Q.4 A vertical magnet is dropped from position on the ends of the wire:
axis of a fixed metallic coil as shown in Figure, figure (i).
(A) 2 V (B) 4 V
In figure (ii) the magnet is fixed and horizontal coil is
dropped. The acceleration of the magnet and coil are (C) 0 V (D) none of these
a1 and a2 respectively then
Q.8 A long metal bar of 30 cm length is aligned along a
S north south line and moves eastward at a speed of 10
ms-1. A uniform magnetic field of 4.0 T points vertically
S
downwards. If the south end of the bar has a potential
N of 0 V, the induced potential at the end of the bar is
Fixed Fixed N (A) +12 V

(i) (ii) (B) -12 V


(C) 0 V
(A) a1 > g,a2 > g (B) a1 > g,a2 < g
(D) Cannot be determined since there is not closed
(C) a1 < g,a2 < g (D) a1 < g,a2 > g
circuit

Q.5 Two identical coaxial circular loops carry a current


Q.9 A conducting rod moves with constant velocity
I each circulating in the same direction. If the loops
v perpendicular to the long, straight wire carrying a
approach each other
current I as shown compute that the e.m.f. generated
(A) The current in each will decrease between the ends of the rod.
(B) The current in each will increase
P hysi cs | 22.41

Q.13 When a ’J’ shaped conducting rod is rotating in


r its own plane with constant angular velocity ω , about
I one of its end P, in a uniform magnetic field B (directed
normally into the plane of paper) then magnitude of
l 
e.m.f. induced across it will be

l L
µ0 ν Il µ0 ν Il
(A) (B) Q
πr 2πr
2µ0 ν Il µ0 ν Il
(C) (D) P 
πr 4 πr

Q.10There is a uniform field B normal to the xy plane. A 1


2 2
(A) Bω L + l (B) BωL2
conductor ABC has length AB=l1, parallel to the x-axis, 2
and length BC=l2, parallel to the y-axis. ABC moves
(C)
1
2
( )
Bω L2 + l2
1
(D) Bω l2
2
in the xy plane with velocity v i + v j . The potential x y
difference between A and C is proportional to
Q.14 A metal disc rotates freely, between the poles of
(A) VXl1 + Vy l2 (B) VXl2 + Vy l1 a magnet in the direction indicated. Brushes P and Q
make contact with the edge of the disc and the metal
(C) VXl2 − Vy l1 (D) VXl1 − Vy l2 axle. What current, if any, flows through R?

N
Q.11 A conducting rod PQ of length 5 m oriented as
shown in Figure is moving with velocity 2 î m/s without

( )
R
any rotation in a uniform magnetic field 3ˆj + 4kˆ T. e.m.f. P 
Q
induced in the rod is
y S
Q
2m/s
5m (A) A current from P to Q

53
o (B) A current from Q to P
x (C) No current, because the e.m.f. induced in one side
P
of the disc is opposed by the back e.m.f.
(A) 32 V (B) 40 V (c) 50 V (D) none (D) No current, because the e.m.f. induced in one side
of disc is opposed by the e.m.f. induced in the other
Q.12 The magnetic field in a region is given by side
x ˆ (E) No current, because no radial e.m.f. is induced in
B B0 1 +
= k . A square loop of edge length d is
a the disc
placed with its edge along x & y axis. The loop is moved

with constant velocity V = V0 ˆi . The e.m.f. induced in the Q.15 A rectangular coil of single turn, having area A,
loop is rotates in a uniform magnetic field B with an angular
velocity w about an axis perpendicular to the field. If
V0 B0 d2 V0 B0 d2 initially the plane of coil is perpendicular to the field,
(A) (B)
a 2a then the average induced e.m.f. when it has rotated
V0 B0 a2 through 900 is
(C) (D) none
d
2 2 . 4 2 | Electromagnetic Induction and Electromagnetic Waves

j
C L
A B
A
B

i
(A) Remains stationary
ωBA ωBA ωBA 2ωBA
(A) (B) (C) (D)
π 2π 4π π (B) Is attracted by the loop-A
(C) Is repelled by the loop-A
Q.16 A copper rod AB of length L, pivoted at one end
(D) Rotates about its CM, with CM fixed
A, rotates at constant angular velocity ω , at right angle
to a uniform magnetic field of induction B. The e.m.f.
developed between the midpoint C to of the rod and Q.21 A circular loop of radius R, carrying current I, lies
end B is in x-y plane with its center at origin. The total magnetic
flux through x-y plane is
Bωl2 Bωl2 3Bω l2 3Bω l2
(A) (B) (C) (D)
4 2 4 8 (A) Directly proportional to I
(B) Directly proportional to R
Q. 17 Figure 22.70shows a uniform magnetic field
(C)Directly proportional to R2
B confined to a cylindrical volume and is increasing
at a constant rate. The instantaneous acceleration (D) Zero
experienced by an electron placed at P is
Q.22 Two circular coils can be arranged in any of
the three situations in the Figure 22.72. Their mutual
P inductance will be

(a) (b) (c)


(A) Zero (B) Towards right
(C) Towards left (D) Upwards (A) Maximum in situation (a)
(B) Maximum in situation (b)
Q.18 A small coil of radius r is placed at the center
(C) Maximum in situation (c)
of a large coil of radius R, where R>>r. The coils are
coplanar. The coefficient of mutual inductance between (D) The same in all situations
the coils is
µ0 πr µ0 πr 2 µ0 πr 2 µ0 πr
(A) (B) (C) 2
(D) Previous Years’ Questions
2R 2R 2R 2R 2
Q.1 A conducting square loop of side L and resistance
Q.19 A long straight wire is placed along the axis of R moves in its plane with a uniform velocity ν
circular ring of radius R. The mutual inductance of this perpendicular to one of its sides. A magnetic induction
system is B, constant in time and space, pointing perpendicular
µ0R µ0 πR µ0 to and into the plane of the loop exists everywhere. The
(A) (B) (C) (D) 0
2 2 2 current induced in the loop is  (1989)

Q.20 Two identical circular loops of metal wire are lying


on a table without touching each other. Loop-A carries v
a current which increases with time. In response, the
loop-B
P hysi cs | 22.43

(A) BLv/R clockwise (B) BLv/R anticlockwise the circular region (2000)
(C) 2BLv/R anticlockwise (D) Zero B(t)
P
Q.2 A thin semicircular conducting ring of radius R is r
falling with
 its plane vertical in a horizontal magnetic
induction B. At the position MNQ the speed of the ring
is ν and the potential difference developed across the
ring is (1996) a


B
(A) is zero (B) decreases as 1/r
N (C) increases as r (D) decreases as 1/r2

Q.7 A coil of wire having finite inductance and


M Q
resistance has a conducting ring placed co-axially
(A) Zero within it. The coil is connected to a battery at time t=0,
so that a time dependent current I1 (t) starts flowing
(B) BνπR 2 / 2 and M is at higher potential
through the coil. I2 (t) is the current induced in the
(C) πBRν and Q is at higher potential ring and B (t) is the magnetic field at the axis of the
coil due to I1 (t) then as a function of time (t>0), the
(D) 2RBν and Q is at higher potential
product I2 (t) B(t) (2000)

Q.3 A metal rod moves at a constant velocity in a direction (A) Increases with time
perpendicular to its length. A constant magnetic field (B) Decreases with time
exist in space in a direction perpendicular to the rod as
(C) Does not vary with time
well as its velocity. Select the correct statement (s) from
the following.  (1998) (D) Passes through a maximum
(A) The entire rod is at the same electric potential
Q.8 A metallic square loop ABCD is moving in its
(B) There is an electric field in the rod
own plane with velocity ν in a uniform magnetic field
(C) The electric potential is higher at the center of the perpendicular to its plane as shown in the Figure 22.84.
rod and decrease towards its ends Electrical field is induced  (2001)
(D) The electric potential is lowest at the center of the
rod and increase towards its ends

Q.4 A small square loop of wire of side l is placed A B


inside a large square of wire of side L (L>>l). The loops
are coplanar and their centers coincide. The mutual
inductance of the system is proportional to  (1998) v

(A) l / L (B) l2 / L (C) L / l (D) L2 / l

Q.5 A coil of inductance 8.4 mH and resistance 6 Ω is


connected to a 12 Ω battery. The current in the coil is
1A at approximately the time (1999)
(A) 500s (B) 20s (C) 35 ms (D) 1 ms (A) In AD, but not in BC
(B) In BC, but not in AD
Q.6 A uniform but time-varying magnetic field B (t)
(C) Neither in AD nor in BC
exists in a circular region a and is directed into the plane
of the paper as shown. The magnitude of the induced (D) In both AD and BC
electric field at point P at a distance r from the center of
2 2 . 4 4 | Electromagnetic Induction and Electromagnetic Waves

Q.9 As shown in the Figure, P and Q are two coaxial Q.13 In an LCR circuit as shown below both switches
conducting loops separate by some distance. When are open initially. Now switch S1 is closed, S2 kept open.
the switch S is closed, a clockwise current IP flows in (q is charge on the capacitor and τ =RC is capacitive
P (as seen by E) and an induced current IQ1 flows in Q. time constant). Which of the following statement is
The switch remains closed for a long time. When S is correct? (2013)
opened, a current IQ2 flows in Q. Then the direction IQ1 V
and IQ2 (as seen by E) are (2002)
P
Q R
S1
E
C
S
S2
L
Battery (A) At t=τ, q=CV / 2
(B) At t=2τ, q=CV(1−e−2)
(A) Respectively clockwise and anticlockwise
τ
(B) Both clockwise Attt= =
(C) = , q=CV
q CV(1−e e−) 1 )
(1 − −1
2
(C) Both anticlockwise
(D) Work done by the battery is half of the energy
(D) Respectively anticlockwise and clockwise
dissipated in the resistor.

Q.10 A short-circuited coil is placed in a time varying


Q.14 A circular loop of radius 0.3 cm lies parallel to a
magnetic field. Electric power is dissipated due to the
much bigger circular loop of radius 20 cm. The centre
current induced in the coil. If the number of turns were
of the small loop is on the axis of the bigger loop. The
to be quadrupled (four time) and the wire radius halved,
distance between their centres is 15 cm. If a current
the electrical power dissipated would be (2002)
of 2.0 A flows through the smaller loop, then the flux
(A) Halved (B) The same linked with bigger loop is (2013)
(C) Doubled (D) Quadrupled
(A) 6 × 10−11 weber (B) 3.3 × 10−11 weber
(C) 6.6 × 10−9 weber (D) 9.1 × 10−11 weber
Q.11 An electromagnetic wave in vacuum has the
 
electric and magnetic fields E and B , which are
Q.15 The magnetic field in a travelling electromagnetic
always perpendicular to each other. The direction of
 wave has a peak value of 20 nT. The peak value of
polarization is given by X and that of wave propagation electric field strength is : (2013)

by k . Then : (2012)
          (A) 6 V/m (B) 9 V/m (C) 12 V/m (D) 3 V/m
(A) X ||B and k || B × E (B) X ||E and k || E × B
         
(C) X ||B and k || E × B (D) X ||E and k || B × E Q.16 Match List-I (Electromagnetic wave type) with List-
II (Its association / application) and select the correct
Q.12 A coil is suspended in a uniform magnetic field, option from the choices given below the lists: (2014)
with the plane of the coil parallel to the magnetic lines
of force. When a current is passed through the coil it
List - I List - II
starts oscillating; it is very difficult to stop. But if an
aluminium plate is placed near to the coil, it stops. This (a) Infrared (i) To treat muscular strain
is due to : (2012) waves
(A) development of air current when the plate is placed. (b) Radio waves (ii) For broadcasting
(B) induction of electrical charge on the plate (c) X-rays (iii) To detect fracture of
bones
(C) shielding of magnetic lines of force as aluminium is
a paramagnetic material. (d) Ultraviolet (iv) Absorbed by the ozone
rays layer of the atmosphere
(D) electromagnetic induction in the aluminium plate
giving rise to electromagnetic damping.
P hysi cs | 22.45

(A) (a) → (iii), (b) → (ii), (c) → (i), (d) → (iv) R L

(B) (a) → (i), (b) → (ii), (c) → (iii), (d) → (iv)


(C) (a) → (iv), (b) → (iii), (c) → (ii), (d) → (i)
C
(D) (a) → (i), (b) → (ii), (c) → (iv), (d) → (iii)

Q.17 During the propagation of electromagnetic waves


in a medium: (2014)
2 2
Q Max Q Max

L2 L2
(A) Electric energy density is equal to the magnetic (A) (B)
L1 L1
energy density.
t t
(B) Both electric and magnetic energy densities are
zero.
(C) Electric energy density is double of the magnetic
2 2
Q Max QMax
energy density. (C) (D) L1
Q0 (For both L1 and L0)
(D) Electric energy density is half of the magnetic L2
energy density. t t

Q.18 In the circuit shown here, the point ‘C’ is kept Q.21 Two long current carrying
connected to point ‘A’ till the current flowing through thin wires, both with current I, are L
the circuit becomes constant. Afterward, suddenly, held by insulating threads of 
point ‘C’ is disconnected from point ‘A’ and connected length L and are in equilibrium as
to point ‘B’ at time t=0. Ratio of the voltage across shown in the figure, with threads I I
resistance and the inductor at t=L/R will be equal to : making an angle ' θ ' with the
 (2014) vertical. If wires have mass λ per unit length then the
A R value of I is: (g=gravitational acceleration) (2015)
C
B πλgL πgL
L (A) 2sin θ (B) 2 tan θ
µ0 cos θ µ0

πλgL πλgL
1−e e (C) tan θ (D) sin θ
(A) -1 (B) (C) (D) 1 µ0 µ0 cos θ
e 1−e

Q.19 An inductor (L=0.03 H) and a resistor (R=0.15 kΩ) Q.22 Two identical wires A and B, each of length '  ' ,
are connected in series to a battery of 15 V EMF in a carry the same current I. Wire A is bent into a circle of
circuit shown. The key K1 has been kept closed for a radius R and wire B is bent to form a square of side ‘a’. If
long time. Then at t=0, K1 is opened and key K2 is closed BA and BB are the values of magnetic field at the centres
simultaneously. At t=1 ms, the current in the circuit will BA
of the circle and square respectively, then the ratio
be (e5 ≅ 150)  (2015) BB
(A) 67 mA (B) 6.7 mA is:  (2016)
π2 π2 2 π2
(C) 0.67 mA (D) 100 mA (A) (B) (C) π (D)
16 2 16 8 2 8

Q.20 An LCR circuit is equivalent to a damped pendulum.


Q.23 Arrange the following electromagnetic radiations
In an LCR circuit the capacitor is charged to Q0 and then
per quantum in the order of increasing energy :(2016)
connected to the L and R as shown. If a student plots
2
graphs of the square of maximum charge (QMax ) on the (1) : Blue light (2) : Yellow light
capacitor with time (t) for two different values L1 and L2
(3) : X-ray (4) : Radiowave
(L1 > L2) of L then which of the following represents this
graph correctly? (Plots are schematic and not drawn to (A) (1), (2), (4), (3) (B) (3), (1), (2), (4)
scale) (2015) (C) (2), (1), (4), (3) (D) (4), (2), (1), (3)
2 2 . 4 6 | Electromagnetic Induction and Electromagnetic Waves

Q.24 Hysteresis loops for two magnetic materials A and These materials are used to make magnets for electric
B are given below : generators, transformer core and electromagnet core.
B B Then it is proper to use: (2016)
(A) A for electromagnets and B for electric generators
(B) A for transformers and B for electric generators
H (C) B for electromagnets and transformers
(D) A for electric generators and transformers

(A) (B)

JEE Advanced/Boards

Exercise 1 Q.4 A rectangular loop with a sliding connector of


length l=1.0 m is situated in a uniform magnetic field
B=2T perpendicular to the plane of loop. Resistance of
Q.1 The horizontal component of the earth’s magnetic
connector is r= 2 Ω . Two resistances of 6 Ω and 3 Ω are
field at a place is 3 × 10 −4 T and the dip is tan-1 (4/3). A
connected as shown in Figure. Find the external force
metal rod of length 0.25 m placed in the north- south
required to keep the connector moving with a constant
position is moved at a constant speed of 10 cm/s
velocity V=2m/s.
towards the east. Find the e.m.f. induced in the rod. 
+B
Q.2 A wire forming one cycle sine curve is moved in x-y 6 3

V Vx ˆi + Vy ˆj.  There exist a magnetic
plane with velocity =

field is B = − B0 kˆ . Find the motional e.m.f. develop
Q.5 Two concentric and coplanar circular coils have
across the ends PQ of wire.
radii a and b(>>a) as shown in Figure. Resistance of
y the inner coil is R. Current in the outer coil is increased
 from 0 to i, then find the total charge circulating the
B
inner coil.
P O
x
b
a

Q.3 A conducting circular loop is placed in a uniform


Q.6 A horizontal wire is free to slide on the vertical rails
magnetic field of 0.02 T, with its plane perpendicular to
of a conducting frame as shown in Figure. The wire has
the field. If the radius of the loop starts shrinking at a
a mass m and length l and the resistance of the circuit is
constant rate of 1.0 mm/s, then find the e.m.f. induced
R. If a uniform magnetic field B is directed perpendicular
in the loop, at the instant when the radius is 4 cm.
to the frame, then find the terminal speed of the wire as
it falls under the force of gravity.
C l

m

B
A B
R
P hysi cs | 22.47

Q.7 A metal rod of resistance 20Ω is fixed along a with the rails starts at vertex at the time t=0 & moves
diameter of a conducting ring of radius 0.1 m and lies symmetrically with a constant velocity of 5.2 m/s to the
on x-y plane. There is a magnetic field B= (50T) K̂ . The right as shown in Figure. A 0.35 T magnetic field points
ring rotates with an angular velocity ω=20 rad/s about out of the page. Calculate:
its axis. An external resistance of 10 Ω is connected
across the center of the ring and rim. Find the current B
through external resistance.
5.2m/s
o
90
Q.8 A triangular wire frame (each side=2m) is placed in
a region of time variant magnetic field
Having dB/dt= 3 T/s. The magnetic field is
perpendicular to the plane of the triangle. The base of
the triangle AB has a resistance 1 Ω while the other (i) The flux through the triangle by the rails & bar at
two sides have resistance 2 Ω each. The magnitude of t=3.0s
potential difference between the points A and B will be.
(ii) The e.m.f. around the triangle at that time.

Q.9 A uniform magnetic field of 0.08 T is directed into (iii) In what manner does the e.m.f. around the triangle
the plane of the page and perpendicular to it as shown vary with time?
in the Figure. A wire loop in the plane of the page has
constant area 0.010m2. The magnitude of magnetic Q.14A wire is bent into 3 circular segments of radius
field decrease at a constant rate 3 × 10 −4 Ts-1. Find the r=10cm as shown in Figure. Each segment is a quadrant
magnitude and direction of the induced e.m.f. in the of a circle, ab lying in the xy plane, bclying in the yz
loop. plane & ca lying in the zx plane.
z

c
B

r r
b
Q.10 There exists a uniform cylindrically symmetric y
magnetic field directed along the axis of a cylinder but
varying with time as B=kt. If an electron is released from
rest in this filed at a distance ‘r’ from the axis of cylinder, a
its acceleration, just after it is released would be (e and x
m are the electronic charge and mass respectively) (i) If a magnetic field B points in the positive x direction,
what is the magnitude of the e.m.f. developed in the
Q.11 A uniform but time varying magnetic field B=Kt–C; wire, when B increases at the rate of 3 mT/s?
(0 ≤ t ≤ C/K), where K and C are constants and t is time, (ii) What is the direction of the current in the segment
is applied perpendicular to the plane of the circular bc.
loop of radius ’a’ and resistance R. Find the total charge
that will pass around the loop.
Q.15 Consider the possibility of a new design for an
electric train. The engine is driven by the force due to
Q.12 A charged ring of mass m=50gm, charge 2 the vertical component of the earth’s magnetic field on
coulomb and radius R=2m is placed on a smooth a conducting axle. Current is passed down one coil, into
horizontal surface. A magnetic field varying with at a a conducting wheel through the axle, through another
rate of (0.2t) T/ s is applied on to the ring in a direction conducting wheel & then back to the source via the
normal to the surface of ring. Find the angular speed other rail.
attained in a time t1=10 s.
(i) What current is needed to provide a modest 10-KN
force? Take the vertical component of the earth’s field
Q.13 Two straight conducting rails form a right angle
be 10 µ T & the length of axle to be 3.0 m.
where their ends are joined. A conducting bar contact
2 2 . 4 8 | Electromagnetic Induction and Electromagnetic Waves

(ii) How much power would be lost for each Ω of Q.19 A rectangular frame ABCD made of a uniform
resistivity in the rails? metal wire has a straight connection between E & F
made of the same wire as shown in the figure. AEFD is
(iii) Is such a train realistic?
a square of side 1m & EB= FC=0.5 m. The entire circuit
is placed in a steadily increasing uniform magnetic field
Q.16 A rectangular loop of dimensions l& w and directed into the place of the paper & normal to it. The
resistance R moves with constant velocity V to the right rate change of the magnetic field is 1T/s, the resistance
as shown in the Figure. It continues to move with same per unit length of the wire is 1 Ω/m. Find the current in
speed through a region containing a uniform magnetic segments AE, BE &EF.
field B directed into the plane of the paper &extending
A E B
a distance 3 W. sketch the flux, induced e.m.f. & external
force acting on the as a function of the distance.

3w B

v B D F C

S Q.20 A pair o parallel horizontal conducting rails of


negligible resistance shorted at one end is fixed on a
table. The distance between the rails is L. A conducting
w massless rod of resistance R can slide on the rails
frictionally. The rod is tied to a massless string which
passes over a pulley fixed to the edge of the table. A mass
Q.17 A long straight wire is arranged along the m, tied to the other end of the string hangs vertically.
symmetry a toroidal coil of rectangular cross-section, A constant magnetic field B exists perpendicular to the
whose dimensions are gives in the Figure. The number table. If the system is released from rest, calculate:
of turns on the coil is N, and relative permeability of the
surrounding medium is unity. Find the amplitude of the
e.m.f. induced in this coil, if the current i=im cos ω t flows
along the straight wire.
b

h
m

(i) The terminal velocity achieved by the rod.


(ii) The acceleration of the mass at the instant when the
Q.18 A uniform magnetic field B fills a cylindrical volume velocity of the rod is half the terminal velocity.
radius R. A metal rod CD of length l is placed inside
the cylinder along a chord of circular cross-section as Q.21 A magnetic field B=(B0y/a) K̂ is into the plane
shown in the Figure. If the magnitude of magnetic field of paper in the +z direction. B0 and a are positive
increases in the direction of field at a constant rate constants. A square loop EFGH of side a, mass m
dB/dt, find the magnitude and direction of the E.M.F. and resistance R, in x-y plane, starts falling under the
induced in the rod. influence of gravity. Note the directions of x and y axes
in the Figure. Find

 O x
B
E F
b g
G H
C D
y
P hysi cs | 22.49

(i) The induced current in the loop and indicated its µ0 er 2


direction, (C) α (D) none of these
4 πR
(ii) The total Lorentz force acting on the loop and
indicated its direction, Q.2 A closed planar wire loop of area A and arbitrary
(iii) An expression for the speed of the loop, ν (t) and shape is placed in a uniform magnetic field of
its terminal value. magnitude B, with its plane perpendicular to magnitude
to magnetic field. The resistance of the wire loop is R.
The loop is now turned upside down by 180o so that
Q.22 A square loop of ‘a’ with a capacitor of capacitor its plane again becomes perpendicular to the magnetic
C is located between two current carrying long parallel field. The total charge that must have flowed through
wires as shown. The value of I is given as I=I0 sin ω t . the wire in the process is
(A) <AB/R (B) =AB/R (C) =2AB/R (D) None
a a
Q.3 A square loop of side a and resistance R is moved in
I I the region of uniform magnetic field B (loop remaining
a completely inside field), with a velocity v through a
distance x. The work done is:
(a) Calculate maximum current in the square loop.
B2 vx 2B2 2 vx 4B2 2 vx
(b) Draw a graph between charge on the lower plate of (A) (B) (C) (D) None
R R R
the capacitor v/s time.
Q.4 A metallic rod of length L and mass M is moving
Q.23 A long solenoid of radius a and number of turns under the action of two unequal forces F1 and F2
per unit length n is enclosed by cylindrical shell of radius (directed opposite to each other) acting at its ends
R, thickness d(d<<R) and length L. A variable current along its length. Ignore gravity and any external
i=i0 sin ω t flows through the coil. If the resistivity of the magnetic field. If specific charge of electrons is (e/m),
material of cylindrical shell is ρ , find the induced current then the potential difference between the ends of the
in the shell. rod is steady state must be
R
(A) |F1-F2|mL/eM (B) (F1-F2)mL/eM
a

(C) [mL/eM] ln [F1/F2] (D) None
d

Q.5 A rod closing the current (shown in Figure) moves


along a U shaped wire at a constant speed v under the
L action of the force F. The circuit is in a uniform magnetic
perpendicular to the plane. Calculate F if the rate of
heat generation in the circuit is Q.

F
Exercise 2
Single Correct Choice Type
Q v
(A) F=Qv (B) F= (C) F= (D) F= Qv
Q.1 An electron is moving in a circular orbit of radius v Q
R with an angular acceleration α. At the center of the
orbit is kept a conducting loop of radius r,(r<<R). The Q.6 Two parallel long straight conductors lie on a
e.m.f. induced in the smaller loop due to the motion of smooth surface. Two other parallel conductors rest
the electron is on them at right angles so as to form a square side
(A) Zero, since charge on electron in constant a initially. A uniform magnetic field B exists at right
angles to the plane containing the conductors. They all
µ0 er 2 start moving out with a constant velocity v. If r is the
(B) α
4R
2 2 . 5 0 | Electromagnetic Induction and Electromagnetic Waves

resistance per unit length of the wire the current in the Multiple Correct Choice Type
circuit will be
Bv Br Q.11 A conducting wire is placed in a magnetic field
(A) (B) (C) Bvr (D) Bv
r v which is directed into the paper. The magnetic field is
increasing at a constant rate. The directions of induced
Q.7An equilateral triangle loop ADC of some finite B currents in wire AB and CD are
as shown in the Figure. At time t=0, side DC of loop
is at edge of the magnetic field. Magnetic field is
perpendicular to the paper inwards (or perpendicular
to the plane of the coil). The induced current versus
time graph will be as

A
Figure
(A) B to A and D to C (B) A22.104
to B and C to D
(C) A to B and D to C (D) B to A and C to D

D C Q. 12 Two circular coils P& Q are fixed coaxially & carry


V currents I1 and I2 respectively
i i
I1 I2
(A) (B)
t t
P Q
i i
(A) If I2=0& P moves towards Q, a current in the same
(C) (D) direction as I1 is induced in Q
t t
(B) If I1=0 & Q moves towards P, a current in the opposite
direction to that of I2 is induced in P.
Q.8 A ring of resistance 10 Ω, radius 10cm and 100
turns is rotated at a rate 100 rev/s about its diameter is (C) When I1 ≠ 0 and I2 ≠ 0 are in the same direction
perpendicular to a uniform magnetic field of induction then the two coils tend to move apart.
10mT. The amplitude of the current in the loop will be (D) When I1 ≠ 0 and I2 ≠ 0 are in opposite directions
nearly (take: π2 =10 ) then the coils tends to move apart.
(A) 200A (B) 2A
(C) 0.002 A (D) None of these Q.13 A conducting rod PQ of length L= 1.0 m is moving
with a uniform speed v=20 m/s in a uniform magnetic
field B=4.0T directed into the paper. A capacitor of
Q.9 A long solenoid of N turns has a self-inductance
capacity C= 10 µF is connected as shown in Figure. Then
L and area of cross section A. When a current I flows
P
through the solenoid, the magnetic field inside it has
magnetic B. the current I is equal to:
A
(A) BAN/L (B) BANL B

(C) BN/AL (D) B/ANL
Q

Q.10 A small square loop of wire of side l is placed (A) qA=+800 µC andqB =
−800µC
inside a large square loop of wire of side L (L>>l).
(B) qA =
−800 µ C andqB =
+800µ C 
The loop are co-planner & their centers coincide. The
mutual inductance of the system is proportional to: (C) qA= 0= qB

l l2 L L2 (D) charged stored in the capacitor increases


(A) (B) (C) (D) exponentially with time
L L l l
P hysi cs | 22.51

Q.14 The e.m.f. induced in a coil of wire, which is (A) There will be repulsion between A and B if i is
rotating in a magnetic field, does not depend on increased
(A) The angular speed of rotation (B) There will be attraction between A and B if i is
increased
(B) The area of the coil
(C) There will be neither attraction nor repulsion when
(C) The number of turns on the coil
i is changed
(D) The resistance of the coil
(D) Attraction or repulsion between A and B depends
on the direction of current. It does not depend whether
Q.15 Current growth in two L-R circuit (b) and the current is increased or decreased.
(c) as shown in Figure (a). Let L1,L2, R2 and R2 be the
corresponding value in two circuits, then
Q.19 A bar magnet is moved along the axis of copper
ring placed far away from the magnet. Looking from
i the side of the magnet, an anticlockwise current is
(b) L1 ( c ) R1 L12 R21 found to be induced in the ring. Which of the following
(b) (c) may by true?
t V S V S (A) The south pole faces the ring and the magnet moves
towards it.
(a) (b) (c)
(B) The north pole faces the ring and the magnet moves
(A) R1 > R 2    (B) R1 = R 2   (C) L1 > L2 (D) L1 < L2
towards it.
(C) The south pole faces the ring and the magnet moves
Q.16 The dimension of the ratio of magnetic flux and
away from it.
the resistance is equal to that of:
(D) The north pole faces the ring and the magnet moves
(A) Induced e.m.f. (B) Charge
away from it.
(C) Inductance (D) Current
Q.20 AB and CD are smooth parallel rails, separated
Q.17 Figure 22.73 shows a P by a distance l, and inclined to the horizontal at an
plane figure made of a angel θ. A uniform magnetic field of magnitude B,
conductor located in a Q directed vertically upwards, exists in the region. EF is a
magnetic field along the conductor of mass m, carrying a current i. For EF to be
inward normal to the plane R in equilibrium,
of the figure. The magnetic B F D
field starts diminishing.
Then the induced current
C  B
(A) At point P is clockwise
(B) At point Q is anticlockwise 
E
(C) At point Q is clockwise

(D) At point R is zero A
(A) i must flow from E to F (B) Bil=mg tan θ
Q.18 Two circular coils A and B are facing each
other as shown in Figure. The current I through (C) Bil=mg sin θ (D) Bil=mg
A can be altered
A B Q.21 In the previous question, if B is normal to the
plane of the rails
(A) Bil=mg tan θ
(B) Bil=mg sin θ
(C) Bil=mg cos θ
i

 (D) equilibrium cannot be reached


2 2 . 5 2 | Electromagnetic Induction and Electromagnetic Waves

Q.22 A semicircle conducting ring of radius R is placed Q.25 Two metallic rings A and B, identical in shape
in the xy plane, as shown in the Figure. A uniform and size but having different resistivities ρA and ρB ,
magnetic field is set up along the x-axis. No net e.m.f., are kept on top of two identical solenoids as shown in
will be induced in the ring. If the Figure. When current I is switched on in both the
Y solenoids in identical manner, the rings A and B jump to
heights hA and hB respectively, with hA>hB. The possible
relation (s) between their resistivity and their masses mA
B
and mB is (are)
R

X A B

(A) It moves along the x-axis


(B) It moves along the y-axis
(C) It moves along the z-axis
(D) It remains stationary

Q.23 In the given diagram, a line of


(A) ρA > ρB and mA =mB (B) ρA < ρB and mA =mB
force of a particular force field is
shown. Out of the following options, (C) ρA > ρB and mA > mB (D) ρA < ρB and mA < mB
it can never represent
(A) An electrostatic field Assertion Reasoning Type
(B) A magnetic field
Q.26 Statement-I: A vertical iron rod has a coil of wire
(C) A gravitation field of mass at rest wound over it at the bottom end. An alternating current
(D) An induced electric field flows in the coil. The rod goes through a conducting
ring as shown in the Figure. The ring can float at a
certain height above the coil because
Q.24 The Figure shows certain wire segments joined
together to form a coplanar loop. The loop is placed in
a perpendicular magnetic field in the direction going
into the plane of the figure. The magnitude of the field
increases with time.I1 and I2 are the currents in the
segments ab and cd. Then,

c d
a b

Statement-II: In the above situation, a current is


induced in the ring which interacts with the horizontal
component of the field to produce an average force in
the upward direction.
(A) Statement-I is true, statement-II is true; statement-II
(A) I1 > I2 is a correct explanation for statement-I.
(B) I1 < I2 (B) Statement-I is true, statement-II is true; statement-II
is not a correct explanation for statement-I.
(C) I1 is in the direction ba and I2 is in the direction cd
(C) Statement-I is true, statement-II is false
(D) I1 is in the direction ab and I2 is in the direction dc
(D) Statement-I is false, statement-II is true
P hysi cs | 22.53

Comprehension Type Q.30 The value of magnetic flux in each case is given by

Comprehension-I ( A ) Case I : Φ = π (L2 + 2 ) B


A conducting ring of radius a is rotated about a point (
Case II : Φ = π L2 − 2 B )
(B ) Case I : Φ = π (L )
O on its periphery as shown in the Figure on a plane 2 2
+ B
perpendicular to uniform magnetic field B which exists
everywhere. The rotational velocity is w. Case II : Φ = π (L +  ) B
2 2

P B ( C ) Case I : Φ= (L +  ) B
2 2

Case II : Φ= (L −  ) B
2 2

(D ) Case I : Φ= (L +  ) B
2
O Q
Case II : Φ = π (L −  ) B
2

R
Q.31 The direction of induced current in the case I is
Q.27 choose the correct statement (s) related to the
potential of the points P, Q and R (A) From a to b and from c to d

(A) VP − V0 > 0 and VR − V0 < 0 (B) From a to b and from f to e

(B) V= (C) From b to a and from d to c


P VR > V0

(C) (D) From b to a and from e to f


V0 > VP =
VQ

(D) VQ − VP = VP − V0
Q.32 The direction of induced current in the case II is

Q.28 Choose correct statement (s) related to the (A) From a to b and from c to d
magnitude of potential differences (B) From b to a and from f to e
1 1 (C) From b to a and from c to d
(A) VR − V0 = Bωa2 (B) VP − VQ = Bωa2
2 2
(D) From a to b and from d to c
2 2
(C) VQ − VO =2Bωa (D) VP − VR =2Bωa

Q.33 If I1 and I2 are the magnitudes of induced current
in the cases I and II, respectively, then
Q.29 Choose the correct statement(s) related to the
induced current in the ring (A) I1 =I2 (B) I1 > I2   

(A) Current flows from Q → P → O → R → Q (B) (C) I1 < I2   (D) Nothing can be said
Current flows from Q → R → O → P → Q
(C) Current flows from Q.34 Match the Following Columns

Q → P → O and Q → R → O Column 1 Column 2


(D) No current flows (A) Dielectric (P) Time independent
ring uniform electrostatic field
Comprehension–II The adjoining Figure 22.80 shows charged out of system
two different arrangements in which two square wire (B) Dielectric (Q) Magnetic field
frames of same resistance are placed in a uniform ring uniform
constantly decreasing magnetic field B. charged
g Rotating with
h h g angular velocity.
e e f
f (C) Constant (R) Induced electric filed
current i0 in ring
c d d l c
b l b (D) Current i=i0 cos (S) Magnetic moment
a a
L L ω t in ring
I II
2 2 . 5 4 | Electromagnetic Induction and Electromagnetic Waves

Previous Years’ Questions (C) I1 is in the direction ba and I2 is in the direction cd

Q.1 An infinitely long cylinder is kept parallel to a (D) I1 is in the direction ab and I2 is in the direction dc
uniform magnetic field B directed along positive z-axis.
The direction of induced as seen from the z-axis will be Q.3 A thin flexible wire of length L is connected to
 (2005) two adjacent fixed points and carries a current l in the
clockwise direction, as shown in the Figure. When the
(A) Clockwise of the+ ve z-axis
system is put in a uniform magnetic field of straight B
(B) Anticlockwise of the + ve z-axis going into the plane of the paper, the wire takes the
(C) Zero shape of a circle. The tension in the wire is  (2010)

(D) Along the magnetic field

Q.2 The Figure shows certain wire segment joined


together to from a coplanar loop. The loop is placed in
a perpendicular magnetic field in the direction going
into the plane of the figure. The magnitude of the field.
The magnitude of the field increases with time. I1 and
I2 are the currents in the segments ab and cd. Then,
 (2009)
IBL IBL IBL
c d (A) IBL (B) (C) (D)
π 2π 4π
a b
Q.4 You are given many resistances, capacitors and
inductors. These are connected to a variable DC voltage
source (the first two circuits) or an AC voltage source
of 50Hz frequency (the next three circuits) in different
ways as shown in column II. When a current I (steady
state for DC or rms for AC) flows through the circuit, the
corresponding voltage V1 and V2 (indicated in circuits)
(A) I1 > I2 are related as shown in column I. (2010)
(B) I1 > I2

Column I Column I
V1 V2
(A) I ≠ 0, V1 is proportional to I

6mH 3F
(p)

V
V1 V2
(B) I ≠ 0, V2 > V1

6mH 2
(q)

V
P hysi cs | 22.55

V1 V2
(C)=
V1 0,
= V2 V

6mH 2
(r)

V1 V2
(D) I ≠ 0, V1 is proportional to I

6mH 3F
(s)
V
V1 V2

1k 3F
(s)
V

Passage I Q.6 After the capacitor gets fully charged, S1 is opened


and S2 is closed that the inductor in series with the
The capacitor of capacitance C and be charged (with
capacitor. Then, (2006)
the help of a resistance R) by a voltage source V, by
closing switch S2 open. The capacitor can be connected (A) At t=0, energy stored in the circuit is purely in the
in series with an inductor L by closing switch S2 and form of magnetic energy
opening S1 (See fig.).
(B) At any time t>0, current in the circuit is in the same
V direction
(C) At t>0, there is no exchange of energy between the
R
C S1 inductor and capacitor
(d) At any time t>0, maximum instantaneous current in
c
S2 the circuit may be V
L

Q.7 If the total charge stored in the LC circuit is Q0, then


Q.5 Initially, the capacitor was uncharged. Now, switch
for t ≥ 0  (2006)
S1 is closed and S2 is kept open. If time constant of this
circuit is t, then  (2006) π t 
=
(A) The charge on the capacitor is Q Q0 cos  + 
(A) After time interval t, charge on the capacitor is CV/2 2 Lc 
(B) After time interval 2t, Charge on the capacitor is CV π t 
(1-e-2) =
(B) The charge on the capacitor is Q Q0 cos  − 
2 Lc 
(C) The work done by the voltage source will be half of
the heat dissipated when the capacitor is fully charged d2Q
(C) The charge on the capacitor is Q = − LC
(D) After time interval 2t, charge on the capacitor is CV dt2
(1-e-1) 2
(D) The charge on the capacitor is Q = − 1 d Q
2
LC dt
2 2 . 5 6 | Electromagnetic Induction and Electromagnetic Waves

Q.8 Two different coils have self-inductances L1=8 mH (B) The emf induced in the loop is finite if the current
and L2 =2 mH. The current in one coil is increased at is constant
a constant rate. The current in the second coil is also
(C) The emf induced in the loop is zero if the current
increased at the same constant rate. At a certain instant
decreases at a steady rate
of time, the power given to the coils is the same, At that
time, the current the induced voltage and the energy (D) The emf induced in the loop is finite if the current
stored in the first soil are i1,V1 and W1 respectively. decreases at a steady rate
Corresponding value for the second coil at the same
instant are i2, V2 and W2 respectively. (1994) Q.12 If the direct transmission method with a cable of
Then resistance 0.4 Ω km−1 is used, the power dissipation (in
i1 i1 %) during transmission is  (2013)
1 W1 1 V1
(A) = (B) = 4 (C) = (D) =4
i2 4 i2 W2 4 V2 (A) 20 (B) 30 (C) 40 (D) 50

Q.13 In the method using the transformers, assume


Q.9 A series R-C circuit is connected to AC voltage
that the ratio of the number of turns in the primary to
source. Consider two cases; (A) when C is without a
that in the secondary in the step-up transformer is 1 :
dielectric medium and (B) when C is filled C is filled
10. If the power to the consumers has to be supplied at
with dielectric of constant 4. The current IR through the
200 V, the ratio of the number of turns in the primary to
resistor and VC across the capacitor are compared in the
that in the secondary in the step-down transformer is
two cases. Which of the following is/are true?  (2011)
 (2013)
A B A B A B A B
(A) IR > IR (B) IR < IR (C) VC > IC (D) VC < IC (A) 200 : 1 (B) 150 : 1 (C) 100 : 1 (D) 50 : 1

Q.14 A conducting loop in the shape of a right angled


Q.10 A circular wire loop of radius R is placed in the x-y
isosceles triangle of height 10 cm is kept such that the
plane centered at the origin O. A square loop of side
90o vertex is very close to an infinitely long conducting
a (a < < R) having two turns is placed with its centre
wire (see the figure). The wire is electrically insulated
at z = 3 R along the axis of the circular wire loop, as from the loop. The hypotenuse of the triangle is parallel
shown in figure. The plane of the square loop makes to the wire. The current in the triangular loop is in
an angle of 45o with respect to the z-axis. If the mutual counterclockwise direction and increased at a constant
µ0 a2 rate of 10 A s-1. Which of the following statement(s)
inductance between the loops is given by , then
21/2 R is(are) true? (2016)
the value of p is  (2012)
z
o
90
o 10 cm
45

 µ0 
 (A) The magnitude of induced emf in the wire is  
3R volt  π 

(B) If the loop is rotated at a constant angular speed


R  µ0 
y about the wire, an additional emf of   volt is
O
induced in the wire  π 
(C) The induced current in the wire is in opposite
direction to the current along the hypotenuse
Q.11 A current carrying infinitely long wire is kept along
the diameter of a circular wire loop, without touching it. (D) There is a repulsive force between the wire and the
The correct statement(s) is (are) (2012) loop

(A) The emf induced in the loop is zero if the current is


constant.
P hysi cs | 22.57

Q.15 Two inductors L1 (inductance 1 mH, internal Which of the following schematic plot(s) is(are) correct?
resistance 3 Ω) and L2 (inductance 2 mH, internal (Ignore gravity)
resistance 4 Ω), and a resistor R (resistance 12 Ω) are all
I(x) F(x)
connected in parallel across a 5V battery. The circuit is
switched on at time t=0. The ratio of the maximum to
the minimum current (Imax/Imin) drawn from the battery (A) (B) L
is  (2016) x x
0 L 2L 3L 4L 0 2L 3L 4L

Q.16 A rigid wire loop of square shape having side of


length L and resistance R is moving along the x-axis I(x)
with a constant velocity v0 in the plane of the paper. v(x)
At t=0, the right edge of the loop enters a region of v0
length 3L where there is a uniform magnetic field B0 3L 4L
x
(C) (D)
into the plane of the paper, as shown in the figure. 0 L 2L x
For sufficiently large v0, the loop eventually crosses 0 L 2L 3L 4L
the region. Let x be the location of the right edge of
the loop. Let v(x), I(x) and F(x) represent the velocity
of the loop, current in the I(x)
loop, and force on the loop, F(x)
respectively, as a function of x. Counter-clockwise
current is taken as positive. (2016)
(A) (B) L
x x
0 L 2L 3L 4L 0 2L 3L 4L

I(x)
L v(x)
v0 v0
3L 4L
(C)
0 xx (D)
0 L 2LL 2L
3L 4L x
0 L 2L 3L 4L

PlancEssential Questions
JEE Main/Boards JEE Advanced/Boards

Exercise 1 Exercise 1
Q.4 Q.7 Q.9
Q.5 Q.8 Q.9
Q.13 Q.14 Q.19
Q.18
Q.20

Exercise 2 Exercise 2
Q.3 Q.4 Q.7
Q. 1 Q.2 Q.8
Q.13 Q. 15 Q.24
Q.11 Q.14
Q.25 Q.26
2 2 . 5 8 | Electromagnetic Induction and Electromagnetic Waves

Answer Key

JEE Main/Boards Q.12 µr =1000

Q.13 Along PSRQP


Exercise 1
Q.16 emax=0.6032 V and eav=0
Q.2 0.4 V
Q.18 F=0.00375 N
Q.3 8 × 10 −5 Wb. 8 × 10 −3 Wb
Q.20 (i) Mutual inductance
Q.5 5.03 × 10 −4 H
(ii) The current product in coil B depends on:
Q.6 6.25 × 10−3 H
(a) Number of turns in the coil
Q.7 0.1 H
   (b) Natural of material
Q.8 Clockwise Direction    (c) geometry of coil
Q.9 2.56 V Q.21 (ii) 625 × 10 −4 V
3
Q.10 By decreasing current from 2 A to zero in 0.28s
Q.24 (i) Same
Q.11 1.58 × 10 −4 H
(ii) Current in copper loop is more than aluminum loop

Exercise 2

Single correct choice type

Q.1 C Q.2 A Q.3 A Q.4 C Q.5 A Q.6 D

Q.7 A Q.8 A Q.9 B Q.10 C Q.11 A Q.12 A

Q.13 C Q.14 A Q.15 D Q.16 D Q.17 B Q18 B

Q.19 D Q.20 C Q.21 D Q.22 A

Previous Years’ Question


Q.1 D Q.2 D Q.3 B Q.4 B Q.5 D Q.6 B

Q.7 D Q.8 D Q.9 D Q.10 D Q.11 C Q.12 D

Q.13 B Q.14 D Q.15 A Q.16 B Q.17 A Q.18 D

Q.19 C Q.20 D Q.21 A Q.22 C Q.23 D Q.24 C

JEE Advanced/Boards
Exercise 1
Q.1 10 µV Q.2 λVyB0

Q.3 5.0 µV Q.4 2N


2
µ0ia π Rmg
Q.5 Q.6
2Rb B2  2
P hysi cs | 22.59

1
Q.7 A Q.8 0.4V
3
Q.9 3 µV ,clockwise
erk
Q.10 directed along tangent to the circleof radius r, whose center lies on the axis of cylinder
2m
πa2C
Q.11 Q.12 200 rad/s
R
Q.13 (i) 85.22 Tm2; (ii)56.8 V (iii) Linearly Q.14 (i) 2.4 × 10−5 V (ii) from c to b

Q.15 (i) 3.3 × 108 A, (ii) 4.1 × 107 W, (iii) totally unrealistic Q.16 
E w s
3w 4w

 s
F
s

2
µ0ihωimN b  dB 2 
Q.17 ln Q.18 R −
2π a 2 dt 4

7 3 1 1 Bωr 2 1 − e−Rt/L 
Q.19
= iEA = Al; iBE = A; iFE A 2
Q.20 (i) E= Bωr (ii) I =  
22 11 22 2 2R
mgR g
Q.21 (i) Vterminal = ;(ii)
2 2
BL 2
µ0 a
CI0 ω2 ln2,
( µ0ni0 cos ωt ) πa2 (Ld)
Q.22 (a) Imax= Q.23 I =
π ρ2πR

(b) Qo
  
/ 2/
t
/2 3/2
-Qo

Exercise 2

Single Correct Choice Type

Q.1 B Q.2 C Q.3 D Q.4 A Q.5 B Q.6 A

Q.7 B Q.8 B Q.9 A Q.10 B

Multiple Correct Choice Type

Q.11 A Q.12 B, D Q.13 A Q.14 D Q.15 B, D Q.16 B

Q.17 A, B, D Q.18 A Q.19 B, C Q.20 A, B Q.21 B Q.22 A, B, C, D

Q.23 A, C Q.24 D Q.25 B, D


2 2 . 6 0 | Electromagnetic Induction and Electromagnetic Waves

Assertion Reasoning Type

Q.26 C

Comprehension Type

Q.27 B, D Q.28 A, C Q.29 D Q.30 C Q.31 C Q.32 B

Q.33 B

Match the Column Type

Q.34 A → P; B → P, Q, S; C → Q, S; D → Q, R, S

Previous Years’ Questions


Q.1 C Q.2 D Q.3 C Q.5 B Q.6 D Q.7 C

Q.8 A, C, D Q.9 B, C Q.10 7 Q.11 A, C Q.12 B Q.13 A

Q.14 A, D Q.15 8 Q.16 C, D

Solutions

JEE Main/Boards di
N∈=– M
dt
Exercise 1 m0nNA=M

M=4p×10–7 × 50 × 102 × 200 × 4 × 10–4 =5.03 × 10–4 H


Sol 1: No, as the voltmeter also gets induced emf.
∆I
dφ nB dA Sol 6 : ε = M
Sol 2: E.m.f.= = ∆T
dt dt
(∆A=2A as it turned through 180o) 4
50 × 10–3= M.
1
4 × 10 –4 × 103 × 500 × 10 –4 × 2 2
= = 0.4V
1
50 × 10 –3
10 M= =6.25 × 10–3H
8
dφ di
Sol 3: =L ∆I
dt dt Sol 7: ε = L
∆T
φ=Li; φ=20 × 10–3 × 4 × 10–3=80 µ Wb 4
4 × 104= L .
Total flux=hφ=100 × 80 mWb=8000 mWb 10 × 10 –6
L=0.1 Henry
Sol 4: Field is perpendicular outwards the paper. As
the loop area increases, net flux increases, so induced µ0i
Sol 8: B=
current tries to reduce flux. So it flow clock wise. 2πr
dφ=B.dA=B.dr
Sol 5: B=m0ni
µ 0 i
dB di dφ= dr
ε =– A =– m0nA 2πr
dt dt
P hysi cs | 22.61

µ 0 i r2 4 × π × 10 –7 × 10 × 0.2 0.1 ⇒Di increases. i increases


φ= ln = ln
2π r1 2π 0.05 \bulb becomes brighter

dφ 2.77 × 10 –7 After completely removing, the current again decreases


ε= = =1.39 × 10–5 v = 2.77 × 10–7Wb.
dt 2 × 10 –2 V
as steady state current is I0= , which was also initial
current r
Current will be in clockwise direction.

∆A Sol 15: The voltage induced across a conductor when


Sol 9: ε = Bn (∆A=2A as it turns 180º)
∆T it is exposed to a varying magnetic field in called
Bn2A 0.4 × 100 × 2 × (8 × 10 –2 )2 inducedemf.
= = = 2.56V
t 0.2 dφ
ε=–
∆i dt
Sol 10: ε = –L
∆t ⇒ dV=– B v(d )
(–2) ⇒ dV=– Bω r dr
50 = –5
∆t BωL2
V=
T=0.28 2

Current should reduce to 0 in 0.28.


Sol 16: A=A0 cos ωt

Sol 11: B=m0n1i φ=BnA = nB.A0 cos wt

di dφ
ε =– L e=– =nBA0 w sin wt
dt dt

di = 20 × 3 × 10–2 × π(8 × 10–2)2 × 50=0.6 V


 d   d 
ε = n2  – B.A  = n2  – µ0 in1 πr 2  = n2m0n1pr2 dt ⇒ eav=0 as in one complete rotatior, Se= 0
 dt   dt 
1000
⇒ M=m0 n1n2pr2 = 4p×10–7× ×100×π(2×10–2)2 Sol 17: If a current i in a coil changes with time on e.m.f.
100 × 10 –1
= 1.58 × 10 H –4 is induced in the coil. The self – induced rmf is ∈L=– L
di
Sol 12: L ∝ m dt

L2 µ2 B=m0 in
= di
L1 µ1 ε =– m0npr2 dt ∴ L=m0npr2

m1=m0, m2=mrm0
N µ Nπr 2
L2
10 π= ∴ L= 0
⇒ mr= = =1000  
L1 0.01
\ mr=1000 Sol 18: ε =Blv
B V
Sol 13: It flows anti-clock wise to increase flux along i=
R
outside the plane. Hence it flow PSRQP. B2 2 V (0.15)2 (0.5)2 (2)
F=iB = = =3.75×10–3 N
R 3
di
Sol 14: ε = –L
dt Sol 19: The currents induced in a solid conducting
Solenoid tries to go back to initial state i.e. If an action body as it passes through a magnetic field is called
produce a change Dε1, solenoid tries to produce a eddy current.
change Dε2 such that Dε2 is in Opposite direction of Dε1. Eddy currents lead to heating up of Transformer core.
When you remove iron core, L keeps decreasing Eddy current is reduced by making transformer with
thin slabs.
2 2 . 6 2 | Electromagnetic Induction and Electromagnetic Waves

Exercise 2
Sol 1 : (C) A=pR2 =π(R0 + t)2
dA
=2π (R0 + t)
dt
–BdA
ε= =– 2πB(R0 + t)
dt
Sol 20: (i) The Principal involved is mutual inductance \2π(R0 + t)B is induced anticlockwise.
(ii) The current produced in coil B depends on Note: To have clarity about clockwise or anticlockwise,
(a) number of turns in the coil, remember as flux increases, it tries to reduce net
magnetic field B. Hence voltage is induced. It leads to
(b) Nature of material current in direction of voltage, which reduces magnetic
(c) geometry of coil field.

BA
Sol 21: (i) Faraday’s law of electromagnetic induction Sol 2 : (A) E=
∆t
An emf is induced in the loop when the number
of magnetic field lines that pass through the loop is 20 × (0.1)2
10=
changing. ∆t
(ii) ε =Blv tanθ
∴ Dt=20 ms
5 1
=5×10 ×25×1800×
–4
×
18 3 Sol 3 : (A) [MA–1T–2]
625
= ×10 V
–4

3 µ0i
Now B= (for circular wire)
2r
Sol 22: (a) The current induced in a solid conducting
body as it passes through a magnetic field is called eddy [B][r] [MA –1 T –1 ][L]
⇒ [m0]= = = MLA–2T–2
current. It is used in induction stove, water heaters, etc. [ i] [A]
(b) (i) ∈=Blv
Sol 4 : (C) Induced emf tries to push the coil upward in
∈ B v B2  2 v
(ii) i= = ; F=iB= case II and magnet in case-I, to present sudden change
R R R
in net flux.
∈2 B2 2 v 2
(iii) Power dissipated P= = \a1, a2< g
R R

Sol 23: If a current i in a coil charges with time, an emf Sol 5 : (A) For a circular loop B at center is greater than
B at any point along the axis.
di
is induced in the coil. The self-inducedemf is ∈L=– L
dt When both the loops approach each other, magnetic
S.I unit Henry–H. field (B) starts increasing at center. To compensate it,
Current decreasing.
Sol 24: (i) Inducedemf is same →
Sol 6 : (D) Let the triangle travel a distance x along v
ε = 2pr2w B in time t.
(ii) Current in copper is more, as its resistance is less. Area of triangle in magnetic field
1
Sol 25: It induces current in opposite direction. A= x(2x) = x2
2
A=v2t2
Sol 26: Emf induces Anticlockwise as seen from north.
Both Magnets produce current in same direction. –BdA
E=
dt
P hysi cs | 22.63

d 2 2 Sol 13 : (C) Let displacement of PQ be x.dx be small


iR= B (v t )
displacement along dv
dt
2Bv 2 dE=vBdx
i= t
R v=xω
\i∝t \dE=ωB xdx
2
→ → WB 2 x0 ωBx0
Sol 7 : (A) E=  v × B  .  ⇒ E= x ⇒ E=
  2 0 2
 
x20 =l2 + L2
 ( ) (  ) ( )
=  2iˆ + 3ˆj + Kˆ × ˆi + 2ˆj  2kˆ =  –2iˆ + ˆj + kˆ  2kˆ 
  
E=2V \E=
(
ωB L2 + 2 )
2
→ →
    ( )( )
Sol 8 : (A) E=  V× B  . l= 10iˆ × 4kˆ  0.3ˆj = 12 V
Sol 14 : (A) Current is from P to Q
 
(A)
(
Sol 9 : (B) E= V × B . l ) π π
µ0 I Sol 15 : (D) wt= ⇒ t=
B= 2 2ω
2πr
µ0 I BDA B A 2ωBA
E= . V. l Avg. E.m.f= = =
2πr Dt π π
ω
→ 2
Sol 10 : (C) B=B K ; V= v x ˆi + v y ˆj ; L= 1ˆi +  2ˆj
Sol 16 : (D) dε = vB dl


ˆ (ˆ )
E=(v × B). () = Vx i + Vy j × Bk .  – ˆi +  2ˆj
ˆ 
  
V = lw
ˆ ˆ
= + Vx B, j + v yBl2 i \d∈=Bwldl
2
⇒ VA – VB=VyBl2 B ω 2
E= Bω∫ d =
2
VC – VB=Vx Bl1 1

L
VA – VC=VyBl2 –VxBl1 l2=L; l 1=
2
\VA – VC∝ (Vxl2 – Vyl1)
Bω  2  L   3Bω2
2
\Î= L –  =
2   2   8

Sol 11 : (A) V =2i
=5 cos θ î + 5 sin θ ˆj =3 î + 4 ˆj Sol 17 : (B) Electric field is induced to left
→ → \it accelerates to right (B)
E=  V× B  . = 2iˆ × (3ˆj + 4kˆ  3iˆ + 4ˆj
     
 
µ0i
Sol 18 : (B) B=
= 6kˆ – 8ˆj 3iˆ + 4ˆj = 32 Volts 2R
  
dB µ0 di
=
Sol 12 : (A) φ=B.dA dt 2R dt
– µ0 di
x 2 E= . pr2
φ= B0 1 + d 2R dt
a

dφ dφ dx di
E= –L
= . dt
dt dv dt

B0 d2 µ0 πr 2
⇒ L=
= V 0. 2R
a
2 2 . 6 4 | Electromagnetic Induction and Electromagnetic Waves

d Sol 4: (B) Magnetic field produced by a current I in a


Sol 19 : (D) E= – B.dA
dt large square loop at its centre,
B.dA=0 Q E=0 ⇒ L=0 i
B∝
Note: Simply we can say. The magnetic field vectors will L
be along the plant. i
Say B=K
L
B.dA=0
∴ Magnetic flux linked with smaller loop
\E=0
φ=B.S
⇒ L=0
 i
φ=  K  (l2)
Sol 20 : (C) Current increases  L
Therefore, the mutual inductance
⇒ magnetic field increases at a giver point. Magnetic
field also decreases radially. Hence to nullify the φ 2 2
M= =K or M ∝
increases magnetic field, loop B repels. i L L
Note Dimensions of self-inductance (L) or mutual
Sol 21 : (D) φ= ∫ B.dA =K inside loop, – K outside loos inductance (M) are:
\total φ=0 [Mutual inductance]=[Self-inductance]=[m0][length]
Similarly dimensions of capacitance are :
Sol 22 : (A) In (a), magnetic field in perpendicular to
[capacitance]=[e0][length]
plane others along plane,
From this point of view options (b) and (d) may be
∴ in others it is minimum, maximum in (a)
correct

Previous Years’ Questions Sol 5 : (D) The current-time (i – t) equation in L-R circuit
is given by [Growth of current in L-R circuit]
Sol 1 : (D) Net change in magnetic flux passing through –t/tL
the coil is zero. i=i0( 1 – e ) … (i)

∴ Current (or emf) induced in the loop is zero


V 12
whereiL= = =2 A
R 6

Sol 2 : (D) Induced motional emf in MNQ is equivalent


L 8.4 × 10 –3
and i0= = = 1.4 × 10–3 S
to the motional emf in an imaginary wire MQ i.e., R 6
and i=1A(given), t=?
eMNQ=eMQ=Bvl=Bv(2R) [=MQ=2R]
Substituting these values in Eq. (i), we get
Therefore, potential difference developed across the
ring is 2RBv with Q at higher potential. t=0.97 × 10–3 s
or t=0.97 ms ⇒ t=1 ms
Sol 3 : (B) A motional emf, e=Blv is induced in the
rod. Or we can say a potential difference is induced
Sol 6 : (B)
between the two ends of the rod. AB with A at higher
potential and B at lower potential. Due to this potential →→
dϕ dB
difference, there is an electric field in the rod. ∫ E d = dt
=S
dt

dB
or E(2pr)=pa2 dt for r ≥ a

a2 dB
∴ E=
2r dt

1
Induced electric field ∝
r
P hysi cs | 22.65

For r ≤ a Sol 9: (D) When switch S is closed magnetic field lines


dB passing through Q increases in the direction from right to
E(2pr)=pr2 dt left. So according to Lenz’s law induced current in Q i.e.,
IQ will flow in such a direction, so that the magnetic field
1
lines duet to IQ passes from left to right through Q. This
r dB 1
Or E= or E ∝ r is possible when IQ flows in anticlockwise direction as
2 dt 1
seen by E. Opposite is the case when switch S is opened
a dB i.e., IQ will be clockwise as seen by E.
At r=a, E= 2
2 dt
Therefore, variation of E with r (distance from centre) e2
Sol 10 : (D) Power P=
will be as follows R
 dϕ 
Here, e=induced emf=–  
 dt 
whereφ=NBA

 dB 
E=– NA  
 dt 
1
Also, R ∝
r2
Sol 7: (D) The equations of l1(t), l2(t) and B(t) will take Where R= resistance, r=radius,
the following form : =length.
–k 2t
I1(t)=K1(1 – e ) → current growth in L-R circuit ∴ P ∝ N2r2
–k 2t
B(t)=K3(1– e ) → (t) ∝ I1(t) P2
∴ =4
P1
I2(t)=K4 e –k 2t

 e dI dI  Sol 11: (C) Direction of polarization is parallel to


= 2 and e2 ∝ 1 e=
I2 (t) 2 –m 1  magnetic field,
 R dt dt   
–k t
∴ X || B
Therefore, the product I2(t)B(t)=K5 e–k2t (1 – e 2 ). The  
value of this product is zero at t=0 and t=∞. Therefore, and direction of wave propagation is parallel to E × B
  
the product will pass through a maximum value. The ∴ K || E × B
corresponding graphs will be as follows :
Sol 12 : (D) Oscillating coil produces time variable
magnetic field. It cause eddy current in the aluminium
plate which causes anti–torque on the coil, due to
which is stops.

Sol 13 : (B) Charge on the capacitor at any time ‘t’ is


=q CV (1 − e− t/ τ )

At t = 2τ

=q CV (1 − e−2 )

2 1
Sol 14 : (D)
Sol 8 : (D) Electric field will be induced in both AD and r
BC.
R d
2 2 . 6 6 | Electromagnetic Induction and Electromagnetic Waves

Let M12 be the coefficient of mutual induction between


loops 
φ =M 12i2 Fl

µ0i2R 2 2
⇒ πr =
M12i2
2(d2 + R 2 )3/2 lg
µ0R 2 πr 2 Sol 22 : (C)
⇒ M12 =
2(d2 + R 2 )3/2 µ0
2πi
BA =
φ2= M12i1 ⇒ φ2= 9.1 × 10 −11 weber 4 π (  / 2π )
 µ0 i 
=BB  (sin 45o + sin 45o ) × 4
8 −9
= 6 V /m  4π  / 8 
Sol 15 : (A) E0 = CB0 =3 × 10 × 20 × 10
BA 2
π
=
BB 8 2
Sol 16 : (B)
Infrared waves → To treat muscular strain
Sol 23 : (D)
radio waves → for broadcasting
Radiation energy per quantum is
X-rays → To detect fracture of bones
E=hv
Ultraviolet rays → Absorbed by the ozone layer of the
atmosphere; As per EM spectrum, the increasing order of frequency
and hence energy is
Sol 17 : (A) Energy is equally divided between electric Radio wave < Yellow light < Blue light < X Ray
and magnetic field.
Sol 24 : (C) For electromagnet and transformer, the
Sol 18 : (D) Since resistance and inductor are in parallel, coercivity should be low to reduce energy loss.
so ratio will be 1.

Sol 19 : (C) When K1 is closed and K2 is open, JEE Advanced/Boards


E
I0 = Exercise 1
R
when K1 is open and K2 is closed, current as a function 4
Sol 1 : e=B tan θ × v .l =3 × 10–4× ×0.1×0.23
of time ‘t’ in L.R. circuit. 3
−R
t
1 −5 1 =10–5 . V=10mV
I = I0 e=L =e = 0.67 mA
10 1500
Sol 2 : E = (V × B).
Sol 20 : (D) V= v x ˆi + v y ˆi
1 2 1 2 1 2
As L1 > L2 , therefore L i > L > L 2i , B=– B0kˆ
2 1 2 2
∴ Rate of energy dissipated through R from L1 will be Here = λ î we us are taking a cross PQ
slower as compared to L2.
\E=[( v x ˆi + v y ˆj ) × B0kˆ ] (li)=λ VyB0

Sol 21 : (A)
 µ I2  Sol 3 : A=pr2
 0 
E  4 πL sin θ 
tan=
θ =   dA dA dr dr
= . =2pr.
λ g λ g dt dr dt dt

πλLg –BdA
⇒ I = 2 sin θ E ⇒=–
µ0 cos θ dt
P hysi cs | 22.67

dr Sol 7 :
=– B2pr . =(0.02) . 2π (4 × 10–2) . (1 × 10–3) =5 mV
dt

Sol 4 : Consider the sum as two loops, one with 6r and


other 3r.
E=BLV =2 × 1 × 2=4V
Similarly in loop with 3r also 4 V is induced.
Let AB be diameter rod, CD be external resistor CD is
hence the circuit can be shown as
fixed
R2
area of part I be A1= θ
2
dA BR 2 dθ
E= B =
dt 2 dt
BR 2 w 500 × (0.1)2 × 20
E= = =5 V
2 2
4 R
⇒ 1= = 1A 20
1 RAC= AB = =10 W
2+ 2 2
1 1
+
6 3
⇒ F=ilb = 1 × 1 × 2 = 2 N

µ0i µ0i
Sol 5 : B= =
2r 2b
Area of small coil Ai=pa2
5 1
d
2
d πa .µ0i i= = A
∈= BA = 1 3
dt dt 2b 10 +
1 1
+
πa2µ0 di 10 10
∈= 1
2b dt Current through external resistance is A
3
dQ
∈ =iR = R
dt dB 3 2 dB 3
Sol 8 : E= A = a . = (a)2. 3
2 dt 4 dt 4
dQ πa µ0 di
⇒R = E=3V
dt 2b dt
Emf induced is 3V
πa2µ0 πa2µ0
⇒DQ= ∆i = i
2bR 2bR E 3
Current induced I= = =0.6 B
8 5
E
Sol 6 : Let terminal velocity be V Voltage induced in each side V1=
3
E=– Blv
V1=1 V
b V
I=– Now each side acts like a battery with a resists
R
B2  2 v
Force due to magnetic field fi=iLB=–
R
\VAB=V1=iRAB =1 – 0.6 (1)=0.4 V
Force due to gravity (f2)=mg
f1 + f2=0 dB
Sol 9 : =– 3 × 10–4 (here is taken positive)
dt
B2  2 v Rmg AdB
⇒ mg – =0 ⇒ v= E=–
R B2  2 dt
2 2 . 6 8 | Electromagnetic Induction and Electromagnetic Waves

E=– 10–2 × (– 3 × 10–4)=3 µ V F qr  dB  1


α= =  .
It is induced clock wise. mr 2  dt  mr
Note: It you get confused with direction, remember
q  dB 
the induced emf produces current, which produces ⇒ α=  
magnetic field. This field will be opposite to direction of 2m  dt 
change. i.e, if B1=B – ∆B1, then induced B will produces q
∆B2 such that it opposite sign of ∆B1. ⇒ α=
2m
( 0.2t )
Sol 10 : The electron experiences the force tangentially, dω q
⇒ = (0.2t)
along the circular paths of induced emf. dt 2m
dB t
⇒ E=pr2. 0.1q 0.1q t2 0.1 × 2 × 102
dt
a=
m ∫ t.d t = =
m 2 50 × 10 –3 × 2
=200 rad/sec.
0
E=pr2K
Sol 13 : Let perpendicular distance of bar from vertex
E=2pr.E
be x
F =qE x=vt
F 1
Acceleration a= Area of triangle A= x (2x) =x2
m 2
qE A=V2t2
a=
M (i) These φ=BA
E rK
⇒E= = φ(t)=BV2t2=0.35 × (5.2)2t2
2πr 2
φ(3)=9.464 (3)2 =85.22 Tm2
q q rK erK
a= E= = dφ
m m 2 2m (ii) emf e=–
dt
(charge of electron is e)
C(t)=– 2BV2t=– 18.93 t
dB
Sol 11 : e=– A e(3)=– 18.93 (3)=56.8 V
dt
dQ dB |e(3)|=56.8V
R =– A
dt dt (iii) e(t)=– 2BV2t
–A  πa2  E(t)=Kt
⇒DQ = ∆B= –  (–c) 
R  R  It varies linearly
 
2
πa C
DQ= πr 2
R Sol 14 : (i) A=
4
dB dB
Sol 12 : e=pr2. =3 × 10–4
dt dt
dB πr 2 dB π
E πr 2 dB e=– A = = × (0.1)2. 3 × 10–4
Ε= = . dt 4 dt 4
2πr 2πr dt
e=2.4 × 10–5 V
r  dB  Induced emf is 2.4 × 10–5V
e=  
2  dt 
(ii)It flows from c to b, to reduce the increasing emf.
qr  dB 
F=qE=   (F is tangential at every point)
2  dt  Sol 15 : (i) f=ilb

F=Ia 10 × 103=i × 3 × 10 × 10–6
F . r=mr2 .a i=3.3×108A
P hysi cs | 22.69

(ii) P=i2R e=– Blv 0< x < w=0 w < x < 3w=Blvw < x < 4w
P 2
=i = 4.1 × 107 W
R Sol 17 : ∫ B.ds =m0ienc
(iii) Totally unrealistic
B.2pr=m0im in cos wt
 w µ0Nim cos ωt
Sol 16 : φ=Blvt 0 < t <  B=
 v 2πr
(downwards positive) dφ=B.dA=B. h.dr
w 3w  µ0N imcos ωt
=Blw  <t<  dr
v v  dφ= h
2π r
 4w   3w 4w  µ0N imcos ωt
=Blv  – t   <t<  b
 v v  ⇒ φ= h/n
 v  2π a

e= –dφ wµ0N imhsin ωt b
dt ∈= = ln
dt 2π a
 w
⇒ e=– Blv 0 < t < 
 v µ0N ωhim b
Amplitude= ln
2π a
w 3w 
=0  <t< 
v v 
Sol 18 :
 3ω 3ω 
=Blv  <t< 
 v v 
∈→
F=iB= B
R
– 2B2 V  w
⇒ E= 0 < t <  dB
R  v e= A
dt
w 3w  dB
= 0  <t<  ⇒ e1= A1
v v  dt
dB
– 2B2 v  3w 4w  ⇒ e2= A2
=  <t<  dt
R  v v 
⇒ e1 is along CD and ⇒ 2 along DC
(here=– )
dB
x=vt \e (A1 – A2) along CD
dt

 2 2
A1 – A2= R –
2 4

 2 2 dB
\e= R –
2 4 dt

Sol 19 : Take loop AEFD


DB
⇒ e1=A1. =1 × 1=1V
DT
Take loop EBCT
DB 1
⇒ e2= A2 = × 1 = 0.5 V
Dt 2
2 2 . 7 0 | Electromagnetic Induction and Electromagnetic Waves

Lets use superposition of current

(ii)

 –Rt 
I= I0 1 – e L 

 
 

I0=steady state current


2
e B ωr
= =
R 2R

B ωr 2  
–Rt
I= 1 – e L 
2R  
 
dTm=rdE
iAE=i11 + i21iEF=i13 – i23iBE= i12 +i22
(Tm=torque due to magnetic field)
e1 3
i11= = A dFm=Bidl
1 11
3+
1 Bidr
1+
2
(tm=magnetic force)
2 2
i13= i11= A
2+1 11 d.Tm=Birdr
1 Bir 2
i12=i11 – i13= A ⇒ Tm=
11 2
e2 2
ωB 2 g 4  – 
Rt
i22= = A
1 11 ⇒ Tm= 1 – e L 
2+ 4R  
1  
1+
3 fg=mg cos θ (fg=force of graving)
3 3
i23= i22= A fgr  r  mgr cos(θ)
3+1 22 Tg= = mg cos θ   =
2 2 2
1
i21=i22 – i23= A
22 mgr cos θ ωB2r 4  – 
Rt
\ T= + 1 – e L 
3 1 7 2 4R  
 
iAE= + = A
11 22 22
Sol 21 : e=BLV(V is terminal velocity)
2 3 1
iEf= – = A
11 22 22 e BLV
i= =
R R
1 2 3
iEB= + = A Fm=iLB (fm=force due to magnetic field)
11 11 11
B2L2 V
=
Sol 20 : (i)d∈=Bvdr R

de=Brωdr Fg=mg (fg=force due to grouts)


2
B ωr
e= B2L2 V
2 mg=
R
B ωr 2
Emfacross the terminals of switch is
2
P hysi cs | 22.71

mgR Integrating on both sides.


⇒ v= 2 2
BL B0 av
g–
–mR mR =t
B2L2  V  mg ln
2 2 2 2
(ii) Fm=  = B0 a B a v
R 2 2 g– 0 0
mR
fg=mg ⇒ v0=0 (initially dropped fromrest)
mg mg
f=fg – fm=mg – = B20 a2 v
2 2 g– 2 2
ln mR =– B0 a t
F=ma g mR
g
\a=  –B2a2t 
2 B20 a2 v
 0

g ⇒ = g  1 – e mR 
\acceleration of the mass is mR  
2  
B0 y  –B2 2 
(iii) (a) dφ=B.dy = . a. dy oa t
mgR  
a ⇒ v= 1 – e mR 
B
(
φ= y 22 – y12
2
) B20 a2 


(y1, y2 are instantaneous heights of edges parallel to


µ0i
x-axis) Sol 22 : (a) B=
2πr
B0 a
φ= (y 2 + y1 ) ( y2 – y1=a) Let magnetic field due to upward current be B,
2
µ0 I
dφ B0 d d B 1=
= (y + y1) 2πr
dt 2 dt 2
Force due to it be φ,
B0 a dy dy1 dy 2 µ0 I
= (2V) (v= = = ) dφ,= .a.dr
2 dt dt dt 2πr
dφ µ0 Ia 2a
=B0aV f 1= ln
dt 2π a
ε dφ 1 B0 aV µ0 Ia
i= = . = f1= ln 2
R dt R R 2πr
(b) fm=SiB (fn is magnetic force) µ0 Ia
Similarly f2= ln 2 (f2 is by downward current)

B y B y  B av  B 
= i a  0 2 – 0 1  = 0 .a  0 (y 2 – y1 ) \φ=f1 + f2=m0Ialn 2
 a a  R a 

B20 a2 v µ0 I0 a.ln2sin ωt
F m= φ=
R π

dφ µ ωI aln2
fg – fm e=– =– 0 0 cos wt
(c)a= dt π
m
Q=CV
B20 a2 v
mg – dQ dv (µ ω2 I aln2)
dv R i= =C =C 0 0 sin wt
= dt dt π
dt m
µ0 a
dv Imore= CI0w2 ln2
=at π
B20 a2 v – µ0 ωI0 acln2
g– (b) Q=CV= cos wt
mR π
2 2 . 7 2 | Electromagnetic Induction and Electromagnetic Waves

Sol 3 : (D) Work done is zero as magnetic fields is


uniform

Sol 4 : (A) Let voltage induced be V.


eN
Total charge a=
m
V
Electric field E=
µ0 ωI0 a.cln2 L
Q0=
π QE=|f1 – f2|

Sol 23 : B=m0 in eM V
. =|f1 – f2|
M L
φ=B.A=m0 in pa2
V=|f1 – f2| ML/eM
φ=m0 npa2 i0 sin wt
–dφ Sol 5 : (B) Here power Supplied=Heat generated as no
∈= =– m0npa2ωI0 cos wt
dt other element is using I,
e. e.2πR Q
Resistance of shell, r5= = ⇒F . V=Q ⇒ F=
A L.d V
∈ Sol 6 : (A) area of loop A=a2
i=
rs
dA da
= 2a = 2a (2V)= 4av

∴I=
(µ ni ω cos ωt)πa (Ld))
0 0
2 dt dt
dA
ρ.2πR B =iR
dt
B
Exercise 2 ⇒ i=
R
=(4aV)

⇒ R=4ar
Sol 1 : (B) Let angular velocity be ω.
Bv
ωe ⇒ i=
i=– r

µ0i – µ 0 ωe Sol 7 : (B) Let height of triangle be a at time t, area


B= = inside the magnetic field
2R 4 πR
1 2 (a – vt)2
φ=B.A=
– µ 0 ωe
. pr =
2
– µ0 ωer 2 A=
2
( a – vt ) ( a – vt ) =
πR 4R 3 3
2 2 dA
–dφ µ0 er dw µ0 er α ∈=– B
∈= = = dt
dt 4R dt 4R
d (a – vt)2
iR=– B
dA BdA dt 3
Sol 2 : (C) e= B
⇒ iR=
dt dt
B 2
dQ BdA B i= . (a – vt).v
R. = ⇒ dQ= dA R 3
dt dt R
2BV 2BVa 2BV 2
B i= (a – vt)= –
⇒DQ= DA R 3 R 3 R 3
R
i=C1 – C2t
DA=2A (as it is rotated by 180º)
2AB
∴DQ=
R
P hysi cs | 22.73

Sol 8 : (B) A=A0 cosq Sol 13 : (A) a=Blv


θ=wt Q
=– BLv
φ=nBA c
Q=BlVC=4 × 1 × 20 × 10 × 10–6=800 µC
φ=nBA0 cos wt
Phas greater potential than Q as
–dφ
e= =nωBA0 sin wt
dt [V × B] is directed towards P
e nωBA0 Hence qA is the
i= = sin wt
R R \qA=+800 mc qB=– 800 mc
nωBA0
Amplitude=
R Sol 14 : (D) it is independent or resistance
ω=100 (2π)=200 p
e=qvB
⇒Amplitude
e=q.rωB
100 × 200π × 10 × 10 –3 × π × (10 –1 )2
= = 2A
10  – 
Rt
Sol 15 : (B, D) i= i0  1 – e L 
di  
Sol 9 : (A) e= –L  
dt V
i 0=
⇒ – ∫ e.dt=Li
∈ .dt R

d i0 is same,
⇒ e= – BAN
dt ⇒ R1=R2

∫ ∈e at
dt=BAN
Time constant t=
L
R
⇒ Li=BAN
tC>tB
i=BAN/L
⇒ L2> L1
1
Sol 10 : (B) B ∝ φ φ t ∈
L Sol 16 : (B) = . = × t=it=Q
R t R R
A ∝ l2 Here charge (B)
2
∴L ∝
L Sol 17 : (A, B, D) For both P, Q it is induced inward
hence clockwise.
Sol 11 : (A) Induced current is along DC for loop DC. iR=0 which is obvious
For loop AB it should be along AB but since area of CD
loop is greater than AB loop, hence current is along BA.
Sol 18 : (A) If i increases B increases, to reduce B, they
(A) repel

–dB
e=
dt
( ACD – A AB ) Sol 19 : (B, C) Antilockwise means field should increase
into plane.
∴A cos DC
Sol 20 : (A, B) Magnetic force fm=iB
Sol 12 : (B, D) Opposite currents (anti parallel currents)
Gravity force fg=mg
repel
fm cos θ=mg sin q
Hence (D)
ilB=mg Tan q
I2 induces opposite current to oppose the increase flux
is show from E to F.
(B)
2 2 . 7 4 | Electromagnetic Induction and Electromagnetic Waves

Sol 21 : (B) fm would be along rats \f1=(L2 + l2)B


fm=mg sin q Case II : A=L2 – l2
∴B=mg sin q \d=(L2 – l2)B
Sol 31 : (C) Clockwise current is induced
Sol 22 : (A, B, C, D)
Current from  to c and b to a
B is along the plane of ring Hence, it cannot t be
induced, irrespective of directions of motion
Sol 32 : (B) Clockwise overall current

Sol 23 : (A, C) \f toe, b to a

Its common knowledge Reason will be taught in higher


Sol 33 : (B) f2<f1
classes.
∴I2< I1
Sol 24 : (D) Induced current is anti-clockwise hence i2
along dc, i, along ab
Sol 34 : (A) Di-electric ring which is uniformly charged
i1=i1 since there are in same wire has stationary any charges. Hence time independent
electrostatic field out of system
Sol 25 : (B, D) Assume mA=mB A→P
Then iA>iB (hA>hB) (B) Rotating charge produce magnetic field within the
⇒ PA> PB system, and hence induced electric field. But outside
remains unchanged as it is
Now is mA<mB and PA> PB then surely hA>hB
Di-electric

Sol 26 : (C) Opposite current will induce in the upper B → P, Q, S


ring and it will get repelled by the coil at the bottom (C) Current in a ring produces magnetic field hence
induced electric field
Sol 27 : (B, D) C → Q, S
V α horizontal displacement (D) Current carrying ring has magnetic field and
\VQP=VPO, VPO=VRO induced electric field.
\VQ= – VP=VP – VO, VP=VR>V0 µ=Ipr2
µ=pr2I0 cos wt
Sol 28 : (A, C) d∈=Bvdr
µ charges with time,
de=Bω rdr
⇒ magnetic moment charge Q, R, S
B ωr 2
⇒ e=
2
Bωa2 Previous Years’ Questions
VP–V0=
2
Sol 1 : (C) In uniform magnetic field, change in magnetic
Bw(2a)2
VQ – V0= =2Bwa2 flux is zero. Therefore, induced current will be zero.
2
\correct answer is (c)
Vp – VR=0

Sol 2 : (D)
Sol 29 : (D) No current flows. As it doesn’t form a
closed circuit. Cross × magnetic field passing from the closed loop is
increasing. Therefore, from Lenz’s law induced current
Sol 30 : (C) φ= ∫ B.dA will produce dot • magnetic field. Hence, induced
current is anticlockwise.
Case I : A=L2 + l2
P hysi cs | 22.75

Sol 3 : (C) Sol 6 : (D)


From conservation of energy,
1 2 1
LI = CV2
2 max 2
C
∴ Imax=V
L

L=2pR Sol 7 : (C)


L Comparing the LC oscillations with normal SHM, we get
\R=
2π d2Q
=– w2Q
2T sin(dθ)=Fm dt 2

From small angles, sin(dθ)=dq 1 d2Q


Here, ω2= ∴ Q=– LC
LC dt2
\2T(dθ)=I(dL)B sin 90°= I(2R. dθ) . B
ILB Sol 8 : (A, C, D) From Faraday’s law, the induced voltage
∴ T=IRB=

 di 
∴Correct option is (c) V ∝ L, if rate of change of current is constant  V = –L 
 dt 
V2 L2 2 1 V1
Sol 4 : A → r, s, t ; B → q, r, s, t ; ∴ = = = or =4
V1 L1 8 4 V2
C → q, p ; D → q, r, s, t
Power given to the two coils is same, i.e.
In circuit (p) : I can’t be non-zero in steady state.
i1 V2 1
In circuit (q) V1i1=V2i2 or = =
i2 V1 4
V1=0 and V2=2I=V (also)
1
In circuit (r): V1=XLI=(2pfL) I Energy stored W= Li2
2
= (2π × 50 × 6 × 10–3) I=1.88 I 2
W2 L   i2  1 W1 1
V2=2I ∴ =  2    =   (4)2or =
W1  L1   i1  4 W2 4
In circuit (s): V1=XLI =1.88 I

 1   1  Sol 9 : (B, C)
V2=XCI=  I =  l=(1061) I
2 πfC –3 
   2π × 50 × 3 × 10  2
 1 
Z= R 2 + X 2C = R 2 +  
In circuit (t):  ωC 
V1=IR=(1000) I In case (b) capacitance C will be more. Therefore,
V2=XCI=(1061) I impedance Z will be less. Hence, current will be more.
Therefore the correct options are as under ∴ Option (B) is correct
(A) → r, s, t ; (B) → q, r, s, t ; Further,
(C) → q, p; (D) → q, r, s, t VC= V 2 – VR2 = V 2 – (IR)2

Sol 5 : (B) In case (b), since current I is more.


Charge on capacitor at time t is Therefore, VC will be less.
q=q0(1 – e–t/ τ ) ∴ Option (C) is correct
Here, q0=CV and t=2t ∴Correct options are (B) and (C)
∴ q=CV(1 – e–2τ / τ )=CV(1 – e–2)
2 2 . 7 6 | Electromagnetic Induction and Electromagnetic Waves

Sol 10 : (7) Sol 15 : (8)


Assume circular wire loop as primary and square loop At t=0, current will flow only in 12 Ω resistance
as secondary coil 5
∴ Imin =
2µ0iR 2 12
=
φsec ondary × a2 × cos 45o
2(3R 2 + R 2 )3 / 2 At t → ∞ both L1 and L2 behave as conducting wires

µ0iR 2 2 3
= × a2 × ∴ R e ff =
2 × 8R 3 2 2

10
φsec ondary µ0 a2 Imax =
=M = 3
i 23 × 21/2 R
Imax
µ0 a2 =8
M= Imin
7/2
2 R
1mH 3

Sol 11 : (A, C)
Total flux associate with loop=0 2mH 4
Therefore emf=0 in any case.

Sol 12 : (B) 12


For direct transmission
P = i2R = (150)2 (0.4 × 20) = 1.8 × 105 W
5V
1.8 × 105
Fraction (in =
%) × 100
= 30 %
6 × 105 S

40000 Sol 16 : (C, D)


Sol 13 : (A) = 200
200
For right edge of loop from x=0 to x=L
Sol 14 : (A, D) vBL
i= +
R
o o
45 45
vB2L2
h=10 cm 1 F iLB
= = (leftwards)
dr R

dv vB2L2
−mv =
dx R
h
µ0 I µ0 Ih B2L2
∴ v(x) =
v0 − x
=
φ w ∫ =
2πr
2rdr
π mR
0
v 0BL B3L3
i(x)
= − x
µ 0h R mR 2
So, Mutual inductance Mw =
π
v 0B2L2 B 4L4
µ0h di µ0 F(x)
= − x (leftwards)
∴ =
εw = R mR 2
π dt π
Due to rotation there is no change in flux through the
wire, so there is no extra induced emf in the wire. From
Lenz’s Law, current in the wire is rightward so repulsive
force acts between the wire and loop.
2017-18 100 &
op kers
Class 12 T
By E ran culty
-JE Fa r
IIT enior emie .
S fP r es
o titut
Ins

PHYSICS
FOR JEE MAIN & ADVANCED
SECOND
EDITION

Exhaustive Theory
(Now Revised)

Formula Sheet
9000+ Problems
based on latest JEE pattern

2500 + 1000 (New) Problems


of previous 35 years of
AIEEE (JEE Main) and IIT-JEE (JEE Adv)

5000+Illustrations and Solved Examples


Detailed Solutions
of all problems available

Topic Covered Plancess Concepts


Tips & Tricks, Facts, Notes, Misconceptions,
Alternating Current Key Take Aways, Problem Solving Tactics

PlancEssential
Questions recommended for revision
23. A LT E R N AT I N G
CURRENT

1. INTRODUCTION
A majority of electrical power in the world is generated, distributed, and consumed in the form of 50-Hzor60-Hz
sinusoidal alternating current (AC) and voltage. It is used for household andindustrial applications.
AC has several advantages over DC. The major advantage of AC is the fact that it can be transformedinto any form,
whereas direct current (DC) cannot. A transformer permits voltage to be stepped up or down for the purpose of
transmission. Transmission of high voltage (in terms of KV) implies that less current is required to produce the same
amount of power. Less current permits thinner wires to be used for transmission.
In this chapter, we will introduce a sinusoidal signal and its basic mathematic equation. We will discuss and analyse
circuits where currents i(t) and voltages v(t) vary with time. The phasor analysis techniques will be used to analyse
electronic circuits under sinusoidal steady-state operating conditions. The chapter will conclude with single-phase
power.

2. SINUSOIDAL WAVEFORMS
AC, unlike DC, flows first in one direction, then in the
opposite direction. The most common AC waveform is a RMS value
sine (or sinusoidal) waveform. Peak value

In discussing AC signal, it is necessary to express the


current and voltage in termsof maximum or peak values, Peak-to-Peak
peak-to-peakvalues, effective values, average values, or
instantaneous values. Each of these values has a different
meaning and is used to describe a different amount of
current or voltage. V(t)=V0 sin wt. Where Vo is the peak
voltage, ω = 2π f is the angular frequency expressed in
radian per second (rad/s), f is the frequencyexpressed in
Hertz (Hz), t is time expressed in second (s). 1 cycle

Figure 23.1: Sinusoidal Waveform.


2.1 Instantaneous Value
The instantaneous value of an AC signal is the value of voltage or current at one particular instant. The value may
be zero, if the particular instant is the time in the cycle at which the polarity of the voltage is changing. It may also
be the same as the peak value, if the selected instant is the time in the cycle at which the voltage or current stops
increasing and starts decreasing. There are actually an infinitenumber of instantaneous values between zero and
the peak value.
2 3 . 2 | Alternating Current

PLANCESS CONCEPTS

It is always advisable to find symmetries in functions while calculating rms and average value to reduce
the period of integration.It helps a lot in avoiding unnecessary calculations when functions are defined
part by part.
Yashwanth Sandupatla (JEE 2012, AIR 821)

2.2 Average Value t2

∫ fdt t2
t1
Average value of a function,from t1 to t2, is defined as <f>=
t2 − t1
. We can find the value of ∫ fdt graphically if the
graph is simple. It is the area of f-t graph from t2 − t1 . t1

∫ idt
0
Iavg =
t
,where i is the instantaneous value of the current.

∫ dt
0

2.2.1 For Sinusoidal Variation of Current and Voltages


t

∫ io sin ( ωτ + θ) dt
Case I: Average value over complete cycle 0 t
. Similarly Vavg = 0

∫ dt
0
t/2

∫ io sin ( ωτ + θ ) dt
0 2io 2 io
Case II: Average value=
over half cycle Iavg = ; Similarly Vavg =
t/2 π π
∫ dt
0

Illustration 1: An electric heater draws 2.5 A current from a 220-V, 60-Hz power supply. Find  (JEE MAIN)
(a) The average current
(b) The average of the square of the current
(c) The current amplitude
(d) The supply voltage amplitude

Sol: In AC circuit, the average value of current over a long time interval is zero but I2 is not zero. The r.m.s. value of
Imax Vmax
current and voltage is=
given by Irms = and Vrms .
2 2
(a) The average of sinusoidal AC values over any whole number of cycles is zero.
P hysi cs | 23.3

(b) RMS value of current = Irms =2.5 A so, =


I2 ( ) av
(=
Irms )
2
6.25 A2

Im
( c ) Irms = ; So, current amplitude I =
m
= 2 Irms 2 ( 2.5A ) 3.5 A
=
2

Vm
( d) = =
Vrms 220V ; So, supply voltage amplitude
= V m 2 ( Vrms )
= ( 220V ) 311 V.
2=
2

2.3 Effective Value (RMS Value)


This is the value of AC signal that will have the same effect on a resistance as a comparable value of direct voltage
or current will have on the same resistance. It is possible to compute the effective value of a sine wave of current to
a good degree of accuracy by taking equally spacedinstantaneous values of current along the curve and extracting
the square root of the average of the sum of the squared values. For this reason, effective value is sometimes called
RMS value. t 2

∫ f 2dt
t1
Root mean square value of a function, from t1 to t2 is defined as frms =
t2 − t1
T T

∫ I2dt ∫ I02 sin2 ( ωτ ) dt


I20
The magnitude of Irms is given by
= 2
Irms 0
= 0
=
T T 2
∫ dt0 ∫0 dt

I0 V0
I= =
eff Irms = 0.707 I0 =
Where I0 is the peak value of the current. Similarly Ve f f or Vrms = 0.707 E0 A
2 2

PLANCESS CONCEPTS

RMS value is actually more important because in the context of power transmission,the loss in energy
due to a resistor plays an important role. And the power is given by i2R, where R is the resistance.
Yashwanth Sandupatla (JEE 2012, AIR 821)

π π 3π
Illustration 2:Find the RMS value of current I = Im sin ω t from (i) t=0 to t= (ii)t= to t =  (JEE MAIN)
ω 2ω 2ω

Sol: In AC circuit over time interval 0 ≤ t ≤ T the RMS value of current is given by

T T

∫ I2dt ∫ I20 sin2 ( ωτ ) dt


0 0 I0 2π
=
Irms = = where T =
T T
2 ω
∫ dt 0 ∫0 dt
2 3 . 4 | Alternating Current


π 2π


2
ω Im sin2 (ωt)dt
∫ Im2 sin2 (ωt)dt 2
Im I
π 2
Im Im
(i) =
Irms 0
= = m (ii) =
Irms 2
= = A
π 2 2 π 2 2
ω ω

PLANCESS CONCEPTS

The RMS value of one cycle or half cycle (either a positive or negative cycle) is same.
GV Abhinav (JEE 2012, AIR 329)

2.4 Difference between Sine and Cosine Representation of AC Signal


The sine and cosine are essentially the same function, but with a 900 phase difference. For example, sin ωt =cos
( ωt − 90 ) . Multiples of 360 may be added to or subtracted from the argument of any sinusoidal function, without
0 0

changing the value of the function. To realize this, let us consider

=V1 VP1 cos 10t + 20


= (
0
)
VP1 sin 10t + 900 + 200 ( ) 
… (i)

(
= VP1 sin 10t + 1100 = )
Leads V2 VP2 sin 10t − 40
0
( )  … (ii)

by 1500. It is also correct to say that v1lags v2by 2100, since v1 may be written as

= (
V1 VP1 sin 10t − 2500 ) V … (iii)

v
VP

VPsin(t+) Vpsin t

t


-VP

Figure 23.2: Representation of voltage as sine and cosine function


P hysi cs | 23.5

3. POWER IN AC CIRCUITS t

∫ vidt
Average power in alternating current circuit over time t is defined as Pavg = 0
, where V andiare the
t

∫ dt
0

instantaneous values of voltage and current respectively. Let V= V0 sin ω


= ( )
t; i i0 sin ωt − φ , Average power over a cycle
T T
 1 
∫ v oio sin ωt.sin ( ωt − φ ) dr v oio ∫  sin2 ωt cos φ − sin2ω t sin φ  dt
0 0 
2  1
Pavg = ;= = V=
i cos φ Vrmsirms cos φ
T T 2 00
∫ dt
0
The term cos φ is known as power factor.
If the current leads voltage, it is said to be leading, whereas, if it lags voltage, it is said to be lagging. Thus, a power
factor of 0.5 lagging means the current lags voltage by 60o (as cos-10.5 = 60o). The product of Vrms and irms gives
the apparent power, while the true power is obtained by multiplying the apparentpower by the power factor cosφ

Thus, apparent power = Vrms × irms and true power=apparent power × power factor

For φ = 0o, the current and voltage are in phase. The power is thus, maximum (Vrms × irms). For φ = 90o the power is
zero. The current is then stated wattless. Such a case will arise when resistance in the circuits is zero. The circuit is
purely inductive or capacitive. The case is similar to that of a frictionless pendulum, where the total work done by
gravity upon the pendulum cycle is zero.
We shall discuss more about the power and power factor later, shortly after we define impedance and its properties.

Illustration 3: When a voltage Vs = 200 2 sin (ωt+15o) is applied to an AC circuit, the current in thecircuit is found
to be I=2 sin (ωt+π/4) then average power consumed in the circuit is  (JEE MAIN)

(A) 200 W (B) 400 2 W (C) 100 2 W (D) 200 2 W

Sol: Power in any AC circuit is calculated


= as Pav Vrms Irms cos φ where φ is phase angle between V and I.

=Pav Vrms=
Irms cos φ
200 2 2
2
. =
2
cos 300 ( )
100 6 W

4. SIMPLE AC CIRCUITS

4.1 Purely Resistive Load Vs = Vm sin t

Writing KVL along the circuit (see Fig. 23.3), VS iR=0



Vs Vm sin ωt i
Or =I = = Im sin ωt . R
R R
Figure 23.3: AC voltage applied to
We see that the phase difference between potential differences across resistive load
resistance, VR and iR is 0.
2 3 . 6 | Alternating Current

Vm Vrms Vrms2
Im = ⇒ Irms = <P> = Vrms Irms cos φ =
R R R

4.2 Purely Capacitive Vs = Vm sin t

Writing KVL along the circuit shown in Fig. 23.4



q i
VS − 0 And current in the circuit is
=
c C

dq d ( cv ) d ( cvm sin ωt ) Vm Figure 23.4: AC voltage


I = = = = cvmω sin ω=
t cos ωt.
dt dt dt 1 / ωC applied to capacitive load

Vm
= cos=
ωt Im cos ωt
XC
1 V-
T
where X C =
where and is called capacitive reactance. Its unit is Ohm ( Ω ).
ωC t
From the graph of current versus time and voltage versus time,
i
T
it is clear that current attains its peak value at a timebefore the
4
T t
time at which voltage attains its peak value. Corresponding to phase difference.
4
2π T π
= ω� t = = i leadsvcby π / 2 diagrammatically (phasediagram) represented as Figure 23.5
T 4 2 c

Since φ =900 ,<P>= Vrms Irms cos φ =0 Im


The current leads the voltage by π / 2 in a capacitive circuit
Vm
PLANCESS CONCEPTS
<P>=0 doesn’t mean it is zero in any period less than the time period. In actuality, first the capacitor
gets charged up, gaining energy during the first half cycle, and loses it for the next half cycle.So overall,
power becomes zero.Same goes for the inductor in a different fashion (magnetic field plays a role there).
Yashwanth Sandupatla (JEE 2012, AIR 821)

4.3 Pure Inductive Circuit


di di Vs = Vm sin t
Writing KVL along circuit, Vs − L
dt
=;L
0=
dt ∫ Ldi
Vm sin ωt;= ∫ Vm sin ωt dt ;

Vm
i =− cos ωt + C ; <i>=0 ; C=0; i
ωL
L
V Vm
∴i = − m cos ωt Im = From the graph of current versus time and voltage Figure 23.6: AC voltage
ωL XL applied to inductive load
T
versus time, it is clear that voltage attains its peak value at a time before the time
4
T 2π T 2π π
at which current attains its peak value. Corresponding to , the phase difference = ω� t = = =
4 T 4 T 2
P hysi cs | 23.7

V T
t

Figure 23.7: Variation of current and voltage with respect to time

Vm
Diagrammatically (See Fig. 23.7) it is represented as

Im
iL lags behind VL by π / 2 since φ =90 , <P>=Vrms Irms cos φ =0. The current lags voltage by π / 2 in a purely inductive
0

circuit.

Applied Induced
voltage voltage

Figure 23.8: AC voltage applied to purely inductive circuit

5. IMPEDANCE
We have already seen that the inductive reactance XL = ωL and capacitance reactance X=
C 1 / ωL play the role of
an effective resistance in apurely inductive and capacitive circuit respectively. In the series RLC circuit, the effective

( )
2
resistance is the impedance,defined as Z = R + XL − X C …(iv)

The relationship between Z, XL, and XC can be represented by the diagram shown in Fig. 23.9.
Following is a diagrammatic representation of the relationship between Z, XL and XC. 
Z
The impedance has SI unit of Ω. In terms of Z the current may be rewritten as I(t) XL - XC

V0
sin ( ωt − φ ) …(v)

=
Z R

Figure 23.9: Impedance


Notice that the impedance Z also depends on the angular frequency ω ,as do XL and XC.
Triangle
Using the above equations for phase φ and Z, we may readily recover the limit for
simple circuit (with only one element).
2 3 . 8 | Alternating Current

PLANCESS CONCEPTS

By now, students should get a clear idea of individual behaviour of inductor, capacitor and resistor and
be able to visualize phasors. They should never get confused whetherinductor, capacitor is leading, etc.
Chinmay S Purandare (JEE 2012, AIR 698)

The upcoming series of circuits would be easy to understand because they are just a superposition of individual
phasor diagrams.

6. MIXED AC CIRCUITS
6.1 LR Circuit
If VR, VL and Vsare the RMS voltage across are R, L and the AC source respectively. Then,

=
VS VR2 =
+ VL2 I2 R 2 + XL2 Where Is is r.m.s value of source current.

The total opposition to the current is called impedance and it is denoted by Z.


VS
Z= = R2 + X
=L
2
R 2 + ω2L2
IS

VL
R VR

V S VS
L VL

VR IS
(a) (b)

Figure 23.10: (a) AC voltage applied to LR circuit (b) Phasor diagram of voltage drops across R and L

−1 −1 X   ωL 
The phase angel φ by which the applied voltage leads the current
= is φ tan
= L
  tan  
R
   R 

Illustration 4: An alternating voltage of 220V RMS at a frequency of 40 cycles/second is supplied to a circuit


containing a pure inductance of 0.01 H and a pure resistance of 6Ω in series. Calculate (a) The current, (b) Potential
difference across the resistance, (c) Potential different across inductance, (d) The time lag. (JEE MAIN)

Vrms
R 2 + ( ωL ) . The RMS value of the current is Irms =
2
Sol: Theimpedance of LR circuit is=
Z . In LR circuit, the
Z
 ωL 
current lags the applied voltage by phase angleφ obtained as φ =tan−1  .
 R 
The impedence of the L-R series circuit is given by:
1/2 1/2
Z2= R 2 + ( ωL )
2  2 2
= (R ) + ( 2πfL ) 


P hysi cs | 23.9

1/2
= 62 + ( 2 × 3.14 × 40 × 0.01 ) 
2
= 6.504 Ω
 

V 220
(a) RMS value of the current: Irms==
rms
= 33.83 A
Z 6.504
(b) The potential difference across the resistance is given by: VR=Irms × R=33.83 × 6= 202.83 V
(c) Potential difference across the inductance is given by:
( )
VL = Irms × ( ωL ) = 33.83 × 2 × 3.14 × 0.01 = 96.83 V

 ωL 
(d) Phase angle φ =tan−1   ; so, φ = tan (0.4189=22.46)
-1

 R 

φ 22.46
Now time lag= = T= = 0.0623 s.
360 360

9
Illustration 5: A H inductor and a 12 Ω resistance are connected in a series to a 225 V, 50 Hz ac source.
100π
Calculate the current in the circuit and the phase angle between the current and the source voltage. (JEE MAIN)

Vrms
R 2 + ( ωL ) . The RMS value of the current is Irms =
2
Sol: Theimpedance of LR circuit is =
Z . In LR circuit, the
Z
 ωL 
current lags the applied voltage by phase angleφ obtained as φ =tan−1  .
 R 
9
Here XL= ωL =2 πfL = 2π × 50 × = 9Ω
100π

So, Z = R 2 + XL2 = 122 + 92 =15Ω

V 225 −1  XL  −1  9  −1
(a) l=
= and (b) φ tan
= 15A = = =
  tan =
  tan 3 / 4 370o
Z 15 R
  12
 
i.e., the current will lag the applied voltage by 370 in phase.

Illustration 6: A chokecoil is needed to operate an arc lamp at 160 V (RMS) and 50 Hz. The arc lamp has an
effective resistance of 5Ω when running of 10 A (RMS). Calculate the inductance of the choke coil. If the same arc
lamp is to be operated on 160V (dc), what additional resistance is required? Compare the power losses in both
cases.
 (JEE ADVANCED)

Sol: The choke coil is a LR circuit having large inductanceand small


Ark lamp
resistance. The potential difference across the resistor and inductor is added
vectorially:V2=VR2+VL2.
L R
As for the lamp, VR=IR = 10 × 5 = 50V , so when it is connected to 160 V ac source
VL VR
though a choke in series,V2=VR2+VL2, VL= 1602 − 502 =
152 V

VL
And as, VL=IXL=I ωL = 2πfLI L=
2πfI
=
152
2 × π × 50 × 10
=4.84 ×10 −2 H 
V = V0sin t

Now the lamp is to be operated at 160 V dc; instead of choke, if additional Figure 23.11
resistance r is I put in a series with it, V = I(R+r), i.e. 160 = 10(5+r) i.e. r = 11Ω In
case of AC, as choke has no resistance, power loss in the choke will be zero, while
2 3 . 1 0 | Alternating Current

the bulb will consume P=I2 R=102 × 5=500 W. However, in case of DC,as resistance r is to be used instead of choke,
the power loss in the resistance r will be PL=102 X 11=1100 W
While the bulb will still consume 500 W, i.e., when the lamp is run on resistance r instead of choke, more than
double the power consumed by the lamp is wasted by the resistance r.

6.2 RC Circuits
VR
IS
R VR 

V S VC
VS
C -
- VC

(a) (b)

Figure 23.12: (a) AC voltage applied to RC circuit (b) Phasor diagram of voltage drops across R and C

If Vs, VR and VC are RMS voltages across a source, resistance and capacitor respectively

VS = VR2 + VC2 =IS = R 2 + X 2C

VS 1
Impedance of circuit, Z= = R 2 + X 2C = R2 +
IS 2 2
ωC

−1 −1 X   1 
VS leads=
IS by φ tan
= 
C
 tan  
 R   ωCR 
The current leads the applied voltage by angel φ .

Illustration 7:An ac source of angular frequency ω is fed across a resister R and a capacitor C in series. The current
registered is I. If now, the frequency of source is changed to w/3 (but maintaining the same voltage), the current in
the circuit is found to be halved. Calculate the ratio of reactance to resistance at the original frequency w.
 (JEE MAIN)
Sol: The impedance of RC circuit is:

2
 1  Vrms
=
Z R2 +   . The RMS current is Irms =
 ωC  Z

V V
According to the given problem, I= = … (i)
Z  2 2
1/2


R + ( 1 / C ω ) 

ω I V
And for frequency of , = … (ii)
3 2  2 2
1/2


R + ( 3 / C ω ) 

Substituting the value of I from equation (i) in (ii),

 1  9 1 3
4  R2 + R2 +
= i.e.. = R2
2 2  2 2 2 2 5
 C ω  C ω C ω
P hysi cs | 23.11

1/2
3 2
 R 
X (1 / cω )  5  3
So that,
= = =
R R R 5

Illustration 8: In an RC series circuit, the RMSvoltage of source is 200V, and its frequency is 50 Hz. If R = 100 Ω
100
and
= C µF , find
π 220V50HZ
(a) Impedance of the circuit (b) Power factor angle 
(c) Power factor (d) Current
(e) Maximum current (f) Voltage across R
C R
(g) Voltage across C (h) Max voltage across R Figure 23.13
(i) Maxvoltage cross C ( j)<P>
(k) <PR> (l) <PC> (JEE ADVANCED)

Sol: The impedance of RC circuit is

2
2  1 
=Z R + 
 ωC 

Vrms XC
The RMS current is Irms = . The phase angle between current and voltage is given by tan φ = . The RMS
Z R
I0 V0
value of current and voltage
= is Irms = and Vrms =
. Power developed in circuit is P Vrms Irms cos φ .
2 2

106
=
XC = 100 Ω
100
π
( )
2 π50

Z = R 2 + XC2 = 1002 + (100 ) = 100 2Ω


2
(a)

XC
(b) tan=
φ = 1 φ 450
∴=
R
1
(c) Power factor= cos φ =
2

Vrms 200
(d) Current=
Irms = = 2A
Z 100 2

(e) Maximum current =Irms 2 = 2 A

(f) Voltage across R=VR,rms=Irms R= 2 × 100 V

(g) Voltage across C= VCrms = IrmsXC = 2 × 100 V

(h) Max voltage across R= 2 VR,rms=200 V

(i) Max voltage across C= 2 VCrms=200 V


2 3 . 1 2 | Alternating Current

1
( j) <P>= Vrms Irms cos=
φ 200 × 2 × W
2
(k) <PR>Irms 2R=200 W

(l) <Pc>=0

PLANCESS CONCEPTS

We observed here that inductor’s reactance is directly proportional to the frequency used in the circuit
and vice-versa for capacitor. So a combined circuit of them can be used as a frequency filter. High
frequencies can be received by noting the voltage across capacitor and low frequencies can be noted
using the inductor.
Nitin Chandrol (JEE 2012, AIR 134)

Vs = V sint XL
 V
V
90
o I
I
XC
L C
(a) (b)
Figure 23.14: (a) AC voltage applied to LC circuit (b) Phasor diagram for voltage drops across L and R

6.3 LC Circuits

( )
From the phasor diagram V = I XL − X C = I Z; φ =900

6.4 RLC Circuits

VR2 + ( VL − VC )
2
For LCR series circuits Vs =

2
2  1 
R + ( XL − X C )
2 2
Impedance of circuits Z = = R +  ωL − 
 ωc 

 1 
 X − X   ωL − 
VS leads IS by φ =tan−1  L −1 ωc 
 = tan 
C

 R   R 
 
 
R
Power in LCR circuit= Vrms=
Irms cosf V=
rms Irms VR Irms
Z
P hysi cs | 23.13

VL -
R VR

VL-VC
VS
VS L VL

VR IS
C VC VC-

(a) (b)

Figure 23.15: (a) AC voltage applied to LCR circuit. (b) Phasor diagram of voltage drops across L, C and R

Where cos φ is called the power factor of the LCR circuit.

6.4.1 Resonance in RLC Circuits

1
At a particular angular frequency ωο of the source, when XL = X C or ωο L= , the impedance of the circuit
ω0 C
becomes minimum and equal to R and therefore, the current will be maximum. The circuit is then said to be

in resonance. The resonance angular frequency ωο andfrequency V0 given by


1 1
ω0= .ν0=
2 2π LC
The variation of RMS current with the frequency of the applied voltage is shown
in the Fig. 23.16. If the applied voltage consists of a number of frequency Irms
components, the current will be large for the components having frequency V0.
ω0L
The Q factor of an LCR series circuit is given by Q= . A direct current of a
R
flows uniformly throughout the cross-section of theconductor. An alternating

current on the other hand, flows mainly along the surface of the conductor.
This effect is known as the skin effect. The reason is that when ac flows through
V0 V
aconductor, the flux change in the inner part of the conductor is higher.
Figure 23.16

PLANCESS CONCEPTS

The idea of resonance is used in TV channelsfor clarity: a particular frequency is assignedto a channel
and when this frequency is received by the receiver,the current corresponding to this frequency becomes
maximum. This helps in maximum possible separation of channels, thus increasing their individual clarity.
It is also used by intelligence agencies to intercept the signals of anti-social elements. They generally use
frequency of a very high order.
Nivvedan (JEE 2009, AIR 113)
2 3 . 1 4 | Alternating Current

Illustration 9: In the circuit shown in the Fig. 23.17, find  (JEE MAIN)
(a) The reactance of the circuit 200V, 50Hz
(b) Impedance of the circuit 
2 100
(c) The current H 100  F
(d) Reading of the ideal AC voltmeters
(These are hot wire instruments and read RMS values) V1 V2 V3
V4

R 2 + ( X C − XL )
2
Sol: In series LCR circuit, the impedance is Z = where XC V5

and XL are the capacitive reactance and inductive reactance respectively. Figure 23.17

2 1
(a) XL = 2πfL = 2π × 50 × = 200Ω XC= =100 Ω
π 100 −6
2π50 × 10
π
∴ The reactance of the circuit X=XL-XC=200-100=100 Ω

Since XL>XC, the circuit is called inductive.

(b) Impedance of circuit Z= R 2 + X=


2
1002 + 100=
2
100 2 Ω

vrms 200
(c) The current =
Irms = = 2A
Z 100 2
(d) Readings of ideal voltage

V1 : Irms XL = 200 2 V

V2 : IrmsR = 100 2 V

V3 : Irms X c = 100 2 V

V4 : Irms R 2 + XL2 =
100 10 V
, which also happens to be the voltage of source.
V4 : Irms Z = 200 V,

Illustration 10: A resistance R, inductance L and a capacitor C all are connected in series with ac supply. The
resistance of R is 16 Ω and for a given frequency, the inductive reactance of L is 24 Ω and capacitive reactance of C
is 12 Ω. If the current in the circuit is 5 amp, find: (JEE MAIN)
(a) The potential difference across R, L and C
(b) The impedance of the circuit
(c) The voltage of ac supply
(d) Phase angle

R 2 + ( X C − XL ) where XC and XL are the capacitive reactance and


2
Sol: In series LCR circuit, the impedance is Z =
 X − XC 
inductive reactance respectively. The phase angle between voltage and current is given by φ =tan−1  L  .
 R
 
P hysi cs | 23.15

(a) Potential difference across resistance: VR=iR = 5 × 16 =


80 V
( )
Potential difference across inductance: VL=i × ωL = 5 × 24 = 120 V
i (1 / ωC ) =5 × 12 =60 V
Potential difference across capacitor: Vc =×

  1  
2

(16 ) + (12) =
2 2
(b) Z= R 2 +  − ωL  = 20 Ω

  ωC  

(c) The voltage of ac supply is given by: V =IZ =5 × 20 =100 V

 ωL − ( 1 / ωc ) 
−1  24 − 12 
(d) φ =tan 

=
R
 tan−1=
 
16
=
 tan
−1
( 0.75) 360 46"
   

Illustration 11: An oscillating voltage drives an alternating current through a resistor, an R


inductor, and a capacitor that are all connected in series. Calculate the RMS voltage across Vrms
each another by multiplying the reactance or resistance of each element by the RMS current. 
To calculate the RMS current, divide the RMS voltage by the impedance.  (JEE ADVANCED)
L
lrms C

R 2 + ( X C − XL )
2
Sol: In series LCR circuit, the impedance is Z = where XC and XL are the Figure 23.18

capacitive reactance and inductive reactance respectively. The phase angle between voltage
 X − XC 
and current is given by φ =tan−1  L  . Find the current in the series circuit, and multiply the resistance or
 R
 
reactance of each element with the currrent to find the voltage drop across it.

1 1
1. Calculate XC;  X
= = =17.68 k Ω
C
ωC 2π ( 60.0Hz ) 0.15µF

( )( )
2. Calculate XL ; XL =ωL = 2π 60.0Hz 25mH = 9.42π Ω

3. Calculate the impedance:

R 2 + ( XL − X C=
) ( 9.9k Ω ) + ( 0.00942k Ω − 17.68k Ω)
2 2 2
=
Z = 20.25 K Ω

Vrms 115 V
4. Divide the voltage by the impedance: =
Irms = = 5.7 mA
Z 20.25 k Ω

= Irms=
5. Multiply the current by the resistance: Vrms.R R 5.68 mA(9.9k =
Ω) 56 V

= Irms=
6. Multiply the current by the inductive reactance: Vrms.L XL 5.68 mA( 9.42k=
Ω) 54 V

7. Multiply the current by the capacitive reactance:

= Irms=
Vrms.C X C 5.68 m A(17.68k =
Ω ) 100V
= 0.10 KV
2 3 . 1 6 | Alternating Current

6.5 Parallel RCL Circuits


Consider the parallel RLC circuit illustrated in Fig. 23.19. 
V(t) R L C
The voltage source is V (t) =V0 sin ωt .
Unlike the series RLC circuit, the instantaneous voltage acrossall three circuit
elements R, L, and C are the same, and each voltage is in phase with the current
through the resistor. However, the current through each element will be different. Figure 23.19 Parallel LRC circuit

In analysing this circuit, we make use of the results derived before. The current
V(t) V0
in the resistor is IR (t)= = = sin ωt= IR0 sin ωt  … (i)
R R
dIL
Where IR0 = V0 /R. The voltage across the inductor is VL (t)
= V(t)
= V0 sin ω=
t L … (ii)
dt 
t
V0 V0 V0  π  π
which gives=
IL (t) ∫L sinωt'dt'
=
ωL
=
cos ωt
XL
sin  ωt −=

 IL0 sin  ωt −  
2  2
… (iii)
0

where IL0 = V0 / XL andXL = ωL is the inductive reactance.

Similarly, the voltage across the capacitor is VC (t)=V0 sin ωt =Q(t)/c, which implies

dQ V  π  π
IC (t) = =ωCV0 cos ωt = 0 sin  ωt +  =IC0 sin  ωt +   … (iv)
dt XC  2  2

= V0 / XC and X=
where IC0 C 1 / ωL is the capacitive reactance.

Using Kirchhoff’s junction rule, the total current is simply the sum of all three currents.

I(t) = IR (t) + IL (t) + Ic (t) = IR0 sin ωt + IL0 sin  ωt − π  + IC0 sin  ωt + π   … (v)
 2  2

The current can be represented with the phasor diagram shown in Fig. 23.20


ICO

  
I0 ICO + ILO

 
IRO V0

ILO

Figure 23.20: Phase difference between current and voltage

   
From the phasor diagram, we see that. I0 = IR0 + IL0 + IC0  … (vi)

And the maximum amplitude of the total current, I0 , can be obtained as


P hysi cs | 23.17

2 2
      1  1 1  1 1 
I0 = I0 = IR0 + IL0 + IC0 = I2R0
+ (Ic0 − I=
L0 ) V0
2
+  ωC − =  V0 + −  … (vii)
ωL  
 X C XL  
2 2
R  R
Note however, since IR(t), IL (t) and IC(t) are not in phase with one another, I0is not equal to the sum of the maximum
amplitudes of the three currents: I0 ≠ IR0 + IL0 + IC0  … (viii)
With I0 = V0 /Z, the (inverse) impedance of the circuit is given by:
2 2
1 1  1  1  1 1 
= +  ωC − = + −   … (ix)
Z R 2
 ω L R 2  X C XL 

The relationship between Z, R, XL and XC is shown in Fig. 23.21 which shows a relationship between Z, R, XL and XC
in a parallel RLC circuit.

1/Z 1 -1
XC XL

1/R

Figure 23.21: Impedance triangle


V0 V0

 IC0 − IL0  X C XL V V 
From the phasor diagram, we see that the phase can be obtained as: tan
= φ  =  = R 0 − 0 
 I  V0 X 
 R0   C XL 
 π 
= R  ωt −  R … (x)
 2

The resonance condition for the parallel RLC circuit is given by φ =0, which implies:
1 1
=  … (xi)
X C XL
1
The resonant frequency is: ω0 =  … (xii)
LC
which is the same as for the series RLC circuit. From Eq. (xii), we readily see that 1/Z is minimum (or Z is maximum)
at resonance. The current in the inductor exactly cancels out the current in the capacitor, so that the total current
V0
in the circuit reaches minimum, and is equal to the current in the resistor: I0 =  … (xiii)
R

As in the series RLC circuit, power is dissipated only through the resistor. The average power is

V02 V02 V02 Z


P(t) = IR (t)V(t) = IR2 (t)R = sin2 ωt = = =   … (xiv)
R 2R 2Z  R 

Thus, the power factor in this case is

P(t) Z 1
Power factor = = = = cos φ  … (xv)
V02 / 2Z R  R 
2
1 +  RωC − 
 ω L
2 3 . 1 8 | Alternating Current

Illustration 12: The image shows an inductor (L=0.22 mH) in series with a 15 Ω L R
resistor. These elements are in parallel with a second 15 Ω resistor. An AC generator
powers the circuit with an RMS voltage of 65V. R
In the limit of high frequency, the inductor behaves like a very large resistor. In such a
case, nearly all of the current flows through the branch with the lone resistor. Calculate
the current by dividing the RMS voltage by the single resistor. 
Figure 23.22
In the limit of low frequency, the reactance of the inductor approaches zero.
In such a case, the current flows through each resistor equally. Calculate the
equivalent resistor and divide the voltage by the equivalentresistance to determine the current.(JEE ADVANCED)

Sol: For very high source frequency, the reactance of the inductor becomes practically infinite so that the current
doesn’t flow through the inductor. Thus, the inductor acts as an open circuit. For very low source frequency, the
reactance of the inductor becomes practically zero, and theinductor behaves as a short circuit.
Vrms 65 V
1. Calculate the current at high frequency: =
Irms = = 4.3 A
R 15 Ω
−1
1 1 R 15Ω
2. Calculate the equivalent resistance at low frequency: R eq =
 +  == = 7.5Ω 3.
R R  2 2
Vrms 65V
Divide the voltage by the equivalent resistance: =
Irms = = 8.7 A
R eq 7.5Ω

Illustration 13: For the circuit shown in Fig. 23.23, current in inductance is 0.8 A while its capacitance is 0.6A. What
is the current drawn from the source? (JEE ADVANCED)

IL
C
I IC

Figure 23.23

Sol: For LC circuit, total current in the circuit= ( )


is I I0 sin ωt + φ = IL + IC. The current in the inductor lags the applied
π π
voltage by phase difference of while in capacitor, the current leads applied voltage by In parallel ac circuit,
2 2
=V V0 sin ωt is applied across both the inductorand capacitor, current in inductor lags the applied voltage while
current in capacitor leads the applied voltage.

V  π V  π
So, IL= ; IC
sin  ωt −  =−0.8 cos ωt= sin  ωt += 0.6 cos ωt
XC  2 XC  2

So, the current drawn from the source, I =IL + IC =−0.2cos ωt , i.e. I0 = 0.2 A

7. MORE ON POWER FACTOR


(a) The factor cos φ present in the relation for average power of an ac circuit is called power factor.
Pac Pavg
So, cos φ = = . Thus, ratio of average power and virtual power in the circuit is equal to power factor.
Erms Irms pV
P hysi cs | 23.19

(b) Power factor is also equal to the ratio of the resistance and the impedance of the Z
ac circuit.
1
R
Thus, cos φ = C
Z 
(c) Power factor depends upon the nature of the components used in the circuit.(d) If 0 R
a pure resistor is connected in the ac circuit then, Figure 23.24
E0 I0 E02
=φ 0,cos
= φ 1 ; p=
av = = Erms Irms
2 2R
Thus, the power loss is maximum and electrical energy is converted in the form of heat.
(e) If a pure inductor or capacitor are connected in the ac circuit, then
φ ≠ 90o , cosφ = 0 ∴ Pav =0 (minimum)
Thus is no loss of power.
(f) If a resistor and an inductor or a capacitor are connected in an ac circuit, then φ ≠ 0 or φ ≠ 90o. Thus φ is in
between 0 & 90o.
(g) If the components L, C and R are connected in series in a circuit, then

X ( ωL − 1 / ωC ) R R R
=
tan φ= = and cos φ= = ; Power factor cos φ
R R Z  2 2
1/2 Z
(
R ωL − 1 / ωC  )
(h) Power factor is a unit less quantity.

(i) If there is only an inductance coil in the circuit, there will be no loss of power, and energy will be stored in the
magnetic field.

( j) If a capacitor is only connected in the circuit, there will also be no loss of power, and energy will be stored in the
electrostatic field.

(k) In reality, an inductor and capacitor do have some resistance. So, there is always some loss of power.
(l) In the state of resonance, the power factor is one.

8. WATTLESS CURRENT
(a) The component of current whose contribution to the average power is nil, is called wattless current.
(b) The average wattle of power iszero because the average of the second Irms cos
component of instantaneous power for a full cycle will be 0

E

E0 sin ωt (l0 sin φ ) sin ( ωt − π / 2 ) = 0



X
(c) The component of current associated with this part is called Wattless Z
current. Thus the current

(l0 sinφ) sin ( ωt − π / 2) is a wattless current whose amplitude is l0 sinφ . Irms sin Irms

Figure 23.25
(c) If RMS value of current in the circuit is Irms, then the RMS value of a
wattless current will be Irms, sinφ . A wattless current lags or leads the e.m.f.
by an angle π / 2 . RMS value of wattless current:
2 3 . 2 0 | Alternating Current

l0 l0 X X
Irms=
sin φ sin φ ; = . Since sin φ = , where X is the resultant reactance of the circuit.
2 2Z Z

9. TRANSFORMERS
A transformer is a device used to convert low alternating voltage at higher current into high alternating voltage
at lower current, and vice-versa. In other words, a transformer is an electrical device used to increase or decrease
alternating voltage.

9.1 Types of Transformers


(a) Step-up transformers: The transformerwhich converts low alternating voltage at higher current into a high
alternating voltage at lower current is called a step-up transformer.
(b) Step-down transformers: The transformer which converts high
alternating voltage at lower current into a low alternating voltage
athigher current is called a step-down transformer. EP P S ES Load

Principle: A transformer is based on the principle of mutual induction.


An e.m.f. is induced in a coil, when a changing current flows through its
Core
nearby coil.
Figure 23.26
Construction: Itconsists of two separate coils of insulated wires wound
on the same iron core. One of the coil connected to a.c. input is called primary (p) and the other winding giving
output is called secondary (S) winding or coil.
Theory: When an alternating source of e.m.f. Ep is connected to the primary coil, an alternating current flows
through it. Due to the flow of alternating current in the primary coil, an alternating magnetic flux induces an
alternating e.m.f. in the secondary coil (Es). Let Np and Ns be the number of turns in the primary and secondary coil
respectively. The iron core is capable of coupling the whole of the magnetic flux φ produced by the turns of the
primary coil with the secondary coil.
According to Faraday’s law of electromagnetic induction, the induced e.m.f in the primary coil,

EP = −NP  ... (i)
dt

The induced e.m.f in the secondary coil. ES = −NS ... (ii)
dt 
ES NS NS
Dividing (ii) by (i), we get = ; Where =K the transformation ratio or ratio.
Ep NP NP
ES NS
Then, = =K
Ep NP

K< 1 for step down transformer. In this case, NS < NP and ES < Ep i.e. Ep , and output alternating voltage <input
alternating voltage.

K>1 for step up transformer. In this case, NS > NP and ES > Ep i.e., output alternating voltage is greater than the
input alternating voltage.
For an ideal transformer (in which there in no energy losses), output power= input power .... (iii)
Let Ip and Is be the current in the primary and secondary coil respectively.
Es IP 1
Then output power= Es IS ; input power= Ep Ip ; from equation (iii) Ep = Es or = ; In general, E ∝ . For
Ep IS I
same power transfer, voltage increases with the decrease in current and vice-versa. Thus, whatever is gained in
voltage ratio is lost in the current ratio and viceversa. So, astep-up transformer increases the alternating voltage by
P hysi cs | 23.21

decreasing the alternating current, and a step- down transformer decreases the alternating voltage by increasing
the alternating current.

ouputpower Es IS
For a transformer, efficiency, n= = For an ideal transformer, efficiency, n is 100%. But in a real
inputpower EP IP
transformer, the efficiency varies from 90-99%. This indicates that there are some energy losses in the transformer.

10. CHOKING COIL

Let us consider a choke coil of large inductance L and low resistance R. Then, the power factor of the given circuit
R R
will be given by cos φ = = (as R<< ωL )
R 2 + ω2L2 ωL

Now, as we know that R<<wL, the power factor is small and hence the power absorbed will be very small. And also,
on account of its large impedance (large inductance), current passing through the coil is very small. Hence, such a
coil is preferred in electrical circuits for the purpose of adjusting the current to any desired value without having a
significant energy waste.

Illustration 14: An ac circuit consists of a 220 Ω resistance and a 0.7 H choke. Find the power absorbed from a 220V
and 50 Hz source connected in this circuit if the resistance and choke are joined, (a) in series (b) in parallel
 (JEE ADVANCED)

Sol: For a seriesLR circuit, impedance is=


Z R 2 + ω2L2 and average power dissipated in circuit is calculated as
=P Vrms Irms cos φ .

1 1 1 1
In parallel LR circuit = + . But for a choke, L is very large, so ≈0.
2 2 2 2
Z R ωL ω2L2
Z
(a) in series the impedance of the circuit is: XL

R 22 + ( 2πfL=
))2 (( 220 )2 + ( 2 × 3.14 × 50 × 0.7=
))2 311
2 2 2
=
Z R 22 + ω22=
L22 Ω 
=
Z R + ω= L R + ( 2πfL= 220 ) + ( 2 × 3.14 × 50 × 0.7= 311Ω
V 220 Figure 23.27
∴ Irms = Vrms = 220 = 0.707A R 220
∴ Irms = rmsZ = 311 = 0.707A , cos φ= = = 0.707
Z 311 Z 311

=
and the power absorbed in the circuit,
= P Vrmsirms cos φ (=
220 )( 0.707 )( 0.707 ) 110.08 W

(b) When the resistance and choke are in parallel, the entire power is absorbed in resistance, as the choke (having
( 220 )
2
V 2rms
zero resistance) absorbs no power. =
∴P = =220W
R 220
2 3 . 2 2 | Alternating Current

PROBLEM-SOLVING TACTICS

(a) In this chapter, we have seen how a phasor provides a powerful tool for analysing the AC circuits.
Below are some important tips:
1. Keep in mind the phase relationship for simple circuits.
(i) For a resistor, the voltage and phase are always in phase.
(ii) For an inductor, the current lags the voltage by900.
(iii) For a capacitor, the current leads the voltage by 900.
(b) When circuit elements are connected in series, the instantaneous current is the same for allelements, and
instantaneous voltages across the elements are out of phase. On the otherhand, when circuit elements are
connected in parallel, the instantaneous voltage is the same for all elements, and the instantaneous currents
across the elements are out of phase.

(c) For a series connection, draw a phasor diagram for the voltage. The amplitude of the voltage drop across
all the circuit elements involved should be represented with phasors. In Fig. 23.28, the phasor diagram for a
series RLC circuit is shown for both the inductive case XL > X C and the capacitive case XL < X C . Below is a
phasor diagram for the series RLC circuit for (a) XL > X C (b) XL < X C .


VLO 
VLO
  
V0 VLO +VCO  
 I0 VRO
    
I0 VRO  VLO +VCO
V0

VCO 
VCO

(a) (b)
Figure 23.28: Phase angle between applied voltage and current (a) in RC circuit, (b) in LC circuit

From Fig. 23.28(a), we see that VL0 > VC0 in the inductive case and V 0 leads I0 by a phase φ . On the other
hand, in the capacitive case shown in Fig. 23.28(b), VC0 > VL0 and I0 leads V 0 by a phase φ .
(d) Students should directly learn the formula for reactance, impedance, etc.to solve any problem easily.
(e) For parallel connection, draw a phasor diagram for the currents. The amplitudes of the current across all the
circuit elements involved should be represented with phasors. In the following Fig. 23.29, the phasor diagram
for a parallel RLC circuit is shown for both the inductive case XL > X C and the capacitive case XL < X C .
 
ICO ICO

  
  I0 ICO + ILO
IRO V0 
    
 ICO + ILO IRO V0
I0
 
ILO ILO

(a) (b)
Figure 23.29
P hysi cs | 23.23

(f) Phasor diagram for the parallel RLC circuit for (a) XL > X C And (b) XL < X C : From Fig. 23.29(a), we see that
IL0 > IC0 in the inductive case and V0 lead I0 by a phase φ .On the other hand, in the capacitive case shown
in Fig. 23.29 (b), IC0 > IL0 and I0 leads V0 by a phase φ .

FORMULAE SHEET

(a) In an AC circuit, sinusoidal voltage source of amplitude V0 is represented as:V(t) =V0 sinwt.
The current in the circuit has amplitude I0 and lags the applied voltage by phase angle φ .
Current is represented as: I(t) = I0 sin ( ωt − φ )
(b) For a single-element circuit (a resistor, a capacitor or an inductor) connected to the AC voltage source, we
summarise the results in the below table:

Circuit elements Resistance/Reactance Current Amplitude Phase angel φ

R R 0
V0
IR =
0 R

L Inductive Reactance
XL = ωL (π/2)
V0
IL = i.e.,current lags
0 XL
voltage by 900

C Capacitive Reactance
1 (- π / 2 )
XC = V0
ωC IC = i.e. current leads
0 XC
voltage by 900

(c) For a circuit having more than one circuit element connected ina series,we summarise the results in the below
table:

Circuit elements Impedance Z Current amplitude Phase angle φ

R C V0 π
R 2 + XL2 I0 = 0<φ< 
2
R + XL2 2

R C V0
2 I0 =  π
R + X 2C R 2 + X 2C −  < φ < 0
 2

R L C φ > 0 if
V0
R + ( XL − X C )
2
2
I0 = XL > X C
R + ( XL − X C )
2 2
φ < 0 if XL < X C
2 3 . 2 4 | Alternating Current

(d) For series LCR circuit,

(i) the impedance is Z = R 2 + (XL − X C )2

(ii) the current lags the voltage by phase angle φ =tan−1


( XL − XC )
R
1
(iii) the resonant frequency is ω0 = .
LC

At resonance, the current in the series LCR circuit is maximum, while that in parallel LCR circuit is minimum.

(e) Impedance for parallel LCR circuit, is given by


2 2
1 1  1  1  1 1 
= + − ωC=
 + − 
Z R 2
 ω L  R 2  XL X C 

The phase angle by which the current lags the voltage is

 1 1   1 
=
φ tan−1 R = −  tan−1 R  − ωC 
X 
 L XC   ωL 

(f) The RMS (root mean square) value of voltage and current in an AC circuit are given as
V0 I0
Vrms = , and Irms =
2 2
R
(g) Average power of an AC circuit
= is P(t) Irms Vrms cos φ where cos φ = is the power factor of the circuit.
Z
ω0L 1 L
(h) Quality factor Q of LCR circuit is=
Q =
R R C
V2 N2
(i) For a transformer, the ratio of secondary coil voltage to that of primary coil voltage is =
V1 N1
where N1 is number of turns in primary coil, and N2 is number of turns in secondary coil.
For the step-up transformer, N2 > N1 ; for step down transformer, N2 < N1 .
P hysi cs | 23.25

Solved Examples

JEE Main/Boards Example 2: A circuit draws a power of 550 W from a


source of 220 V, 50Hz. The power factor of the circuit is
Example1: A resistance R, inductance L and a capacitor 0.8 and the current lags in phase behind the potential
C all are connected ina series with an AC supply. The difference. To make the power factor of circuit as 1.0,
resistance of R is 16 Ω, and for a given frequency, the what capacitance will be connected in the circuit?
inductive reactance of L is 24 Ω,and capacitive reactance
of C is 12 Ω. If the current in the circuit is 5 A, find Sol: In series LR circuit, the current lags the applied
(a) The potential difference across R, L and C
voltage by angle φ and the power factor of circuit is
(b) The impedance of the circuit R
cos φ = . When capacitor is connected
(c) The voltage of AC supply R 2 + ω2L2
(d) Phase angle in series in the circuit, the impedance of the circuit is

R 2 + ( X C − XL )
2
Z= and the power factor of the
Sol: In a series LCR circuit, the impedance of circuit is R
circuit is cos φ = .
R 2 + ( X C − XL ) where XC and XL are the capacitive
2
Z= R 2 + ( XL − X C ) 
2

 
and inductive reactances respectively. Phase difference
 X − XC  We want to find the value of the capacitor to make the
between voltage and current is φ =tan−1  L . circuit’s power factor 1.0
 R 
 
Potential drop across resistance is IR and that across (A) Find the value resistance and inductive reactance.
reactance is IX. For a LR circuit, current lags behind voltage in phase.
(a) Potential difference across The power in AC circuit is given as
(i) Resistance VR =I × R =5 × 16 =80 V
V 2rms × cos φ  … (i)
( )
(ii) Inductor  VL = I × ωL = 5 × 24 = 120 V P=
Z
...(1)

I (1 / ωC ) =5 × 12 =60 V
(iii) Capacitor VC =×
( 220 )
2
V 2rms × cos φ × 0.8
⇒ Z= = = 70.4 Ω
(b) The impedance of the circuit P 550

 1 
2 R
Power factor cos φ = , so we get value of resistance as
(16 ) + ( 24 − 12)
2 2 2
=
Z R +  ωL − = Z
 ωC 
= 20 Ω R = Z × cos φ = 70.4 × 0.8 = 56.32 Ω

Inductive Reactance is
(c) The voltage of AC supply is given by
E =I × Z =5 × 20 =100 V ω
= L (Z 2 2
− R= ) ( 70.4 ) − (56.32)
2 2

ω=
L 42.2 Ω
(d) Phase angle between voltage & current is
(B) Capacitance needed to be connected in circuit to
 ωL − ( 1 / ωC )   24 − 12  make power factor = 1.0
−1
φ =
tan   tan−1  
 R   16  When the capacitor is connected in the circuit.
Impedance
( )
= tan−1 0.75 = 360 52'
 1  
2
R 2 +   ωL − …(ii)
=
Z   ...(2)
 ωC  
  
2 3 . 2 6 | Alternating Current

and power factor is given by as, U= P × t= mc∆θ= ( TC ) ∆θ ;


cos φ =
R
=t
( TC=
) × ∆θ 2 × 10
= 348 348 sec
secs= �5.8 min
5.8min.
 1  
2
P 0.0575
R 2 +   ωL −  
 ωC  
 
Example 4: A 100 V ac source of frequency 500 Hz
1
When cos φ= 1, ωL= …(iii) is connected to a series LCR circuit with L=8.1 mH,
ωC C = 12.5 µ F and R= 10 Ω . Find the potential different
1 1 across the resistance.
From (iii) we get
= C =
ω ( ωL ) 2πf ( ωL ) Sol: For LRC circuit, total potential difference is
1
= 75 × 10−6 F VR2 + ( VC − VL ) .
2
= V=
( 2 × 3.14 × 50 ) ( )
× 42.2
= 75 µF. Inductive reactance,

Therefore to make a circuit with power factor = 1, 75 µF XL = 2π × 500 × 8.1 × 10−3 = 25.45 Ω
capacitor is to be connected in a series with resistance
and inductor. Capacitive reactance,
106
=
XC = 25.45Ω
2π × 500 × 12.5
Example 3: A 750 Hz, 20 V source is connected to
a resistance of 100 ohm, an inductance of 0.1803 ⇒ XL =
XC
Henry and a capacitance of 10 microfarad all in series.
Calculate the time in which the resistance (thermal This is the condition of resonance. This means that total
capacity 2J/°C) will get heated by 10°C. potential drop occurs across the resistance only.

( )
2
∴ V = VR2 + VL − VC = VR = 100 V
Sol: For an LCR circuit, the average power dissipated as
2
Vrms The total potential difference across resistance is the
heat is =
Pav × R , where Z is the impedance of the same as the applied voltage across circuit.
2
Z
circuit.
Product of power and time equals the heat generated. Example 5: A 0.21 H inductor and a 12 Ω resistor
are connected ina series to a 20 V, 50 Hz ac source.
XL = ωL = 2πfL = 2π × 750 × 0.1803
= 849.2 Ω and Calculate the current in the circuit and the phase angle
between the current and the source voltage.
1 1
=
X C =
ωC 2πfC
Sol: In series LR circuit, the current lags voltage by phase
1
= = 21.2Ω  ωL 
2π × 750 × 10−5 angle φ =tan−1   . And RMS value of the current is
 R 
V
So X = XL − X C = 849.2 − 21.2 = 828 Ω Irms = rms where Z is impedance of the circuit.
Z

(100 ) + ( 828 ) =
2 2
And hence=
Z R2 + X2 = 834Ω Impedance =
Z R 2 + (ωL)2 ;

But as in case of ac,


122 + ( 2 * 3.14 * 50 * 0.21 )
2
R Vrms
P= Vrms Irms cos=
φ Vrms × ×
(12 ) + (65.94 )=
av 2
Z Z = 2
67Ω
2 2
 Vrms   20 
i.e.=
Pav   =
×R  100 0.00575W And
 ×= V 200
 Z   834  Current Irms = =
rms
= 3.28A
Z 67

Phase angle φ
P hysi cs | 23.27

 ωL  −1  65.94 
VR V VL
tan−1   = tan  ;
 R   12  20

tan = (5.495 ) = 78.69 ° l


VS
Example 6: A current of 4 A flows in a coil when
connected to a 12 V dc source. If the same coil is
connected to a 22 V, 50 rad/sec ac source, a current of (a) P.D. across R,L and C
2.4 A flows in the circuit. Determine the inductance of (b) Impedance of circuit
the coil. Also, find the power developed in the circuit if
a 2500 µ F condenser is connected in a series with the (c) Voltage of AC supply and
coil. (d) Phase angle

Sol: For dc supply, the coil is purely resistive; inductance Sol: For the LCR circuit, impedance is
does not come into picture. For AC voltage source,the
reactance of the inductor is non-zero. When a capacitor Z= R 2 + (X C − XL )2 .
is connected in a series in a circuit, the impedance of
The phase angle between voltage and current is given
R + ( XL − X C ) .
2 2
circuit is Z =
 X − XC 
The real power in the circuit is by φ =tan−1  L  .
 R
 
V2
P I2=
= R R. (a) P.D. across each component is found below
Z2
12 VL =I XL =5 × 24 =120 V,
Resistance of the coil, R= = 3Ω VR =5 × 16 =80 V
4 VC =I X C =5 × 12 =60 V
( Reactance of inductor in dc circuit is zero)
(b) Using the formula of Impedance
12
Impedance of coil, Z= = 5Ω ;
2.4 Z= R 2 + ( XL − X C )
2

Now, Z=
2
R 2 + ω2L2 ;
(16 ) + ( 24 − 12)
2 2
Z= =20 Ω
2 2
Z −R 4
or=
L = = 0.08 H
ω 50
(c) Voltage of AC source is
Reactance of the capacitor E = IZ = 5 × 20 =
100 V
1 1 (d) Phase angle is
X C= = = 8Ω
ωL 50 × 2500 × 10−6
−1 ( L
X − XC )  24 − 12 
= Φ tan
= tan−1  
R  16 
∴ When the capacitor is connected in series,

Z = R 2 + ( XL − X C ) = 32 + ( 8 − 4 ) =5 Ω
2 2 ( )
= tan−1 0.75 = 360 87'

Example 8: A coil of resistance 20 Ω and inductance


R 3
Power factor, cos φ = = ; 0.5H is switched to dc 200 V supply. Calculate the rate
Z 5 of increase of current:
( 2.4 )
2 2
Power developed P= Irms =
Z cos φ × 3 =17.28 W. (a) At the instant of closing the switch
(b) After one time constant
Example 7: A resistance R, an inductance L, and (c) Find the steady state current in the circuit
capacitor C are connected in series with an AC supply
where R=16 Ω . Inductive reactance X= L 24 Ω and Sol: The current in the LR circuit attains constant value
capacitive reactance X= 12 Ω . If the current in the over a long period of time. Generally, the current in the
circuit is 5 A, find
C
2 3 . 2 8 | Alternating Current

circuit is given by
constant.
( )
=i i0 1 − e− t/ τ where τ is one time
(c) The quality factor Q of the circuit
(d) The amplitude of the voltage across the inductor at
(a) Current at any time is given by: the resonant frequency.

 − 
Rt Sol: When the LCR circuit is set to resonance, the
=i i0  1 − e L   ...(1) … (i)
resonant frequency is f = .
  1 1
  2π LC
Differentiating above equation w.r.t. t, we get
ω0L 1 L
Quality factor is=
Q = .

Rt R R C
dI  V R  − RtL  V
=dI  V . R  e= L ∴ i0 V   ...(2) … (ii)
(a) Using formula of resonant frequency
=dt  R . L  e= ∴ i0 R  ...(2)
dt  R L   R
dI V 200 The resonant frequency, for the circuit is given by
At =
t 0, dI= V= 200= 400 A / s
At =
t 0, dt= L= 0.5= 400 A / s ω0
dt L 0.5 1 1
=f =
L 2π 2π LC
(b) Current after one time constant τ =
R
1 1
From equation (ii) = = 5033Hz

dI
( )(
2π 10 × 10−3 H 100 × 10−9 F )
= 400
= e−1 147.15 A / s
dt (b) At resonance current is Maximum i.e. I0
(c) For steady state t = ∞
V0 200
So from (i) we get i(∞ ) = i0 = 400 A =
I0 = = 10.0 A
R 20.0Ω

Example 9: What is average and RMS current over (c) The quality factor Q of the circuit is given by
half cycle if instantaneous current is given by i=4
sin ωt + 3cos ωt. ?
=
Q
(
ω0L 2π 5033s
=
−1
)(
10.0 × 10−3 H )
R ( 20.0Ω )
Sol: Reduce the given expression of current in standard
( )
=i i0 sin ωt + φ , where i0 is the maximum current
form
= 15.8
in the circuit.
(d) At resonance, the amplitude of the voltage across
Given i = 4 sin ωt + 3cos ωt. the inductor is
4 3 
t  5sin ( ωt + α )
= 5  sin ωt + cos ω= VL = I0 XL= I0 ω0L
0
5 5 
where cos α =
4 3
and sin α = ;
= ( )(
(10.0A ) 2π 5033 s−1 10.0 × 10−3 H )
5 5 = 3.16 × 103 V
Comparing with
=i i0 sin ωt + φ ( )
Example 2: Consider the circuit shown in figure. The
 5   10  sinusoidal voltage source is V (t) = V0 sinωt . If both
i0 = 5 A ; ⇒ irms =  A ; iavg =   A
 2  π  switches s1 and s2 are closed initially, find the following
quantities, ignoring the transient effect and assuming
that R, L, V0 and w are known:
JEE Advanced/Boards (a) The current I(t)as a function of time
(b) The average power delivered to the circuit
Example 1: A sinusoidal voltage V(t) = (200 V) sin ωt is
applied to a series LCR circuit with L=10.0 mH, C=100 (c) The current as a function of time, a long time after
nF and R=20.0 Ω . Find the following quantities: only S1 is opened
(a) The resonant frequency
(b) The amplitude of current at resonance
P hysi cs | 23.29

V Thus, the current as a function of time is


V0  ωL 
=
I(t) I0 sin(ωt −=
φ) sin  ωt − tan−1 
R 2 + ω2L2  R 
R₀
A C Note that in the limit of vanishing resistance R=0,
B
R φ = π / 2 , and we recover the expected result for a

purely inductive circuit.


(d) The capacitance C if both s1 and s2 are opened for a
long time, with the current and voltage in phase.
(d) If both the switches are opened, then this would be
(e) The impedance of circuit when both s1 and s2 are
opened. a driven RLC circuit, with the phase angle φ given by tan
1
(f) The maximum energy stored in the capacitor during XL − X C ωL −
oscillations. φ= = ωc
R R
(g) The maximum energy stored in the inductor during If the current and voltage are in phase, lthen= φ ,
oscillations.
implying tan φ =0. Let the corresponding angular
(h) The phase difference between the current and the
voltage if the frequency of V (t) is doubled. 1
frequency be ω0 ; we then obtain. ω0L = And the
ω0 c
(i) The frequency at which the inductive reactance XL is 1
capacitance is C =
And the capacitance
equal to half the capacitive reactance X C . ω20L
(e) From (d), we see that both switches are opened; the
Sol: In LCR circuit explained above, when the switches circuit is at resonance with XL = XC. Thus, the impedance
of the circuit becomes
are closed, the current follows path of least resistance
R 2 + ( XL − X C ) = R
2
i.e., L and C are short-circuited. Impedance of series Z=

R 2 + ( X C − XL ) . The energy stored


2
LCR circuit is Z =
(e) The electric energy stored in the capacitor is
1 2
in inductor is UL = LI and that stored in capacitor is = UE =
1 2 1
CVC C ( IX C ) It attains maximum when the
2
2 2 2
1
UC = CVc2 . current in at its maximum I0:
2
2
(a) When both switches s1 and s2 are closed, the current 1 2 2 1  V0  1 V02L
goes through only the generator and the resistor, so =U =CI X C  =
C,max
2 0 C 2  R  ω2 C2 2R 2
the total impedance of the circuit is R and the current 0

V0 Where we have used ω0 = 2


1 / LC.
is =
IR (t) sin ωt
R (g) The maximum energy stored in the inductor is given
(b) The average power is given by: by.
V 20 2 V 2
0 1 2 LV0
2
= P(t) =
IR (t)V(t) sin= ωt = =
is given by.UL,max LI
R 2R 2 0 2R 2
(c) If only S1 is opened, after a long time a current
will pass through the generator, the resistor and the (h) If the frequency of the voltage source is double, i.e.,
inductor. For this RL circuit, the impedance becomes ω = 2ω = 1 / LC , then the phase becomes
0
1 1
=Z =  ωL − 1 / ωC 
R 2 + XL2 R 2 + ω2L2 φ =tan−1  
 R 
 ωL 
And the phase angle φ is φ =tan−1 
 R 
 ( ) (
 2 / LC L − LC / 2C
= tan−1 
) 
R 
 
 
 3 L
= tan−1  
 2π C 
 
φ =tan  
 R 
( ) (
 2 / LC L − LC / 2C
. 3 0−1| Alternating Current
2=3tan
) 
 R 
 
 
 3 L V 10 1
= tan−1   =
IrmsinL 2 = − = ;
 2π C  ωL 2 2π × 50 × 10 100π
 
1 1 3
(i) If the inductive reactance in one-half the capacitive Irmsincircuit = + =
50π 100π 100π
reactance,
1 1 1  Example 4: A series LCR circuit containing a resistance
XL = X ; ⇒ ωL =  ; of 120 Ω has angular frequency 4 × 105 rads–1. At
2 C 2  ωC 
resonance, the voltage across resistance and inductance
1 ω0
Then
= ω = are 60 V and 40 V respectively. Find the value of L and
2LC 2 C. At what frequency does the current lag the voltage
by 45o?
Example 3: Two inductances of 5.0 H and 10.0 H
Sol: At resonance, XL = XC. The phase angle by which
are connected in parallel circuit. Find the equivalent
inductance and RMScurrent in each inductor and in  X − XC
the current lags the voltage is φ =tan−1  L


mains circuit when connected to source of 10 V AC.  R
 
I1 For resistance VR = IrmsR;
5.0 H VR 60
or I=
rms
= = 0.5 A
I
I2 10.0 H R 120
For inductor V= Irms ω0L
 V
L

10 V AC
40 = 0.5 × 4 × 105 × L ⇒ L = 2 × 10−4 H
Sol: When two inductors are connected in parallel, the 1
LL
At resonance, XL = XC i.e. ω0L =
net inductance is L = 1 2 . If V is the RMS value of ω0 C
L1 + L2
1 1 1
applied voltage, then RMScurrent through inductor is =
C = = µF
ω20L
( 4 × 10 ) 32
2
V
5
× 2 × 10−4
I= .
XL
When the current lags behind the voltage by = 45o,
=
Let E E0 sin ωt , then current drawn from supply is, XL − X C
using tan φ = , gives
R
 π  E0  π 1
=I I0 sin  ωt −=  sin  ωt −  (Since current lags ωL −
 2  ω L  2  1  ω2 L 
1= ω C ⇒ R =ωL − =ωL −  o 
π
by ) R ωC  ω 
2  

Where L is equivalent inductance of circuit. ∴ωR = ω2L − ω2oL

= I1 + I2
I =
E0
 π
sin  ωt − 
(
120 ω = 2 × 10−4 ω2 − (4 × 105 )2 )
ωL1  2 VL

Source Voltage
E0  π E  π
= sin  ωt −  + 0 sin  ωt −  VL - VC
ωL1  2  ω L 2  2 

I I I 1 1 15 3
= + = + = = ;
o
⇒ 45
L L1 L2 5 10 50 10 VR
i

10
⇒L= H
3 VC
V 10 1
=
IrmsinL1 =− = ;
ωL1 2π × 50 × 5 50π
P hysi cs | 23.31

On solving the above equation,we get V 10


=I V00 sin ( ω=
t + φ ) 10 sin ( 314 t + π / 2 )
ω = 8 × 105 or ω = −2 × 105 =I Z sin ( ω= t + φ ) 19.3 sin ( 314 t + π / 2 )
Z cos 314t 19.3
∵ Frequency can’t be negative = 0.52
= 0.52 cos 314t
∴ Ignoring negative root we have ω = 8 × 105 Hz

Example 5: An inductor of 20mH, a capacitor 100 µF Example 6: A choke coil is needed to operate an arc
and a resistor 50 Ω are connected in a series across lamp at 160 V (rms) and 50 Hz. The lamp has an effective
a source of e.m.f. V=10 sin (314t). Find the energy resistance of 5 Ω when running at 10 A (RMS). Calculate
dissipated in the circuit in 20 minutes. If resistance is the inductance of the choke coil. If the same arc lamp is
removed from the circuit and the value of inductance is to be operated on 160 V (dc), what additional resistance
doubled, then find the variation of current with time in is required? Compare the power losses in both cases.
the new circuit. L R

Sol: For the LCR circuit, the energy dissipated over a Choke Lamp
long
= time is U ( )
Vrms Irms cos φ t . When resistance is VL VR
removed,the circuit becomes LC circuit, the impedance
and hence current changes.

V = V0 sin t
The circuit is as shown in figure. One time cycle
Sol: Choke coil has large inductance and low internal
2π 2π
T= = = 0.02s. So, we have to calculate the resistance, sothere is no power loss in the choke coil.
ω 314 Hence, when alamp of some resistance is connected
average energy at time t>>T. in series with the coil, the net RMS voltage in circuit

(=
Vrms ) ( Vrms )R + ( Vrms )L .
2 2 2
L R C is When the same lamp

is operated on dc, additional resistance in a series is


required to limit the current in the lamp to 10 A.

Voltage drop across the lamp is
10 sin 314t
( Vrms )R = ( Irms )(R ) = 10 × 5 = 50 V Voltage drop across
Energy dissipated in time t
choke coil is
 I V R
U = ( Vrms Irms cos φ ) t =  0 × 0 ×  t
∴ ( Vrms ) = ( Vrms ) − ( Vrms )R
2 2
 
 2 2 Z L

=∴U
V02R
=

t  I0
V0 

= (160 ) − (50 ) =
2 2
152 V
2Z 2
 Z 
As (=
Vrms ) (irms
= ) XL (irms )( 2πfL ) ;
L
102 × 50 × 20 × 60
=∴U = 864.2 J
2 × 3153.7 ( Vrms )L
∴L =
When resistance is removed,and inductance is doubled, ( 2πf )(irms )
then cos φ = 0 ⇒ φ = π / 2
Substituting the values
Value of impedance is 152
L= = 4.84 × 10−2 H
=
Z'
1
− ωL'
=
1
− 314 × 40 × 10−3 Ω ( )( )( )
2 π 50 10
ωC 314 × 10 −4
When lamp is operated on DC supply with a resistance
=19.3 Ω
R’ in series, then voltage drop across the circuit is
And the current in the circuit is found to be
V i (R + R' ) or 160=10(5+R’);
=
∴ R=' 11Ω
2 3 . 3 2 | Alternating Current

Choke coil has no resistance.Therefore,for ac circuit The capacitive reactance


power loss in choke coil is zero, while in case of dc, the
loss due to additional resistance R’ is 1 1 100
= X = C
=
Ω Ω
ωC −6 π
(=
10 ) (11 )
2 2π × 50 × 100 × 10
P= i2R'
= 1100 W

The inductive reactance


Example 7: A series AC circuit contains an inductor
(20 mH), a capacitor (100 µF ) and resistance (50 Ω). AC XL = ωL = 2π × 50 × 20 × 10 −3 Ω = 2π Ω.
source of 12 V (RMS), 50 Hz is applied across the circuit.
Find the energy dissipated in the circuit in 1000 s. 1
The net reactance is X= − ωL
ωC
Sol: The average power dissipated in series LCR circuit
100
is Pav Vrms Irms cos φ . For time t � T , the energy =
= Ω − 2π=
Ω 25.5 Ω
π
dissipated is U = Pavt.
(50 Ω ) + ( 25.5 Ω ) =
2 2
Thus, Z 2 = 3150 Ω2
The time period of the source is,
From (i), average power
T=1/f=20 ms.
and t = 1000 s � T 7200
=
Pav = 2.3 W
The average power dissipated is 3150
∴ The energy dissipated int = 1000s is
( )( )
2
2
Vrms R RVrms 50Ω 12V
=Pav V= = s 2.3 × 103 J
rms
Z Z Z2 Z2 U = Pav × 1000 =

7200
Pav = ...(i) … (i)
Z2 

JEE Main/Boards

Q.5 What is the relation between peak value and root


Exercise 1
mean square value of alternating e.m.f?

Q.1 The resistance of coil for direct current (dc)is 10 Ω . Q.6 Is there any device which may control the direct
When alternating current (ac) is sent through it; will its current without dissipation of energy?
resistance increase, decrease or remain the same?

Q.7 What is the phase relationship between current


Q.2 Prove that an ideal inductor does not dissipate and voltage in an inductor?
power in an A.C. circuit.

Q.8 Find the reactance of a capacitance C at f Hz.


Q.3 What is impedance? Derive a relation for it in an
A.C. Series LCR circuit. Show it by a vector.
Q.9 Prove that an ideal capacitor connected to an A.C.
source does not dissipate power.
Q.4 An A.C. supply E = E0 sinω t is connected to a series
combination of L, C and R. Calculate the impedance
of the circuit and discuss the phase relation between Q.10 State the principle of an A.C. generator.
voltage and current.
Q.11 How are the energy losses reduced in a
transformer?
P hysi cs | 23.33

Q.12 Discusses the principle, working and use of a (c) Determine the rms potential drops across the three
transformer for long distance transmission of electrical elements of the circuit. Show that the potential drop
energy. across the LC combination is zero at the resonating
frequency.
Q.13(a) What will be instantaneous voltage for A.C.
supply of 220 V and 50 Hz? Q.22 A circuit containing a 80 mH inductor and a 60 µF
capacitor in series is connected to 230 V, 50Hz supply.
(b) In an A.C. circuit, the rms voltage is 100 2 V , find
The resistance of the circuit is negligible. (a) Obtain
the peak value of voltage and its mean value during a
the current amplitude and rms values. (b) Obtain the
positive half cycle.
rms value of potential drops across each element, (c)
What is the average transferred to the inductor? (d)
Q.14 What should be the frequency of alternating 200 What is the average power transferred to the capacitor?
V so as to pass a maximum current of 0.9 A through an (e) What is the total average power absorbed by the
inductance of 1 H? circuit? [‘average’ ‘implies’ averaged over one cycle;].

Q.15 An alternating e.m.f of 100 V (r.m.s), 50 Hz is Q.23 Answer the following questions: (a) in any A.C.
applied across a capacitor of 10 µF and a resistor of 100 circuit, is the applied instantaneous voltage equal to the
W in series.Calculate (a) The reactance of the capacitor; algebraic sum of the instantaneous voltage across the
(b) The current flowing (c) the average power supplied. series element of the circuit? Is the same true for rms
voltage? (b) A capacitor is used in the primary circuit of
Q.16 The effective value of current in a 50 cycle A.C. an inductor coil. (c) A supplied voltage signal consists
circuit 5.0 A. What is the value of current 1/300s after of a super position of a D.C voltage and A.C. voltage
it is zero? of high frequency. The circuit consists of an inductor
and a capacitor in series. Show that the D.C. signal will
appear across C and the A.C. signal across L. (c) An
Q.17 A pure capacitor is connected to an ac source of
applied voltage signal consists of a superposition of a
220 V, 50 Hz, what will be the phase difference between
D.C. voltage and an A.C. Voltage of high frequency. The
the current and applied emf in the circuit?
circuit consists of an inductor and a capacitor in series.
Show that the D.C. signal will appear across C and the
Q.18 A 100 Ω resistance is connected to a 220 V, 50 Hz A.C. signal across L. (e) Why is choke coil needed in the
A.C. supply. use of florescent tubes with A.C. mains? Why can we
(a) What is the rms value of current in the circuit? not use an ordinary resistor instead of the choke coil?

(b) What is the net power consumed over a full cycle?


Q.24 An inductance of negligible resistance, whose
reactance is 22 Ω at 200 Hz is connected to a 220 V, 50
Q.19 A pure inductance of 1 H is connected across hertz power line, what is the value of the inductance
a 110V, 70 Hz source, find (a) reactance (b) current and reactance?
(c) peak value of current.

Q.25 An electric lamp market 220 V D.C. consumes a


Q.20 A series circuit contains a resistor of 10 Ω , a current of 10 A. It is connected to 250 V-50 Hz A.C.
capacitor, an ammeter of negligible resistance. It is main through a choke. Calculate the inductance of the
connected to a source 220V-50 Hz, if the reading of choke required.
an ammeter is 2.0 A, calculate the reactance of the
capacitor.
Q.26 A 2 µF capacitor, 100 Ω resistor and 8H inductor
are connected in series with an A.C. source. What
Q.21 A series LCR circuit connected to a variable should be the frequency of this A.C source, for which
frequency 230V source and L=5.0 H,C=80 µF , R=40 Ω . the current drawn in the circuit is maximum? If the peak
(a) Determine the source frequency which drives the value of e.m.f of the source is 200 V, find for maximum
circuit in resonance. current, (i) The inductive and capacitive reactance of
the circuit; (ii) Total impedance of the circuit; (iii) Peak
(b) Obtain the impedance of the circuit and the value of current in the circuit ; (iv) The phase relation
amplitude of the current at the resonating frequency. between voltages across inductor and resistor; (v) The
2 3 . 3 4 | Alternating Current

phase difference between voltage across inductor and and minimum current that can flow in the circuit is
capacitor. 10V
.S
( )
Q.27 A step-down transformer converts a voltage of 0.1 H 10
2200 V into 220 V in the transmission line. Number
of turns in primary coil is 5000. Efficiency of the
transformer is 90% and its output power is 8 kW. 10
Calculate (i) Number of turns in the secondary coil (ii)
input power.
(A) 2 Amp (B) 3 Amp

Q.28 What will be the effect on inductive reactance XL (C) 1 Amp (D) Nothing can be concluded
and capacitive XC, if frequency of ac source is increased?
Q.4 The ratio of time constant in build-up and decay in
Q.29 The frequency of ac is doubled, what happens to the circuit shown in figure is
(i) Inductive reactance (ii) Capacitive reactance? R

2R L
Exersice 2
Single Correct Choice Type V
(A) 1:1 (B) 3:2 (C) 2:3 (D) 1:3
Q.1 A rectangular loop with a sliding connector of
length 10 cm is situated in uniform magnetic field
Q.5 A current of 2A is increased at a rate of 4 A/s
perpendicular to plane of loop. The magnetic induction
through a coil of inductance 2H. The energy stored in
is 0.1 tesla and resistance of connecter (R) is 1 Ω. The
the inductor per unit time is
sides AB and CD have resistance 2 Ω and 3 Ωrespectively.
Find the current in the connecter during its motion with (A) 2 J/s (B) 1 J/s (C) 16 J/s (D) 4 J/s
constant velocity of 1 meter/sec.
A D Q.6 The current in the given circuit is increased with a
rate a=4 A/s. The charge on the capacitor at an instant
when the current in the circuit is 2 amp will be:
2 3
E=4V
R
B C R=1
1 1 1 1
( A ) 110 A (B ) 220 A ( C ) 55 A (D ) 440 A
L=1H
)
C=3F

Q.2 For L-R circuit, the time constant is equal to (A) (A) 4 µC (B) 5 µC
Twice the ratio of the energy stored in the magnetic (C) 6 µC (D) None of these
field to the rate of dissipation of energy in the resistance.
(B) Ratio of the energy stored in the magnetic field to Q.7 A coil of inductance 5H is joined to a cell of emf 6 V
the rate of dissipation of energy in the resistance. through a resistance 10 Ω at time t=0. The emf across
(C) Half the ratio of the energy stored in the magnetic the coil at time t= 2 s is:
field to the rate of dissipation of energy in the resistance.
(A) 3V (B) 1.5V (C) 0.75V (D) 4.5V
(D) Square of the ratio of the energy stored in the
magnetic field to the rate of dissipation of energy in Q.8 The network shown in the figure is part of a
the resistance. complete circuit. If at a certain instant, the current I is
5A and it is decreasing at a rate of 103As -1 then VB-VA
Q.3 In the adjoining circuit, initially the switch S is open. equals.
The switch‘s’ is closed at t=0. The difference between
P hysi cs | 23.35

1 + 5 mH L S
A B
I 15 V A B

(A) 20 V (B) 15 V (C) 10 (D) 5 V


R1 R2
12v
Q.9 In the previous question, if I is reversed in direction,
then VB-VA equals
(A) 5 V (B) 10 V (C) 15 V (D) 20 V (A) VL=12 V; point A is at the higher potential
(B) VL=12 V; point B is at the higher potential
Q.10 Two resistors of 10 Ω and 20 Ω and an ideal
inductor of 10 H are connected to a 2 V battery as (C) VL=6 V; point A is at the higher potential
shown in figure. The key K is inserted at time t=0. The (D) VL=6 V; point B is at the higher potential
initial (t=0) and final (t>=00) current through battery
are
Q.13 The power factor of the circuit shown in figure is
10 H
1/ 2 . The capacitance of the circuit is equal to

V=2sin(100t)
10 20


2V 10 0.1H
1 1 1 1
(A) A, A (B) A , A C
15 10 10 15
(A) 400 µF (B) 300 µF
2 1 1 2 (C) 500 µF (D) 200 µF
(C) A, A (D) A , A
15 10 15 25
Q.14 In the circuit, as shown in the figure, if the value
of R.M.S current is 2.2 ampere, the power factor of the
Q.11 In the circuit shown, the cell is ideal. The coil has
box is
an inductance of 4H and zero resistance. F is a fuse zero
resistance and will blow when the current through it
reaches 5A. The switch is closed at t=0. The fuse will 100 C
blow
1/ Henry


fuse
L Box

Sw

-1
Vrms=220 volt, -100  s
2V
1 3 1
(A) Just after t=0 (B) After 2 (A) (B) 1 (C) (D)
2 2 2
(C) After 5s (D) After 10s
Q.15 When 100 V DC is applied across a solenoid, a
Q.12 The circuit shown has been operating for a long current of 1 A flows in it. When 100 V AC is applied
time. The instant after the switch in the circuit labeled across the same coil, the current drops to 0.5 A. If the
S is opened, what is the voltage across the inductor VL frequency of the AC source is 50 Hz, the impedance
and which labeled point (A or B) of the inductor is at a and inductance of the solenoid are:
higher potential? Take R1=4.0 Ω , R2=8.0 Ω and L= 2.5 (A) 100 Ω , 0.93 H (B) 200 Ω , 1.0 H
H.
(C) 10 Ω , 0.86 H (D) 200 Ω , 0.55 H
2 3 . 3 6 | Alternating Current

Q.16 An ac current is given by I =+


I0 I1 sin ωt then its Previous Years’ Questions
rms value will be
Q.1 When an AC source of emf e=E0sin (100 t) is
(A) I02 + 0.5112 (B) I02 + 0.5102
connected across a circuit, the phase difference between
(C) 0 (D) I0 / 2 the emf and the current i in the circuit is observed to be
π
ahead, as shown in the figure. If the circuit consists
Q.17 The phase difference between current and voltage 4
in an AC circuit is π / 4 radians. If the frequency of AC possibly only of R-C or R-L or L-C in series, find the
is 50 Hz, then the phase difference is equivalent to the relationship between the two elements:  (2003)
time difference:
e i
(A) 0.78 s (B) 15.7 ms
(C) 0.25 s (D) 2.5 ms t

Q.18 Power factor an L-R series circuit is 0.6 and that of


a C-R series circuit is 0.5. If the element (L, C, and R) of
the two circuits are joined in series, the power factor of (A) R=1 K Ω ,C=10 µF (B) R=1 K Ω ,C=1 µF
this circuit is found to be 1. The ratio of the resistance (C) R=1 K Ω ,L=10H (D) R=1 K Ω ,L=1H
in the L-R circuit to the resistance in the C-R circuit is

(A) 6/5 (B) 5/6 (C) 4 (D) 3 3 Q.2 The current I4 through the resistor and voltage vC
3 3 4 across the capacitor are compared in the two cases.
Which of the following is/are true?  (2011)
Q.19 The effective value of current i=2 sin100 π t+2 sin
(100 π t+300) is: (a) IRA > IBR (B) IRA < IRB

(A) 2A (B) 2 2 + 3 A B
(C) IC > IC (D) IC < IC
A B

(C) 4 (D) None of these


Q.3 The network shown in Figure is part of a complete
Q.20 In a series R-L-C circuit, the frequency of the circuit. If at a certain instant the current (I) is 5A and is
source is half of the resonance frequency. The nature of decreasing at a rate of 103 A/s then VB − VA = .........V
the circuit will be  (1997)

(A) Capacitive i
(B) Inductive A 1 B
15 V 5 mH
(C) Purely resistive
(D) Data insufficient Q.4 An arc lamp requires a direct current of 10 A and 80
V to function. If it is connected to a 220 V (rms), 50 Hz
AC supply, the series inductor needed for it to work is
close to: (2016)
(A) 0.08 H (B) 0.044 H
(C) 0.065 H (D) 80 H
P hysi cs | 23.37

JEE Advanced/Boards

Exercise 1
L R

Q.1 In the given circuit, find the ratio of i1 to i2 where i1


is the initial current (at t=0), i2is steady state (at t=∞)
current through the battery.
E S

6 2 mH Q.7 Two coils, 1 & 2, have a mutual inductance = M and


resistance R each. A current flows in coils 1, which varies
with time as: I1 = kt2, where k is constant ‘t’ is time. Find
4 4 the total charge that has flown through coil 2, between
10V t = 0 and t = T.

Q.8 Find the value of an inductance which should be


L connected in series with a capacitor of 5 F, resistance
Q.2 Find the dimension of the quantity , where
symbols have usual meaning. RCV of 10 Ω and an ac source of 50 Hz so that the power
factor of the circuit is unity.
Q.3 In the circuit shown, initially the switch is in position
1 for a long time. Then the switch is shifted to position Q.9 In an L-R series A.C circuit the potential difference
2 for long time. Find the total heat produced in R2. across an inductance and resistance joined in series
are respectively 12 V and 16 V. Find the total potential
R2
difference across the circuit.
2 L
1 S Q.10 A 50W, 100V lamp is to be connected to an ac
mains of 200V, 50Hz. What capacitance is essential to
E R1 be put in series with lamp.

Q.4 Two resisters of 10 Ω and 20 Ω and an ideal Q.11 In the circuit shown in the figure, the switched
inductor of 10 H are connected to a 2V battery as S1 and S2 are closed at time t=0. After time t = (0.1) In
shown in figure. The key K is shorted at time t=0. Find 2sec, switch S2 is opened. Find the current in the circuit
the initial (t=0) and final (t->∞) current through battery. at time t = (0.2) ln 2sec.
L = 10 H
S1

R = 10 40 S2
20
K 100 V 10

1H
Q.5 An emf of 15 V is applied in a circuit containing
5 H inductance and 10 Ω resistance. Find the ratio of
the current at time t=∞ and t=1 second. Q.12 Find the value of i1and i2

S i1 30
Q.6 In the circuit in shown in figure, switch S is closed i2
at time t=0. Find the charge which passes through the 20
battery in one time constant.
100 V
2 3 . 3 8 | Alternating Current

(i) Immediately after the switch S is closed.


V
(ii) Long time later, with S closed.
(iii) Immediately after switch S is open
(iv) Long time after S is opened.

Q.13 Suppose the emf of the battery in the circuit


O T/4 T/2 3T/4 T
shown varies with time t so the current is given by
i(t) = 3+5t, where i is in amperes & t is in seconds. Take
R=4 Ω , L=6H & find an expression for the battery emf
as a function of time.
Find the amplitude of current in the steady state and
R i(t) obtain the phase difference between the current and
the voltage. Also plot the variation of current for one
cycle on the given graph.
 L

Exercise 2
Q.14 An LCR series circuit with 100 Ω resistance Single Correct Choice Type
is connected to an ac source of 200 V and angular
frequency 300rad/s. When only the capacitance is Q.1 A square coil ABCD is placed in x-y plane with its
removed, the current lags behind the voltage by 600. centre at origin. A long straight wire, passing through
When only the inductance is removed, the current origin, carries a current in negative Z-direction. Current
leads the voltage by 600. Calculate the current and the in this wire increases with time. The induced current in
power dissipated in the LCR circuit. the coil is

Q.15 A box P and a coil Q are connected is series with B


y
C
an ac source of variable frequency. The emf source at
10V. Box P contains a capacitance of 1 µ F in series with
a resistance of 32 Ω . Coil Q has a self-inductance 4.9 
mH and a resistance of 68 Ω series. The frequency
x
adjusted so that the maximum current flows in P and Q.
Find the impedance of P and Q atthis frequency. Also
find the voltage across P and Q respectively.
A D

Q.16 A series LCR circuit containing a resister of 120 (A) Clock wise (B) Anti clockwise
Ω has angularresonance frequency 4 × 105 rad s-1. At
resonance, the voltage across resistance and inductance (C) Zero (D) Alternating
are 60V and 40V respectively. Find the values of L and C.
At what frequency current in the circuit lags the voltage Q.2 An electric current i1 can flow in either direction
by 45o? through loop (1) and induced current i2 in loop (2).
Positive i1 is when current is from ‘a’ to ‘b’ in loop (1)
and positive i2 is when the current is from ‘c’ to ‘d’ in
Q.17 In an LR series circuit, a sinusoidal voltage V=V0 loop
sinωt is applied. It is given that
ω Loop (1)
L=
35mH,R =
11Ω, Vrms =
220V, =
50Hz
2π a b
And π =22 / 7 . Loop (2)
c d
P hysi cs | 23.39

(2) In an experiment, the graph of i2 against time ‘t’ is as Q.4 Two identical inductances carry currents that vary
shown below by Figure which one (s) of the following with time according to linear laws (see in figure). In
graphs couldi2have caused i2 to behave as give above. which of the inductances is the self-inductance emf
i2

greater?
i2
(A) 1 (B) 2
(A) t
(A) 0 t (C) Same (D) Data is insufficient to decide
t

i2 i2 Q.5 L, C and R represents physical quantities inductance,


i2 capacitance and resistance. The combination which has
the dimensions of frequency?
11 RR
(A)
(A) t (B)
(B) t (A)
11
and
and
RR
and (B) and
and
and
(B) t RC
RC LL RC
RC LL

1 C
i i2 (C) (D)
i22 LC L

Q.6 In the circuit shown, X is joined to Y for a long time,


and then X is joined to Z, the total heat produced in R2
(C)
(C) (D)
(B) t t
(C) t t
is:
i1 R2
i2
X Z
Q.3 In an L-R circuit connected to a battery of constant
e.m.f. E, switch S is closed at time t = 0. If e denotes the L Y
magnitude of induced e.m.f.
(C) t across inductor and i the
current in the circuit at anytime t. Then which of the E R1
following graphs shows the variation of e with i?
i1 i2
I

(A) (B)

2
1 t t

t i3 i4

e e
(C) (D)
(A) (B)
t t
t t
LE2 LE2 LE2 LE2R 2
(A) (B) (C) (D)
e e 2R12 2R 22 2R1R 2 2R12
(C) (D)
Q.7 An induction coil stores 32 joules of magnetic
energy and dissipates energy as heat at the rate of 320
t t
watt when a current of 4 amperes is passed through it.
Find the time constant of the circuit when the coil is
joined across a battery.
(A) 0.2s (B) 0.1s (C) 0.3s (D) 0.4s
2 3 . 4 0 | Alternating Current

Q.8 In an L-R decay circuit, the initial current at t=0 is 1. Q.14 The current I, potential difference VL across the
The total charge that has inductor has reduced to one- inductor and potential difference VC across the capacitor
fourth of its initial value is in circuit as shown in the figure are best represented
vectorially as.
(A) LI / R (B) LI / 2R (C) LI / 2R (D) None
VC
VC
Q.9 An inductor coil stores U energy when i current
is passed through it and dissipates energy at the rate (A) I (B)
of P. The time constant of the circuit, when the coil is VL VL I
connected across a battery of zero internal resistance is
4U U 2U 2P
(A) (B) (C) (D) VL
P P P U VL
(C) (D)
I I
Q.10 When a resistance R is connected in series with VC VC
an element A, the electric current is found to be
lagging behind the voltage by angle θ1. When the
same resistance is connected in series with element B, Q.15 In the shown AC circuit in figure, phase difference
current leads voltage by θ2. When R, A, B, are connected between current I1 and I2 is
in series, the current now leads voltage by θ. Assume XC
same AC source in used in all cases. Then:
I1
(A) θ = θ1 − θ2 (B) tan=
θ tan θ2 − tan θ1 
I2
θ1 + θ2
(C) θ = (D) None of these XL R
2

Q.11 The power in ac circuit is given by P=ErmsIrms cos φ . π X X − XC


(A) − tan−1 L (B) tan−1 L
The value of cos φ in series LCR circuit at resonance is: 2 R R
1 1
(A) Zero (B) 1 (C) (D) π X X − XC π
2 2 (C) + tan−1 L (D) tan−1 L +
2 R R 2
Q.12 If I1, I2,I3 and I4 are the respective r.m.s values of the
time varying current as shown in figure the four cases
I.II,III and IV in. Then identify the correct relations. Multiple Correct Choice Type

Q.16 A circuit element is placed in a closed box. At time


i i
(I) I0 (II) I0
O t O t t=0, constant current generator supplying a current of
-I0 -I0 1 amp, is connected across the box. Potential difference
across the box varies according to graph shown in
Figure. The element in the box is:
i i
(A) Resistance of 2 Ω (B) Battery of emf 6V
(III) I0 (IV) I0
O t O
-I0 (C) Inductance of 2H (D) Capacitance of 0.5F

(A) I1 = I2 = I3 = I4 (B) I3 > I1 = I2 > I4



(C) I3 > I4 > I2 = I1 (D) I3 > I2 > I1 > I4 8

2
Q.13 In series LR circuit XL=3R. Now a capacitor with XC=R
is added in series. Ratio of new to old power factor is
t(s)
3
(A) 1 (B) 2 (C) 1 (D)
2
2i
P hysi cs | 23.41

Q.17 For L-R circuit, the time constant is equal to (A) The low resistance of P
(A) Twice the ratio of the energy stored in the magnetic (B) The induced-emf in L
field to the rate of the dissipation of energy in the
(C) The low resistance of L
resistance
(D) The high voltage of the battery B
(B) The ratio of the energy stored in the magnetic field
to the rate of the dissipation of energy in the resistance.
Q.20 Two different coils have a self-inductanceof
L
C 8mH and 2mH. The current in one coil is increased at
a constant rate. The current in the second coil is also
VC
VL increased at the same instant of time. The power given
to the two coils is the same. At that time the current,
 the induced voltage and the energy stored in the first
(C) Half of the ratio of the energy stored in the magnetic coil are I1 V1 and W1 respectively. Corresponding values
field to the rate of the dissipation of energy in the for the second coil at the same instant are I2, V2 and W2
resistance. respectively . Then:
(D) Square of the ratio of the energy stored in the I1 1 I
(A) = (B) 1 = 4
magnetic field to the rate of the dissipation of energy I2 3 I2
in the resistance.
W1 V2 1
(C) = 4 (D) =
Q.18 An inductor L, a resistor R and two identical bulbs W2 V1 4
B1 and B2 are connected to a battery through a switch S
as shown in the figure. The resistance of the coil having Q.21 The symbol L, C, R represents inductance,
inductance L is also R. Which of the following statement capacitance and resistance respectively. Dimension of
gives the correct description of the happening when frequency is given by the combination.
the switch S is closed?
1
L B1 (A) 1/RC (B) R/L (C) (D) C/L
LC
R B2
Q.22 An LR circuit with a battery is connected at t=0.
Which of the following quantities is not zero just after
E S the circuit is closed?
(A) Current in the circuit
(A) The bulb B2 lights up earlier then B1 and finally both
the bulbs shine equally bright. (B) Magnetic field

(B) B1 lights up earlier and finally both the bulbs acquire (C) Power delivered by the battery
brightness. (D) Emf induced in the inductor
(C) B2 lights up earlier and finally B1 shines brighter
than B2. Q.23 The switches in figure (a) and (b) are closed at t=0
(D) B1 and B2 lights up together with equal brightness C R L R
all the time. )

Q.19 In figure, a lamp P is in series with an iron-core


inductor L. When the switch S is closed, the brightness
of the lamp rises relatively slowly to its full brightness E E
than it would to without the inductor. This is due to (a) (b)
(A) The charge on C just after t=0 is EC.
P L
(B) The charge on C long after t=0 is EC.

S
(C) The charge on L just after t=0 is E/R.
(D) The charge on L long after t=0 is EC.
B
2 3 . 4 2 | Alternating Current

Q.24 Two coils A and B have coefficient of mutual Comprehension Type Question
inductance M=2H. The Magnetic flux passing through
coil A changes by 4 Weber in 10 seconds due to the Paragraph 1: A capacitor of capacitance C can be
change in current in B. Then charged (with the help of a resistance R) by a voltage
source V, by closing switch s1 while keeping switch s2
(A) Change in current in B in this time interval is 0.5 A
open. The capacitor can be connected in series with an
(B) The change in current in B in this time interval is 2A inductor ‘L’ by closing switch S2 and opening S1.
(C) The change in current in B in this time interval is 8A V
(D) A change in current of 1A in coil A will produce a
change in flux passing through B by 4 Weber.
R S1
Assertion Reasoning Type
C

(A) Statement-I is true, statement-II is true and


statement-II is correct explaining for statement-I.
S2
(B) Statement-I is true, statement-II is true and L
statement-II is not correct explaining for statement-I
(C) Statement-I is true, statement-II is false.
Q.28 After the capacitor gets fully charged, s1 is opened
(D) Statement-I is false, statement-II is true. and S2 is closed so that the inductor is connected in
series with the capacitor. Then,
Q.25 Statement-I: when resistance of rheostat is (A) At t=0, energy stored in the circuit is purely in the
increased, clockwisecurrent is induced in the ring. form of magnetic energy.
Statement-II: Magnetic flux through the ring is out of
the phase and decreasing. (B) At any time t>0, current in the circuit is in the same
direction.
(C) At t>0, there is no exchange of energy between the
inductor and capacitor.
(D) At any time t>0, instantaneous current in the circuit

is V C
L
Q.26 Statement-I: Peak voltage across the resistance
can be greater than the peak voltage of the source in a
series LCR circuit. Q.29 If the total charge stored in the LC circuit is Q0
then for t>=0

Statement-II: Peak voltage across the inductor can be π t 


greater than the peak voltage of the source in a series =
(A) The charge on the capacitor is Q Q0 cos  + 
2 LC 
LCR circuit.
π t 
Q.27 Statement-I: when a circuit having large inductance =
(B) The charge on the capacitor is Q Q0 cos  + 
is switched off, sparking occurs at the switch. 2 LC 

d2Q
(C) The charge on the capacitor is Q = LC
Statement-II: Emf induced in an inductor is given by dt2
di
|e|
∈ =L (A) Statement-I is true, statement-II is true 1 d2Q
dt (D) The charge on the capacitor is Q = −
and statement-II is correct explanation for statement-I. LC dt2
(B) Statement-I is true, statement-II is true and statement-
II is not the correct explanation for statement-I.
(C) Statement-I is true, statement-II is false.
(D) Statement-I is false, statement-II is true.
P hysi cs | 23.43

Paragraph 2: In a series L-R circuit, connected with a ways as shown in column II. When a current I (steady
sinusoidal ac source, the maximum potential difference state for DC or rms for AC) flows through the circuit, the
across L and R are respectively 3 volts and 4 volts corresponding voltage V1 and V2 (indicated in circuits)
are related as shown in column I.  (2010)
Q.30 At an instant, the potential difference across
Column I Column II
resistor is 2 V. The potential difference in volt, across
the inductor at the same instant will be: (A) I ≠ 0, V1 is (p)
V1 V2
(A) 3 cos30o (B) 3 cos60o Proportional to I
(C) 3 cos45o (D) None of these 6 mH 3 F

Q.31 At the same instant, the magnitude of the potential


difference in volt, across the ac source may be V

(B) I ≠ 0, V2 > V1 (q)


4+3 3
(A) 4 + 3 3 (B) V1 V2
2

3 3 6 mH 3 F
(C) 1 + (D) 2 +
2 2

Previous Years’ Questions (C)=


V1 0,=
V2 V (r)
V1 V2

Q.1 A circuit containing a two position switch S is


shown in Figure. 6 mH 2 F

R3 C 
V

2 2  F 1 R5 (D) I ≠ 0, V2 is (t)
R1 E1 V1 V2
Proportional to I
2 12 V 1k 3 F
1 R2
A E2 B

2 S 3V 2 R4
3 V
L
(s)
10 mH V1 V2

(a) The switch S is in two position 1. Find the potential 3 F


difference VA − VB and the rate production of joule heat
6 mH

in R1.

(b) If Now The switch S is put in position 2 at t=0. Find: V

(i) Steady current in R4 and(ii) The time when current


Paragraph 1 (Q.3 to Q.8)
in R4 is half the steady value. Also calculate the energy
stored in the inductor L at that time. (1991) The capacitor of capacitance C can be charged(with the
help of resistance R) by a voltage source V, by closing
Q.2 Match the Columns switch S1 while keeping switch S2 open. The capacitor
You are given many resistances, capacitors and can be connected in series with an inductor L by closing
inductors. They are connected to a variable DC voltage switch S2 and opening S1.
source (the first two circuits) or in AC voltage source of
50 Hz frequency (the next three circuits) in difference
2 3 . 4 4 | Alternating Current

V Q.6 In the circuit shown, A and B are two cells of same


emf E but different internal resistance r1 and r2 ( r1 >r2)
respectively find the value of R such that the potential
S1
difference across the terminals of cell A is zero a long
R
C
time after the key K is closed (2004)
R
S2 R L
L R
A B
R
R
r1 r2
Q.3 Initially, the capacitor was uncharged. Now switch C
s1 is closed and S2 is kept open. If time constant of this R
circuit is τthen  (2006)
S
(A) After time interval τ, charge on the capacitor is CV/2
(B) After time interval 2τ, charge on the capacitor is CV Q.7 In an L-R series circuit, a sinusoidal voltage V = V0
(1-e-2)
sin ωt is applied. It is given that L=35 mH, R=11 Ω ,
(C) The work done by voltage source will be half of the
heat dissipated when the capacitor is fully charged Vrms 220V, ω=
= / 2π 50Hz and
= π 22 / 7.

(D) After time interval 2τ, charge on the capacitor is CV Find the amplitude of current in the steady state and
(1-e-1) obtain the phase difference between the current and
the voltage. Also plot the variation of current for one
cycle on the given graph.  (2004)
Q.4 After capacitor gets fully charged, S1 is opened and
S2 is closed so that the inductor isconnectedin series V
with the capacitor, then (2006)
(A) At t=0, energy stored in the circuit is purely in the
form of magnetic energy. t
T/4 T/2 3T/4 T
(B) At any time t>0, current in the circuit is in the same
direction.
(C) At t>0, there is no exchange of energy between the
inductor and capacitor. Q.8 What is the maximum energy of the anti-neutrino ?
 (2012)
(D) At any time t>0, instantaneous current in the circuit
(A) Zero
C
may V (B) Much less than 0.8 × 106 eV
L
(C) Nearly 0.8 × 106 eV
Q.5 If the total charge stored in the LC circuit is Q0 then
(D) Much larger than 0.8 × 106 eV
for t ≥ 0 (2006)

π t  Q.9 At time t = 0 terminal A in the circuit shown in the


(A) The charge on the capacitor
= is Q Q0 cos  + 
2 LC  figure is connected to B by a key and an alternating
current = I(t) I0 cos (ωt) , with I0 = 1A and ω =500
π t  rad/s starts flowing in it with the initial direction shown
(B) The charge on the capacitor
= is Q Q0 cos  − 
2 LC  7π
in the figure. At t = , the key is switched from B to
d2Q 6ω
(C) The charge on the capacitor is Q = LC D. Now onwards only A and D are connected. A total
dt2
charge Q flows from the battery to charge the capacitor
1 d2Q fully. If C = 20 µF, R =10Ω and the battery is ideal with
(D) The charge on the capacitor is Q = −
LC dt2 emf of 50 V, identify the correct statement(s). (2014)
P hysi cs | 23.45

B D (A) Magnitude of the maximum charge on the capacitor



A before
= t is 1 × 10−3 C

 50 V (B) The current in the left part of the circuit just before
C=20F

t= is clockwise.

R=10 (C) Immediately after A is connected to D, the current
in R is 10 A.
(D) Q= 2 × 10−3 C

PlancEssential Questions
JEE Main/Boards JEE Advanced/Boards

Exercise 1 Exercise 1
Q. 15 Q.21 Q.22 Q. 3 Q.4 Q.7

Q.23 Q.27 Q.14 Q.15 Q.16

Exercise 2 Exercise 2
Q. 1 Q.3 Q. 11 Q.2 Q.3 Q.12

Q.12 Q.14 Q.22 Q.23


Q.28 Q.28 Q.29
Q.30 Q.31

Answer Key

JEE Main/Boards Q.10 It is based up on the principle of electromagnetic


induction.
Exercise 1 Q.11 (i) By using laminated iron core, we minimize loss
V  of energy due to eddy current.
Q.5. Vrms =  0 
(ii) By selecting a suitable materials for the core of a
 2
transformer, the hysteresis loss can be minimized.
Q.6 No
Q.13 (a) ≈ 311sin314t (b) 200V, 127.4V
Q.7 The current lags behind the voltage by phase angle
π/2. Q.14 50Hz
1 1 Q.15 (a) 318.31 Ω (b) 0.527 A (c) 9 W
Q.8 Capacitive reactance, =
XC =
ωC 2πfc Q.16 6.124A
2 3 . 4 6 | Alternating Current

Q.18 (a) 2.20A, (b) 484 W Q.23 (a) Yes. The same is not true for rms voltage,
because voltage across different element may not be
Q.19 0.354A in phase.
Q.20 109.5 A (b) The high induced voltage, when the circuit is broken,
Q.21 (a) 50 rad s-1, (b) 40 Ω , 8.1A, (c) VLcms=1437.5 is used to change the capacitor, thus avoiding sparks,
etc.
 1 
V,Vvcrms=1437.5V,VRms=230 VLCrms= Irms  ω0L − =  0 (c) For dc, impedance of L is negligible and C very
 ω0 C 
high (infinite), so the D.C. signal appears across C. For
Q.22 (a) For
= V V0 sin ωt frequency ac, impedance of L is high and that of C is
low. So, the A.C. signal appears across L.
V0  π
=I sin  ω
= t +  ; If R 0 (e) A choke coil reduces voltage across the tube without
1  2
ωL − wasting power. A resister would waste power as heat.
ωC
Q.24 1.75 × 10−2 H; 5.5Ω
Where- sign appears if ωL >I/ ωC , and+sign appears
if ωL <I ωC . Q.25 0.04H
=I0 11.6A,I
= rms 8.24A Q.26 Resonant frequency=39.79 Hz
(b) VLCrms =207V, VCrms =437 V (i) 2000 Ω (ii) 100 Ω (iii) 2A

(c) Whatever be the current I in L, actual voltage leads (iv) 900 (v)1800
current by π / 2 . Therefore, average power consumed Q.27 (i) 500; (ii) 8.9kW
by L is zero.
(d) For C, voltage lags by π / 2 . Again average power
consumed by C is zero.
(e) Total average power absorbed is zero.

Exercise 2
Q.1 B Q.2 A Q.3 C Q.4 B Q.5 C Q.6 C
Q.7 A Q.8 B Q.9 C Q.10 A Q.11 D Q.12 D
Q.13 C Q.14 A Q.15 D Q.16 A Q.17 D Q18 D
Q.19 B Q.20 A

Previous Years’ Questions


Q.1 A Q.2 B, C Q.3 15V Q.4 C

JEE Advanced/Boards
Exercise 1
LE2
Q.1 0.8 Q.2 [I]−1 Q.3
2R12

1 1 e2 − 1 EL
Q.4 , Q.5 Q.6
15A 10A e 2
eR 2

KLt2 20
Q.7 q = C Q.8 ≅ 2H Q.10 C = 9.2 .F
R π2
P hysi cs | 23.47

Q.11 6.94 A Q.12 (i) i1 = i2 = 10/3A, (ii) i1 = 0, i2 = 30/11A, (iv) i1 = i2 = 0

Q.13 42+20t V Q.14 2A, 400W

1
Q.15 Z = 100 Ω, VQ = 9.8 V Q.16 0.2 mH, µF,8 × 105 rad / s
32

π  1
Q.17 20A, , ∴ Steady state current=20sin π  100t − 
4  4

Exercise 2

Single Correct Choice Type

Q.1 C Q.2 D Q.3 A Q.4 A Q.5 A Q.6 A


Q.7 A Q.8 B Q.9 C Q.10 B Q.11 B Q.12 B
Q.13 D Q.14 D Q.15 A

Multiple Correct Choice Type

Q.16 D Q.17 D Q.18 A Q.19 B Q.20 B, C, D Q.21 A, B, C


Q.22 D Q.23 B, D Q.24 D

Assertion Reasoning Type

Q.25 C Q.26 D Q.27 A

Comprehension Type

Paragraph 1: Q.28 D Q.29 C

Paragraph 2: Q.30 D Q.31 B

Previous Years’ Questions


Q.1 (a) -5v, 24.5w (b) (i) 0.6A (ii) 1.386 × 10–3s, 4.5 × 10–4 J

Q.2 A → r, s, t; B → q, r, s, t; C →q, p; D → q, r, s, t

4
Q.3 B Q.4 D Q.5 C Q.6 R = (r – r )
3 1 2
π
Q.7 Amplitude = 20A, phase difference = Q.8 C
4
Q.9 C, D
2 3 . 4 8 | Alternating Current

Solutions

JEE Main/Boards Im

z
Exercise 1
x
z
Sol 1: In a resistance coil, when an alternating current 
is flown, there will be a magnetic field generated across R
Re
the coil and so there will be an inductance induced into
the coil. Hence it will have more impedance compared
Sol 4: As derived above,
to the one withDC current.
ZR = R
Sol 2: We know that power dissipated = VI cosθ. ZL = iωL
R  ZC = –i/ωC
cos θ =   ⇒ power factor
Z znet=ZR + ZL + ZC (Since they all are in series)
now for an ideal inductor, Z = ωL and R = 0
Now we can write any quantity in phasor notation,
∴ cos θ = 0
for V = V0 cos (ωt + θ)
Hence power = VI (0) = 0
we write this quantity in phasor notation as,

Sol 3: Impedance is the effective resistance of an V = |V| ∠q


electric circuit or component to alternating current, ⇒ V = V0∠θ. [θ is the phase angle].
arising from the combined effect of ohmic resistance
This is very helpful for us.
and reactance.
R L C Now for the given potential, V = V0 sin ωt
π
V = V0 cos (ωt – )
2
V = V0 cos t 
∴ V= V ∠ − π  ........(1) … (i)
0
2
i
We got Znet = ZR + ZL + ZC= R + i ωL –
Now let ‘i’ (iota) be the complex number, square root ωL
of –1.  1 
Znet = R + i  ωL − 
Now, Impedance of resistance ‘R’ = R ≡ ZR  ω C
Impedance of Inductor ‘L’ = i ωL ≡ ZL
2
 1 
 −i  now |Znet| = R 2 +  ωL − 
Impedance of capacitor ‘C’ =   ≡ ZC  ωC 
 ωC 
now net Impedance of the circuit (figure (i)) is  1 
 ωL − 
Znet = ZR+ ZC + ZL tan θ =  ω C
 R 
 
i  1   
=R– + iωL = R + i  ωL − 
ωC  ω C
P hysi cs | 23.49

Im Sol 6: No nothing is perfect. It is impossible to make a


perpetual machine.

z Sol 7: Using the notation used in Q.4 and Q.5;

 L
Re
R

With this we can write V = V0 cos t 


Znet | Znet | ∠θ 
= … (ii)

Now we known that


∴ In phasor notation: V0 = V0∠ 0
V = I × Z [ V = I R]
π
v0∠ −
π ZL = iωL ⇔ ZL = |ZL| ∠
2
I =  V  ; I =

2


Z Z 0 ∠θ [ use complex analysis in maths.]
π
⇔ ZL = ωL ∠ .
=I  V0  π
 ∠ − − θ 2
Z 2
 0  Now we know that V = I Z L

I = I ∠ –  π + θ   … (iii) V0 ∠0
0   = I
2  π
ωL∠
π  2
Phase of current = –  + θ 
 2  I V0 ∠ − π
=
π ωL 2
Phase of voltage = –
2 π
⇒ I = I0 ∠ −
∴ Depending upon the ‘θ’ we can speak more about 2
the relation between fV and φI.
Phase of voltage = ∠ 0 = zero
Sol 5: Let V = V0 sin (wt + θ) be an ac voltage source.
Then
1/2
T 2 
 ∫ V dt 
  V(0)
Vrms =  0 
T
 dt 
 ∫ 
 0  I
1/2
T 2 

2
 ∫ V0 sin (ωt + θ) 

 
-

2
Vrms =  0  π π
 T  Phase of current = ∠ − = −
  2 2
 
Hence current lags behind the voltage by an angle of
now for simplifying the calculation,
π
∴ We put θ = 0, and solve;  .
2
V 
we get Vrms =  0  Sol 8: ω = 2pf
 2
Now as derived in Q.4;
2 3 . 5 0 | Alternating Current

−i −i V0
ZC = = ∴ vrms =
ωC 2πfC
2

Sol 9: V= V0 ∠0 [In phasor] …..(i) V0 = (vrms) 2

C V0 = (220) ( 2)

V0 = 311 V.

V = V0 cos t  And given f = 50 HZ;


ω = 2pf= 2π(50) = 100 p
ω = 314
∴ v = 311 sin (314 t)
−i  1  π
ZC = =  ∠ − (b) Given Vrms = 100 2V;
ωC  ω C  2
V0
 π We know that Vrms =
= Z C | Z C | ∠ θ ; for i → ∠ −  2
 2 
π Comparing both of them;
 −i → ∠− 
 2 V0 = 200 V

I V= V0 ∠0 V = 200 sin (ωt)
Now =
Z  1  π
 ∠ − V = 200 sin (314 t)
 ωC  2

Now; ω =
I = V ωC∠ π T
0
2
 2π t 
⇒ V = 200 sin  
I= I ∠ π …..(ii)  T 
0
2
T/2
Now power dissipated P = V I standard notation get  2πt 
familiar with this ∫ 200 sin 
 T 
 dt
Average = 0
= 127 V.
 T/2
π
P = (V0∠ 0)  I0 ∠ 
 2 ∫ dt
0
π
P = V 0 I0∠ 0 +
2 Sol 14: Let ‘f’ be the required frequency
π
P = V 0 I0∠ ω = 2pf
2
now V = V0cos (2pft)
π
And cos = zero
2 we are given Vrms = 200 V
Hence P = 0. V0
∴ Vrms =
2
Sol 10: Refer to theory.
V0 = 200 2 V

Sol 11: Refer to theory. ⇒ V 200 2∠0 


= ….(i)
ZL = iωL = i(ω) (i)
Sol 12: Refer to theory.
= iw≡ i2pf
π
Sol 13: (a) Instantaneous voltage V = V0 sinωt now V0 is Z L = 2pf ∠
the maximum possible voltage (or amplitude) 2
V
220 V given is the RMS value of voltage now I =
Z
P hysi cs | 23.51

∴ Imax = 5.0 2 A
I = 200 2∠0
π
2πf∠ Let ∴ I = 5 2 sin(100πt)
2
π 1
I = I ∠ − when t = sec
0
2 300
then
200 2
we want I0 = = 0.9 π 3
2πf I = 5 2 sin = 5 2× 2.5 6 A
=
3 2
200 2
∴f= H ≡ 50 Hz
2π(0.9) Z Sol 17: Vrms = 220
V0 = 2 (Vrms)
Sol 15: V0 = Vrms . 2
V0 = 220 2
100 10f ω = 2pf

V = 220 2 cos (2pft)

V = V0 cos t  V= 220 2 ∠ 0
V = V0 cos (t) C


(a) V0 = 100 2
ω = 2π (50) = 100 p
∴ V = 100 2 cos (100 pt) = 100 2 ∠ 0
−i 1 π
ZC = = ∠ − [In phasor notation]
ZR= R = 100 ωC ωC 2
 −i  −i
ZC =  = = –i (318) W  1  π
 ωC  (100π)(10 × 10−6 ) Z C 
= ∠ −
 ω C  2
∴ Resistance of capacitor is |ZC| ≈ 318 W
V V0 ∠0
(b) now Znet = ZR + ZC Now I = =
Z  1  π
C
 ∠ −
Znet = 100 – i (318)  ωC  2
I = V ωC∠ π + 0
0
 −318  2
Znet= (100)2 + (318)2 ∠ tan−1  
 100 
I = V ωC∠ π
Znet = 334 ∠ –72.5° 0
2

I V= 100 2∠0 = 0.42 ∠72.5=0.527 A
= ∴ Phase of current =
π
Z 334∠ − 72.5 2
(c) Pavg= Vrms Irms cos f Phase of voltage = 0
π π
 0.42  ∴ φI– fv= –0=
= (100)   . cos(72.5) = 29.9 cos (72.5) 2 2
 2 
Sol 18: V = 220 2 cos (50 (2π) t)
Pavg = 9 watt
V = 220 2 cos (100 pt)
Sol 16: f = 50 Hz ∴ ω = 2π × 50 = 100 p
=V 220 2∠0
Irms= 5.0 A
2 3 . 5 2 | Alternating Current

R = 100 Sol 20: V = 220 2 cos (2π (50) t)

10 C
220 V  A
50 H
220 V 
50 H
(a) ZR = R = 100 ⇔ ZR = 100 ∠ 0

V = I Z
V = 220 2 cos (100pt)

I V= 220 2∠0
= V = 220 2 ∠ 0  … (i)
Z 100∠0
Now let ‘C’ be the capacitance of the circuit;
=I 2.2 2∠0
−i −i  1  π
ZC = = =  ∠ −  … (ii)
⇒ I = (2.2) 2 cos (100pt) ωC 2πfc  2 πfc  2
ZR = R = 10Ω = 10 ∠ 0  … (iii)
now I0 = (2.2) 2
Now Znet = ZR + ZC
I0 (2.2)( 2)
Irms = = = 2.2 Amp.
2 2 1
Znet= (10 + ZC) = 10 – i
2πfc
(b) Net power over a full cycle
2
 1 
(Vrms )2 (220)2 |=
Znet | (10)2 +  
= = = 484 watt  2πfc 
R 100
 1 
Sol 19: V = 110 2 cos (2π (70) t) −   1 
tan θ =  2πfc  =  − 
 R   2πfcR 
 
1H  
 −1 
θ = tan–1  2πfRC 
 
110 V

10HZ  −1 
∴ Z = (10)2 + (X C )2 ∠ tan−1    … (iv)
 2πfRC 
Now V = 
IZ
V = 110 2 cos (140 pt)= 110 2 ∠ 0 …(i) 
I = V
ZL = iωL = i (140 π) = i (140 π) Z
|ZL| = 440 W 220 2  −1 
=I ∠0 − tan−1  
π 100 + X 2C  2πfRC 
ZL= 440 ∠ …(ii)
2
220 2
 Now I0 =
I V= 110 2∠0
= 100 + X 2C
Z π
440∠ I0 220
2 Irms = =
2 100 + X 2C
1 π  1   π
=I ∠− =   cos  140πt − 
2 2 2 2 2   2 Irms = 2A (Given)
1 220
I0 = = 0.354 Amp. ⇒2=
2 2 100 + X 2C
P hysi cs | 23.53

100 + X 2C =
(110)2 V = – (813 ∠ 0) (40)
V =
−325∠0
XC = 109.5 A
⇔ V = – 325 cos (50 t)
1 −325
Sol 21: Z = ZR + ZL + Z C = 40 + iωL – Vrms = = −230
ωC 2
 1  (b) Inductance:
Z= 40 + i  ωL − 
 ωC 

Now condition for resonance is Imaginary part of


V = − 
IZL( ) = – (8.13 ∠ 0)  50 × 5∠ 2π 
 
Impedance is zero  π
−  2033 ∠  → (x1)
V=
 2 
R = 40 5H 80F
  π 
V=
− 2033cos  50t +  
  2 
 V  (c) Capacitor:

(
V = − I Z c )
  1 π 
V = 230 2 cos(ωt) VC= – ( 8.13∠0 )  ∠ − 
  50 × 80 × 10−6 2 
= 
V 230 2 < 0 → (1) … (i)
 8.13 π
1 VC = −  ∠− 
∴ ωL – =0  50 × 80 × 10 −6 2
ωC
1 1  π
⇒ ω2 = , ωC = VC = − 2033∠ −  → (x2)
LC LC 2
 
1 1 
ω= =  π 
−6 20 × 10−3 ⇔ VC = – 2033cos  50t −  
5 × 80 × 10   2 

1000 Now from equations (x1) and (x2)


= = 50 rad/s
20 we get VL + VC = 0.
w 50 25
f= = = ~
− 8HZ Study more effectively on Resonance conditions.
2π 2π π

Z =40 + i(0) =40 Sol 22: ZL = iωL = i (100 π) (80 × 10–3)

⇔ Z = 40 < 0 →  … (ii) 80 mH 60F


80 mH 60 µF
V
Now from (i) and (ii), I =
Z
230 V 
I 230 =
2∠0 23 2 230 V,50 ~
= ∠0 HZ
40∠0 4 50 Hz

I = 23 2 cos (50t)
= V 230 2 cos(2π(50)t)
4
=V 230 2 cos(100πt)
I = 8.13 cos (50 t)
Now potential drop across =V 230 2 < 0

(a) Resistance: ZL = i (8π)

( )
V = – IR ZL ⇔ 8π <
π
 … (i)
2
2 3 . 5 4 | Alternating Current

−i −i 500i Power transferred to Inductor


ZC = = = −
ωC 100π × 60 × 10 3π
( ) ( I ) = (290 <π)
− 6
= VL
500 π
ZC ⇔ ∠−  … (ii)
3π 2  π
500i  11.6∠  From
From(i)Xand
→ (1)
(ii)and (4)
 2 
= ZL + Z C = 8pi –
Znet


 500  = (290 × 6) ∠
Znet=  8π − i 2
 3π  ⇒ Znet = – 28 i
 3π 
π = 290 × 6 cos  
⇔ Znet = 28 ∠ −  … (iii)  2 
2
V = Zero
Now V = I Znet ⇒ = I
Znet
Similarly zero for the capacitor to.
230 2∠0
⇒ I = Total power absorbed by the circuit is
π
28∠ −
2 P = V( ) ( I ) = ( 230
)
π
2 ∠ 0  11.6 ∠ 
 2 
I 230 2 ∠ π
=
28 2 (
P = 230 2 × 11.6 ∠
π
2
)
I = 6 ∠ π 
P = ( 230 2 × 11.6 ) cos
… (iv) π
2
 2
π
⇒ I = 6 cos  100πt +  P = zero
 2
11.6
I0 = 6 and Irms= = 8.2 amp Sol 23: Explained in the key.
2
Potential drop across;
Sol 24: Initially
(a) Inductor;
XL = 22 at f1= 200 HZ
 π  π
VL= I . ZL =  11.6∠   8π∠  [ω1 = 2π × 200]
 2  2
(XL)A = ω 1L = 22
VL = (11 6 × 8π) <p
VL = 290 ∠ p→ (x1) ⇒ 2π × 200 L = 22  … (i)

VL = 290 cos (100 pt + π) 22


L= = 1.75 × 10–2 H and finally;
400π
VL = 290; VL =
O ( ) O rms
290
= 205 V
2
f2 = 50Hz

ω22 2π (50)
(b) Capacitor
X2 = ω2L  … (ii)
 π   500 π
VC = I( ) ( Z C ) =  11.6∠  
 2   2 π
∠− 
2 
(i) ⇒ x1 =
(1) 2π × 200 × L
(2)
(ii) x 2 2π × 50 × L
11.6 × 500
VC = ∠0 x1
3π =4
x2
VC = 616 ∠ 0 → (x2) x1 22
x=
2 = = 5.5 ohm.
4 4
⇔ VC = 616 cos (100 pt + 0)
616
(VC)O = 616 (VC)rms= =4
2
P hysi cs | 23.55

Sol 25: Resistance of the lamp 90 8kW 8 × 100


= ⇒ Pi = kW
220 V 100 Pi 90
= = 22 ohm.
10
80
Let ‘L’ be the Inductance of the lamp; ⇒ Pi = kW ⇒ Pi= 8.9 kW.
9
XL = ωL = (100 π) L
Znet = 22 + i (100 πL) −i
Sol 28: XL = ωL ; XC =
ωC
 100πL 
Z=
net (22)2 + (100πL)2 ∠ tan−1   Now Now as ω is increased, both XL and XC increase.
 22 


I = V = 250 2∠0 Sol 29: XL = ωL
Znet  100πL  x1 ω1 ω2
(22)2 + (100πL)2 ∠ tan−1   ⇒ x2 = . x1
 22  =
x2 ω2 ω1
250 2  100πL  ⇒ x2= 2x
I = ∠ − tan−1  
484 + (100πL) 2  22 
 −1 
xc =  
250 2 I0  ωC 
I0 = and Irms=
484 + (100πL)2 2 x1 w2 x 
= ⇔ x2 =  1 
x2 w1  2
250
⇒ Irms = Phasor method:-
484 + (100πL)2
Let V = V0 cos (ωt + q1) be the emf of an AC-source,
Put we are given that Irms = 10 A; then can write this is phasor method as,
250 V = |V| ∠ q1⇔ V = V0 ∠ q1
∴ 10 =
484 + (100πL)2 Now for I = I0cos ( ω t + q2)
484 + (100 πL)2 = 625 ⇔ I = I0 ∠ q2
141
100 πL = 141 ⇒ L = Now let Impedance (Z) ;
100π
11.9 ZRe sis tance = R
⇒L= L = 0.04 H.
100π −i
Z capacitor =
ωC
Sol 26: Current drawn in circuit is maximum when the
(i is iota; complex number)
circuit is in Resonance i.e. the Imaginary part of the
circuit is zero. Zinductor = iωL
Now solve this question exactly as solved in Q. 21. Now in a circuit with series RCL;
i
VS NS Znet
= Z=
R Z C + ZL = =
R + i ωL
Sol 27: = ωC
VP NP
 1 
220 NS Znet = R + i  ωL − →  … (i)
=  ωC 
2200 5000
Now let us write this in phasor notation,
NS = 500 turns.
Znet | Znet | ∠ q
=
Output power
n (efficiency) =
Imput power
2
 1 
8 kW | Znet=
| R 2 +  ωL − 
x=  ωC 
Pi
2 3 . 5 6 | Alternating Current

 1  E = (0.1) (0.1) 1 ⇒ E = 10–2 V


 ωL − 
ω C now applying KVL in mesh (i)
θ = tan–1  R
 
  E – i (i) – i1 (2) = 0
 
 1  E = i + 2i1 ... (i)
2
 ωL − 
 1  ωC 
= R 2 +  ωL −
∴ Z net −1
 ∠ tan  In mesh-(ii);
 ωC   R 
 
  ⇒ E – i (i) – 3i2 = 0
Now for a source of emf V = V0 cos (ωt + q1)
⇒ E = i + 3i2 ... (ii)
Img
⇒ i = i1 + i2 ... (iii)
1
From this we get i = A.
 L -
1
C Znet
220

Sol 2: (A) For an L–R circuit,



Re L 
R T (time constant) =  
R 
⇔ V = V0<q1 1
Now energy stored in magnetic field is LI2 and rate of
V V0 ∠θ1 dissipation of energy is I2R. 2
=I =
Z  1 
 ωL −
net

| Znet | ∠ tan  −1 ωC 
Sol 3: (C) At t = 0, inductor is open circuited
 R 
 
  At t = ∞, inductor is short circuited
 1  At t = 0,
 ωL − 
I = V0 –1 
ωC 
∠ q1– tan R 10 V
| Znet |  
 
 
I0 10
For resonance, imaginary part in eq. (i) is zero!

Exercise 2 10
10
=i = 1 amp
10
Sol 1: (B) Emf induced in rod = BLv At t = ∞
A D 10

2  3 10

R
B C

i1 i2 10
i 10V 10V
=i = = 2 amp.
2
1 Rnet 5
3
∴ Difference = (2 – 1) amp= 1 amp.
(1) E (2)
P hysi cs | 23.57

 L  1
Sol 4: (B) T1 (time constant) during build up =   ⇒ i = 0.6   ⇒ i = 0.3 amp
 2R  2
 L 
T2 during decay =   di
Emf across coil = L
 3R  dt
T 3
∴ 1 = di di
T2 2 = i0 (– (–2) e–2t) ⇒ = 2 i0 e–2t
dt dt

di Emf = 2L (0.6) e–2t


Sol 5: (C) Energy stored per unit time = Li
dt
= 2 (2) (4) = 16 J/s. ⇒ E = 6 e–2t ⇒E = 6 e−2ln 2

ln2 −1 1
di E= 6e ⇒E=6× E = 3V
Sol 6: (C) i = 2 amp = 4 amp/s. 2
dt
Applying KVL, Sol 8: (B) i = 5 amp
4V di
= –103 A/S
dt

1
[Since decreasing; –ve sign]

1 5mH

1H A B
3F 15V
di Q di
⇒ 4 – i (1) – L − =0 VA – i(1) + 15 – L = VB
dt C dt
Q di
⇒ 4 – 2 (1) – 1 (4) – =0 VA – VB = i – 15 + L
C dt
⇒Q=–2×3 VA– VB = 5 – 15 + 5 × 10–3 (–10+3)
⇒ Q = 6C.
VA – VB = 5 – 15 – 5 ⇒ VA – VB = –15 V
 −Rt 
Sol 7: (A)
=i i0 1 − e L 
  Sol 9: (C) When ‘i’ is reversed,
 
1 5mH
6V R L
A i B
15V

 di 
VA + i (1) + 15 – L   = VB
 dt 
6v di
i0
= = 0.6 VA – VB = –i – 15 + L
10 dt
 −10t 
=i 0.6  1 − e 5  ⇒ i = 0.6 (1 – e )
–2t
= –5 – 15 + 5 (+10-3) × 103
 
  di
[i is decreasing against the direction of KVL. Hence
Put t = ln 2 dt
= 103].
(
⇒ i 0.6 1 − e
= 2 n 2
) VA – VB = – 5 – 15 + 5
 n 2−1 =i 0.6  1 − 1  VA – VB = –15 V
⇒ i 0.6  1 − e
=  ⇒  
   2
2 3 . 5 8 | Alternating Current

Sol 10: (A) At t = 0, inductor is open circuited, Sol 12: (D) Just before the switch is opened, let us find
the currents,
at t = ∞, it is short circuited
at t = 0, L i2
A B
i1

12 V R1 R2

10 20
l2 V
i=
Rnet
2V
 RR  4×8 8
2V 2 Rnet=  1 2 = = Ω
i=
Rnet
⇒ i1 =
10 + 20  R1 + RL  12 3

12 9
2 1 i= ⇒ i = amp.
i1 =
⇒= amp. 8 2
30 15
3
Finally; at t = ∞ Now just at the instant switch is opened, i would remain
same
L B
A

10 20 12 V R2

R1

2V
9
2V 2 ∴ VR = i R1 = × 4 VR = 18V
i2 = ⇒ i2 = amp 1 2 1
Rnet 20
Now applying KVL;
1 12 + (VB – VA) – 18 = 0 ⇒ VB – VA = 6 V.
i2 = amp.
10
Sol 13: (C) Power factor,
Sol 11: (D) At t = 0, no current flows in the circuit.  
R
cos φ =  
 2 2 
Fuse L = 4H  (X C − XL ) + R 

V=2sin (100t)
S ~

2V
10Ω 0.1H
As time starts, current starts flowing and at t = ∞,
current in the circuit is infinity. C 10 0.1H
Hence at t = 10, i → ∞ so the fuse will get blown
∴ C
[ Infinity is just an unknown number !]
P hysi cs | 23.59


Im I V=
=
220 2
Z 2
z 2  1 
(100) +  100 − 
 100 πC 
(XC-XL)

 R  1 
R  100 − 
100 πC 
1 π –tan–1  100
cos φ = ⇒ θ=  
2 4  
 

| X C − XL | 220
∴ tan φ = ⇒ |XC – XL| = R irms =
R
1  ≡ 2.2
2

(100)2 +  100 − 
XL = ωL = (0.1) (100) ⇒ XL = 10 Ω.  100πC 

⇒ |XC – XL| = R 2
220  1 
= (100)2 +  100 − 
1 2.2  100πC 
= R + XL
ωC
2
1 1  1 
C= ⇒C= ∴ (100) = 100 +  100 −
2 2

ω(R + XL ) 100(20)  100πC 
1 1
C= × 10−3 ⇒ C = 500 µF. ⇒ 100 − =0
2 100πC
 −1 
1 ∴ XC = – 100 ∴ X C = 
Sol 14: (A) ZL = iωL = × 100 π = i 100 W  ωC 
π
1  XC 
H 100 C Now power factor; φ = tan–1  R 
 
 −100  π
φ = tan–1  100  ⇒ φ = −
  4
π 1
Power factor; cos φ = cos   =
 4 2
ZR = 100 W
Sol 15: (D) For 100 V D.C. source, i =1 amp.
−i −i
ZC
= =
ωC 100πC 100
Hence,=
R = 100Ω
1
Znet = ZR + ZL + Z C
Now for AC source of 100 V
i
Znet = 100 + i (100) –
100πC 100 1 100
i=
Znet ⇒ 2 = Z
net
2

2 1 
Znet= (100) +  100 −  ⇒ Znet = 200
 100πC 
Z=
net R 2 + XL2
 1 
 100 − 
−1
tan  100 πC  ∴ R2 + XL2 = (200)2 ⇒ XL2 = (200)2 – (100)2
 100 
  XL = 174 W
 
2 3 . 6 0 | Alternating Current

ω L = 174 tan q2= 3


174
L= ⇒ L = 0.55 H.
100π R1 3 3
∴ =
R2 4
Sol 16: (A) I = I0 + I1 sin ω t
T
(*) Don’t run to catch cos θ.
2
∫I dt

Use tan θ and simplify!
I2rms = O
T=
T ω
Sol 19: (B) i = 2 sin 100 pt + 2 sin (100 pt + 30°)
∫ dt
O
T
It is similar to superimposition of two vectors with an
(I20 I12 2 angle of 30° in between them
∫ + sin ωt + 2I0 I1 sin ωt)dt
= O inet = i0 sin (100 pt + θ)
T

∫ dt i0 = 22 + 22 + 2(2)(2)cos(30°)
O

I12 T i0 =
I20 T + +0 8 + 8 3 ⇒=
i0 2 2 + 3
2 2 I12
Irms = ⇒ Irms
= I20 +
T 2
Phase diagram will be shown as
π π 1
Sol 17: (D) = ωt ;= 100πt ; t = s. o
4 4 400 i2 = 2 sin (100t + 30 )

Sol 18: (D) for LR circuit; ieffective


  /3
R1
cos q1=   = 0.6  
... (i)
 2 2 
 R 1 + XL  i1 = 2 sin 100t

for CR circuit;
Sol 20: (A) We can speak on nature by observing the
 
R2 phase of final Impedance. If the phase of Impedance is
cos q2=   = 0.5  ... (ii)
 2 2  negative then it is capacitive, else it is inductive.
 R2 + XC 
ω 1
Now when L, C, R of two circuits are joined; ∴ ω' = =
2 2 LC
  ∴ ZR = R
R1 + R 2
cos θ =  
 2 2  1 1 L
 (R1 + R 2 ) + (X C − XL )  ZL = i ω L = i . .L=i
2 LC 2 C
Given that cos θ = 1
−i −i L
∴ XC = XL = X ZC = = = −2i
ωC 1 C
.C
X  2 LC
tan q1=  L 
 R1  3i L 3i L
X  ∴ ZL + Z C =
− ; Znet= R −
tan q2=  C 2 C 2 C

 R2 
 −3i L 
tan θ1 XL R 2  R 2  Znet= Z 0 ∠ tan−1  
= . ≡   2R R 
tan θ2 R1 X C  R1   
4 ∴ -ve phase
tan q1=
3 Hence capacitive.
P hysi cs | 23.61

Previous Years’ Questions Sol 4: (C) For the lamp with direct current,
V = IR
π ⇒ R =8Ω and P =80 × 10 =800 W
Sol 1: (A) As the current i leads the emf e by , it is an
R–C circuit. 4
For ac supply
1 2
XC π Erms
tan φ = or tan = ω C \ωCR = 1 =P I 2
= R R
rms
R 4 R Z2
(220)2 × 8
As ω = 100 rad/s ⇒ Z2 =
800
1 –1
The product of C–R should be s ⇒ Z=22Ω
100
⇒ R 2 + ω2L2 =(22)2
2
 1 
Sol 2: (B, C) Z = R 2 + X 2C = R2 +   ⇒ ωL = 420
 ωC  ⇒ L=
0.065 H
In case (b) capacitance C will be more. Therefore,
impedance Z will be less. Hence, current will be more.
∴Option (b) is correct. JEE Advanced/Boards
Further,
Exercise 1
Vc = V 2 – VR2
Sol 1: At t = 0, we can replace the inductor by open
= V 2 – (IR)2 circuit and at t = ∞, the inductor can be short circuited

In case (b), since current I is more. at t = 0,

Therefore, Vc will be less. 6

dI
Sol 3: = 103 A/s
dt 10 4 4
I
A B
1Ω 15 V 5 mH 10
di 10
∴Induced emf across inductance, |e| = L |e| = i1 = = 1 amp.
dt 10
(5 × 10–3) (103) V = 5 V
At t = ∞,
Since, the current is decreasing, the polarity of this emf
6
would be so as to increase the existing current. The
circuit can be redrawn as

10
I = 5A 4 4
A B
1Ω 15 V 5 mH

Now VA – 5 + 15 + 5 = VB 10 10
i2
= = amp
\VA – VB = – 15 V R eff 8
or VB – VA = 15 V i1 1 8
= = = 0.8 amp
i2 10 10
8
2 3 . 6 2 | Alternating Current

L
Sol 2:
RCV
L L
V = IR ⇒ ⇒
RC(IR) R(RC)I 10
Now {RC} = time constant in RC circuit 20

∴ L 
[RC] = [T] and   = time constant in LR circuit 2V
R 
2
I=
∴ L  Rnet
  = [T]
R 
2
I1 =
 L  [T] −1 10 + 20
∴ =  = [I] .
 RCV  [T] [I] 2
I1 = amp.  … (i)
30
Sol 3: Let us calculate the total energy stored in the at t = ∞,
inductor before switch is shifted.

R2
10
20
L
2V
Here the resistor 10 Ω is shorted.
2 2 1
E R1 I2
= = = amp.
Rnet 20 10
1 2
Energy stored in inductor = LI
2 Sol 5: Let us now derive the current in the circuit as a
1 function of time
= L [Iat t = ∞]2
2
R=10 L=5
 E 
It =∞ =  
 R1  i

2 V
1  E 
∴E= L  
2  R1 

LE2
E= at time t = t; current = i amp;
2R12
using KVL;
Now this is the total heat produced in R2.
di di
V – iR – L = 0 ⇒ V – iR = L
dt dt
Sol 4: This is similar to the Questions 1 (Ex. I). 1 di
⇒ dt =
At t = 0; Inductor is open circuited, L V − iR
At t = ∞, Inductor is short circuited. Integrating;
At t = 0; 1
t
di
i

L0∫ dt = ∫
i
V − iR
0
⇒ i = i0 1 − e−Rt/L
( )
P hysi cs | 23.63

At t = 0, i = zero E = 2kLt
At t = ∞, i = i0 = constant E  2kL 
Current in the coil II is = t
R  R 
Now R = 10Ω, L = 5
dq
i=
(
i = i0 1 − e−2t
) dt
t
At t = 1 sec  2KL 
i −2
q= ∫ i dt ⇒q= ∫  R 
 t dt
i = i0 (1 – e–2) ⇒ = (1 − e ) t =0
i0
t
 2KL  t2
 i

 e2 − 1
 = q= .
 R  2
⇒ q=
2KL 2
2R
t( )
 i0  e2 0

KLt2
Sol 6: i = i0 (1 – e–Rt/L) q= C
R
dq
i= ⇒ q = ∫ i dt
dt
Sol 8: Power factor is cos (θ)
 −Rt 
=q ∫ i0  1 − e L  dt Img
 
 
t −RT 
=q i0 ∫ 1 − e L  dt
 XL+XC
t0 
t 
 −Rt  R
⇒ q = i0 t −  − L  e L  R
 
  R  
0 Given that cos θ = 1 ⇒ θ = 0
 −Rt
L 
⇒ q = i0 t + L eL

−  0 + 
∴ |XL + XC| = 0 ⇒ XL = – XC
 R  R 
 XL = ω L
  −Rt  
⇒ q = i0 t − L  1 − e L   XL =
−1
⇒ ωL =
1
 R  ωC ωC
  
1 1
i0L  −Rt  ω = ⇒L=
⇒ q=
i 0t − 1 − e  LC ω2C
R  L 
ω = 2π (50) = 100 p
L 
One time constant ⇒ t =   1 20
R  L= = = 2H.
(100π) C π2 2

L i0L
⇒ q = i0 . −
R R
(
1 − e−1 ) Sol 9: We know that VR and VL will have a phase
π
i0L i0L  1 i0L difference of .
q= − 1 −  ⇒ q = 2
R R  e Re
EL Vnet = VR2 + VC2 = 162 + 122 = 20V.
q=
R 2e
Sol 10: Resistance of Lamp = R
Sol 7: Given mutual inductance between coils = M
 V 2   100 × 100 
And I1 = kt2 R  =
=  =  200Ω
 P   50 
 
dI
∴ EMF induced in second coil= L = L [2kt]
dt Maximum current the lamp can sustain,
2 3 . 6 4 | Alternating Current

Pmax now at t = 0.1 ln 2, S2 is open;


imax =
V

50 1
i=
max = amp. 40
100 2 100 V
Now in the given conditions;
(200 V, 50 HZ ) 10

200V
i= = 1 amp which is greater than 0.5 amp. 1H
200Ω
Hence we need to increase the Impedance by using a
capacitor of capacitance ‘C’. Such that ‘ I’ will be equal

to
1
amp.
∴ inew =
100
50
(
1 − e−50t )
2

∴=
Z
 1 
R +2
2 (
= 2 1 − e−50t 
inew ) … (ii)

 ωC  But this equation; at t’ = 0, we get inew = 0

200 But this is not true; Since there is a current flowing in


I= the circuit at that instant.
2
 1 
R2 +   Also t’ = 0 ⇒ t = 0.1 ln 2 sec.
 100 
∴ t’ = t – 0.1 ln 2
1 1 200
I = amp ⇒ =
2 2 2 ∴ inew = i0 1 − e−50(t −0.1 ln2)  ; t ≥ 0.1 ln 2  ... (iii)
 1   
R2 +  
 ωC  100
2
i0
= = 2 amp.
 1  2
50
R2 +   = (400)
 ω C  using equation (iii) at time t = 0.1 ln 2, i = 0
But this is not true, since there is a current flowing in
2
 1  2 the circuit guided by the equation,
(200)2 +   = (400)
 ω C  i = 10 (1 – e–10t) [from eq.(i)]
ω = 2π (50) = 100 p now at t = 0.1 ln 2
Solving this will give the value of ‘C’.
i = 10 (1 – e–10t(0.1)ln2)
 1
100 
−10t 
=i 10  1 −  ⇒ i = 5 amp.
Sol=
11: i 1 − e e   2
10  
 
∴ inew = 5 + 2 1 − e−50(t −0.1 ln2) 
 
S1
At time t = 0.2 ln2
 ln2−5 
40 S2 inew = 5 + 2 1 − e−50(t −0.1 ln2)  = 5 + 2 1 − e 
100 V  
 1
10 =5 + 2 1 − 
 25 

 31 
1H inew = 5 + 2  
 32 

(
=i 10 1 − e−10t  ) … (i)

inew =  5 +

31 
16
 amp. = 6.94 amp.

P hysi cs | 23.65

Sol 12: After switch is closed; 20


10 30 i3 =
A 11
(i) t = 0; open circuiting the inductor; B
i2
100 10
i1= i2= = amp.
30 3 20
10 i1 30

i2 D C
20
100 V 20 ∴ Hence the current in loop ABCD will be amp.
11
And this current will start decaying to zero
∴ At t = ∞, i = zero.
(ii) now at t = ∞ ;
inductor is short circuited, Sol 13: Applying KVL;
10 i1 30 R i(t)
i2
100 20
E L

100
i1 =
Rnet di
E – i (t) R – L =0
dt
100 50
i1 = = amp di
22 11 i(t) = 3 + 5t ⇒ =5
dt
10 i1 i3 30 E = R i(t) + L(5) ⇒ E = 4(3 + 5t) + 5(6)

i2 E = 42 + 20t

100 20 Sol 14: Now when capacitance is removed;


R L CR L C

and i1= i2 + i3
200 V, ~ 200 V 
2i2
2i2 = 3i3 ⇒ i3= 300 rad/s. 300 rad/s
3
2i2 5i2
i 1 = i2 + ⇒ i1 =
3 3
V = 200 2 cos(300t)
3 3  50  30
i2 = i=  = amp.
5 1 5  11  11 =V 200 2∠0
20
i3 = amp. Znet
= ZR + ZL
11
(iii) Now when switch is open Znet = R + iωL

(a) Immediately after that, current through inductor will  ωL 


be same as just before Znet = R 2 ω2L2 ∠ tan−1  
 R 

I = V
Z net
2 3 . 6 6 | Alternating Current

Sol 15: Maximum current flows when the circuit is in


200 2∠0
I = resonance
P Q
 ωL 
R 2 + ω2L2 ∠ tan−1  
P Q
R
 R  L
L R
200 2  ωL  1µF 32 4.9 H 68
=I ∠ − tan−1   1F 32 4.9 H 68
R 2 + ω2L2  R  ~

Now given that current lags behind voltage by 60°,
 ωL 
∴ tan–1  R  = 60
  =V 10 2 cos(ωt)
ωL =V 10 2∠0
∴ = 3 ⇔ XL = R 3 → (x1)
R 1
i.e. ω =
R 3 100 3 LC
L= ⇒L=
ω 300 1 1
ω= ⇒ ω=
1 1 × 10 −6
× 4.9 × 10 −3
49 × 10−10
L= H.
3 1
ω= × 105 rad/s.
7
Now when the inductance is removed;
By intuition we can say that Impedance of Box P is (32)2 + (X C )2

 XC  π −1 −1
tan–1  R  = XC
= = = –70W
  3 ωC 1 5
× 10 × 10 −6

XC 7
= 3 ⇒ X C = 3 R → (x2)
R
∴ ZP
= (32)2 + (70)2
1 1
=R 3 ⇒ C =
ωC R 3ω | ZP | = 77 ohm,
1 100
C= C
⇒= µF And impedance of coil Q is (68)2 + (XL )2
100. 3 (300) 3 3
1
Now when all together are present XL= ω L = × 105 × 4.9 × 10–3
7
Znet = ZR + ZL + Z C = 100 + iR 3 − iR 3 XL = 70 W
[From X1and X2]
∴ Impedance = (68)2 + (70)2
Znet = 100
| Z Q | = 98 W
Znet
= 100∠0
Znet = 32 – 70 i + 68 + 70 i
I = V 200 2∠0 I 2 2∠0
= ⇒= Znet = 100 W
Z
net
100∠0

=I 10 2 ∠0 ⇒=
I 2
∠0
power = VI
  = (200 2) (2 2)cos(0)
100 10

P = 800 W Voltage across P; VP = (Irms) ( | ZP | )

 200 2  2 2   2
Pavg = Vrms . Irms =      
  10 
 2   2 
  =   . (77)
2
Pavg = 400 W.
VP = 7.7 V
P hysi cs | 23.67

1
Voltage across Q; VQ = (Irms) ( | ZP | ) = (98) 1
10 |XL – XC| = R; ωL − =R
ωC
VQ = 9.8 V.
ω2L − 1
= R ; ω 2L – ωCR – 1 = 0
Sol 16: ωr = 4 × 10 rad/s. 5 ωC

Given Va – Vb = 60 V Solving this would give us

and Vb – VC = 40 V ω = 8 × 105 rad/s.

120
Sol 17: V = 220 2 sin (100 pt)
d
a b c
=V 220 2∠0
 = ZL + ZR = i (100 π × 35 × 10–3) + 11
Znet

Znet = 11 i + 11

We know that during resonance,  11 


Znet= 112 + 112 ∠ tan−1  
VL + VC = 0  11 
∴ π
VC = – 40 V Vc – Vd = – 40V Z
= net 11 2∠
4
(Va – Vb) = irms R 
I V= 220 2∠0 ; I= 20∠ − π
=
1
60 = irms . 120 ⇒ irms = amp. Z π 4
2 11 2∠
4
Now, Vb – VC = (irms) . ZL
 π
⇔ I = 20 sin  100πt − 
40 40  4
40 = (irms) . ( ZL ) ⇒ ZL = = = 80Ω
irms 1
ω L = 80 2
Exercise 2
(4 × 105) L = 80; L = 0.2 mH
Single Correct Choice Type
Now Vc – Vd = – 40
Sol 1: (C) Current is induced by varying magnetic flux.
i.e. irms . Z c = − 40
Here there is no such phenomena as flux linked with
−1 the coil is zero. Hence induced current is zero.
Z c = −80 ; = −80
ωC
1 1 Sol 2: (D)
C= ;C=
80ω 80 × 4 × 105 t
1
C= µF. i1
32
Current i2 is constant and positive i.e. from ‘c’ to ‘d’ have
 | X − XC |
tan θ =  L  i1 has to be from ‘b‘ to ‘a’. Hence negative
 R  di
L
Also i2 = dt
RL

XL+XC di
∴ = constant
cons tant
dt
Hence i1 versus t is as shown.

R di
π Sol 3: (A) Emf induced across inductor = L
Now at θ = dt
4
2 3 . 6 8 | Alternating Current

 Rt  Sol 10: (B) Let Z A be the Impedance of element A, and


i = i0  1 − e − L  ZB be that of element B.
 
 
Initially; when R is connected to A;
di   R  − Rt  di i0R  − L 
Rt
= i0  −  −  e L  ⇒ = e  Znet = R + Z A .
dt   L  dt L  
    Z 

Rt ⇔ Znet= Z 2A + R 2 ∠ tan−1  A 
e = i 0R . e L  … (i)  R 

Rt i = V
i = i 0 – i0 e − L
Z
e V Z 
=i i0 − =i ∠ − tan−1  A
R 
Z 2A + R 2  R 
e
= −i + i0
R Given that current is lagging behind voltage by angle
e = R (–i + i0) [y = –mx + c] ‘q1’
 ZA 
Hence graph A.
∴ tan–1  R  = q1  … (i)
 
di When R is connected to B
Sol 4: (A) Self-induction Emf = –L
dt
Z 
di1 di2 di1 di2 Z= ZB2 + R 2 ∠ tan−1  B 
< ⇒– >–  R 
dt dt dt dt
V Z 
E1> E2. =i ∠ − tan−1  B 
ZB2 +R 2
 R 
L
Sol 5: (A) We know that RC and will have dimensions Given that current leads voltage by ‘q2’
R
1 R  ZB 
of time. Hence and will have dimensions of ∴
RC L q2= –tan–1  R   … (ii)
 
frequency.
Using same method, when R, A, B are connected,
Sol 6: (A) Refer to Questions – 3 (Ex –I JEE Advanced)  Z A + ZB 
θ = –tan–1  R
  … (iii)
 
1 2 tan (– θ) = tan (–q2) + tan q1
Sol 7: (A) LI = 32J  … (i)
2
tan θ = tan q2– tan q1
I2R = 320 … (ii)
(1) L 2 × 32
= = Sol 11: (B) Resonance is a condition of maximum power
(2) R 320
Hence cos φ = 1.
L
τ= = 0.2s.
R
Sol 12: (B) In calculating the rms value, we square each
value.
Sol 8: (B) In an L-R decay circuit, the initial current at
t=0 is 1. The total charge that has inductor has reduced
to onefourth of its initial value is LI/2R

1 2
Sol 9: (C) LI = U
2
A
I2R = P
Hence both A and B have same square value at every
L 2U
T= = point.
R P
P hysi cs | 23.69

2
 1  π
Znet =  ωL −  ∠
 ωC  2

B

i = V ⇒ V0 π
∠−
Z 2 2
Hence irmsA = irmsB net  1 
 ωL − 
 ωC 
Here we have every value greater than that of Irmsin
VL = i Z L
graph A or graph B.
 
 
 V0 π  π
=VL  ∠−  ωL ∠
  2 2  2
  ωL − 1  
  ωC  
 

∴ (irms)C> IA = IB. π π
VL = V1 ∠ −
2 2
Sol 13: (D) Initially in LR circuit; VC = i Z C
 R   R  π π
cos θ1 =   ⇒cos q1=   VC= V1 ∠ − −
 2 2   R 10  2 2
 R + 9R 
1 π π 
P1 = VC= V1 ∠ −  + 
10 2 2 
Now finally Hence phase difference between VL and VC will be π and
XL – XC = 3R – R = 2R π
between VL and I will be ± . Graph D satisfies all the
conditions. 2
 R 
cos θ2 =  
 2 2 
 R + 4R  Sol 15: (A) Let us consider mesh (1);
1
P2 = XC
5
P1 1 P2 i1
⇒ = . 5 ⇒ = 2 (1)
P2 10 P1

i2 (2)
Sol 14: (D) Znet
= ZL + Z C
L C XL R

V V0 < 0
=
1 π
Z=
1 Z=
C ∠−
ωC 2
 i V V0 ∠0
=1 =
V = V0 cos t Z 1 1 π
∠−
 −i  ωC 2
Znet = i ωL +  
 ωC  i1 = V0 ωC∠
π
 ... (i)
2
 1  Now in mesh (2)
⇒ Znet = i  ωL − 
 ωC 
Z 2 = ZR + ZL = R + i ω L
2 3 . 7 0 | Alternating Current

 ωL  B2
Z=
2 R 2 + (ωL)2 ∠ tan−1  
 R 

i V V∠0
=2 = B2
Z 2  ωL 
R 2 + (ωL)2 ∠ tan−1  
 R 
i = i1 ∠ − tan−1  ωL 
2 0  
 R 
At t = ∞;
π  ωL  R
Phase difference between i1 and i2= − tan−1  
i
B2
2  R 
π X 
= − tan−1  L  R
2 R  i
B2

2i
Multiple Correct Choice Type

Sol 16: (D) Using intuition;


Let us go for capacitance in the box Hence B2 lights up early; but finally both B1 and B2 shine
∴ Q = CV with equal brightness.

dq dv
=C Sol 19: (B)
dt dt
dq i1 L i2
Given i = = constant
dt
i
dv
∴ = constant
dt
∴ Graph looks like a straight line.
(1) (2)
dv
i=C
dt Just after switch is closed, Inductor tries to oppose the
8−2 current ‘i1’. Hence i1< i2. As time goes on, the opposition
Slope of the graph = =2 given by inductor reduces.
3
∴ This opposition is due to the induced EMF in ‘L’.
i = 2C = 1 amp
1
C= C = 0.5 C.
2 Sol 20: (B, C, D) Emf induced in coil 1 = L1 di1
dt
L  di
Sol 17: (D) Time constant τ =  E2 = L2 2
R  dt
1 2 di1 di2
Energy stored in magnetic field = LI Given that =
2 dt dt
Power dissipated in resistor = I R 2
E1 L1
∴ = = 4
1 2  E2 L2
 LI 
∴ 22 = τ V2 1
2
 IR  ∴ =
  V1 4
And also given that power given to the two coils is
Sol 18: (A) At t = 0; same,
∴ Vi i1 = V2 i2
P hysi cs | 23.71

i1 V2 i1 1 Assertion Reasoning Type


= ⇒ =
i2 V1 i2 4
Sol 25: (C) Magnetic field is into the page
1 1
W1 = L1 I12 and W2 = L2 I22
2 2
2 2
W1 L   I  W 8 1
=  1   1  ⇒ 1 =    
W2  L2   I2  W2  2   4 
W1 1
∴ = .
W2 4

L
Sol 21: (A, B, C) RC and will have the dimensions
R As resistance is increasing, current decreases
1 R
of time and hence and will have dimensions of
RC L ∴ Magnetic field decreases.
frequency.
Hence there will b e a clockwise current in the ring.
Sol 22: (D) When just after battery is connected, current
Sol 26: (D) In an LCR circuit,
is zero in the circuit, and hence will follow magnetic
1  |Z|= R 2 + (XL − X C )2
field energy  LI2  and power delivered (I2R) is also
zero. 2 
Vmax
imax =
 di 
EMF induced is  L  . Hence there is a finite value. R 2 + (XL − X C )2
 dt 
R . Vmax
(VR)max = ;
Sol 23: (B, D) At time t = 0, capacitor is short circuited, R 2 + (XL − X C )2
Inductor is open circuited. ωL . Vmax
(VL )max =
At t = ∞, capacitor is open circuited, R + (XL − X C )2
2

Inductor is short circuited.


Now (VR)max = Vmax; at resonance condition, (XL – XC = 0),
Hence both the options follow from this.
now for (VL)max; we can set conditions,
diB dφ A
Sol 24: (D) M. = (a) R ?0 and (b) XL = XC;
dt dt
This will lead to (VL)max> Vmax.
∆iB ∆φA
M =
∆t ∆t
Sol 27: (A) When circuit is suddenly switched off, there
∆φA
∴ ∆ iB = will be a change in current, and it will lead to induced
M EMF.
4
∆IB = di
2 |E| = L
dt
∆IB = 2A
Now for large ‘L’, E is also high.
∆φB ∆iA
=M
∆t ∆t
∆φB = 2(1) = 2
But given the values of 4 weber.
Hence options D isn’t true.
2 3 . 7 2 | Alternating Current

Comprehension Type Paragraph 2


L R
Paragraph 1 Sol 30: (D)

Sol 28: (D) Now when S1 is opened and S2 is closed




V = V0 cos t
CV - + CV Znet = R + i ω L
 ωL 
L | Znet
= | R 2 + ω2L2 ;=
Z net | Z net | ∠ tan−1  
 R 
I V V0 ∠0
= =
At t = 0; energy stored is purely in capacitor.In this type Z  ωL 
| Znet | ∠ tan−1  
of circuits, charge and current will be in the form of sin  R 
or cos. Thus oscillatory.
V  ωL 
 1  =I ∠ − tan−1  
q = Q0 cos  t  ; Q0 = CV | Znet |  R 
 LC 
Now potential difference across resistance,
−1
i= Q0 sin ω t
LC VR = i × Z R

Q0 CV C  V  ωL  
=i = = V =  0 ∠ − tan−1    R∠0 
L
LC LC  | Znet |  R  
Hence option D. V0 R  ωL 
VR
= ∠ − tan−1  
| Znet |  R 
 1 
Sol 29: (C) q = Q0 cos  t V0R
 LC  (VR)max = ≡ 4 volts (given) … (i)
R 2 + XL2
dq −Q0  1 
dt
=
LC
sin  t
 LC 
( VL ) = ( i ) ( Z L )
 V X   π
d2q −Q0  1  =  0 ∠ − tan−1  L
= cos  t   ωL∠ 
dt2 LC  LC   | Znet | R    2

 
d2q
= −
1
q, = ( )
VL  V0 XL ∠ π − tan−1  XL  
 2 R
dt2 LC 2
 R + XL
2   
Hence option ‘C’. V0 XL
(VL )max = ≡3V  … (ii)
R 2 + XL2

(i)
(1) R 4
= =
(ii) XL 3
(2)

R 4 3R
∴ = ⇒ XL =  … (iii)
XL 3 4
P hysi cs | 23.73

9R 2 25R 2 Previous Years’ Questions


R 2 + XL2 = R 2 + =
16 16
5R Sol 1: In steady state no current will flow through
R 2 + XL2 =  ... (iv) capacitor. Applying Kirchhoff’s second law in loop 1:
4
In equation (i) 2 i2 2F

V0R V0R
= 4 ; =4
2
R + XL2 5R
1
4 1
i2
V0 = 1 V 2 12 V

you can just start from here if you understand how I i1 - i2 i1


wrote them i1
i1 2
X 
VR = 4∠ − tan−1  L 
R  A 3V 2 B
 X 
⇔ VR = 4 cos  ωt − tan−1  L   3

 R  10
π X 
VL = 3 ∠ − tan−1  L 
2 R  – 2i2 + 2(i1 – i2) + 12 = 0
 π X  \2i1 – 4i2 = – 12
⇔ VL = 3 cos  ωt + − tan−1  L  
 2
 R  or i1 – 2i2 = – 6 …(i)

 X  Applying Kirchhoff’s second law in loop 2:


VL≡ 3 sin  ωt − tan−1 L 
 R  – 12 – 2(i1 – i2) + 3 – 2i1 = 0

Given VR = 2 4i1 – 2i2 = – 9  …(ii)

 X  Solving Equations (i) and (ii), we get


∴ 2 = 4 cos  ωt − tan−1  L  
 i2 = 2.5 A and i1 = – 1A
 R 
Now, VA + 3 – 2i1 = VB
1  X 
= cos  ωt − tan−1  L   or VA – VB = 2i1 – 3
2 
 R 
 XL  π = 2 (–1) – 3 = – 5V
∴ ω t – tan  R –1  = → (X ) PR = (i1 – i2)2 R1 = (– 1 – 2.5)2 (2) = 24.5 W
  3 1
1
(b) In position 2: Circuit is as under
π 3 3
Now VL = 3 sin   ; VL = 3 sin 60° =
3 2
3V 2
3
3 3
Sol 31: (B) Vsource = VL + VR = +2 10
2

4+3 3
Vsource = Steady current in R4:
2
3
i0 = = 0.6 A
3+2
2 3 . 7 4 | Alternating Current

Time when current in R4 is half the steady value Sol 6: After a long time, resistance across an inductor
L –3
(10 × 10 ) becomes zero while resistance across capacitor
t1/2 = τL (In 2) =  n (2) =  n (2) becomes infinite. Hence, net external resistance,
R 5
R
= 1.386 × 10–4 s +R
3R
Rnet = 2 =
1 2 1 2 4
U= Li = (10 × 10–3) (0.3)2 = 4.5 × 104J
2 2 2E
Current through the batteries, i =
3R
+r +r
Sol 2: In circuit (p): I can’t be non-zero in steady state. 4 1 2
Given that potential across the terminals of cell A is
In circuit (q): V1 = 0 and V2 = 2I = V (also) zero.

In circuit (r): V1 = XLI = (2pfL)I ∴E – iri = 0

= (2π × 50 × 6 × 10–3)I = 1.88I  2E 


or E –   r1 = 0
V2 = 2I  3R / 4 + r1 + r2 
4
In circuit (s): V1 = XLI = 1.88 I Solving this equation, we get, R = (r – r )
3 1 2
 1 
V2 = XCI =   Sol 7: Inductive reactance XL = ωL
 2πfC 
= (50) (2π) (35 × 10–3) = 11W
 1 
=  = I = (1061) I
–6 
 2π × 50 × 3 × 10  Impedance Z = R 2 + XL2 = (11)2 + 11)2

In circuit (t): V1 = IR = (1000) I = 11 2 Ω


V2 = XCI = (1061)I Given Vrms = 220 V
Therefore the correct options are as under
Hence, amplitude of valtage V0 = 2 Vrms
(A) → r, s, t (B) → q, r, s, t
= 220 2 V
(C) → q or p, q (D) → q, r, s, t
V0 220 2
∴ Amplitude of current i0 = =
Sol 3: (B) Charge on capacitor at time t is Z 11 2
q = q0 (1 – e ) –t/τ or i0 = 20 A
Here q0 = CV & t = 2t X  –1  11 
Phase difference φ = tan–1  L  = tan  
Here q0 = CV(1 – e–2τ/τ) = CV (1 – e–2) R   11 
π
φ=
4
Sol 4: (B) From conservation of energy,
In L-R circuit voltage leads the current. Hence,
1 2 1 C instantaneous current in the circuit is,
LImax = CV 2 ∴ lmax = V
2 2 L i = (20 A) sin (wt – π/4)
Corresponding i-t graph is shown in figure.
Sol 5: (C) Comparing the LC oscillations with normal
SHM, we get V,I V=
2
dQ
=–wQ 2
20 i=20 sin (t-/4)
dt2 9T/8
O t
1
Here, w = 2
-10 2 T/8 T/2 5T/8
LC

d2Q
\Q = – LC
dt2
P hysi cs | 23.75

Sol 8: (C) When e– has zero kinetic energy total energy


is shared by antineutrino and proton. This time energy
of antineutrino is its maximum possible kinetic energy.
As antineutrino is very light mass in comparison to
proton so it will have almost contribution in total
energy.
∴ Its energy is almost 0.8 × 106 eV

Sol 9: (C, D) As current leads voltage by π / 2 in the


given circuit initially, then ac voltage can be represented
as
=V V0 sin ωt
q CV0 sin=
∴= ωt Q sin ωt
Where, Q= 2 × 10−3 C
3
•• At t = 7 π / 6ω ; I =− I and hence current is
anticlockwise 2 0


•• Current ‘i’ immediately after t = is

Vc + 50
=i = 10 A
R
Charge flow = 2 × 10−6 C
Q final − Q(7 π /6ω) =
2017-18 100 &
op kers
Class 12 T
By E ran culty
-JE Fa r
IIT enior emie .
S fP r es
o titut
Ins

PHYSICS
FOR JEE MAIN & ADVANCED
SECOND
EDITION

Exhaustive Theory
(Now Revised)

Formula Sheet
9000+ Problems
based on latest JEE pattern

2500 + 1000 (New) Problems


of previous 35 years of
AIEEE (JEE Main) and IIT-JEE (JEE Adv)

5000+Illustrations and Solved Examples


Detailed Solutions
of all problems available

Topic Covered Plancess Concepts


Tips & Tricks, Facts, Notes, Misconceptions,
Modern Physics Key Take Aways, Problem Solving Tactics

PlancEssential
Questions recommended for revision
24. MODERN PHYSICS

1. INTRODUCTION
The developments in the post-classical/Newtonian physics, also known as modern physics, has given us revelatory
insights into the structure and nature of fundamental forces/particles in the universe. The wave-particle duality/
paradox, which postulates that every elementary particle exhibits the properties of not only particles, but also
waves is one such insight. For example, when electromagnetic radiation is absorbed by matter, it predominantly
displays particle-like properties. It was de-Broglie who propounded the concept of matter waves, i.e. the particles
exhibiting wave properties. We will be dealing here with the energy, wavelength, and frequency of electromagnetic
waves and the relationship between these quantities. We will also be dealing with the photoelectric effect on which
Einstein’s based his photon theory of light. We will be discussing the Bohr atomic model, the hydrogen spectra, and
the laws describing the characteristics of X-rays.

2. DUAL NATURE OF ELECTROMAGNETIC WAVES


Classical physics always defined motions in terms of particles or waves i.e., it treated particle and waves as distinct
entities. An electron is considered as a particle because it possess mass (9.109×10−31 kilograms), electric charge
(−1.602×10−19 coulomb) and they behave according to the laws of particle mechanics. However, we shall see that
an electron in motion is as much a particle as it is a wave manifestation.
Electromagnetic radiation has properties in common with other forms of waves such as reflection, refraction,
diffraction, and interference. It, however, also has particle-like properties in addition to those associated with wave
motion (Photoelectric effect and Einstein’s theory, black body radiation, Compton effect). Therefore, we can say
that they a have dual nature.
hc
Einstein’s Equation: =
E hf
=
λ

2.1 Electromagnetic Spectrum

n
28 26 24 22 20 18 16 14 12 10 8 6 4 f(10 Hz)
TV & FM

AM Long
Visible

UHF

r Rays X-Rays UV IR Microwaves Short radio


Wave Waves

n
-20 -19 -16 -14 -12 -10 -8 -6 -4 -2 0 2 4 (10 m)

Figure 24.1 Electromagnetic spectrum


2 4 . 2 | Modern Physics

2.2 Electron Emission


Electrons are negatively charged particles. Therefore, though they move about arbitrarily in a conductor at room
temperature, they cannot leave the surface of the conductor due to attraction of positively charged particles(protons).
Therefore, some external energy has to be provided so that the electrons can be ejected from the atoms on the
surface of the conductor. To eject the electrons which are just on the surface of the conductor, only minimal energy
is required. This minimal energy or thermodynamic work that is needed to remove an electron from a conductor/
solid body to a point in the vacuum immediately outside the surface of the solid body/conductor is called the work
function (denoted by W) of the conductor. Work function is the property of the metallic surface.
Heat, light, electric energy etc., can be employed to liberate an electron from a metal surface. Depending on the
source of energy, the following methods are possible:
(a) Thermionic emission: In this method, the metal/conductor is heated to overcome binding potential of the
conductor, and consequentially, free the electrons.
(b) Field emission: The emission of electrons induced by an electrostatic field is called field emission. In this
process, the high electric field acting on the conductor exerts an electric force on the free electrons in the
conductor in the opposite direction of field. This force overcomes the binding potential of the conductor and
the electrons start coming out of the metal’s surface
(c) Secondary emission: Ejection of electrons from a solid that is bombarded by a beam of charged particles
(e.g., electrons or ions) is known as secondary emission.
(d) Photoelectric emission: The photoelectric effect refers to the emission/ejection of electrons from the surface
of a metal in response to incident light (or electromagnetic wave). This happens when the incident light or
electromagnetic wave has greater energy then the work function of the metal. The electrons emitted in this
process are called photoelectrons.

3. PHOTOELECTRIC EFFECT
(a) The photoelectric effect was discovered by Wilhelm Ludwig Franz Hallwachs in 1888, the experimental
verification which was done by Hertz.
(b) The photoelectric effect refers to the emission/ejection of electrons from the surface of a metal in response
to incident light (or electromagnetic wave).
(c) The electron ejected due to photoelectric effect is is called a photoelectron and is denoted by e− .
(d) Current produced as a result of the ejected electrons is called photoelectric current.
(e) Photoelectric effect proves quantum nature of light.
(f) Photoelectric effect can not be explained by the classical wave theory of light. The wave theory is incable of
explaining the first 3 obserations of the photoelectric effect.
(g) Photoelectrons, generally, refer to the free electrons that are in the inter-molecular spaces in the metal.
(h) Explanation for Photoelectric effect was successfulyy explained given by Albert Einstein as being the result
of light energy being carried in discrete quantized packets. For this excellent work he was honored with the
Nobel prize in 1921.
(i) The law of conservation of energy forms the basis for photoelectric effect.

Threshold Frequency ( v 0 ): The minimum frequency of the incident light or radiation that will produce a
photoelectric effect i.e., ejection of photoelectrons from a metal surface is known as threshold frequency for that
metal. Its value, though constant for a specific metal, may be different for different metals.
If v = frequency of incident photon & v 0 = Threshold Frequency
Then
(a) If v < v 0 , there will be no ejection of photoelectron and, therefore, no photoelectric effect.
P hysi cs | 24.3

(b) If v = v 0 , photoelectrons are just ejected from metal surface, in this case the kinetic energy of electron is zero.
(c) If v > v 0 , then photoelectrons will come out of the surface along with kinetic energy.

Threshold Wavelength ( λ0 ): The greatest wavelength of the incident light or radiation for a specified surface for
c
the emission of photoelectrons is known as threshold wavelength λ0 = .For wavelengths above this threshold,
v0
there will be no photoelectron emission.
For λ = wavelength of incident photon, then
(a) If λ < λ0 , then photoelectric effect will take place and ejected electron will posses kinetic energy.
(b) If λ = λ0 , then just photoelectric effect will take place and kinetic energy of ejected photoelectron will be zero.
(c) If λ > λ0 , there will be no photoelectric effect.

3.1 Work Function or Threshold Energy (f)


(a) The minimal energy or thermodynamic work that is needed to remove an electron from a conductor/solid
body to a point in the vacuum immediately outside the surface of the solid body/conductor is called the work
function or threshold energy for the conductor.
hc
= φ hv= 0
λ0
(b) Work function is the characteristic of given metal
(c) If E = energy of incident photon, then
(i) If E < φ , no photoelectric effect will take place.
(ii) If E = φ , just photoelectric effect will take place but the kinetic energy of ejected photoelectron will be
zero.
(iii) If E > φ , photoelectric effect will take place along with possession of the kinetic energy by ejected
electron.

3.2 Laws of Photoelectric Effect


Lenard postulated the following laws regarding photo emission on the basis of his experiments:
(a) For a given substance, there is a minimum value of frequency of incident light called threshold frequency
below which no photoelectric emission is possible, howsoever, the intensity of incident light may be. It is given
c
by v 0 =
λ0
(b) The number of photoelectrons emitted per second (i.e. photoelectric current) is directly proportional to the
intensity of incident light provided the frequency is above the threshold frequency.
(c) The maximum kinetic energy of the photoelectrons is directly proportional to the frequency provided the
frequency is above the threshold frequency. However, the relationship between the wavelength and kinetic
energy is inversely proportional. With increasing frequency of incident light, the kinetic energy of photoelectron
increases but with increasing wavelength it decreases. So v ↑ λ ↓ K.E. of emitted electrons ↑ v ↓ λ ↑ K.E. of
emitted electrons ↓
(d) The maximum kinetic energy of the photoelectrons is independent of the intensity of the incident light.
(e) The process of photoelectric emission is instantaneous, i.e., as soon as the photon of suitable frequency falls
on the substance, it emits photoelectrons.
(f) The photoelectric emission is one-to-one. i.e. for every photon of suitable frequency one electron is emitted.
(g) Value of threshold frequency or threshold wavelength depends upon photo sensitive nature of metal.
2 4 . 4 | Modern Physics

−19
Illustration 1: The work function of silver is 5.26 × 10 J . Calculate its threshold wavelength. (JEE MAIN)
hc
Sol: For any metal to eject photo electron the work function of surface is given as φ =
λ0
hc 6.6 × 10−34 × 3 × 108 °
Threshold wavelength = λ0 = ; ∴ λ = = 3.764 × 10−7 m ; λ =3764 A
φ 5.26 × 10 −19

Illustration 2: The work function of Na is 2.3 eV. What is the maximum wavelength of light that will cause photo
electrons to be emitted from sodium? (JEE MAIN)
hc
Sol: For any metal to eject photo electron the work function of surface is given as φ =
λ0
hc hc
The threshold wavelength λ0 = ; (=
φ hv=
0 ) ;& hc = 1.24 × 10−6 (eV) m
φ λ0
1.24 × 10−6 °
λ0 = m ; λ0 = 0.539 × 10−6 m = 539 nm; λo =5930 A
2.3

3.3 Failure of Wave Theory to Explain Photoelectric Effect


Note - The assumptions of the classical wave theory could not explain some observations of the photoelectric
effect. These aspects of the photoelectric effect were later explained by Albert Einstein’s photon theory. The failures
of the classical wave theory in explaining the photoelectric effect are enumerated below:
(a) The wave theory suggests that the intensity of the radiation should have a proportional relationship with
the resulting maximum kinetic energy. However Kmax = eV0 suggests that it is independent of the intensity of
light.
(b) According to the wave theory, the photoelectric effect should occur for any intense light, regardless of
frequency or wavelength. However, the equation suggests that photo emission is possible only when frequency
of incident light is either greater than or equal to the threshold frequency f0 .
(c) The wave theory states that there should be a delay on the order of seconds between the radiation’s contact
with the metal and the initial release of photoelectrons. It was assumed that between the impinging of the
light on the surface and the ejection of the photoelectrons, the electron should be “soaking up” energy from
the beam until it had accumulated enough energy to escape. However, no detectable time lag has ever been
measured.
In reality, due to collision between atoms inside the metal, some energy is lost. Hence kinetic energy emitted
by electrons is K< Kmax. Hence the term Kmax is used for the actual total kinetic energy.

3.4 Einstein’s Photon Theory


Albert Einstein worked his way around the limitations of the classical wave theory by explaining that lights exists
and travels as tiny packets/bundles called photons. The energy E of a single photon is given by E = hf
Applying the photon concept to the photoelectric effect, Einstein wrote:

hf
= W + Kmax (Already discussed)

Discussed below is how Einstein’s photon hypothesis overcomes the three objections raised against the wave
theory interpretation of the photoelectric effect.
Objection 1: Intensity of the radiation should have a proportional relationship with the resulting maximum kinetic
energy. This objection is overcome by Einstein’s photon theory because, doubling the light intensity merely doubles
the number of photons, thereby doubling the photoelectric current. It does not, however, change the energy of the
individual photons.
P hysi cs | 24.5

Objection 2: Photoelectric effect should occur for any intense light, regardless of frequency or wavelength. The
existence of a minimum frequency level ( in Einstein’s photon theory) follows from equation hf = W + Kmax . If
Kmax equals zero, then hf0 = W , which implies that the photon’s energy will be barely adequate to eject the
photoelectron and that there will be no residual energy to manifest as kinetic energy. The quantity W is the work
function of the metal/substance. If the frequency f is reduced below f0 , the individual photons, irrespective of how
numerous they are(in other words, no matter what the intensity of the incident light/radiation is), will not have
enough energy to eject photoelectrons.
Objection 3: There should be a delay on the order of seconds between the radiation’s contact with the metal
and the initial release of photoelectrons. The absence of a time lag follows from the photon theory because the
required energy is supplied in packets/bundles. Unlike in the wave theory, the energy is not spread uniformly over
a large area.
Therefore, as far as photoelectricity goes, the photon/particle theory seems to be in total contradiction of the wave
theory of light. Modern physicists have reconciled this apparent paradox by postulating the dual nature of light, i.e.,
light behaves as a wave under some circumstances and like a particle, or photon, under others.

3.5 Einstein’s Equation of Photoelectric Effect


Einstein (1905) explained photoelectric effect on the basis of quantum theory.
According to Einstein, when photons fallon a metal surface, they transfer their energy to the electrons of metal.
When the energy of photon is larger than the minimum energy required by the electrons to leave the metal surface,
the emission of electrons take place instantaneously.
He proposed that after absorbing the photon, an electron either leaves the surface or dissipates its energy within
the metal in such a short interval that it has almost no chance to absorb second photon
The energy supplied to the electrons is used in two ways:
(a) Removes the electron from the surface of metal
(b) Supplies some part of kinetic energy to the photoelectron. Therefore, Einstein’s equation of photoelectric
effect can be written as:
If v max is the maximum velocity of emitted electrons then by law of conservation of energy
1 1
hv = φ + mv 2 . If v 0 : Threshold frequency ∴ φ0 = 2
hv 0 , So ⇒ hv = hv 0 + mv max .
2 2

Einstein’s equation explains the following concepts


(a) The frequency of the radiation/incident light is directly proportional to the kinetic energy of the electrons
and the wavelength of radiation/incident light is inversely proportional to the kinetic energy of the electrons.
1 2
If v 0 is threshold frequency then maximum kinetic energy Emax = hv − hv 0 ⇒ mv max
= h(v − v 0 )
2
2h(v − v 0 )
So maximum velocity of photoelectrons: ⇒ v max =
m
m - mass of electron; v - frequency of incident light; v 0 - threshold frequency;
1 1  1 2 1 1 
λ0 - threshold wavelength Emax hc  −
λ - incident wavelength ⇒ =  ⇒ mv
= hc  −
λ λ  2 max  λ λ 
 0   0 
(b) If v = v 0 or λ = λ0 then v = 0
v < v 0 or λ > λ0 ⇒ There will be no emission of photoelectrons.
(c)
(d) Intensity of the radiation or incident light refers to the number of photons in the light beam. More intensity
means more number of photons and vice-versa. Intensity has no bearing on the energy of photons. Therefore,
intensity of the radiation is increased, the rate of emission increases but there will be no change in kinetic
energy of electrons. With increasing number of emitted electrons, value of photoelectric current increases.
2 4 . 6 | Modern Physics

Illustration 3: A light beam of wavelength 4000 Å is directed on a metal whose work function is 2 eV. Calculate the
maximum possible kinetic energy of the photoelectrons.  (JEE MAIN)

Sol: According to photoelectric equation the maximum kinetic energy of photoelectron after being ejected from
metal is EK = hν − φ
hc 19.8 × 10−19
Energy of the incident photon = . Energy of the incident photon in eV = = 3.09 eV ;
λ 4 × 1.6 × 10−19
Kinetic energy of the emitted electron EK = hν − φ = 3.09 – 2.00 = 1.09 eV

Illustration 4: Calculate the maximum kinetic energy of photoelectrons emitted from a metal with a threshold
wavelength of 5800 Å, if the wavelength of the incident light is 4500 Å. (JEE ADVANCED)

Sol: The maximum kinetic energy of photoelectron with which it is ejected from metal is EK = hν − φ .
2EK
max
Therefore maximum velocity of photoelectron is v max =
me

hc[λ0 − λ] [5800 × 10−10 − 4500 × 10−10 ]


Ek = = 6.62 × 10−34 × 3 × 108 = 9.9 × 10−20 J
max λ0 λ 5800 × 4500 × 10−20

9.9 × 10−20 2hc(λ0 − λ ) 2 × 0.62 × 1.6 × 10−19


Ek = = 0.62 eV ; ⇒ v max = = = 4.67 × 105 m / s
max
1.6 × 10−19 me λλ0 9.31 × 10−31

3.6 Photoelectric Current


(a) When light/radiation is directed on a cathode, photoelectrons are emitted and these are attracted by an
anode. The electric current, thus generated, flows in the circuit. This is called a photoelectric current.
(b) Value of photoelectric current depends upon following parameters:
(i) Potential difference between electrodes. (ii) Intensity of incident light.

3.6.1 Intensity of Light (I)


(a) It is quantity of light energy falling normally on a uniform surface area in unit time.
E W hc
or I = where I = Intensity of light in E = total energy incident = nhv = n
A.t m2 λ
n = no. of photons; A = cross sectional area; T = time of exposure
(b) Intensity of light is proportional to saturation current
I
(c) For point source of light Ir ∝
r2
I
(d) For the Linear source of light Ir ∝
r
Where r is the distance of the point from the light source.

3.7 Stopping Potential and Maximum Kinetic Energy


When the frequency f of the light/radiation is greater than the threshold frequency of the metal on which the light
is directed, some photoelectrons are emitted from the metal with substantial initial speeds. Let us assume that E
is the energy of light incident on a metal surface and W (<E) the work function of metal. In this case, as minimum
energy is required to extract electrons from the surface, the emitted electrons will have the maximum kinetic
energy which is E – W. Kmax = E − W
P hysi cs | 24.7

As the potential V is increased, the electrons experience greater resistance/repulsion, and consequentially, less
number of electrons reach the plate Q. This leads to a decrease in the flow of current in the circuit. At a certain value
V0 , the electrons having maximum kinetic energy (Kmax) also stop flowing and current in the circuit becomes zero.
This is called the stopping potential.
(a) In photoelectric cell, when (+)ve voltage is applied on cathode and negative voltage is applied on anode
applied, then the magnitude of photoelectric current decreases as the potential difference between the two
points (cathode and anode) increases.
(b) The stopping potential is the negative potential ( V0 ) applied to the anode Light
where the current gets reduced to zero or stops flowing in the circuit.
(c) When the magnitude of negative potential on anode is greater than
or equal to magnitude of stopping potential the current in the circuit
becomes zero. i

(d) If emitted electrons do not reach from cathode to anode then stopping P Q G
potential is given by
1 2 h(v − v 0 )
eV0 = mv max or Emax = eV0 ; eV
=0 h(v − v 0 ) ; V0 = V
2 e
(e) Value of stopping potential depends upon frequency of incident light. Figure 24.2: Photoelectric effect

(f) Stopping potential also depends upon nature of metal (or work function)
(g) Stopping potential does not depend upon intensity of light
1 2
(h) Example: Suppose stopping potential = -3 V, then mv max = 3eV
2
1 2
If we apply – 5 V, then also there will be zero current in the circuit but mv max ≠ 5eV
2
Because stopping potential is not equal to 5V which cannot be used in Einstein’s equation.

3.7.1 Graphs
(a) Kinetic energy V/s frequency: At v = v 0 , Emax = 0
E max

V0 V
Figure 24.3

(b)   Vmax V/s v : At v = v 0 Vmax = 0


V max

V0 V
Figure 24.4

(c) Saturated Current V/s Intensity:


Intensity

Current

Figure 24.5
2 4 . 8 | Modern Physics

(d) Stopping potential V/s frequency:

 eV=
0 hv − hv 0
V0
tanθ = slope
h V
= (constant for all type of metals)
e Figure 24.6
Intercept on x-axis = v 0
Intercept on y-axis = v
(e) Potential V/s current: (v : constant) Intensity
I3
Current I2 I3  I2  I1
I1

-V₀ Voltage
Figure 24.7

⇒ Stopping potential does not depend upon intensity of light.


(f) Photoelectric current V/s Retarding potential:

Current

v1 v3  v2  v1
v2
v3
vo(3)vo(2)vo(1)
Retarding potential
Figure 24.8

Illustration 5: Calculate the value of the stopping potential if one photon has 25 eV energy and the work function
of material is 7 eV. (JEE MAIN)
E − φ0
Sol: The stopping potential required to stop the photoelectrons to reach cathode is V0 =
e
E − φ0 25 − 7 18 eV
Stopping potential is V=
0 = = ⇒ V=
0 18 V
e e e

3.8 Derivation of de-Broglie Wavelength


h
De broglie equation is given by: λ =
p

Derivation: Let us start with the energy of a photon in terms of its frequency v, E = hv
Albert Einstein’s special theory of relativity gives a new expression with reference to the velocity of light. This
expression is E = mc2 , where m refers to the relativistic mass of light which is non-zero as it is travelling with
velocity c. If it were at rest, it’s mass would be zero.
c
Now, by equating both the energy equations we get E = hν = mc2 . Also, as seen earlier ν =
λ
h h
Wavelength of a photon. ∴ = mc and λ =
λ mc
P hysi cs | 24.9

Analogously, de Broglie argued that a particle with non-zero rest mass m and velocity v would have a wavelength
h
given λ =
mv
h
Also, mv = p, where p is the particle’s momentum. Substituting p for mv we get λ =
p

3.8.1 Criterion for Type of Behaviour


Like electromagnetic waves, moving bodies also exhibit the wave-particle duality and the wave and particle aspects
of moving bodies cannot be observed simultaneously. A moving body will exhibt particle behavior if the wavelength
of the body is negligible in comparison to its dimensionwhereas it will exhibit a wave nature if its wavelength is in
order of the dimension of body.

Illustration 6: Determine Broglie wavelengths of (a) a 46g golf ball with a velocity of 30 m/s and (b) an electron
with a velocity of 107 m/s. (JEE ADVANCED)
h
Sol: The de-Broglie wavelength of the particle of mass m and moving with velocity v is given by λ = , where h
is Planck’s constant mv

(a) Since v<<c, we can let (effective mass = Rest mass) m =m0 .
h 6.62 × 10−34 J.s
Hence λ = = = 4.8 × 10−34 m.
mv (0.04kg)(30m / s)

Thus, we see that the wavelength of the golf ball is so negligible compared with its dimensions that we would
not be able to observe expect to find any wave aspects in its behavior.
(b) Again v << c, so with m = m0 = 9.1 × 10−31 kg,

h 6.62 × 10−34 J.s °


we have λ = = = 7.3 × 10−11 m = 0.73 A
mv 9.1 × 10 −31 × 107 (kg m / s)
The dimensions of atoms are comparable with the radius of the hydrogen atom which in reality is 5.93 × 10−11 m.
So, it it is clear that an electron with a wavelength of 7.3 × 10−11 m would demonstrate a wave behavior. Also, we
can see that the wave character of moving electrons is facilitates the understanding atomic structure and behavior.

4. ENERGY, MOMENTUM, AND WAVELENGTH OF PHOTONS


(a) The quantum theory states that the light photons are undivided energy packets.
(b) Energy of photons is denoted by E = hv, where h is Planck constant, v is the frequency of photons, and E is
the energy of photons.
(c) The velocity of photons and the velocity of light are equal (c). Therefore, c = vλ
hc
Here, λ is the wavelength of wave connected to photon. ∴ E = hv =
λ
(d) The mass of photons at rest is zero but it will be non-zero if the photons are moving.. Assuming m to be the
effective mass of photons, energy of photon according to Einstein:
hc
E = mc2 ⇒ E = hv = = mc2
λ
mc2 E 1  hc  h h
(e) Momentum of moving photon p = mc = = =   = ⇒ p=
c c c λ  λ λ
E hv h p
(f) Effective mass m = = = =
c 2
c 2 c λ c
2 4 . 1 0 | Modern Physics

h h hc
(g) Wavelength connected to moving photons λ= = =
p mc E
(h) From Point (e) and (f):- Momentum of photon p ∝ m p ∝ E Energy of photons E ∝ m
1 1 1
Wavelength of wave connected to photons λ ∝ ; λ∝ ; λ∝
p m E
(i) Graphs

(i) P P

m E

(ii)

E
P m

(iii)
1/E
1/p 1/m
Figure 24.9

( j) There is no charge on photons

m
Figure 24.10

Illustration 7: A Determine the velocity of a light wave, given that frequency of the photon is ν, energy is hv, and
h
momentum is p =  (JEE MAIN)
λ
c
Sol: For light of frequency ν, the energy is E = hν and the frequency of light wave is ν = . Hence speed of light
is easily determined. λ

h
As E = hν and P = ,
λ
hc E
E= = Pc ⇒ c =
λ P


Illustration 8: Determine the mass of a photon witha wavelength of 0.01 A . (JEE MAIN)
h
Sol: Using equation of equivalent mass of photon, m = , we can find the mass of proton.

E h 6.62 × 10−34
m= = = ; m = 2.21 × 10−30 kg
c2 cλ 8
3 × 10 × 10 −12
P hysi cs | 24.11

Illustration 9: Determine the momentum of a photon with a of frequency 109 Hz.  (JEE MAIN)

Sol: The momentum of photon is p =
c
h hν 6.62 × 10−34 × 109
p= = = ; p = 2.2 × 10−33 kg m/s
λ c 3 × 108

Illustration 10: Determine the energy and momentum of a γ-ray photon with a wavelength of 0.01 Å.(JEE MAIN)
hc h E
Sol: For wave of wavelength λ the energy and momentum is given by E = and p= =
λ λ c
hc 1240(eV) × 1 × 10−9
E= = ;
λ 0.01Å

1240 × 10−9 E MeV


E= (eV) = 1.24 × 106 eV ; E = 1.24 MeV ; The momentum is P = = 1.24
10 −2
× 10 −10 c c

1.24 × 106 × 1.6 × 10−19


P= = 6.62 × 10−22 kg m/s
3 × 108

5. ENERGY, MOMENTUM, AND WAVELENGTH OF A MOVING PARTICLE


Suppose the mass of a particle at rest is m and it is moving with velocity v.

(a) Mass at rest = m


m
(b) Effective mass (or relativistic mass) =
1 − v 2 / c2
 
(c) Momentum or p = m v or p = mv ∴=
p mv = 2mE Here E : Kinetic energy
1 p2
(d) Kinetic energy E = mv 2 =
2 2m
h h h
(e) If λ is the wavelength of connected wave to the moving particle, then λ= = =
p mv 2mE

Illustration 11: A body of 10 gm is moving with velocity 2 × 103 m/s. Determine the value of its associated de-
Broglie wavelength.  (JEE ADVANCED)
h
Sol: The de-Broglie wavelength associated with particle moving with speed v is calculated as λ = .
mv
h 6.62 × 10−34
de-Broglie wavelength =
λ = ;
mv 10 × 10−3 × 2 × 103
λ = 3.3 × 10−35 m
1 1 1 1
So λ ∝ ; λ∝ λ ∝ λ∝
p E v E

 

Figure 24.11 E v Figure 24.12
P 1/ E
2 4 . 1 2 | Modern Physics

Moving particle

Charged Uncharged
electrons
protons
-particles, etc.
Photons Neutrons,
Atoms etc.

5.1 Energy, Momentum, and Wavelength of Charged Particle Accelerated by V-volt


(a) Potential difference or electric field can be used to accelerate a charged.

(b) The kinetic energy of a charged particle having charge q, mass m, accelerated by V volt, and a velocity v is
1
denoted= by E = mv 2 qV
2
2qV 2E
(c) Velocity V = =
m m

(d) Momentum p = 2mE = 2mqV

h h
(e) Wavelength λ = =
p 2mqV
(Here it is assumed that initial potential given to electron is zero)
h
If the particle is given some initial potential Vi and if final potential is Vf then, λ=
2mq(Vf − Vi )
From above Relation

1
λ∝  
V

V
Figure 24.13

1
λ2 ∝  
V

V
Figure 24.14

1
λ∝  
V

V
Figure 24.15
P hysi cs | 24.13

Cases:

(a) If the moving charged particle is an electron, then

2eV h 12.27
(i) v e = (ii) pe = 2 meeV (iii) λe = = Å
me 2me eV V
(b) If the moving charged particle is a proton, then

2eV h 0.286
(i) vP = (ii) pP = 2mepV (iii) λP = = Å
mp 2mepV V
(c) If the charged particle is an α -particle, then

2(2e)V eV 1
(i) v α = = = vP (ii) Pα = 2mα eα V = 2 × 8mp × eV = 2 2 pp
mα mP 2
h 0.101
(iii)
= λα = Å
16mp eV V

Illustration 12: Determine the potential to be applied to accelerate an electron such that its de-Broglie wavelength
becomes 0.4 Å. (JEE MAIN)
12.27
Sol: The de-Broglie wavelength of an electron in terms of accelerating potential difference is λe = Å
V0
12.27 12.27
Where V is the applied potential on electron to accelerate it. λ = Å ; 0.4 =
V0 V0

(12.27)2 12.27 × 12.27


Squaring on both the sides we get 0.16 = ⇒ V0 = ; ⇒ V0 = 941.0 V
V0 16 × 10−2

5.2 Wavelength of Wave Connected to Uncharged Particle


(Like neutron atoms, molecules etc.)
1
(a) If m is the mass and v is the velocity of particle, then kinetic energy E = mv 2 , momentum p = mv
2
h h h
(b) Wavelength λ= = =
p mv 2mE
(c) If λ is the wavelength of wave connected to matter particle then particle energy E will be

h2 h2
E= J= eV
2mλ2 (2mλ2 )e
(d) Energy E of particle (e.g., electron, neutron, or atom) at equilibrium temperature TE = (3/2) KT

Here K = Boltzman constant


h h
(e)
=  λ = here m : mass of a single atom.
2mE 3mKT

Illustration 13: Determine the associated de-Broglie wavelength if the energy of a thermal neutron is 0.02 eV,
 (JEE MAIN)
2 4 . 1 4 | Modern Physics

h
Sol: For neutron having kinetic energy K, the associated de-Broglie wavelength is found to be λ =
2mK
h 6.6 × 10−34
de-Broglie Wavelength λ = = ; λ = 2 × 10−10 m = 2Å
2mK 2 × 1.6 × 10 −27
× 0.02 × 1.6 × 10 −19

6. EXPERIMENTAL VERIFICATION OF MATTER WAVES


The Davisson–Germer experiment conducted by American physicists Clinton Davisson and Lester Germer confirmed
the De Broglie hypothesis which says that the particles of matter such as electrons have wave-like properties (see
Fig. 24.16).

Detector
Electron
Gun

Incident beam
of electrons Diffracted

Crystal

Figure 24.16: Diffraction of matter waves

(a) Davisson–Germer’s experiment


(i) Experimental confirmation of De Broglie waves was done by scientist Davisson and Germer by firing
slow-moving electrons at a crystalline nickel target. The diffraction pattern of electron beam through the
nickel crystal was same as those predicted by Bragg for X-rays.
(ii) Since diffraction is the property of waves, the diffraction of electronic beam confirmed that a wave is
connected to moving electron beam.
(iii) The electron gun was used to obtain electrons with different energies. This was done by accelerating the
e− by V volt in the electron gun.
(iv) When these accelerated electrons fall on a crystal, they are diffracted in various directions.
(v) Electrons were collected by a detector of Faraday cup which was connected to an electrometer.
(vi) Electron beams with different energies produced different intensities of diffracted electrons.
(vii) Results of the Davisson–Germer’s experiment
•• Intensity at any angle is proportional to the distance of the curve at that angle from the point of
scattering.

500

54V

Figure 24.17
P hysi cs | 24.15

•• Intensity is maximum at 54 V potential difference and 50° diffraction angle.

I =50o

54
V
Figure 24.18

(viii) From Bragg’s Law :- D sin θ = nλ (constructive interference)

incident
d o
=90
D 2

Diffracted Wave

Figure 24.19

θ : Angle between incident ray and nth maxima.


n: Diffraction order
D: Distance between atoms or 2d sin φ = nλ
D: Distance between lattice planes
φ : Angle between diffraction plane and incident ray.

(ix) The critical value of wavelength of an accelerated electron at 54 V = 1.67Å
Experimental value = 1.65 Å
(x) Any wave or particle is diffracted by crystal plane only when wavelength is in order of distance between
lattice planes of an atom.

Illustration 14: In a Davisson-Germer experiment, a, electron beam of wavelength 1.5 Å is normally incident on a
crystal, having 3Å distance between atoms. Determine the angle at which first maximum occurs. (JEE MAIN)

Sol: According to Davison–Germer’s experiment, when electrons accelerating at some potential difference V are
incident on a crystal, they diffract. The angle at which the first maxima of diffraction pattern occurs can be found
by Bragg’s law i.e., D sin θ= nλ
nλ 1 × 1.5 1
D sinθ = nλ ∴ sinθ = = = , θ= 30°
D 3 2

7. ATOMIC MODELS
Model : A model is simply a testable idea or hypothesis based on logical and scientific facts.
Theory : A model becomes a theory when it is verified by rigorous scientific analysis and experiments. . Otherwise,
the model is simply not accepted.
2 4 . 1 6 | Modern Physics

7.1 Dalton’s Atomic Model


(a) All matter is made of tiny particles called atoms. Atoms are indivisible and indestructible.
(b) All atoms of a given element are identical in mass and properties, while atoms of different elements differ in
mass and properties.
(c) All matter is made up of hydrogen atoms. The mass and radius of heaviest atom is about 250X and 10X of the
than that of the hydrogen atom, respectively.
(d) Atoms are stable and electrically neutral.

Reason of Failure of model: The discovery of electron by J.J. Thomson (1897) proved that atoms are not indivisible.
Hence, the model is no longer valid.

7.2 Thomson’s Atomic Model (or Plum-Pudding Model)


In this model, the atom is composed of electrons (which Thomson
still called “corpuscles”) surrounded by a soup of positive charge to Electron
balance the electrons’ negative charges, like negatively charged
“raisins” surrounded by positively charged “pudding”. Uniformly distributed
positively charged matter
Achievements of model: Explained successfully the phenomenon
of thermionic emission, photoelectric emission, and ionization. Figure 24.20: Thomson’s Atomic Model

Failure of the model:


(a) It could not explain the line spectrum of H-atom.
(b) It could not explain the Rutherford’s α − particle scattering (Rutherford gold foil) experiment.

7.3 Rutherford’s Experiment and Atomic Model

α -Scattering Experiment:

Results of Experiment:
(a) It was seen that in the experiment that when the α - particles were fired at the gold foil, some of the particles
(<1 in 8000) bounced off the metal foil in all directions, some right back at the source. This should have
been impossible according to Thomson’s model; the alpha particles should have all gone straight through.
Obviously, those particles had encountered an electrostatic force far greater than Thomson’s model suggested
they would, which in turn implied that the atom’s positive charge was concentrated in a much tinier volume
than Thomson imagined. This was possible only in the case when there exists a solid positive mass confining
in a very narrow space.
(b) However, most of the α - particles just flew straight through the foil. This suggested that those tiny spheres
of intense positive charge were separated by vast gulfs of empty space.

Gold foil
Figure 24.21: Scattering of alpha particles by gold nucleus
P hysi cs | 24.17

1
(c) N ∝ ⇒ If θ ↑ then N ↓ , N = No. of particles scattered per unit time
4 θ
sin  
2

N Hyperbola

Figure 24.22

Equation indicates that at larger deflection (scattering) angle, number of particles deflected are very-very less.

Figure 24.23

Graph for N & θ show that coulomb’s law holds for atomic distances also.

(d) N ∝ (Nuclear charge)2

Illustration 15: In an α - particle scattering experiment using gold foil, the number of particles scattered at 60° is
1000 per minute. What will be the number of particles per minute scattered at 90° angle? (JEE ADVANCED)

Sol: In Rutherford’s experiment, the number of particles deflected at an angle θ by the gold atoms per minute are
1
best represented by relation N ∝
θ
sin4  
2
Let N = No. of α - particles scattered per minute at an angle 90°.
1
∴ N∝  (1) … (i)
 90 
sin4  
 2 
1
Given that 1000 ∝ (2) … (ii)
 60 
sin4  
 2    60 
sin4  
Taking ratio of (i) to (ii) we get N = 1000 ×  2  = 250 /min
 90 
sin4  
 2 
Rutherford’s Atomic Model
Uniform distribution of
mass & charge

Solid sphere

Electrons (Stationary & randomly located)

Figure 24.24: Thomson’s Atomic Model


2 4 . 1 8 | Modern Physics

Non uniform distribution of mass & charge


Hollow sphere
Electrons
(movable in
circular orbits)
Figure 24.25: Ruthorford’s Atomic Model

(a) The whole positive charge and almost whole mass of an atom (leaving aside the mass of revolving e− in
various circular orbits) remains concentrated in nucleus of radius of the order of 10−15 m.

e
r
Hollow
sphere
Circular orbit
Figure 24.26: Motion of electron in atom

(b)  Σq( + )ve on proton in a nucleus = Σq( −)ve on e− in various circular orbits & hence, the atom is electrically
neutral.
(c) The necessary centripetal force for revolving round the nucleus in circular orbit is provided by coulomb’s
mv 2 k(ze)(e)
electrostatic force of attraction =
r r2
Reason of failure of model
(a) It could not explain the line spectrum of H-atom.
Justification: Asper Maxwell’s electromagnetic theory, every accelerated moving charged particle emits
energy in the form of electromagnetic waves and, therefore, the frequency of an Modern Physics - Solution (1)
while moving in a circular orbit around the nucleas will steadily decline, resulting in the continuous emission
of lines thereby mandating that the spectrum of an atom be continuous, but in reality, one obtains line
spectrum for atoms.
(b) It could not explain the stability of atoms.
Justification : Since revolving electron continuously radiates energy, the radii of circular path will continuously
decrease and in a time of about 10−8 s the revolving electron must fall down in a nucleus by adopting a spiral
path.

Application of Rutherford’s model


Determintion of distance of closest approach: When a positively charged particle approaches a stationary
nucleus (which is the positively charged core of the atom), then due to repulsion between the two (like charges
repel), the kinetic energy of positively charged particle gradually decreases, reaching a stage where its kinetic
energy becomes zero and from where it again starts retracing its original path.
Definition: The distance of closest approach is the
minimum distance of a stationary nucleus from a point z1e z2e
where the kinetic energy of a positively charged particle
approaching the nucleas for a head-on collision becomes A B
zero. Suppose a positively charged particle A of charge q1 Stationary necleus
(=z1e) approaches from infinity towards a stationary z0
nucleus of charge z 2 e then,
Figure 24.27: Distance of closest approach
P hysi cs | 24.19

Let at point B, kinetic energy of particle A becomes zero then by the law of conservation of energy at point A & B.
k(z 1e)(z 2e) k(z1e)(z 2e)
TEA = TEB ; KEA + PEA = KEB + PEB ; E+0=0+ (in joule) ∴ r0 = m
r0 E

Illustration 16: Calculate the distance of closest approach where an α -particle with kinetic energy 10 MeV is
heading towards a stationary point-nucleus of atomic number 50. (JEE MAIN)

Sol: The nucleus of tin (atomic number 50) being more massive than the
alpha particle, remains stationary. So the kinetic energy of the alpha particle A B
is converted into electric potential energy at the distance of closest
approach. -particle
K × (Z1e) × (Z 2e) z0
The electric potential energy of alpha particle is TEα =
r0
1
where K = and r0 is the distance of closest approach of alpha particle Figure 24.28
4 πε0
from nucleus of tin.
K × (2e)(50e)
TEA = TEB ; ∴ 10 × 106 eV =
r0

−14 −4
r0 1.44 × 10 m =
= ; r0 1.44 × 10 Å

Illustration 17:Find the distance of closest approach for a proton moving with a speed of 7.45 × 105 m/s towards
a free proton originally at rest. (JEE MAIN)

Sol: As the moving proton approaches the free proton originally at rest, it exerts an electric force of repulsion on
the proton at rest. At the distance of closest approach, both the protons move with same velocity along the line
of impact. The initial kinetic energy of moving proton is equal to the final kinetic energy of both the protons plus
Ke2
the electric potential energy at the distance of closest approach, given by . Here r0 is the distance of closest
approach. r0

→ V = 7.45 × 105 m/s u = 0


O O
Proton Free proton

Originally

V1 V2

r0
Proton free proton
after movement
Figure 24.29

At the time of distance of closest approach


By the law conservation of energy
1 ke2 1 1
mv 2 + 0= + mv12 + mv12  ... (i)
2 r0 2 2
v
By the conservation of momentum mv + 0 = mv1 + mv1 ∴ v1 =
2
2
1 ke2 v 4 4 × (9 × 109 )(1.6 × 10−19 )2
From equation (i) mv 2 = + m  ; r0 = × ke2 = r0 1.0 × 10 −12 m
=
2 r0 2 mv 2 (1.66 × 10−27 )(7.45 × 105 )2
2 4 . 2 0 | Modern Physics

7.4 Bohr’s Model


Bohr combined the concepts of classical physics with quantum mechanics to propose his model for H or H-like
atoms. This model is based on law of conservation of angular momentum.
(a) According to de Broglie, in a stationary orbit the circumference of Bohr’s orbit must be an integral multiple of
the wavelength associated with the moving particle
or 2πr = nλ (Constructive interference)
nh nh
or 2πr = or mvr = which is Bohr’s quantum condition.
mv 2π
(b) In an orbit, waves are always formed in whole numbers..

7.4.1 Concept of Stable, Stationary, Quantized, Fixed Allowed Radii Orbit, or Maxwell’s
Licensed Orbits

According to Bohr, if an electron revolves in these orbits the electron neither radiates nor absorbs any energy.
v
Total energy, (E) (eV)

Unbound (ionised)
r + atom


0
-0.85 n=5
n=4
Figure 24.30: Bohr radius -1.51 n = 3 Excited
states

-3.40 n=2

Ground state
-13.6 n=1

Figure 24.31: Energy level diagram

(b) Emission of energy


Where n = principle quantum no.
En = energy of e− in nth orbit

En2 n2
hc
E = h = = En1 - En2

En1 n1

Figure 24.32: Emission of energy by electron


P hysi cs | 24.21

(c) Absorption of energy


En2 n2

E E + En1= En2

En1 n1

Figure 24.33: Absorption of energy by electron

h
Electron revolves only in those orbits in which its angular momentum is integer multiple of

h
mvr = Iω = n

mv 2 kZe2
=
r r2

7.4.2 Determination of Radius, Velocity & Energy of e- in Bohr’s Orbit


(a) Determination of radius of circular path (orbit)
nh
 mvr =  … (i)

nh
∴ v=  … (ii)
2πmr

mv 2 kZe2
2
kZe2  n2h2  n2 h2
and = ; ∴ m  nh  = ; rn = v   rn =
r r2  2πmr  r  4 π2mkZe2  ; Z
×
4 π2mke2
 
n2 
r=
n × 0.529 A
Z

Results:
(1)2
(i) r1
 = × 0.529 Å ; ∴ rn = n2r1
z r

n2
Figure 24.34

(ii) ∴ r ∝ n2
r Parabolic

n
Figure 24.35

Illustration 18: The radius of the shortest orbit of a single-electron system is 18 pm. This system can be represented
as (JEE MAIN)

Sol: According to Bohr’s model, the radius of orbit of electron is directly proportional to square of principle quantum
0.529 o
number i.e., rn ∝ n2 . When the electron is in ground state (i.e., for principle quantum number =1) r1 = A
Z
2 4 . 2 2 | Modern Physics

(1)2 
For shortest orbit n = 1; rn = n2r1 ; × 0.529 Å = 18 × 10-2 A
Z
3 system is Li2+ since only single e  is present.
⇒Z=

Illustration 19: What will be the ratio of the area of circular orbits in doubly ionized lithium atom in 2nd & 3rd Bohr
orbit? (JEE MAIN)

Sol: According to Bohr’s theory as rn ∝ n2 , but A ∝ r 2 . Therefore A ∝ n4 .


A2 (2)4 16
∴ = =
A3 (3) 4 81

7.4.3 Determination of Velocity of Electron in Circular Orbit


nh
∴ mvr =  ...(i)

nh mv 2 kZe2 2πkZe2 Z 2πke2 Z


r= ; ⇒ = ⇒ v= ; ⇒ v= × ⇒ v = 2.18 × 106 m/s
2πmv r r2 nh n h n

c Z
v= m/s; where c = velocity of light in vacuum = 3 × 108 m/s
137 n

Results:
1
(i) v ∝ (Z = constant)
n
v hyperbolic

n
Figure 24.36

Illustration 20: What will be the ratio of speed of electrons in hydrogen atom in its 3rd & 4th orbit? (JEE MAIN)
Z
Sol: According to the Bohr’s theory v ∝ where v is the speed of electron in its orbit, n is the principle quantum
n
number and Z is the atomic number of the element.

Z v 4
 v∝ ∴ 3 =
n v4 3

Illustration 21: What will be the the ratio of speed of electron in 3rd orbit of He+ to 4th orbit of Li++ atom?
 (JEE MAIN)
Z
Sol: According to the Bohr’s theory v ∝ , where v is the speed of electron in its orbit n is the principle quantum
n
number and Z is the atomic number of the element.

Here the element in consideration differs in atomic number, i.e., Z(He) = 2 and Z (Li) =3
2
( v3 ) He+ 3 8
∴ =  = .
( v 4 ) Li2+ 3 9
4
P hysi cs | 24.23

7.4.4 Determination of Energy of Electron in Bohr’s Circular Orbit


1 kZe2
(a) Kinetic energy of electron KE = mv 2 ; KE =
2 2r
Results: v
(i) KE of an e− = positive quantity
(ii)  r ↑ , KE ↓ + r
(iii) when, r = ∞ , KE = 0

K( + Ze)( −e) KZe2 Figure 24.37:


(b) Potential energy of an electron PE = ; PE = − Bohr’s Orbit
r r
Results:
(i) Potential energy (PE) of an e− = negative quantity (ii) r ↑ , PE ↑ (c) If r = ∞ , PE = 0

(c) Total energy of electron: The total energy of an electron in any orbit equals the sum of its kinetic and
KZ e2 KZ e2 KZ e2
potential energy in that orbit. TE = KE + PE = − ; TE = −
2r r 2r
Results:
(i) TE of an electron in atom = (-)ve quantity. (-)ve sign indicates that electron is in bound state.
(ii) If r ↑, TE ↑
(iii) if r = ∞ , TE = 0
PE
(iv) TE = -KE = in any H-like atom
2

Total energy of terms of n

k Ze2
TE = −
 n2h2 
2× 2 
 4 π mk Ze2 
 

2π2mk 2 Z 2e4 Z2 Z2
TE =
− ⇒ TE =
− R ch ⇒ TE =
−13.6 ev
n2h2 n2 n2
2π2mk 2e4 me4
where R = Rydberg constant = = = 1.097 × 107 m−1
3
ch 8 ∈0 ch3
2

Note: Rydberg constant is not a universal constant. In Bohr calculation, it is determined by assuming the nucleus
to be stationary
For Bohr Rydberg constant,
= R ∞ 1.097 × 107 m−1 , if nucleus is not assumed stationary then
R
R= ,mN = mass of nucleus
 me 
1 +  
 mN 

7.4.5 Results Based on Total Energy Equation


(a) With the increase in principal quantum number n (relative overall energy of each orbital), both total energy
and potential energy of an electron increases, whereas the kinetic energy decreases.
(b) With the increase in principal quantum number, the difference between any two consecutive energy level
2 4 . 2 4 | Modern Physics

decreases.
(c) Total energy of an electron in any orbit in H-like atom = (Total energy of an electron in that orbit in H-atom
×Z 2 )
(d) PE of an electron in any orbit in H-like atom = (PE of an electron in that orbit in H-atom) ×Z 2 (v) KE of an
electron in any orbit in H-like atom= (KE of an electron in that orbit in H-atom) ×Z 2 (vi) ∆En n in any H-like
1 2
atom= ( ∆En n in H-atom) ×Z 2
1 2

7.4.6 Success of Bohr’s Theory


(a) Bohr successfully combined Rutherford’s model with the Planck hypothesis on the quantified energy states
at atomic level
(b) Bohr’s theory explained the atomic emission and absorption spectra
(c) It explained the general characteristics of the periodic table
(d) Bohr’s theory offered the first “working” model for the atom

7.4.7 Short Coming of Bohr’s Model


(a) Bohr’s model holds true only for atoms with one electron. E.g, H, He+, Li+2, Na+1
(b) Bohr’s model posits circular orbits whereas according to Somerfield these are elliptical.
(c) The model could not explain the intensity of spectral lines.
(d) It assume the nucleus to be stationary, but it also rotates on its own axis.
(e) It failed to account for the minute structure in spectrum line.
(f) The model offered no explanation for the Zeeman effect (splitting up of spectral lines in magnetic field) and
Stark effect (splitting up of spectral lines in electric field)
(g) Doublets observed in the spectrum of some of the atoms like sodium (5890 Å & 5896 Å) could not be
explained by Bohr’s model.

7.4.8 de Broglie’s Explanation of Bohr’s Second Postulate of Quantization

In Bohr’s model of the atom, it is stated that the angular momentum of the electron orbiting around the nucleus is
nh
quantized (that is, Ln = ; n = 1, 2, 3,..…). Why is it that the values of angular momentum are only integral

h
multiples of 
2π 
De Broglie, speculated that nature did not
single out light as being the only matter
which exhibits a wave-particle duality. He
proposed that ordinary ``particles’’ such as
electrons, protons, or bowling balls could Nucleus
also exhibit wave characteristics in certain r
circumstances.C.J. Davisson and L.H. German
later experimentally verified the wave nature
of electron in 1927. It was De Broglie’s
contention (like Bohr) was that an electron in
motion around the nucleas must be seen as a
particle wave. Analogous to waves travelling A standing wave is shown in a circular orbit where four de Broglie
wavelength fit into the circumference of the orbit.
on a string, particle waves too can lead to
standing waves under resonant conditions. Figure 24.38: De broglie model
P hysi cs | 24.25

We know that when a string is peturbed, it generates a number of wavelengths along the length of the string.
Of these, only those wavelengths that have nodes at either ends and form standing waves survive, while other
wavelengths get reflected upon themselves resulting in their amplitudes quickly dropping to zero. Therefore,
standing waves are formed when a wave travels the along the enrire length of the string and back in one, two, or
any integral number of wavelengths. For an electron moving in nth circular orbit of radius rn , the total distance is
the circumference of the orbit, 2πrn .
h
Thus, 2πrn =nλ , n = 1, 2, 3 …….. We have, λ = , where p is the magnitude of the electron’s momentum.
p
If the speed of the electron is much less than the speed of light, the momentum is mvn.
h nh nh
Thus, λ = .; 2πrn = or mv nrn =
mv n mv n 2π

This is the quantum condition proposed by Bohr for the angular momentum of the electron. Thus de Broglie
hypothesis provided an explanation for Bohr’s second postulate for the quantization of angular momentum of the
orbiting electron by postulating the wave nature of matter particles like electrons. The quantized electron orbits
and energy states are due to the wave nature of the electron and only resonant standing waves can persist.

7.4.9 Limitations
(a) The Bohr model is applicable to hydrogenic atoms with a single electron. All attempts to use Bohr’s Model
to analyze atoms with more than one electron failed as Bohr’s model deals only with interaction between
the electron and the positively charged nucleus but does not account for the interaction of an electron with
other electrons as would be the case with multi-electron atoms.(ii) While the Bohr’s model correctly predicts
the frequencies of the light emitted by hydrogenic atoms, it cannot predict the relative intensities of spectral
lines. Some frequencies in the hydrogen emission spectrum, for example, have weak intensity while others
have strong intensity. Bohr’s model is unable to account for the intensity variations.

7.4.10 Some Important Definitions and their Meaning


Energy state

Ground state Excited state


or
Most stable state
n=1

Energy state where electrons Energy state where electrons


experience attractive force of do not experience nuclear
nucleus n= m+1 attraction = Ionised state
If m=number of excited state =
Figure 24.39: Energy level classification

(a) Ionization energy and ionization potential: The ionization energy is the energy necessary to remove an
electron from the neutral atom. It is a minimum for the alkali metals which have a single electron outside a
closed shell. The ionization potential is the potential through which an electron is accelerated for removal an
electron from the neutral atom is called ionization potential.
I.E. = E∞ − E1 =−E1 = Binding energy of e− ( e∞ assumed to be zero)
(b) Excitation energy and excitation potential: The minimum energy required to excite an atom i.e., alteration
from the condition of lowest energy (ground state) to one of higher energy (excited state) is called excitation
energy of the particular excited state and corresponding potential is called excitation potential.
2 4 . 2 6 | Modern Physics

E2 n=2 E+E1=E2
E E=E2-E1

E1 n=1 Excitation energy

E3 n=3 E+E1=E3
E E=E3-E1

E1 n=1 2nd Excitation energy

E3 n=3 E+E2=E3
E E=E3-E2

E2 n=2 Excitation energy of e- for 1st excited state

E4 n=4 E+E2=E4
E E=E4-E2

E2 n=2 2ndExcitation energy of e- for 2nd excited state

E 
E+E1=E=0
E
E=-E1
E1 1 (I.E.)

E 
E+E2=E=0
E E=E-E2=-E2
E2 2 (I.E.) of in 1st excited state

Figure 24.40

If excitation energy and ionization energy are represented in eV, then corresponding value in volt is termed as
excitation potential and ionization potential, respectively.
For Example: Excitation energy and ionization energy for H-atom are 10.2 eV and 13.6 eV, respectively and,
therefore, 10.2V and 13.6V are excitation and ionization potential, respectively.

PLANCESS CONCEPTS

Reduced mass: Both the proton and electron revolve in circular orbits about their common centre of
mass. However, we can account for the motion of the nucleus simply by replacing the mass of electron
m by the reduced mass µ of the electron and the nucleus.
Mm
Here µ = ….(i)
M+m
m
Where M = mass of nucleus. The reduced mass can also be written as, µ =
m
1+
M
Note: If motion of the nucleus is also considered, then m is replaced by µ, where µ = reduced mass of
mM Z2 µ
electron – nucleus system = . In this case, En = ( −13.6eV) .
m+M n2 me
Vaibhav Krishnan (JEE 2009, AIR 22)
P hysi cs | 24.27

8. SPECTRUM

8.1 Types of Line Spectrum


Emission line spectrum: When an electric current passes through a gas which is at less than atmospheric pressure,
it gives energy to the gas. This energy is then given out as light of several definite wavelengths (colours). This
is called a emission line spectrum. These are caused when an electron hops from excited states to lower states.
Different The wavelength of emission lines of different elements have emissions of different wavelengths. For one
element the emission spectrum are unique for each element.
Absorption line spectrum: It is the electromagnetic spectrum, broken by a specific pattern of dark lines or bands,
observed when radiation traverses a particular absorbing medium and through a spectroscope. The absorption
pattern of an element is unique and can be used to identify the substance of the medium. When white light is
passed through a gas, the gas is absorbs light of certain wavelength. The bright background on the photographic
plate is then crossed by dark lines that corresponds to those wavelengths which are absorbed by the gas atoms,
resulting in transition of an atom from lower energy states to higher energy states.
(The emission spectrum consists of bright lines on dark background.)
The spectrum of sunlight has dark lines called Fraunhoffer lines. These lines are produced when the light
emanating from the core of the sun passes through the layer of cooler gas. This layer absorbs light of certain
wavelengths corresponding to the elements present in the cooler gas. This results in dark lines (absorption of
certain wavelengths) on a brighter background. Fraunhoffer lines reveal the composition of the star.

8.2 Time Period and Frequency of Electron’s Motion


2πrn n3 h3 n3
(a) Time period of revolution of an electron in the nth Bohr orbit is Tn = = = 1.5 × 10−16 sec
vn Z2 4 π2mk 2e4 Z2
For H-atom, Z = 1 ; then for n = 1, T= 1.5 × 10−16 sec T1 : T2 : T3 = 1 : 8 : 27
1 ,
1 Z2
(b) Frequency of revolution v n = vn ∝
Tn n3
1 1
For H-atom v=
1 6.6 × 1015 Hz , v1 : v 2 : v 3 = 1 : :
8 27
(c) Current and Magnetic field Due to Electron’s Motion: The motion of an electron in a circular orbit, gives
rise to some equivalent current in the orbit. It is equal to (in the nth orbit) M = current × area; Mn= In .πrn2 ;
nhe eL
Mn = ; Mn =
4 πm 2m
nh
Where L = , angular momentum of the electron in its orbit.

What you must memorise is their dependence on Z and n and order of magnitudes in first Bohr orbit.
n3
Tn ∝ ; T1 ≈ 1.5 × 10−16 s
Z2
Z2
vn ∝ ; v1 ≈ 6.6 × 1015 Hz
3
n
Z2
ωn = 2πv n ; ωn ∝
n3
nh
Ln = ; Ln ∝ n

2 4 . 2 8 | Modern Physics

PLANCESS CONCEPTS

1
Total energy of an electron in an atom = × potential energy of electron = − kinetic energy of electron
2
Nivvedan (JEE 2009, AIR 113)

8.3 Determination of Number of Spectral Lines (Theoretical) in Emission and in


Absorption Transitions

8.3.1 Number of Emission Spectra Lines


When an electron is in an excited state with principal quantum number n, then the electron may go to (n – 1)th state,
………., 2nd state or 1st state from the nthstate. Therefore, there could be (n – 1) possible transitions starting from the
nth state. The electron reaching (n – 1)th state may make (n – 2) different transitions. Similarly for other lower states,
the total number of possible transitions is
n (n − 1)
(n – 1) + (n – 2) + (n – 3) + ……… 2 + 1 =
2

8.3.2 Number of Absorption Spectral Line


At ordinary temperatures almost all the atoms remain in their lowest energy level (n = 1) and, therefore. absorption
transition can start only from the lowest energy level i.e., n = 1 level (not from n = 2, 3, 4, …… levels). Hence, only
Lyman series is found in the absorption spectrum of hydrogen atom (which as in the emission spectrum, all the
series are found)
Number of absorpton spectral lines = (n – 1)
Remember: The absorpton spectrum of sun has Balmer series also besides the Lyman series. Many H-atoms
remain in n = 2 also due to very high temperature.

8.4 Explanation of H-Spectrum and Spectral Line Formula


In a single-electron atom, the transition of an electron from any higher energy state n2 to any lower energy state
n1 causes a photon of frequency v or wavelength λ to be emitted.

hc Z2 Z2 n2
Then ∆E= hv = = En − En ; E = −Rch J = −13.6 eV En2
λ 2 1
n2 n2
1 v l
RchZ 2  RchZ 2 
21 
∴ ∆E = − −  ⇒ ∆E Rch Z 
= −  n1
n22  n2   n2 n2  En1
 1   1 2 
Figure 24.41
hc 1 1  1 1 1 
⇒ h=
ν = RchZ 2  −  ⇒ ν= = R Z 2  − 
λ  n2 n2  λ  n2 n2 
 1 2   1 2 

v = wave number = number of wave in unit length v = cv

1 1 1 
For H-atom, Z= 1 & there for,
= R − 
λ n2 2 
 1 n2 

8.5 Hydrogen Spectral Series


(a) Lyman series: n1 = 1 , n2 = 2 , 3, 4, ….. ∞
P hysi cs | 24.29

3
2
1
Absorption line
Figure 24.42


Last line
or series limit
3
2
First line
1
Emission line
Figure 24.43

1 1 1 4
For 1st line or series beginning n1 = 1 , n2 = 2=
; R  −  ; λmax = = 1216 Å
λ 12 2
2  3R

1 1 1  1
For series limit or last line n1 = 1 , n2 = ∞=
; R − ; λmin = =912.68 Å
λ 2 2 R
1 ∞ 
* Remember – Lyman series is found in UV region of electromagnetic spectrum
(b) Balmer series:

Series limit

4
3
First line
2

n=1
Energy level
Figure 24.44

n1 = 2, n2 = 3, 4, 5, 6, …… ∞ Wavelength of first line

1 1 1
i.e. maximum wavelength 6563Å
= R  2 − 2  ; ∴ λmax =
λmax 2 3 
Wavelength of last line or series limit i.e. minimum wavelength

1 1  4
λmin = R  − ; λmin = = 3646 Å
2 2 R
2 ∞ 
* Balmer series is found only in emission spectrum.
* Balmer series lies in the visible region of electromagnetic spectrum. Only the first four lines of Balmer series
lies in visible region. Rest of them lie in the infrared region of EM spectrum.
2 4 . 3 0 | Modern Physics

(c) Paschen series:=


n1 3,=
n2 4, 5, 6 . . . .∞


5
4
3

n=2

n=1
Figure 24.45

1 1 1 
n1 3,=
For first line= n2 4 , then R  −
=× 
2
λmax 3 42 

18751 Å For last line or series limit


λmax =



5
4
n=3
n=2
n=1
Figure 24.46

1 1 1  9
n1 = 3 , n2 = ∞ ; = R  2 − 2  ; λmin = = 8107 Å
λmin 3 ∞  R

* Paschen series is also found only in emission spectrum.


* Paschen series is obtained in infrared region of electromagnetic spectrum.

n1 4,
(d) Brackett series –= = n2 5, 6, 7 . . .∞


6
5
4
3
2
n1=1
Figure 24.47

1 1 1
For first list 40477 Å
= R  2 − 2  ; λmax =
λmax 4 5 
P hysi cs | 24.31

1 1 1  16
For last line or series limit = R  2 − 2  ; λmin = = 14572 Å
λmin 4 ∞  R

* Brackett series is also found only in emission spectrum.

* Brackett series is also obtained in infrared region of electromagnetic spectrum.

(e) Pfund series-

last line

6
first line
5

4
3
2
n=1

Figure 24.48

n1 = 5 , n2 = 6 , 7, 8, …... ∞

1 1 1
For first line = R  2 − 2  ; λmax = 74515 Å For last line or series limit
λmax 5 6 

1 1 1  25
= R  − 2  ⇒ λmin = = 22768 Å

λmin 5 ∞  R

* Pfund series is also obtained only in emission spectrum.

* Pfund series is situated in the infrared region of electromagnetic spectrum.

PLANCESS CONCEPTS

The minimum wavelength of a series (Lyman, Balmer, Paschen, Brackett etc.) correlates with the ionization
potential of the electron from that shell.
Chinmay S Purandare (JEE 2012, AIR 698)

General Point for Spectral Lines in Every Spectral Series


(a) Wavelength of first line is maximum and last line is minimum.
(b) As the order of spectral series increases, wavelength also usually increases
λPF > λBR > λP > λB > λL

(c) Frequency of energy emission in Lyman transitions are highest among all other series.
2 4 . 3 2 | Modern Physics

PLANCESS CONCEPTS

1
Total energy of an electron in an atom = * Potential energy of electron
2

= - Kinetic energy of electron
* If an electron jumps from then ∆=
E Ehigh − Elow
Where Elow is the low-energy state from where the jump begins and Ehigh is the high-energy state where
the jump ends.
Nitin Chandrol (JEE 2012, AIR 134)

Illustration 22: What will be the two highest wavelengths of the radiation emitted when hydrogen atoms make
transitions from higher states to n = 2 states? (JEE ADVANCED)

Sol: For electronic transition from energy state En > E2 (where n = 3, 4, 5…) to E2, the spectral series corresponds
1 1 1
to Balmer series. Therefore the wavelength of this transition is  = R  2 − 2  where n = 3, 4 ,5....∞ and R is
λ 2 n 
Rydberg’s constant.
The highest wavelength corresponds to the lowest energy of transition. This will be the case for the transition n =
3 to n = 2. The second highest wavelength corresponds to the transition n = 4 to n = 2.
E1
The energy of the state n is En =
n2
13.6eV 13.6eV 13.6eV
Thus, E2 =
− −3.4 eV ; E3 =
= − −1.5eV ; and E4 =
= − −0.85 eV
=
4 9 16

hc 1242eV × 1 nm
The highest wavelength is λ1 = = = 654 nm
∆E (3.4eV − 1.5eV)

1242eV × 1 nm
The second highest wavelength is λ2 = = 487 nm.
(3.4 eV − 0.85eV)

Illustration 23: The particle µ-meson has a charge equal to that of an electron and a mass that is 208 times that
of the electron. It moves in a circular orbit around a nucleus of charge +3e. Assume that the mass of the nucleus
is infinite. Supposing that Bohr’s model is applicable to this system, (a) derive an equation for the radius of the nth
Bohr orbit, (b) find the value of n for which the radius of the orbit is approximately the same as that of the first Bohr
orbit for a hydrogen atom (c) find the wavelength of the radiation emitted when the µ-meson jumps from the third
orbit to the first orbit. (JEE ADVANCED)

n2h2 ε0
Sol: According to Bohr’s theory, the radius of nth Bohr’s orbit is rn = and energy of µ-meson in nth orbit
πme2 Z
mZ 2e4
is En = − . If µ-meson jumps from a higher energy orbit to a lower energy orbit, the energy emitted is
8ε20n2h2
1 1
∆E = Z 2 × 13.6 ×  −  eV . To derive the expression for the nth orbit we have to keep in mind that the electrostatic
 nf ni2 
2

force of attraction between µ-meson and the nucleus provides the required centripetal force for circular orbit.
h
According to Bohr’s postulate, the magnitude of angular momentum of µ-meson must be integral multiple of .

P hysi cs | 24.33

2
mv 2 Ze2 2 Ze
(a) We have, = or v r =  … (i)
r 4 πε0r 2 4 πε0m
nh
The quantization rule is vr =
2πm

(vr)2 n2h2 4 πε0m n2h2 ε0


The radius is r = = =  … (ii)
v 2r 4 π2m2 Ze2 Zπme2
n2h2 ε0
For the given system, Z = 3 and m = 208 me .;Thus rµ = .
624 πmee2
h2 ε 0
(b) From (ii), the radius of the first Bohr orbit for the hydrogen atom is rh = .
πmee2
n2h2 ε0 h2 ε0
For rµ = rh , = or, n2 = 624 or, n = 25
2 2
624 πmee πmee
mv 2 Ze2 Ze2
(c) From (i), the kinetic energy of the atom is = and the potential energy is − .
2 8πε0r 4 πε0r
Ze2
The total energy is En = − Using (ii),
8πε0r

9 × 208mee4 1872  mee 


4
Z 2 πme4
En = − = − = −  … (iii)
8πε20n2h2 8ε20n2h2 n2  8ε20h2 

 m e4 
But  − e  is the ground state energy of hydrogen atom and hence is equal to −13.6 eV .
 8ε2h2 
 0 

1872 −25459.2eV
From (iii), En = − × 13.6 eV =
2
n n2
E1
Thus, E1 = - 25459.2 eV and E3 = = -2828.8 eV, The energy difference is E3 − E1 = 22630.4 eV.
9
hc
E3 − E1 =
λ
o
o
hc 12375eV − A
=
⇒λ = = 0.5468 A
E3 − E1 22630.4 eV

Illustration 24: A neutron moving with speed v makes a head-on collision with a stationary hydrogen atom in
ground state. Determine the minimum kinetic energy of the neutron for which inelastic (completely or partially)
collision may take place. The mass of neutron ≈ mass of hydrogen = 1.67 × 10−27 kg . (JEE ADVANCED)

Sol: It is important to remember the hydrogen atom will absorb the kinetic energy lost in an inelastic collision,
causing the atom to reach one of its excited states. The quantum of energy thus absorbed by hydrogen atom will
be equal to what is required to reach a possible excited state, and not more. Since the hydrogen atom is initially
in ground state (n = 1), the minimum energy it can absorb will be equal to that required to reach the first excited
state (n = 2). If the colliding neutron’s kinetic energy is less than this minimum energy, no energy will be absorbed,
i.e., inelastic collision may not take place.
Let us assume that the neutron and the hydrogen atom move at speeds v1 and v 2 after the collision. The collision
will be inelastic if a part of the kinetic energy is used to excite the atom. Suppose an energy ∆E is used in this way.
Considering collision to be inelastic, using conservation of linear momentum and energy,
mv mv1 + mv 2 ...(i)
=
2 4 . 3 4 | Modern Physics

1 1 1
And mv 2
= mv12 + mv 22 + ∆E …(ii)
2 2 2
2∆E 2∆E
From (i), v 2 = v12 + v 22 + 2v1 v 2 ; From (ii), v 2 = v12 + v 22 + Thus, 2v1 v 2 =
m m
4 ∆E 4 ∆E
v2 −
Hence, (v1 − v 2 )2 = (v1 + v 2 )2 −4v1 v 2 = ; As v1 − v 2 must be real, ; v 2 − ≥0;
m m
1
or mv 2 > 2∆E .
2
The minimum energy that can be absorbed by the hydrogen atom in ground state to go in an excited state is 10.2 eV.
1 2
Thus, the minimum kinetic energy of the neutron needed for an inelastic collision is mv min 2 × 10.2eV =
= 20.4 eV
2

1
Illustration 25: The potential energy U of a small moving particle of mass m is mω2r 2 , where ω is a constant and
2
r is the distance of the particle from the origin. Assuming Bohr’s model of quantization of angular momentum and

circular orbits, show that radius of the nth allowed orbit is proportional to n .  (JEE ADVANCED)
dU
Sol: The force acting on the particle in the radial direction Fr = − provides the necessary centripetal acceleration
dr
for the particle to move in a circular orbit.

dU
The force at a distance r is Fr =− =−mω2r  … (i)
dr
mv 2
Suppose the particle moves along a circle of radius r. The net force on it should be along the radius.
r
mv 2
Comparing with (i), =mω2r ⇒ v =rω  ... (ii)
r
nh nh
The quantization of angular momentum gives mvr = or, v =  … (iii)
2π 2πmr
1/2
 nh 
From (ii) and (iii), r =   .
 2πmω 
Thus, the radius of the nth orbit is proportional to n.

9. BINDING ENERGY
Binding energy,is amount of energy required to separate a particle from a system of particles or to disperse all
the particles of the system. Conversely it also defined as the energy released when particles are brought together
to form a system of particles. For example, if an electron and a proton are initially at rest and brought from large
distances to form a hydrogen atom, 13.6 eV energy will be released. The binding energy of a hydrogen atom is,
therefore, 13.6 eV, same as its ionization energy.

10. CONCEPT OF RECOILING OF AN ATOM DETERMINATION OF


MOMENTUM & ENERGY FOR RECOIL ATOMS
When a nuclear particle is emitted or ejected at high velocity from an atom the remainder of the atom recoils with a
velocity inversely proportional to its mass. This happens when an electron makes transition from any higher energy
state to any lower energy state. The atom is recoiled by sharing some energy from the energy evolved during
electronic transition.
P hysi cs | 24.35

If m = mass of recoiled atom, V = velocity of recoiled atom  En n2


2

1 hc
Then mv 2 + =En − En = ∆E
2 λ 2 1 l
h
Recoil momentum of atom = = momentum of photon En n1
λ 1

p2 Figure 24.49
Recoil energy of atom =
2m

Illustration 26: Given that the excitation energy of a hydrogen-like ion in its first excited state is 40.8 eV, determine
the energy needed to remove the electron from the ion. (JEE MAIN)
 1 
Sol: The excitation energy for hydrogen like ion for (n-1)th excited state (nth orbit) is E = hc × R × Z 2  1 −  where
n = 2, 3, 4, ….etc. The energy needed to remove the electron from the ion is E = hc × R × Z 2 .  n2 

The excitation energy in the first excited state (n=2) is


1 1  2 3
=E RhcZ 2  −  = (13.6 eV) × Z × .
12
22  4
Equating this to 40.8 eV, we get Z = 2. So, the ion in question is He+ .
RhcZ 2
The energy of the ion in the ground state is E = − = −4 × (13.6 eV) = −54.4 eV
12
Thus 54.4 eV is required to remove the electron from the ion.

PLANCESS CONCEPTS

The energy of a photon and its wavelength are inversely proportional.


B Rajiv Reddy JEE 2012, AIR 11

11. THE WAVE FUNCTION OF AN ELECTRON


Quantum mechanics has enabled physicists to develop a mathematically and logically rigorous theory which
describes the spectra in a much better way. The following is a very brief introduction to this theory.

We have already seen that to understand the behavior of light, we understand it as both a wave (the electric field E

(as well as a particle (the photon). The energy of a particular ‘photon’ is related to the ‘wavelength’ of the E wave.
Light going in x direction is represented by the wave function.= E(x,t) E0 sin(kx − ωt) In general, if light can go in
   
any direction, the wave function is = E(r , t) E0 sin(k . r − ωt)  ... (i)
 
Where r is the position vector; k is the wave vector.

11.1 Quantum Mechanics of the Hydrogen Atom



The wave function Ψ(r , t) of the electron and the possible energies E of a hydrogen atom or a hydrogen-like ion
are obtained from the Schrodinger’s equation.

−h2  ∂ 2 Ψ ∂ 2 Ψ ∂ 2 Ψ  Ze2 Ψ
 + + − EΨ 
= ... (ii)
8π2m  ∂x2 ∂y 2 ∂z 2  4 πε0r

Here (x, y, z) refers to a point with the nucleus as the origin and r is the distance of this point from the nucleus. E
refers to energy. The constant Z is the number of protons in the nucleus. For hydrogen, we have to put Z = 1. There
2 4 . 3 6 | Modern Physics


are infinite number of functions Ψ(r ) which satisfy equation (ii). These functions, which are solutions of equation
(ii), may be characterized in terms of three parameters n, l and ml With each solution Ψnl m , there is associated
l
a unique value of the energy E of the atom or the ion. The energy E corresponding to the wave function Ψnl m
l
mZ 2e4
depends only on n and may be written as En = −
8ε20h2n2

12. LASER

12.1 Basic Process of Laser


The basic strategy to get light amplification by stimulated emission is as follows: 
E2 E2
A system is chosen which has a metastable state at having an energy E2 (See Fig. 24.50).
There is another allowed energy E1 which is less than E2. The system could be any of the Incident
following: a gas or a liquid in a cylindrical tube or a solid in the shape of a cylindrical rod. photon
Let us assume that the number of atoms in the metastable state E2 is increased to more
than that in E1. Let us also assume that a photon of light of energy E2-E1 is incident on one
of the atoms in the metastable state E2. Then this atom drops to the state E1 i.e., emitting a
photon in the same phase, energy, and direction as the first one. Then these two photons E1 E1
interact with two more atoms in the state E2 and so on. Therefore, the number of photons Stimulated
keeps on increasing. All these photons will have the same phase, the same energy, and the absorption
same direction. Thus, the amplification of light is achieved.
Figure 24.50: Laser

12.2 Working
When power is suppliedy and the electric field is established, E2
some of the atoms of the mixture get ionized. These ionized Metastable state
atoms release some electrons which are accelerated by the high
electric field. Consequentially, these electrons collide with helium
atoms to take them to the metastable state at energy E3 . These
atoms collide with a neon atom and transfer the extra energy to E1
it. As a result, the helium atom returns to its ground state and the
Figure 24.51: metastable state of electron
neon atom is excited to the state at energy E2. This process keeps
looping so that the neon atoms are continuously pumped to the
state at energy E2, keeping the population (of atoms) of this state large.

12.3 Uses of Laser


(a) Spectroscopy: Most lasers being inherently pure source of light, emit near monochromatic light with a very
clear range of wavelengths. This makes the laser ideal for spectroscopy.
(b) Heat Treatment: In laser heat treating, energy is transmitted to the material’s surface in order to create
a hardened layer by metallurgical transformation. The use of lasers results in little or no distortion of the
component and, as such, eliminates much of part reworking that is currently done. Therefore, the laser heat
treatment system is cost-effective.
(c) Lunar laser ranging: The Apollo astronauts planted retroreflector arrays on te moon to make possible the
Lunar Laser Ranging Experiment. In this experiment laser beams are focused, through large telescopes on
Earth, on the arrays, and the time taken for the beam to be reflected back to Earth is measured to determine
the distance between the Earth and Moon with high accuracy.
(d) Photochemistry: Extremely brief pulses of light – as short as picoseconds or femtoseconds (10 – 12 to 10 – 15
s) – produced by some laser systems are used to initiate and analyze chemical reactions. This technique is
known as photochemistry.
(e) Laser Cooling: This technique involves atom trapping, wherein a number of atoms are enclosed in a specially
shaped arrangement of electric and magnetic fields.
P hysi cs | 24.37

(f) Nuclear Fusion: Powerful and complex arrangements of lasers and optical amplifiers are used to produce
extremely high-intensity pulses of light of extremely short duration. These pulses are arranged to impact
pellets of tritium-deuterium, simultaneously, from all directions, hoping that the compression effect of the
impacts will induce atomic fusion in the pellets.

13. X-RAYS
X-radiation is a form of electromagnetic radiation. Most X-rays have a wavelength ranging from 0.01 to 10
nanometers, corresponding to frequencies in the range 30 petahertz to 30 exahertz (3×1016 Hz to 3×1019 Hz) and
energies in the range 100 eV to 100 keV. X-radiation is also referred to as Röntgen radiation, after Wilhelm Röntgen,
who is usually credited as its discoverer, and who had named it X-radiation to signify an unknown type of radiation
produced when electron collided with the walls of the tube.
The wave nature of X-rays, was established by Laue who demonstrated that they are diffracted by crystals.

13.1 Production
High P.D.
The modern X-ray tube, called Coolidge tube, is shown in the
Fig. 24.52. A heated element emits electronswhich are
accelerated towards a cooled copper anode under a high Target
potential difference. A target metal of high atomic number Filament
Cooled
and high melting point is lodged on the anode. copper anode

The intensity of the X-ray beam is controlled by The filament


current controls the intensity of the X-ray beam by regulating Concave focusing
the number of electrons striking the target per unit time. The cathode
potential difference between the cathode and anode controls X-rays
the penetrating power of the beam.
Figure 24.52: X- Ray tube

13.2 X-Ray Spectra


A typical X-ray spectrum given by a target is shown in the Fig.24.53. The spectrum is basically continuous range of
wavelengths starting from a minimum value. A line spectrum having sharp wavelengths is superimposed on this.

Characteristic spectrum
Intensity

Continuous spectrum

min 
Figure 24.53: X-Ray spectra

13.3 Origin Of Characteristic Spectrum


If an incoming electron knocks out an electron in one of the inner shells, the exiting electron creates a vacancy
in that shell. This vacancy gets fille by another electron from a higher shell that makes a transition to this shell,
creating another vacancy in the higher shell. This hopping of electrons from higher to lower shells continues till the
inner shells are filled up. This process produces a series of radiations, some of which pertain to the X-ray region.
These radiations are typical of the target element. The X-ray spectrum of a substance is classified into K-series,
L-series, M-series etc.
2 4 . 3 8 | Modern Physics

13.4 Moseley’s Experiment and the Concept of Atomic Number


Moseley’s experiment involved the analysis of the X-ray spectra of 38 different
elements, ranging from aluminum to gold. He measured the frequency of v
principal lines of a particular series (the α-lines in the K-series) of the spectra
and was able to show that the frequencies of certain characteristic X-rays
emitted from chemical elements are proportional to the square of a number
which was close to the element’s atomic number (Z). He presented the
following relationship: =
ν a (Z − b)
z
where v = frequency of X-rays, Z = atomic number, a and b are constants. On -b
plotting the values of square root of the frequency against atomic numbers
Figure 24.54
of the elements producing X-rays on a graph, a straight line was obtained.
When electron is knocked out from n1 energy state and it is filled with electron from n2 energy state wavelength
1 1 1
of X-ray emitted is R (Z − b)2  − 
=
λ  n1 n22 
2

For K-series b = 1, n1 = 1 and n2 = 2,3,......... For L-series : b = 7.4, n1 = 2 , n2 = 3, 4,.......


For K α -line, the electron jumps from L-level to a vacancy in the K-kevel. So for the L electron, there are Z protons
in the nucleus and an electron in the K-shell which screens off the positive charge. So the net charge the L electron
faces can be taken as (Z – 1)e.

2 3
Now, E = Rhc ( Z − 1 ) = ⇒ h ν R hc ( Z − 1 ) ×  
2
=
4
3R c
=v (Z − 1)
4

13.5 Origin of Continuous Spectrum


When an incident electron comes very close to a target nucleus, it is suddenly accelerated due to the electrostatic
field around the nucleus (Coulomb field) and emits electromagnetic radiation. This radiation is referred to as
breaking radiation and is continuous. The minimum wavelength (and maximum frequency) correlates with an
electron losing all its energy in a single collision with a target atom.
hc hc 12420
If V is the acceleration p.d., then hνmax = = eV or λmin = = Å
λmin eV V

13.6 Properties of X-rays


(a) X-rays ionize the material which they penetrate.
(b) They produce the same effect on photographic plates as visible light.
(c) They cause fluorescence when they act on certain chemical compounds like zinc sulphide.
(d) X-rays penetrate matter and get absorbed as they pass through it. If I0 is the intensity of incident relation

and I is the intensity after travelling through a distance x, then I = I0 e−µx where µ is called the absorption

coefficient of the material. The atomic number of the material and its absorption coefficient are directly
proportional. This is the basis of radiography.
(e) They cause photoelectric emission.
(f) Electric and magnetic fields have no effect on X-rays as they contain no charged particles.

Note: X-Rays are not affected by electric or magnetic fields. Intensity of X-rays depends on number of electrons
in the incident beam.
P hysi cs | 24.39

PROBLEM-SOLVING TACTICS

(a) This section of Physics is more fact-based. The key to answering questions of these sections is establish a lonk
between the known and asked quantities
(b) One has to be very conversant with the formulae and standard scientific constants.
(c) In this section, graphical questions seeking relationship between various fundamental quantities are usually
asked. Assign the dependent variable as y and the independent variable as x and then look for a relation
between them.
(d) One must not get confused about approaching the questions from a wave nature or particle nature or try
to combine both. Just solve questions on the basis of the known and asked quantities and the relationship
between the two.
(e) It is important to learn the scientific constants in various units to avoid unnecessary unit conversion. (e.g.,
if energy of a photon is in eV units and wavelength asked in angstrom, one can directly use the relation =
12400/E, here 12400 is the product of Planck’s constant and speed of light.)
(f) Analytical questions pertaining to H-atom can be solved easily if one knows proportionality relation between
quantities. They need not be learnt by heart. They can be derived without bothering about constants appearing
in these relations. (e.g., radius of nth shell is directly proportional to n2 , keeping Z constant.)

FORMULAE SHEET

Speed of E.M.W. in vacuum c = 3 × 108 m / s = νλ


hc
Each photon having a frequency ν and energy E = hν = where h = 6.63 × 10−34 Js is Planck’s Constant
λ

Einstein’s Photo Electric Equation:


Photon energy = K.E. of electron + work function.
1

= mv 2 + φ
2
φ = Work function = energy needed by the electron in freeing itself from the atoms of the metal.
φ =hv 0
The minimum value of the retarding potential to prevent electron emission is:

eVcut off = (KE)max


h
De Broglie wave length given by λ = (wave length of a particle)
p
mv 2 kZe2
The electron in a stable orbit does not radiate energy i.e. =
r r2
A stable orbit is that in which the angular momentum of the electron about nucleus is an integral (n) multiple of
h h
i.e. mvr = n ; n = 1, 2, 3, ….. (n ≠ 0) .
2π 2π
For Hydrogen atom : (Z = atomic number = 1)
h
(i) Ln = angular momentum in the nth orbit = n

2 4 . 4 0 | Modern Physics

n2h2 ε0 −10 2
(ii) rn = radius of nth circular orbit = rn = 2
(0.529 Å) n2 ; (1 Å = 10 m) ; rn ∝ n
πme

−13.6eV 1
(iii) En energy of the electron in the nth orbit = i.e. En ∝
2
n n2
2
e
(iv) nth orbital speed v n =
2ε0nh

Note: Total energy of the electron in an atom is negative, indicating that it is bound.
13.6eV
Binding Energy (BE)n = −En =
n2
(iv) En − En = Energy emitted when an electron jumps from n2 orbit to n1 orbit (n2 > n1 ) .
2 1

1 1
∆E = (13.6 eV)  − 
 n12 n22 

∆E = hν ; ν= frequency of spectral line emitted.

1 1 1
Wave number= ν = = [no. of waves in unit length (1m)] = R  − 
λ  n12 n22 

Where R = Rydberg’s constant for hydrogen = 1.097 × 107 m−1

(v) For hydrogen like atoms of atomic number Z:

Bohr radius 2 n2
= rnz × n = (0.529Å) ;
Z Z

Z2
EnZ = ( −13.6) ev
n2

Note: If motion of the nucleus is also considered, then m is replaced by µ.

mM
Where µ = reduced mass of electron – nucleus system =
m+M

Z2 µ
In this case, En = ( −13.6 eV) .
2 me
n
En − En
2 1
Excitation potential for quantum jump from n1 → n2 =
electron charge

From Mosley’s Law ν= a(z − b) where b (shielding factor) is different for different series.

1 1 1 
For x-rays =R× (Z − b)2 ×  − 
λ n2 2 
 1 n2 
R = R 0 A1/3 . Where R 0 = empirical constant = 1.1 × 10 −15 m ; A = Mass number of the atom.
P hysi cs | 24.41

Solved Examples

JEE Main/Boards KEmax 0.57 × 1.6 × 10−19


V= = = 0.57 V.
e 1.6 × 10−19
Example 1: Calculate the energy of α -particle in the
event of its head-on collision with gold nucleus if the
Example 3: Determine the de Broglie wavelength of an
closest distance of approach is 41.3 Fermi.
electron having kinetic energy of 500 eV?
Sol: The kinetic energy of α -particle is converted into
the electric potential energy at the distance of closest Sol: The de-Broglie wavelength of electron moving
approach in the event of a head-on collision. The kinetic h
with Kinetic energy K is given as λ =
qq 2mK
energy of alpha particle is thus E = 1 2
4 πε0r h h
Using =λ = we get
mv 2mK
Given r0 = 41.3 × 10−15 m, Z = 70, q1= Ze= 79e and q2
= 2e,
6.6 × 10−34
λ=
Ze(2e) 9 × 109 × 79 × 2(1.6 × 10−19 )2 2 × 9.1 × 10−31 × 500 × 1.6 × 10−19
As E = =
4 πε0r 41.3 × 10−15
λ =  0.5467 × 10−10 m .
9 × 79 × 2 × 1.6 × 1.6 × 10−14
= J
41.3 Example 4: If an X-ray tube produces a continuous
spectrum of radiation with its short wavelength end
8.814 × 10−13
= 8.814 × 10−13 J = eV = 5.51 MeV 0.65Å, what is the maximum energy of a photon in the
1.6 × 10−19 radiation?

Example 2: If the wavelength of the incident light is Sol: The energy of radiation having wavelength λ is
5000 Å and the photoelectric work function of the hc
metallic plate is 1.90 eV, find E=
λ
(a) Energy of the photon in eV Given λmin = 0.65Å = 0.65 × 10−10 m,
(b) Kinetic energy of the photoelectrons emitted h = 6.63 × 10−34 Js, c = 3 × 108 ms−1
(c) Stopping potential We know, maximum energy of X-ray photon is
hc
Sol: The energy of photon is E = hν = , where λ is hc 6.63 × 10−34 × 3 × 108
λ Emax =ν
h max = =
the wavelength of the light. This photon knocks out λmin 0.65 × 10−10 × 1.6 × 10−19
photoelectron from the surface of metal with the
= 19.13 × 103 eV = 19.13 keV
maximum kinetic energy Emax = hν − φ0 = e V where
f0 is the work function of metal and V is the stopping
potential. Example 5: If ultra-violet light of λ =2600 Å is incident
on a silver surface with a threshold wavelength for
(a) Energy of the incident photon, photoelectric emission of λ =3800 Å , calculate:
hc 6.6 × 10−34 × 3 × 108 (i) Work function
E = hν= =
λ 5000 × 10−10 (ii) Maximum kinetic energy of the emitted
−19 photoelectrons.
= 3.96 × 10 joule = 2.47 eV
(iii) Maximum velocity of the photoelectrons.
(b) Kinetic energy of the photo-electrons emitted KEmax
1 hc
= mv 2 = hν − φ0 = (2.47 – 1.90) eV = 0.57 eV Sol: The work function of metal is φ = hν th = . The
2 λ th
(c) e V = KEmax Where V is stopping potential kinetic energy with which the photoelectron is ejected
2 4 . 4 2 | Modern Physics

1
from the metal surface is E = hν − φ = mv 2 hc (6.6 × 10−34 ) × (3 × 108 )
2 =
∴ λ2 =
4.8 × 10−19 4.8 × 10−19
hc 6.63 × 10−34 × 3 × 108
(i) φ = hν th = = J
λ th 3800 × 10−10 λ2 4.125 × 10−7 m = 4125 Å
=

5.23 × 10−19
= 5.23 × 10−19 J = eV = 3.27 eV Example 7: A hydrogen-like atom (atomic number Z) in
1.6 × 10−19 a higher excited state of quantum number n can make
a transition to the first excited state by successively
(ii) Incident wavelength λ =2600 Å
emitting two photons of energies 10.20 eV and 17.00
Then KEmax of emitted photoelectrons = hν − φ ; eV, respectively. On the other hand, the atom from the
same excited state can make a transition to the second
hc 6.63 × 10−34 × 3 × 108 excited state by successively emitting two photon of
here h=
ν =
λ 2600 × 10−10 energies 4.25 eV and 5.95 eV, respectively. What are the
values of n and Z. (Ionization energy of hydrogen atom
7.65 × 10−19 = 13.6 eV)?
=7.65 × 10−19 J = =4.78 eV
1.6 × 10−19
Sol: For any hydrogen-like atom, the energy released
KEmax= (4.78 – 3.27) eV; = 1.51 eV in transition from a higher excited state to a lower
1 1
KEmax × 2 1.51 × 1.6 × 10−10 × 2 excited state is ∆E = Z 2 × 13.6 ×  −  eV where nf
(iii) Vmax = = m/s.
m 9.1 × 10−31  n2f ni2 
and ni are principle quantum numbers of final (lower)
= 7.29 × 105 m / s and initial (higher) energy states respectively.
In first case, the excited atom makes a transition from
Example 6: The photocurrent generated when a nth state to n = 2 state and two photons of energies 10.2
surface is irradiated with light of wavelength 4950 Å, eV and 17.0 eV are emitted. Hence, if Z is the atomic
vanishes if a stopping potential greater than 0.6 V is number of H-like atom, then using
applied across the photo tube. When a different source 1 1
of light is used, it is found that the stopping potential ∆E = Z 2 × 13.6 ×  −  eV ;
has changed to 1.1 V. Determine the work function of  n2f ni2 
the emitting surface and the wavelength of second 1 1
source. (10.2 + 17.0)eV =Z 2 × 13.6 ×  −   ...(i)
2
2 n2 
Sol: The maximum kinetic energy of emitted In second case, the excited atom makes a transition
photoelectron is the product of stopping potential and from nth state to n = 3 state and two photons of
electron charge, given by KEmax = eV = hν − φ , where energies 4.25 eV and 5.95 eV are emitted.
ϕ is the work function of the metal. For two different
stopping potentials we have two different wavelengths 1 1
(4.25 + 5.95) eV =Z 2 × 13.6 ×  −   ...(ii)
2
of light used. 3 n2 
Let λ1 = 4950Å, V1 = 0.6 V Dividing equation (i) and (ii), we get
hc hc 27.2 9(n2 − 4) n2 − 4 2n2 − 13
KEmax
= − φ ;  KEmax= eV1 ⇒ =
φ − eV=
1 = or = 1.185 or
λ1 λ1 10.2 2 5
4(n2 − 9) n −9
(6.6 × 10−34 ) × (3 × 108 ) 2.185
− 0.6 =
1.9 eV =
4950 × 10−10 × 1.6 × 10−19 0.185
hc or n2 = 36 or n = 6
(b) = φ + eV2 ;
λ2
Putting in equation (i), we get
hc
λ2
( )
=3.04 × 10−19 + 1.6 × 10−19 × 1.1 = 4.8 × 10−19 J 1 1 
27.2 = Z 2 × 13.6  − 2 27.2 9
 or Z = × = 9
 4 36  13.6 2
or, Z = 3.
P hysi cs | 24.43

Example 8: In a hydrogen sample, if the atoms are


(6.63 × 10−34 ) × (3 × 108 )
excited to states with principal quantum number n, = m = 113.74 Å.
how many different wavelengths may be observed in 108.8 × 1.6 × 10−19
the spectrum?
Example 10: For a hypothetical hydrogen-like atom,
Sol: The hydrogen atom excited to the principal the wavelength in Å for the spectral lines for transitions
quantum number n will emit radiations as the electron
1500p2
hop back to lower energy states. Each transition from n = p to n = 1 are given by λ = , where p
to a lower energy state emits radiation of different p2 − 1
wavelength. Thus, we get a radiation spectrum. = 2, 3, 4, ….

From the nth state, the atom may go to (n – 1) (i) Find the wavelength of the least energetic and the
th
state, ……, n = 2 state or n = 1 state. So there are most energetic photons in this series.
(n – 1) possible transitions starting from the nth state. (ii) Construct an energy level diagram for this element
The atoms reaching (n – 1)th state may make (n – 2) representing at least three energy levels.
different transitions to reach n = 1 state. In the same
(iii) Determine the ionization potential of this element?
way, for other lower states, the total number of possible
transitions is (n – 1) + (n – 2) + (n – 3) + …. 2+1
n(n − 1) Sol: If wavelength of spectral lines for transitions from
= . n = p to n = 1 are given, then the energy of radiation for
2
hc hc 1
each transition is given as = E = (1 − ) . The
Example 9: For a hydrogen-like, doubly ionized λ 1500 p2
lithium atom with atomic number Z=3, determine least energy is obtained from transition from p = 2 to
the wavelength of the radiation required to excite the p = 1 and maximum energy is obtained from transition
electron in Li2+ from the first to the third Bohr orbit. from p=∞ to p=1. The ionization corresponds to the
The ionization energy of hydrogen atom is 13.6 eV. maximum energy in the spectrum.

Sol: The energy required by the hydrogen-like atom for 1500p2 hc


Given λ = and energy is E=
transition from ground state (n=1) to any of the excited 2
p −1 λ
1 hc  1 
is ∆E 13.6 Z 2 (1 − ) .
states (nth orbit)= E
Substituting for λ we get=  1 − 2  × 1010 J
n2 1500  p 
hc
Wavelength of radiation having energy E is, λ = .
E hc  1 
The energy of nth orbit of a hydrogen like atom is given =  1 −  × 1010 eV
(1500)(1.6 × 10−19 )  p2 
13.6
as En = −  1 
n2 = 8.28  1 −  eV.
 p2 

Thus for Li2+ atom, as Z = 3, the electron energies for
8.28
the first and third Bohr orbits are: Hence energy of nth state is given b En = eV
n2
13.6 × (3)2 (i) Maximum energy is released for transition from p
For n = 1, E1 = − eV = −122.4 eV
12 = ∞ to p = 1; hence wavelength of most energetic
photon is 1500 Å.
13.6 × (3)2
For n = 3, E3 = − eV = – 13.6 eV
(3)2 Least energy is released for transition from n = 2 to n =
1 transition. For p = 2 l = 2000Å
Thus the energy required to transfer an electron from
E1 level to E3 level is, (ii) The energy level diagram is shown in the Fig. 24.60.

E E 1 − E3 = – 13.6 – (– 122.4) = 108.8eV


= (iii) The ionization potential corresponds to energy
required to liberate an electron from its ground state.
Therefore, the radiation needed to cause this transition
should have photons of this energy. i.e., ionization energy = 8.28 eV
hc Hence, ionization potential = 8.28 V
hν = 108.8 eV. The wavelength of this radiation is
λ n=3 -0.92 eV
hc
= 108.8 eV or λ = n=2 -2.07 eV
108.8eV
n=1 -8.28 eV
2 4 . 4 4 | Modern Physics

Example 11: A single electron orbiting a stationary 1 1 1 1 


nucleus of charge +Ze, where Z is a constant and e is E4 − E3 = 13.6 Z 2  −  = E1  − 
2 2 2
4 3  3 42 
the magnitude of the electronic charge, requires 47.2
eV to excite the electron from the second Bohr orbit to  1 1
= 340  −  eV = 16.53 eV
the third Bohr orbit. Find  32 4 
(i) The value of Z (iii) Minimum energy required to remove electron from
(ii) Energy required to excite the electron from the third first orbit = 340 eV.
to the fourth Bohr orbit. hc
⇒ = 340 × 1.6 × 10−19
(iii) Wavelength of the electromagnetic radiation λ
required to remove the electron from the first Bohr  6.6 × 10−34 × 3 × 108 
orbit to infinity. or λ =   m = 36.40 Å
−19
 340 × 1.6 × 10 
(iv) Kinetic energy, potential energy, and angular
momentum of the electron in the first Bohr orbit. (iv) KE of the electron in the 1st orbit
(v) The radius of the first Bohr orbit.
KE1 = −E1 = 340 eV ; PE1 = 2E1 = − 680 eV
(The ionization energy of hydrogen atom = 13.6 eV.
Angular momentum of the electron in the 1st Bohr orbit
Bohr radius = 5.3 × 1011 m)
h 6.63 × 10−34
= = = 1.055 × 10−34 kg m / s
Sol: For a hydrogen-like atom, the total energy of 2π 2π
13.6 × Z 2 (v) Radius of the 1st Bohr orbit for the given atom
electron in nth orbit is E = − eV and radius of
nth orbit is n2 Bohr radius 5.3 × 10−11
= = = 1.06 × 10−11 m
Z 5
5.3 × 10−11 n2
rn =
Z
The kinetic energy in nth orbit is equal to the magnitude JEE Advanced/Boards
of total energy in nth orbit. The potential energy in nth
orbit is equal to twice the total energy in nth orbit. Example 1: If a hydrogen atom in its ground state is
excited by means of a monochromatic radiation of
The energy required to excite the atom from n1 state
wavelength 975 Å
1 1 
to n2 state
= is E 13.6Z 2  −  eV . To remove the (a) How many different lines are possible in the resulting
n2 2 
 1 n2  spectrum?
hc (b) Calculate the longest wavelength amongst them.
E = 13.6 × Z 2 .
electron from n = 1 state to infinity, =
λ
hc The ionization energy for hydrogen atom is 13.6 eV.
So λ =
13.6 × Z 2
This atom is hydrogen like Sol: First calculate the energy of the incident photon of
 1 
Z = atomic number of the nucleus given wavelength. From the formula
= E 13.6  1 −  eV ,
 n2 
En = Energy of the electron in the nth orbit. find the value of n, i.e., the maximum excited state the
= (Z)2 (energy of the electron in the nth orbit of the hydrogen atom will reach after absorbing the photon of
given wavelength. Longest wavelength in the resulting
13.6 E1 spectrum will correspond to transition from nth orbit to
hydrogen atom) = −(Z)2 eV =
2
n n2 (n-1)th orbit.
Where E1 = Energy of the electron in the 1st Bohr orbit Energy of the ground state
of the given atom.
(n = 1) = − (ionization energy) = −13.6 eV
 1 1 
(i) Given (Z 2 ) (13.6)  −  =47.2 eV
2 2
32  The wavelength of the incident radiation
⇒ Z = 5. = λ = 975Å ∴ Energy of the incident photon
13.6
(ii) E1 = −(25) = −340 eV hc 6.63 × 10−34 × 3 × 108
1 = = = 12.75 eV
λ 975 × 10−10 × 1.6 × 10−19
P hysi cs | 24.45

Let electron be exerted to nth orbit 1


1 1  (b) Kinetic energy of incident electron mv 2 = eV
⇒ 12.75 = 13.6  −  ⇒n=4 2
2
1 n2  2eV
The quantum transitions to the less excited states give Or v =
m
six possible lines as follows:
n = 4 : (4 → 3), (4 → 2), (4 → 1) 2 × 1.6 × 10−19 × 40 × 103
= = 1.19 × 108 m / s
−31
n = 3 : (3 → 2), (3 → 1); n = 2 : (2 → 1) 9.1 × 10

The longest wavelength emitted is for the transition (4


→ 3) where energy difference is minimum Example 3: If one milliwatt of light of wavelength
4560 Å is incident on a cesium surface, calculate the
n=4 photoelectric current liberated assuming a quantum
n=3 efficiency of 0.5%.
n=2 Planck’s constant h = 6.62 × 10−34 Js and velocity of
light 3 × 108 m/s.
n=1
hc
Sol: The energy of one photon of light is E = . The
1 1  λ
Emin (E4 − E3 ) = 13.6  − 
2
3 42  number of photons incident on the surface per second
can be determined by dividing power by energy of one
hc photon. The number of photons multiplied by quantum
= 0.661eV; Thus λmax =
Emin efficiency gives the number of photoelectrons emitted
per second.
6.63 × 10−34 × 3 × 108
= ≈ 18807Å
0.661 × 1.6 × 10−19 The energy of each photon of incident light
hc (6.63 × 10−34 )(3 × 108 )
E= = = 4.35 × 10−19 J
Example 2: An X-ray tube operating at a potential λ 4560 × 10−10
difference of 40 kV produces heat at the rate of 720 W. Number of photons in one milliwatt source
Assuming 0.5% of the energy of the incident electrons
is converted into X-rays, calculate Power of light
=
Energy of one photon
(a) The number of electrons per second striking the
target.
10−3
(b) The velocity of the incident electrons. = = 2.29 × 1015 /s.
−19
4.35 × 10
Sol: When X-Rays are produced in an X-Ray tube, the 0.5
power consumed is denoted by P = IV. Some of this Number of electrons released = 2.29 × 1015 ×
100
power is wasted as heat and the rest is converted to
= 1.14 × 1013 /s
X-Rays. The electron incident per second on target is
n = I/e ∴ Photoelectric current
As 0.5% of energy is converted into X-ray, therefore = Photo charge flowing per second
heat produced per second at the target is P = 0.995
= Total electrons emitted per sec. × charge of one
VI
electron
where, I is current inside tube
= (1.14 × 1013 ) × (1.6 × 10−19 ) = 1.824 µ A
P 720
⇒I= = = 0.018Å
0.995 V 0.995 × 40 × 103 Example 4: Consider the following data: Incident beam:
wavelength 3650 Å; intensity 10−8 W/m2. Surface:
Number of electrons per second incident of the target
Absorption coefficient 0.8; work function 1.6 eV.
I 0.018 Determine the time rate of number of electrons emitted
n= = = 1.1 × 1017 electrons/s.
e 1.6 × 10−19 per m2 , power absorbed per m2 , and the maximum
kinetic energy of emitted photoelectrons.
2 4 . 4 6 | Modern Physics

hc First line of Balmer series corresponds to transition


Sol: The energy of one photon of light is E = . from orbit n = 3 to orbit n = 2. Energy emitted is
λ
Number of photons incident on the surface per m2 1 1
per second is the intensity divided by energy of one ∆E = Z 2 × 13.6 ×  −  . The momentum imparted to
photon. Number of photons × absorption coefficient
4 9
= the number of photons absorbed by the surface. the heavy target during elastic collision is twice the
The remaining number of photons is equal to the momentum of the striking particle.
photoelectrons emitted per m2 per second. Suppose ZA and ZB are the atomic number and mA and
If N is the number of photons crossing per unit area per mB are the mass numbers of hydrogen like atoms A and
unit time, B, respectively.

Number of photons falling per second on unit area Z 2Rhc −Z 2 × 13.6


En =− = eV
n2 n2
Intensity Iλ Energy emitted for first Balmer line of atom A
= =
Energy of one photon hc
 1 1 
−8 −10
∆EA =−Z 2A × 13.6  2 − 2  eV
10 × 3650 × 10 2 3 
= = 18.35 × 109 / m2s
6.62 × 10−34 × 3 × 108 Similarly, energy emitted for first Balmer line of atom B
The number of photons absorbed Nab by the surface  1 1 
∆EB =−ZB2 × 13.6 ×  −  eV
per unit area per unit time 2 2
32 
Nab = absorption coefficient of surface × N According to question, ∆EA − ∆EB = 5.667 eV
9 10 2
= 0.8 × 18.35 × 10 = 1.47 × 10 / m s
 1 1 
Now, assuming that each photon ejects one electron, or 5.667 eV = (ZB2 − Z 2A ) × 13.6  2 − 2  eV
2 3 
the rate of electrons emitted per unit area is given by
5.667 × 36
N − Nab = 1.835 × 1010 − 1.47 × 1010 = 0.37 × 1010 / m2 − s or ZB2=
− Z 2A = 3 ... (i)
13.6 × 5
Power absorbed/ m2
Suppose you represent the initial velocity of each atom
= Absorption coefficient × Intensity of light falling A and B as u.
on surface= 0.8 × 10−8 = 8 × 10−9 W/ m2 . Maximum
Momentum imparted by A to target = 2mAu
kinetic energy is of emitted photoelectron is given by
hc Momentum imparted by B to target = 2mBu
(K.E.)max = −φ
λ Then according to questions,
(6.62 × 10−34 )(3 × 108 ) 2mAu = 2mBu ⇒ 2mA = mB ... (ii)
= eV–1.6eV 
−10 −19
3650 × 10 × 1.6 × 10 In case of both the atoms A and B, number of protons
and neutrons is same separately, hence mB = 2ZB and
= 3.4 eV – 1.6 eV = 1.80 eV mA = 2Z A
Putting mA and mB in equation (ii)
Example 5: Consider two hydrogen-like atoms A and
B of different masses but having equal number of 2ZB = 2(2Z A ) or ZB = 2Z A  ... (iii)
protons and neutrons. The difference in the energies Solving (i) and (iii) Z A = 1 and ZB = 2
between the first Balmer lines emitted by A and B is
5.667 eV. When these atoms, moving with the same i.e., atom A contains 1 proton and 1 neutron, i.e., atom
velocity, strike a heavy target elastically, the atom B A is deuterium (1 H2 ) .
imparts twice the momentum to the target than the Similarly, atom B contains 2 protons and 2 neutrons,
atom A. Identify the atoms A and B. i.e., atom B is singly ionized Helium.

Sol: The energy of hydrogen-like atom for nth orbit is


Example 6: A traveling hydrogen atom in the ground
Z 2 × 13.6 state makes a head-on inelastic collision with a
given by E = − , where Z = atomic number.
n2 stationary hydrogen atom in the ground state. After
collision, they move together. What is the minimum
P hysi cs | 24.47

velocity of the traveling hydrogen atom if one of the


4 × 16.32 × 10−19
atoms is to gain the minimum excitation energy after u2 = = 39.02 × 108
the collisions? 1.0078 × 1.66 × 10−27

u 6.246 × 10 4 m / s
⇒=
Sol: Here we need to consider that the kinetic energy
lost in the inelastic collision will be absorbed by one of
the hydrogen atoms to reach to its next excited state. Example 7: Assuming the potential energy between
As both the hydrogen atoms are initially in ground state
ke2
(n=1), the minimum energy absorbed will be equal to electron and proton at a distance r to be U = ,
that required by one of the atoms to reach the first 3r3
excited state (n=2). If the kinetic energy of the colliding use Bohr’s theory to obtain energy levels of such a
hydrogen atom is less than this minimum energy, no hypothetical atom.
energy will be absorbed, i.e. inelastic collision may not
take place. Sol: The negative of gradient of potential energy
is equal to force on the electron. This force provides
Let u be the velocity of the hydrogen atom before the necessary centripetal acceleration to the electron
collision and v the velocity of the two atoms moving to move in a circular orbit around the proton. The
together after collision. By the principle of conservation magnitude of angular momentum of electron is
of momentum, we have: Mu + M × 0 = 2Mv quantized. The mass of the proton is very large as
compared to the mass of electron, so it will not be
u
or v = . The loss in kinetic energy ∆E due to collision accelerated due to the force exerted on it by the
2
1 1 electron, hence it is assumed to be stationary.
is given by =∆E Mu2 − (2M)v 2
2 2 As we know that negative of potential energy gradient
u is force for a conservative field.
As v =
2 dU ke2
2 − = F. It is given that U =  … (i)
1 1 u dr 3r3
∆E
we have = Mu2 − (2M)  
2 2 2 dU d  ke2  ke2
Hence, force F = − = −   =
=
1 1 1
Mu2 − Mu2 = Mu2 dr dr  3r3  r4
2 4 4
According to Bohr’s theory this force provides the
This loss in energy is due to the excitation of one of the
necessary centripetal force for orbital motion.
hydrogen atoms. The ground state (n = 1) energy of a
hydrogen atom is: ke2 mv 2
=  … (ii)
r4 r
E1 = −13.6eV
Also quantizing angular momentum,
The energy of the first excited level (n = 2) is:
nh
E2 = −3.4 eV mvr =  ... (iii)

nh
Thus the minimum energy required to excite a hydrogen Hence, v =  ... (iv)
atom from ground state to first excited state is: E2 − E1 2πmr
= [– 3.4 – (–13.6)] eV = 10.2 eV = 10.2 × 1.6 × 10−19 J Substituting this value in Eq.(ii), we get
= 16.32 × 10−19 J mn2h2 ke2 4 π2me2k
= or r =
As per problem, the loss in kinetic energy in collision is 4 π2m2r3 r4 n2h2
due to the energy used up in exciting one of the atoms.
Thus. ∆E = E2 − E1 Substituting this value or r in Eq. (iv), we get

1 n3h3
Or Mu2 = 16.32 × 10−19 v=
4 8π3km2e2
4 × 16.32 × 10−19 Total energy E = KE + PE
Or u2 =
M
The mass of the hydrogen atom is 1.0078 amu or 1.0078 1 ke2
× 1.66 × 10−27 kg = mv 2 −
2 3r3
2 4 . 4 8 | Modern Physics

2 3 Example 9: Assuming that the short series limit of the


m  n3h3  ke2  n2h2  Balmer series for hydrogen is 3646 Å, calculate the
=   −  
2  8π3km2e2  3  4ke2mπ2  atomic number of the element, given X-ray wavelength
down to 1.0 A. Identify the element.
(n )6  h
= where   = 
2 2
6(ke ) m 3
 2π  Sol: Balmer series spectra is obtained when an electron
transitions from higher energy orbit to the second orbit
Example 8: When an electron in a tungsten (Z = 74) (n =2). The wave number of radiation emitted is given
target drops from an M shell to a vacancy in the K shell, 1  1 1 
calculate the wavelength of the characteristic X-ray as ν= = R −  . The shortest wavelength will
λ  n 2 n2 
emitted there of.  f i 
correspond to highest energy, i.e. n = ∞.
Sol: In multi electron atoms, the nucleus is shielded The short limit of the Balmer series is given by
from the outer most electron by the inner shell electron
such that the outer most electron experience Zeff charge 1  1 1  R
ν= = R − =
from nucleus. The energy of this outermost electron in λ 2 2
∞2  4
13.6 × Z eff2
nth shell is E = − 4  4  10 −1
n2 ∴ R= =   × 10 m Further the wavelengths
λ  3646 
Tungsten is a multi-electron atom. Due to the shielding
of the nuclear charge by negative charge of the inner of the k a series are given by the relation
core electrons, each electron is subjected to an effective 1 1 1 
nuclear charge Z eff which is different for different shells. ν= = R(Z − 1)2  − 
2
λ 1 n2 
Thus, the energy of an electron in the nth level of a
multi-electron atom is given by
The maximum wave number correspondence to n = ∞
13.6Z 2eff and, therefore, we must have
En = − eV For an electron in the K shell (n = 1),
n2 1
Z eff = (Z – 1) ν= = R(Z − 1)2
λ
Thus, the energy of the electron in the K shell is: 1 3646 × 10−10
Or (Z − 1)2 = =
(74 − 1)2 × 13.6 Rλ 4 × 1 × 10−10
EK = − ≈ −72500 eV
12
For an electron in the M shell (n = 3), the nucleus is = 911.5 (Z – 1) = 911.5 ≅ 30.2
shielded by one electron of the n = 1 state and eight
Or Z = 30.2 ≅ 31
electrons of the n = 2 state, a total of nine electrons, so
that Z eff= Z − 9 Thus, the energy of an electron in the Thus, the atomic number of the element concerned is
M shell is: 31.
(74 − 9)2 × 13.6 The element having atomic number Z = 30 is Gallium.
EM = − ≈ −6380 eV
2
3
Therefore, the emitted X-ray photon has an energy
given by Hv = EM − EK
=–6380 eV −( −72500eV) = 66100 eV
hc
Or = 66100 × 1.6 × 10−19 J
λ
hc
∴ λ= m
66100 × 1.6 × 10−19

(6.63 × 10−34 ) × (3 × 108 )


= m =0.0188 × 10−9 m.
66100 × 1.6 × 10−19
P hysi cs | 24.49

JEE Main/Boards

Exercise 1 Q.7 Draw a schematic diagram of the experimental


arrangement used by Davisson and Germer to establish
the wave nature of electrons. Explain briefly how the
Q.1 Define the terms: (i) work function, (ii) threshold
de-Broglie relation was experimentally verified in case
frequency and (iii) stopping potential, with reference to
of electrons.
photoelectric effect.
Calculate the maximum Kinetic energy of electron
emitted from a photosensitive surface of work function Q.8 Two lines, A and B, in the plot given below show the
3.2 eV, for the incident radiation of wavelength 300nm. 1
variation of de-Broglie wavelength, λ , versus ,
V
Q.2 Derive the expression for the de Broglie wavelength where V is the accelerating B
of an electron moving under a potential difference of V
volt. potential difference. For two
particles carrying the same 
A
Describe Davisson and Germer experiment to establish charge, which one of the two
the wave nature of electrons. Draw a labelled diagram represents a particle of smaller 1/V
of the apparatus used. mass?

Q.3 Two metals A and B have work functions 2eV and Q.9 The following graphs
5eV respectively. Which metal has lower threshold shows the variation of P Q
wavelength? stopping potential V0 with V0
the frequency v of the
Q.4 de-Broglie wavelength associated with an electron incident radiation for two
accelerated through a potential difference V is λ . What photosensitive metals P 0.5 1.0 (x1015s-1)
v
will be its wavelength when the accelerating potential and Q:
is increased to 4 V? (i) Explain which metal has smaller threshold wavelength.
(ii) Explain, giving reason, which metal emits
Q.5 Sketch a graph between frequency of incident photoelectron having smaller kinetic energy.
radiations and stopping potential for a given
photosensitive material. What information can be (iii) If the distance between the light source and metal
obtained from the value of the intercept on the P is doubled, how will the stopping potential change?
potential axis?
A source of light of frequency greater than the Q.10 The stopping potential in an experiment on
threshold frequency is placed at a distance of 1 m from photoelectric effect is 1.5V. What is the maximum
the cathode of a photo-cell. The stopping potential is kinetic energy of the photoelectrons emitted?
found to be V. If the distance of the light source from
the cathode is reduced, explain giving reasons, what Q.11 An α -Particles and a proton are accelerated from
change will you observe in the rest by the same potential. Find the ratio of their de-
(i) Photoelectric current Broglie wavelengths.

(ii) Stopping potential


Q.12 Write Einstein’s photoelectric equation.
State clearly the three salient features observed in
Q.6 Ultraviolet radiations of different frequencies v1 photoelectric effect, which can be explained on the
and v 2 are incident on two photosensitive materials basis of the above equation.
having work functions W1 and W2 ( (W1 > W2 )
respectively. The Kinetic energy of the emitted electron
Q.13 Define the term ‘stopping potential’ in relation to
is same in both cases. Which one of the two radiations
photoelectric effect.
will be of higher frequency?
2 4 . 5 0 | Modern Physics

Q.14 Draw a plot showing the variation of photoelectric Q.23 The ground state energy of hydrogen atom is
current with collector plate potential for two different –13.6eV. What are the kinetic and potential energies of
frequencies, v1 > v 2 ,of incident radiation having the electron in this state?
same intensity. In which case will the stopping potential
be higher? Justify your answer. Q.24 In a Geiger-Marsden experiment, calculate the
distance of closest approach to the nucleus of Z = 80,
Q.15 A proton and an electron have same kinetic when an α -particle of 8 MeV energy impinges on it
energy. Which one has greater de-Broglie wavelength before it comes momentarily to rest and reverse its
and why? direction.
How will the distance of closest approach be affected
Q.16 Define the terms (i) ’cut-off voltage’ and (ii) when the kinetic energy of the α -particle is doubled?
’threshold frequency’ in relation to the phenomenon of
photoelectric effect.
Q.25 A photon and electron have got the same de-
Using Einstein’s photoelectric equation show how the Broglie wavelength. Which has the greater total energy?
cut-off voltage and threshold frequency for a given Explain.
photosensitive material can be determined with the
help of a suitable plot/graph. Q.26 If the intensity of incident radiation of a metal
surface is doubled, what happens to the kinetic energy
Q.17 Derive the expression for the radius of the ground of the electrons emitted?
state orbit of hydrogen atom, using Bohr’s postulates.
Calculate the frequency of the photon, which can excite Q.27 The wavelength of a spectral line is 4000 Å.
the electron to –3.4 eV from –13.6 eV. Calculate its frequency and energy. Given,
c = 3 × 108 ms−1 and =
h 6.6 × 10−34 Js .
Q.18 A stream of electrons travelling with speed ‘v’ m/s
at right angles to a uniform electric field ‘E’, is deflected Q.28 Calculate the longest wavelength of the incident
e v2 radiation, which will eject photoelectrons from a metal
in a circular path of radius ‘r’. Prove that = .
m rE surface, whose work function is 3 eV.

Q.19 In a hydrogen atom, an electron of change ‘e’


revolves in a orbit of radius ‘r’ with a speed ‘v’. Prove Exercise 2
that the magnetic moment associated with the electron
evr Single Correct Choice Type
is given by .
2
Q.1 Let nr and nb be respectively the number of
Q.20 Draw a labeled diagram of experimental setup of
photons emitted by a red bulb and a blue bulb of equal
Rutherford’s alpha particle scattering experiment. Write
power in a given time.
two important inferences drawn from this experiment.
(A) nr = nb (B) nr < nb
Q.21 The ground state energy of hydrogen atom is (C) nr > nb (D) Data insufficient
–13.6eV.
(i) What is the potential energy of an electron in the 3rd Q.2 In a photo-emissive cell, with exciting wavelength
excited state? λ , the maximum kinetic energy of electron is K. If
(ii) If the electron jumps to the ground state from third 3λ
the exciting wavelength is changed to the Kinetic
excited state, calculate the wavelength of the photon 4
emitted. energy of the fastest emitted electron will be:
3K 4K
(A) (B)
Q.22 Drawn a schematic arrangement of the Geiger– 4 3
Marsden experiment. How did the scattering of α 4K 4K
-particle by a thin foil of gold provide an important way (C) Less than (D) Greater than
3 3
to determine an upper limit on the size of the nucleus?
Explain briefly.
P hysi cs | 24.51

Q.3 If the frequency of light in a photoelectric Q.8 By increasing the intensity of incident light keeping
experiment is doubled, the stopping potential will frequency (v > v 0 ) fixed, on the surface of metal
(A) Be doubled (A) Kinetic energy of the photoelectrons increase
(B) Be halved (B) Number of emitted electrons increases
(C) Become more than doubled (C) Kinetic energy and number of electrons increase
(D) Become less than doubled (D) No effect

Q.4 The stopping potential for the photoelectron Q.9 A proton and an electron accelerated by same
emitted from a metal surface of work function 1.7 eV is potential difference have de-Broglie wavelength λp
10.4 V. Identify the energy levels corresponding to the and λe
transition in hydrogen atom which will result in emission
(A) λe =λp (B) λe < λp
of wavelength equal to that of incident radiation for the
above photoelectric effect. (C) λe > λp (D) None of these
(A) n = 3 to 1 (B) n = 3 to 2
Q.10 An electron with initial kinetic energy of 100eV is
(C) n = 2 to 1 (D) n = 4 to 1 accelerated through a potential difference of 50V. Now
the de-Broglie wavelength of electron becomes
Q.5 Radiation of two photon energies twice and
(A) 1Å (B) 1.5 Å (C) 3Å (D) 12.27Å
five times the work functions of metal are incident
successively on the metal surface. The ratio of the
maximum velocity of photoelectrons emitted is the two Q.11 If h is Planck’s constant in SI system, the
cases is momentum of a photon of wavelength 0.01 Å is:

(A) 1 : 2 (B) 2 : 1 (C) 1 : 4 (D) 4 : 1 (A) 10−2 h (B) h (C) 102 h (D) 1012 h

Q.6 Cut off potentials for a metal in photoelectric effect Q.12 Let K1 be the maximum kinetic energy of
for light of wavelength λ1 , λ2 and λ3 is found to be photoelectrons emitted by a light of wavelength λ1
V1 , V2 and V3 volts if V1 , V2 and V3 are in Arithmetic and K 2 corresponding to λ2 . If λ1 = 2λ2 , then:
Progression and λ1 , λ2 and λ3 will be: K
(A) 2K1 = K 2 (B) K1 = 2K 2 (C) K1 < 2 (D) K1 > 2K 2
2
(A) Arithmetic Progression
(B) Geometric Progression Q.13 Imagine a Young’s double slit interference
experiment performed with waves associated with fast
(C) Harmonic Progression moving electrons produced from an electron gun. The
(D) None distance between successive maxima will decrease
maximum if
Q.7 In a photoelectric experiment, the collector plate (A) The accelerating voltage in the electron gun is
is at 2.0V with respect to the emitter plate made of decreased
copper φ=4.5eV). The emitter is illuminated by a source (B) The accelerating voltage is increased and the
of monochromatic light of wavelength 200nm. distance of the screen from the slits is decreased
(A) The minimum kinetic energy of the photoelectrons (C) The distance of the screen from the slits is increased
reaching the collector is 0.
(D) The distance between the slits is decreased.
(B) The maximum kinetic energy of the photoelectrons
reaching the collector is 3.7ev.
Q.14 If the electron in a hydrogen atom was in the
(C) If the polarity of the battery is reversed then answer energy level with n = 3, how much energy in joule would
to part A will be 0. be required to ionize the atom? (Ionization energy of
(D) If the polarity of the battery is reversed then answer H-atom is 2.18 × 10−18 J):
to part B will be 1.7eV. (A) 6.52 × 10–16 J (B) 2.86 × 10–10 J
(C) 2.42 × 10–19 J (D) 3.56 × 10–19 J
2 4 . 5 2 | Modern Physics

Q.15 In hydrogen and hydrogen like atoms, the ratio of theory of hydrogen atom,
difference of energies E4n − E2n and E2n − En varies with
(A) fr 2L is constant for all orbits
its atomic number z and n as:
(B) frL is constant for all orbits
z2 z4 z
(A) (B) (C) (D) z 0n0
n2 n4 n (C) f 2rL is constant for all orbits
(D) frL2 is constant for all orbits
Q.16 In a hydrogen atom, the electron is in nth excited
state. It may come down to second excited state by
Q.22 Radius of the second Bohr orbit of singly ionized
emitting ten different wavelengths. What is the value
helium atom is
of n?
(A) 0.53 Å (B) 1.06 Å (C) 0.265 Å (D) 0.132 Å
(A) 6 (B) 7 (C) 8 (D) 5

Q.23 An electron in Bohr’s hydrogen atom has an energy


Q.17 Monochromatic radiation of wavelength λ
of –3.4 eV. The angular momentum of the electron is
is incident on a hydrogen sample in ground state.
Hydrogen atoms absorb the light and subsequently h h
(A) (B)
emit radiations of ten different wavelengths. The value π 2π
of λ is nh 2h
(C) (n is an integer) (D)
(A) 95nm (B) 103nm (C) 73nm (D) 88nm 2π π

Q.18 In a sample of hydrogen like atoms all of which Q.24 An electron is in an excited state in hydrogen-like
are in ground state, a photon beam containing photons atom. It has a total energy of –3.4eV. If the kinetic energy
of various energies is passed. In absorption spectrum, of the electron is E and its de-Broglie wavelength is λ ,
five dark lines are observed. The number of bright then
lines in the emission spectrum will be (Assume that all (A) E = 6.8eV, λ =6.6 × 10−10 m
transitions take place)
(B) E = 3.4eV, λ = 6.6 x 10−10 m
(A) 5 (B) 10 (C) 15 (D) None of these
(C) E = 3.4eV, λ = 6.6 x 10−11 m

Q.19 When a hydrogen atom, initially at rest emits, a (D) E = 6.8eV, λ = 6.6 x 10−11 m
photon resulting in transition n = 5 → n = 1, its recoil
speed is about Q.25 If radiation of all wavelengths from ultraviolet
−4 −2 to infrared is passed through hydrogen a gas at room
(A) 10 m/s (B) 2 × 10 m/s
temperature, absorption lines will be observed in the:
−2
(C) 4.2 m / s (D) 3.8 × 10 m/s
(A) Lyman series (B) Balmer series

Q.20 The electron in a hydrogen atom makes a (C) Both (A) and (B) (D) Neither (A) nor (B)
transition from an excited state to the ground state.
Which of the following statement is true? Q.26 In the hydrogen atom, if the reference level of
potential energy is assumed to be zero at the ground
(A) Its kinetic energy increases and its potential and
state level, choose the incorrect statement.
total energies decrease.
(A) The total energy of the shell increases with increase
(B) Its kinetic energy decreases, potential energy
in the value of n.
increases and its total energy remains the same.
(B) The total energy of the shell decrease with increase
(C) Its kinetic, and total energies decrease and its
in the value of n.
potential energy increases.
(C) The difference in total energy of any two shells
(D) Its kinetic, potential and total energies decrease.
remains the same.

Q.21 The magnitude of angular momentum, orbit (D) The total energy at the ground state becomes
radius and frequency of revolution of electron in 13.6 eV.
hydrogen atom corresponding to quantum number
n are L, r and respectively. Then according to Bohr’s
P hysi cs | 24.53

Q.27 Choose the correct statement(s) for hydrogen and Q.3 If elements with principal quantum number n >
deuterium atoms (considering motion of nucleus) 4 were not allowed in nature, the number of possible
elements would be  (1983)
(A) The radius of first Bohr orbit of deuterium is less
than that of hydrogen (A) 60 (B) 32 (C) 4 (D) 64
(B) The speed of electron in the first Bohr orbit of
deuterium is more than that of hydrogen. Q.4 Consider the spectral line resulting from the
transition n = 2 → n = 1 in the atoms and ions given
(C) The wavelength of first Balmer line of deuterium is
below. The shortest wavelength is produced by (1983)
more than that of hydrogen
(A) Hydrogen atom
(D) The angular momentum of electron in the first Bohr
orbit of deuterium is more than that of hydrogen. (B) Deuterium atom
(C) Singly ionized helium
Q.28 In a Coolidge tube experiment, the minimum
wavelength of the continuous X-ray spectrum is equal (D) Doubly ionized lithium
to 66.3 pm, then 4
Q.5 Equation: 411 H →2 He2+ + 2e− + 26 MeV
(A) Electron accelerate through a potential difference of
12.75 kV in the Coolidge tube represents (1983)
(B) Electrons accelerate through a potential difference (A) b – decay (B) g – decay
of 18.75 kV in the Coolidge tube
(C) Fusion (D) Fission
(C) de-Broglie wavelength of the electrons reaching the
anticathode is of the order of 10µm. Q.6 For a given plate voltage, the plate current in a
(D) de-Broglie wavelength of the electrons reaching the triode valve is maximum when the potential of (1985)
anticathode is 0.01Å. (A) The grid is positive and plate is negative.

Q.29 The potential difference applied to an X-ray tube (B) The grid is zero and plate is positive.
is increased. As a result, in the emitted radiation: (C) The grid is negative and plate is positive
(A) The intensity increases (D) The grid is positive and plate is positive
(B) The minimum wave length increases
Q.7 The X-ray beam coming from an X-ray tube will be
(C) The intensity decreases
 (1985)
(D) The minimum wave length decreases
(A) Monochromatic
(B) Having all wavelengths smaller than a certain
Previous Years’ Questions maximum wavelength

Q.1 The shortest wavelength of X-rays emitted from an (C) Having all wavelengths larger than a certain
X-ray tube depends on  (1982) minimum wavelength

(A) The current in the tube (D) Having all wavelengths lying between a minimum
and a maximum wavelength
(B) The voltage applied to the tube
(C) The nature of the gas in tube Q.8 Statement-I: If the accelerating potential in
(D) The atomic number of the target material an X-ray tube is increased, the wavelengths of the
characteristic X-rays do not change.
Q.2 Beta rays emitted by a radioactive material are Statement-II: When an electron beam strikes the target
 (1983) in an X-ray tube, part of the kinetic energy is converted
(A) Electromagnetic radiations into X-ray energy.  (2007)

(B) The electrons orbiting around the nucleus (A) If Statement-I is true, statement-II is true; statement-
II is the correct explanation for statement-I.
(C) Charged particles emitted by the nucleus
(B) If Statement-I is true, statement-II is true; statement-
(D) Neutral particles II is not a correct explanation for statement-I.
2 4 . 5 4 | Modern Physics

(C) If statement-I is true; statement-II is false. Q.14 If a strong diffraction peak is observed when
electrons are incident at an angle ‘i’ from the normal to
(D) If statement-I is false; statement-II is true.
the crystal planes with distance ‘d’ between them (see
figure), de Broglie wavelength λdB of electrons can be
Q.9 To produce characteristic X-rays using a tungsten calculated by the relationship (n is an integer)  (2008)
target in an X-ray generator, the accelerating voltage
should be greater than…… V and the energy of the (A) d sin i = nλdB (B) 2d cos i = nλdB
characteristic radiation is ………. eV. (1983) (C) 2d sin i = nλdB (D) d cos i = nλdB
(The binding energy of the innermost electron in
tungsten is 40 keV). Q.15 In an experiment, electrons are made to pass
through a narrow slit of width ‘d’ comparable to their
Q.10 The radioactive decay rate of a radioactive element de Broglie wavelength. They are detected on a screen
is found to be 103 disintegration/second at a certain at a distance ‘D’ from the slit (see figure).  (2008)
time. If the half-life of the element is one second, the
decay rate after one second is ………… and after three
seconds is………  (1983)
y=0
d
Q.11 The maximum kinetic energy of electrons emitted D
in the photoelectric effect is linearly dependent on the
……….. of the incident radiation. (1984)
Which of the following graph can be expected to
Q.12 In the uranium radioactive series the initial represent the number of electrons ‘N’ detected as a
nucleus is 238 206 function of the detector position ‘y’(y = 0 corresponds
92 U and the final nucleus is 82 Pb . When
the uranium nucleus decays to lead, the number to the middle of the slit)?
y
of α -particles emitted is ………. and the number of y
β -particles emitted is………. (1985)
(A) d (B) d
N N
Directions : Q.13, Q.14 and Q.15 are based on the
following paragraph.
y y
Wave property of electrons implies that they will show
diffraction effects. Davisson and Germer demonstrated
this by diffracting electrons from crystals. The law (C) d (D) d
N
governing the diffraction from a crystal is obtained by N
requiring that electron waves reflected from the planes
of atoms in a crystal interfere constructively (see in
figure). Q.16 Two points P and Q are maintained at the
potentials of 10V and -4V respectively. The work done
Incoming
Electrons
Outgoing in moving 100 electrons from P to Q is  (2009)
Electrons
i
(A) –19×10–17 J (B) 9.60×10–17 J

d (C) –2.24×10–16 J (D) 2.24×10–16 J

Q.17 The surface of a metal is illuminated with the light


Crystal plane of 400 nm. The kinetic energy of the ejected photo
electrons was found to be 1.68 eV. The work function of
Q.13 Electrons accelerated by potential V are diffracted the metal is (hc = 1240 eV nm)  (2009)
from a crystal. If d = 1Å and i = 30°, V should be about (A) 3.09 eV (B) 1.41 eV
(h = 6.6 ×10−34 Js, me= 9.1 ×10−31 kg, e = 1.6 ×10−19 C)
(C) 151 eV (D) 1.68 ev
 (2008)
(A) 2000 V (B) 50 V (C) 500 V (D) 1000 V
Q.18 Statement-I: When ultraviolet light is incident
on a photocell, its stopping potential is V0 and the
P hysi cs | 24.55

maximum kinetic energy of the photoelectrons is Kmax. Statement-II: If electrons have wave nature, they can
When the ultraviolet light is replaced by X-rays, both V0 interfere and show diffraction.  (2012)
and Kmax increase.
(A) Statement-I is false, statement-II is true
Statement-II: Photoelectrons are emitted with speeds
(B) Statement-I is true, statement-II is false
ranging from zero to a maximum value because of
the range of frequencies present in the incident light. (C) Statement-I is true, statement-II is the correct
 (2010) explanation for statement-I
(A) Statement-I is true, statement-II is true; statement-II (D) Statement-I is true, statement-II is true, statement-II
is the correct explanation of statement-I. is not the correct explanation for statement-I.
(B) Statement-I is true, statement-II is true; statement-II
is not the correct explanation of statement-I. Q.22 A diatomic molecule is made of two masses
m1 and m2 which are separated by a distance r. If we
(C) Statement-I is false, statement-II is true. calculate its rotational energy by applying Bohr’s rule
(D) Statement-I is true, statement-II is false. of angular momentum quantization, its energy will be
given by (n is an integer) (2012)
Q.19 If a source of power 4 kW produces 1020 photons/ (m1 + m2 )2 n2h2 n2h2
second, the radiation belong to a part of the spectrum (A) (B)
called  (2010) 2m12m22r 2 2(m1 + m2 )r 2

(A) X–rays (B) Ultraviolet rays 2n2h2 (m1 + m2 )n2h2


(C) (D)
(C) Microwaves (D) γ – rays (m1 + m2 )r 2 2m1m2r 2

Q.20 This question has Statement-I and Statement-II. Q.23 The anode voltage of a photocellis kept fixed.
Of the four choices given after the statements, choose The wavelength λ of the light falling on the cathode is
the one that best describes the two statements. gradually changed. The plate current I of the photocell
varies as follows :  (2013)
Statement-I: A metallic surface is irradiated by a
monochromatic light of frequency v > v0 (the threshold I I
frequency). The maximum kinetic energy and the
stopping potential are Kmax and V0 respectively. If the (A) (B)
frequency incident on the surface doubled, both the
Kmax and V0 are also doubled (2011)
O  O 

Statement-II: The maximum kinetic energy and the I


I
stopping potential of photoelectrons emitted from
a surface are linearly dependent on the frequency of
(C) (D)
incident light.
(A) Statement-I is true, statement-II is true; statement-II
is the correct explanation of statement-I. O  O 

(B) Statement-I is true, statement-II is true; statement-II Q.24 In a hydrogen like atom electron make transition
is not the correct explanation of statement-I. from an energy level with quantum number n to another
with quantum number (n – 1). If n>>1, the frequency of
(C) Statement-I is false, statement-II is true.
radiation emitted is proportional to : (2013)
(D) Statement-I is true, statement-II is false.
1 1 1 1
(A) (B) (C) (D)
n n 2 3
n n3
Q.21 This question has statement-I and statement-II. Of
2
the four choices given after the statements, choose the
one that best describes the two statements Q.25 The radiation corresponding to 3 → 2 transition
of hydrogen atoms falls on a metal surface to produce
Statement-I: Davisson – germer experiment established
photoelectrons. These electrons are made to enter a
the wave nature of electrons.
magnetic field of 3 × 10–4 T. If the radius of the largest
circular path followed by these electrons is 10.0 mm,
2 4 . 5 6 | Modern Physics

the work function of the metal is close to  (2014)


(iii) Davison, Germer (r) Wave nature of
(A) 1.8 eV (B) 1.1 eV (C) 0.8 eV (D) 1.6 eV experiment electron
(s) Structure of atom
Q.26 Hydrogen (1H1), Deuterium (1H2), singly ionised
Helium (2He4)+ and doubly ionised lithium (3Li6)++ all have (A) (i) → (p), (ii) → (s), (iii) → (r)
one electron around the nucleus. Consider an electron
(B) (i) → (q), (ii) → (s), (iii) → (r)
transition from n= 2 to n= 1. If the wave lengths of
emitted radiation are λ1, λ2, λ3 and λ4 respectively then (C) (i) → (q), (ii) → (p), (iii) → (r)
approximately which one of the following is correct?
(D) (i) → (s), (ii) → (r), (iii) → (q)
 (2014)
(A) 4 λ1 = 2λ2 = 2λ3 = λ4 (B) λ1 = 2λ2 = 2λ3 = λ4 Q.28 Radiation of wavelength λ , is incident on a
(C) λ1 = λ2 = 4λ3 = 9λ4 (D) λ1 = 2λ2 = 3λ3 = 4λ4
photocell. The fastest emitted electron has speed v If

Q.27 Match List-I (Fundamental Experiment) with List-II the wavelength is changed to , the speed of the
4
(its conclusion) and select the correct option from the
fastest emitted electron will be :  (2016)
choices given below the list : (2015)
1 1
 4 2  4 2
List - I List - II (A) < v   (B) = v  
3 3
(i) Franck-Hertz (p) Particle nature of
1 1
experiment light  3 2  4 2
(C) = v   (D) > v  
(ii) Photo-electric (q) Discrete energy 4 3
experiment levels of atom

JEE Advanced/Boards

Exercise 1 collector. In the same setup, light of wavelength 220


nm, ejects electron which require twice the voltage
V0 to stop them in reaching a collector. Find the
Q.1 When a monochromatic point source of light is at
numerical value of voltage V0 . (Take plank’s constant,
a distance of 0.2 m from a photoelectric cell, the cut off
h = 6.6 × 10−34 J s and 1 eV = 1.6 × 10−19 J)
voltage and the saturation current are respectively 0.6
V and 18.0 mA. If the same source is placed 0.6 m away
from the photoelectric cell, then find Q.4 A small 10W source of ultraviolet light of Wavelength
99 nm is held at a distance 0.1m from a metal surface.
(a) The stopping potential The radius of an atom of the metal is approximately
(b) The saturation current 0.05 nm. Find
(i) The average number of photons striking an atom per
Q.2 663 mW of light from of 540 nm source is incident second.
on the surface of a metal. If only 1 of each 5 x 109
(ii) The number of photoelectrons emitted per unit
incidents photons is absorbed and causes an electron
area per second if the efficiency of liberation of
to be ejected from the surface, the total photocurrent
photoelectrons is 1%.
in the circuit is ________.

Q.5 The surface of cesium is illuminated with


Q.3 Light of Wavelength 330 nm falling on a piece
monochromatic light of various wavelengths and the
of metal ejects electrons with sufficient energy which
stopping potentials for the wavelengths are measured.
requires voltage V0 to prevent a electron from reading
The results of this experiment is plotted as shown in
P hysi cs | 24.57

the figure. Estimate the value of work function of the Q.10 In a photoelectric effect set-up, a point source
cesium and Planck’s constant. of light of power 3.2 × 10−3 W emits mono energetic
photons of energy 5.0 eV. The source is located at a
distance of 0.8 m from the center of a stationary metallic
sphere of work function 3.0 eV & of radius 8.0 × 10−3 .
Stopping potential (volt)

The efficiency of photoelectrons emission is one for


2 0.49 every 106 incident photons. Assume that the sphere is
1 isolated and initially neutral, and that photoelectrons
15
0 v x10 Hz are instantly swept away after emission.
-1 0.5 1.0 1.5
-2 (a) Calculate the number of photoelectrons emitted per
seconds.
(b) Find the ratio of the wavelength of incident light to
the de-Broglie wave length of the fastest photoelectrons
Q.6 A small plate of a metal (work function = 1.17 eV) is emitted.
placed at a distance of 2m from a monochromatic light
(c) It is observed that the photoelectron emissions stops
source of wave length 4.8 × 10−7 m power 1.0 watt. The
at a certain time t after the light source is switched on.
light falls normally on the plate. Find the number of
Why?
photons striking the metal plate per square meter per
second. If a constant uniform magnetic field of strength (d) Evaluate the time t.
10−4 tesla is applied parallel to the metal surface, find
the radius of the largest circular path followed by the Q.11 When photons of energy 4.25eV strike the surface
emitted photoelectrons. of a metal A, the ejected photoelectrons have maximum
kinetic energy Ta eV and de Broglie wavelength λa . The
Q.7 Electrons in hydrogen like atoms (Z = 3) make maximum kinetic energy of photoelectrons liberated
transition from the fifth to the fourth orbit & from from another metal B by photons of energy 4.7eV is
the fourth to the third orbit. The resulting radiations Tb = ( Ta – 1.5) eV. If the de Broglie wavelength of these
are incident normally on a metal plate & eject photoelectrons is λb = 2λa , then find
photoelectrons. The stopping potential for the
(a) The work function of a
photoelectrons ejected by the shorter wavelength is
3.95 V. Calculate the work function of the metal, & the (b) The work function of b
stopping potential for the photoelectrons ejected by (c) Ta and Tb
the longer wavelength. (Rydberg constant = 1.094 ×
107 m−1 ).
Q.12 An electron of mass “m” and charge “e” initially at
rest gets accelerated by a constant electric field E. The
Q.8 A beam of light has three wavelength 4144 Å, 4972 rate of change of de Broglie wavelength of this electron
Å & 6216Å with a total intensity of 3.6 × 10−3 W.m−2 at time t is……………….
equally distributed amongst the three wavelengths.
The beam falls normally on an area 1.0 cm2 of a clean
metallic surface of work function 2.3 eV. Assume that Q.13 A hydrogen atom in a state having a binding
there is no loss of light by reflection and that each energy 0.85eV makes a transition to a state of excitation
energetically capable photon ejects one electron. energy 10.2eV. The wave length of emitted photon is
Calculate the number of photoelectrons liberated in ……………..nm.
time t = 2 s.
Q.14 A hydrogen atom is in 5th excited state. When the
Q.9 A small 10 W source of ultraviolet light of electrons jump to ground state the velocity of recoiling
wavelength 99nm is held at a distance 0.1 m from a hydrogen atom is ……………….m/s and the energy of the
metal surface. The radius of an atom of the metal is photon is ……………eV.
approximately 0.05 nm. Find:
Q.15 The ratio of series limited wavelength of Balmer
(i) The number of photons striking an atom per seconds.
series to wavelength of first line of
(ii) The number of photoelectrons emitted per seconds
Paschen series is …………………..
if the efficiency of liberation of photoelectrons is 1%.
2 4 . 5 8 | Modern Physics

Q.16 A neutron with kinetic energy 25 eV strikes a transition are λ1 , λ2 and λ3 . The value of λ3 in terms
stationary deuteron. Find the de Broglie wavelengths of λ1 and λ2 is given by _______.
of both particles in the frame of their center of mass.

E3 n=3
Q.17 Assume that the de Broglie wave associated with
1
an electron can form a standing wave between the E2 n=2
atoms arranged in a one dimensional array with nodes
2
at each of the atomic sites. It is found that one such
3

E1 n=1
standing wave is formed if the distance ‘d’ between the
atoms of the array is 2 Å. A similar standing wave is
Q.23 Imagine an atom made up of a proton and a
again formed if ‘d’ is increased to 2.5 Å but not for any
hypothetical particle of double the mass of an electron
intermediate value of d. Find the energy of the electrons
but having the same charge as the electron. Apply the
in electron volts and the least value of d for which the
Bohr atom model and consider a possible transition of
standing wave of the type described above can form.
this hypothetical particle to the first excited level. Find
the longest wavelength photon that will be emitted λ
Q.18 A stationary He+ ion emitted a photon (in terms of the Rydberg constant R.)
corresponding to the first line its Lyman series. That
photon liberated a photoelectron from a stationary
Q.24 In a hydrogen atom, the electron moves in an orbit
hydrogen atom in the ground state. Find the velocity of
of radius 0.5 Å making 1016 revolutions per second. The
the photoelectron.
magnetic moment associated with the orbital motion
of the electron is __________.
Q.19 A gas of identical hydrogen like atom has some
atom in the lowest (ground) energy level A & some
Q.25 A hydrogen like atom has its single electron
atoms in a particular upper (excited) energy level B
orbiting around its stationary nucleus. The energy
& there are no atoms in any other energy level. The
excite the electron from the second Bohr orbit to the
atoms of the gas make transition to a higher energy
third Bohr orbit is 47.2 eV. The atomic number of this
level by the absorbing monochromatic light of photon
nucleus is ________________.
energy 2.7eV. Subsequently, the atom emit radiation of
only six different photon energies. Some of the emitted
photons have energy 2.7eV. Some have energy more Q.26 A single electron orbits a stationary nucleus of
and some have less than 2.7eV. charge Ze where Z is a constant and e is the electronic
charge. It requires 47.2eV to excite the electron from
(i) Find the principle quantum of the initially excited
the 2nd Bohr orbit to 3rd Bohr orbit. Find
level B.
(i) The value of Z
(ii) Find the ionization energy for the gas atoms.
(ii) Energy required to excite the electron from third to
(iii) Find the maximum and the minimum energies of
the fourth orbit
the emitted photons.
(iii) The wavelength of radiation required to remove the
electron from the first orbit to
Q.20 A hydrogen atom in ground state absorbs a
photon of ultraviolet radiation of wavelength 50nm. (iv) Infinity the Kinetic energy, potential energy and
Assuming that the entire photon energy is taken up by angular momentum in the first Bohr
the electron, with what kinetic energy will the electron
(v) Orbit the radius of the first Bohr orbit.
be ejected?

Q.27 A hydrogen like atom (atomic number Z) is in


Q.21 An electron joins a helium nucleus to form a He+
higher excited state of quantum number n. This excited
ion in ground state. The wavelength of the photon
atom can make a transition to the first excited state by
emitted in this process if the electron is assumed to
successive emitting two photons of energy 22.95eV and
have had no kinetic energy when it combines with
5.15eV respectively. Alternatively, the atom from the
nucleus is _________ nm.
same excited state can make transition to the second
excited state by successive emitting two photons of
Q.22 Three energy levels of an atom are shown in the energies 2.4eV and 8.7eV respectively. Find the value
figure. The wavelength corresponding to three possible of n and Z.
P hysi cs | 24.59

Q.28 Find the binding energy of an electron in the Q.6 An electrons collides with a fixed hydrogen atom
ground state of a hydrogen like atom in whose in its ground state. Hydrogen atom gets excited
spectrum the third of the corresponding Balmer series and the colliding electron loses all its kinetic energy.
is equal to 108.5 nm. Consequently the hydrogen atom may emit a photon
corresponding to the largest wavelength of the Balmer
Q.29 Which level of the doubly ionized lithium has series. The min. K.E. of colliding electron will be
the same energy as the ground state energy of the (A) 10.2eV (B) 1.9 eV
hydrogen atom? Find the ratio of the two radii of
(C) 12.1 eV (D) 13.6 eV
corresponding orbits.

Q.7 A neutron collides head on with a stationary


Q.30 A 20 KeV energy electron is brought to rest
hydrogen atom in ground state
in an X-ray tube, by undergoing two successive
bremsstrahlung events, thus emitting two photons. (A) If kinetic energy of the neutron is less than 13.6 eV,
The wavelength of the second photon is 130 x 10−12 collision must be elastic.
m greater than the wavelength of the first emitted
(B) If kinetic energy of the neutron is less than 13.6 eV,
photon. Calculate the wavelength of the two photons.
collision may be inelastic.
(C) Inelastic collision takes place when initial kinetic
Exercise 2 energy of neutron is greater than 13.6 eV.

Single Correct Choice Type (D) Perfectly inelastic collision cannot take place.

Q.1 10−3 W of 5000 Å light is directed on a photoelectric Q.8 An electron in hydrogen atom first jumps from
cell. If the current in the cell is 0.16 µA, the percentage second excited state to first excited state and then,
of incident photons which produce photoelectrons, is from first excited state to ground state. Let the ratio of
wavelength, momentum and energy of photons in the
(A) 0.4% (B) 0.04% (C) 20% (D) 10% two cases be x, y and z, then select the wrong answer(s):
1 9 5 5
Q.2 Photons with energy 5eV are incident on a cathode (A) z = (B) x = (C) y = (D) z =
x 4 27 27
C, on a photoelectric cell. The maximum energy of the
emitted photoelectrons is 2 eV. When photons of energy
6 eV are incident on C, no photoelectrons will reach the Multiple Correct Choice Type
anode A if the stopping potential of A relative to C is
Q.9 In photoelectric effect, stopping potential depends on
(A) 3V (B) –3V (C) –1 V (D) 4 V
(A) Frequency of the incident light
Q.3 In a hydrogen atom, the binding energy of the (B) Intensity of the incident light by varying source
electron of the nth state is En , then the frequency of (C) Emitter’s properties
revolution of the electron in the nth orbit is:
(D) Frequency and intensity of the incident light
2En 2Enn En Enn
(A) (B) (C) (D)
nh h nh h
Q.10 Two electrons are moving with the same speed v.
One electron enters a region of uniform electric field
Q.4 Difference between nth and (n+1)th Bohr’s radius
while the other enters a region of uniform magnetic
of ‘H’ atom is equal to it’s (n – 1)th Bohr’s radius. The
field, then after sometime if the de-Broglie wavelengths
value of n is:
of the two are λ1 and λ2 , then:
(A) 1 (B) 2 (C) 3 (D) 4
(A) λ1 =λ2 (B) λ1 > λ2
(C) λ1 < λ2 (D) λ1 > λ2 or λ1 < λ2
Q.5 Electron in a hydrogen atom is replaced by an
identically charged particle muon with mass 207 times
that of electron. Now the radius of K shell will be Q.11 A neutron collides head-on with a stationary
hydrogen atom in ground state. Which of the following
(A) 2.56 × 10−3 Å (B) 109.7 Å
statements are correct (Assume that the hydrogen
(C) 1.21 × 10−3 Å (D) 22174.4 Å atom and neutron has same mass):
2 4 . 6 0 | Modern Physics

(A) If kinetic energy of the neutron is less than 20.4 eV (A) A minimum wavelength
collision must be elastic.
(B) A continuous spectrum
(B) If kinetic energy of the neutron is less than 20.4 eV
(C) Some discrete comparatively prominent wavelength
collision may be inelastic.
(D) Uniform density over the whole spectrum
(C) Inelastic collision may be take place only when initial
kinetic energy of neutron is greater than 20.4 eV.
Assertion Reasoning Type
(D) Perfectly inelastic collision cannot take place.

Q.16 Statement-I: Figure shows Vs


Q.12 A free hydrogen atom in ground state is at rest. A graph of stopping potential
neutron of kinetic energy ‘K’ collides with the hydrogen and frequency of incident light
atom. After collision hydrogen atom emits two photons in photoelectric effect. For v
in succession one of which has energy 2.55 eV. (Assume values of frequency less than (0,0) v0
that the hydrogen atom and neutron has same mass) threshold frequency ( v0 )
(A) Minimum value of ‘K’ is 25.5 eV. stopping potential is negative.
(B) Minimum value of ‘K’ is 12.75 eV. Statement-II: Lower the value of frequency of incident
light (for v > v 0 ) the lower is the maxima of kinetic
(C) The other photon has energy 10.2eV.
energy of emitted photoelectrons.
(D) The upper energy level is of excitation energy
(A) Statement-I is true, statement-II is true and
12.75 eV.
statement-II is correct explanation for statement-I.

Q.13 A particular hydrogen like atom has its ground (B) Statement-I is true, statement-II is NOT the correct
state binding energy 122.4eV. It is in ground state. explanation for statement-I.
Then: (C) Statement-I is true, statement-II is false.
(A) Its atomic number is 3 (D) Statement-I is false, statement-II is true.
(B) An electron of 90eV can excite it.
(C) An electron of kinetic energy nearly 91.8eV can be Q.17 Statement-I: Two photons having equal
almost brought to rest by this atom. wavelengths have equal linear momenta.

(D) An electron of kinetic energy 2.6eV may emerge Statement-II: When light shows its photons character,
from the atom when electron of kinetic energy 125eV h
each photon has a linear momentum λ = .
collides with this atom. p
(A) Statement-I is true, statement-II is true and
statement-II is correct explanation for statement-I.
Q.14 A beam of ultraviolet light of all wavelengths
pass through hydrogen gas at room temperature, in (B) Statement-I is true, statement-II is NOT the correct
the x-direction. Assuming all photons emitted due explanation for statement-I.
to electron transition inside the gas emerge in the (C) Statement-I is true, statement-II is false.
y-direction. Let A and B denote the lights emerging
from the gas in the x and y directions respectively. (D) Statement-I is false, statement-II is true.

(A) Some of the incident wavelengths will be absent in A.


Q.18 Statement-I: In the process of photoelectric
(B) Only those wavelengths will be present in B which emission, all the emitted photoelectrons have same K.E.
are absent in A.
Statement-II: According to Einstein’s photoelectric
(C) B will contain some visible light. equation
(D) B will contain some infrared light. KEmax = hv − φ .
(A) Statement-I is True, statement-II is True, statement-II
Q.15 X-rays are produced by accelerating electrons is a correct explanation for statement-I.
across a given potential difference to strike a meta
target of high atomic number. If the electrons have (B) Statement-I is True, statement-II is True, statement-II
same speed when they strike the target, the X-ray is NOT a correct explanation for statement-I.
spectrum will exhibit
P hysi cs | 24.61

(C) Statement-I is True, statement-II is False Previous Years’ Questions


(D) Statement-I is False, statement-II is True
Q.1 A single electron orbits around a stationary nucleus
of charge +Ze where Z is a constant and e is the
Q.19 Statement-I: Work function of aluminum is 4.2
magnitude of the electronic charge. It requires 47.2 eV
eV. If two photons each of energy 2.5 eV strikes on a
to excite the electron from the second Bohr orbit to the
piece of aluminum, the photoelectric emission does
third Bohr orbit. (1981)
not occur.
Find:
Statement-II: In photoelectric effect a single photon
interacts with a single electron and electron is emitted (a) The value of Z.
only if energy of each incident photon is greater than
(b) The energy required to excite the electron from the
the work function.
third to the fourth Bohr orbit.
(A) Statement-I is True, statement-II is True, statement-II
(c) The wavelength of the electromagnetic radiation
is a correct explanation for statement-I.
required to remove the electron from the first Bohr
(B) Statement-I is True, statement-II is True, statement-II orbit to infinity.
is NOT a correct explanation for statement-I.
(d) The kinetic energy, potential energy and the angular
(C) Statement-I is True, statement-II is False momentum of the electron in the first Bohr orbit.
(D) Statement-I is False, statement-II is True (e) The radius of the first Bohr orbit.
(The ionization energy of hydrogen atom = 13.6 eV, Bohr
Q.20 Statement-I: An electron and a proton are radius = 5.3 × 10−11 m , velocity of light = 3 × 108 m / s .
accelerated through the same potential difference. The Planck’s constant = 6.6 × 10−34 J s ).
de-Broglie wavelength associated with the electron is
longer.
Q.2 Hydrogen atom in its ground state is excited by
Statement-II: de-Broglie wavelength associated means of monochromatic radiation of wavelength
h 975° Å. How many different lines are possible in the
with a moving particle is λ = where, p is the linear resulting spectrum? Calculate the longest wavelength
p
momentum and both have same K.E. amongst them. You may assume the ionization energy
for hydrogen atom as 13.6 eV. (1982)
(A) Statement-I is True, statement-II is True, statement-II
is a correct explanation for statement-I.
Q.3 How many electrons, protons and neutrons are
(B) Statement-I is True, statement-II is True, statement-II there in a nucleus of atomic number 11 and mass
is NOT a correct explanation for statement-I. number 24? (1982)
(C) Statement-I is True, statement-II is False (a) Number of electrons =
(D) Statement-I is False, statement-II is True (b) Number of protons =
(c) Number of neutrons =
Q.21 Statement-I: In a laboratory experiment, on
emission from atomic hydrogen in a discharge tube,
Q.4 A uranium nucleus (atomic number 92, mass
only a small number of lines are observed whereas
number 238) emits an alpha particle and the resulting
a large number of lines are present in the hydrogen
nucleus emits β -particle. What are the atomic number
spectrum of a star.
and mass number of the final nucleus? (1982)
Statement-II: The temperature of discharge tube is
(a) Atomic number =
much smaller than that of the star.
(b) Mass number =
(A) Statement-I is True, statement-II is True, statement-II
is a correct explanation for statement-I.
Q.5 Ultraviolet light of wavelengths 800 Å and 700 Å
(B) Statement-I is True, statement-II is True, statement-II
when allowed to fall on hydrogen atoms in their ground
is NOT a correct explanation for statement-I.
state is found to liberate electrons with kinetic energy
(C) Statement-I is True, statement-II is False 1.8 eV and 4.0 eV respectively. Find the value of Planck’s
constant.  (1983)
(D) Statement-I is False, statement-II is True
2 4 . 6 2 | Modern Physics

Q.6 the ionization energy of a hydrogen like Bohr atom Q.11 The wavelength of light emitted in the visible
is 4 Rrydberg. region by He+ ions after collisions with H-atoms is
 (2008)
(a) What is the wavelength of the radiation emitted
when the electron jumps from the first excited state to (A) 6.5 × 10−7 m (B) 5.6 × 10−7 m
the ground state?  (1984)
(C) 4.8 × 10−7 m (D) 4.0 × 10−7 m
(b) What is the radius of the first orbit for this atom?
1 rH
Now, as r ∝ Radius of first orbit of this atom, r1 = 1 Q.12 The ratio of the kinetic energy of the n = 2 electron
Z Z for the H atom to that of He+ ion is  (2008)
0.529
= = 0.2645 Å
2 1 1
(A) (B) (C) 1 (D) 2
4 2
Q.7 A doubly ionized lithium atom is hydrogen-like
with atomic number 3. Q.13 Some laws/processes are given in column I. Match
(a) Find the wavelength of the radiation required to these with the physical phenomena given in column II.
excite the electron in Li2+ from the first to the third  (2006)
Bohr orbit. (Ionization energy of the hydrogen atom
equals 13.6 eV.)
Column I Column II
(b) How many spectral lines are observed in the emission (A) Nuclear fusion (p) Converts some
spectrum of the above excited system? (1985) matter into energy.
(B) Nuclear fission (q) Generally possible
Q.8 There is a stream of neutrons with a kinetic energy for nuclei with low
of 0.0327 eV. If the half-life of neutrons is 700s, what atomic number
fraction of neutrons will decay before they travel a
(C) (r) Generally possible
distance of 10 m? (1986) β -decay
for nuclei with higher
atomic number
Q.9 A particle of charge equal to that of an electron (D) Exothermic (s) Essentially proceeds
–e, and mass 208 times of the mass of the electron nuclear reaction by weak nuclear
(called a mu-meson) moves in a circular orbit around a forces
nucleus of charge +3e. (Take the mass of the nucleus to
be infinite). Assuming that the Bohr model of the atom
is applicable to this system. (1988) Q.14 The threshold wavelength for photoelectric
emission from a material is 5200 Å. Photoelectrons
(a) Derive an expression for the radius of the nth Bohr will be emitted when this material is illuminated with
orbit. monochromatic radiation from a  (1982)
(b) Find the value of n for which the radius of the orbit (A) 50W infrared lamp (B) 1 W infrared lamp
is approximately the same as that of the first Bohr orbit
(C) 50W ultraviolet lamp (D) 1W ultraviolet lamp
for the hydrogen atom.
(c) Find the wavelength of the radiation emitted when
Q.15 The allowed energy for the particle for a particular
the mu-meson jumps from the third orbit to the first
value of n is proportional to  (2009)
orbit. (Rydberg’s constant = 1.097 × 107 m−1 )
(A) a-2 (B) a-3/2 (C) a-1 (D) a2
+
Paragraph 1: (Q.10-Q.12) In a mixture of H − He gas
( He+ is single ionized He atom), H atoms and He+ Q.16 If the mass of the particle is m = 1.0 ×10−30 kg and
ions excited to their respective first excited states. a = 6.6 nm, the energy of the particle in its ground state
Subsequently, H atoms transfer their total excitation is closest to  (2009)
energy of He+ ions (by collisions). Assume that the
(A) 0.8 MeV (B) 8 MeV (C) 80 MeV (D) 800 MeV
Bohr model of atom is exactly valid.

Q.17 The speed of the particle that can take discrete


Q.10 The quantum number n of the state finally
values is proportional to (2009)
populated in He+ ions is  (2008)
(A) n−3/2 (B) n-1 (C) n1/2 (D) n
(A) 2 (B) 3 (C) 4 (D) 4
(A) (B)
P hysi cs | 24.63
 
Q.18 An α -particle and a proton are accelerated from V0 V0
rest by a potential difference of 100V. After this, their
de-Broglie wavelength are λ α and λp respectively. The
(C) (D)
λp
ratio , to the nearest integer, is : (2010)
λα 1/ 1/

Q.19 The wavelength of the first spectral line in the



Balmer series of hydrogen atom is 6561 A . The Q.25 In a historical experiment to determine Planck’s
wavelength of the second spectral line in the Balmer constant, a metal surface was irradiated with light of
series of singly-ionized helium atom is  (2010) different wavelengths. The emitted photoelectron
    energies were measured by applying a stopping
(A) 1215 A (B) 1640 A (C) 2430 A (D) 4687 A
potential. The relevant data for the wavelength ( λ ) of
incident light and the corresponding stopping potential
Q.20 A proton is fired from very far away towards a (V0) are given below:  (2016)
nucleus with charge Q = 120 e, where e is the electronic
charge. It makes a closest approach of 10 fm to the V0 (Volt)
λ(µ m)
nucleus. The de Broglie wavelength (in units of fm) of
the proton at its start is: (take the proton mass, mp =
0.3 2.0
1
(5/3) × 10-27 kg; h/e = 4.2 × 10-15 J.s / C; = 9 × 109
4πε0 0.4 1.0
m/F; 1 fm = 10-15)  (2012) 0.5 0.4

Q.21 A pulse of light of duration 100 ns is absorbed Given that c = 3 × 108 m s-1 and e = 1.6 × 10-19 C,
completely by a small object initially at rest. Power of Planck’s constant (in units of J s) found from such an
the pulse is 30 mW and the speed of light is 3 × 108 experiment is
m/s. The final momentum of the object is (2013)
(A) 6.0 × 10-34 (B) 6.4 × 10-34
(A) 0.3 × 10 -17
kg ms -1
(B) 1.0 × 10
-17
kg ms -1
(C) 6.6 × 10-34 (D) 6.8 × 10-34
(C) 3.0 × 10-17 kg ms-1 (D) 9.0 × 10-17 kg ms-1
Q.26 Highly excited states for hydrogen-like atoms
Q.22 The work functions of Silver and Sodium are 4.6 (also called Rydberg states) with nuclear charge Ze are
and 2.3 eV, respectively. The ratio of the slope of the defined by their principal quantum number n, where n
stopping potential versus frequency plot for Silver to >> 1. Which of the following statement(s) is(are) true?
that of Sodium is  (2013)  (2016)
(A) Relative change in the radii of two consecutive
Q.23 Consider a hydrogen atom with its electron in the orbitals does not depend on Z
nth orbital. An electromagnetic radiation of wavelength
(B) Relative change in the radii of two consecutive
90 nm is used to ionize the atom. If the kinetic energy
orbitals varies as 1/n
of the ejected electron is 10.4 eV, then the value of n is
(hc = 1242 eV nm) (2015) (C) Relative change in the energy of two consecutive
orbitals varies as 1/n3
Q.24 For photo-electric effect with incident photon (D) Relative change in the angular momenta of two
wavelength λ , the stopping potential is Vo. Identify the consecutive orbitals varies as 1/n
correct variation(s) of V0 with λ and 1/ λ . (2015)
Q.27 A hydrogen atom in its ground state is irradiated
V0 V0 by light of wavelength 970A. Taking hc/e = 1.237 × 10-6
eVm and the ground state energy of hydrogen atom as
(A) (B) –13.6 eV, the number of lines present in the emission
spectrum is (2016)
 
Q.28 A glass tube of uniform internal radius (r) has
V0 V0
a valve separating the two identical ends. Initially,

(C) (D)
2 4 . 6 4 | Modern Physics

the valve is in a tightly closed position. End 1 has a


1  h2  1  h2 
hemispherical soap bubble of radius r. End 2 has sub- (A)   (B)  
hemispherical soap bubble as shown in figure. Just n2  8π2 I  n  8π2 I 
after opening the valve,  (2008)
 h2   h2 
Figure: (C) n   (D) n2  
 8π2 I   8π2 I 
   

Q.31 It is found that the excitation frequency from


ground to the first excited state of rotation for the CO
2 1 4
molecule is close to × 1011 Hz . Then the moment of
(A) Air from end 1 flows towards end 2. No change in π
the volume of the soap bubbles inertia of CO molecule about its centre of mass is close

(B) Air from end 1 flows towards end 2. Volume of the to (Take h = 2π × 10−34 Js)
soap bubble at end 1 decreases (A) 2.76 ×10−46 kg m2 (B) 1.87 ×10−46 kg m2
(C) No changes occurs (C) 4. 67 ×10−47 kg m2 (D) 1.17 ×10−47 kg m2
(D) Air from end 2 flows towards end 1. volume of the
soap bubble at end 1 increases Q.32 In a CO molecule, the distance between C (mass =
12 a.m.u) and O (mass = 16 a.m.u.),
5
Q.29 Photoelectric effect experiments are performed where 1 a.m.u. = × 10−27 kg, is close to
3
using three different metal plates p, q and r having
work functions φp = 2.0 eV, φq = 2.5 eV and φr = 3.0 eV, (A) 2.4 ×10−10 m (B) 1.9 ×10−10 m
respectively. A light beam containing wavelengths of (C) 1.3 ×10−10 m (D) 4.4 ×10−11 m
550 nm, 450 nm and 350 nm with equal intensities
illuminates each of the plates. The correct I-V graph for
Q.33 A silver sphere of radius 1 cm and work function
the experiment is (Take hc = 1240 eV nm) (2009)
4.7 eV is suspended from an insulating thread in free
l l space. It is under continuous illumination of 200 nm
P
q wavelength light. As photoelectrons are emitted, the
r sphere gets charged and acquires a potential. The
(A) (B) Pq maximum number of photoelectrons emitted from the
r sphere is A ×10z (where 1 < A < 10). The value of ‘Z’ is
V V  (2011)
l r l
q Q.34 Two bodies, each of mass M, are kept fixed with a
(C) P (D) rq separation 2L. A particle of mass m is projected from the
midpoint of the line joining their centres, perpendicular
P to the line. The g ravitational constant is G. The correct
V V statement(s) is (are) (2013)

Paragraph for questions 30 to 32 (2010) (A) The minimum initial velocity of the mass m to

The key feature of Bohr’s theory of spectrum of hydrogen escape the gravitational field of the two bodies is 4
GM
atom is the quantization of angular momentum when L
an electron is revolving around a proton. We will extend (B) The minimum initial velocity of the mass m to escape
this to a general rotational motion to find quantized
the gravitational field of the two bodies is GM
rotational energy of a diatomic molecule assuming it to 2
L
be rigid. The rule to be applied is Bohr’s quantization
condition. (C) The minimum initial velocity of the mass m to escape
2GM
the gravitational field of the two bodies is
Q.30 A diatomic molecule has moment of inertia I. By L
Bohr’s quantization condition its rotational energy in
the nth level (n = 0 is not allowed) is (D) The energy of the mass m remains constant.
P hysi cs | 24.65

Q.35 A metal surface is illuminated by light of two


different wavelengths 248 nm and 310 nm. The Light
maximum speeds of the photoelectrons corresponding
to these wavelengths are u1 and u2, respectively. If
the ratio u1 : u2 = 2 : 1and hc = 1240 eV nm, the work
function of the metal is nearly  (2014) Electrons

(A) 3.7 eV (B) 3.2 eV


(C) 2.8 eV (D) 2.5 eV
v
- +
Q.36 Light of wavelength λph falls on a cathode plate
inside a vacuum tube as shown in the figure. The work (A) For large potential difference (V >> φ /e), λe is
function of the cathode surface is φ and the anode is a approximately halved if V is made four times
wire mesh of conducting material kept at a distance d
from the cathode. A potential difference V is maintained (B) λe decreases with increase in φ and λph
between the electrodes. If the minimum de Broglie (C) λe increases at the same rate as λph for λph < hc/ φ
wavelength of the electrons passing through the anode
(D) λe is approximately halved, if d is doubled
is λe , which of the following statement(s) is(are) true?
 (2016)

PlancEssential Questions
JEE Main/Boards JEE Advanced/Boards
Exercise 1 Exercise 1
Q.10 Q.17 Q.23 Q.4 Q.8 Q.10
Q.27 Q.28 Q.19 Q.22 Q.30

Exercise 2 Exercise 2
Q.2 Q.4 Q.7 Q.1 Q.6 Q.8
Q.12 Q.13 Q.14
Q.16

Previous Years’ Questions


Q.10
2 4 . 6 6 | Modern Physics

Answer Key

JEE Main/Boards
Exercise 1
λ
Q.1 (iii)1.49 × 10−19 J Q.4 Q.8 Line B represent a particle of
smaller mass. 2

Q.9 (i) Metal Q, (ii) Metal P, (iii) Stopping potential remains unchanged.
Q.10. 1.5 eV Q.11 1 : 2 2 Q.17 2.46 × 1015 Hz
Q.21 (i) –1.7 eV (ii) 972.54 Å Q.23 +13.6 eV, –27.2 eV Q.24 28.8 × 10−15 m
Q.27 7.5 × 1014 Hz, 3.094 eV Q.28 4137.5 Å

Exercise 2

Single Correct Choice Type


Q.1 C Q.2 D Q.3 C Q.4 A Q.5 A Q.6 C
Q.7 B Q.8 B Q.9 C Q.10 A Q.11 D Q.12 C
Q.13 B Q.14 C Q.15 D Q.16 A Q.17 A Q.18 C
Q.19 C Q.20 A Q.21 B Q.22 B Q.23 A Q.24 B
Q.25 A Q.26 B Q.27 A Q.28 B Q.29 D

Previous Years’ Questions


Q.1 B Q.2 C Q.3 A Q.4 D Q.5 C Q.6 D
Q.7 C Q.8 B Q.9 30×103V, 30×103eV Q.10 500 dps, 125 dps
Q.11 Frequency Q.12 8, 6 Q.13 B Q.14 B Q.15 D Q.16 D
Q.17 B Q.18 D Q.19 A Q.20 C Q.21 C Q.22 D
Q.23 D Q.24 D Q.25 B Q.26 C Q.27 C Q.28 D

JEE Advanced/Boards
Exercise 1
15
Q.1 (a) 0.6 V, (b) 2.0 mA Q.2 5.76 × 10−11 A Q.3 V
8
5 1019
Q.4 , Q.5 2eV, 6.53 × 10−34 J s Q.6 4.8×1016, 4.0 cm
16 8 π

Q.7 2 eV, 0.754 V Q.8 1.1 × 1012
5 5
Q.9 (i) photon/s, (ii) electrons/s Q.10 (a) 105 s−1 ; (b) 286.76 ; (d) 111.1s
16 1600
P hysi cs | 24.67

h
Q.11 (a) 2.25 eV, (b) 4.2 eV, (c) 2.0 eV, 0.5 eV Q.12 −
eEt2
Q.13 487 nm Q.14 4.26 m/s, 13.2 eV Q.15 7 : 36

Q.16 λdeutron = 8.6 pm


λneutron = Q.17 KE ≅ 148.4 eV, dleast = 0.5 Å Q.18 3.1 × 106 m / s

Q.19 (i) 2 ; (ii) 23.04 × 10−19 J ; (iii) 4 → 1, 4 → 3 Q.20 11.24 eV
λ1 λ2 18
Q.21 22.8 nm Q.22 Q.23
λ1 + λ2 5R

Q.24 1.257 × 10−23 Am2 Q.25 5

Q.26 (i) Z = 5, (ii) E= 16.5 eV, (iii) λ=36.4 A, (iv) K.E = 340 eV, P.E = –680 eV, (v) Radius r = 0.1058 Å

Q.27 Z = 3, n = 7 Q.28 54.4 eV Q.29 n = 3, 3 : 1


 
Q.30 λ1 =0.871 A and λ2 =2.17 A

Exercise 2

Single Correct Choice Type

Q.1 B Q.2 B Q.3 A Q.4 D Q.5 A Q.6 C

Q.7 A Q.8 B

Multiple Correct Choice Type

Q.9 A, C Q.10 A, D Q.11 A,C Q.12 A, C, D Q.13 A, C, D

Q.14 A, C, D Q.15 A, B, C

Assertion Reasoning Type

Q.16 D Q.17 C Q.18 D Q.19 D Q.20 A Q.21 A

Previous Years’ Questions

kgm2
Q.1 (a) 5 (b) 16.53 eV (c) 36.4 Å (d) 340 eV, –680 eV, –340 eV, 1.05 × 10−34 (e) 1.06 × 10−11 m
s

Q.2 Six, 1.875 µm Q.3 0, 11, 13 Q.4 (a) 91 (b) 234

Q.5 6.6 × 10−34 J s Q.6 (a) 300 Å (b) 0.2645 Å Q.7 (a) 113.74 Å (b) 3

n2h2 ε0 
Q.8 3.96 × 10−6 Q.9 (a) rn = (b) n = 25 (c) 0.546 A
624 πmee2

Q.10 C Q.11 C Q.12 A Q.13 A → p, q; B → p, r; C → p, s; D → q

Q.14 C,D Q.15 A Q.16 B Q.17 D Q.18 3 Q.19 A

Q.20 7 fm Q.21 B Q.22 1 Q.23 2 Q.24 A Q.25 B

Q.26 A, B, D Q.27 6 Q.28 B Q.29 A Q.30 D Q.31 B

Q.32 C Q.33 7 Q.34 B Q.35 A Q.36 A


2 4 . 6 8 | Modern Physics

Solutions

JEE Main/Boards

Exercise 1 l2 =
h
=
h
=
λ1
=
λ
2mKE2 2me × 4v 2 2

Sol 1: (i) Work function – It is the minimum energy


of incident photon below which no ejection of Sol 5: For graph, refer theory.
photoelectron from a metal surface will take place is
If the distance is reduced, intensity of light will increase
known as work function or thresholds energy for that
and number of electrons will increase and current will
metal
increase. Stopping potential will not be affected by
φ = hV0 distance.
(ii) Threshold frequency – it is the minimum frequency
of incident photon below which no ejection of Sol 6: hν1 = W1 + KE
photoelectron from a metal surface will take place is hν2 = W2 + KE
known as threshold frequency for that metal.
W1 > W2
(iii) Stopping potential – The negative potential (V0)
applied to the anode at which the current gets reduced ⇒ ν1 > ν2
to zero is called stopping potential.
KEmax = E – φ Sol 7: Refer theory

12400 h
E= = 4.13 eV Sol 8: λ =
3000 2meV

KEmax = 4.13 – 3.2 = 0.93 eV = 1.49 × 10–19 J h


Slope =
2me
Sol 2: KE = eV Slope of B > Slope of A

h h h ⇒ mB < m A
ld = =. .=
p 2mKE 2meV
Sol 9: (i) Metal P has greater threshold wavelength
6.6 ×10−34 12.27 because P has lower threshold frequency.
=. = Å
2 × 9.1 ×10−31 ×1.67 ×10−19 × V V (ii) KEmax = hν – hν0
For Davisson and Germer’s experiment, refer theory. Metal P emits electrons with less kinetic energy as P has
less threshold frequency.
hc
Sol 3: φA = 2 eV = (iii) If distance is doubled, there is no change in stopping
λA potential.
hc
φB = 5 eV =
λB Sol 10: KEmax = eV = 1.5 eV
λA > λB
λα h h
Sol 11: = ÷
Sol 4: K.E. = eV λp 2mα (KE)α 2mp (KE)p
h h
l1 = = =λ
2mKE1 2meV mp (KE)p 1 eV 1
= = · =
mα (KE)α 4 2eV 2 2
P hysi cs | 24.69

Sol 12: hν = hn0 + KE Sol 20: Refer theory


Refer theory
Sol 21: (i) PE = 2 T.E.
Sol 13: Refer theory −13.6 × 2
PE = = – 0.85 × 2 = – 1.7 eV
16
Sol 14: Refer theory for the graph
1 1  15
hν1 = hν0 + eV; hν2 = hν0 + eV (ii) E = 13.6  −  = ×13.6 eV
2 2
1 4  16
Stopping potential V is higher for ν1 frequency by the
above equations. E = 12.75 eV
12400
h h λ= = 972.54 Å
Sol 15: λp = ; λe = 12.75
2mp (KE) 2me (KE)
mp > me Sol 22: Refer theory

lp < le
Sol 23: T.E. = – 13.6 eV

Sol 16: (i) Cut-off voltage is the negative potential KE = |T.E.| = 13.6 eV
applied to the anode at which the current gets reduced P.E = 2T.E = – 13.6 × 2 = –27.2 eV
to zero.
K × (2e)(80e)
Refer theory for second part of question. Sol 24: 8 × 106 =
r

Sol 17: Refer theory 9 ×109 ×160 ×1.6 ×10−19


r= = 28.8 × 10–15 m
6
8 ×10
E = – 3.4 eV – (– 13.6 eV)
If kinetic energy is doubled then closest distance will
E = 10.2 eV
become half of the original.
hν = 10.2 × 10–19 × 1.6 J
h
ν = 2.46 × 1015 Hz Sol 25: λ =
2m(KE)
2
mv h h
Sol 18: eE = . . =λe =
r
2me (KE)e 2mp (KE)p
V
me(KE)e = mp(KE)p
eE
m p > me
⇒ (KE)e > (KE)p

Sol 26: Kinetic energy = hν – hn0

e v2 It is independent of the intensity of light.


=
m rE c 3 ×108
Sol 27: ν = = = 0.75 × 1015 = 7.5 × 1014 Hz
λ −7
Sol 19: Magnetic moment of a charged particle moving 4 ×10
in a circle is given by Energy = hν

µ = IA = 6.6 × 10–34 × 7.5 × 1014 = 49.5 × 10–20 J= 3.094 eV

e
I = charge flowing per sec
= ω Sol 28: Longest wavelength ⇒ minimum frequency

photon = n0
eω evr
=
µ × πr 2 = 12400
2π 2 = 4137.5 Å
3
2 4 . 7 0 | Modern Physics

Exercise 2 [As energy is greater than 12.1 eV so 1 state has to be


ground state]
Single Correct Choice Type 1 1
12.1 = 13.6  − 
 12 n22 
Sol 1: (C) Intensity of both bulb is same i.e. I1 = I2
12.1 1
Er = Energy of red colour photon = 1−
13.6 n22
Eb = Energy of blue colour photon
1 12.1 1.5
Eb > Er 1−
= =
n22 13.6 13.6
I1
nr = 13.6
Er n22 =
1.5
I2
nb = n2 = 3
Eb
⇒ nb < nr
Sol 5: (A) E1 = 2φ
E2 = 5φ
hc
Sol 2: (D) = φ +K  … (i)
λ E1 = φ + KE1
4hc 2 φ – φ = KE1
= φ + K2  … (ii)
3λ 1
By (i) and (ii) KE1 = φ = mv12
2
4 E2 = φ + KE2
[φ + K] = φ + K2
3
5 φ – φ = KE2
4φ 4k
+ = φ + K2 1
3 3 KE2 = 4φ = mv22
2
4K φ
K=
2 +
3 3 2φ
v1 m =1
=
Sol 3: (C) hν = φ + KE v2 8φ 2
m
KE = eV
V = stopping potential
hc
hν = φ + eV  … (i) Sol 6: (C) = φ + KE1
λ1
h2ν = φ + eV2  … (ii)
KE1 = eV1
By (i) and (ii)
hc  hc  1
2(φ + eV) = φ + eV2 = φ + eV1 ⇒ V
=1  − φ 
λ1  λ1 e
φ + 2eV = eV2
hc  hc  1
φ = φ + eV2 ⇒ V
=2  − φ 
V2 = 2V +
e
λ2  λ2 e
hc  hc  1
= φ + eV3 ⇒ V =3  − φ 
Sol 4: (A) E = φ + KE λ3 λ e
 3 
KE = eV, where V is stopping potential V1 V2 and V3 are in A.P.
E = 1.7 eV + 10.4 eV ⇒ V1 + V3 = 2V2
E = 12.1 eV  hc  1  hc  1  hc  1
1 1 2  − φ 
12.1 = 13.6 × (1)2  −   − φ  +  − =
φ

2 2  λ1  e  λ3 e  λ2 e
 n1 n2 
P hysi cs | 24.71

hc hc 2hc 2hc
+ = = φ + K2
λ1 λ3 λ2 λ1

1 1 2 By (i) and (ii)


+ =
λ1 λ3 λ2 2(φ + K1) = φ + K2 ⇒ φ + 2K1 = K2

Sol 7: (B) φ = 4.5 eV K2 φ


K1 = −
2 2
Wavelength of light = 2000Å
12400 124 Sol 13: (B) Distance between two successive maxima in
Energy of photon = = = 6.2eV
2000 20 λD
Young’s double slit experiment is
K.E. of emitted electron = hν – φ d
= 6.2 – 4.5 = 1.7 eV Distance will decrease if D will decreases.
As electrons are further accelerated by 2V
Sol 14: (C)
so final kinetic energy = 1.7 eV + 2eV
2.18 ×10−18
Energy required = = 2.42 × 10–19 J
= 3.7 eV 9

Sol 8: (B) hν = hn0 + KE Sol 15: (D) In some Hydrogen like atom
Kinetic energy depends only on the energy of incident  13.6 13.6  2
photon. Number of emitted electrons ∝ intensity of light. E4n – E2n =  − + Z
2 2 
 (4n) (2n) 
h h
Sol 9: (C) λp= = 13.6 Z 2  1 
pp 2mp (KE)p  
=  − + 1
4n2  4 
h h
λe= = 3 ×13.6 Z 2
pe 2me (KE)e  =
16n2
mp > me
As proton and electron both are accelerated by same  13.6 13.6  2
E2n – En =  − + Z
potential difference so KEp = KEe  (en)
2
n2 
lp < le
13.6 z 2  1  3 ×13.6 Z 2
  =  − + 1  =
n2  4  4n2
Sol 10: (A) Initial KE is 100 eV
After accelerating through potential difference of 50 v E4n − E2n 1
Ratio = =
final KE is 150 eV. E2n − En 4

150 150 Ratio is independent of Z and n.


λd = = =1 Å
V 150
Sol 16: (A) n’ = n + 1
h h
λ = ; p = 1012 h
Sol 11: (D) = n’ = 3
p 0.01 × 10−10
(n + 1 − 3) ( (n + 1 − 3) + 1 )
hc No. of lines =
Sol 12: (C) = φ + K1  … (i) 2
λ1
hc (n − 2)(n − 1)
= φ + K2  =
… (ii) = 10
λ2 2

λ1 (n – 2) (n – 1) = 20,
l2 =
2 n=6
2 4 . 7 2 | Modern Physics

Sol 17: (A) Ten different wavelengths are emitted so So P.E. will decrease
n(n − 1) −13.6 Z 2
= 10 ⇒ n (n – 1) = 20 T.E. =
2
n2
⇒n=5
T.E. will decrease with decrease in n.
1 1
Energy of incident radiation is 13.6  −  nh
 12 52  Sol 21: (B) Angular momentum = = mvr

24 1
= ×13.6 = 13.056 eV f ∝
25 n3
12400
λ= = 949.75 Å r ∝ n2
13.056
1
f rL ∝ × n2 × n =1 ⇒ independent of n.
3
Sol 18: (C) Five dark line corresponds to transitions so n
highest state of electron is n = 6
n(n − 1) 0.529n2 0.529 × 4
So no of lines in emission spectrum = Sol 22: (B) r = = = 1.058 Å
2 Z 2
6× 5
= = 15 −13.6
2 Sol 23: (A) Energy En = = – 3.4 (n = 2)
n2
n=6 nh 2h h
n=5 angular momentum = = =
n=4 2π 2π π
n=3 −13.6
Sol 24: (B) En = −3.4 =
= T.E.
n=2 n2
n=2
n=1
Kinetic energy = |T.E.| = 3.4 eV
1 1 
Sol 19: (C) Energy of photon = 13.6  −  h 6.6 ×10−34
 12 25  =λ =
2mKE 2 × 9.1 ×10−31 × 3.4 × 1.6 × 10−19
24
= × 13.6 eV
25 6.6 ×10−34 25
= ×10
= 6.6 × 10 –10 m
Momentum of photon 9.9

h h E 24 13.6 ×1.6 ×10−19


= = = = × Sol 25: (A) Since some photons have energy greater
λ hc c 25 3 ×108
than 13.6 eV so electrons in hydrogen atoms will come
E
out of hydrogen atom. So only Lyman series absorbtion
By momentum conservation will be observed. Electron will not excite in other excited
mv = 6.96 × 10–27 states as there are only few photons of required energy
for the transition. So Balmer series will not be observed.
1.67 × 10–27 × v = 6.96 × 10–27
v = 4.169 m/s Sol 26: (B) Difference of energy between any shell is
independent of the reference level.
1
Sol 20: (A) Velocity ∝
n T.E. = K.E. + P.E.
So kinetic energy will increase KE at ground state = 13.6 eV

−2 ×13.6 Z 2 So T.E. at ground state = 13.6 + 0 = 13.6 eV


P.E. =
n2 1
Sol 27: (A) r ∝
m
P hysi cs | 24.73

Mass of dueterium > mass of hydrogen Sol 6: (D) For a given plate voltage, the plate current
in a triode valve is maximum when the potential of the
⇒ rd < rh
grid is positive and plate is positive
Velocity is same for both.
Energy of dueterium > Energy of hydrogen Sol 7: (C) The X-ray beam coming from an X-ray tube
will be having all wavelengths larger than a certain
⇒ Wavelength of dueterium < wavelength of
minimum wavelength
hydrogen
nh Sol 8: (B) Cut-off wavelength depends on the
Angular momentum =mvr =
2π accelerating voltage, not the characteristic wavelengths.
is independent of mass. Further, approximately 2% kinetic energy of the
electrons is utilized in producing X-rays. Rest 98% is
hc 12420 lost in heat.
Sol 28: (B) lmin = = Å
eV V
Sol 9: Minimum voltage required is corresponding to n
12420
=V = 18.75kV = 1 to n = 2. Binding energy of the innermost electron
0.663 is given as 40 keV i.e., ionization potential is 40 kV.
Therefore,
12420
Sol 29: (D) λmin =
V 1 1 
40 × 103  2 − 2 
If V increase lmin will decrease. Vmin
= 1 2 
= 30 × 103 V
1 1
 2 − 
1 ∞

Previous Years’ Questions The energy of the characteristic radiation will be


30 × 103 eV .
n
Sol 1: (B) Shortest wavelength or cut-off wavelength 1
Sol 10: R = R 0  
depends only upon the voltage applied in the Coolidge 2
tube.
Here R 0 = initial activity
Sol 2: (C) Beta particles are fast moving electrons which = 1000 disintegration/s
are emitted by the nucleus.
and n = number of half-lives.
At t = 1 s, n = 1
Sol 3: (A) The maximum number of electrons in an
orbit are 2n2 . If n > 4, is not allowed, then the number 31
of maximum electrons that can be in first four orbit are:
∴ R 10
= =   500 disintegration/s
2
2(1)2 + 2(2)2 + 2(3)2 + 2(4)2 At t = 3 s, n = 3
= 2 + 8 + 18 + 32 = 60 3
1
R = 103   = 125 disintegration/s
Therefore, possible elements can be 60. 2

Sol 4: (D) Shortest wavelength will correspond to Sol 11: Kmax = hν − W


maximum energy. As value of atomic number (Z)
increases, the magnitude of energy in different energy Therefore, Kmax is linearly dependent on frequency of
states gets increased. Value of Z is maximum for doubly incident radiation.
ionized lithium atom (Z = 3) among the given elements.
Hence, wavelength corresponding to this will be least. Sol 12: Number of α-particles emitted
238 − 206
n1 = =8
Sol 5: (C) During fusion process two or more lighter 4
nuclei combine to form a heavy nucleus.
and number of β -particles emitted are say n2 , then
2 4 . 7 4 | Modern Physics

92 − 8 × 2 + n2 =82 Sol 19: (A) 4 x 103 = 1020 x hf


∴ n2 = 6 4 × 103
f=
1020 × 6.023 × 10 –34
Sol 13: (B)
f = 6.03 x 1016 Hz
The obtained frequency lies in the band of X–rays.

i Sol 20: (C) KEmax = hυ – hυ0

hυ – hυ0 = e × ∆v
2d cos i = nλ
hυ hυ0
h V0 = –
2d cos i = e e
2meV
'υ ' is doubled
v = 50 volt
KEmax = 2hυ – hυ0
Sol 14: (B) 2d cos i = nλdB 2hυ hυ0
V0' = ( ∆V)' = –
e e
Sol 15: (D) Diffraction pattern will be wider than the
KEmax
slit. may not be equal to 2
KEmax

Sol 16: (D) W = QdV = Q(Vq- VP) V0 '


⇒ may not be equal to 2
= -100 ×(1.6 ×10-19) ×(– 4 – 10) V0

= + 100 ×1.6 ×10-19×14 = +2.24 10-16 J. Ke = max = hυ – hυ0

hυ hυ0
Sol 17: (B) V= –
e e
1 2 hc 1240evnm
mv= eV= 1.68 eV ⇒ h=
ν = = 3.1 eV ⇒ 3.1 eV= W + 1.6 eV
2 o
λ 400nm Sol 21: (C)o Davisson – Germer experiment showed that
1240evnm electron beams can undergo diffraction when passed
= 3.1 eV ⇒ 3.1 eV
= Wo + 1.6 eV through atomic crystals. This shows the wave nature
400nm
of electrons as waves can exhibit interference and
∴ Wo = 1.42 eV
diffraction.

Sol 18: (D) Since the frequency of ultraviolet light is m2 f m1r


Sol 22: (D) r1 = ; r2 =
less than the frequency of X–rays, the energy of each m1 + m2 m1 + m2
incident photon will be more for X–rays.
nh
K.Ephotoelectron = hv - φ (l1 + l2 )ω = n
=

1 (m1 + m2 )n2h2
Stopping potential is to stop the fastest photoelectron K.E.
= ( I
2 2 2
+ I ) ω
=2

hv φ 2m1m2r 2
V=
0 −
e e
So, K.Emax and V0 both increases. Sol 23: (D) As λ is increased, there will be a value of λ
above which photoelectrons will be cease to come out
But K.E ranges from zero to K.Emax because of loss of so photocurrent will become zero. Hence, (D) is correct
energy due to subsequent collisions before getting answer .
ejected and not due to range of frequencies in the
incident light.
P hysi cs | 24.75

Sol 24: (D) ∆E = hν 4 41  1


⇒ w +  mv 2  =w + m(v')2
3 32  2
∆E  1 1 k2n
ν=
= k = – 
h  (n– 1)
2
n  n (n– 1)2
2 2 1 w 41
⇒ m(v')2 =+ mv 2
2 3 32
2k 1
≈ ∝ 1 41 
n 3
n3 ⇒ m(v')2 >  mv 2 
2 32 
mυ 4
Sol 25: (B) r = ⇒ v' > v
qB 3
2meV
=
eB
JEE Advanced/Boards
1 2m Br e 2 2
= = V ⇒=
B e V
2m
= 0.8 V Exercise 1

For transition between 3 to 2, Sol 1: (a) Stopping potential is a property of material so


it will remain same.
1 1 13.6 × 5
E = 13.6  –  = = 1.88 eV 1
4 9 36 (b) Saturation current ∝
Work function = 1.88 eV – 0.8 eV r2
r is thrice of initial distance so
= 1.08 eV = 1.1 eV
1
Saturation current = ×18mA =
2mA
1 9
1 1
Sol 26: (C) = RZ 2  – 
λ  n12 n2  Sol 2: λ = 540 nm
1
⇒λ∝ for given n1 & n2 12400 62
Z2 Energy of photon
= E = eV
5400 27
⇒ λ1 = λ2 = 4λ3 = 9λ 4
Power = 663 mW
663 ×10−3 27 × 663
Sol 27: (C) (i) Frants – Hertz Experiment is associated No. of photon per sec = = ×1016
6.2 −19 62 ×1.6
with Discrete energy levels of atom ×1.6 ×10
27
(ii) Photo electric experiment is associated with particle
27 × 663 ×1016
nature of light. No. of it e¯ per sec =
62 ×1.6 ×5 ×109
(iii) Davison – Germer experiment is associated with
wave nature of electron. = 3.61 × 108 e¯/sec =3.61 × 108 × 1.6 × 10–19 A
= 5.776 × 10–11 A
hc 1
Sol 28: (D) = w + mv 2  … (i)
λ 2
Sol 3: λ = 330 nm
hc 1 hn1 = hn0 + KE
= w + m(v ')2
λ' 2
KE = eV0
hc 1 hn1 = hn0 + eV0
= w + m(v ')2  … (ii)
 3λ  2
  hn2 = hn0 + 2eV0
 4
h(n2 – n1) = eV0
 4
Equation (i) ×  – (ii)
 3  h(ν2 − ν1 ) E2 − E1
V0 = =
4hc 4 hc 4 41  1 e e
– = w +  mv 2  – w – m(v')2
3λ 3 λ 3 32  2
2 4 . 7 6 | Modern Physics

12400 12400 Sol 6: φ = 1.17 eV


E2 = eV ; E1 = eV
2200 3300 d = 2m

62 41.3 λ = 4.8 × 10-7 m = 4800 Å


E2 = eV ; E1 = eV
11 11 P = 1W
p 1 1
62 − 41.3 Intensity of light = = =
V0 = eV = 1.88 V 4 πd 2 4π × 4 16 π
11e
12400
Energy of 1 photon = = 2.58 eV
4800
Sol 4: λ = 990 Å
d = 0.1 m Number of photons striking per square meter per sec

r = 0.05 nm = 5 × 10–11 m 1
= = 4.81 × 1016
10 −11 2 16π × 2.58 ×1.6 ×10−19
(i) Intensity of light = × π (5 × 10 )
4 π (0.1)2 hν = hn0 + KE
= 250 × 25 × 10–22
E = hν
= 6250 × 10–22
E = hn0
= 6.25 × 10–19 J
1
12400 mv 2 = hν – hn0 = E – E0
Energy of photon = = 12.52 eV 2
990 12400
− 1.17
  = 20 × 10–19 J 4800

6.25 ×10−19 1
5 m v 2 = 2.58 – 1.17 = 1.41 eV …. (i)
Average no. of photon = = 2 e
20 ×10−19 16
2.82 ×1.6 ×10−19
v2 =
10 1 1 9.1 ×10−31
(ii) No. of electron = × ×
4 π (0.1)2 −19
(20 × 10 J) 100 v2 = 0.495 × 1012

10 ×1018 5 1019 v = 7.04 × 105 m/s


= = ×1018 =
4 π× 2 4π 8π mv 2
Magnetic force = eVB =
R
Sol 5: hν = hn0 + KE By (i)

KE = eV0 mv2 = 2.82 eV

hν = hn0 + eV0 2.82eV


eVB =
R
eV0 = hν – hn0
2.82
y =V0 ; x = n 7.04 × 105 m/s × 10–4 =
R
ey = hx – hn0 R = 4 cm
hx φ
y
= −
e e Sol 7: Z = 3
Work function φ = 2eV 1 1 
Energy of E1 = 13.6 × 9  − 
h 2  16 25 
Slope = =
e 0.49 ×1015
E1 = 2.754 eV
−19
2 ×1.6 ×10
h= = 6.53 × 10–34 J-s 1 1 
0.49 ×10 15 Energy E2 = 13.6 × 9  − 
 9 16 
P hysi cs | 24.77

E2 = 5.95 eV Sol 10: P = 3.2 × 10–3 W


E2 = φ + KE2 (a) Energy of photons = 5 eV;
KE2 = 3.95 eV 12400
∴λ = = 2480 Å
φ = E2 – KE2 5
Distance = 0.8 m
= 5.95 – 3.95
φ = 3 eV
φ = 2 eV
Radius = 8 × 10–3 m
E1 = φ + KE1
1
KE1 = 2.754 – 2 Efficiency = electrons per photon
106
eV = 0.754 eV Power incident on atom
V = 0.754 Volts
3.2 ×10−3
= × π (8 ×10−3 )2
12400 4 π (0.8)2
Sol 8: l1 = 4144 Å; E1 = eV = 2.99 eV
4144
10−3
12400 = × 0.8 × 0.8 ×10−4
l2 = 4972 Å; E2 = eV = 2.49 eV 0.8
4972

12400 n=5
l3 = 6216 Å; E3 = eV = 1.99 eV E1
6216 n=4
E2
φ = 2.3 eV n=3
E2 > E1
−3
3.6 ×10
Intensity I1 = I2 = I3 = Wm–2 × 10–4
3 = 0.8 × 10–7 = 8 × 10–8 W
= 1.2 × 10–7 W
8 ×10−8
No electrons will be emitted by 6216 Å wavelength No. of photons =
5 ×1.6 ×10−19
photons as E3 < φ.
N = No. of electrons
No. of photons in light of wavelength l2 is
1.2 ×10−7
= 3 × 1011 photons / sec 8 ×10−8
2.49 ×1.6 ×10 −19 = = 105 s−1
106 × 5 ×1.6 ×10−19
No of photons in light wavelength l1 is
h
(b) ld =
1.2 ×10 −7 mv
= 2.5 × 1011 photons / sec
2.99 ×1.6 ×10−19 p2
KE = 2eV =
2m
No of electrons liberated in 2 seconds
p2 = 2 × 1.6 × 10–19 × 2 × 9.1 × 10–31
= 2 (3 + 2.5) × 10 11
p2 = 58.24 × 10–50
= 11 × 10 electrons.
11
p = 7.63 × 10–25

Sol 9: (i) Refer Sol 4 Exercise-I JEE Advanced 6.6 ×10−34


ld = = 0.86 ×10−9
−25
5 7.63 ×10
(ii) No. of photons = per second
16
λ 2480 ×10−10 248
5 1 = = = 286.76
No. of electrons = × per second λd 0.86 ×10 −9 0.86
16 100
5 (c) After some time sphere gets positively charged and
= per second
1600 it will create electric field which will stop the further
emission of electrons.
2 4 . 7 8 | Modern Physics

(d) KE = hν – hφ =5 eV – 3 eV = 2 eV EB = φB + Tb
K.E. = eV = 2 eV φB = 4.7 – 0.5 = 4.2 eV
V = 2 volts
Sol 12: Force on electron = eE
Potential at the surface of sphere
eE
Kq K(Nt) ×1.6 ×10−19 9 ×109 × 105 t ×1.6 ×10−19 Acceleration =
= = = m
r 8 × 10−3 8 × 10−3 eE
Velocity = t ; p = eEt
= 9 × 0.2 × 10–2 × t m
h
2 = 1.8 × 10–2 × t ld =
p
2
t
So time required= ×100 = 111.1 sec dλ d h dp −heE −heE −h
1.8 =– = = =
dt p 2 dt (eEt)2 2 2 2
eE t eEt2
Sol 11: For metal A
13.6
Energy of photons = 4.25 eV Sol 13: B.E. = 0.85 eV =
n2
Maximum KEA = Ta
n=4
de Broglie wavelength = la
For metal B 1  1 1  1 1 
= R − = R − 
2 
λ 2 2
4   4 16 
KEB max = Tb = Ta – 1.5
Energy of photons = 4.7 eV 1 12R 3R 16R
= = ⇒ λ= = 487 nm
De-Broglie wavelength = lb = 2la λ 4 ×16 16 3
2
p2  h  1
KE =
=   × Sol 14: 5th excited state
2m  λd  2m

 h 
2 ⇒n=6
1
=Ta   ×
 λ  2m m = mass of atom
 a
2 v = velocity of atom
 h  1
=Tb  ×
 2λ  2m (mv 2 ) hc 13.6
 a + = E6 − E0 = – + 13.6
2m λ 36
Tb = Ta – 1.5 35
= ×13.6
2 36
 h  1 h2 1 h
  × − × −1.5
= Momentum conservation, = mv
 2λa  2m λa 2m
2
λ
2

 h 
2

  ×
1 3
× =−1.5
1  h  hc
  +
2m  λ  λ
=
35
36
×13.6 × 10−19 ×1.6 J ( )
λ
 a 2m 4
⇒ λ = 939.4 Å
Ta ÷ 2 = 1
Energy = 13.2 eV
Ta = 2 eV
h
Tb = 2 – 1.5 = 0.5 eV v= ⇒ v = 4.26 m/s

From metal A
13.6
Sol 15: Energy of series limit of Balmer is = 3.4
EA = φA + Ta 4
φA = 4.25 – 2 = 2.25 eV hc
λB =
For metal B 3.4
P hysi cs | 24.79

1 1  h 6.6 ×10−34
Energy of first line of Paschen is 13.6  2 − 2  λ1 = = = 8.6 pm
3 4  mv1cm 1.67 × 10−27 × 4.6 ×10 4
hc 13.6 × 7
= = 0.661 h 6.6 ×10−34
λp 9 ×16 l2= = =8.6 pm
2mv 2cm 2 × 1.67 ×10−27 × 2.3 ×10−4
hc
λP =
0.661 nλ
Sol 17: = 2Å
2
λB 0.661 7
Ration = =
λP 3.4 36 (n + 1)λ λ
= 2.5 Å ⇒ = 0.5 Å
2 2
1 h
Sol 16: 25 eV = m v2 ⇒ λ = 1Å ⇒ λ =
2 n p
6.6 ×10−34
V p= = 6.6 × 10–24
−10
m 2m 1 ×10

p2 (6.6)2 ×10−48 2.39 ×10−17


Energy = = = = 148.4eV
2m 2 × 9.1 ×10−31 1.61 ×10−19
50 ×1.6 ×10−19
v2 = = 47.9 × 108
1.67 ×10−27 Sol 18: (D) Energy of photon from He+
V = 6.92 × 104 m/s
1 1
= 13.6 Z2  − 
m1 v1 + m2 v 2 m× V + 0 V 1
2
22 
Vcm = = =
m1 + m2 3m 3
3
= 13.6 × 4 × = 13.6 × 3 eV
= 2.3 × 10 m/s 4
4
By momentum conservation Energy of photon from H = 13.6 eV
Energy of photoelectron = 13.6×3-13.6 = 13.6×2 eV
V1 V2 1
m 2m K.E. = mv2 = 13.6 × 2 × 1.6 × 10–19
2

mV = mV1 + 2mV2 4 ×13.6 ×1.6 ×10−19


v2 = ⇒ v2 = 9.56 × 1012
−31
9.1 ×10
V = V1 + 2V2
By energy conservation v = 3.09 × 106 m/s

1 1 1 n(n − 1)
=mV 2 mV12 + 2mV22 Sol 19: (i) =6
2 2 2 2

V2 = V12 + 2V22 ⇒ V2 = (V – 2V2)2 + 2V22 ⇒ n (n – 1) = 12 ⇒ n = 4


V2 = V2 + 6V22 – 4V2V ⇒ 6V2 = 4V since emitted photons are of energy less, equal and
2V more than 2.7 eV
V2 =
3 So level B must be n = 2
4V V
V1 = V – 2V2 = V – =– 1 1 
3 3 (ii) 2.7 = 13.6 Z2  − 
 22 42 
−2V
Velocity of 1 w.r.t. C.M. is V1cm = = – 4.6 × 104 m/s
3 13.6 Z 2 3
V 2.7 =  
Velocity of 2 w.r.t. CM is V2cm = =2.3 × 104 m/s 4 4
3
2 4 . 8 0 | Modern Physics

2.7 ×16 13.6 Z 2 ×5


Ionisation energy = 14.4 eV = 14.4×1.6×10–19 J 47.2 × 2 =
3 36
= 23.04 × 10–19 J Z2 = 25 ⇒ Z = 5

1 1  1 1
(iii) Maximum energy= 13.6 Z2  −  ; 4 → 1 Sol 26: 47.2 = 13.6 Z2  − 
2
 12 42  2 32 

(i) Z = 5
1 1 
Minimum energy = 13.6 Z2  −  ; 4 → 3
 32 42  1 1 
(ii) E = 13.6 × 25  −  = 16.5 eV
 9 16 
Sol 20: λ = 500 Å
1 1 
12400 (iii) E = 13.6 × 25  − 
Energy = = 24.8eV  1 ∞2 
2
500
Energy required to take out electron from atom = 13.6 E = 13.6 × 25 eV
eV E = 340 eV
KE = 24.8 – 13.6 = 11.2 eV 12400
λ= = 36.4 Å
340
Sol 21: Energy of photon = 13.6 Z2 (iv) KE = |T.E.|
= 13.6 × 4 = 54.4 KE = 13.6 Z2 = 13.6 × 25 = 340 eV
12400 eV P.E. = – 2 |T.E.|
Wavelength = = 227.94 Å = 22.8 nm
54.4 eV
P.E. = – 2 × 340 = – 680 eV
h
Sol 22: E3 – E2 + E2 – E1 = E3 – E1 (v) Angular momentum = mvr = Iω = n

12400 12400 12400 0.529n2
+ = Radius = = 0.1058 Å
λ1 λ2 λ3 z

1 1 1 λ λ
+ = ⇒ λ3 = 1 2 Sol 27:
λ1 λ2 λ3 λ1 + λ2
n n
n1 n2
Sol 23: Energy of new atom = 2 × energy of hydrogen
n=2 n=3
atom
Energy gap between quantum states n and 2 is
hc 1 1
=13.6 × 2  −  22.95 + 5.15 = 28.1 eV
λ  22 32 
Energy gap between quantum state n and 3 is
1 1 1 18
= 2R  −  ⇒ λ = 2.4 + 8.7 = 11.1 eV
λ 2 2
2 3  5R
Energy gap between n = 2 and n = 3 is

Sol 24: r = 0.5 Å ; ω = 2π × 1016 rad / sec. 28.1 – 11.1 = 17 eV


1 1 13.6 Z 2 ×5
qVr 17 = 13.6 Z2  −  =
Magnetic moment = 2 2 36
2 2 3 
Z2 = 9 ⇒Z=3
eωr 2 1.6 ×10−19 × 2π×1016 1
= = × ×10 −20 = 0.4 × 10–23 1 1
2 2 4 28.1 = 13.6 × 9  − 
 22 n2 
= 1.25 × 10–23 Am2 1 1 1
− = 0.229 ⇒ = 0.25 − 0.229
1 4 n 2
n2
1
Sol 25: 47.2 = 13.6 Z2  − 
22
32  ⇒ n2 = 48.96 ⇒ n = 7
P hysi cs | 24.81

1 1 Exercise 2
Sol 28: E = 13.6 Z2  2 − 2 
2 5 
Single Correct Choice Type
12400 1 1
= 13.6 Z 2  −  10−3
1085  22 52  Sol 1: (B) No. of photons =
Energy of 1photon
12400  21  12400
= 13.6 Z 2   Energy of 1 photon = = 2.48 eV
1085  100  5000

⇒ Z2 = 4 ⇒ Z = 2 10−3
np = No. of photons =
Binding energy = 13.6 Z2 = 13.6 × 4 = 54.4 eV 2.48 ×1.6 ×10−19

−13.6 × 32 = 0.25 × 1016 = 2.5 × 1015 photon


Sol 29: Energy of n state of lithium ion =
th

n2 0.16 µA 1.6 ×10−7


= – 13.6 eV (n = 3) ne = No. of electron = =
1.6 ×10−19 1.6 ×10−19
0.529n2
Radius = = 1012 electron
2
ne 1012
0.529 × 9 Efficiency = ×100 = × 100
R 2+ np 2.5 ×1015
=Li
= 3 3 4
1
RH 0.529 = = = 0.04%
2.5 ×10 100

Sol 30: Total energy of the photons is 20 KeV


Sol 2: (B) KEmax = 2eV
Let their wavelength be λ 1, l2 Å
E1 = φ + KEmax 1
12400 12400
+ 20000
=
λ1 λ2 5eV = φ + 2eV
1 1 φ = 3eV
+ 1.613 
= … (i)
λ1 λ2 E2 = φ + KEmax 2 ; KEmax 2 = eV2
6 eV = 3 eV + eV2 ⇒ 3 eV = eV2
λ2 − λ1 =1.3  … (ii)
V2 = 3V
By (i) and (ii)
So stopping potential of A w.r.t. C is – 3V
1 1
+ = 1.613 2π2mk 2 Z 2e4
λ1 1.3 + λ1 Sol 3: (A) En =
n2h2
1.3 + 2λ1
= 1.613 Z 2 . 4 π2 mk 2e4
λ1 (1.3 + λ1 ) Frequency ν =
n3 .h3
1.3 + 2l1 = 1.613 × 1.3 l1 + 1.613 l12
1.6l12 + 0.096 l1 – 1.3 = 0 ν Z 2 × 4 π2 mk 2e4 n2h2 ν 2
= ⇒ =
En 2 3 2
n h 2π mk Z e 2 2 4 En nh
−0.096 ± (0.096)2 + 4 × 1.3 × 1.6
λ1 = 2En
3.2 ⇒ ν=
nh
l1 = 0.871 Å
l2 = 2.17 Å n2
Sol 4: (D) Bohr radius = × 0.529
Z

rn+1 – rn = [(n + 1)2 – n2] 0.529 = rn–1


2 4 . 8 2 | Modern Physics

= (2n + 1) 0.529 = (n – 1)2 × 0.529 field as in magnetic field kinetic energy does not
change. Kinetic energy of electron in electric field may
⇒ (n – 1)2 = 2n + 1
increase, remain same or decrease that’s why l2 can
n2 + 1 – 2n = 2n + 1 ⇒ n2 = 4n increase or decrease.
⇒n=4
λ1 > λ2 or λ2 < λ1 or λ1 = λ2
2 2 2
n h e0 0.529n
Sol 5:=
(A) rn =
pme2 Z Z Sol 11: (A, C) Minimum energy required for transition
to happen from ground state is 10.2 eV.
n =1, Z = 1
If the total loss in energy is less than 10.2 eV no transition
0.529n2 o will occur. Either there can be loss of energy greater
For mean r’n = = 2.56 × 10–3 A than 10.2 eV or no loss in energy since the energy of
207 Z
neutron is 20.4 eV the maximum loss in energy due
to inelasticity will be less then 10.2 eV. Therefore only
Sol 6: (C) Hydrogen emit a photon corresponding to
option is no loss in energy which means elastic collision.
the largest wavelength of the Balmer series. This implies
So (A and C).
electron was excited to n = 3
Energy required for transition n = 1 → 3 is
Sol 12: (A, C, D) Photon of energy 2.55 eV is emitted
 1 1  13.6 × 8 when transition is from n = 4 to n = 2
13.6  −  = = 12.088 eV
2 2
1 3  9 So other photon corresponds to n = 2 → n = 1

Minimum kinetic energy = 12.1 eV Energy absorbed by hydrogen atom = 10.2 + 2.55 =
12.75 eV

Sol 7: (A) Refer Q-11 (in Exercise II JEE Advanced) Minimum Kinetic energy of photon is when
collision is perfectly inelastic i.e. when
1 1  5 K = 25.5 eV
Sol 8: (B) n = 3 → 2; E1 = 13.6  −  = ×13.6
 4 9  36
Refer Q. 11
 1 3
n = 2→ 1; E2 = 13.6 1 −  = ×13.6
 4  4 Sol 13: (A, C, D) 13.6 Z2 = 122.4

hc hc Z=3
λ1 = ; λ2 =
E1 E2 For n = 1, E1 = – 122.4 eV
λ1 E2 3 27 n = 2 E2 = – 30.6 eV
x= = = × 36 =
λ2 E1 4 ×5 5 E2 – E1 = 91.8 eV
E1 5 If 125 eV energy electron collides with this atom then
Z
= =
E2 27 122.4 eV will be used to take out the electron and
kinetic energy of electron will be 125 – 122.4 = 2.6 eV
p1 h / λ1 λ2 5
y
= = = =
p2 h / λ2 λ1 27
Sol 14: (A, C, D) Some incident wavelengths will be
absent in A as some of them will be absorbed by
Multiple Correct Choice Type the hydrogen atom. B will emit photons of Energy
Corresponding to transitions in the hydrogen atom.
Sol 9: (A, C) Stopping potential ∝ kinetic energy This energy will lie in visible and infrared region.

Kinetic energy depends on the frequency of light


Sol 15: (A, B, C) Having electrons of same speed won’t
hν = hn0 + KE matter because electrons get decelerated to different
velocities ( just like electrons with random velocities)
h giving photons of different wavelength. (Read theory).
Sol 10: (A, D) λd =
p

de Broglie wavelength l1 will not change in magnetic


P hysi cs | 24.83

Assertion Reasoning Type


= 0.0364 × 10−7 m = 36.4 Å

Sol 16: (D) For frequency less than no no electrons are (d) In first orbit, total energy = –340eV
emitted. so Statement-I is/false. Kinetic energy = +340 eV

h Potential energy = –2 × 340 eV = –680 eV


Sol 17: (C) Momentum of photon is p =
λ h
and angular momentum =
Sol 18: (D) All emitted electrons do not have same K.E. 2π

There K.E. range from 0 to (hν – φ). 6.6 × 10−34


= = 1.05 × 10−34 kg- m2 /s

Sol 19: (D) If electron will not emit as only one single n2
photon should have energy more than work function. (e) rn ∝
Z
h h Radius of first Bohr orbit
Sol 20: (A) λe = ; λp =
2me (KE) 2mp (KE)
r1H 5.3 × 10−11
mp > me r1 = = = 1.06 × 10−11 m
Z 5
⇒ le > lp
Sol 2: Energy corresponding to given wavelengths:
Sol 21: (A) By Boltzmann’s law (randomization increases
12375 12375
with temperature) electron’s occupy more number of E(in eV) = = = 12.69eV
excited levels at higher temperature. λ (in Å) 975
Now, let the electron excites to nth energy state. Then,
En − E1 =12.69
Previous Years’ Questions
( −13.6)
or – (–13.6) = 12.69
(n2 )
Sol 1: (a) Given E3 − E2 = 47.2 eV
∴n≈ 4
Z2
Since En ∝ (for hydrogen like atoms) i.e., electron excites to 4th energy state. Total number of
n2
lines in emission spectrum would be:
 Z2    Z2  n(n − 1) 4×3
or(–13.6)   –  −(13.6)    =47.2 = =6
 9    4  2 2
    
Longest wavelength will correspond to the minimum
Solving this equation, we get Z = 5
energy and minimum energy is released in transition
(b) Energy required to excite the electron from 3rd to from n = 4 to n = 3.
4th orbit: −13.6 −13.6
E4 −=
3 E4 − E3 = – =0.66 eV
E3−=
4 E4 − E3 (4)2
(3)2

 25    25   ∴ Longest wavelength will be,


= (–13.6)   – ( −13.6)    = 16.53eV
16
    9 
12375 12375
λmax = = Å= 1.875 × 10−6 m = 1.875 µm
(c) Energy required to remove the electron from first E(in eV) 0.66
orbit to infinity (or the ionization energy) will be:
E = (13.6) (5)2 = 340 eV Sol 3: Number of proton = atomic number = 11

The corresponding wavelength would be, Number of neutron = mass number – atomic number
= 13
hc 6.6 × 10−34 × 3 × 108 But note that in the nucleus number of electron will be
λ= =
E 340 × 1.6 × 10−19 zero.
2 4 . 8 4 | Modern Physics

photon required is,


238 α−decay β−decay
Sol 4: 92 U → 90 X 234 →91 Y 234
( −13.6)(3)2  ( −13.6)(3)2 
E1−3 = E3 − E1 = –  
During an α -decay atomic number decreases by 2 and (3)2  (1)2 
mass number by 4. During a β -decay, atomic number
increases by 1 while mass number remains unchanged. = 108.8 eV
Corresponding wavelength will be,
Sol 5: When 800 Å wavelength falls on hydrogen atom
(in ground state) 13.6 eV energy is used in liberating 12375 12375
λ (in Å)= =
the electron. The rest goes to kinetic energy of electron. E(in eV) 108.8
Hence, K = E – 13.6 (in eV) or = 113.74 Å
hc (b) From nth orbit total number of emission lines can be ,
(1.8×1.6× 10−19 )= –13.6×1.6× 10 −19 ….(i)
−10
800 × 10 n(n − 1)
.
Similarly for the second wavelength: 2
3(3 − 1)
∴ Number of emission lines = =3
hc 2
(4.0×1.6× 10−19 )= –13.6×1.6× 10 −19 …(ii)
700 × 10−10
2K
Sol 8: Speed of neutrons =
Solving these two equations, we get m

h ≈ 6.6 × 10 −34 J s− s  1 2
 From K = mv 
 2 
Sol 6: (a) 1 Rydberg = 2.2 × 10−18 J = Rhc
2 × 0.0327 × 1.6 × 10−19
Ionisation energy is given as 4 Rydberg or v = ≈ 2.5 × 103 m / s
1.675 × 10−27
8.8 × 10−18 Time taken by the neutrons to travel a distance of 10 m:
= 8.8 × 10−18 J = = 55 eV
1.6 × 10−19
d 10
t= = = 4.0 × 10−3
∴ Energy in first orbit E1 = −55eV v 2.5 × 103

Energy of radiation emitted when electron jumps from Number of neutrons decayed after time t
first excited state (n = 2) to ground state (n = 1):
N = N0 (1 − e−λt )
E 3
E21 =1 − E1 =− E1 =41.25eV
(2)2 4 ∴ Fraction of neutrons that will decay in this time
interval
∴ Wavelength of photon emitted in this transition
ln(2)
would be, N − × 4.0 × 10−3
= = (1 − e−λt ) = 1 – e 700 = 3.96 × 10−6
12375 N0
λ= = 300 Å
41.25
(b) Let Z be the atomic number of given element. Then Sol 9: If we assume that mass of nucleus >> mass of
mu-meson, then nucleus will be assumed to be at rest,
( 13.6)(Z 2 ) or Z ≈ 2
E1 = ( −13.6)(Z 2 ) or −55 =− only mu-meson is revolving round it.
1 (a) In nth orbit, the necessary
Now, as r ∝
Z centripetal force to the mu-
Radius of first orbit of this atom, meson will be provided by the
rH electrostatic force between the
0.529 +Ze - m = 208 me
r1 = 1 = = 0.2645 Å nucleus and the mu-meson.
Z 2

Z2
Sol 7: Given Z = 3 : En ∝
n2
(a) To excite the atom from n = 1 to n = 3, energy of
P hysi cs | 24.85

mv 2 1 (Ze)(e) Sol 10: (C)


Hence, =  … (i)
r 4 πε0 r 2 n=2 -3.4eV
E = 10.2eV
Further, it is given that Bohr model is applicable to this n=1 -13.6eV
system also. Hence, H-atom
n=4 -3.4eV
nh
Angular momentum in n orbit = th
2π n=3 -6.04eV
h E = 10.2eV
or mvr = n  … (ii) n=2 -13.4eV

We have two unknowns v and r (in nth orbit). After -54.4eV
n=1
n2h2 ε0 He+
solving these two equations, we get r = Z=2
Zπme2
Substituting Z = 3 and m = 208 me , we get Energy given by H-atom in transition from n = 2 to n
= 1 is equal to energy taken by He+ atom in transition
n2h2 ε0 from n = 2 to n = 4.
rn =
624 πmee2

(b) The radius of the first Bohr orbit for the hydrogen Sol 11: (C) Visible light lies in the range, λ1 = 4000 Å to
λ2 = 7000 Å. Energy of photons corresponding to these
h2 ε0
atom is: wavelength (in eV) would be:
πmee2
12375
E1 =
Equating this with the radius calculated in part (a), we 4000 = 3.09 eV
n2 ≈ 624 or n ≈ 25
12375
mv 2 Ze2 E2 =
(c) Kinetic energy of atom = = 7000 = 1.77 eV
2 8πε0r
From energy level diagram of He+ atom we can see that
Ze2 in transition from n = 4 to n = 3, energy of photon
and the potential energy = −
4 πε0r released will lie between E1 and E2.
−Ze2
∴ Total energy En = ∆E43 =−3.4 − ( −6.04) = 2.64 eV
8πε0r

Substituting value of r, calculate in part (a), Wavelength of photon corresponding to this energy,
12375
1872  mee 
4
λ=
264 Å = 4687.5 Å = 4.68×10 m
-7
=En − 
n2  8ε02h2 

 m e4  Sol 12: (A) Kinetic energy K ∝ Z 2


But  − e  is the ground state energy of hydrogen
 8ε20h2  2
KH 1 1
atom and hence is equal to – 13.6 eV. ∴ = =
K + 2 4
He
−1872 25459.2
∴ En = (13.6) eV = – eV
2
n n2 Sol 13: A → p, q; B → p, r; C → p, s; D → q

1 1 
∴ E3 − E1 =−25459.2  −  =22630.4 eV Sol 14: (C, D) For photoemission to take place,
9 1
wavelength of incident light should be less than the
∴ The corresponding wavelength, threshold wavelength. Wavelength of ultraviolet light <
5200Å while that of infrared radiation > 5200 Å.
 12375 
λ(ln)=
A = 0.546 A
22630.4 nλ 2a
Sol 15: (A) a = ⇒λ=
2 n
2 4 . 8 6 | Modern Physics

h h
λdeBroglie = λ=
p mv

2a h nh 4.2 × 10−15 × 1.6 × 10−19 4.2 × 4.8 × 10−34


= ⇒p= =
λ = = 0.07 × 10−13
n p 2a 5
× 10−27 × 331.776 × 1013 57.6 × 5 × 10−21
3
p2 n2h2 λ = 7 × 10 −15 = 7 fm
E= =
2m 8a2m
Sol 21: (B) t = 100 × 10-9 sec, P = 30 × 10-3
⇒ E ∝ 1 / a2
Watt, C = C × 108 m/s

Sol 16: (B) Pt 30×10−3×100×10−9


Momentum
= = = 1.0 × 10−17 kg ms−1
h2 (6.6 × 10−34 )2 C 3 × 108
E= = = 8 meV.
8a2m 8 × (6.6 × 10−9 )2 × 10−30 × 1.6 × 10−19
Sol 22: Slope of graph is h/e = constant ⇒ 1
nh
Sol 17: (D) mv =
2a Sol 23: Ephoton= Eionize atom + Ekinetic energy
nh 1242 13.6
v= ⇒ v ∝n
= + 10.4
2am 90 n2
from this, n = 2
Sol 18: P1 = 2m(100 eV)

h h Sol 24: (A) For photoelectric emission


λP = ⇒ λα =
2m(100 eV) 2(4m)(100 eV)
 hc  1 φ
V0 =   –
λP  e λ e
= 8
λα
hc
λP Sol 25:(B) KEmax
= – φ eV0
=
⇒ The ratio to the nearest integer, is equal to 3. λ
λα
hc hc
⇒ – = e(V1 – V2 )
1  1 1  5R λ1 λ2
Sol 19: (A) =R –  =
6561  4 9  36
 1 1 
⇒ hc  – =1.6 × 10 –19 × 10 –6
1  1 1  3R × 4  0.3 0.4 
= 4R  – =
λ  4 16  16
 0.1  –25
 ⇒ hc  =  1.6 × 10
λ =1215 A  0.12 
1 K(Q)e K(120 e)e
2
Sol 20: 0 + mv= = 1.6 × 10−25 × 1.2
2 10 × 10 −15
10 × 10−15 ⇒ h= =0.64 × 10−33 =6.4 × 10−34
3 × 108

1 5 9 × 109 × 120 × (1.6 × 10−19 )2  1 1 


× × 10−27 v 2 = ⇒ hc  – −19 −6
2 3 10 × 10−15  = (1.6 × 10 ) × 0.6 × 10
 0.4 0.5 
9 × 6 × 109 × 120 × 2.56 × 10−38 0.20 1
⇒ v= ⇒ hc = (0.96 × 10−25 ) × ×
50 × 10−42 0.10 3 × 108

⇒ v
= 331.776 × 1013 1.92
⇒ h = × 10−33 =6.4 × 10−34
3
P hysi cs | 24.87

Sol 26: (A), (B), (D) Orbital radius rn = n2c [c = constant] m2d m1d
r1 = and r2 =
Angular momentum = nh = L m1 + m2 m1 + m2

∆r (n+ 1)2 – n2 2 ∆Ln 1 I = m1 r12 + m2 r22


= = .....[B]; = .... [D]
rn n2 n Ln n
∴ d = 1.3 ×10−10 m.
(A) is correct since it will get cancelled in calculation of
relative charge. hc
Sol 33: (7) Stopping potential = –W
λ
Sol 27: [6] Photon Energy = 6.2 eV −4.7 eV = 1.5 eV

hc 1.237 × 10 –6 1237 Kq
== = × 10eV V
= = 1.5
λ 970 × 10 –10 970 r

Absorption of this photon changes the energy to = – = 1.5 × 10−2


n = 1.05 × 107
13.6 + 12.75 = – 0.85 eV 9 × 10 × 1.6 × 10−19
9

Number of possible transitions from the 4th quantum Z=7


state = 4C2 = 6
−2GMm 1
Sol 34: (B) + mv 2 =
0
Sol 28: (B) P1 = pressure just inside the bubble at the L 2
4T
end 2 = P0 + GM
R ⇒v=
2
L
P2 = pressure just inside the bubble at the end
Note: The energy of mass ‘m’ means its kinetic energy
4T
1 = P0 + (KE) only and not the potential energy of interaction
r
between m and the two bodies (of mass M each) –
R > r ⇒P2< P1 ⇒ Air will flow from end 1 to end 2 which is the potential energy of the system.

Sol 29: (A) VB= (1/e)[(hc/λ) − φ ] hc


–φ
VP= (1/e)[(1240/550) −2]eV = 0.2545 V λ1 u12
Sol 35: (A) =
hc u22
Vq= (1/e)[(1240/450) −2.5]eV = 0.255 V –φ
λ2
Vr = (1/e)[(1240/350) −3]eV = 0.5428 V φ = 3.7 eV
If n is the number of photons in unit time then nhc/λ= I
hc
⇒iP : iq: ir= nP : nq : nr = λP : λq : λr Sol 36: (A) K=
max – φ + eV [Kmax = maximum
λph
energy e– reaching the anode]
nh
Sol 30: (D) L =  hc 
2π h2
⇒ =  – φ  + eV  … (i)
2 2mλ 2e λ 
L2  nh  1  ph 
K.E. = = 
2I  2π  2I From Equation (i) (A) follows
 hc 
if φ increases and λph increases then  – φ
 λph 
Sol 31: (B) hv= k.En=2 −kEn=1 decreases  
As a result λc increases λe is independent of ‘d’ and
I = 1.87 × 10-46 kg m2 clearly λe and λph do not increase at the same rate.

Sol 32: (C)


2017-18 100 &
op kers
Class 12 T
By E ran culty
-JE Fa r
IIT enior emie .
S fP r es
o titut
Ins

PHYSICS
FOR JEE MAIN & ADVANCED
SECOND
EDITION

Exhaustive Theory
(Now Revised)

Formula Sheet
9000+ Problems
based on latest JEE pattern

2500 + 1000 (New) Problems


of previous 35 years of
AIEEE (JEE Main) and IIT-JEE (JEE Adv)

5000+Illustrations and Solved Examples


Detailed Solutions
of all problems available

Topic Covered Plancess Concepts


Tips & Tricks, Facts, Notes, Misconceptions,
Nuclear Physics Key Take Aways, Problem Solving Tactics
and Radioactivity
PlancEssential
Questions recommended for revision
25. NUCLEAR PHYSICS
AND RADIOACTIVITY

NUCLEAR PHYSICS

1. INTRODUCTION
Nuclear physics is the field of physics that studies the constituents and interactions of atomic nuclei. Nuclear
physics is the field of physics that studies the constituents and interactions of atomic nuclei. Nuclear physics is the
field of physics that studies the constituents and interactions of atomic nuclei.

2. PROPERTIES OF ATOMIC NUCLEUS


Nuclear physics is the field of physics that studies the constituents and interactions of atomic nuclei. The most
commonly known applications of nuclear physics are nuclear power generation and nuclear weapons technology, but
the research has provided application in many fields, including those in nuclear medicine and magnetic resonance
imaging, ion implantation in materials engineering and radiocarbon dating in geology and archaeology. The field
of particle physics evolved out of nuclear physics and is typically taught in close association with nuclear physics.
Properties: Atomic nuclei have following properties:

2.1 Composition
All nuclei contain protons and neutrons except ordinary hydrogen atom which has only single proton. Proton has
change +e and neutron is neutral.
Mass no. of nuclei ( A )= Z + N
Where Z = no. of protons in the nucleus; N = no. of neutrons
A
Symbolically atomic nuclei is represented as ZX

Illustration 1: How many electrons, protons, and neutrons are there in nucleus of atomic number 11 and mass
number 24?  (JEE MAIN)

Sol: The atomic number Z of atom represents the number of protons present in the nucleus. The number of
electrons in an atom are same as the number of protons. The Atomic mass number A is sum of proton number Z
and neutron number N.
Number of protons in nucleus = Atomic number = 11
Number of electrons = Number of protons = 11
Number of neutrons = Mass number A – atomic number Z N = 24 – 11 = 13
2 5 . 2 | Nuclear Physics and Radioactivity

2.2 Mass
Nuclear mass has been measured accurately by using mass spectrometer. It is convenient to express mass in terms
1
of amu which is defined as the mass of carbon isotope 126 C
12
= 1.66 × 10−27 kg
1amu
According to Einstein’s equation E = mc2 1amu can be expressed as energy

( )
2
1.66 × 10−27 × 3 × 108
Energy
= equivalence of 1 amu = eV 931MeV
1.6 × 10−19

2.3 Nuclear Radius


The nuclear radius (R) is considered to be one of the basic quantities that any model must predict. For stable nuclei
(not halo nuclei or other unstable distorted nuclei) the nuclear radius is roughly proportional to the cube root of
the mass number (A) of the nucleus, and particularly in nuclei containing many nucleons, as they arrange in more
spherical configurations:
The stable nucleus has approximately a constant density and therefore the nuclear radius R can be approximated
by the following formula, R = r0 A1/3
Where A=Atomic mass number (the number of protons Z, plus the number of neutrons N) and
= 1.25 × 10−15 m .
r0 1.25fm
=

27 125
Illustration 2: The ratio of the radii of the nuclei 13 Al
and 52 Te
is approximately.  (JEE MAIN)

Sol: The radius of the atomic nuclei is directly proportional to the cube root of atomic mass number.

( 27 ) =
1/3
3 6
R Al / R Te= =
5 10
(125)
1/3

64
Illustration 3: The radius of the 30 Zn
nucleus is nearly (in fm) (JEE MAIN)

Sol: The radius of any atomic nucleus is given by R = R 0 A1/3 where Ro = 1.2 × 10−15 is the Fermi radius.

1.2 × 10−15 × ( 64 )
1/3
R=R 0 A1/3 = 1.2 × 10−15 × 4 =
= 4.8fm A ∝ R 3

A = Nucleon number or mass number


A A
Any element X with mass number A and charge number Z can be represented by ZX
or ZX
.
Number of neutron= A − Z Mass number= A= P + N
1 12
1 amu = th Mass of 12gm of 6 C atom.
12

2.4 Nuclear Density


Nuclear density is the density of the nucleus of an atom. The nuclear density for an atom with radius R and molar
A
mass A(mass number) is n =
4 3
πR
3
Typical nucleus can be approximately calculated from the size of the nucleus, which itself can be approximated
based on the number of protons and neutrons in it. The radius of a typical nucleus, in terms of number of nucleons,
is R = A1/3r0 where A is the mass number and r0 is 1.25 fm, with deviations of 0.2 fm from this value.
P hysi cs | 25.3

Illustration 4: Nuclear radius of 16


8 O
is 3 × 10−15 m. Find the density of nuclear matter.  (JEE MAIN)
M
Sol: Considering the nucleus of the oxygen as a sphere of the uniform density ρ, the density can be given as ρ =
V
where M is the atomic mass number (convert it from amu to kg) of the oxygen and V is the volume of the sphere.

1.66 × 10−27 × 16
Use ρ =mass / volume = = 2.35 × 1017 kgm−3
( )
2
( 4 / 3) π 3 × 10 −15

2.5 Nuclear Spin and Magnetism


Many nuclides have an intrinsic nuclear angular momentum or spin and an associated intrinsic nuclear magnetic
moment. Although nuclear angular momenta are roughly of the same magnitude as the angular momenta of
atomic electrons, nuclear magnetic moments are much smaller than typical atomic magnetic moments.

2.6 Types of Nuclei


(a) Isotopes: Nuclei having same atomic number Z but different mass no. are called isotopes.
Ex. 11H , 21H , 31H
(b) Isobars: Nuclei having same mass number A but different atomic number Z are called isobars.
14 14
Ex. 6C
and 7N
.
(c) Isotones: Nuclei having same number of neutrons are called isotones Ex. 31H , 42He .

3. NUCLEAR STABILITY AND RADIOACTIVITY


Nuclear Stability means that nucleus is stable meaning that it does not spontaneously emit any kind of radioactivity
(radiation). On the other hand, if the nucleus is unstable (not stable), it has the tendency of emitting some kind of
radiation, i.e., it is radioactive. Therefore the radioactivity is associated with unstable nucleus:
Stable nucleus -> non-radioactive, Unstable nucleus –> radioactive

PLANCESS CONCEPTS

Keep in mind that less stable means more radioactive and more stable means less radioactive.
We want to know why there is radioactivity. What makes the nucleus a stable one? There are no concrete
theories to explain this but there are only general observations based on the available stable isotopes.
It appears that neutron to proton (n/p) ratio is the dominant factor in nuclear stability. This ratio is close
to 1 for atoms of elements with low atomic number and increase as the atomic number increases. Then
how do we predict the nuclear stability? One of the simplest ways of predicting the nuclear stability is
based on whether nucleus contains odd/even number of protons and neutrons:

Protons Neutrons Number of stable Nuclides Stability


Odd Odd 4
Odd Even 50 least stable
Even Odd 57 ↓
Even Even 167 most stable

•• Nuclides containing odd numbers of both protons and neutrons are the least stable means more
radioactive.
2 5 . 4 | Nuclear Physics and Radioactivity

PLANCESS CONCEPTS

•• Nuclides containing even numbers of both proton and neutrons are most stable means less
radioactive.
•• Nuclides contain odd number of protons and even numbers of neutrons are less stable than
nuclides containing even numbers of protons and odd numbers of neutrons.
In general, nuclear stability is greater for nuclides containing even numbers of protons and neutrons or
both.
Yashwanth Sandupatla (JEE 2012, AIR 821)

4. NUCLEAR FORCE
The force that controls the motions of atomic electrons is the familiar 40
electromagnetic force. To bind the nucleus together, however, there must be
n-p(system)
a strong attractive nuclear force of a totally different kind, strong enough to 20
overcome the repulsive force between the (positively charged) nuclear protons

U(r) (MeV)
and to bind both protons and neutrons into the tiny nuclear volume. The nuclear 0
1 2 3 4 5 6 7 8 r (frn)
force must also be of short range because its influence does not extend very far
beyond the nuclear “surface”. Its range is of the order of 2fm. The present view -20
is that the nuclear force that binds neutrons and protons in the nucleus is not a
-40
fundamental force of nature but is a secondary, or “spillover”, effect of the strong
force that binds quarks together to form neutrons and protons. In much the
-60
same way, the attractive force between certain neutral molecules is a spillover
effect of the Coulomb electric force that acts within each molecule to bind it Figure 25.1
together. This strong force is independent of the charge. This means that the
strong force of proton-proton, neutron-neutron, proton-neutron interactions is the same, apart from the additional
repulsive Coulomb force for the proton-proton interaction. It is customary to talk of the potential energy when
we talk of nuclear forces. Here, the potential energy of interaction of a proton and a neutron is shown in Fig 25.1.

4.1 Properties of Nuclear Forces


(a) These forces are attractive by nature. At very short distance s (< 0.7 fm) these become repulsive.
(b) The nuclear force is short range force. It means that it exist only when particles are very-very close to each
other. In nucleus the separation between particles is 10−15 m or I Fermi. At this infinitesimal small separation,
the nuclear force becomes 100 times stronger than the repulsive than the electric forces between the nucleons.
In the short range force, the force between the particles rapidly decreases. Thus the nuclear force only exists
in the nucleus.
(c) These forces do not obey inverse square law.
(d) Nuclear forces are not central forces. It means that these forces do not depend upon the center of one particle
to another particle.
(e) Strong nuclear forces are the strongest force in nature. In the given range of distance, the nuclear forces are
1038 times stronger than the gravitational forces.

5. MASS DEFECT
It has been observed that actual mass of the nucleus (determined by mass spectrometer of high resolving power)
is always less than the sum of masses of proton and neutrons in Free State.
P hysi cs | 25.5

m  Zmp + ( A − Z ) mn  − M , where Mp is mass of proton; mn is mass neutron; M is mass of nucleus


∆=

Illustration 5: Consider the decay of radium (A=226) atom into an alpha particle and radon (A=222). Then, what
is the mass defect of the reaction.
Mass of radium -226 atom = 226.0256u; Mass of radon -222 atom = 222.0715u and Mass if helium – 4 atom =
4.0026u  (JEE MAIN)
Sol: Mass defect is the difference in masses of parent and daughter nuclei. Mass defect is given by
∆m M(Ra226 ) − M(Rn222 ) − M(α )
=
∆m M(Ra226 ) − M(Rn222 ) − M(α ) = 226.0256 − 222.0175 − 4.00026 = 0.0053u
Mass defect=

6. BINDING ENERGY
It is defined as energy released during formation nucleus as a result of disappearance of mass i.e., mass defect.
( ∆m) c2
Binding energy = (∆m)c ;
2
Binding energy per nucleon=
A

Illustration 6: If mass equivalent to one mass of proton is completely converted into energy then determine the
energy produced?  (JEE MAIN)

Sol: When one proton is converted into its equivalent energy, the energy released during this conversion is given
by E = mc2
1.49 × 10−10
( )( )
2
mc2 =
E= 1.66 × 10−27 3 × 108 1.49 × 10−10 J =
J= MeV =
931.49 MeV  1amu = 931.49MeV
1.6 × 10−13

Variation of B.E. per nucleon with mass no. A 8


If the average binding energy per nucleon is calculated for all
nuclides and the results are plotted against A, the mass number, 6
Binding energy per

a graph shown in Fig. 25.2 is obtained.


nucleon in MeV

It is observed from the graph that binding energy per nucleon 4


(except for He4 , C12 and O16 ) rises first sharply and reaches a
maximum value 8.8 MeV in the neighborhood of A = 50 . The 2
curve falls very slowly after A = 50 and reaches at 8.4 MeV at
about A = 140 . For higher mass number, the energy decreases
0
to about 7.6 MeV. 20 40 60 80 100 120 140 160 180 200 220
Mass number A
Figure 25.2

Illustration 7: Binding energy per nucleon of an α-particle from the following data:
Mass of the helium nucleus = 4.001265amu; Mass of proton = 1.007277amu
Mass of neutron = 1.00866amu; (1amu=931.4812MeV)  (JEE MAIN)

Sol: The binding energy is given by B.E. =∆ m× c2 J = ∆ m× 931.5 Mev


Mass of two protons = 2 × 1.007277 =
2.014554amu
Mass of two neutron = 2 × 1.008666 =
2.017332amu
Total initial mass of two proton and neutrons = 2.014554 + 2.017332 = 4.031886amu
Mass defect
= ∆m 4.031816 − 4.001265,
= ∆m 0.030621amu
2 5 . 6 | Nuclear Physics and Radioactivity

∴ Binding energy of α particle = 0.030621× 931.4812 = 28.5221MeV


Binding energy of nucleon = 28.5221/4=7.10525Mev

PLANCESS CONCEPTS

The energy differences in allowed energy levels of a nucleus are generally large of the order of MeVs.
Hence, it is difficult to excite the nucleus by usual method of supplying energy as heat.
GV Abhinav (JEE 2012, AIR 329)

7. NEUTRON TO PROTON RATIO


According to Pauli Exclusion Principle, each quantum state can contain at most two protons or two neutrons that
too with opposite spin. Hence nuclear forces favor pairing of two protons and two neutrons together. In lighter
nuclei nuclear forces are dominant over coulomb repulsion and hence number of protons and number of neutrons
are nearly the same. In heavier nuclei the case is different, the interaction between nucleon pairs through nuclear
forces is not that effective and Coulomb repulsion dominates. Stability is achieved by having more neutrons as they
are neutral and don’t participate in Coulomb repulsion. That is why N/Z increases with atomic number for stable
209
nuclides. The heaviest stable nuclide is Bi . Bismuth in fact is of radioactive nature but the decay rate is so less
83
that it can be considered stable.

PLANCESS CONCEPTS
Having too many neutrons do not account for higher stability as many of these neutrons won’t have
pairing with protons. It will in fact decrease the stability.
The fact that the binding energy curve “drops” at both high and low mass numbers has very important
practical consequences.
Anurag Saraf (JEE 2011, AIR 226)
P hysi cs | 25.7

RADIOACTIVITY

1. INTRODUCTION
The phenomenon of spontaneous disintegration of nuclei of unstable atoms is defined as radioactivity. Generally
it is exhibited by atoms with A>192 and Z>82. It was discovered by Henry Becqurel. Lead isotope is the stable end
product of any natural radioactive series Radio activity is a nuclear process and not an atomic process Radioactivity
is not associated with the electron configuration of the atom.
Becquerel, in 1896, discovered accidently that uranium salt crystals emit an invisible radiation which affected a
photographic plate even though it was properly covered. Further investigations by Marie and Pierre Curie and
other workers showed that many other substances also emitted similar radiations. This property of spontaneous
emission of radiation is called radioactivity. Subsequent works, notably of Rutherford, suggested that radioactivity
was, in fact, due to decay or disintegration of unstable nuclei.
Emission of α particles: During α -particle emission atomic no. reduces by 2 while mass no. reduces by 4 i.e.
A A−4
Zx → Z −2 y + 42He

Emission of β-particle: When Nuclei has excess neutrons, it emits β -particle to bring n/p ratio into stable region. A
neutron gets converted into proton and β -particle, therefore atomic mass remains constant white atomic number
increases by 1.
γ-Radiation: After emission of α or β particle nuclei are left in excited state, Nucleus comes to stable state by
emitting electromagnetic radiation known as γ radiation. There is no change in A or Z during this process, α and
β emission don’t take place simultaneously while γ radiation can emit along with any of them.

1.1 Properties of Alpha, Beta and Gamma Rays


The comparison of the properties of α , β and γ rays are shown below in the table:

Properties a-rays b-rays g-rays

Nature photons Helium nucleus Fast moving electrons Electromagnetic waves


Nature of charge Positive Negative No change
Magnitude of charge 3.2 × 10–19 coulomb 1.6 × 10–19 coulomb Zero
Mass 6.6 × 10–27 kg 3.1 × 10–31 kg Rest mass zero
Velocity Between 1.4 × 107 m/sec 1% to 99% velocity of light 3 × 108 m/sec.
to 2.2 × 107 m/sec
Effect of electric & Deflected Deflected Not deflected
magnetic fields
Range 2.7to 8.62 cm in air or 1/100 5mm of Al or 1mm of lead 30 cm of iron
mm of Al
Penetrating power Minimum 100 times of α -rays 1000 times of α -rays
Ionising power Maximum Lesser Minimum

1.2 Natural Radioactivity


Natural Radioactivity is the spontaneous disintegration of an unstable atomic nucleus and the emission of particles
or electromagnetic radiation. All naturally occurring elements with atomic numbers greater than 83 as well as some
isotopes of lighter elements show natural radioactivity.
2 5 . 8 | Nuclear Physics and Radioactivity

1.3 Artificial Radioactivity


Radioactivity produced in a substance by bombardment with high-speed particles (as protons or neutrons), also
called as induced radioactivity.

1.4 Parent and Daughter Nuclei


Nucleus which decays in a radioactive decay is called parent nucleus. This parent nucleus transforms to an atom
with a nucleus in a different state, or to a different nucleus containing different numbers of protons and neutrons.
Either of these products is named the daughter nucleus.

1.5 Law of Radioactive Disintegration


(a) Radioactivity is a process in which nuclei of certain elements undergo spontaneous disintegration without
excitation by any external means.
(b) The radioactivity results the emission of powerful radiations known as Alpha ( α ) , Beta ( β ) and Gamma ( γ )
rays.
(c) Radioactivity is a nuclear phenomenon i.e. it is not depend upon no. of electrons present in outer shell.
It was studied by Rutherford and Soddy in 1902. The disintegration of nuclei is purely statistical which means
all nuclei take different time to disintegrate and are independent for radioactive decay. Rate of disintegration is
dN
directly proportional to no. of not decayed nuclei present at that time, i.e. − ∝N=λN  … (i)
dt
Where λ is disintegration or decay constant.
Integrating equation (i)
loge N = −λt + C
 at, t = 0 , N = N0 ⇒ C = loge N
⇒ loge N = −λt + loge N0 or N = N0 e−λt  … (ii)
Equation (ii) shows that no. of nuclei of given radioactive substance decreases exponentially with time. It also shows
that decays occurs rapidly initially and rate of decay decreases with time.
Half-life ( T1/2 ) : The time in which half of radioactive substance decays is known as half-life.
N0 N0 −λT1/2 loge 2 0.693
or t = T1/2 , N = ; ⇒ = N0 e ; ⇒=
T1/2 =  ... (iii)
2 2 λ λ
If t = nT1/2 where n is integer, equation (ii) reduces to N = N0 (1/ 2 )
n

Illustration 8: A count-rate meter is used to measure the activity of a given sample. At one instant the meter shows
4750 counts per minute. Five minutes later it shows 2700 counts per minute. Find:
(a) Decay constant (b) the half-life of the sample. (JEE MAIN)
loge 10 N0
Sol: The decay constant of radioactive element is given by λ = where N0 is the number of radioactive
log
t Nt
nuclei at t=0 and Nt is the number of radioactive nuclei at time t. The half-life of the radioactive element is
0.693
t1/2 = .
λ
Initial activity, A0 = dN / dt at t = 0
Final activity, A t = dN / dt at t = t

dN dN 4750 N0
= IN0 & = INt ; =
dt t =0 dt t =5 2700 Nt
P hysi cs | 25.9

N0 4750 2.303 4750


Using λt =2.303log ; λ (5 ) =
2.303 log = ; λ = log 0.1129 min−1
Nt 2700 5 2700
0.693
t1/2
= = 6.14 min
0.1129

Mean life ( τ ) : Mean life of radioactive substance is defined as sum of life times of all radioactive nuclei divided by
total no. of nuclei.

or τ
=
∫=t dN ∫ tdN

1
or τ =  … (iv)
∫ dN N0 λ

= τ ; N N=
if t = ( ) 0.37N
−λ 1/ λ
0e 0

i.e., In mean life radioactive substance decays by nearly 63%.


T1/2 0.693 τ 
From (iii) and (iv) = … (v)

Illustration 9: The mean lives of a radioactive substance are 1620 and 405 years for α emission and β emission
respectively. Find out the time during which three fourth of a sample will decay if it its decaying both the α emission
and β emission simultaneously. (JEE ADVANCED)
Sol: When substance decays by α and β emission simultaneously, the average rate of λav disintegration is given
by λav = λ α + λβ ; Where λ α and λβ are disintegration constant for α emission and β emission respectively. The
N
average time of the disintegration is given by λav tav =2.303 log o where No is the number of atoms present at
Nt
time t=0 s. And Nt is the number of disintegration atoms present at the time t s.
1 1 1 1 1
Mean life is given by: τm = 1/ λ ; λav = λ α + λβ ; = + = + = 3.08 × 10−3
τav τα τβ 1620 405
100 100
⇒ λav t 2.303 log = ; (3.08 × 10−3 ) t 2.303 log
25 25
1
⇒ t=
2.303 × log 4 =
499.24 years
3.08 × 10−3

Activity of a Radioactive Isotope: The activity of a radioactive substance (or radioisotope) means the rate of
dN
decay per second or the number of nuclei disintegrating per second. It is generally denoted by A. ⇒ A =
dt

If a time t=0 sec, the activity of a radioactive substance is A ο and after time t=t sec it is observed to be A t , then:
dN dN
Aο = = λN0 A t = = λN
dt t =0 dt t =t t

Units of Rate of Decay or Activity: A number of units have been used to express the activity of a radioactive
sample. The more commonly used ones are the following:
(a) Curie (Ci): The activity of a radioactive sample is said to be one curie when 3.7 × 1010 decays take place per
second. Thus 1Ci ≡ 3.7 × 1010 decays / s
This is the approximate activity of 1 g of radium. In practice, the smaller units milli curie and micro curie are
used. 1 mCi ≡ 3.7 × 107 decays / s ; 1 µ Ci ≡ 3.7 × 10 4 decays / s
(b) Becquerel (Bq): The Si unit of activity is called the Becquerel and it represents 1 decay per second. Thus 1 Bq
= 1 decay/s We thus have 1Ci ≡ 3.7 × 1010 Bq
2 5 . 1 0 | Nuclear Physics and Radioactivity

(c) Rutherford (Rd): Another unit for activity is Rutherford and it represents 106 decays per second.
1Rd = 106 decays / s

Illustration 10: Radioisotopes of phosphorus P32 and P38 are mixed in the ratio 2:1 of atoms. The activity of the
sample is 2 m Ci. Find the activity of the sample after 30 days, t1/2 of P32 is 14 days and, t1/2 of P38 is 25 days. 
 (JEE ADVANCED)
Sol: When the radio isotopes are mixed in the proportion 2:1, the compound activity of mixture over time t is given
0.693 × N
by A
= t A1t + A2t . The activity A of any radioactive substance with half-life τ is defined as A =λN = .
τ
Let A ο be the initial activity of the sample,
Let A1ο be initial activity of isotope 1 and A2ο be the initial activity of sample 2
A
= ο A1ο + A2ο
−λ t −λ t
Similarly for final activity (Activity after time t), A t =
A1t + A2t At =
A1ο e 1 + A2ο e 2
Now in the given equation Ao = 2 m Ci ⇒ A0 = A1ο + A2ο = 2 m Ci  ... (i)
Initial ratio of atoms of isotopes=2:1
We know from definition of activity, A = λ N here λ is the decay constant and N is number of radioactive nuclei
present at time instant t s.
A1ο N1ο T2 A1ο 2 25 25
= × where T represents half-life; = × =  … (ii)
A2ο N2ο T1 A 2ο 1 14 7
−λ1t −λ2t
On solving equation (i) and (ii), we get, A1ο = 25 / 16 and A2ο = 7 / 16 =
; A t A1ο e + A 2ο e

 0.693 × 30 
How to solve expression like this? For example, consider the first exponential term exp  − =e−1. 485
 14 
Let y = e−1.485 Therefore, ln y = −1.485 ; log y = − (1.485 / 2.303
= ) y antilog ( −1.485 / 2.303)
−x
So, from above calculations you can derive a general result i.e. e− x = antilog
2.303
25 7
At = × 0.2265 + × 0.4353 = 0.5444 Ci .
16 16

Important Formulae

(a) N = Nο e−λt (b) A = A ο e−λt


N 
2.3027log10  ο 
(c) M = Mο e−λt (d) λ =  N 
t

A  M 
2.3027log10  ο  2.3027log10  ο 
(e) λ =  A  (f) λ =  M 
t t
τα τβ
(g) λ = λ α + λβ (h) τ = (when two particles decay simultaneously)
τα + τβ

Nο Nο Aο
(i) =
N + ( j) A =
n  T   T 
2    
T  T 
2 1/2  2 1/2 
P hysi cs | 25.11

Mο
(k) M =
 T 
 
T 
2 1/2 

2. ALPHA DECAY
In alpha decay, the unstable nucleus emits an alpha particle reducing its proton number Z as well as its neutron N
by 2. The alpha decay process may be represented as
A A−4
ZX → Z −2Y + 42He .

As the proton number Z is changed, the element itself is changed and hence the chemical symbol of the residual
nucleus is different from that of the original nucleus. The nucleus before the decay is called the parent nucleus and
resulting after the decay is called the daughter nucleus. An example of alpha decay is 212 208 4
83Bi → 81Tl + 2 He
.
(a) Characteristics of α -decay:
(i) The spectrum of α -particles is a discrete line spectrum.
(ii) Spectrum of α -particles has fine structure i.e. every spectral line consists of a number of fine lines.
(iii) The α - emitting nuclei have discrete energy levels i.e., energy levels in nuclei are analogous to discrete
energy levels in atoms.
(iv) α -decay is explained on the basis of tunnel effect.
(v) Geiger-Muller law- loge λ= A + Bloge R For radioactive series B is same whereas A is different
(b) Size of the nucleus decreases by α emission

3. BETA DECAY
Beta Decay: Beta decay is a process in which either a neutron is converted into a proton or a proton is converted
into a neutron. Thus, the ratio N/Z is altered in beta decay. If a nucleus is formed with more number of neutrons
than needed for stability, a neutron will convert itself into a proton to move towards stability. Similarly, if a nucleus
is formed with more number of protons than needed for stability, a proton will convert itself into a neutron. Such
transformations take place because of weak forces operating within a neutron or a proton. When a neutron is
converted into a proton, an electron and a new particle named antineutrino are created and emitted from the
nucleus n → p + e + v

3.1 Characteristics β-Decay


(a) The energy spectrum of β -particles is continuous i.e. β -particles of all energies up to a certain maximum are
emitted.
(b) The number of such β -particles is maximum whose energy is equal to the maximum probable energy i.e. at
E = Emp , NB =maximum.
(c) There is a characteristic maximum value of energy in the spectrum of β -particles which is known as the end
point energy (E0 ) .
(d) In β -decay process, a neutron is converted into proton or proton is converted into neutron.


1
0
n
= 1
1
p+ 0
−1
e β particle 1=
p
1
1
0
n + 0e
1
(β particle)
+

(e) The energy of β -particles emitted by the same radioactive material may be same or different.
(f) The number of β -particles with energy E = E0 (end point energy) is zero.
2 5 . 1 2 | Nuclear Physics and Radioactivity

4. GAMMA DECAY
In gamma decay, a nucleus changes from a higher energy state to a lower energy state through the emission of
electromagnetic radiation (photons). The number of protons (and neutrons) in the nucleus does not change in this
process, so the parent and daughter atoms are the same chemical element. In the gamma decay of a nucleus, the
emitted photon and recoiling nucleus each have a well-defined energy after the decay. The characteristic energy is
divided between only two particles. The process is similar to that in a hydrogen atom when an electron jumps from
a higher energy orbit to a lower energy orbit emitting a photon.

4.1 Characteristics γ-Decay


(a) The spectrum of γ -rays is a discrete line spectrum.
(b) Whenever α or β -particles is emitted by a nucleus then the daughter nucleus is left in the excited state. It
suddenly makes transition in the ground state thereby emitting γ -rays.
(c) Knowledge about nuclear energy levels is obtained by γ -spectrum.
(d) γ -rays interact with matter as a consequence of which the phenomena of photoelectric effect, Compton
Effect and pair production happen. (At low energy photoelectric effect and at high energy pair-production is
effective).

5. RADIOACTIVE SERIES
(a) Elements beyond Bismuth are all radioactive in nature. These radioactive elements disintegrate to give new
elements which further disintegrate to form other elements and so on. The process is continued till a non-
radioactive end product is reached.
(b) The whole chain of such elements starting from the parent radioactive elements to the end non-radioactive
element is called “radioactive series or a family.”
( 4n + 1) is artificial series & 4n, ( 4n + 2 ) , ( 4n + 3) are natural series.

S.No. Series Name of the series Initial Final element Nature of No of α & β
element series particles emitted
1. 4n+2 Uranium series 238 206 Natural
U pb 8α ,6β
92 82

2. 4n+3 Actinium series 235 207 Natural


U pb 7α , 4β
92 82

3. 4n Thorium series 232 208 Natural


Th Pb 6α , 4β
90 82

4. 4n+1 Neptunium series 237 209 Artificial 7α , 4β


Np Bi
93 83
P hysi cs | 25.13

5.1 Thorium Series

232 - 228 - 228


(I) Th Ra Ac
90 88 89

228 - 224 - 220


(II) Th Ra Rn
90 88 89

208 216 -
Ti Ac
- 81 - - -
85
208 212 216
Pb Bi Po
82 83 84
212 - - 212 -
- Po Pb
84 82

Figure 25.3

5.2 Uranium Series


238 - 234 - 234 -
U Th Ac
92 90 91
234 - 230 - 226
U Th Ac
92 90 88
-

222(radon)
Rn
86

-
210 214
- Ti - - Pb -
81 214 85
210 218
Pb Bi Po
82 214 83 218 84
- Po - - At -
84 85

-
206
- Ti -
81 206
210
Bi Bi
83 210 82
- Po -
84

Figure 25.4
2 5 . 1 4 | Nuclear Physics and Radioactivity

146 U238
144 Th234
Pa234
142 U234
140 Th230

138 Ra226
136 Rn222
134 Po218
Pt218
132 Pb214 Pn218
Bi214
130 Pb214
Pl210
128 Pb210
Bi210
126 Po210
Ti206
124 Bd206 (Stable nucleus)
Z
81 82 83 84 85 86 87 88 89 90 91 92

Figure 25.5: Uranium series

The mass number of each element in the series is equal to 4n+3. Where n is a positive integer.

5.3 Actinum Series


235 - 231 - 231 -
U 90Th 91Pa
223
87 Fr
- -
227 223 -
89 Ac 88 Ra
- -
227
90Th
219 (Action) - 215 -
Rn 84 Po
86
207
81 Ti
- -
207 211
Pb Bi
83
82 - 211
- 82 Pb
-
211
84 Po

Figure 25.6
P hysi cs | 25.15

PLANCESS CONCEPTS

(a) In all series one element of zero group is present (atomic no=86) in gaseous state which is called
emanation.
208
(b) In all series last product is an isotopes of lead Pb , 206 Pb , 207 Pb respectively.
Pb is found in nature as a mixture of these three isotopes.
(c) The (4n+1) series (Neptunium series):-
• Except the last member all other members of this series have been obtained by artificial
means.
• The series does not contain gaseous emanation.
• The last member of the series is an isotope of Bi and not an isotope of Pb.
Vijay Senapathi (JEE 2011, AIR 71)

6. ELECTRON CAPTURE
Electron capture is a process in which a proton-rich nuclide absorbs an inner atomic electron, thereby changing
a nuclear proton to a neutron and simultaneously causing the emission of an electron neutrino. Various photon
emissions follow, as the energy of the atom falls to the ground state of the new nuclide.
Electron capture is the primary decay mode for isotopes with a relative superabundance of protons in the nucleus,
but with insufficient energy difference between the isotope and its prospective daughter (the isobar with one less
positive charge) for the nuclide to decay by emitting a positron. Electron capture is an alternate decay mode for
radioactive isotopes with sufficient energy to decay by positron emission. It is sometimes called inverse beta decay,
through this term can also refer to the interaction of an electron anti-neutron with a proton.
A free proton cannot normally be changed to a free neutron by this process the proton and neutron must be part
of a larger nucleus. In the process of electron capture, one of the orbital electrons, usually from the K or L electron
shell (K-electron capture, also K-capture, or L-electron capture, L-capture) is captured by a proton in the nucleus
forming a neutron and emitting an electron neutrino.

6.1 Calculation of Number of Alpha and Beta Particles Emitted


m m'
Consider the following general reaction. nX → n'Y + a24 α + b0−1β
Then, m = m'+ 4a + 0b (ii) n =n'+ 2a − b
Solve for a and b
A A
Where a is the number of 42He emitted and b is the number of 0
−1β
emitted ZX →Z1 Y + x 24 α + y β
1
x : no of α -particles emitted y : not of β -particles emitted
1 A − A1
X AZ → Y A1 + xHe24 + ye0−1 ; =
A A1 + 4x ; x=
Z 4
 A − A1 
Z = Z1 + 2x − y y = Z1 − Z + 2x
= y 
 2 
 
(
 − Z − Z1 )
 A − A1   238 − 206 
eg: U238 206 4 0
92 → Pb82 + xHe2 + ye−1
; x=  =  = 8α − particles
 4   4 
 
2 5 . 1 6 | Nuclear Physics and Radioactivity

 A − A1 
y= 
 2 
 
(
 − Z − Z1 = 
 2
)
 238 − 206 
 − ( 92 − 82 ) = 16 − 10 = 6β − particles

7. GROUP DISPLACEMENT LAW


Law is given by Fajan, Soddy and Russel.
The law is basically given for the position of daughter elements in periodic table.
(a)  α-particle emission: When an alpha particle emits the position of daughter element is two places to the left
in the periodic table from parent element.
214 210 213 209
Example: Po → Pb Bi → Tl
84 82 83 81

16   14 15   13


( VIA ) (IVA ) ( VA ) (IIIA )
(b)  β-particle emission: When an β -particle emits the position of daughter element is one place right in the
periodic table from parent element.
27 27 0 14 14 0
Examples: Mg → Al + e C → N + e
12 13 −1 6 7 −1


(IIA ) (IIIA ) (IV A ) ( VA )

8. RADIOACTIVE ISOTOPES
The isotopes of elements which spontaneously decay by emitting radioactivity radiations are defined as radioactive
isotopes.
They are two types.
(a) Natural radioactive isotopes (b) Artificial radioactive isotopes
(b) Natural radioactive isotopes: Those radioactive isotopes which exist naturally are known as natural radioactive
isotopes. e.g. Th232 , Pu240 etc.
(c) Artificial radioactive isotopes: Those isotopes, which are prepared artificially by bombarding fundamental
particles like α , β , γ , p, n etc. no matter, are known as artificial isotopes.

8.1 Uses of Radioactive Isotopes


(a) In Medicine:
(i) For testing blood chromium-51
(ii) For testing blood circulation-Sodium-24
(iii) For detecting brain tumor-Radio mercury-203
(iv) For detecting fault in thyroid gland-Radio iodine-131
(v) For cancer-Cobalt-60
(vi) For blood-Gold_189
(vii) For skin diseases-Phosphorous-31
(b) In Archaeology:
(i) For determining age of archaeological sample (Carbon dating) - C14
P hysi cs | 25.17

(ii) For determining age of meteorites - K 40


(iii) For determining age of earth and isotopes
(c) In Agriculture:
(i) For protecting potato crop from earthworm-Cobalt-60
(ii) For artificial rains Agl
(iii) As fertilizers-Phosphorous-32
(d) As Tracers:
Very small quantity of radio isotopes present in mixture is known as tracer. Tracer technique is used for
studying biochemical reactions in trees and animals.
(e) In Industries:
(i) For detecting leakage in oil or water pipe lines.
(ii) For testing machine parts.
(f) In Research:
(i) In the study of carbon-nitrogen cycle.
(ii) For determining the age of planets.

8.2 Radioactive Dating


Radioactive dating also called carbon dating is used to estimate the age of organic samples. The technique is based
14 14
on the β -activity of the ratio-isotope C14 . C→ Ν + β− + ν
6 7

High energy particles from outer space, called cosmic rays, induce nuclear reactions in the upper atmosphere and
create carbon-14. The carbon dioxide molecule of the earth’s atmosphere has a constant ratio (≈1.3 × 10−12 ) of
C14 and C12 isotope. All living organisms also show the same the same ratio as they continuously exchange CO2
with their surroundings. However, after its death, an organism can no longer absorb CO2 and the ratio C14 / C12
decrease due to the β -decay of C14 Thus by measuring the β -activity per unit mass, it is possible to estimate the
age of a material.
Using such techniques samples of wood, sample of wood, charcoal, bone, etc., have been identified to have lived
from 1000 to 25000 years ago.

9. PROPERTIES AND USES OF NUCLEAR RADIATION

9.1 Alpha Ray


(a) It is a stream of alpha particles, each particle containing two protons and two neutrons. An alpha particle is
nothing but a helium nucleus.
(b) Being made of positively charged particles, alpha ray can be deflected by an electric field as well as by a
magnetic field.
(c) Its penetrating power is low. Even in air, its intensity falls down to very small values within a few centimeters.
(d) Alpha rays coming from radioactive materials travel at large speeds of the order of 106 ms−1
(e) All the alpha particles coming from a particular decay scheme have the same energy.
2 5 . 1 8 | Nuclear Physics and Radioactivity

(f) Alpha ray produces scintillation (flashes of light) when it strikes certain fluorescent materials, such as barium
platinocyanide.
(g) It causes ionization in gases.

9.2 Beta Ray


(a) It is a stream of electrons coming from the nuclei. Thus, the properties of beta ray, cathode ray, thermions,
photoelectrons, etc., are all identical except for their origin. Beta particles are created at the time of nuclear
transformation, whereas, in cathode ray, thermions, etc., the electrons are already present and get ejected.
(b) Being made of negatively charged particles, beta ray can be deflected by an electric field as well as by a
magnetic field.
(c) Its penetrating power is greater than that of alpha ray. Typically, it can travel several meters in air before its
intensity drops to small values.
(d) The ionizing power is less than that of alpha rays.
(e) Beta ray also produces scintillation in fluorescent materials, but the scintillation is weak.
(f) The energy of the beta particles coming from the same decay scheme are not equal. This is because the
available energy is shared by antineutrinos. The energy of beta particles thus varies between zero and a
maximum.

9.3 Beta-Plus Ray


Beta-plus ray has all the properties of beta ray, except that it is made of positively charged particles.

9.4 Gamma Ray


(a) Gamma ray is an electromagnetic radiation of short wavelength. Its wavelength is, in general, smaller than
X-rays. Many of its properties are the same as those of X-rays.
(b) Being charge less, it is not deflected by electric or magnetic field.
(c) It has the least ionizing power and the largest penetrating power among different types of nuclear radiation.
(d) All the photons coming from a particular gamma decay scheme have the same energy.
(e) Being an electromagnetic wave, gamma ray travels in vacuum with the velocity c.

10. NUCLEAR FISSION


It was first observed by German Scientist Otto Hahn and Fritz Strassmann in 1983 in nuclear fission heavy nucleus
splits into two smaller nuclei with liberation of energy. When uranium with Z=92 is bombarded with neutrons, it
splits into two fragments namely barium (Z=56) and krypton (Z=36) and a large amount of energy is released
238 1 138 88
which appears due to decrease in the mass. The reaction is represented as 92 U + 0 n → 56 Ba + 36 Kr + 3 01n + energy
The disintegration process in which heavy nucleus after capturing a neutron splits up into nuclei of nearly equal
mass is called nuclear fission.
Energy released in nuclear fission: The amount of energy released in nuclear fission may be obtained by the
method of mass defect. For example consider the fission of U 235 (Z=92) into Ba 141 (Z=56) and Kr 92 (Z=36) by slow
neutrons. The reaction is given by
235 1 236 141 92 1
U+ n → U→ Ba + Kr + 3 n + Q
92 0 92 56 36 0

Let us estimate the actual masses before and after the fission reactions.
P hysi cs | 25.19

Actual mass before fission reaction


Mass of uranium nucleus 235.124 a. m. u
Mass of neutron 1.009 a. m. u
∴ Total mass 236.133 a. m. u
Actual mass after the fission reaction
Mass of barium nucleus 140.958 a. m. u
Mass of krypton nucleus 91.926 a. m. u
Mass of three neutrons 3.026 a. m. u
∴ Total mass 235.910 a. m. u
Now mass decrease during nuclear reaction = 236.133-235.910=0.223 a. m. u.
∴ Corresponding energy release =0.223 × 931 =200 MeV
If we calculate the energy produced by one gm of uranium its comes out to be
2.28 × 10 4 k. w. h. = 22.8 M watt.
This shows that 1 kg of U236 would give power of 1 M watt for more than two years.

PLANCESS CONCEPTS

The drooping of the binding energy curve at high mass numbers tells us that nucleons are more tightly
bounded when they are assembled into two middle-mass nuclei rather than a single high-mass nucleus.
In other words, energy can be released in the nuclear fission.
Shrikant Nagori (JEE 2009, AIR 30)

10.1 Chain Reaction


A nuclear chain reaction occurs when one nuclear reaction causes an average of one or more nuclear reactions,
thus leading to a self-propagation series of these reactions. The specific nuclear chain reaction releases several
million times more energy per reaction that any chemical reaction.

Energy = E1 Energy = E2 Energy = E3


E2 > E1 > E2

Figure 25.7

E2
10.1.1 Fission Chain Reaction
Fission chain reaction occurs because
of interactions between neutrons and E1
fissile isotopes (such as 235U ). The
Energy
chain reaction requires both the release
released
of neutrons from fissile isotopes
E3
undergoing nuclear fission and the
subsequent absorption of some of Initial Intermediate Final
these neutrons in nuclear fission, a few Figure 25.8
2 5 . 2 0 | Nuclear Physics and Radioactivity

neutrons (the exact number depends on several factors) are ejected from the reaction. These free neutrons will then
interact with the surrounding medium, and if more fissile fuel is present, some may be absorbed and cause more
fission. Thus, the cycle repeats to give a reaction that is self-sustaining.

10.2 Nuclear Reactor


Chain reaction

A nuclear reactor is a device to initiate


and control a sustained nuclear chain
reaction. Nuclear reactors are used
at nuclear power plants for electricity
generation and in propulsion of ships.
Heat from nuclear fission is passed to
a working fluid (water or gas), which
runs through turbines. These either
drive a ship’s propellers or turn electrical
generators. Nuclear generated steam
in principle can be used for industrial
process heat or for district heating. Let
1. A neutron is 2. The uranium nucleus 3. The chain reaction begins :
us see the working of a typical Uranium about to hit splits (fissions) into several those neutrons hit other
nuclear reactor. The volume in the core the nucleus of a smaller atoms, releasing heal nuclei, casing them to
is filled with low-Z material like, D2O uranium atom. and several more neutrons. fission. And so on.
graphite, beryllium etc. This material is
Figure 25.9
called moderator.
When fission takes place in a uranium rod, most of the fast neutrons produced escape from the rod and enter into
the moderator. These neutrons make collisions with the particles of the moderator and thus slow down. About 25
collisions with deuteron (present in heavy water) or 100 collisions with carbon or beryllium are sufficient to slow
down a neutron from 2 MeV to thermal energies. The distances between the rods are adjusted in such a way that
a neutron coming from one rod is generally slowed down to thermal energies before entering the other rod. This
eliminates the possibility of a neutron being absorbed by U238 in 1-100 eV regions. The geometry of the core is
such that out of the average 2.5 neutrons produced per fission, 1 neutron is used to trigger the next fission and
the remaining are lost without triggering any fission. The reaction is then sustained at a constant rate. If the rate of
the loss of neutrons is decreased further, the fission rate will keep on increasing which may lead to explosion. If the
rate of loss of neutrons is increased, the rate of fission will keep on decreasing and ultimately the chain reaction will
stop. The finer control of fission rate is made by the control rods which are made of cadmium and are inserted up
to a certain depth in the moderator. Cadmium is a very good neutron absorber. If the stage is set for stable chain
reaction and the cadmium rods are pushed into the moderator, the reactor will be shut off. Pulling the cadmium
rods out will start the reactor.

Cold water
Pump
Control Turbine
rod Moderator Steam Electric power

Generator

Cold water

Pump Hot water


Condenser
Uranium Water
rod
Hot water

Figure 25.10
P hysi cs | 25.21

10.3 Uranium Fission Reactor


236
The most attractive bid, from a practical point of view, to achieve energy from nuclear fission is to use 92 U
as
the fission material. This nuclide is highly fissionable and hence is not found in nature. Natural uranium contains
238 235
about 99.3% of 92 U
and 0.7% of 92 U
. The technique is to hit a uranium sample by slow-moving neutrons (kinetic
235
energy≈0.04eV, also called thermal neutrons). A 92 U
nucleus has large probability of absorbing a slow neutron
236
and forming 92 U
nucleus. This nucleus then fissions into two parts. A variety of combinations of the middle-
236 137 97
weight nuclei may be formed due to the fission. For example, one may have 92 U → 53I + 39Y + 2n,

236 140 94
And a number of the other combination 92 U → 56 Ba + 36 Kr + 2n

10.4 Breeder Reactors


Although fission generates large amount of energy and the world is heavily depending on fission for its energy
requirement, uranium resources are also limited. The following Table shows that fission can easily take place with
240
Pu besides 236 U. But 239
Pu is not a naturally occurring isotope. However, 238
U can capture a neutron to produce
239
Pu which can be used as a nuclear fuel.
238 239 β β
92 U + n → 92 U  → 239
93Np  → 239
94 Pu

238
Suppose, used uranium rods, which contain only U , are kept in or around a uranium-reactor core. Also suppose,
238
the geometry is such that out of the average 2.5 neutrons produced in fission, one neutron is absorbed by a U
239 239
nucleus in these rods resulting in Pu . Then we produce as much nucleus in these rods resulting in Pu . Then
239 235
we produce as much nuclear fuel in the form of Pu as we consume in the form of U . If more than one
238
neutron can be absorbed by these U rods per fission then we produce more fuel than what we consume. Thus,
apart from nuclear energy, these reactors give us fresh nuclear fuel which often exceeds the nuclear fuel used. Such
a reactor is called a breeder reactor.

11. NUCLEAR FUSION


Binding energy vs. Mass Number a graph shows that when lighter nuclei with A<20 combine to form bigger nuclei
binding energy per nucleon increases. The total binding energy of the product is less than total binding energy of
reactants resulting in release of energy. This process of combining of two lighter nuclei into bigger one is known
as nuclear fusion. i.e.
2 2
1H + 1H → 42He + 24MeV

The following points deserve particular attention concerning nuclear fusion.


(a) The energy 21.62 MeV released in one fusion event is much smaller than about 200 MeV released in one
fission event. This does not mean that fusion is a weaker source of energy than fission. If we compare the
21.62 MeV
energy released per unit mass, we find that one fusion event is accompanied by a release of or 3.6
6 amu
200 MeV
MeV Per amu as against or 0.85 MeV per amu released in one fission.
235
(b) For fusion, positively charged nuclei have to come very close to each other. This requires a very high energy
to be provided to the fusing nuclei to enable them to overcome the strong electrostatic repulsion between
them. Calculations show that the necessary energy can be provided by raising the temperature to about 108
2 5 . 2 2 | Nuclear Physics and Radioactivity

K. Such high temperature can be produced by first inducing a fission event. A fusion reaction is therefore also
called a thermonuclear reaction. This is the basic hydrogen bomb.
(c) Unlike the highly radioactive fission fragments, the end product of the fusion of hydrogen nuclei is safe, non-
radioactive helium.
(d) Unfortunately a sustained and controllable fusion reactor that can deliver a net power output is not yet a
reality. A great deal of effort is currently under way to resolve various difficulties in the development of a
successful device. Nevertheless controlled fusion is regarded as the ultimate energy source because of the
abundant availability of its main fuel: water.

PLANCESS CONCEPTS

The drooping of the binding energy curve at low mass number tells us that energy will be released if
two nuclei of small mass numbers combine to form a single middle-mass nucleus. This is nuclear fusion.
Ankit Rathore (JEE Advanced 2013, AIR 158)

2
Illustration 11: In the nuclear fusion reaction: H + 4 He ⇒ 2 3H in a nuclear reactor, of 200 MW rating. If the
1 2 1

energy from above reaction is used with a 25% efficiency in the reactor, how many grams of deuterium will be

needed per day? (The masses of 3 H and 4 He are 2.0141 and 4.0026 amu respectively) (JEE ADVANCED)
1 2
Sol: The energy absorbed during the nuclear fusion reaction is calculated using Q value equation i.e.
Q =−mc2 =−m × (931) MeV . The number of deuterium atoms required during this reaction is obtained by

Power Required
N= .
Efficiency × (energy released from one fusion reaction)

Let us first calculate the Q value of nuclear function, Q =−mc2 =−m × (931) MeV
Q=
(2 × 2.0141 − 4.0026) × 931MeV = 23.834 × 106 eV. Now efficiency of reactor is 25%
23.834MeV =
So effective energy used = 0.25 × 23.834 × 106 × 1.6 × 10−19=
J 9.534 × 10−13 J
Now 9.534 × 10−13 J energy is released by fusion of 2 deuterium.
= 200 × 106 J / s per second= 200 × 106 × 86400 J / s for 1 days.
Requirement is 200MW

200 × 106 × 86400


No. of deuterium nuclei required= = 3.624 × 1025
9.534
× 10−13
2
g g
Number of deuterium nuclei = × 6 × 1023 ; 3.624 × 1025 = × 6 × 1023
2 2
2 × 3.624 × 1025
=g = 120.83 gm / day.
6 × 1023

11.1 Thermonuclear Fusion


To generate useful amount of energy, nuclear fusion must occur in bulk matter. The best hope for bringing this
about is to raise the temperature of the material until the particles have enough energy-due to their thermal
motions alone-to penetrate the Coulomb barrier. We call this process thermonuclear fusion.
In thermonuclear studies, temperatures are reported in terms of the kinetic energy K of interacting particles via the
relation K=3kT/2
P hysi cs | 25.23

In which K is the average kinetic energy of the interacting particles, k is the Boltzmann constant, and the temperature
T is in kelvins. Thus, rather than saying,
Fusion in Sun: Among the celestial bodies in which energy is produced, the sun is relatively cooler. There are stars
with temperature around 108 K inside. In sun and other stars, where the temperature is less than or around 107 K ,
fusion takes place dominantly by proton-proton cycle as follows:
1
H + 1H → 2H + e+ + ν
2
H + 1H → 3He + γ
3
He + 3He → 4 He + 2 1H
4 1H → 4 He + 2e+ + 2ν + 2γ

Note that the first two reactions should occur twice to produce two 3He nuclei and initiate the third reaction. As
a result of this cycle, effectively, four hydrogen nuclei combine to form a helium nucleus. About 26.7 MeV energy
is released in the cycle. Thus, hydrogen is fuel which ‘burns’ into helium to release energy. The sun is estimated to
have been radiating energy for the last 4.5 × 109 years and will continue to do so till all the hydrogen in it is used
up. It is estimated that the present store of hydrogen in the sun is sufficient for the next 5 × 109 years.

11.2 Lawson Criterion


J. D. Lawson showed that in order to get an energy output greater than the energy input, a fusion reactor should
achieve nτ > 1014 scm−3
Where n is the density of the interacting particles and τ is the confinement time. The quantity nτ in scm−3 is called
Lawson number.
The ratio of the energy output to the energy input is known as Q of the fusion machine. For a viable fusion machine,
Q should be greater than 1.

11.3 Tokamak Design


In one of the method receiving serious attention, one uses
the so-called Tokamak design. The deuterium plasma is
contained in a toroidal region by specially designed magnetic
field. The directions and magnitudes of the magnetic field are
so managed in the toroidal space that whenever a charged
→ →
plasma particle attempts to go out, the q ν× B force tends to
push it back into the toroidal volume. It is a difficult task to
design a magnetic field which will push the particles moving
in random directions with random speeds into a specified
volume, but it is possible and has been done. The plasma is, Path of the ion
therefore, confined by the magnetic field. Such confinement
has been achieved for short durations (≈few microseconds) Figure 25.11
in which some fusion occur. Fusion thus proceeds in bursts
or pulse. The heating is accomplished by passing high frequency oscillating current through the plasma gas. A
schematic design is shown in Fig. 25.11.

11.4 Inertial Confinement


In another method known as inertial confinement, laser beams are used to confine the plasma. A small solid pellet
is made which contains deuterium and tritium. Intense laser beams are directed on the pellet from many directions
distributed over all sides. The laser first vaporizes the pellet converting it into plasma and then compresses it from
all directions because of the large pressure exerted. The density increases by 103 to10 4 times the initial density
and the temperature rises to high values. The fusion occurs in this period. The α -particles (He Nuclei) generated
2 5 . 2 4 | Nuclear Physics and Radioactivity

by the fusion are also forced to remain inside


Mirrors
the plasma. Their kinetic energy is lost into
the plasma itself contributing further rise in
temperature. Again the lasers are operated
in pulses of short duration.
The research in fusion energy is going on.
D-T Pellet
Fusion is the definite and ultimate answer
Leser Beam
to our energy problems. The ‘fuel’ used for
splitter splitter
fusion on earth is deuterium which is available
in natural water (0.03%). And with oceans as
the almost unlimited source of water, we can
be sure of fuel supply for thousands of years.
Secondly, fusion reactions are neat and Figure 25.12
clean. Radioactive radiation accompanying
fission reactors will not be there with fusion reactors.

11.5 Nuclear Holocaust


Nuclear holocaust refers to a possible nearly complete annihilation of human civilization by nuclear warfare. Under
such a scenario, all or most of the Earth is made uninhabitable by nuclear weapons in future word wars.

PROBLEM-SOLVING TACTICS

 1. Problems from this section do not need any mathematically difficult involvement. One only needs to focus on
exponential functions and its properties.
 2. Questions related to energy can easily be solved by thinking.
 3. For e.g. consider energy as money and think of it in terms of loss and gain, But overall total money is conserved
i.e. total energy is conserved; only it is exchanged. One must not be worried with the relation E = mc2 at this
stage and just consider mass and energy as equivalent. So, if more clearly stated this equivalent quantity is
conserved in every process.
 4. Mostly, questions related to basic understanding of Nuclear force are asked rather than which involve
complicated calculations.
 5. Statistics must always be kept in mind while solving a problem of radioactive decay.

FORMULAE SHEET

 1. After n half-lives


n Number of nuclei decayed
 1
(a) Number of nuclei left = N0  
2 N0
n
 1
(b) Fraction of nuclei left =   and
2
Time
2
 1
(c) Percentage of nuclei left = 100  
Figure 25.13
2
P hysi cs | 25.25

 2. Number of nuclei decayed after time=t N0 − N P(Survival)

N0 N0 e−λt =
=− N0 (1 − e−λt )

1
The corresponding graph is as shown in Fig. 25.13.

 3. Probability of a nucleus for survival of time t,


−λt
N N0 e
P(survival)
= = = e−λt
N0 N0
Time
The corresponding graph is shown in Fig. 25.14.
Figure 25.14
 4. Probability of a nucleus to disintegrate in time t is,
P (Disintegration)
P(disintergration) = 1 − e−λt
1 − P(survival) =

The corresponding graph is as shown. 1

 5. Half-life and mean life are related to each other by the relation,
= t1/2 0.693t
= av or tav 1.44 t1/2
Time
 6. As we said in point number (2), number of nuclei decayed in time
t are N0 (1 − e−λt ) . This expression involves power of e. So to avoid Figure 25.15
it we can use, ∆N =λN∆t where, ∆N are the number of nuclei decayed in
time ∆t , at the instant when total number of nuclei are N. But this can be applied only when ∆t << t1/2 .

 7. In same interval of time, equal percentage (or fraction) of nuclei are decayed (or left un decayed).

1. R = R 0 A1/3 2. ∆=
Ebe ∑ (mc2 ) − Mc2 (binding energy)

∆Ebe dN
3. ∆Eben = (binding energy per nucleon.) 4. = −λdt
A N
1
5. N = N0 e−λt (radioactive delay), 6. τ =
λ
ln2
7. Τ1/2 = = τ ln2.
λ

Solved Examples

JEE Main/Boards ∆E
using ∆m = where ∆E is the amount of energy
c2
Example 1: Sun radiates energy in all direction. The released during the day.
average energy received at earth is 1.4 kW/ m2 . The
The sun radiates energy in all directions in a sphere. At a
average distance between the earth and the sun is
distance R, the energy received per unit area per second
1.5 × 1011 m . If this energy is released by conversion of
is 1.4 KJ (given). Therefore the energy released in area
mass into energy, then the mass lost per day by sun is
approximately (use 1 day = 86400 sec) 4 πR 2 per sec is 1400 × 4 πR 2 J the energy released per
day = 1400 × 4 πR 2 × 86400 J
Sol: The sun produces energy by fusion reaction of
R 1.5 × 1011m , thus
Where =
hydrogen atoms. The loss in mass of sun is calculated E 1400 × 4 × 3.14 × (1.5 × 1011 )2 × 86400
∆=
2 5 . 2 6 | Nuclear Physics and Radioactivity

The equivalent mass is ∆m =∆E / C2 Mass defect =


3 × 2.014 − 4.001 − 1.007 − 1.008

1400 × 4 × 3.14 × (1.5 × 1011 )2 × 86400 = 0.026 =


amu 0.026 × 931MeV
∆m = 16
9 × 1016 star 10
Power of the= = W 1016 J / s
14
∆m = 3.8 × 10 kg Number of deuterons used in one second
1016
Example 2: The energy released per fission of uranium = × 3= 7.75 × 1027
0.026 × 931× 106 × 1.6 × 10−19
(U235) is about 200 MeV. A reactor using U235 as fuel is
producing 1000 kilowatts power. The number of U235 Now the time in which the deuterons supply exhausted
nuclei undergoing fission per sec is, approximately
Number of deutrons present in star
t=
Sol: The number of Uranium nuclei undergoing fission number of deutrons used per sec
is obtained by 40
10
Energy produced = = 1.3 × 1012 sec
N= . 7.75 × 1027
Energy realesed during one fission

The energy produced per second is Example 4: The mean lives of a radioactive material
for α and β radiations are 1620 years and 520 years
106
= 1000 × 103 J = eV = 6.25 × 1024 eV respectively. The material decays simultaneously for α
−19
1.6 × 10 and β radiation. The time after which one fourth of the
material remains un-decayed is
The number of fissions should be,

6.25 × 1024 Sol: The mean life of the radioactive material for
=N = 3.125 × 1016
200 × 106 τα τβ
simultaneous α and β decay is τ = . The time in
τα + τβ
40
Example 3: A star initially has 10 deuterons. It 2.303
2 2 3 1
which the 3/4th of material decayed is t = log10 4 .
produces energy via the processes 1 H + 1 H → 1H +1 p λ
1
and 12H +13 H →24 He +10 n . If the average power radiated We know that λ ∝ .
τ
by the star is 1016 W , in how much time the deuteron τα τβ 1620 × 520
supply of the star get exhausted? =τ = = 394 years
τα + τβ 1620 + 520

Sol: The star produces the energy by fusing deuterium N0


time of decay t = τ × 2.303log10
and tritium into the helium, and releasing proton and N
neutron. Thus the mass defect is easily obtained per
one such conversion. The time in which the deuterium t =394 × 2.303 log10 (4) =394 × 2.303 × 0.602
supply is exhausted is found by
t = 546 years
Number of deutrons present in star
t=
number of deutrons used per sec Example 5: A sample contains two substances P and Q,
Here number of deuterons used per second each of mass 10−2 kg. The ratio of their atomic weights
is 1 : 2 and their half-lives are 4 s and 8 s respectively.
N × Power The masses of P and Q that remain after 16s will
n= respectively be-
energy released per reaction

where N is the deuteron used per reaction. Sol: The mass of radioactive element decaying after
N0 M0
Adding the two processes, we get time t is given=by N = ; M where M is the
n
2 2n
mass (in kg) of the radioactive element. As half-lives
3 12H → 4
2 He + 11p + 10n are given, value of n is found as, number of half-life
P hysi cs | 25.27

t In one half-life the number of active nuclei reduces to


n = .
t1/2 half the original. Thus, in two half-lives the number is
 1  1
N0 M0 16 reduced to   ×   of the original number. The
N
= = ; M ; for P,=
n = 4 2 2
2n 2n 4
number of remaining active nuclei is, therefore,
10 −2  1
∴ MP = = 6.25 × 10−4 Kg 6.0 x1018 x   =1.5 x1018
16
4
16 10−2
for Q, =
n = 2 ∴ MQ = = 2.5 × 10 −3
8 22 Example 8: The half-life of radium is about 1600 years.
In how much time will 1 g of radium (a) reduce to 100
mg (b) lose 100 mg ?
Example 6: There is a stream of neutrons with a kinetic
energy of 0.0327 eV. If the half-life of neutrons is 700 Sol: The weight of radioactive nuclei remaining after
sec, what fraction of neutrons will decay before they
W
= 1.675 × 10−27 kg .
travel 10 m? Given mass of neutron n half-lives is calculated as W = o where Wo is the
2n
Sol: The fraction of neutrons decayed in the distance t
mass present originally. Here n = where T is the half-
∆N 0.693 T
of 10 m is calculated by= ∆t . Here T1/2 is the life.
N Τ1/2
half-life of the neutron and ∆t is the time taken =to cover Wo
W = where T 1600 yr.
distance of 10 m. 2 (t/T)
From the given kinetic energy of the neutrons we first (a) Wo= 1gm, W = 0.1 gm. 2(t/T) =1/0.1=10
calculate their velocity, thus
t T 1600
1 2 Or log2
= 1=or t = = 5,333yr
mu= 0.0327 × 1.6 × 10−19 T log2 0.301
2
2 × 0.0327 × 1.6 × 10−19 (b) Wo =
1g, W =−
1 0.1 =0.9gm
∴ u2 =
1.675 × 10−27 1 t
=2(t/T) = or log2 0.0458
= 625 × 10 4 or u = 2500m / s 0.9 T
With this speed, the time taken by the neutrons to 0.0458x1600
travel a distance of 10 m is, = t = 243.3 yr
0.301
10
t= = 4 × 10 −3 s Example 9: The activity of a radioactive sample falls
2500
from 600/s to 500/s in 40 minutes. Calculate its half-life.
The fraction of neutrons decayed in time ∆t second is,
∆N 0.693 Sol: The activity of any radioactive element is found
= λ∆t also, λ=
N Τ1/2
by A = A0 e−λt . The decay constant is found easily by
∆N 0.693
N
=
Τ1/2
t
∆=
0.693
700
( )
× 4 × 10 −3= 3.96 × 10 −6 above equation. The half-life is obtained by T1/2 =
ln2
λ
.
−λt
We have A = A0 e
Example 7: A radioactive sample has 6.0 x1018 active 5
nuclei at a certain instant. How many of these nuclei = or, 500 s−1 (600= s−1 )e−λt or,e−λt
6
will still be in the same active state after two half-lives?
ln(6 / 5) ln(6 / 5)
or,
= λt ln(6 / =5)or, λ =
Sol: The number of radioactive nuclei remaining after t 40min
N ln2
n half-lives is calculated as N = no where No is the  T1/2 = , ∴ The half-life is
2 λ
number of nuclei present initially. ln2
=T1/2 = x40min 152min.
ln(6 / 5)
2 5 . 2 8 | Nuclear Physics and Radioactivity

ratio of the power required and the energy released


JEE Advanced/Boards from one reaction to get the number of reactions
required per second.
Example 1: The disintegration rate of a certain
radioactive sample initially is 4750 disintegrations (a) During fusion
per minute. Five minutes later the rate becomes 2700 (i) Initially 4 11H →24 He + 2 0
and loses 2 bound
+1e
disintegrations per minute. Calculate the half-life of the electrons
sample.
 4 H1 has 4 bound electrons while 
Sol: The decay constant is obtained using  1 
 2 He4 has only 2 bound electrons 
1 A 
λ = loge  0  where Ao is the initial activity and A is
t Energy released in fusion = ∆m×931.5 MeV
 A 
the activity at the time t. As decay constant is obtained
we can easily calculate the half-life of the sample using {
= 4  11H  − 1 24 He   −2
   
0 0
+1 e − 2 −1 e } × 931.5 MeV
loge 2 .
T1/2 = = 4 (1.0078 ) −  4.0026 + 4 × 0.0006  ×931.5
λ
Let N0 is initial no of nuclei and N is no. of nuclei after MeV=24.685 MeV
five minutes (ii) Later the two positrons combine with 2 electrons to
annihilate each other and release energy.
 dN 
Initially −   = λ N0 Energy release = 4 ( 0.00055) × 931MeV = 2.049 MeV
 dt 0
∴ Total energy release per fusion = 24.685 + 2.049
 dN  = 26.734MeV
Five minutes later, −   =λN
 dt t (b) 26.735MeV= 4.277 ×10−12 J
N0  dN   dN  4750 This energy corresponds to 4 (1.007825) a.m.u.
∴ =     = = 1.76
N  dt 0  dt t 2700 i.e., 6.692 ×10−27 kg of 11H

Also N= N0 e−λt 1 MW power = 106 Js−1


Mass of hydrogen required for producing energy of
1  N0  2.3026 106 J
=λ log
=e
 N  log10 (1.76 )
t   5
106 × 6.692 × 10−27
= = 1.565 ×10−9 kg
= 0.11306 per min. 4.277 × 10 −12

loge 2 0.6931 ∴ Rate of consumption of hydrogen required to


Further=
T1/2 = = 6.13minutes.
λ 0.11306 = 1.565 × 10−9 kgs−1
produce 1 MW power

Example 2: In the interior of the sun, a continuous Example 3: The element curium 248 has a mean
96 Cm
process of 4 protons, fusing into a helium nucleus and
life of 1013 seconds. Its primary decay modes are
pair of positron, is going on. Calculate
spontaneous fission and α − decay, the former with a
(a) The release of energy per process probability of 8% and the latter with probability of 92%.
Each fission releases 200 MeV of energy. The masses
(b) Rate of consumption of hydrogen to produce 1 MW
involved in are as follows:
power.
248
Given 1H1 = 1.007825 a.m.u. (atom) 99 Cm = 248.07220u
244
4
= 4.002603 a.m.u. (atom) 94 Pu = 244.064100u
2 He
4
And 2 He = 4.002603u
m
= + m
= − 5.5x10−4 a.m.u.
e e
(Neglect the energy carried away by neutrons)
Calculate the power output from a sample of 1020 Cm
Sol: The energy produced in sun during one fusion  MeV 
atoms. 1amu = 931 
reaction is E = ∆mc2 J =∆m × 931.5 MeV . Take the  c2 
P hysi cs | 25.29

Sol: The energy released in each transformation is At the time of formation of the rock, both isotopes
found by E =∆m × c2 J. As the probabilities of each have the same number of nuclei N0 . Let λ1 and λ2 be
fission is given the total energy Et released in respective the decay constants of the two isotopes. If N1 and N2
transformation is Probability × E where E is the energy are the number of their nuclei after a time t, we have
liberated during any one fission reaction. And the N1
= e( 1 2 ) 
λt λ t λ −λ t
power liberated during the entire process is given by N1 = N0 e 1 and N2 = N0 e 2 … (i)
N2
E
P = T where ET is the total energy released during
τ Let the masses of the two isotopes at time t be m1 and
fission of all the molecules of the sample. m2 and let their respective atomic weights be M1 and
α - decay of Cm takes place as follows: M2 . We have m1 = N1M1 and m2 = N2M2
248 N1 m1 M2
96 Cm → 94Pu244 + 2He4 =  … (ii)
N2 m2 M1
∴ Mass defect = m (M)
∆m ; ∆= − (M) + Mα  
Substituting the value given in the problem, we get
cm  pu 
=∆m ( 248.07220 ) − 244.064100 + 4.002603 N1 100 1 100
= × =
N2 1 1.02 1.02
∆m =
0.005517u
Let t1 and t2 be the mean lives of the two isotopes.
Energy released per α − decay
1 1
= ( 0.005517 )( 931) MeV = 5.136 MeV Then t1
= = and t2
λ1 λ2
Probability of spontaneous fission=8%
t1 − t2 2 × 109 − 4 × 109
Probability of α − decay = 92% Which gives λ1=
− λ2 =

Energy released in each 248


transformation
t1t2 (2 × 10 ) × ( 4 × 10 )
9 9

96 Cm
−0.25 × 10−9
=
= ( 0.08 × 200 + 0.92 × 5.136 ) MeV = 20.725 MeV
Setting this value in Eqn. (i), we get
Energy released by 1020 atoms
 0.25×10−9  t
N1   1 100
= 20.725 × 1020 MeV = e =  ⇒t loge
N2 0.25 × 10 −9 1.02
Mean life time = 1013 sec
= 18.34 × 109 year
20.725 × 1020 MeV
power =
1013 sec Example 5: A small quantity of solution containing
7
= 20.725 × 10 × 1.6 × 10( −13
) joule
= 3.316 × 10−5 watt.
sec
24
11 Na
radioactive nuclei (half-life 15 hours) of activity
1.0 µ Ci is injected into the blood of a person. A sample
of the blood of volume 1 cc taken after 5 hours showed
Example 4: In the chain analysis of a rock, the mass an activity of 296 disintegrations per minute. Determine
ratio of two radioactive isotopes is found to be 100:1. the total volume of blood in the body of the person.
The mean lives of the two isotopes are 4 × 109 year and Assume that the radioactive solution mixed uniformly
2 × 109 year respectively. If it is assumed that at the in the blood of the person.
time of formation of the rock, both isotopes were in
(1 Curie =3.7× 1010 disintegration per second)
equal proportion, calculate the age of the rock. Ratio of
atomic weights of the two isotopes is 1.02:1. Sol: The activity of the radioactive nuclei is given
by Ao = λN0 where λ is the decay constant of the
(log10 1.02 = 0.0086 ) . radioactive nuclei. Find the number of radioactive nuclei
N0 present initially. Also find the number of nuclei in
Sol: The number of the radioactive nuclei remaining
the sample of the blood initially. The ratio of these two
at time t is given as Nt = No e−λt . Here the ratio of the
gives the volume.
masses are given. The ratio of number of atoms are
N m M 0.693
given by 1 = 1 2 . Find the value of t from the ratio We know that T1/2 = or
N2 m2 M1 λ
Nt1 0.693 0.693
.
Nt2 =λ = = 1.283 × 10−5 / sec.  ... (i)
T 15 × 3600 1/2
2 5 . 3 0 | Nuclear Physics and Radioactivity

dN ∆N
Now activity Ao = = λNo  … (ii) 1 second is found by = λN here λ is the decay
dt ∆t
constant and N is the number of nuclei present in 1 g
Where Ao = 1 micro curie = 1× 3.7 × 10 4 sample of radium.
= 3.7 × 10 4 disintegrations / sec Number of atoms in 1g sample is
From equation (ii) we have  1 
N = (
 6.02 × 10
 226 
23
)
=2.66 × 1021 atoms.
3.7 × 10 4 = 1.283 × 10 −5 × N0
3.7 × 10 4 The decay constant is
N0
= = 2.883 × 109
1.283 × 10−5 0.693 0.693
=λ = = 1.35 × 10−11s−1
Let the number of radioactive nuclei present after 5
T1/2 (1620 ) (3.16 × 10 7
)
hours be N1 in 1 cc sample of blood.
Taking 1 yr = 3.16×107 s;
dN 296 0.693
Then = λN1 or = N
dt 60 15 × 3600 1
296 × 15 × 3600
Now,
∆N
∆t
( )(
= λN = 1.35 × 10−11 2.66 × 1021 )
or
= N1 = 3.844 × 105
60 × 0.693 = 3.6 × 1010 s−1
Let N0′ be the number of radioactive nuclei in per cc of Thus, 3.6 × 1010 nuclei decay in one second.
sample, then

Example 7: Determine the age of an ancient wooden


(2)
t/T
Then=
N0′ × N1
piece if it is known that the specific activity of C14
nuclide in it amounts to 3/5 of that in freshly felled
(2) (2)
5/15 1/3
N0′= × N=
1 × 3.844 × 105 trees. The half-life of C 14 nuclide is 5570 years.

= 1.269 × 3.844 × 105 ( 2 ) = 1.269 


1/3
= 4.878 × 105 Sol: Find the age of wooden piece using equation
 
3
N0 2.883 × 109 A = A0 e−λt . Here A is Ao and λ is the decay constant.
Volume of blood = V = 5
N0′ 4.878 × 105 Specify activity is the activity per unit mass of the
=0.5910 × 10 4 cm3 =5.91 litres. substance.
A = A0 e−λt ; Here A = ( 3 / 5) A0
0.693 0.693
=T = = min 6.14min
λ 0.113 5
ln
3
∴ A =
A0 e −λt
or t = 3
Example 6: The half-life of radium is 1620 years. How 5 0 λ
many radium atoms decay in 1s in a 1g sample of
radium? The atomic weight of radium is 226 g/mol. 5  5
= or t ln=  / (ln2 / T ) 5570  ln 
3  3
Sol. Number of radioactive nuclei disintegrated in
= 4.1× 103 years
P hysi cs | 25.31

JEE Main/Boards

Exercise 1 in a second to attain this power level. If this explosion is


designed with nuclear fuel consisting of uranium -235
Nuclear Physics to run a reactor at this power level for one year, then
calculate the amount of fuel needed. You can assume
that the amount of energy released per fission event is
Q.1 Some amount of radioactive substance (half-life= 200 MeV.
10 days) is spread inside a room and consequently the
level of radiation becomes 50 times permissible level
for normal occupancy of the room. After how many Q.7 Draw a diagram to show the variation of binding
days the room will be safe for occupation? energy per nucleon with mass number for different
nuclei. State with reason why light nuclei usually
undergo nuclear fusion.
Q.2 The mean lives of a radioactive substance are
1620 and 405 years for α − emission and β − emission
respectively. Find out the time during which three forth Q.8 Define decay constant of radioactive sample. Which
of a sample will Decay if it is decaying both by α − of the following radiations, α − rays, β − rays, γ − rays
emission and β − emission simultaneously. (i) Are similar to X-rays/
(ii) Are easily absorbed by matter?
Q.3 A radioactive element decays by β − emission. A
detector records n-beta particles in 2 Seconds and in (iii) Travel with greatest speed?
next 2 seconds it records 0.75 n beta particles. Find (iv) Are similar in nature to cathode rays?
mean life correct to nearest whole number. Given
log2=0.6931, log3=1.0986.
Q.9 Calculate the energy released in the following
nuclear reaction:
Q.4 Nuclei of radioactive element A are being produced 6 1
3 Li + 0 n →24 He +13 H (Given; mass of 6
3 Li
=6.015126 u,
at a constant rate ∝ . The element has a decay constant
λ. At time t = 0 there are N0 nuclei of the element. mass of 1
= 1.008665 u, mass of 4
= 4.002604 u,
0n 2 He
(a) Calculate the number N of nuclei of A at time t.
mass of 13 H = 3.016049 u and 1 atomic mass unit (1 u))
(b) If=α 2N0 λ , calculate the number of nuclei of A
after one half-life of A and also the limiting value of N = 931 MeV)
at t → ∞
Q.10 Explain with an example, whether the neutron-
Q.5 Polonium ( ) emits - particles and is
210
84 Po
4

proton ration in a nucleus increases or decreases due
to beta ( β ) decay.
converted into lead ( Pb ) . The Reaction is used for
206
82

producing electric power in a space mission. 210


84 Po
has Q.11 Define the herms; ‘half-life period’ and ‘decay
constant’ of radioactive sample. Derive the relation
half of 138.6days. Assuming an efficiency of 10% of the
between these terms.
210
thermoelectric machine, how much 84 Po
is required
7
to produce 1.2 × 10 J of electric energy per day at the Q.12 When a deuteron of mass 2.0141 u and negligible
end of 693 days? Also find the initial activity of the
210
kinetic energy is absorbed by a lithium ( Li ) nucleus
6
3
material. (Given masses of the nuclei 84 Po
=209.98264
of mass 6.0155 u, the compound nucleus disintegrates
206
amu, 82 Pb
=205.97440 amu, 24 α =4.00260 amu, 1 amu spontaneously into two alpha particles, each of mass
4.0026 u. Calculate the energy in joules carried by each
= 931 MeV and Avogadro number= 6 × 1023 / mol).
(
1u 1.66 × 10−27 kg
alpha particle. = )
Q.6 A nuclear explosion is designed to deliver 1MW of
heat energy, how many fission events must be required
2 5 . 3 2 | Nuclear Physics and Radioactivity

11 Q.22 The mass of a nucleus in its ground state is always


Q.13 A radioactive sample contains 2.2 mg of pure 6 C
less than the total mass of its Constituents neutrons
which has half-life period of 1224 seconds. Calculate
and protons. Explain.
(i) The number of atoms present initially.
(ii) The activity when 5 µg of the sample will be left. Q.23 Draw a plot showing the variation of binding
energy per nucleon versus the mass number A. Explain
with the help of this plot the release of energy in the
Q.14 Define the terms half-life period and decay
processes of nuclear fission and fusion.
constant of a radioactive substance. Write their S.I.
units. Establish the relationship between the two.
Q.24 Define the activity of a radionuclide. Write its S.I.
unit. Give a plot of the activity of a radioactive species
Q.15 A neutron is absorbed by a 36 Li nucleus with the
versus time.
subsequent emission of an alpha particle.
(i) Write the corresponding nuclear reaction.
Q.25 Draw a plot of the binding energy per nucleon as
(ii) Calculate the energy released, in MeV, in this a function of mass number for a large Number of nuclei,
reaction. 2 ≤ A ≤ 240. How do you explain the constancy of
binding energy per nucleon in the range 30 < A < 170
Given mass 36 Li = 6.015126 u;
using the property that nuclear force is short-range?
Mass (neutron) = 1.0086554 u;
Mass (alpha particle) = 4.0026044 u and Radioactivity
Mass (triton) = 3.0100000 u. Take 1 u =931 MeV/ C2 .
Q.26 Classify each of the following nuclides as “beta
Q.16 Define the term ‘activity’ of a radionuclide. Write
its SI unit. ( β) emitter”, or “positron ( β) emitter”:
0
−1
0
−1
49 195
20 Ca 80 Hg

5 150 30 94 84 200 165


8 B 67 Ho 13 Al 36 Kr.
Note: 36 Kr 80 ,Hg
and 67 Ho
are
Q.17 Draw graph showing the variation of potential
energy between a pair of nucleons as a function of their stable.
separation. Indicate the regions in which the nuclear
force is 114 114 114
Q.27 Of the three isobars 48 Cd 49 In
and 50 Sn
, which
(i) Attractive (ii) Repulsive
is likely to be radioactive? Explain your choice.

Q.18 Draw the graph to show variation of binding


energy per nucleon with mass number of different Q.28 Complete the following nuclear equations;
atomic nuclei. Calculate binding energy/nucleon of 14 4
40
nucleus. (a) 7 N +2 He →17
8 O + ........
20 Ca

Q.19 State two characteristic properties of nuclear (b) 94 Be +24 He →12


6 C + ........

force.
(c) 94 Be (p, α ) ........
Q.20 Calculate the energy, released in MeV, in the
30 30
following nuclear reaction (d) 15 P →14 S + .........

238 234
Th + 24 He + Q Massof 238U = 238.05079 u
92 U →90
 92 (e) 13 H →32 He + .........
Massof 234 4
90 Th = 234.043630 u Massof 2 He = 4.002600 u
2
(f) 43
20 Ca ( ∝,...... ) →2146 Sc
1u = 931.5MeV / c

Q.21 Two nuclei have mass numbers in the ration 1:8. Q.29 The activity of the radioactive sample drops to
What is the ration of their nuclear radii? 1/64 of its original value in 2 hr find the decay constant
(λ).
P hysi cs | 25.33

Q.30 The nucleidic ration of 3 H to 11H in a sample of bones. This nuclide has a half-life of 28.1 year. Suppose
water is 8.0 × 10−18 : 1. Tritium undergoes decay tritium one microgram was absorbed by a new-born child,
atoms would 10.0 g of such a sample contains 40 year how much Sr 90 will remain in his bones after 20 years?
after the original sample is collected?
210 206
Q.39 (i) Po decays with α − particle to 82 Pb
with a
125 84
Q.31 The half-life period of I is 60 days. What % of 210
53 half-life of 138.4 day. If 1.0 g of Po is placed in a
84
radioactivity would be present after 240 days? sealed tube, how much helium will accumulate in 69.2
day? Express the answer in cm3 at 1atm and 273K. Also
Q.32 At any given time a piece of radioactive material
210
report the volume of He formed if 1 g of Po is used.
( t1/2 = 30 days ) contains 1012 atoms. 84

Calculate the activity of the sample in dps. (ii) A sample of U238 (half-life = 4.5 × 109 yr) ore is found
to contain 23.8 g of U238 and 20.6 g of Pb206 . Calculate
Q.33 Calculate the age of a vegetarian beverage whose
tritium content is only 15% of the level in living plants. the age of the ore.

Given t1/2 for 3 H = 12.3 years.


1
Q.40 Ac227 has a half-life of 22 year w.r.t radioactive
Q.34 An isotopes of potassium 40
has a half-life of decay. The decay follows two parallel paths, one
19 K
leading the Th227 and the other leading to Fr 223 . The
1.4 × 109 year and decays to Argon 40
18 Ar
which is stable. percentage yields of these two daughters nucleides are
(i) Write down the nuclear reaction representing this 2% and 98% respectively. What is the rate constant in
decay. yr −1 , for each of these separate paths?
(ii) A sample of rock taken from the moon contains both
potassium and argon in the ratio 1/3. Find age of rock.
Exercise 2
Q.35 At a given instant there are 25% undecayed
Nuclear Physics
radioactive nuclei in a sample. After 10 sec the number
of undecayed nuclei remain 12.5%. Calculate : Single Correct Choice Type
(i) mean-life of the nuclei and
Q.1 Let u be denoted one atomic mass unit. One atom
(ii)The time in which the number of undecayed nuclear
of an element of mass number A has mass exactly equal
will further reduce to 6.25% of the reduced number.
to Au
(A) For any value of A
Q.36 Calculate the energy released in joules
and MeV in the following nuclear reaction : (B) Only for A = 1
2 2
H →32 He +10 n Assume that the masses of 2
, (C) Only for A = 12
1 H +1 1H
3 (D) For any value of A provided the atom is stable
2 He
and neutron (n) respectively are 2.020, 3.0160 and

1.0087 in amu. Q.2 The surface area of a nucleus varies with mass
number A as
Q.37 (a) Calculate number of α − and β -particles
(A) A2/3 (B) A1/3 (C) A (D) None
238 206
emitted when 92 U
changes into radioactive 82 Pb
.
Q.3 Consider the nuclear reaction X 200 → A110 + B90
(b) Th234 disintegrates and emits 6β − and 7 α − particles
If the binding energy per nucleon for X,A and B is
to form a stable element. Find the atomic number and
7.4 MeV, 8.2. MeV and 8.2 MeV respectively, what is the
mass number of the stable product.
energy released?

Q.38 One of the hazards of nuclear explosion is the (A) 200 MeV (B) 160 MeV
generation of Sr 90 and its subsequent incorporation in (C) 110 MeV (D) 90 MeV
2 5 . 3 4 | Nuclear Physics and Radioactivity

Q.4 The binding energy per nucleon for C12 is 7.68 Radioactivity
13
MeV and that for C is 7.5 MeV. The energy required
Single Correct Choice Type
to remove a neutron from C13 is
27 29
(A) 5.34 MeV (B) 5.5 MeV Q.11 13 Al
is a stable isotope. 13 Al is expected to be
(C) 9.5 MeV (D) 9.34 MeV disintegrated by
(A) α emission (B) β emission
Q.5 The binding energies of nuclei X and Y are E1 and
(C) Positron emission (D) Proton emission.
E2 respectively. Two atoms of X fuse to give one atom
of Y and an energy Q is released. Then:
Q.12 Loss of a β − particle is equivalent to
Q 2E1 − E2
(A) = (B) Q
= E2 − 2E1
(A) Increase of one proton only
Q 2E1 + E2
(C) = Q 2E2 + E1
(D) =
(B) Decrease of one neutron only

Q.6 There are two radio-nuclei A and B. A is (C) Both (A) and (B)
an alpha emitter and B is a beta emitter. Their (D) None of these
disintegration constants are in the ratio of 1:2. What
should be the ratio of number of atoms of two at
Q.13 Two radioactive material A1 and A2 have decay
time t = 0 so that probabilities of getting α − and
constant of 10λ0 and λ0 . If initially they have same
β − particles are same at time t=0.
1
(A) 2:1 (B)1:2 (C)e (D) e−1 number of nuclei, then after time the ratio of
9λ0
number of their undecayed nuclei will be
Q.7 A certain radioactive substance has a half-life of 5
1 1 1 e
years. Thus for a particular nucleus in a sample of the (A) (B) (C) (D)
element, the probability of decay in ten years is e 2
e 3
e 1

(A) 50% (B) 75% (C) 100% (D) 60% Q.14 The half-life of a radioactive isotopes is three
hours. If the initial mass of the isotope were 256 g, the
Q.8 Half-life of radium is 1620 years. How many radium mass of it remaining undecayed after 18 hours would
nuclei decay in 5 hours in 5 gm radium? (Atomic weight be
of radium = 223) (A) 16.0 g (B) 4.0 g (C) 8.0 (D) 12.0 g
12 15
(A) 9.1× 10 (B) 3.23 × 10
1 2 k k
(C) 1.72 × 1020 (D) 3.3 × 1017 Q.15 A consecutive reaction A → B  → C is
characterised by

Q.9 The decay constant of the end product of a (A) Maxima in the concentration of A
radioactive series is (B) Maxima in the concentration of B
(A) Zero (C) Maxima in the concentration of C
(B) Infinite (D) High exothermicity
(C) Finite (non zero)
(D) Depends on the end product. Q.16 Consider the following nuclear reactions:
238
92 M →YX N + 2 24He; XY N →BA L + 2β +

Q.10 A radioactive nuclide can decay simultaneously The number of neutrons in the element L is
by two different processes which have decay constants
λ1 and λ2 . The effective decay constant of the nuclide is (A) 142 (B)144 (C)140 (D)146
λ , then :
Q.17 The half-life of a radioisotope is four hours. If
(A) λ = λ1 + λ2 λ 1 / 2 ( λ1 + λ1 )
(B)= the initial mass of the isotope was 200 g, the mass
1 1 1 remaining after 24 hours undecayed is
(C) = + (D) λ = λ1λ2
λ λ1 λ2 (A) 1.042 g (B) 2.084 g (C) 3.125 g (D) 4.167 g
P hysi cs | 25.35

Q.18 Helium nuclei combines to form an oxygen Q.23 The ratio of 14C to 12C in a living matter is measured
nucleus. The binding energy per nucleon of oxygen 14
C −12
nucleus is if m0 = 15.834 amu and mHe = 4.0026 amu to be =1.3 X 10 at the present time. Activity of
14
C
(A) 10.24 MeV (B) 0 MeV 12.0 gm carbon sample is 180 dpm. The half-life of 14C
(C) 5.24 MeV (D) 4 MeV is nearly _________x 10−12 sec. [Given: NA = 6x1023 ]
(A) 0.18 (B) 1.8 (C) 0.384 (D) 648
Q.19 A radioactive element gets spilled over the floor
of a room. Its half-life period is 30 days. If the initial Q.24 Which of the following processes represent a
activity is ten times the permissible value, after how gamma – decay?
many days will it be safe to enter the room?
A A
(A) 1000 days (B) 300 days (A) X + y → X + a+b
Z Z −1

(C) 10 days (D) 100 days A 1 A −3


(B) X+ n→ X +c
Z 0 Z −2

Q.20 Which of the following nuclear reactions will A


generate an isotope ? (C) X → A X + ƒ
Z Z

(A) neutron particle emission A A


(D) X+ e→ X +g
Z −1 Z −1
(B) position emission
(C) α-particle emission
Q.25 Let Fpp , Fpn and Fnn denote the magnitudes of net
(D) β-particle emission force by a proton on a proton, by a proton on a neutron
and by a neutron on a neutron respectively. Neglect
Q.21 Read the following: gravitational force. When the separation is 1 fm,

(i) The half-life period of a radioactive element X is same (A) Fpp > Fpn =
Fnn (B) Fpp
= Fpn
= Fnn
as the mean-life time of another radioactive element Y.
Initially both of them have the same number of atoms. (C) Fpp > Fpn > Fnn (D) Fpp < Fpn =
Fnn
Then Y will decay at a faster rate than X.
(ii) The electron emitted in beta radiation originates Q.26 The average (mean) life at a radio nuclide which
from decay of a neutron in a nucleus decays by parallel path is
λ
(iii) The half-life of 215 at is 100 ms. The time taken for 1
A → B; λ1 = 1.8x10 −2 sec−1
the radioactivity of a sample of of 215 At to decay to
1/16th of its initial value is 400 us. λ
2 → C; λ = 10 −3 sec −1
2A  2
(iv) The volume (V) and mass (m) of a nucleus are
related as V ∝ m. (A) 52.63 sec (B) 500 sec
(v) Given a sample of Radium-226 having half-life of (C) 50 sec (D) None
4 days. Find the probability. A nucleus disintegrates
within 2 half-lives is ¾
Q.27 Two radioactive nuclides A and B have half lives
Select the correct code for above. of 50 min respectively. A fresh sample contains the
(A) TTTTT (B) TFTTF nuclides of B to be eight time that of A. How much
time should elapse so that the number of nuclides of A
(C) FTFTF (D) FTTTF becomes double of B
(A) 30 (B) 40 (C) 50 (D) None
Q.22 The radioactive sources A and B of half-lives of t
hours respectively, initially contain the same number of
radioactive atoms. At the end of t hours, their rates of Q.28 A sample of 14 CO2 was to be mixed with ordinary
disintegration are in the ratio: CO2 for a biological tracer experiment. In order that
(A) 2 2 : 1 (B) 1:8 10 cm3 of diluted gas should have 10 4 dis/min, what
activity (in µ Ci) of radioactive carbon is needed to
(C) 2 : 1 (D) n:1
2 5 . 3 6 | Nuclear Physics and Radioactivity

prepare 60 L of diluted gas at STP. [1 Ci= 3.7x 1010 dps] Q.4 During a negative beta decay  (1987)

(A) 270mCi (B)27mCi (C) 2.7mCi (D)2700mCi (A) An atomic electron is ejected
(B) An electron which is already present within the
nucleus is ejected
Q.29 Wooden article and freshly cut tree show activity of
7.6 and 15.2 min−1 gm−1 of carbon ( t1/2 = 5760 years) (C) A neutron in the nucleus decays emitting an electron
respectively. The age of article in years. Is (D) A part of the binding energy of the nucleus is
converted into an electron
 15.2 
(A) 5760 (B) 5760x  
 7.6 
Q.5 A star initially has 10 40 deuterons. It produces
 7.6 
(C) 5760 ×   (D) 5760 x (15.2-7.6) 2
energy via the processes 1H + 1H2 → 1H3 p and
 15.2 
2 3 4
1H + 1H → 2He + n. If the average power radiated by
Q.30 A radioactive sample had an initial activity of the star is 1016 W, the deuteron supply of the star is
56 dpm (disintegration per min it was found to have
an activity of 28 dpm. Find the number of atoms in a exhausted in a time of the order of  (1993)
sample having an activity of 10 dpm. (A) 106 s (B) 108 s (C) 1012 s (D) 1016 s
(A) 693 (B) 1000 (C) 100 (D) 10,000
Q.6 Fast neutrons can easily be slowed down by(1994)
Q.31 The radioactivity of a sample is R1 at a time T1 (A) The use of lead shielding
and R 2 at a time T2 . If the half-life of the specimen is
T, the number of atoms that have disintegrated in the (B) Passing them through heavy water
time (T2 − T1 ) is proportional to (C) Elastic collisions with heavy nuclei
(A) (R1T1 − R 2 T2 ) (B) (R1 − R 2 ) (D) Applying a strong electric field
(C) (R1 − R 2 ) / T (D) (R1 − R 2 ) T/ 0.693
Q.7 Consider α -particles, β -particles and λ -rays each
Previous Years’ Questions having an energy of 0.5 MeV. In increasing order of
penetrating powers, the radiations are (1994)
(A) α , β, γ (B) α , γ , β (C) β, γ , α (D) γ , β, α
Q.1 The half-life of the radioactive radon is 3.8 days. The
time, at the end of which 1/20th of the radon sample will
remain undecayed, is (given log10 3 =0.4343) (1981) Q.8 A radioactive sample S1 having an activity of 5 µ Ci
has twice the number of nuclei as another sample S2
(A) 3.8 days (B) 16.5 days
which has an activity of 10 µ Ci. The half lives of S1 and
(C) 33 days (D) 76 days S2 can be  (2008)
(A) 20 yr and 5 yr, respectively
Q.2 Beta rays emitted by a radioactive material are
(B) 20 yr and 10 yr, respectively
 (1983)
(C) 10 yr each
(A) Electromagnetic radiations
(D) 5 yr each
(B) The electrons orbiting around the nucleus
(C) Charged particles emitted by the nucleus
Q.9 The radioactive decay rate of a radioactive element
(D) Neutral particles is found to be 103 disintegration /second at a certain
time. If the half-life of the element is one second, the
Q.3 The equation ;  (1987) decay rate after one second is ……………. And after three
seconds is ………….. (1983)
4 11H → 42He2+ + 2e− + 26 Me V represents

(A) β -Decay (B) γ -Decay Q.10 In the uranium radioactive series the initial
238 206
(C) Fusion (D) Fission nucleus is 92 U
and the final nucleus is 92 Pb
. When
P hysi cs | 25.37

the uranium nucleus decays to lead, the number of


α -particles emitted is…. And the number of β -particles
emitted is………….  (1985) B C D
E
Eb

Q.11 Consider the reaction: 21H + 21H = 4


2 He + Q . Mass A F

of the deuterium atom = 2.0141u. Mass of helium atom


M
= 4.0024u. This is a nuclear ………… reaction in which the
energy Q released is ……. MeV. (1996) (A) (i) and (iv) (B) (i) and (iii)
(C) (ii) and (iv) (D) (ii) and (iii)
Q.12 This question contains Statement-I and Statement-
II. Of the four choices given after the statements, choose Q.15 The transition from the state n = 4 to n = 3 in
the one that best describes the two statements. a hydrogen like atom results in ultraviolet radiation.
Statement–I: Energy is released when heavy nuclei Infrared radiation will be obtained in the transition
undergo fission or light nuclei undergo fusion. from  (2009)

and (A) 2 →1 (B) 3 →2

Statement–II: For heavy nuclei, binding energy per (C) 4 →2 (D) 5→3
nucleon increases with increasing Z while for light
nuclei it decrease with increasing Z.  (2008) Q.16 The binding energy per nucleon for the parent
nucleus is E1 and that for the daughter nuclei is E2.
(A) Statement-I is false, statement-II is true.
Then  (2010)
(B) Statement-I is true, statement-II is true; statement-II
(A) E2 = 2E1 (B) E1 > E2
is correct explanation for statement-I.
(C) E2 > E1 (D) E1 = 2E2
(C) Statement-Iis true, statement– 2 is true; statement-II
is nota correct explanation for statement-I.
Q.17 The speed of daughter nuclei is (2010)
(D) Statement-I is true, statement-II is False.
∆m 2∆m
(A) C
c (B) cC
Q.13 Suppose an electron is attracted towards the M + ∆m M
origin by a force k/r where ‘k’ is a constant and ‘r’ is the
∆m ∆m
distance of the electron from the origin. By applying (C) cC (D) C
c
Bohr model to this system, the radius of the nth orbital M M + ∆m
of the electron is found to be ‘rn’ and the kinetic energy
of the electron to be Tn. Then which of the following is
Q.18 A radioactive nucleus (initial mass number A and
true?  (2008)
atomic number Z) emits 3 a-particles and 2 positrons.
(A) Tn ∝1/n2, rn ∝n2 (B) Tn independent of n, rn ∝n The ratio of number of neutrons to that of protons in
the final nucleus will be (2010)
(C) Tn ∝1/n, rn ∝n (D) Tn ∝1/n, rn ∝n2
A–Z–8 A–Z–4
(A) (B)
Q.14 The above is a plot of binding energy per nucleon Z–4 Z–8
Eb, against the nuclear mass M; A, B, C, D, E, F correspond
to different nuclei. Consider four reactions:  (2009) A – Z – 12 A–Z–4
(C) (D)
Z–4 Z–2
(i) A + B →C + ε
(ii) C →A + B + ε Q.19 Energy required for the electron excitation in Li++
(iii) D + E →F + ε and from the first to the third Bohr orbit is:  (2011)

(iv) F →D + E + ε (A) 36.3 eV (B) 108.8 eV

where ε is the energy released? In which reactions is ε (C) 122.4 eV (D) 12.1 eV
positive?
2 5 . 3 8 | Nuclear Physics and Radioactivity

Q.20 The half life of a radioactive substance is 20 Q.23 As an electron makes a transition from an excited
minutes. The approximate time interval (t2 – t1) between state to the ground state of a hydrogen - like atom/ion:
2 1  (2015)
the time t2 when of it has decayed and time t1 and
3 3 (A) its kinetic energy increases but potential energy and
of it had decayed is :  (2011) total energy decrease
(A) 14 min (B) 20 min (C) 28 min (D) 7 min (B) kinetic energy, potential energy and total energy
decrease
Q.21 Hydrogen atom is excited from ground state to (C) kinetic energy decreases, potential energy increases
another state with principal quantum number equal to but total energy remains same
4. Then the number of spectral lines in the emission
spectra will be (2012) (D) kinetic energy and total energy decrease but
potential energy increases
(A) 2 (B) 3 (C) 5 (D) 6
Q.24 Half-lives of two radioactive elements A and B are
Q.22 Assume that a neutron breaks into a proton and 20 minutes and 40 minutes, respectively, Initially, the
an electron. The energy released during this process is samples have equal number of nuclei. After 80 minutes,
(Mass of neutron = 1.6725 x 10–27 kg; mass of proton the ratio of decayed numbers of A and B nuclei will be :
= 1.6725 x 10–27 kg; mass of electron = 9 x 10–31 kg)  (2016)
 (2012)
(A) 4 : 1 (B) 1 : 4 (C) 5 : 4 (D) 1 : 16
(A) 0.73 MeV (B) 7.10 MeV
(C) 6.30 MeV (D) 5.4 MeV

JEE Advanced/Boards

Exercise 1 atom and releases the energy by the nuclear reaction,


411H → 42He with 26 MeV of energy released. If the total
Nuclear Physics
output power of the Sun is assumed to remain constant
at 3.9 x 1026 W, find the time it will take to burn all the
Q.1 The binding energies per nucleon for deuteron hydrogen, Take the mass of the Sun as 1.7x 1030 kg.
( 1H2 ) and helium ( 2He4 ) are 1.1 MeV and 7.0 MeV
respectively. The energy released when two deuterons
Q.5 U238 and U235 occur in nature in an atomic ratio
fuse to form a helium nucleus ( 2He4 ) is ________________
140:1. Assuming that at the time of earth’s formation
the two isotopes were present in equal amounts.
40
Q. 2 An isotopes of Potassium 19 K
has a half-life of Calculate the age of the earth.
9 40
1.4 x 10 year and decay to Argon 18 Ar
which is stable. (Halflife of u238 = 4.5 × 109 years and that of U235 = 7.13
(i) Write down the nuclear reaction representing this × 108 years)
decay.
(ii) A sample of rock taken from the moon contains both Q.6 The kinetic energy of an α -particle which flies
potassium and argon in the ratio 1/7. Find age of rock. out of the nucleus of a Ra226 atom in radioactive
disintegration in 4.78 MeV. Find the total energy the
escape of the α -particle.
Q.3 At t=0, a sample is placed in a reactor. An unstable
nuclide is produced at a constant rate R in the sample
by neutron absorption. This nuclide β -decays with Q.7 A small bottle contains powdered beryllium Be &
half-life τ. Find the time required to produce 80% of the gaseous radon which is used as a source of α -particles.
equilibrium quantity of this unstable nuclide. Neutrons are produced when α -particles of the radon
react with beryllium. The yield of this reaction is (1/4000)
i.e. only one α -particle out of 4000 induces the reaction.
Q.4 Suppose that the Sun consists entirely of hydrogen
P hysi cs | 25.39

Find the amount of radon (Rn222 ) originally introduced Radioactivity


into the source. If it produces 1.2 x 106 neutrons per
Q.14 In a nature decay chain series starts with 90Th232
second after 7.6 days. [T1/2 of R a = 3.8 days] and finally terminates at 82Pb208 . A thorium ore sample
was found to contain 8 x 10−5 ml of helium at 1 atm
Q.8 An experiment is done to determine the half-life & 273 K and 5 x 10−7 gm of Th232 . Find the age of
of radioactive substance that emits one β -particle
ore sample assuming that source of He to be only due
for each decay process. Measurement show that an
average of 8.4 β are emitted each second by 2.5 mg of to decay of Th232 . Also assume complete retention of
the substance. The atomic weight of the substance is helium within the ore. (Half-life of Th232 =1.39 x 1010 Y)
230. Find the half-life of the substance.
Q.15 A radioactive decay counter is switched on at t=0.
Q.9 A wooden piece of great antiquity weighs 50 gm and A β -active sample is present near the counter. The
shows C14 activity of 320 disintegrations per minute. counter registers the number of β -particles emitted by
Estimate the length of the time which has elapsed the sample. The counter registers 1 x 105 β -particles at
since this wood was part of living tree, assuming that t=36 s and 1.11 x 105 β -particles at t = 108 s. Find T1/2
living plant show a C14 activity of 12 disintegrations per of this sample.
minute per gm. The half-life of C14 is 5730 yrs.
Q.16 A small quantity of solution containing 24 Na
Q.10 When two deuterons (2 H) fuse to from a helium radionuclide (half-life 15 hours) of activity 1.0 microcurie
1
is injected into the blood of a person. A sample of the
nucleus 2He4 , 23.6 MeV energy is released. Find the
blood of volume 1 cm3 taken after 5 hours shows an
binding energy of helium if it is 1.1 MeV for each
activity of 296 disintegrations per minute. Determine
nucleon of deutrim.
the total volume of blood in the body of the person.
Assume that the radioactive solution mixes uniformly
Q.11 A π+ meson of negligible initial velocity decays to in the blood of the person. (1 Curie=3.7 x 1010
a µ + (muon) and a neutrino. With what kinetic energy disintegrations per second)
(in eV) does the muon move? (The rest mass of neutrino
can be considered zero. The rest mass of the π+ meson
Q.17 A mixture of 239Pu and 240 Pu has a specific
is 150 MeV and the rest mass of the muon is 100 MeV.)
activity of 6 x 109 dis/s/g. The half lives of the isotopes
Take neutrino to behave like a photon.
are 2.44 x 10 4 y and 6.08 x 103 y respectively. Calculate
Take 3 = 1.41. the isotopic composition of this sample.

Q.12 A body of mass m0 is placed on a smooth Q.18 Nuclei of a radioactive element A are being
horizontal surface. The mass of the body is decreasing produced at a constant rate α . The element has a
exponentially with disintegration constant λ . Assuming decay constant λ . At time t=0, there are N0 nuclei of
that the mass is ejected backward with a relative velocity the element.
u. Initially the body was at rest. Find the velocity of body
(a) Calculate the number N of nuclei of A at time t
after time t.
(b) If α = 2N0 λ , calculate the number of nuclei of A after
one half-life of A & also the limiting value of N as t → ∞
Q.13 Show that in a nuclear reaction where the
outgoing particle is scattered at an angle of 900 with
the direction of the bombarding particle, the Q-value is Q.19 In hydrogenation reaction at 25o C , it is observed
expressed as that hydrogen gas pressure falls from 2 atm to 1.2 atm
in 50 min. Calculate the rate of reaction in molarity per
 mp   m1  sec. R=0.0821 litre atm degree −1mol−1
Q = Kp  1 +  − K1 1 + 
 Mo   Mo 
  238
Q.20 U by successive radioactive decays changes
Where, I=incoming particle, P=product nucleus, 92
T=target nucleus, O=outgoing particle. 206
to Pb . A sample of uranium ore was analyzed and
82
2 5 . 4 0 | Nuclear Physics and Radioactivity

found to contain 1.0g of U238 and 0.1g of PB206 . (B) Captures a γ -ray photon of energy 2 MeV
Assuming that all the PB206 had accumulated due to (C) Emits a γ -ray photon of energy 3 MeV
decay of U238 , find out the age of the ore.
(D) Captures a γ -ray photon of energy 3 MeV
238 9
(Half life of U = 4.5x10 years)
Q.2 A certain radioactive nuclide of mass number mx
disintegrates, with the emission of an electron and γ
218 214
Q.21 Po (t /12 = 3.05 min) decay to Pb (t
(t/12 radiation
3.05 min)
== 3.05 only, to give second nuclide of mass number
84 82 /12
my . Which one of the following equation correctly
min) by α -emission, while Pb214 is a β − emitter . In an
relates mx and my ?
218
experiment starting with 1 gm atom of Pure Po , how (A) m= m + 1 (B) m
= m −2
y x y x

much time would be required for the number of nuclei (C) m


=y mx − 1 (D) my = mx
214
of Pb to reach maximum?
82 Q.3 The number of α and β -emitted during the
226
radioactive decay chain starting from 88 Ra
and ending
Q.22 (a) On analysis a sample of uranium ore was found 206
at 82 Pb
is
206 237
to contain 0.277g of Pb and 1.667 g of U . The
82 92
(A) 3α & 6β− (B) 4α & 5β−
238 9
half-life period of U is 4.51 x 10 year. If all the lead
(C) 5α & 4β− (D) 6α & 6β−
238
were assumed to have come from decay of U , what
92

is the age of earth? Q.4 In an α -decay the Kinetic energy of α particle is


48 MeV Q-value of the reaction is 50 MeV. The mass
238 238 236 number of the mother nucleus is : (Assume that
(b) An ore of U is found to contain U and U
92 92 92 daughter nucleus is in ground state)
238
in the weight ratio of 1:0.1. The half-life period of U (A) 96 (B) 100
92
9
is 4.5 x 10 year. Calculate the age of ore. (C) 104 (D) None of these

Q.5 In the uranium radioactive series the initial


Q.23 An experiment requires minimum β -activity
238
206
nucleus is U, and the final nucleus is Pb . When
produced at the rate of 346 β -particles per minute. The 92 82

99 the uranium nucleus decays to lead, the number of


half-life period of Mo which is a β -emitter is 66.6 hr.
42 α -particles emitted is. And the number of β -particles
Find the minimum amount of 99
Mo required to carry emitted.
42
(A) 6, 8 (B) 8, 6
out the experiment in 6.909 hour.
(C) 16, 6 (D) 32, 12
Exercise 2
Q.6 Activity of a radioactive substance is R1 at time t1
Nuclear Physics R
and R 2 at time t2 (t2 > t1 ) . Then the 2 is :
R1
Single Correct Choice Type t2 −λ (t1+ t2 )
(A) (B) e
t1
Q.1 The rest mass of the deuteron, 12 H , is equivalent
to an energy of 1876 MeV, the rest mass of a proton t −t  λ (t1− t2 )
(C) e  1 2  (D) e
is equivalent to 939 MeV and that of a neutron to 940  λ 
MeV. A deuteron may disintegrate to a proton and a
neutron if it : Q.7 A particular nucleus in a large population of
(A) Emits a γ -ray photon of energy 2 MeV identical radioactive nuclei did survive 5 half lives of
P hysi cs | 25.41

that isotope. Then the probability that this surviving formation of Y against time would look like
nucleus will survive the next half-life :
1 1 1 5
(A) (B) (C) (D)
32 5 2 2 (A) (B)

Y
Q.8 The activity of a sample reduces from A0 to A0 3
t t
in one hour. The activity after 3 hours more will be
A0 A0 A0 A0
(A) (B) (C) (D)
3 3 9 9 3 27 (C) (D)

Y
Y
Q.9 The activity of a sample of radioactive material is
t t
A1 at time t1 and A2 at time t2 (t2 > t1 ) . Its mean life
is T.
(E)

Y
A1 − A2
(A) A1t1 = A2 t2 (B) =constant
t2 − t1
t

(t1 − t2 )/T (t1 − T t2 )


(C) A2 = A1e (D) A2 = A1e Q.14 A radioactive substance is dissolved in a liquid
and the solution is heated. The activity of the solution

Q.10 A fraction ƒ1 of a radioactive sample decays in (A) Is smaller than that of element
one mean life, and a fraction ƒ2 decays in one half-life. (B) Is greater than that of element
(A) ƒ1 > ƒ2 (C) Is equal to that of element
(B) ƒ1 < ƒ2 (D) Will be smaller or greater depending upon whether
(C) ƒ1 = ƒ2 the solution is weak or concentrated.

(D) May be (A), (B) or (C) depending on the values of


Q.15 In a certain nuclear reactor, a radioactive nucleus
the mean life and half-life.
is being produced at a constant rate =1000/s. The
mean life of radionuclide is 40 minutes. At steady state,
Q.11 A radioactive substance is being produced at a the number of radionuclide will be
constant rate of 10 nuclei/s. The decay constant of the
substance is 1/ 2sec−1 . After what time the number of (A) 4×104 (B) 24×104 (C) 24 ×105 (D) 24×106
radioactive nuclei will become 10? Initially there are no
nuclei present. Assume decay law holds for the sample. Q.16 In the above question, if there were 20 x 105
radionuclide at t=0, then the graph of N v/s t is
(A) 2.45 sec (B) log (2) sec
1 N N
(C) 1.386 sec (D) sec (A) (B)
log(2) t t
O O
Q.12 The radioactivity of a sample is R1 at time T1 and N N
T2 . If the half-life of the specimen is T. Number of atoms (C) (D)
that have disintegrated in the (T2 − T1 ) is proportional t
O O
to

(A) (R1T1 − R 2 T2 ) (B) (R1 − R 2 )T Q.17 A free neutron is decayed into a proton but a free
proton is not decayed into a neutron. This is because-
(C) (R1 − R 2 ) / T (D) (R1 − R 2 )(T1 − T2 ) (A) Neutron is a composite particle made of a proton
and an electron whereas proton is a fundamental
Q.13 The radioactive nucleus of an element X decays particle
to a stable nucleus of element Y. A graph of rate of (B) Neutron is an uncharged particle whereas proton is
2 5 . 4 2 | Nuclear Physics and Radioactivity

a changed particle Q.22 The graph shown by the side  B


shows the variation of potential A
(C) Neutron has larger rest mass than the proton
energy φ of a proton with its distance r
(D) Weak forces can be operated in a neutron but not ‘r’ form a fixed sodium nucleus, as it O C
in a proton approaches the nucleus, placed at origin O. Then the
portion.
Multiple Correct Choice Type (A) AB indicates nuclear repulsion

Q.18 When a nucleus with atomic number Z and mass (B) AB indicates electrostatic repulsion
number A undergoes a radioactive decay process: (C) BC indicates nuclear attraction
(A) Both Z and A will decrease, if the process is α decay (D) BC represents electrostatic interaction
(B) Z will decrease but A will not change, if the process
is β+ decay Q.23 In β -decay, the Q-value of the process is E. Then

(C) Z will decrease but A will not change, if the process (A) K.E. of a β -particle cannot exceed E.

is β− decay (B) K.E. of antineutrino emitted lies between Zero and E.

(D) Z and A will remain unchanged, if the process is γ (C) N/X ratio of the nucleus is altered.
decay. (D) Mass number (A) of the nucleus is altered.

Q.19 When the atomic number A of the nucleus Q.24 Consider the following nuclear reactions and
increases select the correct statements from the option that
(A) Initially the neutron-proton ratio is constant=1 follow.

(B) Initially neutron-proton ratio increases and later Reaction I: n → p + e− + v


decreases Reaction II: p → n + e+ + v
(C) Initially binding energy per nucleon increases when (A) Free neutron is unstable, therefore reaction I is
the neutron-proton ratio increases. possible
(D) The binding energy per nucleon increases when the (B) Free proton is stable, therefore reaction II is not
neutron –proton ratio increases. possible
(C) Inside a nucleus, both decays (reaction I and II) are
Q.20 Let mp be the mass of a proton, ma the mass of possible
20
a neutron, M1 the mass of a 10 Ne
nucleus and M2 the (D) Inside a nucleus, reaction I is not possible but
40
reaction II is possible
mass of a 20 Ca
nucleus. Then
(A) M2 = 2M1 (B) M2 > 2M1 Q.25 When the nucleus of an electrically neutral atom
(C) M2 < 2M1 (D) M1 < 10(ma + mp ) undergoes a radioactive decay process, it will remain
neutral after the decay if the process is:
Q.21 The decay constant of a ratio active substance is (A) α decay (B) β -decay
0.173 (years)−1 . Therefore : (C) γ decay (D) K-capture
(A) Nearly 63% of the radioactive substance will decay
in (1/0.173) year. Q.26 The heavier nuclie tend to have larger N/Z ratio
(B) Half-life of the radioactive substance is (1/0.173) because-
year. (A) A neutron is heavier than a proton
(C) One-fourth of the radioactive substance will be left (B) A neutron is an unstable particle
after nearly 8 years.
(C) A neutron does not exert electric repulsion
(D) All the above statements are true.
(D) Coulomb forces have longer range compared to the
nuclear forces
P hysi cs | 25.43

Q.27 For nuclei with A>100 cause and the result can be
(A) The binding energy of the nucleus decreases on an (A) A nucleus of excess nucleons is α − active
average as A increases
(B) An excited nucleus of excess protons is β− active
(B) The binding energy per nucleon decreases on an
(C) An excited nucleus of excess protons is β+ active
average a A increases
(D) An nucleus of excess neutrons is β− active
(C) If the nucleus breaks into two roughly equal parts
energy is released
Assertion Reasoning Type
(D) If two nuclei fuse to form a bigger nucleus energy
is released (A) Statement-I is true, statement-II is true and
statement-II is correct explanation for statement-I.
Q.28 A radioactive sample has initial concentration no. (B) Statement-I is true, statement-II is true and statement-
of nuclei- II is NOT the correct explanation for statement-I
(A) The number of undecayed nuclei present in the (C) Statement-I is true, statement-II is false.
sample decays exponentially with time (D) Statement-I is false, statement-II is false.
(B) The activity (R) of the sample at any instant is
directly proportional to the number of undecayed Q.32 Half-life for certain radioactive element is 5 min.
nuclei present in that sample at that time Four nuclei of that element are observed a certain
(C) The no. of decayed nuclei grows exponentially with instant of time. After five minutes
time Statement-I: It can be definitely said that two nuclei
(D) The no. of decayed nuclei grow linearly with time will be left undecayed.
Statement-II: After half-life i.e. 5minutes, half of total
Q.29 A nuclide A undergoes α decay and another nuclei will disintegrate. So only two nuclei will be left
nuclide B undergoes β− decay- undecayed.
(A) All the α -particles emitted by A will have almost
the same speed Q.33 Statement-I: Consider the following nuclear of
14
(B) The α -particles emitted by A may have widely an unstable C6 nucleus initially at rest. The decay
different speeds 14 0
C →14 N + e + ν . In a nuclear reaction total energy
6 7
(C) All the β -particles emitted by B will have almost −1

the same speed and momentum is conserved experiments show that


the electrons are emitted with a continuous range of
(D) The β -particles emitted by B may have widely
kinetic energies upto some maximum value.
different speeds.
Statement-II: Remaining energy is released as thermal
14 energy.
Q.30 A nitrogen nucleus N absorbs a neutron and
7

can transform into lithium nucleus 3 Li7 under suitable Q.34 Statement-I: It is easy to remove a proton from
conditions, after emitting : 40
20 Ca
nucleus as compared to a neutron
(A) 4 protons and 3 neutrons
Statement-II: Inside nucleus neutrons are acted on
(B) 5 protons and 1 negative beta particle
only by attractive forces but protons are also acted on
(C) 1 alpha particle and 2 gamma particles by repulsive forces.
(D) 1 alpha particle, 4 protons and 2 negative beta
particles Q.35 Statement-I: It is possible for a thermal neutron
(E) 4 protons and 4 neutrons to be absorbed by a nucleus whereas a proton or an
α−particle would need a much larger amount of energy
for being absorbed by the same nucleus.
Q.31 The instability of the nucleus can be due to
various causes. An unstable nucleus emits radiations if Statement-II: Neutron is electrically neutral but proton
possible to transform into less unstable state. Then the and α− particle are positively charged.
2 5 . 4 4 | Nuclear Physics and Radioactivity

Comprehension Type
Column I Column II

Paragraph 1: (Q.36) A town has a population of 1 4


(C) Heavy water is (r) 2 He
million. The average electric power needed per person
is 300 W. A reactor is to be designed to supply power By emission of which particle
to this town. The efficiency with which thermal power is 14
(D) the position in the periodic (s) 7 N
converted into electric power is aimed at 25%. table is lowered by 2
When a deuterium is
Q.36 Assuming 200 MeV of thermal energy to come 16
8 O
bombarded on nucleus,
from each fission event on an average the number of (E) (t) D2 O
events that should take place every day. an α particle is emitted, the
product nucleus is
(A) 2.24 × 1024 (B) 3.24 × 1024
(C) 4.24 × 1024 (D) 5.24 × 1024
Q.41

Paragraph 2: A nucleus at rest undergoes a decay


emitting an α particle of de-Broglie wavelength Column I Column II
λ 5.76 × 10−15 m . The mass of the daughter nucleus is
= (A) Nuclear Fusion (p) Some matter
223.40 amu and that of α particle is 4.002 amu. converted into
energy
Q.37 The linear momentum of α particle and that of (B) Nuclear Fission (q) Generally occurs in
daughter nucleus is- nuclei having low
(A) 1.15 × 10−19 N − s & 2.25 × 10−19 N − s atomic number
(C) (r) Generally occurs in
(B) 2.25 × 10−19 N − s & 1.15 × 10−19 N − s β − decay
nuclei having higher
(C) Both 1.15 × 10−19 N − s atomic number.
(D) α − decay (s) Essentially occurs
(D) Both 2.25 × 10−19 N − s
due to weak nuclear
force.
Q.38 The kinetic energy of α particle is-
(A) 0.01 Mev (B) 6.22 MeV Q.42
(C) 0.21 Mev (D) 0.31 MeV
Column I Column II
Q.39 The kinetic energy of daughter nucleus is- (A) 1 Rutherford (p) 1 dis/sec

(A) 3.16 Mev (B) 4.16 MeV (B) 1 Becquerel (q)


3.7 × 1010 dis/sec
(C) 5.16 MeV (D) 0.11 MeV
(C) 1 Curie (r)
106 dis/sec
Match the Columns (D) Activity of 1g (s)
1010 dis/sec
Ra226
Q.40

Column I Column II
Radioactivity

(A) In reaction 12 H +12 H →32 He + X (p) 206


82 Pb Single Correct Choice Type
The X is

Q.43 The analysis of a mineral of Uranium reveals that


238
If U decays by 8 α & 6β 1 ratio of mole of 206 Pb and 238 U in sample is 0.2. If
(B) 92 (q) 0n
effective decay constant of process 238 U → 206 Pb is λ
the resulting nuclei is
then age of rock is
P hysi cs | 25.45

1 5 1 5 1 4 1 6 Q.48 Select correct statement (s):


(A) ln (B) ln   (C) ln (D) ln  
λ 4 λ  1 λ 1 λ 5 (A) The emission of gamma radiation involves transition
between energy levels within the nucleus.

Q.44 The half-life of Tc99 is6.0hr . The delivery of a (B) 24 He is formed due to emission of beta particle
sample of Tc99 that must be shipped in order for the from tritium 13 H .
n
lab to receive 10.0 mg? (C) When positron (o+1e) is emitted, ratio increases.
p
(A) 20.0 mg (B) 15.0 mg (D) In general, adsorption is exothermic process.
(C) 14.1 mg (D) 12.5 mg
Comprehension Type
Q.45 A sample contains 0.1 gram-atom of radioactive
Paragraph 1: Nuclei of a radioactive element ‘A’ are
isotope AZ X ( t1/2 = 5days ) . How many number of atoms being produced at a constant rate, α . The element has
a decay constant, λ . At time, t=0, there are N0 nuclei
will decay during eleventh day? [ NA =Avogadro’s of the element.
number]
 − 0.693×11 −
0.693×10  Q.49 The number of nuclei of A at time‘t’ is
(A) 0.1 −e 5 +e 5 
 
  α
(A) (1 − e−λt ) (B) N0 . eλt
 0.693×11 0.693×10  λ
(B) 0.1 −e 5 +e 5 
  N0 .α   λ  −λt 
  1
(C) [α − (α − λ N0 )e−λt ] (D) 1− 1 −  e 
 − 0.693×11 0.693×10  λ λ   α 

(C) 0.1 −e 5 +e 5  NA
 
 
Q.50 If =α 2N0 λ , the number of nuclei of A after one
 0.693×11 0.693×10  half-life of A becomes

(D) 0.1 −e 5 +e 5  NA
  (A) Zero (B) 2N0 (C) 1.5N0 (D) 0.5N0
 

Multiple Correct Choice Type Paragraph 2: Mass defect in the nuclear reactions
may be expressed in terms of the atomic masses of the
Q.46 Which of the following statements are correct parent and daughter nuclides in place of their nuclides
about half-life period? in place of their nuclear masses.
(A) It is proportional to initial concentration for zero-th
order. Q.51 The mass defect of nuclear reaction:
10
(B) Average life=1.44 half-life for first order reaction 4 Be → 5B10 + e is

(C) time of 75% reaction is thrice of half –life period =


in (A) ∆m At. mass of 10Be − At. mass of 10B
4 5
second order reaction.
= (B) ∆m At. mass of Be − At. mass of 10B − mass of
10
(D) 99.9% reaction takes place in 100 minutes for the one electron
4 5
−1
case when rate constant is 0.0693 min is 0.5
= (C) ∆m At. mass of 10 Be − At. mass of 10 B + mass of
4 5
one electron
Q.47 C14 is a beta active nucleus. A sample of C14H4
gas kept in a closed vessel shows increase in pressure = (D) ∆m At. mass of 10Be − At. mass of 10B – mass of
4 5
with time. This is due to two electron

(A) The formation of N14 H3 and H2


Q.52 The mass defect of the nuclear reaction:
(B) The formation of B11 H3 and H2
8
5B → 4 Be8 + e+ is
(C) The formation of C14 H4 and H2
(D) The formation of C12 H3 , N14 H2 and H2 (A) ∆m At. mass of 58B − At. mass of 8 Be
=
4
2 5 . 4 6 | Nuclear Physics and Radioactivity

(B) ∆m At. mass of 8 8


Be -mass of one Q.5 The element curium 248
96 Cm
has a mean life of 1013 s .
= 5B− At. mass of
4
electron Its primary decay modes are spontaneous fission and
α -decay, the former with a probability of 8% and the
(C) ∆m At. mass of
= 8
5B− At. mass of 8
Be + mass of latter with a probability of 92%, each fission releases
4
one electron 200 MeV of energy. The masses involved in decay are
8 8 as follows:  (1997)
(D) ∆m At. mass of
= 5B− At. mass of Be - mass of
4 248
two electron 96 Cm = 248.072220u,
244
= 94 Pu 244.064100u
= and 42He 4.002603u.
Previous Years’ Questions Calculate the power output from a sample of 1020 Cm
(
atoms. 1u = 931MeV / c2 )
Q.1 There is a stream of neutrons with a kinetic energy
of 0.0327 eV. If the half-life of neutrons is 700 s, what Q.6 Nuclei of a radioactive element A are being
fraction of neutrons will decay before they travel a produced at a constant rate α . The element has a
distance of 10m? (1986) decay constant λ . At time t = 0 , there are N0 nuclei of
the element.  (1998)
Q.2 It is proposed to use the nuclear fusion reaction; (a) Calculate the number N of nuclei of A at time t.
2 2
1H + 1H → 42He
α 2N0 λ , calculate the number of nuclei of A after
(b) If =
In a nuclear reactor 200MW rating. If the energy from
the above reaction is used with a 25 percent efficiency one hale-life of A and also the limiting value of N as
in the reactor, how many grams of deuterium fuel will t→∞.

be needed per day? (The masses of 21H and 24He are


Q.7 In a nuclear reactor 235U undergoes fission
2.0141 atomic mass units and 4.0026 atomic mass units liberating 200 MeV of energy. The reactor has a 10%
respectively.)  (1990) efficiency and produces 1000 MW power. If the reactor
is to function for 10yr, find the total mass of uranium
Q.3 A nucleus X, initially at rest, undergoes alpha-decay required.  (2001)

according to the equation 92A X → 228


zY +α.
Q.8 A radioactive nucleus X decays to a nucleus Y with a
(a) Find the values of A and Z in the above process. decay constant λ x = 0.1s−1 , Y further decays to a stable
(b) The alpha particle produced in the above process is nucleus Z with a decay constant λ y = 1 / 30s−1 . Initially,
found to move in a circular track of radius 0.11m in a there are only X nuclei and their number is N0 = 1020 .
uniform magnetic field of 3T. Find the energy (in Mev)
Set up the rate equations for the populations of X, Y
released during the process and the binding energy of
and z. The population of Y nucleus as a function of time
the parent nucleus X.
is given by
Given
= that m ( Y ) 228.03u;m
= 1
0n ( )
1.009u
Ny (=
t) {N λ / ( λ
0 x x − λy )} exp ( −λ t ) − exp ( −λ t )
y x

= ( )
m 42He 4.003u;m
= 1
1H( ) 1.008u.  (1991)
Find the time at which Ny is maximum and determine
the populations X and Z at that instant. (2001)
Q.4 A small quantity of solution containing Na24 radio
nuclide (half-life=15h) of activity 1.0 microcurie is Q.9 A rock is 1.5 × 109 yr old. The rock contains 238 U
injected into the blood of a person. A sample of the
which disintegrates to from 206 Pb . Assume that there
blood of volume 1 cm3 taken after 5h shows an activity
was no 206 Pb in the rock initially and it is the only
of 296 disintegrations per minute. Determine the total
volume of the blood in the body of the person. Assume stable product formed by the decay. Calculate the ratio
that the radioactive solution mixes uniformly in the of number of nuclei of 238 U to that of 206 Pb in the rock.
blood of the person.
10
Half-life of 238
(
U is 4.5 × 109 yr. 21/3 =
1.259  )
(2004)
(1 curie= 3.7 × 10 disintegrations per second)  (1994)
P hysi cs | 25.47

Q.10 To determine the half-life of a radioactive element, a Q.14 Assume that two deuteron nuclei in the core of
fusion reactor at temperature T are moving towards
dN ( t ) dN ( t )
student plots a graph of ln versus t. Here each other , each with kinetic energy 1.5kT, when the
dt dt separation between them is large enough to neglect
is the rate of radioactive decay at time t. If the number Coulomb potential energy. Also neglect any interaction
of radioactive nuclei of this element decreases by a from other particles in the core. The minimum
factor of p after 4.16yr, the value of p is (2010) temperature t required for them to reach a separation
of 4 × 10−15 m is in the range  (2009)
6
5 Q.15 Assume that the B/A
InldN(t)dt

4 nuclear binding energy


per nucleon (B/A) versus 8
3
mass number (A) is as 6
4
2 shown in the figure. Use
2
1 this plot to choose the
2 3 4 5 6 7 8 correct choice(s) given 0
100 200 A
Years
below.  (2008)
(A) Fusion of two nuclei with mass numbers lying in the
Q.11 The activity of a freshly prepared radioactive
range of 1 < A < 50 will release energy
sample is 1010 disintegrations per second, whose mean
life is 10−9 s. The mass of an atom of this radioisotope (B) Fusion of two nuclei with mass numbers lying in the
range of 51 < A < 100 will release energy
is 10−25 kg . The mass (in mg) of the radioactive sample
is (2011) (C) Fission of a nucleus lying in the mass range of 100 <
A < 200 will release energy when broken into two equal
fragments
Q.12 Some laws and processes are given in column
I. Match these with the physical phenomena given in (D) Fission of a nucleus lying in the mass range of 200
column II. (2006) < A < 260 will release energy when broken into two
equal fragments
Column I Column II
Q.16 Results of calculations for four different designs
(A) Nuclear Fusion (p) Converts some matter
of a fusion reactor using D-D reaction are given below.
into energy
Which of these is most promising based on Lawson
(B) Nuclear Fusion (q) Generally possible for
criterion?  (2009)
nuclei with low Atomic
number (A) Deuteron density = 2.0 ×1012cm−3, confinement
time = 5.0 ×10−3s
(C) (r) Generally possible
β − decay for nuclei with higher (B) Deuteron density = 8.0 ×1014cm−3, confinement
Atomic number. time = 9.0 ×10−1s
(D) Exothermic nuclear (s) Essentially proceeds by (C) Deuteron density = 4.0 ×1023cm−3,confinement time
reaction weak nuclear forces = 1.0 ×10−11s
(D) Deuteron density = 1.0 ×1024cm−3, confinement
Q.13 In the core of nuclear fusion reactor, the gas time = 4.0 ×10−12s
becomes plasma because of  (2009)
(A) Strong nuclear force acting between the deuterons Q.17 A freshly prepared sample of a radioisotope of
half-life 1386s has activity 103 disintegrations per
(B) Coulomb force acting between the deuterons second.
(C) Coulomb force acting between deuteron-electron Given that ln 2 = 0.693, the fraction of the initial number
pairs of nuclei (expressed in nearest integer percentage)
(D) The high temperature maintained inside the reactor that will decay in the first 80 s after preparation of the
core sample is  (2013)
(A) 4% (B) 5% (C) 5.5% (D) 3%
2 5 . 4 8 | Nuclear Physics and Radioactivity

Q.18 A nuclear power plant supplying electrical power Q.23 For a = 0, the value of d (maximum value of ρ as
to a village uses a radioactive material of half life T shown in the figure) is  (2008)
years as the fuel. The amount of fuel at the beginning is 3Ze 3Ze 4Ze Ze
such that the total power requirement of the village is (A) (B) (C) (D)
3 3 3
4 πR πR 3πR 3πR 3
12.5 % of the electrical power available form the plant
at that time. If the plant is able to meet the total power Q.24 The electric field within the nucleus is generally
needs of the village for a maximum period of nT years, observed to be linearly dependent on r. This implies.
then the value of n is  (2014)  (2008)
(A) 2 (B) 5 (C) 3 (D) 4 R 2R
(A) a = 0 (B) a = (C) a = R (D) a =
2 3
Q.19 Match the nuclear processes given in column I
with the appropriate option(s) in column II  (2015) Q.25 To determine the half-life of a radioactive element,
dN(t)
a student plots a graph of log
n versus t. Here
Column I Column II dt
dN(t)
(A) Nuclear fusion (p) Absorption of thermal is the rate of radioactive decay at time t. If the
neutrons by 235 U dt
92
number of radioactive nuclei of this element decreases
(B) Fission in a nuclear (q) 60
Co nucleus by a factor of p after 4.16 years, the value of p is (2009)
27
reactor
6
(C) (r) Energy production
β -decay
in stars via hydrogen 5
conversion to helium 4
(D) (s) Heavy water
γ -ray emission 3

(t) Neutrino emission 2

1
2 3 4 5 6 7 8
12
Q.20 The isotope 5
B having a mass 12.014 u Years
12 12
undergoes β decay to 6 C
. 6 C
has an excited state Q.26 What is the maximum energy of the anti-neutrino?
 (2012)
of the nucleus ( 12
6 C
*) at 4.041 MeV above its ground
(A) Zero
12 12
state. If 5
B decays to 6 C
*, the (1 u = 931.5 MeV/c2),
(B) Much less than 0.8 × 106 eV
where c is the speed of light in vacuum) (2016) (C) Nearly 0.8 × 106 eV
(D) Much larger than 0.8 × 106 eV
Q.21 A radioactive sample S1 having an activity 5µCi
has twice the number of nuclei as another sample S2
which has an activity of 10 µCi. The half lives of S1 and Q.27 If the anti-neutrino had a mass of 3eV/c2 (where c
S2 can be  (2008) is the speed of light) instead of zero mass, what should
be the range of the kinetic energy, K, of the electron?
(A) 20 years and 5 years, respectively  (2012)
(B) 20 years and 10 years, respectively (A) 0 < K < 0.8 × 106 eV
(C) 10 years each (B) 3.0 eV < K < 0.8 × 106 eV
(D) 5 years each (C) 3.0 eV < K < 0.8 × 106 eV
(D) 0 < K < 0.8 × 106 eV
Q.22 The electric field at r = R is  (2008)
(A) Independent of a Q.28 The radius of the orbit of an electron in a
(B) Directly proportional to a Hydrogen-like atom is 4.5 a0 where a0 is the Bohr radius.
3h
(C) Directly proportional to a2 Its orbital angular momentum is . It is given that

(D) Inversely proportional to a h is Planck’s constant and R is Rydberg constant. The
P hysi cs | 25.49

possible wavelength(s), when the atom de-excites, is


List I List II
(are)  (2013)
15
9 9 9 4 (i) Alpha decay (p) 8 O →15
7 N + .....
(A) (B) (C) (D)
32R 16R 5R 3R
258 234
(ii) b+ decay (q) 92 U →90 Th + .....
Direction: The mass of nucleus AZ X is less than the sum
of the masses of (A-Z) number of neutrons and Z number 185
→184
(iii) Fission (r) 83 Bi 82 Pb + .....
of protons in the nucleus. The energy equivalent to the
corresponding mass difference is known as the binding 239
(iv) Proton emission (s) 94 Pu →140
57 La + .....
energy of the nucleus. A heavy nucleus of mass M can
break into two light nuclei of mass m1 and m2 only if (m1
+ m2) < M. Also two light nuclei of masses m3 and m4 Codes:
can undergo complete fusion and form a heavy nucleus
p q r s
of mass M’ only if (m1 + m4) > M’. The masses of some
neutral atoms are given in the table below: (A) (iv) (ii) (i) (iii)

(B) (i) (iii) (ii) (iv)


1 2 (C) (ii) (i) (iv) (iii)
1H 1.007825 u 1H 2.014102 u
(D) (iv) (iii) (ii) (i)
6 7
3 Li 6.015123 u 3 Li 7.016004 u

Q.32 If λcu is the wavelength of K α X-ray line of copper


152 206
64 Gd 151.919803 u 82 Pb 205.974455 u
(atomic number 29) and λMo is the wavelength of the
3 4 K α X-ray line of molybdenum (atomic number 42),
1H 3.016050 u 1 He 4.002603 u
then the ratio λcu / λMo is close to (2014)
70 82
30 Zn 69.925325 u 34 Se 81.916709 u
(A) 1.99 (B) 2.14 (C) 0.50 (D) 0.48
209 210
83 Bi 208.980388 u 84 Po 209.982876 u
Q.33 An electron in an excited state of Li2+ ion has
angular momentum 3h/2 π . The de Broglie wavelength
Q.29 The correct statement is  (2013) of the electron in this state is pπa0 (where a0 is the Bohr
radius). The value of p is (2015)
6
(A) The nucleus 3
Li can emit an alpha particle (A) πa0 (B) 2πa0 (C) 4πa0 (D) 3πa0
210
(B) The nucleus 84
Po can emit a proton.
(C) Deuteron and alpha particle can undergo complete Q.34 For a radioactive material, its activity A and rate
fusion. dN
of change of its activity R are defined as A = – and
(D) The nuclei 70
Zn and 82
Se can undergo complete dt
30 34 dA
fusion. R= dA – , where N(t) is the number of nuclei at time
dt
t. Two radioactive sources P (mean life τ ) and Q(mean
Q.30 The kinetic energy (in keV) of the alpha particle, life 2τ ) have the same activity at t = 0. Their rates of
when the nucleus 210
84
Po at rest undergoes alpha decay, change of activities at t = 2τ are RP and RQ, respectively.
is  (2013) R n
If P = , then the value of n is  (2015)
(A) 5319 (B) 5422 (C) 5707 (D) 5818 RQ e
1 2 3 2
(A) (B) (C) (D)
Q.31 Match List I of the nuclear processes with List II 2e e e 3e
containing parent nucleus and one of the end products
of each process and then select the correct answer Q.35 A fission reaction is given by
using the codes given below the lists: (2013)
236
92 U →140 94
54 Xe +38 Sr + x + y,
where x and y are two
2 5 . 5 0 | Nuclear Physics and Radioactivity

particles. Considering 236


to be at rest, the kinetic are 1.008665 u, 1.007825 u, 15.000109 u and 15.003065
92 U
u, respectively. Given that the radii of both the 15 7 N
and
energies of the products are denoted by KXe, KSr, 15
O nuclei are same, 1 u = 931.5 MeV/c 2
(c is the speed
8
Kx(2MeV) and Ky(2MeV), respectively. Let the binding
of light) and e2/ (4 πε0 ) = 1.44 MeV fm. Assuming that
236
energies per nucleon of , 140
92 U 54 Xe
and 94
38 Sr
be 7.5 the difference between the binding energies of 15 7 N
15
and 8 O is purely due to the electrostatic energy, the
MeV, 8.5 MeV and 8.5 MeV respectively. Considering
different conservation laws, the correct option(s) is(are) radius of either of the nuclei is (1 fm = 10-15 m) (2016)
 (2015) (A) 2.85 fm (B) 3.03 fm
(A) x = n, y = n, KSr = 129MeV, KXe= 86 MeV (C) 3.42 fm (D) 3.80 fm
(B) x = p, y = e-, KSr= 129 MeV, KXe= 86 MeV
(C) x = p, y = n, KSr= 129 MeV, KXe = 86 MeV Q.37 An accident in a nuclear laboratory resulted
in deposition of a certain amount of radioactive
(D) x = n, y = n, KSr= 86 MeV, KXe= 129 MeV material of half-life 18 days inside the laboratory. Tests
revealed that the radiation was 64 times more than
Q.36 The electrostatic energy of Z protons uniformly the permissible level required for safe operation of the
distributed throughout a spherical nucleus of radius R laboratory. What is the minimum number of days after
which the laboratory can be considered safe for use?
3 Z(Z– 1)e2  (2016)
is given by E =
5 4 πε0R
(A) 64 (B) 90 (C) 108 (D) 120
The measured masses of the neutron, 11H, 15
7 N
and 15
8 O

PlancEssential Questions
JEE Main/Boards JEE Advanced/Boards
Exercise 1 Exercise 1
Q.5 Q.13 Q.28 Q. 7 Q.8 Q.13 Q.14
Q.31 Q.34 Q.40 Q.15 Q.17 Q.19 Q.23

Exercise 2
Exercise 2 Q.1 Q.12 Q.13 Q.22
Q.6 Q.11 Q.23 Q.27 Q.37 Q.38 Q.39
Q.24 Q.26 Q.27 Q.43 Q.44 Q.40 Q.49
Q.29 Q.50

Previous Years’ Question


Q.1 Q.7 Q.10 Q.12
Q.14
P hysi cs | 25.51

Answer Key

JEE Main/Boards
Exercise 1
Nuclear Physics

Q.1 56.45 days Q.2 449.94 year Q.3 7s


α
Q.4 Q.5 4.57 × 1021days−1 Q.6 384.5g
λ

Radioactivity

Q.26 beta emitter: 49 Ca, 30 Al, 94 Kr, positron emitter : 195


Hg, 8 B, 150 Ho

Q.27 114
49 In,
odd number of neutrons Q.28 ( a) 11H, (b ) 01n, ( c ) 63Li, ( d) +01e, ( e) −01e, ( f ) +11p
−1
Q.29 λ =2.078hr Q.30 5.05×106 atoms

Q.31 6.25% Q.32 2.67×105 sec-1

40 40
Q.33 33.67 years Q.34 (i) 19 K → 18 Ar +10 e + v (ii) 2.8 × 109 years

Q.35 (i) tmeans = 14.43s (ii) 40 sec Q.36 ∆E =14.25 Mev


206
Q.37 (a) No.of α -particles=8, No.of β -particles=6; (b) Pb
82

Q.38 6.13 × 10−7 g Q.39 (i) 31.25 cm3 ,27.104 cm3 (ii) 4.5 × 109 year

Q.40 6.30 × 10−4 yr −1 ,3.087 × 10−2 yr −1

Exercise 2

Nuclear Physics

Single Correct Choice Type


Q.1 C Q.2 B Q.3 B Q.4 A Q.5 B Q.6 A
Q.7 B Q.8 B Q.9 A Q.10 A

Radioactivity

Single Correct Choice Type


Q.11 B Q.12 C Q.13 A Q.14 B Q.15 B Q.16 B
Q.17 C Q.18 A Q.19 D Q.20 A Q.21 A Q.22 C
Q.23 A Q.24 C Q.25 A Q.26 C Q.27 C Q.28 B
Q.29 A Q.30 B Q.31 D
2 5 . 5 2 | Nuclear Physics and Radioactivity

Previous Years’ Question


Q.1. B Q.2 C Q.3 C Q.5 C Q.6 B Q.7 A
Q.8 A Q.9 125decays/sec Q.10 Alpha=8, beta=6 Q.11 24 Mev Q.12 D
Q.13 B Q.14 A Q.15 D Q.16 C Q.17 B Q.18 B
Q.19 B Q.20 B Q.21 D Q.22 A Q.23 A Q.24 C

JEE Advanced/Boards
Exercise 1

Nuclear Physics

40 40 9
Q.1 23.6 Mev Q.2 (i) 19 K → 18 Ar +10 e + v (ii) 4.2 × 10 years

 ln5 
Q.3
= t  τ Q.4 2.73×1018sec Q.8 1.7 × 1010 years
 ln2 
Q.9 5196 yrs Q.10 28 Mev Q.11 9.00×106 eV


α 
0.2E0 αt − 1 − e−λι 
λ 
( )
Q.12 v=-uλt Q.13 ∆T =
ms

Radioactivity

Q.14 t= 4.89 × 109 years Q.15 ( T1/2 = 10.8 sec )

Q.16 6 litre Q.17 239Pu=44.7%, 240Pu=55.3%

3N0
Q.18 (a) N
=
1
λ 
 ( )
α 1 − e−λι + λN0 e−λι  (b)
 2
,2N0

Q.19 0.833×10-5mol/lit sec

t 7.1× 108 years


Q.20 = Q.21 4.125 min

Q.22 (a) 1.143 × 109 year , (b) 7.097 × 108 year Q.23 3.43 × 10−18 mol

Exercise 2

Nuclear Physics

Single Correct Choice Type


Q.1 D Q.2 D Q.3 C Q.4 B Q.5 B Q.6 D
Q.7 C Q.8 B Q.9 C Q.10 A Q.11 C Q.12 B
Q.13 E Q.14 C Q.15 C Q.16 B Q.17 C
P hysi cs | 25.53

Multiple Correct Choice Type


Q.18 A, B, D Q.19 A, C Q.20 C, D Q.21 A, C Q.22 B, C Q.23 A, B, C
Q.24 A, B, C Q.25 C, D Q.26 C, D Q.27 B, C Q.28 A, B, C Q.29 A, D
Q.30 D, E Q.31 A, C, D

Assertion Reasoning Type


Q.32 D Q.33 C Q.34 A Q.35 A

Comprehension Type
Q.36 B Q.37 C Q.38 B Q.39 D

Matric Match Type


Q.40 A→q; B→p; C→t; D→r; E→s
Q.41 A→p, q; B→p, r; C→p, s; D→r, s
Q.42 A→r; B→p; C→q; D→q

Radioactivity

Single Correct Choice Type


Q.43 D Q.44 C Q.45 C

Multiple Correct Choice Type


Q.46 A, B, C, D Q.47 B, C, D Q.48 C, D

Comprehension Type
Q.49 C Q.50 C Q.51 A Q.52 D

Previous Years’ Questions

Q.1 3.96 × 10−6 Q.2 120.26 g Q.3 1823.2 MeV Q.4 V=5.95 L Q.5 3.32 × 10−5 W

Q.7 3.847 × 10 4 kg Q.9 3.861 Q.10 8 Q.11 1

Q.12 A → p, q; B → p, r; C → p, s; D → p, q, r Q.13 D Q.14 T = 1.4 × 109 K

Q.15 B, D Q.16 B Q.17 A Q.18 C

Q.19 A → r, t; B → p, s; C → p, q, r, t; D → p, q, r, t Q.20 9 MeV Q.21 A

Q.22 A Q.23 B Q.24 C Q.25 8 Q.26 C

Q.27 D Q.28 A, C Q.29 C Q.30 A Q.31 C

Q.32 B Q.33 B Q.34 B Q.35 A Q.36 C

Q.37 C
2 5 . 5 4 | Nuclear Physics and Radioactivity

Solutions

JEE Main/Boards ⇒ 2λ = 2ln2 – ln3


⇒ λ = (ln2 – (ln3)/2) sec–1
Exercise 1 Now mean life

ln2  
Sol 1: t1/2 = = 10 1  
λ 1
= =   sec
λ 
 ln2 −
( ln3 ) 

ln2
⇒λ= (days)–1  2 
10
 
N 1  1 
Now, = and N = N0e–λt =   = 6.9 ≈ 7 sec
N0 50  0.6931 − (1.0986) 
 2 
1
⇒ e–λt = ⇒ ln 50 = lt
50
dN dN
10 × ln50 Sol 4: (a) = α – lN ⇒ + lN = a
⇒t= = 56.44 days dt dt
ln2
⇒ ∫ d N.eλt  = ∫ α.e
λt 
. dt
  
1 1
Sol 2: l1 = years–1 and l2 = years–1 t t
1620 405 ⇒ N.eλt  = α .eλt  / λ
 0  0

dN dN ⇒ N . elt – N0 = (α . elt – α)/l


Now, = – (l1t + l2t) ⇒ = – (λ1 + λ2) t
dt dt α
⇒ N = N0 . e–lt + (1 – e–lt)
−λtot .t λ
⇒ N = N0 . e
(B) α =2N0l
2.ln2 2.ln2
So, 2t1/2 = = ln2
λ tot 1 1 After one half life, t1/2 =
+ λ
1620 405
ln2
810.ln2 4 × 810.ln2 So, t =
= = = 449 years λ
1 5
1+ α
4 N = N0 . e− ln2 + . (1 – e− ln2 )
λ
N0 α
Sol 3: N = N0 . e−λt = + . (1 – 1/2)
2 λ
So in 1st 2 sec,
N0 α  α 1
DN1 = N0 . – N0 . e– λ 2 = N0 . (1 – e–2λ) N= + =  N0 +  ×
2 2λ  λ 2
in other 2 sec,
Now, as t → ∞,
DN2 = N0 . e–2λ – N0 . e–4λ = N0 . e–2λ (1 – e–2λ )
α α
−2λ N = N0 (0) + (1 – 0) ⇒ N =
N0 .(1 − e ) n 4 λ λ
Now, = =
N0 .e −2λ
.(1– e −2λ
) 0.75n 3

3 Sol 5: 84Po210 → 2α4 + 82Pb206


⇒ = e–2λ
4 ln2
So, t1/2 = = 138.6 days
4 λ
⇒ e2λ =
3
P hysi cs | 25.55

ln2 = 3.125 × 1016 fissions


⇒λ= (days)–1
138.6 Number of fissions in 1 year
Now, Mass defect = 3.125 × 1016 × 365 × 24 × 60 × 60
= 209.98264 – (205.97440 + 4.00260) = 9.855 × 1023
= 0.00564 amu Moles of uranium required = 1.637 moles
= 5.251 MeV. Mass of Uranium = 384.5 g
1.6 × 10 –19
J = 1 eV
Sol 7:
So, Mass defect = 1.6 × 10–19 × 106 × 5.251
= 8.4 × 10–13 J
So to produce 1.2 × 107 J energy (at 0.1 efficiency)
Number of reactions B.E.
 dN 
8.4 × 10–13π   × 0.1 = 1.2 × 10
7

 dt  20 40 60 80
 dN  1.2 1 Mass numbers A
⇒   = × 1021 = × 1021
 dt  8.4 7
Now higher the BE/nucleon higher the stability.
dN 1 BE
Now, = lN = × 1021 So light nuclei try to get high ratio by going
dt 7 Nucleon
1 through nuclear fusion and hence increasing their
⇒ λ.N0.e–lt = × 1021
7 atomic number.
1 1
⇒ N0 = × 1021 × elt .
7 λ Sol 8: Now number of particles decaying is directly
 ln2 ×693  proportional to the number of particles present in the
1 1
= × 1021 ×  e138.6  × ×138.6 reaction.
7   ln2
  dN
i.e. ∝N
= 28.56 × 32 × 10 21
dt
N0= 9.13 × 1023 ⇒ This is equated by a constant known as decaying
constant.
No
Now, number of moles = = 1.52 dN
6 × 1023 = λN
dt
So mass = 1.52 × 210 gm = 319.2 gm (i) X-rays and gamma rays both electromagnetic.
Initial activity = lN0 (ii) γ-rays
ln2 (iii) γ-rays
= × 9.13 × 1023 = 4.6 × 1021 days-1
138.6
(iv) β-rays (Both (–) ve)

Sol 6: Energy per fission = 200 MeV


= 200 × 10 eV 6
( ) ( )
Sol 9: Mass defect m 36 Li + m 10 n – m
 ( H) + m ( H)
4
2
3
1

= 200 × 106 eV ⇒ (6.015126 + 1.008665)

= 200 × 106 × 1.6 × 10–19 J – (4.002604 + 3.016049)

= 3.2 × 10–11 J = 0.010697 amu = 9.96 MeV

Now, number of fissions required / time


Sol 10: n/p ratio decreases due to beta decay.
1× 106 10
= = × 1018 i.e. 6C14 → 7N14 + β–1 + ν
3.2 × 10 −11 3.2
2 5 . 5 6 | Nuclear Physics and Radioactivity

n 8 4 n 7 = mass before reaction – mass after reaction


⇒ = = ,| = =1
p 6 3 p 7 =[(6.015126) + (1.0086554)] – [4.0026044 + 3.0100000]
= 0.011177 m
Sol 11: Decay constant refer ans. 8
So energy released = 0.011177 × 931 MeV
Half-life period: the time taken by a disintegration
reaction to half the total number of particles in a sample. = 10.405 MeV

Sol 12: Mass defect = 2.0141 + 6.0155 – 2x (4.0026) Sol 16: Activity = rate of change of number of particles
in a disintegration reaction.
= 0.0244 m
dN
So energy transferred to KE SI unit ⇒
dt
= (0.0244) × 931 MeV
⇒ SI Unit = sec–1
= 22.7164 MeV
So energy for each particle
Sol 17: (i) Graph
22.7164
= = 11.36 MeV
2
= 11.36 × 1.6 × 10–19 × 106
Potential energy (MeV)
= 18.176 × 10–13 = 1.8176 × 10–12 J
100
−3
2.2 × 10
Sol 13: Number of moles =
11 0

= 0.2 × 10–3 = 0.2 × 10–4 moles


-100
(i) Number of moles × A0 = Number of particles r0 1 2 3
= 6.0022 × 1023 × 2 × 10–4 r(fm)

= 12.044 × 1019 (ii) For r > r0 Attraction


dN (iii) For r < r0 Repulsion
(ii) Activity = lN =
dt

 N  Sol 18: Refer Q.7


 λ
ln2 5 × 10−6 Mass defect = 20 × mass of proton + 20 × mass of
So = × × 6 × 1023 = 1.54 × 1014 40
1224 11 neutron – [mass of 20 Ca ]
= 20×[1.0007825 + 1.008665] – 39.962589
Sol 14: Half-life period: sec
= 0.226361 u
dN
Decay constant ⇒ sec – =4N×λ
–1
So, energy = (Dmc2)
dt
⇒ N = N0 . e–lt 931
= 0.226361 × MeV × c2
Now, N = N0 / 2 c2
λt1/2 = 210 MeV
⇒ e =2
⇒ lt1/2 = ln2 Sol 19: (a) Nuclear forces are short-ranged. They are
ln2 most effective only up to a distance of the order of a
⇒ t1/2 = femtometre or less.
λ
(b) Nuclear forces are much stronger than electro-
Sol 15: (i) 36 Li + 10n → 24 α + triton magnetic forces.
(c) Nuclear forces are independent of charge.
(ii) Mass defect
P hysi cs | 25.57

Sol 20: Mass defect = – [mass of 234


90 Th
Radioactivity

+ mass of 24 He ] + mass of 238


92 U Sol 26: For same atomic No.

= – [234.043630 + 4.002600] + 238.05079 If mass No. of an isotope > mass no. of most stable
isotope
= 0.00456 u
Then isotope is a beta emitter → n/p ratio increases
So energy released = 0.00456 × 931.5 MeV = 4.25 MeV otherwise positron emitter → n/p ratio decreases
49 30 94
Sol 21: Radius = R0[A]1/3 20 Ca ; 13 Al ; 36 Cr → beta emitter

1/3
R1 [A1 ]1/3 R1  1
So, = ⇒ =   Sol 27: Odd no. of neutrons
R2 [A2 ]1/3 R2 8 
14
R1 1 Sol 28: (a) 7 N + 24 He → 17
8 O + 11H
=
R2 2
(b) 24 Be + 24 He → 12
6 C + 10 n

Sol 22: (a) It is because of the fact that the binding (c) 94 Be (p, α) → 36 Li + 24 He + 10 P
energy of the particle has to be (+) ve. [i.e. every
system tries to minimise its energy] some of its mass is 30 30 0
converted into energy. (d) 15 P → 14 S + +1e

Sol 23: For stability, binding energy/nucleon should be (e) 13 H → 32 He + 0


−1e

high. Since it is highest at some intermediate atomic


43
number, the elements with large atomic number try (f) 20 Ca + 24 He → 46
21 Sc + 11P
to increase the binding energy/nucleon by fission.
Similarly elements with small atomic number tries to 1 1
increase B.E./nucleon using fusion. Sol 29: =
64 26

Sol 24: Refer Q.16 6t1/2 = 2 × 3600 s


t1/2 = 1200 sec
dN
Plot: – = lN = activity
dt ln2 ln2
t1/2 = ;λ=
λ t1 / 2
0.693
λ=
1200
λ = 5.775 × 10–4 sec–1
λ = 2.079 hr–1
t
40
Sol 30: n = = 3.252
Sol 25: For A > 30, the stability of the nucleus increases 12.3
as more and more nucleons are introduced because
10
of minimization of potential energy. Because of this, × NA × 8 × 10 −18
initially the B.E./nucleon increases. No. of atoms = 1
23.252
But at high mass number, the size of the nucleus starts
to increase and because of this, as the nucleons are = 5.05 × 106 atoms.
weaker for larger distance, the electrostatic repulsion
between the protons starts to dominate over them 1
Sol 31: % of radiation = 100 × % =6.25%
and thus on further increase in the mass number (the 24
nucleus starts to become unstable).
2 5 . 5 8 | Nuclear Physics and Radioactivity

0.693 Sol 36: 12 H + 12 H → 32 He + 10 n


Sol 32: k =
t1
2 Dm = 2 × 2.020 – (3.0160 + 1.0087)
t 1 = 30 days Dm = 0.0153 amu
2
0.0153 × 10−3
0.693 Dm = = 2.54 × 10-29 kg
k= sec−1 6.022 × 1023
30 × 24 × 60 × 60
E = Dm × c2 = 2.28 × 10-12 J
As N = 1011 atoms
dN 2.28 × 10−12
− kN
= E=
dt 1.6 × 10−19
E = 14.25 MeV
dN 0.693 × 1011
− = sec−1
dt 30 × 24 × 60 × 60
Sol 37: (a) 92U238 → 82Pb206 + 8 24 He + 6 0
−1e
= 2.67 × 105 sec‒1 α-Particles = 8
β-Particles = 6
1 15
Sol 33: = 206
t 100 (b) 90Th234 → 82 Pb + 7 24 He + 6 0
−1e
t1/2
2
t 20

t1/2 100 Sol 38: Remains of Sr40 = 1 × 10-6 × 2 28.1
⇒ 2 =
15 = 0.613 mg
 100 
ln  
t 15  Sol 39: (i) 84Po210 → 82Pb206 + 24 He
= 
t1/2 ln2
 1 1
Moles of helium produced = 1 −  ×
 100   2  210
t1/2 × ln  
 15  
t= 1  1
ln2 1 − × × 8.314 × 273
nRT 2  210
V= =  = 31.25 cm3
t = 33.66 yrs P 1.01325 × 105

V × mPoO
Sol 34: t1/2 = 1.4 × 109 (ii) V’ = 2
= 27.104 cm3
mPo
Nuclear reaction: -
(iii) U238 t1/2 = 4.5 × 109 yrs
40 40
(i) 19 K → 18 Ar +10 e+v
–0.1 mole U238 0.1 mole Pb206
(ii) Age = 2t1/2 = 2.8 × 1018 years Age of ore = t1/2 = 4.5 × 109 yr.

Sol 35: t1/2 = 10 sec ln2


Sol 40: λ = λ 1 + l2 ; l1 + l2 =
t1/2
(i) tmean = 1.443 × t1/2 = 14.43 sec
ln2 1
100 l1 + l2 = ; λ1 = l
(ii) 2 = n
22 49 2
6.25
2n = 16 ln2
l1 = = 6.301 × 10–4 year–1
22 × 50
n=4
t = 4 t1/2 = 40 sec 49 × ln2
l2 = = 3.087×10–2 year–1
22 × 50
P hysi cs | 25.59

Exercise 2 Sol 8: (B) λ =


ln2
=
ln2
(years)–1
t1/2 1620
Single Correct Choice Type
ln2
= (hours)–1
1620 × 365 × 24
Sol 1: (C) Carbon-12 is taken as standard
Now, N = N0.e–lt
Sol 2: (B) R = R0 . A
1/3
5
where N0 = × 6.022 × 1023
223
Sol 3: (B) 0
So N = N0.e–l×5
Energy released
So decayed particles
= (8.2 × 90 + 8.2 × 110 – 7.4 × 200) MeV
= N0 – N = N0(1 – e–λ5)
= (0.8 × 200) MeV = 160 MeV
5
× 6.022 × 1023  
5ln2
= −
1620×365×24 
223 1 − e
Sol 4: (A) Energy = (7.5 × 13 – 12 × 7.68) MeV  
 
= 5.34 MeV = 3.29 × 10 15

Sol 5: (B) 2X → Y + Q Sol 9: (A) The end product of radioactive series is stable
and hence the decay constant is zero.
Binding energy is the (–) ve energy
1
From energy conservation C

–2E1 = – E2 + Q ⇒ Q = E2 – 2E1 Sol 10: (A) A


2
Sol 6: (A) We know that half life is given as
dN
0.693 Now, = – (l1N + l2N)
T= dt
λ
dN
Given that λ ' =1 : 2 ⇒ = –(l1 + l2)N.
dt
T λ' 2
∴ = =
T' λ 1
Thus, for probabilities of getting α and β particles at the Radioactivity
same time t = 0, the ratio will be the same 2 : 1
Single Correct Choice Type

Sol 7: (B) Half-life = 5 years, time given = 10 years =


2 half-lives Sol 11: (B) 29 27
+ 2 11n + 2 0
13 Al → 13 Al −1B
n
N 1
= 
N0  2 
Sol 12: (C) 10 n → 11p + 0
−1β
2
1
Or N =   N0 −10 λ0 t
2 N1 A1 e e−10/9
Sol 13: (A) = = = = e–1
N2 A2 e
−λ0 t
e−1/9
1
Or
= N = N 0.25 N0
4 0 256
Sol 14: (B) m = g = 4g
∴ 25% substance left hence probability of decay 26
−k1t −k 2 t
= 100 ‒ 25 = 75% k1e k 2e k1 (k1−k 2 )t
= ; = e
(k 2 − k1 ) (k 2 − k1 ) k 2
2 5 . 6 0 | Nuclear Physics and Radioactivity

k  dN
ln  1  Sol 22: (C) R = = -lN
dt
tmax. =  k 2  = ln(k 2 / k1 )
(k1 − k 2 ) (k 2 − k1 ) ln2 λ t1/2
2
l= ; 1 = =2
t1/2 λ2 t1/2
1
Sol 15: (B) Reaction need not be exothermic. N1 N/2
=
N2 N/ 2
x
Sol 16: (B) 238 4
92 M → y N +2 2 He ; xy N → BA L + 2 10 β+
R1 λ1N1 1
= =2× = 2
X = 230 A = 230 R2 λ2N2 2
Y = 88 B = 86
Neutrons = 230 – 86 = 144 ln2 180
Sol 23: (A) ×N=
t1/2 60

200 200 ln2


Sol 17: (C) m = = = 3.125 g × 6.022 × 1023 × 1.3 × 10-12 = 3
6
2 64 t1/2

6.022 × 1.3 × ln2


Sol 18: (A) 4 24 He → 16 t1/2 = × 10-11
8 O 3
Dm = 4 × 4.0026 – 15.834 = 1.808 × 1011 = 0.18 × 10-12 sec
= 16.0104 – 15.834 = 0.1764 amu
Sol 24: (C) In a γ decay energy of atom is reduced,
B.E. per nucleon atomic mass and atomic number remains the same.
1
= × 0.1764 × 931 MeV = 10.24 MeV
16
Sol 25: (A) 1 fm << radius of atom
1 1
Sol 19: (D) ≤ ∴ Repulsive forces dominate.
2n 10
Fpp > Fpn = Fnn
2t/30 ≥ 10
Fpn and Fnn would be negligible compared to repulsive
⇒ t/30 ≥ log210
forces of protons.
30ln10
⇒t≥ λ
ln2 1
Sol 26: (C) A → B l1 = 1.8 × 10-2 sec-1
⇒ t ≥ 99.65 ≈ 100 λ
2 → C l = 10-3 sec-1
2A  2

Sol 20: (A) ba X → a−1


+ 10 n λ = l1 + 2l2 = 18 × 10-3 + 2 × 10-3
bX
= 2 × 10-2
t1/2y 1 1
Sol 21: (A) (i) t1/2x = tmean = = = 50 sec
ln2 λ 2 × 10−2
t1/2x > t1/2y
Sol 27: (C) Initially, NB = 8NA
∴ Y Decays faster.
Finally, N’A = 2N’B
(ii) True
NA NB
(iii) 4t1/2 = 400 ms = 2t/50 = 2t/10
N'A N'B
(iv) v ∝ m
8NA NB
3 =
(v) No. of disintegrated nucleus= N0 N'A 2N'B
4
3 16 × 2t/50 = 2t/10
Probability =
4
P hysi cs | 25.61

t t H2 + 1H3 → 2He4 + n
+4 t t 1
250 = 210 ⇒ - = -4 31H2 → 2He4 + n + p
50 10
4t Mass defect
= 40 ⇒ t = 50 min
50 Dm = (3×2.014– 4.001–1.007–1.008) amu
3
60 × 10 = 0.026 amu
Sol 28: (B) A = 104 ×
10 Energy released = 0.026 × 931 MeV
= 6 × 107 dis/min = 106 dps
= 0.026 × 931 × 1.6 × 10–13 J
6
10
Activity = Curie = 27 µCi = 3.87 × 10–12 J
3.7 × 1010
This is the energy produced by the consumption of
Sol 29: (A) Age = t1/2 three deuteron atoms.
∴ Total energy released by 1040 deuterons
Sol 30: (B) t1/2 = 69.3 min. 10 40
= × 3.87 × 10–12 J
0.693 1 3
λ= = min-1 = min-1
69.3 100 = 1.29 × 1028 J
lN = 10; N = 10/λ = 1000 atoms The average power radiated is P = 1016 W or 1016 J/s
Therefore, total time to exhaust all deuterons of the star
Sol 31: (D) R1 = lN1 will be
R2 = lN2 1.29 × 1028
t= = 1.29 × 1012 s ≈ 1012 s
Atoms disintegrated = (N1 – N2) 1016
 R − R2   R1 − R 2 
=  1  =   T Sol 6: (B) Heavy water is used as moderators in nuclear
 λ   ln2  reactors to slow down the neutrons.

Previous Years’ Questions Sol 7: (A) Penetrating power is maximum for γ-rays,
then of β-particles and then α-particles because
basically it depends on the velocity. However, ionization
Sol 1: (B) Using N = N0e–lt power is in reverse order.
ln2 ln(2) N –
ln(2)
t
where λ = = ∴ 0 =N0 e 3.8 1
t1/2 3.8 20 Sol 8: (A) Activity of S1 = (activity of S2)
2
Solving this equation with the help of given data we 1 λ N
or l1N1 = (l2N2) or 1 = 2
find: t = 16.5 days 2 λ2 2N1

T1 2N1 ln2
Sol 2: (C) Beta particles are fast moving electrons which or = (T = half-life= )
are emitted by the nucleus. T2 N2 λ

T1
Given N1 = 2N2 ∴ =4
Sol 3: (C) During fusion process two or more lighter T2
nuclei combine to form a heavy nucleus.
∴ Correct option is (A).

Sol 4: Following nuclear reaction takes place n


Sol 9: R = R0  1 
 
0
n1 → 1H1 + –1e0 + ν 2
ν is antineutrino Here R0 = initial activity =1000 disintegration/s
and n = number of half-lives.
Sol 5: (C) The given reaction are :
At t = 1s, n = 1
1
H2 + 1H2 → 1H3 + p
2 5 . 6 2 | Nuclear Physics and Radioactivity

 1 M 2
∴ R = 103   = 500 disintegration/s ∆mc2 =V
2 2 1
At t = 3s, n = 3
2∆mc2
3 = V12
 1 M
R = 103  2  = 125 disintegration/s
  2∆m
V1 = c
M
Sol 10: Number of α-particles emitted
238 – 206 Sol 18: (B) In positive beta decay a proton is transformed
n1 = =8 into a neutron and a positron is emitted.
4
and number of β-particles emitted are say n2, then 92 p+ → n0 + e+
– 8 × 2 + n2 = 82 no. of neutrons initially was A – Z
\ n2 = 6 no. of neutrons after decay (A – Z) – 3 x 2 (due to alpha
particles) + 2 x 1 (due to positive beta decay)
Sol 11: Q = (Dm in atomic mass unit) × 931.4 MeV
The no. of proton will reduce by 8. [as 3 x 2 (due to
= (2 × mass of 1H2 – mass of 2He4) × 931.4 MeV alpha particles) + 2(due to positive beta decay)]
= (2 × 2.0141 – 4.0024) × 931.4 MeV Hence atomic number reduces by 8.
Q ≈ 24 MeV Z2
Sol 19: (B) En = –13.6
n2
Sol 12: (D) Binding energy per nucleon increases for 9
lighter nuclei and decreases for heavy nuclei. E ++= –13.6 ×= –122.4eV
Li 1
9
k mv 2 E = –13.6 ×= –13.6eV
Sol 13: (B) = Li+++ 9
r r
∆E =–13.6 – (–122.4)
mv2 = k (independent or r)
 h  1 2 = 108.8 eV
n =  mvr ⇒ r ∝ n and T = mv is independent
 2π  2
of n. Sol 20: (B) t 1 = 20 minutes
2
−λt2
N N0 e
= λt1 ln3
=
Sol 14: (A) 1st reaction is fusion and 4th reaction is
fission. 2 −λt2 1
= N0 N= 0e t1 ln3
 1  3 λ
1
Sol 15: (D) IR corresponds to least value of  − 
 n2 n2  2 −λt
 1 2  N = N0 e 2
3 0
i.e. from Paschen, Bracket and Pfund series. Thus the
1 3
transition corresponds to 5 →3. t2 = ln
λ 2

Sol 16: (C) After decay, the daughter nuclei will be 1 3  1  1  0.693
t2 − =
t1 ln − ln3
=  ln  =
 = 20 min
more stable hence binding energy per nucleon will be λ 2  λ 2 λ
more than that of their parent nucleus.

M M
Sol 17: (B) Conserving the momentum 0 = V1 – V2 Sol 21: (D) Number of spectral lines from a state n to
2 2
V 1 = V2 … (i) n(n– 1)
ground state is = =6
2
1 M 2 1 M 2
∆mc2 = ⋅ V + ⋅ ⋅V  … (ii)
2 2 1 2 2 2 Sol 22: (A) ∆m(mp + me ) – mn = 9 x 10–31 kg.
P hysi cs | 25.63

Energy released = (9 x 10–31 kg)c2 joules R


N. elt = . [elt – 1]
9 × 10 −31
× (3 × 10 )8 2 λ
= MeV = 0.73 MeV.
1.6 × 10−13 R
⇒N= [1 – e–λt]
2
λ
Z
Sol 23: (A) KE ∝   as n decreases KE increases and So for eq. as t → ∞, N → R/λ,
n
TE, PE decreases So for N = 0.8 R/l
R R
0.8 = [1 – e–lt]
λ λ
Sol 24: (C)
4 1
A B ⇒ = 1 – e–lt ⇒ = e–lt
5 5
TA = 20 min TB = 40 min ⇒ lt = ln5
1 ln5
 N 1 1− ⇒
1 −  1 − t/t 80 1 15 λ
2 1−
 N0 A 2 1/2 20 16 16 5 ln2 ln5
= = = = = and give, λ = t
⇒= ×τ
 N  1 1 1 3 4 τ ln2
 1 −  1 − t/t 1− 1−
80 4 4
 N0 B 2 1/2 2
40 Sol 4: 4 hydrogen atom produces 26 MeV energy.
⇒ 4g (4 moles) hydrogen atom produces
JEE Advanced/Boards ⇒ [26 × 6.022×1023] MeV(energy)
= 26 × 6.022 × 1023 × 1.6 × 10–19 × 106 Joule
Exercise 1
= 26 × 6.022 × 1.6 × 1010 Joule
= 250.51 × 1010 Joules
Sol 1: 1H2 +1 H2 →2 He4
⇒ 1.7 × 1030 kg = 1.7 × 1033 g H produces
Binding energy of deuteron
250.51× 1010
= (1. 1) × 2 MeV = × 1.7 × 1033
4
= 2.2 MeV
250.51× 1.7
= × 1043 Joules
Binding energy of helium 4
= 2He4 = 7 × 4 = 1.065 × 1045 Joules
= 28 MeV Now power × time = total energy
So total energy released 1.065 × 10 45
⇒ time = = 2.73 × 1018 sec
= 28 – 2.2 × 2 = 28 – 4.4 3.9 × 1026
= 23.6 MeV
Sol 5: We have, N= N0.e–lt
40 40 0
Sol 2: (i) 19 K → 18 Ar + +1e +ν So, NU = N0 .e
−λ1t
235

(ii) 4.2 × 109 years −λ2t


NU = N0 .e
238

dN NU
Sol 3: = R – lN 238 ( λ1−λ2 )t
dt = e
NU
dN 235
+ lN = R
dt 140 ( λ −λ )t
Given = e 1 2
N,t t 1
⇒ ∫
0,0
(
d N.eλt ) = ∫ R.e
0
λt
.dt ⇒ ln 140 = (l1 – l2) × t
2 5 . 6 4 | Nuclear Physics and Radioactivity

Now, initial activity = λ.N0


ln140
⇒t=
λ1 − λ2 Activity at some t = λ.N0.e–lt
So, 6.4 = λ.N0.e–lt and 12 = lN0
ln2 ln2
Now, l1 = , l2 = ⇒ e–lt = 6.4/12
7.13 × 108 4.5 × 109
⇒ lt = ln (12/6.4)
ln140 5730 × ln(12 / 6.4)
So t = years ⇒ t= = 5196 years
 1 1  ln2
ln2  8
− 9
 7.3 × 10 4.5 × 10 
Sol 10: 12 H +12 H → 24 H + 23.6 MeV
ln140
= × 109 years = 6.21 × 109 years Now we have, B.E. of He > B.E. of deuterium for the
 10 1  reaction to happen.
ln2  − 
 7.3 4.5 
⇒ B.E. of helium = B.E. of Deuterium + 23.6 MeV
Sol 6: Now, as the momentum of the nucleus and = 4 × 1.1 + 23.6 = 4.4 + 23.6 = 28 MeV
α-particle are same (momentum conservation)
P2 Sol 11: π+ → µ+ + ν →P
Energy = , is divided in the inverse ratio of their (meson) (muon) (neutrino)
2m 150MeV 100MeV
respective masses. Now assume momentum of ν = P
So, let the energy of nucleus after disintegration be K, Now 150 = 100 + KEµ+ + KEν (energy conservation)
then
⇒ 50 = KEµ+ + KEν
4.78 222 111
= = Also using momentum conservation
K 4 2
2 × 4.78 2m + KE = P and KEν = Pc
⇒K= = 0.086 MeV µ µ+
111
Total Energy = (4.78+0.086) MeV = 4.87 MeV ⇒ KEν = c. 2m + KE
µ µ+

dN
Sol 7: We have, = lN = λ.N0.e–lt ⇒ 50 = KEµ+ + 2m + c2KE
dt µ µ+

⇒ 50 = KEµ+ + 200KEµ+
Sol 8: We have Number of particles

2.5 × 10−3 ⇒ 50 = KEµ+ + 10 2KEµ+


= × 6.022 × 1023
230
⇒ (50 – x)2 = 200 x (x = KE)
dN
So = (–lN) ⇒ lN = 8.4 sec–1 ⇒ x2 – 100x – 200 x + 2500 = 0
dt
⇒ x2 – 300x – 2500 = 0
1 N
⇒ = sec 300 ± 90000 − 10000
λ 8.4 x=
2
ln2 ln2 × N
⇒ = t1/2 = sec
λ 8.4
−3 23
=
300 ± 200 2
2
(
= 150 ± 100 2 MeV )
ln2 × 2.5 × 10 × 6.022 × 10
=
230 × 8.4 × 365 × 24 × 60 × 60
years = 1.7 × 1010 years
(
= 150 − 100 2 MeV )
= (150 – 141) MeV= 9 MeV
Sol 9: Activity / gm = 320/50 = 68.4 min–1
ln2 ln2
λ= = (years)-1 Sol 12: m v dm m-dm
t1/2 5730 V+dv
(v+dv–)
P hysi cs | 25.65

Momentum conservation ln 0.783


T1/2 × = -t
mv = (m – dm) (v + dv) + dm(v + dv – u) ln2
mv = mv + mdv – dmv + dmv – u.dm 1.39 × 1010 × 0.244
t=
0.693
⇒ mdv =u.dm
v m t = 4.906 × 109 yrs
dm
⇒ ∫ dv = u ∫
0 m0
m
Sol 15: N0 (1 – e-36λ) = 105
⇒ v = u . ln m/m0
N0 (1 – e-108λ) = 1.11 × 55
Now, m = m0.e –lt
⇒ m/m0 = e –lt

1 − e−36λ 100
⇒ v = u. (–lt) = – ult =
1− e −108 λ 111
So v is opposite to u.
Let e–36λ = t e–108λ = t3
m0
1− t 100
Px =
Sol 13: mI mT mp Px 1− t 3 111
111 – 111 t = 100 – 100 t3
Py 100 t3 – 111 t + 11 = 0
From the conservation of momentum the above (t – 1) (100 t2 + 100 t – 11) = 0
diagram can be deduced
t ≠ 1 t = –1/2 + 3/5
Now, Q = K.E. after collision – K.E. before collision
t = 1/10

=
Py 2
+
(P x
2
+ Py 2 )– Px 2 1
2m0 2mp 2mI e–36λ = ⇒ –36λ = –ln 10
10

=
Py 2
+
Px 2

Px 2
+
(P x
2
+ Py 2 )– Px 2 λ=
ln10
36
2m0 2m0 2m0 2mp 2mI
ln2
P 2 + P 2   m  P 2 P 2 T1/2 = × 36
ln10
= 
x y
 . 1 + p  – x – x
 2mp   m0  2m0 2mI T1/2 = 10.8 sec
   
 mp  P 2  mI  A 1
= Kp . 1 + – x 1 +  Sol 16: =
 m  2mI
0  m0  A0 21/3

 mp   mI  296
Q = Kp . 1 +  – KI . 1 +  60 1
 m0   m0  =
1 21/3
3.7 × 10 4 ×
V
Radioactivity
3.7 × 10 4 × 60
V= ⇒ V = 5952.753 cm3
296 × 21/3
Sol 14: 90Th232 → 82Pb208 + 6 2He4 + 4 0+ β
V = 5.592 m3
1.01325 × 105 × 8 × 10−5 × 10−6
nHe =
8.314 × 273 0.693
Sol 17: l1 = = 2.84 × 10-5 yr-1
nHe = 3.571 × 10 -9 2.44 × 10 4

nTh = 2.155 × 10–9 0.693


l2 = = 1.139 × 10-4 yr-1
3
nTh = 2.75 ×10 -9 6.08 × 10
0
−t
nTh T1/2
l1 = 9 × 10-13 ; l2 = 3.61 × 10-12
= 0.783 = 2
nTh
0
2 5 . 6 6 | Nuclear Physics and Radioactivity

A = l1 N1 + l2 N2 dm d(n / v) 1 dPH
= =
6 × 10 =6.02×10
9 23
dt dt RT dt
 9 × 10−13 × x 3.61× 10−12 (1 − x)  1
 +  = × - 0.0102 × 1.2
 239 240  RT
 
90x 361(1 − x) 0.0102 × 1.2
1= + =– molarity/min
239 240 0.0821× 298
−64679x + 86279 = 0.833 × 10-5 molarity/sec.
1=
239 × 240
1 0.1
x = 0.447 Sol 20: nU = , nPb =
238 206
%239Pu = 44.7% t
t1/2 n0
%240Pu = 55.3% 2 =
n

dN t ln(n0 / n)
Sol 18: (a) = α - lN =
dt t1/2 ln2
dn t1/2 × ln(n0 / n)
= dt
α − λN t1 =
l ln2
 α − λN  n 
ln   = –lt n  0  1/ 238 
 α − λN  n  = ln   =0.119
 0     1/ 238 − 0.1/ 206 
α − λN
= e−λt 4.5 × 109 × 0.1227
α − λN0 t=
ln2
1
N= [α – (α – lN0) e–lt] t = 7.75 × 108 years
λ
(b) t = t1/2 214 214
218 λ1 λ2
Sol 21: 84 Po →82 Pb  →83 Pb + −1ο β
1  (α − λN0 ) 
N= α −
  dN
λ  2  = l1N1 – λ 2N
dt
1  α λN0  1 dN −λ t
=  +  = (1.5 lN0) = λ1No e 1 − λ2N
λ 2 2 λ dt
 
N = 1.5 N0
N=
λ1N0 e ( −λ1t
−e
−λ2t
)
α (λ2 − λ1 )
lim N
= = 2No
t →∞ λ dN
=0
dt
dPH
Sol 19: = - kPH ⇒ l1N0 e
−λ1t
= l2 N
dt

1.2
−50
t1/2 l1N0
−λ t
e 1 =
λ1λ2N0 e ( −λ1t
−e
−λ2t
)
= 2 (λ 2 − λ1 )
2
 2  50 −λ1t −λ1t −λ2t
ln   = ln 2 (l2 – l1) e = l2 e - l2 e
 1.2  t1/2
−λ1t −λ2t
50 ln2 (l1) e = l2 e
t1/2 = = 67.84 min
ln(2 / 1.2) λ1
( λ1−λ2 )t
e =
ln2 λ2
k= = 0.0102
t1/2 ln(λ1 / λ2 )
t=
(λ1 − λ2 )
P hysi cs | 25.67

ln ( 2.68 / 3.05 ) Solving for n0, we get


t= = 4.12 min
 1 1  2.3818 × 10−19 mol
ln2  −  n0
= = 3.43 × 10 −18 mol
 3.05 2.68  1 − 0.9306
 1.667 
Sol 22: (a) Given at time t; 238
92 U = 1.667g =   mole
 238  Exercise 2
206  0.277 
83 Pb
= 0.277g
=   mole Single Correct Choice Type
 206 
Since all lead has been formed from U238 and therefore Sol 1: (D) 2
→ 1P1 + 0 n1 + Energy
1H
1876MeV 939MeV 940MeV
 0.277 
moles of U decayed = Moles of Pb formed =  
 206  So energy conservation gives
∴ Total moles of U before decay (N0) = moles of U at ⇒ 1876 = 939 + 940 + E
time t (N)
⇒ E = – 3MeV
1.667 0.277 2.303 N So a γ ray has to be absorbed
= × t = log 0
238 206 λ N

 1.667   0.277  Sol 2: (C) Mass number is constant as no nucleoid is


9 +  emitted.
2.303 × 4.51 × 10  238   206 
= log
0.693  1.667 
  Sol 3: (D) Mass of 20 is released and charge of 6 is
 238 
released from nucleus 20 mass ⇒ 5a.
(a) t = 1.143 × 109 year
(b) 7.097 × 108 year
Sol 4: (B) Given that k.Eα = 48 Mev, Q = 50 Mev
A − 4
Sol 23: Minimum β-activity required = 346 min‒1 We know that k.Eα = Q  
 A 
Number of β-activity required to carry out the
Here, A is the mass number of mother nucleus
experiment for 6.909 h = (346 min‒1) (6.909 × 60 min)
= 143431 Putting the values, we get
Amount of β-activity required A − 4
⇒ 48 = 50   ⇒ 48 A = 50 A ‒ 200
143431  A 
= = 2.3818 × 10−19 mol ⇒ A = 100
6.022 × 10 mol−1
23

Now, the rate constant of radioactive decay is Sol 5: (B) In the uranium radioactive series  the initial
0.693 0.693 nucleus is 8 alpha and 6 beta particles are released as it
=λ = = 0.010404 h−1 is a 4n + 2 series.
t1 66.6 h
2 dN
Now using the integrated rate expression Sol 6: (D) Activity = =N×l
dt
n0 − nconsumed λt So λ . N0 . e–lt = acitivity (R)
log = − ,
n0 2.303
– λ t2
R2 λ.N0 .e λ (t1− t2 )
n0 − 2.3818 × 10 −19
mol = = e
We get log R1 λ.N0 .e
– λt1
n0

(0.010404 h−1 ) (6.909 h) Sol 7: (C) Just like tossing of a coin, S heads won’t
=
− − 0.03121 or
=
2.303 change probability of next outcome, after any half-life,
1
n0 − 2.3818 × 10−19 mol there is probability of any atom surviving.
= 0.9306 2
n0
2 5 . 6 8 | Nuclear Physics and Radioactivity

A0 dN
Sol 8: (B) = A0.e–lt = A0.e–l Sol 13: (E) Rate = = lN(t) = lN0.e–lt ⇒(E)
3 dt
1
⇒λ= ln3 Sol 14: (C) depends on the number of elements and
2
activities inside nucleus.
Activity = lN = lN0.e–4λ = A0.e–4λ = A0/9

Sol 15: (C) Mean life = 1/λ


Sol 9: (C) So act. (t1) = lN1 = λ.N0.e–lt1 = A1
1
⇒ λ = 1/40 (min–1) = sec–1
So act. (t2) = lN2 = λ.N0.e–lt2 = A2 2400

dN
A1 So = – lN + R
So = e
λ (t2 − t1)
⇒ A2 = A1 . e
(t2 − t1)/T dt
A2
R 
⇒ N= . 1 − e−λt 
λ  
Sol 10: (A) f1 > f2 ⇒ 63º decays in mean life At steady state, t → ∞,
R 103
dN ⇒ N= = = 24 × 105
Sol 11: (C) = R – lN λ 1/ 2400
dt
dN Sol 16: (B) at t = 0, N and at t → ∞, N = const.
⇒ lN + =R
dt
N,t t
Sol 17: (C) Because neutron has larger rest mass than
⇒ ∫ d N.eλt  = ∫ R.eλt .dt proton.
 
0,0 0

R  λt  Multiple Correct Choice Type


N.elt = . e −1
λ  

R  Sol 18: (A, B, D)  nuclear attraction is there, (no rep.)


⇒N= . 1 − e−λt 
λ   (B) as r ↑ the energy ↓ ⇒ it is electrostatic
So, N = 10, R = 10 (C)  Nuclear attraction

10  (D) 
⇒10 = 1 − e− t/2 
1/ 2  
Sol 19: (A, C) Refer theory.
⇒ e–t/2 = 1/2
⇒ ln2 = t/2
Sol 20: (C, D) Due to mass defect (which is finally
⇒ t = 2ln2 = 0.693 responsible for the binding energy of the nucleus),
mass of a nucleus is always less than the sum of masses
Sol 12: (B) R1 = λ.N0. e
−λT1
and R2 = λ.N0. e
−λT2 of its constituent particles.
20
10 Ne is made up of 10 protons plus 10 neutrons.
ln2 ln2 20
t1/2 = T = ⇒ λ= Therefore, mass of 10 Ne nucleus, M1 < 10 (mp + mn)
λ T
Also, heavier the nucleus, more is the mass defect.
Number of atoms disintegrated = N1 – N2
Thus, 20(mn + mp) – M2 > 10(mp + mn) – M1
−λT1 −λT2
= N 0. e – N0. e Or 10(mp + mn) > M2 – M1
Or M2< M1 + 10(mp + mn)
R1 − R 2 T(R1 − R 2 )
= = Now since M1 < 10(mp + mn)
λ ln2
∴ M2 < 2M1
P hysi cs | 25.69

0.693
Sol 21: (A,=
C) T = 2 (C)  N = N0(1 – e–lt)
λ
(D) 
∴ Decay time = n × Half life.
8
∴ n= = 2n Sol 29: (A, D) A → A−4
+ 24 He2+
ZA z −2 A
N 41 1
∴ =  = A A
N0  2  4 ZB → z +1B + β+ ν

(A)  Now as the α-particle is alone, the energy could


Sol 22: (B, C) A, B, negative slope, +F; transfer to the α-particle only and from momentum
B, C, positive slope, -F cons., the particles will have same v.
(C) 
Sol 23: (A, B, C) (A) true (cons. of energy) (D)  Now during β-decay, the anti-neutrino is also
(B) True (cons. of energy). [energy can’t be generated emitted with the β-particle and thus energy can be
from anywhere alse] distributed between them.

(C)  as either N ↓ (β–) and P ↑ or N ↑ or P ↓ (β+) Sol 30: (D, E) 14


+ 10 n → 7
+ ‘ some more ’
7 N 3 Li
(D) mass number is const. so (ABC) elements

Sol 24: (A, B, C) (A)  free neutron is unstable Now mass number should be same

(B) free proton is stable 14 + 1 = 7 + x ⇒ x = 8

(C)  B–and B+ decay (So the products should have mass number = 8) (D) 
and (E) 
(D) both are possible ABC
Now charge also has to balance in D and E.
Sol 25: (C, D) (A) as 24 He2+ has charge in it 1α ⇒ 2 + 4 – 2 = 4
(B) as (+) 1 charge is there in neutron Similarly, (E) is also correct and 4Pº + 2B
(C)  γ decay (no charge transfer) So (D), (E)
(D)  inside the atom, no change in charge.
Sol 31: (A, C, D) (A) more nucleons ⇒ release of
(C, D) nucleons as α particles
(B) Protons in excess ⇒ B+ release ⇒ (C) is 
Sol 26: (C, D)
(D) β– is reduced then protons are increased and
(C) 
neutrons are decreased in a nucleus.
(D) 
⇒ (A), (C), (D)
Because there is comparatively more distance between
protons inside the nucleus, electric repulsion is more Assertion Reasoning Type
because nuclear forces are small as compared to
electrostatic when distance is high. Sol 32: (D) Because the statement is valid for large
number of nuclei
Sol 27: (B, C)
(B)  Sol 33: (C) Remaining energy is given to the anti-
neutrino particles
(C)  At high A, BE /nucleon is more

Sol 34: (A) True exp.


Sol 28: (A, B, C) (A)  N = N0.e–lt
dN Sol 35: (A) True exp.
(B)  = – lN
dt
2 5 . 7 0 | Nuclear Physics and Radioactivity

Comprehension Type Match the Columns

Paragraph 1 Sol 40: (A) → q charge balance


(B) → p (238 – 32) = 206 Þ (B)
Sol 36: (B) 1 million = 10,00,000 = 106 person
(C) → t (Theory)
Electric power = 300 × 106 = 3 × 108 watts
(D) → r (Znew = Z – 2)
3 × 108
Thermal power = = 12 × 108 watts (E) → s 16 + 2 – 4 = 14 (mass no.)
0.25
t = 24 × 60 × 60
Sol 41: (A) → p and q
So N × 200 × 106 × 1.6 × 10–19 J = 12 × 108 × t
Matter into energy (mass defect is observed)
12 × 108 × 24 × 60 × 60
⇒N= Materials combine (low atomic no.)
200 × 1.6 × 10−13
(B) → (p) mass defect
12 × 24 × 60 × 60
= ×1021 = 3.24 × 1024 (r) big nucleus disintegrates into smaller ones
2 × 1.6
(C) → (p) mass defect
Paragraph 2 (r) weak nuclear forces are Responsible
h (D) → (r)
Sol 37: (C) Now, = 5.76 × 10–15 m
p (s)
−34
6.6 × 10
⇒P= = 1.15 × 10–19 N-s
5.76 × 10 −15 Sol 42: (A) → r (from def.)
As from cons. of momentum, their mom should be (B) → p (from def.)
same
(C) → q (from def.)
2
P (D) → q (from def.)
Sol 38: (B) P = mv = = KE
2m

(1.15 × 10−19 )2 Radioactivity


⇒ KE= J
2 × 4 × 1.6 × 10−27 Single Correct Choice Type
2 −38 2 −11
(1.15) × 10 (1.15) × 10 t
= J = eV 1 5 t 6
8 × 1.6 × 10−27 8 × 1.6 × 1.6 × 10−19 Sol 43: (D) = ⇒ 2 1/2 =
t 6 5
t1/2
(1.15)2 2
= ×108 eV = 6.22 ×106
8 × 1.6 × 1.6 t ln6 / 5
=
t1/2 ln2
P2
Sol 39: (D) KE =
2m t1/2 ln6 / 5 1
t= = ln 6/5
(1.15 × 10 −19 2
) ln2 λ
= J
2 × 223.4 × 1.6 × 10−27
Sol 44: (C) For Tc99 = t1/2
(1.15)2 × 10−11
= eV t1/2 = 6.0 hr
2 × 223.4 × 1.6 × 1.6 × 10−19
Let the minimum amount be x
(1.15)2
= ×10 eV
8
x
2 × 223.4 × (1.6)2 Concentration after 3 hr =
2
x
= 0.11 MeV ≥ 10.0 mg ⇒ x ≥ 10.0 × 2 mg
2
x ≥ 14.1 mg
P hysi cs | 25.71

Sol 45: (C) N11 = 0.1 e–l×11 α – lN = (α – lN0) e-lt ⇒ lN = α – (α – lN0) e-lt
N10 = 0.1 e–l×10 1
⇒N= [α – (α – lN0) e-lt]
Atoms decaying during 11th day λ

= N10 – N11 = 0.1 (e–10λ – e–11λ) ln2


Sol 50: (C) t1/2 =
λ
 − ln2×11 −
ln2×10 
= 0.1  −e 5 + e 5  1 −λ
ln2
1   α − λN0 
  N= (α – (α – lN0) e λ
)=  α −   
  λ λ  2
  

1  α λN0  1  2N0 λ λN0 


Multiple Correct Choice Type =  +  =  + 
λ 2 2  λ  2 2 
Sol 46: (A, B, C, D) (A) t1/2 ∝ C1-n
N = 1.5 N0
where n is the order of the reaction.
1 t Paragraph 2
(B) tavg = = 1/2
λ ln2
10 10
dN Sol 51: (A) 4 Be
→ 5 Be
+ e-
(C) = -lN2
dt Dm = (4mp + 6mn) – (5mp + 5mn) – me
dN 1 1 +4me –5me +me
= -ldt; = lt; N =
N2 N λt
= At. mass of 4Be10 – At mass of 5B10
0.693
(D) t1/2 = = 10 min
0.0693
100 100 Sol 52: (D) 58 Be → 84 Be + e+
% Reactant = = = 0.098
210 1024 Dm = (5mp + 3mn) – (4mp + 4mn) – me
Reaction completions = 99.92% +5me –4me – me
Dm = At. mass of 5B8 – At mass of 4Be8
14 14 0
Sol 47: (B, C, D) 6 C → 7 N + −1β
– mass of two electrons
2
C H4 → 2N H3 + H2
14 14

(A)
Previous Years’ Questions
Sol 48: (C, D) (A) Within the atom not nucleus
Sol 1: Speed of neutrons
(B) 13 H → 32 He + 0
−1β

(C) 11P → 0
+1β + 10 n 2K  1 
=  from=
K mν2 
∴ n/p ratio increases m  2 

(D) True 2 × 0.0327 × 1.6 × 10 –19


or v = ≈ 2.5 × 103m/s
1.675 × 10 –27
Comprehension Type
Time taken by the neutrons to travel a distance of 10 m:
Paragraph 1 d 10
t= = = 4.0 × 10–3
v 2.5 × 103
dN Number of neutrons decayed after time
Sol 49: (C) = a - lN;
dt t : N = N0(1 – e–lt)
dN
= dt ∴ Fraction of neutrons that will decay in this time
α − λN interval
ln(2)
−1  α − λN  α − λN N – × 4.0×10 –3
ln
n  = t ⇒ = e-lt = = (1 – e–lt) = 1 – e 700 = 3.96 × 10–6
λ  α − λN α − λ N N0
 0  0
2 5 . 7 2 | Nuclear Physics and Radioactivity

Sol 2: Mass defect in the given nuclear reaction: Total binding energy of daughter products
Dm = 2(mass of deuterium) – (mass of helium) = [92× (mass of proton) + (232 – 92) (mass of neutron)
– (mγ) – (mα)] ×931.48 MeV
= 2(2.0141) – (4.0026) = 0.0256
= [(92×1.008) + (140)(1.009) – 228.03 – 4.003]
Therefore, energy released
×931.48 MeV
∆E = (Dm)(931.48)MeV = 23.85 MeV
= 1828.5 MeV
= 23.85 × 1.6 × 10–13 J = 3.82× 10–12J
∴ Binding energy of parent nucleus
Efficiency is only 25%, therefore,
 25  = binding energy of daughter products –
25% of ∆E =   (3.82 × 10 ) J
–12

 100  energy released


= 9.55 × 10–13 J = (1828.5 – 5.3) MeV = 1823.2 MeV
i.e., by the fusion of two deuterium nuclei, 9.55 × 10 –13
J
energy is available to the nuclear reactor. Sol 4: λ = Disintegration constant
Total energy required in one day to run the reactor with a 0.693 0.693 –1
= h = 0.0462 h–1
given power of 200 MW: t1/2 15
Etotal = 200×106×24×3600=1.728×1013J
Let R0 = initial activity = 1 microcurie
∴ Total number of deuterium nuclei required for this
= 3.7 × 104 disintegration per second.
purpose
r = Activity in 1 cm3 of blood at t = 5 h
Etotal 2 × 1.728 × 1013
n= = =0.362 ×10 26
296
∆E / 2 9.55 × 10 –13 = disintegration per second
60
∴ Mass of deuterium required
= 4.93 disintegration per second, and
= (Number of g-moles of deuterium required)
R = Activity of whole blood at time t = 5 h
×2g
 0.362 × 1026  Total volume of blood should be
=   × 2 = 120.26 g.
 6.02 × 1023 
  R R e – λt
V= = 0
r r
Sol 3: (a) A – 4 = 228
Substituting the values, we have
∴ A = 232  3.7 × 10 4  –(0.0462)(5)
V=  e cm3
92 – 2 = Z or Z = 90  4.93 
 
(b) From the relation, V = 5.95 × 103 cm3
2Km or V = 5.95 L
r=
Bq

r 2B2q2 (0.11)2 (3)2 (2 × 1.6 × 10 –19 )2 Sol 5: The reaction involved in α-decay is
Kα = =
2m 2 × 4.003 × 1.67 × 10 –27 × 1.6 × 10 –13 248 244
96 Cm → 94 Pu + 24 He
= 5.21 MeV
Mass defect
From the conservation of momentum, 248 244
Dm = mass of 96 Cm – mass of 94 Pu – mass of 24 He
or pγ = pα or 2K γ mγ = 2K αmα (248.072220 – 244.064100 – 4.002603) u

m  = 0.005517 u
4.003
∴Kγ =  α  Kα = ×5.21=0.09 MeV
 mγ  228.03 Therefore, energy released in α-decay will be
 
Eα = (0.005517 × 931) MeV = 5.136 MeV
∴ Total energy released = Kα +Kγ = 5.3 MeV
Similarly, Efission = 200 MeV (given)
P hysi cs | 25.73

1 or 200 ×1.6 × 10–13 J or 3.2 × 10–11 J of energy. So, number


Means life is given as tmean = 10 s =
13
of uranium atoms needed are
λ
3.1536 × 1018
∴ Disintegration constant λ = 10–13 s–1 = 0.9855× 1029
3.2 × 10 –11
Rate of decay at the moment when number of nuclei
Or number of kg-moles of uranium needed are
are 1020 = lN = (10–13)(1020)
0.9855 × 1029
= 107 disintegration per second n= = 163.7
6.02 × 1026
Of these disintegrations. 8% are in fission and 92% are
Hence, total mass of uranium required is
in α-decay
m = (n)M = (163.7)(235) kg
Therefore, energy released per second
or m ≈ 38470 kg or m = 3.847 × 104 kg
= (0.08 × 107 × 200 + 0.92 × 107 × 5.136) MeV
= 2.074 × 108 MeV Sol 8: (a) Let at time t = t, number of nuclei of Y and Z
∴ Power output (in watt) are NY and NZ. Then
= energy released per second (J/s) Rate equations of the populations of X, Y and Z are
= (2.074 × 108) (1.6 × 10–13)  dNx 
  = – lX NX ... (i)
∴ Power output = 3.32 × 10–5 W  dt 
 dNY 
Sol 6: (a) Let at time t, number of radioactive nuclei are   = lXNX – lYNY ... (ii)
 dt 
N. Net rate of formation of nuclei of A
 dN 
dN and  Z  = lYNY ... (iii)
= α – lN  dt 
dt
N t N0 λ X –λ t –λ t
dN dN (b) Given NY(t) = [e Y – e X ]
or
α – λN
= dt or ∫ α – λN ∫0
= dt λX – λY
N0
solving this equation, we get For Ny to be maximum
dNY (t)
1 =0
N= [α – (α – lN0) e–lt]  ... (i) dt
λ
ln(2) i.e., lXNX = lYNY ... (iv)
(b) (i) Substituting α = 2lN0 and t = t1/2 in equation
(i) we get, λ [from Equation (ii)]

3 N0 λ X – λY t – λX t
N= N or lX(N0 e– λ X t ) = lY [e – e ]
2 0 λX – λY
(ii) Substituting α = 2lN0 and t → ∞ in Equation (i), we λX – λY e
– λY t
λX ( λ X – λ Y )t
get or = – 1 ; = e
λY – λX t λY
e
α
N= = 2N0 or N = 2N0 λ 
λ or (lX – lY)t ln(e) = ln  X 
 λY 
Sol 7: The reactor produces 1000 MW power or 109 1 λ 
J/s. The reactor is to function for 10 yr. Therefore, total or t = ln  X 
λX – λY  λY 
energy which the reactor will supply in 10 yr is
E = (power)(time) Substituting the values of lX and lY, we have

= (109 J/s)(10 × 365 × 24 × 3600 s) 1  0.1 


t= ln   = 15 ln (3)
= 3.1536 × 10 J 17 (0.1– 1/ 30)  1/ 30 
But since the efficiency of the reactor is only 10%, or t = 16.48 s.
therefore actual energy needed is 10 times of it or 3.1536
× 1018 J. One uranium atom liberates 200 MeV of energy (c) The population of X at this moment,
2 5 . 7 4 | Nuclear Physics and Radioactivity

NX = N0 e– λ X t = (1020)e–(0.1)(16.48)  1 
 dN 
NX = 1.92 ×1019 Sol 11: Activity  –  = lN =   × N
 dt   tmean 
NX λ X
NY = [From Equation (iv)]  dN 
λY ∴ N = –  × tmean = Total number of atoms
 dt 
(0.1)
= (1.92 × 1019) = 5.76 × 1019 Mass of one atom is 10–25 kg = m(say)
(1/ 30)
∴ Total mass of radioactive substance
NZ = N0 – NX – NY = 1020 – 1.92 × 1019 – 5.76 × 1019
= (number of atoms) × (mass of one atom)
or Nz = 2.32 × 1019
 dN 
= –  (tmean)(m)
 dt 
Sol 9: Let N0 be the initial number of nuclei of 238U.
Substituting the values, we get
n
After time t, NU = N0  1  Total mass of radioactive substance = 1 mg
 
2
∴ Answer is 1.
Here n = number of half-lives
t 1.5 × 109 1 Sol 12: A → p, q; B → p, r; C → p, s; D → p, q, r
= = =
t1/2 4.5 × 10 9 3
1/3 Sol 13: (D) It is only due to collision between high
NU = N0  1  energy thermal deuterons which get fully ionized and
 
2 release energy which increases the temperature inside
 1/3  the reactor
and NPb = N0 – NU = N0 1–  1  
 
 2 
 
Sol 14: From conservation of mechanical energy, we
1/3
 1 have
NU
 
2
∴ =   = 3.861 Ui + Ki = Uf + Kf
NPb  1
3
1–   1 (e)(e)
0 + 2(1.5 KT) = . +0
2 4πε0 d
Substituting the values, we get
dN
Sol 10: = |Activity of radioactive substance| T = 1.4 × 109 K
dt
= lN = lN0e–lt
Sol 15: (B, D) If (BE)final −(BE)initial > 0
Taking log both sides
Energy will be released.
dN
ln = ln(lN0) – lt
dt Sol 16: (B) nt0 > 5 ×1014 (as given)
dN
Hence, ln versus t graph is a straight line with
dt Sol 17: (D) f = (1– e−λt ) = 1– e−λt ≈ (1 − λt) = λ t
slope – λ. From the graph we can see that, f = 0.04
1 Hence % decay ≈ 4%
λ= = 0.5 yr–1
2
t
Now applying the equation, A  1 T
Sol 18: (C) = 
N = N0e–lt = N0e–0.5 × 4.16 A0  2 
N0 Where, A0 is the initial activity of the radioactive material
= N0e–2.08 = 0.125 N0 =
8 and A is the activity at t.
t
i.e., nuclei decreases by a factor of 8.
12.5  1  T
So, =  ∴ t = 3T
Hence, the answer is 8. 100  2 

P hysi cs | 25.75

Sol 19: (C) Sol 26: (C) KEmax of β-


(A) → (r, t) ; (B) → (p, s) ; (C) → (p, q, r, t); (D) → (p, q, r, t) Q = 0.8 × 106 eV
KEP + KEβ- + KEν = Q
12
Sol 20: (9) 5 B →12 –
6 C * +e + v KEp is almost zero
12
We take the mass of 6 C as 12 amu When KEβ- = 0

Rest energy of 12
* = 12 × 931.5 MeV + 4.041 MeV Then KEν = Q – KEp ≅ Q
6 C

12
Energy of 5 B = 12 × 931.5 MeV + 0.014 × 931.5 Sol 27: (D) 0 < KE − < Q – KEP – KEν
β
∴ Value of the reaction = 13.041 MeV – 4.041 MeV = 0 < KE <Q
β−
9 MeV
Maximum e- energy = 9 MeV Sol 28: (A, C) Given data
n2
ln2 4.5a0 = a0  … (i)
Sol 21: (A) 5µCi = (2N0 ) Z
T1
ln2 nh 3h
10µCi = (N0 ) =  … (ii)
T2 2π 2π
So n = 3 and z = 2
Dividing we get T1= 4T2
So possible wavelength are
Sol 22: (A) The electric field at r = R 1 1 1 9
= RZ 2  = –  ⇒ λ1
KQ λ1 2
1 3 2 32R
E=
R2 1 1 1 1
= RZ 2  = –  ⇒ λ2
Q Total
= charge within then nucleus Ze λ2 2
1 2 2 3R

KZe 1 1 1 9
So, E = = RZ 2  = –  ⇒ λ3
R 2 λ3 22 2
3  5R

So electric field is independent of a.


Sol 29: (C) 36 Li →24 He +12 H
R
d Q
x2dx Ze = 6.015123 − 4.002603 − 2.014102
Sol 23: (B)
= q ∫ R (R – x) 4π= C2
0
0 = – 0.001582 < 0
3Ze
d= So no α -decay is possible
πR 3
2 4
1 H +2 He →36 Li
Sol 24: (C) If within a sphere ρ is constant E ∝ r Q
= 2.014102 + 4.002603 − 6.015123 = 0.001582 > 0
C2
Sol 25: (8) N = N0 e−λt So, this reaction is possible
70 82
ln | dN/dt | = ln(N0 λ) –λt 30 Zn +34 Se →152
64 Gd

1 Q
From graph, λ = per year = 69.925325+ 81.916709 - 151.919803 = -0.077769 < 0
2 C2
0.693 So this reaction is not possible
t1/2
= = 1.386 year
1/ 2
4.16 yrs = 3t1/2 Sol 30: (A) 210
→24 He +82
206
84 Po Pb
∴p=8
Q = (209.982876 – 4.002603 – 205.97455)C2
= 5.422 MeV
2 5 . 7 6 | Nuclear Physics and Radioactivity

from conservation of momentum Sol 35: (A) Q value of reaction

2K1(4) = 2K 2 (206) = (140 + 94) × 8.5 –236 × 7.5 = 219 Mev


So, total kinetic energy of Xe and Sr
4K1 = 206K2
= 219 –2 –2 = 215Mev
103
∴ K1 = K So, by conservation of momentum, energy, mass and
2 2
charge, only option (A) is correct
K1 + K2 = 5.422
2 Sol 36: (C) (BE)15 = 7 mp + 8 mn − m15
K1 + K =5.422
103 1 7 N 7 N

(BE)15 = 8 mp + 7 mn − m15
105 O N
⇒ K =
5.422 8 8
103 1  
⇒ ∆(BE) = (mn +mp )+  m15 − m15 
∴ K1 = 5.319MeV = 5319 KeV  8 O 7 N

= 0.00084 + 0.002956 = 0.003796 u
Sol 31: (C) P → (ii); Q → (i); R → (iv); S → (iii)
3 14 × 1.44 MeV fm
15 ⇒ × R
=
8 O →15
7 N +10 β (Beta decay) 5 0.003796 × 931.5 MeV
238 234
Th + 24 He (Alpha decay) ⇒R =
3.42 fm
92 U →90
185
83 Bi →184 1
82 Pb +1 H (Proton emission)
Sol 37: (C) Activity A ∝ N (Number of atoms)
239
94 Ph →140
57
99
La +37 Rb (fission) n
 1
N = N0  
λCu  Z − 1
2 2
Sol 32: (B) =  Mo
λMo  Z − 1  where n → Number of half lives
 Cu 
N0
nh 3h If N =
Sol 33: (B) mvr
= = 64
2π 2π
n
de-Broglie Wavelength  1 N
N0   = 0
h 2πr 2π a0 (3)
2 2 64
λ= = = = 2πa0 n 6
mv 3 3 zLi  1 1  1
 = =  
2 64  2 
1 1
Sol 34: (B) λP = ;λ = n=6
τ Q 2τ
RP (A0 λP )e
−λP t time = n × T1/2
=
RQ −λQ t time = 6 × 18 days = 108 days
A0 λQ e

RP 2
At t = 2τ ; =
RQ e
2017-18 100 &
op kers
Class 12 T
By E ran culty
-JE Fa r
IIT enior emie .
S fP r es
o titut
Ins

PHYSICS
FOR JEE MAIN & ADVANCED
SECOND
EDITION

Exhaustive Theory
(Now Revised)

Formula Sheet
9000+ Problems
based on latest JEE pattern

2500 + 1000 (New) Problems


of previous 35 years of
AIEEE (JEE Main) and IIT-JEE (JEE Adv)

5000+Illustrations and Solved Examples


Detailed Solutions
of all problems available

Topic Covered Plancess Concepts


Tips & Tricks, Facts, Notes, Misconceptions,
Semi-conductor and Key Take Aways, Problem Solving Tactics
Communication System
PlancEssential
Questions recommended for revision
26. S EMI CON D U C TOR A N D
CO M M U NI C ATI ON
S YS TE M

SEMICONDUCTOR

1. INTRODUCTION
Certain substances like germanium, silicon etc. are neither good conductors like copper nor insulators like glass. In
other words, the resistivity of these materials lies in between conductors and insulators. Such substances are classified
as semiconductors. These substances have crystalline structure and are formed by covalent bonds. An important
property of a semiconductor is that by adding a controlled amount of suitable impurity to it, its conductivity can
be changed appreciably. This useful property is responsible for the widespread use of semiconductors in electronic
devices. In this chapter, we shall discuss the electrical properties of semiconductors.

2. CLASSIFICATION OF SOLIDS
On the basis of electrical conductivity ( σ ) or resistivity ( ρ= 1 / σ ) , the solids can be classified into the following
three classes:
(a) Metal conductors. These are those solids which possess high conductivity or low resistivity. This is due to the fact
that metals have a large number of free electrons. The conductivity of metal conductors lies between 102 and 108
Sm-1 while their resistivity is in between 10-2 and 10-8 Ω m. Examples of metal conductors are: Al, Cu, Ag, Au, etc.
(b) Insulators. These are those solids which possess very low conductivity or very high resistivity. This is due to the
fact that insulators have practically no free electrons. The conductivity of insulators lie between 10-11 and 10-19
Sm-1 while their resistivity is between 1011 and 1019 Ω m. Examples of insulators are: glass, rubber, plastic etc.
(c) Semiconductors. These are those solids which possess conductivity and resistivity in between metallic
conductors and insulators. This is due to the fact that semiconductors have very few free electrons at room
temperature and can be regarded as insulators for all practical purposes. The conductivity of semiconductors
lies between 10-5 and 100 Sm-1 while their resistivity in between 105 and 0.5 Ω m. Examples of semiconductors
are: germanium, silicon, carbon, etc.

3. ENERGY BANDS IN SOLIDS


(a) Overlapping energy levels are termed as energy bands
(b) The energy band formed by the overlapping of valence electrons is known as valence energy band.
(c) The energy band formed by the overlapping of conduction electrons is known as conduction energy band.
2 6 . 2 | Semiconductor and Communication System

(d) Electrical conduction in solid can take place only when electron remains present in its conduction energy
band.
(e) The minimum energy required for exciting an electron from valence
energy band to conduction energy band is known as forbidden energy
( )
gap ∆Eg ; ∆=Eg CEBmin − VEBmax Conduction band

Band energy
We have seen that energy levels of an isolated atom are converted into
corresponding energy bands when the atom is in solid form. There is no Forbidden
appreciable modification in the energy levels of electrons in the inner energy
gap
orbits but there is a considerable modification of energy levels of valence
electrons. It is because valence electrons are shared by more than one atom
Valence band
in the crystal. Therefore, valence electrons can be considered to be in either
of the two energy bands viz lower valence band or upper or conduction
band as shown in Fig. 26.1. No electron can have energy in the forbidden
Figure 26.1
energy gap between two bands. Normally, the electrons reside in the
valence band where they are held rather tightly to the individual atoms.
(a) Valence band. The range of energies (i.e. band) possessed by valence electrons is known as valence band.
The electrons in the outermost orbit of an atom are known as valence electrons. In a normal atom, valence
band has the electrons of highest energy. This band may be completely or partially filled. For instance, in case
of inert gases, the valence band is full whereas for other materials, it is only partially filled. The partially filled
band can accommodate more electrons.
(b) Conduction band. In certain materials (e.g. metals), the valence electrons are loosely attached to the nucleus.
Even at ordinary temperature, some of the valence electrons may get detached to become free electrons.
In fact, it is these free electrons which are responsible for the conduction of current in a conductor. For this
reason, they are called conduction electrons.
The range of energies (i.e. band) possessed by conduction band electrons is known as conduction band.
All electrons in the conduction band are free electrons. If a substance has empty conduction band, it means
current conduction is not possible in that substance. Generally, insulators have empty conduction band. On
the other hand, it is partially filled for conductors.
(c) Forbidden energy gap. The separation between conduction band and valence band on the energy level
diagram is known as forbidden energy gap (Eg).
No electron of a solid can stay in a forbidden energy gap as there is no allowed energy state in this region. The
width of the forbidden energy gap is a measure of the bondage of valence electrons to the atom. The greater
the energy gap, more tightly the valence electrons are bound to the nucleus. In order to push an electron
from valence band to the conduction band (i.e.to make the valence electron free), external energy equal to
the forbidden energy gap must be supplied.

4. SEMICONDUCTORS
These are solids in which the forbidden energy gap
between the valence band the conduction band is small, of Conduction
the order of 1eV. At 0 kelvin, the valence band is completely band
filled and the conduction band is completely empty. At -le V } Forbidden } }
0K, it behaves like an insulator (electron cannot absorb in band
infinitesimal energy because there is a forbidden gap just Valence
at the top of the valence band). At a finite temperature, band
(room temperature), some electrons gain energy due to
OK TK
thermal motion and jump from the top of the valence
band to the conduction band. These electrons contribute Figure 26.2
to the conduction of electricity in a semiconductor.
P hysi cs | 26.3

The forbidden gap in a semiconductor is small ~ 1eV. At finite temperature, some valance electrons go to conduction
band. Then the formlessly is in middle of the gap
The energy gap is some semiconductors is as follows:
∆Eg ( Silicon) =
1.12eV ; ∆Eg ( germanium) =
0.7 eV
∆Eg ( Indium antimonide ) =
0.17 eV ; ∆Eg ( Gallium arsenide ) =
1.43eV ; ∆Eg ( Tellurium )=
0.33eV
The energy gap decreases slightly with increases in temperature.

Table 26.1: Comparison between conductors, insulators and semiconductors

Conductors Insulators Semiconductors


1 Valence band is partially filled or Completely empty conduction band At 0K, the conductive band is empty
valence band conduction band are separated from completely filled while valence band is full separated
overlapped. valence by small energy gap
2. There is no forbidden energy gap The forbidden gap is large Separated by small energy gap Eg,
Si=1.12 eV
Eg (diamond)~ 6 eV

Eg (diamond) ~ 6 eV
3. At room temperature, all electrons At room temperature, electrons do At room temperature, many
remains in the partially filled valence not get sufficient thermal energy to electrons have sufficient energy to
band or over lapped band cross over and the forbidden energy go to the conduction band.
band remains empty
4. Conducts electric current. Very small Does not conduct electric current May conduct electric current but
resistivity (negligible conduction) very large conduction is small. Medium
ρ ( ohm meter ) resistively (ohm meter) resistivity and medium conductivity
ρ ( glass )  1011 − 1012 Ωm ρ ( Si )= 2100Ωm ρ ( Ge ) = 0.47Ωm
ρ ( Cu) =1.7 × 10 −8 Ωm
ρ ( diamond)  10 Ωm 14
σ ( Ge )  2.13 σ ( Si )  4.7 × 10 −4
ρ ( Ag) =1.6 × 10−2 Ωm
very low conductivity (ohm/m).
The conductivity is high
−10 −14
σ  10 to 10 Ωm
σ ≈ 107 to 10−9 ohm / m
(or Siemen/m) very low conductivity
σ  10−10 to 10−15 ohm / m (or
Siemen/m)
5. Only electrons are the current carrier No current carrier (the electric Both electrons and holes contribute
Number of free electrons (in Cu) conduction is almost zero for all to current conduction. Number of
 1028 per m3 practical purposes, see σ mentioned free electrons (at room temperature)
before) is in Ge  1019 per m3 in Si  1016
per m3
6. Conductivity decreases with Conductivity negligibly small Conductivity increases with
temperature. however increases slightly at very temperature (the resistivity /
high temperatures. resistance decreases with
temperature). The temperature
Conductivity coefficient of resistance of a

 semiconductor is negative
 (Resisivity)

Temperature

Temperature

Temperature
2 6 . 4 | Semiconductor and Communication System

4.1 Classification of Semiconductors


The semiconductors are of two types
A semiconductor in a pure form is called intrinsic semiconductor. The impurity must be less than 0.01 ppm
(parts per million) i.e., less than 1 in 108 parts of semiconductor. At low temperature, the electrons are present in
valence bonds of the semiconductor. As the temperature is increased a few electrons are raised to conduction
band.

In intrinsic semiconductors: ne(0)= nn(0)= ni= AT3/2 e−∆Eg /2KT

Where ne(0) is electron density in conduction band, ne(0) is hole density in valance
band and ni is the density of intrinsic charge carriers.
At absolute zero temperature, all the valence electrons are tightly bound and
no free electron is available for electrical condition. In the band picture, at
absolute zero temperature, the conduction band is completely empty while,
the valence band is fully filled. The semiconductor behaves as a perfect
insulator at absolute zero temperature. At room temperature (~300K), some
of the electrons may gain sufficient thermal energy and moved away from the
influence of the nucleus, i.e. the covalent bond may be broken. The electron,
so obtained is free to move in the crystal and conduct electricity (see Fig 26.4).
The vacancy created in the covalent bond is called a hole.
Figure 26.3

Conduction
Free electrons band
Conduction
band Empty
Gap Eg
Eg (gap)
Valence band
with holes Filled

Valence band
Holes
Figure 26.4

When a covalent band is broken, the electron hole pair is created. Thus, in an intrinsic semiconductor, Number of
holes=number of free electrons nh=ne

4.2 Intrinsic Semiconductors


In intrinsic semiconductor, the number of free electrons and holes are equal. Both electrons and holes contribute
in current conduction. For the purpose of flow of current, a hole,
behaves like a positively charged particle having some effective mass.
Electron flow
Therefore, while the electron moves from the negative electrode of the
hole flow
battery to the positive electrode through the semiconductor, the hole
moves on opposite side.
The hole exists only inside a semiconductor. There are no holes in a
metal. There, electric conduction through holes takes place inside the
semiconductor only. Outside, in the metal wires, the electric current
+ -
flow is due to electrons only. (In cell current flow is due to the motion
of positive and negative ions).
Figure 26.5
P hysi cs | 26.5

In an intrinsic semiconductor the current flow occurs due to the motion of both, the electrons and the holes. Let
e=magnitude of charge on the electrons, nh= number density of holes, µe =mobility of electrons an µh =mobility of
holes, then the conductivity of intrinsic semiconductor is= σ e (neµe + nh + µh ) Consider a block of semiconductor
of length area of cross section A and having density of electron and holes as ne as and nh respectively when a
potential difference say V is applied across it, current I flows through it as shown in Fig. 26.6 The current I is made
of electron current Ie and hole current Ih.
Thus, I = Ie + Ih  … (i)
v
If ve is drift velocity of electrons, Then Ie =ene Ave  …(ii)
Similarly, the hole current is given by Ih= ene Avh  … (iii) Area ‘A’

Using equation (ii) and (iii), the equation (i) becomes


-
I=eA (neve+nhvh)  … (iv) -

-
If R is the resistance offered by the semiconductor to the flow of current, then
l
V V
I= or = eA (neve+nhvh)  … (v) Figure 26.6
R R
V
The electric field set up across the semiconductor is given
= by E =or V E
l
El E
Therefore, equation (v) becomes = eA (neve+nhvh) or E = e (neve+nhvh)
R A
R
l
A E
But R =ρ = resistivity of the material of the semiconductor. Therefore = e (neve+nhvh) … (vi)
l ρ
Mobility of electrons or holes is defined as the drift velocity acquired per unit electric field.
ve vh
Therefore mobility of electrons and holes is given =
by µe and
= µh
E E
From equation (iv), we have

1  ve v 
= e ne . + nh . h  or σ =e (ne µ e+nh µ h)  … (vii)
ρ  E E 

1
Where σ =e is called conductivity of the material of the semiconductor and µe , µh are electron and hole
ρ
mobilities respectively.

PLANCESS CONCEPTS

•• In pure semiconductors, at any temperature t, the carrier concentration ne=nh=n and the conductivity
is determined by the value of Eg (width of the forbidden band)(see relations given above).
•• In metal, however, the value of n is almost the same at different temperatures. The resistance arises
due to interaction of free (conduction) electrons with the lattice vibrations.
•• At absolute zero temperature, n=0, σ =0 i.e., the pure semiconductor behaves like a perfect insulator.
However, as temperature increases both n and σ increases. In germanium at T ≈ 300K, ne = nh
= 2.5 × 1019 per m3. The higher is the temperature, higher is the conductivity and lower is the resistivity.
•• The temperature coefficient of the resistance of a semiconductor is negative.
•• Pure semiconductors are of little use (may be used as heat or light sensitive resistance).
Vaibhav Krishnan (JEE 2009, AIR 22)
2 6 . 6 | Semiconductor and Communication System

Illustration 1: Which one is more sensitive to heat, germanium or silicon?  (JEE MAIN)

Sol: The band gap between conduction band and valance band in germanium (0.68 eV) is less than silicon
(1.1 eV). Thus the electron in the valance band in germanium require less thermal energy (Order of KT) to transit
from valance band to conduction band compared to silicon.
Germanium is more sensitive to heat. Electrons from the valence band of germanium require less energy to move
from valence band to conduction band.

4.3 Extrinsic Semiconductors


A semiconductor in which impurities have been added in
a controlled manner is called extrinsic semiconductor. The
Si Si Si
process of deliberately adding impurities to a semiconductor
is called doping. The impurity atoms are either from V group Fifth electron
such as arsenic (as), antimony (Sb), phosphorus (P) etc. or
from III group (such as Aluminum (AI), gallium (Ga), indium Si Si Si
(In) etc. There are two types of extrinsic semiconductors, (i) Impurity atom
N-type (extrinsic) semiconductor and (ii) P-type (extrinsic)
semiconductor. The conductivity of extrinsic semiconductor
Si Si Si
is controlled by the amount of doping, I part of a donor
impurity per 109 parts of germanium increases its conductivity
by a factor of nearly 103. The compounds of trivalent and
pentavalent elements also behave like semiconductors, Figure 26.7
(indium antimonite), in P, GaP.

4.3.1 N-Type Semiconductor


N-Type (n-type) semiconductor is obtained by adding a small Free electrons
amount of pentavalent (V group) impurity to a sample of intrinsic
semiconductor. The pentavalent impurities are P (phosphorus
+ + + n-type
Z=15), As (Z=3), Sb (z=51), Bi(Z=83).
semi
In the energy band picture we say that impurity atoms produce conductor
+ + +
donor energy levels just below the conduction band. The electrons
from these levels jump to the conduction band easily by gaining Immobile positive
thermal energies (at room temperature). They may also break some ions (impurity)
covalent bonds producing electron hole pair, but their number is
small. So in this type of extrinsic semiconductor, there are a large Figure 26.8
number of free electrons (donated by impurity atoms) and a
negligible number of holes from broken covalent bond.
Conduction
The impurity atom on donating electrons becomes positive ions. band
However the overall charge on the semiconductor is zero. The
negative charge of the immobile positive charge of the immobile Majority
positive ions. The majority charge carriers are electrons (negative electrons
charge). Therefore, this type of extrinsic semiconductor is called
n-type. The Fermi level does not lie in the middle of band gap,
but it shifts towards the conduction band. The few holes formed Donor
by covalent bond breaking are called minority charge carries. The 0.01 (Ge) level
conductivity of the n-type semiconductors is controlled by the
0.05 (Si)
amount of impurity atoms added in it.
(eV)
Valence band
Figure 26.9
P hysi cs | 26.7

4.3.2 P-Type Semiconductor


P-Type (type) semiconductor is obtained by adding a small
amount of trivalent (III group) impurity to intrinsic semiconductor.
The impurities may be Boron (Z=5), AI(Z=13), Ga (Z=31), In (Z=49), Ge Ge Ge
Acceptor
TI (Z=81). For each acceptor iron there exist a hole in this type
atom
of semiconductor, there are a large number of holes present. The
majority charge carriers are holes. Therefore it is called a P-type hole
Ge Ge Ge
semiconductor.
In the picture, we say that accepter energy levels lie just above
the valence band. These levels accept electrons from the valence Ge Ge Ge
band and create holes. The breaking of covalent bonds may create
electron-hole pairs but their number is very little. The majority Hole
carriers are holes. The minority carriers are electrons. migrates
Ge Ge Ge
The conduction takes place mainly through the motion of holes Acceptor
nh>>ne ; σ ≈ eµh nh atom
The overall charge on p-type semiconductor is zero. It is Ge Ge Ge
represented as shown in Fig. 26.11. The positive charge of free
holes is balanced by the negative charge of immobile impurity
ions. Figure 26.10

Conduction
band Free holes

Minority - - -
electrons
- - -
Acceptor level
p-type semi conductor
Immobile negative
Majority ions (impurity)
Valence band Holes
Figure 26.11

PLANCESS CONCEPTS

When temperature is increased, covalent bonds break. This increases minority charge carriers. At very
high temperature, it may happen when number of electron-hole pair obtained from bond breaking, far
exceeds the charge carriers from impurities. Then the semiconductor behaves like intrinsic semiconductor.
The critical temperature at which this happens is 850C germanium and 2000C for silicon.
Chinmay S Purandare (JEE 2012, AIR 698)

Illustration 2: Calculate the conductivity and the resistivity of silicon crystal at 300K. It is given that µe =1350 cm2 / V s,
10
µh = 480 cm2/ V s, and at 300K, the election- hole pair concentration is 1.072 × 10 per cm3.  (JEE MAIN)
Sol: The conductivity of silicon is given as σ = e × (neµe + nµh ) where ne and nh are electron and hole densities
1
respectively. Here ne = nh = ni is the hole-electron pair concentration. The resistivity of the silicon is ρ = .
σ
The conductivity for intrinsic semiconductor is σ = e × (neµe + nµh )
2 6 . 8 | Semiconductor and Communication System

Given, n= 1.072 × 1010 per cm3 = 1.072 × 1016 per cm3


Also ne=nh for intrinsic semiconductor, further,
µe =1350cm2 / V s, = 0.1350 m2/ volt ; µh =0.048 m2/ V s,
σ 1.6 × 10 −19 × 1.072 × 1016 × ( 0.135 + 0.048
Therefore, = = ) 3.14 × 10−4 Ω m
= 3.14 × 10−4 Siemen per meter
1
The resistivity r= = 10−4 /3.14= 3185 Ω m.
σ

Illustration 3: The concentration of acceptor atoms in a p-type germanium crystal is 4 × 1015 per cm3. Find the
conductivity of the crystal at 300K. The µh for germanium at 300K is 1900 cm2 / V s. It is assumed that all the
acceptor atoms are ionized at this temperature.  (JEE MAIN)
Sol: In p-type germanium hole density is greater than electron density. The conductivity of the p-type germanium
is given by=σ nh eµh
σ nh eµh
For extrinsic semiconductor (p-type)=
Given σ= nh = 4 × 1015 per cm3 = 4 × 1021 per m3
µh =1900cm2 /volt –sec
Thus σ= 4 × 1021 × 1.6 × 10−19 × 0.190= 1.216 × 102 ohm / m = 1.21.6 siemen / m

Table 26.2: Comparison of intrinsic and extrinsic semiconductors

S. No. Intrinsic semiconductor Extrinsic semiconductor


1. It is a pure semiconductor with no impurity for this It is an impure semiconductor i.e. a controlled
reason, it is called intrinsic (i.e. pure) semiconductor. pentavalent or trivalent impurity added to a pure
(intrinsic) semiconductor.
2. The number of free electrons in the conduction band In an n-type semiconductor, free electrons exceed the
is equal to the number of holes in the valence band. hole. A p-type semiconductor, it is the reverse.
3. Its electrical conductivity is low. Its electrical conductivity is high.
4. Its electrical conductivity depends on temperature Its electrical conductivity depends on temperature and
alone. the amount of doping.
5. It is of no practical use. It is used in electronic devices.

Table 26.3: Comparison of n-type and p-type semiconductors

S. No. n-type semiconductor p-type semiconductors


1 It is produced by adding controlled amount of It is produced by adding controlled amount of trivalent
pentavalent impurity to pure (intrinsic) semiconductor. impurity to pure (intrinsic) semiconductor.
2 The number of free electron far exceeds the number The number of free holes far exceeds the number of
of holes. electrons.
3 The current conduction is predominant by free The current conduction is predominant by holes.
electrons.
4 The donor energy level is just below the bottom of the The accepter energy level is just above the top of
conduction band. valence band.
P hysi cs | 26.9

Illustration 4: An intrinsic germanium has a resistivity of 0.47 Ω m at room temperature. Find the intrinsic carrier
concentration if the mobility of electrons and holes are 0.39 m2/ V s, and 0.19 m2/volt-sec respectively. (JEE MAIN)
1
Sol: The resistivity of the intrinsic semiconductor is ρ = . Here σ is conductivity of the germanium and is given by
σ
σ ni ( µe + µh )
= where ni is the concentration of electron-hole pair
Let ni be the intrinsic carrier concentration.
1 1 σ
Electrical conductivity, σ= = σ ni ( µe + µh ) Or ni =
Now =
ρ 0.47 e ( µe + µh )

Here σ =1/0.47 S/m, µe = 0.39 m2/volt-sec, µh =0.19 m2/volt-sec, e= 1.6 × 10−19 C

1
ni
= = 2.3 × 10−19 / m3
0.47 × 1.6 × 10 −19
( 0.39 + 0.19 )

Illustration 5: The resistivity of n-type germanium is 0.01 Ω m at room temperature. Find the donor concentration
if the mobility of electrons 0.39m2 /volt-sec. (JEE MAIN)
1
Sol: For n-type germanium the donor concentration is given by nd =
e ρµe
1
Let nd be the donor concentration. nd =
e ρµe
1 1
Conductivity, σ= = = 100S / m Now σ = e Nd µ ……….n-type semiconductor
ρ 0.01 e

Or =
nd σ 100
= = 1.6 × 10−21 / m3
e µe 1.6 × 10−19 × 0.39

Illustration 6: Mobilities of electrons and holes in a sample of intrinsic germanium at room temperature are
0.36m2 / V s and 0.17 m2 / V s respectively. If the electron and hole densities are each equal to 2.5 × 1019 /m3 ,
calculate the conductivity.  (JEE ADVANCED)
Sol: The conductivity of intrinsic semiconductor is given by= σ e (neµe + nhµh ) where ne and nh are the electrons
and hole densities. As here both are equal then n=i n=
e nh and conductivity is given by=σ ni e ( µe + µh ) .
The conductivity of an intrinsic semiconductor is given by;=σ ni e ( µe + µh )
Here ni = 2.5 × 10−19 / m3 ;e = 1.6 × 10−19 C; µe = 0.36 m2 / volt-sec; µh =0.17m2 / volt-sec
σ 2.5 × 1019 × 1.6 × 10−19 ( 0.36 + 0.17=
= ) 2.12 S/ m

Illustration 7: A semiconductor is known to have an electron concentration of 8 × 1013 per cm3 and a hole
concentration of 5 × 1012 per cm3.
(i) Is the semiconductor n-type or p-type?
(ii) What is the resistivity of the sample if the electron mobility is 23000 cm2/ V s and hole mobility is 100 cm2/ V s?
 (JEE ADVANCED)
Sol: For the semiconductor sample, if the hole density is less than electron density the semiconductor is N type in
1
nature. The resistivity of the sample is given as ρ = where σ is the conductivity of the sample.
σ

( ) (
(i) Since electron density ne = 8 × 1013 per cm3 is greater than the hole density nh = 5 × 1012 per cm3 , the )
semiconductor is n-type.
σ e (neµe + nhµh )
(ii) The conductivity of the sample is given by;=
2 6 . 1 0 | Semiconductor and Communication System

Here ne= 8 × 1013 per cm3 ; n = 5 × 1012 per cm3


h

= cm2 / Vs; µh 100cm2 / Vs


µe 23000=

∴ ( ) 1.6 × 184.05 × 10−3 S cm−1


= 1.6 × 10 −19 × 8 × 1013 × 23000 + 5 × 1012 × 100 S cm−1 =
σ
1 1
∴ Resistivity of the sample is given by; ρ= = = 3.396 Ωcm
σ 1.6 × 184.05 × 10−3

Illustration 8: Determine the number density of donor atoms which have to be added to an intrinsic germanium
semiconductor to produce an n-type semiconductor of conductivity 5 Ω -1cm-1, given that mobility of conduction
electrons in n-type Ge is 3900 cm2/ Vs. Neglect the contribution of holes to conductivity.  (JEE ADVANCED)

Sol: In n type semiconductor the number density of electrons is much greater than number density of holes. Thus
we can neglect the number density of holes. Thus to produce the n type semiconductor, the donor of number
= ene µe .
density to added is found by σ
(
σ e neµe + nhµh
The conductivity of a semiconductor is given by;= )
= eneµ=
Neglect the contribution of holes to conductivity, we have, σ e eNdµe
 N=
d ne 

∴Number density of donor atoms or electron density is Nd = σ


e µe
−1 −1 2 −19
Here σ = 5Ω cm µ
; e= 3900 cm / Vs; e
= 1.6 × 10 C
5
Nd
∴= = 8.01 × 1015 cm−3
3900 × 1.6 × 10−19

Illustration 9: Suppose a pure Si crystal has 5 × 1028 atoms m-3. It is doped by 1 ppm concentration of penta-valent
impurity of Arsenic (As). Calculate the number of electrons and holes. Given that ni = 1.5 × 1016 m-3.
 (JEE ADVANCED)

Sol: The arsenic contains the 1 free electron in its conduction band and acts as donor impurity. The number of
electron given by As to the semiconductor is given by
Number of Si atom in parent crystal × 10−6
N= .
| y ppm|
ni2
The number of holes in the semiconductor sample is given by nh = here ni is the concentration of holes and
electron pair.
ne

1
1 ppm =1 part per million =
106
5 × 1028
∴ Number of penta-valent atoms doped in Si crystal= = 5 × 1022 m−3
6
10
As one penta-valent impurity atom donates 1 free election to the crystal,
22 −3
∴ Number of free elections in the crystal is ne = 5 × 10 m

(1.5 × 10 )=
2
16
ni2
∴ Number of holes, n=
h = 4.5 × 109 m−3
ne 5 × 1022
Illustration 10: The mean free path of conduction electrons in copper is about 4 × 10−8 m. Find the electric field
which can give, on an average, 2 eV energy to a conduction electron in a block of copper.  (JEE ADVANCED)

Sol: The band gap between conduction and valance band is given as 2 eV. The work done to lift electron from
P hysi cs | 26.11

conduction band to the valance band is given by W= F × d where F=qE, the electric force applied on the electron.
Mean free path, d= 4 × 10−8 m; energy of electron =2eV
If E is the required electric field, then force on the electron is F=eE
The work done by the electric field on electron before it collides with copper atom is Fd. This work done is equal to
the energy to the energy transferred to electron.
2V 2V
∴ Fd=2 eV or eE ×d =2eV∴ E= = = 5 × 107 V / m
d 4 × 10 −8

4.4 Effect of Temperature on Semiconductors


The electrical conductivity of a semiconductor changes appreciably with temperature variations. This is a very
important point to keep in mind.
(a) At absolute zero. At absolute zero temperature, all the electrons are tightly held by the semiconductor atoms.
The inner orbit electrons are bound whereas the valence electrons are engaged in covalent bonding. At this
temperature, the covalent bonds are very strong and there are no free electrons. Therefore, the semiconductor
crystal behaves as a perfect insulator.
In terms of energy band description, the valence band is filled and there is a large energy gap between
valence band and conduction band. Therefore, no valence electron can reach the conduction band to become
free electron. It is due to the non-availability of free electrons that a semiconductor behaves as an insulator.
(b) Above absolute zero. When the temperature is raised, some of the covalent bonds in the semiconductor
break due to the thermal energy supplied. The breaking of bonds sets those electrons free which are engaged
in the formation of these bonds. The result is that a few free electrons exist in the semiconductor. These free
electrons can constitute a tiny electric current if potential difference is applied across the semiconductor
crystal. This shows that the resistance of a semiconductor decreases with the rise in temperature i.e. it has
negative temperature coefficient of resistance. It may be added that at room temperature, current through a
pure semiconductor, is too small to be of any practical value.
As the temperature of a semiconductor increases, some of the valence electrons acquire sufficient energy to enter
into the conduction band and thus become free electrons. Under the influence of electric field, these free electrons
will constitute electric current. It may be noted that each time a valence electron enters into the conduction band, a
hole is created in the valence band. As we shall see in the next article, holes also contribute to current. In fact, hole
current is the most significant concept in semiconductors.
Barrier
Effect of temperature on conductivity of semiconductors P-type Electric field N-type
The electrical conductivity σ of a semiconductor is given by;
σ e (neµe + nhµh )
- -
- -

O O O O
= - - - -
-
-

As the temperature increases, the values of µe (electron mobility)


- -
O O O O
and µh (hole mobility) decrease due to increase in their collision - - - - - -
-
-
- - -

frequency. But with the increase in temperature, there is a large


increase in charge carrier concentrations ( µe and µh ) due to the O
- -
O O O
increase in breaking of covalent bonds. In fact, the increase in - - - - - -
-
-

carrier concentration is so large that the decrease in the values


of µe and µh has no significant effect. Hence, the conductivity of
Hole Donor
a semiconductor increases with the increase in temperature and Depletion
Acceptor Free
vice-versa. region ion
ion electron

Figure 26.12
5. P-N JUNCTION
When a piece of p-type material and piece of n-type material are joined in such a manner that crystal structure
remain continuous at the boundary, then a pn junction is formed. It is also called a pn junction (PN junction) diode.
2 6 . 1 2 | Semiconductor and Communication System

AP-N junction cannot be made by simply pushing the two pieces together as it would not lead to a single crystal
structure. There are special fabrication techniques to form a PN junction.
Immediately after a PN junction is formed, the following processes are initiated:
(a) The negative ions on P-sides and positive ions on N-sides are immobile. The majority holes from P region
diffuse into N region, and the majority electrons from N region diffuse into P region,
(b) Due to the above, the electrons and holes at the junction region recombine and disappear (i.e. covalent bonds
are completed).
(c) As a result, a layer of negative ions on P-side and a layer of positive ions on N-side is formed at the junction.
In this region, due to recombination of electrons and holes, depletion of free charge carriers occurs. So this
region is called depletion region. The charge density on the two sides of the junction (due to ion layers) is
shown in Fig. 26.13 (A).
(d) The uncompensated ion layers in the depletion region generate an electric field in this region. The electric
field points from N side to P side. This electric field prevents further diffusion of holes from P-sides. It also
prevents further diffusion of electrons from the N side to P side. The electric field is called barrier electric field.
(e) The barrier electrical field gives rise to a difference of potential from one side to the other side. This is called
barrier potential (or potential barrier). For silicon PN junction the barrier potential is about 0.7 V while for
germanium PN junction, it is about 0.2V.
P N-type
P-type
P-type V N-type P-type V N-type
+
(A) (B) (C)
Depletion
- region Electron static potential Potential barrier for
energy barrier for holes electrons
Charge density (potential barrier for holes)

Figure 26.13

(f) For holes the potential on the N-sides is higher. Holes cannot cross the depletion region because of this
barrier potential. Figure 26.13 (B). For electrons the potential barrier is shown in Figure 26.13 (C).
(g) On the average the potential barrier height in PN junction is ~0.5V and the width of the depletion region
V 0.5
 1µ m or 10−6 m. The barrier electric field is thus E= =
~ = 5 × 105 Volt / m
d 10−6

5.1 P-N Junction with Forward Bias


(a) When the position terminal of a battery is connected to the P-side E
and the negative terminal to the N-side of a PN-junction, then it is P-side EB N-side
said to be forward biased (See Fig 26.14). O
- - -
- - - - -

-
-

(b) The holes are repelled from the positive terminal and completed to
O - -
+ - -
-

move towards the junction. The electrons are also repelled from the
O
- - - -
-

O
negative terminal and move towards the junction. This reduce the - -
-

depletion region for a forward biased PN-junction O


- -
- - - -
-

(c) The potential barrier is reduced. More charge carriers diffuse across
the junction.
(d) In the P-type material, near the positive terminal, an electron breaks Depletion region
the covalent bond and goes to the battery. As a result a hole is + -
created in P-sides. At the same time an electron enters the N sides Figure 26.14
from the negative terminal. The current in the P-region arises due
to flow of the holes while the current in the N-region is due to electrons.
P hysi cs | 26.13

(e) The electric field at the barrier, due to the Battery is from P side to N side (forward bias). This is an opposition
to the barrier electric field.
(f) If battery potential is increased, the potential barrier is further reduced. More majority carriers diffuse across
the junction and the current increases.

5.2 P-N Junction with Reverse Bias


(a) When the positive terminal of a battery is connected to the N-side and negative terminal is connected to the
P-side of the PN junction, then it is said to be reverse biased
(b) The holes in the P-region are attracted towards the negative terminal and the electrons in the N-region are
attracted towards the positive terminal. Thus the majority carriers E
move away from the junction. The depletion region increases for a P-side EB N-side
reverse biased PN-junction. O
- - -

- - - - -
-

-
(c) The barrier potential increases, this makes it more difficult for the O - -
+ - -

-
majority carriers to diffuse across the junction. -
O
-
- -

-
O

(d) A very little current called reverse saturation current flows due to - -

-
- -
minority carrier flow. It is of the order nano amperes (10-9 A) for
O
- - - -

-
silicon and micro amperes (10-6 A) for germanium PN-diodes.
(e) In reverse bias situation, the junction behaves like a high resistivity
material sandwiched in between two regions. Depletion region
+ -
(f) The effective capacitance of PN junction in the reverse bias condition
is of the order of few pico farads. Figure 26.15

v
6. SEMICONDUCTOR DIODE + - l
+ -
6.1 Forward and Reverse Bias Characteristics + P N -
+
Forward bias characteristics: The circuit diagram for studying the V-I - R
characteristics of a PN junction diode in forward bias is shown in the
Fig. 26.16.
In forward bias the depletion region decreases, the barrier potential
decrease, and the current flows due to diffusion of charge carriers Figure 26.16
across the junction. Majority holes from P side cross over to N side,
and majority electrons from N sides cross over to P sides. The current l
voltage characteristic is shown in Fig. 26.17. (mA)

The diode current is negligibly small for first few tenths of a volt. The 0.7 V 0.2 V
reason is that the diode does not conduct till the external voltage V, for Si
overcomes the barrier potential. The voltage at which the current starts for Ge
to increase rapidly is called cut-in or knee voltage (V0) of the diode. For (Volt)
O V
a silicon diode V0~0.7 volt for germanium V0~0.2 volt junction diode in
reverse bias is shown in Fig. 26.18. Figure 26.17
v
- + l
- +
P N+
-
- A
+
R

P N
Figure 26.18
2 6 . 1 4 | Semiconductor and Communication System

Reverse Bias characteristics: In reverse bias state, the depletion region increases and
potential barrier also increases. The majority holes in P region and majority electrons P N
in N region, now do not cross the junction. This does not give rise to any current. Easy direction for
conventional current
In reverse bias a very small current flows. This arises due to the flow of minority charge
carriers across the junction. The reverse current is only few µ A for germanium diodes
and only a few nA for silicon diodes. It remains small and almost constant for all Figure 26.19
reverse bias voltages less than the break down voltage Vz. At breakdown, the current
increases rapidly for small increase in voltage. The full characteristics, forward and v2 v
reverse bias are shown in the Fig. 26.19. The PN junction diode thus is a unidirectional O
device. Large current (mA) flows in one direction, but negligible current flows in the
reverse direction. l
Reverse (A)
The symbol used for PN junction diode is shown in Fig. 26.20.
breakdown
(
µ I Iο eeV /kT − 1
The equation for diode current is= )
Where Iο is called saturation current, V is positive for forward and negative reverse bias, Figure 26.20
k is Boltzmann constant, T is temperature and e= 1.6 × 10 −19 C .

6.2 Reverse Breakdown


If the reverse bias voltage is made too high, the current through the PN junction increases rapidly at Vz (see Fig
26.21). The voltage at which this happens is called breakdown voltage or zener voltage.
There two mechanism which causes this breakdown. One is called zener breakdown and the other is called
avalanche breakdown.
Zener breakdown: When reverse bias is increased the electric field at the junction
(mA)
also increases. At some stage the electric field becomes so high that it breaks the
covalent bonds creating electron-hole pairs. Thus a large number of carriers are l
generated. This causes a large current to flow. This mechanism is known as zener v2
breakdown. V
Avalanche breakdown: At high reverse voltage, due to high electric field, the
minority charge carriers, while crossing the junction acquires very high velocities. (A)
These by collision breaks down the covalent bonds, generating more carriers. A Figure 26.21
chain reaction is established, giving rise to high current. This mechanism is called
avalanche breakdown. In other words the covalent where the junction break down liberating a large number of
electron hole pairs. Then the reverse current increases abruptly to high value. This is called avalanche break down
and may damage the junction. This phenomenon is used to zener diode and used in voltage regulator.

Illustration 11: In a pure germanium sample, the electron and hole mobilities at room temperature are
3600cm2 / V s, 1700 cm2/ V s respectively. If the electron-hole concentration is 2.5 × 1019 m−3 , then the conductivity
of Ge is  (JEE MAIN)

σ nie ( µe + µn )
Sol: As the electron-hole concentration is given the conductivity of the germanium is given by =
where µe and µh are electron and hole mobilities respectively.

σ nie ( µe + µn ) ; ( ni = pi )
=

σ= 2.5 × 1019 × 1.6 × 10−19 ( .36 + .17=


) 2.12  / m
P hysi cs | 26.15

7. JUNCTION DIODE AS RECTIFIER


A device which converts alternating current (a.c.) into direction current (d.c.) is known as rectifier. The process of
converting a.c. into d.c is known a rectification.
Principle: Junction diode conducts only when forward biased and not conduct when reverse biased. It acts as a
valve. This fact makes the junction diode to work as a rectifier.

7.1 Junction Diode as a Half Wave Rectifier


The Rectifier which converts only one half of a.c. into Junction diode
d.c. is called halfwave rectifier. The circuit diagram of
half wave rectifier is shown in Fig. 26.22.
The a.c. input signal to be rectified is fed to the
primary (P) coil of the transformer. The secondary (S)
Input
A.C
 P P RL
Output
voltage
coil is connected to the junction diode through a load
resistance RL. The output signal is obtained across the
load resistance RL.
Transformer
Working: Since upper end of secondary coil is
connected to p-region and lower end is connected Figure 26.22
to the n-region of the junction diode, so the junction
diode is forward biased during the positive half of A.C Input

Input
+ +
input a.c. The output voltage is obtained across the O  t
- 2 -
load resistance RL. Upper end of the secondary coil
becomes negative and lower and becomes positive.
So the junction diode is reverse biased. Hence the Output
junction does not conduct and we get no output +
t
across the load resistance during negative half of O  2 D.C Output
input a.c.
Figure 26.23
Disadvantages:
(a) Since the output signal is discontinuous, so the efficiency of half wave rectifier is small.
(b) The output is not pure d.c. but it is a fluctuating (or pulsating a.c.) which contains a.c. components or ripples
also.
Expression for output d.c. voltage: Output d.c. voltage = Mean load current × load resistance i.e. Vd.c=Id.c. × RL
Iο I
But Id.c. = , where Iο is the maximum value of the secondary half wave current∴Vd.c. =ο × RL
π π

7.2 Junction Diode as a Full Wave Rectifier


Full wave rectifier converts both halves a.c. input signal to d.c. D1
output. S1
The p-regions of both the diode the D1 and D2 are connected S
to the two ends of the secondary coil (s). The load resistance RL A.C  P
across which output voltage is obtained is connected between Source D.C voltage
output
common point of n region of diodes and central tapping of the
S2
secondary coil.
Centre tap type D2
Working: The upper end of the secondary coil becomes
positive while the lower end becomes negative. Thus, diode transformer
D1 is forward biased and diode D2 is reverse biased, so the Figure 26.24
current due to diode D1 flows through the circuit in a direction
2 6 . 1 6 | Semiconductor and Communication System

shown by arrows (above RL).The output voltage which varies in A.C at S1


accordance with the input half cycle is obtained across the load

Input
+ +
resistance (RL). O  t
- 2 3 - 4
During negative half cycle of input a.c. signal. Diode D1 is reverse A.C at S2
biased and diode D2 flows through the circuit in a direction + +
shown by arrows (below RL). The output voltage is obtained t
 - 2 3 - 4
across the load resistance (RL).
D1 D2 D3 D4
Since both the halves of input a.c. (wave) are rectified, so the

Output
junction diode is called a full wave rectifier.
t
O  2 3 4
Advantage: In full wave rectifier, output is continuous, so its
efficiency is more than that of the half wave rectifier. D.C. Output

However, the output is again fluctuating (or pulsating d.c.) Figure 26.25
which can be smoothened by using a filter circuit.
Expression for output d.c. voltage: Output d.c. voltage=Mean load current × load resistance i.e. Vd.c.
= Id.c. × RL
2Iο
But Id.c. = , where Iο is the maximum value of the secondary full wave current
π
I
∴Vd.c. =ο × RL
π
Thus, output d.c. voltage in case of full wave rectifier is twice the output d.c. voltage in case of half wave rectifier.

PLANCESS CONCEPTS

S. No. Half wave rectifier Full wave rectifier

Iο 2Iο
1. Iac
= Idc
= Iac =
π π

Vο 2Vο
2. E=
ac E=
dc Eac =
π π

3. r 1.21 ∴Iac > Idc


= ∴Iac < Idc
r= 0.48,

0.406 rp 0.812 rp
4. =η ;1+ =η ;1+
rp RL rp RL

5. Form Factor =1.57 1.11

6. Ripple frequency =w 2w

input pulse frequency


7. Pulse frequency = Pulse frequency = input pulse frequency
2

Yashwanth Sandupatla (JEE 2012, AIR 821)


P hysi cs | 26.17

7.3 V-I Characteristics of Junction Diode


Volt-ampere or V-I characteristic of a junction diode (i.e. pn junction) is the curve between voltage across the diode
and current through the diode. Since a junction diode may be forward biased or reverse biased, it has two type of
V-I characteristics viz.

(a) Forward characteristics (b) Reverse characteristics.


(a) Forward characteristics. It is the graph between forward voltages (VF) applied across the junction diode
and the resulting forward current (IF) through the diode. Figure 26.26(A) shows the circuit arrangement for
determining the forward characteristics of a junction diode. Note that R is current limiting resistance and
prevents the forward current from exceeding the permitted value.
The forward current is due to the majority carriers.

Forward current (mA)


7
6
K
5
mA
4 B
V 3
V
2 Knee
A
1 voltage
VF
O 0.2 0.4 0.6 0.8 1.0 1.2
Forward bias (V)
(A) (B)
Figure 26.26

The forward voltage (VF) across the junction diode is increased from zero in steps and the corresponding
values of forward current (IF) through the diode are noted. If we plot the graph between VF and IF we get the
forward characteristic OAB of the junction diode as shown in Fig. 26.26 (B). The shape of this curve can be
explained as under:
When the applied forward voltage is zero i.e. circuit is open at K, the barrier potential V0 at the junction does
not permit current flow. Therefore, the forward current IF is zero as indicated by point 0 in Fig. 26.26 (b). As the
forward voltage is increased from zero, the forward current increases very slowly (curve 0A) until the forward
voltage across the diode reaches V0 (=0.7V for silicon diode and 0.3V for germanium diode) at the knee of
the curve. The forward voltage corresponding to knee of the curve is called knee voltage. Once the applied
forward voltage exceeds the knee voltage, the forward current increases rapidly (curve AB).
The forward voltage at which the current through the diode starts to increase rapidly with increase in forward
voltage is called knee voltage. For silicon diode, knee voltage=0.7V while for germanium diode, knee voltage=
0.3V.
Below the knee voltage, the curve is non-linear. But once the forward voltage exceeds the knee voltage, the
diode behave like an ordinary conductor. Therefore, forward current rises sharply with increase in forward
voltage (curve AB). The curve is now almost linear.
(b) Reverse characteristics. It is the graph between the reverse voltage (VR) applied across the junction diode
and the reverse current (IR) through the diode. Figure 26.27(A) shows the circuit arrangement for determining
the reverse characteristics of a junction diode. Note that the diode is reverse biased.
The reverse voltage (VR) across the junction diode is increased from Fig. 26.27 (B) zero in steps and the
corresponding value of reverse current (IR) are noted. If we plot the graph between VR and IR, we get the
reverse characteristics OCD of the junction diode as shown in Fig. 26.27(B). The shape of reverse characteristic
of the diode can be explained as under:
Since the diode is reverse biased, its resistance is very high and practically no current flows through the circuit.
2 6 . 1 8 | Semiconductor and Communication System

However, in practice, a very small reverse current (of the order of µA ) flows with reverse bias as shown in Fig.
26.27(B). This is called reverse saturation current because its value practically remains constant until reverse
breakdown voltage (VBR) is reached. The reverse saturation current is due to minority carriers. It may be
recalled that are a few free electrons in p-type material and a few holes in n-type material. These undesirable
free electrons in p-type and hole in n-type are called minority carriers. To these minority carriers, the applied
reverse bias appears as forward bias. Therefore, a small reverse current (IR) flows in the circuit.
15 10 5
VR(volts)
O
K VBR C 100
V
200
V V Breakdown
D voltage 300
400
lR(A)

(A) (B)
Figure 26.27

As shown in Fig. 26.27(b) when the reverse voltage becomes equal to reverse breakdown voltage VBR, the
reverse current increases very rapidly. Now reverse voltage remains approximately constant at VBR but reverse
current IR increases very sharply resulting in overheating and possible damage. This large value of IR is due to
the fact that the kinetic energy of electrons (minority carriers) becomes high enough to knock out electrons
from semiconductor atoms. Thus there is sudden decrease in resistance of the junction and abrupt rise of
reverse current.

7.4 Dynamic or A.C. Forward Resistance of Junction Diode


It is the opposition offered by the junction diode to the changing forward current and may be defined as under:
The ratio of change in forward voltage across the diode to the resulting change in current through it is called a.c.
forward resistance of the diode.
change in forward voltage across the diode
a.c. forward resistance, rf =
corresponding change in current through diode

The a.c. forward resistance is more significant as the diodes are generally used with lF
alternating voltages. The a.c. forward resistance can be determined from the forward
characteristic as shown in Fig. 26.28. If P is the operating point at any instant, then
forward voltage is ob and forward current is OE. To find the a.c. forward resistance, F
vary the forward voltage on both sides of the operating point equally as shown in E
D
Fig. 26.28. where AB=BC. It is clear from this figure that:
For forward voltage OA, circuit current is OD. For forward voltage OC, circuit current
vF
is of. O ABC

∴ a.c.
change in forward voltage OC−−oa
oc OA acAC Figure 26.28
forward resistance
= is; rf = = = =
change in forward current OF−−od
of OD dfDF

It may be mentioned here that forward resistance of a crystal diode is very small. Ranging from 1 to 25 Ω. Note that
above the knee point in the forward characteristic, the curve is linear. Therefore, above knee point, r, is independent
of the forward applied voltage.
A.C. reverse resistance. The a.c. reverse resistance of a junction diode is very large and may be considered infinite
for all practical purposes. For reason, a reverse diode practically conducts no current.
P hysi cs | 26.19

8. ZENER DIODE
A property doped P-N junction diode which works in the breakdown region without damaging itself is called a
zener diode.
Zener diode is also known as breakdown diode. It is mainly as a voltage regular.
Symbolic representation of zener diode is made as
The breakdown voltage zener voltage Vz depends on the concentration of doping. Both n and p regions of zener
diode are heavily doped. The depletion layer is very thin. Since electric field,
E=-dV/dr, the electric field across the junction will be very high.
Volt-ampere characteristics of zener diode

+V - -V +



+
mA
- A



- +
+ - - +

(A) Forward biased (B) Reverse biased


Figure 26.29

Zener diode operates in the breakdown region (reverse bias).


Zener Diode as a voltage regulator: A zener diode can be used as
Forward
voltage regular or stabilizer to provide a constant voltage from a source. current Ge
The zener diode is connected across the fluctuating voltage source (mA) Si
through a dropping resistor of resistance Rs. The constant voltage Reverse voltage
supply is obtained across the load RL. V, (Si) V,(Ge)
The zener diode of zener voltage Vz is reverse connected across the
Forward voltage
load RL across which constant output is desired. The series resistance
Rs absorbs the output voltage fluctuation so as to maintain constant
voltage across the load RL. Reverse voltage
(a) Suppose the input voltage increases. Since the zener is in the (A)
breakdown region, the zener diode is equivalent to a battery Vz Figure 26.30
as shown in Fig. 26.32 (ii) It is clear that output voltage remains
constant at Vz (=E0). The excess voltage is dropped across the series
resistance RS. This will cause an load current remains constant. Hence, output voltage E0 remains constant
irrespective of the changes in the input voltage Ein.

RS

Fluctuating or Constant
voltage voltage
VZ RL output
(Input)

Figure 26.31

(b) Now suppose that input voltage is constant but the load resistance RL decreases. This will cause an increase
in load current. The extra current cannot come from the source because drop in RS (and hence source current
2 6 . 2 0 | Semiconductor and Communication System

I) will not change as the zener is within its regulating range. The additional load current will come from a
decrease in zener current Iz. Consequently, the output voltage stays at constant value.

RS l RL RS l RL
- -

Ein RL EO Ein VZ EO
RL
lZ
+ +
(i) (ii)
Figure 26.32

Voltage drop across RS=Ein-E0


Current through RS, I=IZ+IL
Ein − E0
Applying ohm’s law, we have, R s =
I Z + IL

9. PHOTO DIODE
The junction diode which conducts when charge carriers are generated by the photons i.e., light incident on it is
known as optoelectronic junction device.
A reverse biased special p-n junction diode having transparent window is known as photo diode and when it is
illuminated with light, the reverse diode current varies linearly with the light flux.
Construction: A reverse biased p-n junction diode is enclosed in a clear plastic envelope. Light is allowed to fall on
the surface of the plastic facing the diode. The output voltage is taken across the load resistance RL.
Symbolic representation of a photo diode is shown in the Fig. 26.33.
Principle. When a rectifier diode is reverse biased, it has a very small reverse leaked current. Cathode
The same is true for a photo-diode. The reverse current is produced by thermally generated
electron-hole pairs which are swept across the junction by the electric field create by the
reverse voltage. In a rectifier diode, the reverse current increases with temperature due
to an increase in the number of electron hole pairs. A photo-diode differs from a rectifier
diode in that when its pn junction is exposed to light, the reverse current increases with the Anode
increase in light intensity and vice-versa. This is explained as follows. When light (photons)
falls on the pn junction, the energy is imparted by the photons to the atoms in the junction. Figure 26.33
This will create more free electrons (and more holes). These additional free electrons will
increase the reverse current. As the intensity of light incident on the pn junction increases,
the reverse current also increases. In other words, as the incident light intensity increases, the resistance of the
device (photo-diode) decreases. Figure 26.33 shows the schematic symbol of a photo-diode. The inward represent
the incoming light.
Photo-diode operation. Figure 26.33 shows the basic photo-diode circuit. The circuit has reverse biased photos-
diode, resistor R and d.c. supply. The operation of photo-diode is as under:
(a) When no light is incident on the pn junction of photo-diode, the reverse current Ir is extremely small. This is
called dark current.
The resistance of photo-diode with no incident light is called dark resistance (RR).
VR
Dark resistance of photo-diode, RR =
Dark current
(b) When light is incident on the junction of the photo-diode, there is a transfer of energy from the incident light
P hysi cs | 26.21

(photons) to the atoms in the junction. This will create more free electrons (and more holes). These additional
free electrons will increase the reverse current.
(c) As the intensity of light increases, the reverse current IR goes on increasing till it becomes maximum. This is
called saturation current.
Reverse current versus illumination curve. lR(A)
Figure 26.34 shows the graph between reverse lR
current (IR) and illumination (E) of a photo-diode.
The reverse current is shown on the vertical 100
axis and is measured in µ A. The illumination is R
30
indicated on the horizontal axis and is measured v
in m W/cm2. Note that graph is a straight line VR 10

 
passing through the origin.
mW
E
m E Where m= slope of the straight line
2
∴ IR = 0 1 3 10 cm
The quantity m is called is called the sensitivity
Figure 26.34
of the photo-diode.
Volt- ampere characteristics of photo diode. When photo diode is reverse biased,
then a constant current known as saturation current I0 due to thermally generated
minority carriers flows in the circuit. This current is also known as dark current.
When light of energy (hv) more than the energy gap (Eg) of semiconductor falls on the Photo diode
photo-diode, additional electron-hole pairs are formed. The electron-hole pairs formed
Figure 26.35
are proportional to the intensity of the incident light or the number of incident photons.
These electrons holes diffuse through the junction and hence current Is also flows in
addition to the dark current I0. Thus the electric current I0 is proportional to the intensity
of incident light. Hence, the total reverse current is given by. I=I0+Is
RL
Total reverse current in a photo-diode increase with the increase in the intensity of the
incident light.

Uses of photo diodes: Figure 26.36


(a) Photo diodes are used as photo detectors intensity of
radiation. Reverse voltage
(in volts)
(b) They are used as light operated switches.
Dark current
(c) They are used in optical communication equipment’s. Photo-diode
(d) They are used in fast reading of film sound tracks and current (in A)
tapes. Intensity (l1)
(e) They are used in logic circuits.
Intensity (l2)
(f) They are used as optical demodulators. l2>l1

Figure 26.37

PLANCESS CONCEPTS

Photo-diodes are operated in reverse photo-diode is used to detect photo radiation.


Light variations affect minority carrier based reverse current much more than the forward current
Anand KJEE 2011, AIR 47
2 6 . 2 2 | Semiconductor and Communication System

10. LIGHT EMITTING DIODE (LED)


A special heavily doped P-N junction diode which emits spontaneous radiation when forward
biased is known as light emitting diode (LED).
The symbolic representation of LED is snown in the Fig. 26.38:
LEDs made of elemental semiconductor like germanium (Ge) and silicon (Si) emit energy in the LED
form of infra-red (or heat) radiation. Figure 26.38
LEDs made of Ga As of Eg~1.4 eV emits infrared radiations.
LED made of Ga As 0.6 P 0.4 eV Eg~1.9 eV emits red light.
Theory: When a p-n junction diode is forward biased (See Fig. 26.39), the Visible Visible
electrons injected to p-side of the junction diode falls from the conduction light light
band to the valence band and recombine with the holes in the valence p n
band. [This is equivalent to the jumping of electrons from higher energy
state (i.e., conduction band) to lower energy state (i.e., valence band)]. + +
Hence, energy is known as electro-luminescence. The energy of the photon -+ +-
of visible light by hv =Eg, where Eg is the energy gap between conduction
band and valence band v is the frequency of emitted visible radiation. The
hc hc Figure 26.39
wavelength of the emitted light is given=
by Eg or λ
λ Eg
Advantage of LED
(a) Light emitting diodes are easily manufactured.
(b) LEDs have low cost.
(c) LED works at low voltage as compared to the incandescent bulb.
(d) LED has longer life than the incandescent bulb.
(e) They can be switched on and off very fast so they can be used as blinkers.
(f) Now warm up time is taken by them,
(g) They can emit monochromatic light as well as white light.

Uses of light emitting diodes


Light emitting diodes have the following uses:
(a) They are used as indicator lamps.
(b) They are used in digital displays in watches and calculators.
(c) Light emitting diodes which emit infra-red light are used in burglar alarm.
(d) They are used in remote control schemes.
(e) They are used as blinkers.
(f) They are used as decorating lights.
(g) They are used as light lamp and in torches.

11. SOLAR CELL OR PHOTO-VOLTAIC DEVICE


A special p-n junction diode which converts solar energy (sun light) into electrical is known as solar cell or photo-
voltaic device. Junction surface of these diodes is kept large so that large radiations are caught.
A simple solar cell consists of a p-n junction of which n-region is very thin and p-region is thick.
P hysi cs | 26.23

Nickel plated Solar Energy or light energy


contact
of hv > Eg
l
n

RL p Junction

Nickel plated
contact

Figure 26.40

Action:
(a) When solar energy or light energy falls on the cells, electron-hole pairs are

Electric current
generated in both n-region and p-region of the junction in diode.
(b) The electrons from p-region diffuse through the junction to n-region and holes
from n-region diffuse through the junction to the p-region due to electric field
of depletion layer.
(c) If p-n junction diode is open circuited, then holes and electrons will collect or Intensity of sun light
accumulate on the two sides of the junction. This gives rise to an open circuit
voltage V0. Figure 26.41

V-I characteristics: A typical V-I characteristic of a solar cell is shown in Fig.26.42. V0


is open circuit voltage of the solar cell and Is is the maximum current i.e., short circuit
which can be drawn from the cell. ll l
VR
Uses of solar cell: (V)
(a) Solar cells are used in street lights
lll lS lV
(b) They are used in solar heaters.
(l)
(c) They are used in power supply of satellites and space vehicles.
Figure 26.42
(d) They are used in calculators.

Illustration 12: A diode used in the circuit shown in Fig. 26.43. has a constant R
voltage drop of 0.5 V at all currents and a maximum power rating of 100 mW.
What should be the value of resistance R connected in series with diode for
obtaining maximum current?  (JEE MAIN)

Sol: The power dissipated across diode is given by Pmax = V.Imax where Imax is the
maximum current through the diode. The value of resistance across the diode
is given by Ohm’s law.
Figure 26.43
Applied voltage, E=1.5V
Voltage drop across diode Vd = 0.5V
Max. Power rating of diode, Pmax=100 mW =0.1 W
Pmax 0.1
The maximum current (Imax) that diode can carry safely is Imax
= = = 0.2A
Vd 0.5
Voltage drop across resistance R=E-Vd=1.5-0.5=1.0V
Voltage drop across R 0.1
∴R= = =5Ω
Imax 0.2
2 6 . 2 4 | Semiconductor and Communication System

Illustration 13: A battery of 2 V is connected across the points A and B as shown in Fig. 26.44. Find the current
drawn from the battery if the positive terminal is connected to (i) The points A and (ii) The point B. Assume that the
resistance of each diode is zero in forward bias and infinite in reverse bias.  (JEE ADVANCED)

10 10 20


20

2V 2V
A B
Figure 26.44

Sol: When the diode is connected in forward bias condition the resistance of diode is zero and hence the current
through the diode is maximum. While the diode is in the reverse biased condition the resistance is infinite thus the
circuit acts as open circuit. Thus the effective current in the circuit is obtained using Ohm’s Law.
(a) When positive terminal of the battery is connected to point A, diode D1 is forward biased and offers zero
resistance while diode D2 is reverse biased and offers infinite resistance therefore, diode D1 may be replaced
by a wire while diode D2 is open- circuited.
The circuit then reduced to that shown in Fig. 26.44.
2V
∴ Current drown from battery, I = 0.2A
10Ω
(b) When the positive terminal of the battery is connected to point B, diode D1 is reverse biased and offers infinite
resistance while diode D2 is forward biased and offers zero resistance. The circuit then reduces to that shown
in Fig. 26.44.
2V
∴ Current drown from battery, I = 0.1A
20Ω

Illustration 14: In Fig, 26.45 what is the voltage needed at the source, to maintain 15 V across the load resistance
RL of 2k Ω , assuming that the series resistance R is 200 Ω and the zener requires a minimum current of 10 mA to
work satisfactory? What is the zener rating required?  (JEE MAIN)

Sol: Current through the load resistor of resistance 2k Ω is found using Ohm’s law. As
10 mA current is still flowing through the zener when it is connected in reverse biased
condition while the rest of current passes through load resistor, the rating of the zener
lR
R
diode should be higher than total current passing through the zener. The rating is given
by IR+IL.
Voltage across RL, is VL=15 V lZ vZ RL lL
VL 15
Current through RL , IL= = 7.5 10−3 A =
=× 7.5mA
RL 2 × 10 −3

Zener current, Iz=10 mA ∴ Current through R, IR= Iz+IL=10+7.5mA Figure 26.45

Voltage drop across R, VR= IR × R=17.5 × 10-3×200=3.5 V


Input voltage required, V= VR+VL =3.5+15=18.5V
Therefore, zener diode should have a current rating of 17.5 mA and a breakdown voltage of 15V.

Illustration 15: An a.c. supply of 230 V is applied to a half-wave rectifier circuit through a transformer of turn ratio
10:1 Find (i) The output d.c. voltage and (ii) The peak inverse voltage. Assume the diode to be ideal. 
 (JEE ADVANCED)
P hysi cs | 26.25

Sol: The transformer is the step down transformer, thus the 10 : 1 Ideal
secondary voltage will be less than primary voltage applied. The
DC output voltage obtained at the end of secondary coil is given
N2
by V=
sm Vpm × where Vpm is peak voltage at the primary coil.
N1

340 V RL
The inverted DC voltage obtained at the secondary coil is given by
Vsm
′ =
Vdc .
π
Primary to secondary turns is [see fig.26.46] Figure 26.46

N1
= 10 R.M.S primary voltage =230 V
N2
∴ Maximum primary voltage is V
=pm ( 2 ) × r.m.s. primary voltage = ( 2 ) × 230 = 325.3V Max. Secondary voltage
is
N2 1
Vsm = Vpm × = 325.3 × = 32.53V
N1 10
Vsm 32.3
(i) =′
Vdc = = 10.36V
π π
(ii) During the negative half-cycle of a.c. supply, the diode is reverse biased and hence conducts no current.
Therefore, the maximum secondary voltage appears across the diode.
∴ Peak inverse voltage = 32.53 V

Illustration 16: A crystal diode having internal resistance ri=20 Ω is used for half-wave rectification. If the applied
voltage
= V 50 sin ωt and load resistance = RL 800 Ω, find:
(i) Im, Idc, Irms (ii) a.c. power input and d.c power output (iii) d.c output voltage (iv) Efficiency of rectification 
 (JEE MAIN)
Sol: The equation of AC voltage is given by= V Vo sin ωt where Vo is the maximum voltage from the source. As
a load resistance is applied in series to the supply, the current through the resistance and output voltage are

obtained by Ohm’s law. The power is given by = P Ieff2 × R eff where Reff is the effective resistance in the circuit. The
PDC
efficiency of the diode is given by ε = where PDC and PAC are DC and AC powers dissipated in the circuit.
PAC

Given that, V=50 sin ωt ; ri = 20 Ω, and RL = 800


∴Maximum voltage, Vm=50V
Vm 50
(i) Im
= = = 0.061A
= 61 mA; I= Im /=
π 61 /=
π 19.4 mA
dc
rf + RL 20 + 800
Irms Im
= = / 2 61
= / 2 30.5 mA
2
 30.5 
a.c. power input= ( Irms ) × (rf +=
RL )   × ( 20 + 800 ) 0.763 watt
2
(ii) =
 1000 
2
 19.4 
(iii) d.c power output = I2dc=
× RL  800 0.301 watt
 ×=
 1000 

(iv) d.c. output voltage ==


IdcRL 19.4mA × =
800Ω 15.52 volts

0.301
(v) Efficiency of rectification== × 100= 39.5%
0.763
2 6 . 2 6 | Semiconductor and Communication System

Illustration 17: A full wave rectifier uses two diodes, the internal resistance of each diode may be assumed
constant at 20 Ω. The transformer r.m.s secondary voltage from center tap to each end of secondary is 50 V and
load resistance is 980 Ω find.
(i) The mean load current (ii) The r.m.s. value of load current  (JEE MAIN)

2Im
Sol: Mean load current is given by Idc = where Im is the maximum current through load and rms current is
π
Im
given by Irms
= = 0.707 Im .
2

rf =20Ω, RL =980Ω ; Max. a.c. voltage, Vm= 50 × 2 =


70.7V

Vm 70.7V
=
Max. Load current, Im = = 70.7mA
rf + RL ( 20 + 980 ) Ω
2Im 2 × 70.7
(i) Mean load current, =
Idc = = 45mA
π π
Im 70.7
(ii) R.M.S value of load current is Irms
= = = 50mA
2 2

Illustration 18: In the center- tap circuit shown in Fig. 26.47, the diodes are assumed to be ideal i.e. having zero
internal resistance. Find:
(i) d.c output voltage (ii) peak inverse voltage  (JEE ADVANCED)
2Vm
Sol: The average dc current in the circuit is Idc = where Vm is the maximum voltage across secondary coil
πRL
of center tap circuit. And thus the DC output voltage is obtained using Ohm’s law. The peak inverse voltage is

Vo′ = 2 Vrms Ideal


5:1
Primary to secondary turns, N1/N2=5
R.M.S. primary voltage = 230V Vm
∴ R.M.S. secondary voltage = (
230 × 1 / 5 =46V )

340 V
Maximum voltage across half secondary winding is
Vm
Vrms
Vm
= = 32.5 V
2
2Vm 2 × 32.5
Idc
(i) Average current,= = = 0.207 A Ideal
πRL π × 100
Figure 26.47
∴ d.c. output voltage, Vdc = Idc × RL = 0.207 × 100 = 20.7V
(ii) The peak inverse voltage (PIV) is equal to the maximum secondary voltage i.e. PIV=65V

Illustration 19: In a zener regulated power supply, a zener diode with Vz= 6V is used for regulation. The load
current is to be 4.0 mA and unregulated input is 10.0 V what should be the value of series resistor Rs?
 (JEE ADVANCED)

Sol: The current through the series resistance is =I I z + IL where IZ is the zener current and IL is the current through
Veff
load resistance. The value of resistance is R S = where Veff is the effective resistance across the resistance.
I
Here, IL=4.0 mA; Vz=6V; Ein =10.0 V; Rs=?
P hysi cs | 26.27

For good regulation, the value of Rs should be such that current through the zener diode is much larger than the
load current. Choosing zener current Iz to be 5 times the load current,
Iz=5IL= 5 × 4.0 =
20mA ; ∴ Current through Rs, is I = I z + IL = 20 + 4 = 24 mA

Voltage across R s 4V
Voltage across R s =Ein − Vz =10.0 − 6.0 = 4;  ∴ R s = = = 167Ω
I 24mA

Illustration 20: Figure 26.48 .shows the forward characteristic of a junction diode. Determine the d.c. and a.c.
resistance of the diode when it operates at 0.3 V.  (JEE MAIN)

Sol: The DC resistance of the diode is obtained by the slope of the curve at 0.3 C
V. The ac resistance is obtained by taking the ratio of the voltage difference at 6
two point with their corresponding current. B
4
Referring to Fig. 26.48 when VF =0.3 V; IF=4.3 mA=4.3 ×10 −3 A.

lF (mA)
A
VF 0.3VA
∴ D.C resistance rdc =
= = 69.77Ω 2
IF 4.3 × 10 −3
Referring to Fig. 26.48, ∆VF = VC − VA and ∆IF = IC − I A 0 0.1 0.2 0.3 0.4
∆VF VC − VA 0.35 − 0.25 VF(volts)
∴ A.C. resistance, rac= = = = 33.33Ω
∆IF IC − I A ( 6 − 3) × 10−3 Figure 26.48

12. JUNCTION TRANSISTOR


A transistor is an electronic device formed by p and n-type of semiconductor which is used in placed of a triode
valve. It was discovered in 1948 by American scientists Bardeen, Shockley and Brattain. Transistors are of two type:
p-n-p transistor and n-p-n transistor.

12.1 P-N-P Transistor Emitter Base Collector


E C
It consists of a very thin layer of
n-type semiconductor sandwiched P n P
E C
between two small p-type semi
conductor (See Fig. 26.49). The (A) (B) (B)
p-n-p
central slice is called the ‘base’ while B
the left and right crystals are called
- + - +
the ‘emitter’ and the ‘collector’
respectively. The emitter is given a Figure 26.49
positive potential while the collector
is given a negative potential with Emitter base collector
respect to the base. Thus, the emitter-base (p-n) junction on the left is under p n p
forward-bias (high resistance). While the base-collector (n-p) junction on the
E C
right is reverse-bias. The symbol for this transistor is shown in Fig. 26.49 (B) in
which the direction of the arrow indicates the direction of current (direction
of flow of holes). B
Forward Reserve
Working: A ‘common-base’ circuit of p-n-p transistor is shown in Fig. 26.50. iE biased
iB
biased
iC
The emitter-base (p-n) junction on the left is given a small forward bias + - + -
(fraction of a volt) while the base-collector (n-p) junction is given a large
VEB VCB
reverse-bias (a few volts).
Hole (+)
Holes are the charge-carriers with in the p-n-p transistor, while electrons are Electron (-)
the charge-carriers in the external circuit.
Figure 26.50
2 6 . 2 8 | Semiconductor and Communication System

The small current which leaves the base terminal B is called the ‘iB’ ‘base-current’ the larger current which leaves
the collector terminal is called the ‘collector-current’ iC. Both these currents combine to enter the emitter terminal
E and constitute the emitter-current iE. Clearly, iE=iB+iC
The base being very thin, the number of hole-electron combinations in it is very small, and almost all the holes
entering the base from the emitter reach the collector. Hence the collector-current iC is only very slightly less than
the emitter current iE.

12.2 N-P-N Transistor


It consists of a very thin slice of p-type semiconductor (Fig. 26.51). In this transistor the emitter is given a negative
potential while the collector is given a positive potential with respective to the base. Again, the emitter-base (n-p)
junction on the left is under forward-bias, while the base collector (p-n) junction on the right is denser reverse-bias.
*The symbol for the p-n-p transistor is shown in Fig. 26.51. (B) in which the direction of the arrow indicates the
direction of current (opposite of the direction of flow of electrons).
Emitter Base Collector
E C

P n P
E C
(A) (B) (B)
p-n-p
B
- + - +
Figure 26.51

A transistor can be connector in a circuit in three different ways. They are:


(i) Common- base configuration, (ii) Common-emitter configuration and (iii) Common-collector configuration.
The word ‘common’ is related with that electrode which is common in input and output circuit. This common
electrode is generally grounded. Hence the above three configuration of connection are also called respectively as
grounded- base configuration, grounded-emitter configuration, and groundcollector of connection are also called
configuration. Each configuration has its own characteristics.
Working: A circuit known as’ common-base’ circuit of n-p-n transistor is Emitter base collector
shown in Fig. 26.52. The two n-regions contain the mobile electrons while p n p
the central thin p-region contains the mobile (positive) holes. The emitter-
base by means of a battery VEB, while the base-collector (p-n) junction on E C
the right has been given a large reverse-bias by means of battery VCB.
The electrons are charge-carriers with in the n-p-n transistor as well as in
B
the external circuit (whereas holes are the charge- carriers with in p-n-p iE
Forward
iB
Reserve
iC
transistor). biased biased
- + - +
The small current entering the base terminal B is the base current iB, while
the larger current entering that collector terminal C is the collector-current VEB VCB
iC. Both currents combine to leave the emitter terminal E and constitute the Hole (+)
emitter current iE. Thus, iE=iB+iC Electron (-)
Figure 26.52
13. TRANSISTOR ACTION
There are four possible ways of biasing the two PN junctions (emitter junction and collector junction) of a transistor.
There are tabulated below.

Emitter junction bias Collector junction bias Transistor operation


Forward Reverse Active
P hysi cs | 26.29

Emitter junction bias Collector junction bias Transistor operation


Forward Forward Saturation
Reverse Reverse Cut off
Reverse Forward Inverted

iE=iB+iC; iB<< iC, and iB<< iE N P N


Thus iC is always less than iE, but the difference is small.
Since the emitter junction is forward biased its resistance is
lE
small, while the collector junction is reverse biased, therefore its
resistance is large. Thus, a transistor is a device which transfers E C
iE current from low resistance circuit to a high resistance circuit
( IC < IE ) . Thus it is, lB
lE B lC
Transfer+ resistor =transistor
lB
(The name transistor originated from the action of the transistor). - + - +
Figure 26.53

14. TRANSISTOR CHARACTERISTICS AND USE AS AN AMPLIFIER


A junction diode cannot amplify a signal. A transistor consisting of two p-n junctions, one is forward-biased and
the other is reverse-biased can however, be used for amplifying a weak signal the forward-biased junction has
a low-resistance path whereas the reverse-biased junction has a high-resistances path. The weak input signal is
applied across the forward-biased (low resistance) junction and the output currents are signal is taken appears with
a much higher voltage. The transistor thus acts as an amplifier.
p-n-p
p-n-p
C
E C
B E Output
Input  B Output Input 

+ - + - + -
+ -
(A) Common-base (B) Common-emitter
p-n-p
E
B C Output
Input 

- + - +
(C) Common-collector
Figure 26.54

When a transistor is to be operated as amplifier, three different basic circuit connections are possible, as illustrated
in Fig. 26.54. These are
(a) Common-base
(b) Common emitter and
(c) Common-collector circuits.
2 6 . 3 0 | Semiconductor and Communication System

14.1 Transistor as Common-Base Amplifier (n-p-n)


Figure 26.55 shows the common- n-p-n
iE C iC
base amplifier circuit using an n-p-n E
transistor. The base is made common
to the input and output circuits. RL
The emitter-base circuit is forward- B VCB iC
biased by a low-voltage battery VEB so +
iB VCC
that the resistance of the input circuit Input Output
is small. The collector-base output a-c - a-c
circuit is reverse-biased by means Signal - + i Signal
of a high-voltage battery VCC so the E iC
VEB
resistance at the output circuit is quite Figure 26.55
large. RL is a load resistor connected
in the collector-base circuit. The weak input ac voltage signal is applied across the emitter-base circuit and the
amplified output signal is obtained across the collector-base circuit.
Let iE, iB and ic be the emitter-current, base-current and collector-current irrespective when no ac voltage signal is
applied to the input circuit. (The arrows represent the direction of hole current that is conventional current which
is opposite to the direction of electron current). By Kirchhoff’s first law, we have iE=iB+iC  … (i)
Due to the collector-current ic, the voltage drop across RL is iC RL. Therefore, the collector-to-base voltage (potential
difference between collector and base) VCB would be given by
VCB= VCC - ic RL …. (ii)
When the input ac voltage signal is applied across the emitter-base circuit, it changes the emitter-base voltage
and hence the emitter-current iE which, in turn, changes the collector current iC .consequently the collector to base
voltage VCB varies in accordance with equation (ii). This variation in VCB, when the input signal is applied, appears
as an amplified output.
Phase relationship between input and output voltage signal in CB circuit:
The output voltage signal is in with the input voltage signal in the common-base amplifier.

14.1.1 Gains in Common-Base Amplifier


The various gains in a common-base amplifier are as follow:
(a) ac current gain: It is defined as the ratio of the change in the collector- current to the change in the emitter-
current at a constant collector-to-voltage, and is denoted by a. Thus

 ∆iC 
α( ac ) =
 
 ∆iE  VCB

The value of α is slightly less than 1 (actually there is a little current loss).
(b) ac Voltage Gain: It is defined as the ratio of the changes in the output voltage to the change in the input
voltage, and is denoted by AV.
Suppose, on applying ac input voltage signal, the emitter current changes by ∆iC and correspondingly the
collector-current changes by ∆iC . If Rin and Rout the resistances of the input and the output circuits respectively,
then

∆iC × R out ∆iC R out


A
= v = ×
∆iE × Rin ∆iE Rin

Now, ∆iC / ∆iE is the ac current-gain and R out / R in is called the ‘resistance gain’
 A v = α × Resistance gain
P hysi cs | 26.31

Since the resistance gain is quite high, AV is also quite high although a is slightly less than 1.
(i) ac Power Gain: It is defined as the ratio of the change in the output power to the change in the input power.
Since power=current × voltage, we have
ac power gain =ac current gain×ac voltage-gain= α2 ×resistance gain

14.2 Common Base Amplifier Using a p-n-p Transistor


Common-Emitter using an n-p-n transistor: Figure 26.56 show the common-emitter amplifier circuit using an
n-p-n transistor. The emitter is made common to the input and output circuits.
p-n-p
iE E C iC

RL

B VCB - iC
iB VCC
Input + Output
ac ac
Signal + - Signal
iE iC
VEB

Figure 26.56

The input (base-emitter) circuit is forward-biased by a low-voltage battery VBE so that the resistance of the input
circuit is small. The output (collector-emitter) circuit is reverse-biased by means of a high voltage battery VCC so
that the resistance at the output circuit is high. RL is a load resistance connected in the collected-emitter output
circuit. The weak input ac signal is applied across the base-emitter circuit and the amplified output signal is across
the collector-emitter circuit.
Let iE, iB and ic be the emitter-current, base-current and collector-current irrespective when no ac voltage signal is
applied to the input circuit. (The arrows represent the direction of hole current, that is conventional current which
is opposite to the direction of electron current). By Kirchhoff’s first law, we have iE=iB+iC  ... (i)

C
iB B n-p-n
RL
E iC
VCE
+ VCC
iE Output
Input - ac
ac Signal
+ - iB iC
Signal VBE

Figure 26.57

Due to the collector-current ic, (which is only slightly smaller that iE ), the voltage drop across RL is iC RL. Therefore,
the collector-to-base voltage (potential difference between collector and base) VCB would be given by VCE = VCC - ic
R  …. (ii)
When the input ac voltage signal is applied across the emitter-base circuit, it changes the emitter-base voltage
and hence the emitter-current iE which, in turn, changes the collector current iC .consequently the collector to base
voltage VCE varies in accordance with equation (ii). This variation in VCE, when the input signal is applied, appears as
an amplified output.
2 6 . 3 2 | Semiconductor and Communication System

Phase Relationship between input and output voltage signals: In a common-emitter amplifier the input voltage
signal and the output voltage signal obtained across the collector and the emitter are out of phase with each other.
The output voltage signal is 180o out of phase with the input voltage signal in the common-emitter amplifier.

14.3 Common-Emitter Amplifier Using a p-n-p Transistor

14.3.1 Gains in Common-Emitter Amplifier


The various gains in a common-emitter amplifier are as follows:
(a) dc current Gains: It is defined as the ratio of the collector current to the current, and is denoted by b (dc).
iC
Thus β ( dc ) =
iB
In a typical transistor, a small base-current ( ≈ 10µA ) produces large collector-current ( ≈ 500µA ).

C iC
iB B p-n-p
RL
E iC
VCE
VCC Output
iE -
Input + ac
ac Signal
- + iB iC
Signal VBE

Figure 26.58

500
Thus β ( dc ) = = 50
10
(b) ac current Gain: It is defined as the ratio of the collector-current to the change in the base-current at a
constant collector to emitter voltage, and is denoted by β(ac) .
 ∆iC 
Thus β ( ac ) =
 
 ∆iB VCE
(c) Voltage gain: Suppose, on applying ac input voltage signal, the base-current changes by ∆iB and
correspondingly the output collector-current change by ∆ic . If Rin and Rout be the resistance of the input and
the output circuits respectively,
∆iC × R out ∆iC R out
then A
= v = ×  ... (i)
∆iE × Rin ∆iE Rin
Now, ∆iC / ∆iE is the ac current-gain (ac) and R out / R in is the ‘resistance gain’
β(ac) × resis tance gain 
∴ Av = ... (ii)
Since β(ac) >>a (ac), the ac voltage gain in common-emitter amplifier is larger compared to the common-
base amplifier., although the resistance gain is smaller.
From equation (i) and (ii), it follows that A v =gm ×R out

(d) ac power gain: It is defined as the ratio of the change in the output power to the change in the input power.
Since power=current×voltage, we have
P hysi cs | 26.33

ac power gain=ac current gain × ac voltage gain= β(ac) × Av

= β(ac) × {β(ac) × resis tance gain} = β2 (ac) × resis tance gain

Since β(ac) >> α(ac) , the ac power gain in common-emitter amplifier is extremely large compared to that in
common-base amplifier.
(e) Trans conductance: (gm) is defined as the ratio of small change in the output current (i.e., collector current)
to the corresponding small change in the voltage (VB) at constant output voltage (Vc).

∆I0 ∆IC ∆IC ∆IC ∆IB


i.e gm
= = V0 =cons tant or gm
= = × (Multiplying and dividing by ∆VB )
∆V1 ∆VB V0 =cons tant
∆VB ∆VB ∆VB

 ∆I   ∆IB  ∆Ic ∆V
=  C  ×   But βa.c. and B =
= input resistance (Ri) ∴ gm =
βa.c. / R i
 ∆IB   ∆VB  ∆VB ∆IB
R0
Since voltage gain, A V =
βa.c. × ∴ Using eqn. (i), we have AV =
gm × R 0 For R 0 =RL ∴ A V =gmRL
Ri

14.3.2 Relation Between α and β


CB current gain (α): CB current gain (a) is the ratio of output current to the input current in common base

IC ∆Ic
configuration of a transistor. αdc= ; αac=
IE ∆IE

CE current gain (β): CE current gain (β) is the ratio of the output current to the input current in emitter configuration

IC ∆Ic
of the transistor. βdc= ; βac=
IB ∆IB

The CB current gain a and CE current gain b are related by the following relations.

1 1 β α
= 1+ ; α= ; β=
α β β +1 1−α

The above relations are applicable for both dc and ac current gains.

PLANCESS CONCEPTS
The value or a is always less than 1.a~0.9 to 0.99 or more. The value of b is always much greater than 1.
b ~ 95 to 99 or so.
Chinmay S Purandare (JEE 2012, AIR 698)

Illustration 21: The dc current gain of a resistance in CB configuration is 0.99. Find its dc current gain in CE
configuration.  (JEE MAIN)
Sol: As the gain in common base configuration αdc is given, the gain in common emitter configuration is given by
α
βdc = dc .
1 − αdc
2 6 . 3 4 | Semiconductor and Communication System

αdc 0.99 0.99


Given αdc 0.99 therefore, =
βdc = = or βdc 99
=
1 − αdc 1 − 0.99 0.1

Illustration 22: In certain resistance αdc =0.98 and IE =1 mA. Find the corresponding values of base current and
collector current.  (JEE MAIN)
C I
Sol: The collector current is found by αdc = . In the circuit the base current is IB= IE − IC .
IE
IC IC
=
(i) αdc or 0.98
= Thus IC =0.98mA
IE I (mA )

(ii) Using IB =
IE − IC ; we get IB = (
1 − 0.98 mA = )
0.02mA

Illustration 23: In a common base connection, current amplified factor is 0.9. If the emitter current is 1mA,
determine the value of base current.  (JEE MAIN)
C I
Sol: The collector current is found by αdc = . In the circuit the base current is IB= IE − IC .
IE
IC
Here α = 0.9, IE = 1 mA. Now α = Or IC =α IE =0.9 × 1 =0.9mA . Also IE= IB + IC
IE
lC
∴ Base current, IB =IE − IC =1 − 0.9 =0.1 mA RB C RC
B
 vO
lB E
vi  VCC
Illustration 24: For a CE amplifier (see Fig. 26.59), the audio lE
VBB
signal voltage across the collector resistance = R C 2.0 k Ω is
2.0 V. Suppose the current amplification factor of the transistor is
100. What should be the value RB in series with VBB supply of 2.0 V
if d.c. base current has to be to 10 time the supply current? Also
calculate the d.c. collector current. (JEE ADVANCED) Figure 26.59

ic
Sol: In the CE configuration the base current is obtained as ib = where ic is the AC collector current. The DC base
β
and collector current are obtained as i=
B 10 × ib and IC = β IB .

The output a.c. voltage is 2.0V.

∴ a.c. collector current 2.0V 2.0V


= iC = = 1 mA
RC 2000Ω
∴ a.c. base current =
ib iC /= β 1 / 100
= 0.01 mA
∴ d.c. base current, iB = 10 × ib = 10 × 0.01 = 0.1 mA

Applying Kirchhoff’s voltage law to the base circuit,

VBB − VBE ( 2.0 − 0.6 )


VBB =
IBRB + VBE or RB = = = 14kΩ (Assume VBE =0.6V)
IB 0.1 mA

d.c collector current IC= β IB = 100 × 0.1= 10mA

Illustration 25: In an NPN transistor, 1010 electrons enter the emitter in 10-6s. If 2% electrons are lost in the base,
calculate the current transfer ratio and current amplification factor.  (JEE MAIN)
IC IC
Sol: The amplification factor is given by β = and current transfer ratio is given by .
IB IE
P hysi cs | 26.35

Current =charge /time


N 1010 × 1.6 × 10 −19
∴ Emitter current , IE =e = 1.6 × 10 −3 A =
= 1.6mA
t 10 −6
2
Base current, IB = 2% of IE = × 1.6 = 0.032mA
100
In a transistor, the currents relation is IE = IB + IC or IC = IE − IB = 1.6 − 0.032 = 1.568 mA

C I 1.568
∴ Current transfer ratio=
= = 0.98
IE 1.6
IC 1.568
Current amplification factor, β= = = 49
IB 0.032

15. TRANSISTOR OSCILLATOR


Oscillator is a device which delivers a.c. output waveform of desired frequency from d.c. power even without input
signal excitation.
The electric oscillations are produced by L-C circuit (i.e. tank circuit containing inductor and capacitor in parallel).
L-C circuit producing L-C oscillations consists of an inductor inductance L and a capacitor of variable capacitance C.
Using positive feedback arrangement inductors L and L’ are inductively coupled as both the coils are around same
core acts as a positive feedback arrangement.
1 Feedback circuit
Working: When switch is closed, emitter-base junction is forward
biased and the collector-emitter base junction reverse biased. The L’
emitter current and hence collector current begins to flow. The
inductor L’ opposes the growth of collector current to its maximum 2 lb C
value. Therefore, the current I grows slowly. As a result of this, the
magnetic flux linked with L’ changes. Since coil L’ is inductively B
coupled with coil L, so magnetic flux linked with coil L also changes. 3 E n-p-n
Due to the change in magnetic flux linked with coil L, an induced Output LC
e.m.f is set up across the coil L’. As a result of this positive feedback,
4
collector current IC is further increased. The process of increasing
the collector current continued till the magnetic flux linked with
the coil L’ becomes maximum (i.e., constant). At this stage, collector Switch
current IC becomes maximum (transistor becomes saturated) and Figure 26.60
the change in magnetic flux linked with the coil L’ ceases. As a result
of this, there is also no change in the magnetic flux linked with the coil L and hence the induced e.m.f. across the
coil L becomes zero (transistor is in the cut off region). The capacitor C starts discharging through the coil L.
Now, the forward bias of emitter-base junction decreases. Hence the emitter current IE and consequently collector
current IC begin to decrease. As the collector current IC decreases,
again magnetic flux linked with the coil L’ decreases. Consequently, the
A C
lC
magnetic flux linked with coil L’ also decrease. Hence, induced e.m.f. is
set up across the coil L’ but now in opposite direction. The forward bias Transistor D t
across emitter-base junction is further decreased and hence the emitter O cut off
current IE and collector current IC is further decreased. The process
continues till the collector becomes zero. At this stage, capacitor gets
e
discharged through coil L but now in the opposite direction. t
Now the emitter current and hence collector current increases in the
opposite direction. This process repeats and the collector current Figure 26.61
oscillates between maximum and minimum values.
1
The frequency of oscillation is given by v =
2π LC
2 6 . 3 6 | Semiconductor and Communication System

16. TRANSISTOR AS A SWITCH


We have already discussed that a crystal diode behaves like a switch. When the diode is forward biased, it conducts
current easily and behaves like a closed switch. However, when diode is reverse biased, it practically conducts no
current and behaves like an open switch. A transistor can also be used as a switch by making emitter-base junction
either reverse biased or sufficiently forward biased. Figs. 26.62 (a) and (b) illustrate the operation of a transistor as
a switch.
+VCC +VCC +VCC +VCC

RC lC=0 RC RC lC(sat) RC lC(sat)


RB C RB C
V +VBB
E lB E

(I) (II) (I) (II)


Open switch
Closed switch
(a) (b)
Figure 26.62

(a) Transistor as open switch. In Fig. 26.62 (a), the base-emitter junction is not forward biased. Therefore, base

( )
current IB=0. As a result, collector current IC = β I is also zero. Under this condition, the transistor behaves
B
as an open switch. In other words, there is an open between the collector and emitter as indicated in Fig.
26.62(a).
(b) Transistor as closed switch. In Fig. 26.62.(b), the base-emitter junction is sufficiently forward biased so that
base current IB is made large enough to cause maximum collector current to flow. This maximum value of
collector current is called saturation current IC(sad). Under this collector and emitter as shown in Fig. 26.62.

Illustration 26: An LC oscillation has a tank circuit with L1= 58.6 µ H and C1=300 PF. Calculate the frequency of
oscillations.  (JEE MAIN)
1
Sol: The frequency of oscillation is given by f = .
2π L1C1

L1 = 58.6 × 10−6 H;C1 =


58.6H = 300 × 10−12 F

1 1
The frequency of oscillations f is given
= by; f = Hz
2π L1C1 2π 58.6 × 10−6 × 300 × 10−12

=1199 × 103 Hz =1199 kHz

lC
Illustration 27: In Fig. 26.63, the VBB supply can RB B C RC +
be varied from 0V to 5.0 V. The Si transistor has vO


lB E -
βdc = 250 and RB = 100kΩ, R C = 1kΩ, VCC = 5.0 V . Assume that vi VCC
lE


when the transistor is saturated, VCE=0V and VBE =0.8V. Calculate VBB
(a) the minimum base current for which the transistor will reach
saturation. Hence (b) determine Vi when the transistor is switched
on. (c) Find the range of Vi for which the transistor is switched off
and switched on.  (JEE ADVANCED) Figure 26.63
P hysi cs | 26.37

VCC − VCE
Sol: When the transistor reach saturation the collector current is maximum and given by IC = . The base
RC
IC
current is IB = where βdc is the DC gain of the circuit. The value of Vi is obtained by applying Kirchhoff’s law to
βdc
the circuit.
(a) When the transistor is saturated, the collector current is maximum at saturation, it is given that VCE=0V; VBE=0.8
V.
VCC − VCE (5.0 − 0 ) V
Now, V=CC V CE + I R
C C ∴= I C = = 5.0 mA
RC 1kΩ
IC 5.0mA
Corresponding to saturation current IC=5.0 mA, the base current IB is I=
B = = 20 µA
βdc 250
(b) The input voltage at which the transistor goes into saturation is
V=
i IBRB + VBE= 20µA × 100 kΩ + 0.8= 2.8V
(c) We know that for Si transistor; the transistor will remain off if Vi is less than 0.6 V. Therefore, between 0Vand 0.6
V, the transistor will be in the switched off state. However, between 2.8V and 5.0 V, the transistor will be in switched
on state.

17. LOGIC GATES


A logic gate is a digital circuit which is based on certain logical relationship between the input and the output
voltage of the circuit.
The logic gates are built using the semiconductor diodes and transistors. Each logic gate is represented by its
characteristic symbol. The operation of a logic gate is indicated in a table, known as truth table. This table contains
all possible combination of inputs and the corresponding outputs. A logic gate is also represented by a Boolean
algebraic expression. Boolean algebra is a method of writing equation showing how an output depends upon the
combination of inputs. Boolean algebra was invented by George Boole.
Basic logic gates: (1) OR gate, (2) AND gate, and (3) NOT gate
The OR gate: The output and an OR gate attains the state 1 if one or more inputs attain the state
(a) Logic symbol of OR gate
The Boolean expression of OR gate is Y=A+B, read as Y equals A or B.
Truth table of a two input OR gate

Input Output
AB Y
00 0 A
Y = A+B
B
01 1
Figure 26.64
10 1

11 1

(b) The AND gate: The output of an AND gate attains the state 1 if and only if all the inputs are in state

A
Logic symbol of AND gate B
Y = AB

Figure 26.65
2 6 . 3 8 | Semiconductor and Communication System

The Boolean expression of AND gate is Y=A.B it is read Y equal A and B


Truth table of a two input AND gate

Input Output
AB Y
00 0

01 0

10 0

11 1

(c) The NOT gate: The output of a NOT gate attains the state 1 if and only input does not attains the state 1.
Logic Symbol of NOT gate:

The Boolean expression is Y= A , read as Y equal Not A. A Y

Truth table of NOT gate Figure 26.66

Input Output
A B

0 1

1 0

Combination of Gates: The three basic gates (OR, AND and NOT) when connected in various combinations give
us logic gates such as NAND gates, which are the universal building blocks of digital circuits.

(a) The NAND gate: Logic symbol of NAND gate  A


The Boolean expression on NAND gate is Y= AB Y
Truth table of a NAND gate B

Input Output Figure 26.67

AB Y
00 1

01 1

10 1

11 0

A
(b) The NOR gate: Logic symbol of NOR gate Y = A+B
B
The Boolean expression of NOR gate is Y= A + B Figure 26.68
Truth table of a NOR gate

Input Output
AB Y
P hysi cs | 26.39

Input Output
00 1

01 0

10 0

11 0

Universal gates: The NAND or NOR gate is the universal building block of all digital circuits. Repeated use of
NAND gates (or NOR gates) gives other gates. Therefore, any digital system can be achieved entirely from NAND
or NOR gates. We shall show how the repeated use of NAND (and NOR) gates will gives use different gates.
(a) The NOT gate from a NAND gates: When all the input of a NAND gate are connected together, as shown
in the figure, we obtain a NOT gate
A
Truth table of signal-input gate Y
B

Input Output Figure 26.69


Y
A B=(A)
1
0 0
0
1 1

(b) The AND gate from a NAND gates: If a NAND gate is A Y’


Y
followed by a NOT gate (i.e., a single input NAND gate), the B
resulting circuits is an AND gates as shown in figure and
truth the table given show how an AND gates has been Figure 26.70
obtained from NAND gates.
Truth table

A B Y’ Y
0 0 1 0

0 1 1 0

1 0 1 0

1 1 0 1

(c) The OR gate from NAND gates: If we invert the A


and B and then apply them to the NAND gate, the A
A
resulting circuit is an OR gate.
Y
Truth table
B
B
A B A B Y
0 0 1 1 0
Figure 26.71
0 1 1 1 0 1
1
0 0 1 1

1 0 0 1
A y
(d) The NOT gate from NOR gate: When all the inputs of a NOR gate are connected
together as shown in the figure, we obtain a NOR gate. B
Figure 26.72
2 6 . 4 0 | Semiconductor and Communication System

(e) The AND gate from NOR gates: If we invert A and B and
then apply them to the NOR gate, the resulting circuit is an A
AND gate. Y

B
(f) The OR gate from NOR gate: If a NOR gate is followed by
a single input NOR gate (NOT gate), the resulting circuit is an Figure 26.73
OR gate.

A
Y
XOR AND XNOR gate: (i) The exclusive-OR gate (XOR gate) B

The output of a two-input XOR gate attains the state 1 if one and input Figure 26.74
attains the state 1.

Logic symbol of XOR gate: A


Y
The Boolean expression of XOR gate is Y = A + B + AB or Y = A ⊕ B B

Figure 26.75
Truth table of a XOR gate

Input Output
AB Y
00 0

01 1

10 1

11 0

Exclusive: NOR gate (XNOR gate). The output is in state 1 when it’s both input are the same that is, both 0 or
both 1. A
Y
(a) Logic symbol of XNOR gate: The Boolean expression of XNOR B

Figure 26.76
gate is Y=A.B+ Y= A.B or Y= A ⊕ B

Truth table of a XNOR gate

Input Output
A B Y
00 1

01 0

10 0

11 1

Law of Boolean algebra: Basic OR, AND, and NOT operations are given below:
OR AND NOT
A+0=A A.0=0 A+ A =1
A+1=1 A.1=A A. A =0
A+A=A A. A=A A. A=A
Boolean algebra obeys commutative, associative and distributive laws as given below:
P hysi cs | 26.41

(i) Commutative laws: AB=BA


(ii) Associative laws: A+ (B+C) = (A+B) + C; A. (B.C)=(A.B).C
(iii) Distributive laws: A (B+C) = AB+AC

Some other useful identities:


(i) A + AB = A;
(ii) A.(A + B) = A.
(iii) A+ AB = A+B
(iv) A.( A +B) = AB
(v) A + BC = AB + AC
(vi) ( A + B)(A + B) = B
(b) De Morgan’s Theorem: First theorem A + B = A.B . Second theorem: = A.B = A + B

PLANCESS CONCEPTS

The NAND gate is a universal gate because its repeated use can produce other logic gates.
Ankit Rathore (JEE Advanced 2013, AIR 158)

Illustration 28: The output of an OR gate is connected to both the inputs of a NAND gate. Draw the logic circuit
of this combination of gates and write its truth table. (JEE MAIN)

Sol: When output of OR gate is connected to the input of NAND gate the circuit behaves as NOR gate.
The logic circuit of the combination of the two gates is shown in Fig.26.77. It is clear that: Y ' =
A + B and Y =
A +B
This means that NOR gate is formed. The truth table of the given logic circuit is given below:

A B Y’=A+B Y= A •B A
y’
0 0 0 1
y
B
1 0 1 0
Figure 26.77
0 1 1 0

1 1 1 0

Illustration 29: The output of an OR gate is connected to both the inputs of a NOR gate. Draw the logic circuit of
this combination of gates and write its truth table. (JEE MAIN)

Sol: When output of OR gate is connected to the input of NOR gate the circuit behaves as NAND gate.
The logic circuit of the combination of two gates is shown is Fig. 26.78. It is clear that:

Y ' = A + B and Y = (A + B) + (A + B) = A + B

The truth table of the given logic circuit is shown below:


2 6 . 4 2 | Semiconductor and Communication System

A B Y’ Y A y’ = A+B y’ = A+B
0 0 0 1 or nor
B
1 0 1 0
0 1 1 0 Figure 26.78
1 1 1 0

Illustration 30: Identify the logic gates marked X, Y in Fig. 26.79. Write down the output at y, when A=1, B=1 and
A=0, B=0.  (JEE ADVANCED)

A B Y’ Y
A
1 1 0 1 X Y y
B y’
0 0 1 0

Figure 26.79
Sol: The gate X is AND gate while the gate Y is NOT gate.
The logic gate marked as X is NAND gate while the gate marked as Y is NOT gate. It is clear that:

Y ' = A.B and Y = y ′

Therefore, the output at y will be as shown in the table.

COMMUNICATION SYSTEMS

1. INTRODUCTION
In the most fundamental sense communication involves the transmission of information from one point to another
through a series of processes. In the early 1900s radio communications – the transmission and reception of voice
and music through the air was probably the only application of electronics of any significance. The telephones at
our hands, the radios and televisions in our living rooms, the computer terminals in our offices and homes are all
capable of providing communications from every corner of the earth. In this chapter, we shall focus our attention
on the principles of communication.

2. BASIC ELEMENTS OF A COMMUNICATION SYSTEM


Irrespective of the form of communication system being considered, there are three basic elements of every
communication system viz, (i) Transmitter (ii) Communication channel (iii) Receiver
Figure shows the block diagram of the basic elements of a communication system.
Communication system

Information Message Transmitted Received Message User


signal Transmitter Channel Receiver
source signal signal signal of information

Noise

Figure 26.80
P hysi cs | 26.43

Generally, the transmitter is located at one place and the receiver at some other place. The communication channel
is links the transmitter and the receiver.
(a) Transmitter. The function of the transmitter is to transform the message or information (e.g. music, speech,
picture, written message etc.) into a suitable form and transmit it over the communication channel. Generally,
the information is not electrical in nature. The transmitter first converts the message into equivalent electrical
variations. It is then called signal. The signal modulates a high frequency wave called carrier wave and the
resultant waves are called modulated waves. The actual method of modulation varies from one communication
system to another. After modulation the modulated waves are transmitted over the communication channel.
(b) Communication channel. It is the medium by which the modulated waves are transmitted from the transmitter
to the receiver. The communication channel or transmission medium may be free space, transmission lines or
optical fibers. For example, in case of radio and TV transmission, the communication channel is the free space.
However, in case of telegraphy and telephony, communication channel is transmission lines.
In the process of transmission, signals are contaminated by noise signals. This is called channel noise. Noise
is unwanted energy usually of random character generated by numerous natural or man-made events e.g.
lightning, turning on or off electrical equipment etc.
(c) Receiver. The function of the receiver is to receive the modulated waves transmitted by the transmitter and
to do demodulation or detection or decoding. In this process, the original signal is separated from the carrier.
This process is called demodulation or detection and is the reverse of the modulation process done in the
transmitter. The recovered signal is then utilized as the situation demands.
Basic modes of communication: There are two basic modes of communication viz.
(i) Point-to-Point communication mode
(ii) Broadcast mode
(i) In Point-to-point communication mode the message is transmitted over a link between a single transmitter
and single receiver. Conversation between two persons through a telephone is an example of point-to-point
communication.
(ii) In broadcast mode (or point-to-many points communication) there is a single transmitter and a large number
of receivers. Radio broadcasting and television telecast are the examples of this mode of communication.

3. COMMONLY USED TERMS IN ELECTRONIC COMMUNICATION SYSTEMS


In electronic communication system, the following terms are frequently used:
(a) Transducer. A device that converts one form of energy into another form of energy is called a transducer. For
example, a microphone converts sound energy into electrical energy. Therefore, microphone is a transducer.
Similarly, a loudspeaker is a transducer because it converts electrical energy into sound energy.
(b) Signal. The information converted into electrical form that is suitable for transmission is called a signal.
For example, in a radio station, music, speech etc. are converted into electrical form by a microphone for
transmission into space. This electrical form of sound (music, speech etc.) is the signal. The signal can be of
two type’s viz. (i) analog signal (ii) digital signal
(i) Analog signal. A continuously varying signal (voltage or current) is called an analog signal. For example,
an alternating voltage varying sinusoidally is an analog signal (see figure). If such an analog signal is
applied to the input of a transistor amplifier, the output voltage will also vary sinusoidally. This is the
analog operation i.e., the output voltage can have an infinite number of values. Due to many-valued
output, the analog operation is less reliable.
(ii) Digital signal. A signal (voltage or current) that can have two discrete values is called a digital signal.
For example, a square wave is a digital signal (see figure). It is because this signal has only two values viz,
+5V and 0V and no other value. These values are labeled as High and Low. The high voltage is +5V and
the low voltage is 0V. If proper digital signal is applied to the input of a transistor the transistor can be
2 6 . 4 4 | Semiconductor and Communication System

driven between cut off and v v


saturation. In other words,
the transistor will have
two-state operations i.e.
output is either low or high. t t
Since digital operation has 0 0
only two states (i.e., ON or
OFF), it is far more reliable
than many-valued analog
operation. It is because with Figure 26.81
two states operation all the
signals are easily recognized as either low or high.
(c) Noise. The unwanted signal is called a noise. The noise is undesirable because it disturbs the transmission and
processing of signals in a communication system. The source generating the noise may be located inside or
outside the system. Efforts should be made to minimize the noise level in a communication system.
(d) Transmitter. An electronic system that broadcasts modulated electromagnetic signals toward one or more
distant receivers is called a transmitter. In a transmitter the signal is processed to produce modulated waves.
These modulated waves contain the signal and are sent to the receiver through the channel (e.g. space).
(e) Receiver. Any electronic system that enables the desired modulated wave to be separated from all other
modulated waves coming into the antenna is called a receiver. In a receiver, the signal is extracted from the
modulated wave.
(f) Attenuation. The loss of strength of the signal while propagating through the medium is known as attenuation.
It occurs because the medium distorts, reflected and refracts the signal as it passes through it.
(g) Amplification. The process of raising the strength of a signal is called amplification and it is done by an
electronic circuit called amplifier. Amplification is necessary to compensate for the attenuation of the signal
in a communication system. The energy required for additional signal strength is obtained from a d.c. power
source.
(h) Range. The range of a signal is the distance between the source and the destination up to which the signal
can be received in sufficient strength.
(i) Bandwidth. The bandwidth of an electronic circuit is the range of frequencies over which it operates nicely.
Fox example, suppose an amplifier has a bandwidth of 300 Hz to 3100Hz. It means that the amplifier will
amplify the signals nicely (i.e. with least distortion) in this frequency range. For signals outside this range, the
amplification will be drastically reduced.
( j) Modulation. The signals in communication system (e.g. music, speech etc.) are low frequency signal and
cannot be transmitted to large distances. In order to transmit the signal to large distances, it is superimposed
on a high frequency wave (called carrier wave). This process is called modulation. Modulation is done at the
transmitter and is an important part of communication system.
(k) Demodulation. The process of extracting signal from the modulated wave is called demodulation and is
carried out in the receiver. This is reverse process of modulation.
(l) Repeaters. Repeaters Repeaters
are signal boosters
installed at suitable
locations in between
the transmitter and
the receiver. Each
repeater receives the
transmitted signal, Transmitting Receiving
amplifies the signal and antenna Earth
antenna
transmits the amplified
signal to the next
Figure 26.82
P hysi cs | 26.45

repeater (see figure). Obviously, a repeater is a combination of receiver, amplifier and transmitter. Repeaters
are used to extend the range of a communication system. A communication satellite is essentially a repeater
station in space.

4. BANDWIDTH OF SIGNALS
In electronic communication, message signals converted in the electrical form are transferred from one point to
another point. These message signals are of two types viz.
1. Analog signals 2. Digital signals
(a) Bandwidth of analog signals. An analog signal is that in which the voltage or current varies continuously
with time. In analog communication, the information or message to be transmitted is generally in continuous
waveform. The range of frequencies which are necessary for satisfactory transmission of information or
message contained in the analog signal is called Bandwidth of the analog signal.
Examples of analog signals are speech, music, sound produced by vibrating strings, picture (video) signals etc.
These analog signals are converted into electrical form with suitable transducers and then transmitted to the
required destination.
Different types of analog signals require different bandwidths.
(i) Bandwidth of speech signals. Speech signals contain frequencies between 300 Hz. And 3100 Hz.
Therefore, speech signals require bandwidth=3100 Hz-300 Hz=2800 Hz.
(ii) Bandwidth of music signals. The audio range of frequencies produced by musical instruments is from
20 Hz to 20 kHz. Therefore, music signals require a bandwidth of about 20 kHz (20 kHz-20 Hz).
(iii) Bandwidth of video signals. For transmission of pictures, the video signals require a bandwidth of about
4.2 MHz. Since a TV signal contains both audio and video signals, it is usually allocated a bandwidth of
6 MHz for the transmission of TV signals.
(b) Bandwidth of digital signals. A digital signal
Fundamental wave
is that in which voltage or current can have only
Voltage/Current

Harmonic wave
two discrete values. Therefore, a digital signal
is in the form of rectangular / square waves
or pulses. Each pulse has two levels of voltage
or current represented by 0 and 1. Examples
of digital signals are: letters printed in a book,
out-put of digital computer etc. 0
Time

Theoretically infinite bandwidth is required for


digital signals. This is illustrated in figure. We
have seen that digital signals are in the form
of rectangular waves. It can be shown that a
rectangular waves. It can be shown that a Figure 26.83
rectangular waves can be considered as the
superposition of a large number of sinusoidal waves of frequencies f0, 2 f0, 3 f0, 4 f0, …..n f0 where n is an integer
ranging from 1 to infinity. Here f0 = 1/t0 is called the fundamental frequency and 2 f0, 3 f0, 4 f0,.… are called
second harmonic, third harmonic, fourth harmonic, …. Therefore, the bandwidth of a digital signal is infinite.
However, for all practical purpose, higher harmonics (e.g. 4 f0, 5 f0 …..) can be neglected because contribution
of higher harmonics to shape of the wave is very small. Therefore, if the available bandwidth for a digital signal
is large enough to accommodate a few harmonics the information contained in the digital signal is not lost
and the rectangular digital signal is more or less recovered.
2 6 . 4 6 | Semiconductor and Communication System

5. BANDWIDTH OF TRANSMISSION MEDIUM


Transmitter and receivers have some transmission medium between them so that message or information may
be transferred from the transmitter to the receiver. Some commonly used transmission media are coaxial cables,
optical fibers, free space etc. Different types of transmission media are suitable for different bandwidths.
(a) Coaxial cables. These are used for signals below 18 GHz. The commonly used coaxial cables have a bandwidth
of 750 MHz
(b) Optical fibers. Optical fibers are suitable for microwaves and UV waves. The bandwidth of an optical fiber is
about 1011 Hz.
(c) Free space. In radio communication, free space acts as the transmission medium. The frequency range of
space communication is from 105 Hz to 109 Hz. This frequency range is further subdivided and allocated for
different services.

Table 26.4: Some important wireless communication frequency bands

Service Frequency bands Comments


Standard AM broadcast 540-1600 kHz
FM broadcast 88-108 MHz
Television 54-72 MHz VHF (very high frequencies)
76-88 MHz TV
174-216 MHz UHF (ultra high frequencies)
420-890 MHz TV
Cellular Mobile Radio 896-901 MHz Mobile to base station
840-935 MHZ Base station to mobile
Satellite Communication 5.925-6.425 GHz Uplink
3.7-4.2 GHz Downlink

6. PROPAGATION OF EM WAVES IN ATMOSPHERE


In radio communication, an antenna at the transmitter radiates the electromagnetic waves which travel through the
space and reach the receiving antenna at the other end. Several factors influence the propagation of electromagnetic
waves and the path they follow.
(a) As the electromagnetic waves travel away from the transmitter, their power goes on decreasing.
(b) The electromagnetic waves are little affected by the surrounding atmosphere, rain, snow etc and are able to
penetrate non-metallic objects easily.
(c) The electromagnetic waves are stopped dead by metals or fine mesh screens.
The earth’s atmosphere plays an important role in the propagation of electromagnetic waves. Therefore, it is
desirable to discuss the composition of earth’s atmosphere.

6.1 Earth’s Atmosphere


The gaseous envelope around the earth is called atmosphere. It extends up to a height of about 400km above
the surface of the earth. The earth’s atmosphere is mainly composed of nitrogen (78%) and oxygen (21%). It also
contains minute quantities of carbon dioxide, neon, water vapors, dust particles, etc. the density of air (ρ) goes on
decreasing as we go up. The earth’s atmosphere has been divided into the following regions (see figure).
P hysi cs | 26.47

(a) Troposphere: This region is up to a height


of 12km from the earth’s surface. It is in
Appleton
this region that clouds are formed. The

Ionosphere
layer
temperature in troposphere decreases
with height at the rate of about 6.50C per
kilometer to a value of about -500C at its
Kennelly
upper boundary. The density of air falls
heaviside
from ρ to ρ /10.
layer 400 km
(b) Stratosphere: Above the troposphere
lies the stratosphere. This region is from Thermosphere
12km to 50km from the earth’s surface. An
important part of stratosphere is the ozone Mesosphere
layer which extends from 30km to 50km
Ozone layer 80 km
from the earth surface. The ozone layer
contains ozone in abundance. It absorbs Stratosphere 50 km
most of the ultraviolet radiation coming
from the sun. The temperature falls from Troposphere 12 km
280 K to 220 K and the density of air falls
Earth surface
from ρ /10 to ρ /1000.
(c) Mesosphere: This region is from 50km to Figure 26.84
80km from the earth’s surface. Temperature
falls from 220 K to 180 K. Density of air falls from ρ /1000 to ρ /105.
(d) Ionosphere: This region extends from 80 km to 400 km from the earth’s surface. In this region, the constituent
gases are ionized by ultraviolet radiation and X-rays from the sun. There are main layers viz, Heavy side layer
and Appleton layer, in this region. The heights of these layers vary with the season and the day. Further, the
layers are not fixed, but are irregular and of varying thickness.
Layers of Ionosphere: The ionosphere plays an important role Outer atmosphere
in the propagation of radio waves in space communication.
Because of the variation of the composition of air, there
are several regions of varying ionization density within the
ionosphere, dividing the ionosphere into several layers. The F2 layer
important layers of ionosphere are D, E, F1 and F2 layers as shown
in figure. The ionization density (i.e. number of ions or electrons F1 Layer
per unit volume) of each layer varies with time of the day, season, km
220
altitude etc. km E1 Layer
110 D Layer
(a) The lowest layer, called D layer, exists only in the day time
at an altitude of 50 to 90 km above the earth’s surface. Troposphere
Ionization in this region is relatively weak and does not Earth
affect the direction of travel of radio waves. Figure 26.85
(b) The next layer, the E layer, is in a region of about 90-140 km
above the surface of earth. It has a maximum density at noon but is only weakly ionized at night.
(c) The last layer, the F layer, is quite variable. At night, it exists as a single layer in a region of about 140-400 km
above the earth’s surface. However in day time, it splits into two layers F1 and F2 as shown in figure.
When radio waves from a transmitter are directed towards the ionosphere, then radio waves in the frequency
range 2 MHz to 40 MHz are reflected back to earth by the ionosphere. However, radio waves of frequencies greater
than 40 MHz are not reflected back to earth by the ionosphere; they penetrate into ionosphere and escape. Thus
ionosphere acts as a reflector for certain types of radio waves and helps in the long distance transmission of radio
waves.
2 6 . 4 8 | Semiconductor and Communication System

6.2 Classification of Radio Waves


In radio communication, free space acts as the transmission medium. The radio waves have a very wide frequency
range from 500 kHz to 1000 MHz. This frequency range is divided into various categories as shown in the table
below.

Frequency Band Frequency Range Wavelength Range Typical Uses


Very low frequency (v.l.f.) 10-30 kHz 30,000-10,000 m Long-distance point-to-point
communication
Low frequency (l.f.) 30-300 kHz 10,000-1,000 m Marine, navigational aids
Medium frequency (m.f.) 300-3,000 kHz 1,000-100 m Broadcasting, marine
High frequency (h.f.) 3-30 MHz 100-10 m Communication of all types
Very high frequency (v.h.f.) 30-300 MHZ 10-1 m Television, f.m. broadcasting radar, air
navigation, short-wave broadcasting
Ultra-high frequency (u.h.f.) 300-3,000 MHz 1m-10 cm Radar, microwave relays, short-distance
communication
Super-high frequency (s.h.f.) 3,000-30,000 MHz 10-1 cm Radar, radio relay, navigation,
experimental
Extremely high frequency (e.h.f.) 30,000-300,000 MHz 1-0.1 cm Experimental

7. SPACE COMMUNICATION
The phenomenon of sending, receiving and processing information through space is called space communication.
In space communication, the signal is carried by high frequency electromagnetic wave (called carried wave) from
the transmitter to the receiver in free space. Since no wires are used, it is also called wireless communication.
The frequencies used in space communication lie in the range 104 Hz to 1011 Hz. Radio, television and satellite
communication fall under this category.
Types of radio wave propagation. In space communication, the radio waves travel from the transmitting antenna
to the receiving antenna in free space. Depending upon the frequency of radio waves, the distance between the
transmitter and receiving antennas and the path (or paths) by which radio waves reach the receiving antenna, the
radio wave propagation can be carried out in the following three ways.
(a) Ground or surface wave propagation
(b) Sky wave or ionosphere wave propagation
(c) Space wave or direct wave or tropospheric propagation.

7.1 Ground Wave Propagation


When the radio waves from the transmitting antenna propagate along the surface of earth so as to reach the
receiving antenna, it is called ground wave propagation or surface wave propagation.
In ground wave propagation, the radio waves travel along the surface of earth as shown in figure. This mode
of propagation is possible only when the transmitting and receiving antennas are close to the surface of the
earth. As the ground wave glides over the surface of
earth, it induces current in the earth. This gives rise to Transmitting Receiving
resistance losses and dielectric losses in the ground. antenna antenna
The energy required to supply these losses must
come from the ground wave. Therefore, the energy of Earth
a ground wave decreases as it passes over the earth
surface. The ground losses increase rapidly with the
Figure 26.86
P hysi cs | 26.49

increase in frequency. Moreover, ground wave is also attenuated due to diffraction effect. As the wave propagates
over the earth, it gradually tilts over. At some appreciable distance from the transmitting antenna, the wave lies
down and dies.
We have seen that loss of power of a ground wave increases rapidly with the increase in frequency as well as with
the increase in distance from the transmitting antenna. Therefore, ground wave propagation is limited to low
frequency signals (500 kHz to 1500 kHz) and for short range communication. It cannot be used for high frequency
and long range transmission.
(a) Below 500 kHz, reliable communication can be obtained over distance upto 1500 km by ground waves alone.
(b) Amplitude modulated radio waves in the medium frequency band are transmitted primarily via ground waves.
Advantages: Ground wave propagation has the following advantages:
(a) Ground wave propagation has excellent reliability.
(b) Reception is not affected by changing atmospheric conditions :
(c) Given enough transmitting power, ground wave communication can be maintained with any place in the
world.
Disadvantages: Ground wave propagation has the following disadvantages:
(a) Ground wave propagation requires high transmitting power.
(b) Ground losses increase very rapidly with the increase in frequency of the signal.
(c) Ground losses very with surface material and composition.

7.2 Sky Wave Propagation


When the radio waves from the transmitting antenna reach the receiving antenna after reflection from the
ionosphere, it is called Sky wave propagation or Ionospheric propagation.
We have already discussed that ionosphere has layers viz D, E, F1 and F2 layers of varying ionization density (i.e.
number of ions or electrons per unit volume). These layers of ionosphere act as “radio mirror” for certain radio
frequencies. It has been found that if radio waves in the frequency range 2 MHz to 30 MHz are directed towards
the ionosphere, these waves are reflected back to earth by the various layers of the ionosphere as shown in figure.
If the frequency of the radio waves is more than 30 MHz, these are not reflected back to earth from the ionosphere
and penetrate into the ionosphere and escape. For this reason, we use sky wave escape. For this reason, we use sky
wave propagation for radio waves lying in the frequency range
2 MHz to 30 MHz. The sky wave propagation is also known as
ionosphere propagation because sky waves reach the receiver
after reflection from the ionosphere. Ionospheric

The sky wave propagation is quite unreliable. It is because


whether or not a radio wave is reflected back to earth by the
ionosphere depends upon several factors including
Sky
(a) Frequency of radio wave
R1 R2 waves
(b) Ionization density of ionosphere and R3
(c) The angel of incidence at which the radio wave enters the
ionosphere. Earth
Yet it is primary means of around the world short-wave Figure 26.87
communication.
(a) Critical frequency. As the frequency of sky waves increases, the ionosphere becomes progressively less
effective in reflecting the waves back to earth. At a certain maximum frequency of radio wave, the wave is not
at all reflected back to earth. The highest frequency above which the ionosphere no longer returns the sky
wave back to earth when transmitted in vertical direction is called critical frequency.
2 6 . 5 0 | Semiconductor and Communication System

Since the critical frequency depends on the density of ionization, it will clearly vary with the time of day
and season of the year. Furthermore, it is possible for a particular frequency to pierce the E layer but still be
returned from the F layer because it has higher density of ionization. Of course, a still higher frequency will
pierce both layers and be lost. The value of critical frequency (ƒc) is given by ; ƒc = 9 (Nmax ) 1/ 2 Where Nmax =
maximum electron density of ionosphere.
(b) Critical angle. As the vertical angle of the sky wave w.r.t. earth is increased, the ionosphere layers are no
longer capable of reflecting sky waves back to earth. For a given frequency, the vertical angle above which the
sky wave no longer returns to earth but travels outward into space is called critical angle.
Sky waves at or above critical angle may be refracted (bent) by ionosphere but they are not reflected back to
earth. The critical angle primarily depends on density of ionization and on the frequency of the wave.
(c) Maximum usable frequency (MUF). It is defined as the highest frequency of the radio waves which when
sent at a certain angle towards the given layer of ionosphere gets reflected from that layer and returns to
earth. It is given by ;
ƒc
Maximum usable frequency, MUF
= = ƒc sec θ
cos θ
Where θ = Angle between normal and direction of incident waves.
Since the maximum usable frequency (MUF) depends on the density and height of ionosphere layers, it will
vary from hour to hour, from day to day and from location to location.
(d) Skip distance. The distance between the transmitting aerial and the point where the sky wave is first received
after returning to earth is called skip distance.
(e) Fading. When signals are received via sky waves, it often happens that the signal strength will increase and
decrease periodically. In certain cases, the signal may be lost completely or may be drowned in the noise level.
It is called fading. Fading is caused due to the following reasons:
(i) Multiple-path reception. For example, consider a receiving antenna that receives ground waves as well
as sky waves. The signal strength will be the resultant of the two. This interference causes fading of the
signal.
(ii) Ionospheric conditions. As the conditions in the ionosphere change, the phasing of sky wave will shift,
and the resultant strength of the signal will also vary. This condition leads to fading of the signal.
(iii) Receiving antenna at the edge of skip distance. Fading also occurs when the receiving antenna is
located at the edge of the skip distance. A slight change in the conditions of ionosphere may place the
receiving antenna inside or beyond the skip distance. As a result, fading of signal may take place.

7.3 Space Wave Propagation (Line of Sight Propagation)


At frequencies above 30 MHz radio transmission Transmitting
cannot be carried out by ground waves or sky waves. antenna Straight-line path
It is because ground waves are quickly attenuated
at such high frequencies and at frequencies above
30 MHz, the ionosphere is unable to reflect the sky
waves back to earth. Therefore, we use direct waves
(also called space waves) which travel directly in Geometrical Receiving
straight lines from the transmitting antenna to the antenna
straight-line distance
receiving antenna as shown in figure. This mode
of propagation is called space wave propagation
or line of sight (LOS) propagation or tropospheric
propagation.
When the radio waves from the transiting antenna Figure 26.88
travelling in straight line directly reach the receiving
antenna, it is called space wave propagation.
P hysi cs | 26.51

Wave propagation or line of Sight Propagation.


As the frequency increases, the radio waves tend to travel more and more in straight lines. Therefore, the receiving
antenna must receive the signal directly from the transmitting antenna. The television frequencies lie in the range
100 MHz to 200 MHz. Therefore, reception of TV signals is possible only if the receiving antenna directly intercepts
the signal. Similarly, radar, microwave relays and many other services solely depend on space wave propagation.
The range of space wave propagation is limited by
(a) The line of sight distance i.e. the distance at which the transmitting and receiving antennas can see each other.
(b) The curvature of earth.
The space waves of direct waves travel essentially Transmitting
in a line of sight path. Due to curvature of antenna 1
earth, line of sight distance will depend upon 2 4
the antenna height. Therefore, the maximum
3
range of communication through space waves is Earth
determined by the height of the transmitting and
receiving antennas. This is illustrated in figure.
At receiving location 1, the height of the antenna
is within the line of sight distance. For the same Figure 26.89
antenna height, receiving location 2 would be the
maximum range of communication for direct waves. However, if we increase the height of the receiving antenna,
the communication range can be increased to reach the location 4.

7.4 Range of TV Transmission


The TV signals are in the 100-200 MHz range. Therefore, transmission of such signals via ground waves or sky
waves is not possible. In such situations, we use line of sight transmission i.e., TV signals are transmitted by directed
waves.
Consider a TV transmitting antenna OP of height h located at point O on the surface P
of earth as shown in figure. When TV broadcast is made, the signal can reach up
to tangent point A and B on the surface of earth. There will be no reception of the
signal beyond points A and B. therefore, distance h
On the earth surface is the range of TV transmission (d) and the height (h) of the
transmitting antenna.
B O
If C is the center of earth and R (=AC) is its radius, the from right angled triangle 90o A
PAC, we have, R
R
(PC ) = (PA ) + ( AC ) (h + R)2 = (PA ) + R 2
2 2 2 2
or
Since the height (h) of the transmitting antenna is very small as compared to the C
radius (R) of the earth, PA = PB  d. Figure 26.90
2 2 2 2 2
∴ (h + R) = d + R or d = h + 2Rh
2
=∴d 2Rh =
or d 2Rh
Note that the range of TV transmission depends upon the height of the transmitting antenna. The greater the
height of the transmitting antenna, the larger is the range of TV transmission. For this reason, TV broadcasts are
made from tall transmitting antennas.

Area covered by TV signal πd2 =π (2Rh) [∴ d = 2Rh]


Population covered by TV signal = Population density x Area covered.
2 6 . 5 2 | Semiconductor and Communication System

7.5 Maximum Communication Range for Space Waves


Figure shows space wave propagation between the dM
transmitting antenna and the receiving antenna.
dr dR
Since space waves or direct waves follow the C
straight line path, they get blocked at some point C
due to the curvature of earth. Thus earth presents
a horizon to space wave propagation called radio hr Earth surface
hr
horizon. The distance dT is called radio horizon
of the transmitting antenna while the distance dR
is called radio horizon for the receiving antenna. Figure 26.91
Therefore, maximum line of sight distance dM
between the transmitting and receiving antennas is
If hT and hR are the heights of transmitting and receiving antennas respectively and R is the radius of earth, then,

dr = 2Rhr and dR = 2RhR ; ∴ dM = 2Rhr + 2RhR

Clearly, if the signal is to be received beyond the horizon, then the height of the receiving antenna must be
large enough to intercept the space or direct waves. However, space wave communication is greater than 100
km is hardly used in commercial communication. For an example, if hT and hR are 100m each, then using above
formula, dM comes out to be 71.4 km only. Sometimes, the range of TV transmission is increased by using repeater
transmitting stations.

8. MODULATION
As discussed earlier, a high carrier wave is used to carry the audio signal. The question arises how the audio signal
should be “added” to the wave. The solution lies in changing some characteristic of carrier wave in accordance with
the signal. Under such conditions, the audio signal will be contained in the resultant wave. This process is called
modulation and may be defined as under:
The process of changing some characteristic (e.g. amplitude, frequency or phase) of a carrier wave in accordance
with the intensity of the signal is known as modulation. Modulation means “to change”. In modulation, some
characteristic of carrier wave is called modulated wave or radio wave and contains the audio signal.
Need for modulation. Audio signals have a frequency range from 20 Hz to 20 kHz. These low frequency signals
cannot be transmitted directly (i.e. without modulation) into space for the following reasons:
(a) Practical antenna length. For efficient transmission and reception, the transmitting and receiving antennas
must have a length at least equal to λ / 4 where λ is the wavelength of the signal. For an audio signal of 15
kHz, the length of the antenna required will be:
c 3x108 λ 20000
Wavelength, λ= = = 20000m. ∴ Length of antenna, l= = = 5000m
ƒ 15x103 4 4
To set up a vertical antenna of this length is practically impossible. However, if a carrier wave of 1 MHz is used
to carry the signal (i.e. modulation is done), the length of the antenna comes out of be 75 m only and this
size can be easily constructed. Length of the antenna comes out to be 75m only and this size can be easily
constructed.
(b) Effective power radiated by antenna. For a linear antenna of length l, it has been found that
2
l
Power radiated, P ∝   .Therefore, for the same antenna length l, the power radiated will be large for signals
λ
of shorter wavelength or higher frequency. For good transmission, we need high powers. This requires the
transmission to be carried out at high frequencies. For this reason, we use high frequency carrier wave to carry
the audio signal.
(c) Mixing up of signals from different transmitters. All the audio signals from different transmitters have
the same frequency range i.e. 20 Hz to 20 kHz. Therefore, if the audio signals from various transmitters are
P hysi cs | 26.53

transmitted directly, they will get mixed up and there is no way to distinguish between them. This difficulty is
solved by allotting different carrier frequencies to different transmitting stations. The above discussion shows
the need for frequency transmission. For this purpose, the audio signal is superimposed on the high frequency
waves. Hence modulation permits the transmission to occur at high frequency while it simultaneously allows
the carrying of the audio signal.

8.1 Types of Modulation


As you will recall, modulation is the process of changing amplitude or frequency or phase of a carrier wave in
accordance with the intensity of the signal. The carrier wave is a sinusoidal wave and can be represented as:
=ec EC cos ( ωc t + φ ) where, ec= Instantaneous voltage of carrier wave
EC= Amplitude of carrier wave;
wc =2πfc Angular velocity at carrier frequency fc
φ = Phase angle
Depending upon whether we change EC (amplitude), (frequency) or φ (phase), modulation is of three types viz.
(a) Amplitude Modulation
(b) Frequency Modulation
(c) Phase Modulation
In India amplitude modulation is used in radio broadcasting. However, in television transmission, frequency
modulation is used for sound signal and amplitude modulation for picture signal. We shall discuss these types of
modulation in turn.

8.2 Amplitude Modulation eC


When the amplitude of high frequency carrier wave is changed in accordance with
the intensity of the signal, it is called amplitude modulation.
In amplitude modulation, only the amplitude of the carrier wave is changed t
in accordance with the intensity of the signal. However, the frequency of the 0
modulated wave remains the same i.e., carrier frequency. Figure shows the principle
Signal
of amplitude modulation. Figure shows the audio electrical signal whereas Figure
(ii) shows a carrier wave a constant amplitude. Figure (iii) shows the amplitude eC (i)
modulated (AM) wave.
Note that the amplitude of both positive and negative half-cycles of carrier wave is
changed in accordance with the signal. For instance, when the signal is increasing in
the positive sense, the amplitude of carrier wave also increases. On the other hand,
during negative half-cycle of the signal, the amplitude of carrier wave decreases. t
0
Amplitude modulation is done by electronic circuit called modulator.
Carrier
The following points are worth noting in amplitude modulation:
(ii)
(i) The amplitude of the carrier wave changes according to the intensity of the
signal.
(ii) The amplitude variation of the carrier wave is at the signal frequency fs.
(iii) The frequency of the amplitude modulated wave remains the same i.e., carrier
frequency fc. t
0
Modulation Factor. An important consideration in amplitude modulation is to
describe the depth of modulation i.e., the extent to which the amplitude of carrier
A.M. Wave
wave is changed by the signal. This is described by a factor called modulation
factor which may be defined as under:
(iii)
Figure 26.92
2 6 . 5 4 | Semiconductor and Communication System

The ratio of change of amplitude of carrier wave to the v


amplitude of normal carrier wave is called the modulation
factor ma i.e.
Es maEC
Amplitude change of carrier wave E
Modulation factor, ma = S
= vmax
Normal carrier wave (unmodulated) EC EC
vmin
Where ES is the amplitude of the signal and Ec is the t
0
amplitude of normal carrier wave. Clearly, the modulation
factor depends upon the amplitudes of the signal and carrier.
Obviously, ES = maEC

Figure shows the waveform of amplitude modulated wave.


Figure 26.93
If the maximum and minimum voltage of AM wave are
Vmax and Vmin respectively, then it is clear from figure that:

Vmax + Vmin Vmax − Vmin E V −V


=EC = ; ES ∴ Modulation factor, ma = S = max min
2 2 EC Vmax + Vmin

Modulation factor generally lies between 0 and 1.

8.2.1 Analysis of Amplitude Modulated Wave


A carrier wave may be represented by; ec EC cos ωc t
=
Where e= instantaneous voltage of carrier. EC = Amplitude of carrier
ωC = 2π fc = Angular velocity at carrier frequency fc.
In amplitude modulation, the amplitude EC
of the carrier wave is varied in accordance
with the intensity of the signal as shown in Signal
figure. Suppose the modulation factor is ma. MaEC
It means that signal produces a maximum Es EC
change of maEC in the carrier amplitude. fS
Obviously, the amplitude of signal is maEC . EC
Therefore, the signal can be represented by: Carrier A.M. Wave

=es maEC cos ωs t Figure 26.94

Where es = Instantaneous voltage of signal


maEC = Amplitude of signal
ωs = 2 fs = Angular velocity at frequency fs.

The amplitude of the carrier wave varies at signal frequency fs. Therefore, the amplitude of AM wave is given by

= EC + maEC cos ωs t= EC ( I + ma cos ωs t )

The instantaneous voltage of AM wave is: e= Amplitude x cos ωc t

= EC ( I + ma cos ωc t ) cos ωs=


t EC cos ωc t + maEC cos ωs t cos ωc t
maEC maEC *
= EC cos ωc t +
2
( 2cos ωs t cos ωc=
t ) EC cos ωc t +
2
cos ( ωc + ωs ) t + cos ( ωc − ωs ) t 
 
maEC maEC
= EC cos ωc t + cos ( ωc + ωs ) t + cos ( ωc − ωs ) t
2 2
P hysi cs | 26.55

The following points may be noted from the above equation of amplitude modulated wave:
(a) The AM wave is equivalent to the summation of three sinusoidal e
waves; one having and the third having amplitude EC and
frequency fc., the second having amplitude ma EC / 2 and
frequency fc+fs) and the third having amplitude ma EC / 2 and
frequency (fc – fs).
EC
(b) The AM wave contains three frequencies viz. fc., (fc + fs). The
first frequency is the carrier frequency. Thus, the process of
MaEC
modulation does not change the original carrier frequency but
produces two new frequencies (fc + fs) and (fc – fs) which are 2
called sideband frequencies. 0 fC - fS fC fC + fS
From trigonometry, we have the expansion formula: Frequency
2cos A cosB
= cos(A + B) + cos(A − B)
Figure 26.95
ωc ωc + ωs ωc − ωs
ƒc
= , ƒc +=
ƒs , ƒc −=
ƒs
2π 2π 2π
(c) The sum of carrier frequency and signal frequency i.e., ƒc − ƒs i.e., the difference between carrier and signal
frequencies.

8.2.2 Sideband Frequencies in AM Wave


In an amplitude modulated wave the sideband frequencies are of our interest. It is because the signal frequency ƒs
is contained in the sideband frequencies. Figure shows the frequency spectrum of an amplitude modulated wave.
The frequency components in the AM wave are shown by vertical lines. The height of each vertical line is equal
to the amplitude of the components present. It may be added here that in practical radio transmission, carrier
frequency ƒc is many times greater than signal frequency ƒs . Hence, the sideband frequencies are generally
close to the carrier frequency. It may be seen that a carrier modulated by a single frequency is equivalent to three
simultaneous signals; the carrier itself and two other steady frequencies
i.e., ƒc + ƒs and ƒc − ƒs .
Let us illustrate sideband frequencies with an example. Suppose the carrier frequency is 400 kHz and the signal
frequency is 1 kHz. The AM wave will contain three frequencies viz, 400 kHz, 401 kHz and 399 kHz. It is clear that
upper sideband frequency (401 kHz) and sideband frequency (399 kHz) are very close to the carrier frequency
(400 kHz).

( ) (
Bandwidth: In an AM wave, the bandwidth is from ƒc − ƒs to ƒc + ƒs i.e., 2 ƒs . Thus in the above example, )
bandwidth is form 399 to 401 kHz which is twice the signal frequency. Therefore, we arrive at a very important
conclusion that in amplitude modulation, bandwidth is twice the signal frequency. The tuned amplifier which is called
upon to amplify the modulated wave must have the required bandwidth to include the sideband frequencies. If the
turned amplifier has insufficient bandwidth, the upper sideband frequencies may not be reproduced by the radio
receiver.

8.2.3 Power in AM Wave


The power dissipated in any circuit is a function of the square of voltage across the circuit and the effective
resistance of the circuit. Equation of AM wave reveals that it has three components of amplitude EC ,maEC / 2 and
maEC / 2 . Clearly, power output must be distributed among these components.

(=
E / 2)
2
C E2C
Carrier power =
PC  … (i)
R 2R

(m E ) + (m E )
2 2
a C /2 2 a C /2 2 m2a E2C ma2 EC2 ma2 EC2
Total power of =
sidebands PS = + =  … (ii)
R R 8R 8R 4R
2 6 . 5 6 | Semiconductor and Communication System

E2 m2 E2 E2  m2  E2  2 + m2a 
Total power of AM wave P=T PC + PS ; =C + a C =C 1 + a  or PT = C   …. (iii)
2R 4R 2R  2  2R  2 
   

P Exp. (ii) m2a


Fraction of total power carried by sidebands
= is S =  … (iv)
PT Exp. (iii) 2 + m2
a

As the signal is contained in the sideband frequencies, therefore, useful power is in the sidebands. Inspection of
exp. (iv) reveals that sideband power depends upon the modulation factor ma. The greater the value of ma, the
greater is the useful power carried by the sidebands. This emphasizes the importance of modulation factor is

(a) When ma =0, power carried by sidebands= 02 / 2 + 0=


2
0
(0.5)2
(b) When ma =
= 0.5 , power carried by sidebands = 11.1% of total power of AM wave
2 + (0.5)2
(1)2
(c) When ma =1, power carried by sidebands
= = 33.3% of total power of AM wave.
2 + (1)2
As an example, suppose the total power of an AM wave is 600 watts and modulation is 100%. Then sideband power
is 600/3=200 watts and carrier power will be 600-200=400 watts.
The sideband power represents the signal content and the carrier power is that power which is required as the
means of transmission.
E2C m2a E2C PS 1 2 1 2
Note. PC
= = =
PS ∴ = ma or PS m P  …. (v)
2R 4R PC 2 2 a C

Expression (v) gives the relation between total sideband power PS ( ) and carrier power (PC ) .

8.2.4 AM Modulator
A circuit which does amplitude modulation is called AM modulator. Figure shows the circuit of a simple AM
modulator. It is essentially a CE amplifier having a voltage gain of A. the carrier signal is the input to the amplifier.
The modulating signal is applied in the emitter resistance circuit.
Working: The carrier voltage ec is applied at the amplifier and the modulating signal es is applied in the emitter
resistance circuit. The amplifier circuit amplifies the carrier by a factor “A” so that the output is Aec. Since the
modulating signal is a part of the biasing circuit, it produces low-frequency variations in the emitter circuit. This in
turn causes *variations in “A”. The result is that amplitude of the carrier varies in accordance with the strength of
the signal. Consequently, amplitude modulated output is obtained across RL. It may be noted that carrier should
not influence the voltage gain A; only the modulating signal should be this. To achieve this objective, carrier should
have a small magnitude and signal should have a large magnitude.
+VCC

8.2.5 Limitations of Amplitude Modulation


RC
Although theoretically highly effective, amplitude R1 CC

modulation suffers from the following drawbacks:


Cm
(a) Noisy reception: In an AM wave, the signal is in
RL
the amplitude variations of the carrier. Practically
ec
all the natural and man-made noises consist of
electrical amplitude disturbances. As a radio  R2 RE
Carrier
receiver cannot distinguish between amplitude CE

variations that represent noise and those that
Signal
contain the desired signal, therefore, reception is
generally noisy. Figure 26.96
P hysi cs | 26.57

(b) Low efficiency: In amplitude modulation, useful power is in the sidebands as they contain the signal. As
discussed before, an AM wave has low sideband power. For example, if modulation is 100%, the sideband
power is only-third of the total power of AM wave. Hence the efficiency of this type of modulation is low.
(c) Small operating quality. Due to low efficiency of amplitude modulation, transmitters employing this method
have a small operating range i.e., message cannot be transmitted over larger distances.
(d) Lack of audio quality: This is a distinct disadvantage of amplitude modulation. In order to attain high-fidelity
reception, all audio frequencies up to 15 kHz must be reproduced. This necessitates bandwidth of only 10 kHz
to minimize the interference from adjacent broadcasting stations. This means that the highest modulating
frequency can be 5 kHz which is hardly sufficient to reproduce the music properly.

8.3 Demodulation
The process of recovering the audio signal from the modulated wave is known as demodulation or detection.
At the broadcasting station, modulation is done to transmit the audio signal over larger distance to a receiver.
When the modulated wave is picked up by the radio receiver, it is necessary to recover the audio signal from it. This
process is accomplished in the radio receiver and is called demodulation.
Necessity of demodulation. It was noted that Receiving
amplitude modulated wave consists of carrier aerial
and sideband frequencies. The audio signal is
contained in the sideband frequencies which
are radio frequencies. If the modulated wave Station R.F.
after amplification is directly fed to the speaker No sound
selector Amplifier
as shown in figure, no sound will be heard. It
is because diaphragm of the speaker is not at
all able to respond to such high frequencies. Figure 26.97
Before the diaphragm is able to move in one
direction, the rapid reversal of current tends to move it in the opposite direction i.e. diaphragm will not move at all.
Consequently, no sound will be heard.
From the above discussion, it follows that audio signal must be separated from the carrier at a suitable stage in the
receiver. The recovered audio is then amplified and fed to the speaker for conversion into sound.

8.3.1 Essentials in Demodulation


In order that a modulated wave is audible, it is necessary to change the nature of modulated wave. This is
accomplished by a circuit called detector. A detector circuit performs the following two functions:
(a) It rectifies the modulated wave. i.e.
negative half of the modulated wave e
is eliminated. As shown in figure (i) a Positive
modulated wave has positive and negative
half
halves exactly equal. Therefore, average e
current is zero and speaker cannot respond. 0
Negative
If the negative half of this modulated
wave is eliminated as shown in figure, the half
average value of this wave will not be zero 0 t
t
since the resultant pulses are now all in one
(i) (ii)
direction. The average value is shown by
the dotted line in figure (ii). Therefore, the Figure 26.98
diaphragm will have definite displacement
corresponding to the average wave is similar to that of the modulation envelope. As the signal is of the same
shape as the envelope, therefore, average wave shape is of the same form as the signal.
2 6 . 5 8 | Semiconductor and Communication System

(b) It separates the audio signal from the carrier. The rectified modulated wave contains the audio signal and
the carrier. It is desired to recover the audio signal. This is achieved by a filter circuit which removes the carrier
frequency and allows the audio signal to reach the load i.e., speaker.

8.4 A. M. Diode Detector


Figure below shows a simple detector circuit employing junction diode and filter circuit. The modulated wave of
desired frequency is selected by the parallel turned circuit L1C1 and is applied to the junction diode. During the
positive half-cycles of modulated wave, the diode conducts while during negative half-cycles, it does not. The result
of this rectifying action is that output of consists of positive half-cycles of modulated wave as shown.

L L1 C

Speaker

Rectified Audio
A.M. Wave
current pulses output

Figure 26.99

The rectified modulated wave contains radio frequency and the signal cannot be fed to the speaker for sound
reproduction. If done so, no sound will be heard due to the inertia of speaker diaphragm. The r.f. (radio frequency)
component is filtered by the capacitor C shunted across the speaker. The value of this capacitor is sufficiently large
to present low reactance to the r.f. (radio frequency) component while presenting a relatively high reactance to the
audio signal. The result is that the r.f. (radio frequency) component is by-passed by the capacitor C and the signal
is passed on to the speaker for sound reproduction.

Illustration 1: Calculate the length of a half-wave dipole antenna at (i) 30 MHz (ii) 300 MHz and (iii) 3000 MHz.
What inference do you draw from the results? (JEE MAIN)
c
Sol: The wavelength of half-wave dipole antenna at certain frequency is given by λ =
f
Speed of radio waves, c = 3x108 ms−1
c 3x10 −8
(i) λ= = = 10m ∴ Length of half-wave dipole antenna is l=
λ/2=
10 / 2 =
5m
ƒ 30x106
3x108
(ii) λ= c = = 1m ∴ Length of half-wave dipole antenna is l =
λ/2=
1/2=
0.5m
ƒ 300x106
3x108
(iii) λ= c = = 0.1m ∴ l =
λ/2=
0.1 / 2 =
0.05m
ƒ 3000x106
It is clear that length of the dipole antenna decreases as the frequency of carrier wave increase.

Illustration 2: The maximum peak-to-peak voltage of an amplitude modulated wave is 16 mV and the minimum
peak-to-peak voltage is 4 mV. Calculate the modulation factor. (JEE MAIN)
Vmax − Vmin
Sol: The modulation factor is ma = .
Vmax + Vmin
P hysi cs | 26.59

Figure shows the conditions of the problem. V


16
Maximum voltage of AM wave, Vmax
= = 8mV
2
4 16mV
Minimum voltage of AM wave, Vmin= = 2mV
2 t
0 4mV
Vmax − Vmin 8 − 2 6
∴m
= = = = 0.6
a
Vmax + Vmin 8 + 2 10

Figure 26.100
Illustration 3: An AM wave is represented by the expression:
=V 5(1 + 0.6 cos6280 t) sin 211x10 4 t Volts  (JEE ADVANCED)
(i) What are the minimum and maximum amplitudes of the AM wave?
(ii) What frequency components are contained in the modulated wave and what is the amplitude of each component?

Sol: Comparing the equation with standard wave equation V =EC (1 + ma cos ωs t) sin ωc t we get the maximum
carrier amplitude EC, the modulation factor ma and ω. Using this we get the minimum and maximum amplitude of
ω
the wave as Ec − maEc and Ec + maEc respectively. The frequency is given by f = .

V 5(1 + 0.6 cos 6280 t) sin 211x10 4 t volts 
The AM wave equation is given by = … (i)

Compare it with standard AM wave V =EC (1 + ma cos ωs t) sin ωc t  ... (ii)


From eqs. (i) and (ii), we get, EC = Carrier amplitude = 5V; ma = Modulation factor = 0.6
ƒs = Signal frequency = ω=
s / 2π 6280
= / 2π 1kHz
4
ƒc = Carrier frequency =
= ωs / 2π 211x10
= / 2π 336kHz
(i) Minimum amplitude of AM wave = Ec − maEc =
5 − 0.6x5 =
2V
Maximum amplitude of AM wave =
Ec + maEc =
5 + 0.6 x5 =
δV
(ii) The AM wave will contain three frequencies viz.
ƒc − ƒs ƒc , ƒc + ƒs
or 336-1 336, 336+1
or 335 kHz, 336 kHz, 337 kHz
The amplitudes of the three components of AM wave are:
ma Ec ma Ec
Ec
2 2
0.6 x5 0.6 x5
or 5,
2 2
or 1.5 V, 5 V, 1.5 V

Illustration 4 : An 50 kW carrier is to be modulated to a level of (i) 80% (ii) 10%. What is the total sideband power
in each case? (JEE MAIN)
1 2
Sol: The power used in modulation ma is given by Ps = m P where PC is the power of carrier wave.
2 a c
1 2 1 2
(i) Ps
= =m P (0.8)
= x50 16kW
2 a c 2
1 2 1 2
(ii) Ps
= = ma Pc (0.1)
= x50 0.25kW
2 2
2 6 . 6 0 | Semiconductor and Communication System

Note the effect of modulation factor on the magnitude of sideband power. In the first case (ma = 80%), we generated
and transmitted 50 kW carrier in order to send 16 kW of intelligence. In the second case (ma = 10%), the same
carrier of operation decreases rapidly as modulation factor decreases. For this reason, in amplitude modulation, the
value of ma is kept as close to unity as possible.

Illustration 5: Consider an optical communication system operating at λ =800nm . Suppose, only 1% of optical
source frequency is the available channel bandwidth for optical communication. How many channels can be
accommodated for transmitting (i) audio signals requiring a bandwidth of 8 kHz (ii) video TV signals requiring an
approximate bandwidth of 4.5 MHz? (JEE MAIN)

Sol: The number of the channels for audio or video signal is obtained as
Total bandwidth 0.01 × c
N= . The total bandwidth is obtained as 0.01 × f = .
Bandwidth of signal λ

=
Optical wavelength, λ = 8x10−7 m
800nm

3x108
Frequency of optical source is ƒ= c= = 3.75 x1014 Hz
λ 8x10 −7

∴ Total bandwidth of the channels = 1% of 3.75 ×1014 = 3.75 ×1012 Hz


Total bandwidth of channel  3.75 x1012
=
(i) Number of channels for audio signals = = 4.7 x 108
Bandwidth of audio signal 8x103
3.75 x1012
(ii) Number of channels for video TV=
signals = 8.3 x 105
4.5 x106

7.5. Frequency Modulation


When the frequency of carrier wave is changed in accordance with the intensity of the signal, it is called frequency
modulation.
In frequency modulation, only the frequency of the carrier wave is changed in B F
accordance with the signal. However, the amplitude of the modulated wave A
remains the same i.e. carrier wave amplitude. The frequency variations of carrier C E G Signal
wave depend upon the instantaneous amplitude of the signal as shown in figure D
(iii). When the signal voltage is zero as a A, C, E and G, the carrier frequency (I)
is uncharged. When the signal approaches its positive peaks as a B and F, the
carrier frequency is increased to maximum as shown by the closely spaced cycles.
However, during the negative peaks of signal as at D, the carrier frequency is Carrier
reduced to minimum as shown by the widely spaced cycles. The following points
(II)
may be noted:
(a) All the signals having the same amplitude will change the carrier frequency
by the same amount irrespective of their frequencies. F.M. Wave
(b) All modulating signals of the same frequency, say 2 kHz, will change the
carrier at the same rate of 2000 times per second irrespective of their A B C D EF G
individual amplitudes. (III)
Figure 26.101
7.5.1 Advantages and Disadvantages of FM
Advantages
(a) It gives noiseless reception. As discussed before, noise is a form of amplitude variations and a FM receiver will
reject such signals.
P hysi cs | 26.61

(b) The operating range is quite large.


(c) It gives high-fidelity reception,
(d) The efficiency of transmission is very high.
(e) Since FM has a large number of sidebands, it can be used for stereo sound transmission.

Disadvantages
(a) A much wider bandwidth is required by FM. The bandwidth required is 7 to 8 times as large as for AM.
(b) FM transmitting and receiving equipment’s are complex, particularly for modulation and demodulation.
Therefore, FM is more expensive than AM.
(c) FM reception is limited to line-of-sight.

PROBLEM-SOLVING TACTICS

 1. For long distance transmission, we use electrical signals because they can be transmitted at very high speeds
( = 3x10 ms )
8 −1

 2. The energy of a wave is directly proportional to its frequency. This permits modulated waves to carry the
signals to long distances.
 3. In amplitude modulation (AM), the amplitude of high frequency wave is changed in accordance with the
intensity of the signal.

Amplitude change of carrier wave


Modulation factor, ma =
Normal carrier wave (unmodulated)

The value of m depends upon the amplitudes of carrier and signal.


 4. In frequency modulation (FM), the frequency of high frequency wave (carrier) is changed in accordance with
the intensity of the signal.
Maximum frequency deviation
Modulation Index, mƒ =
Modulating signal frequency

 5. In AM, the power level of the carrier is not affected by the modulation index m.
 6. In phase modulation, the phase angle of the high frequency wave (carrier) is changed in accordance with the
strength of the modulating signal.
2 6 . 6 2 | Semiconductor and Communication System

FORMULAE SHEET

 1. In an n-type semiconductor, ne ≅ Nd >> nh where Nd is the number density of donor atoms.

In a p-type semiconductor, nh ≅ Na >> ne where Na is the number density of acceptor atoms.


In a doped semiconductor (n type or p-type). nenh >> ni2
Where ni is number density of intrinsic carriers?
 2. In amplitude modulation (AM), the amplitude of high frequency wave is changed in accordance with the
intensity of the signal.

Amplitude change of carrier wave


Modulation factor, ma =
Normal carrier wave (unmodulated)

 3. In frequency modulation (FM), the frequency of high frequency wave (carrier) is changed in accordance with
the intensity of the signal.

Maximum frequency deviation


 4. Modulation Index, mƒ =
Modulating signal frequency
 5. σ e (neµe + nhµh )
=
1
Where σ = is called conductivity of the material of semiconductor and µe, µh are electron and hole mobilities
ρ
respectively.

 6. The equation for diode current


= (
is I Io ecV /kT − 1 )
Where Io is called saturation current, V is positive for forward and negative for reverse bias, k is Boltzmann
constant, T is temperature and e = 1.6 x10−19 C .
 7. Half wave Rectifier
Expression for output D.C. Voltage
Output d.c. voltage = Mean load current x load resistance i.e. Vd.c. = Id.c. RL . But
Io
Where Io is the maximum value of the secondary half wave current ∴ Vd.c. = x RL
π
 8. Full-wave Rectifier
Expression for output D.C. Voltage
I
Output D.C. voltage = Mean load current x load resistance i.e. Vd.c. = Id.c. RL but Id.c. = o where Io is the
π
Io
maximum value of the secondary half wave current ∴ Vd.c. = x RL
π
Thus, output D.C. voltage in case of full wave rectifier is twice the output D.C. voltage in case of half wave rectifier.
change in forward voltage across diode
 9.  a.c. forward resistance, rƒ =
corresponding change in current through diode
 10. Zener diode voltage regulation
Voltage drop across R=
S Ein − Eo ; Current through R S '=
I I z + IL
Ein − Eo
Applying Ohm’s law, we have R S =
lz + lL
Where R S is the series resistance that absorbs voltage fluctuations, RL is the load resistance across which
output regulated voltage is desired, Iz is the zener current and lL is the load current.
P hysi cs | 26.63

 11. For a photodiode, ∴ IR = mE Where m = slope of the straight line


The quantity m is called the sensitivity of the photo-diode.
IR is the reverse current and E is the illumination of the photo diode.

 12. For a transistor, where lE = lB + lC is emitter current, lB is base current and lC is collector current.
 13. Gains in Common-Base Amplifier
The various gains in a common-base amplifier are as follow:

(i) ac Current Gain: It is defined as the ratio of the change in the collector-current to the change in the
emitter-current at a constant collector-to-base voltage, and is denoted by α .

 ∆i 
Thus α(ac) = C
 ∆i 
 B VCE
The value of α is slightly less than 1 (actually, there is a little current loss).

(ii) ac Voltage Gain: It is defined as the ratio of the changes in the output voltage to the change in the input
voltage, and is denoted by A.

Suppose on applying an ac input voltage signal, the emitter current changes by ∆i and correspondingly
the collector-current changes by ∆iC . If Rin and R out be the resistances of the input and output circuits
respectively, then

∆iC xR in ∆iC R in
=AV = x
∆iE xR out ∆iE R out

Now, ∆iC / ∆iE is the ac current-gain and R in / R out is called the ‘resistance gain’.

∴ Av = α × Resistance gain
Since the resistance gain is quite high AV is also high although α is slightly less than 1.
(iii) ac Power Gain: It is defined as the ratio of the change in the output power to the change in the input
power.

Since power=current x voltage, we have ac power gain = ac current gain x ac voltage-gain = a2 ×

Resistance gain

 14. Gain in Common emitter amplifier

(i) dc current Gains: It is defined as the ratio of the collector current to the base current, and is denoted by
iC
β ( dc ) =
iB
In a typical transistor, a small base-current ≈ 10µA ( ) ( )
produces a large collector-current ≈ 500µA . Thus
500
β ( dc ) = = 50
10
(ii) ac Current Gain : It is defined as the ratio of the change in the collector-current to the change in the base-
current at a constant collector to emitter voltage, and is denoted by
 ∆iC 
β(ac) . Thus β ( ac ) =
 
 ∆iB VCE
(iii) Voltage gain : Suppose, on applying an ac input voltage signal, the input base-current

Charges by ∆iB and correspondingly the output collector-current changes by ∆iC . If Rin and
R out be the resistance of the input and the output circuits respectively, then.
2 6 . 6 4 | Semiconductor and Communication System

∆iC xR out ∆iC R out


=AV = x ... (i)
∆iB xR in ∆iB R in

Now, ∆iC / ∆iB is the ac current gain (ac) and R in / R out is the resistance gain
∴ Av =
β(ac) × resis tance gain  ... (ii)
Since β(ac) >> α(ac) , the ac voltage gain in common-emitter amplifier is larger compared

To the common-base amplifier, although the resistance gain is smaller.


From equation (i) and (ii), it follows that A v = gm xR out

(iv) ac Power gain : It is defined as the ratio of the change in the output power to the change in the input
power.

Since power=current x voltage, we have ac power gain = ac current gain × ac voltage gain
β(ac) × {β(ac) × resis tance gain} =
β(ac) × A v =
= β2 (ac) × resis tance gain

Since β(ac) >> α(ac) , the ac power gain in common-emitter amplifier is extremely large
Compared to that in common-base amplifier.
1
 15. The frequency of oscillations is given by v =
2π LC

 16. Value of critical frequency in sky wave propagation is given by ƒc = 9 (Nmax )


1/2

Where Nmax =Maximum electron density of ionosphere.

ƒc
 17. Maximum usable frequency, MUF
= = ƒc sec θ
cos θ

Where θ = Angle between normal and direction of incident waves.

Amplitude change of carrier wave ES Vmax − Vmin


 18. Modulation factor, m= = =
Normal carrier wave (unmodulated) EC Vmax + Vmin
a

 19. The instantaneous Voltage of AM wave is


maEC maEC
= EC cos ωc t + cos ( ωc + ωs ) t + cos ( ωc − ωs ) t
2 2

( ) (
 20. In an AM wave, the bandwidth is form ƒc − ƒs to ƒc + ƒs i.e, 2ƒs . )
 21. Power In AM Wave
The power dissipated in any circuit is a function of the square of voltage across the circuit and the
effective resistance of the circuit. Equation of AM wave reveals that it has three components of amplitude
EC ,ma EC / 2 and ma EC / 2. Clearly, power output must be distributed among these components.

=
Carrier power, PC
(=
E / 2)
C E2c
… (i)
R 2R 

(=
m E / 2 2) (m E )
2 2
a C a C /2 2 m2 E2 m2 E2 m2 E2
a C a C a C
Total power of =
sidebands PC = + =  …. (ii)
R R 8R 8R 4R

E2C m2aE2C E2C  m2 


Total power of AM wave,    P
  T = PC + PS = + = 1 + a 
2R 4R 2R  2 
 
P hysi cs | 26.65

Solved Examples

JEE Main/Boards + 0.5 V -


A R = 100 B
Example 1: For a common base transistor if the values
of IE and IC are 103 µA and 0.96 mA respectively then
the value of IB will be-

Sol: The base current in the circuit is given by IB= IE − IC .


D + - C
∴ IE =IB + IC ; IB =IE − IC E = 4.5 V

= 1000X10−6 − 0.96X10−3 V=4.5-0.5=4.0V


=1mA − 0.96mA = 0.04mA Current in the circuit,
V 4.0
Example 2: Mobility of electronics in Germanium of =l = = 0.04A
= 0.04x1000mA
= 40mA
R 100
N types & their conductivity are 3900 cm2/ V s &
5 mho/cm respectively. If effect of holes are negligible
then concentration of impurity will be- Example 5: Calculate the emitter current for which
lb= 20µA, β= 100
σn
Sol: The density of the electron is given by nd =
eµd Sol: The emitter current in the transistor circuit is given
µa = 2
3900 cm / volt sec by le= lb + lc where Ib and Ic are the base and collector
current respectively.
5 mho / cm
σn =
Here β= 100, lb= 20µA
nd x1.6x10−19 x3900
σn =
lc
5 β= ∴ lc = βlb = 100x20 = 2000µA
=∴ nd = 8x1015 lb
1.6x39x10−17
Using le= lb + lc , we get
Example 3: What will be conductance of le = 20 + 2000 = 2020µA = 2.02x10−3 A = 2.02mA
pure silicon crystal at 300 K Temp. If electron
hole pairs per cm3 is 1.072/1010 at this Temp; Example 6: Transistor with β =75 is connected
2
µn 1350 cm= / volt sec & µh 480cm2 / volt sec to common-base configuration. What will be the
maximum collector current for an emitter current of 5
Sol: The conductivity of the sample is given by mA?
σ nie ( µe + µh ) where ni is the concentration of the
=
hole-electron pair. Sol: As value of CE gain is given we find the value of
α
σ = nieµe + nieµh = nie ( µe + µh ) CB gain by β = . The value of collector current is
1− α
= 3.14x10−6 mho / cm found as Ic = α Ie .
Here
= β 75,
= lc 5mA
Example 4: Figure shows a diode connected to an
α
external resistance and an e.m.f. assuming that the Step-l Using β = ,
barrier potential developed in diode is 0.5V obtain the 1− α
value of current in the circuit in milli-ampere. α
we α
we get
get 75 = 1 − α or
75 = or 75 − 75
75 − 75αα= =αα
Sol: The current in the circuit is obtained by Ohm’s 1− α
75
75
=
law, as the diode resistance is zero in forward biased
= or
or 76 α 75
76 α 75 == or
or α
α 76
condition. 76
llc 75
Step ll l l 75
Here,=E 4.5 V,R
= 100Ω , voltage in the circuit, Step − ll α = lc ∴ lcc = αlee = 76 ×
− α = ∴ = α = ×5 = 4.93A
5= 4.93A
lee 76
2 6 . 6 6 | Semiconductor and Communication System

Example 7: In NPN transistor circuit, the collector JEE Advanced/Boards


current is 10 mA. If 95% of the electrons emitted reach
the collector, what is the base current? Example 1: A potential barrier of 0.5V exists across a
p-n junction
Sol: The base current in NPN transistor circuit is
(i) If the depletion region is 5x10-7m wide. What is the
Ib= Ie − Ic where Ie is the emitter current and Ic is the
intensity of the electric field in this region?
collector current.
(ii) An electron with speed 5x106m/s approaches the
Step-l Ic=95% Ic=0.95Ie
p-n junction from the n-side with what speed will it
Ic 10 enter the p-side.

= Ie =
0.95 0.95
(=
 Ic 10mA )
V
Sol: The intensity of electric field is given by E =.
= 10.53mA ∆L
Step ll Now le= lc + lb The energy required by electron to move from N side
to the P side is greater than or equal to the barrier
∴ lb =le − l=
c 10.53 − 10= 0.53mA potential.
V 0.5V
Example 8: In a pure silicon sample, 1013 atoms of E
(i)= = ,
∆L 5X10−7
phosphorus are doped per cm. If all the donor atoms
produce carriers and µc = 1200 cm2 / volt-sec then, V
Depletion layer= ∆L ; E = 106
calculate the resistivity of the sample. m
1 1 (ii) Work energy theorem
Sol: The resistivity is obtained as ρ= =
σ eneµe 1 1
2
13 MV= eV + MVi2
Given ne 10
= = per cm3 1019 per m3 2 i
2
µc =0.12m2 / volt sec MVi2 − 2eV
=Vf = 2.7x105 m / s
Therefore, for doped, n-type semiconductor M

= eneµ=
σ e 1.6x10−19 x1019 x0.12
= 0.192 ohm m−1
Example 2: The base current is 100 µA and collector
1 current is 3 mA.
The resistivity=
is ρ = 5.2ohm meter
0.192
(a) Calculate the values of β and α

Example 9: What will be the antenna current when (b) A change of 20 µA in the base current produces a
modulation is 40%? change of 0.5mA is the collector current. Calculate βac

Sol: The antenna current is obtained using equation l


Sol: The value of α and β are given as β = c and
2 lb
PT I  m2a α ∆l
= T  = 1 + where PT is the power of transmitter β= . The value of βac is found as βac = c .
PC  IC  2 1− α ∆lb
and PC is the power of carrier wave. Here lb = 100µA= 0.100mA ; lc = 3mA
1 lc
Ps = m2a Pc (a) Using
= β , we get
= β
3
= 30
2 lb 0.100
 m m2a 
2
P
PTT = =P +P
Pcc + =P
Pss = Pcc 1 1+ + a  α
α ,
 2
2  Us
Usiin
ng
g β
β== ,
  1
1− α
− α
2 2
2 2
P
PTT m
maa2 
 IITT  m
m2aa α
α = 30 − 30α = α

∴ =
= 1
1++ or
or   = = 1
1++ we
we get
get 30
30 =
= = 30 − 30α = α
P
PCC 2
2  IIC  2
2 1−
1 −αα
C
GiventhatI
=
GiventhatI
= cc 8A
=
8A
= ;m
;maa 0.4 0.4 30
or 31α
= 30 ∴ α
= = 0.97
2
2
 IIT  2 31
(0.4)
(0.4)2

∴  T  = = 1
1 +
+ l lc 3x31
 8  2 Using α = c , we get l= = = 3.1mA
 8  2
le e
α 30
or (IT // 8)
=
or =
(I 8) 1.08 =
1.08
= orI
orITT 8 1.08
1.08 8.31A
8=
= 8.31A
T
P hysi cs | 26.67

2
=µe 0.135m
= / Vs; µh 0.048m2 / Vs and
(b) Here ∆lb = 20µA = 0.02mA ; ∆lc =0.50mA
16 −3
n
=i 1.5 × 10 m For germanium,
∆lc 0.5 2
∴ β=
ac = = 25 =µe 0.39m= / Vs , µh 0.19m2 / Vs and
∆lb 0.02
ni 2.4 × 1019 m−3 .
=
Example 3: The Concentration of hole – electron pairs
in pure silicon at T=300 K is 7 × 1015 per cubic meter. Sol: The electron and hole current in semiconductor
Antimony is doped in silicon in proportion of 1 atom in block is given by=le neeAµe Ε and=lh nheAµhΕ where
107 Si atoms. Assuming that half of the impurity atoms A is the area, ne and nh are the number densities of
contribute electron. In the conduction band, calculate electron and holes and µe and µh are electron and
the factor by which the number of charge carriers
hole mobilities, in the semiconductor block. The total
increases due to doping the number of silicon atoms
current is given by I=Ie+Ih.
per cubic meter is 5 × 1028 .
e.m.f. 2
=
Sol: The factor to increase the number of charge carriers Electric field = l 0.1
n − ninitial = 20Vm−1
from initial to final is given by final where nfinal
ninitial
Cross-sectional area, A = 1× 10−4 m2
and ninitial are the charge concentration after and before
For silicon block
doping the silicon with antimony.
In pure semiconductor electron-hole pair = 7 × 1015 ; ne= nh= n=
i 1.5 × 1016 m−3
ninitial = nh + ne = 14 × 1015 = µe 0.135m= 2
/ Vs; µh 0.048m2 / Vs
After doping: Donor Impurity, Electron Current,
5 × 1028 21
ND = = 5 × 10  νe 
107 le neeA=
= νe neeAµe Ε  µ
= 

e
Ε 
ND
According to question n=
e = 2.5 × 1021
2 = (1.5 × 1016 ) × (1.6 × 10−19 ) × (1× 10−4 ) × 0.135 × 20
so nfinal
= nh + ne
=0.648 × 10−16 A =0.648µA
Here ne >> nh
Hole current,
so nfinal ≈ ne ≈ 2.5 × 1021
 νh 
nfinal − ninitial 2.5 × 1021 lh nheA=
= νh nheAµhΕ  µ
= 
Factor = ≈ 1.8 × 105
= 
h
Ε 
ninitial 14 × 1015
= (1.5 × 1016 ) × (1.6 × 10−19 ) × (1× 10−4 ) × 0.048 × 20
Example 4: In Figure below a battery of e.m.f. 2V is
used. The length of the block is 0.1 m and the area is
0.23 × 10−16 A =
= 0.23µA
1× 10−4 m2 . If the block is of intrinsic silicon at 300 K,
Total current l = le + lh = 0.648 + 0.23 = 0.878µA
find the electron and hole current. What will be the
magnitude of total current if germanium is used instead For germanium block
of silicon? For silicon
ne = nh = ni = 2.4 × 1019 m−3 ; µe = 0.39m2 / Vs; µh
l = 0.1 m
= 0.19m2 / Vs
le neeAµe Ε
Electron Current, =

= (2.4 × 1019 ) × (1.6 × 10−19 ) × (1.0 × 10−4 ) × 0.39 × 20


=2.995 × 10−3 A =2.995mA
lh nheAµhΕ
Hole current,=

2V
2 6 . 6 8 | Semiconductor and Communication System

= (2.4 × 1019 ) × (1.6 × 10−19 ) × (1.0 × 10−4 ) × 0.19 × 20


A B A B Y = A.B
=1.459 × 10−3 A =1.459mA
0 0 1 1 0
Total current, l = le + lh = 2.995 + 1.459 = 4.454mA
1 0 0 1 1

Example 5: Explain how OR gate is realized using 0 1 1 0 1


NAND gate.
1 1 0 0 1

Sol: The OR gate is made using the NAND gate.


For this purpose, we use three NAND gates; the first
The equation explaining the structure is given by
two NAND gates (I and II) are operated as NOT gates
Y (
=
 ) ( ) (A • B)
A • A • B •B 

where A and B are the and their outputs are fed to the third NAND gate (III).
The resulting circuit is OR gate. This is proved as under:
output of the two NAND gates used as input in third
NAND gate Output of gate I = A.A = A + A = A Output of gate
II = B.B = B + B = B
A Output of gate III, Y = A .B = A + B = A + B
A l
Since Y=A+B is the Boolean expression for OR gate,
lll y
the circuit shown in Fig. is OR gate, its truth table is also
shown.
B ll
B

JEE Main/Boards

Exercise 1 Q.5 In the following diagrams, write which of the diodes


are forwards biased and which are reverse biased.
Semiconductor +5V +10V

Q.1 What is meant by the term doping of an intrinsic (A)


R
(B)
semiconductor? How does it affect the conductivity of
a semiconductor? +5V

Q.2 If the output of a 2-input NAND gate is fed as


the input to a NOT gate (i) name the new logic gate
R -12V
obtained and (ii) write down its truth table.
-10V
(C) (D)
R
Q.3 Draw the energy band diagram of a p-type
semiconductor. Deduce an expression for the -5V
conductivity of a p-type semiconductor. Q.6 The output of a 2-input NAND gate is fed to a NOT
gate. Write down the truth table for the output of the
Q.4 How does the width of the depletion region of p-n combination for all possible inputs of A and B.
junction vary, if the reverse bias applied to it?
Q.7 Name the gate obtain from the combination of
gates shown in the figure. Draw its logic symbol. Write
the truth table of the combination.
P hysi cs | 26.69

A Y’ (d) A photodiode is fabricated from a semiconductor


Y with a band gap of 2.8 eV. Can detect wavelength of
B 6000nm? Justify.

Q.8 How is a p-type semiconductor formed? Name


Q.15 Explain (i) Forward biasing, (ii) Reverse biasing of
the major charge carriers in it. Draw the energy band
a P-N junction diode. With the help of a circuit diagram,
diagram of p-type semiconductor.
explain the use of this device as a half-wave rectifier.

Q.9 Draw a labeled circuit diagram of a common base


Q.16 What are energy bands? How are these formed?
amplifier using a p-n-p transistor. Define the term
Distinguish between a conductor, an insulator and a
`voltage gain’ and write an expression for it.
semiconductor on the basis of energy band diagram.
OR
Q.10 Draw the voltage-current characteristic of a Zener
diode. Explain the function of base region of a transistor.
Why this region is made thin and lightly doped?
Draw a circuit diagram to study the input and output
Q.11 Give the logic symbol for an OR gate. Draw the characteristics of n-p-n transistor in a common
output wave from for input wave forms A and B for this `emitter (CE) configuration. Show these characteristics
gate. graphically. Explain how current amplification factor of
the transistor is calculated using output characteristics.

A
Q.17 Two semiconductor materials X and Y shown
(Inputs) in the given figure, are made by doping germanium
crystal with indium and arsenic respectively. The two
B are joined end to end and connected to a battery as
shown.
X Y
Q.12 With the help of a labeled circuit diagram, explain
how an n-p-n transistor can be used as an amplifier in
common emitter configuration. Explain how the input
and output voltages are out of phase by 180° for a
common-emitter transistor amplifier. OR
For an n-p-n transistor in the common-emitter
configuration, draw a labeled circuit diagram of an
arrangement for measuring the collector current as a
function of collector-emitter voltage for at least two (i) Will the junction be formed biased or reverse biased?
different values of base current. Draw the shape of the
curves obtained. Define the terms: (i) output resistance (ii) Sketch a V-I graph for this arrangement.
and (ii) current amplification factor.
Q.18 Draw the circuit diagram of a common emitter
Q.13 On the basis of the energy band diagrams amplifier using n-p-n transistor. What is the phase
distinguish between metals, insulators and difference between the input signal and output voltage?
semiconductors. State two reasons why a common emitter amplifier is
preferred to a common base amplifier.

Q.14 (a) With the help of a circuit diagram explain the


working of transistor as oscillator. Q.19 Explain the formation of energy band in solids.
Draw energy band diagram for (i) a conductor (ii) and
(b) Draw a circuit diagram for a two input OR gate intrinsic semiconductor.
and explain its working with the help of input, Output
waveforms. OR
Q.20 State the reason, why GaAs is most commonly
(c) Explain briefly with the help of a circuit diagram how used in making of a solar cell.
V-I characteristics of p-n junction diode are obtained in
(i) forward bias, and (ii) reverse bias.
2 6 . 7 0 | Semiconductor and Communication System

Q.21 The input A and B are inverted by using two NOT Q.27 (i) Identify the logic gates marked P and Q in the
gates and their outputs are fed to the NOR gate as given logic circuit.
shown below.
A
P
A (1) B Q X

(ii) Write down the output at X for the inputs A=0, B=0
B (2)
and A=1, B=1.

Analyses the action of the gates (1) and (2) and identify Q.28 What happens to the width of depletion layer of
the logic gate of the complete circuit so obtained. p-n junction when it is (i) forward biased, (ii) reverse
Given its symbols and the truth table. biased?

Q.22 Draw the labeled circuit diagram of a common- Q.29 Draw a labeled diagram of a full wave rectifier
emitter transistor amplifier. Explain clearly how the circuit. State its working principle. Show the input-
input and output signals are in opposite phase. Or output waveforms.
State briefly the underlying principle of a transistor
oscillator. Draw a circuit diagram showing how the Q.30 You are given a circuit below. Write its truth
feedback is accomplished by inductive coupling. table. Hence, identify the logic operation carried out
Explain the oscillator action. by this circuit. Draw the logic symbols of the gate it
corresponds to.
Q.23 Give the logic symbol of NOR gate.
A X
Q.24 With the help of a suitable diagram, explain the
formation of depletion region in p-n junction. How Z
does its width change when the junction is (i) forward
biased, and (ii) reverse biased? B
Y

Q.25 Give a circuit diagram of a common emitter Q.31 Describe briefly with the help of a circuit diagram,
amplifier using an n-p-n transistor. Draw the input and how the flow of current carriers in a p-n-p transistor
output waveforms of the signal. Write the expression is regulated with emitter-base junction forward biased
for its voltage gain. and base-collector junction reverse biased.

Q.26 (a) Draw the circuit diagrams of p-n junction Q.32 (a) Describe briefly, with the help of diagram, the
diode in (i) forward bias, (ii) reverse bias. How are these role of the two important processed involved in the
circuits used to study to study the V-I characteristics. formation of a p-n junction.
(b) What is a light emitting diode (LED)? Mention (b) Name the device which is used as a voltage regulator.
two important advantages of LED’s over conventional Draw the necessary circuit diagram and explain its
lamps. Or working.
(a) Draw the circuit arrangement for studying the input
and output characteristics of an n-p-n transistor in CE Q.33 (a) Explain briefly the principle on which a
configuration. With the help of these characteristics transistor-amplifier works as an oscillator. Draw the
define (i) input resistance, (ii) current amplification necessary circuit diagram and explain its working.
factor.
(b) Describe briefly with the help of a circuit diagram (b) Identify the equivalent gate for the following circuit
how an n-p-n transistor is used to produce self- and write its truth table.
sustained oscillations.
P hysi cs | 26.71

Communication System Exercise 2


Q.34 A 40 kW carrier is to be modulated to a level of
Semiconductor
100%.
(i) What is the carrier power after modulation? Single Correct Choice Type
(ii) How Much audio power is required if the efficiency
of the modulated RF amplifier is 72%? Q.1 The forbidden energy gap of a germanium
semiconductor is 0.75 eV. The minimum thermal energy
Q.35 The load current in the transmitting antenna of an of electrons reaching the conduction band from the
unmodulated AM transmitter is 8 A. What will be the valence band should be
antenna current when modulation is 40%? (A) 0.5 eV (B) 0.75 eV
Carrier (40 kW) (C) 0.25 eV (D) 1.5 eV
RF Excitation
Modulated RF
Q.2 The energy of a photon of sodium light ( λ =5890A )
AM wave Sideband (20 kW)
amplifier
=72%
equal the band gap of a semiconductor. The minimum
energy required to create an electron-hole pair is
Audio modulation + (A) 0.026 eV (B) 0.75 eV
power
(C) 2.1 eV (D) 6.4 eV
Q.36 The antenna current of an AM transmitter is 8 A
when only carrier is sent but it increase to 8.93 A when
Q.3 On increasing temperature the specific resistance
the carrier is sinusoidally modulated. Find the % age
of a semiconductor
modulation.
(A) Decreases (B) Increases
Q.37 The r.m.s value of carrier voltage is 100V. After (C) Remains constant (D) Become zero
amplitude modulation by a sinusoidal a.f. (audio
frequency) voltage, the r.m.s. value becomes 110 V. Q.4 In a good conductor the energy gap between the
Calculate the modulation index. conduction band and the valence band is-
(A) Infinite (B) Wide
Q.38 An AM wave consists of the following components:
Carrier component =5.V peak value. Lower sideband (C) Narrow (D) Zero
component = 2.5 V peak value. Upper sideband
component =2.5 V peak value. If the AM wave drives a Q.5 In a semiconducting material the mobilities of
2K Ω resistor, find the power delivered to the resistor electrons and holes are µe and µh respectively. Which
by (i) carrier component (ii) Lower sideband component of the following is true
and (iii) upper sideband component. What is the power
delivered? (A) µe > µh (B) µe < µh
(C) µe =µh (D) µe < 0 ; µh > 0
Q.39 When modulation is 75% and AM transmitter
produces 10kw. What would be percentage power Q.6 Those materials in which number of holes in valence
saving if the carrier and one of sidebands were band is equal to number of electrons in conduction
suppressed before transmission took place? band are called
(A) Conductors
Q.40 Is it necessary for a transmitting antenna to be
at the same height as that of the receiving antenna (B) Intrinsic semiconductors
for line-of-sight communication? A TV transmitting (C) p-type semiconductors
antenna is 81 m tall. How much service area can it cover
if the receiving antenna is at the ground level? (D) n-type semiconductors
2 6 . 7 2 | Semiconductor and Communication System

Q.7 In p-type semiconductor holes move in (i) -10 V, -5 V (ii) -5 V, -10 V (iii)-4 V, -12 V
(A) Forbidden region (B) Conduction band (A) (i)<(ii)<(iii) (B) (iii)<(ii)<(i)
(C) Valence band (D) all the above regions (C) (ii)=(iii)<(i) (D) (i)=(iii)<(i)

Q.8: Fermi level of energy of intrinsic semiconductor Q.13 Which is the correct diagram of a half-wave
lies- rectifier –
(A) In the middle of forbidden gap +
(B) Below the middle of forbidden gap (A)
(C) Above the middle of forbidden gap
-
(D) Outside the forbidden gap +

(B)
Q.9 The electron mobility in N-type germanium is 3900
cm2 / V .s and its conductivity is 6.24 mho/cm, then
-
impurity concentration will be if the effect of cotters is +
negligible-
(C)
(A) 1015 cm3 (B) 1013 / cm3
(C) 1012 / cm3 (D) 1016 / cm3 -
+

Q.10 The approximate radio of resistances in the (D)


forward and reverse bias of the PN-junction diode is-
-
(A) 102 : 1 (B) 10−2 : 1
Q.14 In the diagram, the input is across the terminals
(C) 1: 10−4 (D) 1: 10 4
A and C and the output is across the terminals B and S,
then the output is
Q.11 The diode used in the circuit shown in the figure
has a constant voltage drop of 0.5 V at all current and a B
maximum power rating of 100 milliwatts. What should
be the value of the resistor R, connected in series with
the diode for obtaining maximum current-? A C
R 0.5V

(A) Zero (B) Same as input


15V
(C) Full wave rectifier (D) Half wave rectifier
(A) 1.5Ω (B) 5Ω
(C) 6.67 Ω (D) 200 Ω
Q.15 An n-p-n transistor circuit is arranged as shown
in figure It is-
Q.12: In the following circuits PN-junction diodes D1,
D2 and D3 are ideal for the following potential of A and
B, the correct increasing order of resistance between A n
and B will be p RL
Vout
Vin n
D1 R
D2
R D3 R R
4 4
A B
P hysi cs | 26.73

(A) A common-base amplifier circuit at a time t=CR are respectively.


(B) A common- emitter amplifier circuit V V
+ - + -
(C) A common-collector amplifier circuit
(D) None of the above

Q.16 The combination of `NAND’ gates shown here


under (See below figure) are equivalent to R R
(A) VC, VC (B) VC/e, VC
A (C) VC, VC/e (D) VC/e, VC/e
C

Q.21 In the given circuit V0 & V0 are


B 1 2

(A) 11.3 V & 0.3 V (B) 0.3 V & 11.3 V

A C (C) 11.3 V & 11.3 V (D) 0.3 V & 0.3 V


B
Q.22 In Figure the current supplied by the battery is
(A) An OR gate and an AND gate respectively Si 4.7 k
12 V V 02
(B) An AND gate and a NOT gate respectively V 01
Ge
(C) An AND gate and an OR gate respectively
(D) An OR gate and a NOT gate respectively (A) 0.1 A (B) 0.2 A (C) 0.3 A (D) 0.4 A

Q.17 For the given combination of gates, if the logic Q.23 Which of the following frequencies will be
states of inputs A,B, C are as follows A=B=C=0 and suitable for beyond-the horizon communication using
A=B=1, C=0 then the logic states of output D are- sky waves?
A
B G G2 D A
1
A’
C
Y
(A) 0, 0 (B) 0, 1 (C) 1, 0 (D) 1, 1
B’
B
Q.18 Which of the following gates will have an output
of 1- (A) 10 kHZ (B) 10 MHz
(C) 1 GHz (D) 1000 GHz
1 (B) 0
(A)
0 1
Q.24 Frequencies in the UHF rage normally propagate
0
(C) 1 (D) 0 by means of :
1
(A) Ground waves (B) Sky waves
Q.19 This symbol represents- (C) Surface waves (D) Space waves
A y
B
(A) NOT gate (B) OR gate
Previous Years’ Questions
(C) AND gate (D) NOR gate Semiconductor

Q.20 Two identical capacitors A and B are charged to Q.1 Photoelectric effect experiments are performed
the same potential V and are connected in two circuits using three different metal plates p, q and r having
at t=0 as shown in Figure The charge of the capacitors work functions= φp 2.0 eV,=φq 2.5eV,and= φr 3.0 eV ,
2 6 . 7 4 | Semiconductor and Communication System

respectively. A light beam containing wavelengths of (C) It is a pnp transistor with R as emitter
550 nm, 450 nm and 350 nm with equal intensities
(D) It is an npn transistor with R as collector
illuminates each of the plates. The correct I-V graph for
the experiment is (Take hc=1240 eV nm)  (2009)
Q.4 In the circuit below, A and B represent two inputs
and C represents the output. The circuit represents
l p l
q  (2008)
(A) r
(B) p A
q
r
V V

l l
r B
q
(C) p (D) r
q
p (A) NOR gate (B) AND gate
V V
(C) NAND gate (D) OR gate
Q.2 Truth table for system of four NAND gates as shown
in Figure is  (2010) Q.5 A p-n junction (D) shown in the figure can act as a
rectifier. An alternating current source (V) is connected
A in the circuit. (2009)

Y
D
R
B v 
A B C A B Y A B Y A B Y
0 0 0 0 0 0 0 0 1 0 0 1
0 1 1 0 1 0 0 1 1 i 0 1 0 i
(A) (B)
1 0 1 1 0 1 1 0 0 1 0 0
r r
1 1 0 1 1 1 1 1 0 i 1 1 1 i
(C) (D)
(A) (B) (C) (D)
r r
B Y A B Y A B Y
Q.6 The logic circuit shown below has the input
0 0 0 0 1 0 0 1 waveforms ‘A’ and ‘B’ as shown. Pick out the correct
1 0 0 1 1 0 1 0 output waveform.  (2009)
0 1 1 0 0 1 0 0
1 1 1 1 0 1 1 1 A
Y
B) (C) (D)
B
Q.3 A working transistor with its three legs marked P,
Q and R is tested using a multi meter. No conduction Input A
is found between P and Q. By connecting the common
(negative) terminal of the multi meter to R and the
other (positive) terminal to P or Q, some resistance is Input B
seen on the multi meter. Which of the following is true
for the transistor?  (2008)
(A) It is an npn transistor with R as base (A) (B)
(B) It is a pnp transistor with R as collector
(C) (D)
P hysi cs | 26.75

Q.7 The combination of gates shown below yields Q.12 A signal of 5 kHZ frequency is amplitude
 (2010) modulated on a carrier wave of frequency 2 mHz. The
A frequencies of the resultant signal is/are :  (2015)

X (A) 2 MHz only


(B) 2005 kHz, and 1995 kHz
B
(C) 2005 kHz, 2000 kHz and 1995 kHz
(A) OR gate (B) NOT gate (D) 2000 kHz and 1995 kHz
(C) XOR gate (D) NAND gate
Q.13 A red LED emits light at 0.1 watt uniformly around
it. The amplitude of the electric field of the light at a
Q.8 This question has Statement-I and Statement-II. Of
distance of 1 m from the diode is :  (2015)
the four choices given after the statements, choose the
one that best describes the two statements. (2011) (A) 1.73 V/m (B) 2.45 V/m
Statement-I: Sky wave signals are used for long (C) 5.48 V/m (D) 7.75 V/m
distance radio communication. These signals are in
general, less stable than ground wave signals. Q.14 The temperature dependence of resistances of Cu
Statement-II: The state of ionosphere varies from hour and undoped Si in the temperature range 300 – 400 K,
to hour, day to day and season to season. is best described by : (2016)

(A) Statement-I is true, statement-II is true; statement-II (A) Linear increase for Cu, exponential increase for Si
is the correct explanation of statement-I. (B) Linear increase for Cu, exponential decrease for Si
(B) Statement-I is true, statement-II is true; statement-II (C) Linear decrease for Cu, linear decrease for Si
is not the correct explanation of statement-I.
(D) Linear increase for Cu, linear increase for Si
(C) Statement-I is false, statement-II is true.
(D) Statement-I is true, statement-II is false. Q.15 Choose the correct statement : (2016)
(A) In amplitude modulation the frequency of high
Q.9 A radar has a power of 1 Kw and is operating at a frequency carrier wave is made to vary in proportion to
frequency of 10 GHz. It is located on a mountain top the amplitude of the audio signal
of height 500 m. The maximum distance upto which it
can detect object located on the surface of the earth (B) In frequency modulation the amplitude of the high
(Radius of earth = 6.4 x 106 m) is (2012) frequency carrier wave is made to vary in proportion to
the amplitude of the audio signal.
(A) 80 km (B) 16 km (C) 40 km (D) 64 km
(C) In frequency modulation the amplitude of the high
frequency carrier wave is made to vary in proportion to
Q.10 The magnetic field in a travelling electromagnetic the frequency of the audio signal
wave has a peak value of 20 nT. The peak value of
electric field strength is : (2013) (D) In amplitude modulation the amplitude of the high
frequency carrier wave is made to vary in proportion to
(A) 3V/m (B) 6V/m (C) 9V/m (D) 12 V/m the amplitude of the audio signal

Q.11 The forward biased diode connection is  (2014) Q.16 Arrange the following electromagnetic radiations
per quantum in the order of increasing energy: (2016)
+2 V -2 V
(A)
A : Blue light B : Yellow light
+3 V -3 V C : X-ray D : Radio wave
(B)
(A) A, B, D, C (B) C, A, B, D
2V 4V
(C) (C) B, A, D, C (D) D, B, A, C

-2 V +2 V
(D)
2 6 . 7 6 | Semiconductor and Communication System

Q.17 Identify the semiconductor devices whose


characteristics are given below, in the order (a),(b),(c),(d) d
 (2016)
l l c

b
v v
a

(A) (B) x
(A) AND (B) OR
(A) Zener diode, simple diode, Light dependent
resistance, Solar cell (C) NAND (D) NOT
(B) Solar cell, Light dependent resistance, Zener diode,
simple diode Q.19 For a common emitter configuration, if α and β
have their usual meanings, the incorrect relationship
(C) Zener diode, Solar cell, Simple diode, Light between α and β is.  (2016)
dependent resistance
β β
(D) Simple diode, Zener diode, Solar cell, Light (A) α = (B) α =
1– β 1+ β
dependent resistance.
β2 1 1
(C) α = (D) = +1
2 α β
1+ β
Q.18 If a, b, c, d are inputs to a gate and x is its output,
then, as per the following time graph, the gate is:
 (2016)

JEE Advanced/Boards

Exercise 1 Q.3 For a transistor, β = 45 and voltage drop across


1kΩ which is connected in the collector circuit is 1 volt.
Find the base current for common emitter connection.
Semiconductor

Q.1 If resistivity of pure silicon is 3000 ohm-meter and Q.4 A silicon transistor amplifier circuit is given below:
the mobilities of electrons and holes are 0.12 m2 / V − s If the current amplification factor β =100 , determine:
and 0.025 m2 / V − s respectively, find
(a) Base current iB
(i) The resistivity of a specimen of the material when
1019 atoms phosphorous added per ms, (b) Collector current iO

(ii) The resistivity of specimen if further 2 × 1019 boron (c) Collector –emitter voltage
atoms per m3 are also added. (d) Collector base-voltage.
Take the voltage drop between drop between base and
Q.2 Determine the current i in the circuit shown in emitter as 0.7 V.
Figure Assume diodes are made of silicon (V0 = 0.7V). n-p-n
R1=8.6k R2=100
iB
2k D1 4V 10V

20.7V D2 5V iE
P hysi cs | 26.77

Q.5 Construct the truth table for the function Y of A and Q.20 State the function of a Zener diode in a circuit.
B represented by Figure.
Q.21 What type of charge is on n-type semiconductor?
On p-type semiconductor? Why?

A Y Q.22 Write down the truth table for a NOR gate.


B

Q.23 In half-wave rectification, what is the output


Q.6 If ni is density of intrinsic charge carriers; nh and ne frequency if the input frequency is 50Hz? What is the
are densities of hole and electrons in extrinsic output frequency of a full-wave rectifier for the same
semiconductor, what is the relation among them? input frequency?

Q.7 What type of impurity is added to obtain n-type Q.24 Which logic gate is represented by Boolean
semiconductor?
expression Y= A + B?

Q.8 Distinguish between n-type and p-type


semiconductors Q.25 Name the logic gate whose repetitive use can
make digital circuits.

Q.9 Name the fundamental gates.


Q.26 For a CE-transistor amplifier, the audio signal
voltage across the collected resistance of 2kΩ is
Q.10 What is the width of depletion layer in p-n junction 2V. Suppose the current amplification factor of the
diode? transistor is 100, find the input signal voltage and base
current, if the base resistance is 1kΩ .
Q.11 Show diagrammatically a forward biased and a
reverse biased p-n junction. Q.27 You are given the two circuits as shown in figure.
Show that circuit (a) acts as OR gate while the circuit (b)
Q.12 Distinguish between forward biasing and reverse acts as AND gate.
biasing in p-n junction. Discuss its use.
A
Q.13 What is the net charge on (i) p-type semiconductor Y Y
B
(ii) n-type semiconductor?

(a) (b)
Q.14 Name of the type of charge carriers in p-n junction
diode when forward biased?
A
Y Y
B
Q.15 Which type of biasing result in very high resistance of
a p-n-junction diode? Draw a diagram showing
(a) this bias. (b)

Q.28 Write the truth table for a NAND gate connected


Q.16 Draw the circuit diagram of a half wave rectifier
as given in figure Hence identify the exact logic
using a junction diode.
operation carried out by this circuit.

Q.17 Name the type of charge carriers in p-n-junction


A Y
when reverse biased.

Q.29 You are given two circuits as shown in figure which


Q.18 What are charge carriers in p-n-p transistor?
consist of NAND gates. Identify the logic operation
carried out by the two circuits.
Q.19 Draw a labeled circuit diagram showing use of p-n
junctions as full wave rectifier.
2 6 . 7 8 | Semiconductor and Communication System

A A (C) Recombination of electron and holes


Y Y
B (D) None of these
B

(a) (b) Q.3 When reverse bias in a junction diode is increased,


the width of depletion layer
A (A) Increase (B) Decreases
Y Y
(C) Does not change (D) Fluctuate
B

(b) Q.4 In the given transistor circuit, the base current is


35 µA. The value of Rb is
Q.30 In the following circuits shown if the input wave
from is as shown in figure, What will be the output E C
waveform (assume diode is ideal)
B RL

+5V Rb
7V
0V R
(A) 100kΩ (B) 200kΩ
-5V (C) 300kΩ (D) 400kΩ

Q.5 Given below are four logic gate symbol (figure).


+5V Those for OR, NOR and NAND are respectively
p
A y A y
0V (1) (2) B
B
n
A
(3) A
-5V y y
(4)
B B
(i) Across R (See figure)
(A) 1, 4, 3 (B) 4, 1, 2 (C) 1, 3, 4 (D) 4, 2, 1
(ii) Across the diode (in figure)
Assume that the diode is ideal. Q.6: The following truth table corresponds to the logic
gate

Exercise 2 A 0 0 1 1
B 0 1 0 1
Semiconductor X 0 1 1 1

Single Correct Choice Type (A) NAND (B) OR (C) AND (D) XOR

Q.1 A potential barrier of 0.50 V exists across a P-N Q.7 In the circuit shown in 12 V
junction. If the depletion region is 5.0 × 10−7 m wide, the figure, Voltage V0 is
intensity of the electric field in this region is-
(A) 1.0 × 106 V / m (B) 1.0 × 105 V / m
Si Ge
(C) 2.0 × 105 V / m (D) 2.0 × 106 V / m

V0 2.2k
Q.2 The main cause of avalanche breakdown is
(A) Collision ionization
(A) 11.7 volt (B) 11.3 volt
(B) High doping
(C) 0 (D) None
P hysi cs | 26.79

Q.8 Determine current I in the configuration


A
10K

10 V Si l 30 V 10K 10K
10
Si B

(A) 0 (B) 5 volt


(A) 1A (B) 0 A (C) Less than 1A (D) None
(C) 10 volt (D) 15 volt
Q.9 A cube of germanium is placed between the poles
of a magnet and a voltage is applied across opposite Multiple Correct Choice Type
faces of the cube as shown in Figure. Magnetic field is
directed vertical downward in the plane of the paper: Q.12 In a p-n junction-

Top p-Type germanium (A) New holes and conduction electrons are produced
continuously throughout the material
Front
(B) New holes and conduction electrons are produced
continuously throughout the material except in the
Applied voltage depletion region
(C) Holes and conduction electrons recombine
continuously throughout the material
What effect will occur at the surface of the cube? (D) Holes and conduction electrons recombine
continuously throughout the material except in the
(A) The top surface of cube will become negatively
depletion region
charged
(B) The front surface of the cube will become positively
Assertion Reasoning Type
charged
(A) Statement-I is true, statement-II is true and
(C) The front surface of the cube will become negatively
statement-II is correct explanation for statement-I.
charged
(B) Statement-I is true, statement-II is true and statement-
(D) Both top and front surface of cube will become
II is NOT the correct explanation for statement-I.
positively charged.
(C) Statement-I is true, statement-II is false.
Q.10 A full wave rectifier circuit along with the output (D) Statement-I is false, statement-II is true.
is shown in the following diagram. The contribution(s)
from the diode (1) is (are): Q.13 Statement-I: Conductivity
V Semiconductors decreases with increase in temperature.
Statement-II: More electrons go from valance band to
t
1 Output conduction band, with increase in temperature.
0

V Q.14 Statement-I: In semiconductors current is


obtained due to motion of electrons and holes.
A B C D
t Statement-II: Breaking up of covalent bond produces
0
holes in valence band and electrons in conduction
(A) C (B) A, C band.
20
(C) B, D (D) A, B, C, D

Q.11 In the given figure potential difference between 10


A and B is;
2V
2 6 . 8 0 | Semiconductor and Communication System

Q.15 Statement-I: Doping concentration is maximum


in emitter in(A)
transistor. (A)
(B) (B)
Statement-II: Maximum number of electrons flows
emitter to base in n-p-n transistor.

Comprehension(A) Type
(C) (D)
(B)
(C) (D)
The circuit shown in diagram contains two diodes each
with a forward resistance of 50 ohm and with infinite
reverse resistance of 50 ohm and with infinite reverse
resistance. If the battery voltage is 6V then. (D)
(C)
D1 150

D2 50
Q.2 In the circuit below, A and B represent two inputs
and C represents the output.

6V 100
The circuit represents (2008)
Q.16 Current through 100 Ω resistance is
A
(A) 0 (B) 0.02 amp
(C) 0.03 amp (D) None of these
B
Q.17 Current through 50 Ω resistance is
(A) 0 (B) 0.02 amp (A) NOR gate (B) AND gate
(C) 0.03 amp (D) None of these (C) NAND gate (D) OR gate

Q.3 A radar has a power of 1 Kw and is operating at a


frequency of 10 GHz. It is located on a mountain top
Previous Years’ Questions of height 500m. The maximum distance upto which it
can detect object located on the surface of the earth
Q.1 The logic circuit shown below has the input (Radius of earth = 6.4 × 106 m ) is  (2012)
waveforms ‘A’ and ‘B’ as shown. Pick out the correct (A) 80 km (B) 16 km
output waveform. (2009)
(C) 40 km (D) 64 km
A
Y
Q.4 To find the distance d over which a signal can be
B
seen clearly in foggy conditions, a railways engineer
uses dimensional analysis and assumes that the
distance depends on the mass density ρ of the fog,
Input A intensity (power/area) S of the light from the signal and
its frequency f. The engineer finds that d is proportional
to S1/n. The value of n is  (2014)
Input B
(A) 4 (B) 2 (C) 3 (D) 5

(A) (B)

(C) (D)
P hysi cs | 26.81

PlancEssential Questions
JEE Main/Boards JEE Advanced/Boards
Exercise 1 Exercise 1
Q.5 Q.14 Q.16 Q.1 Q.2 Q.4
Q.27 Q.30 Q.32
Q.26 Q.30
Exercise 2
Exercise 2 Q.5 Q.7 Q.8
Q.1 Q.5 Q.8 Q.11 Q.16 Q.17
Q.9 Q.11 Q.12
Q.17 Q.21
Previous Years’ Questions
Previous Years’ Questions Q.14
Q.1 Q.2 Q.3
JEE Main/Boards

Answer Key

Exercise 1

Semiconductor
Q.2 (i) AND gate
(ii) Truth Table of AND gate

A B Y
0 0 0
0 1 0
1 0 0
1 1 1

Q.4 Increases.
Q.5 Diodes (a), (c) and (d) are forward biased while (b) is reverse bias.
2 6 . 8 2 | Semiconductor and Communication System

Q.6

A B Output NAND gate, Y Final output,(Y)’=Y’


0 0 1 0
0 1 1 0
1 0 1 0
1 1 0 1

Q.7 NOR gate.


Truth Table of NOR gate

Input Output
A B Y
0 0 1
0 1 0
1 0 0
1 1 0

Communication Systems
Q.34 (i) 40 kW; (ii) 27.8 kW Q.35 8.31 A
Q.36 70.1% Q.37 0.648
Q.38 (i) 6.25 mW; (ii) 1.562 mW; (iii) 9.374 mW Q.39 88.8%
2
Q.40 3258 km

Exercise 2
Semiconductor

Single Correct Choice Type


Q.1 B Q.2 C Q.3 B Q.4 D Q.5 A Q.6 B
Q.7 C Q.8 A Q.9 D Q.10 D Q.11 B Q.12 C
Q.13 B Q.14 C Q.15 B Q.16 A Q.17 D Q.18 C
Q.19 A Q.20 B Q.21 A Q.22 A Q.23 D Q.24 B

Previous Years’ Questions


Q.1 A Q.2 A Q.3 A Q.4 D Q.5 C Q.6 A
Q.7 A Q.8 A Q.9 A Q.10 B Q.11 A Q.12 C
Q.13 B Q.14 B Q.15 D Q.16 D Q.17 D Q.18 B
Q.19 A, C
P hysi cs | 26.83

JEE Advanced/Boards
Exercise 1
Semiconductor
Q.1 (i) 5.2 Ω-m, (ii) 25 Ω-m Q.2 8 mA
Q.3 0.0222 mA. Q.4 (a) 0.5 mA, (b) 50 mA, (c) 5 V, (d) 4.3 V
Q.5

A B Y
0 0 0
0 1 0
1 0 1
1 1 1

Q.6 nenh = ni2 Q.7 Pentavalent atoms.



Q.9 OR, AND and NOT gates. Q.10 10−6 m
Q.13 (i) Zero (ii) Zero. Q.14 Majority change carriers: electrons and holes.
Q.17 Minority charge carriers: electrons and holes. Q.23 50 Hz for half-wave, 100 Hz for full –wave
Q.26 V=
1 0.01V;IB= 10µA
Q.28 NOT; Q.29 (a) AND (b) OR

A Y
0 1
1 0

Exercise 2
Semiconductor

Single Correct Choice Type


Q.1 A Q.2 A Q.3 A Q.4 B Q.5 C Q.6 B
Q.7 A Q.8 A Q.9 B Q.10 C Q.11 C

Multiple Correct Choice Type


Q.12 A, D

Assertion Reasoning Type


Q.13 D Q.14 A Q.15 C

Comprehension Type
Q.16 B Q.17 A

Previous Years’ Questions


Q.1 A Q.2 D Q.3 A Q.4 C
2 6 . 8 4 | Semiconductor and Communication System

Solutions

JEE Main/Boards

Exercise 1 Sol 6:

Semiconductor A B Output NAND gate, Final


Y output,(Y)’=Y’

Sol 1: The process of deliberately adding impurities to 0 0 1 0


a semiconductor is called doping. 0 1 1 0
The conductivity of extrinsic semiconductor is controlled 1 0 1 0
by the amount of doping, 1 part of a donor impurity
per 109 parts of germanium increases its conductivity 1 1 0 1
by a factor of nearly 103

y’ Sol 7:
A y A B Y' Y
Sol 2:
B 0 0 0 1
0 1 1 0
A B Y' Y
1 0 1 0
0 0 1 0
0 1 1 0 1 1 1 0

1 0 1 0 NOR gate
1 1 0 1 A
y= A  B
B
Y represents the AND gate output.
So this logic gate is AND gate So y is an output of NOR gate.

Sol 3: Refer page-10 Sol 8: Refer theory

Sol 4: The holes in the P-region are attracted towards Sol 9: Voltage gain: It is defined as the ratio of the
the negative terminal and the electrons in the N-region changes in the output voltage to the change in the
are attracted towards positive terminal. Thus the input voltage, and is denoted by AV
majority carriers more away from the junction. The ∆iC R out
depletion region increases for a reverse biased PN- AV = ×
∆iE R in
junction.
Sol 10:
Sol 5: (a) Forward biased
Forward Ge
(b) Reverse biased current
(c) Forward biased (mA)
V2(Ge)
(d) Forward biased
Forward voltage

reverse voltage
(A)
P hysi cs | 26.85

A y=A+B IBE
Sol 11: ( mA)
B

A
VBE (V)
Input characteristics
B

IC

out put
of OR gate
VCE=VCC–ICRC
Sol 12: Phase shift can be explained as follows
VCE
•• the input voltage and currents are in phase. Current amplification factor
•• the input and output currents are in phase. I
can be found by B = C
Therefore, output currents is in phase with the IB
input voltage.
Sol 17: (i) Indium is a group 13 element and arsenic is
•• An increase in output current results in a decrease
a group is element
in output voltage; and vice versa (as given Vout = VC
= VCC – ICRC)
P n

Sol 13: Refer theory

Sol 14: (a) Refer theory

(b) OR gate
y=A+B It’s reverse biased
A
B V2 V O

for input output waveform refer Q–11 Exercise-I in (ii)
sheet Reverse
I (A)
(c) Refer theory breakdown
(d) Photodiode can detect light of energy greater than
2.8 eV
12400 Sol 18: A common emitter amplifier is preferred over
Energy of incident light = = 0. 2 eV common base amplifier because of higher gain and
60000
high impedance in common emitter amplifier.
Photodiode will not detect light as energy of incident
light is less than band gap.
Sol 19: Refer theory
Sol 15: Refer theory
Sol 20: It’s because GaAs naturally performs better at
converting the sun’s energy into electricity than other
Sol 16: The base activates the transistor Base region is
materials. Further, GaAs solar calls deliver more energy
made very thin and lightly doped to make the transistor
in high heat or low light GaAs solar cells are highly
more efficient. This reduces the opportunity for an
efficient than any other solar cells.
electrons to recombine with a hole and be lost for the
circuit refer page 29 section 16.5
2 6 . 8 6 | Semiconductor and Communication System

Sol 21:
y1
A (1)
IC

y
Ib
y2 2

B (2) Ib
1

VCE
A B y1 y2 y current amplification factor
0 0 0 0 1 I
= C in this curve
1 1 1 1 0 IB
1 0 1 0 0
0 1 0 1 0 Sol 27: (i) P is NAND gate
Q is OR gate
so this is a NAND gate
A B X' x
Gates (1) and (2) have the same output as the input.
(ii) 0 0 1 1
1 1 0 1
Symbol

A x’
truth table x
B
A B Y
0 0 1
1 0 0
Sol 28: Refer theory
1 1 0
0 1 0
Sol 29: Refer theory
Sol 22: Refer theory
Sol 30:

Sol 23: NOR gate A B X Y Z


0 0 1 1 0
0 1 1 0 0
1 0 0 1 0
Sol 24: Refer theory
1 1 0 0 1
∆iC R out
Sol 25: Voltage gain AV = × Function of this logic circuit is same as that of AND gate
∆iB R in

Sol 26: A y
AND gate
IBE B

Sol 31: Refer theory

Sol 32: Refer theory

VBE Sol 33: Refer theory


Input resistance is the
reciprocal of the slope
Communication System
of this curve
Sol 34: (i) Since the carrier itself is unaffected by the
modulating signal, there is no change in the carrier
power level.
P hysi cs | 26.87

∴ PC =
40kW Sol 38: Figure (i) shows the spectrum of AM wave
whereas figure (ii) shows the equivalent circuit.
Carrier (40 kW)
RF Excitation v
Modulated RF 2.5V peak
amplifier AM wave Sideband (20 kW) 
Upper sideband
=72% 5V
5V peak
Carrier 
2.5V
Audio modulation + 2.5V peak
power 
f Lower sideband
0 fc - fs fc - fc+ fs
1 2 1 2
(ii) PS=
2
maPC=
2
(1) × 40= 20kW (I) (II)

(=
r.m.s.voltage ) ( 0.707 × peak value)
2 2

PS 20 =Power
∴ Paudio = = = 27.8 kW R R
0.72 0.72
(i) Power delivered by the carrier,
1
( 0.707 × 5)
2
Sol 35: PS = ma2PC
2 PC = 6.25 × 10−3 W =
= 6.25mW
2000
 m22 
PT = PC + PS = PC 1 + ma2a  (ii)Power delivered by lower sideband component,
PT = PC + PS = PC 1 + m2 
PT = PC + PS = PC 1 + m22a2a 
PT P= PC + PSm=22 PC 1I +m2a  m22 ( 0.707 × 2.5)
2
PT P=T P=+ C 1+ PSm= a PC   2 1 maa
∴ or 1IT+ =+ Plower = =1.562 × 10−3 W =1.562mW
∴ PT =+ 1 m2a2 or  IT  =+ 2 1 2
   m2 2000
∴P C
T
PCT =+ 1 m22a2a or  IICCTT  =+ 1 m22a2a
Given PCT =+ 1 mI2Ca or8A;
that  I m  =+ m2a
0.4

Given
=
that
1 I2C or8A;
=  CT m a1
∴ PC =+ = =

 IC  =+a 1 0.42 (iii) Power delivered by upper sideband component,
Given PC 2that I2C 0.48A;  )IC2 m  a 0.42
(
= = 2
 I 
1 II+C ( 0.4 )2 m
2
Given  IT  that
= 8A;
= 0.4
( 0.707 × 2.5 )
∴ = 2
Given
∴  I8T 2that
= = 1 + 8A;
=
C ( 0.4
2 ) maa 0.4
 2 1.562 × 10−3 W
∴  I8TT 2 = 1 + ( 0.4 2 )22 Pupper
= =
 I8T  = 21 + ( 0.4 2= )or I 8= 2000
or(I8=
∴ // 8 ) 1 + 1.08 1.08 8.31A
or(I8= )
2
∴ T 
 8 = 1.082= or ITT 8= 1.08 8.31A = 1.562mW
T  2
or(I= T / 8 )2
2
 1.08 = or IT 8= 1.08 8.31A
or (I= / 8 )2 1.08 = or IT 8= 1.08 8.31A Total power delivered by the AM wave
or (I= T / 8)
T 1.08 = or IT 8= 1.08 8.31A
=6.25+1.562+1.562=9.374 mW
2
I  m2a
Sol 36:  T  = 1 + Sol 39: Modulation factor, ma
 IC  2
=75%=75/100=0.75
GiventhatI
= T 8.93A;I
= C 8A;m
= a ? Total power in AM wave, PT =10kW
2
 8.93  m2a Total power in sidebands,
 1+
 = 1 + m2a / 2
or1.246 =
 8  2
 0.75 2 
m2a /2=
0.246 or ma =2 × 0.246 =
0.701 =
70.1% PS =
 m2 
PT  a
=10 × 
( ) = 2.19kW
 2
( )
2
 2 + ma   2 + 0.75 
2 Power in sideband,
PT m2  V  m2a
Sol 37: 1 + a or  T
=  1
= + P'S P=
PC 2 V 2 = S /2 2.19
= / 2 1.095kW
 C 
2PS 2 × 2.19
Giventhat
= VT 110V;
= VC 100V;m
= a ? PC
Power in carrier, = = = 7.786kW
2
m2a ( 0.75)2
 110  m2
∴  = 1 + m2a / 2
1 + a or1.21 = Power saved due to suppression of carrier and one
 100  2
sideband
or m2a /=
2 0.21or m=
a 0.21×=
2 0.648 =PC + P'S =7.786 + 1.095 =8.88kW
8.88
∴ %power saving = × 100 = 88.8%
10
2 6 . 8 8 | Semiconductor and Communication System

Sol 40: However, for line-of-sight (LOS) communication, Sol 8: (A) For intrinsic semiconductors fermi level of
the receiving antenna must be able to directly intercept energy lies in the middle of forbidden gap.
the signals radiated by the transmitting antenna.
Here, h=81m; Radius of earth, R=6.4X106 m Sol 9: (D) σ = ρ(mene + mhnh)
∴Radius (d) of the area covered by TV transmitting ne >>nh
antenna is σ 6.24
So, σ = emene ⇒ ne = =
ρµe 3900 × 1.6 × 10 –19
d= 2Rh = 2 × 6.4 × 106 × 81 = 3.2 × 10 4 m
= 1016/cm3
∴ Service area covered by TV transmitting antenna is

( )
2
A = π2 = π × 3.2 × 10 4 = 3258 × 106 m2 =3258 km2 Sol 10: (D) Resistance in forward bias is much less than
resistance in the reverse bias and the ratio of resistances
= 3258 km2 is approximately 1 : 104 (Theoretical fact)
Exercise 2
Sol 11: (B) Given:
Semiconductor - Diode’s Voltage Drop, Vd = 0.5V 

Single Correct Choice Type - Maximum Power Rating, P = 100mW = 0.1 Watts 
- Battery EMF, E = 1.5V 
Sol 1: (B) Eg = 0. 75 eV - Resistance = R 

Sol 2: (C) Minimum energy required to create e–h pair 1) Finding Current in the Circuit:

12400 So it’s given E = 1.5V and Vd = 0.5V 


= = 2.1 eV
5890 Formula: P = I*Vd => I = (P/Vd)
I = (0.1/0.5) = 0.2A
Sol 3: (B) On increasing temperature, energy of electron
increases so they can easily jump in conductor band so 2) Finding Potential Drop Across R 
resistance increase.
V = E – Vd = 1.5 - 0.5 = 1V
Sol 4: (D) In a good conductor, energy band gap is zero 3) Finding Value of R
as conduction and valence band overlap.
From above, 
Sol 5: (A) Movement of holes involve breaking and V = 1V and I = 0.2 A 
joining of bonds so mobility of holes is less than that V = IR R = V / I  = 1 / 0.2  = 5 ohms 
of electrons.
Hence the answer is B.
Sol 6: (B) No of electron and hole pairs are equal in
intrinsic semiconductors. Sol 12: (C) (i)

Sol 7: (C)

3R
Resistance =
2

Holes move in valence band in p-type semiconductors.


P hysi cs | 26.89

(ii) Sol 15: (B) It is a common-emitter amplifier circuit as


emitter is connected to both base and collector

Sol 16: (A)

y1
A
C
Resistance = R
y2
(iii) B

A B y1 y2 C
0 0 1 1 0
0 1 1 0 1 OR gate
1 0 0 1 1
1 1 0 0 1
Resistance = R

A y1
Sol 13: (B) C
B
A B Y1 C
0 0 1 0
AND gate
0 1 1 0
1 0 1 0
Sol 14: (C)
1 1 0 1


Sol 17: (D) A = B = C = 0
G1 = 0, C = 0, D = 1
For the upper half wave
A = B = 1, C = 0
G1 = 1, C = 0
D=1

Sol 18: (C)


1
0
Circuit is as shown in figure 0
B
0
0
1
A C
0
1
1

0
D 0
1
For lower half wave circuit is shown in figure.
So it is a full wave rectifier
2 6 . 9 0 | Semiconductor and Communication System

Sol 19: (A) A Y Diode 1 is reverse biased and diode 2 is forward bias

0 1 2
i= = 0. 1 A
1 0 20

So NOT gate Sol 21: (B) Frequency of 10 kHz cannot be used because
it will require very large transmitting antenna. Also
frequencies of 1 GHz and 1000 GHz cannot be used
Sol 20: (B) V
+ because these will penetrate through the ionosphere;

the ionosphere cannot reflect these high frequencies.

Sol 24: (D) Frequencies in UHF range (0.3 GHz to 3
GHz) normally propagate by means of space waves.
Therefore, choice (d) is correct.

R
1 VC
Q(t = CR) = Q0 e τ = VCe1 =
e Previous Years’ Questions
V
+ – Sol 1: (A) VB = (1/e)[(hc/λ) – f]
VP = (1/e)[(1240/550) – 2]eV = 0.2545 V
Vq = ( 1/e)[1240/450–2.5]eV = 0.255 V
Vr = (1/e)[(1240/350) – 3] eV = 0.5428 V
If n is the number of photons in unit time per unit area
R
then nhc/λ = I, (Intensity)
No current will flow so charge of the capacitor will not
change. ⇒ ip : iq : ir = np : nq : nr = lp : lq : lr

Sol 21: (A) Sol 2: (A)

0.7V 4.7K
V0 A B y y1 y2 y
12V V0 2
1
0.3V 0 0 1 1 1 0
0 1 1 1 0 1
1 0 1 0 1 1
V0 = 11.3V 1 1 0 1 1 0
1
V0 = 0.3 V
2
Sol 3: (B) When the multimeter is connected across P
Sol 22: (A) and Q, there is no conduction. Hence both P and Q are
2 20 n-type of p-type semiconductors. It is therefore, clear
that R is base. When common end in connected to R
and the other end connected to P or Q, conduction is
10 obtained. Hence it is clear that transistor is n-p-n and
R is base.
1

Sol 4: (D)

A B C

2V 0 0 0
P hysi cs | 26.91

(h + R)2 = d2 + R2+
0 1 1
1 0 1
⇒ d2 = h2 + 2Rh

1 1 1 Since h << R1 ⇒ d2 = 2 hR

⇒ d = 2(500)(6.4 × 106 ) =
80 = km
Sol 5: (C) Given figure is half wave rectifier
  
Sol 10: (B) E= B × C
Sol 6: (A)   
|=
E | | B | × | C | = 20 × 10-9 × 3 × 108 = 6 V/m.
Truth Table
A B Y Sol 11: (A)
1 1 1
1 0 0
0 1 0 p n
0 0 0 For forward Bias, p-side must be at higher potential
than n-side.
A
Sol 12: (C)
Y
fc = 2MHz = 2000 KHz
B
fm = 5KHz
Resultant frequencies are
Sol 7: (A) Truth table for given combination is
= fC + fm, fc, fc – fm
A B X = 2005 KHz, 2000, 1995 KHz
0 0 0
P
0 1 1
Sol 13: (B) Intensity I =
4 πr 2
1
1 0 1 I = ∈0 E02 × C
2
1 1 1
P 1
So = ∈0 E20 × C
This comes out to be truth table of OR gate 4 πr 2 2

2P 2 × 0.1× 9 × 109
= E20 =
Sol 8: (A) Since ionospheric properties change with 4 π ∈0 r 2C 1× 3 × 108
time, these signals are in general less stable than
ground wave signals. E
= =6 2.45 V / m
0

Sol 9: (A) Maximum distance on earth where object


can be detected is d, then Sol 14: (B) For conductor (Cu) resistance increases
linearly and for semiconductor resistance decreases
Exponentially in given temperature range.

Sol 15: (D) In amplitude modulation amplitude of


carrier wave (high frequency) is varied in proportion to
the amplitude of signal.

In frequency modulation frequency of carrier wave


(high frequency) is varied in proportion to amplitude
of signal.
2 6 . 9 2 | Semiconductor and Communication System

Sol 16: (D) 1


= e(mene + mhnh)
ρ
Y X U V l M R
1
= e(10 × 0. 12 + 1019 × 0. 025)
ρ
 increasing
1
= e × 1019 × 0. 025
Hence energy of radio wave will be minimum and ρ
maximum for X ray. ρ = 25 Ωm

Sol 17: (D) From standard data Sol 2:


- + 4V
Sol 18: (B) whenever we have 1 at input, output is 1. Reverse biased
So the gate is or
D2
1 1 Across D2 voltage at cathode in greater than voltage at
Sol 19: (A, C) = +1
α β anode so it will not conduct.
β -
α= 4.7 + 4
β +1 Forward biased
D1
Since, across D1 voltage at anode is greater than
JEE Advanvced/Boards cathode, so it will conduct.
Equivalent circuit is
Exercise 1
0.7V
Sol 1: me = 0. 12 m2/V–s 4.7 V
2KΩ +4V
mh = 0. 025 m2/V–s
20.7V
(i) When phosphorus is added
ne = 1019/m2
ne. nh = ni2 20.7 – 4.7 16
Current i = = = 8 ×10–3 A
2000 2000
ni = 1010
1020 1
nh = = 10 Sol 3: Collector current = = 1 mA
1000
1019
1 1 mA
= e(mene + mh. nh) base current = = 0. 0222 mA
ρ 45
= e(0.12 × 1019 + 0.025 × 10) Sol 4: Equivalent base circuit is
= 1.6 × 10–19 × 0.12 × 1019
8.6 K
1 0.7V
= 1.6 × 0.12
ρ
1 (a)
ρ= = 5.2 Ω m 5V
1.6 × 0.12

(ii) When further 2 × 1019 boron atoms are added


the some holes and electrons with recombine.
So finally 5 – 0.7
i= = 0.5 mA
8.6
nh = 1019/m2
nenh = 1020 (b) iC = biB = 100 × 0.5 mA = 50 mA
50
ne = 10/m2 (c) VCE = 10 – iCRC= 10 – × 100 = 5 V
1000
P hysi cs | 26.93

(d) VCB = VC – VB Sol 12: Refer theory


VCB = 5 – 0.7 = 4.3 V
Sol 13: Net charge is zero in both the cases but number
of electron and hole pairs are different in both.
Sol 5: A B y' y
0 0 0 0
Sol 14: Holes and electrons are majority change carriers
0 1 1 0
in P-type and N-type respectively.
1 0 1 1
1 1 1 1
Sol 15: Reverse biasing results in very high resistance
for diagram refer Q. – 11.

A y
Sol 16:
B y’
junction diode

Sol 6: ne . nh = ni2

Input output
Sol 7: Pentavalent atoms are added to obtain n-type A.C
~ P S RL
voltage
semiconductor

Sol 8: n-type extrinsic semiconductors with large


Transforms
electron concentration than the hole concentration
are known as n-type semiconductors, electrons are the
majority carriers and holes are the minority carriers Sol 17: Electrons and holes are minority change carriers
in the P & N regions respectively.
P-type semiconductors: As opposed to n-type
semiconductors, P-type semiconductors have a large
hole concentration than the electron concentration. Sol 18: Holes are the charge-carriers within the P–n–P
Holes are majority carriers and electrons are minority transistor.
carriers.
Sol 19: Refer page 25.17.
Sol 9: OR
Sol 20: Refer page 25.20.

AND
Sol 21: In n-type semiconductor the impurity atoms on
donating electrons becomes positive ions. However the
NOT
overall charge on the semiconductor is zero.

Sol 10: Width of depletion region is around 1 mm. In p-type semiconductor, the overall charge is zero. The
positive charge of majority hole pairs is balanced by the
Sol 11: E EB negatively charged semiconductors.
P-side N-side

- - + + Sol 22: A B y
+
- 0 0 1
+ - - -
+ +
- 0 1 0
- + +
- 1 0 0
- - + +
1 1 0
Depletion region
A
y
B
+ -
2 6 . 9 4 | Semiconductor and Communication System

Sol 23: In half wave rectifier frequency does not change


so frequency = 50 Hz Sol 29: y’ x
In full wave rectifier frequency becomes double so
frequency = 100 Hz This circuit act as NOT gate

A y= A B A y’
Sol 24: (a) y
B B

Sol 25: Any digital system can be achieved entirely A B y' y


from NAND or NOR gates. 0 0 1 0
0 1 1 0
Sol 26: RC = 2 kW 1 0 1 0
2 1 1 0 1
ic = = 1 mA
2kΩ
So this acts like AND gate
β = 100
y1
iC A
iB = = 10µA
β (b) y
y2
voltage = iBRB B
= 10 µA × 103
= 10 mV A B y1 y2 y
0 0 1 1 0
A y’
Sol 27: y 0 1 1 0 1
B 1 0 0 1 1
1 1 0 0 1
A B y' y
0 0 1 0 It acts as OR gate
0 1 0 1
1 0 0 1
Sol 30: (i) Diode will allow only positive voltage to pass
1 1 0 1
(ii) 5V
y corresponds to the output of OR gate

A O
A
y OV

B B
–5V

A B A B y
0 0 1 1 0
Exercise 2
0 1 1 0 0
1 0 0 1 0 Single Correct Choice Type
1 1 0 0 1
V
Sol 1: (A) Intensity of electric field E =
Output y is same of the output of AND gate d
0.5
A y = = 106 V/m
5 × 10 –7
Sol 28: 0 1
1 0
Sol 2: (A) In Avalanche breakdown the covalent bonds
It will function as NOT gate where the junction break down liberating a large
P hysi cs | 26.95

number of electron hole pairs. So the main cause is 10


I= = 1 amp
collision ionisation. 10Ω
→ →
Sol 3: (A) The holes in the p-region are attracted Sol 9: (B) f = q V × B so positive charge will accumulate
towards the negative terminal and the electrons in the on the front surface and negative charge on the back
N-region are attracted towards the positive terminal. surface.
Thus the majority carriers move away from the junction. front
The depletion region increases for a reverse biased PN- i
junction.

V 7 1
Sol 4: (B) Rb = = = ×106 = 200 KΩ B
ib 35 × 10 –6 5

Sol 5: (C)
OR gate Sol 10: (C) During negative half cycle diode 1 is forward
biased and diode 2 is reverse biased so output will be
due to diode 1 so it corresponds to B and D
NAND gate

Sol 11: (C) Diode is forward bias so it will conduct


NOR gate Equivalent resistance of circuit
10 × 10
Req = 10 + = 15KΩ
Sol 6: (B) A B X 10 + 10
30
0 0 0 i= = 2 mA
0 1 1 15KΩ
1 0 1 So, voltage difference between A and B is
1 1 1 VAB = 1 mA × 10 KΩ = 10 Volt

It corresponds to OR gate
Multiple Correct Choice Type

Sol 7: (A) Case-I both diodes conducts but it is Sol 12: (A, D) New holes and conduction electrons are
not possible as voltage at V can’t be 11.3 and 11.7 produced continuously throughout the material.
simultaneously.
Holes and conduction electrons recombine continuously
Case-II Only Si diode conduct. Then V0 = 11.3 Volt and throughout the material except in the depletion region
Ge diode should conduct so, it is not possible because due to the electric field holes and electrons
Case-III Only Ge diode conducts, Then V0 = 11.7 Volt become immobile in depletion region.
and Si diode is off in this condition. So this case is
possible. So V0 = 11.7 Volt. Assertion Reasoning Type

Sol 8: (A) I Sol 13: (D) Conductivity of semiconductors increases


Si as more electron goes from valence band to conduction
10V
10 bond with increase in temperature.
Si
Sol 14: (A) Holes move in the direction of electric
field and electrons move in the opposite direction. So
I holes and electrons both contribute for the current
10V Si in semiconductors. Covalent bond breaks up and
10 produces holes in valence band and electrons in
Si conduction band.
2 6 . 9 6 | Semiconductor and Communication System

Sol 15: (C) In transistors, maximum no of electrons Sol 2: (D)


flows from emitter to collector in n-p-n transistors. Only
a small fraction of electrons flow from emitter to base. A B C
0 0 0
Comprehension Type 0 1 1

Sol 16: (B) Diode D1 is forward biased and D2 is reverse 1 0 1


based so 1 1 1
50 150
Sol 3: (A)

50

6V 100

6
current = = 0. 02 A
300

Sol 17: (A) Current is zero across 50 Ω resistor. Maximum distance on earth where object can be
detected is d, then
(h + R)2 = d2 + R2
Previous Years’ Questions
⇒ d2 = h2 + 2Rh
Sol 1: (A) since h << R, ⇒ d2 = 2hR
Truth table
⇒d= 2(500)(6.4 × 106 ) = 80 km
A B C
1 1 1 Sol 4: (C) d ∝ ρx S yFz

1 0 0 [ML−3 ]x [MT −3 ]y [T −1 ]z
⇒ [L] =
0 1 0
⇒ x + y = 0, –3x = 1, –3y–z=0
0 0 0
–1 1
⇒ x= , y= , z= –1
3 3
A 1
⇒y=
3
Y
⇒n=3
B
2017-18 100 &
op kers
Class 12 T
By E ran culty
-JE Fa r
IIT enior emie .
S fP r es
o titut
Ins

PHYSICS
FOR JEE MAIN & ADVANCED
SECOND
EDITION

Exhaustive Theory
(Now Revised)

Formula Sheet
9000+ Problems
based on latest JEE pattern

2500 + 1000 (New) Problems


of previous 35 years of
AIEEE (JEE Main) and IIT-JEE (JEE Adv)

5000+Illustrations and Solved Examples


Detailed Solutions
of all problems available

Topic Covered Plancess Concepts


Tips & Tricks, Facts, Notes, Misconceptions,
Experimental Physics Key Take Aways, Problem Solving Tactics

PlancEssential
Questions recommended for revision
27. E X P E R I M E N TA L
PHYSICS

1. PREVIEW
This lesson aims to make the student familiar with the basic approach and observations of physics experiments and
activities. It covers the basics of all the physics experiments for class 11th and 12th based on the latest NCERT pattern.
This is indeed very beneficial for JEE aspirants as it has a direct 20% weightage in the JEE MAIN examination.
Therefore, it is recommended that the student goes through this material thoroughly for his/ her benefit in JEE
MAIN examination.

Experiment 1: Vernier Callipers – It is Used to Measure Internal and External Diameter and
Depth of a Vessel

What is a vernier caliper?


(a) A vernier caliper has two scales - one main scale and a vernier scale - which slides along the main scale. The
main scale and vernier scale are divided into small divisions of different magnitudes.
The main scale is graduated in cm and mm. It has two fixed jaws, A and C, projected at right angles from the
scale. The sliding vernier scale has jaws (B.D) projecting at right angles from it as also the main scale and a
metallic strip (N). The zero of the main scale C D
and the vernier scale coincide when the jaws
are made to touch each other. The jaws and S
metallic strip are designed to measure the
distance/ diameter of objects. Knob P is used M
to slide the vernier scale on the main scale.
Screw S is used to fix the vernier scale at a N
0 2 3 5 6 14 15
desired position.
(b) The least count of a common scale is 1mm.
It is difficult to further divide it to improve P Inset
the least count of the scale. A vernier scale 2 3
enables this to be achieved.
The difference in the magnitude of one main
A B
scale division (M.S.D) and one vernier scale
division (V.S.D) is known as the least count of Figure 27.1
the instrument, as it is the smallest distance
that can be measured using the instrument.
nV.S.D.= (n− 1)M.S.D.
2 7 . 2 | Experimental Physics

Procedure
(a) Measuring the diameter of a small spherical or cylindrical body.
(i) Keep the jaws of the Vernier Calipers closed. Observe the zero mark on the main scale; it must perfectly
coincide with that of the vernier scale. If not, account for the zero error for all observations to be made
while the instrument is being used. Look for the division on the vernier scale that coincides with a
division of main scale. Use a magnifying glass, if available and note the number of division on the vernier
scale that coincides with that on the main scale. Position your eyes directly over the division mark so as
to avoid any parallax error.
(ii) Gently loosen the screw to release the movable jaw. Slide it enough to hold the sphere/ cylindrical
body gently (without any undue pressure) between the lower jaws AB. The jaws should be perfectly
perpendicular to the diameter of the body. Now, gently tighten the screw in order to clamp the instrument
in this position to the body.
(iii) Carefully note the position of the zero mark of the vernier scale against the main scale. Usually, it will not
perfectly coincide with any of the small divisions on the main scale. Record the main scale division just
to the left of the zero mark of the vernier scale.
(iv) Start looking for exact coincidence of a vernier scale division with that of a main scale division in the
vernier window from the left end (zero) to the right. Note its number (say) N, carefully.
(v) Multiply ‘N’ by the least count of the instrument and add the product to the main scale reading noted
in step 4. Ensure that the product is converted into proper units (usually cm) for addition to be valid.
(vi) Repeat steps 3-6 to obtain the diameter of the body at different positions on its curved surface. Take
three sets of readings in each case.
(vii) Record the observations in the tabular form with proper units. Apply zero correction if required.
(viii) Find the arithmetic mean of the corrected readings of the diameter of the body. Express the results in
suitable units with appropriate number of significant figures.
(b) Measuring the internal diameter and depth of the given beaker (or similar cylindrical object) to find
its internal volume.
(i) Adjust the upper jaws ‘CD’ of the Vernier Calipers so as to touch the inner wall of the beaker without
exerting undue pressure on it. Gently tighten the screw to keep the Vernier Calipers in this position.
(ii) Repeat the steps 3-6 as in Part (a) to obtain the value of the internal diameter of the beaker/ calorimeter.
Do this for two different (angular) positions of the beaker.
(iii) Keep the edge of the main scale of Vernier Calipers on the peripheral edge of the beaker in order to
determine the depth of the beaker. This should be done in such a way that the tip of the strip is able to
go freely inside the beaker along its depth.
(iv) Keep sliding the moving jaw of the Vernier Calipers until the strip just touches the bottom of the beaker.
Take care that it does so while being perfectly perpendicular to the bottom surface. Now tighten the
screw of the Vernier Calipers.
(v) Repeat steps 4-6 of Part (a) of the expertiment to obtain the depth of the given beaker. Take the readings
for depth at different positions of the breaker.
(vi) Record the observations in tabular form with proper units and significant figures. Apply zero corrections,
if required.
(vii) Find out the mean of the corrected readings of the internal diameter and depth of the given beaker.
Express the result in suitable units and proper significant figures.
Follow similar procedure for calculating the external diameter and by adjusting the upper jaws CD of the
Vernier Caliper so as to touch the walls of the beaker from the outside.
P hysi cs | 27.3

Experiment 2: Screw Gauge – Its Use to Determine Thickness/ Diameter of Thin Sheet/
Wire

What is a screw gauge? D


0.5
C R
A B 5
With Vernier Calipers, you are usually able to 0
measure length accurately up to 0.1mm. More LS 95
ST S
accurate measurement of length up to 0.01mm or Sl CS
0.005mm may be made by using a screw gauge.
As such, a screw gauge is an instrument of higher
precision than Vernier Calipers. You may have
observed an ordinary screw [see Fig 27.2 (a)]. There
are threads on a screw. The separation between
U-Frame
any two consecutive threads is the same. The screw
can be moved backward or forward in this nut by p
rotating it clockwise or anticlockwise [Fig 27.2. (b)].
The distance generated by the screw when it
makes its one complete rotation is the separtation
between two consecutive threads. This distance is
called the pitch of the screw. Fig 27.2. (a) shows the (a) (b)
pitch (p) of the screw. It is usually 1mm or 0.5mm. A screw (a) without nut (b) with nut
The first Fig 27.2 shows a gauge. It has a screw
‘S’ which advances forward or backward as one Figure 27.2
rotates the head ‘C’ through rachet ‘R’. There is a
linear scale ‘LS’ attached to limb ‘D’ of the U frame. The smallest division on the linear scale is 1mm (in one type of
screw gauge). There is a circular scale ‘CS’ on the head which can be rotated. There are 100 divisions on the circular
scale. When the end ‘B’ of the screw touches the surface ‘A’ of the stud ‘ST’, the zero marks on the main scale and
the circular scale should coincide with each other.
ZERO ERROR: When the end of the screw and the surface of the stud are in contact with each other, the linear
scale and the circular scale reading should be zero. In case this is not so, the screw gauge is said to have an error
called zero error.
Fig 27.3. Shows an enlarged view of a screw gauge with its faces A and B in contact. Here, the zero mark of LS and
CS coincide with each other.

5
0
95

U-Frame
Figure 27.3
2 7 . 4 | Experimental Physics

When the reading on the circular scale across the linear scale is more than zero (or positive), the instrument has
Positive Zero Error, as shown in Fig 27.4. (a).
When the reading of the circular scale across the linear scale is less than zero (or negative), the instrument is said
to have Negative Zero Error, as shown in Fig 27.4. (b).

0 0 0 10
90 5
95 0

(a) (b)
Showing a positive zero error Showing a positive zero error

Figure 27.4

Taking the Linear Scale Reading: The mark on the linear scale which lies close to the left edge of the circular scale
is the linear scale reading. For example, the linear scale reading as shown in the Fig 27.5.

Taking Circular Scale Reading: The division of circular scale which coincides with the main scale line is the
reading of circular scale. For example, in the adjacent Fig 27.5, the circular scale reading is 2.

Total Reading
Wire
Total reading A B
= linear scale reading + circular scale reading 0 5
least count 0
95
= 0.5+2*0.001 ST S
= 0.502 cm

Figure 27.5

Principle: The linear distance moved by the screw is directly proportional to the rotation given to it. The linear
distance moved by the screw when it is rotated by one division of the circular scale is the least distance that can be
measured accurately by the instrument. It is called the least count of the instrument.
pitch
Least count =
No. of divisions on circular scale
For example, for a screw gauge with a pitch of 1mm and 100 divisions on the circular scale, the least count is
1mm/100=0.01mm
This is the smallest length that can be measured with this screw gauge.
P hysi cs | 27.5

(a) Measurement of Diameter of a Given Wire

Procedure
(i) Take the screw gauge and make sure that the rachet R on the head of the screw functions properly.
(ii) Rotate the screw through, say, ten complete rotations and observe the distance through which it has
receded. This distance is the reading on the linear scale marked by the edge of the circular scale. Then,
find the pitch of the screw, i.e., the distance moved by the screw in one complete rotation. If there are n
distance moved by the screw in one complete rotation. If there are n number of divisons on the circular
scale, then distance moved by the screw when it is rotated thorugh one division on the circular scale is
pitch
called the least count of the screw gauge, that is, Least count =
n
(iii) Insert the given wire between the screw and the stud of the screw gauge. Move the screw forward by
rotating the rachet till the wire is gently gripped between the screw and the stud as shown in the Fig
27.6. Stop rotating the rachet the moment you hear a click sound.
Take the readings on the linear scale and the A B A B
circular scale.
d1
(iv) From these two readings, obtain the diameter
of the wire. d1 d2

(v) The wire may not have an exactly circular cross


section. Therefore, it is necessary to measure
the diameter of the wire for two positions
at right angles to each other. For this, first (a) (b)
record the reading of diameter d1 [Fig 27.6 Figure 27.6
(a)] and then rotate the wire through 90 at
the same cross-sectional position. Record the
reading for diameter d2 in this position [Fig 27.6(b)].
(vi) The wire may not be truly cylindrical. Therefore, it is necessary to measure the diameter at several
different places and obtain the average value of the diameter. For this, repeat the steps 3-6 for three
more positions of the wire.
(vii) Take the mean of the different values of the diameter so obtained.
(viii) Substract zero error, if any, with proper sign to get the correct value for the diameter of the wire.
(b) Measurement of Thickness of a Given sheet

Procedure
(i) Insert the given sheet between the studs of the screw gauge and determine the thickness at five different
positions.
(ii) Find the average thickness and calculate the correct thickness by applying zero error by following the
steps followed earlier.

Experiment 3: Simple Pendulum – Dissipation of Energy by Plotting a Graph between


Square of Amplitude and Time

Principle: When a simple pendulum executes simple harmonic motion, the restoring force F is given by
F(t) = − kx(t)  (i)
Where x(t) is the displacement at time t and k = mg / L . The displacement is given by
=x(t) A0 cos ( ωt − θ )  (ii)
2 7 . 6 | Experimental Physics

Where ω is the (angular) frequency and θ is a constant. A0 is the maximum ..displacement in each oscillation,
which is called the amplititude. The total energy of the pendulum is given as
1
E = kA20  …(iii)
2
The total energy remains a constant in an ideal pendulum, because its amplitude remain constant.
But in a real pendulum, the amplitude never remains constant. It decreases with time due to serveral factors like
air drag, some play at the point of suspension, imperfection in rigidity of the string and suspension, etc. Therefore,
the amplitude of A0 falls with time at each successive oscillation. The amplitude becomes a function of time and
is given by
A(t) = A0 e−λt/2  …(iv)
Where A0 is the initial amplitude and λ is a contant which depends on damping and the mass of the bob. The
total energy of the pendulum at time t is then given by
1 2

= E(t) = kA (t) E0 e−λt  …(v)
2
Thus, the energy falls with time, because some of the energy is being lost due to the surroundings.
The frequency of a damped oscillator does not depend much on the amplitude. Therefore, instead of measuring
the time, we can also measure the number of oscillations n. At the end of n oscillations, t = nT , where T is the time
period. Then Eq. (v) can be written in the form En = E0 e−αn
where α = λT  …(vi)
and En is the energy of the oscillator at the end of n oscillations.

Procedure
(a) Find the mass of the pendulum bob.
(b) Fix a metre scale just below the pendulum, such that the zero mark of the scale is just below the bob at rest.
(c) When the pendulum oscillates, you have to observe the point on the scale above which the bob rises at its
maximum displacement. When doing so, do not worry about millimeter marks. Take observations only upto
0.5cm.
(d) Pull the pendulum bob so that it is above the 15cm mark.
Thus, the initial amplitude will
= be. A0 15cm
= atn 0 . Leave
the bob gently so that it starts oscillating.
2
An
2
(e) Keep counting the number of oscillations when the bob is at (m )
its maximum displacement on the same side.
(f) Record the amplitude An after every ten oscillations.
(g) Plot a graph of An2 versus n, and interpret the graph.
(h) Stick a piece of cotton or a small strip of paper to the bob n
so as to increase the damping, and repeat the experiment.
Figure 27.7

Experiment 4: Metre Scale – Mass of a Given Object by Principle of Moments

Principle: For a body free to rotate about a fixed axis in equilibrium, the sum of the clockwise moments is equal to
the sum of the anticlockwise moments.
If M1 is the known mass, suspended at a distance l 1 on one side from the centre of gravity of a beam and M2 is
the unknown mass, suspended at a distance l2 on the other side from the centre of gravity, and the beam is in
equilibrium, then M2l2 = M11
l
P hysi cs | 27.7

Procedure:
(a) Create a raised platform on a table. One can use a wooden or a metal block to do so. However, the platform
must be a sturdy. Place a wedge on a laboratory stand about 20cm above the table top. With the help of a
spirit level, set the level of the wedge horizontal.
(b) Use two loops of thread to suspend the unknown mass and the weights from the metre scale (beam). Insert
the loops at about 10 cm from the edge of the metre scale from both sides.
(c) Place the metre scale with thread loops on the wedge and adjust it till it is balanced. Mark two points on the
scale above the wedge where the scale is balanced. Join these two points with a straight line which would help
pin point the location of balance position even if the scale topples off the wedge for some reason. This line is
passing through the centre of gravity of the scale.
(d) Take the unknown mass in one hand. Pick a weight from the weight box that is nearly equal to the unknown
mass when it is held in the other hand.
(e) Suspend the unknown mass from either of the two Wedge
loops of thread attached to the metre scale. Suspend A G B
the known weight from the other loop as shown in
Fig 27.8.
(f) Adjust the position of the known weight by moving Unknown x y
the loop till the metre scale is balanced on the sharp Mass, m m Known standard
wedge. Make sure that in the balanced position, the W = mg mass
line drawn in Step 3 is exactly above the wedge. Also
ensure that the thread of the two loops passing over Figure 27.8: Experimental
Experimental set upset
forup for determination
determination of of
the scale is parallel to this line. mass of a given body
mass of a given body
(g) Measure the distance of the position of the loop from the line drawn in Step 3. Record your observations.
(h) Repeat the activity at least twice, each time with a slightly lighter and a heavier weight. Note the distance of
unknown mass and weight from line drawn in Step 3 in each case.

Experiment 5: Young’s Modulus of Elasticity of the Material of a Metallic Wire


Principle:
The apparatus works on the principle of Hookes’ Law. If l is the extension in a wire of length L and radius r due to
MgL
force F (=mg), the Young’s Modulus of the material of the given wire, Y, is Y =
πr 2l
Procedure:
(a) Suspend weights from both the hooks so that the two wires are stretched and become free from any kinks.
Attach only the constant weight W on the reference wire to keep it tight.
(b) Measure the length of the experimental wire from the point of its support to the point where it is attached
to the frame.
(c) Find the least count of the screw gauge. Determine the diameter of the experimental wire at about five points,
and at each point in two mutually perpendicular directions. Find the mean diameter and subsequently, the
radius of the wire.
(d) Find the pitch and the least count of the miocrometer screw attached to the frame. Adjust it such that the
bubble in the spirit level is exactly in the centre. Take the reading of the micrometer.
(e) Place a load on the hanger attached to the experimental wire and increase it in increments of 0.5kg. For each
load, bring the bubble of the spirit level to the centre by adjusting the micrometer screw and the note its
reading. Take precautions to avoid backlash error.
(f) Take about 8 observations for increasing load.
(g) Decrease the load in reductions of 0.5kg, and each time, take a reading on the micrometer screw as in step 5.
2 7 . 8 | Experimental Physics

Experiment 6: Surface Tension of Water by Capillary Rise and Effect of Detergents

Principle
When a liquid rises in a capillary tube [Fig 27.9], the weight of the column of the liquid of density ρ below the
meniscus is supported by the upward force of surface tension 0
acting around the circumference of the points of contact.
10
Therefore,
2πrT = πr 2hρg (approx.) for distilled water in contact with a clean 20
glass capillary. A M
hρgr 30
or T = C
2
where T = Surface tension of the liquid 40
P
h = Height of the liquid column and 50
r = Inner radius of the capillary tube B

Procedure
(a) Do the experiment in a well-lit place. For example, near a
window or use an incandescent bullb. Rise of liquid in capillary tube

(b) Clean the capillary tube and beaker successively in caustic Figure 27.9
soda and nitric acid and rinse thoroughly with water.
(c) Fill the beaker with water and measure its temperature.
(d) Clamp the capillary tube near its upper end, keeping it above the beaker. Set it up vertically with the help of a
plumbline held near it. Move down the tube so that its lower end dips into the water in the beaker.
(e) Push a pin P through a cork C, and fix it on another clamp such that the tip of the pin is just above the water
surface as shown in the Fig 27.9. Ensure that the pin does not touch the capillary tube. Slowly, lower the pin
till the tip just about touches the water surface. This can be done by matching the tip of the pin with its image
in water.
(f) Now, focus the travelling microscope M on the meniscus of the water in capillary A, and move the microscope
until the horizontal crosswire is tangential to the lowest point of the meniscus, which is seen inverted in M. If
there is any difficulty in focusing the meniscus outside the capillary tube, then focus it first, as a guide. Note
the reading of the travelling microscope.
(g) Mark the position of the meniscus on the capillary with a pen. Now carefully remove the capillary tube from
the beaker, and then the beaker without disturbing the pin.
(h) Focus the microscope on the tip of the pin and note the microscope reading.
(i) Cut the capillary tube carefully at the point marked on it. Fix the capillary tube horizontally on a stand.
Focus the microscope on the transverse cross section of the tube and take a reading to measure the internal
diameter of the tube in two mutually perpendicular directions.

AIM: To study the effect of detergent on surface tension of water by observing capillary rise.

Principle: Substances that can be used to separate grease, dust and dirt sticking to a surface are called detergents.
When added to water, detergents lower its surface tension due to additional intermolecular interactions.
The lowering of surface tension by addition of detergent in water can be observed by capillary rise method.
For a vertically placed capillary tube of radius r in a water – filled shallow vessel, the rise of water in capillary tube
2S cos θ hρgr
given by: h =
h (Fig. 27.10) is= or s
ρgr 2cos θ
P hysi cs | 27.9

r
Where S is surface tension of the water vapour film; θ is the contact
angle (Fig. 27.10), ρ is the density of water and g is the acceleration
due to gravity. For pure or distilled water in contact with a clean glass
1
capillary tube θ =00 or cos θ =1 . Thus, =S hρgr
2 h
Using this result, the surface tension of different detergent solutions
(colloidal) in water can be compared. In a detergent solution, the
capillary rise (or the surface tension) would be lower than that for pure Water
and distilled water. And an increase in a detergent’s concentration would
result in a further lowering of the rise of the solution in the capillary.
A detergent for which the capillary rise is minimum (or the one that Rise of water in capillary tube
causes maximum lowering of surface tension), is said to be more
Figure 27.10
cleansing.

Procedure
(a) Take a capillary tube of uniform bore, and clean and rinse it with distilled water. Use water to clean and rinse
the beaker as well. Pour enough water to fill half the beaker. Make sure that the capillary tube is dry and free
of grease, oil, etc. Also cleck that the top of the capillary tube is open and not blocked by anything.
(b) Take a plastic scale and mount the capillary tube on it
using rubber bands.
Rubber
Capillary tube
(c) Hold the scale with the capillary in the vertical position band
with the help of a clamp stand.
Scale h2
(d) Place the half-filled beaker below the lower end of the
scale and gradually lower the scale till its lower end is
h
immersed below the surface of water in the beaker as
shown in the Fig 27.11. Rubber
band h1
(e) Read the position of the water level inside and outside the
h h2 − h1 .
capillary tube on the scale. Let the position be=
Water with
(f) Rinse the capillary thoroughly in running water and dry it. detergent
(g) Take a small amount of the given detergent and mix it with
the water in the beaker. To study capillary in water and
(h) Repeat the experiment with detergent solution and find detergent mixed in it
the capillary rise again. Let it be h’. Figure 27.11

Experiment 7: Coefficient of Viscosity of a Given Viscous Liquid by Measuring Terminal


Velocity of a Given Spherical Body

Principle: When a spherical body of radius r and density σ falls freely through a viscous liquid of density ρ
and viscosity η with terminal velocity υ , then the sum of the upward buoyant force and viscous drag, force F, is
balanced by the downward weight of the ball (Fig 27.12).
Gravitational force = Buoyant force on the ball + viscous force

4 3 4
πr σg = πr 3ρg + 6πηr υ  … (i)
3 3
4 3
πr ( σ − ρ ) g
3= 2 r ( σ − ρ ) g 
2
or υ
= …(ii)
6πηr 9 η
2 7 . 1 0 | Experimental Physics

where υ is the terminal velocity, the constant velocity acquired by a body while
3
4/3r g
moving through viscous fluid under application of constant force.
The terminal velocity depends directly on the square of the size (diameter) of the
spherical ball. Therefore, if several spherical balls of different radii are made to fall
freely through the viscous liquid, then a plot of υ vs r 2 would be a straight line 6 r
as illustrated in Fig 27.12. 
3
υ 4/3r g
The shape of this line will give an average value of which may be used to find
the coefficient of viscosity η of r2
Figure 27.12

2 r2 2 ( σ − ρ ) g Nsm−2 (poise) 
the given liquid. Thus η=
9
( σ − ρ ) g. =
υ 9 ( slope of line )
…(iii)

The relation given by Eq. (iii) holds good if the liquid through which the spherical body falls freely is in a cylindrical
vessel of radius R>>r, and the height of the cylinder is enough to let the ball attain terminal velocity. At the same
time, the ball should not come in contact with the walls of the vessel.

Procedure: Graph between terminal υelocity υ , and square of radius of ball r 2


(a) Find the least count of the stop-watch.
(b) Note the room temperature, using a thermomerter.
(c) Take a wide bore tube of transparent glass or acrylic (of diameter Y
about 4cm and of length approximately 1.25m). Fit a rubber stopper R
at one end of the tube and ensure that it is air-tight. Fill it with the
given transparent viscous liquid (say glycerine). Fix the tube vertically  -1
in the clamp stand as shown in the Fig 27.13. Ensure that there are no (ms )
air bubbles inside the viscous liquid in the wide bore tube.
(d) Put three rubber bands A, B, and C around the wide bore tube, thus S
T
dividing it into four portions (Fig. 27.13), such that AB=BC, each
about 30cm. The rubber band A should be around 40cm below the
X
mouth of the wide bore tube (length sufficient to allow the ball to O 2 2
r (m )
attain terminal velocity).
Figure 27.13
(e) Separate a set of clean and dry steel balls of different radii. The set
should include four or five identical steel balls of same known radii
(r1 ) . Rinse these balls thorughtly with the experimental viscous liquid
(glycerine) in a petridish or a watch glass, else these balls may develop
air bubbles on their surface as they enter the liquid column.
(f) Fix a short inlet tube vertically at the open end of the wide tube
through a rubber stopper fixed to it. Alternatively, one may also use
a glass funnel instead of an inlet tube as shown in the Fig 27.14. With A
the help of forceps, hold one of the balls of radius r1 near the top of
tube. Allow the ball to fall freely. After passing through the inlet tube,
the ball will fall along the axis of the liquid column.
B
(g) Take two stop watches and start both of them simultaneously as the Viscous liquid
spherical ball passes through the rubber band A. Stop one the watches (glycerine)
as the ball passes through the band B. Allow the second stopwatch to
C
continue and stop it when the ball crosses the band C.
(h) Note the times t1 and t2 as indicated by the two stop watches. t1 is
then the time taken by the falling ball to travel from A to B and t2 is
the time taken by it in falling from A to C. If terminal velocity had been

Figure 27.14
P hysi cs | 27.11

attained before the ball crosses A, then t2 = 2t1. If it is not so, repeat the experiment with a steel ball of same
radii after adjusting the position of rubber bands.
(i) Repeat the experiment for other balls of different diameters.
( j) Obtain terminal velocity for each ball.
(k) Plot a graph between terminal velocity υ and square of the radius of spherical ball, r 2 . It should be a straight
line. Find the slope of the line and thus determine the coefficient of viscosity of the liquid using the relation
given by Eq. (iii).

Experiment 8: Plotting a Cooling Curve for the Relationship between the Temperature of a
Hot Body and Time

20 30 40 50 60 70 80 90 100

20 30 40 50 60 70 80 90 100
T2 T1
Desctription of Apparatus: As shown in the Fig 27.15, the
Newton’s law of cooling apparatus has a double-walled container, Stirrer
which can be sealed by an insulating lid. Water filled between these
double walls ensures that the temperature of the environment Lid
surrounding the calorimeter remains constant. Temperature of
the liquid and the calorimeter also remains constant for a fairly

10

10
long period of time, so that temperature measurement is feasible. Double walled
Temperature of water in calorimeter is feasible. Temperature of container
water in calorimeter and that of water between the double walls
of the container is recorded by two thermometers. Calorimeter

Theory: The rate at which a hot body loses heat is directly


proportional to the difference between the temperature of the
hot body and that of its surroundings, and depends on the nature
of the material and the surface area of the body. This is Newton’s Figure 27.15
law of cooling.
For a body of mass m and specific heat s, at its initial temperature θ higher than the temperature of its surroundings
dQ
θ0 , the rate of loss of heat is , where dQ is the amount of head lost by the hot body to its surrooundings in a
small interval of time. dt

Following Newton’s law of cooling we have


dQ
Rate of loss of heat, = −k(θ − θ0 )  … (i)
dt
dQ  dθ 
Also = ms    … (ii)
dt  dt 
Using Eqs and the rate of fall of termperature is given by
dθ k
dt
=−
ms
( θ − θ0 )  … (iii)

Where k is the constant of proportionality and k ' = k/ ms is another constant (The term ‘ms’ also includes the water
equivalent to the calorimeter with which the experiment is performed). Negative sign appears in Eqs. (ii) and (iii)
because loss of heat implies temperature decrease. Eq. (iii) may be rewritten as
dθ =− k ' ( θ − θ0 ) dt

on int ergrating, we get ∫θ−θ = − k ' ∫ dt
0

or ln ( θ − θ0=) loge ( θ − θ0=


) k 't + c
2 7 . 1 2 | Experimental Physics

or ln ( θ − θ0=) 2.303log10 ( θ − θ0=


) k 't + c  … (iv)

Where c is the constant of integration.


Eq. (iv) shows that the shape of a plot between log10 ( θ − θ0 ) and t will be a straight line.

Procedure T1
(a) Find the least counts of thermometers T1 and T2 . Take some water in a beaker and measure its temperature (at
room temperature θ0 ) with one (say T1 ) of the thermometers.
(b) Find the least count of the stopwatch/ clock by examining its functions.
(c) Pour water into the double-walled container (enclosure) at room temperature. Insert the other thermometer
T2 in the water contained in it, with the help of the clamp stand.
(d) Heat some water separately to a temperature of about 400 C above the room temperature θ0 . Pour hot water
in the calorimeter right to the top.
(e) Put the calorimeter filled with hot water back into the enclosure and cover it using the lid with holes. Insert
the thermometer T1 and the stirrer in the calorimeter through the holes provided in the lid.
(f) Note the initial temperature of the water between the enclosures of double wall with the thermometer T2 .
When the difference of readings of two thermometers . T1 and T2 is about 300 C , note the initial reading of the
thermometer.
(g) Keep on stirring the water gently and constantly. Note the reading of thermometer T1 – first, after about every
half a minute, then after about one minute and finally after two minutes duration or so.
(h) Keep on simultaneously noting the reading of the stopwatch and that of the thermometer T1 , while stirring
water gently and constantly, till the temperature of the water in the calorimeter falls to a temperature of about
50 C above that of the enclosure. Note the temperature of the enclosure, by the thermometer T2 .
(i) Record observations in tabular form. Find the excess of temperature ( θ − θ0 ) and also log10 ( θ − θ0 ) for each
reading, using logarithmic tables. Record these values in the corresponding columns in the table.
( j) Plot a graph between time t, taken along x-axis and log10 ( θ − θ0 ) taken along y-axis. Interpret the graph.

Plotting Graph

(i) Plot a graph between ( θ − θ0 ) and t as shown in the Fig 27.16, taking t along x-axis and ( θ − θ0 ) along
y-axis. This is called cooling curve.
(ii) Also plot a graph between log10 ( θ − θ0 ) and time t, as shown in Fig 27.16, taking time t along x-axis and
log10 ( θ − θ0 ) along y-axis. Choose suitable scales on these axes. Identify the shape of the cooling curve
and the other graph.
y y
Temperature,-0(oC)

Log10(-0)

x x
0 Time,t (min) 0
t (s)
Figure 27.16
P hysi cs | 27.13

Experiment 9: Speed of Sound in Air at Room Temperature Using a Resonance Tube

Principle: When a vibrating tuning fork of known frequency V is held over the
top of an air column in a glass tube AB (Fig 27.17), a standing wave pattern
can be formed in tube. Under the right conditions, a superposition between A A
a forward moving and reflected wave occurs in the tube to cause resonance.
This gives a very noticeable rise in the amplitude, or loudness, of the sound.
In a closed organ pipe like a resonance tube, there is a zero amplitude point
at the closed end (Fig 27.17). For resonance to occur, a node must be created
at the closed end and an antinode must be created at the open end. Let the
first loud sound be heard at length l1 of the air column [Fig 27.17. (a)], i.e.,
when the natural frequency of the air column of length l1 becomes equal to
the natural frequency of the tuning fork, so that the air column vibrates with
maximum amplitude. In fact, the length of the air column vibrates is slightly
longer than the length of the air column in tube AB.
λ B B
Thus, = l +e
4 1 (a) (b)
Where e (= 0.6r, where r = radius of the glass tube) the end correction for Figure 27.17
the resonance is tube and λ is the wavelength of the sound produced by
the tuning fork.
Now, on further lowering the closed end of the tube AB, let the second
11
resonance positon be heard at length l2 of the air column in the tube.

= l2 + e [Fig 27.18 (b)]. This length l2 would approximately be equal to
4 12
three quarters of the wavelength. That is, … (ii)

λ 2 (l2 − l1 ) 
Subtracting Eq. (1) from Eq. (2) gives = …(iii)
Thus, the velocity of sound in air at room temperature ( υ = vλ ) would be

=υ 2v (l2 − l1 ) .
B B
(a) (b)
Procedure Figure 27.18

Adjustment Of Resonance Tube: The apparatus usually consists of a narrow glass tube about a metre long and
5cm in diameter, rigidly fixed in its vertical position with a wooden stand. The lower end of this tube is attached to
a reservoir by a rubber tube. Using a clamp, the reservoir can be made to slide up or down along a vertical rod. In
order to keep the water level (or the length of air column) fixed in the tube, a pinch cock is provided with the rubber
tube. A metre scale is also fixed along the tube. The whole apparatus is fixed on a horizontal wooden base that can
be levelled using the screws provided at the bottom. There is water in both the reservoir and the tube. When the
reservoir is raised, the length of the air column in the tube goes up. Now:
(a) Set the resonance tube vertical with the help of a spirit level, and level the screws provided at the bottom of
the wooden base of the apparatus.
(b) Note the room temperature with a thermometer.
(c) Note the frequency v of given tuning fork.
(d) Fix the reservoir to the highest point of the vertical rod with the help of the clamp.
Determination of First Resonance Position
(e) Fill the water in the reservoir such that the level of water in the tube reaches the top.
(f) Close the pinch clock and lower the position of the reservoir on the vertical rod.
2 7 . 1 4 | Experimental Physics

(g) Gently strike the given tuning fork on a rubber pad and put it nearly one cm above the open end of the tube.
Keep both the prongs of the tuning fork parallel to the ground and place one above the other so that the
prongs vibrate in the vertical plane. Try to listen the sound being produced in the tube. It may not be audible
in the position.
(h) Slowly loosen the pinch cock to let the water level fall in the tube very slowly. Keep bringing the tuning fork
near the open end of the resonance tube; notice the increasing volume of the sound.
(i) Repeat steps 7 and 8 till you get the exact position of water level in the tube for which the intensity of sound
being produced in the tube is maximum. This corresponds to the first resonance postion or fundamental
node, if the length of the air column is minimum. Close the pinch cock at this position and note the positon of
the water level or length l1 of the air column in the tube. This is the determination of first resonance positions
while the level of water is falling in the tube.
( j) Repeat steps 5-9 to confirm the first resonance position.
(k) Next, find out the first resonance postion by gradually raising the level of water in resonance tube, and
holding the vibrating tuning fork continuously on top of its open end. Fix the tube at the position by gradually
raising the level of water in resonance tube, and holding the vibrating tuning fork continuously on top of its
open end. Fix the tube at the position where the sound of maximum intensity is heard.
Determination of second resonance position
(l) Lower the position of the water level further in the resonance tube by sliding down the position of the reservoir
on the vertical stand and opening the pinch cock till the length of the air column in the tube increases about
three times of the length l1 .
(m) Find out the second resonance position and determine the length of air column l2 in the tube with the same
tuning fork with frequency v1 and confirm the length l2 by taking four readings – two when the water level is
falling and the other two when the level of water is rising in the tube.
(n) Repeat steps 5-13 with a second tuning fork of frequency v 2 and determine the first and second resonance
positions.
(o) Calculate the velocity of sound in each case.

Experiment 10: Specific Heat Capacity of a Given (i) Solid and (ii) Liquid by Method of
Mixtures

Principle/ Theory: For a body of mass m and specific heat s, the amount of heat Q lost/ gained by it when its
temperature falls/ rises by ∆t is given by ∆Q = ms ∆t
Specific heat capacity: It is the amount of heat required to raise the temperature of unit mass of a substance
through 1 C. Its S.I. unit is Jkg−1K −1 .
Principle of calorimetry: If bodies of different temperatures are brought in thermal contact, the amount of heat
lost by the body at higher termperature is equal to the amount of heat gained by the body at lower temperature
at thermal equilibrium, provided no heat is lost to the surrounding.
(a) Specific heat capacity of given solid by method of mixtures.

Procedure
(i) Set the physical balance and make sure there is no zero error.
(ii) Weigh the empty calorimeter with a stirrer and lid with the physical balance/ spring. Ensure that the
calorimeter is clean and dry.
Note the mass m1 of the calorimeter. Pour the given water in the calorimeter. Make sure that the quantity
of water taken is sufficient to completely submerge the given solid in it. Weigh the calorimeter with
water along with the stirrer and the lid and note it’s mass m2 . Place the calorimeter in its insulating cover.
P hysi cs | 27.15

(iii) Dip the solid in water and take it out. Now shake it out to get rid of the water sticking to its surface.
Weigh the wet solid with the physical balance and note its mass m3 .
(iv) Tie the solid tightly with the thread in the middle. Ensure that it can be lifted by holding the thread
without slipping.
Place a 250mL beaker on the wire gauze kept on a tripod stand as shown in the Fig 27.19 (a).
Fill the beaker containing water by tying the other end of the thread to a laboratory stand. The solid
should be completely submerged in the water and should be atleast 0.5cm below the surface. Now, heat
the water with the solid suspended in it [Fig 27.19 (a)].

Thermometer
(0oC-100oC) in 0.5oC
Thread
Copper stirrer
Boiling water
Solid
(Metal piece) Felt or glasswool
Burner
Water

Calorimeter
(a) Outer jacket

(b)

Figure 27.19

(v) Note the least count of the thermometer. Measure the temperature of the water taken in the calorimeter.
Record the temperature t1 of the water.
(vi) Let the water in the beaker boil of about 5-10 minutes. Now measure the temperature t2 of the water
with the other thermometer, and record the same. Holding the solid with the thread tied to it, remove
it from the boiling water, and shake it to get rid of the water on it and quickly put it the water in the
calorimeter, immediately replacing the lid [Fig. 27.19(b)]. Stir the water with the stirrer. Measure the
temperature of the mixture till it becomes constant. Record this temperature as t3 .
(b) Specific heat capacity of given liquid by method of mixtures.

Procedure
(i) Set the physical balance and make sure there is no zero error.
(ii) Weigh the empty calorimeter with the stirrer and the lid with the physical balance/ spring balance.
Ensure that the calorimeter is clean and dry. Note the mass m1 of the calorimeter. Pour the given liquid in
the calorimeter. Make sure that the quantity of liquid taken would be sufficient to completely submerge
the solid in it. Weigh the calorimeter with liquid along with the stirrer and the lid and note its mass m2 .
Place the calorimeter in its insulating cover.
(iii) Take a metallic cylinder whose specific heat capacity is known. Dip it in water in a container and shake
it to get rid of the water sticking to its surface. Weigh the wet solid with the physical balance and note
down its mass m3
(iv) Tie the solid tightly with the thread at its middle. Make sure that it can be lifted by holding the thread
without slipping.
2 7 . 1 6 | Experimental Physics

Place a 250 ml beaker on the wire gauze kept on a tripod stand as shown in Fig 27.19 (a). Fill the beaker
halfway with water. Now suspend the solid in the beaker containing water by tying the other end of the
thread to a laboratory stand. The solid should be completely submerged in water and should be atleast
0.5cm below the surface. Now heat the water with the solid suspended in it.
(v) Note the least count of the thermometer. Measure the temperature of the water taken in the calorimeter.
Record the temperature t1 of the water.
(vi) Let the liquid in the beaker boil for about 5-10 minutes. Now measure the temperature t2 of the liquid
with the other thermometer and record the same. Holding the solid with the thread tied to it, remove it
from the boiling water, shake it to get rid of the excess water on it and quickly put it in the liquid in the
calorimeter, replacing the lid immediately. Stir with the stirrer. Measure the temperature of the mixture
becomes constant. Record this temperature as t3 .

Experiment 11: Resistivity of the Material of a Given Wire Using Meter Bridge

Description of Apparatus Meter Bridge


It consists of one metre long constantan wire ‘AC’ of uniform cross-sectional area, mounted on a wooden board
with a scale (Fig 27.20). The two ends of the wire are attached to terminals A and C. Thick metal strips bent at right
angles are used to provide two gaps E and F to connect resistors forming a Wheatstone bridge (Fig 27.20). The
terminal B between the gaps is used for connecting the galvanometer, and the other end of the galvanometer is
connected to a jockey J.
K1
+ - ()
Rh
R
S
RBOX
B
E F

0 10 20 30 40 50 60 70 80 90 100
Q D P
Jockey J
A C
lcm (100-l)cm
Figure 27.20

Principle: A meter bridge works on the principle of Wheatstone’s bridge. As shown in the Fig 27.21, it consists
of four resistors P, Q, R and S connected in the form of a network ABCD. The terminals A and C are connected to
two terminals of a cell through a key K1 . Terminals B and C are connected to a sensitive galvanometer G through
a key K 2 .

If there is no deflection in the galvanometer G,


P R
Then balance condition for Wheatstone’s bridge is =
Q S
We use this relation to determine S, as P, Q and R are known. The unknown resistance S is connected in the gap
E and a resistance box (RBOX ) in gap F of the meter bridge. The terminal B is connected to one terminal of the
galvanometer G. The other terminal of the galvanometer is connected to a jockey J which slides along the wire AC.
A source of dc voltage is connected between A and C through a key K1 so as to provide a constant potential drop
along AC (Fig 27.21).
P hysi cs | 27.17

A resistor (or wire) of known resistance is inserted in the gap F by taking out B
corresponding key from the resistance box (RBOX ) . The jockey is moved on the
wire AC to obtain a condition of no-deflection in the galvanometer. It happens
P Q

()
when the jockey is kept at a point D called the null point. In this condition: i1 K2
i1
P R Resis tance wire of length DC A
= = G C
Q S Resis tance of wire of length AD
i2
Unknown resistance S of the wire, having uniform cross-sectional area, is then R S
i2
l
given by  S = R  … (i) i i
100 − l D
The reason for this is that for a wire of uniform cross-sectional area, resistance is K1
proportional to length. + - ()
Thus, knowing l and R, and using Eq. (1), the unknown resistance S can be Figure 27.21
determined.
a
Resistivity: The specific resistance or resistivity ρ of the material of the given wire is ρ =S where S is the resistance
l
of the wire of length L and a = πr 2 (r being the radius) is the area of cross-section.

Procedure
(a) Find the average diameter of the wire with a screw gauge. From this, obtain the value of its radius r.
(b) Clean the insulation at the ends of the connecting wires with a piece of sand paper. Tighten all plugs of the
resistance box (RBOX ) by pressing each plug.
(c) Set up the circuit as shown in Fig 27.21 with unknown resistance wire of known length in gap E.
(d) Next, introduce some resistance R in the circuit from the resistance box. Bring the jockey J in contact with
terminal A first and then with terminal C. Note the direction in which pointer of the galvanometer gets
deflected in each case. Make sure that the jockey remains in contact with the wire for a fraction of a second.
If the galvanometer shows deflection on both sides of its zero mark for these two points of contact of the
jockey, null point will be somewhere on the wire AC. If it is not so, adjust resistance R so that the null point is
somewhere in the middle of the wire AC, say between 30cm and 70cm.
(e) If there is one-sided deflection, check the circuit again, especially junctions, for their continuity.
(f) Repeat step 4 for four different values of resistance R.
(g) Interchange the position of the resistances S and R and repeat steps 4 to 6 for the same five values of R. While
interchanging S and R, ensure that the same length of wire of resistance S is now in the gap F. The interchange
takes care of unaccounted resistance offered by terminals.

Experiment 12: Resistance of a Given Wire Using Ohm’s Law

Principle: Ohm’s Law states that the electric current flowing through a conductor is directly proportional to the
potential difference across its ends, provided that the physical state of the conductor remains uncharged.
If l is the current flowing through the conductor and V is the potential difference across its ends, then according to
Ohm’s Law
V ∝ I and hence V=RI  … (i)
Where R is the constant of proportionality, and is termed as the electrical resistance of the conductor. If V is
expressed in volts and I in amperes, then R is expressed in Ohm’s. The resistance R depends upon the material
and dimensions of the conductor. For a wire of uniform cross-section, the resistance depends on the length l
2 7 . 1 8 | Experimental Physics

and the area of cross-section A. It also depends on the temperature of the conductor. At a given temperature,
the resistance
l
R = ρ  … (ii)
A
Where ρ is the specific resistance or resistivity and is characteristic to the material of wire.
Combining Eqs. (1) and (2), we have
y
l
V = ρ I  … (iii)
A

(Ampere)
A linear relationship is obtained between V and l, i.e., the graph A
between V and l will be a straight line passing through the origin
as shown in Fig 27.22. The slope of the graph is 1/ R from Eq. 1 I
(equation of straight line passing through origin is y = mx where m
is the slope of graph)
1 1
Slope= ⇒ R= C B
R slope
O x
If l is the length of wire then the resistance per unit length of the V(Volt)
R
wire . Figure 27.22
l

Procedure
(a) Clean the ends of the connecting wires with the help of sand paper in order to remove any insulating coating
on them.
E
(b) Connect various components – resistance, rheostat, + - K Rh
battery, key voltmeter and ammeter as shown in Fig 27.23. ( )
(c) Note whether pointers in milliammeter and voltmeter +
coincide with the zero mark on the measuring scale. If mA
it is not so, adjust the pointer to coincide with the zero V
mark by adjusting the screw provided near the base of the - + -
needle using a screw driver.
(d) Note the range and least count of the given voltmeter and
milliammeter. R
Circuit to find the relation between
(e) Insert the key K and slide the rheostat contact to one of
current l and potential difference,
its extreme ends, so that the current passing through the
V for a give wire
resistance wire is minimum.
Figure 27.23
(f) Note the milliammeter and voltmeter readings.
(g) Remove the key K and allow the wire to cool, if heated. Once again, insert the key. Shift the rheostat contact
slightly to increase the applied voltage. Note the milliammeter and voltmeter reading.
(h) Repeat step 7 for four different settings of the rheostat. Record your observation in a tabular form.

Experiment 13: Potentiometer


(a) Comparison of EMF of Two Primary Cells
In the circuit shown, the cell of emf E1 is balanced for a length l1 on the potentiometer wire by connecting A to C.
In the next event, the cell of emf E2 is balanced for a length l2 on the potentiometer wire by connecting B to C. Let
E l1
x be the potential gradient across the potentiometer wire.= Then E1 l1= x and E2 l2 x ∴ 1 =
E2 l2
P hysi cs | 27.19

The e.m.f of a cell can be determined only when e.m.f of another cell K Rb
+ - ()
is known to us.
(b) Determination of Internal Resistance of a Cell
P Q
Principle: When a resistance R is connected across a cell of emf E and
E1
E
internal resistance r, then the current l in the circuit is I =  … (i) + - A
R +r
C G
The potential difference V (=Rl) across the two terminals of the cell is + - B
E
V= R  … (ii) E2
R+r
Figure 27.24
E r E  E l0
Thus = 1 + or =r  − 1 R Also =  … (iii)
V R V  V l

l0 − l
From the above equations, r = R  … (iv)
l

Procedure
(a) Connect different electrical components R K2 P
as shown in the circuit (Fig 27.25.). After RBOX1 () RBOX1
checking the circuit connections, close key E
K1 . - + K3
+ - ()
(b) With keys K 2 and K 3 open and a protective A
E1 G
high resistance P form the RBOX 2 , find the
position of the balance point. For final A
K1
()

reading, short circuit the resistance P by J

closing the key K 3 and find the balance


length l0 . Rh
(c) Take R R
= ( )
10Ω from RBOX1 , close the key
= 10
B
K 2 and quickly measure the new balance
Figure 27.25
length l. Open K 2 as soon as this has been
done.
(d) Keep the reading in the ammeter constant throughout the above observation.
(e) Reduce the value of R in equal steps of 1 Ω and for each value of R obtain the balance length l.
(f) At the end of the experiment, open key K 2 and repeat step 2 to find l0 again.

Experiment 14: Resistance and Figure of Merit of a Galvanometer by Half Deflection


Method

Principle Galvanometer: Galvanometer is a sensitive device used to detect very low current. Its working is based
on the principle that a coil placed in a uniform magnetic field experiences a torque when an electric current is set
up in it. The deflection of the coil is determined by a pointer attached to it, moving on the scale.
When a coil carrying current I is placed in a radial magnetic field, the coil experiences a deflection θ which is
related to I as I = kθ  ... (i)
Where k is a constant of proportionality and is termed as Figure of merit of the galvanometer.
2 7 . 2 0 | Experimental Physics

The circuit arrangement required for finding the resistance G of the galvanometer by half deflection method is
shown in Fig 27.26.

High resistance box


E K1
- + RBOX1 ()
R

l l
G

lg

Low resistance box


RBOX2 ()
l-lg
K2

Figure 27.26

E
When a resistance R is introduced in the circuit, the current Ig flowing through it is given by Ig =  … (ii)
R+G
In this case, the key K 2 is kept open. Here E is the emf of battery, G is the resistance of the galvanometer whose
resistance is to be determind.
If the current Ig produces a deflection θ in the galvanometer, then from equation (i) we get Ig = kθ  … (iii)
E
Combining equations (ii) and (iii) we get = k  … (iv)
R+G
On keeping both the keys K1 and K 2 closed and by adjusting the value of shunt resistance S, the deflection of
1
the galvanometer needle becomes (half). As G and S are in parallel combination and R in series with it, the total
2
GS
resistance of the circuit. R=' R +  … (v)
G+S
E
The total current, I due to the emf E in the circuit is given by I =  … (vi)
GS
R+
G+S
IS
If l'g is the current through the galvanometer of resistance G, then Gl'=
g S l − l'g Ig = ( ) G+S
 … (vii)

Substituting the value of l from Equation (v), in equation (vii) the current l'g is given by

IS E S ES
I'g
= = ⋅ ⇒ I'g =  … (viii)
G+S GS G + S R G + GS + RS
R+
G+S
For galvanometer current l'g if the deflection through the galvanometer is reduced to half of its initial value

1 θ ES
= then
= I'g k=
 
2   R ( G + S ) + GS
2

Ig E R ( G + S ) + GS
On dividing Eq. (ii) by Eq. (viii), = × 2
=
I'g R + G ES
P hysi cs | 27.21

⇒ R(G+ S) + GS
= 2S(R + G)
⇒ RG =RS + GS
⇒ G(R − S) =
RS
RS
or, G =  … (ix)
R −S

By knowing the values of R and S, the galvanometer resistance G can be determined. Normally R is chosen very
high (10kΩ ) in comparison to S (100 Ω ) .
The Figure of merit (k) of the galvanometer is defined as the current requied for deflecting the pointer by one
division.
For determining the Fig of merit of the galvanometer the key K 2 is opened in the circuit arrangement. Using Eqs
(ii) and (iii) the Figure of merit of the galvanometer is given by
1 E
k=  … (x)
θR+G
By knowing the values of E, R, G and θ the Figure of merit of the galvanometer can be calculated.

Procedure
(a) Clean the connecting wires with sand paper and make neat and tight connections as per the circuit diagram.

(
(b) From the high resistance box RBOX 1 ) (1 − 10kΩ ) , remove 5kΩ key and then close the key K1. Adjust the
resistance R from this resistance box to get full scale deflection on the galvanometer dia. Record the values
of resistance, R and deflection θ .
(c) Insert the key K 2 and keep R fixed. Adjust the value of shunt resistance S to get the deflection in the
galvanometer which is exactly half of θ . Note down S. Remove plug K 2 after noting down the value of shunt
resistance, S.
(d) Take five sets of observations by repeating steps 2 and 3 so that θ is even number of divisions and record
the observations for R,

1
S, θ and in tabular form.
2
(e) Calculate the galvanometer resistance G and Figure of merit k of galvanometer using Eqs (ix) and (x)
respectively.

Experiment 15: Focal Length of Different Mirrors


(i) Convex Lens (ii) Convex Mirror (iii) Concave Mirror Using Parallax Method

Convex lens

Parallax: This is employed in the location of image of an object. For example, as shown in the Fig.27.27 (a) O and
I are the object and image points for a mirror / lens.
An object point O and its real image ‘I’ are conjugate points i.e., any of the two may be considered as object and the
other as its image. Thus it helps in accurate adjustment to check for no parallax at both the points. If we say there
is no parallax between an object O (pin) and its image I, then by moving the eye through which we are observing
to the left and then to the right, object and its image both appear to move together relative to the lens / mirror.
It implies that the position of both are same on the optical bench [Fig 27. 27. (d) and (e)]. If their positions are not
same then in one position they may appear to coincide and in another they will appear separate [Fig 27. 27. (b)
and (c)].
2 7 . 2 2 | Experimental Physics

This method of locating the position of an image on the optical bench by a pin, is called the method of parallax.

Y
(a) X
O l (d) Y
X C F P

Y
(b) X V
O l l’
V’
L
(e) X Y
F P
C u’
P1
P2
(c) X Y
O l u
l’

L
Figure 27. 27

Principle: For an object placed at a distance u from the pole of a concave mirror of focal length f, the image is
formed at a distance v from the pole. The relation between these distances (for a concave mirror) is

1 1 1 uv
= + or f=
f u v u+ v

If an object (say, a pin) is placed in front of the reflecting surface B


of the concave mirror such that the object’s position lies in
between the principal focus of the mirror, F and the centre
A’
of curvature C, then a real, inverted and magnified image is
2F A
formed in between the centre of curvature C of the mirror and F
infinity (Fig 27.28).
Thus, the image formed in such a case would be clearer and
easier to be seen. The focal length of the mirror, using the B’
above relation, can be determined by placing the object in
between the point 2F and focus F. Figure 27.28

Procedure
(a) Obtain approximate value of the focal length of the concave mirror, by focusing the image of a distant
object. Obtain bright and clear image of distant building or tree on a plane wall or a sheet of paper and
measure the distance between the mirror and the image which gives the approximate focal length of the
concave mirror.
(b) Place the optical bench on a rigid table. Make it horizontal using a spirit level and levelling screws.
(c) Clamp the concave mirror on an upright and mount it vertically near one end of the optical bench. Move an
object pin P1 on the optical bench back and forth so that its image is formed at the same height. Make slight
adjustments of the height of the pin or the mirror inclination. This procedure ensures that the principal axis of
the mirror is parallel to the optical bench.
P hysi cs | 27.23

(d) Place another vertically mounted sharp and bright pin P2 in front of the reflecting surface of the concave
mirror. Adjust the pins P1 and P2 so that the height of the tips of these pins become equal to the height of the
pole P of mirror from the base of the optical bench.
(e) To determine index correction, a thin straight index needle is placed so that its one end A1 touches the tip of
the pin and the other end B1 touches the pole P of the mirror. The positions of the uprights are read on the
scale. Their difference gives the observed distance between the tip of the pin and the pole of the mirror. Lenth
of the needle A1B1 is measured by placing it on the scale which is the actual distance between the points in
question. The difference between the two gives the correction to be applied to the observed distance. Find
the index correction for both the pins P1 and P2 for all measurements.
(f) Move the pin P1 away from the mirror and place it almost at 2F. An inverted image of same size as the pin
should be visible.
(g) Now place another pin P2 on the bench. Adjust its height to be almost the same as the earlier pin. Place a piece
of paper on the tip of one pin, take this as the object pin.
(h) Place the pin with paper at a distance lying between F and 2F.
(i) Locate the image of the pin using the other pin. Remember that parallax has to be removed between the
image and the pin.
( j) Note the values of u and v i.e. the distances of the object and image pins from the mirror respectively.
(k) Repeat the experiment for at least five different positions of the object and determine the corresponding
values of V. Record your observations in tabular form.
(l) After doing index correction, record the corrected values of u and v. Find the value of focal length, f.

Focal length of a convex mirror: Since a convex mirror L


always forms a virtual image, its focal length cannot be M
found directly. For this purpose, an indirect method is
used as described below.
O C
An auxiliary convex lens L is introduced between the P
convex mirror M and object needle O. Now the object
needle is kept at a distance which is roughly 1.5 times
the focal length of the convex lens. The position of the
convex mirror is now adjusted, such that a real inverted R
image of object needle O is formed at O itself. This will
be possible only when the rays incident on the convex Figure 27.29
mirror are incident normally on it, hence retrace their
path and when produced rightward must pass through +Y
the centre of curvature C of the mirror.
To locate the position of C, convex mirror is removed
(without disturbing the object needle O and convex -X u(cm) O
B
lens L). An image needle I is put behind the convex lens o
45
and moved to a position at which there is no parallax
between tip of the inverted image of the object needle
I gives position of centre of curvature C of the mirror M. D
Q
R PI
Then, PC=PI=R and =
f = (2f, 2f)
2 2
V(cm)
Focal length of concave mirror: We can calculate the
focal length of a concave mirror by using the following
three graphical methods.

-Y
Figure 27.30
2 7 . 2 4 | Experimental Physics

Method 1: u-v Method


Let us select a suitable but the same scale to represent u along-X axis and v along-Y axis. According to sign
convention, in this case, u and v are both negative. Plot the various points for different sets of values of u and v
from the observation table. The graph then comes out to be a rectangular hyperbola as shown.
Draw a line OQ making an angle of 45° with either axis and L
meeting the curve at point Q. Draw QB and QD perpendicular M
on X and Y axes respectively. The values of u and v will be same
for point Q. So the coordinates of point Q must be (2f, 2f). This O
P
is due to the fact that, for a concave mirror, u and v are equal
only when the object is placed at the centre of curvature. So,
u=v=R=2f
1 1 1 2 Figure 27.31
From mirror formula applied to point Q + = = ;
u v f R
1 2 2 u v
Since u=v ∴ = = ∴ f= or
f u v 2 2
Hence, half the values of either coordinates of Q (i.e., distance OD or OB) gives the focal length of the concave
mirror.
OD
f= − −...cm =
= f1(say);
2
OB
Also f=− = −...cm = f2 (say)
2
f +f 
So, mean value of f =  1 2  = −...cm
 2 

Method 2: u-v Graph Method


Select a suitable but the same scale to represent u along X axis (or X’ axis) and v along-Y axis (or Y’ axis). Mark the
points at distance u1, u2, u3,….. etc. along the OX’ axis and the
corresponding points at distance v1, v2, v3,….etc. along the u(cm)
OY’ axis for different sets of observations. U4 U3 U2 U1 L O
X’ o X
Draw straight lines joining u with v , u with v u with v etc. 45
1 1 2 2, 3 3
These lines will intersect at a point K as shown in the graph.
Draw KL and KM perpendicular on X’ and Y’ axis respectively, M
then V4
V3
OL=OM= – f ∴ f=
−...........cm
V2 v(cm)
The above argument can be justified from the fact that the
1 1 1 V1
mirror formula, + = is satisfied by extreme values
u v f
(f, ∞ ) and ( ∞ , f). The straight lines corresponding to extreme
values intersect at a point K having co-ordinates (f, f). For Y’
lines LK, u=f, v → ∞ and for line MK, v=f, u → ∞ . Figure 27.32

1 1
Method 3: and Graph Method
u v
1 1
Select a suitable but the same scale to represent along – X axis (or X’ axis) and along-Y axis (or Y’ axis). By sign
u v
1 1 1 1
convention, both and are negative (for real images). For different sets of values of and plot the graph,
u v u v
which comes to be a straight line as shown in Fig 27.33.
P hysi cs | 27.25

The straight line cuts the two axes X’ and Y’ at an angle Y


of 45° at points A and B respectively and also has equal -1
1/u(cm )
intercepts for both the axis. Measures are distance OA and
OB. A O
X’ o
45 X
1 1
The focal length, f=
− =
− f (say)
=
OA OB 0
∴ f=
f0 − cm

The above agreement can be explained as follows. -1


1/v(cm )
1 1 1
From mirror formula, + =
u v f
o
45
(a) If object is placed at infinity, then u→∞
B
1 1
∴ = Y’
v f
1 1 Figure 27.33
So, intercept OB= =
v f
1 1
(b) If image is formed at infinity, then v → ∞ ∴ =
u v
1 1
So, intercept OA= =
u f

Experiment 16: Plot of Angle of Deviation vs. Angle of Incidence for a Triangular Prism
A
Principle: A triangular prism has three rectangular lateral
surfaces and two triangular bases. The line along which any two
faces (refracting surfaces) of the prism meet, is the refracting M

edge of the prism and the angle between them is the angle of i r’ R e
the prism. For this experiment, it is convenient to place the prism Q r
with its rectangular surfaces vertical. The prinicipal section ABC of
P S
the prism is obtained by a horizontal plane perpendicular to the
refracting edge (Fig. 27.34).
B C
A ray of light PQ (from air to glass) incident on the first face AB = (i - r) + (e - r’)
at an angle is refreacted at angle r along QR and finally, emerges =i+e-A
along RS. The dotted lines in the Fig 27.34 represent the normal
Figure 27.34
to the surfaces. The angle of incidence (from glass to air) at the
second face AC is r’ and the angle of refraction (or emergence)
is e. The angle between the direction of incident ray PQ (produced forward) and direction of emergent ray RS
(produced backward) is the angle of deviation δ .
A 
From geometrical considerations we have r + r ' = … (i)
δ = (i − r) + (e − r ') = i + e − A  … (ii)
At the position of the prism for minimum deviation δm' the light ray passes through the prism symmetrically, i.e.
parallel to the base so that when
δ =δm' i = e which implies r = r '.
The advantage of putting the prism in minimum deviation position is that the image is brightest in this position.

Procedure
(a) Fix a white sheet of paper on a drawing board with the help of cellotape or drawing pins.
(b) Draw a straight line XY, using a sharp pencil nearly in the middle and parallel to the length of the paper.
2 7 . 2 6 | Experimental Physics

(c) Mark points O1 ,O2 ,O3 ,......... on the straight line XY at suitable distance of about 8 to 10cm and draw normal
N1O1 ,N2O2 , N3O3 ...... on these point (Fig. 27.35).

N1 N2 N3
P1
P2
i1 i2 i3
P3
Q1 Q1 Q2 Q2 Q3 Q3
X A B A B A T3 B Y
T1 T1
3
2
R1 1 R2 R3

S1 S2 S3

Figure 27.35

(d) Draw straight lines, P1O1 , P2 O2 , P3 O3 ,......... corresponding to the incident rays, making angles of incidence
at 350 , 400 , 450 , 500 , ....600 respectively with the normal, using a protractor. Write the values of the angles
∠P1O1N1 , ∠P2O2N2 , ∠P3O3N3 ,....... on the white paper sheet (Fig. 27.35)
(e) Place the prism with its refracting face AB on the line XY with point O1 in the middle of AB as shown in the
Fig 27.35. Draw the boundary of the prism with a sharp pencil.
(f) Fix two alpins P1 and Q1 with sharp tips vertically about 10cm apart, on the incident ray line P1Q1
such that pin Q1 is close to point O1 . Close one eye (say left) and looking through the prism, bring your right
eye in line with the images of the pins P1 and Q1 . Fix alpins R1 and S1 about 10cm apart vertically on the white
paper sheet, with their tips in line with the tips in line with the tips of the images of pins P1 and Q1 . In this
way pin R1 and S1 will become collinear with the images of pin P1 and Q1 .
(g) Remove the pins R1 and S1 and encircle their pin pricks on the white paper sheet with the help of a sharp
pencil. Remove the pins P1 and Q1 and encircle their pin pricks also.
(h) Join the points (or pin pricks) R1 and S1 with the help of a sharp pencil and scale, to obtain the emergent ray
R1 S1 . Produce it backwards to meet the incident ray P1Q1 (produced forward) at T1 . Draw arrowheads on P1Q1
and R1 S1 to show the direction of the rays.
(i) Measure the angle of deviation δ1 and the 65
angle BAC (angle A) of the prism (Fig. 27.36)
with a protractor and write the values of o
60 n=1.60, A=60
these angles indicated in the diagram.
( j) Repeat steps 5 to 9 for different values of 55
angle of incidence (40o, 45o, 50o .....) and
measure the corresponding angles of
 (in degrees)

deviation δ2 , δ3 ..... with the protractor, and 50


indicate them in the respective diagrams. m
(k) Record observation in tabular form with 45
proper units and significant Fig 27.36.
Sample graph looks like the adjacent Fig 40
27.36. 30 40 50 60 70 80 90
i (in degrees)
Figure 27.36
P hysi cs | 27.27

Experiment 17: Refractive Index of a Glass Slab Using a Travelling Microscope

Principle
(a) A travelling microscope (Fig 27.37) consists of an ordinary compound
microscope M which is capable of moving along both vertical and
horizontal directions. The movement in any direction can be read by
a fixed main scale S and a sliding vernier scale V. There is a horizontal
metal platform which can be levelled by means of the levelling screws
L and a small spirit level over the base. The vertical motion is controlled
by two screws, one is called the fixing screw and other as tangent
screw, and the later imparts slow motion only when the former is fixed.
Similarly, there are two screws for the horizontal motion.
(b) The compound microscope has the usual objective O and eyepiece E.
There is a cross-wire which can be sharply focused by the eyepiece in
the field of view. While focusing the microscope, adjustments are to be
made so that there is no parallax between the cross wire and the image
of the object to be seen.
(c) When a ray of light is allowed to travel from the rarer medium (air) to
the denser medium (glass) obliquely, there is a change in both, the
265
path, as well as speed of the light. The ratio of the speed of light in air
medium to that in the glass medium is called refractive index of glass Figure 27.37
medium with respect to air medium.
(d) Let abcd be a glass slab (Fig 27.38) below which a mark A is put. Eye
After refraction, the rays from A will appear to come from B. As
a result, B is the apparent image of A. Then refractive index of
air w.r.t glass is given by
sini CD/AD BD
g µ= = = r
a
sinr CD/BD AD
1 AD a C D b
Hence refractive index of glass w.r.t air will be a µ=
g =
µ
g a BD
r i
(e) If the point D is considered to be very close to point C, then
ADAC=Real thickness and BD ≅ BC = Apparent thickness B i
(f) So, the refractive index of glass with respect to air is Glass
slab
AC Real Thickness c
a µg= = d
BC Apparant Thickness A
Figure 27.38
Procedure
(a) The platform of the travelling microscope is levelled by a spirit level by adjusting the levelling screws.
(b) The least count of the vertical scale is determined by using the common principle of the vernier scale.
S
Example: LG=1MSD-1VSD=
n
Where S = smallest main scale division and n = total number of divisions in the vernier scale.
1 0.05
If S = mm=0.05cm, n=50, then LC = =0.001 cm R1
2 50
(c) The eyepiece is focused to get the distinct images of the cross wires in the field of view. At first, the microscope
is focused to a cross mark O on a piece of paper fixed on the platform. The corresponding reading R1 is noted
on the vernier scale.
2 7 . 2 8 | Experimental Physics

(d) The glass slab is then placed on the mark O and the cross M
wire is focused through the microscope which has to be
raised up. The corresponding reading R 2 is noted on the

7
vernier scale. R3
(e) Without disturbing the slab, a small quantity or lycopodium M

6
powder is sprinkled on the upper surface of the glass slab.
Microscope is raised further up, to focus the granules of M

5
R2
lycopodium powder on the surface of the glass slab. The
final reading R 3 is noted.

4
(f) Repeat steps 3 to 5 three times by putting mark A at different R1

3
places on the paper.

2
1

1
0

0
A A A
(I) (II) (III)
Figure 27.39

Experiment 18: Characteristic Curves of a P-N Junction Diode in Forward and Reverse Bias

Theory:
Characteristics of diode: Graphical relationship between the voltage applied across a diode and the current
through the diode is called characteristics of diode. The graph plotted with current as ordinate and potential
applied across it ends as abscissa, shows the characteristics of the diode.
Forward biasing: A p-n junction diode gets forward biased when its p side is connected to the positive terminal of
the supply voltage and n to the negative terminal. Initially for voltages up to 0.4V, there is not much rise in current
due to the opposition by barrier potential. Beyond this, the current starts rising in a p-n junction.
Knee Voltage: The forward voltage when the current starts rising, i.e., is termed as the knee voltage. It is represented
as Vk. It is about 0.7V for silicon.
+
Reverse biasing : A p-n junction is reverse biased when the p side of + +
the junction is connected to the negative terminal of supply voltage + +
Anode
and n side terminal is connected to positive terminal of battery.
+ +
Reverse saturation current: As the applied voltage is increased in the + P+
reverse bias, starting from zero value, the current increases, but soon +
becomes constant. This current is very small (a few microamperes). It
P +++
+
is called the reverse saturation current. Conducts + +
PN Junction Diode and its characteristics N -
- -
PN junction diode is symbolically represented as shown in the picture. - -
The direction of the arrow is the direction of conventional current flow Cathode N
-
(under forward bias). Now let’s try applying an external voltage to
- - - -
the pn junction diode. The process of applying an external voltage is - -
called “biasing”. There are two ways in which we can bias a pn junction Diode (P-N junction)
-
diode. forward based
-
(1) Forward bias and (2) Reverse bias
Figure 27.40
P hysi cs | 27.29

The basic difference between a forward bias and reverse bias is in the direction of applying external voltage. The
direction of external voltage applied in reverse bias is opposite to that of external voltage applied in forward bias.

Forward biasing a PN Junction diode


vext

depletion
p-type zone n-type
- +
- +
- +
- +
0-ext
Figure 27.41

Plotting the characteristics of a p-n junction


What we are going to do is, vary the voltage
i
across diode by adjusting the battery. We
start from 0 volts, then slowly move 0.1 volts, Breakdown Reverse Forward
0.2 volts and so on till 10 volts. Let’s note
the readings of voltmeter and ammeter each
time we adjust the battery (in steps of 0.1
volts). Finally, after taking the readings, plot a Vbr
graph with voltmeter readings on X-axis and Vd v
corresponding Ammeter readings on Y axis.
Join all the dots in the graph paper and you + v -
will see a graphical representation as shown
below. This is what we call “characteristics of
a pn junction diode” or the “behavior of diode i
under forward bias”.
Figure 27.42
How to analyse the characteristics of a pn
junction diode? 60mA
From the “characteristics graph” we have just drawn,
we are going to make conclusions about the behavior 50mA
of pn junction diode. The first thing that we will be
40mA
interested in, is “barrier potential”. From the graph,
we observe that the diode does not conduct at all
30mA
in the initial stages. From 0 volts to 0.7 volts, we are
seeing the ammeter reading as zero! This means the
20mA
diode has not started conducting current through it.
From 0.7 volts and up, the diode starts conducting 10mA
and the current through diode increases linearly with 120v 90v 60v 30v
increase in voltage of battery. The barrier potential of
silicon diode is 0.7 volts.  The diode starts conducting 1v 2v 3v 4v 5v
at 0.7 volts and current through the diode increases 600A
linearly with increase in voltage. So that’s the forward
bias characteristics of a pn junction diode. It conducts 1200A
current linearly with increase in voltage applied across
the 2 terminals  (provided the applied voltage crosses
barrier potential). Junction diode characteristics
Figure 27.43
2 7 . 3 0 | Experimental Physics

What happens inside the pn junction diode when we apply forward bias?
We have seen the characteristics of pn junction diode through its graph. We know a diode has a depletion region
with a fixed barrier potential. This depletion region has a predefined width, say W. This width will vary for a Silicon
diode and a Germanium diode. The width highly depends on the type of semiconductor used to make pn junction,
the level of doping etc. When we apply voltage to the terminals of diode, the width of the depletion region slowly
starts decreasing. The reason for this is, in forward bias we apply voltage in a direction opposite to that of barrier
potential. We know the p-side of diode is connected to positive terminal and n-side of diode is connected to
negative terminal of battery. So the electrons in n-side gets pushed towards the junction (by force of repulsion)
and the holes in p-side gets pushed towards the junction. As the applied voltage increases from 0 volts to 0.7 volts,
the depletion region width reduces from ‘W’ to zero. This means depletion region vanishes at 0.7 volts of applied
voltage. This results in increased diffusion of electrons from n-side to p-side region and the increased diffusion of
holes from p-side to n-side region. In other words, “minority carrier” injection happens on both p-side (in a normal
diode (without bias) electrons are a minority on p-side) and n-side (holes are a minority on n-side) of the diode.

How does the current flow take place in a pn junction diode?


This is another interesting factor, to explain. As the voltage level increases, the electrons from n-side gets pushed
towards the p-side junction. Similarly holes from p-side gets pushed towards the n-side junction. Now there
arises a concentration gradient between the number of electrons at the p-side junction region and the number
of electrons at the region towards the p-side terminal. A similar concentration gradient develops between the
number of holes at the n-side junction region and the number of holes at region near the n-side terminal. This
results in movement of charge carriers (electrons and holes) from a region of higher concentration to a region of
lower concentration. This movement of charge carriers inside pn junction gives rise to current through the circuit.

Reverse biasing a pn junction diode


Why should we reverse bias a pn diode? The i
reason is, we want to learn its characteristics
under different circumstances. By reverse Breakdown Reverse Forward
biasing, we mean applying an external voltage
which is opposite in direction to forward
bias. So here we connect positive terminal of
battery to n-side of the diode and negative Vbr
terminal of the battery to p-side of the diode. Vd v
This completes the reverse bias circuit for pn
+ v -
junction diode. Now to study its characteristics
(change in current with applied voltage), we
need to repeat all those steps again. Connect i
voltmeter, ammeter, vary the battery voltage,
note the readings etc etc. Finally we will get a
graph as shown. Figure 27.44
Analysing the reverse bias characteristics
Here the interesting thing to note is that, diode does not conduct with change in applied voltage. The current
remains constant at a negligibly small value (in the range of micro amps) for a long range of change in applied
voltage. When the voltage is raised above a particular point, say 80 volts, the current suddenly shoots (increases
suddenly). This is called as “reverse current” and this particular value of applied voltage, where reverse current
through diode increases suddenly is known as “break down voltage“.

What happens inside the diode?


We connected p-side of the diode to the negative terminal of the battery and n-side of the diode to the positive
terminal of the battery.  One thing is clear, we are applying external voltage in the same direction of barrier
potential. If applied, external voltage is V and barrier potential is Vx , then total voltage across the pn junction will
be V+Vx. The electrons at n-side will get pulled from junction region to the terminal region of n-side and similarly
the holes at p-side junction will get pulled towards the terminal region of p-side. This results in increasing the
P hysi cs | 27.31

depletion region width from its initial length, say ‘W’ to some ‘W+x’. As width of depletion region increases, it
results in increasing the electric field strength.

How does reverse saturation current occur and why does it exist?
The reverse saturation current is the negligibly small current (in the range of micro amperes) shown in the graph,
from 0 volts, to break down voltage. It remains almost constant (negligible increase do exist) in the range of 0 volts,
to reverse breakdown voltage. How does it occur? We know, as electrons and holes are pulled away from junction,
they dont diffuse each other across the junction. So the net “diffusion current” is zero! What remains is the drift
due to the electric field. This reverse saturation current is the result of drifting of charge carriers from the junction
region to the terminal region. This drift is caused by the electric field generated by depletion region.

What happens at reverse breakdown?


At breakdown voltage, the current through the diode shoots rapidly. Even for a small change in applied voltage,
there is a high increase in net current through the diode. For each pn junction diode, there will be a maximum net
current that it can withstand. If the reverse current exceeds this maximum rating, the diode will get damaged.

Conclusion about PN junction characteristics


To conclude about pn junction characteristics, we need to get an answer to the first question we have raised – what
is the use of pn junction? From the analysis of both, forward bias and reverse bias, we can arrive at one fact – a pn
junction diode conducts current only in one direction – i.e., during forward bias. During forward bias, the diode
conducts current with increase in voltage. During reverse bias, the diode does not conduct with increase in voltage
(break down usually results in damage of diode).

Experiment 19: Characteristic Curves of a Zener Diode and Finding Reverse Breakdown
Voltage

Theory and Circuit diagram


- v +
On application of reverse bias to a diode, depletion layer
widens and the bias increases the barrier potential. As a result
of this, there is no flow of current in the diode. As the reverse
bias increases to a certain value, the applied electric field pulls R
electrons directly out of their bonds and an increased current +
flow occurs. The effect is called Zener effect and the reverse
voltage applied is called Zener current. At breakdown voltage, - 0-100 V
+
the current suddenly increases to a high value (maintaining A
the voltage constant). That is why zener diodes are used in 50 mA
-
voltage regulators. Zener diodes with breakdown voltage
2.7V to a few hundred volts are available. Figure 27.44

Experiment 20: Characteristic Curves of a Transistor and Finding Current Gain and Voltage
Gain

Theory:
In most of the transistor circuits, out of the common base, common collector and common emitter, the configuration
generally used is common emitter. In such connections, the emitter is common to both the input and the output.
For ascertaining the common emitter characteristics, the variables studied are:
(a) IB vs. VBE keeping VCE constant (Input characteristics)
(b) Ic vs. VCE keeping IB constant. (Output characteristics)
(c) IC vs. IB keeping VC constant. (Transfer characteristics)
Transistor is said to be a current device.
2 7 . 3 2 | Experimental Physics

Input Characterisitcs: Input characteristics show interdependence of the base current on the base potential for
fixed values of as shown in the figure.

Vi
IC
t
IB Input
(A)
RL
Vi Input VCE
VBE IE V0

EB EC t
Output
(B)

Figure 27.46

The a.c. input resistance ( ri ) of the transistor in common emitter circuit is

 ∆VBE 
=ri =  v c cons tant ri is only a few 100 ohms.
 ∆IB 

Output characteristics: These characteristics show the dependence of Ic on VCE when IB value is fixed as shown in
figure and is generally operated beyond the sharp change of slope. The a.c. output resistance (ro) of transistor in
common emitter circuit is
 ∆VC 
=ro = I cons tant
 ∆I  B
 C 
The value of ro varies from a 1000 ohms to a few 10 kilo-ohms.

Transfer characteristics: These characteristics show the variation of Ic to base current IB corresponding to a point
P on the transfer characteristics is termed as direct current gain β .
Therefore,
Collector current at given value of Vc I
Current gain – β = or β=C
Base current at same value IB
Alternating current Amplification –
In transfer characteristics, a small change in base current ∆IB at a given value of Vc produces a large change ∆IC in
collector current, then,
∆IC QR
A.C. Current gain, β=' = as shown in Fig = ……………. (ii)
∆IB PR
Voltage Gain –
Corresponding to a small voltage change ∆v i in the emitter base (i.e., input), if the change in the output voltage at
the collector is ∆Vo , then the ratio of ∆Vo to ∆Vi is termed as voltage gain, i.e.,
ro ∆Ic ro
But ∆Vi =ri ∆IB and ∆Vo =ro ∆IC ⇒ Avg. gain = . = β
ri ∆IB ri
Where ri is input resistance and ro is the output resistance of the transistor and β is the current gain,
ro ∆Ic r
∴ Av = . = β o
ri ∆IB ri
P hysi cs | 27.33

Exercises

Sound and Heat Q.14 Why do both the prongs vibrate when we strike
only one?

Sound
Q.15 Can sound also travel in vacuum?

Q. 1 Do the prongs and stem of a tuning fork execute Q.16 In which medium is the velocity of sound higher,
same type of vibrations? in oxygen or hydrogen?

Q.2 Is the frequency of these vibrations different? Q.17 Which particular column in your apparatus, do
you call resonance column?
Q.3 What about the amplitude of two kinds of
vibrations? Q.18 What is the role of water in this apparatus?

Q.4 How does the frequency of tuning fork vary with Q.19 Why do you use water; can’t you use mercury
increase of length of prong? instead of water?

Q.5 Suppose you have been given two tuning forks of Q.20 What types of waves are there in air above water
the same metal on which, marks of the frequency have in this resonance column?
disappeared. How will you detect the one with higher
frequency?
Q.21 How are these waves produced?

Q.6 If the prong of the fork are rubbed with a file


slightly, will it affect the frequency? Q.22 How do you get the position of first resonance?

Q.7 If a prong of the tuning fork is loaded with wax, will Q.23 Where are the nodes and antinodes situated?
its frequency change?
Q.24 Is antinode situated exactly at the open end?
Q.8 Why are the forks made of some standard
frequencies like 256, 288, 320, 341.5, 384, 426.6, 484, Q.25 How can you get wavelength of sound by this first
512, Hz etc.? resonating length?

Q.9 What is the significance of the letters of English Q.26 Can we use a resonance tube of square cross-
alphabet engraved on the running forks? section for the experiment?

Q.10 What do you mean by a note? How does it differ Q.27 Why do we use a long tube?
from a tone?
Q.28 How do you keep the vibrating tuning fork near
Q.11 The vibrations of a fork stop when its prongs are the open end of the tube?
touched but they do not stop if the stem is touched.
Why? Q.29 What do you mean by second resonance?

Q.12 Why a tuning fork has two prongs? Q.30 Why is the second resonance found feebler than
the first?
Q.13 Why should a tuning fork not be struck with a
great force?
2 7 . 3 4 | Experimental Physics

Q.31 Where are the nodes and antinodes in this case? Q.38 In the resonating air column experiment, l1
represents 1st resonating length and l2 represents 1
Q.32 What will be the wavelength of sound in this case? 2nd resonating length, the relation between 1st and 2nd
resonating length is

Q.33 Can’t you eliminate this end correction? (A) l2 = l1 (B) l2 =2 l1


(C) l2 =3 l1 (D) l2 =4 l1
Q.34 Can you also determine the velocity of sound by
this method? Q.39 In resonating air column apparatus the
approximate relation between end correction and
Q.35 If n is the frequency of the tuning fork used to diameter of the tube is
excite the air column in resonating air column apparatus (A) 0.3d (B) 0.15d (C) 0.6d (D) 0.4d
l1 first resonating length and l2 second resonating
length, then the velocity of sound in air is given by the Q.40 In resonance column apparatus the reason for
formula hearing booming sound is because
v n (l2 − l1 )
(A)= v n (l1 − l2 )
(B)= (A) The air column in the tube and the tuning fork
vibrate with the same frequency
(c)
= v 2n (l2 − l1 ) (d)
= v 2n (l1 − l2 )
(B) The air column in the tube vibrates with frequency
which is greater than the frequency of the tuning fork
Q.36 In resonating air column apparatus while
comparing the frequencies of the two tuning forks, if (C) The air column in the tube vibrates with frequency
which is less than the frequency of the tuning fork.
l1 Is the 1 resonating length of air column with 1 fork
st st

of frequency n1 (D) Velocity of sound in air column is greater than the


velocity of sound in atmosphere air
l 2 is the 2nd resonating length of air column with 1st
fork of frequency n2
Q.41 The resonating lengths of an air column in the
l1' is the 1st resonating length of air column with 2nd fork first and second modes of vibration are 37 cm and 97
of frequency n2 cm. If velocity of sound is 300 ms−1 the frequency of the
l'2 is the 2nd resonating length of air column with 2nd tuning fork is
fork of frequency n2 (A) 1000 Hz (B) 500 Hz
Then: (C) 250 Hz (D) 125 Hz
n1 l2 − l1 n1 l'2 − l2
(A) = (B) = Q.42 The end correction of a resonance tube is 1cm. If
n2 l'2 − l1' n2 l1' − l1
shortest resonating length is 15cm, the next resonating
n1 l1' − l1 n1 l'2 − l1' length will be
(C) = (D) =
n2 l'2 − l2 n2 l2 − l1 (A) 47 cm (B) 45 cm (C) 50 cm (D) 33 cm

Q.37 The end correction (e) is ( l1 = length of air column Q.43 A tuning fork of frequency 340Hz is excited
at first resonance and l2 is length of air column at and held above a cylindrical tube of length 120cm.
second resonance) It is slowly filled with water. The minimum height of
water column required for resonance to be first heard
l2 − 3l1 l1 − 3l2 (Velocity of sound=340 ms−1 ) is
(A) e = (B) e =
2 2 (A) 25 cm (B) 75 cm (C) 45 cm (D) 105 cm

l2 − 2l1 l1 − 3l2
(C) e = (D) e = Q.44 Two unknown frequency tuning forks are used in
2 2 resonance column apparatus. When only first tuning
fork is excited the 1st and 2nd resonating lengths noted
are 10cm and 30cm respectively. When only second
tuning fork is excited the 1st and 2nd resonating lengths
P hysi cs | 27.35

noted are 30 cm and 90 cm respectively. The ratio of dropped in a copper calorimeter (of water equivalent
the frequency of the 1st to 2nd tuning fork is 0.025kg) containing 150cm3 of water at 27°C. The final
temperature is 40°C. The specific heat of the metal is
(A) 1:3 (B) 1:2 (C) 3:1 (D) 2:1
(A) 0.1 Jg–1 oC–1 (B) 0.2 Jg–1 oC–1
Q.45 In a resonance tube, the first resonance is (C) 0.3 calg–1 oC–1 (D) 0.1 calg–1 oC–1
obtained when the level of water in the tube is at
16 cm from the open end. Neglecting end correction,
Q.54 Is newton’s law of cooling true for all differences
the next resonance will be obtained when the level of
of temperature between the body losing heat and that
water from the open end is
of its surroundings?
(A) 24 cm (B) 32 cm (C) 48 cm (D) 64 cm
Q.55 How do you express this law mathematically?
Heat
Q.56 What is the shape of the graph of log ( θ − θ0 )
Q.46 Is specific heat of a substance is a constant versus t?
quantity?
Electricity and Magnetism
Q.47 What is meant by thermal capacity of a body?
Q.1 What is the alternative term used for Metre Bridge?
Q.48 What ate the units of water equivalent and thermal
capacity of a body? Q.2 Name the principal on which Metre Bridge is based.

Q.49 Why do we use generally a calorimeter made of Q.3 When is the Wheatstone’s bridge said to be most
copper? sensitive?

Q.50 In an experiment to determine the specific heat of Q.4 Why is the metre bridge suitable for resistances of
aluminum, piece of aluminum weighing 500g is heated moderate values only?
to 100° C. It is then quickly transferred into a copper
calorimeter of mass 500g containing 300g of water
30° C. The final temperature of the mixture is found to Q.5 For determination of resistance of a coil, which of
be 46.8° C. If specific heat of copper 0.093 cal g–1° C–1, two methods is better Ohm’s Law method or metre
then the specific heat of aluminium is bridge method?

(A) 0.11 cal g–1 oC–1 (B) 0.22 cal g–1 oC–1
Q.6 Why should the battery key be pressed before the
(C) 0.33 cal g–1 oC–1 (D) 0.44 cal g–1 oC–1 galvanometer key?

Q.51 The mass of a copper calorimeter is 40g and Q.7 Sometimes it is advisable to shunt the galvanometer
its specific heat in SI units is 4.2 × 102 J kg–1° C–1 . The while trying for a balance point. Why?
thermal capacity is
(A) 4 J oC–1 (B) 18.6 J Q.8 What is the material of the wire of metre bridge?
(C) 16.8 J/KG (D) 16.8 J C o –1

Q.9 Why is constant an used for the bridge wire?


Q.52 When 0.2 kg of brass at 100°C is dropped into
0.5 kg of water at 20°C, the resulting temperature 23°C. Q.10 Can we measure a resistance of the order of 0.160
The specific heat of brass is Ω using a Whaeatstone’s bridge? Support your answer
with reasoning?
(A) 0.41 103 J kg–1 oC–1 (B) 0.41 102 J kg–1 oC–1
(C) 0.41 104 J kg–1 oC–1 (D) 0.41 J kg–1 oC–1
Q.11 In a Wheatstone bridge, three resistances P, Q
and r connected in the three arms and the fourth arm
Q.53 In an experiment to determine the specific heat is formed by two resistances S1 and S2 connected in
of a metal, a 0.20kg block of the metal at 105°C is
2 7 . 3 6 | Experimental Physics

parallel. The condition for the bridge to be balanced new position of the null point from the same end, if one
will be decides to balance a resistance of 4X against Y will be at

P R ( S1 + S2 ) P R (A) 50 cm (B) 80 cm (C) 40 cm (D) 70 cm


(A) = (B) =
Q 2S1S2 Q S1 + S2
Q.18 The specific resistance of a wire
P 2R P R ( S1 + S2 )
(C) = (D) = (A) Varies with its length
Q S1 + S2 Q S1S2
(B) Varies with its cross-section
Q.12 A square aluminium rod is 1m long and 5 mm on (C) Varies with its mass
edge. What must be the radius of another aluminium
rod whose length is 1m and which has the same (D) Does not depend on its length, cross-section and mass
resistance as the previous rod?
Q.19 What do you mean by potential gradient?
(A) 5.0 mm (B) 4.2 mm
(C) 2.8 mm (D) 1.4 mm
Q.20 What do you mean by the sensitivity of
potentiometer?
Q.13 The specific resistance and cross-section area of
potentiometer wire is ρ and A respectively. If a current
Q.21 How can it be increased?
Ι is passed through the wire, the potential gradient of
the wire will be
Q.22 How can it be achieved?
Ιρ Ι ΙA
(A) (B) (C) (D) ΙAρ
A ρA ρ
Q.23 On what factors internal resistance of a cell
depends?
Q.14 The resistance in the left and right gaps of a
balanced metre bridge are R1 and R 2 . The balanced
point is 50cm. If a resistance of 24Ω is connected in Q.24 In a potentiometer arrangement, a cell of emf
parallel to R 2 , the balance point is 70cm. The value of 1.25 V gives a balance point at 35 cm length of the wire.
R1 or R 2 is If the cell is replaced by another cell, the balance point
shifts to 63 cm, then emf of the second cell is:
(A) 2Ω (B) 8Ω (C) 8Ω (D) 32Ω
(A) 4.25 V (B) 2.25 V
Q.15 An unknown resistance R1 is connected in series (C) 3.25 V (D) 1.25 V
with a resistance of 10ohm. This combination os
connected to one gap of a metre bridge, while other Q.25 Two cells of emf E1 and E2 (E1 > E2 ) are connected
gap is connected to another resistance R 2 The balance as shown in figure. When a potentiometer connected
point is at 50cm. Now, when the 10ohm resistance is between A and B, the balancing length is 100cm. The
removed, the balance point shifts to 40cm. Then, the ratio E1 and E2 is
value of R1 is
(A) 60Ω (B) 40Ω (C) 20Ω (D) 10Ω A B C
E1 E2
Q.16 Two resistances are connected in the two gaps of (A) 3:2 (B) 4:3 (C) 5:4 (D) 2:1
a metre bridge. The balance point is 20cm from the zero
end. When a resistance 15Ω is connected in series with
the smaller of two resistances, the null point shifts to Q.26 In the experiment of half deflection method the
40cm. The smaller of the two resistances has the value. resistance R should be

(A) 8Ω (B) 9Ω (C) 10Ω (D) 12Ω (A) >S (B) ≈ S (C) <S (D) ≈ G

Q.17 In a metre bridge experiment, null point is obtained Q.27 In the experiment of half deflection method
at 20 cm from one end of the wire when resistance X is if R= 20Ω and S= 12Ω for half deflection then the F
balanced against another resistance Y. If X<Y, then the (galvanometer resistance) is
(A) 20Ω (B) 30Ω (C) 40Ω (D) 25Ω
P hysi cs | 27.37

Q.28 Figure of merit of a galvanometer is 0.5 A per Q.8 Can we use Vernier Callipers to measure the
division. If the galvanometer has total 10 divisions then thickness of a piece of paper?
the current required for full deflection in one side is
(A) 5.0 A (B) 2.0A (C) 2.5 A (D) 3.0 A Q.9 Select the incorrect statement.
(A) If the zero of vernier scale does not coincide with
Q.29 In an experiment to determine the figure of merit the zero of the main scale, then the vernier calipers is
of a galvanometer the circuit shown is used. If E be the said to be having zero error?
emf of the battery, R is the series resistance, G is the
(B) Zero correction has a magnitude equal to zero error
resistance of the galvanometer and θ is the deflection,
but sign is opposite to that of zero error.
then the figure of merit of galvanometer is
(C) Zero error is positive when the zero of vernier scale
E E lies to the left of the zero of the main scale.
(A) (B)
(R − G ) θ (R + G ) θ
EG Eθ Q.10 1 cm on the main scale of a vernier calipers is
(C) (D) divided into 10 equal parts. If 10 divisions of vernier
Rθ R+G
coincide with 8 small divisions of main scale, then the
least count of the caliper is
Q.30 In half deflection method, a high resistance box is
connected in series with the battery so that (A) 0.01cm (B) 0.02cm

(A) The deflection of the galvanometer is brought (C) 0.05cm (D) 0.005cm
within the scale
(B) Power losses are minimized Q.11 The vernier constant of a travelling microscope
is 0.01cm. If 49 main scale divisions coincide with 50
(C) High resistance values are easily available vernier scale divisions, then the value of 1 main scale
(D) None of the above division is
(A) 0.1mm (B) 0.5mm (C) 0.4mm (D) 1mm
Vernier Callipers and Screw Gauge
Q.12 If the error in the measurement of the diameter of
the sphere is 1%, then the error in the measurement of
Q.1 What is meant by Least Count (L.C.) of a measuring the volume is
instrument?
(A) 3% (B) 1% (C) 6% (D) 9%

Q.2 How do you define Least Count for Vernier


Callipers? Q.13 Which of the following has the largest least count?
(A) Spherometer (B) Vernier calipers
Q.3 (a) What is the vernier constant of Vernier Callipers? (C) Screw guage (D) Metre scale
(b) What is the value of the least count of a commonly
available Vernier Callipers? Q.14 1 cm of main scale of a vernier caliper is divided
into 10 divisions. The least count of the callipers is
Q.4 What is meant by zero error of Vernier Callipers? 0.005cm, then the vernier scale must have
(A) 10 divisions (B) 20 divisions
Q.5 When is the zero error positive? (C) 25 divisions (D) 50 divisions

Q.6 What is zero correction? How does it differ from Q.15 Each division on the main scale is 1mm. Which of
zero error? the following vernier scales give vernier constant equal
to 0.01mm?
Q.7 How is zero error taken into account for getting
(A) 9 mm divided into 10 divisions
true reading?
(B) 90 mm divided into 100 divisions
2 7 . 3 8 | Experimental Physics

(C) 99 mm divided into 100 divisions Q.29 What is negative zero error of screw guage?
(D) 9 mm divided into 100 divisions
Q.30 What is the principle of a screw guage?
Q.16 Vernier constant is the
Q.31 The pitch of screw guage is
(A) Value of one MSD divided by total number of
divisions on the main scale (A) 1 mm (B) 0.1mm (C) 1cm (D) 0.1cm
(B) Value of one MSD divided by total number of
divisions on the vernier scale Q.32 Least count of screw guage is defines as

(C) Total number of divisions on the main scale divided Dis tance moved by thimble on main scale
(A)
by total number of divisions on the vernier scale Number of rotation of thimble

(D) Difference between value of one main scale division Pitch of the screw
and one vernier scale division
(B)
Number of circular scale divisions on head scale

Number of rotation thimble


Q.17 The error in the measurement of radius of (C)
the spheres by using vernier callipers is 0.3%. The Number of circular scale divisions
permissible error in the measurement of surface area is
(D) None of the above
(A) 0.6%   (B) 1.2%   (C) 1.8%   (D) 0.9%
Q.33 Screw guage is said to have a negative error
Q.18 In a Vernier callipers 1cm of the main scale is
(A) When head scale zeroth division coincides with
divided into 20 equal parts. 19 divisions of the main
base line of main scale
scale coincide with 20 divisions on the vernier scale.
Find the least count of the instrument. (B) When head scale zeroth division is above with base
line of main scale
Q.19 Why is screw gauge commonly known as (C) When head scale zeroth division is below with base
micrometer? line of main scale
(D) None of the above
Q.20 Why is a screw guage so named?
Q.34 For positive error, the correction is
Q.21 What is the main characteristic of a screw?
(A) Positive (B) Negative

Q.22 Define the pitch of a screw. (C) Nil (D) None of these

Q.23 What is the relation between the pitch and the Q.35 The diameter D of a wire is measured using screw
least count? guage of zero error. Then
(A) D=main scale reading + circular scale reading x L.C.
Q.24 What is the cause of backlash error? (B) D=circular scale reading + main scale reading x L.C.
(C) D=main scale reading + vernier scale reading x L.C.
Q.25 How can you minimize backlash error?
(D) None of the above

Q.26 Of what metal is the screw of the screw guage


made and why? Q.36 In a screw guage value of each main scale division
and circular scale division is

Q.27 Which screw guage is more accurate-the one with (A) 1 mm and 0.01 cm (B) 0.01 cm and 1 cm
a pitch of 0.5mm or the one with a pitch of 1 mm, both (C) 0.01 cm and 1 mm (D) 0.01 cm and 1 mm
having the same number of circular scale divisions?

Q.28 What is zero error of the screw guage?


P hysi cs | 27.39

Q.37 The pitch of a screw guage is 0.5 mm. Its head Q.12 An object is placed in front of the convex mirror
scale contains 50 divisions. The lease count of the screw at a distance of 50 cm. A plane mirror is introduced
guage is covering the lower half of the convex mirror. If the
distance between the two mirrors is 10 cm, it is found
(A) 0.01 mm (B) 0.001 mm
that there is no parallax between the images formed
(C) 0.02 mm (D) 0.002 mm by the two mirrors. Then the focal length of the convex
mirror is
Q.38 The circular scale (head scale) of a screw guage (A) 12.5 cm (B) -12.5 cm
is divided into 100 equal parts and it moves 0.5 mm
ahead in one revolution. Find the pitch and the least (C) 25 cm (D) 75 cm
count.
Q.13 In the determination of focal length of convex
Optics and Modern Physics mirror by using a convex lens the object is generally
placed from the convex lens at a distance x times the
focal length of the convex lens, then x equals
Q.1 How will you distinguish between two given
spherical mirrors, as concave or convex? (A) 4 (B) 0.25 (C) 1.5 (D) 0.5

Q.2 What is meant by ‘parallax’? Q.14 In an experiment to find focal length of a concave
mirror, a graph is drawn between the magnitudes of u
Q.3 How is the parallax removed? and v. The graph look like.

Q.4 Why is a concave mirror of large focal length often Q.15 Which of the following graph is applicable
used as shaving glass? 1 1
between and for a concave mirror?
v u
Q.5 For what purpose does a surgeon use a concave Q.16 The graph showing correctly the variation of
mirror? image distance (v) as a function of object distance (u)
in case of a concave mirror is
Q.6 What type of mirrors are used in headlights of
vehicles and in searchlights? Q.17 How do you distinguish a convex lens from a
concave lens?
Q.7 Which mirror is used as a driving mirror in
automobiles and why? Q.18 What is meant by principal axis of a lens.

Q.8 Can you determine the focal length of a convex Q.19 If a lens is plano-convex, what is the radius of
1 1 curvature of its plane surface? Where does the centre
mirror by plotting versus .
v u of curvature of its plane surface lie?

Q.9 In determining focal length of a convex mirror


using auxillary convex lens, can a convex lens of any Q.20 Then, how do you define principal axis for a
focal length be used? plano-convex lens?

1 1
Q.10 What is spherical aberration? Q.21 A graph is drawn with along x-axis and along
u v
the y-axis. If the intercept on the x-axis is 0.5−1 , the
Q.11 Which method is more accurate in the focal length of the lens is (in meter)
determination of f for a concave mirror.
(A) 2.00 (B) 0.50 (C) 0.20 (D) 1.00
1 1
(i) u vs. v, or (ii) vs. graphs?
u v Q.22 What happens to a ray of light when it passes
through a rectangular glass slab?
2 7 . 4 0 | Experimental Physics

Q.23 Does the lateral displacement depend upon the (A) 13 cm (B) 11 cm (C) 9 cm (D) 7.1 cm
thickness of the slab?
Q.31 The diffusion dominant mechanisms for motion
Q.24 What causes earlier rise of the sun than the actual of charge carriers in forward and reverse biased silicon
rise p-n junction are
(A) Drift in forward bias, diffusion in reverse bias
Q.25 What causes formation of a rainbow after rains?
(B) Diffusion in forward bias, drift in reverse bias

Q.26 If a plane glass plate is placed over letters different (C) Diffusion in both forward and reverse bias
colours, then which coloured letters appears to be most (D) Drift in both forward and reverse bias
raised
(A) Blue (B) Green Q.32 The cause of the potential barrier in a p-n junction
diode is
(C) Violet (D) Red
(A) Depletion of positive charges near the junction
Q.27 A small pin fixed on a table top is viewed from (B) Concentration of positive charges near the junction
above from a distance of 50 cm. The distance by which
(C) Concentration of negative charges near the junction
the pin appears to be raised, if it is viewed from the
same point through a 15 cm thick glass slab held (D) Concentration of positive and negative charges
parallel to the table is (Refractive index of glass=1.5) near the junction
(A) 1 cm (B) 2 cm (C) 3 cm (D) 5 cm
Q.33 The reverse voltage at which the current increases
steeply is called
Q.28 A mark is made at the bottom of a beaker and
a microscope is focused on it. The microscope then (A) Threshold voltage (B) Knee voltage
raised through 1.5 cm. To what height water must be
(C) Breakdown voltage (D) Stopping voltage
poured into the beaker to bring the mark again into
4
focus? Given, Refractive index of water = . Q.34 The forward biased characteristics of a p-n
3
junction diode is:
(A) 6 cm   (B) 7 cm   (C) 8 cm   (D) 9 cm
Q.35. The reverse biased characteristics of a p-n
Q.29 A glass is filled with water upto 10 cm. the junction diode is.
apparent depth of an object lying at the bottom of the
glass measured by a microscope is 8.2 cm.
Q.36 Zener breakdown occurs in junction which is
(i) What is the refractive index of water?
(A) Heavily doped and has wide depletion layer
(ii) If water is replaced by a liquid of refractive index
(B) Lightly doped and has wide depletion layer
1.7 upto the same height, by what distance would
the microscope have to be moved to focus the object (C) Moderately doped and has narrow depletion layer
again>
(D) Heavily doped and has narrow depletion layer
(A) (i) 1.2 (ii) 2.32 cm
(B) (i) 1.2 (ii) 3.32 cm Q.37 At breakdown region of zener diode which of the
following quantities does not change much?
(C) (i) 1.4 (ii) 4.32 cm
(A) Voltage (B) Current
(D) (i) 1.4 (ii) 5.32 cm
(C) Dynamic impedance (D) Capacitance
Q.30 A beaker of depth 20 cm is half-filled with a liquid
of refractive index 1.4 and half-filled with liquid of Q.38 The avalanche breakdown occurs at
refractive index 1.7. What is the apparent depth of a (A) Higher reverse voltage
coin lying at the bottom of the beaker assuming that
liquids do not mix with each other? (B) Lower reverse voltage
P hysi cs | 27.41

(C) Lower forward voltage Q.47 In a common-emitter transistor amplifier circuit


b =100, input resistance R1 = 1kΩ, output resistance
(D) Higher forward voltage
R2 = 10kΩ. The voltage gain of circuit is
Q.39 Zener diode is always used in (A) 100 (B) 1000 (C) 10 (D) 5000
(A) Forward bias mode
Q.48 While using a transistor as an amplifier
(B) Reverse bias mode
(A) The collector junction is forward biased and emitter
(C) May be forward of reverse mode junction reverse biased
(D) Unbiased mode (B) The collector junction is reverse biased and emitter
junction is forward biased.
Q.40 Zener diode is used as a/an
(C) Both the junctions are forward biased
(A) Amplifier (B) Rectifier
(D) Both the junctions are revere biased
(C) Oscillator (d) Voltage regulator
Q.49 For a transistor the value of α =0.9, the value if
Q.41 In the zener diode circuit, total current is i, load β is
current is iL , zener current is iZ then
(A) 1 (B) 100 (C) 90 (D) 9
(A) iZ = iL (B) iZ = i + iL
Q.50 Multimeter is used to
(C) iZ = i − iL (D) iZ − iL =
i
(A) Check whether a given diode or a transistor is in
working order
Q.42 In a voltage regulating circuit of Zener diode, the
graph of output voltage V0 versus input voltage Vi is (B) Identify the base of a transistor and terminals of an IC.
(C) Distinguish between p-n-p and n-p-n transistor
Q.43 The current gain for common emitter amplifier is
(D) Verify all the above
59. If the emitter current is 6.0 mA, the base current and
collector current is
Q.51 While checking the nature of components of a
(A) 0.1 mA, 5.9 mA (B) 0.2 mA, 6.9 mA
transistor, if multimeter shows low resistance then
(C) 0.3 mA, 3.9 mA (D) 0.4 mA, 1.9 mA nature of the components connected to the positive
and negative terminals of multimeter respectively are
Q.44 In a transistor, the collector current is 5.488 mA (A) p-type, n-type (B) n-type, p-type
for an emitter current of 5.60 mA. The value of current
(C) n-type, n-type (D) p-type, p-type
amplification factor ( β ) will be
(A) 51 (B) 48 (C) 49 (D) 50
Q.52 The extreme terminals of a transistor are connected
with two probes of a multimeter. If it shows less deflection
Q.45 The value of β or more resistance, then central terminal is
(A) Is always less than 1 (A) Base (B) Emitter
(B) Lies between 20 and 200 (C) Collector (D) Any one
(C) Is always greater than 200
(D) Is always infinity Q.53 If a semi conductor device has more than three
legs (or) pins, it is a / an:

Q.46 The current gain α of a transistor is 0.95. The (A) Transistor (B) Junction diode
change in emitter current is 10 mA. The change in base (C) Zener diode (D) Integrated circuit
current is
(A) 9.5 mA (B) 0.5 mA
Q.54 Multimeter ends are connected to emitter base of
 200  a working order transistor, then multimeter shows
(C) 10.5 mA (D)   mA
 19 
2 7 . 4 2 | Experimental Physics

(A) Large deflection in one direction and small deflection Q.7 Will the time period of a simple pendulum change
in other direction if we take it from Earth to the surface of Moon?
(B) Large deflection in both cases
Q.8 Why does it increases?
(C) Small deflection in both cases
(D) None of the above Q.9 How does the value of g vary from place to place
on the surface of the Earth?
Q.55 A multimeter in connected to LED, then deflection
before and after reversing the probes is Q.10 Is g a scalar or a vector quantity?

(A) Same in both cases


Q.11 Why should the amplitude be small for a simple
(B) Large and then falls to zero pendulum experiment?
(C) Almost zero in both cases
Q.12 Does the time period depend upon the mass, the
(D) Very small in one case and very large in reverse case size and the material of the bob?
or vice versa and emits light

Q.13 What will happen to the time period if the bob is


Q.56 If the diode is spoiled then multimeter shows made to oscillate in water (neglect viscosity)?
(A) Large deflection in one case
(B) Small deflection in reverse case Q.14 How will the amplitude of oscillation change, if
the pendulum oscillate in vacuum?
(C) Deflection is large in one case and small in reverse
case
Q.15 Why do you avoid rotational motion of the bob
(D) Large or small deflection in both cases during the course of your experiment?

Q.57 How do we find whether a diode is working or not Q.16 What type of graph do you expect between (i) L
by using a multimeter? and T and (ii) L and T2 ?
(A) Forward bias resistance in more
(B) Reverse bias resistance is less Q.17 Why do the pendulum clocks go slow in summer
and fast in winter?
(C) In both the cases resistance is more
(D) Forward bias resistance is less and reverse bias Q.18 Why do we use Invar material for the pendulum
resistance more of good clocks?

Simple Pendulum Q.19 A simple pendulum has a bob which is a hollow


sphere full of sand and oscillates with certain period. If
Q.1 What is an idea simple pendulum? all that sand is drained out through a hole at its bottom,
then its period
Q.2 Is there any difference between gravity and (A) Increase (B) Decreases
acceleration due to gravity? (C) Remains same (D) Is zero

Q.3 What is the difference between g and G. Q.20 The second’s pendulum is taken from earth to
moon, to keep the time period constant
Q.4 What is meant by effective length of the pendulum?
(A) The length of the second’s pendulum should be
decreased
Q.5 What is second’s pendulum?
(B) The length of the second’s pendulum should be
increased
Q.6 What is the approximate effective length of a
second’s pendulum? (C) The amplitude should increase
(D) The amplitude should decrease
P hysi cs | 27.43

Q.21 The length of a second’s pendulum at the surface Q.33 Three identical wires A, B and C made of
of earth is 1 m. The length of second pendulum at different materials attached with same load have
YA = 20 × 1011 Nm−2 andYc =
0.2 × 1011 Nm−2 . The wire
the surface of moon where g is (1/ 6 ) that at earth’s
th
having maximum elongation is
surface is
1 1 (A) A
(A) m (B) 6 m (C) m (D) 36 m
6 36 (B) B
(C) C
Q.22 The time period of a second’s pendulum is 2 sec.
The spherical bob which is empty from inside has a (D) Same elongation for all three wires
mass 50 g. This is now replaced by another solid bob of
same radius but having a different mass of 100 g. The Q.34 Approximation to perfectly elastic body is:
new time period will be
(A) Quartz fibre (B) Optical fibre
(A) 4 s (B) 1 s (C) 2 s (D) 8 s
(C) Human bone (D) Copper rod

Q.23 The time period of a simple pendulum is doubled,


Q.35 Which of the following is wrong regarding Searle’s
when
apparatus method in finding Young’s modulus of a
(A) its length is doubled given wire?
(B) Mass of bob is doubled (A) Average elongation of wire will be determined
with a particular load while increasing the load and
(C) Length is made four times
decreasing the load.
(D) The mass of bob and length of pendulum are double
(B) Reference wire will be just taut and experimental
wire will undergo for elongation.
Elasticity
(C) Air bubble in the spirit level will be disturbed from
the central position due to relative displacement
Q.24 How do you differentiate between elastic and
between the wires due to elongation.
plastic bodies?
(D) Average elongation of the wires is to be determined
Q.25 What is meant by elastic limit? by increasing the load attached to both the wires.

Q.26 What is yield point? Q.36 Q wire of area of cross-section 10−6 m2 is increased
in length by 0.1%. The tension produced is 1000N. The
young’s modulus of wire is
Q.27 What is breaking point?
(A) 1012 Nm−2 (B) 1011Nm−2
Q.28 What is breaking stress? (C) 1010 Nm−2 (D) 109 Nm−2

Q.29 Is there any effect of temperature change on the Q.37 Which of the following is most elastic?
value of Y? (A) Rubber (B) Wet clay
(C) Plastic (D) Steel
Q.30 Why do you use long wires in searle’s experiment?

Q.38 Q wire can support a load W without breaking.


Q.31 Why is it advisable not to take readings of
It is cut into equal parts. The maximum load that each
micrometer screw immediately after loading or
part can support is
unloading?
W W
(A) (B) (C) W (D) 2W
Q.32 Radius of the experiment wire is to be measure 4 2
with ___________ in Searle’s experiment to determine Y.
Q.39 The following four wires are made of the same
(A) Ordinary scale (B) Vernier caliper material. Which of these will have the largest extension
(C) Spherometer (D) Screw gauge when the same tension is applied on all?
2 7 . 4 4 | Experimental Physics

(A) Length=50 cm, diameter=0.5 mm Q.49 For determining the mass of a given body using
a meter scale by principal of moments, the mass of
(B) Length=100 cm, diameter=1.3 mm
weight in the paper pan is 30 g and the length of the
(C) Length= 300 cm, diameter=2mm weight arm is 20 cm. If the fixed length of the power
(D) Length=300 cm, diameter=3mm arm is 25cm, then the unknown mass is
(A) 18 g    (B) 20 g   (C) 24 g   (D) 28 g
Q.40 Two wires of copper have lengths in the ratio 1:2
and radii in the ratio 2:1. Their Young’s modulli are in Q.50 A balance is made of rigid rod free to rotate about
the ratio a point not at the centre of the rod. When an unknown
(A) 1:1 (B) 1:8 (C) 8:1 (D) 1:4 mass m is placed in the left hand pan, it is balanced by
a mass m1 placed in the right hand pan, and similarly
when the mass m is placed in the right hand pan, it is
Q.41 Young’s modulus for a perfectly rigid body is balanced by a mass m2 in the left hand pan. Neglecting
(A) 0 (B) 1 (C) ∞    (D) None of these the masses of the pans, m is
1
Q.42 Three wires A, B and C made of different material,
(A) (m + m2 )
2 1
(B) m1m2

have same length and cross-section. They are stretched m2 + m22


by the same force. It is found that wire A is stretched 1 1
(C) m1m2 (D)
least and comes back to its original length when the 2 2
force is withdrawn. Wire B is stretched more than A and
also comes back to its original length. Wire C is stretched Q.51 A false balance has equal arms. An object weights
most but does not return to its original length. Which x when placed in one pan and y in the other pan. The
of the three materials has the largest Young’s modulus? true weight of the object is equal to
(A) A x+y
(A) xy (B)
(B) B 2

(C) C x2 + y 2
(C) ( x + y ) (D)
2
(D) No conclusion can be drawn from the given
information
Surface Tension and Viscosity
Principal of Moments
Q.52 Why are the small spheres wetted in glycerine
Q.43 What is the principal of moments? before making them to fall through glycerine?

Q.44 The bottom of a ship is made heavy. Why? Q.53 Why are the small spheres centrally dropped in
the glass jar?
Q.45 Why does a girl lean towards right while carrying
a bag in her left hand? Q.54 Why do bubbles of air or gas rise up through
water or any other liquid?
Q.46 Some heavy boxes are to be loaded along with
some empty boxes on a cart. Which boxes should be Q.55 Why does the water not rise in an ordinary glass
put in the cart first and why? tube as it does in a capillary tube?

Q.47 Standing is not allowed in deck of a double decker Q.56 What formula are you using to determine the
bus. Why? surface tension?

Q.48 Why cannot we rise from a chair without bending Q.57 While measuring the height of water column
a little forward? in the capillary, up to what point of meniscus do you
measure the height?
P hysi cs | 27.45

Q.58 Which of the following is a better cleaning agent? Q.65 A clean capillary tube is dipped vertically in
breaker containing water and the water rises 6cm in
(A) Cold water (B) Hot water
it. If another capillary tube of same radius and length
(C) Soap water (D) Hot soap water 3cm is dipped vertically in the same beaker containing
water (assume angle of contact of water is 0°C ) then
Q.59 A capillary tube is dipped in water and the top is the water
closed then the rise of water (A) Will flow out like a fountain
(A) Increases (B) Decreases (B) Will rise to a height of 3cm only and angle of
(C) Remains same (D) Water does not rise contacts is 0°C
(C) Will rise to a height of 3cm only and angle of contact
Q.60 If a capillary tube of insufficient length is used, is 60° C
the liquid (D) Will not rise at all
(A) Rises and over flows slowly
(B) Does not overflow Q.66 When a partially soluable impurity is added to a
liquid, then the surface tension
(C) Rises and comes out like fountain
(A) Increases (B) Decreases
(D) Does not rise into the tube
(C) Remains same (D) Cannot be measured

Q.61 What happens to the length of liquid column in


the capillary tube if it is inclined with respect to vertical? Q.67 When a completely soluable impurity is added to
a liquid, then the surface tension
(A) Decreases (B) Increases
(A) Increases (B) Decreases
(C) Remains same (D) Overflows
(C) Remains same (D) Become zero

Q.62 h is the height of the liquid column in the capillary


tube when it is clamped vertically. If the tube is making Q.68 If temperature is increase, then angle of contact

an angle ( θ ) with vertical. Then length of the liquid (A) Decreases (B) Increases

column (l) is (C) Become zero (D) Remains same

hcos θ
=l hcos θ
(A) (B) l = Q.69 If radius of the tube is decreased, then the height
2
of the liquid column
h
(C) l 2hcos θ
= (D) l = (A) Increases (B) Decreases
cos θ
(C) Unchanged (D) May increase or decrease
Q.63 Two students A and B determine the surface
tension in summer and winter as TA and TB respectively Q.70 If the angle of contact is 90° then the liquid level
9other conditions are same in the experiment). Then: in a capillary tube will
(A) TA > TB (B) TA < TB (A) Increase (B) Decrease
TB
(C) TA = TB (D) TA = (C) Neither rise nor fall (D) May increase of decrease
2

Q.71 What happens to the surface tension if detergents


Q.64 A student A does the capillary rise experiment at
are added to the water?
equator and gets the surface tension value TA . Another
student does the same experiment in same conditions (A) Increases (B) Decreases
at poles and gets the surface tension value TB . Then: (C) Remains same (D) Become zero
(A) TA > TB (B) TA < TB
(C) TA = TB (D) None of these
2 7 . 4 6 | Experimental Physics

Q.72 What happens to the angle of contact if detergents Q.78 A solid sphere falls with a terminal velocity of
are added to the water? 20ms−1 in air. If it is allowed to fall in vaccum
(A) Decreases (B) Increases (A) Terminal velocity will be 20ms−1
(C) Become zero (D) Remains same (B) Terminal velocity will be less than 20 ms−1
(C) Terminal velocity will be greater than 20 ms-1
Q.73 A liquid rises to a height h in a capillary tube on
(D) There will be no terminal velocity
the earth. The height to which the same liquid would
rise in the same tube on the moon is about
Q.79 Eight spherical rain drops of the same mass and
h
(A) 6h (B) h (C)    (D) None of these radius are falling down with a terminal speed v. If they
6 coalesce to form one big drop. Its terminal speed will
be (neglect buoyancy due to air)
Q.74 At critical temperature the surface tension of a
v
liquid (A)    (B) v   (C) 4v   (D) 8v
4
(A) Is zero
(B) Is infinity Q.80 A spherical steel ball released at the top of a long
column of glycerine of length L, falls through a distance
(C) Is the same as that at any other temperature
L/2 with accelerated motion and remaining distance
(D) Cannot be determined L/2 with a uniform velocity. If t1 and t2 denote the times
taken to cover the first and second half and w1 and w2
Q.75 A liquid will not wet the surface of a solid if the are the respective works done against gravity in the
angle of contact is two halves then

(A) Acute (B) Obtuse (A) t1 < t2 , w1 > w2 (B) t1 > t2 , w1 < w2

π (C)
= t1 t=
2 , w1 w2 (D) t1 > t2 , w1 =
w2
(C) Zero (D)
2
Q.81 A tiny sphere of mass m and density p is dropped
Q.76 A water-proofing agent changes the angle of in a tall jar of glycerine of density σ . When the sphere
contact from acquires terminal velocity, the magnitude of the viscous
forecasting on it is
(A) An obtuse to an acute value
mgp mgσ
(B) An acute to an obtuse value (A) (B)
σ p
π
(C) An obtuse value to  σ  p
2 (C) mg 1 −  (D) mg 1 − 
π  p  σ
(D) An acute value to
2
Q.82 The viscous force on a spherical body moving
Q.77 A rain drop falls near the surface of the earth with through a fluid depends upon
almost uniform velocity because (i) The mass of the body
(A) Its weight is negligible (ii) The radius of the body
(B) The force of surface tension balances its weight (iii) The velocity of the body
(C) The viscous drag and the upthrust due to air (iv) The viscosity of the fluid
balances its weight
(A) (i), (ii), and (iii) are true
(D) The drops are charged and atmospheric electric
field balances its weight (B) (i), (iii) and (iv) are true
(C) (ii), (iii) and (iv) are true
(D) (ii) and (iv) are true
P hysi cs | 27.47

Q.83 When the upthrust on a body is negligible Q.85 A steel ball of density ρ1 and radius r falls vertically
compared to its weight, the terminal velocity of a small through a liquid of density ρ2 . Assume that viscous
spherical body falling through a viscous liquid depends force acting on the ball is F=Krv, where k is a constant
upon and v is velocity. The terminal velocity of the ball is
(i) The density of the body 4 πgr 2 ( ρ1 − ρ2 ) 4 πr 2 ( ρ1 − ρ2 )
(A) (B)
(ii) The diameter of the body 3K 3gK
(iii) The viscosity of liquid 4 πr ( ρ1 + ρ2 ) 4 πgr
(iv) The acceleration due to gravity
(C)
2
(D)
3K
( ρ1 + ρ2 )
3gr K

(A) (i) and (ii) are true


Q.86 For a body falling with terminal velocity, the net
(B) (ii) and (iii) are true force on it is
(C) (ii), (iii) and (iv) are true (A) Buoyant force
(D) All are true (B) Weight of the body
(C) Difference of viscous force and weight of the body
Q.84 A small metal sphere of radius r and density p falls
(D) Zero
from rest in a viscous liquid of density σ and coefficient
of viscosity η . Due to friction heat is produced. The
expression for the rate of production of heat when the Q.87 An air bubble of radius 1.0 cm rises with a
sphere has acquired the terminal; velocity is constant speed of 3.5 mms−1 through a liquid of density
1.75 × 103 kgm−3. Neglecting the density of air, the
 8πg
(A) 
 27η
( 
)
p − σ2  r 5

 8πg
(B) 
 27η
(
2
p − σ )  r5
 coefficient of viscosity of the liquid (in kgm−1s−1 )
(A) 54.5 (B) 109
 8πg2   8πg2 
(C)  ( p − σ )  r5 (D)  (p − σ )  r5 (C) 163.5 (D) 218
 27η
2
  27η 

Answer Key

Sound
Q.35 C Q.36 D Q.37 A Q.38 C Q.39 A Q.40 A
Q.41 C Q.42 A Q.43 C Q.44 C Q.45 C Q.50 B
Q.51 C Q.53 D

Electricity and Magnetism


Q.11 D Q.12 C Q.13 A Q.14 D Q.15 C Q.15 B
Q.17 A Q.18 D Q.24 B Q.25 A Q.26 A Q.27 B
Q.28 C Q.29 B Q.30 A
2 7 . 4 8 | Experimental Physics

Vernier Callipers and Screw Gauge


Q.3. (B) 0.01 cm Q.9 C Q.10 B Q.11 B Q.12 A Q.13 D
Q.14 B Q.15 C Q.16 D Q.17 A Q.18 0.025cm Q.31 A
Q.32 B Q.33 B Q.34 B Q.35 A Q.36 A Q.37 A
Q.38 0.5 mm, 0.005 mm

Optics and Modern Physics


Q.12 D Q.13 C Q.14 C Q.15 B Q.16 A Q.21 A
Q.26 C Q.27 D Q.28 A Q.29 A Q.30 A Q.31 A
Q.32 D Q.33 C Q.34 C Q.35 C Q.36 D Q.37 A
Q.38 A Q.39 B Q.40 D Q.41 C Q.42 A Q.43 A
Q.44 C Q.45 B Q.46 B Q.47 B Q.48 B Q.49 D
Q.50 D Q.51 A Q.52 A Q.53 D Q.54 A Q.55 D
Q.56 D Q.57 D

Simple Pendulum, Elasticity, Principle of Moments, Surface Tension and Viscocity


Q.19 C Q.20 A Q.21 A Q.22 C Q.23 C Q.32 D
Q.33 C Q.34 A Q.35 D Q.36 A Q.37 D Q.38 C
Q.39 A Q.40 C Q.42 A Q.49 C Q.50 B Q.51 B
Q.58 D Q.59 C Q.60 B Q.61 B Q.62 D Q.63 B
Q.64 B Q.65 C Q.66 B Q.67 A Q.68 A Q.69 A
Q.70 C Q.71 B Q.72 A Q.73 A Q.74 A Q.75 B
Q.76 B Q.77 C Q.78 D Q.79 C Q.80 D Q.81 C
Q.82 C Q.83 D Q.84 B Q.85 A Q.86 D Q.87 B

You might also like